You are on page 1of 757

EBD_7418

• Corporate Office : 45, 2nd Floor, Maharishi Dayanand Marg, Corner Market,
Malviya Nagar, New Delhi-110017
Tel. : 011-49842349 / 49842350

by
Sanjeev Kumar Jha

Typeset by Disha DTP Team

DISHA PUBLICATION
All Rights Reserved

© Copyright Publisher
No part of this publication may be reproduced in any form without prior permission of the publisher. The author and
the publisher do not take any legal responsibility for any errors or misrepresentations that might have crept in. We have
tried and made our best efforts to provide accurate up-to-date information in this book.

For further information about the books from DISHA,


Log on to www.dishapublication.com or email to info@dishapublication.com

(ii)
Contents
S. No. Chapters Page No.
1. Physical World 1-4
2. Units and Measurements 5-18
3. Motion in a Straight Line 19-32
4. Motion in a Plane 33-48
5. Laws of Motion 49-64
6. Work, Energy and Power 65-80
7. System of Particles and Rotational Motion 81-96
8. Gravitation 97-110
9. Mechanical Properties of Solids 111-120
10. Mechanical Properties of Fluids 121-136
11. Thermal Properties of Matter 137-152
12. Thermodynamics 153-168
13. Kinetic Theory 169-182
14. Oscillations 183-198
15. Waves 199-214
16. Electric Charges and Fields 215-230
17. Electrostatic Potential and Capacitance 231-248
18. Current Electricity 249-266
19. Moving Charges and Magnetism 267-282
20. Magnetism and Matter 283-294
21. Electromagnetic Induction 295-308
22. Alternating Current 309-324
23. Electromagnetic Waves 325-336
24. Ray Optics and Optical Instruments 337-354
25. Wave Optics 355-270
26. Dual Nature of Radiation and Matter 371-384
27. Atoms 385-398
28. Nuclei 399-412
29. Semiconductor Electronics : Materials, Devices and Simple Circuits 413-428
30. Communication Systems 429-440

Hints & Solutions


S. No. Chapters Page No.
1. Physical World 441-441
2. Units and Measurements 442-448
3. Motion in a Straight Line 449-457
4. Motion in a Plane 458-469
(iii)
EBD_7418
5. Laws of Motion 470-481
6. Work, Energy and Power 482-494
7. System of Particles and Rotational Motion 495-507
8. Gravitation 508-517
9. Mechanical Properties of Solids 518-524
10. Mechanical Properties of Fluids 525-534
11. Thermal Properties of Matter 535-545
12. Thermodynamics 546-553
13. Kinetic Theory 554-561
14. Oscillations 562-574
15. Waves 575-586
16. Electric Charges and Fields 587-596
17. Electrostatic Potential and Capacitance 597-607
18. Current Electricity 608-620
19. Moving Charges and Magnetism 621-630
20. Magnetism and Matter 631-637
21. Electromagnetic Induction 638-644
22. Alternating Current 645-655
23. Electromagnetic Waves 656-661
24. Ray Optics and Optical Instruments 662-674
25. Wave Optics 675-683
26. Dual Nature of Radiation and Matter 684-692
27. Atoms 693-702
28. Nuclei 703-711
29. Semiconductor Electronics : Materials, Devices and Simple Circuits 712-719
30. Communication Systems 720-726

MOCK TEST

Mock Test - 1 MT-1-3

Mock Test - 2 MT-4-6

Mock Test - 3 MT-7-9

Mock Test - 4 MT-10-11

Mock Test - 5 MT-12-13

Hints and Solutions Mock Test (1-5) MT-14-26

(iv)
1
Chapter
PHYSICAL WORLD
EBD_7418
2 PHYSICS

Exercise 1 : Topic-wise MCQs

Topic 1: Physics, Technology and Society I. Optics deal with the phenomena involving light.
II. Unification means physical phenomena in terms of
1. Macroscopic domain includes few concepts and laws.
(a) phenomena at the laboratory III. Macroscopic domain of Physics deals with the
(b) terrestrial scales constitution and structure of matter at the minute
(c) astronomical scales scales of atoms and nuclei.
(d) All of the above (a) Only I (b) Only II
2. The man who is known as the Father of Experimental Physics (c) I and II (d) II and III
is 10. Match the Column I and Column II.
(a) Newton (b) Albert Einstein Column I Column II
(c) Galileo (d) Rutherford (A) Johannes Kepler (1) Nuclear model of the atom
3. The person who has been awarded the title of the Father of (B) Tycho Brahe (2) Planetary motion
Physics of 20th century is (C) Nicolas Copernicus (3) Elliptical orbit theory
(a) Madame Curie (b) Sir C.V. Raman (D) Ernest Rutherford (4) Circular orbit theory
(c) Neils Bohar (d) Albert Einstein (a) (A)®(2); (B)®(4); C®(3); (D)®(1)
4. Science is exploring, ...x... and ...y... from what we see (b) (A)®(1); (B)®(2); C®(3); (D)®(4)
around us. Here, x and y refer to (c) (A)®(2); (B)®(1); C®(4); (D)®(3)
(a) qualitative, modify (b) experiment, predict (d) (A)®(3); (B)®(2); C®(4); (D)®(1)
(c) verification, predict (d) reasoning, quantitative 11. C.V. Raman got Nobel Prize for his experiment on
5. The branch of science which deals with nature and natural (a) dispersion of light (b) reflection of light
phenomena is called (c) deflection of light (d) scattering of light
(a) Sociology (b) Biology 12. Louis de-Broglie is credited for his work on
(c) Civics (d) Physics (a) theory of relativity
6. Assertion : The concept of energy is central to Physics (b) electromagnetic theory
and expression for energy can be written for every (c) matter waves
physical system. (d) law of distribution of velocities
Reason : Law of conservation of energy is not valid for 13. Madam Marie Curie won Nobel Prize twice which were in
all forces and for any kind of transformation between the field of
different forms of energy. (a) Physics and chemistry(b) Chemistry only
(a) Assertion is correct, reason is correct; reason is a (c) Physics only (d) Biology only
correct explanation for assertion. 14. A scientific way of doing things involve
(b) Assertion is correct, reason is correct; reason is not a (a) identifying the problem
correct explanation for assertion (b) collecting data
(c) Assertion is correct, reason is incorrect (c) hypothesising a possible theory
(d) Assertion is incorrect, reason is correct. (d) All of the above
7. Which of the following is true regarding the physical science? 15. Two Indian born physicists who have been awarded Nobel
Prize in Physics are
(a) They deal with non-living things
(a) H. J. Bhabha and APJ Kalam
(b) The study of matter are conducted at atomic or ionic
(b) C.V. Raman and S. Chandrasekhar
levels
(c) J.C. Bose and M.N. Saha
(c) Both (a) and (b)
(d) S. N. Bose and H. J. Bhabha
(d) None of these 16. Column I Column II
8. The scientific principle involves in production of ultra high (A) Galileo Galilei (1) Explanation of
magnetic fields is photoelectric effect.
(a) super conductivity (b) digital logic (B) JC Bose (2) Law of inertia.
(c) photoelectric effect (d) laws of thermodynamics (C) Albert Einstein (3) Discovery of Ultra short
9. Consider the following statements and select the correct radio waves.
statement(s). (D) JJ Thomson (4) Discovery of electron.
PHYSICAL WORLD 3

(a) (A)®(2); (B)®(3); C®(1); (D)®(4) Flash of


Lead
(b) (A)®(1); (B)®(2); C®(4); (D)®(3) block light
(c) (A)®(1); (B)®(2); C®(3); (D)®(4) Microscope
(d) (A)®(3); (B)®(4); C®(1); (D)®(2) a Fluorescent
q
17. The man who has won Nobel Prize twice in physics is screen
A Scattering
(a) Einstein (b) Bardeen angle
(c) Heisenberg (d) Faraday B
18. Prof. Albert Einstein got nobel prize in physics for his work
on (a) polonium sample and aluminium foil
(b) polonium sample and gold foil
(a) special theory of relativity
(c) uranium sample and gold foil
(b) general theory of relativity
(d) uranium sample and aluminium foil
(c) photoelectric effect
(d) theory of specific heats Topic 2: Fundamental Forces in Nature
19. Which of the following is wrongly matched ?
25. The exchange particles for the electromagnetic force are
(a) Barometer-Pressure
(a) gravitons (b) gluons
(b) Lactometer-Milk
(c) photons (d) mesons
(c) Coulomb’s law-charges
26. Which of the following is the weakest force?
(d) Humidity-Calorimeter
(a) Nuclear force (b) Gravitational force
20. Column I Column II
(c) Electromagnetic force (d) None of these
(A) J.C Maxwell (1) Verified experimentally
27. Assertion : Electromagnetic force is much stronger than
the prediction of
the gravitational force.
electromagnetic force.
Reason : Electromagnetic force dominates all phenomena at
(B) Cario Rubia (2) Unified electricity,
atomic and molecular scales.
magnetism and optics,
(a) Assertion is correct, reason is correct; reason is a
showed that light is an
correct explanation for assertion.
EM waves.
(b) Assertion is correct, reason is correct; reason is not a
(C) Isaac Newton (3) Unified celestial and
correct explanation for assertion
terrestrial mechanics.
(c) Assertion is correct, reason is incorrect
(D) Michael Faraday (4) Showed that electric
(d) Assertion is incorrect, reason is correct.
and magnetic
28. When we hold a book in our hand, we are balancing the
phenomenon i.e.,
gravitational force on the book due to
electromagnetism.
(a) normal force provided by our hand
(a) (A)®(1); (B)®(2); C®(4); (D)®(3) (b) friction force provided by our book
(b) (A)®(2); (B)®(1); C®(3); (D)®(4) (c) both (a) and (b)
(c) (A)®(2); (B)®(3); C®(4); (D)®(1) (d) None of these
29. Which of the following has infinite range?
(d) (A)®(2); (B)®(1); C®(4); (D)®(3)
(a) Gravitational force (b) Electromagnetic force
21. Who gave general theory of relativity? (c) Strong nuclear force (d) Both (a) and (b)
(a) Einstein (b) Marconi 30. Which of the following statements is/are correct?
(c) Ampere (d) Newton I. Strong nuclear force binds protons and neutrons in a
22. Who discovered X-rays? nucleus.
(a) Chadwick (b) Roentgen II. In twentieth century, silicon chip triggered a
(c) Thomson (d) Madam Curie
revolutionary changes in technology of computer
23. The field of work of S. Chandrashekar is
system.
(a) theory of black hole (b) Cosmic rays
III. The fossil fuels of the planet are dwindling fast and
(c) theory of relativity (d) X-rays
there is urgent need to discover new source of energy.
24. In Rutherford, alpha particle scattering experiment as
(a) Only I (b) Only III
shown in given figure, A and B refer to
(c) I and II (d) I, II and III
EBD_7418
4 PHYSICS

31. Which of the following is the correct decreasing order of (b) Strong nuclear force > weak nuclear force >
the strengths of four fundamental forces of nature ? electromagnetic force > gravitational force
(a) Electromagnetic force > weak nuclear force > (c) Gravitational force > electromagnetic force > strong
gravitational force > strong nuclear force nuclear force > weak nuclear force
(d) Strong nuclear force > electromagnetic force > weak
nuclear force > gravitational force

ANSWER KEYS
Exercise 1 : Topic-wise MCQs
1 (d) 5 (d) 9 (a) 13 (a) 17 (b) 21 (a) 25 (c) 29 (d)
2 (c) 6 (c) 10 (a) 14 (d) 18 (c) 22 (b) 26 (b) 30 (d)
3 (d) 7 (c) 11 (d) 15 (b) 19 (d) 23 (a) 27 (a) 31 (d)
4 (b) 8 (a) 12 (c) 16 (a) 20 (b) 24 (b) 28 (a)
2
Chapter
UNITS AND MEASUREMENTS

Trend
Analysis of NEET and AIIMS (Year 2010-2018)
5

4
Number of Questions

3
AIPMT/NEET
2 AIIMS

0
2010 2011 2012 2013 2014 2015 2016 2017 2018
Year

Trend
Analysis of JEE Main and BITSAT (Year 2010-2018)

4
Number of Questions

3
JEE Main
2 BITSAT

0
2010 2011 2012 2013 2014 2015 2016 2017 2018
Year

Chapter Utility Score (CUS)


Exam Weightage Important Concepts Difficulty Level CUS (Out of 10)

NEET 4 Errors in measurements,


AIIMS 5 Dimensional Analysis and 3.5/5 7.5/10
JEE Main 4 its Applications
BITSAT 3
EBD_7418
6 PHYSICS
UNITS AND MEASUREMENTS 7
EBD_7418
8 PHYSICS

Exercise 1 : Topic-wise MCQs


(c) the distance travelled by light in free space in one year.
Topic 1: Unit of Physical Quantities
(d) time taken by earth to go once around the sun.
1. Which one of the following pairs of quantities and their 12. The unit of the coefficient of viscosity in S.I. system is
units is a proper match? (a) m/kg-s (b) m-s/kg2 (c) kg/m-s2 (d) kg/m-s
(a) Impulse – N/sec (b) Magnetic flux – weber 13. The unit of the Stefan-Boltzmann's constant is
(c) Power – farad (d) Capacitance – henry (a) W/m2K4 (b) W/m2 (c) W/m2K (d) W/m2K2
2. One second is defined to be equal to
(a) 1650763.73 periods of the krypton clock 14. The wrong unit conversion among the following is
(b) 652189.63 periods of the krypton clock (a) 1 angstrom = 10–10m
(c) 1650763.73 periods of the cesium clock (b) 1 fermi = 10–15m
(d) 9192631770 periods of the cesium clock (c) 1 light year = 9.46 × l015m
3. One yard in SI unit is equal to (d) 1 astronomical unit = 1.496 × 10 –11m
(a) 1.9144 metre (b) 0.9144 metre 15. Unit of specific resistance is
(c) 0.09144 kilometre (d) 1.0936 kilometre (a) ohm/m2 (b) ohm m3 (c) ohm – m (d) ohm/m
4. One pico farad is equal to 16. Unit of magnetic moment is
(a) 10–24 farad (b) 10–18 farad
(a) ampere–metre2 (b) ampere–metre
(c) 10–12 farad (d) 10–6 farad
5. ‘Torr’ is the unit of (c) weber–metre2 (d) weber/metre
(a) pressure (b) volume (c) density (d) flux 17. Surface tension of a liquid is 70 dyne/cm. Its value in SI is
6. Very large distances such as the distance of a Planet or (a) 70 N/m (b) 7 × 10–2 N/m
2
(c) 7 × 10 N/m (d) 7 × 103 N/m
a star from Earth can be measured by
(a) Spectrograph 18. Assertion : When we change the unit of measurement of a
(b) Millikan’s oil drop method quantity, its numerical value changes.
(c) Parallax method Reason : Smaller the unit of measurement smaller is its
(d) All of these numerical value.
7. Joule second is a unit of
(a) Assertion is correct, reason is correct; reason is a
(a) energy (b) torque
(c) power (d) angular momentum correct explanation for assertion.
8. Temperature can be expressed as derived quantity in terms (b) Assertion is correct, reason is correct; reason is not a
of correct explanation for assertion
(a) length and mass (b) mass and time (c) Assertion is correct, reason is incorrect
(c) length, mass and time (d) None of these (d) Assertion is incorrect, reason is correct.
9. One unified atomic mass unit is equal to 19. For the given figure solid angle, dW is equal to
(a) 12 times the mass of one carbon-12 atom
1 r
(b) of the mass of 12 atoms of C-12
12 dW dA
1
(c) of the mass of one atom of C-12
12
(d) 12 times the mass of 12 atoms of C-12
10. Match the columns I and II (a) r2dA steradian (b) dA/r2 steradian
Column I Column II
(A) Practical unit (1) radian r2
(c) steradian (d) dA/r steradian
(B) Base unit (2) light year dA
(C) Derived unit (3) kg-ms–1 20. Column I Column II
(D) Complementary unit (4) second (A) Distance between earth & stars (1) micron
(a) (A)®(4); (B)®(2); C®(1); (D)®(3) (B) Inter-atomic distance in a solid (2) angstrom
(C) Size of the nucleus (3) light year
(b) (A)®(2); (B)®(4); C®(3); (D)®(1)
(D) Wavelength of infrared laser (4) fermi
(c) (A)®(3); (B)®(2); C®(4); (D)®(1) (5) kilometre
(d) (A)®(2); (B)®(4); C®(1); (D)®(3) (a) (A)®(1); (B)®(3); C®(4); (D)®(2)
11. Light year is (b) (A)®(3); (B)®(2); C®(4); (D)®(1)
(a) light emitted by the sun in one year. (c) (A)®(5); (B)®(2); C®(3); (D)®(1)
(b) time taken by light to travel from sun to earth. (d) (A)®(2); (B)®(4); C®(1); (D)®(3)
UNITS AND MEASUREMENTS 9

21. Which of the following represents incorrect unit? 33. The correct unit of thermal conductivity is
(a) Pressure = N/m2 (b) Surface tension = N/m (a) Jm–2 sec–1(oC)–1 (b) Jm–1 sec–1(oC)–2
Stress (c) J-sec (d) Jm–1 sec–1(oC)–1
(c) Energy = kg-m/sec (d) =N/m2 34. If unit of length and force are increased 4 times. The unit of
Strain
energy
22. Which of the following is the unit of molar gas constant?
(a) is increased by 4 times
(a) JK–1 mol–1 (b) Joule
(c) JK–1 (d) J mol–1 (b) is increased by 16 times
23. Number of fundamental units in SI system is (c) is increased by 8 times
(a) 4 (b) 7 (c) 3 (d) 5 (d) remains unchanged
24. Consider the following statements and select the correct 35. The density of a material in CGS system of units is 4g/cm3.
statement(s). In a system of units in which unit of length is 10 cm and unit
I. 1 calorie = 4.18 joule of mass is 100 g, the value of density of material will be
o (a) 0.4 (b) 40 (c) 400 (d) 0.04
II. 1A = 10–10 m
III. 1 MeV = 1.6 × 10–13 joule Topic 2: Dimensions of Physical Quantities
IV. 1 newton = 10–5 dyne 36. Dimensional analysis can be applied to
(a) I, II and III (b) III and IV
(a) check the dimensional consistency of equations
(c) I only (d) IV only
(b) deduce relations among the physical quantities
25. Which one of the following is not measured in units of
(c) to convert from one system of units to another
energy ?
(a) Couple × angle (d) All of these
(b) Moment of inertia × (angular velocity)2 37. The dimensional formula for relative density is
(c) Force × distance (a) [ML–3] (b) [MoL–3]
o
(c) [M L T ]o –1 (d) [MoLoTo]
(d) Impulse × time
26. The numerical values of young's modulus in S.I. unit is b. 38. The dimensions of speed and velocity are
What is its numerical value in cgs system? (a) [L2T], [LT –1] (b) [LT–1], [LT –2]
(a) b (b) 10b (c) b/10 (d) 100 b (c) [LT], [LT] (d) [LT–1], [LT –1]
–1 –1
39. [MLT ] + [MLT ] = .............
æ a ö
27. In the eqn. ç P + ÷ (V - b) = constant, the unit of a is
è V2 ø (a) [M°L°T°] (b) [MLT–1]
(c) 2[MLT ] –1 (d) None of these
(a) dyne × cm5 (b) dyne × cm4
(c) dyne/cm 3 (d) dyne × cm2 40. The dimensions of pressure is equal to
28. In equation, r = m sin pt , where t represents time. If the unit
2 (a) force per unit volume (b) energy per unit volume
of m is N, then the unit of r is (c) force (d) energy
(a) N (b) N2 (c) Ns (d) N2 s 41. The dimensional formula of angular velocity is
2
29. If x = at + bt , where x is the distance travelled by the body (a) [MLT–1] (b) [M0L0T]
in kilometers while t is the time in seconds, then the unit of 0
(c) [ML T ] –2 (d) [M0L0T–1]
b is 42. The physical quantity that has no dimensions is
(a) km/s (b) kms (c) km/s2 (d) kms2 (a) strain (b) angular velocity
30. The unit of permittivity of free space, eo is (c) angular momentum (d) linear momentum
(a) coulomb2/(newton-metre)2 43. Which is dimensionless?
(b) coulomb/newton-metre (a) Force/acceleration (b) Velocity/acceleration
(c) newton-meter2/coulomb2 (c) Volume/area (d) Energy/work
(d) coulomb2/newton-metre2 44. Column I Column II
r A (A) Joule (1) MLT –2
31. The electric field is given by E = 3 iˆ + Byjˆ + Cz 2kˆ . The SI
x (B) Newton (2) ML–1 T –2
units of A, B and C are respectively: [where x, y and z are in m] (C) Hertz (3) ML2 T –2
(D) Pascal (4) M0L0 T –1
N - m3
(a) , V/m2, N/m2-C(b) V-m2, V/m, N/m2-C (a) (A)®(4); (B)®(2); C®(1); (D)®(3)
C
(b) (A)®(1); (B)®(2); C®(4); (D)®(3)
(c) V/m2, V/m, N-C/m2 (d) V/m, N-m3/C, N-C/m
(c) (A)®(3); (B)®(1); C®(4); (D)®(2)
32. E, m, J and G denote energy, mass, angular momentum and
(d) (A)®(2); (B)®(4); C®(1); (D)®(3)
gravitational constant respectively, then the unit of
45. Dimensions of specific heat are
EJ 2
is (a) [ML2 T–2 K] (b) [ML2 T–2 K–1]
m5 G 2
(a) newton (b) metre (c) kilogram (d) second (c) [ML2 T2 K–1] (d) [L2 T–2 K–1]
EBD_7418
10 PHYSICS

46. The dimensions of torque are 59. Assertion : The time period of a pendulum is given by the
(a) [MLT–2] (b) [ML2T–2] formula, T = 2p g / l
(c) [ML T ] 2 –1 (d) [M2L2T–2] Reason : According to the principle of homogeneity of
47. The dimensional formula for magnetic flux is dimensions, only that formula is correct in which the
(a) [ML2T–2A–1] (b) [ML3T–2A–2] dimensions of L.H.S. is equal to dimensions of R.H.S.
0 –2
(c) [M L T A ] 2 –2 (d) [ML2T–1A2] (a) Assertion is correct, reason is correct; reason is a
48. The dimensions of magnetic field in M, L, T and C (coulomb) correct explanation for assertion.
is given as (b) Assertion is correct, reason is correct; reason is not a
(a) [MLT–1 C–1] (b) [MT2 C–2] correct explanation for assertion
(c) [MT C ]–1 –1 (d) [MT–2 C–1] (c) Assertion is correct, reason is incorrect
49. The dimensions of coefficient of self inductance are (d) Assertion is incorrect, reason is correct.
(a) [ ML2 T–2 A–2 ] (b) [ ML2 T–2 A–1 ] 60. If P,Q, R are physical quantities, having different dimensions,
(c) [ MLT A ] –2 –2 (d) [ MLT–2 A–1 ] which of the following combinations can never be a meaningful
50. The division of energy by time is X. The dimensional formula quantity?
of X is same as that of (a) (P – Q) / R (b) PQ – R
(a) momentum (b) power (c) PQ / R (d) (PR – Q2) / R
(c) torque (d) electric field 61. Match the columns I and II.
51. Which of the following is a dimensional constant? Column I Column II
(a) Refractive index (b) Dielectric constant (A) Angle (1) ML2T–3
(b) Relative density (d) Gravitational constant (B) Power (2) M0L0T 0
52. The dimensions of mobility are (C) Work (3) ML2T–2
(a) [M–2T2A] (b) [M–1T2A] (D) Energy/Length (4) MLT–2
–2
(c) [M T A] 3 (d) [M–1T3A] (a) (A)®(3); (B)®(4); C®(1); (D)®(3)
53. Which one of the following represents the correct (b) (A)®(2); B®(1); (C)®(5); (D)®(4)
dimensions of the coefficient of viscosity? (c) (A)®(3); (B)®(1); C®(4); (D)®(2)
(a) [ML–1T–1] (b) [MLT–1] (d) (A)®(2); (B)®(1); C®(3); (D)®(4)
(c) [ML T ]–1 –2 (d) [ML–2T–2] 62. Dimensions of ‘resistance’ are same as (where h is Planck’s
1 constant and e is charge)
54. Assertion: Formula for kinetic energy is K = mu2 = ma
2 h h2 h h2
1 (a) (b) (c) (d)
Reason : Both the equation K = mv2 and k = ma are
e e e2 e2
2 63. If L denotes the inductance of an inductor through which a
dimensionally incorrect. current i is flowing, the dimensions of L i2 are
(a) Assertion is correct, reason is correct; reason is a (a) [ML2 T–2] (b) [MLT–2]
correct explanation for assertion. (c) [M2 L2 T–2] (d) Not expressible in M, L, T
(b) Assertion is correct, reason is correct; reason is not a a
correct explanation for assertion 64. If v = + bt 3 where v = velocity and t is time
t
(c) Assertion is correct, reason is incorrect The dimensional formula of a and b are
(d) Assertion is incorrect, reason is correct. (a) [T], [T–3] (b) [L], [LT–4]
55. The ratio of the dimensions of Planck’s constant and that (c) [T–3], [T] (d) [LT–4], [L]
of the moment of inertia is the dimensions of 65. The displacement of a body at a particular second n is given
(a) time (b) frequency a
(c) angular momentum (d) velocity by the expression Snth = u + (2n - 1) . The dimensional
2
56. Two quantities A and B have same dimensions which formula of Snth in this equation is
mathematical operation given below is physically meaningful? (a) [M1L0T1] (b) [M0L1T0] (c) [M0L1T–1] (d) [M0L0T0]
(a) A/B (b) A + B2 (c) A2 – B (d) A = B2 aV
57. The solar constant is defined as the energy incident per RT RT
66. In the equation P = e
unit area per second. The dimensional formula for solar V-b
constant is V = volume, P = pressure, R = universal gas constant,
(a) [M0L0T0] (b) [MLT–2] and T = temperature
(c) [ML T ]2 –2 (d) [ML0T–3] The dimensional formula of a is same as that of
(a) V (b) P (c) T (d) R
58. Distance travelled by a particle at any instant ‘t’ can be
67. Let Q denote the charge on the plate of a capacitor of
represented as S = A (t + B) + Ct 2. The dimensions of B are
(a) [M0L1T–1] (b) [M0L0T1] Q2
capacitance C. The dimensional formula for
is
0
(c) [M L T ] –1 –2 (d) [M0L2T–2] C
(a) [L2M2T] (b) [LMT2] (c) [L2MT–2] (d) [L2M2T2]
UNITS AND MEASUREMENTS 11

68. Suppose the kinetic energy of a body oscillating with 78. The thrust developed by a rocket-motor is given by F = mv
amplitude A and at a distance x is given by + A(P1 – P2) where m is the mass of the gas ejected per unit
Bx time, v is velocity of the gas, A is area of cross-section of
K= 2
x + A2 the nozzle, P1 and P2 are the pressures of the exhaust gas
The dimensions of B are the same as that of and surrounding atmosphere. The formula is dimensionally
(a) work/time (b) work × distance (a) correct (b) wrong
(c) work/distance (d) work × time (c) sometimes wrong, sometimes correct
69. If the dimensions of a physical quantity are given by (d) Data is not adequate
Ma Lb Tc, then the physical quantity will be 79. A, B, C and D are four different physical quantities having
(a) velocity if a = 1, b = 0, c = – 1 different dimensions. None of them is dimensionless. But
(b) acceleration if a = 1, b = 1, c = – 2 we know that the equation AD = C ln (BD) holds true. Then
(c) force if a = 0, b = – 1, c = – 2 which of the combination is not a meaningful quantity?
(d) pressure if a = 1, b = – 1, c = – 2
C AD 2
70. If force (F), length (L) and time (T) are assumed to be (a) - (b) A2 –B2C2
fundamental units, then the dimensional formula of the mass BD C
will be A (A - C)
(c) -C (d)
(a) [FL–1T2] (b) [FL–1T–2] B D
–1
(c) [FL T ] –1 (d) [FL2T2] 80. If electronic charge e, electron mass m, speed of light in
71. Which one of the following represents the correct vacuum c and Planck’s constant h are taken as fundamental
dimensions of the gravitational constant? quantities, the permeability of vacuum m0 can be expressed
(a) [M–1L3T–2] (b) [MLT–1] in units of
(c) [ML T ]–1 –2 (d) [ML–2T–2] æ h ö æ hc ö
72. A force is given by F = at + bt2, where t is time, the dimensions (a) ç 2÷ (b) ç 2÷
è me ø è me ø
of a and b are
æ h ö æ mc 2 ö
(a) [M L T–4] and [M L T–1] (c) ç 2÷ (d) çç 2 ÷÷
è ce ø è he ø
(b) [M L T–1] and [M L T0]
(c) [M L T–3] and [M L T–4] Topic 3: Errors in Measurements, Significant Figures
(d) [M L T–3] and [M L T0] 81. The magnitude of the difference between the individual
b- x 2 measurement and true value of the quantity is called
73. Write the dimensions of a × b in the relation E = , (a) absolute error (b) relative error
at
(c) percentage error (d) None of these
where E is the energy, x is the displacement and t is time 82. Systematic errors can be
(a) ML2T (b) M–1L2T 1 (c) ML2T–2 (d) MLT–2 (a) positive only
r r r
74. The dimensions of the quantity E ´ B where E represents (b) negative only
r (c) either positive or negative
the electric field and B the magnetic field may be given
as: (d) None of these
(a) [MT3] (b) [M2LT –5A–2] 83. Random error can be eliminated by
2 –3 2 –1
(a) careful observation
(c) [M L T A ] (d) [MLT–2A–2] (b) eliminating the cause
75. What are the dimensions of A/B in the relation F = A x + Bt 2, (c) measuring the quantity with more than one instrument
where F is the force, x is the distance and t is time? (d) taking large number of observations and then their
(a) ML2T–2 (b) L–1/2T 2 (c) L–1/2T–1 (d) LT–2 mean
84. The _____ is the error associated with the resolution of
76. A physical quantity x depends on quantities y and z as
the instrument.
follows : x = Ay + B tan Cz, where A, B and C are constants.
(a) parallax error (b) systematic error
Which of the following do not have the same dimensions : (c) random error (d) least count error
(a) x and B (b) C and z–1 (c) y and B/A (d) x and A 85. The smallest value that can be measured by the measuring
DV instrument is called
77. A quantity X is given by e 0 L where Î0 is the permittivity
Dt (a) least count (b) parallax
(c) accuracy (d) precision
of the free space, L is a length, DV is a potential difference
86. Which of the following is not a systematic error?
and Dt is a time interval. The dimensional formula for X is
(a) Instrumental error
the same as that of (b) Imperfection in experimental technique
(a) resistance (b) charge (c) Personal error
(c) voltage (d) current (d) None of these
EBD_7418
12 PHYSICS

87. The _____ is a measure of how closed the measured 97. Error in the measurement of radius of a sphere is 1%. Then
value is to the true value of quantity. error in the measurement of volume is
(a) Precision (b) accuracy (a) 1% (b) 5% (c) 3% (d) 8%
(c) Error (d) None of these 98. Column I Column II
88. Consider the following statements and select the correct (A) Length (1) burette
option.
I. Every measurement by any measuring instrument (B) Volume (2) Vernier callipers
has some error (C) Diameter of a thin wire (3) screw gauge
II. Every calculated physical quantity that is based on (D) Mass (4) common balance
measured values has some error
III. A measurement can have more accuracy but less (a) (A)®(4); (B)®(2); C®(1); (D)®(3)
precision and vice-versa (b) (A)®(3); (B)®(2); C®(4); (D)®(1)
(a) I and II (b) II and III (c) (A)®(4); (B)®(2); C®(3); (D)®(1)
(c) II and III (d) I, II and III (d) (A)®(2); (B)®(1); C®(3); (D)®(4)
89. When two quantities are divided, the relative error in the 99. The respective number of significant figures for the number
result is given by 23.023, 0.0003 and 2.1 × 10–3 are respectively.
(a) the product of the relative error in the individual (a) 5, 1 and 2 (b) 5, 1 and 5
quantities (c) 5, 5 and 2 (d) 4, 4 and 2
(b) the quotient of the relative error in the individual 100. The value of resistance is 10.845 W and the value of current is
quantities 3.23 A. The potential difference is 35.02935 volt. Its value in
(c) the difference of the relative error in the individual significant number would be
quantities (a) 35 V (b) 35.0 V (c) 35.03 V (d) 35.029 V
(d) the sum of the relative error in the individual 101. Mass of a body is 210 gm and its density is 7.981 g/cm3
quantities what will be its volume, with regard to significant digits?
DZ (a) 26.312 cm3 (b) 26 cm3
90. If Z = A3, then = ––––– 3
Z (c) 27 cm (d) 26.3 cm3
3 102. Which of the following statements is/are correct?
DA3 æ DA ö
(a) (b) ç ÷ I. 345.726 has six significant figures.
A è A ø
II. 0.004289 has seven singificant figures.
1/3
æ DA ö æ DA ö III. 125000 has three significant figures.
(c) 3ç ÷ (d) ç ÷
è A ø è A ø IV. 9.0042 has five significant figures.
91. The pitch and the number of circular scale divisions in a (a) I only (b) II only
screw gauge with least count 0.02 mm are respectively (c) I, III and IV (d) II, III and IV
(a) l mm and 100 (b) 0.5 mm and 50 103. Resistance R = V/I, here V = (100 ± 5)V and I = (100 ± 0.2 )
(c) 1 mm and 50 (d) 0.5 mm and 100 A. Find percentage error in R.
92. Which of the following statements is/are correct? (a) 5% (b) 2% (c) 7% (d) 3%
I. Change of units does not change the number of 104. If x = a – b, then the maximum percentage error in the
significant digits. measurement of x will be
II. All the non-zero digits are significant. æ Da Db ö æ Da Db ö
III. All the zero between two non-zero digits are (a) ç + ÷ ´ 100% (b) ç - ÷ ´ 100%
è a b ø è a b ø
significant.
(a) I only (b) II only æ Da Db ö æ Da Db ö
(c) ç + ÷ ´ 100% (d) ç - ÷ ´ 100%
(c) II and III (d) I, II and III èa-b a -bø èa-b a -bø
93. If L = 2. 331 cm, B = 2.1 cm, then L + B = 105. A wire has a mass 0.3 ± 0.003 g, radius 0.5 ± 0.005 mm and
(a) 4.4 cm (b) 4 cm (c) 4.43 cm (d) 4.431 cm length 6 ± 0.06 cm. The maximum percentage error in the
94. What is the correct number of significant figures in measurement of its density is
0.0003026? (a) 1 (b) 2 (c) 3 (d) 4
(a) Four (b) Seven (c) Eight (d) Six 106. The pitch of the screw gauge is 0.5 mm. Its circular scale
95. Which of the following is the most accurate? contains 50 divisions. The least count of the screw gauge is
(a) 200.0 m (b) 20 × 101 m (a) 0.001 mm(b) 0.01 mm (c) 0.02 mm (d) 0.025 mm
(c) 2 × 102 m (d) 0.2 × 103 m 107. In a vernier callipers N division of vernier coincide with
4.327 g (N – 1) divisions of main scale in which length of a division
96. Number of significant figures in expression is
2.51 cm3 is 1 mm. The least count of the instrument in cm is
(a) 2 (b) 4 (c) 3 (d) 5 1
(a) N (b) N – 1 (c) (d) (1 / N) – 1
10N
UNITS AND MEASUREMENTS 13

108. Find equivalent resistance when R1 = (100 ± 3)W and Quantity P is calculated as follows
R2 = (200 ± 4)W when connected in series
a 3b 2
(a) (300 ± 7)W (b) (300 ± 1)W P= % error in P is
(c) (100 ± 7)W (d) None of these cd
(a) 10% (b) 7% (c) 4% (d) 14%
DZ
109. If Z = A4 B1/3/ CD3/2, than relative error in Z. is 116. The length of one rod l1 = 3.323 cm and the other is
Z l2 = 3.321 cm. Both rods were measured with one measuring
equal to instrument with least count 0.001 cm Then (l1 – l2) is
4
æ DA ö æ DB ö
1/3
æ DC ö æ DD ö
3/2 (a) (0.002 ± 0.001) cm (b) (0.002 ± 0.000) cm
(a) ç ÷ +ç ÷ –ç ÷-ç ÷ (c) (0.002 ± 0.002) cm (d) None of these
è A ø è B ø è C ø è D ø 117. The number of significant figures in a number
æ DA ö æ 1 öæ DB ö æ DC ö æ 3 öæ DD ö “1700.00200” is
(b) 4 ç ÷ + ç ÷ç ÷ +ç ÷ + ç ÷ç ÷ (a) 3 (b) 7 (c) 9 (d) 10
è A ø è 3 øè B ø è C ø è 2 øè D ø
118. The internal and external diameter of a hollow cylinder are
æ DA ö 1 æ DB ö æ DC ö æ 3 öæ DD ö measured with the help of vernier callipers. Their values are
(c) 4ç ÷+ ç ÷-ç ÷ - ç ÷ç ÷
è A ø 3 è B ø è C ø è 2 øè D ø 4.23 ± 0.01 cm and 3.87 ± 0.01 cm respectively. The thickness
4 of the wall of the cylinder is
æ D A ö 1 æ D B ö æ D C ö 3 æ DD ö
(d) ç ÷ + ç ÷+ç ÷+ ç ÷ (a) 0.36 ± 0.02 cm (b) 0.18 ± 0.02 cm
è A ø 3è B ø è C ø 2è D ø (c) 0.36 ± 0.01 cm (d) 0.18 ± 0.01 cm
110. Two masses MA and MB (MA < MB) are weighed using same 119. In a vernier callipers, ten smallest divisions of the vernier
weighing machine. Absolute error and relative error in two scale are equal to nine smallest division on the main scale. If
measurement are (Assume only systematic errors are the smallest division on the main scale is half millimeter,
involved) then the vernier constant is
(a) absolute error same for both, relative error greater for (a) 0.5 mm (b) 0.1 mm (c) 0.05 mm (d) 0.005 mm
MA and lesser for MB. 120. The heat generated in a circuit is given by Q = I2 Rt, where
(b) absolute error same for both, relative error greater for I is current, R is resistance and t is time. If the percentage
MB and lesser for MA. errors in measuring I, R and t are 2%, 1% and 1% respectively,
then the maximum error in measuring heat will be
(c) relative error same for both, absolute error greater for
(a) 2% (b) 3% (c) 4% (d) 6%
MA and lesser for MB. 121. A screw gauge has 200 equal divisions marked along the
(d) relative error same for both, absolute error greater for circumference of the disc and one full rotation of the disc
MB and lesser for MA. advances on the main scale by 0.05 cm. The least count of
111. One centimetre on the main scale of a vernier callipers is the screw gauge is
divided into 10 equal parts. If 10 divisions of vernier coincide (a) 2.5 ×10–4 cm (b) 2.5 × 10–3 cm
with 8 small divisions of the main scale, the least count of (c) 6.25 × 10–4 cm (d) 6.25×10–3 cm
vernier callipers is 122. A physical quantity P is described by the relation
(a) 0.01 cm (b) 0.02 cm (c) 0.05 cm (d) 0.005 cm P = a1/2 b2 c3 d –4. If the relative errors in the measurement of
112. Relative density of a metal may be found with the help of a, b, c and d respectively, are 2%, 1%, 3% and 5%, then the
spring balance. In air the spring balance reads (5.00 ± 0.05) relative error in P will be :
N and in water it reads (4.00 ± 0.05) N. Then, the relative (a) 8% (b) 12% (c) 32% (d) 25%
123. The pressure on a square plate is measured by measuring
density along with the maximum permissible percentage error the force on the plate and length of the sides of the plate by
would be F
(a) (5.00 ± 0.05) (b) (5.00 ± 11%) using the formula P = . If the maximum errors in the
l2
(c) (5.00 ± 0.10) (d) (5.00 ± 6%) measurement of force and length are 6% and 3% respectively,
113. The refractive index of water measured by the relation then the maximum error in the measurement of pressure is
real depth (a) 1% (b) 2% (c) 12% (d) 10%
m= is found to have values of 1.34, 1.38, 124. An atomic clock has an accuracy of 1 part is 1010. If two
apparent depth such clocks are operated with precision, then after running
1.32 and 1.36; the mean value of refractive index with for 2500 years these will record a difference of nearly.
percentage error is (a) 1 sec (b) 8 sec (c) 5 sec (d) 10 sec
(a) 1.35 ± 1.48 % (b) 1.35 ± 0 % 125. Resistance of a given wire is obtained by measuring the
(c) 1.36 ± 6 % (d) 1.36 ± 0 % current flowing in it and the voltage difference applied across
it. If the percentage errors in the measurement of the current
114. A quantity is represented by X = Ma Lb Tc. The % error
and the voltage difference are 3% each, then error in the
in measurement of M, L and T are a%, b% and g% value of resistance of the wire is
respectively. The % error in X would be (a) 6% (b) zero (c) 1% (d) 3%
(a) (aa + bb + gc)% (b) (aa – bb + gc)% 126. Which of the following is most accurate?
(c) (aa – bb – gc) × 100% (d) None of these (a) A screw gauge of least count 0.001 mm
115. In an experiment four quantities a, b, c and d are measured (b) A screw gauge having pitch 1 mm and 50 divisions on
with percentage error 1%, 2%, 3% and 4% respectively. circular scale
EBD_7418
14 PHYSICS

(c) A vernier callipers of least count 0.01 mm 129. Diameter of a steel ball is measured using a Vernier callipers
(d) Vernier callipers having 20 divisions on the sliding scale which has divisions of 0.1 cm on its main scale (MS) and 10
(vernier scale) coinciding 19 divisions on the main divisions of its vernier scale (VS) match 9 divisions on the
millimetre scale main scale. Three such measurements for a ball are given
127. A thin copper wire of length l metre increases in length by below:
2% when heated through 10°C. What is the percentage
S.No. MS(cm) VS divisions
increase in area when a square copper sheet of length
l metre is heated through 10°C? 1. 0.5 8
(a) 4% (b) 8% 2. 0.5 4
(c) 16% (d) None of these 3. 0.5 6
128. A physical quantity z is calulated using the formula
If the zero error is – 0.03 cm, then mean corrected diameter is
1 2 1/ 3 (a) 0.52 cm (b) 0.59 cm (c) 0.56 cm (d) 0.53 cm
z= xy / z , where x, y and z are experimentally
10 130. ln a simple pendulum experiment, the maximum percentage
measured quantities. If the fractional error in the error in the measurement of length is 2% and that in the
measurement of x, y and z are 2 %, 1% and 3% respectively, observation of the time-period is 3%. Then the maximum
then the fractional error in z will be percentage error in determination of the acceleration due
(a) 0.5% (b) 5% (c) 6% (d) 7% to gravity g is
(a) 5% (b) 6% (c) 1% (d) 8%

Exercise 2 : Exemplar & Past Year MCQs


NCERT Exemplar MCQs 8. You measure two quantities as A = 1.0 m ± 0.2 m, B = 2.0 m ±
1. The number of significant figures in 0.06900 is 0.2 m. We should report correct value for AB as
(a) 5 (b) 4 (c) 2 (d) 3 (a) 1.4 m ± 0.4 m (b) 1.41 m ± 0.15 m
2. The sum of the numbers 436.32, 227.2 and 0.301 in appropriate (c) 1.4 m ± 0.3 m (d) 1.4 m ± 0.2 m
significant figures is 9. Which of the following measurement is most precise?
(a) 663.821 (b) 664 (c) 663.8 (d) 663.82 (a) 5.00 mm (b) 5.00 cm
(c) 5.00 m (d) 5.00 km
3. The mass and volume of a body are 4.237 g and 2.5 cm 3,
10. The mean length of an object is 5 cm. Which is the following
respectively. The density of the material of the body in measurement is most accurate?
correct significant figures is (a) 4.9 cm (b) 4.805 cm
(a) 1.6048 g cm–3 (b) 1.69 g cm–3 (c) 5.25 cm (d) 5.4 cm
(c) 1.7 g cm –3 (d) 1.695 g cm–3 11. Young's modulus of steel is 1.9 × 1011 N/m2. When expressed
4. The numbers 2.745 and 2.735 on rounding off to 3 significant in CGS units of dyne/cm2, it will be equal to (1N = 105 dyne,
figures will give 1 m2 = 104 cm2)
(a) 2.75 and 2.74 (b) 2.74 and 2.73 (a) 1.9 × 1010 (b) 1.9 × 1011 (c) 1.9 × 1012 (d) 1.9 × 1013
(c) 2.75 and 2.73 (d) 2.74 and 2.74 12. If momentum (p), area (A) and time (T) are taken to be
5. The length and breadth of a rectangular sheet are 16.2 ± 0.1 fundamental quantities, then energy has the dimensional
cm and 10.1 ± 0.1 cm, respectively. The area of the sheet in formula
appropriate significant figures and error is (a) [pA–1T1] (b) [p2AT] (c) [pA–1/2T] (d) [pA1/2T–1]
(a) 164 ± 3 cm2 (b) 163.62 ± 2.6 cm2
(c) 163.6 ± 2.6 cm 2 (d) 163.62 ± 3 cm2 Past Year MCQs
6. Which of the following pairs of physical quantities does 13. If force (F), velocity (V) and time (T) are taken as fundamental
not have same dimensional forrmula? units, then the dimensions of mass are : [AIPMT 2014, S]
(a) Work and torque (a) [F V T– 1] (b) [F V T– 2]
(b) Angular momentum and Planck's constant –
(c) [F V T ] 1 – 1 (d) [F V– 1 T]
(c) Tension and surface tension
æ1ö
(d) Impulse and linear momentum 14. The dimensions of ç ÷ e0 E 2 (e0: permittivity of free space,
7. Measure of two quantities along with the precision of è 2ø
E: electric field) are [AIIMS 2014, A]
respective measuring instrument is –1 2 –2
A = 2.5 ms–1 ± 0.5 ms–1, (a) [MLT ] (b) [ML T ]
B = 0.10 s ± 0.01 s. The value of AB will be (c) [ML–1T–2] (d) [ML2T–1]
(a) (0.25 ± 0.08) m (b) (0.25 ± 0.5) m 15. A student measured the length of a rod and wrote it as
(c) (0.25 ± 0.05) m (d) (0.25 ± 0.135) m 3.50 cm. Which instrument did he use to measure it?
[JEE Main 2014, C]
UNITS AND MEASUREMENTS 15

(a) A screw gauge having 50 divisions in the circular scale e2 h hc


and pitch as 1 mm. (a) u= (b) u=
2
a0 e a0
(b) A meter scale.
2
(c) A vernier calliper where the 10 divisions in vernier scale e c e2 a 0
(c) u= (d) u =
matches with 9 divisions in main scale and main scale ha 0 hc
has 10 division in 1 cm. 24. Assertion : The number of significant figures depends on
(d) A screw gauge having 100 divisions in the circular scale the least count of measuring instrument.
and pitch as 1 mm. Reason : Significant figures define the accuracy of
16. Thecurrent voltage relation of diode is given by I = (e1000V/T –1) measuring instrument. [AIIMS 2016, C]
mA, where the applied voltage V is in volt and temperature T (a) Assertion is correct, reason is correct; reason is a
is in kelvin. If a student makes an error measuring ± 0.01 V correct explanation for assertion.
while measuring the current of 5 mA at 300 K, what will be (b) Assertion is correct, reason is correct; reason is not a
the error in the value of current in mA? correct explanation for assertion
[JEE Main 2014, S] (c) Assertion is correct, reason is incorrect
(a) 0.05 mA (b) 0.2 mA (c) 0.02 mA (d) 0.5 mA (d) Assertion is incorrect, reason is correct.
17. In a vernier callipers, n divisions of its main scale match with 25. A screw gauge with a pitch of 0.5 mm and a circular scale
(n + 1) divisions on its vernier scale. Each division of the with 50 divisions is used to measure the thickness of a thin
main scale is a units. Using the vernier principle, calculate sheet of Aluminium. Before starting the measurement, it is
its least count. [BITSAT 2014, A] found that when the two jaws of the screw gauge are brought
a a a a in contact, the 45th division coincides with the main scale
(a) (b) (c) (d) line and the zero of the main scale is barely visible. What is
n +4 n +2 n+3 n +1
the thickness of the sheet if the main scale reading is 0.5 mm
18. If energy (E), velocity (V) and time (T) are chosen as the
and the 25th division coincides with the main scale line?
fundamental quantities, the dimensional formula of surface [JEE Main 2016, A, BN]
tension will be : [AIPMT 2015, S] (a) 0.70 mm (b) 0.50 mm (c) 0.75 mm (d) 0.80 mm
(a) [EV T ] –1 –2 (b) [EV T–2]
–2
26. A student measures the time period of 100 oscillations of a
(c) [E–2V–1T–3] (d) [EV–2T–1] simple pendulum four times. The data set is 90 s, 91 s, 95 s, and
19. If dimensions of critical velocity ucof a liquid flowing through 92 s. If the minimum division in the measuring clock is
a tube are expressed as [hxryrx], where h, r and r are the 1 s, then the reported mean time should be: [JEE Main 2016, A]
coefficient of viscosity of liquid, density of liquid and radius (a) 92 ± 1.8 s (b) 92 ± 3s
of the tube respectively, then the values of x, y and z are (c) 92 ± 2 s (d) 92 ± 5.0 s
given by : [AIPMT 2015 RS, A] 27. The frequency of vibration of string is given by
Here p is number of segments in the ½
(a) –1, –1, 1 (b) –1, –1, –1 p éFù
v =
(c) 1, 1, 1 (d) 1, –1, –1
string and l is the length. 2l êë m úû
The dimensional formula for m will be
20. The least count of a stop watch is 0.2 second. The time of 20
oscillations of a pendulum is measured to be 25 second. The [BITSAT 2016, A]
percentage error in the measurement of time will be (a) [M0LT–1] (b) [ML0T–1]
[AIIMS 2015, S] (c) [ML–1T0] (d) [M0L0T0]
(a) 8% (b) 1.8% (c) 0.8% (d) 0.1% 28. A physical quantity of the dimensions of length that can be
L e2
21. The period of oscillation of a simple pendulum is T = 2p formed out of c, G and is [c is velocity of light, G is
g 4pe0
Measured value of L is 20.0 cm known to 1 mm accuracy and universal constant of gravitation and e is charge]
time for 100 oscillations of the pendulum is found to be 90 s [NEET 2017, S]
using a wrist watch of 1s resolution. The accuracy in the 1/ 2 1/ 2
determination of g is : [JEE Main 2015, S] 2
é e2 ù 1 é e2 ù
(a) c êG ú (b) ê ú
(a) 1% (b) 5% (c) 2% (d) 3% êë 4pe 0 úû c 2 êë G4pe0 úû
22. If e is the charge, V the potential difference, T the temperature,
1/ 2
eV 1 e2 1 é e2 ù
then the units of are the same as that of [AIIMS 2016, A] (c) G (d) ê G ú
T c 4pe0 c 2 êë 4pe 0 úû
(a) Planck’s constant (b) Stefan’s constant
29. A force F is applied onto a square plate of side L. If the
(c) Boltzmann constant (d) Gravitational constant
percentage error in determining L is 2% and that in F is 4%,
23. If the capacitance of a nanocapacitor is measured in terms
the permissible percentage error in determining the pressure
of a unit ‘u’ made by combining the electric charge ‘e’, Bohr
is [AIIMS 2017, A]
radius ‘a0’, Planck’s constant ‘h’ and speed of light ‘c’ then
[AIIMS 2016, S] (a) 2% (b) 4% (c) 6% (d) 8%
EBD_7418
16 PHYSICS

30. Assertion: In the measurement of physical quantities direct 33. A student measured the diameter of a small steel ball using
and indirect methods are used. [AIIMS 2017, C] a screw gauge of least count 0.001 cm. The main scale
Reason : The accuracy and precision of measuring reading is 5 mm and zero of circular scale division coincides
instruments along with errors in measurements should be with 25 divisions above the reference level. If screw gauge
taken into account, while expressing the result. has a zero error of –0.004 cm, the correct diameter of the ball
(a) Assertion is correct, reason is correct; reason is a is [NEET 2018, A, BN]
correct explanation for assertion. (a) 0.521 cm (b) 0.525 cm (c) 0.529 cm (d) 0.053 cm
(b) Assertion is correct, reason is correct; reason is not a 34. The velocity of water waves (v) may depend on their
correct explanation for assertion wavelength l, the density of water r and the acceleration
(c) Assertion is correct, reason is incorrect due to gravity, g. The method of dimensions gives the relation
between these quantities is [AIIMS 2018, A]
(d) Assertion is incorrect, reason is correct.
(a) v (b) v 2 µ g l
31. The following observations were taken for determining
surface tensiton T of water by capillary method : (c) v 2 µ g l 2 (d) v2 µ g -1 l 2
Diameter of capilary, D = 1.25 × 10–2 m 35. The density of a material in the shape of a cube is determined
Rise of water, h = 1.45 × 10–2 m by measuring three sides of the cube and its mass. If the
Using g = 9.80 m/s2 and the simplified relation relative errors in measuring the mass and length are
rhg respectively 1.5% and 1%, the maximum error in determining
T= ´ 103 N/m, the possible error in surface tension is the density is: [JEE Main 2018, A]
2
closest to : [JEE Main 2017, S] (a) 2.5% (b) 3.5% (c) 4.5% (d) 6%
(a) 2. 4 % (b) 10 % (c) 0.15% (d) 1.5% 36. If C, the velocity of light, g the acceleration due to gravity
32. In the formula X = 3 YZ2, X and Z have dimensions of capaci- and P the atmospheric pressure be the fundamental
tance and magnetic induction respectively. The dimensions quantities in MKS system, then the dimensions of length
of Y in MKS system are : [BITSAT 2017, A] will be same as that of [BITSAT 2018, A]
–3 –2 –2 –4 –2
C C C2
(a) [M L T A ] (b) [ML ] (a) (b) (c) PCg (d)
g P g
(c) [M–3L–2A4T8] (d) [M–3L2A4T4]

Exercise 3 : Try If You Can


DZ
ML2 5. If Z = A3 × A3/2, then relative error = –––––
1. Which of the following units denotes the dimensions , Z
Q2 9/2 1/3
where Q denotes the electric charge : [weber (Wb); henry (H)] DA3/2 æ DA ö 9 æ DA ö æ DA ö
(a) (b) ç ÷ (c) ç ÷ (d) ç ÷
A è A ø 2è A ø è A ø
Wb H 6. Given that (a/pb) = az/KB q where p is pressure, z is distance,
(a) Wb (b) 2 (c) H (d)
m m2 KB is Boltzmann constant and q is temperature, the
1.53 ´ 0.9995 dimensions of b are
2. Find the value of with due regard for significant
1.592 (a) [L0M 0 T 0] (b) [L1M –1T 2]
figures – 2
(c) [L M T ]0 0 (d) [L–1 M 1T –2]
(a) 0.961 (b) 0.123 (c) 0.921 (d) 0.913 7. If x and R stands for distance then which of the following is
3. The volume of a liquid of density r and viscosity h Rdx
flowing in time t through a capillary tube of length l and dimensionally same as ò 2 ?
x
radius R, with a pressure difference P, across its ends is R R2
proportional to : (a) Rx2 (b) 2xR (c) (d) -
x x
(a) P2R2t/hl2 (b) PR4/hlt (c) PR4t/hl (d) hR4/lt 8. The moment of inertia of a body rotating about a given axis
is 6.0 kg m2 in the SI system. What is the value of the moment
b k .q.t 3 of inertia in a system of units in which the unit of length is 5
4. A formula is given as P = 1+
a m.a cm and the unit of mass is 10 g?
where P = pressure; k = Boltzmann’s constant; (a) 2.4 × 103 (b) 2.4 × 105 (c) 6.0 × 103 (d) 6.0 × 105
9. The momentum of an electron in an orbit is h/l where h is a
q = temperature; t = time; ‘a’ and ‘b’ are constants.
constant and l is wavelength associated with it. The nuclear
Dimensional formula of ‘b’ is same as magneton of electron of charge e and mass me is given as µn
(a) Force (b) Linear momentum eh
= . The dimensions of µn are (A ® current)
(c) Angular momentum (d) Torque 3672 pme
(a) [ML2A] (b) [ML3A] (c) [L2A] (d) [ML2]
UNITS AND MEASUREMENTS 17

10. Intensity observed in an interference pattern is I = I0 sin2q. 16. In an experiment to determine the inertial mass of an object
At q = 30° intensity I = 5 ± 0.0020 W/m2. Find percentage using Newton’s second law, following graph is obtained
error in angle if I0 = 20 W/m2. between net force on the object and the acceleration
4 2 produced in it. The mass of the object within error limits is
(a) 3 ´ 10 -2 % (b) 3 ´ 10 -2 % a(m/s2)
p p
1 3 (a) 1.0 kg
(c) 3 ´ 10 -2 % (d) 3 ´ 10 -2 % 3
p p
11. The frequency (f) of a wire oscillating with a length l, in p (b) 1 kg 2

p T 1
loops, under a tension T is given by f = where µ = (c) (1.0 ± 0.1) kg
2l m
F(N)
linear density of the wire. If the error made in determining (d) (1.0 ± 0.2) kg 1 2 3
length, tension and linear density be 1%, –2% and 4%, then 17. The relative uncertainty in the period of a satellite orbiting
find the percentage error in the calculated frequency. around the earth is 10–2. If the relative uncertainty in the
(a) – 4% (b) – 2% (c) –1% (d) –5% radius of the orbit is negligible, the relative uncertainty in
V the mass of the earth is
12. The resistance of a metal is given by R = , where V is (a) 3× 10–2 (b) 10–2 (c) 2 × 10–2 (d) 6 × 10–2
I
potential difference and I is the current. In a circuit the 18. The percentage errors in quantities P, Q, R and S are 0.5%,
potential difference across resistance is V = (8 ± 0.5) V and 1%, 3% and 1.5% respectively in the measurement of a
current in resistance, I = (2 ± 0.2) A. What is the value of P3 Q 2
resistance with its percentage error ? physical quantity A = .
RS
(a) 4W ± 16.25% (b) (4 ± 0.7) W
The maximum percentage error in the value of A will be
(c) 4W ± 0.7% (d) 4W ± 7%
13. A physical quantity Q is related to four observables x, y, z (a) 8.5% (b) 6.0% (c) 7.5% (d) 6.5%
19. The accompanying diagram represents a screw gauge.
x 2 / 5 z3 The circular scale is divided into 50 divisions and the
and t by the relation Q = . The percentage errors of linear scale is divided into millimeters. If the screw
y t
advances by 1 mm when the circular scale makes 2
measurement in x, y, z and t are 2.5%, 2%, 0.5% and 1% complete revolutions, the least count of the instrument
respectively. The percentage error in Q will be and the reding of the instrument in figure are respectively.
(a) 5% (b) 4.5% (c) 8% (d) 7.75%
14. The space shuttle astronauts use a massing chair to measure
their mass. The chair is attached to a spring and is free to 40
oscillate back and forth. The frequency of the oscillation is 35
30
measured and that is used to calculate the total mass m 25
0 1 2 3
attached to the spring. If the spring constant of the spring k 20
is measured in kg/s2 and the chair ’s frequency f is
0.50 s–1 for a 62 kg astronaut, what is the chair’s frequency (a) 0.01 mm and 3.82 mm (b) 0.02 mm and 3.70 mm
for a 75kg astronaut ? The chair itself has a mass of 10.0kg. (c) 0.11 mm and 4.57 mm (d) 1.0 mm and 5.37 mm
(a) 0.46 s–1 (b) 0.12 s–1 (c) 0.78 s–1 (d) 0.92 s–1 20. The characteristic distance at which quantum gravitational
15. A metal sample carrying a current along X-axis with density Jx effects are significant, the Planck length, can be determined
is subjected to a magnetic field Bz (along z-axis). The electric from a suitable combination of the fundamental physical
field Ey developed along Y-axis is directly proportional to Jx constants G, h and c . Which of the following correctly
gives the Planck length?
as well as Bz. The constant of proportionality has SI unit. 1
m2 m3 m2 As æ Gh ö 2 1
(a) (b) (c) (d) (a) G2hc (b) ç 3 ÷ (c) (d) Gh2c3
A As As m3 èc ø G 2 h 2c
EBD_7418
18 PHYSICS

ANSWER KEYS
Exercise 1 : Topic-wise MCQs
1 (b) 14 (d) 27 (b) 40 (b) 53 (a) 66 (b) 79 (d) 92 (d) 105 (d) 118 (b)
2 (d) 15 (c) 28 (b) 41 (d) 54 (d) 67 (c) 80 (c) 93 (a) 106 (b) 119 (c)
3 (b) 16 (a) 29 (c) 42 (a) 55 (b) 68 (b) 81 (a) 94 (a) 107 (c) 120 (d)
4 (c) 17 (b) 30 (d) 43 (d) 56 (a) 69 (d) 82 (c) 95 (a) 108 (a) 121 (a)
5 (a) 18 (c) 31 (a) 44 (c) 57 (d) 70 (a) 83 (d) 96 (c) 109 (b) 122 (c)
6 (c) 19 (b) 32 (d) 45 (d) 58 (b) 71 (a) 84 (d) 97 (c) 110 (a) 123 (c)
7 (d) 20 (b) 33 (d) 46 (b) 59 (d) 72 (c) 85 (a) 98 (d) 111 (b) 124 (b)
8 (d) 21 (c) 34 (b) 47 (a) 60 (a) 73 (b) 86 (d) 99 (a) 112 (b) 125 (a)
9 (c) 22 (a) 35 (b) 48 (c) 61 (d) 74 (b) 87 (b) 100 (b) 113 (a) 126 (a)
10 (b) 23 (b) 36 (d) 49 (a) 62 (c) 75 (b) 88 (d) 101 (b) 114 (a) 127 (a)
11 (c) 24 (a) 37 (d) 50 (b) 63 (a) 76 (d) 89 (d) 102 (c) 115 (d) 128 (b)
12 (d) 25 (d) 38 (d) 51 (d) 64 (b) 77 (d) 90 (c) 103 (c) 116 (c) 129 (b)
13 (a) 26 (b) 39 (b) 52 (b) 65 (c) 78 (a) 91 (c) 104 (c) 117 (c) 130 (d)
Exercise 2 : Exemplar & Past Year MCQs
1 (b) 5 (a) 9 (a) 13 (d) 17 (d) 21 (d) 25 (d) 29 (d) 33 (c)
2 (b) 6 (c) 10 (a) 14 (c) 18 (b) 22 (c) 26 (c) 30 (a) 34 (b)
3 (c) 7 (a) 11 (c) 15 (c) 19 (d) 23 (d) 27 (c) 31 (d) 35 (c)
4 (d) 8 (d) 12 (d) 16 (b) 20 (c) 24 (b) 28 (d) 32 (c) 36 (d)
Exercise 3 : Try If You Can
1 (c) 3 (c) 5 (c) 7 (c) 9 (c) 11 (a) 13 (a) 15 (b) 17 (c) 19 (a)
2 (a) 4 (b) 6 (c) 8 (b) 10 (a) 12 (a) 14 (a) 16 (c) 18 (d) 20 (b)
3
Chapter
MOTION IN A STRAIGHT LINE

Trend
Analysis of NEET and AIIMS (Year 2010-2018)
5

4
Number of Questions

3
AIPMT/NEET
2 AIIMS

0
2010 2011 2012 2013 2014 2015 2016 2017 2018
Year

Trend
Analysis of JEE Main and BITSAT (Year 2010-2018)
5

4
Number of Questions

3
JEE Main

2 BITSAT

0
2010 2011 2012 2013 2014 2015 2016 2017 2018
Year

Chapter Utility Score (CUS)


Exam Weightage Important Concepts Difficulty Level CUS (Out of 10)

NEET 4 Acceleration, Kinematic


AIIMS 5 equations & Motion under 3/5 5.5/10
JEE Main 2 gravity
BITSAT 2
EBD_7418
20 PHYSICS
MOTION IN A STRAIGHT LINE 21
EBD_7418
22 PHYSICS

Exercise 1 : Topic-wise MCQs

(c) Both P and Q move with uniform speeds but the speed
Topic 1: Distance, Displacement & Uniform Motion
of P is more than the speed of Q
1. A body is moving along a straight line path with constant (d) Both P and Q move with uniform speeds but the speed
velocity. At an instant of time the distance travelled by it of Q is more than the speed of P.
is S and its displacement is D, then 11. The distance travelled by a body is directly proportional to
(a) D < S (b) D > S (c) D = S (d) D £ S the time taken. Its speed
2. The location of a particle has changed. What can we say (a) increases (b) decreases
about the displacement and the distance covered by the (c) becomes zero (d) remains constant
particle? 12. The slope of velocity-time graph for motion with uniform
(a) Neither can be zero (b) One may be zero velocity is equal to
(c) Both may be zero (d) One is +ve, other is –ve (a) final velocity (b) initial velocity
3. The displacement of a body is zero. The distance covered (c) zero (d) none of these
(a) is zero 13. The ratio of the numerical values of the average velocity
(b) is not zero and average speed of a body is
(c) may or may not be zero (a) unity (b) unity or less
(d) depends upon the acceleration (c) unity or more (d) less than unity
4. In 1.0 s, a particle goes from point A to point B, moving in a 14. The slope of the tangent drawn on position-time graph at
semicircle of radius 1.0 m (see Figure). The magnitude of the any instant is equal to the instantaneous
average velocity is A (a) acceleration (b) force
(c) velocity (d) momentum
(a) 3.14 m/s 15. The displacement-time graphs of two particles A and B are
1.0m
straight lines making angles of 30º and 60º respectively
(b) 2.0 m/s with the time axis. If the velocity of A is vA and that of B is
vB, the value of vA/vB is
(c) 1.0 m/s
(a) 1/2 (b) 1 / 3 (c) 3 (d) 1/3
(d) Zero 16. What is the rate of change of velocity of an object in uniform
B motion ?
5. The numerical ratio of displacement to distance for a moving (a) Always equal to zero
object is (b) Always less than one
(a) always less than 1 (b) always equal to 1 (c) Always greater than one
(c) always more than 1 (d) equal to or less than 1 (d) Either less than or equal to one.
6. Which of the following can be zero, when a particle is in 17. Choose the wrong statement from the following.
motion for some time? (a) The motion of an object along a straight line is a
(a) Distance (b) Displacement rectilinear motion.
(c) Speed (d) None of these (b) The speed in general is less than the magnitude of the
7. If distance covered by a particle is zero, what can you say velocity.
about its displacement? (c) The slope of the displacement-time graph gives the
(a) It may or may not be zero (b) It cannot be zero velocity of the body.
(c) It is negative (d) It must be zero (d) The area under the velocity-time graph gives the
8. Which of the following is a one dimensional motion ? displacement of the body.
(a) Motion of snake 18. An athlete completes one round of a circular track of radius
(b) Motion of air particle R in 40 sec. What will be his displacement at the end of
(c) Motion of satellite 3 min. 20 sec ?
(d) Motion of train running on a straight track (a) Zero (b) 2 R (c) 2 pR (d) 7 pR
9. A man of height h walks in a straight path towards a lamp 19. Which of the following graph cannot possibly represent
post of height H with velocity v. Then velocity of the edge one dimensional motion of a particle?
of the shadow on the ground will be x x
hv hv H+h (H – h )
(a) (b) (c) (d)
H+h H–h Hv Hh (a) t (b)
10. The fig given shows the x P t
Q
time-displacement curve of
two particles P and Q. speed
Which of the following O t
statement is correct? (c) (d) All of these
(a) Both P and Q move with uniform equal speed t
(b) P is accelerated Q is retarded
MOTION IN A STRAIGHT LINE 23

20. Consider the following statements and select the incorrect (a) zero (b) 2 km h–1
statements. 1
I. The magnitude of instantaneous velocity of a particle (c) 10 km s–1 (d) km s -1
2
is equal to its instantaneous speed. 30. A point traversed half of the distance with a velocity v0.
II. The magnitude of the average velocity in an interval The half of remaining part of the distance was covered with
is equal to its average speed in that interval. velocity v1 & second half of remaining part by v2 velocity.
III. It is possible to have a situation in which the speed of
The mean velocity of the point, averaged over the whole
the particle is never zero but the average speed in an
interval is zero. time of motion is
IV. It is possible to have a situation in which the speed of v 0 + v1 + v 2 2 v 0 + v1 + v 2
(a) (b)
particle is zero but the average speed is not zero. 3 3
(a) II, III and IV (b) I and II v 0 + .2 v1 + 2 v 2 2 v 0 (v1 + v 2 )
(c) II and III (d) IV only (c) (d)
3 (2v 0 + v1 + v 2 )
21. The total distance travelled by the body in the given time
is equal to Topic 2: Non-Uniform Motion
(a) the area which v– t graph encloses with displacement axis
(b) the area which x – t graph encloses with time axis 31. What determines the nature of the path followed by the
(c) the area which v – t graph encloses with time axis particle?
(d) the area which a – t graph encloses with axis (a) Speed (b) Velocity
22. Choose the correct equation to determine distance in a (c) Acceleration (d) Both (b) and (c)
straight line for a body with uniform motion. 32. The graph between displacement and time for a particle
v moving with uniform acceleration is a/an
(a) s = (b) s = v2t (a) straight line with a positive slope
t
1 2 (b) parabola
(c) s = ut + at (d) s = v × t2 (c) ellipse
2 (d) straight line parallel to time axis
23. A particle moves 2m east then 4m north then 5 m west. 33. The acceleration of a moving body can be found from
The distance is (a) area under velocity - time graph
(a) 11 m (b) 10 m (c) –11 m (d) 5 m (b) area under distance -time graph
24. A body moves in straight line with velocity v1 for 1/3rd (c) slope of the velocity- time graph
time and for remaining time with v2. Find average (d) slope of distance-time graph
velocity. 34. Acceleration of a particle changes when
v1 2v 2 v1 v 2 2v1 v 2 2v 2 (a) direction of velocity changes
(a) + (b) + (c) + (d) v1 + (b) magnitude of velocity changes
3 3 3 3 3 3 3 (c) speed changes
25. Select the incorrect statements from the following. (d) Both (a) and (b)
I. Average velocity is path length divided by time interval. 35. The area under acceleration time graph gives
II. In general, speed is greater than the magnitude of (a) distance travelled (b) change in acceleration
the velocity. (c) force acting (d) change in velocity
III. A particle moving in a given direction with a non- 36. The incorrect statement(s) from the following is/are
zero velocity can have zero speed. I. A body having zero velocity will not necessarily
IV. The magnitude of average velocity is equal to the have zero acceleration.
average speed. II. A body having zero velocity will necessarily have
(a) II and III (b) I and IV zero acceleration.
(c) I, III and IV (d) I, II, III and IV III. A body having uniform speed can have only
26. A particle moves in straight line with velocity 6 m/s and uniform acceleration.
3 m/s for time intervals which are in ratio 1: 2 . Find IV. A body having non-uniform velocity will have zero
average velocity. acceleration.
(a) 2 m/s (b) 3 m/s (c) 4 m/s (d) 5 m/s
27. A particle moves from (2,3) m to (4,1) m. The magnitude (a) II, III and IV (b) I and II
(c) II and III (d) IV only
of displacement is 37. Which of the following is the correct expression of
(a) 2 m (b) 2 3 m (c) 2 2 m (d) 3 2 m instantaneous acceleration?
28. Which of the following is not possible for a body in
Dv dv
uniform motion? (a) a = 2
(b) a =
( Dt) dt
Displacement
Displacement

2
d2 v æ Dv ö
(c) a = 2 (d) a = ç ÷
dt è Dt ø
(a) (b) 38. If a body travels with constant acceleration, which of the
following quantities remains constant ?
Time Time (a) Displacement (b) Velocity
(c) Both (a) & (b) (d) None of these (c) Time (d) None of these
29. A man leaves his house for a cycle ride. He comes back to 39. Velocity time curve for a body projected vertically upwards is
his house after half-an-hour after covering a distance of (a) parabola (b) ellipse
one km. What is his average velocity for the ride ? (c) hyperbola (d) straight line
EBD_7418
24 PHYSICS

40. A bus starts moving with acceleration 2 m/s2. A cyclist 96 48. The velocity-time graph of a body is shown in fig. The ratio
m behind the bus starts simultaneously towards the bus at of average acceleration during the intervals OA and AB is
20 m/s. After what time will he be able to overtake the bus? (a) 1 v (m/s)
(a) 4 sec (b) 8 sec (c) 18 sec (d) 16 sec
41. The velocity time graph of the motion of the body is as 1 D C
40
shown below (b)
2
A B 1
u
(c)
v(m/s)

3 30º 60º
C O E
D E
o (d) 3 A B t (s)
t1 t2 t3 t (s)
49. A bullet fired into a wooden block loses half of its velocity
The total distance travelled by the body during the
after penetrating 40 cm. It comes to rest after penetrating a
motion is equal to ____.
further distance of
1 1 22 40 20 22
(a) (AD + BE) × OC (b) (OA + BC) × OC (a) cm (b) cm (c) cm (d) cm
2 2 3 3 3 5
1 1 50. A body covers 26, 28, 30, 32 meters in 10th, 11th, 12th and
(c) (OC + AB) × AD (d) (OA + AB) × BC
2 2 13th seconds respectively. The body starts
42. Stopping distance of a moving vehicle is directly proportional (a) from rest and moves with uniform velocity
to (b) from rest and moves with uniform acceleration
(a) square of the initial velocity (c) with an initial velocity and moves with uniform
(b) square of the initial acceleration acceleration
(c) the initial velocity (d) with an initial velocity and moves with uniform velocity
(d) the initial acceleration 51. The displacement x of a particle at the instant when its
43. Which of the following graphs gives the equation velocity is v is given by v = 3x + 16 . Its acceleration and
1 2 initial velocity are
x = vot + at (a) 1.5 units, 4 units (b) 3 units, 4 units
2
(c) 16 units, 1.6 units (d) 16 units, 3 units
52. The distance time graph of a particle at time t makes angles
v0 45° with the time axis. After one second, it makes angle 60°
x with the time axis. What is the acceleration of the particle?
(a) (b) v (a) 3 - 1 (b) 3 + 1 (c) 3 (d) 1
t 53. A particle experiences constant acceleration for 20 seconds
t after starting from rest. If it travels a distance s1 in the first
10 seconds and distance s2 in the next 10 seconds, then
v0 (a) s2 = s1 (b) s2 = 2 s1 (c) s2 = 3 s1 (d) s2 = 4 s1
(c) v (d) None of these 54. The distance travelled by a particle starting from rest and
4
t moving with an acceleration ms-2 , in the third second is:
44. The displacement of a particle is represented by the 3
following equation: S = 3t 3 + 7t 2 + 5t + 8 where 5 is in 10 19
(a) 6 m (b) 4 m (c) m (d) m
meter and t in second. The acceleration of the particle at 3 3
t = 15 is 55. If a car at rest accelerates uniformly to a speed of 144 km/h
(a) 14 m/s2 (b) 18 m/s2 (c) 32 m/s2 (d) zero in 20 s, it covers a distance of
45. If a train travelling at 20 m/s is to be brought to rest in (a) 2880 m (b) 1440 m (c) 400 m (d) 20 m
a distance of 200 m, then its retardation should be 56. The displacement x of a particle along a straight line at time
(a) 1 m/s2 (b) 2 m/s2 a1t a 2 2
2
(c) 10 m/s (d) 20 m/s2 t is given by : x = a0 + + t . The acceleration of the
46. Assertion : A body may be accelerated even when it is 2 3
particle is
moving uniformly.
a2 2a 2 a1 a
Reason : When direction of motion of the body is changing, (a) (b) (c) (d) a0 + 2
the body must have acceleration. 3 3 2 3
(a) Assertion is correct, reason is correct; reason is a 57. The dependence of velocity of a body with time is given by
correct explanation for assertion. the equation v = 20 + 0.1t 2 . The body is in
(b) Assertion is correct, reason is correct; reason is not (a) uniform retardation
a correct explanation for assertion (b) uniform acceleration
(c) Assertion is correct, reason is incorrect (c) non-uniform acceleration
(d) Assertion is incorrect, reason is correct. (d) zero acceleration.
47. A body starts from rest and travels ‘s’ m in 2nd second, 58. A car accelerates from rest at a constant rate a for some
then acceleration is time, after which it decelerates at a constant rate b and
2 3 comes to rest. If the total time elapsed is t, then the maximum
(a) 2s m/s2 (b) 3s m/s2 (c) s m/s2 (d) s m/s2 velocity acquired by the car is
3 2
MOTION IN A STRAIGHT LINE 25

1 -2
æ a 2 + b2 ö
t
æ a 2 - b2 ö
ç ÷ (a + b)t abt (a) ms towards north
(a) ç ab ÷ (b) ç ab ÷ (c) (d) 2
è ø è ø ab a +b 1
59. The deceleration experienced by a moving motorboat after
(b) ms - 2 towards north - east
2
dv 1
its engine is cut off, is given by = -KV3 where K is (c) ms - 2 towards north - west
dt 2
constant. If V0 is the magnitude of the velocity at cut-off, (d) zero
the magnitude of the velocity at a time t after the cut-off is 66. It is given that t = px2 + qx, where x is displacement and t is
V0 time. The acceleration of particle at origin is
(a) (b) V0e–Kt 2p 2q 2p 2q
2
(2V0 Kt + 1) (a) - (b) - (c) (d)
3 3
q 3
p q p3
(c) V0/2 (d) V0
60. Assertion : A particle starting from rest and moving with 67. An object, moving with a speed of 6.25 m/s, is decelerated
uniform acceleration travels' a length of x and 3x in first two dv
at a rate given by: = -2.5 v where v is the instantaneous
and next two-seconds. dt
Reason : Displacement is directly proportional to velocity. speed. The time taken by the object, to come to rest, would be
(a) Assertion is correct, reason is correct; reason is a (a) 2 s (b) 4 s (c) 8 s (d) 1 s
correct explanation for assertion. 68. The position of a particle along the x-axis at certain times is
(b) Assertion is correct, reason is correct; reason is not
given below
a correct explanation for assertion t(s) 0 1 2 3
(c) Assertion is correct, reason is incorrect
(d) Assertion is incorrect, reason is correct. x(m) –2 0 6 16
61. The displacement of a particle as a function of time is shown Which of the following describes the motion correctly?
in figure. It indicates that (a) uniform acceleration
(b) uniform retardation
Displacement in m

(c) non-uniform acceleration


30 (d) there is not enough data for generalization
69. A bike accelerates from rest at a constant rate 5 m/s2 for
20 some time after which it decelerates at a constant rate 3 m/s2
10 to come to rest. If the total time elapsed is 8 second, the
maximum velocity acquired by the bike is given by
20 30 40 50 10 (a) 5 m/s (b) 10 m/s (c) 12 m/s (d) 15 m/s
Time in sec 70. A metro train starts from rest and in 5 s achieves 108 km/h.
(a) the velocity of the particle is constant throughout After that it moves with constant velocity and comes to
(b) the acceleration of the particle is constant throughout rest after travelling 45 m with uniform retardation. If total
(c) the particle starts with a constant velocity and is distance travelled is 395 m, find total time of travelling.
accelerated (a) 12.2 s (b) 15.3 s (c) 9 s (d) 17.2 s
(d) the motion is retarded and finally the particle stops 71. A car, starting from rest, accelerates at the rate f through a
62. Assertion : The equation of motion can be applied only distance S, then continues at constant speed for time t and
if acceleration is along the direction of velocity and is
constant. f
then decelerates at the rate to come to rest. If the total
Reason : If the acceleration of a body is zero then its 2
motion is known as uniform motion. distance traversed is 15 S , then
(a) Assertion is correct, reason is correct; reason is a 1 1 1 2
correct explanation for assertion. (a) S = ft 2 (b) S = f t (c) S = ft 2 (d) S = ft
(b) Assertion is correct, reason is correct; reason is not 6 4 72
a correct explanation for assertion 72. A particle starting with certain initial velocity and uniform
(c) Assertion is correct, reason is incorrect acceleration covers a distance of 12 m in first 3 seconds
(d) Assertion is incorrect, reason is correct. and a distance of 30 m in next 3 seconds. The initial velocity
63. A particle moves along a straight line OX. At a time t (in of the particle is
second) the distance x (in metre) of the particle from O is (a) 3 ms–1 (b) 2.5 ms–1 (c) 2 ms–1 (d) 1 ms–1
given by x = 40 + 12t – t3. How long would the particle
travel before coming to rest? Topic 3: Ralative Motion in One Dimension
(a) 24 m (b) 40 m (c) 56 m (d) 16 m 73. Two trains, each 40 m long are travelling in opposite
64. A particle moves a distance x in time t according to equation
direction with equal velocity 20 m/s. The time of crossing is
x = (t + 5)–1. The acceleration of particle is proportional to
(a) 1s (b) 2s (c) 3s (d) Zero
(a) (velocity) 3/2 (b) (distance)2
74. The relative velocity VAB or VBA of two bodies A and B
(c) (distance)–2 (d) (velocity)2/3
may be
65. A particle is moving eastwards with a velocity of 5 ms–1. In
I. greater than velocity of body A
10 seconds the velocity changes to 5 ms–1 northwards.
II. greater than velocity of body B
The average acceleration in this time is III. less than the velocity of body A
EBD_7418
26 PHYSICS

IV. less than the velocity of body B 83. A ship A is moving Westwards with a speed of 10 km h–1
(a) I and II only (b) III and IV only and a ship B 100 km South of A, is moving Northwards with
(c) I, II and III only (d) I, II, III and IV a speed of 10 km h–1. The time after which the distance
75. The graph shown below represent between them becomes shortest, is
object B (a) 5 h (b) 5 2 h (c) 10 2 h (d) 0 h
x02 84. A bus is moving with a velocity of 10 ms–1 on a straight
Position

object A
road. A scootorist wishes to overtake the bus in one minute.
x01
If the bus is at a distance of 1.2 km ahead, then the velocity
with which he has to chase the bus is
Time (a) 20 ms–1 (b) 25 ms–1 (c) 60 ms–1 (d) 30 ms–1
(a) A and B are moving with same velocity in opposite
Topic 4: Motion Under Gravity
directions
(b) velocity of B is more than A in same direction 85. A body is thrown vertically upwards. If air resistance is
(c) velocity of A is more than B in same direction to be taken into account, then the time during which the
(d) velocity of A and B is equal in same direction body rises is
(a) equal to the time of fall
76. Two cars A and B approach each other at the same (b) less than the time of fall
speed, then what will be the velocity of A if velocity of (c) greater than the time of fall
B is 8 m/s? (d) twice the time of fall
(a) 16 m/s (b) 8 m/s 86. A body is thrown upwards and reaches half of its maximum
(c) – 8 m/s (d) Can’t be determined. height. At that position
77. A train of 150 m length is going towards north direction at (a) its acceleration is minimum
a speed of 10 ms–1. A parrot flies at a speed of 5 ms–1 (b) its velocity is maximum
towards south direction parallel to the railway track. The (c) its velocity is zero
time taken by the parrot to cross the train is equal to (d) its acceleration is constant
87. Velocity-time curve for a body projected vertically
(a) 12 s (b) 8 s (c) 15 s (d) 10 s upwards is
78. A boat takes 2 hours to travel 8 km and back in still water (a) parabola (b) ellipse
lake. With water velocity of 4 km h–1, the time taken for (c) hyperbola (d) straight line
going upstream of 8 km and coming back is 88. An object accelerated downward under the influence of
(a) 160 minutes (b) 80 minutes force of gravity. The motion of object is said to be
(c) 100 minutes (d) 120 minutes (a) uniform motion
79. A car is moving on a road and rain is falling vertically. (b) free fall
Select the correct answer. (c) non uniformly accelerated motion
(a) The rain will strike the back screen only (d) None of these
(b) The rain will strike the front screen only 89. Free fall of an object (in vacuum) is a case of motion with
(c) The rain will strike both the screens (a) uniform velocity
(d) The rain will not strike any of the screens (b) uniform acceleration
80. If a boat can travel with a speed of v in still water, which of (c) variable acceleration
the following trips will take the least amount of time ? (d) constant momentum
(a) travelling a distance of 2d in still water 90. A ball thrown vertically upwards after reaching a maximum
(b) travelling a distance of 2d across (perpendicular to) height h, returns to the starting point after a time of l0 s. Its
the current in a stream displacement is
(c) travelling a distance d downstream and returning a (a) h (b) 2 h (c) 10 h (d) zero
distance d upstream 91. The ball is projected up from ground with speed 30 m/sec.
(d) travelling a distance d upstream and returning a What is the average velocity for time 0 to 4 sec?
(a) 10 m/sec (b) 20 m/sec
distance d downstream
r (c) 15 m/sec (d) zero
81. An object has velocity v1 relative to the ground. 92. A body is thrown vertically upwards with a velocity u.
r
An observer moving with a constant velocity v0 relative Select the incorrect statements from the following.
r
to the ground measures the velocity of the object to be v 2 I. Both velocity and acceleration are zero at its highest
(relative to the observer). The magnitudes of these point.
velocities are related by II. Velocity is maximum and acceleration is zero at the
(a) v0 £ v1 + v 2 (b) v1 £ v 2 + v0 highest point.
III. Velocity is maximum and acceleration is g downwards
(c) v2 £ v0 + v1 (d) All of these at its highest point.
82. Two trains are each 50 m long moving parallel towards each (a) I and II (b) II and III
other at speeds 10 m/s and 15 m/s respectively. After what (c) I and III (d) I, II and III
time will they pass each other? 93. Column I Column II
(A) Distance travelled (1) zero acceleration
2
(a) 5 sec (b) 4 sec (c) 2 sec (d) 6 sec by a body
3
MOTION IN A STRAIGHT LINE 27

1 (a) TA > TB
(B) Uniform velocity (2) ut + at2 (b) TA = TB
2
(C) Speedometer (3) instantaneous speed (c) TA < TB
u2 (d) their time of flights depend on their masses.
(D) Height of a vertically (4) 102. A ball is released from the top of tower of height h metre. It
2g
thrown body takes T second to reach the ground. What is the position
in (m) from the ground of the ball in T/3 second ?
(a) (A)®(2, 3); (B)®(2); C®(3, 4); (D)®(1, 5)
(b) (A)®(1, 2); (B)®(3); C®(5); (D)®(4) h 7h 8h 17h
(a) (b) (c) (d)
(c) (A)®(1, 5); (B)®(1); C®(3); (D)®(4, 5) 9 9 9 18
103. A ball is dropped vertically from a height d above the ground.
(d) (A)®(2); (B)®(4); C®(1); (D)®(3)
It hits the ground and bounces up vertically to a height d/
94. The equation represented by the graph below is :
2. Neglecting subsequent motion and air resistance, its
1 velocity v varies with the height h above the ground as
(a) y = gt
2
-1
(b) y = gt t(s)
2 O d
(a) h (b)
1 2 y h
(c) y = gt d
2 (m)
-1 2
(d) y = gt
2
95. A body is projected vertically upwards. If t1 and t2 be the
times at which it is at height h above the projection while
ascending and descending respectively, then h is d d h
(c) h (d)
1
(a) gt1t2 (b) gt1t2 (c) 2gt1t2 (d) 2hg
2
96. Two balls A and B of same mass are thrown from the top of 104. A stone is dropped into a well in which the level of water is
the building. A thrown upward with velocity v and B,
h below the top of the well. If v is velocity of sound, the
thrown down with velocity v, then
(a) velocity A is more than B at the ground time T after which the splash is heard is given by
(b) velocity of B is more than A at the ground æ 2hö h
(c) both A &B strike the ground with same velocity (a) T = 2h/v (b) T= ç ÷ +
(d) None of these è g ø v
97. A rocket is fired upward from the earth’s surface such that
æ 2h ö h æ h ö 2h
it creates an acceleration of 19.6 m s - 2 . If after 5 s, its (c) T= ç ÷+ (d) T = çç ÷÷ +
engine is switched off, the maximum height of the rocket è v ø g è 2g ø v
from earth’s surface would be 105. A ball is thrown vertically upwards. It was observed, at a
(a) 980 m (b) 735 m (c) 490 m (d) 245 m height h twice with a time interval Dt. The initial velocity of
98. A man throws balls with same speed vertically upwards one the ball is-
after the other at an interval of 2 sec. What should be the speed 2
of throw so that more than two balls are in air at any time ? æ gDt ö
(a) 8gh + g 2 (Dt)2 (b) 8gh + ç ÷
(a) Only with speed 19.6 m/s è 2 ø
(b) More than 19.6 m/s 1
(c) At least 9.8 m/s (c) 8gh + g 2 ( Dt) 2 (d) 8gh + 4g 2 ( Dt)2
(d) Any speed less then 19.6 m/s. 2
99. A ball is dropped from a high rise platform at t = 0 starting 106. The balls are released from the top of a tower of height H at
from rest. After 6 seconds another ball is thrown downwards regular interval of time. When first ball reaches at the
th
from the same platform with a speed v. The two balls meet æ n + 1ö
at t = 18s. What is the value of v? (take g = 10 m/s2) ground, the n th ball is to be just released and ç ÷ ball
è 2 ø
(a) 75 m/s (b) 55 m/s (c) 40 m/s (d) 60 m/s is at same distance ‘h’ from top of the tower. The value of h is.
100. A stone falls freely under gravity. It covers distances h1, 2 3 4 5H
h2 and h3 in the first 5 seconds, the next 5 seconds and the (a) H (b) H (c) H (d)
next 5 seconds respectively. The relation between h 1, h2 3 4 5 6
and h3 is 107. A stone is dropped from a rising balloon at a height of 76 m
h h above the ground and reaches the ground in 6s. What was
(a) h1 = 2 = 3 (b) h2 = 3h 1 and h3 = 3h2 the velocity of the balloon when the stone was dropped?
3 5
(c) h1 = h2 = h3 (d) h1 = 2h 2 = 3h 3 Take g = 10 m/s2.
101. From a building two balls A and B are thrown such that A is æ 52 ö æ 52 ö
thrown upwards and B downwards (both vertically). If TA (a) ç ÷ m/s upward (b) ç ÷ m/s downward
è 3 ø è 3 ø
and TB are their respective time of flights then (c) 3 m/s (d) 9.8 m/s
EBD_7418
28 PHYSICS

108. Let A, B, C, D be points on a vertical line such that AB = BC (a) 4 s (b) 3.26 s (c) 3.48 s (d) 2.828 s
= CD. If a body is released from position A, the times of 118. Two bodies of masses m1 and m2 fall from heights h1 and
h2 respectively. The ratio of their velocities, when they hit
descent through AB, BC and CD are in the ratio.
the ground is
(a) 1 : 3 - 2 : 3 + 2 (b) 1 : 2 - 1 : 3 - 2
(c) 1 : 2 - 1 : 3 (d) 1 : 2 : 3 - 1 h1 h1 m1h1 h12
(a) (b) (c) (d)
109. Water drops fall at regular intervals from a tab which is h2 h2 m1h 2 h 22
hm above the ground. After how many seconds does the
first drop reach the ground? 119. A stone falls from a balloon that is descending at a uniform
h 2h rate of 12 m/s. The displacement of the stone from the point
2h h
(a) (b) (c) 2g (d) g of release after 10 sec is
g 2g
(a) 490 m (b) 510 m (c) 610 m (d) 725 m
110. If two balls of masses m1 and m2(m1= 2m2) are dropped 120. A body thrown vertically so as to reach its maximum
from the same height, then the ratio of the time taken by height in t second. The total time from the time of
them to reach the ground will be projection to reach a point at half of its maximum height
(a) m1 : m2 (b) 2m2 : m1 (c) 1 : 1 (d) 1 : 2 while returning (in sec) is
111. A boy standing at the top of a tower of 20m height drops a æ 1 ö 3t t
stone. Assuming g = 10 ms–2, the velocity with which it hits (a) 2t (b) ç1 + ÷ø t (c) (d)
è 2 2 2
the ground is
121. The ratio of distances traversed in successive intervals of
(a) 10.0 m/s (b) 20.0 m/s (c) 40.0 m/s (d) 5.0 m/s
time when a body falls freely under gravity from certain
112. What will be the ratio of the distances moved by a freely height is
falling body from rest on 4th and 5th seconds of journey ?
(a) 4 : 5 (b) 7 : 9 (c) 16 : 25 (d) 1 : 1 (a) l : 2 : 3 (b) l : 5 : 9
113. A ball released from a height falls 5 m in one second. In 4 (c) 1 : 3 : 5 (d) 1: 2 : 3
seconds it falls through 122. A body dropped from top of a tower fall through 40 m
(a) 20 m (b) 1.25 m (c) 40 m (d) 80 m during the last two seconds of its fall. The height of tower
114. From a balloon moving upwards with a velocity of 12 ms -1, is (g = 10 m/s2)
a packet is released when it is at a height of 65 m from the (a) 60 m (b) 45 m (c) 80 m (d) 50 m
ground. The time taken by it to reach the ground is 123. A stone thrown upward with a speed u from the top of the
(g = 10 ms–2) tower reaches the ground with a velocity 3u. The height of
(a) 5 s (b) 8 s (c) 4 s (d) 7 s the tower is
115. A ball dropped from a point A falls down vertically to C, (a) 3u2/g (b) 4u2/g (c) 6u2/g (d) 9u2/g
through the midpoint B. The descending time from A to B 124. A stone thrown vertically upwards with a speed of 5 m/sec
and that from A to C are in the ratio attains a height H1. Another stone thrown upwards from
(a) 1 : 1 (b) 1 : 2 (c) 1 : 3 (d) 1: 2 the same point with a speed of 10 m/sec attains a height H2.
116. A ball is dropped from the top of a tower of height 100 m The correct relation between H1 and H2 is
and at the same time another ball is projected vertically (a) H2 = 4H1(b) H2 = 3H1(c) H1 =2H2 (d) H1 = H2
upwards from ground with a velocity 25 ms–1. Then the 125. From a pole of height 10 m, a stone is thrown vertically
distance from the top of the tower, at which the two balls upwards with a speed 5 m/s. The time taken by the stone,
meet is to hit the ground, is n times that taken by it to reach the
(a) 68.4 m (b) 48.4 m (c) 18.4 m (d) 78.4 m
117. A body released from the top of a tower falls through half highest point of its path. The value of n is
the height of the tower in 2 s. In what time shall the body [take g = 10 m/s2]
fall through the height of the tower ? (a) 2 (b) 3 (c) 4 (d) 5

Exercise 2 : Exemplar & Past Year MCQs


NCERT Exemplar MCQs x x
1. Among the four graph shown in the figure there is only
one graph for which average velocity over the time interval (c) (d)
(O, T) can vanish for a suitably chosen T. Which one is it?
x x t t
2. A lift is coming from 8th floor and is just about to reach 4th
floor. Taking ground floor as origin and positive direction
(a) t (b) upwards for all quantities, which one of the following is correct?
(a) x < 0, v < 0, a > 0 (b) x > 0, v < 0, a < 0
t (c) x > 0, v < 0, a > 0 (d) x > 0, v > 0, a < 0
MOTION IN A STRAIGHT LINE 29

3. In one dimensional motion, instantaneous speed v satisfies 10. Two stones are thrown up simultaneously from the edge of
0 £ v < v0. a cliff 240 m high with initial speed of 10 m/s and 40 m/s
(a) The displacement in time T must always take non- respectively. Which of the following graph best represents
negative values the time variation of relative position of the second stone
with respect to the first?
(b) The displacement x in time T satisfies – v0T < x < v0T (Assume stones do not rebound after hitting the ground
(c) The acceleration is always a non-negative number and neglect air resistance, take g = 10 m/ s2)
(d) The motion has no turning points (The figures are schematic and not drawn to scale)
4. A vehicle travels half the distance l with speed v1 and the [JEE Main 2015, C]
other half with speed v2, then its average speed is
(y2 – y1) m (y2 – y1) m
v1 + v2 2v1 + v2 2v1v2 L(v1 + v2 ) 240 240
(a) (b) (c) (d) (a) (b)
2 v1 + v2 v1 + v2 v1v2
5. The displacement of a particle is given by x = (t – 2)2 where t(s)
8 12 t(s)
x is in metre and t in second. The distance covered by the 8 12
particle in first 4 seconds is (y2 – y1) m (y2 – y1 ) m
(a) 4 m (b) 8 m (c) 12 m (d) 16 m 240 240
(c) (d)
Past Year MCQs
t(s) t(s)
6. From a tower of height H, a particle is thrown vertically t® 8 12 12
upwards with a speed u. The time taken by the particle, to 11. A body is thrown vertically upwards from A, the top of the
hit the ground, is n times that taken by it to reach the highest tower, reaches the ground in time t1. If it is thrown vertically
point of its path. The relation between H, u and n is: downwards from A with the same speed, it reaches the ground
[AIPMT 2014, S] in time t2. If it is allowed to fall freely from A, then the time it
(a) 2gH = n u 2 2 (b) gH = (n – 2)2u2d takes to reach the ground is given by [BITSAT 2015, A]
2
(c) 2gH = nu (n – 2) (d) gH = (n – 2)u2 t1 + t 2 t1 - t 2
7. A body starts from rest at time t = 0, the acceleration time (a) t= (b) t=
2 2
graph is shown in the figure. The maximum velocity attained
by the body will be [AIIMS 2014, C] t1
(c) t = t1t 2 (d) t=
Acceleration t2
2
(m/s ) 10 12. If the velocity of a particle is v = At + Bt2, where A and B are
constants, then the distance travelled by it between 1s and
2s is : [NEET 2016, C]
3 3 7 A B
11 Time (a) A + 4B (b) 3A + 7B (c) A + B (d) +
2 2 3 2 3
(sec.) 13. Two bodies begin a free fall from the same height at a time
(a) 110 m/s (b) 55 m/s (c) 650 m/s (d) 550 m/s interval of N s. If vertical separation between the two bodies
8. A particle of unit mass undergoes one-dimensional motion is 1 after n second from the start of the first body, then n is
such that its velocity varies according to v(x) = bx–2n equal to [AIIMS 2016, S]
where b and n are constants and x is the position of the
1 1 N 1 N
particle. The acceleration of the particle as d function of x, (a) nN (b) (c) + (d) -
is given by: [AIPMT 2015, S, BN] gN gN 2 gN 4
(a) –2nb2x–4n–1 (b) –2b2 x–2n+1 14. Preeti reached the metro station and found that the escalator
was not working. She walked up the stationary escalator in
(c) –2nb2 e–4n+1 (d) –2nb2x–2n–1 time t1. On other days, if she remains stationary on the
9. Assertion : Velocity-time graph for an object in uniform moving escalator, then the escalator takes her up in time t2.
motion along a straight path is a straight line parallel to the The time taken by her to walk up on the moving escalator
time axis. will be: [NEET 2017, Exemplar, S]
Reason : In uniform motion of an object velocity increases as t1t 2 t t
1 2 t +t
the square of time elapsed. [AIIMS 2015, C] (a) (b) (c) t1 – t2 (d) 1 2
t 2 - t1 t 2 + t1 2
(a) Assertion is correct, reason is correct; reason is a
correct explanation for assertion. 15. From a balloon moving upwards with a velocity of 12 ms–1,
(b) Assertion is correct, reason is correct; reason is not a packet is released when it is at a height of 65 m from the
a correct explanation for assertion ground. The time taken by it to reach the ground is (g = 10
(c) Assertion is correct, reason is incorrect ms–2) [AIIMS 2017, A]
(d) Assertion is incorrect, reason is correct. (a) 5 s (b) 8 s (c) 4 s (d) 7 s
EBD_7418
30 PHYSICS

16. A body is thrown vertically upwards. Which one of the 19. The water drops fall at regular intervals from a tap 5 m
following graphs correctly represent the velocity vs time? above the ground. The third drop is leaving the tap at an
[JEE Main 2017, C] instant when the first drop touches the ground. How far
above the ground is the second drop at that instant ? (Take
g = 10 m/s2) [AIIMS 2018, S]
(a) 1.25 m (b) 2.50 m (c) 3.75 m (d) 5.00 m
(a) (b) 20. All the graphs below are intended to represent the same
motion. One of them does it incorrectly. Pick it up.
[JEE Main 2018, C]
distance
velocity
(c) (d)
(a) position (b) time

17. A toy car with charge q moves on a frictionless horizontal


position velocity
r
plane r
surface under the influence of a uniform electric field
E . Due to the force q E , its velocity increases from 0 to
6 m/s in one second duration. At that instant the direction time time
of the field is reversed. The car continues to move for two (c) (d)
more seconds under the influence of this field. The average
velocity and the average speed of the toy car between 0 to 21. A boy running on a horizontal road at 8 km/h finds the rain
3 seconds are respectively [NEET 2018, A] falling vertically. He increases his speed to 12 km/h and
(a) 2 m/s, 4 m/s (b) 1 m/s, 3 m/s finds that the drops makes 30° with the vertical. The speed
(c) 1.5 m/s, 3 m/s (d) 1 m/s, 3.5 m/s of rain with respect to the road is [BITSAT 2018, S]
18. The motion of a particle along a straight line is described (a) 4 7 km/h (b) 9 7 km/h
by equation :
x = 8 + 12t – t3 (c) 12 7 km/h (d) 15 7 km/h
where x is in metre and t in second. The retardation of the 22. A juggler keeps on moving four balls in the air throwing the
particle when its velocity becomes zero, is : balls after intervals. When one ball leaves his hand (speed
[AIIMS 2018, A] = 20 ms–1) the position of other balls (height in m) will be
(a) 24 ms–2 (b) zero (c) 6 ms–2 (d) 12 ms–2 (Take g = 10 ms–2) [BITSAT 2018, S]
(a) 10, 20, 10(b) 15, 20, 15(c) 5, 15, 20 (d) 5, 10, 20

Exercise 3 : Try If You Can


1. A particle when thrown, moves such that it passes from g ( H 2 + L2 ) gH 2
same height at 2 and 10 seconds, then this height h is : (a) (b)
(a) 5g (b) g (c) 8g (d) 10g 2H H 2 + L2
2. A particle moving along x-axis has acceleration f, at time t,
g H 2 + L2 2gH 2
(c) (d)
given by f = f0 æç 1 - t ö÷ , where f0 and T are constants. H H 2 + L2
è Tø
4. Which graph corresponds to an object moving with a
The particle at t = 0 has zero velocity. In the time interval
constant negative acceleration and a positive velocity ?
between t = 0 and the instant when f = 0, the particle’s
velocity (vx) is (a) (b)
1 1
(a) f T 2 (b) f0 T 2 (c) f T (d) f0T Velocity
2 0 2 0 Velocity
3. A hunter tries to hunt a monkey with a small, very poisonous
arrow, blown from a pipe with initial speed v0. The monkey is Time
Time
hanging on a branch of a tree at height H above the ground.
The hunter is at a distance L from the bottom of the tree. The (c) (d)
monkey sees the arrow leaving the blow pipe and immediately
loses the grip on the tree, falling freely down with zero initial Velocity Velocity
velocity. The minimum initial speed v0 of the arrow for hunter
to succeed while monkey is in air is
Distance Distance
MOTION IN A STRAIGHT LINE 31

5. A particle starts sliding down a frictionless inclined plane. 10. A ball is dropped from a height of 5 m onto a sandy floor and
If Sn is the distance travelled by it from time t = n – 1 sec to penetrates the sand upto 10 cm before coming to rest. Find
t = n sec, the ratio Sn/Sn+1 is the retardation of the ball in sand assuming it to be uniform.
(a) 490 m/s2 (b) 610 m/s2 (c) 720 m/s2 (d) 810 m/s2
2n - 1 2n + 1 2n
(a) (b) (c) (d) 2n + 1 11. A bird flies with a speed of 10 km/h and a car moves
2n + 1 2n 2n + 1 2n - 1 with uniform speed of 8 km/h. Both start from B towards
6. Starting from rest a particle moves in a straight line with A (BA = 40km) at the same instant. The bird having reached
acceleration a = (25 – t2)1/2 m/s2 for 0 £ t £ 5s, A, flies back immediately to meet the approaching car. As
3p soon as it reaches the car, it flies back to A. The bird repeats
a= m/s2 for t > 5s. The velocity of particle at t = 7s is: this till both the car and the bird reach A simultaneously.
8
(a) 11 m/s (b) 22 m/s (c) 33 m/s (d) 44 m/s The total distance flown by the bird is
7. The acceleration of a particle, starting from rest, varies with (a) 80 km (b) 40 km (c) 50 km (d) 30 k m
12. The displacement of a particle is given by
time according to the relation a = -s w2 sin w t . The y = a + b t + c t2 – d t4
displacement of this particle at a time t will be The initial velocity and acceleration are respectively
(a) s sin w t (b) s w cos w t (a) b, – 4 d (b) – b, 2 c (c) b, 2 c (d) 2 c, – 4 d
13. The displacement x of a particle varies with time t as x =
1 2 2
(c) s w sin w t (d) - (s w sin wt ) t ae-at + bebt, where a, b, a and b are positive constants.
2 The velocity of the particle will
8. The displacement ‘x’ (in meter) of a particle of mass ‘m’ (in
(a) be independent of a and b
kg) moving in one dimension under the action of a force, is
related to time ‘t’ (in sec) by t = x + 3 . The displacement (b) drop to zero when a = b
of the particle when its velocity is zero, will be (c) go on decreasing with time
(a) 2 m (b) 4 m (c) 0 m (d) 6 m (d) go on increasing with time
9. The distance travelled by a body moving along a line in 14. A point moves with uniform acceleration and V1, V2, V3
time t is proportional to t3. The acceleration-time (a, t) graph denote the average velocities in three successive intervals
for the motion of the body will be of time t1, t2, t3. Which of the following relations is correct?
(a) V1 – V2 : V2 – V3 = t1 – t2 : t2 + t3
a (b) V1 – V2 : V2 – V3 = t1 + t2 : t2 + t3
a
(c) V1 – V2 : V2 – V3 = t1 – t2 : t2 – t3
(a) (b) (d) V1 – V2 : V2 – V3 = t1 – t2 : t1 – t3
t t 15. Two fixed points A and B are 20 metres apart. At time t = 0,
the distance between a third point C and A is 20 meters and
the distance between C and B is 10 metres. The component
a a of velocity of point C along both CA and CB at any instant
(c) (d) is 5m/s. Then the distance between A and C at the instant
all the three points are collinear will be
t t (a) 5 m (b) 15 m (c) 10 m (d) 25 m
EBD_7418
32 PHYSICS

ANSWER KEYS
Exercise 1 : Topic-wise MCQs
1 (c) 14 (c) 27 (c) 40 (b) 53 (c) 66 (a) 79 (b) 92 (d) 105 (c) 118 (b)
2 (a) 15 (d) 28 (c) 41 (c) 54 (c) 67 (a) 80 (a) 93 (c) 106 (b) 119 (c)
3 (c) 16 (a) 29 (a) 42 (a) 55 (c) 68 (a) 81 (d) 94 (d) 107 (a) 120 (b)
4 (b) 17 (b) 30 (d) 43 (c) 56 (b) 69 (d) 82 (b) 95 (a) 108 (b) 121 (c)
5 (d) 18 (a) 31 (d) 44 (c) 57 (c) 70 (d) 83 (a) 96 (c) 109 (a) 122 (b)
6 (b) 19 (d) 32 (b) 45 (a) 58 (d) 71 (d) 84 (d) 97 (b) 110 (c) 123 (b)
7 (d) 20 (a) 33 (c) 46 (a) 59 (d) 72 (d) 85 (b) 98 (b) 111 (b) 124 (a)
8 (d) 21 (c) 34 (c) 47 (c) 60 (c) 73 (a) 86 (d) 99 (a) 112 (b) 125 (c)
9 ( b) 22 (c) 35 (d) 48 (c) 61 (d) 74 (d) 87 (d) 100 (a) 113 (d)
10 (c) 23 (a) 36 (a) 49 (b) 62 (d) 75 (d) 88 (b) 101 (a) 114 (a)
11 (d) 24 (a) 37 (b) 50 (c) 63 (c) 76 (c) 89 (b) 102 (c) 115 (d)
12 (c) 25 (c) 38 (d) 51 (a) 64 (a) 77 (d) 90 (d) 103 (a) 116 (d)
13 (b) 26 (c) 39 (d) 52 (a) 65 (c) 78 (a) 91 (a) 104 (b) 117 (d)
Exercise 2 : Exemplar & Past Year MCQs
1 (b) 4 (c) 7 (b) 10 (b) 13 (c) 16 (a) 19 (c) 22 (b)
2 (a) 5 (b) 8 (a) 11 (c) 14 (b) 17 (b) 20 (b)
3 (b) 6 (c) 9 (c) 12 (c) 15 (a) 18 (d) 21 (b)
Exercise 3 : Try If You Can
1 (d) 3 (a) 5 (a) 7 (a) 9 (b) 11 (c) 13 (d) 15 (b)
2 (c) 4 (c) 6 (b) 8 (c) 10 (a) 12 (c) 14 (b)
4
Chapter
MOTION IN A PLANE

Trend
Analysis of NEET and AIIMS (Year 2010-2018)
5

4
Number of Questions

AIPMT/NEET
2 AIIMS

0
2010 2011 2012 2013 2014 2015 2016 2017 2018
Year

Trend
Analysis of JEE Main and BITSAT (Year 2010-2018)
5

4
Number of Questions

3
JEE Main

2 BITSAT

0
2010 2011 2012 2013 2014 2015 2016 2017 2018
Year

Chapter Utility Score (CUS)


Exam Weightage Important Concepts Difficulty Level CUS (Out of 10)
NEET 5 Relative Velocity,
AIIMS 4 Projectile Motion and 3.5/5 5.5/10
JEE Main 2 Circular Motion
BITSAT 4
EBD_7418
34 PHYSICS
MOTION IN A PLANE 35
EBD_7418
36 PHYSICS

Exercise 1 : Topic-wise MCQs

Topic 1: Vectors 12. The resultant of A × 0 will be equal to


(a) zero (b) A
1. Which of the following conditions are sufficient and (c) zero vector (d) unit vector
essential for a quantity to be a vector? 13. In a clockwise system
(a) Magnitude and direction
(b) Magnitude and addition, subtraction, multiplication (a) $j × k$ = $i (b) $i × k$ = 0
by ordinary rules of algebra
(c) Magnitude, direction, and addition, subtraction (c) $j × $j = 1 k$ × $i = 1
(d)
multiplication and division by vector laws r r
14. If A = 4iˆ + 3jˆ and B = 3iˆ + 4ˆj then cosine of angle between
(d) Magnitude, direction and combination of vectors by
ordinary rules of algebra r r r
A and A + B is
2. If q is the angle between two vectors, then the resultant 7
vector is maximum, when value of q is 9 2 5 2 5 2
(a) (b) (c) (d)
(a) 0° (b) 90° 5 5 2 49 28
(c) 180° (d) Same in all cases 15. Consider the following statements and select the correct
3. How many minimum number of vectors in different planes statements from the following.
can be added to give zero resultant? I. Addition and subtraction of scalars make sense only
(a) 2 (b) 3 (c) 4 (d) 5 for quantities with same units
4. Let A = iAˆ cos q + ˆjA sin q be any vector. Another vector II. Multiplication and division of scalars with different
B, which is normal to A can be expressed as units is possible
III. Addition, subtraction, multiplication and division of
(a) î B cos q – ĵ B sin q (b) î B cos q + ĵ B sin q scalars with same unit is possible
(a) I and II (b) II and III
(c) î B sin q – ĵ B cos q (d) î B sin q + ĵ B cos q (c) I and III (d) I, II and III
5. If A = î + ĵ + k̂ and B = 2î - ĵ + 4k̂ then the unit vector 16. Which of the following is/are correct?
r r r
along A + B is I. A ´ B = -B ´ A
r r r
3î + 5k̂ 3î - 5k̂ II. A ´ B ¹ B ´ A
r r r r r r
(a) (b) III. A ´ ( B + C ) = ( A ´ B ) + C
24 34
(a) I only (b) II and III
3î + 5k̂ (b) I and III (d) I and II
(c) (d) None of these
34 17. It is found that | A + B | = | A |. This necessarily implies,
r r r r r r (a) B = 0
6. If a , b and c are three unit vectors such that a + b + c = 0
r r r r r r (b) A and B are antiparallel
then the value of a . b + b . c + c . a is (c) A and B are perpendicular
3 3
(a) (b) –1 (c) 0 (d) - (d) A.B < 0
2 2
18. The position vector a of particle is
7. Let q be the angle between two vectors A and B , then r
r = ( a cos q )iˆ + ( a sin q ) ˆj
 ´ B̂
is equal to The distance from origin & direction of the particle is
Â.B̂ (a) a2 directed towards the origin
(a) sin q (b) cos q (c) tan q (d) cot q
(b) a, directed away from the origin
8. If 0.4î + 0.7ˆj + c k̂ is a unit vector, then the value of c is (c) a3, parallel to the position vector
(a) 0.67 (b) 0.12 (b) 1.44 (d) 0.35 (d) a2 directed away from origin.
9. The unit vectors along the three co-ordinate axes are related as 19. Which of the following statements is/are incorrect?
I. A scalar quantity is the one that is conserved in a
(a) ˆi > ˆj > kˆ > 1 (b) ˆi = ˆj = kˆ = 0 process.
(c) ˆi = -ˆj = kˆ = 1 (d) ˆi = ˆj = kˆ = 1 II. A scalar quantity is the one that can never take
negative values.
10. The angle between the direction of î and (iˆ + ˆj) is III. A scalar quantity has the same value for observers
(a) 90° (b) 0° (c) 45° (d) 180° with different orientations of the axes.
11. Acceleration due to gravity (a) I and III (b) II only
(a) a scalar (b) a vector (c) II and III (d) I and II
(c) polar vector (d) None of these
MOTION IN A PLANE 37

20. For the figure, which of the following is correct? 27. Two vectors A and B lie in a plane, a third vector C lies
® ® ® outside this plane, the sum of these vectors A + B + C
(a) A+ B=C
(a) can be zero
® ® ®
C (b) can never be zero
(b) B+ C=A r r
B (c) lies in a plane containing A + B
® ® ®
(c) C+ A=B r r
(d) lies in a plane containing A ´ B
® ® ®
(d) A+ B+C = 0 28. ABCDEF is a regular hexagon. The centre of hexagon is
A a point O. Then the value of
r r
21. Six vectors, a through f have the b is
AB + AC + AD + AE + AF
a c
magnitudes and directions
indicated in the figure. Which of (a) 2AO (b) 4AO (c) 6 AO (d) Zero
d e
the following statements is true? f 29. For two vectors A and B, | A + B | = | A - B| is always
r r r r r r true when
(a) b + c = f (b) d + c = f
r r r r r r (a) | A | = | B | ¹ 0 (b) | A ^ B
(c) d + e = f (d) b + e = f (c) | A | = | B | ¹ 0 and A and B are parallel or anti parallel
22. Three vectors A, B and C add up to zero. Select the correct (d) None of these
statements. Ù Ù Ù
I (A × B) . C is not zero unless B, C are parallel 30. If a vector 2 i + 3 j + 8 k is perpendicular to the vector
II If A,B,C define a plane, (A × B) × C is in that plane Ù Ù Ù
4 j - 4 i + a k , then the value of a is
III (A × B) .C = | A | | B | C | ® C2 = A2 + B2
(a) I and II (b) II and III (a) 1/2 (b) –1/2 (c) 1 (d) –1
r
(c) I and III (d) I, II and III 31. The velocity v of a particle moving in the xy – plane is
r r
23. For which angle between two equal vectors A and B will
the magnitude of the sum of two vectors be equal to the
r
( ) r
given by v = 6t – 4t 2 ˆi + 8ˆj , with v in m/s and t(> 0) in
magnitude of each vector? second.
(a) q = 60° (b) q = 120° Match the following columns :
(c) q = 0° (d) q = 90°
r Column I Column II
24. A can be written in terms of components as (A) Acceleration magnitude is 10 m/s2 (1) 3\4 s
r r
A = Axiˆ + Ayjˆ + Azkˆ . When will | A | be zero at a time
(a) Ax = Ay = 0 & Az ¹ 0 (b) Ax = Ay = Az ¹ 0 (B) Acceleration zero at time (2) never
r (C) velocity zero at time (3) 1 s
(c) Ax = Ay = Az = 0 (d) | A | can never be zero.
(D) The speed 10 m/s at a time (4) 2 s
r r
25. Given two vectors; A = ˆi + ˆj and B = ˆi - ˆj . Then match (a) (A)®(4); (B)®(1); C®(2); (D)®(3)
the following columns : (b) (A)®(2); (B)®(4); C®(3); (D)®(1)
Column - I Column - II (c) (A)®(3); (B)®(2); C®(4); (D)®(1)
r r (d) (A)®(2); (B)®(4); C®(1); (D)®(3)
(A) ( )
A + B /2 (1) î r
r r 32. The component of vector a = 2iˆ + 3jˆ along the vector
(B) ( )
A - B /2 (2) ĵ i + j is
r r 5
(C) ( )
A × B /2 (3) - kˆ (a)
2
(b) 10 2 (c) 5 2 (d) 5
r r
(D) ( )
A ´ B /2 (4) 0 33. What is the area of triangle formed by A = 2 î - 3 ĵ + 4 k̂
(a) (A)®(4); (B)®(1); C®(2); (D)®(2) and B = î - k̂ and their resultant?
(b) (A)®(2); (B)®(4); C®(3); (D)®(1) (a) 13.5 units (b) 13.5 units
(c) (A)®(3); (B)®(2); C®(4); (D)®(1) (c) 38.7 units (d) 38.7 units
(d) (A)®(1); (B)®(2); C®(4); (D)®(3) ˆ
34. If A = 4 î + 6 j and B = 2 î + 3 ĵ . Then
26. Which law is governed by the given figure? (a)
C
A B A .B = 29
(a) Associative law of vector
addition (b) A ´B = 0
R B
(b) Commutative law of vector |B| 2
addition O A (c) =
C |A| 1
(c) Associative law of vector multiplication
(d) Commutative law of vector multiplication (d) angle between A and B is 30°
EBD_7418
38 PHYSICS

r
35. The vector a = a iˆ + 2 ˆj + bkˆ lies in the plane of the vectors 37° with the X-axis. At t = 0 the particle is at the origin
r r r and its velocity is 8.0 m/s along the X-axis. Find the
b = iˆ + ˆj and c = ˆj + kˆ and bisects the angle between b
and cr . Then which one of the following gives possible
position of the particle at t = 4.0 s.
(a) (41.6m, 7.2 m) (b) (50.3 m, 8.2 m)
values of a and b?
(a) a = 2, b = 2 (b) a = 1, b = 2 (c) (60.2 m, 8.2 m) (d) (11.2 m, 8 m)
(c) a = 2, b = 1 (d) a = 1, b = 1 r
43. A particle moving with velocity v == k ( yiˆ + xjˆ), where k
36. The vectors of magnitude 51 , making equal angles with the = constant. The general equation for its path is
r 1 r
vectors ar = 1 (iˆ - 2 ˆj + 2 kˆ ) ; b = ( -4iˆ - 3kˆ) and c = ˆj [C = constant]
3 5 (a) y = x2 + C (b) y2 = x + C
are (c) xy = C (d) y2 = x2 + C
51 ˆ 51 ˆ 44. The coordinates of a particle moving in x–y plane at any
(a) ± (8i + 4 ˆj - 3kˆ) (b) ± (7i - 4 ˆj + 3kˆ)
89 74 instant of time t are x = 4t2; y = 3t2. The speed of the
particle at that instant is
(c) ± 51 (-7iˆ + ˆj + 9kˆ) (d) None of these
131 (a) 10 t (b) 5 t (c) 3 t (d) 2 t
37. The position vector of a particle is 45. For motion in two or three dimensions, the angle between
r velocity and acceleration is
r= (a cos wt )î + (a sin wt ) ĵ. The velocity vector of the (a) 0°
particle is (b) 90°
(a) directed towards the origin (c) 180°
(b) directed away from the origin (d) Any angle between 0° & 180°
(c) parallel to the position vector 46. A particle moves in a plane with a constant acceleration
(d) perpendicular to the position vector in a direction different from the initial velocity. The path
of the particle is a/an
Topic 2: Motion in a Plane with Constant Acceleration & (a) straight line (b) arc of a circle
Projectile Motion (c) parabola (d) ellipse
47. The shape of trajectory of the motion of an object is
38. A particle crossing the origin of co-ordinates at time t = 0,
determined by
moves in the xy-plane with a constant acceleration a in the
(a) acceleration (b) initial position
y-direction. If its equation of motion is y = bx2 (b is a
(c) initial velocity (d) All of these
constant), its velocity component in the x-direction is
48. If t m is the time taken by a projectile to achieve the
2b a a b maximum height, then the total time of flight Tf related to
(a) (b) (c) (d)
a 2b b a tm as
r (a) tm = 2 Tf (b) Tf = tm
39. The position of particle is given by r = 2 t 2ˆi + 3tjˆ + 4k,
ˆ
(c) Tf = 2tm (d) None of these
where t is in second and the coefficients have proper units
r r 49. If u is the initial velocity of a projectile and v is the velocity
for r to be in metre. The a(t) of the particle at t = 1 s is at any instant, then the maximum horizontal range Rm is
(a) 4 m s–2 along y-direction equal to
(b) 3 m s–2 along x-direction
2 v2
(c) 4 m s–2 along x-direction (a) Rm = u sin 2q (b) Rm =
(d) 2 m s–2 along z-direction g g
40. A body moves in X-Y plane under the action of acceleration v2 sin 2q u2
1 (c) Rm = g
(d) Rm = g
given by (6tiˆ + 4tjˆ) . Assuming that the body is at rest at
3 50. In the projectile motion, if air resistance is ignored, the
time t = 0, the velocity of body at t = 3 sec is horizontal motion is at
(a) 9î + 6ˆj (b) 18î + 6 ĵ (a) constant acceleration (b) constant velocity
(b) variable acceleration (d) constant retardation
(c) 18î + 12ˆj (d) 12î + 68 ĵ 51. A bullet is dropped from the same height when another
41. A particle has an initial velocity 3iˆ + 4 ˆj and an bullet is fired horizontally. They will hit the ground
(a) one after the other
acceleration of 0.4iˆ + 0.3 ˆj . Its speed after 10 sec is (b) simultaneously
(a) 7 2 units (b) 7 units (c) depends on the observer
(c) 8.5 units (d) 10 units (d) None of these
42. A particle moves in the X-Y plane with a constant 52. The time of flight of a projectile on an upward inclined
plane depends upon
acceleration 1.5 m/s2 in the direction making an angle of
MOTION IN A PLANE 39

(a) angle of inclination of the plane the vertical velocity component remains same. Ball stops
(b) angle of projection after hitting the ground. Match the statement of column I
(c) the value of acceleration due to gravity with the distance of the wall from the point of throw in
(d) all of the above. column II.
53. At the highest point on the trajectory of a projectile, its
Column I Column II
(a) acceleration is minimum
(b) velocity is maximum (A) Ball strikes the wall directly (1) 8 m
(c) acceleration is maximum (B) Ball strikes the ground at x = 12 m (2) 10 m
(d) velocity is minimum. from the wall
54. In a projectile motion, velocity at maximum height is (C) Ball strikes the ground at x = 10m (3) 0 m
u cos q from the wall
(a) (b) u cos q
2 (D) Ball strikes the ground at x = 5 m (4) 25 m
u sin q from the wall
(c) (d) None of these
2 (a) (A)®(1,2); (B)®(1); C®(2); (D)®(4)
55. For angle ...X..., the projectile has maximum range and it
is equal to ...X.... Here, X and Y refer to (b) (A)®(2); (B)®(3); C®(1); (D)®(2)
(c) (A)®(2); (B)®(3); C®(1); (D)®(4)
p v2 p v
(a) and 0 (b) and 0 (d) (A)®(3); (B)®(4); C®(3); (D)®(2)
4 2g 2 g 60. The velocity of a projectile at the Y

(c)
p
and 0
v2
(d)
p
and 0
v2 ( )
initial point A is 2$i + 3 $j m/s its
4 g 2 g velocity (in m/s) at point B is B
A X
56. At the top of the trajectory of a projectile, the acceleration
is (a) -2$i + 3 $j (b) 2$i - 3$j
(a) maximum (b) minimum (c) 2$i + 3$j (d) -2$i - 3 $j
(c) zero (d) constant (g)
61. A body is projected from the ground with a velocity 50 m/s
57. A particle is projected with some angle from the surface of at an angle of 30°. It crosses a wall after 3 sec. How far
the planet. The motion of the particle is described by the beyond the wall the stone will strike the ground?
equation; x = t, y = t – t2. Then match the following columns:
[take g = 10 m/s2]
Column I Column II
(a) 86.6 m (b) 96.2 m
(quantity) (magnitude only)
(A) Velocity of projection (1) 1 (c) 100.1 m (d) 111.1 m
(B) Acceleration (2) 62. A stone is projected horizontally with a 5 m/s from the top
2
of a plane inclined at an angle 45o with the horizontal.
(C) Time of flight (3) 2
How far from the point of projection will the particle strike
1 the plane?
(D) Maximum height attained (4)
4
(a) (A)®(4); (B)®(1); C®(2); (D)®(2) (a) 5 2m (b) 11 2 m
(b) (A)®(2); (B)®(3); C®(1); (D)®(2)
(c) 12 2 m (d) 15 2 m
(c) (A)®(2); (B)®(3); C®(1); (D)®(4)
(d) (A)®(3); (B)®(4); C®(3); (D)®(2) 63. Assertion : The horizontal range is same when the angle
58. If V1 is velocity of a body projected of projection is greater than 45° by certain value and less
from the point A and V2 is the V than 45° by the same value.
1 C Reason : If q = 45° + a, then
velocity of a body projected from
point B which is vertically below the V2 u 2 sin2(45° + a ) u 2 cos 2a
highest point C. if both the bodies R1 = =
A g g
collide, then B
u 2sin2(45° - a ) u 2 cos 2a
1 1 If q = 45° – a, then R2 = =
(a) V1 = V2 (b) V2 = V1 g g
2 2
(c) V1 = V2 (d) Two bodies can't collide. (a) Assertion is correct, reason is correct; reason is a
correct explanation for assertion.
59. A ball is thrown at an angle 75° with the horizontal at a
(b) Assertion is correct, reason is correct; reason is not
speed of 20 m/s towards a high wall at a distance d. If the
a correct explanation for assertion
ball strikes the wall, its horizontal velocity component (c) Assertion is correct, reason is incorrect
reverses the direction without change in magnitude and (d) Assertion is incorrect, reason is correct.
EBD_7418
40 PHYSICS

64. A stone is projected horizontally with velocity u from a 75. Two pegs A and B thrown with speeds in the ratio
height H. It’s time of flight is: 1 : 3 acquired the same heights. If A is thrown at an angle
(a) 2H / g (b) 2 H / gu of 30° with the horizontal, the angle of projection of B
will be
(c) H / 2gu (d) 4 H /g
65. A body is projected horizontally with velocity 3 m/s from a æ1ö
(a) 0° (b) sin -1 ç ÷
height 5m. It’s horizontal range is è8ø
(a) 3 m (b) 5 m (c) 10 m (d) 12 m
66. Two bullets are fired horizontally with different velocities æ1ö -1 æ 1 ö
(c) sin -1 ç ÷ (d) sin ç ÷
from the same height. Which will reach the ground first? è6ø è2ø
(a) Slower one
76. A body projected at an angle with the horizontal has a
(b) Faster one
(c) Both will reach simultaneously range 300 m. If the time of flight is 6 s, then the horizontal
(d) It cannot be predicted component of velocity is
67. Two balls are projected at an angle q and (90º – q) to the (a) 30 m s–1 (b) 50 m s–1
horizontal with the same speed. The ratio of their maximum (c) 40 m s–1 (d) 45 m s–1
vertical heights is 77. A particle of unit mass is projected with velocity u at an
(a) 1 : 1 (b) tan q : 1 (c) 1 : tan q (d) tan2 q : 1 inclination a above the horizon in a medium whose
68. A body is thrown with a velocity of 9.8 ms–1 making an resistance is k times the velocity. Its direction will again
angle of 30º with the horizontal. It will hit the ground make an angle a with the horizon after a time
after a time
1 ì 2ku ü 1 ì 2 ku ü
(a) 3.0 s (b) 2.0 s (c) 1.5 s (d) 1 s (a) log í1 - sin a ý (b) log í1 + sin a ý
69. The velocity of projection of a body is increased by 2%. k î g þ k î g þ
Other factors remaining unchanged, what will be the
1 ì ku ü 1 ì 2 ku ü
percentage change in the maximum height attained? (c) log í1 + sin a ý (d) log í1 + sin a ý
(a) 1% (b) 2 % (c) 4 % (d) 8 % k î g þ k î 3 g þ
70. A cricket ball is hit with a velocity 25 ms–1, 60° above the 78. The greatest range of a particle, projected with a given
horizontal. How far above the ground, ball passes over a velocity on an inclined plane, is x times the greatest vertical
fielder 50 m from the bat (consider the ball is struck very altitude above the inclined plane. Find the value of x.
close to the ground)? (a) 2 (b) 4 (c) 3 (d)1/2
Take 3 = 1.7 and g = 10 ms–2 79. A body is projected vertically upwards with a velocity u,
(a) 6.8 m (b) 7 m (c) 5 m (d) 10 m after time t another body is projected vertically upwards
71. A missile is fired for maximum range with an initial velocity from the same point with a velocity v, where v < u. If they
of 20 m/s. If g = 10 m/s2, the range of the missile is meet as soon as possible, then choose the correct option
(a) 40 m (b) 50 m (c) 60 m (d) 20 m u - v + u 2 + v2 u - v + u2 - v2
72. A stone is thrown from a point with a speed 5 m/s at an (a) t= (b) t =
g g
elevation angle of q. From the same point and at the same
instant, a person starts running with a constant speed 2.5 u + v + u2 - v2 u - v + u2 - v2
m/s to catch the stone. If the person will be able to catch (c) t= (d) t =
g 2g
the ball then, what should be the angle of projection q?
80. The range of a projectile is R when the angle of projection
(a) 75° (b) 30° (c) 60° (d) 45°
is 40°. For the same velocity of projection and range, the
73. A projectilel can have the same range R for two angles of other possible angle of projection is
projection. If t1 and t2 be the times of flight in two cases, (a) 45° (b) 50° (c) 60° (d) 40°
then what is the product of two times of flight?
81. If the angles of projection of a projectile with same initial
(a) t1t2 µ R (b) t1t2 µ R2
velocity exceed or fall short of 45° by equal amounts , then
(c) t1t2 µ1/R (d) t1t2 µ 1/R2 the ratio of horizontal ranges is
74. A particle of mass m is projected with a velocity u making (a) l : 2 (b) l : 3 (c) 1 : 4 (d) 1 : 1
an angle of 30° with the horizontal. The magnitude of (Vh
× h) of the projectile when the particle is at its maximum gx 2
82. The equation of a projectile is y = 3x -
height h 2
The angle of projection is given by
3 u2 1
(a) (b) zero (a) tan q = (b) tan q = 3
2 g 3
u3 3 u3
(c)
2g
(d)
16 g (c) p (d) zero.
2
MOTION IN A PLANE 41

83. A plane flying horizontally at a height of 1500 m with a identical with the trajectory of the projectile fired from the
velocity of 200 ms–1 passes directly overhead on antiaircraft earth. The value of the acceleration due to gravity on the
gun. Then the angle with the horizontal at which the gun planet is (in ms– 2) given g = 9.8 m/s2
should be fired from the shell with a muzzle velocity of (a) 3.5 (b) 5.9 (c) 16.3 (d) 110.8
400 ms–1 to hit the plane, is
(a) 90° (b) 60° (c) 30° (d) 45° Topic 3: Relative Velocity in Two
84. The equation of trajectory of projectile is given by Dimensions & Uniform Circular Motion
x gx 2
y= - , where x and y are in metre. 92. A river flow with a speed more than the maximum speed
3 20 with which a person can swim in the still water. He intends
The maximum range of the projectile is to cross the river by shortest possible path (i.e., he wants
8 4 3 3 to reach the point on the opposite bank which directly
(a) m (b) m (c) m (d) m
3 3 4 8 opposite to the starting point). Which of the following
85. A stone is just released from the window of a moving train correct?
moving along a horizontal straight track. The stone will (a) He should start normal to the river bank.
hit the ground following a (b) He should start in such a way that, he moves normal
(a) straight line path (b) circular path to the bank, relative to the bank.
(c) parabolic path (d) hyperbolic path (c) He should start in a particular (calculated) direction
86. A ball is thrown from rear end of the compartment of train making an obtuse angle with the direction of water
to the front end which is moving at a constant horizontal current.
velocity. An observer A sitting in the compartment and (d) The man cannot cross the river, in that way.
another observer B standing on the ground draw the 93. A moves with 65 km/h while B is coming back of A with
trajectory. They will have 80 km/h. The relative velocity of B with respect to A is
(a) equal horizontal and equal vertical ranges (a) 80 km/h (b) 60 km/h (c) 15 km/h (d) 145 km/h
(b) equal vertical ranges but different horizontal ranges 94. A boat crosses a river with a velocity of 8 km/h. If the
(c) different vertical ranges but equal horizontal ranges resulting velocity of boat is 10 km/h then the velocity of
(d) different vertical and different horizontal ranges river water is
87. Two balls are projected simultaneously in the same vertical
(a) 4 km/h (b) 6 km/h (c) 8 km/h (d) 10 km/h
plane from the same point with velocities v1 and v2 with
angle q1 and q2 respectively with the horizontal. If v1 cos 95. A person aiming to reach the exactly opposite point on the
q1 = v2 cos q2, the path of one ball as seen from the position bank of a stream is swimming with a speed of 0.5 m/s at
an angle of 120° with the direction of flow of water. The
of other ball is :
speed of water in the stream is
(a) parabola (a) 1 m/s (b) 0.5 m/s (c) 0.25 m/s (d) 0.433 m/s
(b) horizontal straight line
(c) vertical straight line 96. A boat is moving with a velocity 3 î + 4 ĵ with respect to
(d) straight line making 45° with the vertical ground. The water in the river is moving with a velocity
88. A bullet is fired with a speed of 1500 m/s in order to hit a
–3 î – 4 ĵ with respect to ground. The relative velocity of
target 100 m away. If g = 10 m/s2. The gun should be
the boat with respect to water is
aimed
(a) 15 cm above the target (a) 8 ĵ (b) –6 î – 8 ĵ (c) 6 î + 8 ĵ (d) 5 2
(b) 10 cm above the target 97. A boat which has a speed of 5 km/hr in still water crosses
(c) 2.2 cm above the target a river of width 1 km along the shortest possible path in
(d) directly towards the target 15 minutes. The velocity of the river water in km/hr is
89. A projectile is thrown in the upward direction making an
angle of 60° with the horizontal direction with a velocity (a) 3 (b) 4 (c) 21 (d) 1
of 147 ms–1. Then the time after which its inclination with 98. A boat B is moving upstream
with velocity 3 m/s with vb
the horizontal is 45°, is B
(a) 15 s (b) 10.98 s (c) 5.49 s (d) 2.745 s respect to ground. An observer j
90. A particle is projected with a velocity v such that its standing on boat observes that v
oS w i
range on the horizontal plane is twice the greatest height a swimmer S is crossing the
attained by it. The range of the projectile is (where g is river perpendicular to the
acceleration due to gravity) direction of motion of boat. If river flow velocity is 4 m/s
and swimmer crosses the river of width 100 m in 50 sec,
4v2 4g v2 4v 2
(a) (b) (c) (d) then
5g 5v 2 g 5g (a) velocity of swimmer w.r.t ground is Ö13 m/s
91. A projectile is fired from the surface of the earth with a (b) drift of swimmer along river is zero
velocity of 5 ms–1 and angle q with the horizontal. Another (c) drift of swimmer along river will be 50 m
projectile fired from another planet with a velocity of (d) velocity of swimmer w.r.t ground is 2 m/s
3 ms– 1 at the same angle follows a trajectory which is
EBD_7418
42 PHYSICS

99. Centripetal acceleration is 108. A particle moves in a circle of radius 4 cm clockwise at


(a) a constant vector constant speed 2 cm/s. If x̂ and ŷ are unit acceleration
(b) a constant scalar vectors along X and Y-axis respectively (in cm/s2), the
(c) a magnitude changing vector acceleration of the particle at the instant half way between
(d) not a constant vector P and Q is given by y
100. Two stones are moving with same angular speeds in the (a) -4(xˆ + y)
ˆ P
radii of circular paths 1 m and 2 m. The ratio of their
linear speed is ...X.... Here, X refers to (b) 4(xˆ + y)
ˆ
(a) 2 (b) 1/2 (c) 1/3 (d) 3 O x
(c) -(xˆ + y)
ˆ / 2 Q
101. The direction of the angular velocity vector is along
(a) the tangent to the circular path (d) (xˆ - y)ˆ /4
(b) the inward radius 109. Assertion : The magnitude of velocity of two boats relative
(c) the outward radius to river is same. Both boats start simultaneously from same
(d) the axis of rotation point on one bank may reach opposite bank simultaneously
102. If ar and at represent radial and tangential accelerations, moving along different paths.
the motion of particle will be uniformly circular, if Reason : For boats to cross the river in same time. The
(a) ar = 0 and at = 0 (b) ar = 0 but at ¹ 0 component of their velocity relative to river in direction
(c) ar ¹ 0 and at = 0 (d) ar ¹ 0 and at ¹ 0 normal to flow should be same.
103. In uniform circular motion (a) Assertion is correct, reason is correct; reason is a
(a) both velocity and acceleration are constant correct explanation for assertion.
(b) acceleration and speed are constant but velocity (b) Assertion is correct, reason is correct; reason is not
changes a correct explanation for assertion
(c) both acceleration and velocity change (c) Assertion is correct, reason is incorrect
(d) both acceleration and speed are constant (d) Assertion is incorrect, reason is correct.
104. A body is moving with a constant speed v in a circle of 110. A boat which has a speed of 5 km h –1 in still water crosses
radius r. Its angular acceleration is a river of width 1 km along the shortest possible path in 15
(a) vr (b) v/r (c) zero (d) vr2 minutes. The velocity of the river water is
105. A stone of mass m is tied to a string of length l and rotated (a) 1 km h –1 (b) 3 km h –1
in a circle with a constant speed v, if the string is released
the stone flies (c) 4 km h –1 (d) 41 km h -1
(a) radially outward 111. If rain falls vertically with a velocity Vr and wind blows
(b) radially inward With a velocity vw from east to west, then a person standing
(c) tangentially outward on the roadside should hold the umbrella in the direction
(d) with an acceleration mv2/l
V V
106. Choose the correct statement(s) from the following. (a) tan q = w (b) tan q = r
I. If speed of a body in a curved path is constant it has Vr Vw
zero acceleration Vrw Vr
II. When a body moves on a curved path with a constant (c) tan q = (d) tan q =
speed, it has acceleration perpendicular to the direction
2
Vr + Vw 2
Vr + Vw 2
2

of motion 112. If Vr is the velocity of rain falling vertically and Vm is the


(a) I only (b) II only velocity of a man walking on a level road, and q is the
(c) I and II (d) None of these angle with vertical at which he should hold the umbrella
107. A swimmer wants to cross a river straight. He swim at to protect himself, than the relative velocity of rain w.r.t.
5 km/hr in still water. A river 1 km wide flows at the rate the man is given by:
of 3 km/hr. Which of the following figure shows the correct
direction for the swimmer along which he should strike? (a) Vrm = Vr 2 + Vm 2 + 2VrVm cos q
(Vs ® velocity of swimmer, Vr ® velocity of river, V®
resultant velocity) (b) Vrm = Vr 2 + Vm 2 - 2VrVm cos q
B C
(c) Vrm = Vr2 + Vm 2
V V Vr
1 km
1 km

(a) (b)
VS (d) Vrm = Vr2 - Vm 2
O V A VS 113. An aircraft executes a horizontal loop of radius 1.00 km
r
with a stedy speed of 900 km/h. The ratio of centripetal
acceleration to acceleration due to gravity is [g = 9.8
m/s2]
1 km

1 km

(c) (d) (a) 6.38 (b) 9.98 (c) 11.33 (d) 12.13
MOTION IN A PLANE 43

114. A particle is going parallel to x-axis with constant speed


(a) 0 m s–1 (b) 2 2m s - 1
v at a distance a from the axis. Then its angular velocity
about an axis passing through the origin O, at the instant (c) 8 m s–1 (d) 4 m s–1
when radial vector of the particle makes angle q with the æ 20 ö
x-axis is 119. A particle moves along a circle of radius ç ÷m with
è p ø
v 2 v a v2 constant tangential acceleration. It the velocity of particle
(a) sin θ (b) cos 2 θ (c) sin θ (d) sin 2 θ is 80 m/sec at end of second revolution after motion has
a 2a v a2 begun, the tangential acceleration is
115. Two particles A and B separated by a distance 2R are
(a) 40 p m/sec2 (b) 40 m/sec2
moving counter clockwise along the same circular path of 2
radius R each with uniform speed v. At time t = 0, A is (c) 640 p m/sec (d) 160 p m/sec2
72v 2 120. A stone tied to the end of a string of 1 m long is whirled in
given a tangential acceleration of magnitude a = a horizontal circle with a constant speed. If the stone makes
25 pR
22 revolution in 44 seconds, what is the magnitude and
then direction of acceleration of the stone?
(a) p2 m s–2 and direction along the radius towards the
6 pR centre.
(a) the time lapse for the two bodies to collide is
5v (b) p2 m s–2 and direction along the radius away from
(b) the angle covered by A is 11p/6 the centre.
11v (c) p2 m s–2 and direction along the tangent to the circle.
(c) angular velocity of A is (d) p2/4 m s–2 and direction along the radius towards
5R
the centre.
(d) radial acceleration of A is 289v2/5R 121. A cone filled with water is revolved in a vertical circle of
116. The length of second’s hand in a watch is 1 cm. The change radius 4 m and the water does not fall down. What must
in velocity of its tip in 15 seconds is: be the maximum period of revolution?
(a) 2 s (b) 4 s (c) 1 s (d) 6 s
p
(a) zero (b) cm/s 122. A particle moves along a circle of radius R with constant
30 2 angular velocity w. Its displacement magnitude in time t
is :
p p 2 (a) wt (b) 2R sin wt
(c) cm/s (d) cm/s
30 30 wt
(c) 2R cos wt (d) 2 R sin
117. For a particle in uniform circular motion, the acceleration 2
r 123. Two bodies are moving in concentric orbits of radii 2cm
a at a point P(R,q) on the circle of radius R is (Here q is
measured from the x-axis) and 4 cm such that their time periods are the same. The
ratio of their centripetal accelerations is :
n2 n2
(a) - cos q iˆ + sin q ˆj 1 1 3 4
R R (a) (b) (c) (d)
2 8 2 9
n2 n2 124. A particle P is moving in a circle of radius ‘a’ with a
(b) - sin q iˆ + cos q ˆj
R R uniform speed v. C is the centre of the circle and AB is a
n2 n2 diameter. When passing through B the angular velocity
(c) - cos q iˆ - sin q ˆj of P about A and C are in the ratio:
R R
(a) 1 : 1 (b) 1 : 2 (c) 2 : 1 (d) 4 : 1
n2 ˆ n2 ˆ 125. A particle moves in a circle of radius 30 cm. Its linear
(d) i+ j
R R speed is given by : V = 2t, where t in second and v in m/s.
118. A particle describes uniform circular motion in a circle of Find out its radial and tangential acceleration at t = 3 sec
radius 2 m, with the angular speed of 2 rad s–1 . The respectively.
p (a) 220 m/sec2, 50 m/sec2 (b) 110 m/sec2, 5 m/sec2
magnitude of the change in its velocity in s is
2
(c) 120 m/sec2, 2 m/sec2 (d) 110 m/sec2, 10 m/sec2
EBD_7418
44 PHYSICS

Exercise 2 : Exemplar & Past Year MCQs


NCERT Exemplar MCQs Past Year MCQs
1. The angle between A = iˆ + ˆj and B = iˆ - ˆj is 9. A particle is moving such that its position coordinate
(a) 45° (b) 90° (c) – 45° (d) 180° (x, y) are
(2m, 3m) at time t = 0
2. Which one of the following statements is true?
(6m, 7m) at time t = 2 s and
(a) A scalar quantity is the one that is conserved in a
(13m, 14m) at time t = 5s.
process r
(b) A scalar quantity is the one that can never take Average velocity vector (Vav ) from t = 0 to t = 5s is :
negative values [AIPMT 2014, A]
(c) A scalar quantity is the one that does not vary from 1 7
(a) (13iˆ + 14j)
ˆ (b) (iˆ + ˆj)
one point to another in space 5 3
(d) A scalar quantity has the same value for observers 11 ˆ ˆ
with different orientation of the axes (c) 2(iˆ + ˆj) (d) (i + j)
5
3. Figure shows the orientation of two Y 10. Two projectiles are fired from the same point with the
vectors u and v in the xy-plane. same speed at angles of projection 60º and 30º respectively.
If u = aiˆ + bjˆ and v = piˆ + qjˆ u
v Which one of the following is true? [AIIMS 2014, C]
Which of the following is correct? (a) Their maximum height will be same
(a) a and p are positive while b and (b) Their range will be same
q are negative O X (c) Their landing velocity will be same
(b) a, p and b are positive while q is negative (d) Their time of flight will be same
(c) a, q and b are positive while p is negative 11. A projectile is fired with a velocity u making an angle q
(d) a, b, p and q are all positive with the horizontal. What is the magnitude of change in
4. The component of a vector r along X-axis will have
maximum value if velocity when it is at the highest point– [BITSAT 2014, A]
(a) r is along positive Y-axis (a) u cos q (b) u (c) u sin q (d) u cos q – u
(b) r is along positive X-axis r r wt wt
(c) r makes an angle of 45° with the X-axis 12. If vectors A = cos wtiˆ + sinwtjˆ and B = cos ˆi + sin ˆj
2 2
(d) r is along negative Y-axis are functions of time, then the value of t at which they are
5. The horizontal range of a projectile fired at an angle of orthogonal to each other is : [AIPMT 2015 RS, C]
15° is 50 m. If it is fired with the same speed at an angle of p p
45°, its range will be (a) t = (b) t =
2w w
(a) 60 m (b) 71 m (c) 100 m (d) 141 m p
6. Consider the quantities, pressure, power, energy, impulse, (c) t = 0 (d) t =
gravitational potential, electrical charge, temperature, area. r4w
13. The position vector of a particle R as a function of time is
Out of these, the only vector quantities are : given by:
(a) impulse, pressure and area r
(b) impulse and area R = 4sin(2pt)iˆ + 4cos(2pt) ˆj
(c) area and gravitational potential Where R is in meter, t in seconds and î and ĵ denote unit
(d) impulse and pressure vectors along x-and y-directions, respectively.
7. In a two dimensional motion, instantaneous speed v0 is a
Which one of the following statements is wrong for the
positive constant. Then, which of the following are
necessarily true? motion of particle? [AIPMT 2015 RS, C]
(a) The average velocity is not zero at any time v2
(a) Magnitude of acceleration vector is , where v is
(b) Average acceleration must always vanish R
(c) Displacements in equal time intervals are equal the velocity of particle
(d) Equal path lengths are traversed in equal intervals (b) Magnitude of the velocity of particle is 8 meter/second
8. In a two dimensional motion, instantaneous speed v0 is a (c) path of the particle is a circle of
r radius 4 meter.
positive constant. Then, which of the following are (d) Acceleration vector is along - R
necessarily true? 14. A ship A is moving Westwards with a speed of 10 km h–1
(a) The acceleration of the particle is zero and a ship B 100 km South of A, is moving Northwards
(b) The acceleration of the particle is bounded with a speed of 10 km h –1. The time after which the distance
(c) The acceleration of the particle is necessarily in the between them becomes shortest, is : [AIPMT 2015, S]
plane of motion
(d) The particle must be undergoing a uniform circular motion (a) 5 h (b) 5 2 h (c) 10 2 h (d) 0 h
MOTION IN A PLANE 45

15. Assertion : The magnitude of velocity of two boats relative (a) R = 4 H1H 2 (b) R = 4(H1 – H2)
to river is same. Both boats start simultaneously from same
point on one bank may reach opposite bank simultaneously H12
(c) R = 4 (H1 + H2) (d) R =
moving along different paths. H 22
Reason : For boats to cross the river in same time. The 21. The x and y coordinates of the particle at any time are
component of their velocity relative to river in direction x = 5t – 2t2 and y = 10t respectively, where x and y are in
normal to flow should be same. [AIIMS 2015, C] meters and t in seconds. The acceleration of the particle at
(a) Assertion is correct, reason is correct; reason is a
correct explanation for assertion. t = 2s is [NEET 2017, A]
(b) Assertion is correct, reason is correct; reason is not (a) 5 m/s2 (b) –4 m/s2 (c) –8 m/s2 (d) 0
a correct explanation for assertion 22. A boy playing on the roof of a 10 m high building throws a
(c) Assertion is correct, reason is incorrect ball with a speed of 10m/s at an angle of 30º with the
(d) Assertion is incorrect, reason is correct. horizontal. How far from the throwing point will the ball be
16. The position of a projectile launched from the origin at t = 0 at the height of 10 m from the ground? [AIIMS 2017, S]
r
( )
is given by r = 40iˆ + 50 ˆj m at t = 2s. If the projectile was 1
[ g = 10m/s2 , sin 30o = , cos 30o =
3
]
launched at an angle q from the horizontal, then q is
(take g = 10 ms–2) [BITSAT 2015, S] 2 2
2 (a) 5 5 (b) 6 (c) 3 (d) 5 3
-1 -1 3 -1 7 -1 4
(a) tan (b) tan (c) tan (d) tan 23. Two projectiles A and B thrown with speeds in the ratio
3 2 4 5
17. Ar particle moves so that its position vector is given by 1 : 2 acquired the same heights. If A is thrown at an
r = cos wtxˆ + sin wtyˆ . Where w is a constant. Which of angle of 45° with the horizontal, the angle of projection of
the following is true? [NEET 2016, C] B will be [BITSAT 2017, S]
r
(a) Velocity and acceleration both are perpendicular to r (a) 0° r r(b) r60° (c) 30° (d) 45°
r
(b) Velocity and acceleration bothrare parallel to r 24. Given that A + B r = R r
and A = B = R. What should be the
(c) Velocity is perpendicular to r and acceleration is angle between A and B ? [BITSAT 2017, A]
directed towards the origin r (a) 0 (b) p/3 (c)
r 2p/3 (d) p
(d) Velocity is perpendicular to r and acceleration is 25. The moment of the force, F = 4iˆ + 5 ˆj – 6kˆ at (2, 0, –3),
directed away from the origin about the point (2, –2, –2), is given by [NEET 2018, S]
18. If the magnitude of sum of two vectors is equal to the (a) –8iˆ – 4 ˆj – 7kˆ (b) –4iˆ – ˆj – 8kˆ
magnitude of difference of the two vectors, the angle
between these vectors is : [NEET 2016, A] (c) –7iˆ – 4 ˆj – 8kˆ (d) –7iˆ – 8 ˆj – 4kˆ
(a) 0° (b) 90° (c) 45° (d) 180° 26. Assertion : The trajectory of projectile is quadratic in y
19. A ball is thrown from a point with a speed ‘v0 ’ at an and linear in x.
Reason : y component of trajectory is independent of x-
elevation angle of q. From the same point and at the same component. [AIIMS 2018, C]
'v ' (a) Assertion is correct, reason is correct; reason is a
instant, a person starts running with a constant speed 0
2 correct explanation for assertion.
to catch the ball. Will the person be able to catch the ball? (b) Assertion is correct, reason is correct; reason is not
If yes, what should be the angle of projection q? a correct explanation for assertion
[AIIMS 2016, C] (c) Assertion is correct, reason is incorrect
(a) No (b) Yes, 30° (c) Yes, 60° (d) Yes, 45° (d) Assertion is incorrect, reason is correct.
20. A stone projected with a velocity u at an angle q with the 27. A hunter aims his gun and fires a bullet directly at a monkey
horizontal reaches maximum height H1 . When it is on a tree. At the instant the bullet leaves the barrel of the
gun, the monkey drops. Pick the correct statement
æp ö regarding the situation. [BITSAT 2018, C]
projected with velocity u at an angle ç - q÷ with the (a) The bullet will never hit the monkey
è2 ø
horizontal, it reaches maximum height H2. The relation (b) The bullet will always hit the monkey
between the horizontal range R of the projectile, heights (c) The bullet may or may not hit the monkey
H1 and H2 is [BITSAT 2016, S] (d) Can’t be predicted

Exercise 3 : Try If You Can


ur ur ur ur ur 2. If R = P + Q and R makes an angle q1 with P and q2
1. If A = B - C , then, the angle between A and B is
with Q , then
B2 + A 2 – C2 B2 + A 2 – C2 (a) q1 cannot be less than q2
(a) tan –1 (b) sin –1
2AB 2AB
(b) If q1 < q2 then | P | < | Q |
A 2 + B2 – C2 A 2 + B2 – C2 (c) If q1 < q2 then | P | > | Q |
(c) cos –1 (d) sec –1
2AB 2AB
(d) If q1 = q2 then | P | = | Q |
EBD_7418
46 PHYSICS

3. A particle is projected with a certain velocity at an angle 10. A particle is projected from a
37°
a above the horizontal from the foot of an inclined plane tower as shown in figure, then
of inclination 30°. If the particle strikes the plane normally the distance from the foot of the 500 m/s

1500m
3
then a is tower where it will strike the
ground will be
–1 æ 3 ö –1 1 (a) 4000/3 m (b) 2000/3m ///////////////////////////////////////////////////
(a) 30° + tan ç 2 ÷ (b) 30° + tan
è ø 2 (c) 1000/3 m (d) 2500/3 m
(c) 30° + tan–11 (d) 60° 11. A particle moving along the circular path with a speed v
4. A particle is projected over a triangle from one end of a and its speed increases by g in one second. If the radius of
horizontal base and grazing the vertex falls on the other the circular path be r, then the net acceleration of the
end of the base. If a and b be the base angles and q the particle is:
angle of projection, then the correct relation between a, b v2 v2
and q is (a) +g (b) 2
+ g2
r r
(a) sinq = cosa + tanb (b) tanq = tana + tanb 1/ 2 1/ 2
(c) cosq = cosa + cosb (d) sina = sinq + sinb é v4 2ù é v2 ù
(c) ê 2 + g ú (d) ê + g ú
5. Two particles are projected simultaneously from the level ër û ër û
ground as shown in figure. They may collide after a time : 12. A projectile is fired with a velocity v at right angle to the
x sin q 2 x cos q2 slope which is inclined at an angle q with the horizontal.
(a) u1 (b) u2 The range of the projectile along the inclined plane is:
x sin q2 x sin q1 2v 2 tan q v 2 sec q
(c) (a) (b)
u1 sin ( q 2 - q1 ) (d) u2 sin ( q2 - q1 ) g g
6. A balloon starts rising from the surface of the earth. 2v 2 tan q sec q v 2 sin q
The ascension rate is constant and equal to v 0. Due to (c) (d)
g g
the wind the balloon gathered the horizontal velocity
component v x = ay, where a is a constant and y is the 13. AB is an inclined plane of
O
height of ascent. The tangential, acceleration of the inclination 30° with horizontal.
balloon is Point O is 20 m above point A. B 20m
A particle is projected
a 2 y / 1 + ( ay / v0 )
2
a 2 y / v0
30°
(a) (b) horizontally and it collides with A
Horizontal
the plane AB, perpendicularly.
(c) a 2 y / 1 + v02 (d) a 2v0 / 1 + ( 2 y / a )2 Speed of the particle must be (g = 10 m/s2)
7. Consider a collection of a large number of particles each (a) 13 m/s (b) 8 3m / s (c) 4 5m/s (d) 2 5m/s
with speed v. The direction of velocity is randomly 14. A point A moves uniformly with a velocity v in such a way
distributed in the collection. The magnitude of the relative that the direction of its velocity continually points at an-
velocity between a pair of particles averaged over all the other point B, which in turn, moves along a straight line
pairs in the collection is with a uniform velocity u (u < v). At the initial moment u
(a) 4v/p (b) 3v/2p (c) 5v/3p (d) 2v/3p and v are right angles and the points are separated by a
8. A particle is projected horizontally with a speed u from distance l. How soon will the points meet?
the top of a plane inclined at an angle q with the horizontal. (a) vl/(v2 + u2) (b) vl/(v2 – u2)
How far from the point of projection will the particle strike 2
(c) 5vl/(v + u ) 2 (d) 3vl/(v2 + u2)
the plane? 15. You throw a ball with a launch
(a) 2u2 tanq.secq/g (b) u2 sinq.cosq/g r
(c) 3u2 cosq.secq/g (d) 2u2 sin2q/g
velocity of v = 3iˆ + 4ˆj m/s ( )
towards a wall, where it hits at
9. When a shot is projected from a gun at any angle of
elevation, the shot as seen from the point of projection will height h 1 . Suppose that the
appear to descend past a vertical target with launch velocity were, instead,
r
(a) uniform velocity (b) uniform acceleration
(c) non-uniform velocity (d) None of these
( )
v = 5ˆi + 4 ˆj m/s and h2 is height, then
(a) h2 = h1 (b) h2 < h1 (c) h2 > h1 (d) h2 ³ h1
MOTION IN A PLANE 47

ANSWER KEYS
Exercise 1 : Topic-wise MCQs
1 (c) 14 (b) 27 (b) 40 (a) 53 (d) 66 (c) 79 (b) 92 (d) 105 (c) 118 (c)
2 (a) 15 (d) 28 (c) 41 (a) 54 (b) 67 (d) 80 (b) 93 (c) 106 (b) 119 (b)
3 (c) 16 (d) 29 (b) 42 (a) 55 (c) 68 (d) 81 (d) 94 (b) 107 (d) 120 (a)
4 (c) 17 (b) 30 (b) 43 (d) 56 (d) 69 (c) 82 (b) 95 (c) 108 (c) 121 (b)
5 (c) 18 (b) 31 (a) 44 (a) 57 (c) 70 (c) 83 (b) 96 (c) 109 (b) 122 (d)
6 (d) 19 (d) 32 (a) 45 (d) 58 (b) 71 (a) 84 (b) 97 (a) 110 (b) 123 (a)
7 (c) 20 (c) 33 (a) 46 (c) 59 (a) 72 (c) 85 (c) 98 (a) 111 (a) 124 (b)
8 (d) 21 (c) 34 (b) 47 (d) 60 (b) 73 (a) 86 (b) 99 (d) 112 (c) 125 (c)
9 (d) 22 (c) 35 (d) 48 (c) 61 (a) 74 (d) 87 (c) 100 (b) 113 (a)
10 (c) 23 (b) 36 (d) 49 (d) 62 (a) 75 (c) 88 (c) 101 (d) 114 (a)
11 (c) 24 (c) 37 (d) 50 (b) 63 (a) 76 (b) 89 (c) 102 (c) 115 (b)
12 (c) 25 (d) 38 (b) 51 (b) 64 (a) 77 (b) 90 (a) 103 (c) 116 (d)
13 (a) 26 (a) 39 (c) 52 (d) 65 (a) 78 (b) 91 (a) 104 (c) 117 (c)
Exercise 2 : Exemplar & Past Year MCQs
1 (b) 4 (b) 7 (d) 10 (b) 13 (b) 16 (c) 19 (c) 22 (d) 25 (c)
2 (d) 5 (c) 8 (c) 11 (c) 14 (a) 17 (c) 20 (a) 23 (c) 26 (d)
3 (b) 6 (b) 9 (d) 12 (b) 15 (a) 18 (b) 21 (b) 24 (c) 27 (b)
Exercise 3 : Try If You Can
1 (c) 3 (a) 5 (c) 7 (a) 9 (a) 11 (c) 13 (c) 15 (b)
2 (c) 4 (b) 6 (b) 8 (a) 10 (a) 12 (c) 14 (b)
EBD_7418
48 PHYSICS
5
Chapter
LAWS OF MOTION

Trend
Analysis of NEET and AIIMS (Year 2010-2018)
5

4
Number of Questions

3
AIPMT/NEET
2 AIIMS

0
2010 2011 2012 2013 2014 2015 2016 2017 2018
Year

Trend
Analysis of JEE Main and BITSAT (Year 2010-2018)

4
Number of Questions

3
JEE Main
2 BITSAT

0
2010 2011 2012 2013 2014 2015 2016 2017 2018
Year

Chapter Utility Score (CUS)


Exam Weightage Important Concepts Difficulty Level CUS (Out of 10)

NEET 6 Newton's laws of motion,


AIIMS 4 Conservation of momentum, 4.5/5 8/10
JEE Main 3 Friction, Uniform circular
BITSAT 3 motion & Banking of road
EBD_7418
50 PHYSICS
LAWS OF MOTION 51
EBD_7418
52 PHYSICS

Exercise 1 : Topic-wise MCQs


to supply the thrust needed to overcome the weight of rocket is
Topic 1: Ist, IInd & IIIrd Laws of Motion (a) 117.6 kg s–1 (b) 58.6 kg s–1
1. Newton's first law of motion describes the (c) 6 kg s–1 (d) 76.4 kg s–1
(a) energy (b) work 14. An object of mass 10 kg moves at a constant speed of
(c) inertia (d) moment of inertia 10 ms–1. A constant force, that acts for 4 sec on the object,
2. Inertia is the property of a body linked to tendency of a body gives it a speed of 2 ms–1 in opposite direction. The force
(a) to change its position acting on the object is
(b) to change its direction (a) –3 N (b) –30 N (c) 3 N (d) 30 N
(c) to change the momentum 15. A satellite in a force free space sweeps stationary
(d) to resist any change in its state interplanetary dust at a rate (dM/dt) = av. The acceleration
3. Physical independence of force is a consequence of of satellite is
(a) third law of motion (b) second law of motion -2av 2 -av 2 -av 2
(c) first law of motion (d) all of these (a) (b) (c) (d) -av 2
4. A reference frame attached to the earth M M 2M
16. A player stops a football weighting 0.5 kg which comes
(a) is an inertial frame by definition flying towards him with a velocity of 10m/s. If the impact
(b) cannot be an inertial frame because earth is revolving lasts for 1/50th sec. and the ball bounces back with a velocity
round the sun of 15 m/s, then the average force involved is
(c) is an inertial frame because Newton's laws are applicable
(d) is an inertial frame because the earth is rotating (a) 250 N (b) 1250 N (c) 500 N (d) 625 N
about its own axis 17. A large force is acting on a body for a short time. The impulse
5. The direction of impulse is imparted is equal to the change in
(a) same as that of the net force (a) acceleration (b) momentum
(b) opposite to that of the net force (c) energy (d) velocity
(c) same as that of the final velocity 18. China wares are wraped in straw of paper before packing.
(d) same as that of the initial velocity This is the application of concept of
6. A particle of mass m is moving with velocity v1, it is given (a) impulse (b) momentum
an impulse such that the velocity becomes v2 . Then (c) acceleration (d) force
magnitude of impulse is equal to 19. We can derive Newton’s
r r r r
(a) m( v2 - v1 ) (b) m( v1 - v 2 ) (a) second and third laws from the first law
r r r r (b) first and second laws from the third law
(c) m ´ (v 2 - v1 ) (d) 0.5m(v 2 - v1 )
r (c) third and first laws from the second law
7. A force F = 8î - 6 ĵ - 10k̂ newton produces an acceleration (d) All the three laws are independent of each other
of 1 ms–2 in a body. The mass of the body is 20. Swimming is possible on account of
(a) 10 kg (b) 10 2 kg (c) 10 3 kg (d) 200 kg (a) first law of motion
8. A particle of mass 0.3 kg subject to a force F = – kx with (b) second law of motion
k = 15 N/m . What will be its initial acceleration if it is released (c) third law of motion
from a point 20 cm away from the origin ? (d) newton's law of gravitation
(a) 15 m/s2 (b) 3 m/s2 (c) 10 m/s2 (d) 5 m/s2 21. A truck accelerates on a horizontal road due to the force
9. A ship of mass 3 × 107 kg initially at rest, is pulled by a force exerted by the
of 5 × 104 N through a distance of 3m. Assuming that the (a) road (b) engine
resistance due to water is negligible, the speed of the ship is (c) earth (d) driver
(a) 1.5 m/sec.(b) 60 m/sec.(c) 0.1 m/sec.(d) 5 m/sec. 22. Rocket engines lift a rocket from the earth surface,
10. When a body is stationary because hot gases with high velocity
(a) there is no force acting on it (a) push against the air
(b) the force acting on it is not in contact with it (b) push against the earth
(c) the combination of forces acting on it balances each other (c) react against the rocket and push it up
(d) the body is in vacuum (d) heat up the air which lifts the rocket.
23. Consider the following statements and select the incorrect
11. If a stone is thrown out of an accelerated train, then statement(s).
acceleration of the stone at any instant depends on I. To move a football at rest, some one must kick it.
(a) force acting on it at that instant II. To throw a stone upwards, one has to give it an upward push.
(b) acceleration of the train III. A breeze causes the branches of a tree to become
(c) Both (a) & (b) (d) None of these stationary.
12. No force is required for IV. A strong wind can move even heavy objects.
(a) an object moving in straight line with constant velocity (a) Only I (b) Only III (c) III and IV (d) I and II
(b) an object moving in circular motion 24. A cannon after firing recoils due to
(c) an object moving with constant acceleration (a) conservation of energy
(d) an object moving in elliptical path. (b) backward thrust of gases produced
13. A 600 kg rocket is set for a vertical firing. If the exhaust (c) Newton’s third law of motion
speed is 1000 ms–1 , the mass of the gas ejected per second (d) Newton’s first law of motion
LAWS OF MOTION 53

25. A man is standing at the centre of frictionless pond of ice. 34. A block of mass m is placed on a smooth wedge of
How can he get himself to the shore? inclination q. The whole system is accelerated horizontally
(a) By throwing his shirt in vertically upward direction so that the block does not slip on the wedge. The force
(b) By spitting horizontally exerted by the wedge on the block (g is acceleration due to
(c) He will wait for the ice to melt in pond gravity) will be
(d) Unable to get at the shore (a) mg/cos q (b) mg cos q (c) mg sin q (d) mg
26. Law of conservation of momentum follows from 35. The net force on a rain drop falling down with a constant
(a) Newton's first law of motion speed is ________
(b) Newton's second law of motion (a) weight of drop W (b) viscous drag of air F
(c) Newton's third law of motion (c) W + F + force of buoyany (d) zero
(d) Both (b) & (c) 36. A body is imparted motion from rest to move in a straight
27. A body whose momentum is constant must have constant line. If it is then obstructed by an opposite force, then
(a) velocity (b) force (a) the body may necessarily change direction
(c) acceleration (d) All of these (b) the body is sure to slow down
28. In an explosion, a body breaks up into two pieces of unequal (c) the body will necessarily continue to move in the
masses. In this same direction at the same speed
(a) both parts will have numerically equal momentum (d) None of these
(b) lighter part will have more momentum 37. If n bullets each of mass m are fired with a velocity v per
(c) heavier part will have more momentum second from a machine gun, the force required to hold the
(d) both parts will have equal velocity gun in position is
29. A block of mass 4 kg is suspended mv mv
(a) (n + 1) mv (b) 2 (c) (d) mnv
through two light spring balances A n n
A 38. A ball of mass 10 g moving perpendicular to the plane of the
and B. Then A and B will read wall strikes it and rebounds in the same line with the same
respectively : velocity. If the impulse experienced by the wall is 0.54 Ns,
(a) 4 kg and zero kg the velocity of the ball is
B
(b) zero kg and 4 kg (a) 27 ms–1 (b) 3.7 ms–1 (c) 54 ms–1 (d) 37 ms–1
39. Two bodies of masses 1 kg and 2 kg moving with same
(c) 4 kg and 4 kg velocities are stopped by the same force. Then the ratio of
4kg
(d) 2 kg and 2 kg their stopping distances is
30. An explosion breaks a rock into three parts in a horizontal (a) 1 : 2 (b) 2 : 1 (c) 2 : 1 (d) 1 : 2
plane. Two of them go off at right angles to each other. The 40. A hammer weighing 3 kg strikes the head of a nail with a
first part of mass 1 kg moves with a speed of 12 ms–1 and the speed of 2 ms–1 drives it by l cm into the wall. The impulse
second part of mass 2 kg moves with speed 8 ms–1. If the imparted to the wall is
third part flies off with speed 4 ms–1 then its mass is (a) 6Ns (b) 3Ns (c) 2Ns (d) l2 Ns
(a) 5 kg (b) 7 kg (c) 17 kg (d) 3 kg
41. The force required to stop a car of mass 800 kg, moving at a
31. A stationary body of mass 3 kg explodes into three equal
pieces. Two of the pieces fly off in two mutually speed of 20 ms–1 over a distance of 25 m in 2.5 sec is
perpendicular directions, one with a velocity of 3iˆ ms - 1 (a) 1200 N (b) 6400 N (c) 1600 N (d) 1800 N
and the other with a velocity of 4jˆ ms - 1. If the explosion
occurs in 10–4 s, the average force acting on the third piece Topic 2: Motion of Connected Bodies, Pulley &
in newton is Equillibrium of Forces
y
ˆ ´ 10 - 4 42. For the given situation as shown in the figure, the value of
(a) (3iˆ + 4j)
q to keep the system in equilibrium will be
ˆ ´ 10 - 4 (a) 30°
(b) (3iˆ - 4j) 1 + 4ˆj T1
x q
1 + 3iˆ (b) 45°
(c) (3iˆ - 4ˆj) ´ 10 4 +
4j
)
( 3i
(– (c) 0°
(d) - (3iˆ + 4j)
ˆ ´ 104 1× T2
32. A spacecraft of mass 100 kg breaks into two when its velocity (d) 90°
is 104 m s–1. After the break, a mass of 10 kg of the space W = 60N
craft is left stationary. The velocity of the remaining part is 43. When an elevator cabin falls down, the cabin and all the
(a) 103 m s–1 (b) 11.11 × 103 m s–1 bodies fixed in the cabin are accelerated with respect to
(c) 11.11 × 102 m s–1 (d) 104 m s–1 (a) ceiling of elevator (b) floor of elevator
33. A ball is thrown up at an angle with the horizontal. Then (c) man standing on earth (d) man standing in the cabin
the total change of momentum by the instant it returns to 44. A monkey is climbing up a rope, then the tension in the rope
ground is (a) must be equal to the force applied by the monkey on
(a) acceleration due to gravity × total time of flight
the rope
(b) weight of the ball × half the time of flight
(c) weight of the ball × total time of flight (b) must be less than the force applied by the monkey on
(d) weight of the ball × horizontal range the rope.
EBD_7418
54 PHYSICS

(c) must be greater than the force applied by the monkey 51. For the system shown in figure, the correct expression is
on the rope. m3
(d) may be equal to, less than or greater the force applied m2 m1
F3 F2 F1
by the monkey on the rope.
45. Which equation holds true for the given figure?
(a) F1 – F2 = F3 F1
m 3F
(b) F1 + F2 = F3 (a) F3 = F1 + F2 (b) F3 = F + F + F
1 2 3
(c) F1 + F2 + F3 = 0 F2 m 3F m 3F
(c) F3 = m + m + m (d) F3 = m + m
(d) F2 + F3 = F1 F3 1 2 3 1 2
52. Which of the following is true about acceleration, a for the
46. If rope of lift breaks suddenly, the tension exerted by the
surface of lift (a = acceleration of lift) system?
(a) mg (b) m(g + a) (c) m(g – a) (d) 0 m2
47. A spring balance is attached to the ceiling of a lift. A man m1
hangs his bag on the spring and the spring reads 49 N, when T T B
the lift is stationary. If the lift moves downward with an F A
acceleration of 5m / s 2 , the reading of the spring balance (a) Acceleration is more in A, when force is applied on A.
will be (b) Acceleration is more in B, when force is applied on B.
(a) 24 N (b) 74 N (c) 15 N (d) 49 N (c) Acceleration is same and does not depend on whether
48. A light string ABCDE whose extremity A is fixed, has weights the force is applied on m1 or m2
W1 and W2 attached to it at B and C. It passes round a small (d) Acceleration depends on the tension in the string.
smooth peg at D carrying a weight of 300 N at the free end E 53. Two blocks of masses 2 kg and 4 kg are attached by an
as shown in figure. If in the equilibrium position, BC is inextensible light string as shown in the figure. If a force of
horizontal and AB and CD make 150° and 120° with CB. 120 N pulls the blocks vertically upward, the tension in the
Match the columns : string is (take g = 10 ms–2)
F = 120 N
(a) 20 N
4 kg
(b) 15 N
(c) 35 N
D
A
(d) 40 N 2 kg

150° 54. A solid sphere of 2 kg is suspended from a horizontal beam


120°
E by two supporting wires as shown in fig. Tension in each
B C
300 N wire is approximately (g = 10 ms–2)
w1 w2 30º 30º
(a) 30 N T T

Column I Column II (b) 20 N


(A) Tension in portion AB, TAB (1) 150 N (c) 10 N
(B) Tension in portion BC, TBC (2) 173 N (d) 5 N mg
(C) Weight, W1 (3) 260 N 55. Three identical blocks of masses m = 2 kg are drawn by a
(D) Weight, W2 (4) 87 N force F = 10. 2 N with an acceleration of 0. 6 ms-2 on a
frictionless surface, then what is the tension (in N) in the
(a) (A)®(4); (B)®(1); C®(2); (D)®(3) string between the blocks B and C?
(b) (A)®(2); (B)®(1); C®(4); (D)®(3)
(c) (A)®(3); (B)®(4); C®(1); (D)®(3) C B A F
(d) (A)®(4); (B)®(3); C®(1); (D)®(2)
49. A person of mass 60 kg is inside a lift of mass 940 kg and (a) 9.2 (b) 3.4 (c) 4 (d) 9.8
presses the button on control panel. The lift starts moving 56. A 1 kg block and a 0.5 kg block move together on a horizontal
upwards with an acceleration 1.0 m/s2. If g = 10 ms–2, the frictionless surface . Each block exerts a force of 6 N on the
tension in the supporting cable is other. The block move with a uniform acceleration of
(a) 8600 N (b) 9680 N (c) 11000 N (d) 1200 N (a) 3 ms -2 (b) 6 ms -2 F
a
50. If two masses (M & m) are connected on a horizontal plane 1 kg 0.5 kg
and a force is applied on the combination, then the tension (c) 9 ms -2 (d) 12 ms -2
T depends on 57. A spring balance is attached to the ceiling of a lift. A man
(a) the force applied on the system hangs his bag on the spring and the spring reads 49 N,
(b) whether force is applied on M or m when the lift is stationary. If the lift moves downward with
(c) both (a) and (b) an acceleration of 5 m/s2, the reading of the spring balance
(d) Can’t be predicted. will be
(a) 24 N (b) 74 N (c) 15 N (d) 49 N
LAWS OF MOTION 55

58. The elevator shown in fig. is descending with an acceleration 68. Which of the following statements about friction is true?
of 2 m/s2. The mass of the block A = 0.5 kg. The force exerted (a) Friction can be reduced to zero.
by the block A on block B is (b) Frictional force cannot accelerate a body.
(c) Frictional force is proportional to the area of contact
(a) 2 N between the two surfaces.
2 m/s2 (d) Kinetic friction is always greater than rolling friction.
(b) 4 N 69. Select the incorrect statement(s) from the following.
A
I. Limiting friction is always greater than the kinetic friction.
(c) 6 N II. Limiting friction is always less than the static friction.
(d) 8 N B III. Coefficient of static fiction is always greater than the
coefficient of kinetic fiction.
59. A block A of mass 7 kg is placed on a frictionless table. A (a) I only (b) I and III (c) II and III (d) I and II
thread tied to it passes over a frictionless pulley and carries 70. The acceleration of the system shown in the figure is given
a body B of mass 3 kg at the other end. The acceleration of by the expression (coefficient of friction between m1 and
the system is (given g = 10 ms–2) A surface is m)
(a) 100 ms–2 (b) 3 ms–2 (m2 - mm1) a
B (a) a =
(m1 + m2 ) m1 T
(c) 10 ms–2 (d) 30 ms–2
m1g
60. A triangular block of mass M with angles 30°, 60°, and 90° (b) a =
rests with its 30°–90° side on a horizontal table. A cubical (m1 + m2 ) T
block of mass m rests on the 60°–30° side. The acceleration (m1 + mm2 ) a
which M must have relative to the table to keep m stationary (c) a =
(m1+ m2 )g m2
relative to the triangular block assuming frictionless contact is (m1 - m2 )m
g g (d) a = m2 g
(a) g (b) (c) g (d) (m1 + m2 )g
2 3 5 71. The force required to just move a body up the inclined plane
is double the force required to just prevent the body from
Topic 3: Friction sliding down the plane. The coefficient of friction is m. The
inclination q of the plane is
61. Identify the correct statement from the following.
(a) tan–1 m (b) tan–1 (m/2) (c) tan–1 2m (d) tan–1 3m
(a) Friction depends on the area of contact.
(b) Friction depends on the volume of body. 72. The coefficient of static friction ms, between block A of
(c) Friction does not depend on the area in contact. mass 2 kg and the table as shown in the figure is 0.2. What
(d) Friction is always less than the applied force. would be the maximum mass value of block B so that the
62. The coefficient of static friction between two surfaces two blocks do not move? The string and the pulley are
depends upon assumed to be smooth and massless. (g = 10 m/s2)
(a) the normal reaction (a) 0.4 kg 2 kg
(b) the shape of the surface in contact A
(c) the magnitude of applied force (b) 2.0 kg
(d) None of these (c) 4.0 kg
63. If the applied force is doubled, then coefficient of friction is (d) 0.2 kg B
(a) halved (b) tripled
(c) doubled (d) not changed 73. A conveyor belt is moving at a constant speed of 2m/s. A box
64. When a box is in stationary position with respect to train is gently dropped on it. The coefficient of friction between
moving with acceleration, then relative motion is opposed them is µ = 0.5. The distance that the box will move relative to
by the ...X.... Which provides the same acceleration to the belt before coming to rest on it taking g = 10 ms–2, is
box as that of the train, keeping it stationary relative to the (a) 1.2 m (b) 0.6 m (c) zero (d) 0.4 m
train. Here, X refers to
(a) kinetic friction (b) static friction 74. The upper half of an inclined plane of inclination q is per-
(c) limiting friction (d) None of these fectly smooth while lower half is rough. A block starting
65. If ms, mk and mr are coefficients of static friction, kinetic from rest at the top of the plane will again come to rest at the
friction and rolling friction, then bottom, if the coefficient of friction between the block and
(a) ms < mk < mf (b) mk < mr < ms lower half of the plane is given by
(c) mr < mk < ms (d) mr = mk = ms 2 1
66. It is difficult to move a cycle with brakes on because (a) m = (b) m = 2 tan q (c) m = tan q (d) m =
(a) rolling friction opposes motion on road tan q tan q
(b) sliding friction opposes motion on road 75. A horizontal force of 10 N is necessary to just hold a block
(c) rolling friction is more than sliding friction stationary against a wall. The coefficient of friction between
(d) sliding friction is more than rolling friction the block and the wall is 0.2. The weight of the block is
67. Select the incorrect statement(s) about static friction. (a) 20 N
I. Static friction exists on its own.
II. In the absence of applied force static friction is maximum. (b) 50 N
III. Static friction is equal and opposite to the applied force (c) 100 N 10N
upto a certain limit.
(a) I only (b) II and III (c) I and III (d) I and II (d) 2 N
EBD_7418
56 PHYSICS

76. A block B is pushed momentarily along a horizontal surface blocks move together is 12 N, the maximum force that can be
with an initial velocity V. If m is the coefficient of sliding applied to B for the blocks to move together will be:
friction between B and the surface, block B will come to rest (a) 30 N (b) 25 N (c) 27 N (d) 48 N
after a time 87. The two blocks, m = 10 kg and M = 50 kg are free to move as
gm g shown. The coefficient of static friction between the blocks
(a) (b) B is 0.5 and there is no friction between M and the ground.
V V V
V V A minimum horizontal force F is applied to hold m against M
(c) (d) that is equal to
g g (m)
77. A body of mass 2 kg is placed on a horizontal surface having F m
kinetic friction 0.4 and static friction 0.5. If the force applied M Smooth
on the body is 2.5 N, then the frictional force acting on the
body will be [g = 10 ms–2]
(a) 8 N (b) 10 N (c) 20 N (d) 2.5 N (a) 100 N (b) 50 N (c) 240 N (d) 180 N
78. A marble block of mass 2 kg lying on ice when given a velocity 88. A given object takes n times as much time to slide down a
of 6 m/s is stopped by friction in 10 s. Then the coefficient of 45° rough incline as it takes to slide down a perfectly smooth
friction is (Take g = 10 ms–2) 45° incline. The coefficient of friction between the object
(a) 0.06 (b) 0.03 (c) 0.04 (d) 0.01 and the incline is
79. A block rests on a rough inclined plane making an angle of (a) (1 – 1/n2) (b) 1/(1 – n2)
30° with the horizontal. The coefficient of static friction
between the block and the plane is 0.8. If the frictional force (c) (1 - 1/ n2 ) (d) 1/ (1 - n2 )
on the block is 10 N, the mass of the block (in kg) is 89. A block of mass m = 2 kg is placed on a plank of mass M = 10 kg
(take g = 10 m / s 2 ) which is placed on a smooth horizontal plane. The coefficient
(a) 1.6 (b) 4.0 (c) 2.0 (d) 2.5 1
80. The minimum force required to start pushing a body up rough of friction between the block and the plank is m = . If a
3
(frictional coefficient m) inclined plane is F1 while the horizontal force F is applied on the plank, then find the maximum
minimum force needed to prevent it from sliding down is F2. value of F for which the block and the plank move together.
If the inclined plane makes an angle q from the horizontal (Take g = 10 m/s2)
F h/4
such that tan q = 2m then the ratio 1 is
F2 h m
(a) 1 (b) 2 (c) 3 (d) 4 m = 1/3
81. A body starts from rest on a long inclined plane of slope 45°. M F
The coefficient of friction between the body and the plane
varies as m = 0.3 x, where x is distance travelled down the (a) 30 N (b) 4 0 N
plane. The body will have maximum speed (c) 120 N (d) None of the above
(for g = 10 m/s2) when x = 90. In figure, block 1 is one-fourth the length of block 2 and
(a) 9.8 m (b) 27 m (c) 12 m (d) 3.33 m weighs one-fourth as much. Assume that there is no friction
82. A 100 N force acts horizontally on a block of 10 kg placed on between block 2 and the surface on which it moves and that
a horizontal rough surface of coefficient of friction m = 0.5. If the coefficient of sliding friction between blocks 1 and 2 is
the acceleration due to gravity (g) is taken as 10 ms–2, the µk = 0.2. After the system is released find the distance block
acceleration of the block (in ms–2) is 2 has moved when only one-fourth of block 1 still on block
(a) 2.5 (b) 10 (c) 5 (d) 7.5 2. Block 1 and block 3 have the same mass.
83. A block of mass 0.1kg is held against a wall applying a
horizontal force of 5 N on the block. If the coefficient of m
1
friction between the block and the wall is 0.5, the magnitude
of the frictional force acting on the block is: 2
4m
(a) 2.5 N (b) 0.98 N (c) 4.9 N (d) 0.49 N m3
84. Minimum force required on a block of weight w for motion Configuration at t = 0
along rough horizontal surface of coefficient of friction m is
(a) 1/7.47 (b) 2/7.47 (c) 1/3.37 (d) 4/7.47
mw mw w 91. A small mass slides down a fixed inclined plane of inclination
(a) (b) (c) (d) ( m2+ 1)w q with the horizontal. The coefficient of friction is
1+ m 2
1+ m 1 + m2
85. The retarding acceleration of 7.35 m s–2 due to frictional µ = µ0x where x is the distance through which the mass
force stops the car of mass 400 kg travelling on a road. The slides down and µ0 is a constant. Then the speed is maximum
coefficient of friction between the tyre of the car and the after the mass covers a distance of
road is cosq sin q tan q 2 tan q
(a) (b) (c) (d)
(a) 0.55 (b) 0.75 (c) 0.70 (d) 0.65 m0 m0 m0 m0
86. A block A of mass 4 kg is placed on another block B of mass 92. The mass string system shown in the figure is in equilib-
5 kg, and the block B rests on a smooth horizontal table. If rium. If the coefficient of friction between A and the table is
the minimum force that can be applied on A so that both the 0.3, the frictional force on A is:
LAWS OF MOTION 57

99. A car takes a circular turn with a uniform speed u. If the


reaction at inner and outer wheels be denoted by R1 and R2,
then
(a) R1 = R2 (b) R1 < R2
(c) R1 > R2 (d) None of these
100. A cyclist bends while taking turn in order to
(a) reduce friction
(b) provide required centripetal force
(a) 9.8 N (b) 2.04 N (c) 1.96 N (d) 0.59 N (c) reduce apparent weight
93. Two identical smooth surfaced solid cylinders of radius r (d) reduce speed
are placed touching along their lengths on a horizontal 101. The motion of a car on a banked road is shown in the figure.
surface. A third cylinder of same material but twice the radius The centripetal force equation will be given by
of that of the cylinders is placed lengthwise on them so that
mv 2
the system remains at rest. If all three cylinders have the (a) Nsin q + fcos q =
same length, then minimum value of the coefficient of friction R N cos q
2 N q
between smaller cylinders and the surface is:
(b) f = mv v
2
1 R a =
(a) 2 R q
2
2r (c) N cos q + f = mv f
1 R q mg
(b) 2
(d) N sin q + f = mv
3
1 r r R
(c) 102. A particle is acted upon by a force of constant magnitude
3 2
which is always perpendicular to the velocity of the
(d) None of these particle. The motion of the particle takes place in a plane.
94. What is the maximum value of the force F such that the It follows that
block shown in the arrangement, does not move? (a) its velocity is constant
F (b) its acceleration is constant
1 (c) its speed is constant
m=
60º 2 3 (d) it moves in a straight line
m = 3 kg 103. A car moves at a speed of 20 ms–1 on a banked track and
describes an arc of a circle of radius 40 3 m. The angle of
banking is (g = 10 ms–2 )
(a) 20N (b) 10N (c) 12N (d) 15N (a) 25° (b) 60° (c) 45° (d) 30°
95. f is the angle of the incline when a block of mass m just 104. A particle rests on the top of a hemisphere of radius R.
starts slipping down. The distance covered by the block if Find the smallest horizontal velocity that must be
thrown up the incline with an initial speed u0 is imparted to the particle if it is to leave the hemisphere
without sliding down is
(a) u02 / 4g sin f (b) 4u02 / g sin f
(a) gR (b) 2g R (c) 3g R (d) 5g R
(c) u02 / sin f / 4g (d) 4u02 sin f / g 105. A train is moving with a speed of 36 km/hour on a curved
path of radius 200 m. If the distance between the rails is 1.5
m, the height of the outer rail over the inner rail is
Topic 4: Circular Motion & Banking of Road
(a) 1 m (b) 0.5 m (c) 0.75 m (d) 0.075 m
96. What are the effects if force is acting on a moving body in a 106. A car moving on a horizontal road may be thrown out of the
direction perpendicular to the direction of motion? road in taking a turn
(a) The speed changes uniformly (a) by the gravitational force
(b) The acceleration changes uniformly (b) due to the lack of proper centripetal force
(c) The direction of motion changes (c) due to the rolling frictional force between the tyre and road
(d) All of these (d) due to the reaction of the ground
97. On a banked road, which force is essential to provide the 107. A car sometimes overturns while taking a turn. When it
necessary centripetal force to a car to take a turn while driving overturns, it is
at the optimum speed? (a) the inner wheel which leaves the ground first
(a) Component of normal reaction (b) the outer wheel which leaves the ground first
(c) both the wheel leave the ground simultaneously
(b) Component of frictional force
(d) either wheel will leave the ground first
(c) Both (a) & (b) (d) None of these 108. On a railway curve the outside rail is laid higher than the
98. A cyclist taking turn bends inwards while a car passenger inside one so that resultant force exerted on the wheels of
taking the same turn is thrown outwards. The reason is the rail car by the tops of the rails will
(a) car is heavier than cycle (a) have a horizontal inward component
(b) car has four wheels while cycle has only two (b) be vertical
(c) difference in the speed of the two (c) equilibriate the centripetal force
(d) cyclist has to counteract the centrifugal force while in (d) be decreased
the case of car only the passenger is thrown by this force
EBD_7418
58 PHYSICS

109. A car when passes through a bridge exerts a force on it (a) 10.84 m/s (b) 9.84 m/s
which is equal to (c) 8.84 m/s (d) 6.84 m/s
Mv 2 Mv 2 117. A bridge is in the from of a semi-circle of radius 40m. The
(a) Mg + (b) greatest speed with which a motor cycle can cross the bridge
r r
without leaving the ground at the highest point is
Mv 2 (g = 10 m s–2) (frictional force is negligibly small)
(c) Mg – (d) None of these
r (a) 40 m s–1 (b) 20 m s–1 (c) 30 m s–1 (d) 15 m s–1
110. Assertion : Linear momentum of a body changes even 118. A particle tied to a string describes a vertical circular motion
when it is moving uniformly in a circle. of radius r continually. If it has a velocity 3 gr at the
Reason : In uniform circular motion, velocity remains highest point, then the ratio of the respective tensions in
constant. the string holding it at the highest and lowest points is
(a) Assertion is correct, reason is correct; reason is a (a) 4 : 3 (b) 5 : 4 (c) 1 : 4 (d) 3 : 2
correct explanation for assertion. 119. For safe turning, roads are banked with an angle q. If m =
(b) Assertion is correct, reason is correct; reason is not a mass of vehicle, r = radius of turing, v = velocity of vehicle
correct explanation for assertion then, tanq is
(c) Assertion is correct, reason is incorrect (a) µ m (b) µ v2 (c) µ r (d) µ m2
(d) Assertion is incorrect, reason is correct. 120. A person with his hands in his pockets is skating on ice at
111. The minimum velocity (in ms-1) with which a car driver must the velocity of 10 m/s and describes a circle of radius 50 m.
traverse a flat curve of radius 150 m and coefficient of friction What is his inclination with vertical
0.6 to avoid skidding is -1 æ 1 ö -1 æ 3 ö
(a) 60 (b) 30 (c) 15 (d) 25 (a) tan ç ÷ (b) tan ç ÷
112. A mass m is revolving in a vertical circle at the end of a è 10 ø è 5ø
string of length 20 cm. By how much does the tension of the - 1 æ 1ö
(c) tan -1 (1) (d) tan ç ÷
string at the lowest point exceed the tension at the topmost è 5ø
point? 121. Keeping the banking angle same, to increase the maximum
(a) 2 m g (b) 4 m g (c) 6 m g (d) 8 m g speed with which a vehicle can travel on the curved road by
113. A car is moving in a circular horizontal track of radius 10 m 10%, the radius of curvature of the road has to be changed
with a constant speed of 10 m/s. A bob is suspended from from 20 m to :
the roof of the car by a light wire of length 1.0 m. The angle (a) 16 m (b) 18 m (c) 24.2 m (d) 30.5 m
made by the wire with the vertical is 122. A body of mass 0.4 kg is whirled in a vertical circle making
p π π 2 rev/sec. If the radius of the circle is 1.2 m, then tension in
(a) 0° (b) (c) (d) the string when the body is at the top of the circle, is
3 6 4 (a) 41.56 N (b) 89.86 N (c) 109.86 N (d) 115.86 N
114. A particle of mass m rotates with a uniform angular speed w.
123. A bucket tied at the end of a 1.6 m long string is whirled in a
It is viewed from a frame rotating about the z-axis with a
vertical circle with constant speed. What should be the
uniform angular velocity w0. The centrifugal force on the
minimum speed so that the water from the bucket does not
particle is:
spill when the bucket is at the highest position?
æ w + w0 ö (a) 4 m/sec (b) 6.25 m/sec
(a) mw2r (b) mw 20 r (c) m çè ÷ a (d) zero
2 ø (c) 16 m/sec (d) None of these
115. The coefficient of friction between the rubber tyres and the 124. A particle describes a horizontal circle in a conical funnel
road way is 0.25. The maximum speed with which a car can whose inner surface is smooth with speed of 0.5 m/s. What
be driven round a curve of radius 20 m without skidding is is the height of the plane of circle from vertex of the funnel?
(g = 9.8 m/s2) (a) 0.25 cm (b) 2 cm (c) 4 cm (d) 2.5 cm
(a) 5 m/s (b) 7 m/s (c) 10 m/s (d) 14 m/s 125. An aircraft executes a horizontal loop with a speed of 150 m/
116. What will be the maximum speed of a car on a road turn of s with its wings banked at an angle of 12°. The radius of the
radius 30 m if the coefficient of friction between the tyres loop is : (g = 10 m/s2)
and the road is 0.4 (Take g = 9.8 m/s2) (a) 10.6 km (b) 9.6 km (c) 7.4 km (d) 5.8 km

Exercise 2 : Exemplar & Past Year MCQs


NCERT Exemplar MCQs (d) the centre of the ball moves with constant velocity and
the ball spins about its centre uniformly
1. A ball is travelling with uniform translatory motion. This 2. A metre scale is moving with uniform velocity. This implies
means that (a) the force acting on the scale is zero, but a torque about
(a) it is at rest the centre of mass can act on the scale
(b) the path can be a straight line or circular and the ball (b) the force acting on the scale is zero and the torque
travels with uniform speed acting about centre of mass of the scale is also zero
(c) all parts of the ball have the same velocity (magnitude (c) the total force acting on it need not be zero but the
and direction) and the velocity is constant torque on it is zero
(d) neither the force nor the torque need to be zero
LAWS OF MOTION 59

3. A cricket ball of mass 150 g has an initial velocity


r r
u = (3iˆ + 4 ˆj )ms -1 and a final velocity v = -(3iˆ + 4 ˆj )ms -1 , 6
after being hit. The change in momentum (final momentum-
3
initial momentum) is (in kgms–1)
(a) zero (b) -(0.45iˆ + 0.6 ˆj ) 0

F(N)
2 4 6 8
(c) -(0.9 ˆj + 1.2 ˆj ) (d) -5(iˆ + ˆj )iˆ –3
4. In the previous problem (3), the magnitude of the momentum
transferred during the hit is t(s)
(a) zero (b) 0.75 kg-m s–1 (a) 24 Ns (b) 20 Ns (c) 12 Ns (d) 6 Ns
(c) 1.5 kg-m s –1 (d) 1.4 kg-m s–1 12. A system consists of three masses m1, m2 and m3 connected
5. Conservation of momentum in a collision between particles by a string passing over a pulley P. The mass m1 hangs
can be understood from freely and m2 and m3 are on a rough horizontal table (the
(a) Conservation of energy coefficient of friction = m). The pulley is frictionless and of
(b) Newton's first law only negligible mass. The downward acceleration of mass m1 is :
(c) Newton's second law only (Assume m1 = m2 = m3 = m) [AIPMT 2014, A]
(d) both Newton's second and third law
g(1 – gm) m2 m3
6. A hockey player is moving northward and suddenly turns (a)
westward with the same speed to avoid an opponent. The g P
force that acts on the player is 2gm
(b)
(a) frictional force along westward 3
(b) muscle force along southward g(1 – 2m )
(c)
(c) frictional force along sotuh-west 3
g(1 – 2m ) m1
(d) muscle force along south-west (d)
7. A body of mass 2 kg travels according to the law 2
x(t ) = pt + qt 2 + rt 3 where, q = 4 ms–2, p = 3 ms–1 and r = 5 13. A mass is hanging on a spring balance which is kept in a lift.
ms–3. The force acting on the body at t = 2s is The lift ascends. The spring balance will show in its readings
(a) 136 N (b) 134 N (c) 158 N r(d) 68 N [AIIMS 2014, C]
(a) an increase
8. A body with mass 5 kg is acted upon bya force F = (-3iˆ + 4 ˆj) N .
(b) a decrease
If its initial velocity at t = 0 is v = (6iˆ - 12 ˆj ) ms -1 , the time (c) no change
at which it will just have a velocity along the y-axis is (d) a change depending on its velocity
(a) never (b) 10 s (c) 2 s (d) 15 s 14. A block of mass m is placed on a surface with a vertical cross
9. A car of mass m starts from rest and acquires a velocity x3
r
along east, v = viˆ (v > 0) in two seconds. Assuming the section given by y = . If the coefficient of friction is 0.5,
car moves with uniform acceleration, the force exerted on 6
the car is the maximum height above the ground at which the block
mv can be placed without slipping is: [JEE Main 2014, S]
(a) eastward and is exerted by the car engine 1 2 1 1
2 (a) m (b) m (c) m (d) m
mv 6 3 3 2
(b) eastward and is due to the friction on the tyres
2 15. Two stones of masses m and 2 m are whirled in horizontal
exerted by the road r
mv circles, the heavier one in radius and the lighter one in
(c) more than eastward exerted due to the engine and 2
2 radius r. The tangential speed of lighter stone is n times that
overcomes the friction of the road of the value of heavier stone when they experience same
mv
(d) exerted by the engine centripetal forces. The value of n is : [AIPMT 2015 RS, C]
2 (a) 3 (b) 4 (c) 1 (d) 2
Past Year MCQs 16. A plank with a box on it at one end is gradually raised about
10. A balloon with mass ‘m’ is descending down with an the other end. As the angle of inclination with the horizontal
acceleration ‘a’ (where a < g). How much mass should be reaches 30º the box starts to slip and slides 4.0 m down the
plank in 4.0s. The coefficients of static and kinetic friction
removed from it so that it starts moving up with an
between the box and the plank will be, respectively :
acceleration ‘a’? [AIPMT 2014, A]
[AIPMT 2015 RS, A]
2ma 2ma ma ma
(a) g + a (b) (c) (d) g - a
g-a g+a
11. The force ‘F’ acting on a particle of mass ‘m’ is indicated by
the force-time graph shown below. The change in momentum mg
of the particle over the time interval from zero to 8 s is : q
[AIPMT 2014, A]
EBD_7418
60 PHYSICS

(a) 0.6 and 0.5 (b) 0.5 and 0.6 m 2g m 2g


(c) 0.4 and 0.3 (d) 0.6 and 0.6 (c) towards right (d) towards left
2 (m 2 - m1 ) 2 (m 2 - m1 )
17. Three blocks A, B and C of masses 4 kg, 2 kg and 1 kg
22. Given in the figure are two blocks A and B of weight 20 N
respectively, are in contact on a frictionless surface, as and 100 N, respectively. These are being pressed against a
shown. If a force of 14 N is applied on the 4 kg block then wall by a force F as shown. If the coefficient of friction
the contact force between A and B is [AIPMT 2015, A] between the blocks is 0.1 and between block B and the wall
is 0.15, the frictional force applied by the wall on block B is:
A B [JEE Main 2015, C]
C
(a) 120 N
(a) 6 N (b) 8 N (c) 18 N (d) 2 N F
A B
18. A block A of mass m1 rests on a horizontal table. A light (b) 150 N
string connected to it passes over a frictionless pulley at the
edge of table and from its other end another block B of mass (c) 100 N
m2 is suspended. The coefficient of kinetic friction between
the block and the table is µk. When the block A is sliding on (d) 80 N
the table, the tension in the string is [AIPMT 2015, S] 23. In figure, two blocks are separated by a uniform strut
(m 2 – m k m1 )g m1m 2 (1 + m k )g attached to each block with frictionless pins. Block A
(a) (m1 + m 2 ) (b) (m1 + m 2 ) weighs 400N, block B weighs 300N, and the strut AB weigh
200N. If µ = 0.25 under B, determine the minimum coefficient
m1m 2 (1 – m k )g (m 2 + m k m1 )g of friction under A to prevent motion. [BITSAT 2015, S]
(c) (m1 + m 2 ) (d) (m1 + m 2 ) (a) 0.4

//////
B
19. A ball of mass 0.5 kg moving with a velocity of 2 m/sec (b) 0.2

//////
strikes a wall normally and bounces back with the same

//////
speed. If the time of contact between the ball and the wall is (c) 0.8 30°
A

//////
one millisecond, the average force exerted by the wall on 60°
the ball is : [AIIMS 2015, C] (d) 0.1 //////////////////////////////////
(a) 2000 newton (b) 1000 newton
24. The masses of blocks A and B are m and M respectively.
(c) 5000 newton (d) 125 newton Between A and B, there is a constant frictional force F and
20. A smooth inclined plane is inclined at an angle q with B can slide on a smooth horizontal surface. A is set in motion
horizontal. A body starts from rest and slides down the with velocity while B is at rest. What is the distance moved
inclined surface. [AIIMS 2015, C] by A relative to B before they move with the same velocity?
[BITSAT 2015, S]
v0
A m
h B M
mMv20 mMv20
q (a) (b)
F(m - M) 2F ( m - M )
Then the time taken by it to reach the bottom is
mMv02 mMv 02
æ 2h ö æ 2l ö (c)
F (m + M )
(d)
2F ( M + m )
(a) çç ÷÷ (b) çç ÷÷
è g ø è g ø 25. A car is negotiating a curved road of radius R. The road is
banked at an angle q. the coefficient of friction between the
1 2h ( 2h )
(c) (d) sin q tyres of the car and the road is ms. The maximum safe velocity
sin q g g on this road is : [NEET 2016, C]
21. A constant force F = m2g/2 is applied on the block of mass m s + tan q m s + tan q
m1 as shown in fig. The string and the pulley are light and (a) gR 2 (b) gR
the surface of the table is smooth. The acceleration of m1 is 1 - ms tan q 1 - ms tan q
[AIIMS 2015, S] g ms + tan q g ms + tan q
m1 (c) (d)
R 1 - m 2 tan q R 2 1 - ms tan q
F
26. What is the minimum velocity with which a body of mass m
must enter a vertical loop of radius R so that it can complete
m2 the loop ? [NEET 2016, C]
m2g m 2g
(a) towards right (b) towards left (a) gR (b) 2gR (c) 3gR (d) 5gR
2 (m1 + m 2 ) 2 (m1 - m 2 )
LAWS OF MOTION 61

27. A smooth block is released at rest on a 45° incline and then 33. Assertion : A man and a block rest on smooth horizontal
slides a distance ‘d’. The time taken to slide is ‘n’ times as surface. The man holds a rope which is connected to block.
much to slide on rough incline than on a smooth incline. The man cannot move on the horizontal surface.
The coefficient of friction is [AIIMS 2016, S]
1 1
(a) m k = 1- 2 (b) m k = 1-
n n2
1 1
(c) m s = 1- (d) m s = 1- Reason : A man standing at rest on smooth horizontal surface
n2 n2 cannot start walking due to absence of friction (The man is
28. Block A of mass m and block B of mass 2m are placed on a only in contact with floor as shown). [AIIMS 2017, C]
fixed triangular wedge by means of a massless, inextensible
string and a frictionless pulley as shown in figure.

A B
m 2m (a) Assertion is correct, reason is correct; reason is a
correct explanation for assertion.
45° 45° (b) Assertion is correct, reason is correct; reason is not a
correct explanation for assertion
The wedge is inclined at 45° to the horizontal on both the (c) Assertion is correct, reason is incorrect
sides. If the coefficient of friction between the block A and (d) Assertion is incorrect, reason is correct.
the wedge is 2/3 and that between the block B and the wedge 34. Two blocks A and B of masses 3 m and m respectively are
is 1/3 and both the blocks A and B are released from rest, the connected by a massless and inextensible string. The whole
acceleration of A will be [BITSAT 2016, S] system is suspended by a massless spring as shown in
(a) –1 ms (b) 1.2 ms (c) 0.2 ms–2 (d) zero
–2 –2 figure. The magnitudes of acceleration of A and B
29. A frictionless wire AB is fixed on a sphere of radius R. A immediately after the string is cut, are respectively :-
very small spherical ball slips on this wire. The time taken [JEE Main 2017, C]
by this ball to slip from A to B is [BITSAT 2016, A]
g
2 gR (a) ,g (b) g, g
(a) 3
g cos q A
A 3m
cos q
(b) 2 gR . g g g
g q (c) , (d) g, m
3 3 3 B
O
R 35. Which one of the following statements is incorrect?
(c) 2 B R
g [NEET 2018, C]
(a) Rolling friction is smaller than sliding friction.
gR C
(d) (b) Limiting value of static friction is directly proportional
g cos q to normal reaction.
30. One end of string of length l is connected to a particle of mass (c) Coefficient of sliding friction has dimensions of length.
(d) Frictional force opposes the relative motion.
'm' and the other end is connected to a small peg on a smooth
36. A block of mass m is placed on a smooth inclined wedge
horizontal table. If the particle moves in circle with speed 'v' the ABC of inclination q as shown in the figure. The wedge is
net force on the particle (directed towards centre) will be given an acceleration ‘a’ towards the right. The relation
(T represents the tension in the string) : [NEET 2017, A] between a and q for the block to remain stationary on the
wedge is [NEET 2018, C]
mv2 mv 2
(a) T+ (b) T – (c) Zero (d) T A
ℓ ℓ
m
31. A hockey player is moving northward and suddenly turns
westward with the same speed to avoid an opponent. The
force that acts on the player is [AIIMS 2017, C] a
(a) frictional force along westward
(b) muscles force along southward q
(c) frictional force along south-west C B
(d) muscle force along south-west g g
32. The retarding acceleration of 7.35 ms–2 due to frictional (a) a= (b) a=
force stops the car of mass 400 kg travelling on a road. The cosec q sin q
coefficient of friction between the tyre of the car and the (c) a = g tan q (d) a = g cos q
road is [AIIMS 2017, A]
(a) 0.55 (b) 0.75 (c) 0.70 (d) 0.65
EBD_7418
62 PHYSICS

37. What is the direction of force on the wall due to the ball in 39. Assertion: Friction forces are conservative forces.
two cases shown in the figures? [AIIMS 2018, C] Reason: Friction forces are unidirectional [AIIMS 2018, C]
(a) Assertion is correct, reason is correct; reason is a
u correct explanation for assertion.
(b) Assertion is correct, reason is correct; reason is not a
30° correct explanation for assertion
u u 30° (c) Assertion is correct, reason is incorrect
(d) Assertion is incorrect, reason is correct.
u
40. Two masses m1 = 5 kg and m T
m
m2
(a) (b) m2 = 10 kg, connected by an 2

(a) In (a) force is normal to the wall and in (b) force is inextensible string over a
inclined at 30° to the normal. frictionless pulley, are moving T
(b) In (a) force is normal to the wall and in (b) force is as shown in the figure. The
inclined at 60° to the normal. m1
coefficient of friction of
(c) In (a) the force is along the wall and in (b) force is horizontal surface is 0.15. m1g
normal to the wall. The minimum weight m that should be put on top of m2 to
(d) In (a) and (b) both the force is normal to the wall. stop the motion is: [JEE Main 2018, S]
38. A boy, sitting on the topmost birth in the compartment of
a train which is just going to stop on the railway station, (a) 18.3 kg (b) 27.3 kg (c) 43.3 kg (d) 10.3 kg
drops an apple aiming at the open hand of his brother 41. If a stone of mass 0.05 kg is thrown out a window of a train
situated vertically below his own hand at a distance of moving at a constant speed of 100 km/h then magnitude of
2m. The apple will fall [AIIMS 2018, C] the net force acting on the stone is [BITSAT 2018, A]
(a) in the hand of his brother (a) 0.5 N (b) zero (c) 50 N (d) 5 N
(b) slightly away from the hand of his brother in the 42. A body of mass M hits normally a rigid wall with velocity V
direction of motion of the train and bounces back with the same velocity. The impulse
(c) slightly away from the hand of his brother opposite experienced by the body is [BITSAT 2018, C]
to the direction of motion of the train (a) MV (b) 1.5 MV (c) 2 MV (d) zero
(d) None of the above

Exercise 3 : Try If You Can


1. Two monkeys of masses 10 kg and 8 kg are moving
2g 2g 2g g
along a vertical rope which is light and (a) (b) (c) (d)
inextensible, the former climbing up with an 3 6 9 3
acceleration of 2m/s2 while the latter coming down 4. A bob is hanging over a pulley inside car
with a uniform velocity of 2m/s. Find the tension a car through a string. The second a
(in newtons). end of the string is in the hand of a a

(a) 200 N (b) 150 N person standing in the car. The car m
(c) 300 N (d) 100 N is moving with constant acceleration
2. A horizontal uniform rope of length L, resting on a a directed horizontally as shown in
frictionless horizontal surface, is pulled at one end by force figure. Other end of the string is pulled with constant
acceleration a vertically. The tension in the string is equal to
F. What is the tension in the rope at a distance l from the
end where the force is applied? (a) m g2 + a2 (b) m g 2 + a 2 - ma
æ lö æ l ö
(a) F ç 1 – ÷ (b) 2F ç1 – ÷ (c) m g 2 + a 2 + ma (d) m (g + a)
è Lø è 2L ø
F Fæ lö 5. A 20 kg block B is suspended from a cord attached to a 40 kg
(c) (d) ç1 – ÷ cart A. Find the ratio of the acceleration of block in cases (i)
L lè Lø
and (ii) shown in the figure immediately after the system is
3. A smooth ring P of mass m can slide released from rest. (neglect friction)
m
on a fixed horizontal rod. A string 60°
tied to the ring passes over a fixed P
pulley and carries a block Q of mass A A
(m/2) as shown in the figure. At an B B
instant, the string between the ring Q m/2 Case (i) Case (ii)
and the pulley makes an angle 60°
2 3 3
with the rod. The initial acceleration of the ring is (a) (b) 3 2 (c) (d)
3 2 2 2
LAWS OF MOTION 63

6. There are two blocks A and B in 10. A bead of mass m is located on a parabolic wire with its axis
contact with vertical and horizontal vertical and vertex at the origin as shown in the figure and
smooth surfaces respectively, as whose equation is x2 = 4ay. The wire frame is fixed and the
shown in the figure. Acceleration bead can slide on it without friction. The bead is released
of A and B are aA and aB respectively from point y = 4a on the frame from rest. The tangential
along their constrained direction of acceleration of the bead when it reaches the position given
motions. Relation between aA and by y = a is y
m
aB is (Assume sin 23° = 2/5). 3g g
(a) (b)
(a) 2aA = 21 aB (b) 2aA = 5aB 2 2
x
(c) 5aA = 2aB (d) 21 a A = 2aB g g
(c) (d)
7. In the figure acceleration of 5 2
bodies A, B and C are shown 11. A particle is released on a vertical smooth semicircular track
with directions. Values b and from point X so that OX makes angle q from the vertical (see
c are w.r.t. ground whereas a
figure). The normal reaction of the track on the particle
is acceleration of block A w.r.t.
wedge C. Acceleration of vanishes at point Y where OY makes angle f with the
block A w.r.t. ground is horizontal. Then: X
(a) sin f = cos f
Y
(a) (b + c ) 2 + a 2 1 q
(b) sin f = cos q
(b) c – (a + b) cos q 2
f
2
(c) sin f = cos q O
(c) (b + c) 2 + c 2 - 2(b + c).c.cos q 3
3
(d) (b + c) 2 + c 2 + 2(b + c).c.cos q (d) sin f = cos q
4
8. A rocket is fired vertically from the earth with an acceleration 12. A body starts slipping down an incline and moves half metre
of 2g, where g is the gravitational acceleration. On an inclined in half second. How long will it take to move the next half?
plane inside the rocket, making an angle q with the horizontal, (a) 0.21 sec (b) 0.44 sec (c) 1.21 sec (d) 2.11 sec
a point object of mass m is kept. The minimum coefficient of 13. If m be the coefficient of friction
friction mmin between the mass and the inclined surface such between the block and the car t,
that the mass does not move is : horizontal acceleration of the cart that
(a) tan2q (b) tanq (c) 3 tanq (d) 2 tan q is required to prevent block B from
9. Block B has a mass m and is released from rest when it is on falling is:
top of wedge A, which has a mass 3 m. Determine the (a) m /g (b) g / m (c) g (d) (m2 + 1) g
extension of the spring of force constant k while B is sliding 14. Two particles of equal mass are connected to a rope AB of
down on A. Neglect friction : negligible mass such that one is at end A and other dividing
(a) 2 mg cos q/k the length of rope in the ratio 1 : 2 from B. The rope is rotated
about end B in a horizontal plane. Ratio of tensions in the
mg
(b) cos q B smaller part to the other is (ignore effect of gravity)
2k
k (a) 4 : 3 (b) 1 : 4 (c) 1 : 2 (d) 1 : 3
mg q A
15. In figure the mass of the block M = 1 kg, m = 0.5 and
(c) sin 2q
2k sin q = 3/5. The acceleration of the block if F = 10 N is
(d) mg sin 2q/k (a) 4 m/s2
F
(b) 8 m/s2 q
(c) 12 m/s2 M
(d) 16 m/s2
EBD_7418
64 PHYSICS

ANSWER KEYS
Exercise 1 : Topic-wise MCQs
1 (c) 14 (b) 27 (a) 40 (a) 53 (d) 66 (d) 79 (c) 92 (c) 105 (d) 118 (c)
2 (d) 15 (b) 28 (a) 41 (b) 54 (b) 67 (d) 80 (c) 93 (c) 106 (b) 119 (b)
3 (c) 16 (d) 29 (c) 42 (b) 55 (b) 68 (d) 81 (d) 94 (a) 107 (a) 120 (d)
4 (b) 17 (b) 30 (a) 43 (c) 56 (d ) 69 (c) 82 (c) 95 (a) 108 (a) 121 (c )
5 (a) 18 (a) 31 (d) 44 (a) 57 (a) 70 (a) 83 (b) 96 (c) 109 (c) 122 (a)
6 (a) 19 (c) 32 (b) 45 (c) 58 (b) 71 (d) 84 (a) 97 (a) 110 (c) 123 (a)
7 (b) 20 (c) 33 (c) 46 (d) 59 (b) 72 (a) 85 (b) 98 (d) 111 (b) 124 (d)
8 (c) 21 (a) 34 (a) 47 (a) 60 (c) 73 (d) 86 (c) 99 (b) 112 (c) 125 (a)
9 (c) 22 (c) 35 (d) 48 (b) 61 (c) 74 (b) 87 (c) 100 (b) 113 (d)
10 (c) 23 (b) 36 (b) 49 (c) 62 (a) 75 (d) 88 (a) 101 (a) 114 (b)
11 (a) 24 (c) 37 (d) 50 (b) 63 (d) 76 (d) 89 (a) 102 (c) 115 (b)
12 (a) 25 (b) 38 (a) 51 (c) 64 (b) 77 (d) 90 (a) 103 (d) 116 (a)
13 (c) 26 (d) 39 (a) 52 (c) 65 (c) 78 (a) 91 (c) 104 (a) 117 (b)
Exercise 2 : Exemplar & Past Year MCQs
1 (c) 6 (c) 11 (c) 16 (a) 21 (a) 26 (d) 31 (c) 36 (c) 41 (a)
2 (b) 7 (a) 12 (c) 17 (a) 22 (a) 27 (b) 32 (b) 37 (d) 42 (c)
3 (c) 8 (b) 13 (a) 18 (b) 23 (a) 28 (d) 33 (d) 38 (b)
4 (c) 9 (b) 14 (a) 19 (a) 24 (d) 29 (c) 34 (a) 39 (b)
5 (d) 10 (a) 15 (d) 20 (c) 25 (b) 30 (d) 35 (c) 40 (b)
Exercise 3 : Try If You Can
1 (a) 3 (c) 5 (d) 7 (c) 9 (c) 11 (c) 13 (b) 15 (c)
2 (a) 4 (c) 6 (d) 8 (b) 10 (b) 12 (a) 14 (a)
6
Chapter
WORK, ENERGY AND POWER

Trend
Analysis of NEET and AIIMS (Year 2010-2018)

4
Number of Questions

3
AIPMT/NEET
2 AIIMS

0
2010 2011 2012 2013 2014 2015 2016 2017 2018
Year

Trend
Analysis of JEE Main and BITSAT (Year 2010-2018)
5

4
Number of Questions

3
JEE Main
2 BITSAT

0
2010 2011 2012 2013 2014 2015 2016 2017 2018
Year

Chapter Utility Score (CUS)


Exam Weightage Important Concepts Difficulty Level CUS (Out of 10)

NEET 5 Work done by a constant


AIIMS 5 and Variable force, Energy, 4/5 7.5/10
JEE Main 5 Power and Collisions
BITSAT 3
EBD_7418
66 PHYSICS
WORK, ENERGY AND POWER 67
EBD_7418
68 PHYSICS

Exercise 1 : Topic-wise MCQs

Topic 1: Work where t is in second. Work done by the force in 2 second is


3 8 19 5
1. When the force retards the motion of body, the work done is (a) J (b) J (c) J (d) J
8 3 5 19
(a) zero 9. A particle moves under the effect of a force F = cx from
(b) negative x = 0 to x = x1, the work done in the process is
(c) positive
(d) Positive or negative depending upon the magnitude 1 2
of force and displacement (a) cx 12 (b) cx1 (c) 2 cx12 (d) zero
2
2. A man pushes a wall and fails to displace it, he does
(a) negative work 10. A ball of mass m moves with speed V and strikes a wall
(b) positive but not maximum work having infinite mass and it returns with same speed, then
(c) no work at all the workdone by the ball on the wall is
(d) maximum positive work (a) zero (b) mvJ (c) m/v J (d) v/m J
3. A boy carrying a box on his head is walking on a level road 11. If W represents the work done, then match the two columns:
from one place to another is doing no work. This statement Column I Column II
is (A) Force is always along the velocity (1) W = 0
(a) correct (b) incorrect (B) Force is always perpendicular to (2) W < 0
(c) partly correct (d) cannot say velocity
4. No work is done if (C) Force is always perpendicular to (3) W > 0
(a) displacement is zero acceleration
(b) force is zero (D) The object is stationary but the point
(c) force and displacement are mutually perpendicular of application of the force moves on
(d) All of these the object
5. A particle is taken round a circle by application of force. (a) (A)®(1); (B)®(2); C®(3); (D)®(2)
The work done by the force is (b) (A)®(3); (B)®(1); C®(2,3); (D)®(1)
(a) positive non–zero (b) negative non–zero
(c) (A)®(2); (B)®(3); C®(1); (D)®(2)
(c) Zero (d) None of these
(d) (A)®(1); (B)®(2); C®(3); (D)®(1)
6. A small block of mass 200g is kept at the top of a an incline r
which is 10 m long and 3.2 m high. Match the columns 12. Calculate the work done on the tool by F if this
Column I Column II displacement is along the straight line y = x that connects
(A) Work done, to lift the block from (1) 6.4 J these two points.
(a) 2.50 J (b) 500 J (c) 50.6 J (d) 2 J
the ground and put it at the top r
(B) Work done to slide the block (2) 7.2 J 13. Calculate the work done on the tool by F if the tool is first
up the incline moved out along the x-axis to the point x = 3.00m, y = 0 and
then moved parallel to the y-axis to x = 3.00m, y = 3.00 m.
(C) the speed of the block at the (3) 4 m/s
(a) 67.5 J (b) 85 J (c) 102 J (d) 7.5 J
ground when left from the top
14. A particle moving in the xy plane undergoes a displacement
of the incline to fall vertically r r
(D) The speed of the block at the (4) 8 m/s of s = (2 iˆ + 3 ˆj ) while a constant force F = (5iˆ + 2 ˆj) N
ground when side along the incline acts on the particle. The work done by the force F is
(a) (A)®(2); (B)®(3); C®(1); (D)®(4) (a) 17 joule (b) 18 joule (c) 16 joule (d) 15 joule
(b) (A)®(1); (B)®(1); C®(3); (D)®(3) 15. A force F acting on an object F(N)
(c) (A)®(4); (B)®(3); C®(2); (D)®(2) varies with distance x as
(d) (A)®(1); (B)®(3); C®(1); (D)®(2) shown here. The force is in N 3
and x in m. The work done by 2
7. The work done in stretching a spring of force constant k
the force in moving the object 1
from length l1 and l2 is x(m)
1 from x = 0 to x = 6 m is 0
1 2 3 4 5 6 7
(a) k ( l 22 - l 12 ) (b) k (l 22 - l 12 ) (a) 18.0 J (b) 13.5 J (c) 9.0 J (d) 4.5 J
2
k 16. A particle describe a horizontal circle of radius 0.5 m with
(c) k (l 2 - l1 ) (d) (l 2 + l 1 ) uniform speed. The centripetal force acting is 10 N. The
2
8. A body of mass 3 kg is under a constant force which causes work done in describing a semicircle is
1 (a) zero (b) 5 J (c) 5 p J (d) 10 p J
a displacement s in metre in it, given by the relation s = t 2 ,
3
WORK, ENERGY AND POWER 69

17. A force Fx acts on a particle such t in second. The work done during the first 2 seconds is
that its position x changes as (a) 128 mJ (b) 512 mJ (c) 576 mJ (d) 144 mJ
3

Fx (N)
shown in the figure. The work 27. A boy pushes a toy box 2.0 m along the floor by means of a
done by the particle as it moves force of 10 N directed downward at an angle of 60º to the
from x = 0 to 20 m is 0 5 x(m) 15 20 horizontal. The work done by the boy is
(a) 37.5 J (b) 10 J (c) 45 J (d) 22.5 J (a) 6 J (b) 8 J (c) 10 J (d) 12 J
18. Assertion : The work done in moving a body over a closed 28. A spring of force constant 800 N/m has an extension of 5
loop is zero for every force in nature. cm. The work done in extending it from 5 cm to 15 cm is
Reason : Work done depends on nature of force. (a) 16 J (b) 8 J (c) 32 J (d) 24 J.
(a) Assertion is correct, reason is correct; reason is a 29. A body moves a distance of 10 m along a straight line under
correct explanation for assertion. the action of a force of 5 newtons. If the work done is 25
(b) Assertion is correct, reason is correct; reason is not a joules, the angle which the force makes with the direction of
correct explanation for assertion motion of body is
(c) Assertion is correct, reason is incorrect (a) 0º (b) 30º (c) 60º (d) 90º
(d) Assertion is incorrect, reason is correct.
19. A force acts on a 30 g particle in such a way that the position Topic 2: Energy , Conservation of Energy & Momentum
of the particle as a function of time is given by x = 3t – 4t2 + t3, 30. According to work-energy theorem, the work done by the
where x is in metres and t is in seconds. The work done during net force on a particle is equal to the change in its
the first 4 seconds is (a) kinetic energy (b) potential energy
(a) 576mJ (b) 450mJ (c) 490mJ (d) 530mJ (c) linear momentum (d) angular momentum
20. A cord is used to lower vertically a block of mass M, 31. A light and a heavy body have equal momentum. Which
a distance d at a constant downward acceleration of g/4. one has greater K.E.?
The work done by the cord on the block is (a) The lighter body
d d d (b) The heavier body
(a) Mg (b) 3 Mg (c) -3 Mg (d) Mg d
4 4 4 (c) Both have equal K.E.
21. Assertion : A man rowing a boat upstream is at rest with (d) Data given is incomplete
respect to the bank. He is doing no external work. 32. A particle of mass m has momentum p. Its kinetic energy
Reason : Work done by constant force, W = F s cos q. will be
(a) Assertion is correct, reason is correct; reason is a
correct explanation for assertion. p2 p2
(a) mp (b) p2m (c) (d)
(b) Assertion is correct, reason is correct; reason is not a m 2m
correct explanation for assertion 33. Kinetic energy, with any reference, must be
(c) Assertion is correct, reason is incorrect (a) zero (b) positive
(d) Assertion is incorrect, reason is correct. (c) negative (d) both (b) and (c)
22. If a motorcyclist skids and stops after covering a distance 34. Work done by a conservative force is positive if
of 15 m. The stopping force acting on the motorcycle by the (a) P.E. of the body increases
road is 100 N, then the work done by the motorcycle on the (b) P.E. of the body decreases
road is (c) K.E. of the body increases
(a) 1500 J (b) –1500 J (c) 750J (d) Zero (d) K.E. of the body decreases
35. The ...X... energy V(x) of the spring is said to be zero when
23. A ball moves in a frictionless inclined table without
block and spring system is in the ...Y... position.
slipping. The work done by the table surface on the ball
is Here, X and Y refer to
(a) positive (b) negative (a) potential, equilibrium
(c) zero (d) None of these (b) kinetic, equilibrium
24. A spring of spring constant 5 × 103 N/m is stretched initially (c) mechanical, equilibrium
by 5 cm from the unstretched position. Then the work (d) vibrational, left
required to stretch it further by another 5 cm is 36. For a conservative force in one dimension, potential energy
(a) 18.75 J (b) 25.00 J (c) 6.25 J (d) 12.50 J function V(x) is related to the force F(x) as
25. A uniform force of (3iˆ + ˆj) newton acts on a particle of - dV(x) dV(x)
(a) F(x) = (b) F(x) =
mass 2 kg. The particle is displaced from position (2$i + k$ ) dx dx
–dV(x)
meter to position (4$i + 3 $j - k$ ) meter. The work done by the (c) F(x) = V(x) dx (d) F(x) = ò
dx
force on the particle is
37. If stretch in a spring of force constant k is tripled then the
(a) 6 J (b) 13 J (c) 15 J (d) 9 J
ratio of elastic potential energy in the two cases will be
26. The position of a particle of mass 4 g, acted upon by a
(a) 9 : 1 (b) 1 : 6 (c) 3 : 1 (d) 1 : 3
constant force is given by x = 4t2 + t, where x is in metre and
EBD_7418
70 PHYSICS

38. If a shell fired from a cannon, explodes in mid air, then 50. If two particles are brought near one another, the potential
(a) its total kinetic energy increases energy of the system will
(b) its total momentum increases (a) increase (b) decrease
(c) its total momentum decreases (c) remains the same (d) equal to the K.E
(d) None of these 51. Which of the following statements are incorrect?
39. Total ....X.... energy of a system is conserved, if the forces, I. If there were no friction, work need to be done to move
doing work on it, are .....Y..... . a body up an inclined plane is zero.
Here, X and Y refer to II. Kinetic energy.
(a) conservative, mechanical 1
(b) mechanical, conservative Ek = mv2
2
(c) mechanical, non-conservative III. As the angle of inclination is increased, the normal
(d) kinetic, conservative reaction on the body placed on it increases.
40. A bullet is fired and gets embedded in block kept on table. IV. A duster weighing 0.5 kg is pressed against a vertical
If table is frictionless, then board with a force of 11 N. If the coefficient of friction
(a) kinetic energy gets conserved is 0.5, the work done in rubbing it upward through a
(b) potential energy gets conserved distance of 10 cm is 0.55J.
(c) momentum gets conserved (a) I and II (b) I, II and IV
(d) both (a) and (c) (c) I, III and IV (d) I, II, III and IV
41. The speed of an object of mass m dropped from an inclined 52. The K.E. acquired by a mass m in travelling a certain distance
plane (frictionless), at the bottom of the plane, depends on: d, starting form rest, under the action of a constant force is
(a) height of the plane above the ground directly proportional to
(b) angle of inclination of the plane (a) m (b) m
(c) mass of the object 1
(d) All of these (c) (d) independent of m
m
42. One man takes 1 min. to raise a box to a height of 1 metre and 53. A running man has half the kinetic energy of that of a boy
another man takes 1/2 min. to do so. The energy of the of half of his mass. The man speeds up by 1m/s so as to
(a) two is different (b) two is same
have same K.E. as that of the boy. The original speed of the
(c) first is more (d) second is more
man will be
43. If the momentum of a body is increased by 50%, then the
percentage increase in its kinetic energy is (a) 2 m/s (b) ( 2 - 1) m/s
(a) 50% (b) 100% (c) 125% (d) 200% 1 1
44. A metallic wire of length L metre extends by l metre when (c) m/s (d) m/s
stretched by suspending a weight Mg from it. The mechanical
( 2 - 1) 2
energy stored in the wire is 54. A steel ball of mass 5g is thrown downward with velocity 10
m/s from height 19.5 m. It penetrates sand by 50 cm. The
Mg l Mg l change in mechanical energy will be (g = 10 m/s2)
(a) 2 Mg l (b) Mg l (c) (b)
2 4 (a) 1J (b) 1.25 J (c) 1.5 J (d) 1.75 J
45. A ball dropped from a height of 2m reaches to a height of 55. The figure gives the potential
1.5m before hitting the ground. Then the percentage of energy function U(x) for a
potential energy lost is system in which a particle is in
(a) 25 (b) 30 (c) 50 (d) 100 one-dimensional motion. In
46. A body starts from rest and acquires a velocity V in time T. which region the magnitude of
The work done on the body in time t will be proportional to the force on the particle is
V V2 2 V2 V2 2 greatest :
(a) t (b) t (c) t (d) t (a) OA (b) AB (c) BC (d) CD
T T T2 T2
56. A ball is allowed to fall from a height of 10 m. If there is 40%
47. A body of mass 0.5 kg travels in a straight line with velocity loss of energy due to air friction, then velocity of the ball
v = 5x3/2. The work done by the net force during the when it hit the ground is
displacement from x = 0 to x = 2m is
(a) 25 J (b) 50 J (c) 75 J (d) 100 J (a) 190 m/s (b) 180 m/s
48. Four particles given, have same momentum. Which has
maximum kinetic energy (c) 150 m/s (d) 120 m/s
(a) Proton (b) Electron 57. In the non-relativistic region, if the momentum, is increase
(c) Deutron (d) a-particles by 100% , the percentage increase in kinetic energy is
49. The potential energy of a system increases if work is done (a) 100 (b) 200 (c) 300 (d) 400
(a) upon the system by a non conservative force 58. Two bodies A and B having masses in the ratio of 3 : 1
(b) by the system against a conservative force possess the same kinetic energy. The ratio of linear
(c) by the system against a non conservative force momentum of B to A is
(d) upon the system by a conservative force
(a) 1 : 3 (b) 3 : 1 (c) 1: 3 (d) 3 :1
WORK, ENERGY AND POWER 71

59. A particle is placed at the origin and a force F = kx is acting 68. A crate is pushed horizontally with 100 N across a 5 m floor.
on it (where k is positive constant). If U(0) = 0, the graph of If the frictional force between the crate and the floor is 40 N,
U(x) versus x will be (where U is the potential energy then the kinetic energy gained by the crate is
function) : (a) 200 J (b) 240 J (c) 250 J (d) 300 J
69. A particle moves in a straight line with retardation
proportional to its displacement. Its loss of kinetic energy
for any displacement x is proportional to
(a) (b) (a) x2 (b) ex (c) x (d) loge x
70. A rod of mass m and length l is made to stand at an angle
of 60° with the vertical. Potential energy of the rod in this
position is
mgl mgl mgl
(c) (d) (a) mg l (b) (c) (d)
2 3 4
71. A mass of m kg moving with a speed of
60. Figur e shows a bob of mass m C mg 1.5 m/s on a horizontal smooth surface,
suspended from a string of length L. collides with a nearly weightless
TC
The velocity is V0 at A, then the spring of force constant k = 50 N/m. If the maximum
potential energy of the system is compression of the spring is 0.15 m, the value of mass m is
B
_______ at the lowest point A. (a) 0.5 kg (b) 0.15 kg (c) 0.12 kg (d) 1.5 kg
1 72. A 2 kg block slides on a horizontal floor with a speed of 4m/s.
(a) mv 02 (b) mgh TA
2 A
It strikes a uncompressed spring, and compresses it till the
VO
-1 2 block is motionless. The kinetic friction force is 15N and
(c) mv 0 (d) zero mg
2 spring constant is 10,000 N/m. The spring compresses by
61. Calculate the K.E and P.E. of the ball half way up, when a (a) 8.5 cm (b) 5.5 cm (c) 2.5 cm (d) 11.0 cm
ball of mass 0.1 kg is thrown vertically upwards with an
æ kp ö
initial speed of 20 ms–1. 73. Two spring P and Q of force constant kp and k q ç k q = 2 ÷
(a) 10 J, 20 J (b) 10 J, 10 J è ø
(c) 15 J, 8 J (d) 8 J, 16 J are stretched by applying forces of equal magnitude. If the
62. A small body is projected in a direction inclined at 45º to the energy stored in Q is E, then the energy stored in P is
horizontal with kinetic energy K. At the top of its flight, its (a) E (b) 2E (c) E /8 (d) E /2
kinetic energy will be 74. Two springs have their force constant as k 1 and
k2 (k1 > k2). When they are stretched by the same force
(a) Zero (b) K/2 (c) K/4 (d) K / 2
(a) no work is done in case of both the springs.
63. A body accelerates uniformly from rest to a velocity of 1 (b) equal work is done in case of both the springs
ms–1 in 15 seconds. The kinetic energy of the body will be (c) more work is done in case of second spring
2 (d) more work is done in case of first spring.
J when 't' is equal to [Take mass of body as 1 kg] 75. One end of a light spring of spring constant k is fixed to a
9
wall and the other end is tied to a block placed on a smooth
(a) 4s (b) 8s (c) 10s (d) 12s
horizontal surface. In a displacement, the work done by the
64. A bomb of mass 9 kg explodes into the pieces of masses spring is 1/2 k x2. The possible cases are
3 kg and 6 kg. The velocity of mass 3 kg is 16 m/s. The (a) the spring was initially compressed by a distance x,
kinetic energy of mass 6 kg in joule is was finally in its natural length
(a) 96 (b) 384 (c) 192 (d) 768 (b) it was initially stretched by a distance x and was finally
65. An athlete in the olympic games covers a distance of 100 m in its natural length
in 10 s. His kinetic energy can be estimated to be in the (c) it was initially in its natural length and finally in a
range compressed position
(a) 200 J - 500 J (b) 2 × 105 J - 3 × 105 J (d) it was initially in its natural length and finally in the
(c) 20, 000 J - 50,000 J (d) 2,000 J - 5, 000 J stretched position
66. At time t = 0s particle starts moving along the x-axis. If its 76. If the extension in a spring is increased to 4 times then the
potential energy
kinetic energy increases uniformly with time t, the net force
(a) remains the same (b) becomes 4 times
acting on it must be proportional to (c) becomes one fourth (d) becomes 16 times
1 77. A spherical ball of mass 20 kg is stationary at the top of a hill
(a) t (b) constant (c) t (d) of height 100 m. It rolls down a smooth surface to the ground,
t then climbs up another hill of height 30 m and finally rolls
67. Two bodies of masses 4 kg and 5 kg are moving with equal down to a horizontal base at a height of 20 m above the
momentum. Then the ratio of their respective kinetic energies is ground. The velocity attained by the ball is
(a) 4 : 5 (b) 2 : 1 (c) 1 : 3 (d) 5 : 4 (a) 20 m/s (b) 40 m/s (c) 10 30 m/s (d) 10 m/s
EBD_7418
72 PHYSICS

78. A long string is stretched by 2 cm and the potential energy


Topic 3: Power
is V. If the spring is stretched by 10 cm, its potential energy
will be 87. If a force F is applied on a body and it moves with a velocity
(a) V / 25 (b) V/5 (c) 5 V (d) 25 V V, the power will be
79. The potential energy of a conservative system is given by (a) F × v (b) F/v (c) F/v2 (d) F × v2
U = ay2 – by, where y represents the position of the particle 88. Which of the following must be known in order to determine
and a as well as b are constants. What is the force acting on the power output of an automobile?
the system ? (a) Final velocity and height
(a) – ay (b) – by (c) 2ay – b (d) b – 2ay (b) Mass and amount of work performed
80. A block of mass m = 0.1 kg is connected to a spring of (c) Force exerted and distance of motion
unknown spring constant k. It is compressed to a distance (d) Work performed and elapsed time of work
x from its equilibrium position and released from rest. After 89. A body of mass m is accelerated uniformly from rest to a
speed v in a time T. The instantaneous power delivered to
æ xö the body as a function of time is given by
approaching half the distance ç ÷ from equilibrium
è 2ø
position, it hits another block and comes to rest momentarily, mv 2 2 mv 2
(a) .t (b) .t
whilethe other block moves with a velocity 3 ms–1. T2 T2
The total initial energy of the spring is 1 mv 2 2 1 mv 2
(a) 0.3 J (b) 0.6 J (c) 0.8 J (d) 1.5 J (c) .t (d) .t
2 T2 2 T2
81. Figure here shows the frictional force versus displacement
90. A car of mass m starts from rest and accelerates so that the
for a particle in motion. The loss of kinetic energy in travelling
over s = 0 to 20 m will be instantaneous power delivered to the car has a constant
f(N) magnitude po. The instantaneous velocity of this car is
(a) 225 J 15 proportiourl to:
(b) 200 J 10
(a) t 2 p o (b) t 1 2
5 t
(c) 250 J (c) t –1 2 (d)
0 x(m) m
(d) 750 J 0 5 10 20
91. A vehicle is moving with a uniform velocity on a smooth
82. The kinetic energy of particle moving along a circle of radius horizontal road, then power delivered by its engine must be
R depends upon the distance covered S and is given by K = (a) uniform (b) increasing
aS where a is a constant. Then the force acting on the particle
(c) decreasing (d) zero
is
92. How much water, a pump of 2 kW can raise in one minute to
aS 2(aS) 2 aS 2 2aS a height of 10 m, take g = 10 m/s2?
(a) (b) (c) (d)
R R R2 R (a) 1000 (b) 1200 (c) 100 (d) 2000
83. A body falls freely under gravity. Its velocity is v when it 93. The engine of a vehicle delivers constant power. If the
has lost potential energy equal to U. What is the mass of vehicle is moving up the inclined plane then, its velocity,
the body? (a) must remain constant
(a) U2 /v2 (b) 2U2/v2 (c) 2U/v2 (d) U /v2 (b) must increase
84. A particle is moving in a circle of radius r under the action (c) must decrease
of a force F = ar2 which is directed towards centre of the circle. (d) may increase, decrease or remain same.
Total mechanical energy (kinetic energy + potential energy) of
94. A body is moved along a straight line by a machine
the particle is (take potential energy = 0 for r = 0)
delivering a constant power. The distance moved by the
1 3 5 3 4 3
(a) ar (b) ar (c) αr (d) ar3 body in time ‘t’ is proportional to
2 6 3
85. When a body is projected vertically up from the ground (a) t 3/4 (b) t3/2 (c) t 1/4 (d) t1/2
with certain velocity, its potential energy and kinetic energy 95. A body projected vertically from the earth reaches a height
at a point A are in the ratio 2 : 3. If the same body is projected equal to earth's radius before returning to the earth. The
with double the previous velocity, then at the same point A power exerted by the gravitational force is greatest
the ratio of its potential energy to kinetic energy is (a) at the highest position of the body
(a) 9 : 1 (b) 2 : 9 (c) 1 : 9 (d) 9 : 2 (b) at the instant just before the body hits the earth
86. A uniform chain of length 2 m and mass 0.1 kg overhangs a (c) it remains constant all through
smooth table with its two third part lying on the table. Find (d) at the instant just after the body is projected
the kinetic energy of the chain as it completely slips- off the 96. A body of mass 10 kg moves with a velocity v of 2 m/s along
table. a circular path of radius 8 m. The power produced by the
8 12 3 11 body will be
(a) J (b) J (c) J (d) J
9 5 7 3 (a) 10 J/s (b) 98 J/s (c) 49 J/s (d) zero
WORK, ENERGY AND POWER 73

97. Johnny and his sister Jane race up a hill. Johnny weighs 106. When after collision the deformation is not relived and the
twice as much as jane and takes twice as long as jane to two bodies move together after the collision, it is called
reach the top . Compared to Jane (a) elastic collision
(a) Johnny did more work and delivered more power. (b) inelastic collision
(b) Johnny did more work and delivered the same amount (c) perfectly inelastic collision
of power. (d) perfectly elastic collision
(c) Johnny did more work and delivered less power 107. In an inelastic collision, which of the following does not
(d) Johnny did less work and johnny delivered less power. remain conserved?
(a) Momentum
98. A constant power P is applied to a car starting from rest.
(b) kinetic energy
If v is the velocity of the car at time t, then
1 (c) Total energy
1
(a) v µ t (b) v µ (c) v µ t (d) v µ (d) Neither momentum nor kinetic energy
t t 108. The coefficient of restitution e for a perfectly elastic
99. If two persons A and B take 2 seconds and 4 seconds collision is
respectively to lift an object to the same height h, then the (a) 1 (b) 0 (c) ¥ (d) –1
ratio of their powers is 109. The coefficient of restitution e for a perfectly inelastic collision
(a) 1 : 2 (b) 1 : 1 (c) 2 : 1 (d) l : 3 is
100. A 10 H.P. motor pumps out water from a well of depth 20 m (a) 1 (b) 0 (c) ¥ (d) –1
and fills a water tank of volume 22380 litres at a height of 10 110. In case of elastic collision, at the time of impact.
m from the ground. The running time of the motor to fill the (a) total K.E. of colliding bodies is conserved.
empty water tank is (g = 10ms–2) (b) total K.E. of colliding bodies increases
(a) 5 minutes (b) 10 minutes (c) total K.E. of colliding bodies decreases
(c) 15 minutes (d) 20 minutes (d) total momentum of colliding bodies decreases.
101. If a machine gun fires n bullets per second each with kinetic 111. In elastic collision, 100% energy transfer takes place when
energy K, then the power of the machine gun is (a) m1 = m2 (b) m1 > m2
K (c) m1 < m2 (d) m1 = 2m2
(a) nK2 (b) (c) n2K (d) nK 112. A ball of mass m hits the floor making
n
102. An engineer claims to have made an engine delivering 10kW an angle q as shown in the figure. If e
power with fuel consumption of 1 g/s. The calorific value of is the coefficient of restitution, then v q q 1 v1
fuel is 2 kcal/g. This claim is which relation is true, for the velocity
(a) valid component before and after collision?
(b) invalid (a) V1 sin q = V sin q (b) V1 sin q¢ = – sin q
(c) depends on engine design (c) V1 cos q¢ = V cos q (d) V1 cos q¢ = –V cos q
(d) dependent on load 113. Assertion : If collision occurs between two elastic bodies
103. A force applied by an engine of a train of mass 2.05 × 106 kg their kinetic energy decreases during the time of collision.
changes its velocity from 5m/s to 25 m/s in 5 minutes. The Reason : During collision intermolecular space decreases
power of the engine is and hence elastic potential energy increases.
(a) 1.025 MW (b) 2.05 MW (a) Assertion is correct, reason is correct; reason is a
(c) 5 MW (d) 6 MW correct explanation for assertion.
104. A 10 m long iron chain of linear mass density 0.8 kg m –1 is (b) Assertion is correct, reason is correct; reason is not a
hanging freely from a rigid support. If g = 10 ms–2, then the correct explanation for assertion
(c) Assertion is correct, reason is incorrect
power required to left the chain upto the point of support in
(d) Assertion is incorrect, reason is correct.
10 second
114. Before a rubber ball bounces off from the floor, the ball is in
(a) 10 W (b) 20W (c) 30 W (d) 40 W contact with the floor for a fraction of second. Which of the
Topic 4: Collisions following statements is correct?
(a) Conservation of energy is not valid during this period
105. Which one of the following statements is true? (b) Conservation of energy is valid during this period
(a) Momentum is conserved in elastic collisions but not (c) As ball is compressed, kinetic energy is converted to
in inelastic collisions compressed potential energy
(b) Total kinetic energy is conserved in elastic collisions (d) None of these
but momentum is not conserved in elastic collisions 115. A rubber ball is dropped from a height of5m on a plane, where
(c) Total kinetic energy is not conserved but momentum the acceleration due to gravity is not shown. On bouncing
is conserved in inelastic collisions it rises to 1.8 m. The ball loses its velocity on bouncing by
(d) Kinetic energy and momentum both are conserved in a factor of
all types of collisions 16 2 3 9
(a) (b) (c) (d)
25 5 5 25
EBD_7418
74 PHYSICS

r
116. If two equal masses (m1 = m2) collide elastically in one 123. Two particles having the position r1 = (3iˆ + 5j) ˆ m and
dimension, where m2 is at rest and m1 moves with a velocity r r
r2 = (-5iˆ - 3j)
ˆ m move with velocities V = (4iˆ + 3j) ˆ m/s
u1, then the final velocities of two masses are r 1
and V2 = (aiˆ + 7ˆj) m / s . If the particles collide, then value
(a) V1 = 0; V2 = u1 (b) V1 = V2 = 0 of a must be
(c) V1 = 0 and V2 = –u1 (d) V1 = –u1; V2 = 0 (a) 8 (b) 6 (c) 4 (d) 2
117. A particle A suffers an oblique elastic collision with a particle 124. A block of mass m is moved
B that is at rest initially. If their masses are the same, then towards a movable wedge of
after collision mass M = km and height h with
(a) they will move in opposite directions velocity u (All the surface are
(b) A continues to move in the original direction while B smooth).
remains at rest If the block just reaches the top of the wedge, the value of u
(c) they will move in mutually perpendicular directions is
(d) A comes to rest and B starts moving in the direction of 2ghK
the original motion of A (a) 2gh (b)
1+ K
118. Assertion : In an elastic collision of two billiard balls, the
2gh(1 + K) æ 1ö
total kinetic energy is conserved during the short time of (c) (d) 2gh ç1 - ÷
collision of the balls (i.e., when they are in contact). K è K ø
Reason : Energy spent against friction follow the law of 125. A mass of 20 kg moving with a speed of 10m/s collides with
conservation of energy. another stationary mass of 5 kg. As a result of the collision,
(a) Assertion is correct, reason is correct; reason is a the two masses stick together. The kinetic energy of the
correct explanation for assertion. composite mass will be
(b) Assertion is correct, reason is correct; reason is not a (a) 600 (b) 800 (c) 1000 (d) 1200
correct explanation for assertion 126. A mass m moving horizontally (along the x-axis) with velocity
(c) Assertion is correct, reason is incorrect v collides and sticks to mass of 3m moving vertically upward
(d) Assertion is incorrect, reason is correct. (along the y-axis) with velocity 2v. The final velocity of the
119. A ball of mass m moving with a constant velocity strikes combination is
against a ball of same mass at rest. If e = coefficient of 1 ˆ 3 ˆ 1 ˆ 2 ˆ
(a) vi + v j (b) vi + v j
restitution, then what will be the ratio of velocity of two 4 2 3 3
balls after collision? 2 ˆ 1 ˆ 3 ˆ 1 ˆ
(c) vi + v j (d) vi + v j
1- e e -1 1+ e 2+e 3 3 2 4
(a) (b) (c) (d)
1+ e e +1 1- e e -1 127. A body of mass m moving with velocity v collides head on
120. A metal ball of mass 2 kg moving with a velocity of with another body of mass 2m which is initially at rest. The
36 km/h has a head on collision with a stationary ball of ratio of K. E. of colliding body before and after collision will
mass 3 kg. If after the collision, the two balls move together, be
the loss in kinetic energy due to collision is (a) 1 : 1 (b) 2 : 1 (c) 4 : 1 (d) 9 : 1
(a) 140 J (b) 100 J (c) 60 J (d) 40 J 128. An object of mass 2.0 kg makes an elastic collision with
121. A ball moving with velocity 2 m/s collides head on with another object of mass M at rest and continues to move in
another stationary ball of double the mass. If the coefficient the original direction but with one-fourth of its original
of restitution is 0.5, then their velocities (in m/s) after collision speed. What is the value of M?
will be (a) 0.75 kg (b) 1.0 kg
(a) 0, 1 (b) 1, 1 (c) 1, 0.5 (d) 0, 2 (c) 1.2 kg (d) None of these
122. The block of mass M moving on the frictionless horizontal 129. A bullet of mass 20g and moving with 600 m/s collides with
surface collides with the spring of spring constant k and a block of mass 4 kg hanging with the string. What is velocity
compresses it by length L. The maximum momentum of the of bullet when it comes out of block, if block rises to height
block after collision is 0.2 m after collision ?
(a) 200 m/s (b) 150 m/s (c) 400 m/s (d) 300 m/s
kL2
(a) (b) Mk L M 130. A block of mass 0.50 kg is moving with a speed of 2.00
2M ms–1 on a smooth surface. It strikes another mass of 1.00 kg
ML2 and then they move together as a single body. The energy
(c) (d) zero
k loss during the collision is
(a) 0.16 J (b) 1.00 J (c) 0.67 J (d) 0.34 J
WORK, ENERGY AND POWER 75

Exercise 2 : Exemplar & Past Year MCQs

NCERT Exemplar MCQs down from rest, one on each track as shown in figure.
Which of the following statement is correct?
1. An electron and a proton are moving under the influence of A
mutual forces. In calculating the change in the kinetic energy
of the system during motion, one ignores the magnetic force I II
of one on another. This is, because h
(a) the two magnetic forces are equal and opposite, so q1 q2
they produce no net effect B
(b) the magnetic forces do not work on each particle (a) Both the stones reach the bottom at the same time but
(c) the magnetic forces do equal and opposite (but non- not with the same speed
zero) work on each particle (b) Both the stones reach the bottom with the same speed
(d) the magnetic forces are necessarily negligible and stone I reaches the bottom earlier than stone II
2. A proton is kept at rest. A positively charged particle is (c) Both the stones reach the bottom with the same speed
released from rest at a distance d in its field. Consider two and stone II reaches the bottom earlier than stone I
experiments; one in which the charged particle is also a (d) Both the stones reach the bottom at different times
proton and in another, a positron. In the same time t, the and with different speeds
work done on the two moving charged particles is
8. The potential energy function for a V(x)
(a) same as the same force law is involved in the two
experiments particle executing linear SHM is
(b) less for the case of a positron, as the positron moves 1 x
given by V ( x) = kx 2 where k is –xm xm
away more rapidly and the force on it weakens 2
(c) more for the case of a positron, as the positron moves the force constant of the oscillator (Fig.). For k = 0.5 N/m,
away a larger distance the graph of V(x) versus x is shown in the figure. A particle
(d) same as the work done by charged particle on the of total energy E turns back when it reaches x = ±xm. If V
stationary proton and K indicate the PE and KE, respectively of the particle at
3. A man squatting on the ground gets straight up and stand. x = +xm, then which of the following is correct?
The force of reaction of ground on the man during the (a) V = O, K = E (b) V = E, K = O
process is (c) V < E, K = O (d) V = O, K < E
(a) constant and equal to mg in magnitude 9. Two identical ball bearings in contact 1 2 3
(b) constant and greater than mg in magnitude with each other and resting on a
(c) variable but always greater than mg frictionless table are hit head-on by
(d) at first greater than mg and later becomes equal to mg another ball bearing of the same mass v
4. A bicyclist comes to a skidding stop in 10 m. During this moving initially with a speed v as shown in figure.
process, the force on the bicycle due to the road is 200N If the collision is elastic, which of the following (figure) is a
and is directly opposed to the motion. The work done by possible result after collision?
the cycle on the road is
(a) + 2000 J (b) – 200 J 1 1 2 3
(c) zero (d) – 20,000 J (a) (b)
5. A body is falling freely under the action of gravity alone in v=0 v/2 v=0 v
vaccum. Which of the following quantities remain constant
during the fall? 1 2 3 1 2 3
(a) Kinetic energy
(c) (d)
(b) Potential energy
(c) Total mechanical energy v/3 v/1 v/2 v/3
(d) Total linear momentum 10. A body of mass 0.5 kg travels in a straight line with velocity
6. During inelastic collision between two bodies, which of the v = a x3/2 where a = 5 m–1/2s–1. The work done by the net
following quantities always remain conserved? force during its displacement from x = 0 to x = 2 m is
(a) Total kinetic energy (a) 15 J (b) 50 J (c) 10 J (d) 100 J
(b) Total mechanical energy
11. A body is moving unidirectionally under the influence of a
(c) Total linear momentum
source of constant power supplying energy. Which of the
(d) Speed of each body
diagrams shown in figure correctly shown the displacement-
7. Two inclined frictionless tracks, one gradual and the other
time curve for its motion?
steep meet at A from where two stones are allowed to slide
EBD_7418
76 PHYSICS

d d 16. In a shotput event an athlete throws the shotput of mass 10


kg with an initial speed of 1 m s–1 at 45° from a height 1.5 m
(a) (b) above ground. Assuming air resistance to be negligible and
acceleration due to gravity to be 10 m s–2, the kinetic energy
t t of the shotput when it just reaches the ground will be
d d (a) 2.5 J (b) 5.0 J (c) 52.5 J (d) 155.0 J
17. Which of the diagrams in figure correctly shows the change
(c) (d) in kinetic energy of an iron sphere falling freely in a lake
having sufficient depth to impart it a terminal velocity?
t t
12. Which of the diagrams shown in figure most closely shows
the variation in kinetic energy of the earth as it moves once KE KE
around the sun in its elliptical orbit? (a) (b)
KE KE t O t
O Depth Depth

(a) (b)
KE KE
t
(c) (d)
KE KE O t O t
Depth Depth
18. A cricket ball of mass 150 g moving with a speed of 126 km/
(c) (d) h hits at the middle of the bat, held firmly at its position by
the batsman. The ball moves straight back to the bowler
after hitting the bat. Assuming that collision between ball
13. Which of the diagrams shown in figure represents variation
and bat is completely elastic and the two remain in contact
of total mechanical energy of a pendulum oscillating in air
for 0.001s, the force that the batsman had to apply to hold
as function of time?
the bat firmly at its place would be
E E
(a) 10.5 N (b) 21 N
(a) (b) (c) 1.05 × 104 N (d) 2.1 × 104 N
t
t Past Year MCQs
E E 19. A body of mass (4m) is lying in x-y plane at rest. It suddenly
explodes into three pieces. Two pieces, each of mass (m)
move perpendicular to each other with equal speeds (v).
(c) (d)
The total kinetic energy generated due to explosion is :
t t [AIPMT 2014, A]
14. A mass of 5 kg is moving along a circular path of radius 1 m. 3
If the mass moves with 300 rev/min, its kinetic energy would (a) mv2 (b) mv 2 (c) 2 mv2 (d) 4 mv2
2
be 20. If the linear momentum is increased by 5%, the kinetic energy
(a) 250 p2 (b) 100 p2 (c) 5 p2 (d) 0 will increase by [AIIMS 2014, C]
15. A raindrop falling from a height h above ground, attains a (a) 50% (b) 100%
near terminal velocity when it has fallen through a height (c) 125% (d) 10%
(3/4)h. Which of the diagrams shown in figure correctly 21. When a rubber-band is stretched by a distance x, it exerts
shows the change in kinetic and potential energy of the restoring force of magnitude F = ax + bx2 where a and b are
drop during its fall up to the ground? constants. The work done in stretching the unstretched
h PE h PE rubber-band by L is: [JEE-Main 2014, C]

(a)
KE
(b) h/4
(a) aL2 + bL3 (b)
1
2
(aL2 + bL3 )
KE
t t aL2 bL3 1 æ aL2 bL3 ö
x (c) + (d) 2 çç 2 + 3 ÷÷
h 2 3 è ø
PE h KE
22. A small block of mass m is kept on a rough inclined surface
(c) (d) PE
of inclination q fixed in a elevator. The elevator goes up with
KE a uniform velocity v and the block does not slide on the
O t O t
WORK, ENERGY AND POWER 77

wedge. The work done by the force of friction on the block r r r r


r r r r r1 - r2 v 2 - v1
in time t will be : [BITSAT 2014, A] (c) r1 - r2 = v1 - v2 (d) r r = r r
| r1 - r2 | | v 2 - v1 |
(a) zero (b) mgvt cos2q
30. Figure here shows the frictional force versus displacement
(c) mgvt sin2q (d) mgvt sin 2q
for a particle in motion. The loss of kinetic energy in travelling
23. The heart of man pumps 5 litres of blood through the arteries
over s = 0 to 20 m will be [AIIMS 2015, A]
per minute at a pressure of 150 mm of mercury. If the density f(N)
of mercury be 13.6 ×103 kg/m3 and g = 10m/s2 then the
15
power of heart in watt is : [AIPMT 2015 RS, A]
(a) 2.35 (b) 3.0 (c) 1.50 (d) 1.70 10
24. A ball is thrown vertically downwards from a height of 20 m
with an initial velocity v0. It collides with the ground loses 5
50 percent of its energy in collision and rebounds to the
same height. The initial velocity v0 is : [AIPMT 2015 RS, A] 0 x(m)
0 5 10 20
(Take g = 10 ms–2) (a) 250 J (b) 200 J (c) 150 J (d) 10 J
(a) 20 ms–1 (b) 28 ms–1 31. A particle of mass m moving in the x direction with speed 2v
(c) 10 ms–1 (d) 14 ms–1 is hit by another particle of mass 2m moving in the y direction
25. A particle of mass m is driven by a machine that delivers a with speed v. If the collision is perfectly inelastic, the
constant power of k watts. If the particle starts from rest the percentage loss in the energy during the collision is close
to : [JEE Main 2015, C]
force on the particle at time t is [AIPMT 2015, C]
(a) 56% (b) 62% (c) 44% (d) 50%
(a) mk t –1/2 (b) 2mk t –1/2 32. An elastic string of unstretched length L and force constant
k is stretched by a small length x. It is further stretched by
1 mk –1/2 another small length y. The work done in the second
(c) mk t –1/2 (d) t
2 2 stretching is [BITSAT 2015, A]
26. Two similar springs P and Q have spring constants KP and (a) 1/2 Ky2 (b) 1/2 Ky(2x + y)
KQ, such that KP > KQ. They are stretched, first by the (c) 1/2 K(x2 + y2) (d) 1/2 k (x + y)2
same amount (case a,) then by the same force (case b). The 33. A body of mass 1 kg begins to move under the action of a
r
work done by the springs WP and WQ are related as, in time dependent force F=(2tiˆ+3t 2 ˆj) N, where î and ĵ are
case (a) and case (b), respectively [AIPMT 2015, C] unit vectors alogn x and y axis. What power will be developed
(a) WP = WQ; WP = WQ by the force at the time t? [NEET 2016, A]
(b) WP > WQ; WQ > WP (a) (2t2 + 3t3)W (b) (2t2 + 4t4)W
(c) (2t3 + 3t4) W (d) (2t3 + 3t5)W
(c) WP < WQ; WQ < WP
34. A particle of mass 10 g moves along a circle of radius 6.4 cm
(d) WP = WQ; WP > WQ with a constant tangential acceleration. What is the
27. A block of mass 10 kg, moving in x direction with a constant magnitude of this acceleration if the kinetic energy of the
speed of 10 ms–1, is subject to a retarding force F = 0.1 × J/m particle becomes equal to 8 × 10–4 J by the end of the second
during its travel from x = 20 m to 30 m. Its final KE will be: revolution after the beginning of the motion?
[AIPMT 2015, S] [NEET 2016, S]
(a) 450 J (b) 275 J (c) 250 J (d) 475 J (a) 0.1 m/s2 (b) 0.15 m/s2
28. On a frictionless surface a block of mass M moving at speed (c) 0.18 m/s2 (d) 0.2 m/s2
v collides elastically with another block of same mass M 35. A point particle of mass m, moves long the uniformly rough
which is initially at rest. After collision the first block moves track PQR as shown in the figure. The coefficient of friction,
v between the particle and the rough track equals m. The
at an angle q to its initial direction and has a speed . The
3 particle is released, from rest from the point P and it comes
second block's speed after the collision is :
to rest at a point R. The energies, lost by the ball, over the
[AIIMS 2015, S] parts, PQ and QR, of the track, are equal to each other, and
3 3 3 2 2 no energy is lost when particle changes direction from PQ
(a) v (b) v (c) v (d) v
4 2 2 3 to QR. The value of the coefficient of friction m and the
r distance x (= QR), are, respectively close to :
29. Two particles A and B, move with constant velocities v1
r r [JEE Main 2016, A, BN]
and v 2 . At the initial moment their position vectors are r1
r (a) 0.29 and 3.5 m P
and r2 respectively. The condition for particles A and B for
(b) 0.29 and 6.5 m h=2m
their collision is: [AIIMS 2015, C]
30° R
r r r r (c) 0.2 and 6.5 m
r r r r Q
(a) r1 .v1 = r2 .v 2 (b) r1 ´ v1 = r2 ´ v2 Horizontal
(d) 0.2 and 3.5 m Surface
EBD_7418
78 PHYSICS

36. A person trying to lose weight by burning fat lifts a mass of 42. A body of mass m = 10–2 kg is moving in a medium and
10 kg upto a height of 1 m 1000 times. Assume that the experiences a frictional force F = –kv2. Its intial speed is v0 = 10
potential energy lost each time he lowers the mass is 1 2
dissipated. How much fat will he use up considering the ms–1. If, after 10 s, its energy is mv0 , the value of k will be:
8
work done only when the weight is lifted up? Fat supplies [JEE Main 2017, C]
3.8 × 107 J of energy per kg which is converted to mechanical (a) 10–4 kg m–1 (b) 10–1 kg m–1 s–1
energy with a 20% efficiency rate. Take g = 9.8 ms–2 : (c) 10–3 kg m–1 (d) 10–3 kg s–1
[JEE Main 2016, S] 43. A particle of mass m moving in the x direction with speed 2v
(a) 9.89 × 10–3 kg (b) 12.89 × 10–3 kg is hit by another particle of mass 2m moving in the y direction
(c) 2.45 × 10–3 kg (d) 6.45 × 10–3 kg with speed v. If the collision is perfectly inelastic, the
37. Consider elastic collision of a particle of mass m moving percentage loss in the energy during the collision is close
with a velocity u with another particle of the same mass at to [BITSAT 2017, A]
rest. After the collision the projectile and the struck particle (a) 56% (b) 62% (c) 44% (d) 50%
move in directions making angles q1 and q2 respectively 44. A body initially at rest and
with the initial direction of motion. The sum of the angles sliding along a frictionless
q1 + q2, is : [BITSAT 2016, S] track from a height h (as shown h B
(a) 45° (b) 90° (c) 135° (d) 180° in the figure) just completes a
vL
38. Consider a drop of rain water having mass 1 g falling from a A
height of 1 km. It hits the ground with a speed of 50 m/s. vertical circle of diameter AB = D. The height h is equal to
Take 'g' constant with a value 10 m/s2. [NEET 2018, A]
The work done by the (i) gravitational force and the (ii)
resistive force of air is [NEET 2017, A] 3 5 7
(a) D (b) D (c) D (d) D
(a) (i) 1.25 J (ii) –8.25 J 2 4 5
(b) (i) 100 J (ii) 8.75 J 45. A moving block having mass m, collides with another
(c) (i) 10 J (ii) – 8.75 J stationary block having mass 4m. The lighter block comes to
(d) (i) – 10 J (ii) –8.25 J rest after collision. When the initial velocity of the lighter block
is v, then the value of coefficient of restitution (e) will be
39. Figure shows three forces applied to a trunk that moves
[NEET 2018, S]
leftward by 3 m over a smooth floor. The force magnitudes
(a) 0.5 (b) 0.25 (c) 0.4 (d) 0.8
are F1 = 5N, F2 = 9N, and F3 = 3N. The net work done on the
46. Assertion : A quick collision between two bodies is more
trunk by the three forces [AIIMS 2017, A]
violent than slow collision, even when initial and final
velocities are identical.
Reason : The rate of change of momentum determines that
the force is small or large. [AIIMS 2018, C]
(a) If both Assertion and Reason are correct and Reason
is the correct explanation of Assertion.
(b) If both Assertion and Reason are correct, but Reason
(a) 1.50 J (b) 2.40 J is not the correct explanation of Assertion.
(c) 3.00 J (d) 6.00 J (c) If Assertion is correct but Reason is incorrect.
40. Assertion : Graph between potential energy of a spring
(d) If both the Assertion and Reason are incorrect.
versus the extension or compression of the spring is a
straight line. 47. A particle is moving in a circular path of radius a under the
Reason : Potential energy of a stretched or compressed k
spring, proportional to square of extension or compression. action of an attractive potential U = - . Its total energy
2r 2
[AIIMS 2017, C] is: [JEE Main 2018, C]
(a) If both Assertion and Reason are correct and Reason
is the correct explanation of Assertion. k k 3 k
(a) - (b) (c) zero (d) - 2
(b) If both Assertion and Reason are correct, but Reason 4a 2 2a 2 2a
is not the correct explanation of Assertion. 48. In a collinear collision, a particle with an initial speed n0
(c) If Assertion is correct but Reason is incorrect. strikes a stationary particle of the same mass. If the final
total kinetic energy is 50% greater than the original kinetic
(d) If both the Assertion and Reason are incorrect.
energy, the magnitude of the relative velocity between the
41. A time dependent force F = 6t acts on a particle of mass two particles, after collision, is: [JEE Main 2018, A]
1 kg. If the particle starts from rest, the work done by the
force during the first 1 second will be [JEE Main 2017, C] n0 n0 n0
(a) (b) 2n0 (c) (d)
(a) 9 J (b) 18 J (c) 4.5 J (d) 22 J 4 2 2
WORK, ENERGY AND POWER 79

49. The mass of a hydrogen molecule is 3.32×10–27 kg. If 1023 rest, fractional loss of energy is Pc. The values of Pd and Pc
hydrogen molecules strike, per second, a fixed wall of area are respectively: [JEE Main 2018, S]
2 cm2 at an angle of 45° to the normal, and rebound elastically (a) (×89, ×28) (b) (×28, ×89)
with a speed of 103 m/s, then the pressure on the wall is (c) (0, 0) (d) (0, 1)
nearly: [JEE Main 2018, S]
3 2 51. A particle of mass m1 moving with velocity v collides with
(a) 2.35 × 10 N/m (b) 4.70 × 103 N/m2
2 2
a mass m2 at rest, then they get embedded. Just after
(c) 2.35 × 10 N/m (d) 4.70 × 102 N/m2 collision, velocity of the system [BITSAT 2018, C]
50. It is found that if a neutron suffers an elastic collinear (a) increases (b) decreases
collision with deuterium at rest, fractional loss of its energy (c) remains constant (d) becomes zero
is pd; while for its similar collision with carbon nucleus at

Exercise 3 : Try If You Can

1. A block of mass 1 kg is pulled along the curve path ACB by


1 ll 2 g v
a tangential force as shown in figure. The work done by the (a) llv 2 +
frictional force when the block moves from A to B is 2 2
(b) llgv
m = 0.2 1 3 llvg
C (c) lv +
2 2
1 3
(d) llvg + lv
A B 2
x = 10 m
6. A chain of length l and mass m lies on the surface of a
(a) 5 J (b) 10 J smooth hemisphere of radius R > l with one end tied to the
(c) 20 J (d) None of these top of the hemisphere. Potential energy of the chain with
th reference level of the top of the hemisphere is :
æ1ö
2. A bullet looses ç ÷ of its velocity passing through one mR2 g
ènø (a) mg R (b)
plank. The number of such planks that are required to stop l
the bullet can be: mR 2 g é l lù
l
(c) mg R sin (d) sin -
n2 2n 2 R l êë R R úû
(a) (b) (c) infinite (d) n
2n - 1 n -1 7. A man of mass m on an initially stationary boat gets off the
3. A mass of M kg is suspended by a weightless string. The boat by jumping to the left in an exactly horizontal direction.
horizontal force that is required to displace it untill the string Immediately after the jump, the boat of mass M, is observed to
makes an angle of 450 with the initial vertical direction is : be moving to the right at speed v. How much work did the man
(a) Mg( 2 – 1) (b) Mg ( 2 + 1) do during the jump (both on his own body and on the boat)
Mg
(c) Mg 2 (d)
2
1
( M + m )v 2
1æ M2ö 2
(a) (b) ç M+ v
4. A particle of mass m is moving in a circular path of constant 2 2è m ÷ø
radius r such that its centripetal acceleration ac is varying 1 æ Mm ö 2
with time t as ac = k2rt2 where k is a constant. The power (c) ç ÷v (d) None of these
2 è M + mø
delivered to the particles by the force acting on it is 8. The potential energy of particle in a force field is
A B
(a) 2p mk 2 r 2 t (b) mk2r2t U= - , where A and B are positive constants and r is
2 r
r
(mk 4 r 2 t 5 ) the distance of particle from the centre of the field. For stable
(c) (d) zero
3 equilibrium, the distance of the particle is
5. A uniform rope of linear mass density l and length l is (a) B / 2A (b) 2A / B (c) A / B (d) B / A
coiled on a smooth horizontal surface. One end is pulled up 9. An electric pump is used to fill an overhead tank of capacity
with constant velocity v. Then the average power applied 9m3 kept at a height of 10m above the ground. If the pump
by the external agent in pulling the entire rope just off the takes 5 minutes to fill the tank by consuming 10 kW power
horizontal surface is the efficiency of the pump should be (g = 10 ms–2)
(a) 60 % (b) 40 % (c) 20 % (d) 30%
EBD_7418
80 PHYSICS

10. A force of 250 N is required to lift a 75 kg mass through a spring. The other end of spring is fixed to the ground at a
pulley system. In order to lift the mass through 3 m, the rope horizontal distance 2 R from the base of the rod. If the mass
is released at a height 1.5 R, then the velocity of the ring as
has to be pulled through 12m. The efficiency of system is
it reaches the ground is
(a) 50% (b) 75% (c) 33% (d) 90%
11. A car of mass m starts from rest and accelerates so that the (a) gR
instantaneous power delivered to the car has a constant
(b) 2 gR
magnitude P0. The instantaneous velocity of this car is
proportional to : (c) 2gR
t
(a) t 2P 0 (b) t 1/2 (c) t –1/2 (d)
m (d) 3gR
12. A body of mass 5 kg explodes at rest into three fragments 15. A block lying on a smooth surface with spring connected to
with masses in the ratio 1 : 1 : 3. The fragments with equal it is pulled by an external force as shown. Initially the velocity
masses fly in mutually perpendicular directions with speeds of ends A and B of the spring are 4 m/s and 2 m/s
of 21 m/s. The velocity of heaviest fragment in m/s will be respectively. If the energy of the spring is increasing at the
(a) (b) 5 2 (c) 3 2 (d) rate of 20 J/sec, then the stretch in the spring is
7 2 2
13. A stationary particle explodes into two particles of masses k=100 N/m
m1 and m2 which move in opposite directions with velocities
B A
v1 and v2. The ratio of their kinetic energies E1/E2 is
(a) m1v2/m2v1 (b) m2/m1 (c) m1/m2 (d) 1 (a) 1.0 cm (b) 2.0 cm (c) 10 cm (d) 2.0 cm
14. A ring of mass m can slide over a smooth vertical rod as
shown in figure. The ring is connected to a spring of force
constant k = 4 mg / R, where 2 R is the natural length of the

ANSWER KEYS
Exercise 1 : Topic-wise MCQs
1 (b) 14 (c) 27 (c) 40 (c) 53 (c) 66 (d) 79 (d) 92 (b) 105 (c) 118 (d)
2 (c) 15 (b) 28 (b) 41 (a) 54 (b) 67 (d) 80 (b) 93 (a) 106 (c) 119 (a)
3 (a) 16 (a) 29 (c) 42 (b) 55 (d) 68 (d) 81 (a) 94 (b) 107 (b) 120 (c)
4 (d) 17 (c) 30 (a) 43 (c) 56 (d) 69 (a) 82 (d) 95 (b) 108 (a) 121 (a)
5 (c) 18 (b) 31 (a) 44 (c) 57 (c) 70 (d) 83 (c) 96 (d) 109 (b) 122 (b)
6 (b) 19 (a) 32 (d) 45 (a) 58 (c) 71 (a) 84 (b) 97 (b) 110 (c) 123 (a)
7 (b) 20 (c) 33 (b) 46 (d) 59 (a) 72 (b) 85 (c) 98 (a) 111 (a) 124 (c)
8 (b) 21 (a) 34 (b) 47 (b) 60 (d) 73 (d) 86 (a) 99 (c) 112 (a) 125 (b)
9 (b) 22 (d) 35 (a) 48 (b) 61 (b) 74 (c) 87 (a) 100 (c) 113 (b) 126 (a)
10 ( a) 23 (c) 36 (a) 49 (d) 62 (b) 75 (a) 88 (d) 101 (d) 114 (b) 127 (d)
11 (b) 24 (a) 37 (a) 50 (a) 63 (c) 76 (d) 89 (b) 102 (b) 115 (b) 128 (d)
12 ( c) 25 (d) 38 (a) 51 (c) 64 (c) 77 (b) 90 (b) 103 (b) 116 (a) 129 (a)
13 (a) 26 (c) 39 (b) 52 (d) 65 (d) 78 (d) 91 (d) 104 (d) 117 (c) 130 (c)
Exercise 2 : Exemplar & Past Year MCQs
1 (b) 7 (c) 13 (c) 19 (b) 25 (d) 31 (a) 37 (b) 43 (a) 49 (a)
2 (c) 8 (b) 14 (a) 20 (d) 26 (b) 32 (b) 38 (c) 44 (c) 50 (a)
3 (d) 9 (b) 15 (b) 21 (c) 27 (d) 33 (d) 39 (a) 45 (b) 51 (b)
4 (c) 10 (b) 16 (d) 22 (c) 28 (d) 34 (a) 40 (d) 46 (b)
5 (c) 11 (b) 17 (b) 23 (d) 29 (d) 35 (a) 41 (c) 47 (c)
6 (c) 12 (d) 18 (c) 24 (a) 30 (a) 36 (b) 42 (a) 48 (b)
Exercise 3 : Try If You Can
1 (c) 3 (a) 5 (c) 7 (b) 9 (d) 11 (b) 13 (b) 15 (c)
2 (a) 4 (b) 6 (d) 8 (b) 10 (b) 12 (a) 14 (b)
7
Chapter
SYSTEM OF PARTICLES
AND ROTATIONAL MOTION

Trend
Analysis of NEET and AIIMS (Year 2010-2018)
5

4
Number of Questions

3
AIPMT/NEET
2 AIIMS

0
2010 2011 2012 2013 2014 2015 2016 2017 2018
Year
Trend
Analysis of JEE Main and BITSAT (Year 2010-2018)
5

4
Number of Questions

3
JEE Main
2 BITSAT

0
2010 2011 2012 2013 2014 2015 2016 2017 2018
Year

Chapter Utility Score (CUS)


Exam Weightage Important Concepts Difficulty Level CUS (Out of 10)
NEET 6 Centre of mass, Torque &
AIIMS 5 Angular momentum, Moment 4.5/5 9.5/10
JEE Main 4 of Inertia & Rolling Motion
BITSAT 3
EBD_7418
82 PHYSICS
SYSTEM OF PARTICLES AND ROTATIONAL MOTION 83
EBD_7418
84 PHYSICS

Exercise 1 : Topic-wise MCQs


® ® ® ®
Topic 1: Centre of Mass, Centre of Gravity & Principle of Moments ® ® x + y (c) ® ® (d) x - y
(a) x- y (b) ( x- y)
1. The centre of mass of a body 2 2
11. Two spheres A and B of masses m and 2m and radii 2R and
(a) lies always outside the body
(b) may lie within, outside on the surface of the body R respectively are placed in contact as shown. The COM of
the system lies A
(c) lies always inside the body
(a) inside A B
(d) lies always on the surface of the body
2. Position vector of centre of mass of two particles system is (b) inside B 2R R
given by (c) at the point of contact
ur m rr - m rr ur m1rr1.m2 rr2 (d) None of these
11 2 2
(a) R = m + m (b) R = rr + rr 12. The motion of a rigid body which is not pivoted or fixed in
1 2 1 2
ur m1rr1 + m 2 rr2 ur m rr + m rr some way is either a pure ...A... or a combination of
(c) R = r r 11
(d) R = m + m
22 translation and rotation. The motion of a rigid body which
r1 + r2 1 2 is pivoted or fixed in some way is ...B...
3. The position of centre of mass of a system of particles does Here, A and B refer to
not depend upon the (a) rotation and translation
(a) mass of particles (b) translation and rotation
(b) symmetry of the body (c) translation and the combination of rotation and
(c) position of the particles translation
(d) relative distance between the particles (d) None of the above
4. The mass per unit length of a non - uniform rod of length L 13. If the resultant of all external forces is zero, then velocity of
varies as m = lx where l is constant. The centre of mass of centre of mass will be
the rod will be at ; (a) zero (b) constant
3 1 4 (c) either (a) or (b) (d) neither (a) nor (b)
(a) 2 L (b) L (c) L (d) L 14. In rotation of a rigid body about a fixed axis, every ...A...of
3 2 2 3
5. The motion of the centre of mass depends on the body moves in a ...B..., which lies in a plane ...C... to the
(a) total external forces (b) total internal forces axis and has its centre on the axis.
(c) sum of (a) and (b) (d) None of these Here, A, B and C refer to
6. The centre of mass of two particles lies on the line (a) particle, perpendicular and circle
(a) joining the particles (b) circle, particle and perpendicular
(b) perpendicular to the line joining the particles (c) particle, circle and perpendicular
(c) at any angle to this line (d) particle perpendicular and perpendicular
(d) None of these 15. Centre of mass of the earth and the moon system lies
(a) closer to the earth
7. A pulley fixed to the ceiling carries a string with blocks of
(b) closer to the moon
mass m and 3 m attached to its ends. The masses of string (c) at the mid-point of line joining the earth and the moon
and pulley are negligible. When the system is released, its (d) cannot be predicted
centre of mass moves with what acceleration ? 16. Consider the following statements and choose the correct option.
(a) 0 (b) – g/4 (c) g/2 (d) – g/2 I. Position vector of centre of mass of two particles of equal
8. In rotatory motion, linear velocities of all the particles of the mass is equal to the position vector of either particle.
body are II. Centre of mass is always at the mid-point of the line
(a) same (b) different (c) zero (d) cannot say joining two particles
y III. Centre of mass of a body can lie where there is no mass
9. Three identical spheres, each of mass 1
kg are kept as shown in figure, touching (a) I and II (b) II only (c) III only (d) I, II and III
each other, with their centres on a straight P Q R 17. There are some passengers inside a stationary railway
x
compartment. The centre of mass of the compartment itself
line. If their centres are marked
(without the passengers) is C1, while the centre of mass of
P, Q, R respectively, the distance of centre of mass of the the ‘compartment plus passengers’ system is C2. If the
system from P is passengers move about inside the compartment then
PQ + PR + QR PQ + PR (a) both C1 and C2 will move with respect to the ground
(a) (b)
3 3 (b) neither C1 nor C2 will be stationary with respect to the
PQ + QR PR + QR ground
(c) (d)
3 3 ® ®
(c) C1 will move but C2 will be stationary with respect to
10. Two identical particles are located at x and y with the ground
reference to the origin of three dimensional co-ordinate (d) C2 will move but C1 will be stationary with respect to
system. The position vector of centre of mass of the system the ground
is given by
SYSTEM OF PARTICLES AND ROTATIONAL MOTION 85

18. A stick is thrown in the air and lands on the ground at some (c) Assertion is correct, reason is incorrect
distance from the thrower. The centre of mass of the stick (d) Assertion is incorrect, reason is correct.
will move along a parabolic path 25. Two particles of mass m1 and m2 (m1 > m2) attract each
(a) in all cases other with a force inversely proportional to the square of
(b) only if the stick is uniform the distance between them. If the particles are initially held
(c) only if the stick has linear motion but no rotational at rest and then released, the centre of mass will
motion (a) move towards m1 (b) move towards m2
(d) only if the stick has a shape such that its centre of (c) remains at rest (d) None of these
mass is located at some point on it and not outside it 26. A shell following a parabolic path explodes somewhere in
19. The motion of binary stars , S1 and S2 is the combination its flight. The centre of mass of fragments will continue to
of ....X.... and ....Y.... . Here, X and Y refer to move in
s1 (a) vertical direction (b) any direction
(c) horizontal direction (d) same parabolic path
27. A man stands at one end of a boat which is stationary in
water. Neglect water resistance. The man now moves to the
other end of the boat and again becomes stationary. The
centre of mass of the ‘man plus boat’ system will remain
stationary with respect to water
s2 (a) in all cases
(a) motion of the CM and motion about the CM (b) only when the man is stationary initially and finally
(b) motion about the CM and motion of one star (c) only if the man moves without acceleration on the boat
(c) position of the CM and motion of the CM (d) only if the man and the boat have equal masses
(d) motion about CM and position of one star 28. Statement 1 : When you lean behind over the hind legs of
20. Assertion : The centre of mass of a two particle system the chair, the chair falls back after a certain angle.
lies on the line joining the two particle, being closer to the Statement 2 : Centre of mass lying outside the system
heavier particle. makes the system unstable.
Reason : Product of mass of particle and its distance from (a) Statement -1 is false, Statement-2 is true
centre of mass is numerically equal to product of mass of (b) Statement -1 is true, Statement-2 is true; Statement -2 is
other particle and its distance from centre of mass. a correct explanation for Statement-1
(a) Assertion is correct, reason is correct; reason is a (c) Statement -1 is true, Statement-2 is true; Statement -2 is
correct explanation for assertion. not a correct explanation for Statement-1
(b) Assertion is correct, reason is correct; reason is not a (d) Statement -1 is true, Statement-2 is false
correct explanation for assertion 29. A system consists of three particles, each of mass m and
(c) Assertion is correct, reason is incorrect
(d) Assertion is incorrect, reason is correct. located at (1, 1), (2, 2) and (3, 3). The co-ordinates of the
21. Consider a system of two particles having masses m 1 and centre of mass are
m2 . If the particle of mass m1 is pushed towards the centre (a) (1, 1) (b) (2, 2) (c) (3, 3) (d) (6, 6)
of mass particles through a distance d, by what distance 30. Two bodies of masses 2 kg and 4 kg are moving with
would the particle of mass m2 move so as to keep the mass velocities 2 m/s and 10 m/s respectively along same direction.
centre of particles at the original position? Then the velocity of their centre of mass will be
m2 m1 m (a) 8.1 m/s (b) 7.3 m/s (c) 6.4 m/s (d) 5.3 m/s
(a) d (b) d (c) 1 d (d) d 31. Four particles of masses m1,m2,m3 and m4 are placed at the
m1 m1 + m 2 m2
22. Three masses are placed on the x-axis : 300 g at origin, 500 g vertices A,B,C and D as respectively of a square shown.
at x = 40 cm and 400 g at x = 70 cm. The distance of the centre The COM of the system will lie at diagonal AC if
of mass from the origin is (a) m1 = m3 A m1 B m2
(a) 40 cm (b) 45 cm (c) 50 cm (d) 30 cm
23. A body A of mass M while falling vertically downwards (b) m2 = m4
1
under gravity breaks into two parts; a body B of mass M (c) m1 = m2
3 m4 m3
2 (d) m3 = m4 D C
and a body C of mass M. The centre of mass of bodies 32. In the figure shown ABC is a uniform wire. If centre of mass
3
B and C taken together shifts compared to that of body A BC
(a) does not shift of wire lies vertically below point A, then is close to :
A AB
(b) depends on height of breaking
(c) towards body B (a) 1.85 (b) 1.5
(d) towards body C
60°
24. Assertion : The centre of mass of an isolated system has (c) 1.37 (d) 3
a constant velocity. B C
Reason : If centre of mass of an isolated system is already 33. A circular disc of radius R is removed from a bigger circular
at rest, it remains at rest. disc of radius 2R such that the circumferences of the discs
(a) Assertion is correct, reason is correct; reason is a coincide. The centre of mass of the new disc is a / R form
correct explanation for assertion. the centre of the bigger disc. The value of a is
(b) Assertion is correct, reason is correct; reason is not a (a) 1/4 (b) 1/3 (c) 1/2 (d) 1/6
correct explanation for assertion
EBD_7418
86 PHYSICS

34. The centre of mass of three bodies each of mass 1 kg located (a) 30 radians/second2 (b) 1880 degree/second2
(c) 40 radians/second 2 (d) 1980 degree/second2
at the points (0, 0), (3, 0) and (0, 4) in the XY plane is
43. A wheel has a speed of 1200 revolutions per minute and is
æ4 ö æ 1 2ö æ 1 1ö æ 4ö made to slow down at a rate of 4 radians/s2. The number of
(a) çè , 1÷ø (b) çè 3 , 3 ÷ø (c) çè , ÷ø (d) çè1, ÷ø
3 2 2 3 revolutions it makes before coming to rest is
35. Two objects P and Q initially at rest move towards each (a) 143 (b) 272 (c) 314 (d) 722
other under mutual force of attraction. At the instant when 44. When a ceiling fan is switched off, its angular velocity falls
the velocity of P is v and that of Q is 2v, the velocity of to half while it makes 36 rotations. How many more rotations
centre of mass of the system is will it make before coming to rest?
(a) v (b) 3v (c) 2v (d) zero (a) 24 (b) 36 (c) 18 (d) 12
36. A uniform thin rod AB of length L has linear mass density 45. A mass is revolving in a circle which is in the plane of paper.
bx The direction of angular acceleration is
m (x) = a + , where x is measured from A. If the CM (a) upward the radius
L (b) towards the radius
æ 7ö
of the rod lies at a distance of ç ÷ L from A, then a and (c) tangential
è 12 ø (d) at right angle to angular velocity
b are related as : 46. In a bicycle, the radius of rear wheel is twice the radius of
(a) a = 2b (b) 2a = b (c) a = b (d) 3a = 2b front wheel. It rf and rr are the radii and vf and vr are the
speeds of topmost points of wheels then
Topic 2: Angular Displacement, Velocity and Acceleration (a) vr = 2vf (b) vf = 2vr (c) vf = vr (d) vf = 4vr
37. Which of the following is incorrect? 47. A wheel is rotating at 1800 rpm about its own axis. When
r r
(a) vr = w ´r (b) ar = vr ´ rr the power is switched off, it comes to rest in 2 minutes.
Then the angular retardation in rad s–1 is
dw p p
(c) a = (d) None of these (a) 2p (b) p (c) (d)
dt
2 4
38. Concrete mixture is made by mixing cement, stone and sand
in a rotating cylindrical drum. If the drum rotates too fast, Topic 3: Torque, Couple & Angular Momentum
the ingredients remain stuck to the wall of the drum and
48. Which component of force contributes to the torque?
proper mixing of ingredients does not take place. The
(a) Radial component (b) Transverse component
maximum rotational speed of the drum in revolutions per (c) Both (a) and (b) (d) Either radial or transverse
minute (rpm) to ensure proper mixing is close to : 49. A couple produces
(Take the radius of the drum to be 1.25 m and its axle to be (a) linear motion (b) rotational motion
horizontal): (c) both (a) and (b) (d) neither (a) nor (b)
(a) 27.0 (b) 0.4 (c) 1.3 (d) 8.0 50. Which of the following statements about angular momentum
39. A rod PQ of mass M and length L is hinged at end P. The rod is correct?
is kept horizontal by a massless string tied to point Q as (a) It is directly proportional to moment of inertia
shown in figure. When string is cut, the initial angular (b) It is a scalar quantity
acceleration of the rod is (c) both (a) and (b)
(a) g /L (d) None of these
(b) 2g/L 51. The wide handle of screw is based upon
(a) Newton’s second law of motion
2g (b) law of conservation of linear momentum
(c) (c) turning moment of force
3L
(d) None of these
3g 52. Which of the following is an expression for angular momentum?
(d) r r r r
2L r r r r r r r P
40. Angular velocity of each particle of a rotating rigid body (a) P = L ´ r (b) r = P ´ L (c) L = r (d) L = r ´ P
r
about axis of rotation is 53. Torque on a rotating body decreases, then its angular
(a) same (b) different velocity
(c) depends on relative position (a) decreases (b) increases
(d) None of these (c) remains constant (d) becomes zero
41. A wheel rotates with a constant acceleration of 54. When a mass is rotating in a plane about a fixed point, its
2.0 radian/sec2. If the wheel starts from rest, the number of angular momentum is directed along the
(a) radius of orbit
revolutions it makes in the first ten seconds will be approximately
(b) tangent to the orbit
(a) 8 (b) 16 (c) 24 (d) 32 (c) line parallel to plane of rotation
42. The wheel of a car is rotating at the rate of 1200 revolutions (d) line perpendicular to plane of rotation
per minute. On pressing the accelerator for 10 seconds. It 55. Assertion : It is harder to open and shut the door if we
starts rotating at 4500 revolutions per minute. The angular apply force near the hinge.
acceleration of the wheel is
SYSTEM OF PARTICLES AND ROTATIONAL MOTION 87

Reason : Torque is maximum at hinge of the door. 64. Two identical discs of mass m and radius r are arranged as
(a) Assertion is correct, reason is correct; reason is a shown in the figure. If a is the angular acceleration of the
correct explanation for assertion. lower disc and acm is acceleration of centre of mass of the
(b) Assertion is correct, reason is correct; reason is not a lower disc, then relation between acm, a and r is
correct explanation for assertion
(c) Assertion is correct, reason is incorrect (a) acm = a/r
(d) Assertion is incorrect, reason is correct. (b) acm = 2ar
56. A particle moving in a circular path has an angular momentum
of L. If the frequency of rotation is halved, then its angular (c) acm = a r
momentum becomes (d) None of these
L L L 65. A stone of mass m tied to a string of length l is rotating
(a) (b) L (c) (d)
2 3 4 along a circular path with constant speed v. The torque on
57. Consider the following statements and select the correct the stone is
option. (a) mlv (b) mv/l (c) mv2/l (d) zero
I. Moment of a couple depends on the point about which 66. A particle of mass 0.2 kg is moving in a circle of radius 1 m
moment is taken. with f = (2/p) sec–1, then its angular momentum is :
II. Principle of moments holds only when parallel forces (a) 0.8 kg-m2/s (b) 2 kg-m2/s
F1 and F2 are perpendicular to the lever (c) 8 kg-m /s2 (d) 16 kg-m2/s
III. Centre of mass depends on the gravity 67. A wheel having angular momentum 2p kg-m2/s about its
IV. Centre of mass depends on the distribution of mass of vertical axis, rotates at the rate of 60 rpm about this axis, The
the body torque which can stop the wheel’s rotation in 30 sec would
(a) I and II (b) III and IV (c) I, II and III (d) IV only be
58. Assertion: An ice-skater stretches out arms-legs during
performance. p 2p p p
(a) Nm (b) Nm (c) Nm (d) Nm
Reason: Stretching out arms-legs helps the performer to 18 15 12 15
balance his or her body so that he or she does not fall.
(a) Assertion is correct, reason is correct; reason is a Topic 4: Moment of Inertia & Rotational K.E.
correct explanation for assertion.
(b) Assertion is correct, reason is correct; reason is not a 68. The moment of inertia of a ...A... body about an axis ...B... to
correct explanation for assertion its plane is equal to the sum of its moments of inertia about
(c) Assertion is correct, reason is incorrect two ...C... axes concurrent with perpendicular axis and lying
(d) Assertion is incorrect, reason is correct. in the plane of the body.
59. The instantaneous angular position of a point on a rotating Here, A, B and C refer to
wheel is given by the equation q(t) = 2t3 – 6t2. The torque (a) three dimensional, perpendicular and perpendicular
on the wheel becomes zero at (b) planar, perpendicular and parallel
(a) t = 1s (b) t = 0.5 s (c) t = 0.25 s (d) t = 2s (c) planar, perpendicular and perpendicular
60. A disc is given a linear velocity on a rough horizontal surface (d) three dimensional, parallel and perpendicular
then its angular momentum is 69. The moment of inertia of a uniform circular disc of radius ‘R’
(a) conserved about COM only and mass ‘M’ about an axis passing from the edge of the disc
(b) conserved about the point of contact only and normal to the disc is
(c) conserved about all the points 1 3 7
(a) MR2 (b) MR 2 (c) MR 2 (d) MR 2
(d) not conserved about any point. 2 2 2
r r 70. Moment of inertia does not depend upon
61. A force F = a ˆi + 3jˆ + 6kˆ is acting at a point r = 2iˆ - 6ˆj - 12kˆ . (a) distribution of mass
The value of a for which angular momentum about origin is (b) axis of rotation
conserved is (c) point of application of force
(a) 2 (b) zero (c) 1 (d) –1 (d) None of these
62. A particle of mass 2 kg is on a smooth horizontal table and 71. Moment of inertia of a circular wire of mass M and radius R
moves in a circular path of radius 0.6 m. The height of the about its diameter is
table from the ground is 0.8 m. If the angular speed of the (a) MR2/2 (b) MR2 (c) 2MR2 (d) MR2/4.
particle is 12 rad s–1, the magnitude of its angular momentum 72. Moment of inertia of a hollow cylinder of mass M and radius
about a point on the ground right under the centre of the r about its own axis is
circle is 2 2 1 2
(a) 14.4 kg m2s–1 (b) 8.64 kg m2s–1 (a) Mr 2 (b) Mr 2 (c) Mr (d) Mr2
2 –1 3 5 3
(c) 20.16 kg m s (d) 11.52 kg m2s–1 73. Which of the following has the highest moment of inertia when
63. If the angular momentum of a particle of mass m rotating each of them has the same mass and the same outer radius
along a circular path of radius r with uniform speed is L, the (a) a ring about its axis, perpendicular to the plane of the ring
centripetal force acting on the particle is (b) a disc about its axis, perpendicular to the plane of the ring
L2 L2 L L2 m (c) a solid sphere about one of its diameters
(a) (b) (c) (d) (d) a spherical shell about one of its diameters
mr 2 mr mr r
EBD_7418
88 PHYSICS

74. Radius of gyration of a body depends upon (b) Assertion is correct, reason is correct; reason is not a
(a) axis of rotation (b) translational motion correct explanation for assertion
(c) shape of the body (d) area of the body (c) Assertion is correct, reason is incorrect
75. The correct relation between moment of inertia I, radius of (d) Assertion is incorrect, reason is correct.
gyration k and mass M of the body is 84. Of the two eggs which have identical sizes, shapes and
M I weights, one is raw, and other is half boiled. The ratio between
(a) K = I2M (b) K = IM2 (c) K = (d) K = the moment of inertia of the raw to the half boiled egg about
I M central axis is
76. If two circular discs A and B are of same mass but of radii r
(a) one (b) greater than one
and 2r respectively, then the moment of inertia of A is (c) less than one (d) not comparable
(a) the same as that of B (b) twice that of B 85. One solid sphere A and another hollow sphere B are of
(c) four times that of B (d) 1/4 that of B same mass and same outer radii, Their moments of inertia
77. For a given mass and size, moment of inertia of a solid disc is about their diameters are respectively IA and IB, such that
(a) more than that of a ring (a) IA = IB (b) IA > IB
(b) less than that of a ring (c) IA < IB (d) IA / IB = rA = rB
(c) equal to that of a ring Here r A and r B represent their densities.
(d) depend on the material of ring and disc 86. A wheel having moment of inertia 2 kg-m2 about its vertical
78. What is the moment of inertia of a solid sphere about its axis, rotates at the rate of 60 rpm about this axis, The torque
diameter? which can stop the wheel’s rotation in one minute would be
2 1 2 (a) 1.12 Nm (b) 0.83 Nm (c) 0.55 Nm (d) 0.21 Nm
(a) MR2 (b) MR2 (c) MR2 (d) MR2
5 5 3 87. A round disc of moment of inertia I 2 about its axis
79. If Ixy is the moment of inertia of a ring about a tangent in the perpendicular to its plane and passing through its centre is
plane of the ring and Ix¢y¢ is the moment of inertia of a ring placed over another disc of moment of inertia I1 rotating
about a tangent perpendicular to the plane of the ring then with an angular velocity w about the same axis. The final
1 3 3 angular velocity of the combination of discs is
(a) Ixy = Ix¢y¢ (b) Ixy = Ix¢y¢ (c) Ix¢y¢ = Ixy (d) Ixy = Ix¢y¢
2 4 4 I w I1w
80. The moment of inertia of a uniform circular disc (figure) is (a) (I1 + I 2 )w (b) 2 (c) w (d)
I1 I1 + I 2 I1 + I 2
maximum about an axis perpendicular to the disc and passing
through 88. The ratio of the radii of gyration of a circular disc about a
(a) B tangential axis in the plane of the disc and of a circular ring
C
of the same radius about a tangential axis in the plane of
(b) C D
the ring is
B
(c) D
A (a) 1 : Ö2 (b) 1 : 3 (c) 2 : 1 (d) Ö5 : Ö6
89. The moment of inertia of a thin uniform rod of mass M and
(d) A length L about an axis passing through its midpoint and
81. Three particles, each of mass m gram, are situated at the perpendicular to its length is I0. Its moment of inertia about
vertices of an equilateral triangle ABC of side l cm (as shown an axis passing through one of its ends and perpendicular
in the figure). The moment of inertia of the system about a to its length is
line AX perpendicular to AB and in the plane of ABC, in (a) I0 + ML2/2 (b) I0 + ML2/4
(c) I0 + 2ML 2 (d) I0 + ML2
gram-cm2 units will be X
90. Four point masses, each of value m, are placed at the corners
3
ml 2 m C of a square ABCD of side l. The moment of inertia of this
(a)
2 system about an axis passing through A and parallel to
3 BD is
ml 2 l l
(b) (a) 2ml 2 (b) 3ml 2 (c) 3ml 2 (d) ml 2
4 91. Consider a uniform square plate of side ‘a’ and mass ‘m’.
(c) 2 ml2 The moment of inertia of this plate about an axis
5 A B perpendicular to its plane and passing through one of its
(d) ml 2
4 m l m corners is
82. The moment of inertia of the rectangular plate 5 1 7 2
(a) ma 2 (b) ma 2 (c) ma 2 (d) ma 2
ABCD, (AB = 2 BC) is minimum along the axis 6 12 12 3
92. Point masses 1, 2, 3 and 4 kg are lying at the points (0, 0, 0),
(a) G H
(2, 0, 0), (0, 3, 0) and (–2, –2, 0) respectively. The moment of
(b) E F inertia of this system about X-axis will be
(c) B C (a) 43 kg m2 (b) 34 kg m2 (c) 27 kg m2 (d) 72 kg m2
(d) A C 93. The moment of inertia of a circular disc of mass M and
83. Assertion : If polar ice melts, days will be longer. radius R about an axis passing through the centre of mass is I0.
Reason : Moment of inertia decreases and thus angular The moment of inertia of another circular disc of same mass
velocity increases. and thickness but half the density about the same axis is
(a) Assertion is correct, reason is correct; reason is a I0 I0
correct explanation for assertion. (a) (b) (c) 8I0 (d) 2I0
8 4
SYSTEM OF PARTICLES AND ROTATIONAL MOTION 89

94. Consider a thin uniform square sheet made of a rigid material. (a) 2 (b) 3 (c) 4 (d) 5
If its side is ‘a’ mass m and moment of inertia I about one of 102. Four masses are fixed on a massless rod as shown in the
its diagonals, then adjoining figure. The moment of inertia about the dotted
ma 2 ma 2 ma 2 axis is about
(a) I > (b) <I<
12 24 12 0.2 m 0.2 m
2 2
ma ma
(c) I = (d) I =
24 12
95. From a solid sphere of mass M and radius R a cube of 2kg 5kg 5kg 2kg
0.4 m 0.4 m
maximum possible volume is cut. Moment of inertia of cube
about an axis passing through its center and perpendicular
(a) 2 kg m2 (b) 1 kg m2 (c) 0.5 kg m2 (d) 0.3 kg m2
to one of its faces is
103. A solid cylinder of mass 50 kg and radius 0.5 m is free to
4MR 2 4MR 2 MR 2 MR 2
(a) (b) (c) (d) rotate about the horizontal axis. A massless string is wound
9 3p 3 3p 32 2p 16 2p round the cylinder with one end attached to it and other
96. Two discs rotating about their respective axis of rotation hanging freely. Tension in the string required to produce an
with angular speeds 2 rads–1 and 5 rads–1 are brought into angular acceleration of 2 revolutions s– 2 is
contact such that their axes of rotation coincide. Now, the (a) 25 N (b) 50 N (c) 78.5 N (d) 157 N
angular speed of the system becomes 4 rads–1. If the moment
of inertia of the second disc is 1 × 10–3 kg m2, then the Topic 5: Rolling Motion
moment of inertia of the first disc (in kg m2) is
(a) 0.25 × 10–3 (b) 1.5 × 10–3 104. A uniform solid cylindrical roller of mass ‘m’ is being pulled
(c) 1.25 × 10 –3 (d) 0.5 × 10–3 on a horizontal surface with force F parallel to the surface
and applied at its centre. If the acceleration of the cylinder is
97. Two bodies have their moments of inertia I and 2I
‘a’ and it is rolling without slipping then the value of ‘F’ is
respectively about their axis of rotation. If their kinetic
energies of rotation are equal, their angular momenta will 5 3
(a) ma (b) ma (c) ma (d) 2 ma
be in the ratio 3 2
105. A body rolls down an inclined plane. If its kinetic energy of
(a) 2 : 1 (b) 1 : 2 (c) 2 : 1 (d) 1 : 2 rotation is 40% of its kinetic energy of translation motion,
98. M.I of a circular loop of radius R about the axis in figure is then the body is
y
(a) hollow cylinder (b) ring
(a) MR2 (c) solid disc (d) solid sphere
Axis of rotation
R/2 106. A solid cylinder of mass 2 kg and radius 0.1 m rolls down an
(b) (3/4) MR2 x inclined plane of height 3m without slipping. Its rotational kinetic
O
energy when it reaches the foot of the plane would be :
(c) MR2/2 (a) 22.7 J (b) 19.6 J (c) 10.2 J (d) 9.8 J
107. A sphere starts rolling down an incline of inclination 30o.
(d) 2MR2 Find the speed of its centre when it has covered a
99. A rod of mass m and length l is bent in to shape of L. Its 70
moment of inertia about the axis shown in figure distance m. [take g = 10 m/s2]
5
(a) 10 m/s (b) 12 m/s (c) 15 m/s (d) 18 m/s
ml 2 ml 2 m/2 108. The acceleration of a disc (mass ‘m’ and radius ‘R’) rolling
(a) (b)
6 3 down an incline of angle ‘q’ without slipping is
(a) 2/3 g sin q (b) 5/7 g sin q
(c) 1/2 g sin q (d) 7/5 g sin q
ml 2 109. A carpet of mass m made of inextensible material is rolled
(c) (d) None m/2
2 along its length in the form of a cylinder of radius r and kept
100. The moment of inertia of a hollow thick spherical shell of on a rough floor. The decrease in the potential energy of the
mass M and its inner radius R1 and outer radius R2 about its r
system, when the carpet is unrolled to a radius without
diameter is 2
sliding is
2M (R 52 - R15 ) 2M (R 52 - R15 ) (g = acceleration due to gravity)
(a) 5 (R 32 - R13 ) (b) 3 (R 32 - R13 ) 3 5 7 1
(a) mgr (b) mgr (c) mgr (d) mgr
4 8 8 2
4M (R 52 - R15 ) 4M (R 52 - R15 ) 110. A ball rolls without slipping. The radius of gyration of the
(c) 5 (R 32 - R13 ) (d) 3 (R 32 - R13 ) ball about an axis passing through its centre of mass is K. If
radius of the ball be R, then the fraction of total energy
101. The moment of inertia of a uniform semicircular wire of mass
associated with its rotational energy will be
m and radius r, about an axis passing through its centre of
æ kö K2 K2 R2 K2 + R2
mass and perpendicular to its plane is mr 2 ç1 - 2 ÷ then (a) (b) 2 (c) (d
è p ø R2 K2 + R K2 + R 2 R2
find the value of k.
EBD_7418
90 PHYSICS

111. A sphere rolls down on an inclined plane of inclination q. the cylinder rolls without slipping, then linear acceleration
What is the acceleration as the sphere reaches the bottom? of its centre of mass will be
5 3 2 2 20N
(a) g sin q (b) g sin q (c) g sin q (d) g sin q (a) 6.7 m/s2 (b) 10 m/s2 0.1m
7 5 7 5
112. A circular disc of radius R rolls without slipping along the
horizontal surface with constant velocity v0. We consider a (c) 3.3 m/s2 (d) None of these
point A on the surface of the disc, then the acceleration of 120. A toy car rolls down the inclined plane as shown in the fig.
the point A is It loops at the bottom. What is the relation between H and
(a) constant in magnitude as well as direction. h?
(b) constant in direction
(c) constant in magnitude H H h
(d) cannot say (a) =2 (b) =3
h h H
113. A solid sphere of mass 2 kg rolls on a smooth horizontal r D
surface at 10 m/s. It then rolls up a smooth inclined plane of
inclination 30° with the horizontal. The height attained by H H
(c) = 4 (d) =5
the sphere before it stops is h h B
(a) 700 cm (b) 701 cm (c) 7.1 m (d) None of these
114. A hollow smooth uniform sphere A of mass m rolls without 121. The spool shown in figure is placed on a rough horizontal
sliding on a smooth horizontal surface. It collides head on surface has inner radius r and outer radius R.
elastically with another stationary smooth solid sphere B of The angle q between the applied force and the horizontal
the same mass m and same radius. The ratio of kinetic energy can be varied.
of B to that of A just after the collision is
A B The critical angle (q) for which the spool does not roll and
remains stationary is given by F
v0 æ rö
(a) q = cos -1 ç ÷ R
è Rø
r
(a) 1 : 1 (b) 2 : 3 æ 2r ö
(b) q = cos-1 ç ÷
(c) 3 : 2 (d) None of these è Rø
115. A solid sphere, disc and solid cylinder all of the same mass
r æ rö
and made of the same material are allowed to roll down (from (c) q = cos -1 (d) q = sin -1 ç ÷
rest) on an inclined plane, then R è Rø
(a) solid sphere reaches the bottom first 122. A cylinder A rolls without slipping
(b) solid sphere reaches the bottom last on a plank B. The velocities of
(c) disc will reach the bottom first A
center of the cylinder and that of
(d) all reach the bottom at the same time B
116. A solid cylinder of mass m & radius R rolls down inclined the plank are 4m/s and 2m/s
plane without slipping. The speed of its C.M. when it reaches respectively in same direction, with Horizontal floor
the bottom is respect to the ground. Find the angular velocity of the
cylinder (in rad/s) if its radius is 1m.
(a) 2 gh (b) 4gh / 3 h (a) 2 rad/sec(b) 4 rad/sec(c) 6 rad/sec(d) 10 rad/sec
123. A small sphere A of mass m and
(c) 3 / 4 gh (d) 4 gh
radius r rolls without slipping A
117. The least coefficient of friction for an inclined plane inclined inside a large fixed hemispherical R
at angle a with horizontal in order that a solid cylinder will bowl of radius R (>> r) as shown
roll down without slipping is in figure. If the sphere starts from
2 2 5 rest at the top point of the
(a) tan a (b) tan a (c) tan a (d) tan a B
3 7 7 hemisphere find the normal force exerted by the small sphere
118. An annular ring with inner and outer radii R1 and R2 is
rolling without slipping with a uniform angular speed. The on the hemisphere when it is at the bottom B of the
ratio of the forces experienced by the two particles situated hemisphere.
10 17 5 7
(a) mg (b) mg (c) mg (d) mg
on the inner and outer parts of the ring , F1 is 7 7 7 5
F2 124. A cylinder of mass Mc and sphere of mass Ms are placed at
æR ö
2 points A and B of two inclines, respectively (See Figure). If
R2
(a) çç 1 ÷÷ (b) (c) R 1 (d) 1 they roll on the incline without sipping such that their
R
è 2ø R1 R2
119. A tangential force of 20 N is applied on a cylinder of mass sin qc
accelerations are the same, then the ratio is
4 kg and moment of inertia 0.02 kg m2 about its own axis. If sin q s
SYSTEM OF PARTICLES AND ROTATIONAL MOTION 91

125. Two identical uniform solid spherical ball A and B of mass


8 MC m each are placed on a the fixed wedge as shown in figure.
(a) A
7 Ball B is kept at rest and it is released just before two ball
collides. Ball A rolls down without slipping on inclined
15
(b) plane and collide elastically with ball B. The kinetic energy
14 M of ball A just after the collision with B is :
S
B
8 mgh mgh
(c) (a) (b) A
7 qC 7 2
B
15 qS 2mgh 7mgh h
(d) D h/2
14 C (c) (d) q
5 5

Exercise 2 : Exemplar & Past Year MCQs


6. In problem-5, the CM of the plate is now in the following
NCERT Exemplar MCQs
quadrant of x-y plane.
1. For which of the following does the centre of mass lie (a) I (b) II (c) III (d) IV
outside the body? 7. The density of a non-uniform rod of length 1 m is given by
(a) A pencil (b) A shotput (c) A dice (d) A bangle r(x) = a (1 + bx 2) where, a and b are constants and
2. Which of the following points is the likely position of the 0 £ x £ 1. The centre of mass of the rod will be at
centre of mass of the system shown in figure? 3(2 + b) 4(2 + b) 3(3 + b ) 4(3 + b )
Hollow sphere (a) (b) (c) 4(2 + b) (d) 3(2 + b )
(a) A Air 4(3 + b) 3(3 + b)
8. A merry-go-round, made of a ring-like platform of radius R
(b) B R/2 A and mass M, is revolving with angular speed w. A person of
B
(c) C C mass M is standing on it. At one instant, the person jumps off
R/2 the round, radially away from the centre of the round (as seen
D from the round). The speed of the round of afterwards is
(d) D
Sand w
3. A particle of mass m is moving in yz-plane with a uniform (a) 2w (b) w (c) (d) 0
2
velocity v with its trajectory running parallel to +ve y-axis
and intersecting z-axis at z = a in figure. The change in its Past Year MCQs
angular momentum about the origin as it bounces elastically 9. The ratio of the accelerations for a solid sphere (mass ‘m’
from a wall at y = constant is and radius ‘R’) rolling down an incline of angle ‘q’ without
z
(a) mvaeˆx slipping and slipping down the incline without rolling is :
v [AIPMT 2014, A]
(b) 2mvaeˆx a
(a) 5 : 7 (b) 2 : 3 (c) 2 : 5 (d) 7 : 5
(c) ymveˆ x 10. A solid cylinder of mass 50 kg and radius 0.5 m
(d) 2ymveˆ x is free to rotate about the horizontal axis. A massless string
y is wound round the cylinder with one end attached to it and
4. When a disc rotates with uniform angular velocity, which of
other hanging freely. Tension in the string required to
the following is not true?
(a) The sense of rotation remains same produce an angular acceleration of 2 revolutions s– 2 is :
(b) The orientation of the axis of rotation remains same [AIPMT 2014, C]
(c) The speed of rotation is non-zero and remains same (a) 25 N (b) 50 N (c) 78.5 N (d) 157 N
(d) The angular acceleration is non-zero and remains same 11. A wheel is rolling straight on ground without slipping. If the
5. A uniform square plate has a small piece Q of an irregular axis of the wheel has speed v, the instantenous velocity of
shape removed and glued to the centre of the plate leaving a point P on the rim, defined by angle q, relative to the
a hole behind in figure. The moment of inertia about the z- ground will be [AIIMS 2014, C]
axis is then, y y
æ1 ö
(a) increased hole (a) v cos ç q ÷ P
Q Q è2 ø
(b) decreased x x q
P P æ1 ö
(c) the same (b) 2 v cos ç q ÷
(d) changed in unpredicted manner è2 ø
(c) v(1 + sin q)
(d) v(1 + cos q)
EBD_7418
92 PHYSICS

12. An equilateral prism of velocity w0 is minimum, is given by : [AIIMS 2015, S]


mass m rests on a rough F w0
horizontal surface with
coefficient of friction µ. A a a
horizontal force F is ap-
plied on the prism as m1 P m2
shown in the figure. a
If the coefficient of friction is sufficiently high so that the x (L–x)
prism does not slide before toppling, then the minimum
force required to topple the prism is – [BITSAT 2014, C]
mg mg mmg mmg
(a) (b) (c) (d)
3 4 3 4
13. Three identical spherical shells, each of mass m and radius r
m1 m
are placed as shown in figure. Consider an axis XX' which is (a) x= L (b) x= 2L
m2 m1
touching to two shells and passing through diameter of third
shell. Moment of inertia of the system consisting of these m2 L m1L
(c) x = m + m (d) x = m + m
three spherical shells about XX' axis is [AIPMT 2015, S] 1 2 1 2
(a) 3mr2 X 18. An automobile moves on a road with a speed of 54 km h-1. The
radius of its wheels is 0.45 m and the moment of inertia of the
16 2 wheel about its axis of rotation is 3 kg m2. If the vehicle is
(b) mr
5 brought to rest in 15s, the magnitude of average torque
(c) 4mr2 transmitted by its brakes to the wheel is: [AIIMS 2015, S]
(a) 8.58 kg m2 s-2 (b) 10.86 kg m2 s-2
11 2 (c) 2.86 kg m s 2 -2 (d) 6.66 kg m2 s-2
(d) mr X¢
5 19. From a solid sphere of mass M and radius R a cube of
maximum possible volume is cut. Moment of inertia of cube
14. A rod of weight W is supported by two parallel knife edges about an axis passing through its center and perpendicular
A and B and is in equilibrium in a horizontal position. The to one of its faces is : [JEE Main 2015, S]
knives are at a distance d from each other. The centre of 2 2
mass of the rod is at distance x from A. The normal reaction 4MR 4MR MR 2 MR 2
(a) (b) (c) (d)
on A is [AIPMT 2015, C] 9 3p 3 3p 32 2p 16 2p
Wd W(d – x) W(d – x) Wx 20. A ring of mass M and radius R is w
(a) (b) (c) (d) rotating with angular speed w about a
x x d d
fixed vertical axis passing through its
15. A mass m moves in a circle on a smooth horizontal plane with
centre O with two point masses each of
velocity v0 at a radius R0. The mass is attached to string O
which passes through a smooth hole in the plane as shown. M
mass at rest at O. These masses can
v0 8
move radially outwards along two massless rods fixed on
the ring as shown in the figure. At some instant the angular
8
m speed of the system is w and one of the masses is at a
9
3
distance of R from O. At this instant the distance of the
5
The tension in the string is increased gradually and finally other mass from O is [JEE Main 2015, A]
R 2 1 3 4
m moves in a circle of radius 0 . The final value of the (a)
3
R (b)
3
R (c)
5
R (d)
5
R
2
kinetic energy is [AIPMT 2015, A] 21. Distance of the centre of mass of a solid uniform cone from
1 2 1 2 its vertex is z0. If the radius of its base is R and its height is
(a) mv0 (b) 2mv02 (c) mv0 (d) mv02
4 r 2 h then z0 is equal to : [JEE Main 2015, S]
r
16. A force F = a ˆi + 3jˆ + 6kˆ is acting at a point r = 2iˆ - 6ˆj - 12kˆ . 5h 3h 2 h2 3h
The value of a for which angular momentum about origin is (a) (b) (c) (d)
8 8R 4R 4
conserved is : [AIPMT 2015 RS, C]
(a) 2 (b) zero (c) 1 (d) –1 22. A disk and a sphere of same radius but different masses roll
17. Point masses m1 and m2 are placed at the opposite ends of off on two inclined planes of the same altitude and length.
Which one of the two objects gets to the bottom of the
a rigid rod of length L, and negligible mass. The rod is to be plane first ? [NEET 2016, C]
set rotating about an axis perpendicular to it. The position (a) Disk
of point P on this rod through which the axis should pass so (b) Sphere
that the work required to set the rod rotating with angular (c) Both reach at the same time
(d) Depends on their masses
SYSTEM OF PARTICLES AND ROTATIONAL MOTION 93

23. A uniform circular disc of radius 50 cm at rest is free to turn (a) go straight. B D
about an axis which is perpendicular to its plane and passes
through its centre. It is subjected to a torque which produces (b) turn left and right alternately.
a constant angular acceleration of 2.0 rad s–2 . Its net
acceleration in ms–2 at the end of 2.0s is approximately : (c) turn left. O
[NEET 2016, C]
(a) 8.0 (b) 7.0 (c) 6.0 (d) 3.0 (d) turn right.
24. From a disc of radius R and mass M, a circular hole of diameter C
A
R, whose rim passes through the centre is cut. What is the moment R
of inertia of the remaining part of the disc about a perpendicular 29. A circular disc of radius R and thickness has moment
6
axis, passing through the centre? [NEET 2016, A] inertia I about an axis passing through its centre
(a) 15 MR2/32 (b) 13 MR2/32 perpendicular to its plane. It is melted and recasted into a
(c) 11 MR2/32 (d) 9 MR2/32 solid sphere. The moment of inertia of the sphere about its
25. Consider a thin uniform square sheet made of a rigid material. diameter is [BITSAT 2016, S]
If its side is ‘a’ mass m and moment of inertia I about one of 2I I I
its diagonals, then [AIIMS 2016, A] (a) I (b) (c) (d)
8 5 10
ma 2 ma 2 ma 2 30. Which of the following statements are correct ? [NEET 2017, C]
(a) I > (b) <I<
12 24 12 (A) Centre of mass of a body always coincides with the
ma 2 ma 2 centre of gravity of the body
(c) I = (d) I = (B) Centre of mass of a body is the point at which the total
24 12 gravitational torque on the body is zero
26. A small particle of mass m is projected at an angle q with the (C) A couple on a body produce both translational and
x-axiswith an initial velocity n0 in the x-y plane as shown in rotation motion in a body
(D) Mechanical advantage greater than one means that
n0 sin q
the figure. At a time t < , the angular momentum of small effort can be used to lift a large load
g (a) (A) and (B) (b) (B) and (C)
the particle is [AIIMS 2016, C] (c) (C) and (D) (d) (B) and (D)
(a) - mg n 0 t 2 cos q ˆj y 31. Two discs of same moment of inertia rotating about their
ˆ regular axis passing through centre and perpendicular to
(b) mg n 0t cos q k v0
the plane of disc with angular velocities w1 and w2 . They
1
(c) - mg n 0 t cos q kˆ
2 are brought into contact face to face coinciding the axis of
2 rotation. The expression for loss of energy during this
1 q process is:- [NEET 2017, A]
(d) mgn0t 2 cos q iˆ
2 x 1 2
(a) I(w1 - w2 ) (b) I(w1 - w2 )2
27. A particle of mass m is moving y
4
along the side of a square of side 'a', D a C 1 1
with a uniform speed v in the V (c) (w1 - w2 )2 (d) I( w1 + w2 ) 2
a V V a 8 2
x-y plane as shown in the figure : A V
32. A rope is wound around a hollow cylinder of mass 3 kg and
a B
Which of the following statements V radius 40 cm. What is the angular acceleration of the cylinder
is false for the angular momentum O 45° R a if the rope is pulled with a force of 30 N ? [NEET 2017, A]
ur
L about the origin? [JEE Main 2016, C] (a) 0.25 rad/s2 (b) 25 rad/s2
2 (d) 25 m/s2
ur éR ù$ (c) 5 m/s
(a) L = mv êê + a úú k when particle is moving B to C. 33. A solid sphere rolls down two different inclined planes of
ë 2 û same height, but of different inclinations. In both cases
ur mv [AIIMS 2016, C]
(b) L = Rk$ when particle is moving from D to A. (a) speed and time of descent will be same
2
(b) speed will be same, but time of descent will be different
ur mv $ (c) speed will be different, but time of descent will be same
(c) L = – Rk when the particle is moving from A to B.
(d) speed and time of descent both are different
2
ur éR ù 34. Assertion : Radius of gyration of body is a constant
(d) L = mv êê - a ú k$ when particle is moving C to D. quantity.
ë 2 úû Reason : The radius of gyration of a body about an axis
28. A roller is made by joining together two cones at their vertices of rotation may be defined as the root mean square
O. It is kept on two rails AB and CD, which are placed distance of the particle from the axis of rotation.
asymmetrically (see figure), with its axis perpendicular to CD [AIIMS 2017, S]
and its centre O at the centre of line joining AB and Cd (see (a) Assertion is correct, reason is correct; reason is a
figure). It is given a light push so that it starts rolling with its correct explanation for assertion.
centre O moving parallel to CD in the direction shown. As it (b) Assertion is correct, reason is correct; reason is not a
moves, the roller will tend to : [JEE Main 2016, C] correct explanation for assertion
(c) Assertion is correct, reason is incorrect
(d) Assertion is incorrect, reason is correct.
EBD_7418
94 PHYSICS

35. A slender uniform rod of mass M 40. Assertion : When a sphere is rolls on a horizontal table it
and length l is pivoted at one end slows down and eventually stops.
so that it can rotate in a vertical plane Reason : When the sphere rolls on the table, both the sphere
(see figure). There is negligible and the surface deform near the contact. As a result, the
friction at the pivot. The free end is normal force does not pass through the centre and provide
held vertically above the pivot and an angular declaration. [AIIMS 2018, C]
then released. (a) Assertion is correct, reason is correct; reason is a
The angular acceleration of the rod when it makes an angle correct explanation for assertion.
q with the vertical is [JEE Main 2017, S] (b) Assertion is correct, reason is correct; reason is not a
3g 2g correct explanation for assertion
(a) cos q (b) cos q (c) Assertion is correct, reason is incorrect
2l 3l (d) Assertion is incorrect, reason is correct.
3g 2g 41. Seven identical circular planar disks, each of mass M and
(c) sin q (d) sin q
2l 2l radius R are welded symmetrically as shown. The moment of
36. The moment of inertia of a uniform cylinder of length l and inertia of the arrangement about the axis normal to the plane
radius R about its perpendicular bisector is I. What is the and passing through the point P is: [JEE Main 2018, A]
ratio l/R such that the moment of inertia is minimum ?
[JEE Main 2017, S] P
3 3 3 O
(a) 1 (b) (c) (d)
2 2 2
37. A thin but rigid semicircular wire frame of radius r is hinged at O
and can rotate in its own vertical plane. A smooth peg P starts
19 55 73 181
from O and moves horizontally with constant speed v0, lifting (a) MR 2 (b) MR 2 (c) MR 2 (d) MR2
the frame upward as shown in figure. [BITSAT 2017, S, BM] 2 2 2 2
42. From a uniform circular disc of radius R and mass 9 M, a
R
small disc of radius is removed as shown in the figure.
3
The moment of inertia of the remaining disc about an axis
perpendicular to the plane of the disc and passing, through
centre of disc is : [JEE Main 2018, S]
Find the angular velocity w of the frame when its diameter 2R
makes an angle of 60° with the vertical : 3
(a) v0 / r (b) v0 / 2 r (c) 2 v0 / r (d) v0r
38. A solid sphere is in rolling motion. In rolling motion a body R
possesses translational kinetic energy (K t) as well as
rotational kinetic energy (Kr) simultaneously. The ratio Kt :
(Kt + Kr) for the sphere is [NEET 2018, A] 40 37
(a) 4 MR2 (b) MR2 (c) 10 MR2 (d) MR2
(a) 7 : 10 (b) 5 : 7 (c) 2 : 5 (d) 10 : 7 9 9
39. Three objects, A : (a solid sphere), B : (a thin circular disk) 43. A particle is moving with a uniform speed in a circular orbit
and C : (a circular ring), each have the same mass M and of radius R in a central force inversely proportional to the
radius R. They all spin with the same angular speed w about nth power of R. If the period of rotation of the particle is T,
their own symmetry axes. The amounts of work (W) required then: [JEE Main 2018, C]
to bring them to rest, would satisfy the relation (a) T µ R for any n
3/2 (b) T µ R n /2+1
[NEET 2018, C] (c) T µ R (n +1)/2 (d) T µ R n/2
(a) WC > WB > WA (c) WA > WB > WC
44. A hoop rolls down an inclined plane. The fraction of its total
(c) WA > WC > WB (d) WB > WA > WC
kinetic energy that is associated with rotational motion is
[BITSAT 2018, A]
(a) 1 : 2 (b) 1 : 3 (c) 1 : 4 (d) 2 : 3
SYSTEM OF PARTICLES AND ROTATIONAL MOTION 95

Exercise 3 : Try If You Can


1. Particles of masses m, 2m, 3m, ............. nm grams are placed
7. A uniform thin rod is bent in the form y
on the same line at distances l, 2l, 3l, ...... nl cm from a fixed
point. The distance of centre of mass of the particles from of closed loop ABCDEFA as shown C
Semicircle
the fixed point in centimetres is in the figure. The ratio of moment of A B D E
x

(2 n + 1)l inertia of the loop about x-axis to that


1 n(n 2 + 1)l 2l 2r
(a) (b) (c) (d) about y-axis is
3 n +1 2 n(n 2 + 1) Semicircle
2. Three identical rods are hinged at point A as shown. The F

angle made by rod AB with vertical is (a) > 1 (b) < 1 (c) = 1 (d) = 1/2
A
8. A shaft is rotating at a speed of 4000 revolutions per minute.
æ 1 ö If the power expended in driving the shaft is 12 kW, the
(a) tan -1 ç magnitude of the driving torque is :
è 3 ÷ø
B 90° (a) 90 / pNm (b) 90 Nm (c) 180 Nm (d) 36 /p Nm
D 9. A particle of mass m = 5 is moving with a uniform speed
-1 æ 3 ö
(b) tan çè ÷ø
4 v = 3 2 in the XOY plane along the line y = x + 4. The
90° magnitude of the angular momentum of the particle about
(c) tan –1 (1)
the origin is
C
æ 4ö (a) 60 units (b) 40 2units (c) zero (d) 7.5 units
(d) tan -1 ç ÷ 10. A horizontal circular plate is rotating about a vertical axis
è 3ø
passing through its centre with an angular velocity wo. A
3. A spool is pulled horizontally F man sitting at the centre having two blocks in his hands
by two equal and opposite stretches out his hands so that the moment of inertia of the
forces as shown in fig. Which system doubles. If the kinetic energy of the system is K
of the following statements are F initially, its final kinetic energy will be
correct? Rough (a) 2 K (b) K/ 2 (c) K (d) K/ 4
(a) The centre of mass moves towards left. 11. A particle of mass m is moving in a circle of radius r. The
(b) The centre of mass moves towards right centripetal acceleration (ac) of the particle varies with the
(c) The centre of mass remains stationary time according to the relation, ac = Kt2, where K is a positive
(d) The net force about the centre of mass of the spool is zero constant and t is the time. The magnitude of the time rate of
4. Velocity of the centre of a small cylinder is v. There is no change of angular momentum of the particle about the centre
slipping anywhere. The angular velocity of the centre of of the circle is
2 3
the larger cylinder is (a) mKr (b) m Kr (c) mKr (d) mKr2
12. A block of mass m is attached to a pulley disc of equal mass
(a) 2v m and radius r by means of slack string as shown. The
R
(b) v 2R v pulley is hinged about its centre on a horizontal table and
3v the block is projected with an initial velocity of 5 m/s. Its
(c) velocity when the string becomes tight will be
2 (a) 3 m/s m v
(d) None of these
(b) 2.5 m/s m
5. A ring of mass M and radius R is rotating about its axis with (c) 5/3 m/s
angular velocity w. Two identical bodies each of mass m are (d) 10/3 m/s
now gently attached at the two ends of a diameter of the 13. A circular disc of mass m and radius R is rotating on a rough
ring. Because of this, the kinetic energy loss will be : surface having a coefficient of friction µ with an initial angular
Mm velocity w . Assuming a uniform normal reaction on the entire
m( M + 2m) 2 2
(a) w R (b) w2 R2 contact surface, the time after which the disc comes to rest is
M ( M + m) wR 3wR 1 wR 3 wR
(a) (b) (c) (d)
Mm ( M + m) M 2 2 µg 4µg 2 µg 2 µg
(c) w2 R2 (d) w R 14. The free end of a thread wound on a bobbin is passed round
( M + 2m) (M + 2m)
a nail A hammered into the wall. The thread is pulled at a
6. A uniform solid cube of mass M has edge length a. The constant velocity. Assuming pure rolling of bobbin, find
moment of inertia of the cube about its face diagonal will be the velocity v0 of the centre of the bobbin at the instant
3 1 5 7 when the thread forms an angle a with the vertical.
(a) Ma2 (b) Ma 2 (c) Ma 2 (d) Ma 2
2 2 12 12
EBD_7418
96 PHYSICS

vR vR 18. A thin wire of length L and uniform


(a) (b) X X'
R sin a - r R sin a + r linear mass density r is bent into 90º
2vR v a circular loop with centre at O as
(c) (d)
R sin a + r R sin a + r shown. The moment of inertia of
15. A bowling ball rolls without slipping down the loop about the axis XX' is
an inclined plane inclined at an angle q to R g
rL3 rL3 5rL3 3rL3
the horizontal, as shown. The coefficient (a) (b) (c) (d)
of static friction between the ball and the 8p 2 16p 2 16p 2 8p2
surface is µs, and the coefficient of kinetic 19. Two point masses of 0.3 kg and 0.7 kg are fixed at the ends
friction is µk. What is the magnitude of the force of friction of a rod of length 1.4 m and of negligible mass. The rod is set
acting on the ball ? rotating about an axis perpendicular to its length with a
(a) zero (b) µs mg cos q uniform angular speed. The point on the rod through which
7 2 the axis should pass in order that the work required for
(c) µs mg sin q (d) mg sin q rotation of the rod is minimum, is located at a distance of
5 7
16. A frustum of a solid right circular cone 10cm (a) 0.42 m from mass of 0.3 kg
has a base diameter of 20 cm, top diameter (b) 0.70 m from mass of 0.7 kg
of 10 cm and height 20cm. It has an axial (c) 0.98 m from mass of 0.3 kg
20cm
cylindrical hole of diameter 5cm. 5
(d) 0.98 m from mass of 0.7 kg
Determine the position of centre of mass 20. Find the minimum height of the obstacle so that the sphere
of this body 20cm can stay in equilibrium.
(a) 7.6 cm (b) 4.3 cm (c) 12.6 cm (d) 15.3 cm R
17. There is a flat uniform triangular plate ABC (a)
A 1 + cos q m
such that AB = 4 cm, BC = 3 cm and angle
R R
ABC = 90º. The moment of inertia of the plate (b)
about AB, BC and CA as axis is respectively 1 + sin q h
I1, I2 and I3. Which one of the following is 90º
(c) R (1 – sin q) q
(d) R (1 – cos q)
true? B C
(a) I3 > I2 (b) I2 > I1 (c) I3 > I1 (d) I1 > I2
ANSWER KEYS
Exercise 1 : Topic-wiese MCQs
1 (b) 14 (c) 27 (a) 40 (a) 53 (a) 66 (a) 79 (d) 92 (a) 105 (d) 118 (c)
2 (d) 15 (a) 28 (d) 41 (b) 54 (d) 67 (d) 80 (a) 93 (d) 106 (b) 119 (a)
3 (d) 16 (c) 29 (b) 42 (d) 55 (c) 68 (c) 81 (d) 94 (d) 107 (a) 120 (d)
4 (a) 17 (c) 30 (b) 43 (c) 56 (a) 69 (c) 82 (b) 95 (a) 108 (a) 121 (a)
5 (a) 18 (a) 31 (b ) 44 (d ) 57 (d) 70 (c) 83 (c) 96 (d) 109 (c) 122 (a)
6 (a) 19 (a) 32 (c) 45 (a) 58 (c) 71 (a) 84 (b) 97 (d) 110 (b) 123 (b)
7 (c) 20 (a) 33 (b) 46 (c) 59 (a) 72 (d) 85 (c) 98 (b) 111 (a) 124 (d)
8 (b) 21 (c) 34 (d) 47 (c) 60 (b) 73 (a) 86 (d) 99 (a) 112 (a) 125 (a)
9 (b) 22 (a) 35 (d) 48 (b) 61 (d) 74 (a) 87 (d) 100 (a) 113 (c)
10 (b) 23 (a) 36 (b) 49 (b) 62 (a) 75 (d) 88 (d) 101 (c) 114 (c)
11 (c) 24 (b) 37 (b) 50 (a) 63 (a) 76 (d) 89 (b) 102 (b) 115 (a)
12 (b) 25 (c) 38 (a) 51 (c) 64 (b) 77 (b) 90 (c) 103 (d) 116 (b)
13 (c) 26 (d) 39 (d) 52 (d) 65 (d) 78 (a) 91 (d) 104 (c) 117 (c)
Exercise 2 : Exemplar & Past Year MCQs
1 (d) 6 (c) 11 (b) 16 (d) 21 (d) 26 (c) 31 (a) 36 (c) 41 (d)
2 (c) 7 (a) 12 (a) 17 (c) 22 (b) 27 (a) 32 (b) 37 (a) 42 (a)
3 (b) 8 (a) 13 (c) 18 (d) 23 (a) 28 (c) 33 (b) 38 (b) 43 (c)
4 (d) 9 (a) 14 (c) 19 (a) 24 (b) 29 (c) 34 (d) 39 (a) 44 (a)
5 (b) 10 (d) 15 (b) 20 (d) 25 (d) 30 (d) 35 (c) 40 (b)
Exercise 3 : Try If You Can
1 (a) 3 (b) 5 (c) 7 (b) 9 (a) 11 (b) 13 (b) 15 (d) 17 (b) 19 (c)
2 (b) 4 (b) 6 (c) 8 (a) 10 (b) 12 (d) 14 (a) 16 (a) 18 (d) 20 (d)
8
Chapter
GRAVITATION

Trend
Analysis of NEET and AIIMS (Year 2010-2018)
5

4
Number of Questions

3
AIPMT/NEET
2 AIIMS

0
2010 2011 2012 2013 2014 2015 2016 2017 2018
Year
Trend
Analysis of JEE Main and BITSAT (Year 2010-2018)
5

4
Number of Questions

3
JEE Main
2 BITSAT

0
2010 2011 2012 2013 2014 2015 2016 2017 2018
Year

Chapter Utility Score (CUS)


Exam Weightage Important Concepts Difficulty Level CUS (Out of 10)
NEET 4 Universal law of Gravitation,
AIIMS 3 Escape & Orbital Velocity 3.5/5 6.5/10
JEE Main 3 and Variation in Acceleration
BITSAT 4 due to Gravity
EBD_7418
98 PHYSICS
GRAVITATION 99
EBD_7418
100 PHYSICS

Exercise 1 : Topic-wise MCQs


9. The period of moon's rotation around the earth is nearly 29
Topic 1: Kepler's Laws of Planetary Motion
days. If moon's mass were 2 fold its present value and all
æ 4 p2 ö 3 other things remain unchanged, the period of moon's
1. For elliptical orbits, in the equation T2 = çç ÷÷ R rotation would be nearly
è GMs ø (a) 29 2 days (b) 29 / 2 days
R refers to
(a) radius of orbit (b) major axis (c) 29 × 2 days (d) 29 days
(c) semi-minor axis (d) semi-major axis 10. A planet of mass m moves around the sun of mass M in an
2. In planetary motion elliptical orbit. The maximum and minimum distance of the
(a) the angular speed remains constant planet from the sun are r1 and r 2 respectively. The time
(b) the total angular momentum remains constant period of planet is proportional to
3/ 2 3/ 2
(c) the linear speed remains constant æ ö æ ö
(d) neither the angular momentum nor angular speed (a) r12 / 5 (b) ç r1 + r2 ÷
(c) ç r1 – r2 ÷ (d) r 3 / 2
è è ø ø
remains constant 11. Which of the following graphs represents the motion of a
3. Kepler’s second law may be stated as "under the influence planet moving about the sun ?
of central force, in equal interval of time, position vector (a) (b)
sweeps out equal
(a) distance (b) area T2 T2
(c) dispalacement (d) volume
4. The figure shows a planet in elliptical orbit around the sun
S. Where is the kinetic energy of the planet maximum ? R3 R3
P3 (c) (d)

T2 T2
P4 S P2

R3 R3
P1
(a) P 1 (b) P 2 (c) P 3 (d) P 4 12. The figure shows elliptical orbit of a planet m about the sun
5. A planet revolves about the sun in elliptical orbit. The areal S. The shaded area SCD is twice the shaded area SAB. If t 1
is the time for the planet to move from C to D and t2 is the
velocity æç ö÷ of the planet is 4.0 × 1016 m2/s. The least
dA time to move from A to B then
è dt ø m
v
distance between planet and the sun is 2 × 10 12 m. Then the
maximum speed of the planet in km/s is B C
(a) 10 (b) 20 (c) 30 (d) 40
6. In planetary motion the areal velocity of position vector of S
a planet depends on angular velocity (w) and the distance A D
of the planet from sun (r). If so, the correct relation for areal
velocity is (a) t1 = 4t2 (b) t1 = 2t2 (c) t1 = t2 (d) t1 > t2
dA dA
(a) µ wr (b) µ w2 r 13. The time period T of the moon of planet Mars (mass M m) is
dt dt related to its orbital radius R (G = Gravitational constant) as
dA dA
(c) µ wr 2 (d) µ wr 4p 2 R 3 4p 2 GR 3
dt dt (a) T2 = (b) T2 =
7. A planet goes round the sun three times as fast as the earth. GM m Mm
If rp and r e are the radii of orbit of the planet and the earth
2 pR 3 G
respectively then (c) T2 = (d) T2 = 4pMmGR3
Mm
(a) re3 = 8rp3 (b) re3 = 3rp3 14. If the distance of earth is halved from the sun, then the no.
1 of days in a year will be
(c) re3 = 9 rp3 (d) re3 = rp3 (a) 365 (b) 182.5 (c) 730 (d) 129
3
15. If the earth is at one-fourth of its present distance from the
8. The maximum and minimum distances of a comet from the sun, the duration of the year will be
sun are 8 × 1012 m and 1.6 × 1012 m. If its velocity when (a) half the present year
nearest to the sun is 60 m/s, what will be its velocity in m/s (b) one-eighth the present year
when it is farthest (c) one-sixth the present year
(a) 12 (b) 60 (c) 112 (d) 6 (d) one-tenth the present year
GRAVITATION 101

26. In the figure, the direction of gravitational force on m1 due


Topic 2: Newton's Universal Law of Gravitation to m2 is along y
16. Newton's universal law of gravitation applies to m1 r
(a) small bodies only m2
(b) planets only r1
(c) both small and big bodies
(d) only valid for solar system r2
17. For a particle inside a uniform spherical shell, the
gravitational force on the particle is O x
-G m1 m 2 G m1 m2
(a) infinite (b) zero (c) (d)
r2 r2
18. The value of G varies with z
r r r r
(a) height above the earth's surface (a) r1 (b) r2 (c) r (d) - r
(b) depth below the ground 27. Two bodies of masses 4 kg and 9 kg are separated by a
(c) radius of the planet distance of 60 cm. A 1 kg mass is placed in between these
(d) None of these two masses. If the net force on 1 kg is zero, then its distance
19. Force of gravitational attraction is least
(a) at the equator from 4 kg mass is
(b) at the poles (a) 26 cm (b) 30 cm (c) 28 cm (d) 24 cm
(c) at a point in between equator and any pole 28. A body weighs 72 N on the surface of the earth. What is the
(d) None of these gravitational force on it due to earth at a height equal to half
20. Two spheres of masses m and M are situated in air and the the radius of the earth from the surface?
gravitational force between them is F. The space around the (a) 32 N (b) 28 N (c) 16 N (d) 72 N
masses is now filled with a liquid of specific gravity 3. The 29. If masses of two point objects is doubled and distance
gravitational force will now be between them is tripled, then gravitational force of attraction
F F between them will nearly
(a) (b) 3F (c) F (d) (a) increase by 225% (b) decrease by 44%
9 3
21. A mass M is split into two parts m and (M – m), which are (c) decrease by 56% (d) increase by 125%
then separated by a certain distance. What ratio of m/M 30. A central particle M is surrounded by a square array of
maximizes the gravitational force between the two parts? other particles, separated by either distance d or distance d/
1 1 1 1 2 along the perimeter of the square. The magnitude of the
(a) (b) (c) (d) gravitational force on the central particle due to the other
3 2 4 5
22. The distance of the centres of moon and earth is D. The particles is
mass of earth is 81 times the mass of the moon. At what 2m 3m
distance from the centre of the earth, the gravitaional force 4m
will be zero?
D 2D 4D 9D 7m 5m
(a) (b) (c) (d)
2 3 3 10
23. Six stars of equal mass are moving about the centre of mass 3m
of the system such that they are always on the vertices of a M
regular hexagon of side length a. Their common time period 5m 7m
will be
2m
a3 4 3a3 4m 3m
(a) 4p (b) 2p
Gm (
Gm 5 3+4 ) (a)
9 GMm
d2
(b)
5 GMm
d2
(c)
3 GMm
d2
(d)
GMm
d2
31. If three equal masses m are placed at the three vertices of an
(c) 3a3 (d) None of these
4p equilateral triangle of side 1/m then what force acts on a
Gm particle of mass 2m placed at the centroid?
24. Two stars of mass m1 and m2 are parts of a binary system. (a) Gm2 (b) 2Gm2 (c) Zero (d) –Gm2
The radii of their orbits are r1 and r2 respectively, measured 32. Two identical spheres of gold are in contact with each other.
from the C.M. of the system. The magnitude of gravitational The gravitational attraction between them is
force m1 exerts on m2 is (a) directly proportional to the square of the radius
m1m2G m1G m2G ( m1 + m2 ) (b) directly proportional to the cube of the radius
(a) ( r1 + r2 ) (b) ( r1 + r2 ) 2 (c) ( r1 + r2 ) 2 (d) (r1 + r2 ) 2
2 (c) directly proportional to the fourth power of the radius
25. Consider the following statements and select the correct (d) inversely proportional to the square of the radius
statement(s). 33. Two particles of equal mass ‘m’ go around a circle of radius
I. Gravitational force may be attractive or repulsive R under the action of their mutual gravitational attraction.
II. Gravitational force between two particles is The speed of each particle with respect to their centre of
independent of presence of other particles mass is
III. Gravitational force is a short-range force
(a) I only (b) II only (c) II and III (d) I, II and III Gm Gm Gm Gm
(a) (b) (c) (d)
4R 3R 2R R
EBD_7418
102 PHYSICS

41. Assume that the acceleration due to gravity on the surface


Topic 3: Acceleration due to Gravity
of the moon is 0.2 times the acceleration due to gravity on
34. The value of acceleration due to gravity on moving from the surface of the earth. If Re is the maximum range of a
equator to poles will projectile on the earth's surface, what is the maximum range
(a) decrease (b) increase on the surface of the moon for the same velocity of projection
(c) remain same (d) become half (a) 0.2 Re (b) 2 Re (c) 0.5 Re (d) 5 Re
35. As we go down below the earth's surface, the acceleration 42. The height of the point vertically above the earth's surface,
due to gravity decreases by a factor (d ® distance, R ® at which acceleration due to gravity becomes 1% of its value
radius of earth) at the earth's surface is (Radius of the earth = R)
(a) 8 R (b) 9 R (c) 10 R (d) 20 R
d R
(a) 1+ (b) 1 - 43. The speed of earth’s rotation about its axis is w. Its speed is
R d
increases to x times to make the effective acceleration due
d to gravity equal to zero at the equator. Then x is :
(c) 1 - (d) remains constant
R (a) 1 (b) 8.5 (c) 17 (d) 34
36. The weight of a body at the centre of the earth is 44. Where will it be profitable to purchase one kilogram sugar?
(a) zero (a) At poles (b) At equator
(b) infinite (c) At 45° latitude (d) At 40° latitude
(c) same as on the surface of earth 45. Earth is flattened at poles, bulged at the equator. This is due
(d) None of these to
37. weight of body is found less on moon because of (a) the angular velocity of spinning about its axis is less at
(a) smaller value of G (b) smaller value of g equator
(c) smaller value of R (d) smaller value of m (b) the angular velocity of spinning about its axis is more
38. Which of the following graphs shows the correct variation at equator
of acceleration due to gravity with the height above the (c) the centrifugal force is more at the equator than at the
earth's surface? poles
(d) earth revolves round the sun in an elliptical orbit
g 46. A particle is suspended from a spring and it stretches the
g
spring by 1 cm on the surface of earth. By how much amount
the same particle will stretch the same spring at a place 800
km above the surface of earth.
(a) 1.59 cm (b) 2.38 cm (c) 0.79 cm (d) 1.38 cm
(a) (b) 47. The height at which the acceleration due to gravity becomes
g
r (where g = the acceleration due to gravity on the surface
O r O 9
of the earth) in terms of R, the radius of the earth, is
R
g (a) (b) R / 2 (c) (d) 2 R
2R
2
48. Assertion : Moon travellers tie heavy weight at their back
before landing on the moon.
Reason : The acceleration due to gravity on moon is smaller
(c) (d) None of these than that of earth.
(a) Assertion is correct, reason is correct; reason is a
correct explanation for assertion.
O r
(b) Assertion is correct, reason is correct; reason is not a
39. Assertion : If earth suddenly stops rotating about its axis, correct explanation for assertion
then the value of acceleration due to gravity will become (c) Assertion is correct, reason is incorrect
same at all the places. (d) Assertion is incorrect, reason is correct.
Reason : The value of acceleration due to gravity depends 49. The density of a newly discovered planet is twice that of
upon the rotation of the earth. earth. The acceleration due to gravity at the surface of the
(a) Assertion is correct, reason is correct; reason is a planet is equal to that at the surface of the earth. If the
correct explanation for assertion. radius of the earth is R, the radius of the planet would be
(b) Assertion is correct, reason is correct; reason is not a (a) 1/2 R (b) 2 R (c) 4 R (d) 1/4 R
correct explanation for assertion 50. Imagine a new planet having the same density as that of
(c) Assertion is correct, reason is incorrect earth but it is 3 times bigger than the earth in size. If the
(d) Assertion is incorrect, reason is correct.
acceleration due to gravity on the surface of earth is g and
40. The acceleration due to gravity on the planet A is 9 times
the acceleration due to gravity on planet B. A man jumps to that on the surface of the new planet is g’, then
a height of 2m on the surface of A. What is the height of (a) g’ = g/9 (b) g’ = 27g (c) g’= 9g (d) g’ = 3g
jump by the same person on the planet B? 51. Assertion : Generally, the path of a projectile from the earth
is parabolic but it is elliptical for projectiles going to a very
2 2
(a) m (b) m (c) 18 m (d) 6 m large height.
3 9
GRAVITATION 103

Reason : The path of a projectile is independent of the 59. If the density of a small planet is the same as that of earth,
gravitational force of earth. while the radius of the planet is 0.2 times that of the earth,
(a) Assertion is correct, reason is correct; reason is a the gravitational acceleration on the surface of the planet is
correct explanation for assertion. (a) 0.2 g (b) 0.4 g (c) 2 g (d) 4 g
(b) Assertion is correct, reason is correct; reason is not a 60. If the radius of the earth were to shrink by 1%, with its mass
correct explanation for assertion remaining the same, the acceleration due to gravity on the
(c) Assertion is correct, reason is incorrect earth’s surface would
(d) Assertion is incorrect, reason is correct. (a) decrease by 1% (b) decrease by 2%
52. Average density of the earth (c) increase by 1% (d) increase by 2%
(a) is a complex function of g 61. If a person goes to height equal to the radius of the earth,
(b) does not depend on g from its surface, then his weight (W¢) relative to the weight
(c) is inversely proportional to g on earth (W) will be
(d) is directly proportional to g W
(a) W¢ = (b) W¢ = 2W
53. If the earth were to rotates faster than its present speed, 4
the weight of an object will W
(c) W¢ = (d) W¢ = W
(a) increase at the equator but remain unchanged at the 2
poles 62. If value of acceleration due to gravity is 'g' at a height 50 km
(b) decrease at the equator but remain unchanged at the above the surface of earth, then at what depth inside the
earth will the acceleration due to gravity be same as 'g'?
poles
(a) 100 km (b) 50 km (c) 25 km (d) 75 km
(c) remain unchanged at the equator but decrease at the
poles Topic 4: Gravitational Field, Potential and Potential Energy
(d) remain unchanged at the equator but increase at the
poles 63. Consider Earth to be a homogeneous sphere. Scientist A
54. At what height from the ground will the value of g be the goes deep down in a mine and scientist B goes high up in a
balloon. The gravitational field measured by
same as that in 10 km deep mine below the surface of earth?
(a) A goes on decreasing and that by B goes on increasing
(a) 20 km (b) 10 km (c) 15 km (d) 5 km (b) B goes on decreasing and that by A goes on increasing
55. The ratio of radii of earth to another planet is 2/3 and the (c) each decreases at the same rate
ratio of their mean densities is 4/5. If an astronaut can jump (d) each decreases at different rates
to a maximum height of 1.5 m on the earth, with the same 64. In some region, the gravitational field is zero. The
effort, the maximum height he can jump on the planet is gravitational potential in this region.
(a) 1 m (b) 0.8 m (c) 0.5 m (d) 1.25 m (a) must be variable (b) must be constant
56. The value of 'g' at a particular point is 9.8 m/s2. Suppose the (c) cannot be zero (d) must be zero
earth suddenly shrinks uniformly to half its present size 65. In a gravitational field, at a point where the gravitational
without losing any mass. The value of 'g' at the same point potential is zero
(assuming that the distance of the point from the centre of (a) the gravitational field is necessarily zero
(b) the gravitational field is not necesarily zero
the earth does not shrink) will now be
(c) any value between one and infinite
(a) 4.9 m/sec2 (b) 3.1 m/sec2 (d) None of these
(c) 9.8 m/sec2 (d) 19.6 m/sec2 66. The gravitational field strength due to a solid sphere (mass
57. R is the radius of the earth and w is its angular velocity and M, radius R) varies with distance r from centre as
gp is the value of g at the poles. The effective value of g at
E
the latitude l = 60° will be equal to E
1 2 3 2
(a) g p - Rw (b) g p - Rw
4 4
1 R
(c) g p - Rw2 (d) g p + Rw
2 (a) r (b) r
4 R
58. What should be the velocity of rotation of earth due to
rotation about its own axis so that the weight of a person
2
becomes of the present weight at the equator. Equatorial E E
3
radius of the earth is R
1 1
æ 2g ö 2 æ g ö2 (c) R r (d) R r
(a) çè ÷ (b) çè ÷
3R ø 3R ø
1 1
æ g ö2 æ g ö2 67. A particle of mass M is situated at the centre of a spherical
(c) çè ÷ (d) çè ÷
7R ø 5R ø shell of same mass and radius a. The gravitational potential
a
at a point situated at distance from the centre, will be:
2
EBD_7418
104 PHYSICS

3GM 2 GM GM 4 GM value of g factor 1/4


(a) - (b) - (c) - (d) - (B) At depth h = R/2, (2) decreases by a
a a a a
68. Assuming the radius of the earth as R, the change in value of g factor 1/2
gravitational potential energy of a body of mass m, when it (C) At height h = R/2, (3) decreases by a
is taken from the earth's surface to a height 3R above its value of g factor 3/4
surface, is (D) At depth h = R / 4, (4) decreases by a
3 3 value of g factor 2/3
(a) 3 mg R (b) mg R (c) 1 mg R (d) mg R (a) (A) ® (2) ; (B) ® (1) ; (C) ® (3) ; (D) ® (4)
4 2
69. Two concentric spherical shells are as shown in figure. Match (b) (A) ® (2) ; (B) ® (2) ; (C) ® (4) ; (D) ® (3)
the following: (c) (A) ® (4) ; (B) ® (3) ; (C) ® (2) ; (D) ® (1)
B
(d) (A) ® (4) ; (B) ® (3) ; (C) ® (1) ; (D) ® (2)
A 76. The gravitational potential due to a hollow sphere
D
(mass M, radius R) varies with distance r from centre as
C
V V

Column I Column II (a) R r (b) R r


(A) Potential at A (1) greater than B
(B) Gravitational field at A(2) less than B
(C) As one moves (3) potential remains
from C to D constant V V
R
(D) As one moves (4) gravitational field
from D to A decreases
(5) None r (d) R r
(c)
(a) (A) ® (2) ; (B) ® (5) ; (C) ® (3) ; (D) ® (4)
(b) (A) ® (3) ; (B) ® (2) ; (C) ® (1) ; (D) ® (4)
(c) (A) ® (4) ; (B) ® (3) ; (C) ® (2) ; (D) ® (1)
(d) (A) ® (5) ; (B) ® (3) ; (C) ® (1) ; (D) ® (2) 77. If ‘g’ is the acceleration due to gravity on the earth’s surface,
the gain in the potential energy of an object of mass ‘m’
70. The potential energy of a rock, having mass m and rotating raised from the surface of the earth to a height equal to the
at a height of 3.2 × 106 m from the earth surface, is radius ‘R' of the earth is
(a) – 6 mgRe (b) – 0.67 mgRe 1 1
(a) mgR (b) mgR (c) 2 mgR (d) mgR
(c) – 0.99 mgRe (d) – 0.33 mgRe 4 2
71. There are _______ gravitational lines of force inside a 78. Radius of moon is 1/4 times that of earth and mass is 1/81
spherically symmetric shell. times that of earth. The point at which gravitational field
(a) infinitely many due to earth becomes equal and opposite to that of moon, is
(b) zero (Distance between centres of earth and moon is 60R, where
(c) varying number depending upon surface area R is radius of earth)
(d) varying number depending upon volume (a) 5.75 R from centre of moon
72. Intensity of the gravitational field inside the hollow (b) 16 R from surface of moon
spherical shell is (c) 53 R from centre of earth
(a) variable (b) minimum (d) 54 R from centre of earth
(c) maximum (d) zero 79. Let V and E denote the gravitational potential and
73. The gravitational potential energy associated with two gravitational field at a point. It is possible to have
particles separated by a distance r, when r ® ¥, is given by (a) V = 0 and E = 0 (b) V = 0 and E ¹ 0
G m1 m 2 -G m1 m 2 (c) V ¹ 0 and E = 0 (d) All of the above
(a) (b) 80. Assertion: The gain in potential energy of an object of mass m
r r
(c) zero (d) infinity 1
raised to height equal to the radius of earth is mg R
74. Which of the following is always positive? 2
1
(a) Potential energy of an object Reason: Kinetic energy at surface = P.E at the top mv2
2
(b) Total energy of a satellite 1
(c) Kinetic energy and at the top v = gR . \ PE = mgR.
2
(d) None of these (a) Assertion is correct, reason is correct; reason is a
75. On the surface of earth acceleration due to gravity is g and correct explanation for assertion.
gravitational potential is V. Match the following: (b) Assertion is correct, reason is correct; reason is not a
Column I Column -II correct explanation for assertion
(A) At height h = R, (1) decreases by a (c) Assertion is correct, reason is incorrect
(d) Assertion is incorrect, reason is correct.
GRAVITATION 105

81. A uniform spherical shell gradually shrinks maintaining its


2GM 4GM
shape. The gravitational potential at the centre (c) , (d) None of these
(a) increases (b) decreases 3R 3R
(c) remains constant (d) cannot say 90. The radius of the earth is reduced by 4%. The mass of the
82. The gravitational potential at the centre of a square of side earth remains unchanged. What will be the change in
'a' and four equal masses (m each) placed at the corners of a escape velocity?
square is (a) Increased by 2% (b) Decreased by 4%
(c) Increased by 6% (d) Decreased by 8%
Gm 91. Time period of a simple pendulum inside a satellite orbiting
(a) Zero (b) 4 2
a earth is
Gm Gm 2 (a) zero (b) ¥ (c) T (d) 2 T
(c) -4 2 (d) -4 2 92. A person sitting in a chair in a satellite feels weightless
a a
because
(a) the earth does not attract the objects in a satellite
Topic 5: Motion of Satellites, Escape Speed (b) the normal force bythe chair on the person balances
and Orbital Velocity the earth's attraction
(c) the normal force is zero
83. To have an earth synchronous satellite it should be launched (d) the person in satellite is not accelerated.
at the proper height moving from 93. The relay satellite transmits the TV programme continuously
(a) north to south in a polar plane from one part of the world to another because its
(b) east to west in an equatorial plane (a) period is greater than the period of rotation of the earth
(c) south to north in a polar plane (b) period is less than the period of rotation of the earth
(d) west to east in an equatorial plane about its axis
84. A missile is launched with a velocity less than escape (c) period has no relation with the period of the earth about
velocity. The sum of its kinetic and potential energies is its axis
(a) zero (d) period is equal to the period of rotation of the earth
(b) negative about its axis
(c) positive 94. If a satellite is orbiting the earth very close to its surface, then
(d) may be positive, negative or zero. the orbital velocity mainly depends on
85. The orbital speed of Jupiter is (a) the mass of the satellite
(a) greater than the orbital speed of earth (b) the radius of earth
(b) less than the orbital speed of earth (c) the orbital radius
(c) equal to the orbital speed of earth (d) the mass of earth
(d) zero 95. Two satellites of masses m1 and m2 (m1 > m2) are revolving
86. The total energy of a circularly orbiting satellite is round the earth in circular orbits of radii r1 and r2 (r1 > r2)
(a) twice the kinetic energy of the satellite respectively. Which of the following statements is true
(b) half the kinetic energy of the satellite regarding their velocities v1 and v2?
(c) twice the potential energy of the satellite (a) v1 = v2 (b) v1 < v2
(d) half the potential energy of the satellite (c) v1 > v2 (d) (v1 / r2) = (v2 / r2)
87. Geo-stationary satellite is one which 96. The escape velocity of a body depends upon mass as
(a) remains stationary at a fixed height from the earth’s (a) m0 (b) m1 (c) m2 (d) m3.
surface 97. If Ve is escape speed from the earth and Vp is that from a
(b) revolves like other satellites but in the opposite direction planet of half the radius of earth, then
of earth’s rotation
(c) revolves round the earth at a suitable height with same Vp Vp
angular velocity and in the same direction as earth does (a) Ve = Vp (b) Ve = (c) Ve = 2Vp (d) Ve =
2 4
about its own axis
98. Assertion : If an object is projected from earth surface with
(d) None of these
88. An artificial satellite orbiting the earth does not fall down escape velocity path of object will be parabola.
because the earth’s attraction Reason : When object is projected with velocity less than
(a) is balanced by the attraction of the moon escape velocity from horizontal surface and greater than
(b) vanishes at such distances orbital velocity path of object will be ellipse.
(c) is balanced by the viscous drag produced by the (a) Assertion is correct, reason is correct; reason is a
atmosphere correct explanation for assertion.
(d) produces the necessary acceleration of its motion in a (b) Assertion is correct, reason is correct; reason is not a
curved path correct explanation for assertion
89. The minimum and maximum distances of a satellite from (c) Assertion is correct, reason is incorrect
centre of earth are 2R and 4R respectively, where R is the (d) Assertion is incorrect, reason is correct.
radius of earth. The minimum and maximum speeds of the 99. When does the object in a satellite escapes to infinity?
satellite will be (a) When the total energy is positive
(b) When total energy is zero
GM 2GM GM 2GM (c) Both (a) & (b)
(a) , (b) , (d) None of these
R R 6R 3R
EBD_7418
106 PHYSICS

100. What is the distance of a geostationary satellite from the (a) 1.25 km/s (b) 2.49 km/s (c) 3.7 km/s (d) 5.6 km/s
earth's centre? 109. A planet in a distant solar system is 10 times more massive
(a) 4.22 × 104 km (b) 4.22 × 104 m than the earth and its radius is 10 times smaller. Given that
6 (d) 4.22 × 106 m
(c) 4.22 × 10 km the escape velocity from the earth's surface is 11 km s–1, the
101. Select the incorrect statements from the following. escape velocity from the surface of the planet would be
I. The orbital velocity of a satellite increases with the (a) 1.1 km s–1 (b) 11 km s–1 (c) 110 km s–1 (d) 0.11km s–1
radius of the orbit 110. A satellite revolves around the earth of radius R in a circular
II. Escape velocity of a particle from the surface of the orbit of radius 3R. The percentage increase in energy
earth depends on the speed with which it is fired required to lift it to an orbit of radius 5R is
III. The time period of a satellite does not depend onthe (a) 10 % (b) 20 % (c) 30 % (d) 40 %
radius of the orbit 111. The escape velocity from the earth's surface is 11 km/s. The
IV. The orbital velocity is inversely proportional to the escape velocity from a planet having twice the radius and
sequare root of the radius of the orbit. same mean density as that of earth is
(a) I and II (b) I and IV (a) 5.5 km/s (b) 11 km/s
(c) I, II and IV (d) I, II and III (c) 22 km/s (d) None of these
102. Two satellites of masses m and 2m are revolving around a 112. The kinetic energy of a satellite in its orbit around the earth
planet of mass M with different speeds in orbits of radii r is E. What should be the kinetic energy of the satellite so as
and 2r respectively. The ratio of minimum and maximum to enable it to escape from the gravitational pull of the earth?
forces on the planet due to satellites is (a) 4 E (b) 2 E (c) 2 E (d) E
1 113. A satellite of mass ‘m’, moving around the earth in a circular
(a) orbit of radius R, has angular momentum L. The areal velocity
2
r of satellite is (Me = mass of earth)
1 (a) L /2m (b) L /m (c) 2L /m (d) 2L /Me
(b) 2r M
4 114. The radii of circular orbits of two satellites A and B of the
1 earth, are 4R and R, respectively. If the speed of satellite A is
(c) 3 V, then the speed of satellite B will be
3
(d) None of these (a) 3 V/4 (b) 6 V (c) 12 V (d) 3 V/2
103. If suddenly the gravitational force of attraction between 115. The least energy required to launch a satellite of mass 100
earth and a satellite revolving around it becomes zero, then kg from the surface of a planet of mass M and radius 3200
the satellite will km in a circular orbit at an altitude of 6400 km is:
(a) continue to move in its orbit with same velocity GM GM GM GM
(b) move tangentialy to the original orbit in the same (a) (b) (c) (d)
38400 8860 98600 9580
velocity 116. A rocket is fired vertically from the surface of mars with a
(c) become stationary in its orbit speed of 2 km/s. If 20% of its initial energy is lost due to
(d) move towards the earth
martian atmosphere resistance, how far will the rocket go
104. A geostationary satellite is orbiting the earth at a height of
5R above that surface of the earth, R being the radius of the from the surface of mars before returning to it? Mass of
earth. The time period of another satellite in hours at a height mars = 6.4 × 1023 kg, radius of mars = 3395 km, G = 6.67 ×
of 2R from the surface of the earth is 10–11 Nm2/kg2.
6 (a) 1655 km (b) 1200 km
(a) 5 (b) 10 (c) 6 2 (d) (c) 1155 km (d) 1055 km
2
105. The mass of a spaceship is 1000 kg. It is to be launched from 117. A satellite can be in a geostationary orbit around earth at a
the earth's surface out into free space. The value of g and R distance r from the centre. If the angular velocity of earth
(radius of earth) are 10 m/s2 and 6400 km respectively. The about its axis doubles, a satellite can now be in a
required energy for this work will be geostationary orbit around earth if its distance from the
(a) 6.4 × 1011J (b) 6.4 × 108 J centre is
(c) 6.4 × 109 J (d) 6.4 × 1010 J
106. For a satellite moving in an orbit around the earth, the ratio r r r r
of kinetic energy to potential energy is (a) (b) (c) 1/3 (d)
2 2 2 (4) (2)1 / 3
1 1
(a) (b) (c) 2 (d) 2 118. A satellite is launched in the equatorial plane in such a way
2 2 that it can transmit signals upto 60° latitude on the earth.
107. A small satellite is revolving near earth’s surface. It’s orbital The angular velocity of the satellite is
velocity will be nearly
(a) 8 km/sec (b) 11.2 km/sec GM GM
(c) 10 km/sec (d) 6 km/sec (a) 3 (b)
8R 2R 3
108. The moon has a mass of 1/81 that of the earth and a radius
of 1/4 that of the earth. The escape speed from the surface
of the earth is 11.2 km/s. The escape speed from the surface GM 3 3GM
(c) (d)
of the moon is: 4R 3 8R 3
GRAVITATION 107

119. A satellite is revolving round the earth in a circular orbit of 120. A satellite is seen after each 8 hours over equator at a place
radius 'a' with velocity v0. A particle of mass m is projected on the earth when its sense of rotation is opposite to the
from the satellite in forward direction with relative velocity earth. The time interval after which it can be seen at the
é 5 ù same place when the sense of rotation of earth and satellite
V =ê –1ú V0 . During subsequent motion of the particle is same will be –
ë 4 û
total energy is (a) 8 hours (b) 12 hours
(c) 24 hours (d) 6 hours
(a) –3G Me m/8a (b) zero
(c) –5G Me m/6a (d) ¥

Exercise 2 : Exemplar & Past Year MCQs


NCERT Exemplar MCQs gravitational force on it is not central
(c) is not elliptical but will necessarily be a closed curve
1. The earth is an approximate sphere. If the interior contained (d) deviates considerably from being elliptical due to
matter which is not of the same density everywhere, then influence of planets other than the earth
on the surface of the earth, the acceleration due to gravity 6. In our solar system, the inter-planetary region has chunks
(a) will be directed towards the centre but not the same of matter (much smaller in size compared to planets) called
everywhere asteroids. They
(b) will have the same value everywhere but not directed (a) will not move around the sun, since they have very
towards the centre small masses compared to the sun
(c) will be same everywhere in magnitude directed towards (b) will move in an irregular way because of their small
the centre masses and will drift away into outer space
(d) cannot be zero at any point (c) will move around the sun in closed orbits but not obey
2. As observed from the earth, the sun appears to move in an Kepler's laws
approximate circular orbit. For the motion of another planet (d) will move in orbits like planets and obey Kepler's laws
like mercury as observed from the earth, this would 7. Choose the wrong option.
(a) be similarly true (a) Inertial mass is a measure of difficulty of accelerating a
(b) not be true because the force between the earth and body by an external force whereas the gravitational
mercury is not inverse square law mass is relevant in determining the gravitational force
(c) not be true because the major gravitational force on on it by an external mass
mercury is due to the sun (b) That the gravitational mass and inertial mass are equal
(d) not be true because mercury is influenced by forces is an experimental result
other than gravitational force (c) That the acceleration due to gravity on the earth is the
3. Different points in the earth are at slightly different same for all bodies is due to the equality of gravitational
distances from the sun and hence experience different forces mass and inertial mass
due to gravitation. For a rigid body, we know that if various (d) Gravitational mass of a particle like proton can depend
forces act at various points in it, the resultant motion is as if on the presence of neighbouring heavy objects but
a net force acts on the CM (centre of mass) causing the inertial mass cannot
translation and a net torque at the CM causing rotation 8. Particles of masses 2M, m and M are respectively at points A,
around an axis through the CM. For the earth-sun system
1
(approximating the earth as a uniform density sphere). B and C with AB = ( BC ) × m is much-much smaller than
(a) the torque is zero 2
(b) the torque causes the earth to spin M and at time t = 0, they are all at rest as given in figure.
(c) the rigid body result is not applicable since the earth is At subsequent times before any collision takes place.
not even approximately a rigid body
(d) the torque causes the earth to move around the sun A r B C
4. Satellites orbitting the earth have finite life and sometimes 2M m M
debris of satellites fall to the earth. This is because
(a) m will remain at rest
(a) the solar cells and batteries in satellites run out
(b) the laws of gravitation predict a trajectory spiralling (b) m will move towards M
inwards (c) m will move towards 2M
(c) of viscous forces causing the speed of satellite and (d) m will have oscillatory motion
hence height to gradually decrease Past Year MCQs
(d) of collisions with other satellites
5. Both the earth and the moon are subject to the gravitational 9. A black hole is an object whose gravitational field is so strong
force of the sun. As observed from the sun, the orbit of the that even light cannot escape from it. To what approximate
moon radius would earth (mass = 5.98 × 1024 kg) have to be
(a) will be elliptical compressed to be a black hole? [AIPMT 2014, C]
(b) will not be strictly elliptical because the total (a) 10– 9 m (b) 10– 6 m (c) 10– 2 m (d) 100 m
EBD_7418
108 PHYSICS

10. The potential energy of a satellite of mass m and revolving 16. From a solid sphere of mass M and radius R, a spherical
at a height Re above the surface of earth where Re = radius portion of radius R/2 is removed, as shown in the figure.
of earth, is [AIIMS 2014, A] Taking gravitational potential V = 0 at r = ¥, the potential at
-m g R e -m g R e -m g R e the centre of the cavity thus formed is : [JEE Main 2015, S]
(a) – m g Re (b) (c) (d) (G = gravitational constant)
2 3 4
11. Four particles, each of mass M and equidistant from each -2GM -2GM
other, move along a circle of radius R under the action of (a) (b)
3R R
their mutual gravitational attraction. The speed of each
particle is: [JEE Main 2014, C] -GM -GM
(c) (d)
2R R
GM GM 17. An artificial satellite is moving in a circular orbit around the
(a) (b) 2 2
R R earth with a speed equal to half the magnitude of the escape
velocity from the earth. The height (h) of the satellite above
(c)
GM
R
(
1+ 2 2 ) (d)
1 GM
2 R
(
1+ 2 2 ) the earth’s surface is (Take radius of earth as Re)
[BITSAT 2015, A]
12. A spherically symmetric gravitational system of particles (a) h = Re2 (b) h = Re (c) h = 2Re (d) h = 4Re
18. The ratio of escape velocity at earth (ve) to the escape
ìr0 for r £ R
has a mass density r = í0 for r > R velocity at a planet (vp) whose radius and mean density are
î twice as that of earth is : [NEET 2016, A]
where r0 is a constant. A test mass can undergo circular (a) 1 : 2 (b) 1 : 2 2 (c) 1 : 4 (d) 1 : 2
motion under the influence o the gravitational field of par- 19. At what height from the surface of earth the gravitational
ticles. Its speed V as a function of distance r (0 < r < ¥) from potential and the value of g are –5.4 × 107 J kg–1 and 6.0 ms–2
the centre of the system is represented by respectively ? Take the radius of earth as 6400 km :
[BITSAT 2014, C]
V V
[NEET 2016, A]
(a) 2600 km (b) 1600 km (c) 1400 km (d) 2000 km
20. The distance of neptune and saturn from the sun is nearly
1013 and 1012 meter respectively. Assuming that they move
(a) (b)
in circular orbits, their periodic times will be in the ratio
R r R r [AIIMS 2016, A]
V V (a) 10 (b) 100 (c) 10 10 (d) 1000
21. A satellite is revolving in a circular orbit at a height 'h' from
the earth's surface (radius of earth R; h < < R). The minimum
(c) (d) increase in its orbital velocity required, so that the satellite
R r R r could escape from the earth's gravitational field, is close to :
(Neglect the effect of atmosphere.) [JEE Main 2016, C]
13. A satellite S is moving in an elliptical orbit around the earth.
The mass of the satellite is very small compared to the mass (a) gR / 2 (b) gR ( )
2 -1
of the earth. Then, [AIPMT 2015, F] (c) 2gR (d) gR
(a) the total mechanical energy of S varies periodically with
time. 22. What should be the velocity of rotation of earth due to
(b) the linear momentum of S remains constant in rotation about its own axis so that the weight of a person
magnitude. 3
becomes of the present weight at the equator. Equatorial
(c) the acceleration of S is always directed towards the 5
centre of the earth. radius of the earth is 6400 km. [BITSAT 2016, S]
(d) the angular momentum of S about the centre of the (a) 8.7 × 10–7 rad/s (b) 7.8 × 10–4 rad/s
earth changes in direction, but its magnitude remains (c) 6.7 × 10–4 rad/s (d) 7.4 × 10–3 rad/s
constant.
14. A remote - sensing satellite of earth revolves in a circular orbit at 23. Kepler's third law states that square of period of revolution
a height of 0.25 × 106 m above the surface of earth. If earth's (T) of a planet around the sun, is proportional to third power
radius is 6.38 × 106 m and g = 9.8 ms-2, then the orbital speed of of average distance r between sun and planet i.e. T 2 = Kr3
the satellite is: [AIPMT 2015, A] here K is constant. If the masses of sun and planet are M
(a) 8.56 km s-1 (b) 9.13 km s-1 and m respectively then as per Newton's law of gravitation
(c) 6.67 km s-1 (d) 7.76 km s-1 GMm
15. Taking the gravitational potential at a point infinte distance force of attraction between them is F = 2 , here G is
r
away as zero, the gravitational potential at a point A is –5 gravitational constant. The relation between G and K is
unit. If the gravitational potential at point infinite distance
described as [BITSAT 2016, C]
away is taken as + 10 units, the potential at point A is
[AIIMS 2015, S] (a) GMK = 4p2 (b) K = G
(a) – 5 unit (b) + 5 unit (c) + 10 unit (d) + 15 unit 1
(c) K = (d) GK = 4p2
G
GRAVITATION 109

24. The acceleration due to gravity at a height 1 km above the by V(r) = kr (k is a positive constant). Period of revolution of
earth is the same as at a depth d below the surface of earth. the body is proportional to : [BITSAT 2017, C]
Then [NEET 2017, A] (a) R 1/2 (b) R –1/2 (c) R –3/2 (d) R –5/2
3 1 30. The kinetic energies of a planet in an elliptical orbit about
(a) d = 1 km (b) d = km (c) d = 2 km (d) d = km the Sun, at positions A, B and C are KA, KB and KC,
2 2
25. The change in the value of ‘g’ at a height ‘h’ above the respectively. AC is the major axis and SB is perpendicular
surface of the earth is the same as at a depth ‘d’ below the to AC at the position of the Sun S as shown in the figure.
surface of earth. When both ‘d’ and ‘h’ are much smaller Then [NEET 2018, C]
than the radius of earth, then which one of the following is B
correct? [AIIMS 2017, A]
3h h A C
S
(a) d = (b) d = (c) d = h (d) d =2 h
2 2
26. Assertion : Space rocket are usually launched in the (a) KA < KB < KC (b) KA > KB > KC
equatorial line from west to east (c) KB > KA > KC (d) KB < KA < KC
Reason : The acceleration due to gravity is minimum at the 31. If the mass of the Sun were ten times smaller and the
equator. [AIIMS 2017, C] universal gravitational constant were ten times larger in
(a) Assertion is correct, reason is correct; reason is a
magnitude, which of the following is not correct?
correct explanation for assertion.
(b) Assertion is correct, reason is correct; reason is not a [NEET 2018, C]
correct explanation for assertion (a) Raindrops will fall faster
(b) Walking on the ground would become more difficult
(c) Assertion is correct, reason is incorrect
(c) ‘g’ on the Earth will not change
(d) Assertion is incorrect, reason is correct. (d) Time period of a simple pendulum on the Earth would
27. Two astronauts are floating in gravitation free space after decrease
having lost contact with their spaceship. The two will 32. The gravitational field in a region is given by
(a) move towards each other [JEE Main 2017, C] ®
(b) move away from each other g = 5N / kgiˆ + 12N / kgjˆ . The change in the gravitational
(c) become stationary potential energy of a particle of mass 1 kg when it is taken
(d) keep floating at the same distance between them from the origin to a point (7 m, – 3 m) is: [AIIMS 2018, A]
28. What is the minimum energy required to launch a satellite of
mass m from the surface of a planet of mass M and radius R (a) 71 J (b) 13 58J (c) –71 J (d) 1 J
in a circular orbit at an altitude of 2R? [BITSAT 2017, C] 33. Infinite number of masses, each 1 kg are placed along the
5GmM 2GmM GmM GmM x-axis at x = ± 1m, ± 2m, ± 4m, ± 8m, ± 16m ..... the magnitude
(a) (b) (c) (d) of the resultant gravitational potential in terms of
6R 3R 2R 2R
29. A body moves in a circular orbit of radius R under the action gravitational constant G at the orgin (x = 0) is
of a central force. Potential due to the central force is given [BITSAT 2018, A]
(a) G/2 (b) G (c) 2G (d) 4G

Exercise 3 : Try If You Can


1. A straight rod of length L extends from x = a to x = L + a. Find 2
the gravitational force it exerts on a point mass m at x = 0 if the M s r2 M s æ r2 ö
(a) ´ 100 (b) ´ 100
linear density of rod µ =A + Bx2 . M m r1 M m çè r1 ÷ø
é æ1 1 ö ù 2 2
éA ù æ r ö Mm æ r1 ö M s
(a) G m ê + BL ú (b) Gm ê A çè - ÷ + BL ú (c) 2ç 1 ÷ ´ 100 (d) ç ÷ ´ 100
ëa û ë a a + Lø û è r2 ø M s è r2 ø M m
é A ù é Aù
(c) Gm ê BL + (d) G m ê BL - ú 5. If the Earth has no rotational motion, the weight of a person
ë a + L úû ë aû on the equator is W. Determine the speed with which the
2. The true weight of an object in a geostationary satellite earth would have to rotate about its axis so that the person
that weighed exactly 10.0N at the north pole at the equator will weight 3/4W. Radius of the Earth is
(a) 0.23 N (b) 2.33 N (c) 10.0 N (d) 12.1 N 6400 km and g =10 m/s2.
3. Two spherical bodies of mass M and 5M & radii R & 2R (a) 1.1×10–3 rad/s (b) 0.83×10–3 rad/s
respectively are released in free space with initial separation –3
(c) 0.63 × 10 rad/s (d) 0.28×10–3 rad/s
between their centres equal to 12 R. If they attract each other
due to gravitational force only, then the distance covered by 6. The radii of two planets are respectively R1 and R2 and their
the smaller body just before collision is densities are respectively r1 and r2. The ratio of the
(a) 2.5 R (b) 4.5 R (c) 7.5 R (d) 1.5 R accelerations due to gravity at their surfaces is
4. The percentage change in the acceleration of the earth towards r1 r2
the sun from a total eclipse of the sun to the point where the (a) g1 : g 2 = 2 : 2 (b) g1 : g 2 = R1R2 : r1r2
R1 R2
moon is on a side of earth directly opposite to the sun is
(c) g1 : g 2 = R1R2 : R2 r1 (d) g1 : g 2 = R1r1 : R2 r2
EBD_7418
110 PHYSICS

7. Let g be the acceleration due to gravity at earth’s surface (a) (R2g/w2)1/3 (b) (Rg/w2)1/3
and K be the rotational kinetic energy of the earth. Suppose (c) (R w /g)
2 2 1/3 (d) (R2g/w)1/3
the earth’s radius decreases by 2% keeping all other 12. Imagine a light planet revolving around a very massive star
quantities same, then in a circular orbit of radius R with a period of revolution T. If
(a) g decreases by 2% and K decreases by 4% the gravitational force of attraction between the planet and
(b) g decreases by 4% and K increases by 2%
the star proportional to R –5/2, then :
(c) g increases by 4% and K decreases by 4%
(a) T 2 µ R 3 (b) T 2 µ R 7/2
(d) g decreases by 4% and K increases by 4% (c) T 2 µ R3/2 (d) T 2 µ R 3.75
8. Gravitational field at the centre y
13. For a satellite orbiting in an orbit, close to the surface of
of a semicircle formed by a thin
earth, to escape, what is the percentage increase in the
wire AB of mass m and length l
kinetic energy required?
is
A O B x
(a) 41% (b) 61% (c) 81% (d) 98%
Gm Gm 14. Two concentric uniform shells of mass
(a) along +x-axis (b) 2
along +y-axis M1 and M2 are as shown in the figure. A M2
l2
pl
2pGm particle of mass m is located just within
2pGm a M1
(c) along +x-axis (d) along +y-axis the shell M 2 on its inn er surface.
l2 l2 Gravitational force on ‘m’ due to M1 and
9. In a certain region of space, gravitational field is given b
M will be m
by I = –(K/r). Taking the reference point to be at r = r0 2

with V =V0, find the potential. GM 1 m


(a) zero (b)
r r0 b2
(a) K log + V0 (b) K log + V 0 G (M1 + M 2 ) m
r0 r (c) (d) None of these
r r0 b2
(c) K log - V0 (d) log - V0 r 15. A satellite of mass m is orbiting the earth in a circular orbit of
r0 r radius R. It starts losing energy due to small air resistance at
10. The gravitational field due to a mass distribution is the rate of C J/s. Find the time taken for the satellite to reach
3
E = K/x in the x-direction. (K is a constatnt). Taking the the earth.
gravitational potential to be zero at infinity, its value at a
distance x is GMm é 1 1 ù GMm é 1 1 ù
(a) – ú (b) +
(a) K/x (b) K/2x (c) K/x 2 (d) K/2x2 ê
C ëR rû 2C êë R r úû
11. The mean radius of earth is R, its angular speed on its own
axis is w and the acceleration due to gravity at earth's surface GMm é 1 1 ù 2GMm é 1 1 ù
(c) êR – rú (d) +
is g. What will be the radius of the orbit of a geostationary 2C ë û C êë R r úû
satellite?
ANSWER KEYS
Exercise 1 : Topic-wise MCQs
1 (d) 14 (d) 27 (d) 40 (c) 53 (b) 66 (a) 79 (d) 92 (c) 105 (d) 118 (a)
2 (b) 15 (b) 28 (a) 41 (d) 54 (d) 67 (a) 80 (c) 93 (d) 106 (a) 119 (a)
3 (b) 16 (c) 29 (c) 42 (b) 55 (b) 68 (b) 81 (a) 94 (b) 107 (a) 120 (c)
4 (d) 17 (b) 30 (c) 43 (c) 56 (c) 69 (a) 82 (c) 95 (b) 108 (b)
5 (d) 18 (d) 31 (c) 44 (b) 57 (a) 70 (b) 83 (d) 96 (a) 109 (c)
6 (c) 19 (a) 32 (c) 45 (c) 58 (b) 71 (b) 84 (b) 97 (c) 110 (b)
7 (c) 20 (c) 33 (a) 46 (c) 59 (a) 72 (d) 85 (b) 98 (c) 111 (c)
8 (a) 21 (b) 34 (b) 47 (d) 60 (d) 73 (c) 86 (d) 99 (c) 112 (b)
9 (d) 22 (d) 35 (c) 48 (a) 61 (a) 74 (c) 87 (c) 100 (a) 113 (a)
10 (b) 23 (b) 36 (a) 49 (a) 62 (a) 75 (b) 88 (d) 101 (d) 114 (b)
11 (c) 24 (a) 37 (b) 50 (d) 63 (d) 76 (b) 89 (b) 102 (c) 115 (a)
12 (b) 25 (b) 38 (b) 51 (c) 64 (b) 77 (b) 90 (a) 103 (b) 116 (a)
13 (a) 26 (c) 39 (c) 52 (d) 65 (a) 78 (d) 91 (b) 104 (c) 117 (c)
Exercise 2 : Exemplar & Past Year MCQs
1 (d) 5 (b) 9 (c) 13 (c) 17 (b) 21 (b) 25 (d) 29 (a) 33 (c)
2 (c) 6 (d) 10 (b) 14 (d) 18 (b) 22 (b) 26 (b) 30 (a)
3 (a) 7 (d) 11 (d) 15 (b) 19 (a) 23 (a) 27 (a) 31 (c)
4 (c) 8 (c) 12 (b) 16 (d) 20 (c) 24 (c) 28 (a) 32 (d)
Exercise 3 : Try If You Can
1 (b) 3 (c) 5 (c) 7 (c) 9 (a) 11 (a) 13 (a) 15 (c)
2 (a) 4 (c) 6 (d) 8 (d) 10 (d) 12 (b) 14 (b)
9
Chapter
MECHANICAL PROPERTIES
OF SOLIDS

Trend
Analysis of NEET and AIIMS (Year 2010-2018)
5

4
Number of Questions

3
AIPMT/NEET
2 AIIMS

0
2010 2011 2012 2013 2014 2015 2016 2017 2018
Year

Trend
Analysis of JEE Main and BITSAT (Year 2010-2018)
5

4
Number of Questions

3
JEE Main
2 BITSAT

0
2010 2011 2012 2013 2014 2015 2016 2017 2018
Year

Chapter Utility Score (CUS)


Exam Weightage Important Concepts Difficulty Level CUS (Out of 10)
NEET 2 Hooke's Law, Stress-Strain
AIIMS 3 Curve and Elastic 3/5 5.5/10
JEE Main 2 Moduli
BITSAT 2
EBD_7418
112 PHYSICS
MECHANICAL PROPERTIES OF SOLIDS 113

Exercise 1 : Topic-wise MCQs


10. In magnitude hydraulic stress is equal to
Topic 1: Hooke's Law & Young's Modulus
(a) hydraulic force (b) hydraulic pressure
1. The expression of force constant for a spring following (c) restoring force (d) hydraulic strain
Hooke's law is given by 11. Substances which can be stretched to cause large strains
are called
YA YA YA Dl YAl (a) brittle (b) ductile (c) plastic (d) elastomer
(a) k = (b) k = (c) k = (d) k =
l Dl l Dl 12. The reason for the change in shape of a regular body is
2. For an equal stretching force F, the young's modulus (Ys) for (a) volume stress (b) shearing strain
steel and rubber (Yr) are related as (c) longitudinal strain (d) metallic strain
(a) Ys = Yr (b) Ys < Yr (c) Ys > Yr (d) Ys ³ Yr 13. Longidudinal stress depends on
3. which of the following is correct for young's modulus of (a) area (b) length (c) volume (d) mass
elasticity (g)? [where r = radius of cross section of wire] 14. If h = s hear mokulus and g = young's modulus, then in
l = length of wire general
(a) g µ r2 (a) g > h (b) g = h
(b) g µ l3 (c) g < h (d) none of these
(c) g µ 1/r2 15. Which of the following affects the elasticity of a substance?
(d) g µ l2 (a) Hammering and annealing
4. If the length of a wire is reduced to half, then it can hold the (b) Change in temperature
(a) half load (b) same load (c) Impurity in substance
(c) double load (d) one fourth load (d) All of the above
5. The Young’s modulus of a perfectly rigid body is 16. If the load is increased beyond the _______ point, the strain
(a) unity increases rapidly for even a small change in the stress.
(b) zero (a) elastic point (b) yield point
(c) infinity (c) plastic point (d) fracture point
(d) some finite non-zero constant 17. What is the phenomenon of temporary delay in regaining
6. According to Hooke’s law of elasticity, if stress is increased, the original configuration by an elastic body, after the
then the ratio of stress to strain removel of a deforming force?
(a) becomes zero (b) remains constant (a) Elastic fatigue (b) Elasticity
(c) decreases (d) increases (c) Plasticity (d) Elastic after effect
7. The diagram shows a force - extension graph for a rubber 18. Which of the following types of stress causes no change in
band. Consider the following statements : shape?
I. It will be easier to compress this rubber than expand it (a) compressive stress (b) hydraulic stress
II. Rubber does not return to its original length after it is (c) oblique stress (d) none of these
stretched 19. If a mass M produces an elongation of DL in a wire of radius
III. The rubber band will get heated if it is stretched and r and length L, then the young's modulus of the material of
released the wire is given by
Mg Mg ´ DL
(a) Y = (b) Y =
2
(p r ´ DL) (p r 2 ´ L)
Mg ´ L M ´ DL
Extension (c) Y = 2
(d) Y =
(p r ´ DL) (p r 2 ´ L)
20. A vertical metal cylinder of radius 2 cm and length 2 m is
fixed at the lower end and a load of 100 kg is put on it. Find
Force the strain. [Youn's modulus of the metal = 2 × 1011 N/m2]
Which of these can be deduced from the graph? (a) 4 × 10–6 (b) 3 × 10–8 (c) 2 × 10–9 (d) 6 × 10–8
(a) III only (b) II and III 21. The graph given is a stress-strain curve for
(c) I and III (d) I only (a) elastic objects
8. A 2 m long rod of radius 1 cm which is fixed from one end is
Stress (N/ m )

1.0
2

given a force of 8 N. The longitudinal strain developed will (b) plastics


be
[take g = 2.5×1011N/m2] 0.5
(c) elastomers
(a) 10–8 (b) 10–6 (c) 10–5 (d) 10–4
9. The restoring force per unit area is known as 0
(d) None of these 0.5 1.0
(a) strain (b) elasticity(c) stress (d) plasticity Strain
EBD_7418
114 PHYSICS

22. A beam of metal supported at the two edges is loaded at the 29. A steel ring of radius r and cross-section area 'A' is fitted on
centre. The depression at the centre is proportional to to a wooden disc of radius R(R > r). If Young's modulus be
E, then the force with which the steel ring is expanded is
d
R æ R-r ö
(a) AE (b) AE ç ÷
(a) Y 2 (b) Y (c) 1/Y (d) 1/Y 2 r è r ø
23. Column -I Column-II E æ R-r ö Er
A çè A ÷ø
(A) Equal force acting (1) Balance (c) (d)
AR
perpendicular to each the net weight to be
point on a spherical supported 30. Two persons pull a rope towards themselves. Each person
surface exerts a force of 100 N on the rope. Find the Young's modulus
(B) Cross-sectional area (2) Higher modulus of of the material of the rope if it extends in length by 1 cm.
of the rope used in elasticity Original length of the rope = 2 m and the aea of cross-section
giant structures = 2 cm2.
(C) Steel in structural (3) Reduction in volume
(a) 108 N/m2 (b) 107 N/m2
designs without change in shape 6 2
(D) Stress-strain curve (4) Inversely depends on (c) 10 N/m (d) 105 N/m2
the yeild strength 31. If stress/strain is x in eastic region and y in the region of
yield, then
(a) (A) ® (4); (B) ® (1, 3); (C) ® (2) ; (D) ® (1)
(b) (A) ® (3); (B) ® (1, 2); (C) ® (4) ; (D) ® (3, 4) (a) x = y (b) x > y (c) x < y (d) x = 2 y
(c) (A) ® (2); (B) ® (1) ; (C) ® (4) ; (D) ® (3) 32. What per cent of length of wire increases by applying a
(d) (A) ® (3); B ® (1, 4) ; (C) ® (2) ; (D) ® (2) stress of 1 kg weight/mm2 on it?
24. The adjacent graph shows the (Y = 1 × 1011 N/m2 and 1 kg weight = 9.8 newton)
extension (Dl) of a wire of 4 (a) 0.0067% (b) 0.0098%
(l ×10 )m

3 (c) 0.0088% (d) 0.0078%


length 1m suspended from the
–4

2
top of a roof at one end with a 33. Assertion : The stress-strain graphs are shown in the figure
1
for two materials A and B are shown in figure. Young's
load W connected to the other
modulus of A is greater than that of B.
end. if the corss-sectional area 20 40 60 80 W(N)

Stress
A
of the wire is 10–6m2, calculate the Young’s modulus of the B
material of the wire
(a) 2 × 1011 N/m2 (b) 2 × 10–11 N/m2
–12
(c) 2 × 10 N/m 2 (d) 2 × 10–13 N/m2
25. For the given graph, Hooke's law is obeyed in the region Strain
(a) OA C Reason : The Young's modules for small strain is,
A stress
E B Y= = slope of linear portion, of graph; and slope of
(b) C strain
Stress

A is more than slope that of B.


(c) OE (a) Assertion is correct, reason is correct; reason is a
correct explanation for assertion.
(d) OB O Strain
(b) Assertion is correct, reason is correct; reason is not a
correct explanation for assertion
26. If the ratio of radii of two wires of same material is 3 : 1 and (c) Assertion is correct, reason is incorrect
ratio of their lengths is 5 : 1, then the ratio of the normal (d) Assertion is incorrect, reason is correct.
forces that will produce the same extension in the length of 34. An elevator cable is to have a maximum stress of 7 × 107 N/m2
two wires is to allow for appropriate safety factors. Its maximum upward
(a) 2 : 1 (b) 4 : 1 (c) 1 : 4 (d) 1 : 1 acceleratiom is 1.5 m/s2. If the cable has to support the total
27. An iron bar of length l cm and cross section A cm2 is weight of 2000 kg of a loaded elevator, the area of cross–
pulled by a force of F dynes from ends so as to produce an section of the cable should be
elongation Dl cm. Which of the following statement is correct ? (a) 3.28 cm2 (b) 2.38 cm2
(a) Elongation is inversely proportional to length (c) 0.328 cm2 (d) 8.23 cm2
(b) Elongation is directly proportional to cross section A 35. To break a wire, a force of 106 N/m2 is required. If the density
(c) Elongation is inversely proportional to cross-section of the material is 3 × 103 kg/m3, then the length of the wire
(d) Elongation is directly proportional to Young’s modulus which will break by its own weight will be
28. Two wires of equal lengths are made of the same material. (a) 34 m (b) 30 m (c) 300 m (d) 3 m
Wire A has a diameter that is twice as that of wire B. If 36. A rubber cord catapult has cross-sectional area 25 mm2 and
identical weights are suspended from the ends of these initial length of rubber cord is 10 cm. It is stretched to 5 cm
wires, the increase in length is and then released to project a missile of mass 5 gm. Taking
(a) four times for wire A as for wire B Yrubber = 5 × 108 N/m2. Velocity of projected missile is
(b) twice for wire A as for wire B (a) 20 ms–1 (b) 100 ms–1
(c) half for wire A as for wire B (c) 250 ms–1 (d) 200 ms–1
(d) one-fourth for wire A as for wire B
MECHANICAL PROPERTIES OF SOLIDS 115

37. For the same cross-sectional area and for a given load, the 46. The length of elastic string, obeying Hooke’s law is l1
ratio of depressions for the beam of a square cross-section
and circular cross-section is metres when the tension 4N and l 2 metres when the tension
(a) 3 : p (b) p : 3 (c) 1 : p (d) p : 1 is 5N. The length in metres when the tension is 9N is –
38. The length of a metal is l1 when the tension in it is T1 and is (a) 5l1 – 4l2 (b) 5l2 – 4l1
l2 when the tension is T2. The original length of the wire is (c) 9l1 – 8l2 (d) 9l2 – 8l1
47. A steel wire of length l and cross section area A is stretched
l1 + l 2 l1T2 + l 2T1 l1T2 - l 2T1
(a) (b) T + T (c) T - T (d) T1T2 l1l 2 by 1 cm under a given load. When the same load is applied
2 1 2 2 1 to another steel wire of double its length and half of its
39. A light rod of length 2m suspended from the ceiling cross section area, the amount of stretching (extension) is
horizontally by means of two vertical wires of equal length. (a) 0.5 cm (b) 2 cm (c) 4 cm (d) 1.5 cm
A weight W is hung from a light rod as shown in figure. 48. Two wires are made of the same material and have the same
The rod hung by means of a steel wire of volume. However wire 1 has cross-sectional area A and wire
cross-sectional area A1 = 0.1 cm2 and T T2 2 has cross-sectional area 9A. If the length of wire 1 increases
brass wire of cross-sectional area A2 = Steel 1 Brass by Dx on applying force F, how much force is needed to
0.2 cm2. To have equal stress in both 2m stretch wire 2 by the same amount?
wires, T1/T2 = W (a) 16 F (b) 25 F (c) 81 F (d) 64 F
49. The force exerted by a special compression device is given
(a) 1/3 (b) 1/4 (c) 4/3 (d) 1/2 as function of compression x as Fx(x) = kx(x – l) for
40. A metallic rod breaks when strain produced is 0.2%. The 0 £ x £ l, where l is maximum possible compression and k is
Young’s modulus of the material of the rod is 7 × 109 N/m2. a constant. The force exerted by the device under
What should be its area of cross-section to support a load compression is maximum when compression is –
of 104 N ? (a) 0 (b) l/4 (c) l / 2 (d) l/2
(a) 7.1 × 10–8 m2 (b) 7.1 × 10–6 m2 50. What is the minimum diameter of a brass rod if it is to support
a 400N load without exceeding the elastic limit ? Assume
(c) 7.1 × 10–4 m2 (d) 7.1 × 10–2 m2 that the stress for the elastic limit is 379 MPa.
41. A steel wire of original length 1 m and cross-sectional area (a) 1.16 mm (b) 2.32 mm (c) 0.16 mm (d) 1.35 mm
4.00 mm2 is clamped at the two ends so that it lies horizontally 51. The elastic limit of steel is 8 × 108 N/m2 and its Young's
and without tension. If a load of 2.16 kg is suspended from modulus 2 × 1011 N/m2. Find the maximum elongation of a
the middle point of the wire, what would be its vertical half-meter steel wire that can be given without exceeding
depression? the elastic limit.
Y of the steel - 2.0 × 1011N/m2. Take g = 10 m/s2 (a) 2 mm (b) 4mm (c) 5 mm (d) 6mm
(a) 1.5 cm (b) 2.8 cm (c) 3.2 cm (d) 4.1 cm 52. A steel wire and a copper wire of equal length and equal
42. Two wires are made of the same material and have the same cross-sectional area are joined end to end and the
volume. However first wire has cross-sectional area A and combination is subjected to a tension. Find the ratio of
second wire has cross-sectional area 5A. If the length of the stresses developed in the two wires and Y of steel
first wire increases by Dl on applying force f, how much = 2 × 1011 N/m2. Y of copper = 1.3 × 1011 N/m2.
force is needed to stretch second wire by the same amount? (a) 1 (b) 3 (c) 5 (d) 7
(a) 14f (b) 6 f (c) 25 f (d) 9f 53. A force of 6 × 106 Nm–2 is required for breaking a material.
43. When forces are applied on a body such that it is still in Then density r of the material is 3 × 103 kg m–3. If the wire is
static equilibrium, then the extent to which the body gets to break under its own weight, the length of the wire made of
deformed, depends on that material should be (take g = 10 ms–2)
(a) nature of the material (a) 20 m (b) 200 m (c) 100 m (d) 2000 m
(b) magnitude of deforming force 54. A steel wire of cross-sectional area 3 × 10 m2 can with
–6
(c) Both (a) & (b) stand a maximum strain of 10–3. Young's modulus of steel is
(d) None of these
2 × 1011 N/m2 . The maximum mass the wire can hold is:
44. Two wires A and B of same material and of equal length
(a) 40 kg (b) 60 kg (c) 80 kg (d) 100 kg
with the radii in the ratio 1 : 2 are subjected to identical
loads. If the length of A increases by 8 mm, then the 55. A structural steel rod has a radius of 10 mm and length of
increase in length of B is 1.0 m. A 100 kN force stretches it along its length. Young’s
(a) 2 mm (b) 4 mm (c) 8 mm (d) 16 mm modulus of structural steel is 2 × 1011 Nm–2. The percentage
45. The length of an elastic string is a metre when the strain is about
longitudinal tension is 4 N and b metre when the longitudinal (a) 0.16% (b) 0.32% (c) 0.08% (d) 0.24%
tension is 5 N. The length of the string in metre when the 56. A wire elongates by l mm when a load W is hanged from it.
longitudinal tension is 9 N is If the wire goes over a pulley and two weights W each are
1 hung at the two ends, the elongation of the wire will be (in mm)
(a) a – b (b) 5b – 4a (c) 2b – a (d) 4a – 3b
4 (a) l (b) 2l (c) zero (d) l/2
EBD_7418
116 PHYSICS

57. A thick rope of density r and length L is hung from a rigid


support. The Young’s modulus of the material of rope is Y.
(a) r12 / r22 (b) r13 / r23 (c) r24 / r14 (d) r14 / r24
The increase in length of the rope due to its own weight is 67. A copper wire (Y = 1011 N/m2) of length 8 m and steel wire
(a) (1/4) r g L2/Y (b) (1/2) r g L2/Y (Y = 2 × 1011 N/m2) of length 4 m each of 0.5 cm2 cross-
2
(c) r g L /Y (d) r g L/Y section are fastened end to end and stretched with a tension
of 500 N.
Topic 2: Bulk and Rigidity Modulus and Work Column-I Column-II
(A) Elongation in copper (1) 0.25
Done in Stretching a Wire wire in mm
58. Which of the following elastic moduli is used to describe (B) Elongation in steel (2) 1.0
the elastic behaviour of object as they respond to the wire in mm
deforming forces acting on them? (C) Total elongation in mm (3) 0.8
(a) Young's modulus (b) Shear modulus
(c) Bulk modulus (d) All of these 1
(D) Elastic potential energy (4) th the elongation in
59. Which of the following is the correct relation? Y = Young's 4
modulus & G = modulus of rigidity? of the system in joules copper wire
(a) Y < G (b) Y > G (a) (A)®(3), (B)®(4), (C)®(2), (D)®(1)
(c) Y = G (d) None of these (b) (A)®(4), (B)®(2), (C)®(3), (D)®(1)
60. The isothermal bulk modulus of a gas at atmospheric (c) (A)®(1), (B)®(2), (C)®(3), (D)®(4)
pressure is (d) (A)®(2), (B)® (1), (C)®(3), (D)®(4)
(a) 1 mm of Hg (b) 13.6 mm of Hg 68. A metallic wire of length 2.0 m is elongated by 2.0 mm. Area
(c) 1.013 × 105 N/m2 (d) 2.026 × 105 N/m2 of cross-section of the wire is 4.0 mm2. The elastic potential
61. The ratio of shearing stress to the corresponding energy stored in the wire in elongated condition is [young's
shearing strain is called modulus of the metallic wire is = 2 ´ 1011 N/m2]
(a) bulk modulus (b) Young's modulus (a) 8.23 (b) 0.83 (c) 6.23 (d) 0.63
(c) modulus of rigidity (d) None of these 69. Assertion : Bulk modulus of elasticity (k) represents
62. The potential energy U between two atoms in a diatomic incompressibility of the material.
molecules as a function of the distance x between atoms Reason : Bulk modulus of elasticity is proportional to
has been shown in the figure. The atoms are change in pressure.
(a) attracted when x lies between (a) Assertion is correct, reason is correct; reason is a
A and B and are repelled when x correct explanation for assertion.
O
lies between B and C U (b) Assertion is correct, reason is correct; reason is not a
(b) attracted when x lies between correct explanation for assertion
B and C and are repelled when x (c) Assertion is correct, reason is incorrect
lies between A and B (d) Assertion is incorrect, reason is correct.
A B Cx
70. Shearing strain is expressed by
(c) are attracted when they reach B from C (a) angle of twist (b) angle of shear
(d) are repelled when they reach B from A (c) increase in volume (d) decrease in volume
63. The reciprocal of the bulk modulus is called 71. When a pressure of 100 atmosphere is applied on a spherical
(a) modulus of rigidity (b) volume stress
ball, then its volume reduces to 0.01%. The bulk modulus of
(c) volume strain (d) compressibility
64. Which of the following statements is/are wrong? the material of the rubber in dyne/cm2 is
I. Hollow shaft is much stronger than a solid rod of (a) 10 × 1012 (b) 100 × 1012
same length and same mass. (c) 1 × 1012 (d) 10 × 1012
II. Reciprocal of bulk modulus of elasticity is called 72. A uniform cube is subjected to volume compression. If each
compressibility. side is decreased by 1%, then bulk strain is
III. It is difficult to twist a long rod as compared to small (a) 0.01 (b) 0.06 (c) 0.02 (d) 0.03
rod. 73. When a 4 kg mass is hung vertically on a light spring that
(a) I and II (b) II and III (c) III only (d) I only obeys Hooke’s law, the spring stretches by 2 cms. The work
65. Select the correct statement(s) from the following. required to be done by an external agent in stretching this
I. Modulus of rigidity for a liquid is not zero spring by 5 cms will be (g = 9.8 m/sec2)
II. Young's modulus of a material decreases with rise in (a) 4.900 joule (b) 2.450 joule
temperature (c) 0.495 joule (d) 0.245 joule
III. Poisson's ratio is unitless 74. Assertion : Identical springs of steel and copper are equally
(a) I only (b) II only (c) I and II (d) II and III stretched. More work will be done on the steel spring
66. Two rods A and B of the same material and length have their Reason : Steel is more elastic than copper.
radii r1 and r 2 respectively. When they are rigidly fixed at (a) Assertion is correct, reason is correct; reason is a
one end and twisted by the same couple applied at the other correct explanation for assertion.
end, the ratio (b) Assertion is correct, reason is correct; reason is not a
æ Angle of twist at the end of A ö correct explanation for assertion
çè Angle of twist at the end of B ÷ø is (c) Assertion is correct, reason is incorrect
(d) Assertion is incorrect, reason is correct.
MECHANICAL PROPERTIES OF SOLIDS 117

75. Consider four steel wires of dimensions given below


(d = diameter and l = length) : (8.02) 4 - (7.98) 4 (8.02) 2 - (7.98) 2
(a) (b)
(a) l = 1m, d = 1mm (b) l = 2m, d = 2 mm (0.8) 4 (0.8) 2
(c) l = 2m, d = 1mm (d) l = 1m, d = 2 mm
If same force is applied to all the wires then the elastic potential (0.8) 2 (0.8) 2
energy stored will be maximum in wire: (c) (d)
(8.02) 4 - (7.98) 4 (8.02) 3 - (7.98) 2
(a) A (b) B (c) C (d) D
76. If in a wire of Young’s modulus Y, longitudinal strain X is 84. The bulk moduli of ethanol, mercury and water are given as
produced, then the value of potential energy stored in its 0.9, 25 and 2.2 respectively in units of 109 Nm–2. For a given
unit volume will be value of pressure, the fractional compression in volume is
(a) Y X2 (b) 2 Y X2 (c) Y2 X/2 (d) Y X2/2 DV DV
77. The Poisson’s ratio of a material is 0.5. If a force is applied to . Which of the following statements about for
V V
a wire of this material, there is a decrease in the cross-sectional these three liquids is correct ?
area by 4%. The percentage increase in the length is : (a) Ethanol > Water > Mercury
(a) 1% (b) 2% (c) 2.5% (d) 4% (b) Water > Ethanol > Mercury
78. The Young's modulus of the material of a wire is (c) Mercury > Ethanol > Water
2 × 1010 Nm–2. If the elongation strain is 1%, then the energy (d) Ethanol > Mercury > Water
stored in the wire per unit volume in Jm–3 is 85. A material has poisson’s ratio 0.50. If a uniform rod of it
(a) 106 (b) 108 (c) 2 × 106 (d) 2 × 108 suffers a longitudinal strain of 2 × 10–3, then the
79. A 5 metre long wire is fixed to the ceiling. A weight of 10 kg
is hung at the lower end and is 1 metre above the floor. The percentage change in volume is
wire was elongated by 1 mm. The energy stored in the wire (a) 0.6 (b) 0.4 (c) 0.2 (d) Zero
due to stretching is 86. The system is rotated with angular speed w0 (see figure).
(a) zero (b) 0.05 joule (c) 100 joule (d) 500 joule What is the ratio of energy stored in each wire?
80. A wire suspended vertically from one of its ends is
stretched by attaching a weight of 200N to the lower end.
The weight stretches the wire by 1 mm. Then the elastic
energy stored in the wire is
(a) 0.2 J (b) 10 J (c) 20 J (d) 0.1 J (a) 31:9 (b) 50:9 (c) 47:9 (d) 8:9
81. Two, spring P and Q of force constants k p and 87. When a force is applied on a wire of uniform cross-section
æ kp ö area 3 × 10–6 m2 and length 4m, the increase in length is
kQ ç kQ =
è 2 ÷ø are stretched by applying forces of equal 1 mm. Energy stored in it will be (Y = 2 × 1011 N/m2)
magnitude. If the energy stored in Q is E, then the energy (a) 6250 J (b) 0.177 J (c) 0.075 J (d) 0.150 J
stored in P is 88. A metal rod of Young's modulus 2 × 1010 N m–2 undergoes
(a) E (b) 2 E (c) E/2 (d) E/4 an elastic strain of 0.06%. The energy per unit volume
82. A spherical ball contracts in volume by 0.02% when stored in J m–3 is
subjected to a pressure of 100 atmosphere. Assuming one (a) 3600 (b) 7200 (c) 10800 (d) 14400
atmosphere = 105 N m–2, the bulk modulus of the material 89. Steel ruptures when a shear of 3.5 × 108 N m–2 is applied.
of the ball is The force needed to punch a 1 cm diameter hole in a steel
(a) 0.02 × 105 N/m2 (b) 0.02 × 107 N/m2 sheet 0.3 cm thick is nearly:
(c) 50 × 107 N/m2 (d) 50 × 109 N/m2 (a) 1.4 × 104 N (b) 2.7 × 104 N
4
(c) 3.3 × 10 N (d) 1.1 × 104 N
83. A circular tube of mean radius 8 cm and thickness 0.04 cm is
melted up and recast into a solid rod of the same length. The 90. When the load on a wire is increasing slowly from 2 kg to
ratio of the torsional rigidities of the circular tube and the 4 kg, the elongation increases from 0.6 mm to 1 mm. The
solid rod is work done during this extension of the wire is (g = 10 m/s2)
(a) 9 ´ 10–3 J (b) 12 ´ 10–3 J
(c) 14 ´ 10 J–3 (d) 16 ´ 10–3 J

Exercise 2 : Exemplar & Past Year MCQs


NCERT Exemplar MCQs (c) be four times (d) remain same
1. Modulus of rigidity of ideal liquids is 3. The temperature of a wire is doubled. The Young's modulus
(a) infinity of elasticity
(b) zero (a) will also double (b) will become four times
(c) unity (c) will remain same (d) will decrease
(d) some finite small non-zero constant value 4. A spring is stretched by applying a load to its free end. The
2. The maximum load a wire can withstand without breaking, strain produced in the spring is
when its length is reduced to half of its original length, will (a) volumetric (b) shear
(a) be double (b) be half (c) longitudinal and shear (d) longitudinal
EBD_7418
118 PHYSICS

5. A rigid bar of mass M is supported symmetrically by three 11. The Young's modulus of steel is twice that of brass. Two
wires each of length l. Those at each end are of copper and wires of same length and of same area of cross section, one
the middle one is of iron. The ratio of their diameters, if each of steel and another of brass are suspended from the same
is to have the same tension, is equal to roof. If we want the lower ends of the wires to be at the same
Yiron level, then the weights added to the steel and brass wires
(a) Ycopper / Yiron (b) Ycopper must be in the ratio of : [AIPMT 2015, A]
(a) 2 : 1 (b) 4 : 1 (c) 1 : 1 (d) 1 : 2
2
Yiron Yiron 12. Stress vs strain curve for the elastic tissue of the aorta, the
(c) 2 (d) Ycopper large tube (vessel) carrying blood from the heart, will be :
Ycopper
[stress is proportional to square of the strain for the elastic
6. A mild steel wire of length 2L and cross-sectional area A is
tissue of the aorta] [AIIMS 2015, C]
stretched, well within elastic limit, horizontally between two
pillars (figure ). A mass m is suspended from the mid-point of
the wire. Strain in the wire is
2L
x2 x (a) (b)
(a) (b) x
2 L2 L
x2 x2 m
(c) (d)
L 2L
7. A rectangular frame is to be suspended symmetrically by
two strings of equal length on two supports (figure). It can (c) (d)
be done in one of the following three ways;
13. The bulk modulus of a spherical object is 'B'. If it is subjected
to uniform pressure 'p', the fractional decrease in radius is
[NEET 2017, C]
B 3p p p
(a) (b) (c) (a) (b) (c) (d)
The tension in the strings will be 3p B 3B B
(a) the same in all cases (b) least in (a) 14. An iron rod of length 2m and cross-sectional area of 50 mm2
(c) least in (b) (d) least in (c) stretched by 0.5 mm, when a mass of 250 kg is hung from its
8. Consider two cylindrical rods of identical dimensions, one lower end. Young’s modulus of iron rod is [AIIMS 2017, A]
of rubber and the other of steel. Both the rods are fixed (a) 19.6 × 1020 N/m2 (b) 19.6 × 1018 N/m2
rigidly at one end to the roof. A mass M is attached to each (c) 19.6 × 1010 N/m2 (d) 19.6 × 1015 N/m2
of the free ends at the centre of the rods. 15. Assertion: Solids are least compressible and gases are most
(a) Both the rods will elongate but there shall be no compressible.
perceptible change in shape Reason: solids have definite shape and volume but gases
(b) The steel rod will elongate and change shape but the do not have either definite shape or definite volume.
rubber rod will only elongate [AIIMS 2017, C]
(c) The steel rod will elongate without any perceptible
(a) If both Assertion and Reason are correct and Reason
change in shape, but the rubber rod will elongate and
the shape of the bottom edge will change to an ellipse is the correct explanation of Assertion.
(d) The steel rod will elongate, without any perceptible change (b) If both Assertion and Reason are correct, but Reason
in shape, but the rubber rod will elongate with the shape is not the correct explanation of Assertion.
of the bottom edge tapered to a tip at the centre (c) If Assertion is correct but Reason is incorrect.
(d) If both the Assertion and Reason are incorrect.
Past Year MCQs
16. Two wires are made of the same material and have the same
9. Copper of fixed volume ‘V; is drawn into wire of length ‘l’. volume. The first wire has cross-sectional area A and the
When this wire is subjected to a constant force ‘F’, the second wire has cross-sectional area 3A. If the length of
extension produced in the wire is ‘Dl’. Which of the following the first wire is increased by Dl on applying a force F, how
graphs is a straight line? [AIPMT 2014, C]
much force is needed to stretch the second wire by the
1 same amount? [BITSAT 2016, NEET 2018, A]
(a) Dl versus (b) Dl versus l2
l (a) 9 F (b) 6 F (c) F (d) 4 F
1 17. The fractional change in volume of a glass slab when
(c) Dl versus 2 (d) Dl versus l
l subjected to a hydraulic pressure of 10 atm is:
10. A wire is stretched by 1 kg wt. If the radius of the wire is [Bulk modulus of glass = 37 × 109 N/m2] [AIIMS 2018, A]
doubled, its Young's modulus will be [BITSAT 2014, A] (a) 2.74 × 10–5 (b) 5.12 × 10–3
(a) remains unchanged (b) becomes half (c) 8.31 × 10 –4 (d) 6.11 × 10–4
(c) become doubled (d) become four times
MECHANICAL PROPERTIES OF SOLIDS 119

18. A solid sphere of radius r made of a soft material of bulk piston to compress the liquid, the fractional decrement in
modulus K is surrounded by a liquid in a cylindrical æ dr ö
container. A massless piston of area a floats on the surface the radius of the sphere çè ÷ø ,is : [JEE Main 2018, S, BN]
r
of the liquid, covering entire cross-section of cylindrical
container. When a mass m is placed on the surface of the Ka Ka mg mg
(a) (b) (c) (d)
mg 3mg 3Ka Ka

Exercise 3 : Try If You Can


1. A body of mass 10 kg is attached to a wire of radius 3 cm. 2.0 × 1011 Nm–2, the total extension of the composite wire is :
It’s breaking stress is 4.8 × 10 7 Nm –2 , the area of cross- (a) 1.75 mm (b) 2.0 mm (c) 1.50 mm (d) 1.25 mm
section of the wire is 10–6 m 2 . What is the maximum 6. A uniform wire (Young’s modulus 2 × 1011 Nm–2) is subjected
angular velocity with which it can be rotated in the to longitudinal tensile stress of 5 × 107 Nm–2. If the overall
horizontal circle ? volume change in the wire is 0.02%, the fractional decrease
(a) 1 rad sec–1 (b) 2 rad sec–1 in the radius of the wire is close to :
–1 (a) 1.0 × 10–4 (b) 1.5 × 10–4
(c) 4 rad sec (d) 8 rad sec–1 –4
(c) 0.25 × 10 (d) 5 × 10–4
2. A massive stone pillar 20 m high and of uniform cross- 7. Spring constant of a spring varries as:
section rests on a rigid base and supports a vertical load of [where Young's Modulus, l = 1 length, d = diameter of spring
5.0 × 105 N at its upper end. If the compressive stress in the wire]
pillar is not to exceed 1.6 × 106 N m–2, what is the minimum 1
cross-sectional area of the pillar? Density of the stone = 2.5 (a) µ (b) µ d 2 (c) µ l (d) µ g 2
× 103 kg m–3. (Take g = 10 N kg–1) g
8. A thin 1 m long rod has a radius of 5 mm. A force of 50 pkN
(a) 0.15 m2 (b) 0.25 m2 (c) 0.35 m2 (d) 0.45 m2 is applied at one end to determine its Young's modulus (Y).
3. If the ratio of lengths, radii and Young’s modulus of steel Assume that the force is exactly known. If the least count in
and brass wires shown in the figure are a, b, and c, the measurement of all lengths is 0.01 mm, which of the
respectively. The ratio between the increase in lengths of following statements is false ?
brass and steel wires would be (a) The maximum value of Y that can be determined is
b2 a 2 × 1014N/m2.
(a) DY
2c (b) gets minimum contribution from the uncertainty
Y
bc Brass in the length
(b) 2 DY
2a
2kg (c) gets its maximum contribution from the
Y
2 uncertainty in strain
ba
(c) Steel (d) The figure of merit is the largest for the length of the
2c
rod.
a 4kg 9. A uniformly tapering conical wire is made from a material of
(d) 2 Young's modulus Y and has a normal, unextended length L.
2b c
The radii, at the upper and lower ends of this conical wire,
4. One end of a uniform wire of length L and of weight W is have values R and 3R, respectively. The upper end of the
attached rigidly to a point in the roof and W1 weight is wire is fixed to a rigid support and a mass M is suspended
suspended from looser end. If A is area of cross-section of from its lower end. The equilibrium extended length, of this
L wire, would equal :
the wire, the stress in the wire at a height from the upper
4
end is æ 2 Mg ö æ 1 Mg ö
L ç1 + L ç1 +
W1 + W W1 + 3W / 4
(a) è 9 pYR 2 ÷ø (b) è 9 pYR 2 ÷ø
(a) (b)
a a æ 1 Mg ö æ 2 Mg ö
L ç1 + (d) L çè 1 + 3
(c)
W1 + W / 4
(d)
4 W1 + 3W (c) è 3 pYR 2 ÷ø ÷
pYR 2 ø
a a 10. A massless rod of length L is hanging from a horizontal
5. A copper wire of length 1.0 m and a steel wire of length ceiling with help of two wires, of equal length and diameter,
0.5 m having equal cross-sectional areas are joined end to attached to the ends of the rod. The Young's modulus of
end. The composite wire is stretched by a certain load which
the first wire is Y and the second wire is 1.2Y. A mass M is
stretches the copper wire by 1 mm. If the Young’s modulii of
copper and steel are respectively 1.0 × 1011 Nm–2 and attached to the rod at a distance X from the first wire. If the
stretching of the two wires is equal then X/L is
EBD_7418
120 PHYSICS

5 6 13. The Young’s modulus of brass and steel are respectively


(a) (b) 1010 N/m2. and 2 × 1010 N/m2. A brass wire and a steel wire
11 11
Y 1.2Y of the same length are extended by 1 mm under the same
L force, the radii of brass and steel wires are RB and RS
7 9
(c) (d) X respectively. Then
11 11
M (a) RS = 2 R B (b) RS = R B / 2
11. A square frame of ABCD consisting of five steel bars of
cross-section area 400 mm2 and joined by pivot is subjected (c) R S = 4R B (d) R S = R B / 4
to action of two forces P = 40 kN in the direction of the 14. Two bodies of masses 2 kg and 3 kg are connected by a
diagonal as shown. Find change in angle at A if Young’s metal wire of cross-section 0.04 mm2. Breaking stress of
modulus Y = 2 × 105 N/min. metal wire is 2.5 GPa. The maximum force F that can be
1 a applied to 3 kg block so that wire does not break is
(a) rad A
B (Neglect friction)
2000
0.04 mm² F
1 a a 2kg 3kg
(b) rad
1000 ///////////////////////////////////////////////////
D (a) 100 N (b) 150 N (c) 200 N (d) 250 N
2 a C
(c) rad 15. A metal wire of length L1 and area of cross-section A is
1000 attached to a rigid support. Another metal wire of length L2
(d) None 40 kN and of the same cross-sectional area is attached to the free
12. A rod of length 1000mm and coefficient of linear expansion end of the first wire. A body of mass M is then suspended
a = 10–4 per degree is placed symmetrically between fixed from the free end of the second wire. If Y1 and Y2 are the
walls separated by 1001 mm. The Young’s modulus of the young’s moduli of the wires respectively, the effective force
rod is 1011 N/m2. If the temperature is increased by 20°C, constant of the system of two wires is
then the stress developed in the rod is (in N/m2) (Y1Y2 ) A (Y1Y2 ) A
(a) 10
//////////////////////////////////////

(a) 2(Y L + Y L ) (b)


1 2 2 1 ( L1L2 )1/2
1000mm
(b) 108
(Y1Y2 ) A (Y1Y2 )1/2 A
(c) 2 × 108 (c) (d)
Y1 L2 + Y2 L1 ( L2 L1 )1/2
(d) cannot be calculated
1001mm

ANSWER KEYS
Exercise 1 : Topic-wise MCQs
1 (a) 10 (b) 19 (c) 28 (d) 37 (a) 46 (b) 55 (a) 64 (c) 73 (b) 82 (d)
2 (c) 11 (d) 20 (a) 29 (b) 38 (c) 47 (c) 56 (a) 65 (d) 74 (a) 83 (a)
3 (c) 12 (b) 21 (c) 30 (a) 39 (d) 48 (c) 57 (b) 66 (c) 75 (c) 84 (a)
4 (b) 13 (a) 22 (c) 31 (b) 40 (c) 49 (d) 58 (d) 67 (a) 76 (d) 85 (b)
5 (c) 14 (a) 23 (d) 32 (b) 41 (a) 50 (a) 59 (b) 68 (b) 77 (d) 86 (b)
6 (b) 15 (d) 24 (a) 33 (d) 42 (c) 51 (a) 60 (c) 69 (a) 78 (a) 87 (c)
7 (c) 16 (b) 25 (c) 34 (a) 43 (c) 52 (a) 61 (c) 70 (b) 79 (b) 88 (a)
8 (b) 17 (d) 26 (d) 35 (a) 44 (a) 53 (b) 62 (b) 71 (c) 80 (d) 89 (c)
9 (c) 18 (b) 27 (c) 36 (c) 45 (b) 54 (d) 63 (d) 72 (d) 81 (c) 90 (c)
Exercise 2 : Exemplar & Past Year MCQs
1 (b) 3 (d) 5 (b) 7 (c) 9 (b) 11 (a) 13 (c) 15 (b) 17 (a)
2 (d) 4 (c) 6 (a) 8 (d) 10 (a) 12 (a) 14 (c) 16 (a) 18 (c)
Exercise 3 : Try If You Can
1 (c) 3 (d) 5 (d) 7 (b) 9 (c) 11 (b) 13 (b) 15 (c)
2 (d) 4 (b) 6 (c) 8 (a) 10 (b) 12 (b) 14 (d)
10Chapter
MECHANICAL PROPERTIES
OF FLUIDS

Trend
Analysis of NEET and AIIMS (Year 2010-2018)
5

4
Number of Questions

3
AIPMT/NEET
2 AIIMS

0
2010 2011 2012 2013 2014 2015 2016 2017 2018
Year
Trend
Analysis of JEE Main and BITSAT (Year 2010-2018)
5

4
Number of Questions

3
JEE Main
2 BITSAT

0
2010 2011 2012 2013 2014 2015 2016 2017 2018
Year

Chapter Utility Score (CUS)


Exam Weightage Important Concepts Difficulty Level CUS (Out of 10)
NEET 3 Pressure in a Fluid,
AIIMS 3 Fluid Flow, Bernoulli's 4/5 8.5/10
JEE Main 2 Principle, Viscosity and
BITSAT 3 Surface Tension
EBD_7418
122 PHYSICS
MECHANICAL PROPERTIES OF FLUIDS 123
EBD_7418
124 PHYSICS

Exercise 1 : Topic-wise MCQs

Topic 1: Pressure, Density, Pascal's Law equilibrium in the position shown in the figure.Which of
the following is true for r1, r1and r3?
and Archimedes' Principle r1

(a) r3 < r1 < r2


1. Liquid pressure depends upon
(b) r1 > r3 > r2 r3
(a) area of the liquid surface
(b) shape of the liquid surface (c) r1 < r2 < r3
(c) height of the liquid column
(d) directions (d) r1 < r3 < r2
2. Hydraulic lifts and hydraulic brakes are based on 9. A candle of diameter d is floating on a liquid in a cylindrical
(a) Archimedes' principle (b) Bernoulli's principle container of diameter D (D >> d) as shown in figure. If it is
(c) Stoke's law (d) Pascal's law burning at the rate of 2 cm/hour then the top of the candle
3. According to archimedes' principle, loss of weight of a body will
immersed in a liquid is equal to L
(a) remain at the same height
(a) weight of the liquid displaced
(b) weight of the total liquid (b) fall at the rate of 1 cm/hour L
(c) weight of the body
(d) None of these (c) fall at the rate of 2 cm/hour d
4. A vessel with water is placed on a weighing pan and it reads D
600 g. Now a ball of mass 40 g and density 0.80 g cm–3 is (d) go up at the rate of 1 cm/hour
sunk into the water with a pin of negligible volume, as shown 10. Specific gravity of a body is numerically equal to
in figure keeping it sunk. The weighting pan will show a (a) weight of the body in air
reading (b) weight of the body in water
(c) relative density of the body
(a) 600 g (d) density of body in water
11. If a solid floats with 1/4th of its volume above the surface of
(b) 550 g
water, then density of the solid (rs) is related to density of
(c) 650 g water (rw) as
(d) 632 g Weighing pan (a) rs = rw (b) rs = 1 rw
4
3 4
5. _________ and _________ play the same role in case of (c) rs = rw (d) rs = rw
fluids as force and mass play in case of solids. 4 3
12. Assertion: A small iron needle sinks in water while a large
(a) Thrust and density (b) Pressure and density
iron ship floats.
(c) Pressure and thrust (d) Thrust and volume Reason: The shape of iron needle is like a flat surface while
6. Pressure applied to enclosed fluid is the shape of a ship is that which makes it easier to float.
(a) increased and applied to every part of the fluid (a) Assertion is correct, reason is correct; reason is a
(b) diminished and transmitted to wall of container correct explanation for assertion.
(c) increased in proportion to the mass of the fluid and (b) Assertion is correct, reason is correct; reason is not a
then transmitted correct explanation for assertion
(d) transmitted unchanged to every portion of the fluid (c) Assertion is correct, reason is incorrect
and wall of containing vessel. (d) Assertion is incorrect, reason is correct.
7. The pressure at the bottom of a tank containing a liquid 13. Consider an iceberg floating in sea water. The density of
does not depend on sea water is 1.03 g/cc and that of ice is 0.92 g/cc. The fraction
(a) acceleration due to gravity of total volume of iceberg above the level of sea water is
(b) height of the liquid column near by
(c) area of the bottom surface (a) 1.8% (b) 3% (c) 8% (d) 11%
(d) nature of the liquid 14. A block of ice floats on a liquid of density 1.2 in a beaker
then level of liquid when ice completely melt
8. A jar is filled with two non-mixing liquids 1 and 2 having
(a) remains same (b) rises
densities r1 and, r2 respectively. A solid ball, made of a
(c) lowers (d) either (b) or (c)
material of density r3 , is dropped in the jar. It comes to
MECHANICAL PROPERTIES OF FLUIDS 125

15. The total weight of a piece of wood is 6 kg. In the floating 22. A cubical box is to be constructed with iron sheets 1 mm in
1 thickness. What can be the minimum value of the external
state in water its part remains inside the water. On this edge so that the cube does not sink in water?
3
floating piece of wood what maximum weight is to be put [ diron = 8000 kg/m3, dwater = 1000 kg/m3]
such that the whole of the piece of wood is to be drowned in (a) 4.8 cm (b) 5.8 cm (c) 6.9 cm (d) 7.3 cm
the water? 23. In rising from the bottom of a lake, to the top, the temperature
(a) 15 kg (b) 14 kg (c) 10 kg (d) 12 kg of an air bubble remains unchanged, but its diameter gets
16. A vessel contains oil (density = 0.8 gm/cm3) over mercury doubled. If h is the barometric height (expressed in m of
(density = 13.6 gm/cm3). A homogeneous sphere floats with mercury of relative density r) at the surface of the lake, the
half of its volume immersed in mercury and the other half in depth of the lake is
oil. The density of the material of the sphere in gm/cm3 is (a) 8 rh m (b) 7rh m (c) 9 rh m (d) 12 rh m
(a) 3.3 (b) 6.4 (c) 7.2 (d) 12.8 24. A hollow sphere of mass M = 50 kg and radius
17.In a hydraulic lift, compressed air exerts a force F1 on a small 1/ 3
æ 3 ö
piston having a radius of 5 cm. This pressure is transmitted r= ç m is immersed in a tank of water (density
è 40 p ÷ø
to a second piston of radius 15 cm. If the mass of the load to
rw = 103 kg/m3). The sphere is tied to the bottom of a tank
be lifted is 1350 kg, find the value of F1 ? The pressure
by two wires A and B as shown. Tension in wire A is
necessary to accomplish this task is
(g = 10 m/s2)
(a) 1.4 × 105 Pa (b) 2 × 105 Pa
(c) 1.9 × 10 Pa5 (d) 1.9 Pa (a) 125 2N
18. Assertion: The apparent weight of a floating body is zero.
Reason: The weight of the block acting vertically downwards (b) 125 N
is balanced by the buoyant force acting on the block (c) 250 2N
upwards.
(a) Assertion is correct, reason is correct; reason is a (d) 250 N
correct explanation for assertion.
25. The two thigh bones, each of cross-sectional area
(b) Assertion is correct, reason is correct; reason is not a
correct explanation for assertion 10 cm 2 support the upper part of a human body of mass
(c) Assertion is correct, reason is incorrect 40 kg. Estimate the average pressure sustained by the
(d) Assertion is incorrect, reason is correct. bones. Take g = 10 m/s2
19. The force acting on a window of area 50 cm × 50 cm of a (a) 2 × 105 N/m2 (b) 5 × 104 N/m2
submarine at a depth of 2000 m in an ocean, interior of which (c) 2 × 107 N/m2 (d) 3 × 106 N/m2
is maintained at sea level atmospheric pressure is
26. A hemispherical bowl just floats without sinking in a liquid
(Density of sea water = 103 kgm–3, g = 10 ms–2) of density 1.2 × 103 kg/m3. If outer diameter and the density
(a) 106 N (b) 5 × 105 N of the bowl are 1 m and 2 × 104 kg/m3 respectively then the
6
(c) 25 × 10 N (d) 5 × 106 N inner diameter of the bowl will be
20. A concrete sphere of radius R has a cavity of radius r which
is packed with sawdust. The specific gravities of concrete (a) 0.94 m (b) 0.97 m (c) 0.98 m (d) 0.99 m
and sawdust are respectively 2.4 and 0.3 for this sphere to 27. A uniform rod of density r is placed in a wide tank containing
float with its entire volume submerged under water. Ratio of a liquid of density r0(r0 > r). The depth of liquid in the
mass of concrete to mass of sawdust will be tank is half the length of the rod. The rod is in equilibrium,
with its lower end resting on the bottom of the tank. In this
(a) 8 (b) 4 (c) 3 (d) zero position the rod makes an angle q with the horizontal
21. A body B is capable of remaining stationary inside a liquid
1 1 r
at the position shown in Fig. (a). If the whole system is (a) sin q = r0 / r (b) sin q = . 0
gently placed on smooth inclined plane (Fig (b)) and is 2 2 r
allowed to slide down, then (0 < q < 90°). The body will (c) sin q = r / r0 (d) sin q = r0 / r
(a) move up
28. A boy can reduce the pressure in his lungs to 750 mm of
(relative to liquid)
B mercury. Using a straw he can drink water from a glass upto
(b) move down
B the maximum depth of (atmospheric pressure = 760 mm of
(relative to liquid)
(c) remain stationary q mercury; density of mercury = 13.6 gcm–3)
(relative to liquid) (a) (b) (a) 13.6 cm (b) 9.8 cm (c) 10 cm (d) 76 cm
(d) move up for some inclination q and will move down 29. A wooden block, with a coin placed on its top, floats in water
for another inclination q as shown in fig. the distance l and h are shown there. After
some time the coin falls into the water. Then
EBD_7418
126 PHYSICS

(a) l decreases and h increases Coin 36. What is the absolute pressure of the gas above the liquid
surface in the tank shown in fig. Density of oil = 820 kg/m3,
(b) l increases and h decreases
l density of mercury = 13.6 × 103 kg/m3. Given 1 atmospheric
(c) both l and h increases h pressure = 1.01× 105 N/m2.
Pa
(a) 3.81 × 105 N/m2
(d) both l and h decreases
Gas
30. The density r of water of bulk modulus B at a depth y in the (b) 6 × 106 N/m2 1m 0.75 m
ocean is related to the density at surface r0 by the relation
1.5 m
é r gy ù é r gy ù (c) 5 × 107 N/m2
r = r0 ê1 - 0 ú r = r0 ê1 + 0 ú Oil
(a) B û
(b) B û A B
ë ë
é B ù é B ù (d) 4.6 × 102 N/m2 Mercury
(c) r = r0 ê1 + r hgy ú (d) r = r0 ê1 - r hgy ú
ë 0 û ë 0 û
31. A sphere of solid material of specific gravity 8 has a 37. Figure shows a U-tube of uniform cross-sectional area A,
concentric spherical cavity and just sinks in water. The accelerated with acceleration a as shown. If d is the
ratio of radius of cavity to that of outer radius of the sphere separation between the limbs, then what is the difference in
must be the levels of the liquid in the U-tube is
h = h1–h2
71 3 51 3 91 3 31 3
(a) (b) (c) (d) ad ag
2 2 2 2 (a) (b)
32. A vessel containing water is given a constant acceleration g d
a
‘a’ towards the right along a straight horizontal path. Which a dg
of the following diagrams represents the surface of the liquid ? (c) (d)
d a d

Topic 2: Fluid Flow, Reynold's Number


(a) a (b)a a
and Bernoull is Principle
38. Beyond the critical speed, the flow of fluids becomes
(a) streamline (b) turbulent
(c)a a (d) None of these (c) steady (d) very slow
39. Streamline flow is more likely for liquids with
33. An iceberg is floating in ocean. What fraction of its volume (a) high density and low viscosity
is above the water ? (Given : density of ice = 900kg/m3 and (b) low density and high viscosity
density of ocean water = 1030 kg/m3) (c) high density and high viscosity
90 13 10 1 (d) low density and low viscosity
(a) (b) (c) (d)
103 103 103 103 40. For flow of a fluid to be turbulent
34. A cone full of water, is placed on its side on a horizontal (a) fluid should have high density
table, the thrust on its base is x times the weight of the (b) velocity should be large
contained fluid, where 2a is the vertical angle of the cone. (c) reynold number should be less than 2000
Find the value of x. (d) both (a) and (b)
(a) 3 cos a (b) 3 sin a (c) 2 sin a (d) 2 cos a 41. In Bernoulli’s theorem which of the following is conserved?
35. Assertion : Imagine holding two identical bricks under water. (a) Mass (b) Linear momentum
Brick A is completely submerged just below the surface of (c) Energy (d) Angular momentum
water, while Brick B is at a greater depth. The magnitude of 42. A liquid is allowed to flow into a tube of truncated cone
force exerted by the person (on the brick) to hold brick B in shape. Identify the correct statement from the following:
place is the same as magnitude of force exerted by the person (a) the speed is high at the wider end and high at the
(on the brick) to hold brick A in place. narrow end.
Reason : The magnitude of buoyant force on a brick com- (b) the speed is low at the wider end and high at the narrow
pletely submerged in water is equal to magnitude of weight end.
of water it displaces and does not depend on depth of brick (c) the speed is same at both ends in a stream line flow.
in water. (d) the liquid flows with uniform velocity in the tube.
(a) Assertion is correct, reason is correct; reason is a 43. Which of the following statements are true about streamline
correct explanation for assertion. flow?
(b) Assertion is correct, reason is correct; reason is not a I. Path taken by a fluid particle under a steady flow is a
correct explanation for assertion
streamline
(c) Assertion is correct, reason is incorrect
(d) Assertion is incorrect, reason is correct. II. No two streamlines can cross each other
MECHANICAL PROPERTIES OF FLUIDS 127

III. Velocity increases at the narrower portions where the 50. Assertion : Lifting of aircraft is caused by pressure
streamlines are closely spaced difference brought by varying speed of air molecules.
(a) I & II only (b) II & III only Reason : As the wings/ aerofoils move against the wind,
(c) I & III only (d) I, II & III the streamlines crowd more above them than below, causing
44. Consider the following statements : In a streamline flow of a higher velocity above than below.
liquid (a) Assertion is correct, reason is correct; reason is a
I. the kinetic energies of all particles arriving at a given correct explanation for assertion.
point are same. (b) Assertion is correct, reason is correct; reason is not a
II. the momenta of all particles arriving at a given point correct explanation for assertion
(c) Assertion is correct, reason is incorrect
are same.
(d) Assertion is incorrect, reason is correct.
III. the speed of particles are below the critical velocity.
51. In the figure shown the velocity and pressure of theliquid
Which of the statements given above are correct? at the cross section (2) are given by (If P0 is the atmospheric
(a) I & II only (b) II & III only pressure).
(c) I &III only (d) I, II & III
45. A liquid flows through a tube of uniform cross-section with
a constant speed in the direction as shown by the arrows
in fig. The liquid exerts on the tube
(a) a net force to the left h 1
2
(b) a net force to the right 3
(c) an anticlockwise torque A A/2 A/4
(d) a clockwise torque
46. Water flows through a frictionless tube with a varying cross-
section as shown in fig. Pressure P at points along the axis rhg rhg
is represented by (a) 2hg, P0 + (b) hg, P0 +
2 2
hg 3rhg hg rhg
(c) , P0 + (d) , P0 -
2 4 2 4
P P 52. A cylinderical vessel contains a liquid of density r filled
upto a height h. The uper surface of the liquid is in contact
(a) (b) with a pistion of mass m and area of cross-section A. A small
hole is drilled at the bottom of the vessel. (Neglect the
x viscous effects) The speed with which the liquid comes out
P P of the hoe is :
æ mö
(a) 2gh (b) 2g ç h + ÷
(c) (d) è rA ø
x x
æ mö æ 2m ö
47. Air flows horizontally with a speed v = 106 km/hr. A house (c) g çh + ÷ (d) g çh+
è rA ø è rA ø÷
has plane roof of area A = 20m 2. The magnitude of
aerodynamic lift of the roof is 53. A wide vessel with a small hole at the bottom is filled with
water (density r1, height h1) and kerosene (density r2, height
(a) 1.127 × 104 N (b) 5.0 × 104 N h2). Neglecting viscosity effects, the speed with which water
(c) 1.127 × 105 N (d) 3.127 × 104 N flows out is :
48. In which flow the velocity of fluid particles reaching a (a) [2g(h1 + h2)]1/2 (b) [2g(h1r1 + h2r2)]1/2
particular point is same at all time? (c) [2g(h1 + h2(r2/r1))] 1/2
(d) [2g (h1 + h2(r1/r2))]1/2
(a) vigrous flow
(b) steady flow Topic 3: Equation of Continuity, Viscosity and Terminal Velocity
(c) turbulant flow
(d) None of these 54. Figure shows a liquid flowing through a tube at the rate of
0.1 m3/s. The tube is branched into two semicircular tubes
49. A wind with speed 40 m/s blows parallel to the roof of a
of cross - sectional area A/3 and 2A/3. The velocity of
house. The area of the roof is 250 m2. Assuming that the liquid at Q is (the cross-section of the main tube is
pressure inside the house is atmospheric pressure, the force A = 10–2 m2 and vP = 20 m/s) P vP
exerted by the wind on the roof and the direction of the
(a) 5 m/s
force will be (rair = 1.2 kg/m3)
(b) 30 m/s
(a) 4.8 × 105 N, upwards (b) 2.4 × 105 N, upwards (c) 35 m/s
Q
(c) 2.4 × 105 N, downwards (d) 4.8 × 105 N, downwards (d) None of these vQ
EBD_7418
128 PHYSICS

55. Oil is filled in a cylindrical container upto height 4m. A 63. When a ball is released from rest in a very long column of
small hole of are a ‘p’ is punched in the wall of the viscous liquid, its downward acceleration is ‘a’ (just after
container at a height 1.52m from the bottom. The cross- release). Its acceleration when it has acquired two third of
the maximum velocity is a/X. Find the value of X.
p
sectional area of the container is Q. If = 0.1 then v is (a) 2 (b) 3 (c) 4 (d) 5
q
(where v is the velocity of oil coming out of the hole) 64. A solid sphere of density h(>1) times lighter than water is
(a) 5 2 (b) 6 3 (c) 8 2 (d) 7 5 suspended in a water tank by a string tied to its base as
56. As the temperature of a liquid is raised, the coefficient of shown in fig. If the mass of the sphere is m, then the tension
viscosity in the string is given by
(a) decreases æ h -1 ö
(b) increases (a) ç ÷ mg (b) hmg
è h ø
(c) remains the same
(d) may increase or decrease depending on the nature of mg
(c) (d) (h – 1)mg
liquid h -1
57. After terminal velocity is reached, the acceleration of a body 65. A ball of radius r and density r falls freely under gravity
falling through a fluid is through a distance h before entering water. Velocity of ball
(a) equal to g (b) zero does not change even on entering water. If viscosity of
(c) less than g (d) greater than g water is h the value of h is given by
58. According to stokes law, the relation between terminal
velocity (vt) and viscosity of the medium (n) is h
(a) vt = n (b) vt µ n
1
(c) vt µ (d) vt is independent of n.
n
59. Terminal velocity of ball depends on 2 2 æ 1- r ö 2 2 æ r -1 ö
(a) r ç ÷g (b) r ç ÷g
(a) area of the ball 9 è h ø 81 è h ø
(b) density of liquid only
2 2
(c) density of ball only 2 4 æ r -1 ö 2 4 æ r -1 ö
(c) r ç ÷ g (d) r ç ÷ g
(d) density difference of liquid and ball 81 è h ø 9 è h ø
60. A boat with base area 8m2 floating on the surface of a still 66. An air bubble of radius 1 cm rises with terminal velocity 0.21
river is intended to move with a constant speed of 2 m/s by
cm/s in liquid column. If the density of liquid is 1.47 × 103
the application of a horizontal force. If the river bed is 2m
kg/m3. Then the value of coefficient of viscosity of liquid
deep find the force needed, (assuming a constant velocity
gradient) Coefficient of viscosity of water is 0.90 × 10–2 poise. ignoring the density of air, will be
(a) 720 dyne(b) 620 dyne(c) 520 dyne(d) 360 dyne (a) 1.71 × 104 poise (b) 1.82 × 104 poise
4
(c) 1.78 × 10 poise (d) 1.52 × 104 poise
61. Which of the following statements is/are true?
I. Solid friction is independent of area of surface in contact 67. Column I Column II
and viscous drag is also independent of area of layers (A) Terminal velocity (1) Average density becomes
in contact. less than that of liquid
II. Solid friction depends on the relative velocity of one (B) Objects of high (2) Upthrust is zero
body on the surface of another body while viscous density can also float
drag is independent of the relative velocity between (C) A beaker having a (3) Varies with velocity
two layers of the liquid. solid iron under free fall
III. Solid friction is directly proportional to the normal (D) Viscous drag (4) Upthrust and viscous
reaction while viscosity is independent of the normal force
reaction between two layers of the liquid. (a) (A) ® (4) ; (B) ® (2) ; (C) ® (3) ; (D) ® (1)
(a) I, II & III (b) I & II (c) III only (d) II only (b) (A) ® (1) ; (B) ® (2) ; (C) ® (3) ; (D) ® (4)
62. A spherical solid ball of volume V is made of a material of (c) (A)® (4) ; (B) ® (1) ; (C) ® (2) ; (D) ® (3)
density r1. It is falling through a liquid of density r2 (r2< r1). (d) (A) ® (3) ; (B) ® (4) ; (C) ® (2) ; (D) ® (1)
Assume that the liquid applies a viscous force on the ball 68. The relative velocity of two parallel layers of water is 8 cm/sec.
If the perpendicular distance between the layers is 0.1 cm.
that is proportional to the square of its speed v, i.e., Fviscous
Then velocity gradient will be
= –kv2 (k > 0). The terminal speed of the ball is
(a) 80/sec (b) 60 /sec (c) 50/sec (d) 40/sec
Vg (r1 – r2 ) Vgr1 69. If a ball of steel (density r = 7.8 g cm–3) attains a terminal
(a) (b)
k k velocity of 10 cms–1 when falling in a tank of water
Vgr1 Vg (r1 – r2 ) (coefficient of viscosity hwater = 8.5 × 10–4 Pa – s) then its
(c) (d) terminal velocity in glycerine (r = 12 gcm–3, h = 13.2 Pa – s)
k k
would be nearly
MECHANICAL PROPERTIES OF FLUIDS 129

(a) 1.6 × 10–5 cms–1 (b) 6.25 × 10–4 cms–1 77. A small spherical ball falling through a viscous medium of
(c) 6.45 × 10 cms–4 –1 (d) 1.5 × 10–5 cms–1 negligible density has terminal velocity v. Another ball of
70. Assertion : Falling raindrops acquire a terminal velocity. the same mass but of radius twice that of the earlier falling
Reason : A constant force in the direction of motion and a through the same viscous medium will have terminal velocity
velocity dependent force opposite to the direction of motion, (a) v (b) v/4 (c) v/2 (d) 2v
always result in the acquisition of terminal velocity. 78. The velocity of water in a river is 18 km/hr near the surface.
(a) Assertion is correct, reason is correct; reason is a If the river is 5 m deep, find the shearing stress between
correct explanation for assertion. the horizontal layers of water. The co-efficient of
(b) Assertion is correct, reason is correct; reason is not a viscosity of water = 10–2 poise.
correct explanation for assertion (a) 10–1 N/m2 (b) 10–2 N/m2
(c) Assertion is correct, reason is incorrect (c) 10–3 N/m2 (d) 10–4 N/m2
(d) Assertion is incorrect, reason is correct.
71. A liquid is filled upto a height of 20 cm in a cylindrical vessel. Topic 4: Surface Tension, Surface Energy and Capillarity
The speed of liquid coming out of a small hole at the bottom
of the vessel is (g = 10 ms–2) 79. Surface tension of a liquid is due to
(a) 1.2 ms–1 (b) 1 ms–1 (c) 2 ms–1 (d) 3.2 ms–1 (a) gravitational force between molecules
72. A solid ball of volume V experiences a viscous force F when (b) electrical force between molecules
falling with a speed v in a liquid. If another ball of volume (c) adhesive force between molecules
8 V with the same velocity v is allowed to fall in the same (d) cohesive force between molecules
liquid, it experiences a force 80. If a soap bubble formed at the end of the tube is blown very
(a) F (b) l6 F (c) 4 F (d) 2 F slowly, then the graph between excess pressure inside the
73. In the figure, the velocity V3 will be bubble with time will be a
(a) straight line sloping up
A2 = 0.2 m2
–1
(b) straight line sloping down
V1 = 4 ms V2 = 2 ms–1 (c) parabolic curve sloping down
A1 = 0.2 m2 (d) parabolic curve sloping up
A3= 0.4 m2 V3 81. Surface tension may be defined as
(a) the work done per unit area in increasing the surface
(a) Zero (b) 4 ms–1 (c) 1 ms–1 (d) 3 ms–1
area of a liquid under isothermal conditions
74. In the diagram shown, the difference in the two tubes of the
manometer is 5 cm, the cross section of the tube at A and B (b) the work done per unit area in increasing the surface
is 6 mm2 and 10 mm2 respectively. The rate at which water area of a liquid under adiabatic conditions
flows through the tube is (g = 10 ms–2) (c) the work done per unit area in increasing the surface
(a) 7.5 cc/s area of a liquid under adiabatic conditions
(d) free surface energy per unit volume
(b) 8.0 cc/s 5 cm 82. Which of the following graph represents the variation of
surface tension with temperature over small temperature
(c) 10.0 cc/s A B
ranges for water?
(d) 12.5 cc/s S.T. S.T.

75. What is the velocity v of a metallic ball of radius r falling in


(a) (b)
a tank of liquid at the instant when its acceleration is one -
half that of a freely falling body ? (The densities of metal T T
and of liquid are r and s respectively, and the viscosity of S.T. S.T.
the liquid is h ).
r2 g r2 g (c) (d)
(a) ( r - 2 s) (b) (2r - s)
9h 9h T T
r2g 2r 2 g 83. If a water drop is kept between two glass plates, then its
(c) (r - s) (d) ( r - s) shape is
9h 9h
76. A spherical ball of iron of radius 2 mm is falling through a
column of glycerine. If densities of glycerine and iron are (a) (b)
respectively 1.3 × 103 kg/m3 and 8 × 103 kg/m3. h for
glycerine = 0.83 Nm–2 sec, then the terminal velocity is
(a) 0.7 m/s (b) 0.07 m/s
(c) 0.007 m/s (d) 0.0007 m/s (c) (d) None of these
EBD_7418
130 PHYSICS

84. Drops of liquid of density r are floating half immersed in a 90. Column I Column II
liquid of density s. If the surface tension of liquid is T, the (A) Capillaries of smaller (1) Flat meniscus
radius of the drop will be radii
3T 6T
(a) (b) (B) Fc > 2Fa where Fc (2) Greater height difference
g (3r - s) g (2r - s )
and Fa are cohesive and adhesive force
3T 3T
(c) (d) (C) Angle of contact is (3) Drop in level
g (2r - s ) g (4r - 3s)
zero
85. Let T1 be surface tension between solid and air, T 2 be the (D) Lower angle of contact (4) Welding agents
surface tension between solid and liquid and T be the
surface tension between liquid and air. Then in equilibrium, (a) (A) ® (4) ; (B) ® (2) ; (C) ® (3) ; (D) ® (1)
for a drop of liquid on a clean glass plate, the relation is (b) (A) ® (3) ; (B) ® (4) ; (C) ® (1) ; (D) ® (2,3)
(q is angle of contact) (c) (A)® (2) ; (B) ® (1) ; (C) ® (4) ; (D) ® (3)
(d) (A) ® (3) ; (B) ® (4) ; (C) ® (2) ; (D) ® (1)
T 91. For a given volume which of the following will have
liquid air
minimum energy?
q
(a) Cube (b) Cone
T1 Solid T2 (c) Sphere (d) All have same energy
92. Which of the following expressions represents the excess
T T
(a) cos q = (b) cos q = of pressure inside the soap bubble?
T1 + T2 T1 - T2
2s
(a) Pi – Po = s (b) Pi – Po =
T1 + T2 T1 - T2 r r
(c) cos q = (d) cos q = 4s
T T 2s
(c) Pi – Po = + hrg (d) Pi – Po =
86. At critical temperature, the surface tension of a liquid is r r
(a) zero 93. The lower end of a capillary tube of radius r is placed
(b) infinity vertically in water. Then, with the rise of water in the
(c) the same as that at any other temperature capilllary , heat evolved is :
(d) None of these p2r2h2 pr 2 h 2 dg
87. If two soap bubbles of different radii are in communication (a) + dg (b) +
J 2J
with each other then
(a) air flows from the larger bubble into the smaller one pr 2 h 2dg pr 2 h 2dg
(c) – (d) –
until the two bubbles are of equal size 2J J
(b) the size of the bubbles remains the same 94. Assertion : Surface tension of all lubricating oils and paints
(c) air flows from the smaller bubble into the larger one is kept high.
and the larger one grows at the expense of the smaller Reason : Due to high value of surface tension the fluids
one don't get damaged.
(d) the air flows from the larger into the smaller bubble (a) Assertion is correct, reason is correct; reason is a
until the radius of the smaller one becomes equal to correct explanation for assertion.
that of the larger one and of the larger one equal to (b) Assertion is correct, reason is correct; reason is not a
that of the smaller one. correct explanation for assertion
88. Two water droplets merge with each other to form a larger (c) Assertion is correct, reason is incorrect
droplet. In this process (d) Assertion is incorrect, reason is correct.
(a) energy is liberated 95. When a pinch of salt or any other salt which is soluble in
(b) energy is absorbed water is added to water, its surface tension
(c) energy is neither liberated nor absorbed (a) increases
(d) some mass is converted into energy
(b) decreases
89. Consider the following statements and select the true (c) may increase or decrease depending upon salt
statement(s)?
(d) None of these
I. A large soap bubble shrinks while a small soap bubble
96. If more air is pushed in a soap bubble, the pressure in it
expands when they are connected to each other by a
(a) decreases (b) becomes zero
capillary tube, in order to gain equilibrium
(c) remains same (d) increases
II. The raindrops fall on the surface of earth with the same
97. A drop of oil is placed on the surface of water. Which of the
constant velocity
following is correct?
III. A hydrogen filled balloon stops rising after it has
(a) It will remain on it as a sphere
attained a certain height in the sky
(b) It will spread as a thin layer
(a) I only (b) II only (c) III only (d) I & III
(c) It will partly be a spherical droplet and partly a thin film
(d) It will float as a distorted drop on the water surface
MECHANICAL PROPERTIES OF FLUIDS 131

98. Kerosene oil rises up in a wick of a lantern because of 109. Two soap bubbles A and B are kept in a closed chamber
(a) diffusion of the oil through the wick where the air is maintained at pressure 8 N/m 2. The radii of
(b) capillary action bubbles A and B are 2 cm and 4 cm, respectively.Surface
(c) buoyant force of air tension of the soap-water used to make bubbles is 0.04 N/m.
(d) the gravitational pull of the wick Find the ratio nB/nA, where nAand nB are the number of moles
99. Due to capillary action, a liquid will rise in a tube if angle of of air in bubbles A and B, respectively. [Neglect the effect of
contact is gravity]
(a) acute (b) obtuse (c) 90º (d) zero (a) 2 (b) 9 (c) 8 (d) 6
110. An isolated and charged spherical soap bubble has a radius
100. With the increase in temperature, the angle of contact
r and the pressure inside is atmospheric. If T is the surface
(a) decreases
(b) increases tension of soap solution, then charge on drop is X pr 2rTe 0
(c) remains constant then find the value of X.
(d) sometimes increases and sometimes decreases (a) 8 (b) 9 (c) 7 (d) 2
101. Select the false statement(s) about surface tension from the 111. A 20 cm long capillary tube is dipped in water. The water rises
following? up to 8 cm. If the entire arrangement is put in a freely falling
I. Surface tension is the extra energy that the molecules at the elevator the length of water column in the capillary tube will be
interface have as compared to the molecules in the interior (a) 10 cm (b) 8 cm (c) 20 cm (d) 4 cm
II. The value of surface tension is independent of the 112. In a capillary tube, water rises to 3 mm. The height of water
that will rise in another capillary tube having one-third radius
temperature
of the first is
III. Surface tension causes capillary action
(a) 1 mm (b) 3mm (c) 6 mm (d) 9mm
(a) I only (b) II only (c) I & II (d) I, II & III 113. In a surface tension experiment with a capillary tube water
102. A glass capillary tube of inner diameter 0.28 mm is lowered rises upto 0.1 m. If the same experiment is repeated on an
vertically into water in a vessel. The pressure to be applied on artificial satellite, which is revolving around the earth, water
the water in the tube so that water level in the tube is same as that will rise in the capillary tube upto a height of
in the vessel in N/m 2 is (surface tension of water (a) 0.1 m (b) 0.2 m (c) 0.98 m
= 0.07 N/m, atmospheric pressure = 105 N/m2) : (d) full length of the capillary tube
(a) 103 (b) 99 × 103 (c) 100 × 103 (d) 101 × 103 114. Water rises in a capillary tube to a certain height such that the
103. Assertion : Smaller the droplets of water, spherical they are. upward force due to surface tension is balanced by 7.5 × 10–4N
Reason : Force of surface tension is equal, and opposite to force due to the weight of the liquid. If the surface tension of
force of gravity. water is 6 × 10–2Nm–1, the inner circumference of the capillary
(a) Assertion is correct, reason is correct; reason is a correct tube must be
explanation for assertion. (a) 1.25 × 10–2m (b) 0.50 × 10–2m
(b) Assertion is correct, reason is correct; reason is not a (c) 6.5 × 10 m –2 (d) 12.5 × 10–2m
correct explanation for assertion 115. A glass tube of uniform internal radius (r) has a valve
(c) Assertion is correct, reason is incorrect separating the two identical ends. Initially, the valve is in a
(d) Assertion is incorrect, reason is correct. tightly closed position. End 1 has a hemispherical soap bubble
104. A beaker of radius 15 cm is filled with a liquid of surface of radius r. End 2 has sub-hemispherical soap bubble as shown
tension 0.075 N/m. Force across an imaginary diameter on the in figure. Just after opening the valve
surface of the liquid is
(a) 0.075 N (b) 1.5 × 10–2 N
(c) 0.225 N (d) 2.25 × 10–2 N
105. A water film is formed between two straight parallel wires of
10 cm length 0.5 cm apart. If the distance between wires is 2 1
increased by 1 mm. What will be the work done ? (a) air from end 1 flows towards end 2. No change in the
(surface tension of water = 72 dyne/cm) volume of the soap bubbles
(a) 36 erg (b) 288 erg (c) 144 erg (d) 72 erg (b) air from end 1 flows towards end 2. Volume of the soap
106. The work done in increasing the size of a soap film from bubble at end 1 decreases
10 cm × 6 cm to 10 cm × 11 cm is 3 × 10–4J. The surface tension (c) no change occurs
of the film is (d) air from end 2 flows towards end 1. Volume of the soap
(a) 11 × 10–2 N/m (b) 6 × 10–2 N/m bubble at end 1 increases
(c) 3 × 10–2 N/m (d) 1.5 × 10–2 N/m 116. Radius of a capillary tube is 2 × 10–3m. A liquid of weight
107. A mercury drop of radius 1 cm is sprayed into 106 drops of 6.28 × 10–4 N may remain in the capillary then the surface
equal size. The energy expressed in joule is (surface tension
tension of liquid will be
of Mercury is 460 × 10–3 N/m)
(a) 0.057 (b) 5.7 (c) 5.7 × 10–4 (d) 5.7 × 10–6 (a) 5 × 103 N/m (b) 5 × 10–2 N/m
108. Work done in increasing the size of a soap bubble from radius (c) 5 N/m (d) 50 N/m
3 cm to 5 cm is nearly (surface tension of soap solution = 0.03 117. If the surface tension of water is 0.06 Nm –1, then the capil-
Nm–1) lary rise in a tube of diameter 1 mm is (q = 0°)
(a) 0.2 p mJ (b) 2pmJ (c) 0.4p mJ (d) 4 p mJ (a) 1.22 cm (b) 2.44 cm (c) 3.12 cm (d) 3.68 cm
EBD_7418
132 PHYSICS

118. The height upto which liquid rises in a capillary tube is given if it is initially resting on a horizontal water surface. The
2Scos q circumference of the ring is 20 cm and its weight is 0.1 N :
by h = this is for which of the following cases,
hrg (a) 0.125 N
water will be depressed in such a tube? (b) 0.225 N
(a) q is acute (b) q is a right angle
(c) q is zero (d) q is obtuse (c) 0.115 N
119. Two capillary tubes A and B of diameter 1 mm and 2 mm (d) 0.130 N
respectively are dipped vertically in a liquid. If the capillary 124. On heating water, bubbles being formed at the bottom of
rise in A is 6 cm, then the capillary rise in B is the vessel detach and rise. Take the bubbles to be spheres
(a) 2 cm (b) 3 cm (c) 4 cm (d) 6 cm of radius R and making a circular contact of radius r with the
120. Water rises to a height of 10 cm in capillary tube and mercury bottom of the vessel. If r << R and the surface tension of
falls to a depth of 3.1 cm in the same capillary tube. If the water is T, value of r just before bubbles detach is:
density of mercury is 13.6 and the angle of contact for mercury (density of water is rw)
is 135°, the approximate ratio of surface tensions of water rw g
(a) R2
and mercury is 3T
(a) 1 : 0.15 (b) 1 : 3 (c) 1 : 6 (d) 1.5 : 1
121. In a capillary rise experiment, the water level rises to a height 2rw g
(b) R2
of 5 cm. If the same capillary tube is placed in water such 3T
that only 3 cm of the tube projects outside the water level, then rw g
(a) water will begin to overflow through the capillary (c) R2
T R
(b) angle of contact decreases
(c) angle of contact increases 3rw g
(d) R2
(d) the meniscus completely vanishes T 2r
122. A soap film of surface tension 3 × 10–2 formed in a rectangular 125. A thin liquid film formed between a U-shaped wire and a
frame cam support a straw as shown in Fig. If g = 10 ms–12, light slider supports a weight of 1.5 × 10–2 N (see figure).
the mass of the straw is 10 cm
The length of the slider is 30 cm and its weight negligible.
(a) 0.006 g Soap The surface tension of the liquid film is
film
(b) 0.06 g (a) 0.0125 Nm–1 Film
(c) 0.6 g Straw (b) 0.1 Nm–1
(d) 6 g
123. The surface tension of water is 75 dyne/cm. Find the minimum (c) 0.05 Nm–1
vertical force required to pull a thin wire ring up (refer figure) (d) 0.025 Nm–1 W

Exercise 2 : Exemplar & Past Year MCQs


NCERT Exemplar MCQs 2. Which of the following diagrams does not represent a
streamline flow?
1. A tall cylinder is filled with viscous oil. A round pebble is
dropped from the top with zero initial velocity. From the plot
shown in figure, indicate the one that represents the velocity
(v) of the pebble as a function of time (t) .
(a) (b)

(a) v (b) v

t t
(c) (d)

(c) v (d) v

t t
MECHANICAL PROPERTIES OF FLUIDS 133

3. Along a streamline, 10. Two capillary tubes are of the same diameters. One is dipped
(a) the velocity of a fluid particle remains constant in a liquid of relative density 0.8 while the other in a liquid of
(b) the velocity of all fluid particles crossing a given relative density 0.6. If surface tensions of these liquids are
position is constant 60 and 50 milli N/m, respectively, and the angles of the con-
(c) the velocity of all fluid particles at a given instant is tact are equal, compare the rise of liquids in the capillary
constant tubes. [BITSAT2014, A]
(d) the speed of a fluid particle remains constant (a) 9 : 10 (b) 7 : 10 (c) 3 : 10 (d) 1 : 10
4. An ideal fluid flows through a pipe of circular cross-section 11. Water rises to a height 'h' in a capillary tube. If the length of
made of two sections with diameters 2.5 cm and 3.75 cm. capaillary tube above the surface of water is made less than
The ratio of the velocities in the two pipes is 'h' then : [AIPMT 2015, C]
(a) 9 : 4 (b) 3 : 2 (c) 3 : 2 (d) 2: 3 (a) water rises upto the top of capillary tube and stays
5. The angle of contact at the interface of water-glass is 0°, there without overflowing
ethyl alcohol-glass is 0°, mercury-glass is 140° and (b) water rises upto a point a little below the top and stays
methyliodide-glass is 30°. A glass capillary is put in a trough there
containing one of these four liquids. It is observed that the (c) water does not rise at all.
meniscus is convex. The liquid in the trough is (d) Water rises upto the tip of capillary tube and then starts
(a) water (b) ethylalcohol
overflowing like fountain.
(c) mercury (d) methyliodide
12. The cylindrical tube of a spray pump has radius, R, one end
Past Year MCQs of which has n fine holes, each of radius r. If the speed of the
liquid in the tube is V, the speed of the ejection of the liquid
6. A certain number of spherical drops of a liquid of radius ‘r’
through the holes is : [AIPMT 2015, C]
coalesce to form a single drop of radius ‘R’ and volume ‘V’. If ‘T’
is the surface tension of the liquid, then : [AIPMT 2014, S] VR 2 VR 2 V 2R VR 2
æ1 1 ö (a) (b) (c) (d)
(a) energy = 4VT ç - ÷ is released nr 2 n3r 2 nr n2r2
è r Rø 13. The approximate depth of an ocean is 2700 m. The
æ1 1 ö compressibility of water is 45.4 × 10–11 Pa–1 and density of
(b) energy = 3VT ç + ÷ is absorbed
è r Rø water is 103 kg/m3.What fractional compression of water will
æ1 1 ö be obtained at the bottom of the ocean ? [AIPMT 2015, A]
(c) energy = 3VT ç - ÷ is released (a) 1.0 × 10–2 (b) 1.2 × 10–2
è r Rø
(d) energy is neither released nor absorbed (c) 1.4 × 10–2 (d) 0.8 × 10–2
7. A water tank of height 10m, completely filled with water is 14. 1 m3 water is brought inside the lake upto 200 metres depth
placed on a level ground. It has two holes one at 3 m and the from the surface of the lake. What will be change in the
other at 7 m from its base. The water ejecting from volume when the bulk modulus of elastically of water is
[AIIMS 2014, A] 22000 atmosphere? [AIIMS 2015, A]
(a) both the holes will fall at the same spot 3 3
(density of water is 1 × 10 kg/m atmosphere pressure =
(b) upper hole will fall farther than that from the lower hole 105 N/m2 and g = 10 m/s2)
(c) upper hole will fall closer than that from the lower hole
(a) 8.9 × 10–3 m3 (b) 7.8 × 10–3 m3
(d) more information is required
8. There is a circular tube in a vertical (c) 9.1 × 10–4 m3 (d) 8.7 × 10–4 m3
plane. Two liquids which do not 15. Water is flowing on a horizontal fixed surface, such that its
mix and of densities d1 and d2 are flow velocity varies with y (vertical direction) as
filled in the tube. Each liquid æ 2 y2 y3 ö
a d2
subtends 90º angle at centre. v = k ç 2 - 3 ÷ . If coefficient of viscosity for water is h,
ç a a ÷ø
Radius joining their interface è
makes an angle a with vertical. what will be shear stress between layers of water at y =a.
d d1 [BITSAT2015, S]
Ratio 1 is: [JEE Main 2014, S, BN]
d2 hk h
(a) (b)
1 + sin a 1 + cos a a ka
(a) (b) ha
1 - sin a 1 - cos a
(c) (d) None of these
1 + tan a 1 + sin a k
(c) (d) 16. A glass capillary tube of internal radius r = 0.25 mm is
1 - tan a 1 - cos a
9. An open glass tube is immersed in mercury in such a way immersed in water. The top end of the tube projected by 2
that a length of 8 cm extends above the mercury level. The cm above the surface of the water. At what angle does the
open end of the tube is then closed and sealed and the tube liquid meet the tube? Surface tension of water = 0.7 N/m.
is raised vertically up by additional 46 cm. What will be [BITSAT2015, A]
length of the air column above mercury in the tube now?
(a) q = 90° (b) q = 70° (c) q = 45° (d) q = 35°
(Atmospheric pressure = 76 cm of Hg) [JEE Main 2014, S]
(a) 16 cm (b) 22 cm (c) 38 cm (d) 6 cm
EBD_7418
134 PHYSICS

17. Two non-mixing liquids of densities r and nr (n > 1) are put


in a container. The height of each liquid is h. A solid cylinder Cylindrical surface
of water
of length L and density d is put in this container. The cylinder
floats with its axis vertical and length pL(p < 1) in the denser 2T 4T T T
(a) (b) (c) (d)
liquid. The density d is equal to : [NEET 2016, C] R R 4R R
22. Assertion: The pressure of water reduces when it flows
(a) {1 + (n + 1)p}r (b) {2 + (n + 1)p}r
from a narrow pipe to a wider pipe.
(c) {2 + (n – 1)p}r (d) {1 + (n – 1)p}r
Reason: Since for wider pipe area is large, so flow of speed
18. A uniform cylinder of length L and mass M having cross- is small and pressure also reduces proportionately.
sectional area A is suspended, with its length vertical, from [AIIMS 2017, C]
a fixed point by a massless spring such that it is half (a) Assertion is correct, reason is correct; reason is a
submerged in a liquid of density s at equilibrium position. correct explanation for assertion.
The extension x0 of the spring when it is in equilibrium is: (b) Assertion is correct, reason is correct; reason is not a
correct explanation for assertion
[AIIMS 2016, C]
(c) Assertion is correct, reason is incorrect
Mg Mg æ LAs ö (d) Assertion is incorrect, reason is correct.
(a) (b) ç1 – ÷
k k è M ø 23. A mercury drop of radius 1 cm is sprayed into 10 6 drops of
Mg æ LAs ö Mg æ LAs ö equal size. The energy expressed in joule is (surface tension
(c) ç1 – ÷ (d) ç1 + ÷
k è 2M ø k è M ø of Mercury is 460 × 10–3 N/m) [BITSAT 2017, A]
19. A capillary tube of radius r is immersed vertically in a liquid (a) 0.057 (b) 5.7
such that liquid rises in it to height h (less than the length of
(c) 5.7 × 10–4 (d) 5.7 × 10–6
the tube). Mass of liquid in the capillary tube is m. If radius
of the capillary tube is increased by 50%, then mass of liquid 24. A capillary tube of radius R is immersed in water and water
that will rise in the tube, is [BITSAT 2016, S] rises in it to a height H. Mass of water in the capillary tube
2 3 9 is M. If the radius of the tube is doubled, mass of water that
(a) m (b) m (c) m (d) m
3 2 4 will rise in the capillary tube will now be : [BITSAT 2017, C]
20. A U tube with both ends open to the atmosphere, is partially (a) M (b) 2 M (c) M/2 (d) 4 M
filled with water. Oil, which is immiscible with water, is poured
25. A small sphere of radius ‘r’ falls from rest in a viscous
into one side until it stands at a distance of 10 mm above the
water level on the other side. Meanwhile the water rises by liquid. As a result, heat is produced due to viscous force.
65 mm from its original level (see diagram). The density of The rate of production of heat when the sphere attains its
the oil is Pa Pa [NEET 2017, S] terminal velocity, is proportional to [NEET 2018, C]
A
F (a) r3 (b) r2 (c) r4 (d) r5
(a) 425 kg m–3 E 10 mm
Final water level 26. A film of water is formed between two straight parallel wires
65 mm of length 10 cm each separated by 0.5 cm. If their separation
(b) 800 kg m–3 Oil D
Initial water level
is increased by 1 mm while still maintaining their parallelism,
65 mm
(c) 928 kg m–3 B C how much work will have to be done? (Surface tension of
Water water = 7.2 × 10–2 N/m) [AIIMS 2018, S]
(d) 650 kg m–3 –6
(a) 7.22 × 10 joule (b) 1.44 × 10–5 joule
21. If two glass plates have water between them and are (c) 2.88 × 10–5 joule (d) 5.76 × 10–5 joule
separated by very small distance (see figure), it is very 27. Figure shows a capillary rise H. If the air is blown through
difficult to pull them apart. It is because the water in between the horizontal tube in the direction as shown then rise in
forms cylindrical surface on the side that gives rise to lower
pressure in the water in comparison to atmosphere. If the capillary tube will be [BITSAT 2018, S]
radius of the cylindrical surface is R and surface tension of (a) =H
water is T then the pressure in water between the plates is H
(b) >H
lower by [AIIMS 2017, C]
(c) <H
(d) zero
MECHANICAL PROPERTIES OF FLUIDS 135

Exercise 3 : Try If You Can


1. A vessel in the shape of a hollow hemisphere surmounted by 7. A cylindrical vessel filled with water upto a height
a cone is held with the axis vertical and vertex uppermost. If it
be filled with a liquid so as to submerge half the axis of the cone h is connected to a light inextensible string. The
in the liquid, and the height of the cone be double the radius of string is wound on a solid uniform disc of radius
its base, find the value of x, where the resultant downward R. With what velocity will the first water particle
thrust of the liquid on the vessel is x times the weight of the
liquid that the hemisphere can hold. be ejected from the vessel through a small hole
(a) 15/8 (b) 1/8 (c) 5/8 (d) 15/2 made at a depth y = (3/10) R, just after the
2. A rectangular gate shown should dip automatically when disc is spin ed with an an gular speed
the water rises above a certain level. Determine that level in
terms of h. w = g / R and released? Assume that mass
of the disc is equal to the sum of mass of
vessel plus water. Ignore friction anywhere and avoid slipping
h
of the string with the disc.
1m 8
(a) gR (b) 2gR
5
(a) 2.123m (b) 1.212m (c) 1.512m (d) 1.732m
3. A tank is filled with water upto a height H. Water is allowed 8 6
(c) g(R + h) (d) gR
to come out of a hole P in one of the walls at a depth h below 5 5
the surface of water (see fig.) Express the horizontal distance 8. Find the force acting on the blade of an undershot wheel
X in terms H and h. (figure) if the stream after impinging on the blade continues
to move with the velocity of the blade.
(a) X = h (H - h ) h
P
The height of the water head is h, the radius of the wheel is
h R, the angular velocity of the wheel is w and the cross-
(b) X= (H - h ) H
2 section area of the stream is A.
(c) X = 2 h (H - h) (a) r A ( 2 gh - w R ) 2
(d) X = 4 h ( H - h ) x (b) rA ( 2 gh + wR )
2 h

4. –1
Water is flowing at a speed of 1.5 ms through horizontal
tube of cross-sectional area 10–2 m2 and you are trying to (c) rA ( 2 gh + wR ) R

stop the flow by your palm. Assuming that the water stops (d) rA (2 gh + w R )
immediately after hitting the palm, the minimum force 9. A long cylinder of radius R1 is displaced along its axis with
that you must exert should be a constant velocity v0 inside a stationary co-axial cylinder
(density of water = 103 kgm–3) of radius R2. The space between the cylinders is filled with
(a) 15 N (b) 22.5 N (c) 33.7 N (d) 45 N viscous liquid. Find the velocity of the liquid as a function
5. In a cylindrical water tank, there are two small holes A and B of the distance r from the axis of the cylinders. The flow is
on the wall at a depth of h1, from the surface of water and at laminar.
a height of h2 from the bottom of water tank. Surface of v0 ln (r / R2 )
ln (r / R2 )
water is at height of h2 from the bottom of water tank. Surface (a) 2v0 (b)
of water is at heigh H from the bottom of water tank. Water ln ( R1 / R2 ) 2 ln ( R1 / R2 )
coming out from both holes strikes the ground at the same ln ( r / R2 ) 2v0 ln (r / R2 )
(c) v0 (d)
point S. Find the ratio of h1 and h2 ln ( R1 / R2 ) 3 ln ( R1 / R2 )
h1
(a) Depends on H 10. A ring is cut from a platinum tube 8.5 cm internal and 8.7 cm
A external diameter. It is supported horizontally from the pan
(b) 1 : 1 H of a balance, so that it comes in contact with the water in a
B
glass vessel. If an extra 3.97. If is required to pull it away from
(c) 2 : 2 water, the surface tension of water is
h2 (a) 72 dyne cm–1 (b) 70.80 dyne cm–1
(d) 1 : 2 (c) 63.35 dyne cm–1 (d) 60 dyne cm–1
S 11. A large number of liquid drops each of radius r coalesce to
6. A 0.7 kg solid cube having edge length 10 cm floats in water.
from a single drop of radius R. The energy released in the
How much volume of the cube is outside the water? [density process is converted into kinetic energy of the big drop so
of water = 1000 kg/m3] formed. The speed of the big drop is (given, surface tension
(a) 300 cm3 (b) 400 cm3 (c) 500 cm3 (d) 600 cm3 of liquid T, density r)
EBD_7418
136 PHYSICS

T æ1 1 ö 2T æ 1 1 ö l l
- - (a) (b)
r çè r R ÷ø r çè r R ÷ø
(a) (b) p r p r
1+ 0 1+ 0
T 2T
4T æ 1 1 ö 6T æ 1 1 ö
ç - ÷ -
r çè r R ÷ø
(c) (d) l l h
r èr Rø (c) (d)
p r 2 p r
12. A thin tube sealed at both ends is 100 cm long. It lies 1+ 0 1+ 0
horizontally, the middle 20 cm containing mercury and two 4T T
equal ends containing air at standard atmospheric pressure. 14. Two parallel glass plates are dipped partly in the liquid of
If the tube is now turned to a vertical position, by what density 'd' keeping them vertical . If the distance between
amount will the mercury be displaced ? the plates is 'x', surface tension for liquids is T and angle of
(Given : cross-section of the tube can be assumed to be contact is q, then rise of liquid between the plates due to
uniform) capillary will be
(a) 2.95 cm T cos q 2T cos q 2T T cos q
(a) (b) (c) (d)
(b) 5.18 cm xd xdg xdg cos q xdg
l0 l0
(c) 8.65 cm 20 cm 15. A cylinder with movable piston contains air under pressure
(d) 0.0 cm 100 cm P0 and a soap bubble of radius r. The surface tension of
13. A glass capillary of length l and inside radius r(r < < l) is soap solution is T and the temperature of the system is kept
submerged vertically into water. The upper end of the constant. The pressure to which the air should be com-
capillary is sealed. The atmospheric pressure is p0. To what pressed by slowly pushing the piston into the cylinder for
length h has the capillary to be submerged to make the the soap bubble to reduce its size by half is
water levels inside and outside the capillary coincide? é 3T ù é Tù
Assume that temperature of air in the capillary remains (a) 8 ê P0 + ú (b) êë P0 + r úû
constant. ë r û
(Given, surface tension of water = T, angle of contact between é Tù é 7T ù
(c) 8 ê P0 + ú (d) 8 ê P0 +
glass water interface = 0°). ë rû ë r úû

ANSW ER KEYS
Exercis e 1 : Topic-wis e MCQs
1 (c) 14 (b) 27 (a) 40 (d) 53 (c) 66 (d) 79 (d) 92 (d) 105 (c) 118 (d)
2 (d) 15 (d) 28 (a) 41 (c) 54 (a) 67 (c) 80 (c) 93 (a) 106 (c) 119 (b)
3 (a) 16 (c) 29 (d) 42 (b) 55 (a) 68 (a) 81 (a) 94 (d) 107 (a) 120 (c)
4 (c) 17 (c) 30 (b) 43 (d) 56 (a) 69 (b) 82 (b) 95 (a) 108 (c) 121 (c)
5 (b) 18 (a) 31 (a) 44 (d) 57 (b) 70 (a) 83 (c) 96 (b) 109 (d) 122 (c)
6 (d) 19 (d) 32 (c) 45 (c) 58 (c) 71 (c) 84 (c) 97 (b) 110 (a) 123 (d)
7 (c) 20 (b) 33 (b) 46 (a) 59 (d) 72 (d) 85 (d) 98 (b) 111 (c) 124 (b)
8 (d) 21 (d) 34 (b) 47 (a) 60 (a) 73 (c) 86 (a) 99 (a) 112 (d) 125 (d)
9 (b) 22 (a) 35 (a) 48 (b) 61 (c) 74 (a) 87 (c) 100 (a) 113 (d)
10 (c) 23 (b) 36 (a) 49 (b) 62 (a) 75 (c) 88 (a) 101 (b) 114 (a)
11 (c) 24 (c) 37 (a) 50 (a) 63 (b) 76 (b) 89 (c) 102 (b) 115 (b)
12 (c) 25 (a) 38 (b) 51 (c) 64 (d) 77 (c) 90 (b) 103 (c) 116 (b)
13 (d) 26 (c) 39 (b) 52 (b) 65 (c) 78 (b) 91 (c) 104 (d) 117 (b)
Exercis e 2 : Exemplar & Pas t Year MCQs
1 (c) 4 (a) 7 (a) 10 (a) 13 (b) 16 (b) 19 (c) 22 (a) 25 (d)
2 (d) 5 (c) 8 (c) 11 (a) 14 (c) 17 (d) 20 (c) 23 (a) 26 (b)
3 (b) 6 (c) 9 (a) 12 (a) 15 (a) 18 (c) 21 (d) 24 (b) 27 (b)
Exercis e 3 : Try If You Can
1 (a) 3 (c) 5 (a) 7 (d) 9 (c) 11 (d) 13 (b) 15 (a)
2 (d) 4 (a) 6 (a) 8 (a) 10 (a) 12 (b) 14 (b)
11Chapter
THERMAL PROPERTIES
OF MATTER

Trend
Analysis of NEET and AIIMS (Year 2010-2018)
5

4
Number of Questions

3
AIPMT/NEET
2 AIIMS

0
2010 2011 2012 2013 2014 2015 2016 2017 2018
Year

Trend
Analysis of JEE Main and BITSAT (Year 2010-2018)
5

4
Number of Questions

3
JEE Main
2 BITSAT

0
2010 2011 2012 2013 2014 2015 2016 2017 2018
Year

Chapter Utility Score (CUS)


Exam Weightage Important Concepts Difficulty Level CUS (Out of 10)
NEET 5 Thermal Expansion,
AIIMS 5 Specific Heat, Latent Heat, 4/5 8/10
JEE Main 3 Calorimetry, Heat transfer
BITSAT 3 and Newton’s law of Cooling
EBD_7418
138 PHYSICS
THERMAL PROPERTIES OF MATTER 139
EBD_7418
140 PHYSICS

Exercise 1 : Topic-wise MCQs

9. An iron tyre is to be fitted on to a wooden wheel 1m in


Topic 1: Thermometer and Thermal Expansion
diameter. The diameter of tyre is 6 mm smaller than that of
1. The reading of Centigrade thermometer coincides with that wheel. The tyre should be heated so that its temperature
of Fahrenheit thermometer in a liquid. The temperature of increases by a minimum of (the coefficient of cubical
the liquid is expansion of iron is 3.6 × 10–5/°C)
(a) – 40ºC (b) 313°C (c) 0°C (d) 100°C (a) 167°C (b) 334°C (c) 500°C (d) 1000°C
2. Expansion during heating 10. Assertion: The triple point of water is a standard fixed
(a) occurs only in solids point in modern thermometry.
(b) increases the weight of a material Reason: The triple point of a substance is unique i.e. it
(c) generally decreases the density of a material occurs at one particular set of values of pressure and
(d) occurs at the same rate for all liquids and solids temperature.
3. Choose the correct equation for interconversion of (a) Assertion is correct, reason is correct; reason is a
temperature scales. correct explanation for assertion.
TC - 0 TF - 32 (b) Assertion is correct, reason is correct; reason is not a
(a) = correct explanation for assertion
100 180
(c) Assertion is correct, reason is incorrect
TF - 32 T + 273.15
(b) = K (d) Assertion is incorrect, reason is correct.
180 100 11. A metal sheet with a circular hole is heated. The hole
TF - 32 T - 273.15 (a) gets larger
(c) = K (b) gets smaller
180 180
TC - 0 TF - 32 (c) remains of the same size
(d) = (d) gets deformed
180 100
12. A solid ball of metal has a spherical cavity inside it. The ball
4. Which of the following will expand the most for same rise
is heated. The volume of cavity will
in temperature?
(a) decrease (b) increase
(a) Aluminium (b) Glass
(c) remain unchanged (d) have its shape changed
(c) Wood (d) All will expand same
13. On a linear temperature scale Y, water freezes at – 160° Y and
5. If a, b and g are coefficient of linear, area and volume
boils at – 50° Y. On this Y scale, a temperature of 340 K
expansion respectively, then
would be read as : (water freezes at 273 K and boils at 373 K)
(a) g = 3a (b) a = 3g
(a) – 73.7° Y (b) – 233.7° Y
(c) b = 3a (d) g = 3b
(c) – 86.3° Y (d) – 106.3° Y
6. The TF versus Tc graph will be a
14. Assertion : Water kept in an open vessel will quickly
(a) straight line parallel to x-axis
evaporate onthe surface of the moon.
(b) straight line parallel to y-axis
Reason : The temperature at the surface of the moon is
(c) straight line inclined to x-axis much higher than boiling point of the water.
(d) parabolic curve (a) Assertion is correct, reason is correct; reason is a
7. Consider the following statements and select the incorrect correct explanation for assertion.
statements. (b) Assertion is correct, reason is correct; reason is not a
I. Water expands on heating between 0°C & 4°C correct explanation for assertion
II. The density of water is minimum at 4°C (c) Assertion is correct, reason is incorrect
III. Density of water increases on heating above 4°C (d) Assertion is incorrect, reason is correct.
IV. Water contracts on heating between 0°C & 4°C 15. A glass flask is filled up to a mark with 50 cc of mercury at
(a) I and II only (b) III and IV only 18°C. If the flask and contents are heated to 38°C, how much
(c) I , II and III (d) I, II, III and IV mercury will be above the mark? (a for glass is 9 × 10 –6/ºC
8. On heating a liquid of coefficient of cubical expansion g and coefficient of real expansion of mercury is 180 × 10–6/
in a container having coefficient of linear expansion g/3, ºC)
the level of liquid in the container will (a) 0.85 cc (b) 0.46 cc (c) 0.153 cc (d) 0.05 cc
(a) rise 16. A bar of iron is 10 cm at 20°C. At 19°C it will be (a of iron
(b) fall = 11 × 10–6/°C)
(c) remain almost stationary (a) 11 × 10–6 cm longer (b) 11 × 10–6 cm shorter
(d) It is difficult to say (c) 11 × 10 cm shorter (d) 11 × 10–5 cm longer
–5
THERMAL PROPERTIES OF MATTER 141

17. Consider the following statements and select the correct should the liquid occupy so that the volume of the remaining
statement(s). vacant space will be same at all temperatures?
I. Water can never be boiled without heating. (a) 2 : 5 (b) 1 : 4 (c) 1 : 64 (d) 1 : 8
II. Water can be boiled below room temperature by 26. Which of the following circular rods, (given radius r and
lowering the pressure. length l) each made of the same material and whose ends are
III. On releasing the excess pressure water refreezes into maintained at the same temperature will conduct most heat?
ice. (a) r = 2r0; l = 2l0 (b) r = 2r0; l = l0
(a) I only (b) II only (c) r = r0; l = 2l0 (d) r = r0; l = l0
(c) I and II (d) II and III 27. A beaker contains 200 gm of water. The heat capacity of the
beaker is equal to that of 20 gm of water. The initial
18. The coefficient of apparent expansion of mercury in a glass
temperatue of water in the beaker is 20ºC. If 440 gm of hot
vessel is 153 × 10–6/ºC and in a steel vessel is 144 × 10–6/ºC.
water at 92ºC is poured in it, the final temperature, neglecting
If a for steel is 12 × 10–6/ºC , then that of glass is
radiation loss, will be nearest to
(a) 9 × 10–6/ºC (b) 6 × 10–6/ºC
(a) 58ºC (b) 68ºC (c) 73ºC (d) 78ºC
(c) 36 × 10–6/ºC (d) 27 × 10–6/ºC
28. In a vertical U-tube containing a liquid,
19. The value of coefficient of volume expansion of glycerine is
the two arms are maintained at different t1
5 × 10–4 K–1 . The fractional change in the density of
temperatures t 1 and t 2 . The liquid t2
glycerine for a rise of 40°C in its temperature, is: l1
columns in the two arms have heights
(a) 0.020 (b) 0.025 (c) 0.010 (d) 0.015
l1 and l2 respectively. The coefficient l2
20. A rod PQ of length l is pivoted at an end P and freely rotated
of volume expansion of the liquid is
in a horizontal plane at an angular speed w about a vertical
equal to
axis passing through P. Coefficient of linear expansion of
material of rod is a. Find the percentage change in its angular l1 – l2 l1 – l2
(a) (b)
velocity if temperature of system is increased by DT is l2 t1 – l1t2 l1t1 – l2 t2
(a) (aDT× 100)% (b) (2aDT × 100)% l1 + l2 l1 + l2
(c) (3aDT × 100) × 100% (d) (4aDT×100)% (c) (d)
l2 t1 + l1t2 l1t1 + l2 t2
21. The table gives the initial length L, change in temperature 29. A pendulum clock loses 12 s a day if the temperature is 40°C
DT and change in length DL of four rods. The rod, which has and gains 4 s a day if the temperature is 20° C. The
greatest coefficient of expansion? temperature at which the clock will show correct time, and
Rod L(m) DT(ºC) DL(m) the co-efficient of linear expansion (a) of the metal of the
a 2 10 4 × 10–4 pendulum shaft are respectively :
b 1 20 4 × 10–4 (a) 30°C; a = 1.85 × 10–3/°C
c 2 10 8 × 10–4 (b) 55°C; a = 1.85 × 10–2/°C
d 4 5 4 × 10–4 (c) 25°C; a = 1.85 × 10–5/°C
(d) 60°C; a = 1.85 × 10–4/°C
(a) a (b) b (c) c (d) d
30. The apparent coefficient of expansion of a liquid when
22. A piece of metal weighs 45g in air and 25g in a liquid of
heated in a copper vessel is C and that when heated in a
density 1.5 × 103kg – m–3 kept at 30°C. When the temperature
silver vessel is S. If A is the linear coefficient of expansion of
of the liquid is raised to 40°C, the metal piece weighs 27g.
copper, then the linear coefficient of expansion of silver is
The density of liquid at 40°C, is 1.25 × 103 kg –m–3. The
coefficient of linear expansion of metal is C + S - 3A C + 3A - S
(a) (b)
(a) 1.3 × 10–3/°C (b) 5.2 × 10–3/°C 3 3
–3
(c) 2.6 × 10 /°C (d) 0.26 × 10–3/°C S + 3A - C C + S + 3A
(c) (d)
23. On a new scale of temperature (which is linear) and called 3 3
the W scale, the freezing and boiling points of water are 39° 31. The coefficient of linear expansion for a certain metal varies
W and 239° W respectively. What will be the temperature with temperature as a(T). If L0 is the initial length of the
on the new scale, corresponding to a temperature of 39° C metal and the temperature of metal is changed from T0 to T
on the Celsius scale? (T0 > T), then
(a) 78° W (b) 117° W (c) 200° W (d) 139° W
(b) L = L 0 éê1 + ò a (T) dT ùú
T T
(a) L = L0 ò a(T) dT
24. Two marks on a glass rod 10 cm apart are found to increase T0 ë T0 û
their distance by 0.08 mm when the rod is heated from 0°C to é T ù
100°C. A flask made of the same glass as that of rod measures (c) L = L 0 ê1 - ò a (T) dT ú (d) L > L0
ê T ú
a volume of 1000 cc at 0°C. The volume it measures at 100°C ë 0 û
in cc is 32. The length of a metallic rod is 5 m at 0°C and becomes
(a) 1002.4 (b) 1004.2 (c) 1006.4 (d) 1008.2 5.01 m, on heating upto 100°C. The linear expansion of the
25. The ratio of the coefficient of volume expansion of a glass metal will be
container to that of a viscous liquid kept inside the container (a) 2.33 × 10–5/°C (b) 6.0 × 10–5/°C
–5
(c) 4.0 × 10 /°C (d) 2.0 × 10–5/°C
is 1 : 4. What fraction of the inner volume of the container
EBD_7418
142 PHYSICS

33. A pendulum clock is 5 seconds fast at temperature of 15ºC 42. At temperature T, the emissive power and absorption
and 10 seconds slow at a temperature of 30ºC. At what power of a body for certain wavelength are el and al
temperature does it give the correct time? (take time interval = respectively, then
24 hours) (a) el = al
(a) 18ºC (b) 20ºC (c) 22ºC (d) 25ºC (b) el > al
(c) el < al
Topic 2: Calorimetry and Heat Transfer (d) no difinite relation between el and al
43. Which of the following statements regarding specific heat
34. Mass of water which absorbs or emits the same amount
capacity of a substance are correct ? It depends on
of heat as is done by the body for the same rise or fall
I. mass of substance.
in temperature is known as
II. nature of substance.
(a) thermal capacity of the body
III. temperature of substance.
(b) specific heat capacity of the body
IV. volume of substance.
(c) latent heat capacity of the body
(a) I and II (b) II and III
(d) water equivalent of the body
(c) III and IV (d) I and IV
35. If two rods A and B of equal length L, and different areas
44. A quantity of heat required to change the unit mass of a
of cross-section A1 and A2 have one end each at
solid substance, from solid state to liquid state, while the
temperature T1 and T2, have equal rates of flow of heat,
then temperature remains constant, is known as
A1 K1 (a) latent heat (b) sublimation
(a) A1 = A2 (b) = (c) hoar frost (d) latent heat of fusion
A2 K 2
45. A bubble of n mole of helium is submerged at a certain
A1 K 2
(c) = (d) K1 = K2 depth in water. The temperature of water increases by Dt°C.
A2 K1 How much heat is added approximately to helium during
36. Heat is transmitted from higher to lower temperature through
expansion?
actual mass motion of the molecules in nc p n 2c v
(a) conduction (b) convection (a) ncvDt (b) (c) (d) ncpDt
(c) radiation (d) None of these Dt Dt
46. Which of the following is used as a coolent in automobile
37. Good absorbers of heat are radiator as well as a heater in hot water bags?
(a) poor emitters (b) non-emitters (a) Ice (b) Sand
(c) good emitters (d) highly polished (c) Water (d) All of these
38. Three bodies A, B and C have equal area which are 47. Six identical conducting rods are joined as shown in figure.
painted red, yellow and black respectively. If they are at Points A and D are maintained at 200°C and 20°C
same temperature, then respectively. The temperature of junction B will be
(a) emissive power of A is maximum.
(b) emissive power of B is maximum. (a) 120°C (b) 100°C
(c) emissive power of C is maximum. A B C D
(d) emissive power of A, B and C are equal. (c) 140°C (d) 80°C
39. Sweet makers do not clean the bottom of cauldron because 48. 500 g of water and 100 g of ice at 0°C are in a calorimeter
(a) emission power of black and bright surface is more. whose water equivalent is 40 g. 10 g of steam at 100°C is
(b) absorption power of black and bright surface is more. added to it. Then water in the calorimeter is : (Latent heat of
(c) black and rough surface absorbs more heat. ice = 80 cal/g, Latent heat of steam = 540 cal/ g)
(d) transmission power of black and rough surface is (a) 580 g (b) 590 g (c) 600 g (d) 610 g
more. 49. Consider the following statements and select the correct
40. 4200 J of work is required for statement(s).
(a) increasing the temperature of 10 g of water through I. Water and ice have same specific heats
10°C II. Water and ice have different specific heats
(b) increasing the temperature of 100 g of water through III. Specific heat of water is more than that of ice
10°C IV. Specific heat of ice is more than that of water
(c) increasing the temperature of 1 kg of water through (a) I only (b) II only
10°C (c) II and III (d) III and IV
(d) increasing the temperature of 500 g of water through 50. The temperature of the two outer surfaces of a composite
10°C slab, consisting of two materials having coefficients of
41. The latent heat of vaporization of a substance is always thermal conductivity K and 2K and thickness x and 4x,
(a) greater than its latent heat of fusion respectively, are T2 and T1(T2 > T1). The rate of heat transfer
(b) greater than its latent heat of sublimation æ A(T2 - T1 ) K ö
(c) equal to its latent heat of sublimation through the slab, in a steady state is ç
è ÷ø f ,
x
(d) less than its latent heat of fusion with f equal to
THERMAL PROPERTIES OF MATTER 143

x 4x
2 57. In an energy recycling process, 100 g of steam at 100°C
(a) becomes water at 100°C which converts y g of ice at 0°C
3
into water at 100°C. The numeric value of y is
1 (a) 100 (b) 200 (c) 300 (d) 400
(b) 2K T1
2 K 58. In a water-fall the water falls from a height of 100 m. If the
(c) 1 entire K.E. of water is converted into heat, the rise in
1 temperature of water will be
(d) (a) 0.23°C (b) 0.46°C (c) 2.3°C (d) 0.023°C
3
51. Three liquids A, B and C having same specific heat and 59. 19 g of water at 30°C and 5 g of ice at – 20°C are mixed
mass m, 2m and 3m have temperatures 20°C, 40°C and 60°C together in a calorimeter. What is the final temperature of
respectively. Temperature of the mixture when the mixture? Given specific heat of ice = 0.5 cal g–1(°C)–1
and latent heat of fusion of ice = 80 cal g–1
Column I Column II
(a) 0°C (b) –5°C (c) 5°C (d) 10°C
(A) A and B are mixed (1). 33.3°C 60. The heat (Q) supplied to a solid, which is otherwise thermally
(B) A and C are mixed (2) 52°C isolated from its surroundings, is plotted as a function of its
(C) B and C are mixed (3) 50°C absolute temperature, q. It is found that they are related by
(D) A, B and C all three (4) 46.67°C the equation.
are mixed Q = aq2 + bq4. (a, b are constants).
(a) (A) ® (1) ; (B) ® (3) ; (C) ® (2) ; (D) ® (4) The heat capacity of the solid is given by
(b) (A) ® (1) ; (B) ® (2) ; (C) ® (3) ; (D) ® (4) q3 q5
(c) (A)® (2) ; (B) ® (1) ; (C) ® (4) ; (D) ® (3) (a) a +b (b) aq + bq3
3 5
(d) (A) ® (3) ; (B) ® (4) ; (C) ® (2) ; (D) ® (1)
q q3
52. A block of ice at – 10°C is slowly heated and converted to (c) a + b (d) 2aq + 4bq3
steam at 100°C. Which of the following curves represents 3 5
the phenomenon qualitatively 61. A 2 kg copper block is heated to 500°C and then it is placed
on a large block of ice at 0°C. If the specific heat capacity of
copper is 400 J/kg°C and latent heat of fusion of water is 3.5
Temperature
Temperature

× 105 J/kg, the amount of ice that can melt is


(a) (b) (a) (7/8) kg (b) (7/5) kg (c) (8/7) kg (d) (5/7) kg
62. Two spheres of different materials one with triple the radius
Heat supplied Heat supplied and one-fifth wall thickness of the other are filled with ice. If
the time taken for complete melting of ice in the larger sphere
is 30 minute and for smaller one is 20 minute, the ratio of
Temperature
Temperature

thermal conductivities of the materials of larger spheres to


(c) (d) that of smaller sphere is
(a) 1/8 (b) 3/4 (c) 2/3 (d) 1/2
63. A student takes 50gm wax (specific heat = 0.6 kcal/kg°C)
Heat supplied Heat supplied
and heats it till it boils. The graph between temperature and
53. Assuming no heat losses, the heat released by the time is as follows. Heat supplied to the wax per minute and
condensation of x g of steam at 100°C can be used to convert boiling point are respectively
y g of ice at 0°C into water at 100°C, the ratio x : y is :
Temperature (°C)

(a) 1 : 1 (b) 1 : 2 (c) 1 : 3 (d) 3 : 1 (a) 500 cal, 50°C 250


54. A lead bullet strikes against a steel plate with a velocity 200 200
m s–1 . If the impact is perfectly inelastic and the heat (b) 1000 cal, 100°C 150
produced is equally shared between the bullet and the target, 100
then the rise in temperature of the bullet is (specific heat (c) 1500 cal, 200°C 50
capacity of lead = 125 J kg–1 K–1) 0
1 2 3 4 567 8
(a) 80°C (b) 60°C (c) 160°C (d) 40°C (d) 1000 cal, 200°C Time (Minute)
55. Certain amount of heat is given to 100 g of copper to 64. Column I Column II
increase its temperature by 21° C. If the same amount of
heat is given to 50 g of water, then the rise in its (A) Specific heat (1) l1 – l2 = constant for
temperature is capacity S l1a1 = l2a2
(Specific heat capacity of copper = 400 J kg–1 K–1 and that (B) Two metals (l1, a1) (2) Y is same
for water = 4200 J kg–1 K–1) and (l2, a2) are heated
(a) 4° C (b) 5.25° C (c) 8° C (d) 6° C
56. A hammer of mass 1 kg having speed of 50 m/s, hit a iron nail uniformly
of mass 200 gm. If specific heat of iron is 0.105 cal/gm°C and (C) Thermal stress (3) S = ¥ for DT = 0
half the energy is converted into heat. the raise in (D) Four wires of same (4) Y µ Dt
temperature of nail is
(a) 7.1°C (b) 9.2°C (c) 10.5°C (d) 12.1°C material
EBD_7418
144 PHYSICS

(a) (A) ® (3) ; (B) ® (1) ; (C) ® (4) ; (D) ® (2) ( K 2l1T1 + K1l2T2 ) ( K1l2T1 + K 2l1T2 )
(b) (A) ® (1) ; (B) ® (2) ; (C) ® (3) ; (D) ® (4) (c) (d)
( K 2 l1 + K1l2 ) ( K1l2 + K 2 l1 )
(c) (A)® (2) ; (B) ® (1) ; (C) ® (4) ; (D) ® (3) 71. Four identical rods of same material are joined end to end to
(d) (A) ® (3) ; (B) ® (4) ; (C) ® (2) ; (D) ® (1) form a square. If the temperature difference between the
65. Five rods of same dimensions are arranged as shown in the ends of a diagonal is 100°C, then the temperture difference
figure. They have thermal conductivities K1, K2, K3, K4 and between the ends of other diagonal will be
K5. When points A and B are maintained at different 100 100
(a) 0°C (b) °C (c) º C (d) 100°C
temperatures, no heat flows through the central rod if l 2l
(where l is the length of each rod)
(a) K1 = K4 and K2 = K3
72. The figure shows a system of two
K1 K2
(b) K1 K4 = K2K3 concentric spheres of radii r1 and r2 are
kept at temperatures T 1 and T 2 , r1
(c) K1 K2 = K3 K4 K5 B
A
respectively. The radial rate of flow of T1
K1 K 2 heat in a substance between the two
(d) = K3 K4 r2 T2
K 4 K3 concentric spheres is proportional to
D
æ ö (r2 - r1 )
66. Ice starts forming in a lake with water at 0ºC when the (a) In ç r2 ÷ (b)
è r1 ø (r1 r2 )
atomspheric temperature is –10ºC. If the time taken for the
first 1 cm of ice to be formed is 7 hours, then the time taken r1 r2
(c) (r2 – r1) (d)
for the thickness of ice to change from 1 cm to 2 cm is (r2 - r1 )
(a) 7 hours (b) 14 hours 73. 300 gm of water at 25°C is added to 100 g of ice at 0°C. The
(c) 21 hours (d) 3.5 hours final temperature of the mixture is
67. A kettle with 3 litre water at 27°C is heated by operating coil 5 5
heater of power 2 kW. The heat is lost to the atmosphere at (a) – °C (b) – °C (c) 5°C (d) 0°C
3 2
constant rate 130J/sec, when its lid is open. In how much 74. 2
A slab of stone of area 0.36 m and thickness
time will water heated to 97°C with the lid open? (specific 0.1 m is exposed on the lower surface to steam at 100°C. A
heat of water = 4.2 kJ/kg) block of ice at 0°C rests on the upper surface of the slab. In
(a) 472 sec (b) 693 sec one hour 4.8 kg of ice is melted. The thermal conductivity of
(c) 912 sec (d) 1101 sec slab is :
(Given latent heat of fusion of ice = 3.36 × 105 J kg–1.) :
68. On observing light from three different stars P, Q and R, it (a) 1.24 J/m°C (b) 1.29 J/m°C
was found that intensity of violet colour is maximum in the (c) 2.05 J/m°C (d) 1.02 J/m°C
spectrum of P, the intensity of green colour is maximum in 75. A tube shaped like a parallelogram is filled with liquid is kept
the spectrum of R and the intensity of red colour is maximum in a vertical plane. Tube is heated at mid point of ad. The
in the spectrum of Q. If TP, TQ and TR are the respective heat flows only by convection. Then
absolute temperature of P, Q and R, then it can be concluded (a) Td = Ta > Tb = Tc a b
from the above observations that (b) Ta > Tb > Tc > Td
(a) TP > TR > TQ (b) TP < TR < TQ (c) Ta > Tb > Td = Tc
(d) Td > Ta > Tb > Tc d c
(c) TP < TQ < TR (d) TP > TQ > TR 76. In the plot of temperature versus time showing changes
69. The two ends of a metal rod are maintained at temperatures in the state of ice on heating, which part represents
100°C and 110°C. The rate of heat flow in the rod is found to constant temperature?
be 4.0 J/s. If the ends are maintained at temperatures 200°C D
and 210°C, the rate of heat flow will be
10°C B
(a) 16.8 J/s (b) 8.0 J/s (c) 4.0 J/s (d) 44.0 J/s C
70. One end of a thermally insulated rod is kept at a temperature
(in °C)
Temp.

T1 and the other at l2. The rod is composed of two sections


of length l1 and l2 and thermal conductivities K1 and K2 A
respectively. The temperature at the interface of the two 0°C tm
O
section is Time (in sec.)
(a) OA (b) AB
( K1l1T1 + K 2 l2T2 ) T1 l1 l2 T2
(a) (c) CD (d) All of these
( K1l1 + K 2 l2 ) 77. The coefficient of thermal conductivity of copper, mercury
( K 2l2T1 + K1l1T2 ) and glass are respectively Kc, Km and Kg such that Kc > Km
(b) K1 K2
> Kg. If the same quantity of heat is to flow per sec per unit
( K1l1 + K 2 l2 )
area of each and corresponding temperature gradients are
Xc, Xm and Xg then
THERMAL PROPERTIES OF MATTER 145

(a) Xc = Xm = Xg (b) Xc > Xm > Xg and 50ºC the coefficient of thermal conductivity is
(c) Xc < Xm < Xg (d) Xm < Xc < Xg 0.5 cal
. Then the thermal resistance of the rod in
78. A piece of ice falls from a height h so that it melts completely. cm× sec × ºC
Only one-quarter of the heat produced is absorbed by the cal
ice and all energy of ice gets converted into heat during its is
sec × ºC
fall. The value of h is : [Latent heat of ice is 3.4 × 105 J/kg and
g = 10 N/kg] (a) 318 (b) 31.8 (c) 3.18 (d) 0.318
(a) 34 km (b) 544 km (c) 136 km (d) 68 km 85. A metal ball of surface area 200 square cm, temperature 527ºC
79. Consider a pair of insulating blocks with thermal resistances is surrounded by a vessel at 27ºC. If the emissivity of the
R1 and R2 as shown in the figure. The temperature q at the metal is 0.4, then the rate of loss of heat from the ball is
boundary between the two blocks is é joule ù
(a) (q1q2ÖR1R2) / (q1 + q2) (R1 + R2) approximately ê s = 5.67 ´ 10 -8
2 2ú
ë m ´ sec´ K û
(b) (q1R1 + q2R2) / (R1 + R2) q2 q1
R2 R1 (a) 108 joule (b) 168 joule
(c) [(q1 + q2) R1R2] / (R12 + R22) (c) 182 joule (d) 192 joule
(d) (q1 R2+ q2R1) / (R1 + R2) 86. Two rods of same length and transfer a given amount of
heat 12 second, when they are joined as shown in figure (i).
80. 12 identical rods made of same material are arranged in the
But when they are joined as shown in figure (ii), then they
form of a cube. The temperature of P and R are maintained
will transfer same heat in same conditions in
at 90°C and 30°C respectively. Then the temperature of point
V, when steady state is reached is (a) 24 s
T U
(a) 65°C (b) 13 s l
P Q a Fig. (i)
(b) 60°C 90°C (c) 15 s

(c) 20°C W (d) 48 s l l


V Fig. (ii)
87. The rectangular surface of area 8 cm × 4 cm of a black body
(d) 50°C S R
30°C
at temperature 127°C emits energy E per second. If the length
81. A sphere and a cube of same material and same volume are
and breadth are reduced to half of the initial value and the
heated upto same temperature and allowed to cool in the
temperature is raised to 327°C, the rate of emission of energy
same surroundings. The ratio of the amounts of radiations
becomes
emitted will be
3 81 9 81
4p (a) E (b) E (c) E (d) E
(a) 1 : 1 (b) :1 8 16 16 64
3 88. A partition wall has two layers of different materials A and B
1/3 2/3
æ pö 1 æ 4p ö in contact with each other. They have the same thickness
(c) ç ÷ :1 (d) ç ÷ :1 but the thermal conductivity of layer A is twice that of layer
6
è ø 2è 3 ø
B. At steady state the temperature difference across the
82. ABCDE is a regular pentagon of uniform wire. The rate of
layer B is 50 K, then the corresponding difference across
heat entering at A and leaving at C is equal. TB and TD are
the layer A is
temperature of B and D. Find the temperature TC.
(a) 50 K (b) 12.5 K (c) 25 K (d) 60 K
3TB + 2TD 89. The tempertaure of equal masses of three different liquids
(a)
5 A, B and C are 12°C, 19°C and 28°C respectively. The
temperature when A and B are mixed is 16°C and when B and
(b) 3TD – 2TB
C are mixed is 23°C . The temperature when A and C are
(c) 3TD + 2TB mixed is
(a) 18.2°C (b) 22°C (c) 20.2°C (d) 25.2°C
(d) can have any value 90. Assuming the sun to have a spherical outer surface of radius
83. A black body is at a temperature of 2880 K. The energy of r, radiating like a black body at temperature t°C, the power
radiation emitted by this object with wavelength between received by a unit surface, (normal to the incident rays) at a
499 nm and 500 nm is U1, between 999 nm and 1000 nm is U2 distance R from the centre of the sun is
and between 1499 nm and 1500 nm is U3. Wien’s constant r 2s(t + 273)4 16p 2 r 2st 4
b = 2.88 × 106 nm-K. Then (a) (b)
4 pR 2 R2
(a) U1 = 0 (b) U2 = 0 (c) U1 = U2 (d) U2 > U1
r 2s(t + 273)4 4pr 2st 4
84. A cylindrical rod of aluminium is of length 20 cms and radius (c) (d)
2 cms. The two ends are maintained at temperatures of 0ºC R2 R2
where s is the Stefan’s constant.
EBD_7418
146 PHYSICS

91. A cylindrical rod of length l, thermal conductivity K and body to cool from 51°C to 49°C will be
area of cross section A has one end in the furnace at (a) 8 (b) 5 (c) 6 (d) 4
temperature T1 and the other end in surrounding at 98. In a surrounding medium of temperature 10°C, a body takes
temperature T 2 . Surface of the rod exposed to the 7 min for a fall of temperature from 60°C to 40°C. In what time
surrounding has emissivity e. Also T2 = Ts + DT and Ts >> DT. the temperature of the body will fall from 40°C to 28°C?
If T1 – Ts µ DT, then the proportionality constant is (a) 7 min (b) 11 min (c) 14 min (d) 21 min
Ts
99. A container contains hot water at 100°C. If in time T1
T2
temperature falls to 80° C and the time T2 temperature falls
T1 Insulation
to 60°C form 80°C, then
(a) T1 = T2 (b) T1 > T2
Furnace (c) T1 < T2 (d) None of these
æ 1 + 2e slTs3 ö æ 4eslTs2 ö 100. If a body cools down from 80°C to 60°C in 10 min when the
(a) ç ÷ (b) ç1 + ÷
ç k ÷ è k ø temperature of the surrounding is 30°C, then the temperature
è ø
of the body after next 10 min will be
æ 4eslTs3 ö (a) 50°C (b) 48°C
(c) ç1 + ÷ (d) k(1 + 4e slTs ) (c) 30°C (d) None of these
è k ø
101. A block of steel heated to 100ºC is left in a room to cool.
Topic 3: Newton’s Law of Cooling Which of the curves shown in fig., represents the correct
behaviour?
92. Newton’s law of cooling is applicable for
(a) any excess of temperature over the surrounding (a) A

Temperature
(b) small excess of temperature over the surrounding C
(c) large excess of temperature over the surrounding (b) B B
(d) very large excess of temperature over the surrounding
(c) C A
93. Which of the given graphs proves Newton’s law of cooling?
(d) None of these Time
log (T – T0)

102. A body cools from 50°C to 40°C in 5 minutes. The surrounding


temperature is 20°C. In what further time (in minutes) will it
log (T – T0)

(a) (b)
cool to 30°C?
(a) 5 (b) 15/2 (c) 25/3 (d) 10
Time Time 103. According to Newton’s law of cooling, the rate of cooling of
a body is proportional to (Dq)n, where Dq is the difference of
the temperature of the body and the surroundings, and n is
equal to
(a) two (b) three (c) four (d) one
log (T – T0)

(c) (d) None of these 104. A body cools in a surrounding which


is at a constant temperature of q0.
Time Assume that it obeys Newton’s law q2 P
94. A body cools in a surrounding of constant temperature of cooling. Its temperature q is q
30°C. Its heat capacity is 2J/kg °C. Initial temperature of the plotted against time t. Tangents are q1 f2 f1
Q
drawn to the curve at the points 0 q
body is 40°C. Assume Newton's law of cooling is valid. The t
body cools to 38°C in 10 minutes. In further 10 minutes it P(q = q2) and Q(q = q1). These tangents meet the time axis
will cool from 38°C to at angle of f2 and f1, as shown, then
(a) 36°C (b) 36.4°C (c) 37°C (d) 37.5°C tan f 2 q1 - q0 tan f 2 q 2 - q0
95. Rate of cooling of a spherical body is directly proportional (a) = (b) =
to [where T = body temperature T 0 = surrounding tan f1 q 2 - q0 tan f1 q1 - q0
temperature] tan f1 q1 tan f1 q 2
(a) (T – T0) (b) (T2 – T02) (c) T/T0 (d) T0/T (c) = (d) tan f = q
tan f 2 q 2 2 1
96. A body cools from 50.0°C to 49.9°C in 5s. How long will it
take to cool from 40.0°C to 39.9°C? Assume the temperature 105. Consider two hot bodies B1 and B2 which have temperatures
of surroundings to be 30.0°C and Newton’s law of cooling 100°C and 80°C respectively at t = 0. The temperature of the
to be valid surroundings is 40°C. The ratio of the respective rates of
(a) 2.5 s (b) 10 s (c) 20 s (d) 5 s cooling R1 and R2 of these two bodies at t = 0 will be
97. In a room where the temperature is 30°C, a body cools from (a) R1 : R2 = 3 : 2 (b) R1 : R2 = 5 : 4
61°C to 59°C in 4 minutes. The time (in minutes) taken by the (c) R1 : R2 = 2 : 3 (d) R1 : R2 = 4 : 5
THERMAL PROPERTIES OF MATTER 147

Exercise 2 : Exemplar & Past Year MCQs


NCERT Exemplar MCQs shifting to centre of mass below the centre of the bob
(d) decreases as its effective length remains same but the
1. A bimetallic strip is made of aluminium and steel (aAl >
centre of mass shifts above the centre of the bob
asteel). On heating, the strip will
6. Heat is associated with
(a) remain straight
(a) kinetic energy of random motion of molecules
(b) get twisted
(b) kinetic energy of orderly motion of molecules
(c) will bend with aluminium on concave side
(c) total kinetic energy of random and orderly motion of
(d) will bend with steel on concave side
molecules
2. A uniform metallic rod rotates about its perpendicular
(d) kinetic energy of random motion in some cases and
bisector with constant angular speed. If it is heated uniformly
kinetic energy of orderly motion in other
to raise its temperature slightly
7. The radius of a metal sphere at room temperature T is R and
(a) its speed of rotation increases
the coefficient of linear expansion of the metal is a. The
(b) its speed of rotation decreases
sphere heated a little by a temperature DT so that its new
(c) its speed of rotation remains same
temperature is T + DT. The increase in the volume of the
(d) its speed increases because its moment of inertia
sphere is approximately.
increases
(a) 2pRaDT (b) pR2aDT
3. The graph between two temperature scales A and B is shown
(c) 4pR3aDT/3 (d) 4pR3aDT
in figure between upper fixed point and lower fixed point
8. A sphere, a cube and a thin circular plate, all of same material
there are 150 equal division on scale A and 100 on scale B.
and same mass are initially heated to same high temperature.
The relationship for conversion between the two scales is
(a) Plate will cool fastest and cube the slowest
given by
(b) Sphere will cool fastest and cube the slowest
(c) Plate will cool fastest and sphere the slowest
180
(d) Cube will cool fastest and plate the slowest
Temperature (°A)

DtA = 150° Past Year MCQs


9. Steam at 100°C is passed into 20 g of water at 10°C. When
water acquires a temperature of 80°C, the mass of water
present will be: [AIPMT 2014, A]
DtB = 100° [Take specific heat of water = 1 cal g °C– 1 and latent heat
– 1

O Temperature (°B) 100 of steam = 540 cal g– 1]


(a) 24 g (b) 31.5 g (c) 42.5 g (d) 22.5 g
t A - 180 t B t A - 30 t B
(a) = (b) = 10. Certain quantity of water cools from 70°C to 60°C in the first
100 150 150 100 5 minutes and to 54°C in the next 5 minutes. The temperature
t B - 180 t A t B - 40 t A of the surroundings is: [AIPMT 2014, A]
(c) = (d) =
150 100 100 180 (a) 45°C (b) 20°C (c) 42°C (d) 10°C
4. An aluminium sphere is dipped into water. Which of the 11. If the temperature of the sun were to increase from T to 2T
following is true? and its radius from R to 2R, then the ratio of the radiant
(a) Buoyancy will be less in water at 0°C than that in water energy received on earth to what it was previously will be
at 4°C [AIIMS 2014, C]
(b) Buoyancy will be more in water at 0°C than that in (a) 32 (b) 16 (c) 4 (d) 64
water at 4°C 12. A crystal has a coefficient of expansion 13×10–7 in one
(c) Buoyancy in water at 0°C will be same as that in water direction and 231 × 10–7 in every direction at right angles to
at 4°C it. Then the cubical coefficient of expansion is
(d) Buoyancy may be more or less in water at 4°C [AIIMS 2014, A]
depending on the radius of the sphere (a) 462 × 10–7 (b) 244 × 10–7
5. As the temperature is increased, the period of a pendulum
(c) 475 × 10–7 (d) 257 × 10–7
(a) increases as its effective length increases even though
13. Three rods of Copper, Brass and Steel are welded together
its centre of mass still remains at the centre of the bob
to form a Y shaped structure. Area of cross - section of each
(b) decreases as its effective length increases even though
its centre of mass still remains at the centre of the bob rod = 4 cm2. End of copper rod is maintained at 100ºC where
(c) increases as its effective length increases due to as ends of brass and steel are kept at 0ºC. Lengths of the
EBD_7418
148 PHYSICS

copper, brass and steel rods are 46, 13 and 12 cms at wavelength 500 nm is U2 and that at 1000 nm is U3. Wien's
respectively. The rods are thermally insulated from constant, b = 2.88 × 106 nmK. Which of the following is
surroundings excepts at ends. Thermal conductivities of correct ? [NEET 2016, A]
copper, brass and steel are 0.92, 0.26 and 0.12 CGS units (a) U1 = 0 (b) U3 = 0 (c) U1 > U2 (d) U2 > U1
respectively. Rate of heat flow through copper rod is: 20. A piece of ice falls from a height h so that it melts completely.
[JEE Main 2014, S] Only one-quarter of the heat produced is absorbed by the
(a) 1.2 cal/s (b) 2.4 cal/s (c) 4.8 cal/s (d) 6.0 cal/s ice and all energy of ice gets converted into heat during its
14. The value of coefficient of volume expansion of glycerine is fall. The value of h is : [NEET 2016, A]
5 × 10-4 K-1. The fractional change in the density of glycerine [Latent heat of ice is 3.4 × 105 J/kg and g = 10 N/kg]
for a rise of 40°C in its temperature, is: [AIPMT 2015, A] (a) 34 km (b) 544 km (c) 136 km (d) 68 km
21. Steam is passed into 22 g of water at 20°C. The mass of
(a) 0.020 (b) 0.025 (c) 0.010 (d) 0.015
water that will be present when the water acquires a
15. The rate of heat flow through the cross-section of the rod
shown in figure is (T2 > T1 and thermal conductivity of the temperature of 90°C is (Latent heat of steam is 540 cal/gm)
material of the rod is K) [AIIMS 2015, C] [AIIMS 2016, A]
Kpr1r2 (T2 - T1 ) (a) 24.8 gm (b) 24 gm (c) 36.6 gm (d) 30 gm
(a) 22. Two spheres of different materials one with double the radius
L
and one-fourth wall thickness of the other are filled with ice.
Kp(r1 + r2 )2 (T2 - T1 ) If the time taken for complete melting of ice in the larger
(b) sphere is 25 minute and for smaller one is 16 minute, the
4L r1 r2
ratio of thermal conductivities of the materials of larger
Kp(r1 + r1 ) 2 (T2 - T1 ) L spheres to that of smaller sphere is [BITSAT 2016, C]
(c)
L T1 T2 (a) 4 : 5 (b) 5 : 4 (c) 25 : 8 (d) 8 : 25
23. Two rods A and B of different materials are welded together
Kp(r1 + r1 ) 2 (T2 - T1 )
(d) as shown in figure. Their thermal conductivities are K1 and
2L K2. The thermal conductivity of the composite rod will be :
16. The diagram below shows the change in the length X of a [NEET 2017, C]
thin uniform wire caused by the application of stress F at 3(K1 + K 2 )
two different temperatures T1 and T2. The variation shown (a) K1
2 A
suggests that [AIIMS 2015, C] T1 T2
(b) K1 + K2
(a) T1 > T2 T2 (c) 2 (K1 + K2) B K2
F
(b) T1 < T2 T1 K1 + K 2
(d) d
2
(c) T2 > T1
24. A spherical black body with a radius of 12 cm radiates 450
(d) T1 ³ T2 X watt power at 500 K. If the radius were halved and the
17. A solid body of constant heat capacity 1 J/°C is being heated temperature doubled, the power radiated in watt would be :
by keeping it in contact with reservoirs in two ways : [NEET 2017, A]
(a) 450 (b) 1000 (c) 1800 (d) 225
(i) Sequentially keeping in contact with 2 reservoirs such 25. An external pressure P is applied on a cube at 0 oC so that it
that each reservoir supplies same amount of heat. is equally compressed from all sides. K is the bulk modulus
(ii) Sequentially keeping in contact with 8 reservoirs such of the material of the cube and a is its coefficient of linear
that each reservoir supplies same amount of heat. expansion. Suppose we want to bring the cube to its original
In both the cases body is brought from initial temperature size by heating. The temperature should be raised by :
100°C to final temperature 200°C. Entropy change of the [NEET 2017, C]
body in the two cases respectively is : [JEE Main 2015, A] 3a P P
(a) ln2, 2ln2 (b) 2ln2, 8ln2 (a) (b) 3PKa (c) (d)
PK 3a K aK
(c) ln2, 4ln2 (d) ln2, ln2 26. There rods of the same dimensions have thermal
18. Coefficient of linear expansion of brass and steel rods are conductivities 3K, 2K and K. They are arranged as shown
a1 and a2. Lengths of brass and steel rods are l1 and l2 in fig. with their ends at 100°C, 50°C and 20°C. The
respectively. If (l 2 – l 1 ) is maintained same at all temperature of their junction is [AIIMS 2017, C]
temperatures, which one of the following relations holds 50°C
good ? [NEET 2016, C] 2K
(a) a1l 22 = a 2 l12 (b) a12 l 2 = a 22 l1 100°C 3K q
(c) a1 l 2 =a2 l1 (d) a1l1 = a 2l 2
19. A black body is at a temperature of 5760 K. The energy of K
radiation emitted by the body at wavelength 250 nm is U1,
20°C
THERMAL PROPERTIES OF MATTER 149

(a) 60° (b) 70° (c) 50° (d) 35° 3 4 81 256


27. A beaker is filled with water at 4°C. At one time the (a) (b) (c) (d)
4 3 256 81
temperature is increased by few degrees above 4°C and at 31. A solid substance is at 30°C. To this substance heat energy
another time it is decreased by a few degrees below 4°C. is supplied at a constant rate. Then temperature versus time
One shall observe that: [AIIMS 2017, C] graph is as shown in the figure. The substance is in liquid
(a) the level remains constant in each case state for the portion (of the graph) [AIIMS 2018, C]
(b) in first case water flows while in second case its level

Temperature (T°C)
comes down 240
F
(c) in second case water over flows while in first case its D
210
comes down E
60 B
(d) water overflows in both the cases C
28. A copper ball of mass 100 gm is at a temperature T. It is 30 Time
A
dropped in a copper calorimeter of mass 100 gm, filled with
(a) BC (b) CD (c) ED (d) EF
170 gm of water at room temperature. Subsequently, the
temperature of the system is found to be 75°C. T is given by 32. Assertion : It is hotter over the top of a fire than at the same
(Given : room temperature = 30° C, specific heat of copper distance on the sides.
= 0.1 cal/gm°C [JEE Main 2017, A] Reason : Air surrounding the fire conducts more heat
(a) 1250°C (b) 825°C (c) 800°C (d) 885° C upwards. [AIIMS 2018, C]
29. During vapourisation [BITSAT 2017, C] (a) Assertion is correct, reason is correct; reason is a
I. change of state from liquid to vapour state occurs. correct explanation for assertion.
II. temperature remains constant. (b) Assertion is correct, reason is correct; reason is not a
III. both liquid and vapour states coexist in equilibrium. correct explanation for assertion
IV. specific heat of substance increases. (c) Assertion is correct, reason is incorrect
Correct statements are (d) Assertion is incorrect, reason is correct.
(a) I, II and IV (b) II, III and IV 33. Water of volume 2 litre in a container is heated with a coil of
(c) I, III and IV (d) I , II and III 1 kW at 27°C. The lid of the container is open and energy
30. The power radiated by a black body is P and it radiates dissipates at rate of 160 J/s. In how much time temperature
maximum energy at wavelength, l0. If the temperature of will rise from 27°C to 77°C? [Given specific heat of water is
the black body is now changed so that it radiates maximum 4.2 kJ/kg] [BITSAT 2018, A]
(a) 8 min 20 s (b) 6 min 2 s
3
energy at wavelength l0 , the power radiated by it (c) 7 min (d) 14 min
4
becomes nP. The value of n is [NEET 2018, C]

Exercise 3 : Try If You Can


1. A glass sinker has a mass M in air. When weighed in a liquid The distance between the far ends of the rods is maintained
at temperature t1, the apparent mass is M1 and when weighed at 120.0 cm throughout. The temperature is raised until the
in the same liquid at temperature t2, the apparent mass is two rods are barely in contact. The coefficients of linear
M2. If the coefficient of cubical expansion of the glass is gg, expansion of bras and aluminium are 2.0 × 10-5 k-1 and 2.4 ×
then the real coefficient of expansion of the liquid is : 10-5 k-1, respectively. In figure, the ratio of the increase in
æM –M ö 1 the length of the aluminium rod to that of the brass rod, is
(a) g g + ç M – M ÷ × (t – t )
2 1

è 2 ø 2 1
closest to :
(a) 1.12
æM –M ö 1
(b) g g – ç M – M ÷ × (t – t )
2 1 (b) 1.20
è 2 ø 2 1 (c) 1.29
æ M –M ö 1 (d) 1.03
(c) g g – ç M – M ÷ × (t – t )
2

è 2 1 ø 2 1
3. A sinker of weight W0 has an apparent weight W1 when
weighed in a liquid at a temperature t1 and W2 when weight
æ M – M1 ö 1 in the same liquid at temperature t2. The coefficient of cubical
(d) gg + ç 2 ÷×
M
è 2 + M 1 ø ( t 2 – t1 ) expansion of the material of sinker is b. What is the
2. A brass rod is 64.4 cm long and an aluminium rod is 55.4 cm coefficient of volume expansion of the liquid?
long when both rods are at an initial temperature of 0ºC. The W2 - W1 b(W0 - W1 )
rods are placed in line with a gap of 0.2 cm between them. (a) (W - W )(t - t ) + W - W
0 2 2 1 0 2
EBD_7418
150 PHYSICS

W2 - W0 1/ 4
(b) b æ 97 ö T
t 2 - t1 (c) çè ÷ø (d) (97)1/4T
2
W1 - W0 9. Two vessels, each containing the same mass m of water but
(c) b
t 2 - t1 at different temperatures, are connected by a rod of length
W0 W - W1 L, cross – section A, and thermal conductivity K. The ends
(d) b+ 2 b of the rod are inserted into the vessels, while the rest of the
(t 2 - t1 ) (W0 - W)
rod is insulated so that there is negligible loss of heat into
4. The top of an insulated cylindrical container is covered by the atmosphere. The specific heat capacity of water is s,
a disc having emissivity 0.6 and conductivity 0.167 while that of the rod is negligible. The temperature difference
WK–1m–1 and thickness 1 cm. The temperature is maintained between the two vessels reduces to 1/e of its original value
by circulating oil as shown in figure. Find the radiation loss after a time Dt. The thermal conductivity (K) of the rod may
to the surrounding in Jm–2s–1 if temperature of the upper be expressed by
surface of the disc is 27°C and temperature of the
msL emsL msL msL
surrounding is 27°C. (a) (b) (c) (d)
AD t ADt 2eADt 2 AD t
(a) 595 Jm–2s–1
Oil out 10. A hot body placed in the air is cooled down according to
(b) 545 Jm–2s–1 Newton's law of cooling, the rate of decrease of temperature
(c) 495 Jm–2s–1 being k times the temperature difference from the
Oil in
(d) None of these surrounding. Starting from t = 0, find the time in which the
5. A black coloured solid sphere of radius R and mass M is body will lose half the maximum heat it can lose.
inside a cavity with vacuum inside. The walls of the cavity (a) ln 2/k (b) k /l n 2
are maintained at temperature T0. The initial temperature of
(c) kln 2 (d) None of these
the sphere is 3T0. If the specific heat of the material of the
11. A solid copper cube of edges 1 cm each is suspended in an
sphere varies as aT3 per unit mass with the temperature T
evacuated enclosure. Its temperture is found to fall from
of the sphere, where a is a constant, then the time taken for
100°C to 99°C in 100 s. Another solid copper cube of edges
the sphere to cool down to temperature 2T 0 will be (s is
2 cm, with similar surface nature, is suspended in a similar
Stefan Boltzmann constant)
manner. The time required for this cube to cool from 100°C
Ma æ3ö Ma æ 16 ö to 99°C will be approximately
(a) 2
In ç ÷ (b) 2
In ç ÷
4pR s è 2 ø 4pR s è 3 ø (a) 25 s (b) 50 s (c) 200 s (d) 400 s
Ma æ 16 ö Ma æ3ö 12. A circular hole of diameter 2 cm is made in metal plate at 0°C.
(c) 2
In ç ÷ (d) 2
In ç ÷ What will be the diameter at 100°C? [ametal = 2.3 × 10–5 °C–1]
16pR s è ø 3 16pR s è 2 ø (a) 2.004 cm (b) 2.8012 cm
6. A large cylindrical rod of length L is made by joining two (c) 6.0028 cm (d) 2.1208 cm
13. When a block of iron floats in mercury at 0oC, fraction K1 of
identical rods of copper and steel of length æç ö÷ each. The
L
è 2ø its volume is submerged, while at the temperature 60 oC, a
rods are completely insulated from the surroundings. If the fraction K2 is seen to be submerged. If the coefficient of
free end of copper rod is maintained at 100°C and that of volume expansion of iron is gFe and that of mercury is gHg,
steel at 0°C then the temperature of junction is (Thermal then the ratio K1/K2 can be expressed as
conductivity of copper is 9 times that of steel) 1 + 60g Fe 1 – 60g Fe
(a) 90°C (b) 50°C (c) 10°C (d) 67°C (a) (b)
1 + 60g hg 1 + 60g Hg
7. A piece of ice (heat capacity = 2100 J kg–1 °C–1 and latent heat
= 3.36 × 105 J kg–1) of mass m grams is at –5°C at atmospheric 1 + 60g Fe 1 + 60g He
pressure. It is given 420 J of heat so that the ice starts melting. (c) 1 – 60g Hg (d)
1 + 60g Fe
Finally when the ice-water mixture is in equilibrium, it is found 14. Three rods of same dimensions are arranged as shown in
that 1 gm of ice has melted. Assuming there is no other heat figure they have thermal conductivities K1, K2 and K3 . The
exchange in the process, the value of m is points P and Q are maintained at different temeratures for
(a) 4 (b) 8 (c) 2 (d) 5 the heat to flow at the same rate along PRQ and PQ then
8. Three very large plates of same area are kept parallel and which of the following option is correct?
close to each other. They are considered as ideal black R
1
surfaces and have very high thermal conductivity. The first (a) K3 = ( K1 + K 2 )
and third plates are maintained at temperatures 2T and 3T 2
respectively. The temperature of the middle (i.e. second) (b) K3 = K1 + K2 K K
1 2
plate under steady state condition is K1K 2
1/ 4 1/ 4 (c) K3 =
æ 65 ö æ 97 ö K1 + K2
(a) çè ÷ø T (b) çè ÷ø T P
K3
Q
2 4 (d) K3 = –2(K1 + K2)
THERMAL PROPERTIES OF MATTER 151

15. A cylindrical metallic rod in thermal contact with two (a) 2 and 3 (b) 3 and 1 (c) 1 and 5 (d) 2 and 4
reservoirs of heat at its two ends conducts an amount of 18. A rod of length l and cross-sectional area A has a variable
heat Q in time t. The metallic rod is melted and the material is conductivity given by K = aT, where a is a positive constant
formed into a rod of half the radius of the original rod. What and T is temperature in kelvin. Two ends of the rod are
is the amount of heat conducted by the new rod, when
maintained at temperatures T1 and T2 (T1 > T2). Heat current
placed in thermal contact with the two reservoirs in time t?
Q Q Q flowing through the rod will be
(a) (b) (c) 2 Q (d) Aa (T12 - T22 ) Aa (T12 + T22 )
4 16 2 (a) (b)
16. A long metallic bar is carrying heat from one of its ends to l l
the other end under steady–state. The variation of Aa (T12 + T22 ) Aa (T12 – T22 )
temperature q along the length x of the bar from its hot end (c) (d)
3l 2l
is best described by which of the following figures?
19. A sphere of density r, specific heat capacity c and radius r
is hung by a thermally insulating thread in an enclosure
q q
which is kept at a lower temperature than the sphere. The
temperature of the sphere starts to drop at a rate which
(a) (b) depends upon the temperature difference between the
sphere and the enclosure and the nature of the surface of
x x
sphere and is proportional to
c 1 1
(a) r 3r
(b) r 3rc
(c) 3r3 rc (d)
q q rrc
20. Assuming the sun to have a spherical outer surface of radius
r, radiating like a black body at temperature t°C, the power
(c) (d) received by a unit surface, (normal to the incident rays) at a
x x distance R from the centre of the sun is
s is the Stefan’s constant.
17. A cube, a pyramid (with faces identical) and a sphere (all of r 2s(t + 273)4 16p 2 r 2st 4
them hollow) are made from the same material and have (a) (b)
equal mass and bound equal volume. They are heated to 4pR2 R2
the same temperature and then left to cool. After some time,
r 2s(t + 273)4 4pr 2st 4
1. sphere will have the highest temperature (c) (d)
2. pryamid will have the highest temperature. R2 R2
3. cube will have the lowest temperature.
4. sphere will have the lowest temperature.
5. pyramid will have the lowest temperature
Correct option will be :
EBD_7418
152 PHYSICS

ANSWER KEYS
Exercise 1 : Topic-wise MCQs
1 (a) 12 (b) 23 (b) 34 (d) 45 (d) 56 (a) 67 (a) 78 (c) 89 (c) 100 (b)
2 (c) 13 (c) 24 (a) 35 (c) 46 (c) 57 (c) 68 (a) 79 (d) 90 (c) 101 (a)
3 (a) 14 (c) 25 (b) 36 (b) 47 (c) 58 (a) 69 (c) 80 (d) 91 (c) 102 (c)
4 (a) 15 (c) 26 (b) 37 (c) 48 (b) 59 (c) 70 (d) 81 (c) 92 (b) 103 (d)
5 (a) 16 (c) 27 (b) 38 (c) 49 (b) 60 (a) 71 (a) 82 (b) 93 (b) 104 (b)
6 (c) 17 (d) 28 (a) 39 (c) 50 (d) 61 (c) 72 (d) 83 (d) 94 (b) 105 (a)
7 (c) 18 (a) 29 (c) 40 (b) 51 (a) 62 (d) 73 (d) 84 (d) 95 (a)
8 (c) 19 (a) 30 (b) 41 (a) 52 (a) 63 (c) 74 (a) 85 (c) 96 (b)
9 (c) 20 (b) 31 (b) 42 (a) 53 (c) 64 (a) 75 (a) 86 (d) 97 (c)
10 (a) 21 (c) 32 (d) 43 (b) 54 (a) 65 (b) 76 (a) 87 (d) 98 (a)
11 (a) 22 (c) 33 (c) 44 (d) 55 (a) 66 (c) 77 (c) 88 (c) 99 (c)
Exercise 2 : Exemplar & Past Year MCQs
1 (d) 5 (a) 9 (d) 13 (c) 17 (d) 21 (a) 25 (c) 29 (d) 33 (a)
2 (b) 6 (a) 10 (a) 14 (a) 18 (d) 22 (d) 26 (b) 30 (d)
3 (b) 7 (d) 11 (d) 15 (a) 19 (d) 23 (d) 27 (d) 31 (b)
4 (a) 8 (c) 12 (c) 16 (a) 20 (c) 24 (c) 28 (d) 32 (c)
Exercise 3 : Try If You Can
1 (a) 3 (a) 5 (c) 7 (b) 9 (d) 11 (c) 13 (a) 15 (d) 17 (c) 19 (d)
2 (a) 4 (a) 6 (a) 8 (c) 10 (a) 12 (a) 14 (c) 16 (a) 18 (d) 20 (c)
12Chapter
THERMODYNAMIC

Trend
Analysis of NEET and AIIMS (Year 2010-2018)

4
Number of Questions

3
AIPMT/NEET
2 AIIMS

0
2010 2011 2012 2013 2014 2015 2016 2017 2018
Year

Trend
Analysis of JEE Main and BITSAT (Year 2010-2018)

3
Number of Questions

2
JEE Main

BITSAT

0
2010 2011 2012 2013 2014 2015 2016 2017 2018
Year

Chapter Utility Score (CUS)


Exam Weightage Important Concepts Difficulty Level CUS (Out of 10)
NEET 5 Laws of Thermodynamics,
AIIMS 6 Thermodynamic Processes 4/5 8.5/10
JEE Main 3 and Carnot’s Engine
BITSAT 3
EBD_7418
154 PHYSICS
THERMODYNAMICS 155
EBD_7418
156 PHYSICS

Exercise 1 : Topic-wise MCQs

Topic 1: Zeroth law and First Law of Thermodynamics (c) Assertion is correct, reason is incorrect
(d) Assertion is incorrect, reason is correct.
1. First law of thermodynamics states that 12. Thermodynamics is concerned in part with transformations
(a) system can do work between
(b) system has temperature (a) different forms of heat energy
(c) system has pressure (b) internal energy at various temperatures
(d) heat is a form of energy (c) one form of mechanical energy into other forms
2. A system X is neither in thermal equilibrium with Y nor with Z. (d) heat, internal energy and mechnical work
The systems Y and Z 13. The first law of thermodynamics expresses
(a) must be in thermal equilibrium (a) law of conservation of momentum
(b) cannot be in thermal equilibrium (b) law of conservation of energy
(c) may be in thermal equilibrium (c) law of conservation of mass
(d) None of these (d) All of the above
3. Which of the following macroscopic variable is not 14. In changing the state of thermodynamics from A to B state,
measurable? the heat required is Q and the work done by the system is
(a) Pressure (b) Volume W. The change in its internal energy is
(c) Mass (d) None of these Q -W
(a) Q + W (b) Q – W (c) Q (d)
4. Which of the following is a state variable ? 2
(a) Heat (b) Work 15. Which of the following is incorrect regarding first law of
(c) Internal energy (d) All of these thermodynamics?
5. The internal energy of an ideal gas depends upon (a) It is a restatement of principle of conservation of energy.
(a) specific volume (b) pressure (b) It is applicable to cyclic processes
(c) temperature (d) density (c) It introduces the concept of entropy
6. Energy transfer brought about by moving the piston of a (d) It introduces the concept of internal energy
cylinder containing the gas is known as 16. 110 joules of heat is added to a gaseous system, whose
(a) work (b) heat internal energy is 40J; then the amount of external work
(c) pressure (d) temperature done is
7. The state of a thermodynamic system is represented by (a) 150 J (b) 70 J (c) 110 J (d) 40 J
(a) pressure only 17. The change in internal energy of a thermodynamical system
(b) volume only which has absorbed 2 kcal of heat and done 400 J of work is
(c) pressure, volume and temperature (1 cal = 4.2 J)
(d) number of moles (a) 2 kJ (b) 8 kJ (c) 3.5 kJ (d) 5.5 kJ
8. Which of the following parameters does not characterize 18. Which of the following statements are incorrect?
the thermodynamic state of matter? I. If Q > 0, heat is added to the system.
(a) Temperature (b) Pressure II. If W > 0, work is done by the system.
(c) Work (d) Volume III. If W = 0, work is done by the system.
9. At a given temperature the internal energy of a substance (a) II and III (b) I, II and III
(a) in liquid state is equal to that in gaseous state. (c) I and II (d) I and III
(b) in liquid state is less than that in gaseous state. 19. If the amount of heat given to a system is 35 J and the
(c) in liquid state is more than that in gaseous state. amount of work done on the system is 15 J, then the change
(d) is equal for the three states of matter. in internal energy of the system is
10. The variable defined by Zeroth law of thermodynamics is (a) –50 J (b) 20 J (c) 30 J (d) 50 J
(a) temperature (b) internal energy 20. A gas is compressed from a volume of 2m3 to a volume of
(c) work (d) All of these 1m3 at a constant pressure of 100 N/m 2. Then it is heated at
11. Assertion : Zeroth law of thermodynamics explain the constant volume by supplying 150 J of energy. As a result,
concept of energy. the internal energy of the gas:
Reason : Energy depends on temperature. (a) increases by 250 J (b) decreases by 250 J
(a) Assertion is correct, reason is correct; reason is a (c) increases by 50 J (d) decreases by 50 J
correct explanation for assertion. 21. A perfect gas goes from a state A to another state B by
(b) Assertion is correct, reason is correct; reason is not a absorbing 8 × 105 J of heat and doing 6.5 × 105 J of external
correct explanation for assertion work. It is now transferred between the same two states in
THERMODYNAMICS 157

another process in which it absorbs 105 J of heat. In the 31. Which process will increase the temperature of the
second process system without heating it?
(a) work done by gas is 105 J (a) Adiabatic compression
(b) work done on gas is 105 J (b) Adiabatic expansion
(c) work done by gas is 0.5 × 105 J (c) Isothermal compression
(d) work done on the gas is 0.5 × 105 J (d) Isothermal expansion
32. A cube of side 5 cm made of iron and having a mass of 1500
Topic 2: Specific Heat Capacity and g is heated from 25° C to 400°C. The specific heat for iron is
0.12 cal/g°C and the coefficient of volume expansion is
Thermodynamical Processes
3.5 × 10–5/°C, the change in the internal energy of the cube
22. The specific heat of a gas at constant pressure is greater is (atm pressure = 1 × 105 N/m2)
than the specific heat of the same gas at constant volume (a) 320 kJ (b) 282 kJ (c) 141 kJ (d) 423 kJ
because 33. 4 kg of oxygen gas is heated so as to raise its temperature
(a) work is done in the expansion of the gas at constant from 20 to 120°C. If the heating is done at constant pressure,
pressure. the external work done by the gas is (Cp = 0.219 cal/g°C and
(b) work is done in the expansion of the gas at constant Cv = 0.157 cal/g°C)
volume. (a) 628 kJ (b) 104 kJ (c) 366 kJ (d) 206 kJ
(c) the attraction between the molecules increases at 34. Two kg of water is converted into steam by boiling at
constant pressure. atmospheric pressure. The volume changes from 2 × 10–3m3
(d) the molecular attraction increases at constant volume. to 3.34 m3. The work done by the system is about
23. Which of the following formula is wrong? (a) – 340 kJ (b) –170 kJ (c) 170 kJ (d) 340 kJ
R γR 35. A point on P – V diagram represents
(a) C v = (b) C p = (a) the condition of a system
γ -1 γ -1
(b) work done on or by the system
(c) Cp / Cv = g (d) Cp – Cv = 2R
(c) work done in a cyclic process
24. If CP and CV are specific heat capacities at constant
(d) a thermodynamic process
pressure and constant volume respectively, then for an
36. A sample of gas expands from volume V1 to V2. The amount
adiabatic process of an ideal gas
of work done by the gas is greatest when the expansion is
(a) PV = constant (b) PV–g = constant
(a) isothermal (b) isobaric
P (c) adiabatic (d) equal in all cases
(c) PVg = constant (d) = constant
Vg 37. When heat is given to a gas in an isothermal change, the
25. The specific heat of a gas in an isothermal process is result will be
(a) infinite (b) zero (a) external work done
(c) negative (d) remains constant (b) rise in temperature
26. The work done in an adiabatic change in a particular gas (c) increase in internal energy
depends only upon (d) external work done and also rise in temperature
38. Which of the following statements about a thermodynamic
(a) change in volume
process is wrong ?
(b) change in temperature (a) For an adiabatic process DEint = – W
(c) change in pressure (b) For a constant volume process DEint = + Q
(d) None of these (c) For a cyclic process DEint = 0
27. Which one of the following is an isoentropic process? (d) For free expansion of a gas DEint > 0
(a) Isothermal (b) Adiabatic 39. In a cyclic process, work done by the system is
(c) Isochoric (d) Isobaric (a) zero
28. During isothermal expansion, the slope of P-V graph (b) equal to heat given to the system
(a) decreases (b) increases (c) more than heat given to the system
(c) remains same (d) may increase or decrease (d) independent of heat given to the system
29. Which of the following processes is adiabatic ? 40. The slopes of isothermal and adiabatic curves are related as
(a) Melting of ice (a) isothermal curve slope = adiabatic curve slope
(b) Bursting of tyre (b) isothermal curve slope = g × adiabatic curve slope
(c) Motion of piston of an engine with constant speed (c) adiabatic curve slope = g × isothermal curve slope
(d) None of these 1
(d) adiabatic curve slope = ´ isothermal curve slope
30. Which of the following holds good for an isochoric 2
process? 41. A mass of diatomic gas (g = 1.4) at a pressure of 2
(a) No work is done on the gas atmospheres is compressed adiabatically so that its
(b) No work is done by the gas temperature rises from 27°C to 927°C. The pressure of the
(c) Both (a) and (b) gas in final state is
(d) None of these (a) 28 atm (b) 68.7 atm (c) 256 atm (d) 8 atm
EBD_7418
158 PHYSICS

42. A diatomic gas initially at 18ºC is compressed adiabatically 49. What will be the final pressure if an ideal gas in a cylinder
to one eighth of its original volume. The temperature after
compression will be 1
is compressed adiabatically to rd of its volume?
(a) 18ºC (b) 668.4ºK (c) 395.4ºC (d) 144ºC 3
43. 2 k mol of hydrogen at NTP expands isobarically to twice its (a) Final pressure will be three times less than initial
initial volume. The change in its internal energy is (Cv = 10 pressure.
kJ/kg.K and atm pressure = 1 × 105 N/m2) (b) Final pressure will be three times more than initial
(a) 10.9 MJ (b) 9.10 MJ (c) 109 MJ (d) 1.09 MJ pressure.
44. When a system is taken from state a (c) Change in pressure will be more than three times the
f initial pressure.
i to state f along the path iaf, it is
found that Q =50 cal and W = 20 (d) Change in pressure will be less than three times the
cal. Along the path ibf Q = 36 cal. initial pressure.
W along the path ibf is i b 50. Four curves A, B, C and D are drawn in the figure for a given
amount of a gas. The curves which represent adiabatic and
(a) 14 cal (b) 6 cal (c) 16 cal (d) 66 cal isothermal changes are
45. Choose the incorrect statement related to an isobaric
(a) C and D respectively B
process. P C
(b) D and C respectively A
V D
(a) = constant (c) A and B respectively
T
(b) W = PDV (d) B and A respectively V
(c) Heat given to a system is used up in raising the 51. An ideal gas is initially at P1, V1 is expanded to P2, V2 and
temperature only. then compressed adiabatically to the same volume V1 and
(d) DQ > W pressure P3. If W is the net work done by the gas in complete
46. The P-V diagram of 0.2 mol of a diatomic ideal gas is shown process which of the following is true?
in figure. Process BC is adiabatic, g = 1.4. (a) W > 0 ; P3 > P1 (b) W < 0 ; P3 > P1
P B (c) W > 0 ; P3 < P1 (d) W < 0 ; P3 < P1
52. For an ideal gas graph is shown Work done (magnitude)
for three processes. Process 1, 2
and 3 are respectively.
3
1 atm
A
(a) Isobaric, adiabatic, isochoric 2
455K C
V (b) Adiabatic, isobaric, isochoric
Column I Column II 1
(c) Isochoric, adiabatic, isobaric
(A) DQAB (J) (1) 602
(d) Isochoric, isobaric, adiabatic Temperature change
(B) DWBC (J) (2) – 644
(C) DUCA (J) (3) 1246 53. In the P-V diagram, I is the initial state and F is the final
(D) DUBC (J) (4) – 602 state. The gas goes from I ot F by (i) IAF, (ii) IBF, (iii) ICF.
(a) (A) – (1), (B) – (3), (C) – (4), D – (2) The heat absorbed by the gas is P F
(b) (A) – (3), (B) – (1), (C) – (2), D – (4) (a) the same in all three processes
(c) (A) – (3, 4), (B) – (3), (C) – (2), (D) – (1) (b) the same in (i) and (ii) A B C
(d) (A) – (1), (B) – (2), (C) – (3), (D) – (4) (c) greater in (i) than in (ii)
47. Which of the following statements are correct about (d) the same in (i) and (iii) I
V
isothermal and adiabatic changes?
I. Isothermal system is thermally conducting to the 54. A closed system undergoes a process 1 ® 2 for which the
surroundings. values W1-2 and Q1-2 are 50 kJ and –20 kJ respectively. If
II. Adiabatic system is thermally insulated from the the system is returned to state 1 and Q2®1 is + 10 kJ the
surroundings. work done W2®1 is
III. Internal energy changes in isothermal process. (a) 40 kJ (b) 50 kJ (c) –60 kJ (d) –50 kJ
(a) I and III (b) II and III 55. During an adiabatic process of an ideal gas, if P is
(c) I and II (d) I, II and III 1
proportional to , then the ratio of specific heat
48. A thermodynamic system undergoes cyclic process ABCDA V1.5
as shown in fig. The work done by the system in the cycle is capacities at constant pressure to that at constant volume
for the gas is
(a) P0V0 P (a) 1.5 (b) 0.25 (c) 0.75 (d) 0.4
C B
3P0
(b) 2P0V0 56. The relation between U, P and V for an ideal gas in an
2P0
adiabatic process is given by relation U = a + bP V. Find the
P0 V0 P0
(c) A D value of adiabatic exponent (g) of this gas.
2 b +1 b +1 a +1 a
(d) Zero V0 2V0 V (a) (b) (c) (d)
b a b a+b
THERMODYNAMICS 159

57. During an adiabatic process an object does 100J of work P P Adiabatic


and its temperature decreases by 5K. During another
process it does 25J of work and its temperature decreases

Adiabatic

Isothermal
Isothermal
Adiabatic
by 5K. Its heat capacity for 2nd process is
(a) 20 J/K (b) 24 J/K (c) 15 J/K (d) 100 J/K (c) (d)
58. Heat energy absorbed by a system in going through a cyclic Adiabatic
process shown in the given figure is
V V

30
64. One mole of an ideal gas at an initial temperature of T K
does 6R joules of work adiabatically. If the ratio of specific
heats of this gas at constant pressure and at constant
volume is 5/3, the final temperature of gas will be
V (in litre)

(a) (T – 4) K (b) (T + 2.4) K


10
(c) (T – 2.4) K (d) (T + 4) K
10 30 65. There are two processes ABC and DEF. In which of the
P(KPa) process is the amount of work done by the gas greater?
(a) 107 p J (b) 104 p J (c) 102 p J (d) 10–3 p J
59. The pressure inside a tyre is 4 times that of atmosphere. If the 24 C
tyre bursts suddenly at temperature 300 K, what will be the 21
new temperature? B E
P 18 F
(a) 300 (4)7/2 (b) 300 (4)2/7 (c) 300 (2)7/2 (d) 300 (4)–2/7
60. In pressure-volume diagram, the isochoric, isothermal, 15 D
isobaric and iso-entropic parts respectively, are 12 A

(a) BA, AD, DC,CB A B

6 9 12 15
(b) DC, CB, BA, AD C V
P (a) ABC (b) DEF
(c) AB, BC, CD, DA D
(c) Equal in both processes (d) It cannot be predicted
(d) CD, DA, AB, BC V 66. In a thermodynamic process, pressure of a fixed mass of gas
61. We consider a thermodynamic system. If DU represents the is changed in such a manner that the gas molecules gives
increase in its internal energy and W the work done by the out 30 joules of heat and 10 J of work is done on the gas. If
system, which of the following statements is true? the initial internal energy of the gas was 40J, then final
(a) DU = – W in an adiabatic process
internal energy will be
(b) DU = W in an isothermal process
(c) DU = – W in an isothermal process (a) zero (b) 80 J (c) 20 J (d) –20 J
(d) DU = W in an adiabatic process 67. Calculate the work done when 1 mole of a perfect gas is
62. Assertion : When a bottle of cold carbonated drink is compressed adiabatically. The initial pressure and volume
opened a slight fog forms around the opening. of the gas are 105 N/m2 and 6 litre respectively. The final
Reason : Adiabatic expansion of the gas causes lowering of volume of the gas is 2 litres. Molar specific heat of the gas at
temperature and condensation of water vapours. constant volume is 3R/2. [Given (3)5/3 = 6.19]
(a) Assertion is correct, reason is correct; reason is a (a) –957 J (b) +957 J (c) – 805 J (d) + 805 J
correct explanation for assertion. 68. A one mole sample of an ideal gas is carried around the
(b) Assertion is correct, reason is correct; reason is not a thermodynamic cycle shown in the figure. The cycle consists
correct explanation for assertion of three steps : (i) an isothermal expansion (a ® b), (ii) an
(c) Assertion is correct, reason is incorrect isobaric compression (b ® c), and (iii) a constant volume
(d) Assertion is incorrect, reason is correct. increases in pressure (c ® a). If Ta = 400 K, Pa = 4 atm and Pb
63. Which of the following is the P-V curve for isothermal = Pc = 1 atm, the work done by the gas per cycle is
and adiabatic process of an ideal gas? P
P P
Isothermal a
Adiabatic Pa
T = 400 K
Isothermal
Isothermal

Adiabatic

Adiabatic

(a) (b) b
P=
b
Pc c
Isothermal V
Va= Vc Vb
V V (a) 211.6 J (b) 1158 J (c) 705.3 J (d) 2116 J
EBD_7418
160 PHYSICS

69. The pressure-volume diagram P 74. A diatomic ideal gas undergoes a thermodynamic change
shows six curved paths that can a according to the P-V diagram shown in the figure. The total
be followed by the gas (connected b heat given to the gas is nearly (use ln 2 = 0.7)
by vertical paths). Which two of c P
them should be part of a closed d
cycle if the net work done by the e B
2P0
gas is to be its maximum positive f isothermal
value? V
A
(a) af (b) ae (c) ac (d) ce P0 C
70. A thermodynamic system is P
taken from state A to B along B
ACB and is brought back to A P2 C
along BDA as shown in the PV V
V0 2V0
diagram. The net work done P1 D
during the complete cycle is A (a) 2.5 P0V0 (b) 1.4 P0V0
given by the area A¢ B¢ V (c) 1.1 P0V0 (d) 3.9 P0V0
(a) P1ACBP2P1 (b) ACBB'A'A 75. In P-V diagram shown in figure ABC is a semicircle. The
(c) ACBDA (d) ADBB'A'A work done in the process ABC is 2
P(N/m )
71. Unit mass of a liquid with volume V1 is completely changed (a) 4 J
into a gas of volume V2 at a constant external pressure P 3 A
-p
and temperature T. If the latent heat of evaporation for the (b) J B
given mass is L, then the increase in the internal energy of 2
p 1
the system is (c) J C
2
(a) Zero (b) P(V2 – V1) 0 V(m )
3
(d) zero 1 2
(c) L – P(V2 – V1) (d) L
72. A gas undergoes a process according to the graph. P is 76. On P-V coordinates, the slope of an isothermal curve of a
pressure, V is volume, W is work done by the gas, DU is gas at a pressure P = 1MPa and volume V = 0.0025 m3 is
change in internal energy of the gas and DQ is heat given to equal to –400 MPa/m3. If Cp / Cv = 1.4, the slope of the
the system. Matchf the two columns. adiabatic curve passing through this point is :
(a) –56 MPa/m3 (b) –400 MPa/m3
P 3
(c) –560 MPa/m (d) None of these
C B 77. The P-V diagram of a gas A D
2
system undergoing cyclic 2 × 10 –2
process is shown here. The P(N m 2) B C
10
work done during isobaric
compression is

D A (a) 100 J (b) 200 J


0 1 2 3
1/V (c) 600 J (d) 400 J V(m–3)
Column I Column II 5
(A) For process AB (1) DU > 0, DQ > 0 78. 1 gm of water at a pressure of 1.01 × 10 Pa is converted into
(B) For process BC (2) DU < 0, DQ < 0 steam without any change of temperature. The volume of 1
(C) For process CD (3) DQ × DU × W = 0 g of steam is 1671 cc and the latent heat of evaporation is
(D) For process DA (4) DQ × DU < 0 540 cal. The change in internal energy due to evaporation of
(a) (A) – (1), (B) – (2), (C) – (4), D – (3) 1 gm of water is
(b) (A) – (3), (B) – (1), (C) – (2), D – (2) (a) »167 cal (b) » 500 cal (c) 540 cal (d) 581 cal
(c) (A) – (2), (B) – (1), (C) – (4), (D) – (3) 79. A certain amount of ideal monoatomic gas undergoes,
(d) (A) – (4), (B) – (3), (C) – (2), (D) – (1) process given by UV1/2 = C where U is the internal energy
73. In the following P-V diagram two P of the gas. The molar specific heat of the gas for the process
adiabatics cut two isothermals at will be
a (a) R/2 (b) 3R (c) 5R/2 (d) –R/2
temperatures T1 and T2 (fig). The b
T1 80. An ideal gas goes thr ough a V
Va c
value of will be reversible cycle a®b®c®d has b
Vd d c the V - T diagram shown below.
Vb Vc T2
(a) (b) Process d®a and b®c are d
Vc Vb a
adiabatic. T
Vd V
(c) (d) Vb Vc Va Vd Vb Vc The corresponding P - V diagram for the process is
Va
THERMODYNAMICS 161

P P (c) isothermal compression


d c a b (d) adiabatic compression
(a) (b) 86. “Heat cannot by itself flow from a body at lower temperature
a b d c
to a body at higher temperature” is a statement or
consequence of
V V
P P (a) second law of thermodynamics
d c a b (b) conservation of momentum
(c) conservation of mass
(c) a b (d) d c (d) first law of thermodynamics
87. Air conditioner is based on the principle of
V V (a) Carnot cycle
81. A system changes from the state (P1, V1) to (P2, V2) as shown (b) refrigerator
in the figure. What is the work done by the system? (c) first law of thermodynamics
(d) None of these
6×10 5 (P2, V2) 88. Which of the following processes is reversible?
5
Pressure in N/m2

5×10
4×10
5 (a) Transfer of heat by conduction
5
3×10 (b) Transfer of heat by radiation
5
2×10 (c) Isothermal compression
5
1×10 (P1, V1) (d) Electrical heating of a nichrome wire
89. In a reversible cyclic process of a gaseous system
(a) DQ = DU (b) DU = DW
1 2 3 4 5 (c) DW = 0 (d) DU = 0
Volume in metre 3 90. The correct relation between coefficient of performance
(a) 7.5 × 105 joule (b) 7.5 × 105 erg and efficiency of refrigerator is
5
(c) 12 × 10 joule (d) 6 × 105 joule 1+ h 1- h
82. When the state of a gas adiabatically changed from an (a) b = (b) b =
equilibrium state A to another equilibrium state B an amount h h
of work done on the stystem is 35 J. If the gas is taken from (c) b = 1 + h (d) None of these
state A to B via process in which the net heat absorbed by 91. Which of the following processes is irreversible?
the system is 12 cal, then the net work done by the system (a) Transfer of heat by radiation
is (1 cal = 4.19 J) (b) Adiabatic changes performed slowly
(a) 13.2 J (b) 15.4 J (c) 12.6 J (d) 16.8 J (c) Extremely slow extension of a spring
83. Carbon monoxide is carried around a closed cycle abc in (d) Isothermal changes performed slowly
which bc is an isothermal process as shown in the figure . 92. Heat engine is a device by which a system is made to
The gas absorbs 7000 J of heat as its temperture increases undergo a ...X... process that result in conversion of ...Y...
from 300 K to 1000 K in going from a to b. The quantity of into work.
heat rejected by the gas during the process ca is
Here, X and Y refer to
(a) 4200 J P (a) isothermal and heat (b) cyclic and heat
P2 b (c) cyclic and work (d) adiabatic and heat
(b) 5000 J 93. Which of the following statements is incorrect?
(a) All reversible cycles have same efficiency
(c) 9000 J P1 a c (b) Reversible cycle has more efficiency than an irreversible
O V one
(d) 9800 J V1 V2 (c) Carnot cycle is a reversible one
84. Two cylinders A and B fitted with pistons contain equal (d) Carnot cycle has the maximum efficiency in all cycles
amounts of an ideal diatomic gas at 300 K. The piston of A is 94. A refrigerator is a
free to move while that of B is held fixed. The same amount (a) heat engine
of heat is given to the gas in each cylinder. If the rise in
(b) an electric motor
temperature of the gas in A is 30 K, then the rise in
temperature of the gas in B is (c) heat engine working in backward direction
(a) 30 K (b) 18 K (c) 50 K (d) 42 K (d) air cooler
95. The coefficient of performance of a refrigerator is given
Topic 3: Carnot Engine, Refrigerator and by
Second Law of Thermodynamics q2 q1 q1 - q2 q1 - q2
(a) (b) (c) (d)
85. The first operation involved in a Carnot cycle is q1 - q2 q1 - q2 q 2 q1
(a) isothermal expansion 96. A Carnot engine works between a source and a sink
(b) adiabatic expansion maintained at constant temperatures T 1 and T 2 . For
efficiency to be the greatest
EBD_7418
162 PHYSICS

(a) T1 and T2 should be high (a) 4.2 × 106 J (b) 8.4 × 106 J
(b) T1 and T2 should be low (c) 16.8 × 106 J (d) zero
(c) T1 should be low and T2 should be high 106. A diatomic ideal gas is used in a car engine as the working
(d) T1 should be high and T2 should be low substance. If during the adiabatic expansion part of the cycle,
97. The efficiency of a Carnot engine operating between the volume of the gas increases from V to 32 V, the efficiency of
temperatures of 100ºC and –23ºC will be the engine is
100 - 23 (a) 0.5 (b) 0.75 (c) 0.99 (d) 0.25
(a) 100 + 23 (b) 107. A Carnot engine operating between temperatures T1 and T2
100 100
373 - 250 1
(c) 373 + 250 (d) has efficiency .When T2 is lowered by 62 K its efficiency
6
373 373
1
98. The temperature of source and sink of a heat engine are increases to . Then T1 and T2 are, respectively
127ºC and 27ºC respectively. An inventor claims its efficiency 3
to be 26%, then: (a) 372 K and 330 K (b) 330 K and 268 K
(a) it is impossible (c) 310 K and 248 K (d) 372 K and 310 K
(b) it is possible with high probability 108. If the co-efficient of performance of a refrigerator is 5 and
(c) it is possible with low probability operates at the room temperature 27°C, the temperature
(d) data are insufficient. inside the refrigerator is
99. A steam engine takes 0.1 kg of steam at 100°C per minute (a) 240 K (b) 250 K (c) 230 K (d) 260 K
and cools it down to 20°C. What is the heat rejected by the 109. The temperature-entropy diagram of a reversible engine
steam engine per second? [Latent heat of vaporization of cycle is given in the figure. Its efficiency is
steam = 540 cal g–1] T
(a) 6.2 × 104 cal/sec 1 1
(a) (b) 2T0
(b) 1.03 × 103 cal/sec 4 2
(c) 62 × 103 cal/sec T0
(d) None of these 2 1
(c) (d) S
100. Even Carnot engine cannot give 100% efficiency because 3 3 S0 2S0
we cannot
110. A Carnot engine whose efficiency is 50% has an exhaust
(a) prevent radiation
(b) find ideal sources temperature of 500 K. If the efficiency is to be 60% with the
(c) reach absolute zero temperature same intake temperature, the exhaust temperature must be
(d) eliminate friction (in K)
101. In all natural processes, the entropy of the universe (a) 800 (b) 200 (c) 400 (d) 600
(a) remains constant 111. Assertion : When a glass of hot milk is placed in a room and
(b) always decreases allowed to cool, its entropy decreases
(c) always increases Reason : Allowing hot object to cool does not violate the
(d) may increase or decrease second law of thermodynamics.
102. If an air conditioner is put in the middle of a room and (a) Assertion is correct, reason is correct; reason is a
started working correct explanation for assertion.
(a) the room can be cooled slightly (b) Assertion is correct, reason is correct; reason is not a
(b) the temperature of the room will not change correct explanation for assertion
(c) the room will become slightly warmer (c) Assertion is correct, reason is incorrect
(d) the same temperature will be attained in the room as (d) Assertion is incorrect, reason is correct.
by putting it on the window in the standard position 112. A Carnot engine is working between 127°C and 27°C. The
103. If the temperatures of source and sink of a Carnot engine increase in efficiency will be maximum when the temperature of
having efficiency h are each decreased by 100 K, then the (a) the source is increased by 50°C
efficiency (b) the sink is decreased by 50°C
(a) remains constant (b) becomes 1 (c) source is increased by 25°C and that of sink is
(c) decreases (d) increases decreased by 25°C
1 (d) both source and sink are decreased by 25°C each.
104. A refrigerator with coefficient of performance releases 113. When 1 kg of ice at 0°C melts to water at 0°C, the
3
200 J of heat to a hot reservoir. Then the work done on resulting change in its entropy, taking latent heat of ice to
the working substance is be 80 cal/°C, is
100 200 (a) 273 cal/K (b) 253 cal/K
(a) J (b) 100 J (c) J (d) 150 J
3 3 (c) 263 cal/K (d) 293 cal/K
105. A Carnot engine takes 3 × 106 cal. of heat from a reservoir at 114. If the energy input to a Carnot engine is thrice the work it
627°C, and gives it to a sink at 27°C. The work done by the performs then, the fraction of energy rejected to the sink is
engine is
THERMODYNAMICS 163

1 1 2 2 when it transfers heat from r eservoir to sink is


(a)
3
(b) (c)
5
(d)
3 12.6 × 106J, the quantity of heat absorbed by the engine
4
115. A Carnot engine operating between temperatures T 1 and from the reservoir is
T2 has efficiency 0.2. When T2 is reduced by 50 K, its (a) 16.8 × 106 J (b) 4 × 106 J
6
(c) 7.6 × 10 J (d) 4.2 × 106 J
efficiency increases to 0.4. Then T1 and T2 are respectively
(a) 200 K, 150 K (b) 250 K, 200 K 121. A body of mass m with specific heat C at temperature Th
(c) 300 K, 250 K (d) 300 K, 200 K is brought into thermal equilibrium by contact with an
116. Assertion : The efficiency of a reversible engine is identical body which was initially at a lower temperature
maximum. Tt. No heat is lost to the surroundings and the
Reason : In such a device no dissipation of energy takes temperatures are in kelvin. The change in entropy of the
place. system is
(a) Assertion is correct, reason is correct; reason is a 2 2
mC ln
( T h + Ti )
mC ln
( T h - Ti )
correct explanation for assertion. (a) (b)
4Ti Th 4Ti Th
(b) Assertion is correct, reason is correct; reason is not a
correct explanation for assertion Th
(c) mC ln (d) None of these
(c) Assertion is correct, reason is incorrect Ti
(d) Assertion is incorrect, reason is correct. 122. A Carnot engine whose low temperature reservoir is at 7°C
117. Two Carnot engines A and B are operated in series. The has an efficiency of 50%. It is desired to increase the
engine A receives heat from the source at temperature T1 efficiency to 70%. By how many degrees should the
and rejects the heat to the sink at temperature T. The second temperature of the high temperature reservoir be increased?
engine B receives the heat at temperature T and rejects to its (a) 840 K (b) 280 K (c) 560 K (d) 380 K
sink at temperature T2. For what value of T the efficiencies 123. A body of mass m with specific heat C at temperature 500
of the two engines are equal? K is brought into contact with an identical body at
T1 + T2 T -T temperature 100 K. The system is isolated from the
(a) (b) 1 2 (c) T1T2 (d) T1T2 surroundings during the process. The change in entropy
2 2
118. A Carnot engine absorbs 1000 J of heat energy from a of the system is
reservoir at 127°C and rejects 600 J of heat energy during (a) mC ln 5 (b) mC ln (9/5)
each cycle. The efficiency of engine and temperature of (c) mC ln 3 (d) mC ln (5/3)
sink will be: 124. Two bodies each of mass M and temperature
(a) 20% and – 43°C (b) 40% and – 33°C independent specific heat C are initially at temperatures
(c) 50% and – 20°C (d) 70% and – 10°C 100 K and 400 K. A reversible engine is used to extract
119. The efficiency of an ideal gas with adiabatic exponent 'g' for heat with the hotter body as the source and cooler body
as the sink. Both the bodies attain the same final
the shown cyclic process would be
temperature. Then,
( 2 ln 2 - 1) (a) the final temperature of the two bodies is 250K.
(a) v
g / (g - 1) (b) the work done by the engine is 500 MC
2V0 C
(1 - 2 ln 2) (c) total change in entropy of the system is zero.
(b) g /( g - 1) (d) the total heat extracted from the hotter body is 150 MC
V0 125. A monoatomic ideal gas goes through a process p = p0 – aV
( 2 ln 2 + 1) A
(c) B where p0 and a are positive constants and V is its volume.
g /( g - 1) At what volume will the entropy of gas be maximum?
T0 2T0 T
( 2 ln 2 - 1)
(d) 5 p0 p0 p0 5 p0
g /( g + 1) (a) (b) (c) (d)
120. In a Carnot engine, the temperature of reservoir is 927°C 6a 2a 4a 8a
and that of sink is 27°C. If the work done by the engine

Exercise 2 : Exemplar & Past Year MCQs


NCERT Exemplar MCQs (a) 4 (b) 3 (c) 2 (d) 1
1. An ideal gas undergoes four different 2. If an average person jogs, he produces 14.5 × 103 cal/min.
processes from the same initial state 4 This is removed by the evaporation of sweat. The amount
(figure). Four processes are adiabatic, p 3 of sweat evaporated per minute (assuming 1 kg requires
isothermal, isobaric and isochoric. 2 580 × 103 cal for evaporation) is
Out of 1, 2, 3 and 4 which one is 1
adiabatic? (a) 0.025 kg (b) 2.25 kg (c) 0.05 kg (d) 0.20 kg
V
EBD_7418
164 PHYSICS

3. Consider p-V diagram for an ideal gas p 1 8. A system goes from A to B via two processes I and II as
shown in figure. shown in figure. If DU1 and DU2 are the changes in internal
constant
Out of the following diagrams, which p=
V
energies in the processes I and II respectively, then
figure represents the T-p diagram? [AIIMS , 2014, C]
T T 2
2 2 V p II
(i) (ii) A B
1 1 I
p p
T T v
(a) relation between DU1 and DU2 can not be determined
2 1 1 2
(b) DU1 = DU2
(iii) (iv) (c) DU1 < DU2
(d) DU1 > DU2
p p 9. Assertion : Adiabatic expansion is always accompanied by
(a) (iv) (b) (ii) (c) (iii) (d) (i) fall in temperature.
4. An ideal gas undergoes cyclic p Reason : In adiabatic process, volume is inversely
D C
process ABCDA as shown in 2p0 proportional to temperature. [AIIMS, 2014, C]
given p-V diagram. Th e (a) Assertion is correct, reason is correct; reason is a
p0 B
amount of work done by the A correct explanation for assertion.
gas is (b) Assertion is correct, reason is correct; reason is not a
V
V0 3V0 correct explanation for assertion
(a) 6p0V0 (b) –2p0 V0 (c) +2p0V0 (d) +4p0V0 (c) Assertion is correct, reason is incorrect
5. Consider two containers A and B containing identical gases (d) Assertion is incorrect, reason is correct.
at the same pressure, volume and temperature. The gas in 10. One mole of a diatomic ideal gas
container A is compressed to half of its original volume undergoes a cyclic process ABC as
isothermally while the gas in container B is compressed to shown in figure. The process BC is B
800 K
half of its original value adiabatically. The ratio of final adiabatic. The temperatures at A, B
pressure of gas in B to that of gas in A is and C are 400 K, 800 K and 600 K P
g-1 2 2 respectively. Choose the correct
æ1ö æ 1 ö æ 1 ö
(a) 2 g-1(b) ç ÷ (c) ç ÷ (d) ç ÷ statement: [JEE Main 2014, A] 600 k
è2ø è 1 - g ø è g -1 ø A
400 K
C
(a) The change in internal energy
6. Three copper blocks of masses M 1 , M2 and M 3 kg
in whole cyclic process is V
respectively are brought into thermal contact till they reach
250 R.
equilibrium. Before contact, they were at T1, T2, T3 (T1 > T2
> T3). Assuming there is no heat loss to the surroundings, (b) The change in internal energy in the process CA is 700 R.
the equilibrium temperature T is (s is specific heat of copper) (c) The change in internal energy in the process AB is -350 R.
(d) The change in internal energy in the process BC is – 500 R.
T +T +T
(a) T = 1 2 3 11. One mole of O2 gas having a volume equal to 22.4 Litres at
3 0°C and 1 atmospheric pressure in compressed isothermally
M1T1 + M 2T2 + M 3T3 so that its volume reduces to 11.2 litres. The work done in
(b) T =
M1 + M 2 + M 3 this process is- [BITSAT 2014, A]
M 1T1 + M 2T2 + M 3T3 (a) 1672.5 J (b) 1728 J
(c) T= (c) –1728 J (d) –1572.5 J
3( M1 + M 2 + M 3 )
12. An ideal gas is compressed to half its initial volume by
M 1T1 s + M 2T2 s + M 3T3 s
(d) T = means of several processes. Which of the process results
M1 + M 2 + M 3 in the maximum work done on the gas? [AIPMT 2015, C]
Past Year MCQs (a) Isobaric (b) Isochoric
(c) Isothermal (d) Adiabatic
7. A monoatomic gas at a pressure P, having a volume V expands 13. The coefficient of performance of a refrigerator is 5. If the
isothermally to a volume 2V and then adiabatically to a volume inside temperature of freezer is -20°C, then the temperature of
5 the surroundings to which it rejects heat is [AIPMT 2015, A]
16V. The final pressure of the gas is : (take g = )
3 (a) 41°C (b) 11°C (c) 21°C (d) 31°C
[AIPMT 2014, C]
P 14. Consider a spherical shell of radius R at temperature T. The
(a) 64P (b) 32P (c) (d) 16P black body radiation inside it can be considered as an ideal
64 U
gas of photons with internal energy per unit volume u =
V
THERMODYNAMICS 165

1æUö P
µ T4 and pressure p = ç ÷ . If the shell now undergoes 9P0 V0 9P0 V0
3è V ø (a) (b) A
2nR nR 2P0
an adiabatic expansion the relation between T and R is :
9P0 V0 3P0 V0 P0 B
1 1 (c) (d)
(a) T µ (b) T µ 3 [JEE Main 2015, S] 4nR 2nR
R R V0 2V0 V
(c) T µ e–R (d) T µ e–3R 20. Thermodynamic processes are indicated in the following
15. 0.5 mole of an ideal gas at diagram : [NEET 2017, C]
constant temperature 27°C Match the following
P
kept inside a cylinder of length L Column I Column II
IV
L and cross-section area A P. Process I A. Adiabatic i f
I III
closed by a massless piston. L Q. Process II B. Isobaric f 700
The cylinder is attached with a conducting rod of length L , R. Process III C. Isochoric f II 500 k
k
S. Process IV D. Isothermal f 300k
cross-section area (1/9) m2 and thermal conductivity k, whose V
other end is maintained at 0°C. If piston is moved such that (a) P ® C, Q ® A, R ® D, S ® B
rate of heat flow through the conducing rod is constant (b) P ® C, Q ® D, R ® B, S ® A
then velocity of piston when it is at height L/2 from the (c) P ® D, Q ® B, R ® A, S ® C
bottom of cylinder is : [BITSAT 2015, S] (d) P ® A, Q ® C, R ® D, S ® B
[Neglect any kind of heat loss from system ] 1
21. A carnot engine having an efficiency of as heat engine,
æ kö æ k ö 10
(a) çè ÷ø m / sec (b) çè ÷ m / sec is used as a refrigerator. If the work done on the system is
R 10R ø 10 J, the amount of energy absorbed from the reservoir at
æ k ö æ k ö lower temperature is : [NEET 2017, A]
(c) çè ÷ m / sec (d) çè ÷ m / sec
100R ø 1000R ø (a) 90 J (b) 99 J (c) 100 J (d) 1 J
16. A gas is compressed isothermally to half its initial volume. 22. The internal energy change in a system that has absorbed
The same gas is compressed separately through an adiabatic 2 kcals of heat and done 500 J of work is [AIIMS 2017, A]
process until its volume is again reduced to half. Then : (a) 6400 J (b) 5400 J (c) 7900 J (d) 8900 J
[NEET 2016, C] 23. In a heat engine, the temperature of the source and sink are
(a) Compressing the gas isothermally will require more work 500 K and 375 K. If the engine consumes 25 × 105 J per
to be done. cycle, the work done per cycle is [AIIMS 2017, A]
(b) Compressing the gas through adiabatic process will (a) 6.25 × 105 J (b) 3 × 105 J
require more work to be done. (c) 2.19 × 105J (d) 4 × 104 J
(c) Compressing the gas isothermally or adiabatically will 24. Assertion : The heat supplied to a system is always equal
require the same amount of work. to the increase in its internal energy.
(d) Which of the case (whether compression through Reason : When a system changes from one thermal
isothermal or through adiabatic process) requires more equilibrium to another, some heat is absorbed by it.
work will depend upon the atomicity of the gas. [AIIMS 2017, C]
17. A refrigerator works between 4°C and 30°C. It is required to (a) Assertion is correct, reason is correct; reason is a
remove 600 calories of heat every second in order to keep correct explanation for assertion.
the temperature of the refrigerated space constant. The (b) Assertion is correct, reason is correct; reason is not a
power required is: (Take 1 cal = 4.2 joules) [NEET 2016, A] correct explanation for assertion
(a) 2.365 W (b) 23.65 W (c) 236.5 W (d) 2365 W (c) Assertion is correct, reason is incorrect
(d) Assertion is incorrect, reason is correct.
18. Figure below shows two paths
P 25. One mole of an ideal gas is taken P
that may be taken by a gas to 6×104 Pa B C
from state A to state B by three B
go from a state A to a state C.
different processes,
In process AB, 400 J of heat is
(i) ACB (ii) ADB (iii) AEB as shown
added to the system and in 2×104 Pa C D E
A in the P-V diagram. The heat
process BC, 100 J of heat is
absorbed by the gas is
added to the system. The heat A

2 × 10 3 m3 4 × 10–3 m3 (a) greater in process (ii) than in (i) V
absorbed by the system in the V
(b) the least in process (ii) [BITSAT 2017, C]
process AC will be [AIIMS 2016, A]
(c) the same in (i) and (iii)
(a) 500 J (b) 460 J (c) 300 J (d) 380 J
(d) less in (iii) than in (ii)
19. 'n' moles of an ideal gas undergoes a process A ® B as
26. A cyclic process ABCD is shown in the figure P-V diagram.
shown in the figure. The maximum temperature of the gas
Which of the following curves represent the same process
during the process will be : [JEE MAIN 2016, C]
[BITSAT 2017, C]
EBD_7418
166 PHYSICS

P A B
30. A diatomic gas which has initial volume of 10 liter is
isothermally compressed to 1/15th of its original volume
C where initial pressure is 105 Pascal. If temperature is 27°C
D them find the work done by gas. [AIIMS 2018, A]
V (a) –2.70 × 103 J (b) 2.70 × 103 J
P P
A B (c) –1.35 × 103 J (d) 1.35 × 103 J
A B 3
31. If 1 cm of water is vaporized (latent heat of vaporization = 5.40
(a) C (b) C cal/g°C) at P = 1 atm. If the volume of steam formed is 1671 cm3
D D calculate increase internal energy. [AIIMS 2018, A]
T T (a) 373 cal (b) 473 cal (c) 573 cal (d) 673 cal
P
B P 32. Assertion: Energy of an isolated particles system is
A A B constant.
(c) (d) Reasion : Isolated system do not allow exchange of energy.
D C D C [AIIMS 2018, C]
T T (a) Assertion is correct, reason is correct; reason is a
27. A sample of 0.1 g of water at 100°C and normal pressure correct explanation for assertion.
(1.013 × 105 Nm–2) requires 54 cal of heat energy to convert (b) Assertion is correct, reason is correct; reason is not
to steam at 100°C. If the volume of the steam produced is
167.1 cc, the change in internal energy of the sample, is a correct explanation for assertion
[NEET 2018, A] (c) Assertion is correct, reason is incorrect
(a) 104.3 J (b) 208.7 J (c) 84.5 J (d) 42.2 J (d) Assertion is incorrect, reason is correct.
28. The volume (V) of a monatomic gas V 33. In the following P-V diagram
varies with its temperature (T), as of an ideal gas, two adiabates
shown in the graph. The ratio of work B cut two isotherms at T1 =
done by the gas, to the heat absorbed A
300K and T2 = 200K. The
by it, when it undergoes a change from value of VA = 2 unit, VB = 8
O T
state A to state B, is
unit, VC = 16 unit. Find the
[NEET 2018, C]
value of VD.
2 2 2 1 (a) 4 unit [BITSAT 2018, C]
(a) (b) (c) (d)
5 3 7 3 (b) < 4 unit
29. The efficiency of an ideal heat engine working between the (c) > 5 unit
freezing point and boiling point of water, is [NEET 2018, A] (d) 5 unit
(a) 26.8% (b) 20% (c) 12.5% (d) 6.25%

Exercise 3 : Try If You Can


1. An ideal gas has temperature T1 3. Figure shows the variation of internal energy (U) with the
at the initial state i shown in the P
pressure (P) of 2.0 mole gas in cyclic process abcda. The
T2
P-V diagram. The gas has a higher a temperature of gas at c and d are 300 K and 500 K. Calculate
temperature T2 at the final states the heat absorbed by the gas during the process.
a and b, which it can reach the T1 U
paths shown. The change in T2 (a) 400 R ln 2 a d
i b
entropy: V
(a) greatest in a (b) greatest in b (b) 200 R ln 2
(c) same in a and b (d) nothing can be said b c
2. A monatomic ideal gas, initially at temperature T1, is enclosed (c) 100 R ln 2
in a cylinder fitted with a frictionless piston. The gas is P
(d) 300 R ln 2 P0 2P0
allowed to expand adiabatically to a temperature T2 by
releasing the piston suddenly. If L1 and L2 are the length of 4. One mole of an ideal gas at temperature T was cooled
the gas column before and after expansion respectively, then P
T1 isochorically till the gas pressure fell from P to
. Then, by
n
T2 is given by an isobaric process, the gas was restored to the initial
2/3 2/3 temperature. The net amount of heat absorbed by the gas in
æ L1 ö L1 L2 æ L2 ö
(a) çè L ÷ø (b) L (c) (d) ç ÷ the process is
2 2 L1 èL ø
1
THERMODYNAMICS 167

10. A Carnot freezer takes heat from water at 0°C inside it and
RT
(a) nRT (b) rejects it to the room at a temperature of 27°C. The latent
n heat of ice is 336 × 103 J kg–1. If 5 kg of water at 0°C is
(c) RT (1 – n–1) (d) RT (n – 1) converted into ice at 0°C by the freezer, then the energy
5. An ideal gas goes from state A to consumed by the freezer is close to :
state B via three different processes A
P
1 (a) 1.51 × 105 J (b) 1.68 × 106 J
as indicated in the P-V diagram : 2
7
If Q1, Q2, Q3 indicate the heat a B (c) 1.71 × 10 J (d) 1.67 × 105 J
3
absorbed by the gas along the three 11. The pressure of an ideal gas varies with volume as P = aV,
processes and DU1, DU2, where a is a constant. One mole of the gas is allowed to
V
undergo expansion such that its volume becomes ‘m’ times
DU3 indicate the change in internal energy along the three its initial volume. The work done by the gas in the process
processes respectively, then is
(a) Q1 > Q2 > Q3 and DU1 = DU2 = DU3
(b) Q3 > Q2 > Q1 and DU1= DU2 = DU3
(a)
aV 2
2 (
m -1 ) (b)
a 2V 2 2
2
(
m -1 )
(c) Q1 = Q2 = Q3 and DU1 > DU2 > DU3
(d) Q3 > Q2 > Q1 and DU1> DU2 > DU3
(c)
2
(
a 2
m -1 ) (d)
2
(
aV 2 2
m -1 )
6. Pressure p, volume V and temperature T for a certain gas 12. The equation of state for a gas is given by PV = nRT + aV,
AT - BT 2 where n is the number of moles and a is a positive constant.
are related by p = , where A and B are constants. The initial temperature and pressure of one mole of the gas
V contained in a cylinder are To and Po respectively. The work
The work done by the gas as its temperature changes from
T1 to T2 while pressure remains constant is done by the gas when its temperature doubles isobarically
will be:
B Po To R Po To R
(a) A - (T2 - T1 )
2 (a) (b) (c) PoToR In 2 (d) PoToR
Po - a Po + a
(b) A(T2 - T1 ) - B (T22 - T12 ) 13. The pressure of a monoatomic gas increases linearly from
A 2 B 4 × 105 N/m2 to 8 × 105 N/m2 when its volume increases from
(c) (T2 - T12 ) - (T23 - T13 ) 0.2 m3 to 0.5 m3. The work done by the gas and increase in
T 3
internal energy are given by
B
(d) A(T2 - T1 ) 2 - (T2 - T1 )3 (a) 1.8 × 105 J, 1.8 × 105 J (b) 4.8 × 105 J, 4.8 × 105 J
3
7. The specific heat capacity of a metal at low temperature (T) (c) 1.8 × 105 J, 4.8 × 105 J (d) 4.8 × 105 J, 1.8 × 105 J
3 14. An ideal gas is subjected to cyclic process involving four
æ T ö
is given as C p (kJK –1kg –1 ) = 32 ç ÷ . A 100 g vessel of thermodynamic states, the amounts of heat (Q) and work
è 400 ø
this metal is to be cooled from 20 K to 4 K by a special (W) involved in each of these states
refrigerator operating at room temperature (27°C). The Q1 = 6000 J; Q2 = –5500 J; Q3 = –3000 J; Q4 = + 3500 J
amount of work required to cool in vessel is
W1 = 2500 J; W2 = –1000 J; W3 = –1200 J; W4 = xJ
(a) equal to 0.002 kJ
The ratio of the net work done by the gas to the total heat
(b) greater than 0.148 kJ
absorbed by the gas is h. The values of x and h are
(c) between 0.148 kJ and 0.028 kJ
respectively
(d) less than 0.028 kJ
8. The temperature of 5 moles of a gas which was held at (a) 500; 7.5% (b) 700; 10.5%
constant volume was changed from 100º to 120ºC. The (c) 1000; 21% (d) 1500; 15%
change in the internal energy of the gas was found to be 80 15. Two identical containers A and B have frictionless pistons.
joule, the total heat capacity of the gas at constant volume They contain the same volume of an ideal gas at the same
will be equal to temperature. The mass of the gas in A is mA and that in B is
(a) 8 joule per K (b) 0.8 joule per K mB. The gas in each cylinder is now allowed to expand
(c) 4.0 joule per K (d) 0.4 joule per K isothermally to double the initial volume. The changes in
9. An ideal monatomic gas with pressure P, volume V and the pressure in A and B are found to be Dp and 1.5 Dp
temperature T is expanded isothermally to a volume 2V and
respectively.
a final pressure Pi. If the same gas is expanded adiabatically
(a) 4mA = 9mB (b) 2mA = 3mB
P
to a volume 2V, the final pressure is Pa. The ratio a is (c) 3mA = 2mB (d) 9mA = 4mB
Pi
(a) 2–1/3 (b) 21/3 (c) 22/3 (d) 2–2/3
EBD_7418
168 PHYSICS

ANSWER KEYS
Exercise 1 : Topic -wise MCQs
1 (d) 14 (b) 27 (a) 40 (c) 53 (c) 66 (c) 79 (d) 92 (b) 105 (b) 118 (b)
2 (c) 15 (c) 28 (a) 41 (c) 54 (c) 67 (a) 80 (b) 93 (a) 106 (b) 119 (a)
3 (d) 16 (b) 29 (b) 42 (b) 55 (a) 68 (d) 81 (c) 94 (c) 107 (d) 120 (a)
4 (c) 17 (b) 30 (c) 43 (a) 56 (a) 69 (b) 82 (b) 95 (a) 108 (b) 121 (a)
5 (c) 18 (a) 31 (a) 44 (b) 57 (c) 70 (c) 83 (d) 96 (d) 109 (d) 122 (d)
6 (a) 19 (d) 32 (b) 45 (c) 58 (c) 71 (c) 84 (d) 97 (d) 110 (c) 123 (b)
7 (c) 20 (a) 33 (b) 46 (b) 59 (d) 72 (b) 85 (a) 98 (a) 111 (b) 124 (c)
8 (c) 21 (d) 34 (d) 47 (c) 60 (d) 73 (a) 86 (a) 99 (b) 112 (b) 125 (d)
9 (b) 22 (a) 35 (a) 48 (d) 61 (a) 74 (d) 87 (b) 100 (c) 113 (d)
10 (a) 23 (d) 36 (a) 49 (c) 62 (a) 75 (c) 88 (c) 101 (c) 114 (d)
11 (d) 24 (c) 37 (a) 50 (c) 63 (c) 76 (c) 89 (d) 102 (c) 115 (b)
12 (d) 25 (a) 38 (a) 51 (b) 64 (a) 77 (d) 90 (b) 103 (d) 116 (a)
13 (b) 26 (b) 39 (b) 52 (d) 65 (b) 78 (b) 91 (a) 104 (d) 117 (d)
Exercise 2 : Exemplar & Past Year MCQs
1 (c) 5 (a) 9 (c) 13 (d) 17 (c) 21 (a) 25 (d) 29 (a) 33 (a)
2 (a) 6 (b) 10 (d) 14 (a) 18 (b) 22 (c) 26 (a) 30 (a)
3 (c) 7 (c) 11 (d) 15 (c) 19 (c) 23 (a) 27 (b) 31 (a)
4 (b) 8 (b) 12 (d) 16 (b) 20 (a) 24 (d) 28 (a) 32 (d)
Exercise 3 : Try If You Can
1 (b) 3 (a) 5 (a) 7 (c) 9 (d) 11 (d) 13 (c)
2 (d) 4 (c) 6 (b) 8 (c) 10 (d) 12 (a) 14 (b) 15 (c)
13Chapter
KINETIC THEORY

Trend
Analysis of NEET and AIIMS (Year 2010-2018)

4
Number of Questions

3
AIPMT/NEET
2 AIIMS

0
2010 2011 2012 2013 2014 2015 2016 2017 2018
Year

Trend
Analysis of JEE Main and BITSAT (Year 2010-2018)
Number of Questions

JEE Main
2
BITSAT

0
2010 2011 2012 2013 2014 2015 2016 2017 2018
Year

Chapter Utility Score (CUS)


Exam Weightage Important Concepts Difficulty Level CUS (Out of 10)
NEET 2 Ideal gas equation, RMS
AIIMS 4 speed of gas molecules, 3.5/5 6.5/10
JEE Main 3 Law of equipartition, Heat
BITSAT 3 capacity & Mean free path
EBD_7418
170 PHYSICS
KINETIC THEORY 171
EBD_7418
172 PHYSICS

Exercise 1 : Topic-wise MCQs

Topic 1: Kinetic Theory of an Ideal Gas and Gas Laws 11. According to kinetic theory of gases, at absolute zero
temperature
1. Kinetic theory of gases provide a base for (a) water freezes
(a) Charle’s law (b) liquid helium freezes
(b) Boyle’s law (c) molecular motion stops
(c) Both Charle’s law and Boyle’s law (d) liquid hydrogen freezes
(d) None of these 12. At a given temperature the force between molecules of a
2. When do real gases approach the ideal gas behaviour ? gas as a function of intermolecular distance is
(a) At low pressure and low temperature (a) always constant
(b) At low pressure and high temperature (b) always decreases
(c) At high pressure and high temperature (c) always increases
(d) At high pressure and low temperature (d) first decreases and then increases
3. According to the kinetic theory of gases, the pressure
exerted by a gas on the wall of the container is measured as 13. For Boyle’s law to hold, the gas should be
(a) rate of change of momentum imparted to the walls per (a) perfect and of constant mass and temperature
second per unit area. (b) real and of constant mass and temperature
(b) momentum imparted to the walls per unit area (c) perfect and constant temperature but variable mass
(c) change of momentum imparted to the walls per unit area. (d) real and constant temperature but variable mass
(d) change in momentum per unit volume 14. Boyle' law is applicable for an
4. When a gas is in thermal equilibrium, its molecules (a) adiabatic process. (b) isothermal process.
(a) have the same average kinetic energy of molecules (c) isobaric process. (d) isochoric process
(b) have different energies which remain constant 15. The deviation of gases from the behaviour of ideal gas is
(c) have a certain constant energy due to
(d) do not collide with one another (a) colourless molecules
5. The equation which should be satisfied exactly at all (b) covalent bonding of molecules
pressures and temperatures to be an ideal gas is (c) attraction of molecules
(a) PV = mRT (b) PV = kBNT (d) absolute scale of temp.
(c) P = kB nT. (d) All of these 16. The given P-V curve is predicted by
6. Molecules of a ideal gas behave like (a) Boyle’s law 1.4
(a) inelastic rigid sphere
1.2
(b) perfectly elastic non-rigid sphere
(c) perfectly elastic rigid sphere (b) Charle’s law 1.0 T1 > T2 > T3
P 0.8
(d) inelastic non-rigid sphere T1
7. In kinetic theory of gases, it is assumed that molecules 0.6
(c) Avogadro’s law 0.4 T2
(a) have same mass but can have different volume
(b) have same volume but mass can be different 0.2 T3
(c) have different mass as well as volume 0
(d) Gaylussac’s law 20 60 100 140 160 200
(d) have same mass but negligible volume.
8. The internal energy of a gram-molecule of an ideal gas V
depends on 17. What will be the ratio of number of molecules of a
(a) pressure alone monoatomic and a diatomic gas in a vessel, if the ratio of
(b) volume alone their partial pressures is 5 : 3 ?
(c) temperature alone (a) 5 : 1 (b) 3 : 1 (c) 5 : 3 (d) 3 : 5
(d) both on pressure as well as temperature
9. The phenomenon of Browninan movement may be taken 18. Assertion: On reducing the volume of the gas at constant
as evidence of temperature, the pressure of the gas increases.
(a) kinetic theory of matter Reason: It happens because on reducing the volume, the no.
(b) electromagnetic theory of radiation of molecules per unit volume increases and as a result more
(c) corpuscular theory of light collisions with walls exert greater pressure on the walls.
(d) photoelectric phenomenon (a) Assertion is correct, reason is correct; reason is a
10. The density (r)versus pressuure (P) of a given mass of an correct explanation for assertion.
ideal gas is shown at two temperatures T1 and T2 (b) Assertion is correct, reason is correct; reason is not a
Then relation between T1 correct explanation for assertion
T1 and T2 may be (c) Assertion is correct, reason is incorrect
T2 (d) Assertion is incorrect, reason is correct.
(a) T1 > T2 r 19. If the pressure and the volume of certain quantity of ideal
(b) T2 > T1 gas are halved, then its temperature
(a) is doubled (b) becomes one-fourth
(c) T1 = T2 (c) remains constant (d) is halved
(d) All the three are possible P
KINETIC THEORY 173

20. If a gas is heated at constant pressure, its isothermal 32. P, V, T respectively denote pressure, volume and
compressibility temperature of two gases. On mixing, new temperature and
(a) remains constant
(b) increases linearly with temperature volume are respectively T and V. Final pressure of the
(c) decreases linearly with temperature mixture is
(d) decreases inversely with temperature (a) P (b) 2P (c) zero (d) 3P
21. Assertion : A gas can be liquified at any temperature by 33. If two vessels A and B contain the same gas but the
increase of pressure alone. volume of vessel A is twice that of B and temperature and
Reason : On increasing pressure the temperature of gas pressure of gas A is twice that of gas in B, then the ratio
decreases. of gas molecules in A and B is
(a) Assertion is correct, reason is correct; reason is a
correct explanation for assertion. (a) 1 : 2 (b) 1 : 4 (c) 4 : 1 (d) 2 : 1
(b) Assertion is correct, reason is correct; reason is not
a correct explanation for assertion Topic 2: Speed of Gas, Pressure and Kinetic Energy
(c) Assertion is correct, reason is incorrect
(d) Assertion is incorrect, reason is correct. 34. In a mixture of gases at a fixed temperature
22. A gas in a container A is in thermal equilibrium with antoher (a) heavier molecule has higher average speed
gas of the same mass in container B. If we denote the (b) lighter molecule has lower average speed
corresponding pressures and volumes by the suffixes A (c) heavier molecule has lower average speed
and B, then which of the following statement is most likely (d) None of these
to be true? 35. The average kinetic energy per molecule of any ideal gas
(a) PA = PB , VA ¹ VB (b) PA ¹ PB , VA = VB is always equal to
(c) PA / VA = PB / VB (d) PA VA = PB VB
23. In a cubical vessel are enclosed n molecules of a gas each 2 3 3
(a) k BT (b) k B T (c) k B T (d) 3kBT
having a mass m and an average speed v. If l is the length 3 4 2
of each edge of the cube, the pressure exerted by the gas 36. The average kinetic energy of gas molecules depends
will be upon which of the following factor?
n m2 v m n v2 (a) Nature of the gas (b) Temperature of the gas
n mv 2 nmv
(a) (b) (c) 3 (d) (c) Volume of the gas (d) Both (b) & (c)
3
l 3 2l 3l 2l 37. Let v denote the rms speed of the molecules in an ideal
24. If the critical temperature of a gas is 100ºC, its Boyle diatomic gas at absolute temperature T.
temperature will be approximately The mass of a molecule is ‘m’ neglecting vibrational energy
(a) 337.5ºC (b) 500ºC (c) 33.3ºC (d) 1000ºC terms, the false statement is
25. A vessel has 6g of hydrogen at pressure P and temperature (a) a molecule can have a speed greater than 2v
500K. A small hole is made in it so that hydrogen leaks out.
How much hydrogen leaks out if the final pressure is P/2
(b) v is proportional to T
and temperature falls to 300 K ?
(a) 2g (b) 3g (c) 4g (d) 1g 1 2
(c) the average rotational K.E. of a molecule is mv
26. If the intermolecular forces vanish away, the volume 4
occupied by the molecule contained in 4.5 kg water at 5 2
standard temperature and pressure will be given by (d) the average K.E. of a molecule is mv
6
(a) 5.6 litre (b) 4.5 litre (c) 11.2 litre (d) 11.2 litre 38. The temperature of a gas is a measure of
27. A balloon contains 1500 m3 of helium at 27oC and 4 (a) the average kinetic energy of the gaseous molecules
atmospheric pressure. The volume of helium at – 3oC (b) the average potential energy of the gaseous molecules
temperature and 2 atmospheric pressure will, (c) the average distance between the molecules of the gas
(a) 1500 m3 (b) 1700 m3 (c) 1900 m3 (d) 2700 m3 (d) the size of the molecules of the gas
28. A perfect gas at 27°C is heated at constant pressure so as 39. In the isothermal expansion of 10g of gas from volume V to
to double its volume. The final temperature of the gas will 2V the work done by the gas is 575J. What is the root mean
be, close to square speed of the molecules of the gas at that temperature?
(a) 327°C (b) 200°C (c) 54°C (d) 300°C (a) 398m/s (b) 520m/s (c) 499m/s (d) 532m/s
29. The number of molecules in each cubic metre of a gas at 40. In a diatomic molecules, the rotational energy at a given
temperature
1atm and 27oC is (a) obeys Maxwell’s distribution
(a) 2.4 × 1025(b) 5.2 × 1029(c) 6.1 × 1030 (d) 8.3 × 1031 (b) have the same volue for all molecules
30. At a particular temperature and pressure 22.4 L of N2-gas (c) equals the translational kinetic energy for each molecule.
contains 1 mol of its’ molecule. For the same temperature
and pressure 22.4 L of N2-gas contains how much (d) None of these
number of molecules 41. The speed of sound in oxygen (O2) at a certain temperature
(a) 1 mol (b) 2 mol (c) 3 mol (d) 4 mol is 460 ms–1. The speed of sound in helium (He) at the same
31. Ratio of volume occupied by a gas H2 of mass 22 gm at temperature will be (assume both gases to be ideal)
two different pressure p and 2p if temperature remains (a) 1421 ms–1 (b) 500 ms–1 (c) 650 ms–1 (d) 330 ms–1
constant is 42. Match Column I (Physical Variables) with Column II
(a) 1 : 2 (b) 2 : 1 (c) 1 : 4 (d) 4 : 1 (Expressions) . (n = number of gas molecules present per
unit volume, k = Boltzmann constant, T = absolute
temperature, m = mass of the particle) :
EBD_7418
174 PHYSICS

Column I Column II 50. Maxwell's laws of distribution of velocities shows that


(A) Most probable velocity (1) nkT (a) the number of molecules with most probable velocity
(B) Energy per degree of (2) Ö (3kT / m) is very large
freedom (b) the number of molecules with most probable velocity
(C) Pressure (3) Ö (2kT / m) is very small
(D) R.M.S. velocity (4) kT / 2 (c) the number of molecules with most probable velocity
(a) (A) ® (3) ; (B) ® (4) ; (C) ® (1) ; (D) ® (2) is zero
(b) (A) ® (1) ; (B) ® (2) ; (C) ® (3) ; (D) ® (4) (d) the number of molecules with most probable velocity
(c) (A)® (4) ; (B) ® (3) ; (C) ® (1) ; (D) ® (2) is exactly equal to 1
(d) (A) ® (3) ; (B) ® (4) ; (C) ® (2) ; (D) ® (2) 51. For mean kinetic energy per molecule, a vessel filled with
43. Assertion : When we place a gas cylinder on a moving two different gases.
train, its internal kinetic energy increases. I. Mean kinetic energy per molecule for both gases
Reason : Its temperature remains constant. will be equal.
(a) Assertion is correct, reason is correct; reason is a II. Mean kinetic energy per molecule of gas with higher
correct explanation for assertion. mass will be more.
(b) Assertion is correct, reason is correct; reason is not III. Mean kinetic energy per moleculers of gas with
a correct explanation for assertion lower mass will be more.
(c) Assertion is correct, reason is incorrect Select the correct statement(s).
(d) Assertion is incorrect, reason is correct. (a) I only (b) II only
44. Cooking gas containers are kept in a lorry moving with (c) I and II (d) I, II and III
uniform speed. The temperature of the gas molecules inside 52. A gas is enclosed in a cube of side l. What will be the
will. change in momentum of the molecule, if it suffers an
(a) increase elastic collision with the plane wall parallel to yz-plane
(b) decrease and rebounds with the same velocity ?
(c) remains the same [(Vx, Vy & Vz) initial velocities of the gas molecules]
(d) decrease for some and increase for others (a) mvx (b) zero
45. Pressure exerted by a gas is (c) – mvx (d) – 2mvx.
(a) independent of density of the gas 53. A flask contains a monoatomic and a diatomic gas in the
(b) inversely proportional to the density of the gas ratio of 4 : 1 by mass at a temperature of 300 K. The ratio
(c) directly proportional to the square of the density of of average kinetic energy per molecule of the two gases is
the gas (a) 1 : 1 (b) 2 : 1 (c) 4 : 1 (d) 1 : 4
(d) directly proportional to the density of the gas 54. At constant volume, temperature is increased then
46. At a given temperature which of the following gases (a) collision on walls will be less
possesses maximum r.m.s. velocity? (b) number of collisions per unit time will increase
(a) Hydrogen (b) Oxygen (c) collisions will be in straight lines
(c) Nitrogen (d) Carbon dioxide (d) collisions will not change.
47. The internal energy of an ideal gas is 55. Gas at a pressure P0 is contained in a vessel. If the
(a) the sum of total kinetic and potential energies. masses of all the molecules are halved and their speeds
(b) the total translational kinetic energy. are doubled, the resulting pressure P will be equal to
(c) the total kinetic energy of randomly moving molecules.
P0
(d) the total kinetic energy of gas molecules. (a) 4P0 (b) 2P0 (c) P 0 (d)
48. The root mean square speed of a group of n gas molecules, 2
56. Which of the following is/are incorrect statement(s)
having speed v1 , v 2 , v3 ,..., vn is regarding the law of equipartition of energy ?
1 I. The gas possess equal energies in all the three
(a) (v1 + v 2 + v 3 + ... + v n ) 2 directions x,y and z-axis.
n
II. The total energy of a gas is equally divided between
1
(b) (v12 + v 2 2 + v 32 + ... + v n 2 ) kinetic and potential energies.
n III. The total kinetic energy of a gas molecules is equally
1 divided among translational and rotational kinetic
(c) (v12 + v 2 2 + v3 2 + ... + v n 2 ) energies.
n
(a) I only (b) II and III
é (v1 + v 2 + v3 + ... v n ) 2 ù (c) I and III (d) I, II and III
(d) ê ú 57. The temperature at which the average transitional kinetic
êë n úû energy of a gas molecule is equal to the energy gained by
49. If E is the translational kinetie energy, then which of the an electron in accelerating from rest through a potential
following relation holds good ? difference of 1 volt is
3 (a) 4.6 × 103 K (b) 11.6 × 103 K
(a) PV = E (b) PV = E (c) 23.2 × 10 K 3 (d) 7.7 × 103 K
2
58. The temperature at which protons in hydrogen gas would
2
(c) PV = 3E (d) PV = E have enough energy to overcome Coulomb barrier of
3 4.14 × 10–14 J is
KINETIC THEORY 175

(Boltzmann constant = 1.38 × 10–23 JK–1) 69. The average transitional energy and the rms speed of
(a) 2 × 109 K (b) 109 K molecules in a sample of oxygen gas at 300 K are 6.21 × 10–21 J
(c) 6 × 10 K9 (d) 3 × 109 K and 484 m/s respectively. The corresponding values at 600
59. The temperature at which oxygen molecules have the same K are nearly (assuming ideal gas behaviour)
root mean square speed as that of hydrogen molecules at (a) 12.42 × 10–21 J, 968 m/s (b) 8.78 × 10–21 J, 684 m/s
300 K is (c) 6.21 × 10–21 J, 968 m/s (d) 12.42 × 10–21 J, 684 m/s
(a) 600 K (b) 2400 K (c) 4800 K (d) 300 K
60. Assertion : If a gas container in motion is suddenly Topic 3: Degree of Freedom, Specific Heat Capacity and
stopped, the temperature of the gas rises. Mean Free Path
Reason : The kinetic energy of ordered mechanical motion
is converted in to the kinetic energy of random motion of 70. If a gas has ‘n’ degrees of freedom, the ratio of the specific
gas molecules. heats g of the gas is
(a) Assertion is correct, reason is correct; reason is a 1+ n n 1 2
correct explanation for assertion. (a) (b) 1 + (c) 1 + (d) 1 +
(b) Assertion is correct, reason is correct; reason is not 2 2 n n
a correct explanation for assertion æ 5ö
71. If one mole of monoatomic gas ç g = ÷ is mixed with one
(c) Assertion is correct, reason is incorrect è 3ø
(d) Assertion is incorrect, reason is correct. æ 7ö
mole of diatomic gas ç g = ÷ , the value of g for the mixture is
61. One kg of a diatomic gas is at a pressure of 8 × 104N/m2. è 5ø
The density of the gas is 4kg/m3. What is the energy of the (a) 1.40 (b) 1.50 (c) 1.53 (d) 3.07
gas due to its thermal motion? 72. How is the mean free path (l) in a gas related to the
(a) 5 × 104 J (b) 6 × 104 J interatomic distance ?
(c) 7 × 104 J (d) 3 × 104 J (a) l is 10 times the interatomic distance
62. The kinetic theory of gases states that the average squared (b) l is 100 times the interatomic distance
velocity of molecules varies linearly with the mean molecular (c) l is 1000 times the interatomic distance
weight of the gas. If the root mean square (rms) velocity of 1
oxygen molecules at a certain temperature is 0.5 km/sec. (d) l is times of the interatomic distance
The rms velocity for hydrogen molecules at the same 10
73. Mean free path of a gas molecule is
temperature will be :
(a) 2 km/sec (b) 4 km/sec (c) 8 km/sec (d) 16 km/sec (a) inversely proportional to number of molecules per unit
63. Find the temperature at which the Vrms for O2 molecule is volume
(b) inversely proportional to diameter of the molecule
483 ms–1
(c) directly proportional to the square root of the absolute
(a) 300 K (b) 400 K (c) 500 K (d) 600 K temperature
64. The r.m.s. velocity of oxygen molecule at 16ºC is (d) directly proportional to the molecular mass
474 m/sec. The r.m.s. velocity in m/s of hydrogen molecule 74. If the pressure in a closed vessel is reduced by drawing
at 127ºC is out some gas, the mean-free path of the molecules
(a) 1603 (b) 1896 (c) 2230.59 (d) 2730 (a) is decreased
65. Assertion : For an ideal gas, at constant temperature, the (b) is increased
product of pressure and volume is constant. (c) remains unchanged
Reason : The mean square velocity of the molecules is (d) increases or decreases according to the nature of the gas
inversely proportional to mass. 75. From the following statements, concerning ideal gas at any
(a) Assertion is correct, reason is correct; reason is a given temperature T, select the correct one(s)
correct explanation for assertion. I. The coefficient of volume expansion at constant
(b) Assertion is correct, reason is correct; reason is not a pressure is same for all ideal gas.
correct explanation for assertion II. The average translational kinetic energy per molecule
(c) Assertion is correct, reason is incorrect of oxygen gas is 3 KT (K being Boltzmann constant).
(d) Assertion is incorrect, reason is correct. III. In a gaseous mixture, the average translational kinetic
66. The root mean sqaure velocity of the molecules in a sample energy of the molecules of each component is same.
of helium is 5/7 th that of the molucules in a sample of IV. The mean free path of molecules increases with
hydrogen. If the temperature of hydrogen sample is 0oC, decrease in pressure.
that of helium sample is about (a) I, II and III (b) III and IV only
(a) 0 o C (b) 283 K (c) 0K (d) 100 o C (c) I, III and IV (d) I, II, III and IV
67. The root mean square velocity of hydrogen molecules at 76. The specific heat of a gas
300 K is 1930 metre/sec. Then the r.m.s velocity of oxygen (a) has only two values cp and cv
molecules at 1200 K will be (b) has a unique value at a given temperature
(a) 482.5 metre/sec (b) 965 metre/sec (c) can have any value between 0 and ¥
(c) 1930 metre/sec (d) 3860 metre/sec (d) depends upon the mass of the gas
68. At what temperature the mean speed of the molecule of 77. The total internal energy of one mole of rigid diatomic gas is
hydrogen gas (H2) equals the escape speed from the earth?
3 7 5 9
(a) 11800 K (b) 1500 K (c) 1800 K (d) 2000 K (a) RT (b) RT (c) RT (d) RT
2 2 2 2
EBD_7418
176 PHYSICS

78. The specific heats at constant pressure is greater than (a) 10 15 (b) 8.3 × 10 12 (c) 1.1 × 106 (d) 1.23 × 1010
that of the same gas at constant volume because 91. How many degrees of freedom are associated with 2 grams
(a) at constant pressure work is done in expanding the gas
of He at NTP ?
(b) at constant volume work is done in expanding the gas
(c) the molecular attraction increases more at constant (a) 3 (b) 3.01 × 1023 (c) 9.03 × 1023 (d) 6
pressure 92. Modern vacuum pumps can evacuate a vessel down to a
(d) the molecular vibration increases more at constant pressure of 4.0 × 10–15 atm. at room temperature (300 K).
pressure Taking R = 8.0 JK–1 mole–1, 1 atm = 105 Pa and NAvogadro
79. One mole of a gas occupies 22.4 lit at N.T.P. calculate the = 6 × 1023 mole–1, the mean distance between molecules of
difference between two molar specific heats of the gas J = gas in an evacuated vessel will be of the order of:
4200 J/kcal
(a) 1.972 kcal/kmol K (b) 2.378 kcal/kmol K (a) 0.2 mm (b) 0.2 mm (c) 0.2 cm (d) 0.2 nm
(c) 4.569 kcal/kmol K (d) 3.028 kcal/kmol K 93. The ratio of molar specific heat at constant pressure CP,
80. As per the law of equi-partition of energy each vibrational to molar specific heat at constant volume Cv for a
mode gives how many degrees of freedom ? monoatomic gas is
(a) 1 (b) 2 (c) 3 (d) 0 CP 3 CP 7
81. A fly moving in a room has ...X... degree of freedom. (a) = (b)
CP 5
= (c) = (d) CP = 9
CV 5 CV 3 C V 9 CV 7
Here, X refers to
(a) one (b) two (c) three (d) four 94. The ratio of the molar heat capacities of a diatomic gas at
constant pressure to that a constant volume is
82. Consider the following statements and select the correct option.
I. The ratio of CP / CV for a diatomic gas is more than 7 3 3 7
(a) (b) (c) (d)
that of a monoatomic gas. 2 2 5 5
II. The ratio of CP / CV is more for helium gas than for 95. If CP and CV denote the specific heats of nitrogen per unit
hydrogen mass at constant pressure and constant volume
(a) I only (b) II only respectively, then
(c) I and II (d) None of these (a) CP – CV = 28R (b) CP – CV = R/28
83. The number of degrees of freedom for each atom of a
(c) CP – CV = R/14 (d) CP – CV = R
monatomic gas is
(a) 3 (b) 5 (c) 6 (d) 1 96. The molar specific heat at constant pressure of an ideal gas
84. The total number of degree of freedom of a CO2 gas is (9/2)R. The ratio of specific heat at constant pressure to
molecule is that at constant volume is
(a) 3 (b) 6 (c) 5 (d) 4 (a) 1.58 (b) 1.82 (c) 1.28 (d) 1.44
85. A polyatomic gas with n degrees of freedom has a mean 97. The molar specific heats of an ideal gas at constant pressure
energy per molecule given by and volume are denoted by Cp and Cv, respectively. If
nkT nkT nkT 3kT Cp
(a) (b) (c) (d) g= and R is the universal gas constant, then Cv is
N 2N 2 2
86. The degree of freedom of a molecule of a triatomic gas is Cv
equal to
(a) 2 (b) 4 (c) 6 (d) 8
87. For hydrogen gas Cp – C v = a and for oxygen gas
Cp – Cv = b. So, the relation between a and b is given by R ( g - 1) 1+ g
(a) (b) (c) gR (d)
(a) a = 16 b (b) 16 a = b (c) a = 4 b (d) a = b ( g - 1) R 1- g
88. P-V diagram of a diatomic gas is a straight line passing
through origin. The molar heat capacity of the gas in the 98. A gaseous mixture consists of 16 g of helium and 16 g of
process will be Cp
4R oxygen. The ratio of the mixture is
(a) 4 R (b ) 2.5 R (c) 3 R (d) Cv
3
89. For a gas, difference between two specific heats is 5000 J/ (a) 1.62 (b) 1.59 (c) 1.54 (d) 1.4
mole°C. If the ratio of specific heat is 1.6, the two specific 99. A cylinder rolls without slipping down an inclined plane,
heats are in J/mole-°C
the number of degrees of freedom it has, is
(a) C P = 1.33 ´10 4 , C V = 2.66 ´ 10 4
(a) 2 (b) 3 (c) 5 (d) 1
(b) C P = 13.3 ´ 10 4 , C V = 8.33 ´103 100. The value of CP – CV is 1.00 R for a gas sample in state A is
1.06 R in state B. Let PA, PB denote the pressure and TA, TB
(c) C P = 1.33 ´ 10 4 , C V = 8.33 ´103 denote the temperature of states A and B respectively, then
(d) C P = 2.6 ´ 10 4 , C V = 8.33 ´10 4 (a) PA< PB and TA> TB (b) PA> PB and TA< TB
90. The number of collisions per second suffered by a molecule (c) PA– PB and TA< TB (d) PA> PB and TA– TB
in a sample of hydrogen at S.T.P is:
[Mean free path = 1.38 × 10–5 cm]
KINETIC THEORY 177

Exercise 2 : Exemplar & Past Year MCQs


6. 1 mole of H2 gas is contained in a box of volume V = 1.00 m3
NCERT Exemplar MCQs at T =300 K. The gas is heated to a temperature of T = 3000
1. A cubic vessel (with face horizontal + vertical) contains an K and the gas gets converted to a gas of hydrogen atoms.
ideal gas at NTP. The vessel is being carried by a rocket The final pressure would be (considering all gases to be ideal)
which is moving at a speed of 500 m s–1 in vertical direction. (a) same as the pressure initially
The pressure of the gas inside the vessel as observed by (b) 2 times the pressure initially
us on the ground (c) 10 times the pressure initially
(a) remains the same because 500 ms–1 is very much (d) 20 times the pressure initially
smaller than vrms of the gas
(b) remains the same because motion of the vessel as a 7. A vessel of volume V contains a mixture of 1 mole of
whole does not affect the relative motion of the gas hydrogen and 1 mole oxygen (both considered as ideal).
molecules and the walls Let f1(v)dv, denote the fraction of molecules with speed
2
(c) will increase by a factor equal to (vrms + (500)2 ) / vrms
2 between v and (v + dv ) with f2(v)dv, similarly for oxygen.
where vrms was the original mean square velocity of Then,
the gas (a) f1 (v) + f 2 (v ) = f (v ) obeys the Maxwell's distribution
(d) will be different on the top wall and bottom wall of the law
vessel (b) f1(v), f2(v) will obey the Maxwell's distribution law
2. 1 mole of an ideal gas is contained in a cubical volume V, separately
ABCDEFGH at 300K (figure). One face of the cube (EFGH) (c) neither f 1(v), nor f 2(v) will obey the Maxwell's
is made up of a material which totally absorbs any gas distribution law
molecule incident on it. At any given time, (d) f2(v) and f1(v) will be the same
B 8. An inflated rubber balloon contains one mole of an ideal
C
A gas, has a pressure p, volume V and temperature T. If the
D
temperature rises to 1.1 T, and the volume is increased to
G
F 1.05 V, the final pressure will be
E
H (a) 1.1 p (b) p
(a) the pressure on EFGH would be zero (c) less than p (d) between p and 1.1
(b) the pressure on all the faces will the equal
(c) the pressure of EFGH would be double the pressure Past Year MCQs
on ABCD
9. The mean free path of molecules of a gas, (radius ‘r’) is
(d) the pressure on EFGH would be half that on ABCD
inversely proportional to : [AIPMT 2014, C]
3. Boyle's law is applicable for an
(a) r3 (b) r2 (c) r (d) r
(a) adiabatic process (b) isothermal process 10. Assertion : Mean free path of a gas molecules varies
(c) isobaric process (d) isochoric process inversely as density of the gas.
4. A cylinder containing an ideal gas is in vertical position Reason : Mean free path varies inversely as pressure of
and has a piston of mass M that is able to move up or down the gas. [AIIMS 2014, C]
without friction (figure). If the temperature is increased (a) If both Assertion and Reason are correct and Reason
(a) both p and V of the gas will M is the correct explanation of Assertion.
change (b) If both Assertion and Reason are correct, but Reason
(b) only p will increase according is not the correct explanation of Assertion.
to Charles' law (c) If Assertion is correct but Reason is incorrect.
(c) V will change but not p (d) If both the Assertion and Reason are incorrect.
(d) p will change but not V 11. 4.0 g of a gas occupies 22.4 litres at NTP. The specific heat
5. Volume versus temperature graphs for a given mass of an capacity of the gas at constant volume is 5.0JK-1. If the
speed of sound in this gas at NTP is 952 ms–1, then the heat
ideal gas are shown in figure. At two different values of
capacity at constant pressure is (Take gas constant
constant pressure. What can be inferred about relation
R = 8.3 JK-1 mol-1) [AIPMT 2015, C]
between p1 and p2? () p2 (a) 7.5 JK-1 mol-1 (b) 7.0 JK-1 mol-1
(a) p1 > p2 (c) 8.5 JK-1 mol-1 (d) 8.0 JK-1 mol-1
40
12. Two vessels separately contain two ideal gases A and B at
(b) p1 = p2 30 the same temperature. The pressure of A being twice that
p1
20 of B. Under such conditions, the density of A is found to
(c) p1 < p2 be 1.5 times the density of B. The ratio of molecular weight
10
of A and B is : [AIPMT 2015, C]
(d) Data is insufficient 100 200300 400 500 (a) 3/4 (b) 2 (c) 1/2 (d) 2/3
T(K)
EBD_7418
178 PHYSICS

13. A gas mixture consists of molecules of type 1, 2 and 3, with CP - C C - CV


molar masses m1 > m2 > m3. vrms and K are the r.m.s. speed (a) n= (b) n =
C - CV C - CP
and average kinetic energy of the gases. Which of the CP C – CP
following is true? [AIIMS 2015, C] (c) n = (d) n =
CV C – CV
(a) (vrms)1 < (vrms)2 < (vrms)3 and ( K )1 = ( K ) 2 = ( K )3
æ Cp ö
(b) (vrms)1 = (vrms)2 = (vrms)3 and ( K )1 = ( K ) 2 > ( K )3 20. The value of g ç = ÷ for diatomic gas with vibrational
è Cv ø
(c) (vrms)1 > (vrms)2 > (vrms)3 and ( K )1 < ( K ) 2 > ( K ) 3 mode is [BITSAT 2016, C]
(d) (vrms)1 > (vrms)2 > (vrms)3 and ( K )1 < ( K ) 2 < ( K )3 (a) 1.66 (b) 1.28 (c) 1.99 (d) 1.4
14. Consider an ideal gas confined in an isolated closed 21. A gas mixture consists of 2 moles of O2 and 4 moles of Ar at
chamber. As the gas undergoes an adiabatic expansion, temperature T. Neglecting all vibrational modes, the total
the average time of collision between molecules increases internal energy of the system is : [NEET 2017, C]
as Vq, where V is the volume of the gas. The value of q is : (a) 15 RT (b) 9 RT (c) 11 RT (d) 4 RT
æ Cp ö 22. If the root mean square velocity of the molecules of
çg = ÷ [JEE Main 2015, C] hydrogen at NTP is 1.84 km/s. Calculate the root mean
è Cv ø square velocity of oxygen molecule at NTP, molecular weight
g +1 g -1 3g + 5 3g - 5 of hydrogen and oxygen are 2 and 32 respectively
(a) (b) (c) (d)
2 2 6 6 [AIIMS 2017, A]
15. A vessel of volume 20L contains a mixture of hydrogen (a) 1.47 km/sec (b) 0.94 km/s
and helium at temperature of 27°C and pressure 2 atm. The (c) 1.84 km/s (d) 0.47 km/sec
mass of mixture is 5g. Assuming the gases to be ideal, the 23. Assertion : Air pressure in a car tyre increases during
ratio of mass of hydrogen to that of helium in the given driving.
mixture will be [BITSAT 2015, A] Reason : Absolute zero temperature is not zero energy
(a) 1 : 2 (b) 2 : 3 (c) 2 : 1 (d) 2 : 5 temperature. [AIIMS 2017, C]
16. The molecules of a given mass of a gas have r.m.s. velocity (a) If both Assertion and Reason are correct and Reason
is the correct explanation of Assertion.
of 200 ms–1 at 27°C and 1.0 × 105 Nm–2 pressure. When the
(b) If both Assertion and Reason are correct, but Reason
temperature and pressure of the gas are respectively, 127°C is not the correct explanation of Assertion.
and 0.05 × 105 Nm–2, the r.m.s. velocity of its molecules in (c) If Assertion is correct but Reason is incorrect.
ms–1 is : [NEET 2016, A] (d) If both the Assertion and Reason are incorrect.
400 100 2 100 24. The temperature of an open room of volume 30 m3 increases
(a) 100 2 (b) (c) (d) from 17°C to 27°C due to sunshine. The atmospheric pressure
3 3 3
17. A thermally insulated vessel contains an ideal gas of in the room remains 1 × 105 Pa. If ni and nf are the number of
molecular mass M and ratio of specific heats g. It is moving molecules in the room before and after heating, then nf – ni
with speed v and its suddenly brought to rest. Assuming will be : [JEE Main 2017, A]
no heat is lost to the surroundings, its temperature` (a) 2.5 × 1025 (b) –2.5 × 1025
increases by [AIIMS 2016, C] (c) –1.61 × 1023 (d) 1.38 × 1023
( g –1) 2 25. Cp and Cv are specific heats at constant pressure and
Mv 2 K gMv
(a) (b) K constant volume respectively. It is observed that
2gR 2R Cp – Cv = a for hydrogen gas
( g –1) ( g –1) Cp – Cv = b for nitrogen gas
(c) Mv 2 K (d) Mv 2 K
2R 2( g + 1) R The correct relation between a and b is:[JEE Main 2017, C]
18. Assertion : One mole of any substance at any temperature 1
or volume always contains 6.02 × 1023 molecules. (a) a = 14 b (b) a = 28 b (c) a = b (d) a = b
14
Reason : One mole of a substance always refers to S.T.P. 26. Five gas molecules chosen at random are found to have
conditions. [AIIMS 2016, C]
speeds of 500, 600, 700, 800 and 900 m/s:
(a) If both Assertion and Reason are correct and Reason
is the correct explanation of Assertion. [BITSAT 2017, A]
(b) If both Assertion and Reason are correct, but Reason (a) The root mean square speed and the average speed
is not the correct explanation of Assertion. are the same.
(c) If Assertion is correct but Reason is incorrect. (b) The root mean square speed is 14 m/s higher than the
(d) If both the Assertion and Reason are incorrect.
average speed.
19. An ideal gas undergoes a quasi static, reversible process
in which its molar heat capacity C remains constant. If (c) The root mean square speed is 14 m/s lower than the
during this process the relation of pressure P and volume V average speed.
is given by PVn = constant, then n is given by (Here CP and (d) The root mean square speed is Ö14 m/s higher than
CV are molar specific heat at constant pressure and constant the average speed.
volume, respectively) : [JEE Main 2016, C]
KINETIC THEORY 179

27. At what temperature will the rms speed of oxygen molecules (b) Assertion is correct, reason is correct; reason is not
become just sufficient for escaping from the Earth’s a correct explanation for assertion
atmosphere? (c) Assertion is correct, reason is incorrect
(Given : (d) Assertion is incorrect, reason is correct.
Mass of oxygen molecule (m) = 2.76 × 10–26 kg 30. Two moles of an ideal monoatomic gas occupies a volume
Boltzmann’s constant kB = 1.38 × 10–23 JK–1) [NEET 2018, A] V at 27°C. The gas expands adiabatically to a volume 2 V.
(a) 2.508 × 104 K (b) 8.360 × 104 K Calculate (i) the final temperature of the gas and (ii) change
(c) 1.254 × 10 K4 (d) 5.016 × 104 K in its internal energy. [JEE Main 2018, S]
28. Choose the false statement(s) from the following. (a) (i) 189 K (ii) 2.7 kJ (b) (i)195 K (ii) –2.7 kJ
I. CP – CV = R is true for monoatomic gases only. (c) (i)189 K (ii) –2.7 kJ (d) (i)195 K (ii) 2.7 kJ
II. Specific heat of a gas at constant pressure is greater 31. The mass of H2 molecule is 3.32 × 10–24 g. If 1023 hydrogen
than specific heat at constant volume. molecules per second strike 2 cm 2 of wall at an angle of 45o
III. Mean free path of molecules of a gas decrease with with the normal, while moving with a speed of 10 5 cm/s, the
increase in number density of the molecules.
pressure exterted on the wall is nearly. [BITSAT 2018, S]
[AIIMS 2018, A]
(a) I only (b) II only (a) 1350 N/m2 (b) 2350 N/m2
(c) I and II (d) I, II and III (c) 3320 N/m 2 (d) 1660 N/m2
29. Assertion : Each vibrational mode gives two degrees of freedom.
Cp
Reason : By law of equipartition of energy, the energy for 32. The ratio of the specific heats of a gas is = g1.66 , Then
each degree of freedom in thermal equilibrium is 2kB T. Cv
[AIIMS 2018, C] the gas may be [BITSAT 2018, S]
(a) Assertion is correct, reason is correct; reason is a (a) CO2 (b) He (c) H2 (d) NO2
correct explanation for assertion.

Exercise 3 : Try If You Can


1. By what percentage should the pressure of a given mass of 5. The pressure of an ideal gas varies according to the law
a gas be increased so as to decrease its volume by 10% at P = P0 – AV2, where P0 and A are positive constants. Then
a constant temperature? (a) the highest temperature Tmax that can be attained by
(a) 8.1 % (b) 9.1 % (c) 10.1 % (d) 11.1 %
2. At 10° C the value of the density of a fixed mass of an ideal 2 P0 16
gas divided by its pressure is x. At 110°C this ratio is: the gas is
3nR 3 A
383 10 283
(a) x (b) x (c) x (d) x P0
283 110 383 (b) for Tmax, V =
3. For a gas sample with N0 number of molecules, function 3A
N(V) is given by :
3P0 P0
dN é 3N0 ù 2 (c) Tmax =
N (V ) = =ê 2nR 3 A
ú V for 0 £ V £ V0 and N(V) = 0 for
dV êë V03 úû (d) None of these
6. A closed hollow insulated cylinder is filled with gas at 0°C
V > V0 where dN is number of molecules in speed range
and also contains an insulated piston of negligible weight
V to V + dV. The rms speed of the gas molecule is -
and negligible thickness at the middle point. The gas on one
2 3 side of the piston is heated to 100°C. If the piston moves 5
(a) V0 (b) V0 (c) 2V0 (d) 3V0
5 5 cm, the length of the hollow cylinder is
4. Three containers of the same volume contain three different
(a) 13.65 cm (b) 27.3 cm (c) 38.6 cm (d) 64.6 cm
gases. The masses of the molecules are m1, m2 and m3 and the
number of molecules in their respective containers are N1, N2 and 7. An experiment requires a gas with g = 1.50. This can be
N3. The gas pressure in the containers are P1, P2 and P3 achieved by mixing together monatomic and rigid diatomic
respectively. All the gases are now mixed and put in one of these ideal gases. The ratio of moles of the monatomic to diatomic
containers. The pressure P of the mixture will be gas in the mixture is
P + P + P3 (a) 1 : 3 (b) 2 : 3 (c) 1 : 1 (d) 3 : 4
(a) P < (P1 + P2 + P3) (b) P = 1 2
3 8. Consider a gas with density r and c as the root mean
(c) P = P1 + P2 + P3 (d) P > (P1 + P2 + P3) square velocity of its molecules contained in a volume. If
EBD_7418
180 PHYSICS

the system moves as whole with velocity v, then the pressure 14. In two vessels of same volume atomic hydrogen and helium
exerted by the gas is at pressure 1 atm. and 2 atm. are filled. If temperature of
1 2 1 both the samples is same, then average speed of hydrogen
(a) rc (b) r( c + v ) 2
3 3 atom v H will be related to helium v He as
1 1 (a) vH = 2vHe (b) v H = v He
(c) r(c – v ) 2 (d) r(c –2 – v )2
3 3 (c) v H = 2 v He (d) v H = vHe / 2
9. A vessel of volume V contains a mixture of 1 mole of 15. The density of a gas is 6 × 10–2 kg/m3 and the root mean
hydrogen and 1 mole of oxygen (both considered as ideal). square velocity of the gas molecules is 500 m/s. The
Let f1(v)dv, denote the fraction of molecules with speed pressure exerted by the gas on the walls of the vessel is
between v and (v + dv). Similarly for oxygen with f2(v)dv. Then 3 2
(a) 5 ×10 N / m (b) 1.2 ×10 –4 N / m2
(a) f1(v) + f2(v) = f (v) obeys the Maxwell’s distribution
law (c) 0.83×10 –4 N / m2 (d) 30N / m2
(b) f1(v), f2(v) will obey the Maxwell’s distribution law
16. The density of air at pressure of 105 Nm–2 is 1.2 kg m–3.
separately
Under these conditions, the root mean square velocity of
(c) neither f 1 (v) nor f 2 (v) will obey the Maxwell’s the air molecules in ms–1 is
distribution law (a) 500 (b) 1000 (c) 1500 (d) 3000
(d) f2(v) and f1(v) will be same 17. Figure shows a parabolic graph between T and 1/V for a
10. N (< 100) molecules of a gas have velocities 1, 2, 3, ........ N mixture of a gas undergoing an adiabatic process. What is
km/s respectively. Then ratio of rms speed and average the ratio of Vrms of molecules and speed of sound in mixture?
speed is T
(2N + 1) (N + 1)
(a) 1 (b) (a) 3/ 2
6N
2T0
(2N + 1) (N + 1) 2N + 1 (b) 2 T0
(c) (d) 2
6 6(N + 1)
11. Graph of specific heat at constant volume for a monatomic (c) 2/3 1/V
1/V0 4/V0
gas is
(a) Y (b) Y (d) 3
3R
18. N moles of a diatomic gas in a cylinder are at a temperature
cv cv
T. Heat is supplied to the cylinder such that the temperature
X
O
T O
T
X remains constant but n moles of the diatomic gas get
(c) Y 3 (d) Y converted into monoatomic gas. What is the change in the
—R
2 total kinetic energy of the gas ?
cv cv
1 3 5
O X O X (a) nRT (b) 0 (c) nRT (d) nRT
T T 2 2 2
12. Air is filled at 60°C in a vessel of open mouth. The vessel is 19. Using equipartition of energy, the specific heat
heated to a temperature T so that 1/4th part of air escapes. (in J kg–1 K–1) of aluminium at room temperature can be
Assuming the volume of vessel remaining constant, the estimated to be (atomic weight of aluminium = 27)
value of T is (a) 410 (b) 25 (c) 1850 (d) 925
(a) 80°C (b) 444°C (c) 333°C (d) 171°C 20. Two gases occupy two containers A and B the gas in A, of
13. 1 mole of a monatomic and 2 mole of diatomic gas are mixed, volume 0.10m3, exerts a pressure of 1.40 MPa and that in B
now the resulting gas is taken through a process in which of volume 0.15m3 exerts a pressure 0.7 MPa. The two
molar heat capacity was found 3R. Polytropic constant in containers are united by a tube of negligible volume and
the process is the gases are allowed to intermingle. Then if the temperature
(a) –1/5 (b) 1/5 remains constant, the final pressure in the container will be
(c) 2/5 (d) None of these (in MPa)
(a) 0.70 (b) 0.98 (c) 1.40 (d) 210
KINETIC THEORY 181

ANSWER KEYS
Exercise 1 : Topic -wise MCQs
1 (c) 11 (c) 21 (d) 31 (b) 41 (a) 51 (a) 61 (a) 71 (b) 81 (c) 91 (c)
2 (b) 12 (d) 22 (d) 32 (b) 42 (a) 52 (d) 62 (a) 72 (b) 82 (b) 92 (b)
3 (a) 13 (a) 23 (c) 33 (d) 43 (c) 53 (a) 63 (a) 73 (a) 83 (a) 93 (b)
4 (a) 14 (b) 24 (a) 34 (c) 44 (c) 54 (b) 64 (c) 74 (b) 84 (c) 94 (d)
5 (c) 15 (c) 25 (d) 35 (c) 45 (d) 55 (b) 65 (b) 75 (c) 85 (c) 95 (b)
6 (c) 16 (a) 26 (b) 36 (b) 46 (a) 56 (b) 66 (a) 76 (c) 86 (c) 96 (c)
7 (d) 17 (c) 27 (d) 37 (c) 47 (d) 57 (d) 67 (b) 77 (c) 87 (d) 97 (a)
8 (a) 18 (a) 28 (a) 38 (a) 48 (c) 58 (a) 68 (a) 78 (a) 88 (c) 98 (a)
9 (a) 19 (b) 29 (a) 39 (c) 49 (d) 59 (c) 69 (d) 79 (a) 89 (c) 99 (a)
10 (b) 20 (a) 30 (a) 40 (a) 50 (a) 60 (a) 70 (d) 80 (b) 90 (d) 100 (a)
Exercise 2 : Exemplar & Past Year MCQs
1 (b) 5 (a) 9 (b) 13 (a) 17 (c) 21 (c) 25 (a) 29 (c)
2 (d) 6 (d) 10 (a) 14 (a) 18 (c) 22 (d) 26 (b) 30 (c)
3 (b) 7 (b) 11 (d) 15 (d) 19 (d) 23 (b) 27 (b) 31 (b)
4 (c) 8 (d) 12 (a) 16 (b) 20 (b) 24 (b) 28 (a) 32 (b)
Exercise 3 : Try If You Can
1 (d) 3 (b) 5 (b) 7 (c) 9 (b) 11 (c) 13 (a) 15 (a) 17 (b) 19 (d)
2 (d) 4 (c) 6 (d) 8 (a) 10 (d) 12 (d) 14 (c) 16 (a) 18 (a) 20 (b)
EBD_7418
182 PHYSICS
14Chapter
OSCILLATIONS

Trend
Analysis of NEET and AIIMS (Year 2010-2018)
5

4
Number of Questions

3
AIPMT/NEET
2 AIIMS

0
2010 2011 2012 2013 2014 2015 2016 2017 2018
Year

Trend
Analysis of JEE Main and BITSAT (Year 2010-2018)
4

3
Number of Questions

JEE Main
2
BITSAT

0
2010 2011 2012 2013 2014 2015 2016 2017 2018
Year

Chapter Utility Score (CUS)


Exam Weightage Important Concepts Difficulty Level CUS (Out of 10)
NEET 3 Simple Harmonic motion, Velocity,
AIIMS 5 Acceleration & Energy in SHM, 4/5 7.5/10
JEE Main 4 System executing SHM – Simple
BITSAT 4 Pendulum & Compound Pendulum
EBD_7418
184 PHYSICS
OSCILLATIONS 185
EBD_7418
186 PHYSICS

Exercise 1 : Topic-wise MCQs

12. Which of the following is a simple harmonic motion?


Topic 1: Displacement, Acceleration, Velocity and Phase in SHM
(a) Particle moving through a string fixed at both ends.
(b) Wave moving through a string fixed at both ends.
1. The equation of SHM is y = a Sin (2 pnt + a), then its phase (c) Earth spinning about its axis.
at time ‘t’ is (d) Ball bouncing between two rigid vertical walls.
(a) a (b) 2pnt 13. A particle moves on the X-axis according to the equation
(c) 2pnt + a (d) 2pt x = A cos (wt + kx) + B sin (wt + kx). The motion is simple
2. In SHM, the acceleration is directly proportional to harmonic with amplitude
(a) time (b) linear velocity (a) A (b) B (c) A + B (d) A 2 + B 2
(c) displacement (d) frequency Velocity
3. The displacement of a particle in simple harmonic motion in 14. The graph shown in figure represents
one time period is [A = amplitude] (a) S.H.M.
(a) A (b) 2 A (c) 4 A (d) Zero (b) circular motion
4. For a body executing simple harmonic motion, which –a O +a Displacement
(c) rectillinear motion
parameter comes out to be non-periodic ?
(d) uniform circular motion
(a) Displacement (b) Velocity
(c) Acceleration (d) None of these 15. Suppose a tunnel is dug along a diameter of the earth. A
5. A system exhibiting SHM must possess particle is dropped from a point, a distance h directly above
(a) inertia only the tunnel, the motion of the particle is
(b) elasticity as well as inertia (a) simple harmonic (b) parabolic
(c) elasticity, inertia and an external force (c) oscillatory (d) non-periodic
(d) elasticity only 16. The graph plotted between the velocity and displacement
6. A particle moves in a circular path with a uniform speed. Its from mean position of a particle executing SHM is
motion is (a) circle (b) ellipse
(a) periodic (b) oscillatory
(c) parabola (d) straight line
(c) simple harmonic (d) angular simple harmonic
7. The displacement of a particle is given by 17. The acceleration of a particle undergoing SHM is graphed
r r r in figure. At point 2 the velocity of the particle is
r = A( i cos wt + j sin wt ) . The motion of the particle is
(a) simple harmonic (a) zero
(b) on a straight line (b) negative
(c) on a circle (c) positive
(d) with constant acceleration. (d) None of these
8. Which of the following quantities are always negative in a
simple harmonic motion ? 18. A particle of mass 1 kg is moving in S.H.M. with an amplitude
rr rr
(a) F. r . (b) v.r . 0.02 and a frequency of 60 Hz. The maximum force acting on
rr the particle is
(c) a.r. (d) Both (a) & (c)
(a) 144 p2 (b) 188 p2
9. The motion which is not simple harmonic is
(c) 288 p2 (d) None of these
(a) vertical oscillations of a spring
19. A particle starts with S.H.M. from the mean position as
(b) motion of simple pendulum
shown in figure below. Its amplitude is A and its time period
(c) motion of a planet around the Sun is T. At one time, its speed is half that of the maximum speed.
(d) oscillation of liquid column in a U-tube What is the displacement at that time ?
10. Select the incorrect statement(s) from the following.
I. A simple harmonic motion is necessarily periodic. 2A 3A
(a) (b)
II. A simple harmonic motion may be oscillatory 3 2
III. An oscillatory motion is necessarily periodic
(a) I only (b) II and III 2A 3A
(c) (d)
(c) I and III (d) I and II 3 2
11. The phase difference between the instantaneous velocity
20. Match the column I and column II.
and acceleration of a particle executing simple harmonic
motion is Column I Column II
(a) p (b) 0.707p (c) zero (d) 0.5p (A) Max. positive (1) 0
displacement
OSCILLATIONS 187

T p
(B) Max. positive velocity (2) (a) -p < f < -
2
2
3p
(C) Min. negative (b) p < f <
T 2
acceleration (3) 3p
4 (c) – < f < –p
2
(D) Max. positive (4) T p
(d) <f<p
acceleration 2
27. A body executing linear simple harmonic motion has a
3T
(E) Zero displacement (5) velocity of 3 m/s when its displacement is 4 cm and a velocity
4 of 4 m/s when its displacement is 3 cm. What is the amplitude
(a) (A) - (2), (B) - (4), (C) - (3), (D) - (5), (E) - (1) of oscillation ?
(b) (A) - (1), (B) - (4), (C) - (5), (D) - (3), (E) - (2) (a) 5 cm (b) 7.5 cm (c) 10 cm (d) 12.5 cm
(c) (A) - (3), (B) - (1, 4), (C) - (3), (D) - (5), (E) - (1, 2, 4) 28. The ratio of maximum acceleration to maximum velocity in a
(d) (A) - (1, 3), (B) - (3, 4), (C) - (5), (D) - (1, 2), (E) - (5, 4) simple harmonic motion is 10 s–1. At, t = 0 the displacement
is 5 m. The initial phase is p . What is the maximum
21. A graph of the square of the velocity against the square of
the acceleration of a given simple harmonic motion is 4
acceleration?
v2 v2
(a) 500 m/s2 (b) 500 2 m/s2
(a) (b) (c) 750 m/s 2 (d) 750 2 m/s2
29. The displacement of a particle in SHM is
a2 a2
æ pö
v2 v2 x = 10sin ç 2t - ÷ metre. When its displacement is 6 m,
è 6ø
the velocity of the particle (in m s–1) is
(c) (d) (a) 8 (b) 24 (c) 16 (d) 10
a2 a2 30. Two particles are executing simple harmonic motion of the
same amplitude A and frequency w along the x-axis. Their
22. For a particle moving according to the equation
mean position is separated by distance X0(X0 > A). If the
x = a cos p t, the displacement in 3 s is maximum separation between them is (X0 + A), the phase
(a) 0 (b) 0.5a (c) 1.5a (d) 2a difference between their motion is
23. A body oscillates with SHM according to the equation p p p p
(a) (b) (c) (d)
æ pö 3 4 6 2
(in SI units), x = 5 cos ç 2p t ÷ . 31. Assertion : A particle executing simple harmonic motion
è 4ø
Its instantaneous displacement at t = 1 second is comes to rest at the extreme positions .
Reason : The resultant force on the particle is zero at these
2 1 5 1 positions.
(a) m (b) m (c) m (d) m
3 2 2 (a) Assertion is correct, reason is correct; reason is a
5
correct explanation for assertion.
24. A point mass oscillates along the x-axis according to the law (b) Assertion is correct, reason is correct; reason is not a
x = x0 cos(wt - p / 4) . If the acceleration of the particle is correct explanation for assertion
written as a = A cos(wt + d), then (c) Assertion is correct, reason is incorrect
(a) A = x0w2, d = 3p / 4 (b) A = x0 , d = – p / 4 (d) Assertion is incorrect, reason is correct.
32. The amplitude of a particle executing SHM is 4 cm. At the
(c) A = x0w2, d = p / 4 (d) A = x0w2, d = –p / 4
mean position the speed of the particle is 16 cm/sec. The
25. Assertion : The force acting on a particle moving along
distance of the particle from the mean position at which the
x-axis is F = – k(x + v0t), where k is a constant.
Reason : To an observer moving along x-axis with constant speed of the particle becomes 8 3 cm/s, will be
velocity v0, it represents S.H.M. (a) 2 3 cm (b) 3 cm (c) 1 cm (d) 2 cm
(a) Assertion is correct, reason is correct; reason is a 33. Two simple harmonic motions are represented by the
correct explanation for assertion. p
(b) Assertion is correct, reason is correct; reason is not a equations y1 = 0.1 sin æç100pt + ö÷ and y 2 = 0.1 cos pt .
è 3ø
correct explanation for assertion
The phase difference of the velocity of particle 1 with respect
(c) Assertion is correct, reason is incorrect to the velocity of particle 2 is
(d) Assertion is incorrect, reason is correct.
26. The displacement vs time of a particle executing SHM is p -p p -p
(a) (b) (c) (d)
shown in figure. The initial phase f is 3 6 6 3
EBD_7418
188 PHYSICS

34. Two particles are oscillating along two close parallel straight 42. Starting from the origin a body oscillates simple harmonically
lines side by side, with the same frequency and amplitudes. with a period of 2 s. After what time will its kinetic energy be
They pass each other, moving in opposite directions when 75% of the total energy?
their displacement is half of the amplitude. The mean
1 1 1 1
positions of the two particles lie on a straight line (a) s (b) s (c) s (d) s
perpendicular to the paths of the two particles. The phase 6 4 3 12
difference is 43. A body executes simple harmonic motion. The potential
(a) 0 (b) 2p/3 (c) p (d) p/6 energy (P.E.), the kinetic energy (K.E.) and total energy (T.E.)
are measured as a function of displacement x. Which of the
Topic 2: Energy in Simple Harmonic Motion following statement is true?
(a) P.E. is maximum when x = 0.
35. The total energy of a particle executing S.H.M. is (b) K.E. is maximum when x = 0.
proportional to (c) T.E. is zero when x = 0.
(a) displacement from equilibrium position (d) K.E. is maximum when x is maximum.
(b) frequency of oscillation 44. If < E > and < U > denote the average kinetic and the average
(c) velocity in equilibrium position potential energies respectively of mass describing a simple
(d) square of amplitude of motion harmonic motion, over one period, then the correct relation
36. The potential energy of a particle (Ux) executing S.H.M. is is
given by (a) < E > = < U > (b) < E > = 2< U >
k (c) < E > = –2< U > (d) < E > = – < U >
(a) Ux = (x - a) 2 (b) U x = k1 x + k 2 x 2 + k 3 x 3
2 45. What do you conclude from the graph about the frequency
of KE, PE and SHM ?
(c) U x = A e - bx (d) Ux = a constant Energy
Total energy
37. If a is the amplitude of SHM, then K.E. is equal to the P.E. at
............ distance from the mean position. A B
a KE
a a PE
(a) (b) (c) (d) a
2 2 4
38. Which of the following is true about total mechanical energy 0 t
T/4 2T/4 3T/4 4T/4
of SHM ?
(a) It is zero at mean position. (a) Frequency of KE and PE is double the frequency of
(b) It is zero at extreme position. SHM.
(c) It is always zero. (b) Frequency of KE and PE is four times the frequency
(d) It is never zero. SHM.
39. The total mechanical energy of a harmonic oscillator is given (c) Frequency of PE is double the frequency of K.E.
as (d) Frequency of KE and PE is equal to the frequency of
1 1 2 2 2 SHM.
(a) mw2 A2 (b) m w A 46. A particle is executing simple harmonic motion with amplitude
4 4
A. When the ratio of its kinetic energy to the potential energy
1 1 2 2 2 1
(c) mw2 A2 (d) m w A is , its displacement from its mean position is
2 2 4
40. Assertion : The graph of total energy of a particle in SHM 2 3 3 1
(a) A (b) A (c) A (d) A
w.r.t. position is a straight line with zero slope. 5 2 4 4
Reason : Total energy of particle in SHM remains constant 47. A mass of 4 kg suspended from a spring of force constant
throughout its motion. 800 N m–1 executes simple harmonic oscillations. If the total
(a) Assertion is correct, reason is correct; reason is a energy of the oscillator is 4 J, the maximum acceleration
correct explanation for assertion. (in m s–2) of the mass is
(b) Assertion is correct, reason is correct; reason is not a (a) 5 (b) 15 (c) 45 (d) 20
correct explanation for assertion 48. A particle of mass m executes simple harmonic motion with
(c) Assertion is correct, reason is incorrect amplitude a and frequency n. The average kinetic energy
(d) Assertion is incorrect, reason is correct. during its motion from the position of equilibrium to the end
41. In S.H.M. the ratio of kinetic energy at mean position to the is
potential energy when the displacement is half of the
amplitude is (a) 2p 2 ma 2 n 2 (b) p2 ma 2 n 2
4 2 4 1 1
(a) (b) (c) (d) (c) ma 2 n 2 (d) 4p 2 ma 2 n 2
1 3 3 2 4
OSCILLATIONS 189

49. For a particle executing SHM the displacement x is given by 60. The ratio of energies of oscillations of two exactly identical
x = A cos wt. Identify the graph which represents the variation pendulums oscillating with amplitudes 5 cm and 10 cm is :
of potential energy (P.E.) as a function of time t and (a) 1 : 2 (b) 2 : 1 (c) 1 : 4 (d) 4 : 1
displacement x. 61. A child swinging on swing in sitting position stands up.
The time period of the swing will
(a) increase
(b) decrease
(c) remain same
(d) increase if the child is tall and decrease if the child is
(a) I, III (b) II, IV (c) II, III (d) I, IV short.
50. The total energy of the particle executing simple harmonic 62. A spring-mass system oscillates with a frequency n. If it is
motion of amplitude A is 100 J. At a distance of 0.707 A from taken in an elevator slowly accelerating upward, the
the mean position, its kinetic energy is frequency will
(a) 25 J (b) 50 J (c) 100 J (d) 12.5 J (a) increase (b) decrease
51. When the displacement of a particle executing simple (c) remain same (d) become zero
harmonic motion is half of its amplitude, the ratio of its kinetic 63. Select the correct statement(s) from the following
energy to potential energy is ` I. Motion of a satellite and a planet is periodic as well
(a) 1 : 3 (b) 2 : 1 (c) 3 : l (d) l : 2 as SHM.
II. Mass suspended by a spring executes SHM.
Topic 3: Time Period, Frequency, Simple Pendulum III. Motion of a simple pendulum is always SHM.
(a) I only (b) II only
and Spring Pendulum
(c) I and II (d) I, II and III
52. If a particle takes 0.5 sec to reach position of minimum 64. A simple spring has length l and force constant K. It is cut
velocity from previous such position, then into two springs of lengths l1 and l2 such that l1 = n l2
(a) T = 6 sec, n = 1/6 Hz (b) T = 2 sec, n = 1 Hz (n = an integer). The force constant of spring of length l1 is
(c) T = 3 sec, n = 3 Hz (d) T = 1 sec, n = 1 Hz (a) K (1 + n) (b) (K/n) (1 + n)
53. A simple harmonic motion has an amplitude A and time period (c) K (d) K/(n + 1)
T. The time required by it to travel from x = A to x = A/2 is
65. Frequency of oscillation is proportional to
(a) T/6 (b) T/4 (c) T/3 (d) T/2
54. A particle executes simple harmonic motion between x = – A k 2k
and x = + A. The time taken for it to go from O to A/2 is T1 m
and T1 and to go from A/2 to A is T2. Then
(a) T1 < T2 (b) T1 > T2 (c) T1 = T2 (d) T1 = 2T2 3k k 2k m
55. If the maximum velocity and acceleration of a particle executing (a) (b) (c ) (d)
m m m 3k
SHM are equal in magnitude, the time period will be
(a) 1.57 sec (b) 3.14 sec (c) 6.28 sec (d) 12.56 sec 66. A pendulum is undergoing S.H.M. The velocity of the bob
56. A simple pendulum oscillates in air with time period T and in the mean position is v. If now its amplitude is doubled,
amplitude A. As the time passes keeping the length same, its velocity in the mean position
(a) T and A both decrease will be
(b) T increases and A is constant (a) v/2 (b) v (c) 2 v (d) 4 v
(c) T remains same and A decreases 67. A simple pendulum is made of a body which is a hollow
(d) T decreases and A is constant sphere containing mercury suspended by means of a wire.
57. Which of the following will change the time period as they If a little mercury is drained off, the period of pendulum will
are taken to moon? (a) remain unchanged
(a) A simple pendulum (b) A physical pendulum
(c) A torsional pendulum (d) A spring-mass system (b) increase
58. For an oscillating simple pendulum, the tension in the string is
(a) maximum at extreme position (c) decrease
(b) maximum at mean position
(c) constant throughout the motion (d) become erratic
(d) cannot be predicted 68. The bob of a simple pendulum of mass m and total energy E
59. A simple pendulum is set into vibrations. The bob of the will have maximum linear momentum equal to
pendulum comes to rest after some time due to
(a) Air friction 2E
(a) (b) 2mE (c) 2mE (d) mE2
(b) Moment of inertia m
(c) Weight of the bob
(d) Combination of all the above
EBD_7418
190 PHYSICS

69. Choose the correct statement 77. Two pendulums of lengths 1.44 m and 1 m start to swing
(a) Time period of a simple pendulum depends on together. The number of vibrations after which they will
amplitude. again start swinging together is
(b) Time shown by a spring watch varies with acceleration (a) 4 (b) 3 (c) 5 (d) 2
due to gravity g. 78. When two springs A and B with force constants kA and kB
(c) In a simple pendulum time period varies linearly with are stretched by the same force, then the respective ratio of
the length of the pendulum. the work done on them is
(d) The graph between length of the pendulum and time
(a) kB : kA (b) kA : kB (c) kAkB : 1 (d) kB : kA
period is a parabola.
70. A particle of mass 2 kg is moving on a straight line under the 79. Two oscillating simple pendulums with time periods T and
action of force F = (8 – 2x) N. The particle is released at rest 5T
from x = 6m. For the subsequent motion match the following are in phase at a given time. They are again in phase
4
(All the values in the column II are in their S.I. units) after an elapse of time
Column I Column II (a) 4T (b) 3T (c) 6T (d) 5T
(A) Equilibrium position at x (1) p/4 80. The length of a simple pendulum executing simple harmonic
(B) Amplitude of SHM is (2) p/2 motion is increased by 21%. The percentage increase in the
(C) Time taken to go directly (3) 4 time period of the pendulum of increased length is
from x = 2 to x = 4 (a) 11% (b) 21% (c) 42% (d) 10%
(D) Energy of SHM is (4) 2 81. A particle is executing SHM along a straight line. Its velocities
(a) (A) - (3), (B) - (4), (C) - (2), (D) - (3) at distances x1 and x2 from the mean position are V1 and V2,
(b) (A) - (4), (B) - (3), (C) - (2), (D) - (1) respectively. Its time period is
(c) (A) - (1), (B) - (2), (C) - (3), (D) - (4)
(d) (A) - (2), (B) - (4), (C) - (1), (D) - (3) x 22 – x12 V12 + V22
(a) 2p (b) 2p
71. Assertion: If amplitude of simple pendulum increases then V12 – V22 x12 + x 22
the motion of pendulum is oscillatory but not simple harmonic.
Reason: For larger amplitude q is large and then sin q ¹ q, V12 – V22 x 2 – x 22
(c) 2p 2 2
(d) 2p 12
so the motion is no longer SHM. x1 – x 2 V1 – V22
(a) Assertion is correct, reason is correct; reason is a 82. A block of mass m is projected towards a spring with velocity
correct explanation for assertion. v0. The force constant of the spring is k. The block is
(b) Assertion is correct, reason is correct; reason is not a projected from a distance l from the free end of the spring.
correct explanation for assertion The collision between block and the wall is completely
(c) Assertion is correct, reason is incorrect elastic. Match the following columns :
(d) Assertion is incorrect, reason is correct.
72. A particle executing simple harmonic motion covers a k
v0
distance equal to half of its amplitude in one second. Then
the time period of the simple harmonic motion is l
(a) 4 s (b) 6 s (c) 8 s (d) 12 s Column I Column II
73. The maximum velocity of a particle, executing simple kv 02
harmonic motion with an amplitude 7 mm, is 4.4 m/s. The (A) Maximum compression of (1) –
the spring m
period of oscillation is
(a) 0.01 s (b) 10 s (c) 0.1 s (d) 100 s mv0 2
(B) Energy of oscillations of block (2)
74. A body is executing S.H.M. When its displacement from the k
mean position are 4 cm and 5 cm, it has velocities 10 cm s –1 1
mv 02
(C) Time period of oscillations (3)
and 8 cm s–1 respectively. Its periodic time is 2
(a) p/2 s (b) p s (c) 3 p/2 s (d) 2 p s é 2l mù
(D) Maximum acceleration of the (4) ê +p ú
75. The displacement of an object attached to a spring and v
ë 0 k û
executing simple harmonic motion is given by x = 2 × 10–2 block
cos pt metre.The time at which the maximum speed first (a) (A) - (2), (B) - (3), (C) - (1), (D) - (4)
occurs is (b) (A) - (2), (B) - (3), (C) - (4), (D) - (1)
(a) 0.25 s (b) 0.5 s (c) 0.75 s (d) 0.125 s (c) (A) - (1), (B) - (4), (C) - (3), (D) - (2)
76. A particle at the end of a spring executes S.H.M with a (d) (A) - (1), (B) - (2), (C) - (3), (D) - (4)
period t1 , while the corresponding period for another spring 83. A simple harmonic wave having an amplitude a and time
is t2. If the period of oscillation with the two springs in period T is represented by the equation y = 5sin p (t + 4)m .
series is T then Then the value of amplitude (a) in (m) and time period (T) in
-1 -1 -1 second are
(a) T = t1 + t 2 (b) T 2 = t12 + t 22 (a) a = 10, T = 2 (b) a = 5, T = 1
(c) T = t1 + t2 (d) T -2 = t1-2 + t -22 (c) a = 10, T = 1 (d) a = 5, T = 2
OSCILLATIONS 191

84. What will be the force constant of the spring 93. A simple harmonic oscillator of angular frequency 2 rad s–1
system shown in figure? is acted upon by an external force F = sin t N. If the oscillator
k1 k1
-1 is at rest in its equilibrium position at t = 0, its position at
k1 é 1 1 ù later times is proportional to
(a) + k2 (b) ê + ú
2 ë 2k1 k 2 û 1 1
-1 k2 (a) sin t + cos 2t (b) cos t - sin 2t
é 1 1 ù é2 1 ù 2 2
(c) ê 2k + k ú (d) ê + ú 1 1
ë 1 2û ë k1 k 2 û (c) sin t - sin 2t (d) sin t + sin 2t
2 2
85. A particle of mass m oscillates with a potential energy 94. A cylindrical block of wood (density = 650 kg m–3), of
U = U0 + a x2, where U0 and a are constants and x is the base area 30cm2 and height 54 cm, floats in a liquid of
displacement of particle from equilibrium position. The time
period of oscillation is density 900 kg m–3. The block is depressed slightly and
then released. The time period of the resulting oscillations
m m 2m m of the block would be equal to that of a simple pendulum
(a) 2p (b) 2p (c) p (d) 2p 2 of length (nearly)
a 2a a a (a) 52 cm (b) 65 cm (c) 39 cm (d) 26 cm
86. The height of liquid column in a U tube is 0.3 m. If the liquid 95. A body executes simple harmonic motion under the action
in one of the limbs is depressed and then released, then the 4
time period of the liquid column will be of a force F1 with a time period s. If the force is changed
(a) 1.1 sec (b) 19 sec (c) 0.11 sec (d) 2 sec 5
3
87. A block rests on a horizontal table which is executing SHM to F2, it executes S.H.M. with time period s. If both the
in the horizontal plane with an amplitude 'a'. If the coefficient 5
forces F1 and F2 act simultaneously in the same direction
of friction is 'm', then the block just starts to slip when the on the body, its time period in second is
frequency of oscillation is 12 7 24 5
a (a) (b) (c) (d)
1 mg mg a 25 5 25 7
(a) (b) (c) 2p (d) m g 96. If the differential equation for a simple harmonic motion is
2p a a mg
88. On earth, a body suspended on a spring of negligible mass d 2 y2
+ 2y = 0, the time-period of the motion is
causes extension L and undergoes oscillations along length dt 2
of the spring with frequency f. On the Moon, the same 2s p
(a) p 2s (b) s (c) s (d) 2ps
quantities are L/n and f ' respectively. The ratio f '/f is p 2
1 97. Time period of a simple pendulum attached with the celling
(a) n (b) (c) n –1/2 (d) 1 of a lift falling freely is
n
89. A second’s pendulum is placed in a space laboratory orbiting (a) 0 (b) ¥
around the earth at a height 3 R from the earth’s surface (c) 1 sec (d) None of these
where R is earth’s radius. The time period of the pendulum 98. A pendulum made of a uniform wire of cross sectional area
will be A has time period T. When an additional mass M is added to
its bob, the time period changes to TM. If the Young's
(a) zero (b) 2 3 (c) 4 sec (d) infinite
1
90. A bent tube of uniform cross-section area A has a non- modulus of the material of the wire is Y then is equal to:
viscous liquid of density r. The mass of liquid in the tube is Y
(g = gravitational acceleration)
m. The time period of oscillation of the liquid is
é æ T ö2 ù A é æ T ö2 ù A
m m ê1 - ç ÷ ú
M
(a) 2p (b) 2p (a) ê1 - ç ÷ ú (b)
rgA 2rgA ë è T ø û Mg êë è TM ø úû Mg
30° 30°
2m éæ T ö2 ù A éæ T ö2 ù Mg
(c) 2p (d) None of these (c) êç
M
÷ - 1ú (d) êç M ÷ - 1ú
rgA ëè T ø û Mg ëè T ø û A
91. The bob of a simple pendulum executes simple harmonic 99. Masses MA and MB hanging from the ends of strings of
motion in water with a period t, while the period of oscillation lengths LA and LB are executing simple harmonic motions. If
of the bob is t0 in air. Neglecting frictional force of water and their frequencies are fA = 2fB, then
given that the density of the bob is (4/3) × 1000 kg / m3. (a) LA = 2LB and MA = MB/2
What relationship between t and t0 is true (b) LA = 4LB regardless of masses
(a) t = 2t0 (b) t = t0 / 2 (c) t = t0 (d) t = 4t0
(c) LA = LB/4 regardless of masses
92. A wall clock uses a vertical spring-mass system to measure
(d) LA = 2LB and MA = 2MB
the time. Each time the mass reaches an extreme position, the
100. A simple pendulum has time period T1. The point of
clock advances by a second. The clock gives correct time at
suspension is now moved upward according to the relation
the equator. If the clock is taken to the poles it will
y = Kt2, (K = 1 m/s2) where y is the vertical displacement.
(a) run slow (b) run fast
(c) stop working (d) give correct time T12
The time period now becomes T2. The ratio of is
T22
(g = 10 m/s2)
EBD_7418
192 PHYSICS

(a) 5/6 (b) 6/5 (c) 1 (d) 4/5 (a) free oscillations (b) coupled oscillations
101. A U-tube is of non uniform cross-section. The area of cross- (c) forced oscillations (d) maintained oscillations
sections of two sides of tube are A and 2A (see fig.). It 107. In case of a forced vibration, the resonance wave becomes
contains non-viscous liquid of mass m. The liquid is very sharp when the
displaced slightly and free to oscillate. Its time period of (a) quality factor is small
oscillations is A 2A A 2A
(b) damping force is small
(c) restoring force is small
y/2 (d) applied periodic force is small
y 108. What is the amplitude of simple harmonic motion at
m resonance in the ideal case of zero damping?
(a) Zero (b) – 1 (c) 1 (d) Infinite
m m
r
(a) T = 2p T = 2p 109. A particle oscillating under a force F = - kx - bu is a (k and
3rgA
(b) 2rgA b are constants)
m (a) simple harmonic oscillator
(c) T = 2p (d) None of these
rgA (b) non linear oscillator
102. A rod of mass M and length L is hinged at its centre of mass (c) damped oscillator
so that it can rotate in a vertical plane. Two springs each of (d) forced oscillator
stiffness k are connected at its ends, as shown in the figure. 110. In case of sustained forced oscillations the amplitude of
The time period of SHM is oscillations
(a) decreases linearly
M M
(a) 2 p (b) 2 p (b) decreases sinusoidally
6k 3k
(c) decreases exponentially
(d) always remains constant
ML M 111. The frequency of the simple harmonic motion attained in
(c) 2 p (d) p
k 6k forced oscillations, after the forced oscillation die out, is
103. Four massless springs whose force constants are 2k, 2k, k (a) the natural frequency of the particle
and 2k respectively are attached to a mass M kept on a (b) the frequency of the driving force
frictionless plane (as shown in figure). If the mass M is (c) double the frequency of the driving force
displaced in the horizontal direction, then the frequency of (d) double the natural frequency of the particle
the system is 112. Column I Column II
d2y d2y
1 k
2k
k (A) = v2 dx 2 (1) Resonant vibration
(a) 2k dt 2
2p 4M M
d2y
2k (B) + w2y = 0 (2) Free vibration
dt 2
1 4k 1 k 1 7k d2y dy
(b) (c) (d) (C) + 2k + w2y = 0 (3) Damped vibration
2p M 2p 7M 2p M dt 2 dt
104. The bob of a simple pendulum executes simple harmonic d2y dy
motion in water with a period t, while the period of oscillation (D) + 2k + w2y (4) Forced vibration
dt 2 dt
of the bob is t0 in air. Neglecting frictional force of water and = F sin pt
(5) Progressive wave
given that the density of the bob is ( 4 / 3 ) ´ 1000 kg / m .3
(a) (A) - (1), (B) - (3), (C) - (2, 4), (D) - (5)
What relationship between t and t0 is true? (b) (A) - (1, 3), (B) - (2, 5), (C) - (3), (D) - (4, 5)
(a) t = t0 (b) t = t0 / 2 (c) t = 2t0 (d) t = 4t0 (c) (A) - (5), (B) - (2), (C) - (3), (D) - (1, 4)
105. A small ball of density 4r0 is released from rest just below (d) (A) - (1), (B) - (2), (C) - (3), (D) - (4)
the surface of a liquid. The density of liquid increases with 113. Choose the false statement(s) for a forced oscillation.
depth as r = r0 (1 + ay) where a = 2m–1 is a constant. Find I. Displacement amplitude of an oscillator is independent
the time period of its oscillation. (Neglect the viscosity of the angular frequency of the driving force.
effects). II. The amplitude tends to infinity when the driving
2p p 3p frequency equals the natural frequency.
p
(a) sec (b) sec (c) sec (d) sec III. Maximum possible amplitude for a given driving
5 5 2 5 2 5 frequency is governed by the driving frequency and
Topic 4: Free, Damped, Forced Oscillations and Resonance the damping.
(a) I only (b) II only
106. If a body oscillates at the angular frequency wd of the driving (c) I and II (d) I, II and III
force, then the oscillations are called
OSCILLATIONS 193

114. Assertion: Amplitude of a forced vibration can never reach 121. Column I Column II
infinity. (A) Motion of a satellite (1) Damped oscillations
Reason: The driving frequency cannot be increased beyond (B) Motion of a simple (2) Resonant oscillations
a certain limit. pendulum
(a) Assertion is correct, reason is correct; reason is a (C) Oscillation of (3) Periodic motion
correct explanation for assertion. stretched string in air
(b) Assertion is correct, reason is correct; reason is not a
(D) Flying off of a paper (4) Simple harmonic
correct explanation for assertion
(c) Assertion is correct, reason is incorrect rider placed on the motion
(d) Assertion is incorrect, reason is correct. stretched string
115. A block connected to a spring oscillates vertically. A damping (a) (A) - (2), (B) - (3), (C) - (4), (D) - (1)
force Fd , acts on the block by the surrounding medium. (b) (A) - (3), (B) - (2), (C) - (4), (D) - (1)
Given as Fd = – bV b is a positive constant which depends (c) (A) - (1), (B) - (3), (C) - (2), (D) - (4)
on : (d) (A) - (3), (B) - (4), (C) - (1), (D) - (2)
(a) viscosity of the medium 122. A particle, with restoring force proportional to displacement
(b) size of the block and resisting force proportional to velocity is subjected to a
(c) shape of the block force F sin wt. If the amplitude of the particle is maximum for
(d) All of these w = w1 and the energy of the particle is maximum for
rd w = w2, then
æ 1ö
116. The amplitude of a damped oscillator becomes ç ÷ in 2
è 3ø (a) w1 = w0 and w2 ¹ w0 (b) w1 = w0 and w2 = w0
1 (c) w1 ¹ w0 and w2 = w0 (d) w1 ¹ w0 and w2 ¹ w0
seconds. If its amplitude after 6 seconds is times the
n 123. A particle of mass m is attached to a spring (of spring
original amplitude, the value of n is constant k) and has a natural angular frequency w0. An
(a) 32 (b) 33 (c) 3 3 (d) 23 external force F(t) proportional to cos wt(w ¹ w0) is applied
117. When an oscillator completes 100 oscillations its amplitude to the oscillator. The displacement of the oscillator will be
1 proportional to
reduces to of its initial value. What will be its amplitude, 1 1
3
when it completes 200 oscillations? (a) 2 2 (b) m (w 2 - w 2 )
m (w 0 + w ) 0
1 2 1 1
(a) (b) (c) (d) m m
8 3 6 9 (c) 2 2 (d) 2 2
118. A forced oscillator is acted upon by a force F = F0 sin wt. w0 -w (w 0 +w )
124. The amplitude of velocity of a particle is given by, Vm = V0/
55
The amplitude of oscillation is given by . (aw2 – bw + c) where V0, a, b and c are positive :
2w 2 - 36w + 9 The condition for a single resonant frequency is
The resonant angular frequency is
(a) 2 unit (b) 9 unit (c) 18 unit (d) 36 unit (a) b2 < 4ac (b) b2 = 4ac
119. A pendulum with time period of 1s is losing energy. At (c) b2 = 5ac (d) b2 = 7ac
certain time its energy is 45 J. If after completing 15 125. If a simple pendulum has significant amplitude (upto a factor
oscillations, its energy has become 15 J, its damping of 1/e of original) only in the period between t = 0s to t = ts,
constant (in s–1) is :
then t may be called the average life of the pendulum. when
1 1 1 the spherical bob of the pendulum suffers a retardation (due
(a) (b) ln3 (c) 2 (d) ln3
2 30 15 to viscous drag) proportional to its velocity with b as the
120. In damped oscillations, the amplitude of oscillations is constant of proportionality, the average life time of the
reduced to one-third of its inital value a0 at the end of 100 pendulum is (assuming damping the small) in seconds
oscillations. When the oscillator completes 200 oscillations, 0.693 1 2
its amplitude must be (a) (b) b (c) (d)
b b b
(a) a0/2 (b) a0/4 (c) a0/6 (d) a0/9
EBD_7418
194 PHYSICS

Exercise 2 : Exemplar & Past Year MCQs


8. Figure shows the circular motion of a particle. The radius of
NCERT Exemplar MCQs
the circle, the period, sense of revolution and the initial
1. The displacement of a particle is represented by the equation position are indicated on the figure. The simple harmonic
æp ö motion of the x-projection of the radius vector of the rotating
y = 3cos ç - 2wt ÷ . The motion of the particle is particle P is y
è 4 ø
(a) simple harmonic with period 2p/w æ 2pt ö p(t = 0)
(a) x(t ) = B sin ç ÷ T = 30s
(b) simple harmonic with period p/w è 30 ø
(c) periodic but not simple harmonic æ pt ö B
(b) x(t ) = B cos ç ÷ x
(d) non-periodic è 15 ø
2. The displacement of a particle is represented by the equation æ pt p ö
y = sin3 wt. The motion is (c) x(t ) = B sin ç + ÷
(a) non-periodic è 15 2 ø
(b) periodic but not simple harmonic æ pt p ö
(d) x(t ) = B cos ç + ÷
(c) simple harmonic with period 2p/w è 15 2 ø
(d) simple harmonic with period p/w 9. The equation of motion of a particle is x = a cos(at)2. The
3. The relation between acceleration and displacement of four motion is
particles are given below. Which, one of the particle is (a) periodic but not oscillatory
exempting simple harmonic motion?
(a) ax = +2x (b) ax = +2x2 (b) periodic and oscillatory
(c) ax = –2x 2 (d) ax = –2x (c) oscillatory but not periodic
4. Motion of an oscillating liquid column in a U-tube is (d) neither periodic nor oscillatory
(a) periodic but not simple harmonic 10. A particle executing SHM maximum speed of 30 cm/s and a
(b) non-periodic maximum acceleration of 60 cm/s2. The period of oscillation is
(c) simple harmonic and time period is independent of the p p
density of the liquid (a) p sec (b) sec (c) 2p sec (d) sec
2 t
(d) simple harmonic and time period is directly proportional
to the density of the liquid Past Year MCQs
5. A particle is acted simultaneously by mutually perpendicular 11. The oscillation of a body on a smooth horizontal surface is
simple harmonic motion x = a cos wt and y = a sin wt. The represented by the equation,
trajectory of motion of the particle will be X = A cos (wt)
(a) an ellipse (b) a parabola where X = displacement at time t
(c) a circle (d) a straight line w = frequency of oscillation
6. The displacement of a particle varies with time according to Which one of the following graphs shows correctly the
the relation y = a sin wt + b cos wt. variation of ‘a’ with ‘t’? [AIPMT 2014, C]
(a) The motion is oscillatory but not SHM
a a
(b) The motion is SHM with amplitude a + b O O
(a) T t (b) T t
(c) The motion is SHM with amplitude a2 + b2
(d) The motion is SHM with amplitude a 2 + b2 a a
7. Four pendulums A, B, C and D are O O
(c) (d)
suspended from the same elastic G G T t T t
support as shown in figure. A and C are
of the same length, while B is smaller é pt ù
12. y = 2 (cm) sin ê + fú what is the maximum acceleration of
than A and D is larger than A. If A is ë 2 û
given a transverse displacement, C B A the particle doing the S.H.M. [AIIMS, 2014, C]
D
(a) D will vibrate with maximum amplitude 2
p p
(b) C will vibrate with maximum amplitude (a) cm/s2 (b) cm/s2
2 2
(c) B will vibrate with maximum amplitude p2 p
(d) All the four will oscillate with equal amplitude (c) cm/s2 (d) cm/s2
4 4
OSCILLATIONS 195

13. Resonance is an example of [AIIMS 2014, C] E KE E


(a) tuning fork (b) forced vibration PE
(c) free vibration (d) damped vibration
(a) d (b) KE
14. Assertion : In simple harmonic motion, the velocity is
maximum when the acceleration is minimum.
Reason : Displacement and velocity of S.H.M. differ in phase PE
p E E PE
by . [AIIMS 2014, C] KE
2
(a) Assertion is correct, reason is correct; reason is a (c) PE (d) KE
correct explanation for assertion. d d

(b) Assertion is correct, reason is correct; reason is not a


20. The displacement of a particle is given at time t, by:
correct explanation for assertion
(c) Assertion is correct, reason is incorrect x = A sin( -2w t ) + B sin 2 w t Then, [BITSAT 2015, C]
(d) Assertion is incorrect, reason is correct. (a) the motion of the particle is SHM with an amplitude of
15. A particle moves with simple harmonic motion in a straight
line. In first ts, after starting from rest it travels a distance a, B2
A2 +
and in next ts it travels 2a, in same direction, then: 4
[JEE Main 2014, C]
(b) the motion of the particle is not SHM, but oscillatory
(a) amplitude of motion is 3a
with a time period of T = p/w
(b) time period of oscillations is 8t
(c) the motion of the particle is oscillatory with a time period
(c) amplitude of motion is 4a
of T = p/2w
(d) time period of oscillations is 6t
(d) the motion of the particle is a periodic.
16. At the corners of an equilateral triangle of side a (1 metre),
21. A load of mass m falls from a height h on to the scale pan
three point charges are placed (each of 0.1 C). If this system
hung from the spring as shown in the figure. If the spring
is supplied energy at the rate of 1 kw, then calculate the constant is k and mass of the scale pan is zero and the mass
time required to move one of the mid-point of the line joining m does not bounce relative to the pan, then the amplitude
the other two. [BITSAT 2014, A] of vibration is [BITSAT 2015, S]
A (a) mg/d
mg æ 1 + 2hk ö K
(b) k çè mg ÷ø

a a
mg mg æ 1 + 2hk ö
+
k çè mg ÷ø
(c) k m
mg æ 1 + 2hk mg ö h
D -
k çè mg k ÷ø
(d)
B a C
22. The period of oscillation of a mass M suspended from a
(a) 50 h (b) 60 h (c) 48 h (d) 54 h spring of negligible mass is T. If along with it another mass
17. A particle is executing a simple harmonic motion. Its maximum M is also suspended, the period of oscillation will now be
acceleration is a and maximum velocity is b. (a) T (b) T / 2 [AIIMS 2016, A]
Then its time period of vibration will be : [AIPMT 2015, A] (c) 2T (d) 2T
23. A particle moves with simple harmonic motion in a straight
a b2 2pb b2
(a) (b) (c) (d) line. In first t s, after starting from rest, it travels a distance
b a a a2 a, and in next t s, it travels 2a in same direction then
18. Two particles are executing S.H.M. of same amplitude and [AIIMS 2016, A]
frequency along the same straight line path. They pass each (a) amplitude of motion is 3a
other when going in opposite directions, each time their (b) time period of oscillations is 8t.
displacement is half of their amplitude. What is the phase (c) amplitude of motion is 4a.
difference between them ? [AIIMS 2015, A] (d) time period of oscillations is 6t.
(a) 5 p/6 (b) 2 p/3 24. A particle performs simple harmonic mition with amplitude
(c) p/3 (d) p/6 A. Its speed is trebled at the instant that it is at a distance
19. For a simple pendulum, a graph is plotted between its kinetic
energy (KE) and potential energy (PE) against its displacement 2A
from equilibrium position. The new amplitude of the
d. Which one of the following represents these correctly? 3
(graphs are schematic and not drawn to scale) motion is : [JEE Main 2016, A]
[JEE Main 2015, C] 7A A
(a) A 3 (b) (c) 41 (d) 3A
3 3
EBD_7418
196 PHYSICS

25. A pendulum clock loses 12 s a day if the temperature is 40°C


and gains 4 s a day if the temperature is 20° C. The
(a) (b)
temperature at which the clock will show correct time, and
the co-efficient of linear expansion (a) of the metal of the
pendulum shaft are respectively : [JEE Main 2016, S]
(a) 30°C; a = 1.85 × 10–3/°C (c) (d)
(b) 55°C; a = 1.85 × 10–2/°C
32. The following figure depict a
(c) 25°C; a = 1.85 × 10–5/°C y
(d) 60°C; a = 1.85 × 10–4/°C circular motion. The radius of T = 4s P(t = 0)
26. A 1 kg mass is attached to a spring of force constant the circle, the period of a45°
x
600 N/m and rests on a smooth horizontal surface with other revolution, the initial position
end of the spring tied to wall as shown in figure. A second and the sense of revolution
mass of 0.5 kg slides along the surface towards the first at are indicated on the figure.
3m/s. If the masses make a perfectly inelastic collision, then The simple harmonic motion of the x-projection of the radius
find amplitude and time period of oscillation of combined vector of the rotating particle P can be shown as :
mass. [BITSAT 2016, S]
[BITSAT 2017, A]
///////////////

æ 2p t p ö æ pt p ö
(a) x(t ) = a cos ç + ÷ (b) x(t ) = a cos ç + ÷
3m/s è 4 4ø è 4 4ø
0.5kg 1 kg
//////////////////////////////////////////////////// (c) æ 2p t p ö (d) x(t ) = a cos æ p t + p ö
x(t ) = a sin ç + ÷ çè ÷
è 4 4ø 3 2ø
p p
(a) 5cm, s (b) 5cm, s 33. A pendulum is hung from the roof of a sufficiently high
10 5 building and is moving freely to and fro like a simple
2p p harmonic oscillator. The acceleration of the bob of the
(c) 4cm, s (d) 4cm, s pendulum is 20 m/s2 at a distance of 5 m from the mean
5 3
position. The time period of oscillation is [NEET 2018, A]
27. A point particle of mass 0.1 kg is executing S.H.M. of
(a) 2p s (b) p s (c) 1 s (d) 2 s
amplitude of 0.1 m. When the particle passes through the 34. A particle performs SHM on x-axis with amplitude A and
mean position, its kinetic energy is 8 × 10–3 Joule. Obtain time period T. The time taken by the particle to travel a
the equation of motion of this particle if this initial phase of
A
oscillation is 45º. [BITSAT 2016, A] distance starting from rest is [AIIMS 2018, A]
5
æ pö æ pö
(a) y = 0.1sin ç ±4t + ÷ (b) y = 0.2sin ç ±4t + ÷ T T æ4ö
è 4 ø è 4ø (a) (b) cos-1 ç ÷
20 2p è5ø
æ pö æ pö
(c) y = 0.1sin ç ±2t + ÷ (d) y = 0.2sin ç ±2t + ÷
è 4ø è 4ø T æ1ö T æ1ö
(c) cos-1 ç ÷ (d) sin -1 ç ÷
28. A particle executes linear simple harmonic motion with an 2p è5ø 2p è5ø
amplitude of 3 cm. When the particle is at 2 cm from the
35. A particle of mass is executing oscillations about the origin
mean position, the magnitude of its velocity is equal to that
on the x-axis. Its potential energy is V(x) = k | x |3, where k is
of its acceleration. Then its time period in seconds is
[NEET 2017, A] a positive constant. If the amplitude of oscillation is a, then
its time period T is [AIIMS 2018, S]
5 4p 2p 5 1
(a) (b) (c) (d) (a) proportional to (b) proportional to a
2p 5 3 p a
29. A spring of force constant k is cut into lengths of ratio 3
1 : 2 : 3. They are connected in series and the new force
(c) independent a2 (d) None of these
constant is k'. Then they are connected in parallel and force
36. Assertion : If the amplitude of a simple harmonic oscillator
constant is k¢¢. Then k' : k¢¢ is [NEET 2017, A]
is doubled, its total energy becomes four times.
(a) 1 : 9 (b) 1 : 11 (c) 1 : 14 (d) 1 : 6
Reason : The total energy is directly proportional to the
30. The amplitude of a damped oscillator decreases to 0.9 times
its original magnitude in 5s. In another 10s it will decrease to square of the amplitude of vibration of the harmonic
a times its original magnitude, where a equals oscillator. [AIIMS 2018, C]
[AIIMS 2017, A] (a) Assertion is correct, reason is correct; reason is a
(a) 0.7 (b) 0.81 (c) 0.729 (d) 0.6 correct explanation for assertion.
(b) Assertion is correct, reason is correct; reason is not a
31. A particle is executing simple harmonic motion with a time
correct explanation for assertion
period T. At time t = 0, it is at its position of equilibrium. The (c) Assertion is correct, reason is incorrect
kinetic energy-time graph of the particle will look like: (d) Assertion is incorrect, reason is correct.
[JEE Main 2017, C]
OSCILLATIONS 197

37. A silver atom in a solid oscillates in simple harmonic motion 38. Two oscillators are started simultaneously in same phase.
in some direction with a frequency of 1012/sec. What is the After 50 oscillations of one, they get out of phase by p, that
force constant of the bonds connecting one atom with the is half oscillation. The percentage difference of frequencies
other? (Mole wt. of silver = 108 and Avagadro number of the two oscillators is nearest to [BITSAT 2018, A]
= 6.02 ×1023 gm mole –1) [JEE Main 2018, S] (a) 2% (b) 1% (c) 0.5% (d) 0.25%
(a) 6.4 N/m (b) 7.1 N/m (c) 2.2 N/m (d) 5.5 N/m

Exercise 3 : Try If You Can


1. A simple pendulum attached to the ceiling of a stationary pendulum of shorter length has completed oscillations
lift has a time period T. The distance y covered by the lift [nT1=(n–1)T2, where T1 is time period of shorter length &
moving upwards varies with time t as y = t 2 where y is in T2 be time period of longer wavelength and n are no. of
metres and t in seconds. If g = 10 m/s2 , the time period of oscillations completed]
pendulum will be (a) 5 (b) 1 (c) 2 (d) 3
4 5 5 6 6. A mass M is suspended from a spring of negligible mass.
(a) T (b) T (c) T (d) T The spring is pulled a little and then released so that the
5 6 4 5
mass executes SHM of time period T. If the mass is increased
2. A simple harmonic motion along the x-axis has the following
5T m
properties : amplitude = 0.5 m, the time to go from one extreme by m, the time period becomes . Then the ratio of is
position to other is 2 s and x = 0.3 m at t = 0.5 s. The general 3 M
25 16 5 3
equation of the simple harmonic motion is (a) (b) (c) (d)
9 9 3 5
é pt ù
(a) x = ( 0.5 m ) sin ê + 8º ú 7. If the mass shown in figure is slightly displaced and then let
ë2 û go, then the system shall oscillate with a time period of
é pt ù
(b) x = ( 0.5 m ) sin ê - 8º ú m
ë2 û (a) 2p
3k k
é pt ù
(c) x = ( 0.5 m ) cos ê + 8º ú 3m
ë2 û (b) 2p
2k
é pt ù
(d) x = ( 0.5 m ) cos ê - 8º ú 2m
ë2 û (c) 2p k k
3. A particle of mass m = 2 kg 3k
executes SHM in xy plane
3k
between points A and B under A (d) 2p m
the action of force (2, 2) m
r 8. A 1 kg block attached to a spring vibrates with a frequency
ˆ ˆ
F= Fx i + Fy j . Minimum time
of 1 Hz on a frictionless horizontal table. Two springs
taken by the particle to move
from A to B is 1 s. At t = 0 the identical to the original spring are attached in parallel to an
B 8 kg block placed on the same table. So, the frequency of
particle is at x = 2 and y = 2. Then (–2, –2)
Fx as function of time t is vibration of the 8 kg block is :
1 1 1
(a) –4p2 sin pt (b) –4p2 cos pt (a) Hz (b) Hz (c) Hz (d) 2 Hz
4 2 2 2
(c) –4p2 cos pt (d) None of these
9. In an engine the piston undergoes vertical simple harmonic
4. Two particles are performing simple harmonic motion in a motion with amplitude 7 cm. A washer rests on top of the
straight line about the same equilibrium point. The amplitude
piston and moves with it. The motor speed is slowly
and time period for both particles are same and equal to A
and T, respectively. At time t=0 one particle h as increased. The frequency of the piston at which the washer
no longer stays in contact with the piston, is close to :
-A
displacement A while the other one has displacement (a) 0.7 Hz (b) 1.9 Hz (c) 1.2 Hz (d) 0.1 Hz
2 10. Two bodies of masses 1 kg and 4 kg are connected to a
and they are moving towards each other. If they cross each
vertical spring, as shown in the figure. The smaller mass
other at time t, then t is:
executes simple harmonic motion of angular frequency 25
5T T T T
(a) (b) (c) (d) rad/s, and amplitude 1.6 cm while the bigger mass remains
6 3 4 6 stationary on the ground. The maximum force exerted by
5. Two simple pendulums of length 0.5 m and 20 m respectively
are given small linear displacement in one direction at the the system on the floor is (take g = 10 ms–2)
same time. They will again be in the phase when the
EBD_7418
198 PHYSICS

(a) 20 N 1 kg m m m
(a) 2p (b) p +p
(b) 10 N k k k/2
(c) 60 N m m m
(c) p (d) p +p
(d) 40 N 4 kg 3k / 2 k 2k
11. A mass is suspended separately by 14. A uniform cylinder of length L and mass M having cross-
two different springs in successive sectional area A is suspended, with its length vertical, from
order then time periods is t1 and t2 a fixed point by a massless spring, such that it is half
respectively. It is connected by both k1 k2 submerged in a liquid of density s at equilibrium position.
springs as shown in fig. then time When the cylinder is given a downward push and released,
period is t0, the correct relation is it starts oscillating vertically with a small amplitude. The
m time period T of the oscillations of the cylinder will be :
1
(a) t 02 = t 12 + t 22 (b) t 0-2 = t 1-2 + t 2-2 é
(a) Smaller than 2p ê
M ù 2
ú
(c) t 0-1 = t1-1 + t -2 1 (d) t 0 = t1 + t 2 ë (k + Asg ) û
12. The angular frequency of the damped oscillator is given by, M
(b) 2p
k
æk r2 ö 1
w= ç - ÷ é M ù 2
ç m 4m2 ÷ where k is the spring constant, m is the (c) Larger than 2p ê ú
è ø ë (k + Asg ) û
mass of the oscillator and r is the damping constant. If the 1
r2 é M ù 2
ratio is 8%, the change in time period compared to the (d) 2p ê ú
mk ë (k + Asg ) û
undamped oscillator is approximately as follows: 15. The amplitude of a simple pendulum, oscillating in air with a
(a) increases by 1% (b) increases by 8% small spherical bob, decreases from 10 cm to 8 cm in 40
(c) decreases by 1% (d) decreases by 8% seconds. Assuming that Stokes law is valid, and ratio of the
13. A mass m is suspended from a spring of coefficient of viscosity of air to that of carbon dioxide is 1.3.
k
force constant k and just touches another The time in which amplitude of this pendulum will reduce
identical spring fixed to the floor as shown in m from 10 cm to 5 cm in carbon dioxide will be close to
the figure. The time period of small (In 5 = 1.601, In 2 = 0.693).
k
oscillations is (a) 231 s (b) 208 s (c) 161 s (d) 142 s

ANSWER KEYS
Exercise 1 : Topic -wise MCQs
1 (c) 14 (a) 27 (a) 40 (a) 53 (a) 66 (c) 79 (d) 92 (d) 105 (a) 118 (b)
2 (c) 15 (c) 28 (b) 41 (a) 54 (a) 67 (b) 80 (d) 93 (c) 106 (c) 119 (d)
3 (d) 16 (b) 29 (c) 42 (a) 55 (c) 68 (b) 81 (a) 94 (c) 107 (b) 120 (d)
4 (d) 17 (a) 30 (a) 43 (b) 56 (c) 69 (d) 82 (b) 95 (a) 108 (d) 121 (d)
5 (b) 18 (c) 31 (c) 44 (a) 57 (a) 70 (a) 83 (d) 96 (a) 109 (c) 122 (c)
6 (a) 19 (b) 32 (d) 45 (a) 58 (b) 71 (a) 84 (b) 97 (b) 110 (a) 123 (b)
7 (c) 20 (c) 33 (b) 46 (a) 59 (a) 72 (d) 85 (b) 98 (c) 111 (b) 124 (b)
8 (d) 21 (d) 34 (b) 47 (d) 60 (c) 73 (a) 86 (a) 99 (c) 112 (c) 125 (d)
9 (c) 22 (d) 35 (d) 48 (b) 61 (b) 74 (b) 87 (a) 100 (b) 113 (a)
10 (b) 23 (c) 36 (a) 49 (a) 62 (c) 75 (b) 88 (d) 101 (a) 114 (c)
11 (d) 24 (a) 37 (a) 50 (b) 63 (b) 76 (b) 89 (d) 102 (a) 115 (d)
12 (b) 25 (a) 38 (d) 51 (c) 64 (b) 77 (c) 90 (a) 103 (b) 116 (b)
13 (d) 26 (a) 39 (c) 52 (d) 65 (a) 78 (a) 91 (a) 104 (c) 117 (d)
Exercise 2 : Exemplar & Past Year MCQs
1 (b) 5 (c) 9 (c) 13 (b) 17 (c) 21 (b) 25 (c) 29 (b) 33 (b) 37 (b)
2 (b) 6 (d) 10 (a) 14 (b) 18 (b) 22 (d) 26 (a) 30 (c) 34 (b) 38 (b)
3 (d) 7 (b) 11 (c) 15 (d) 19 (d) 23 (d) 27 (a) 31 (b) 35 (a)
4 (c) 8 (a) 12 (b) 16 (a) 20 (a) 24 (b) 28 (b) 32 (a) 36 (b)
Exercise 3 : Try If You Can
1 (b) 3 (b) 5 (b) 7 (b) 9 (b) 11 (b) 13 (d) 15 (d)
2 (b) 4 (d) 6 (b) 8 (c) 10 (c) 12 (b) 14 (a)
15Chapter
WAVES

Trend
Analysis of NEET and AIIMS (Year 2010-2018)
5

4
Number of Questions

3
AIPMT/NEET
2 AIIMS

0
2010 2011 2012 2013 2014 2015 2016 2017 2018
Year

Trend
Analysis of JEE Main and BITSAT (Year 2010-2018)

3
Number of Questions

2
JEE Main

BITSAT

0
2010 2011 2012 2013 2014 2015 2016 2017 2018
Year

Chapter Utility Score (CUS)


Exam Weightage Important Concepts Difficulty Level CUS (Out of 10)
NEET 6 Standing & Progressive waves,
AIIMS 5 Principle of superposition, Beats 4.5/5 9.2/10
JEE Main 5 and Doppler’s effect
BITSAT 4
EBD_7418
200 PHYSICS
WAVES 201
EBD_7418
202 PHYSICS

Exercise 1 : Topic-wise MCQs

13. The diagram below shows the propagation of a wave. Which


Topic 1: Basic of Mechanical Waves,
points are in same phase ?
Progressive and Stationary Waves
E
1. When a sound wave goes from one medium to another, the H
quantity that remains unchanged is
A
D
(a) frequency (b) amplitude B
(c) wavelength (d) speed F G
C
2. For which of the following do the longitudinal waves
exist? (a) F and G (b) C and E
(a) Vacuum (b) Air
(c) B and G (d) B and F
(c) Water (d) Both (b) and (c)
3. Sound travels in rocks in the form of 14. The reason for introducing Laplace correction in the
(a) longitudinal elastic waves only expression for the velocity of sound in a gaseous medium is
(b) transverse elastic waves only (a) no change in the temperature of the medium during
(c) both longitudinal and transverse elastic waves the propagation of the sound through it
(d) non-elastic waves (b) no change in the heat of the medium during the
4. From a point source, if amplitude of waves at a distance r is propagation of the sound through it
A, its amplitude at a distance 2r will be (c) change in the pressure of the gas due to the
(a) A (b) 2 A (c) A/2 (d) A/4 compression and rarefaction
5. The property of a medium necessary for wave propagation is (d) change in the volume of the gas
(a) inertia (b) elasticity
15. Which of the following changes at an antinode in a stationary
(c) low resistance (d) All of the above
6. The equation y = A sin2(kx – wt) represents a wave with wave?
(a) amplitude A, frequency w/2p (a) Density only
(b) amplitude A/2, frequency w/p (b) Pressure only
(c) amplitude 2A, frequency w/4p (c) Both pressure and density
(d) it does not represent a wave motion (d) Neither pressure nor density
7. When a wave is reflected from a denser medium, the change 16. The equation of plane progressive wave motion is
in phase is

(a) 0 (b) p (c) 2 p (d) 3 p y = a sin ( vt - x ) . Velocity of the particle is
8. The ratio of the speed of a body to the speed of sound is λ
called dv dy dv dy
(a) Mach number (b) Doppler ratio (a) y (b) v (c) - y (d) -v
dx dx dx dx
(c) sonic index (d) refractive index 17. A progressive wave y = A sin(kx – wt) is reflected by a rigid
9. The relation between velocity of sound in a gas (v) and wall at x = 0. Then the reflected wave can be represented by
r.m.s. velocity of molecules of gas (vr.m.s.) is (a) y = A sin(kx + wt) (b) y = A cos(kx + wt)
(a) v = vr.m.s.(g/3)1/2 (b) vr.m.s. = v(2/3)1/2
(c) v = vr.m.s. (d) v = vr.m.s.(3/g)1/2 (c) y = –A sin(kx – wt) (d) y = –A sin(kx + wt)
10. Of the following, the equation of plane progressive wave is 18. What is the effect of humidity on sound waves when
(a) y = r sin wt (b) y = r sin (wt – kx) humidity increases?
a (a) Speed of sound waves is more
a
(c) y = sin (ωt - kx) (d) y = sin (ωt - kr) (b) Speed of sound waves is less
r r
(c) Speed of sound waves remains same
11. Sound waves are travelling in a medium whose adiabatic
elasticity is E and isothermal elasticity E¢. The velocity of (d) Speed of sound waves becomes zero
sound waves is proportional to 2p
19. The equation y = a sin (vt - x ) is expression for
E l
(a) E¢ (b) E (c) E' (d) (a) stationary wave of single frequency along x-axis
E'
12. Sound waves transfer (b) a simple harmonic motion
(a) energy (c) a progressive wave of single frequency along x-axis
(b) momentum (d) the resultant of two SHMs of slightly different
(c) both energy and momentum frequencies
(d) only energy not momentum
WAVES 203

20. Consider the three waves z 1, z2 and z3 as 27. The bulk modulus of a liquid of density 8000 kg m–3 is
z1 = A sin (kx – wt) 2 × 109 N m–2. The speed of sound in that liquid is (in m s–1)
z2 = A sin (kx + wt) (a) 200 (b) 250 (c) 100 (d) 500
z3 = A sin (ky – wt) 28. The rate of transfer of energy in a wave depends
Which of the following represents a standing wave? (a) directly on the square of the wave amplitude and square
(a) z1 + z2 (b) z2 + z3 of the wave frequency
(c) z3 + z1 (d) z1 + z2 + z3 (b) directly on the square of the wave amplitude and root
of the wave frequency
21. Ultrasonic, infrasonic and audible waves travel through a
(c) directly on the wave amplitude and square of the wave
medium with speeds vu, vi, va respectively, then
frequency
(a) vu, vi, va are nearly equal (d) None of these
(b) vu ³ va ³ vi 29. For a sinusoidal wave represented by y (x, t) = a sin
(c) vu £ va £ vi (kx – wt + f), for a given a, the factor determines the
(d) va, £ vu and vu » vi displacement of the wave of any position and at any
22. A circular loop of rope of length L rotates with uniform angular instant is
velocity w about an axis through its centre on a horizontal (a) constant k (b) angular velocity w
smooth platform. Velocity of pulse (with respect to rope) (c) time interval t (d) phase (kx – wt + f)
produced due to slight radial displacement is given by 30. The velocity of sound in hydrogen is 1224 m/s. Its velocity
w
in a mixture of hydrogen and oxygen containing 4 parts by
wL volume of hydrogen and 1 part oxygen is
(a) wL (b) (a) 1224 m/s (b) 612 m/s (c) 2448 m/s (d) 306 m/s
2p
31. When a sound wave of frequency 300 Hz passes through a
medium, the maximum displacement of a particle of the
wL wL
(c) (d) medium is 0.1 cm. The maximum velocity of the particle is
p 4p 2 equal to
23. Which of the following statements is/are correct about the (a) 60 pcms–1 (b) 30 pcms–1
standing wave? (c) 30 cms –1 (d) 60 cms–1
I. In a standing wave the disturbance produce is confined 32. The velocity of sound in a container of air at –73ºC is
to the region where it is produced. 300 m/s. It temp. of container were raised to 127ºC, what
II. In a standing wave, all the particles cross their mean would be the velocity of sound ?
position together.
III. In a standing wave, energy is transmitted from one (a) 300 m/s (b) 300 2 m/s
region of space to other. (c) 300 / 2 m/s (d) 600 m/s
(a) I and II (b) Only II
33. Velocity of sound waves in air is 330 m/s. For a particular
(c) Only III (d) I, II and III
sound wave in air, a path difference of 40 cm is equivalent to
24. Two waves are represented by the equations y1 = a sin (wt
phase difference of 1.6p. The frequency of this wave is
+ kx + 0.57) m and y2 = a cos (wt + kx) m, where x is in meter
(a) 165 Hz (b) 150 Hz (c) 660 Hz (d) 330 Hz
and t in sec. The phase difference between them is
34. A wave travelling in the +ve x-direction having displacement
(a) 1.0 radian (b) 1.25 radian
1
(c) 1.57 radian (d) 0.57 radian along y-direction as 1m, wavelength 2p m and frequency
p
25. A transverse wave is represented by y = A sin ( w t – kx). For Hz is represented by
what value of the wavelength is the wave velocity equal to (a) y = sin (2px – 2pt) (b) y = sin (10px – 20pt)
the maximum particle velocity? (c) y = sin (2px + 2pt) (d) y = sin (x – 2t)
pA 35. The displacement y of a particle in a medium can be
(a) (b) p A (c) 2pA (d) A
2
26. Assertion : Solids can support both longitudinal and -6 æ pö
expressed as, y = 10 sin ç100t + 20 x + ÷ m where t is in
transverse waves but only longitudinal waves can è 4ø
propagate in gases. second and x in meter. The speed of the wave is
Reason : For the propagation of transverse waves, medium (a) 20 m/s (b) 5 m/s (c) 2000 m/s (d) 5p m/s
must also necessarily have the property of rigidity. 36. A progressive sound wave of frequency 500 Hz is travelling
(a) Assertion is correct, reason is correct; reason is a through air with a speed of 350 ms –1. A compression
correct explanation for assertion. maximum appears at a place at a given instant. The minimum
(b) Assertion is correct, reason is correct; reason is not a time interval after which the rarefraction maximum occurs at
correct explanation for assertion the same point, is
(c) Assertion is correct, reason is incorrect 1 1 1
(d) Assertion is incorrect, reason is correct. (a) 200 s (b) s (c) s (d) s
250 500 1000
EBD_7418
204 PHYSICS

37. A hospital uses an ultrasonic scanner to locate tumours in a 46. A wave travelling on a string is described by
tissue. The operating frequency of the scanner is 4.2 MHz. y (x, t) = 0.005 sin (80.0x – 3.0t)
The speed of sound in a tissue is 1.7 km/s. The wavelength The period of the wave is
of sound in tissue is close to (a) 3.00 s (b) 2.09 s (c) 0.48 s (d) 0.05 s
(a) 4 × 10–4 m (b) 8 × 10–4 m 47. Frequencies of sound produced from an organ pipe open at
(c) 4 × 10 m–3 (d) 8 × 10–3 m both ends are
38. When two sound waves travel in the same direction in a (a) only fundamental note
medium, the displacements of a particle located at 'x' at time
(b) only even harmonics
‘t’ is given by :
(c) only odd harmonics
y1 = 0.05 cos (0.50 px – 100 pt)
(d) even and odd harmonics
y2 = 0.05 cos (0.46 px – 92 pt) 48. The fifth harmonic for vibrations of a stretched string is
where y1, y2 and x are in meters and t in seconds. The speed shown in figure. How many nodes are present here?
of sound in the medium is :
(a) 92 m/s (b) 200 m/s (c) 100 m/s (d) 332 m/s
39. At t = 0, the shape of a travelling pulse is given by (a) 4 (b) 6 (c) 5 (d) 10
49. A closed organ pipe (closed at one end) is excited to support
4 ´ 10-3 the third overtone. It is found that air in the pipe has
y (x, 0) =
8 - (x)-2 (a) three nodes and three antinodes
where x and y are in metres. The wave function for the (b) three nodes and four antinodes
travelling pulse if the velocity of propagation is 5 m/s in the (c) four nodes and three antinodes
x direction is given by (d) four nodes and four antinodes
4 ´ 10-3 4 ´ 10-3 50. If there are six loops for 1 m length in transverse mode of
(a) y (x, t) = (b) y (x, t) = Melde’s experiment., the no. of loops in longitudinal mode
8 - (x 2 - 5t) 8 - (x - 5t)2 under otherwise identical conditions would be
4 ´ 10-3 4 ´ 10-3 (a) 3 (b) 6 (c) 12 (d) 8
(c) y (x, t) = (d) y (x, t) =
8 - (x + 5t)2 8 - (x 2 + 5t) 51. If n1, n2 and n3 are the fundamental frequencies of three
40. A plane progressive simple harmonic sound wave of segments into which a string is divided, then the original
angular frequency 680 rad/s moves with speed 340 fundamental frequency n of the string is given by
m/s in the direction which makes equal angle with each 1 1 1 1
x, y and z-axis. The phase difference (f1 – f2) between (a) = + +
n n1 n 2 n 3
the oscillations of the particle in the medium located at
the positions (Ö3, Ö3, Ö3) and (2Ö3, 2Ö3, 2Ö3) is 1 1 1 1
(b) = + +
(assume cos q > 0) n n1 n2 n3
(a) 2 radian (b) 3 radian (c) 4 radian (d) 6 radian
(c) n = n1 + n 2 + n 3
Topic 2: Vibrations of String and Organ Pipe (d) n = n1 + n2 + n3
41. A string fixed at both ends is vibrating in two segments. 52. Tube A has both ends open while tube B has one end closed,
The wavelength of the corresponding wave is otherwise they are identical. The ratio of fundamental
(a) l/4 (b) l/2 (c) l (d) 2l frequency of tube A and B is
42. If vibrations of a string are to be increased by a factor of (a) 1 : 2 (b) 1 : 4 (c) 2 : 1 (d) 4 : 1
two, then tension in the string must be made
53. If the length of a stretched string is shortened by 40% and
(a) half (b) twice
(c) four times (d) eight times the tension increased by 44% then the ratio of the final and
43. The fundamental frequency of a closed end organ pipe is n. initial fundamental frequencies is
Its length is doubled and radius is halved. Its frequency will (a) 3 : 4 (b) 4 : 3 (c) 1 : 3 (d) 2 : 1
become nearly 54. The number of possible natural oscillation of air column in a
(a) n/2 (b) n/3 (c) n (d) 2 n pipe closed at one end of length 85 cm whose frequencies
44. The fundamental frequency of an organ pipe is 512 Hz. If its lie below 1250 Hz are : (velocity of sound = 340 ms– 1)
length is increased, then frequency will (a) 4 (b) 5 (c) 7 (d) 6
(a) decrease (b) increase 55. The fundamental frequency of a closed organ pipe of length
(c) remains same (d) cannot be predicted 20 cm is equal to the second overtone of an organ pipe open
45. What is the effect of increase in temperature on the frequency at both the ends. The length of organ pipe open at both the
of sound produced by an organ pipe?
ends is
(a) increases (b) decreases
(a) 100 cm (b) 120 cm (c) 140 cm (d) 80 cm
(c) no effect (d) erratic change
WAVES 205

56. The extension in a string, obeying Hooke’s law, is x. The 65. A stretched wire 60 cm long is vibrating with its fundamental
speed of sound in the stretched string is v. If the extension frequency of 256 Hz. If the length of the wire is decreased to
in the string is increased to 1.5x, the speed of sound will be 15 cm and the tension remains the same. Then the
(a) 1.22v (b) 0.61v (c) 1.50v (d) 0.75v fundamental freuqency of the vibration of the wire will be
57. Choose the false statement(s) about speed of transverse (a) 1024 (b) 572 (c) 256 (d) 64
waves on a stretched string.
I. It depends on linear mass density of the string 66. A string is stretched between fixed points separated by 75.0
II. It depends on the tension in the string cm. It is observed to have resonant frequencies of 420 Hz and
III. It depends on the frequency of the wave 315 Hz. There are no other resonant frequencies between these
(a) I only (b) II only two. Then, the lowest resonant frequency for this string is
(c) III only (d) I, II and III (a) 105 Hz (b) 1.05 Hz (c) 1050 Hz (d) 10.5 Hz
58. An organ pipe P1 closed at one end vibrating in its first 67. The fundamental frequency of an open organ pipe is 300
overtone and another pipe P2 open at both ends vibrating Hz. The first overtone of this pipe has same frequency as
in third overtone are in resonance with a given tuning fork. first overtone of a closed organ pipe. If speed of sound is
The ratio of the length of P1 to that of P2 is 330 m/s, then the length of closed organ pipe is
(a) 8/3 (b) 3/8 (c) 1/2 (d) 1/3 (a) 41 cm (b) 37 cm (c) 31 cm (d) 80 cm
59. Length of a sonometer wire between two fixed ends is 110 68. In a standing wave formed as a result of reflection from a
cm. If the fundamental frequencies are in the ratio of surface, the ratio of the amplitude at an antinode to that at
1 : 2 : 3, then what is the ratio of lengths of these node is x. The fraction of energy that is reflected is
segments of the wire?
2 2 2 2
(a) 3 : 2 : 1 (b) 6 : 3 : 2 (c) 6 : 2 : 3 (d) 2 : 3 : 6 é x - 1ù é x ù é x - 1ù é1ù
(a) ê ú (b) ê ú (c) ê ú (d) ê ú
60. The equation of a wave on a string of linear mass density ë x û ë x + 1û ë x + 1û ëxû
0.04 kg m–1 is given by 69. Two strings A and B, made of same material, are stretched
é æ t x öù by same tension. The radius of string A is double of radius
y = 0.02(m)sin ê 2p ç – ÷ú of B. A transverse wave travels on A with speed vA and on
ë è 0.04( s) 0.50(m) ø û B with speed vB. The ratio vA / vB is
The tension in the string is
(a) 1/2 (b) 2 (c) 1/4 (d) 4
(a) 4.0 N (b) 12.5 N (c) 0.5 N (d) 6.25 N
61. Assertion : Two waves moving in a uniform string having 70. In the sonometer experiment, a tuning fork of frequency 256
uniform tension cannot have different velocities. Hz is in resonance with 0.4 m length of the wire when the
Reason : Elastic and inertial properties of string are same for iron load attached to free end of wire is 2 kg. If the load is
all waves in same string. Moreover speed of wave in a string immersed in water, the length of the wire in resonance would
depends on its elastic and inertial properties only. be (specific gravity of iron = 8)
(a) Assertion is correct, reason is correct; reason is a (a) 0.37 m (b) 0.43 m (c) 0.31 m (d) 0.2 m
correct explanation for assertion. 71. Two identical piano wires kept under the same tension T
(b) Assertion is correct, reason is correct; reason is not a
correct explanation for assertion have a fundamental frequency of 600 Hz. The fractional
(c) Assertion is correct, reason is incorrect increase in the tension of one of the wires which will lead to
(d) Assertion is incorrect, reason is correct. occurrence of 6 beats/s when both the wires oscillate
62. A sonometer wire of length 1.5 m is made of steel. together would be
The tension in it produces an elastic strain of 1%. What (a) 0.02 (b) 0.03 (c) 0.04 (d) 0.01
is the fundamental frequency of steel if density and 72. The length of the wire between two ends of a sonometer is
elasticity of steel are 7.7 × 103 kg/m3 and 2.2 × 1011 N/m2 100 cm. What should be the positions of two bridges below
respectively ? the wire so that the three segments of the wire have their
(a) 188.5 Hz (b) 178.2 Hz (c) 200.5 Hz (d) 770 Hz fundamental frequencies in the ratio of 1 : 3 : 5?
63. A sonometer wire supports a 4 kg load and vibrates in 1500 2000
cm, cm 1500 500
fundamental mode with a tuning fork of frequency 416 Hz. (a) (b) cm, cm
The length of the wire between the bridges is now doubled. 23 23 23 23
In order to maintain fundamental mode, the load should be 1500 300 300 1500
changed to (c) cm, cm (d) cm, cm
23 23 23 23
(a) 1 kg (b) 2 kg (c) 4 kg (d) 16 kg
64. Two pulses in a stretched string whose centres are initially Topic 3: Beats, Interference and Superposition of waves
8 cm apart are moving towards each other as shown in the
figure. The speed of each pulse is 2 cm/s. After 2 s, the total 73. When two sound waves are superimposed, beats are
energy of the pulses will be produced when they have different
(a) amplitudes and phases
(a) Zero
(b) velocities
(b) Purely kinetic
(c) phases
8 cm
(c) Purely potential (d) frequencies
(d) Partly kinetic and partly potential
EBD_7418
206 PHYSICS

74. Beats are produced by two waves: 85. The equation of a resultant wave obtained after
Y1 = a sin 100 pt and y2 = a sin 998 pt superposition of two waves is given by y (x, t) = 2a sin
The number of beats heard/sec is: kx cos wt. The position of nodes will be given by
(a) 0 (b) 2 (c) 1 (d) 4 (a) sin kx = – 1 (b) sin kx = 0
75. Maximum number of beats frequency heard by a human np
being is (c) sin kx = 1 (d) sin kx =
2
(a) 10 (b) 4 (c) 20 (d) 6 86. Two sound waves travel in the same direction in a medium.
76. Reverberation time does not depend upon The amplitude of each wave is A and the phase difference
(a) temperature (b) volume of room
between the two waves is 120°. The resultant amplitude will be
(c) size of window (d) carpet and curtain
77. The essential condition for the formation of beats is (a) 2A (b) 2A (c) 3A (d) A
(a) difference in frequencies of two sources should be ³ 10 87. Two vibrating tuning forks produce progressive waves
(b) difference in frequencies of two sources should be < 10 given by Y1 = 4 sin 500 pt and Y2 = 2 sin 506 pt. Number of
(c) difference in frequencies of two sources should be > 10 beats produced per minute is
(d) difference in frequencies of two sources should be £ 10 (a) 360 (b) 180 (c) 60 (d) 3
78. The intensity of harmonic wave 88. Two tuning forks P and Q when set vibrating, give 4 beats
(a) depends upon its frequency and not on its amplitude per second. If a prong of the fork P is filled, the beats are
(b) depends upon its amplitude and not on its frequency reduced to 2/5 what is frequncy of P, if that of Q is 250 Hz?
(c) depends upon both, its frequency and not on (a) 246 Hz (b) 250 Hz (c) 254 Hz (d) 252 Hz
amplitude
89. The wavelength of two waves are 50 and 51 cm respectively.
(d) depends neither on frequency nor on its amplitude
If the temperature of the room is 20°C then what will be the
79. If the intensities of two interfering waves be I1 and I2, the
contrast between maximum and minimum intensity is number of beats produced per second by these waves, when
maximum, when the speed of sound at 0°C is 332 m/s?
(a) I1 > > I2 (b) I1 << I2 (a) 24 (b) 14
(c) I1 = I2 (d) either I1 or I2 is zero (c) 10 (d) none of these
80. Three sound waves of equal amplitudes have frequencies 90. In the figure shown the wave speed is v. The velocity of car
(n –1), n, (n + 1). They superpose to give beats. The number is v0. The beat frequency for the observer will be
of beats produced per second will be :
(a) 3 (b) 2 (c) 1 (d) 4
81. Two tones of frequencies n 1 and n2 are sounded together.
The beats can be heard distinctly when
(a) 10 < (n1 – n 2) < 20 (b) 5 < (n1 – n 2) > 20 2 f 0 vv0 2 f0v2 2 f 0 vv0 f 0 vv0
(a) v + v0
2 2 (b) (c) v - v0 (d) v - v0 2
(c) 5 < (n1 – n 2) < 20 (d) 0 < (n1 – n 2) < 10 v - v0
2 2 2 2 2

82. Two travelling waves y1 = A sin [k (x – ct)] and y2 = A sin [k 91. Two sound sources S2 and S1 emit pure sinusoidal coherent
(x + ct)] are superimposed on string. The distance between waves in phase. If the speed of sound is 340 m/s, then find
adjacent nodes is out the frequencies for which constructive interference
(a) ct / p (b) ct / 2p (c) p /2k (d) p /k occurs at P. S
2
83. What will be the frequency of beats formed from the
superposition of two harmonic waves shown below? (a) 170 Hz
2 10 m 3m
y
(b) 340 Hz
P
1.0
(c) 510 Hz 4m
0
t(s)
–1.0 (d) All of these
(a) S1
y 92. A tuning fork arrangement (pair) produces 4 beats/sec with
one fork of frequency 288 cps. A little wax is placed on the
1.0 unknown fork and it then produces 2 beats/sec. The
0 frequency of the unknown fork is
t(s)
–1.0 (b)
(a) 286 cps (b) 292 cps (c) 294 cps (d) 288 cps
(a) 20 Hz (b) 11 Hz (c) 9 Hz (d) 2 Hz 93. A source of sound gives 5 beats per second when sounded
84. Two factories are sounding their sirens at 800 Hz. A man with another source of frequency 100 per second. The
goes from one factory to other at a speed of 2m/s. The second harmonic of the source, together with a source of
velocity of sound is 320 m/s. The number of beats heard by frequency 205 per second gives 5 beats per second. What
the person in one second will be: is the frequency of the source?
(a) 2 (b) 4 (c) 8 (d) 10
(a) 95 Hz (b) 100 Hz (c) 105 Hz (d) 205 Hz
WAVES 207

94. A tuning fork of frequency 512 Hz makes 4 beats per second


æ vö æ v ö
with the vibrating string of a piano. The beat frequency (a) f ç1 + ÷ (b) f ç1 – s ÷
decreases to 2 beats per sec when the tension in the piano è vs ø è vø
string is slightly increased. The frequency of the piano æ v ö æ vö
string before increasing the tension was (c) f ç1 + s ÷ (d) f ç1 – ÷
è vø è vs ø
(a) 510 Hz (b) 514 Hz (c) 516 Hz (d) 508 Hz 103. Choose the false statement(s) from the following.
95. In a large room, a person receives direct sound waves from I. Change in frequency due to Doppler effect will be
a source 120 m away from him. He also receives waves from positive if the distance between source and listener
the same source which reach him, being reflected from the increases.
25 m high celling at a point halfway between them. The two II. Change in frequency due to Doppler effect will be
waves interfere constructively for wavelength of negative if the distance between source and listener
20 20 (a) I only (b) II only
(a) 20, , etc (b) 10, 5, 2.5 etc (c) I and II (d) None of these
3 5
(c) 10, 20, 30 etc (d) 15, 25, 35 etc 104. If wind blows from a stationary sounding object to a
stationary listener, then the apparent frequency n' and actual
Topic 4: Musical Sound and Doppler’s Effect frequency n are related as
(a) n¢ ³ n (b) n¢ < n (c) n¢ = n (d) n¢ > n
96. The loudness and pitch of a sound depends on
105. A car is moving towards a high cliff. The car driver sounds
(a) intensity and velocity
a horn of frequency f. The reflected sound heard by the
(b) frequency and velocity driver has as frequency 2f. If v be the velocity of sound,
(c) intensity and frequency then the velocity of the car, in the same velocity units, will be
(d) frequency and number of harmonics (a) v /2 (b) v /Ö2 (c) v /3 (d) v /4
97. A source of sound is moving with a uniform speed along a 106. Column I Column II
circle. The frequency of sound as heard by listener stationed
(A) Change in apparent (1) Beats
at the centre of the path frequency due to the
(a) increases relative motion between
(b) decreases source and listner
(c) remains the same (B) Intensity of sound (2) Transverse Wave
(d) may increase and decrease alternately varies with
98. Doppler's effect in sound takes place when source and (C) Sound waves in air (3) Doppler’s effect
observer are (D) Light waves (4) Longitudinal wave
(a) stationary (a) (A) ® (1) ; (B) ® (2) ; (C) ® (3) ; (D) ® (4)
(b) moving with same velocity (b) (A) ® (2) ; (B) ® (3) ; (C) ® (4) ; (D) ® (1)
(c) in relative motion
(c) (A)® (3) ; (B) ® (1) ; (C) ® (4) ; (D) ® (2)
(d) None of these (d) (A) ® (3) ; (B) ® (4) ; (C) ® (1) ; (D) ® (2)
99. Doppler phenomena is related with
107. Assertion : Doppler formula for sound wave is symmetric
(a) Pitch (Frequency) (b) Loudness
with respect to the speed of source and speed of observer.
(c) Quality (d) Reflection Reason : Motion of source with respect to stationary
100. Doppler shift in frequency does not depend upon observer is not equivalent to the motion of an observer with
(a) frequency of the wave produced respect to stationary source.
(b) velocity of the source (a) Assertion is correct, reason is correct; reason is a
(c) velocity of the observer correct explanation for assertion.
(d) distance from the source to the listener (b) Assertion is correct, reason is correct; reason is not a
101. For which of the following cases, there will be no Doppler correct explanation for assertion
effect? (c) Assertion is correct, reason is incorrect
(a) If source and listener, both move in the same (d) Assertion is incorrect, reason is correct.
direction with same speed. 108. The musical interval between two tones of frequencies 320
(b) If one of the source/listener is at the centre of a circle, Hz and 240 Hz is:
while the other is moving on it. (a) 80 (b) (4/3) (c) 560 (d) 320 ´ 240
(c) When both the source and listener are at rest. 109. A is singing a note and at the same time B is singing a note
with exactly one-eight the frequency of the note of A. The
(d) All of these
energies of the two sounds are equal. The amplitude of the
102. A train moving at a speed vs towards a stationary observer note of B is:
on a platform emits sound of frequency f and velocity v. (a) same as that of A (b) twice that of A
Then the apparent frequency heard by him is (c) four times that of A (d) eight times that of A
EBD_7418
208 PHYSICS

110. A whistle of frequency 385 Hz rotates in a horizontal circle speed of the motorcycle, if it is given that he does not observe
of radius 50 cm at an angular speed of 20 radians s–1. The any beats is
lowest frequency heard by a listener a long distance away Police car Motorcycle
at rest with respect to the centre of the circle, given velocity
of sound equal to 340 ms–1, is
(a) 396 Hz (b) 363 Hz (c) 374 Hz (d) 385 Hz Stationary siren
111. A band playing music at frequency f is moving towards 22 m/s v (165 Hz)
(176 Hz)
a wall at a speed vb. A motorist is following the band with
a speed vm. If v is the speed of sound, the expression for (a) 33 m/s (b) 22 m/s (c) zero (d) 11 m/s
the beat frequency heard by the motorist is 116. Two trains are moving towards each other with speeds of
v + vm v + vm 20m/s and 15 m/s relative to the ground. The first train
(a) f (b) f sounds a whistle of frequency 600 Hz. The frequency of the
v + vb v - vb
whistle heard by a passenger in the second train before the
2v b (v + v m ) 2v m (v + v b ) train meets, is (the speed of sound in air is 340 m/s)
(c) f (d) f
v 2 - v 2b v2 - vm 2
(a) 600 Hz (b) 585 Hz (c) 645 Hz (d) 666 Hz
112. A whistle producing sound waves of frequencies 9500 HZ
117. Two trains move towards each other with the same speed.
and above is approaching a stationary person with speed
The speed of sound is 340 m/s. If the height of the tone of
vms–1. The velocity of sound in air is 300 ms–1. If the
the whistle of one of them heard on the other changes 9/8
person can hear frequencies upto a maximum of 10,000 HZ,
times, then the speed of each train should be
the maximum value of v upto which he can hear whistle is
(a) 20 m/s (b) 2 m/s (c) 200 m/s (d) 2000 m/s
15
(a) 15 2 ms -1 (b) ms -1 118. A source producing sound of frequency 170 Hz is
2 approaching a stationary observer with a velocity 17 ms–1.
(c) 15 ms–1 (d) 30 ms–1 The apparent change in the wavelength of sound heard by
113. The power of a sound from the speaker of a radio is 20 the observer is (speed of sound in air = 340 ms–1)
milliwatt. By turning the knob of the volume control the (a) 0.1 m (b) 0.2 m (c) 0.4 m (d) 0.5 m
power of sound is increased to 400 milliwatt. The power 119. A train approaching a hill at a speed of 60 km/hour sounds
increases (indecibel) as compared to the original power is: a whistle of frequency 600 Hz when it is at a distance of 1 km
(a) 13 (b) 10 (c) 20 (d) 800 from the hill. Wind is blowing in the direction of the train
114. A sound source emits frequency of 180 Hz when moving with a speed of 60 km/h. Find the frequency of the whistle
towards a rigid wall with speed 5 m/s and an observer is heard by an observer on the hill: (Velocity of sound in
moving away from wall with speed 5 m/s. Both source and air = 1200 km/h)
observer moves on a straight line which is perpendicular to
the wall. The number of beats per second heard by the (a) 610 Hz (b) 620 Hz (c) 630 Hz (d) 650 Hz
observer will be [Speed of sound = 355 m/s] 120. A man is watching two trains, one leaving and the other
(a) 5 beats/s (b) 10 beats/s coming in with equal speeds of 4m/sec. If they sound their
(c) 6 beats/s (d) 8 beats/s whistles, each of frequency 240 Hz, the number of beats
115. A police car moving at 22 m/s, chases a motorcyclist. The heard by the man (velocity of sound in air = 320 m/sec) will
policeman sounds his horn at 176 Hz, while both of them be equal to
move towards a stationary siren of frequency 165 Hz. The (a) 6 (b) 3 (c) 0 (d) 12

Exercise 2 : Exemplar & Past Year MCQs


(c) increases with increase in humidity
NCERT Exemplar MCQs
(d) decreases with increase in humidity
1. Water waves produced by a motorboat sailing in water are 4. Change in temperature of the medium changes
(a) neither longitudinal nor transverse (a) frequency of sound waves
(b) both longitudinal and transverse (b) amplitude of sound waves
(c) only longitudinal (c) wavelength of sound waves
(d) only transverse (d) loudness of sound waves
2. Sound waves of wavelength l travelling in a medium with a 5. With propagation of longitudinal waves through a medium,
speed of v m/ s enter into another medium where its speed in the quantity transmitted is
2v m/s. Wavelength of l sound waves in the second medium is (a) matter
(a) l (b) (c) 2l (d) 4 l
(b) energy
3. Speed of sound wave2 in air
(a) is independent of temperature (c) energy and matter
(b) increases with pressure (d) energy, matter and momentum
WAVES 209

6. Which of the following statements are true for wave motion? 12. A fork of frequency 256 Hz resonates with a closed organ
(a) Mechanical transverse waves can propagate through pipe of length 25.4 cm. If the length of pipe be increased by
all mediums. 2 mm, the number of beats/sec. will be [AIIMS 2014, C]
(b) Longitudinal waves can propagate through solids only . (a) 4 (b) 1 (c) 2 (d) 3
(c) Mechanical transverse waves can propagate through 13. The equation of a progressive wave is
solids only.
(d) Longitudinal waves can propagate through vacuum. é t x ù
y = 0.02 sin 2p ê -
7. A sound wave is passing through air column in the form of
ë 0. 01 0 .30 úû
compression and rarefaction. In consecutive compressions
and rarefactions, Here x and y are in metre and t is in second. The velocity of
(a) density remains constant propagation of the wave is [AIIMS 2014, C]
(b) Boyle's law is obeyed (a) 300 m s–1 (b) 30 m s–1
(c) bulk modulus of air oscillates (c) 400 m s–1 (d) 40 m s–1
(d) there is no transfer of heat
14. Assertion : Doppler formula for sound wave is symmetric
8. Equation of a plane progressive wave is given by
with respect to the speed of source and speed of observer.
æ xö
y = 0.6sin 2p ç t - ÷ . On reflection from a denser medium Reason : Motion of source with respect to stationary
è 2ø
2 observer is not equivalent to the motion of an observer with
its amplitude becomes of the amplitude of the incident respect to stationary source. [AIIMS 2014, C]
3
wave. The equation of the reflected wave is (a) If both Assertion and Reason are correct and Reason
æ xö æ xö is the correct explanation of Assertion.
(a) y = 0.6sin 2p ç t + ÷ (b) y = -0.4sin 2p ç t + ÷
è 2ø è 2ø (b) If both Assertion and Reason are correct, but Reason
æ xö æ xö is not the correct explanation of Assertion.
(c) y = 0.4sin 2p ç t + ÷ (d) y = -0.4 sin 2 p ç t - ÷ (c) If Assertion is correct but Reason is incorrect.
è 2ø è 2ø
9. A string of mass 2.5 kg is under tension of 200 N. The length (d) If both the Assertion and Reason are incorrect.
of the stretched string is 20.0 m. If the transverse jerk is 15. A pipe of length 85 cm is closed from one end. Find the
struck at one end of the string, the disturbance will reach the number of possible natural oscillations of air column in the
other end in 200 N = T pipe whose frequencies lie below 1250 Hz. The velocity of
sound in air is 340 m/s. [JEE Main 2014, C]
(a) 1 s (a) 12 (b) 8 (c) 6 (d) 4
(b) 0.5 s 16. When the wavelength of sound changes from 1 m to 1.01 m,
(c) 2 s 20 m the number of beats heard are 4. The velocity of Sound is
(d) data given is insufficient [BITSAT 2014, C]
10. A train whistling at constant frequency is moving towards a (a) 404 m/s (b) 4.04 m/s (c) 414 m/s (d) 400 m/s
station at a constant speed v. The train goes past a stationary 17. A source of sound S emitting waves of frequency 100 Hz
observer on the station. The frequency n' of the sound as and an observor O are located at some distance from each
heard by the observer is plotted as a function of time t other. The source is moving with a speed of 19.4 ms-1 at an
(figure). Identify the expected curve. angle of 60° with the source observer line as shown in the
n n figure. The observor is at rest. The apparent frequency
observed by the observer is (velocity of sound in air
(a) (b) 330 ms-1) [AIPMT 2015, A]
(a) 103 Hz
t t
(b) 106 Hz
n n
(c) 97 Hz 60°
(d) 100 Hz S O
(c) (d) 18. A string is stretched between two fixed points separated by
t t 75.0 cm. It is observed to have resonant frequencies of 420
Hz and 315 Hz. There are no other resonant frequencies
Past Year MCQs between these two. The lowest resonant frequency for this
string is : [AIPMT 2015, C]
11. A speeding motorcyclist sees trafic jam ahead of him. He
(a) 205 Hz (b) 10.5 Hz (c) 105 Hz (d) 155 Hz
slows down to 36 km/hour. He finds that traffic has eased
and a car moving ahead of him at 18 km/hour is honking at a 19. Two waves of wavelengths 99 cm and 100 cm both travelling
frequency of 1392 Hz. If the speeds of sound is 343 m/s, the with velocity 396 m/s are made to interfere. The number of
frequency of the honk as heard by him will be: beats produced by them per second is [AIIMS 2015, A]
[AIPMT 2014, A] (a) 1 (b) 2
(a) 1332 Hz (b) 1372 Hz (c) 1412 Hz (d) 1464 Hz (c) 4 (d) 8
EBD_7418
210 PHYSICS

20. Assertion : Two waves moving in a uniform string having 27. Two similar open organ pipe of length 50 cm and 50.5 cm
uniform tension cannot have different velocities. produce 3 beats per second when sounded together. The
Reason : Elastic and inertial properties of string are same for velocity of sound in air is [AIIMS 2016, A]
all waves in same string. Moreover speed of wave in a string (a) 303 m/s (b) 330 m/s
depends on its elastic and inertial properties only.
[AIIMS 2015, C] (c) 151.5 m/s (d) 603 m/s
(a) If both Assertion and Reason are correct and Reason 28. A whistle S of frequency f revolves in a circle of radius R at
is the correct explanation of Assertion. a constant speed v. What is the ratio of largest and smallest
frequency detected by a detector D at rest at a distance 2R
(b) If both Assertion and Reason are correct, but Reason
is not the correct explanation of Assertion. from the centre of circle as shown in figure ?
(c) If Assertion is correct but Reason is incorrect. (take c as speed of sound) [AIIMS 2016, C]
(d) If both the Assertion and Reason are incorrect.
21. A train is moving on a straight track with speed 20 ms–1. It
is blowing its whistle at the frequency of 1000 Hz. The D S
percentage change in the frequency heard by a person R
standing near the track as the train passes him is (speed of
sound = 320 ms–1) close to : [JEE Main 2015]
(a) 18% (b) 24% (c) 6% (d) 12%
2R
22. Two tuning forks with natural frequencies 340 Hz each move
relative to a stationary observer. One fork moves away from æ c + vö æ c + vö
çè ÷ 2ç
the observer, while the other moves towards the observer
(a)
c - vø
(b) è c - v ÷ø
at the same speed. The observer hears beats of frequency 3 (c + v)
Hz. Find the speed of the tuning forks. [BITSAT 2015, S] (c) 2 (d)
(a) 1.5 m/s (b) 2 m/s (c) 1 m/s (d) 2.5 m/s c 2
23. The frequency of a sonometer wire is 100 Hz. When the 29. Assertion : The base of Laplace correction was that
weights producing the tensions are completely immersed in exchange of heat between the region of compression and
water, the frequency becomes 80 Hz and on immersing the rarefaction in air is negligible.
weights in a certain liquid, the frequency becomes 60 Hz. Reason : Air is bad conductor of heat and velocity of sound
The specific gravity of the liquid is [BITSAT 2015, S, BN] in air is quite large. [AIIMS 2016, C]
(a) 1.42 (b) 1.77 (c) 1.82 (d) 1.21 (a) If both Assertion and Reason are correct and Reason
24. An air column, closed at one end and open at the other, is the correct explanation of Assertion.
resonates with a tuning fork when the smallest length of the (b) If both Assertion and Reason are correct, but Reason
column is 50 cm. The next larger length of the column is not the correct explanation of Assertion.
resonating with the same tuning fork is : [NEET 2016, C] (c) If Assertion is correct but Reason is incorrect.
(a) 66.7 cm (b) 100 cm (c) 150 cm (d) 200 cm
(d) If both the Assertion and Reason are incorrect.
25. A siren emitting a sound of frequency 800 Hz moves away
from an observer towards a cliff at a speed of 15ms–1. Then, 30. A uniform string of length 20 m is suspended from a rigid
the frequency of sound that the observer hears in the echo support. A short wave pulse is introduced at its lowest end.
reflected from the cliff is : [NEET 2016, A] It starts moving up the string. The time taken to reach the
(Take velocity of sound in air = 330 ms–1) supports is : (take g = 10 ms–2) [JEE Main 2016, S]
(a) 765 Hz (b) 800 Hz (c) 838 Hz (b) 885 Hz (a) 2 2s (b) 2s (c) 2p 2 s (d) 2 s
26. A massless rod of length L is suspended by two identical
31. A pipe open at both ends has a fundamental frequency f in
strings AB and CD of equal length. A block of mass m is
air. The pipe is dipped vertically in water so that half of it is
suspended from point O such that BO is equal to ‘x’. Further
in water. The fundamental frequency of the air column is
it is observed that the frequency of 1st harmonic in AB is
now : [JEE Main 2016, A]
equal to 2nd harmonic frequency in CD. ‘x’ is
f 3f
(a) 2f (b) f (c) (d)
A C [AIIMS 2016, S] 2 4
32. A string of length l is fixed at both ends. It is vibrating in its
3rd overtone with maximum amplitude 'a'. The amplitude at a
distance l/3 from one end is [BITSAT 2016, A]

O 3a a
B L
D (a) a (b) 0 (c) (d)
x 2 2
m 33. The two nearest harmonics of a tube closed at one end and
open at other end are 220 Hz and 260 Hz. What is the
fundamental frequency of the system? [NEET 2017, A]
L 4L 3L L
(a) (b) (c) (d) (a) 20 Hz (b) 30 Hz (c) 40 Hz (d) 10 Hz
5 5 4 4
WAVES 211

34. Two cars moving in opposite directions approach each other successive resonances are produced at 20 cm and 73 cm of
with speed of 22 m/s and 16.5 m/s respectively. The driver of column length. If the frequency of the tuning fork is 320 Hz,
the first car blows a horn having a frequency 400 Hz. The the velocity of sound in air at 27°C is [NEET 2018, A]
frequency heard by the driver of the second car is [velocity (a) 330 m/s (b) 339 m/s (c) 300 m/s (d) 350 m/s
of sound 340 m/s]: [NEET 2017, A] 40. The fundamental frequency in an open organ pipe is equal
(a) 361 Hz (b) 411 Hz (c) 448 Hz (d) 350 Hz to the third harmonic of a closed organ pipe. If the length of
35. A train moving at a speed of 220 ms–1 towards a stationary the closed organ pipe is 20 cm, the length of the open organ
object, emits a sound of frequency 1000 Hz. Some of the pipe is [NEET 2018, A]
sound reaching the object gets reflected back to the train as
echo. The frequency of the echo as detected by the driver (a) 13.2 cm (b) 8 cm (c) 16 cm (d) 12.5 cm
of the train is (speed of sound in air is 330 ms–1) 41. Vibrations are produced in a vertical tube of length 150cm
[AIIMS 2017, A] closed at one end by a tuning fork of frequency 340Hz. Now
(a) 3500 Hz (b) 4000 Hz water is filled slowly in the tube. If the speed of sound in air
(c) 5000 Hz (d) 3000 Hz is 340 m/s then the minimum height of water required for
36. Assertion : The fundemental frequency of an open organ resonance is [AIIMS 2018, C]
pipe increases as the temperature is increased. (a) 90cm (b) 75cm (c) 50cm (d) 25cm
Reason : As the temperature increses, the velocity of sound 42. Three waves of equal frequency having amplitudes 10 mm,
increases more rapidly than length of the pipe. 4 mm and 7 mm arrive at a given point with successive phase
[AIIMS 2017, C] p
(a) If both Assertion and Reason are correct and Reason difference of . The amplitude of the resulting wave in mm
2
is the correct explanation of Assertion. is given by : [AIIMS 2018, C]
(b) If both Assertion and Reason are correct, but Reason (a) 7 (b) 6 (c) 5 (d) 4
is not the correct explanation of Assertion. 43. A granite rod of 60 cm length is clamped at its middle point
(c) If Assertion is correct but Reason is incorrect. and is set into longitudinal vibrations. The density of granite
(d) If both the Assertion and Reason are incorrect. is 2.7 × 103 kg/m3 and its Young's modulus is 9.27×1010 Pa.
37. A sonometer wire resonates with a given tuning fork forming What will be the fundamental frequency of the longitudinal
standing waves with five antinodes between the two bridges vibrations? [JEE Main 2018, S]
when a mass of 9 kg is suspended from the wire. When this (a) 5 kHz (b) 2.5 kHz (c) 10 kHz (d) 7.5 kHz
mass is replaced by a mass M, the wire resonates with the 44. The wavelength of two waves are 50 and 51 cm respectively.
same tuning fork forming three antinodes for the same positions If the temperature of the room is 20°C then what will be the
of the bridges. The value of M is [BITSAT 2017, A] number of beats produced per second by these waves, when
(a) 25 kg (b) 5 kg (c) 12.5 kg (d) 1/25 kg the speed of sound at 0°C is 332 m/s? [BITSAT 2018, A]
(a) 24 (b) 14
38. A tuning fork of frequency 392 Hz, resonates with 50 cm (c) 10 (d) none of these
length of a string under tension (T). If length of the string is 45. 10 forks are arranged in increasing order of frequency in
decreased by 2%, keeping the tension constant, the number such a way that any two nearest tuning forks produce 4
of beats heard when the string and the tuning fork made to beats/sec. The highest frequency is twice of the lowest.
vibrate simultaneously is : [BITSAT 2017, A] Possible highest and the lowest frequencies (in Hz) are
(a) 4 (b) 6 (c) 8 (d) 12 [BITSAT 2018, C]
39. A tuning fork is used to produce resonance in a glass tube. (a) 80 and 40 (b) 100 and 50
The length of the air column in this tube can be adjusted by (c) 44 and 22 (d) 72 and 36
a variable piston. At room temperature of 27°C two

Exercise 3 : Try If You Can


1. A transverse sinusoidal wave moves y 2. In a transverse wave the distance between a crest and
along a string in the positive x-direction neighbouring trough at the same instant is 4.0 cm and the
at a speed of 10 cm/s. The wavelength P distance between a crest and trough at the same place is 1.0
of the wave is 0.5 m and its amplitude is cm. The next crest appears at the same place after a time
x
10 cm. At a particular time t, the snap– interval of 0.4s. The maximum speed of the vibrating particles
in the medium is :
shot of the wave is shown in figure. The
3p 5p p
velocity of point P when its displacement is 5 cm is (a) cm/s (b) cm/s (c) cm/s (d) 2p cm/s
2 2 2
3p ˆ 3p 3. The total length of a sonometer wire between fixed ends is
(a) jm/s (b) - ĵ m / s 110 cm. Two bridges are placed to divide the length of wire
50 50
in ratio 6 : 3 : 2. The tension in the wire is 400 N and the mass
3p 3p per unit length is 0.01 kg/m. What is the minimum common
(c) î m / s (d) - î m / s
50 50 frequency with which three parts can vibrate?
(a) 1100 Hz (b) 1000 Hz (c) 166 Hz (d) 100 Hz
EBD_7418
212 PHYSICS

4. A uniform tube of length 60.5 cm is held vertically with its 10. A source of sound is travelling 100 ms –1
lower end dipped in water. A sound source of frequency 100 –1 A 3
at m s along a road, 3 m S
500 Hz sends sound waves into the tube. When the length 3 q
of tube above water is 16 cm and again when it is 50 cm, the towards a point A. When the
tube resonates with the source of sound. Two lowest source is 3 m away from A, a 4m
person standing at a point O
frequencies (in Hz), to which tube will resonate when it is
on a road perpendicular to the
taken out of water, are (approximately). track hears a sound of
(a) 281, 562 (b) 281, 843 (c) 276, 552 (d) 272, 544 frequency n'. The distance of O
5. While measuring the speed of sound by performing a O from A at that time is 4 m. If the original frequency is 640 Hz,
resonance column experiment, a student gets the first then the value of n' is (given : velocity of sound = 340 m s–1)
resonance condition at a column length of 18 cm during
(a) 620 Hz (b) 680 Hz (c) 720 Hz (d) 840 Hz
winter. Repeating the same experiment during summer, she 11. An earthquake generates both transverse (S) and
measures the column length to be x cm for the second longitudinal (P) sound waves in the earth. The speed of S
resonance. Then waves in about 4.5 km/s and that of P waves is about
(a) 18 > x (b) x > 54 8.0 km/s. A seismograph records P and S waves from an
(c) 54 > x > 36 (d) 36 > x > 18 earthquake. The first P wave arrives 4.0 min. before the first
6. A and B are two sources generating sound waves. A listener S wave. The epicenter of the earthquake is located at a
is situated at C. The frequency of the source at A is 500 Hz. distance about
A, now, moves towards C with a speed 4 m/s. The number of (a) 25 km (b) 250 km (c) 2500 km (d) 5000 km
beats heard at C is 6. When A moves away from C with 12. A uniform rope of length L and mass m1 hangs vertically
speed 4 m/s, the number of beats heard at C is 18. The speed from a rigid support. A block of mass m2 is attached to the
of sound is 340 m/s. The frequency of the source at B is : free end of the rope. A transverse pulse of wavelength l1 is
produced at the lower end of the rope. The wavelength of
A C B
the pulse when it reaches the top of the rope is l2 the ratio
l2/l1 is
(a) 500 Hz (b) 506 Hz (c) 512 Hz (d) 494 Hz
7. A wave travelling along the x-axis is described by the m1 m1 + m 2
(a) m2 (b) m2
equation y(x, t) = 0.005 cos (a x – bt). If the wavelength and
the time period of the wave are 0.08 m and 2.0s, respectively, m2 m1 + m 2
then a and b in appropriate units are (c) m1 (d) m1
0.08 2.0 13. A train is moving in an elliptical orbit in anticlockwise sense
(a) a = 25.00 p , b = p (b) a= ,b =
p p with a speed of 110 m/s. Guard is also moving in the given
direction with same speed as that of train. The ratio of the length
0.04 1.0 p
(c) a = ,b = (d) a = 12.50p, b = of major and minor axes is 4/3. Driver blows a whistle of 1900 Hz
p p 2.0 at P, which is received by guard at S. The frequency received by
8. A train has just completed U-curve guard is (velocity of sound v = 330 m/s)
in a track which is a semi-circle. The P
(a) 1900 Hz
engine is at the forward end of the
semi-circular part of the track while (b) 1800 Hz R
O
S
Observer a
the last carriage is at the rear end of (c) 2000 Hz b
the semi circular track. The driver (d) 1500 Hz Q
blows a whistle of frequency 14. In a standing wave experiment, a 1.2 kg horizontal rope is
V Engine
200 Hz. Velocity of sound is fixed in place at its two ends (x = 0 and x = 2.0 m) and made
340 m/sec Then the apparent frequency as observed by a to oscillate up and down in the fundamental mode, at
passenger in the middle of the train, when the speed of the frequency 5.0 Hz. At t = 0, the point at x = 1.0 m has zero
train is 30 m/sec is displacement and is moving upward in the positive direction
of y axis with a transverse velocity 3.14 m/s.
(a) 181 Hz (b) 200 Hz (c) 188 Hz (d) 210 Hz Tension in the rope is
v (a) 60 N (b) 100 N (c) 120 N (d) 240 N
9. An engine running at speed sounds a whistle of
10 15. A siren is fitted on a car going towards a vertical wall at a
frequency 600 Hz. A passenger in a train coming from the speed of 36 km/hr. A person standing on the ground behind
v the car, listens to the siren sound coming directly from the
opposite side at speed experiences this whistle to be of
15 source as well as that coming after reflection from the wall.
frequency f. If v is speed of sound in air and there is no The apparent frequency of the wave coming directly from
wind, f is near to the siren to the person and coming after reflection
respectively are (Take the speed of sound to be 340 m/s.)
(a) 710 Hz (b) 630 Hz (c) 580 Hz (d) 510 Hz
(a) 515 Hz, 486 Hz (b) 486 Hz, 515 Hz
(c) 510 Hz, 490 Hz (d) 490 Hz, 510 Hz
WAVES 213

ANSWER KEYS
Exercise 1 : Topic-wise MCQs
1 (a) 13 (d) 25 (c) 37 (a) 49 (d) 61 (d) 73 (d) 85 (b) 97 (c) 109 (b)
2 (d) 14 (b) 26 (a) 38 (b) 50 (a) 62 (b) 74 (c) 86 (d) 98 (c) 110 (c)
3 (c) 15 (d) 27 (d) 39 (b) 51 (a) 63 (d) 75 (a) 87 (b) 99 (a) 111 (c)
4 (c) 16 (d) 28 (a) 40 (d) 52 (c) 64 (b) 76 (d) 88 (a) 100 (d) 112 (c)
5 (d) 17 (d) 29 (d) 41 (c) 53 (d) 65 (a) 77 (d) 89 (b) 101 (d) 113 (a)
6 (b) 18 (a) 30 (b) 42 (b) 54 (d) 66 (a) 78 (c) 90 (c) 102 (c) 114 (a)
7 (b) 19 (c) 31 (a) 43 (a) 55 (b) 67 (a) 79 (c) 91 (d) 103 (c) 115 (b)
8 (a) 20 (a) 32 (d) 44 (a) 56 (a) 68 (c) 80 (b) 92 (b) 104 (c) 116 (d)
9 (a) 21 (b) 33 (c) 45 (a) 57 (c) 69 (a) 81 (d) 93 (c) 105 (c) 117 (a)
10 (b) 22 (b) 34 (d) 46 (b) 58 (b) 70 (a) 82 (d) 94 (d) 106 (c) 118 (a)
11 (b) 23 (a) 35 (b) 47 (d) 59 (b) 71 (a) 83 (d) 95 (b) 107 (d) 119 (c)
12 (c) 24 (a) 36 (d) 48 (a) 60 (d) 72 (a) 84 (d) 96 (c) 108 (d) 120 (a)
Exercise 2 : Exemplar & Past Year MCQs
1 (b) 6 (c) 11 (c) 16 (a) 21 (d) 26 (a) 31 (b) 36 (a) 41 (d)
2 (c) 7 (d) 12 (c) 17 (a) 22 (a) 27 (a) 32 (c) 37 (a) 42 (c)
3 (c) 8 (b) 13 (b) 18 (c) 23 (b) 28 (a) 33 (a) 38 (c) 43 (a)
4 (c) 9 (b) 14 (d) 19 (c) 24 (c) 29 (c) 34 (c) 39 (b) 44 (b)
5 (b) 10 (c) 15 (c) 20 (d) 25 (c) 30 (a) 35 (c) 40 (a) 45 (d)
Exercise 3 : Try If You Can
1 (a) 3 (b) 5 (b) 7 (a) 9 (a) 11 (c) 13 (b) 15 (b)
2 (b) 4 (d) 6 (c) 8 (b) 10 (b) 12 (b) 14 (d)
EBD_7418
214 PHYSICS
16 Chapter
ELECTRIC CHARGES
AND FIELDS

Trend
Analysis of NEET and AIIMS (Year 2010-2018)
5

4
Number of Questions

3
AIPMT/NEET
2 AIIMS

0
2010 2011 2012 2013 2014 2015 2016 2017 2018
Year

Trend
Analysis of JEE Main and BITSAT (Year 2010-2018)
4

3
Number of Questions

2
JEE Main
BITSAT
1

0
2010 2011 2012 2013 2014 2015 2016 2017 2018
Year

Chapter Utility Score (CUS)


Exam Weightage Important Concepts Difficulty Level CUS (Out of 10)
NEET 3 Coulomb’s law,
AIIMS 4 Electric field and 4/5 7.5/10
JEE Main 3 Gauss’s law
BITSAT 4
EBD_7418
216 PHYSICS
ELECTRIC CHARGES AND FIELDS 217
EBD_7418
218 PHYSICS

Exercise 1 : Topic-wise MCQs


(d) loses part of the charge on it
Topic 1: Electric Charges and Coulomb’s Law 9. On charging by conduction, mass of a body may
1. A body is positively charged, it implies that (a) increase (b) decreases
(a) there is only positive charge in the body. (c) increase or decrease (d) None of these
(b) there is positive as well as negative charge in the body 10. Select the correct statements, Coulomb's law correctly
but the positive charge is more than negative charge describes the electric force that
I. binds the electrons of an atom to its nucleus.
(c) there is equal positive and negative charge in the body
II. binds the protons and neutrons in the nucleus of an
but the positive charge lies in the outer regions
atom.
(d) negative charge is displaced from its position III. binds atoms together to form molecules.
2. On rubbing, when one body gets positively charged and (a) I and II (b) I and III
other negatively charged, the electrons transferred from (c) II and III (d) I, II and III
positively charged body to negatively charged body are 11. The figure shows a charge + q at point P held in equilibrium
(a) valence electrons only in air with the help of four + q charges situated at the vertices
(b) electrons of inner shells of a square. The net electrostatic force on q is given by
(c) both valence electrons and electrons of inner shell (a) Newton’s +q
(d) yet to be established
(b) Coulomb’s law
3. Quantisation of charge implies
(a) charge cannot be destroyed (c) Principle of superposition +q +q
(b) charge exists on particles
(d) Net electric flux out the P
(c) there is a minimum permissible charge on a particle +q
position of +q.
(d) charge, which is a fraction of a coulomb is not possible. +q
4. What happens when some charge is placed on a soap 12. Which of the following graphs shows the correct variation
bubble?
of force when the distance r between two charges varies ?
(a) Its radius decreases (b) Its radius increases
(a) (b)
(c) The bubble collapses (d) None of these
5. Which of the following statements is incorrect? F F
Force

Force
I. The charge q on a body is always given by q = ne,
where n is any integer, positive or negative.
II. By convention, the charge on an electron is taken to
be negative. Distance r
Distance r
III. The fact that electric charge is always an integral (c) (d)
multiple of e is termed as quantisation of charge.
F F
IV. The quatisation of charge was experimentally
Force

Force

demonstrated by Newton in 1912.


(a) Only I (b) Only II
(c) Only IV (d) Only III
6. Three charges +q, +2q and +4q are connected by strings as Distance r Distance r
shown in the figure. What is ratio of tensions in the strings 13. Assertion : The property that the force with which two
AB and BC ? charges attract or repel each other are not affected by the
A B C presence of a third charge.
d d Reason : Force on any charge due to a number of other
charge is the vector sum of all the forces on that charge
+q +2q +4q due to other charges, taken one at a time.
(a) 1 : 2 (b) 1 : 3 (c) 2 : 1 (d) 3 : 1 (a) Assertion is correct, reason is correct; reason is a
7. 1 C charge is equivalent to charge on how much number of correct explanation for assertion.
protons? (b) Assertion is correct, reason is correct; reason is not a
(a) 6 × 1018 (b) 7 × 1019 correct explanation for assertion
(c) 8 × 10 20 (d) 9 × 1021 (c) Assertion is correct, reason is incorrect
8. When a body is charged by induction, then the body (d) Assertion is incorrect, reason is correct.
(a) becomes neutral 14. The metal knob of a gold leaf electroscope is touched with
(b) does not lose any charge a positively charged rod. When it is taken away the leaves
(c) loses whole of the charge on it stay separated. Now the metal knob is touched by negatively
charged rod. The separation between the leaves
ELECTRIC CHARGES AND FIELDS 219

(a) increases 22. The electric charge required to expand a soap bubble to
(b) decreases twice its dimension is
(c) remains same (a) 8p Î0 r3 (7 Pr + 12 T) (b) 8p Î0 r 2 (7 Pr + 12 T)
(d) first increases then decreases.
15. Two identical metal spheres A and B are supported on (c) 8p Î0 r 3 (6 Pr + 12 T) (d) 8p Î0 r 3 (Pr + T)
insulating stands and placed in contact. What kind of charges 23. A large nonconducting sheet M is given a uniform charge
will A and B develop when a negatively charged ebonite rod density. Two uncharged small metal rods A and B are placed
is brought near A? near the sheet as shown in figure. Then
(a) A will have a positive charge and B will have a negative M
charge (a) M attracts A
(b) A will have a negative charge and B will have a positive A B
(b) M attracts B
charge
(c) A attracts B
(c) Both A and B will have positive charges
(d) Both A and B will have negative charges (d) All of these
16. The force of repulsion between two electrons at a certain 24. Assertion : Consider two identical
distance is F. The force between two protons separated by charges placed distance 2d apart,
the same distance is (mp = 1836 me) along x-axis.
F The equilibrium of a positive test
(a) 2 F (b) F (c) 1836 F (d) charge placed at the point O
1836
17. The force between two small charged spheres having midway between them is stable for
displacements along the x-axis.
charges of 1 × 10–7 C and 2 × 10–7 C placed 20 cm apart
Reason: Force on test charge is zero.
in air is
(a) Assertion is correct, reason is correct; reason is a
(a) 4.5 × 10–2 N (b) 4.5 × 10–3 N
–2 correct explanation for assertion.
(c) 5.4 × 10 N (d) 5.4 × 10–3 N
(b) Assertion is correct, reason is correct; reason is not a
18. Two charge q and –3q are placed fixed on x–axis
correct explanation for assertion
separated by distance d. Where should a third charge 2q
(c) Assertion is correct, reason is incorrect
be placed such that it will not experience any force?
(d) Assertion is incorrect, reason is correct.
q –3q 25. Three charges –q1 , +q2 and –q3 are place as shown in the
A d B figure. The x - component of the force on –q1 is proportional
to
d - 3d d + 3d q2 q3
(a) (b)
2 2 (a) - 2 cos q Y
b2 a -q 3
d + 3d d - 3d q2 q3
(c) (d) (b) + 2 sin q
2 2 b2 a
19. Two insulated charged metalic sphere P and Q have their q 2 q3 a
(c) + 2 cos q
centres separated by a distance of 60 cm. The radii of P b2 b
a
and Q are negligible compared to the distance of q2 q3
separation. The mutual force of electrostatic repulsion if (d) - 2 sin q -q 1 +q 2 X
b2 a
the charge on each is 3.2 × 10–7 C is
26. A charged ball B hangs from a silk thread S, which makes an
(a) 5.2 × 10–4 N (b) 2.5 × 10–3 N
–3
angle q with a large charged conducting sheet P, as shown
(c) 1.5 × 10 N (d) 3.5 × 10–4 N in the figure. The surface charge density s of the sheet is
20. If a charge q is placed at the centre of the line joining two proportional to
equal charges Q such that the system is in equilibrium then
(a) cot q P
the value of q is q
(a) Q/2 (b) –Q/2 (c) Q/4 (d) –Q/4 (b) cos q S

21. Two positive ions, each carrying a charge q, are separated (c) tan q
by a distance d. If F is the force of repulsion between the (d) sin q B
ions, the number of electrons missing from each ion will be 27. Two balls of same mass and carrying equal charge are hung
(e being the charge of an electron) from a fixed support of length l. At electrostatic equilibrium,
4pe0 Fd 2 4 pe0 Fe 2 assuming that angles made by each thread is small, the
(a) (b) separation, x between the balls is proportional to :
e2 d2
(a) l (b) l 2 (c) l 2/3 (d) l 1/3
4pe0 Fd 2 4 pe0 Fd 2 28. A solid conducting sphere of radius a has a net positive
(c) (d) charge 2Q. A conducting spherical shell of inner radius b
e2 q2 and outer radius c is concentric with the solid sphere and
EBD_7418
220 PHYSICS

has a net charge – Q.The surface charge density on the 35. In fig., two equal positive point charges q1 = q2 = 2.0 µC
inner and outer surfaces of the spherical shell will be interact with a third point charge Q = 4.0 µC. The magnitude,
2Q Q as well as direction, of the net force on Q is
(a) - ,
(a) 0.23 N in the
4pb 4pc 2
2
+
q1 = 2.0 µ C
+x-direction
Q Q a 0.50 m
(b) - , (b) 0.46 N in the 0.30 m
Q = 4.0 µ C
4pb 4pc 2
2 b +x-direction a + x
0.40 m
Q c (c) 0.23 N in the 0.30 m
(c) 0, –x-direction
4pc 2 (d) 0.46 N in the
+
q2 = 2.0 µ C
(d) None of the above –x-direction
29. Two pith balls carrying equal charges are suspended from a 36. Force between two identical charges placed at a distance of
common point by strings of equal length. The equilibrium r in vacuum is F. Now a slab of dielectric of dielectric contrant
separation between them is r. Now the strings are rigidly 4 is inserted between these two charges. If the thickness of
clamped at half the height. The equilibrium separation be- the slab is r/2, then the force between the charges will become
tween the balls now become 3 4 F
(a) F (b) F (c) F (d)
5 9 2
æ r ö æ 2r ö
(a) ç3 ÷ (b) ç ÷ 37. Two particle of equal mass m and charge q are placed at a
è 2ø è 3ø y
distance of 16 cm. They do not experience any force. The
2 q
æ 2r ö æ r ö y/2 value of is
(c) ç ÷ (d) ç ÷ m
è 3 ø è 2ø G
pe0
30. Two equal point charges each of 3mC are separated by a (a) l (b) (c) 4pe 0 (d) 4pe0 G
G
certain distance in metres. If they are located at 38. A uniformly charged conducting sphere of 4.4 m diameter
(iˆ + ˆj + kˆ ) and (2iˆ + 3 ˆj + kˆ) , then the electrostatic force has a surface charge density of 60 mC m–2. The charge
between them is on the sphere is
(a) 9 × 103 N (b) 16 × 10–3 N (a) 7.3 × 10–3C (b) 3.7 × 10–6 C
–3 (d) 9 × 10–2 N –6
(c) 10 N (c) 7.3 × 10 C (d) 3.7 × 10–3 C
31. A total charge Q is broken in two parts Q1 and Q2 and they
are placed at a distance R from each other. The maximum Topic 2: Electric Field, Electric Field Lines
force of repulsion between them will occur. when and Electric Dipole
Q Q Q 2Q
(a) Q2 = , Q1 = Q - (b) Q2 = , Q1 = Q - 39. Two point charges + Q and + q are separated by a certain
R R 4 3
distance. If + Q > + q then in between the charges the electric
Q 3Q Q Q field is zero at a point
(c) Q2 = , Q1 = (d) Q1 = , Q2 =
4 4 2 2 (a) closer to + Q
32. Two particles A and B having equal charges are placed at a
(b) closer to + q
distance d apart. A third charged particle placed on the
perpendicular bisection of AB at distance x. The third particle (c) exactly at the mid-point of line segment joining + Q and + q.
experiences maximum force when (d) no where on the line segment joining + Q and + q.
d d d 40. If an electron has an initial velocity in a direction different
d
(a) x= (b) x= (c) x= (d) x= from that of an electric field, the path of the electron is
2 2 2 2 3 2
(a) a straight line (b) a circle
33. Among two discs A and B, first have radius 10 cm and charge (c) an ellipse (d) a parabola
10–6 µC and second have radius 30 cm and charge 10–5C. 41. If one penetrates a uniformly charged spherical cloud,
When they are touched, charge on both q A and q B electric field strength
respectively will, be (a) decreases directly as the distance from the centre
(a) q A = 2.75μC, q B =3.15μC (b) increases directly as the distance from the centre
(b) q A = 1.09μC, q B = 1.53μC (c) remains constant
(c) q A = q B = 5.5 μC (d) None of these
(d) None of these 42. Electric lines of force about a negative point charge are
34. An isolated charge q1 of mass m is suspended freely by a (a) circular anticlockwise (b) circular clockwise
thread of length l. Another charge q2 is brought near it (c) radial, inwards (d) radial, outwards
(r >> l). When q1 is in equilibrium, tension in thread will be 43. Electric lines of force
(a) exist everywhere
(a) mg (b) exist only in the immediate vicinity of electric charges
(b) > mg l (c) exist only when both positive and negative charges
(c) < mg are near one another
q2
r
q1 (d) are imaginary
(d) None of these
ELECTRIC CHARGES AND FIELDS 221

44. For distance far away from centre of dipole the change in (a) a translational force only in the direction of the field
magnitude of electric field with change in distance from the (b) a translational force only in the direction normal to the
centre of dipole is direction of the field
(a) zero. (c) a torque as well as a translational force
(b) same in equatorial plane as well as axis of dipole. (d) a torque only
(c) more in case of equatorial plane of dipole as compared 53. An electric dipole has a pair of equal and opposite point
to axis of dipole. charges q and –q separated by a distance 2x. The axis of
(d) more in case of axis of dipole as compared to equatorial the dipole is
plane of dipole. (a) from positive charge to negative charge
45. The electric field at a point on equatorial line of a dipole (b) from negative charge to positive charge
and direction of the dipole moment (c) perpendicular to the line joining the two charges
(a) will be parallel drawn at the centre and pointing upward direction
(b) will be in opposite direction (d) perpendicular to the line joining the two charges
(c) will be perpendicular drawn at the centre and pointing downward direction
(d) are not related 54. The electric field near a conducting surface having a
r uniform surface charge density is given by
r of dipole moment p is placed in a uniform electric
46. If a dipole
s
field E , then torque acting on it is given by (a) and is parallel to the surface
r rr r r r e0
(a) t = p.E r (b) t = p ´ E
r r r r r (b)
2s
and is parallel to the surface
(c) t = p + E (d) t = p - E
e0
47. A metallic sphere is placed in a uniform electric field. The s
line of force follow the path (s) shown in the figure as (c) and is normal to the surface
e0
(a) 1 1 1 2s
2 2 (d) and is normal to the surface
(b) 2
3 3 e0
(c) 3 4 4 55. Select the correct statements from the following
(d) 4 I. Inside a charged or neutral conductor, electrostatic
48. Three infinitely long charge sheets are placed as shown in field is zero.
figure. The electric field at point P is II. The electrostatic field at the surface of the charged
2s conductor must be tangential to the surface at any
Z
(a) e k̂ point.
0 s III. There is no net charge at any point inside the
Z = 3a
4s conductor.
(b) e k̂ P
0
(a) I and II (b) I and III
-2s (c) II and III (d) I, II and III
2s ˆ Z=a
(c) - e k 56. In the figure, charge q is placed at origin O. When the charge q
0 X
4s ˆ is displaced from its position the electric field at point P changes
(d) - k -s Z = -a
e0 (a) at the same time when y
49. If Ea be the electric field strength of a short dipole at a q is displaced.
point on its axial line and Ee that on the equatorial line at P
the same distance, then OP
(b) at a time after where c is
(a) Ee= 2Ea (b) Ea = 2Ee c q x
(c) Ea = Ee ur (d) None of these the speed of light. O q
ur electric
50. Whenuran electric dipole P is placed inur a uniform
field E then at what angle between P and E the value OP cos q
of torque will be maximum? (c) at a time after
c
(a) 90° (b) 0° (c) 180° (d) 45°
51. Figure shows some of the electric field lines corresponding OP sin q
(d) at a time after
to an electric field. The figure suggests that c
57. Which of the following graphs shows the correct variation in
magnitude of torque on an electric dipole rotated in a uniform
A B C
electric field from stable equillibrium to unstable equillibrium?
(a) (b)
t t
Torque

Torque

(a) EA > EB > EC (b) EA = EB = EC


(c) EA = EC > EB (d) EA = EC < EB
52. An electric dipole is placed at an angle of 30° to a
non-uniform electric field. The dipole will experience
Angle rotated q Angle rotated q
EBD_7418
222 PHYSICS

64. A hollow insulated conduction sphere is given a positive


(c) (d)
charge of 10 mC. What will be the electric field at the centre
t t of the sphere if its radius is 2 m?
(b) 5 mCm–2 (c) 20 mCm–2 (d) 8 mCm–2

Torque
(a) Zero
Torque

65. The number of electric lines of force that radiate outwards


from one coulomb of charge in vacuum is
(a) 1.13 × 1011 (b) 1.13 × 1010
(c) 0.61 × 10 11 (d) 0.61 × 109
Angle rotated q Angle rotated q 66. The electric intensity due to a dipole of length 10 cm and
58. Assertion : A point charge is brought in an electric field, the having a charge of 500 mC, at a point on the axis at a distance
field at a nearby point will increase or decrease, depending 20 cm from one of the charges in air, is
on the nature of charge. (a) 6.25 × 107 N/C (b) 9.28 × 107 N/C
(c) 13.1 × 10 N/C11 (d) 20.5 × 107 N/C
Reason : The electric field is independent of the nature of
67. Intensity of an electric field (E) depends on distance r, due
charge.
to a dipole, is related as
(a) Assertion is correct, reason is correct; reason is a
correct explanation for assertion. 1 1 1 1
(a) E µ (b) E µ 2 (c) E µ 3 (d) E µ 4
(b) Assertion is correct, reason is correct; reason is not a r r r r
correct explanation for assertion 68. If the dipole of moment 2.57 × 10–17 cm is placed into an
(c) Assertion is correct, reason is incorrect electric field of magnitude 3.0 × 104 N/C such that the fields
(d) Assertion is incorrect, reason is correct. lines are aligned at 30° with the line joining P to the dipole,
59. A rod of length 2.4 m and radius 4.6 mm carries a negative what torque acts on the dipole?
charge of 4.2 × 10–7 C spread uniformly over it surface. (a) 7.7 × 10–13 Nm (b) 3.855 × 10–13 Nm
The electric field near the mid–point of the rod, at a point (c) 3.855 × 10 Nm –15 (d) 7.7 × 10–15 Nm
on its surface is
69. An electric dipole is placed at an angle of 30° with an electric
(a) –8.6 × 105 N C–1 (b) 8.6 × 104 N C–1 field of intensity 2 × 105 NC–1, It experiences a torque of 4 Nm.
(c) –6.7 × 10 N C 5 –1 (d) 6.7 × 104 N C–1 Calculate the charge on the dipole if the dipole length is 2 cm.
60. If electric field in a region is radially outward with magnitude (a) 8 mC (b) 4 mC (c) 8 mC (d) 2 mC
E = Ar, the charge contained in a sphere of radius r centred 70. On decreasing the distance between the two charges of a
at the origin is dipole which is perpendicular to electric field and decreasing
1 1 A 4 p eo A the angle between the dipole and electric field, the torque
(a) A r 3 (b) 4peoAr3 (c) 3 (d) on the dipole
4 p eo 4 p eo r r3
(a) increases (b) decreases
61. The electric field intensity just sufficient to balance the earth’s
(c) remains same (d) cannot be predicted
gravitational attraction on an electron will be: (given mass
71. Assertion : On going away from a point charge or a small
and charge of an electron respectively are 9.1 ´10-31 kg electric dipole, electric field decreases at the same rate in
both the cases.
and 1.6 ´ 10-19 C .) Reason : Electric field is inversely proportional to square
(a) –5.6 × 10–11 N/C (b) –4.8 × 10–15 N / C of distance from the charge or an electric dipole.
(c) –1.6 × 10–19 N/C (d) –3.2 × 10–19 N / C (a) Assertion is correct, reason is correct; reason is a
correct explanation for assertion.
62. The insulation property of air breaks down when the electric
(b) Assertion is correct, reason is correct; reason is not a
field is 3 × 106 Vm–1. The maximum charge that can be given
correct explanation for assertion
to a sphere of diameter 5 m is approximately
(c) Assertion is correct, reason is incorrect
(a) 2 × 10–2 C (b) 2 × 10–3 C (d) Assertion is incorrect, reason is correct.
(c) 2 × 10 C –4 (d) 2 × 10–5 C 72. n identical point charges are kept symmetrically on the
63. ABC is an equilateral triangle. Charges +q are placed at periphery of the circle x2 + y2 = R2 in xy plane. The resultant
each corner as shown in fig. The electric intensity at centre electric field at (0, 0, R) is E1 and at (0, 0, 2R) is E2. The ratio
O will be E
+q A of 1 is
1 q E2
(a) 5 5 5 5 5 5
4 p Îo r (a) (b) (c) (d)
4 2 2 4 ur
r ur 2
2
1 q 73. Two point dipoles of dipole moment p1 and p 2 are at a
(b) O ur ur
4 p Îo r 2 r r distance x from each other and p1 || p 2 . The force between
+q +q the dipoles is :
1 3q 1 4 p1 p2
(c) B C 1 3 p1 p2
4 p Îo r 2 (a) (b)
4pe0 x 4 4pe 0 x 4
(d) zero 1 6 p1 p2 1 8 p1 p2
(c) (d)
4pe0 x 4 4pe0 x 4
ELECTRIC CHARGES AND FIELDS 223

y
74. Let r (r ) =
Q
r be the charge density distribution for a (a) -14.4iˆ + 10.8jˆ O q = –8.0 nC x

p R4 (b) -14.4iˆ - 10.8jˆ
solid sphere of radius R and total charge Q. For a point ‘P’

r=
1.6 m

2.0
inside the sphere at distance r1 from the centre of the sphere, (c) -10.8iˆ + 14.4ˆj

m
the magnitude of electric field is :
(d) -10.8iˆ - 14.4ˆj
Q Qr12 Qr12 1.2 m
(a) (b) (c) (d) 0
4p Î0 r12 4p Î0 R4 3p Î0 R4 81. Two very long line charges of uniform charge density +l
and –l are placed along same line with the separation
75. In the figure the electric lines on the right have twice the
between the nearest ends being 2a, as shown in figure. The
separation of those on the left. If a charge particle takes time
electric field intensity at point O is
t to move a distance x in left region, then it will take time to a
a
travel the same distance in the right side region is : – +
O ++++++
l l l
(a) (b) 0 (c) (d)
t 2pe 0 a pe 0 a 4pe 0 a
(a) (b) t (c) 2t (d) 2t
2 82. Figure shows an electric quadrupole, with quadrupole
76. A small metal ball is brought into contact alternatively with moment (Q = 2ql2). The electric field at a distance from its
points A, B and C of the charged body shown in figure. centre at the axis of the quadrupole is given by
After each contact the charge of the ball is determined
approximately by touching it against an electroscope. If qA,
qB and qC are the charges, then
(a) qA > qB > qC æ 1 ö Q æ 1 ö 2Q
(b) qA < qB < qC (a) ç 4p Î ÷
è 4 (b) ç 4p Î ÷ 4
0ø r è 0ø r
(c) qA = qB, qC = 0 æ 1 ö 3Q
(d) qA = 0, qB = qC (c) ç 4p Î ÷ (d) None of these
è 0ø r4
77. An electric dipole, consisting of two opposite charges of 83. Two identical electric dipoles are arranged on x-axis as
2 ´ 10 -6 C each separated by a distance 3 cm is placed in an shown in figure. Electric field at the origin will be
y
electric field of 2 ´ 10 5 N/C. Torque acting on the dipole is (a) Zero
kp 2 ˆ
(a) 12 ´ 10 -1 N - m (b) 12 ´ 10 -2 N - m (b) j
r3
(c) 12 ´ 10 -3 N - m (d) 12 ´ 10 -4 N - m
- kp 2 ˆ x
78. Point charge q moves from point P to point S along the path (c) j 45° 45°
PQRS (as shown in fig.) in a uniform electric field r3
- kp ˆ kp ˆ
E pointing co-parallel to the positive direction of X-axis. (d) i- 3 j r r
The coordinates of the points Y
r3 r
P, Q, R and S are (a, b, 0), 84. A liquied drop having 6 excess electrons is kept stationary
(2 a, 0, 0), (a, –b, 0) and (0, 0, 0) P under a uniform electric field of 25.5 KVm–1. The density of
respectively. E liquid is 1.26 × 103 kg m –3. The radius of the drop is (neglect
S Q buoyany )
The workdone by the field in
R
(a) 4.3 × 10–7 m (b) 7.3 × 10–7 m
the above case is given by the (c) 0.078 × 10–7 m (d) 3.4 × 10–7 m
expression
(a) q E A (b) – q E A Topic 3: Electric Flux and Gauss’s Law

(c) q E A 2
2
(d) q E [(2 a) + b ]
2 85. The Gaussian surface
(a) can pass through a continuous charge distribution.
79. A pendulum bob of mass m carrying a charge q is at rest with
(b) cannot pass through a continuous charge distribution.
its string making an angle q with the vertical in a uniform (c) can pass through any system of discrete charges.
horizontal electric field E. The tension in the string is (d) can pass through a continuous charge distribution as
mg qE mg qE well as any system of discrete charges.
(a) and (b) and 86. Gauss's law is valid for
sin q cos q cos q sin q (a) any closed surface
qE mg (b) only regular close surfaces
(c) (d) (c) any open surface
mg qE
(d) only irregular open surfaces
80. A point charge q = – 8.0 nC is located at the origin. The 87. The total electric flux emanating from a closed surface
electric field (in NC –1) vector at the point x = 1.2 m, y = – 1.6 m, enclosing an a-particle is (e-electronic charge)
as shown in Fig., is 2e e e0 e
(a) (b) (c) ee0 (d)
e0 e0 4
EBD_7418
224 PHYSICS

r r
88. For a given surface the Gauss's law is stated as Ñò E . dA = 0 . 98. A charge +q is at a distance L/2 above a square of side L.
From this we can conclude that Then what is the flux linked with the surface?
q 2q q 6q
(a) E is necessarily zero on the surface (a) 4e (b) 3e (c) 6e (d) e
(b) E is perpendicular to the surface at every point 0 0 0 0
(c) the total flux through the surface is zero 99. A point charge is kept at the centre of metallic insulated
(d) the flux is only going out of the surface spherical shell. Then
89. The electric field inside a spherical shell of uniform (a) electric field out side the sphere is zero
surface charge density is (b) electric field inside the sphere is zero
(a) zero (c) net induced charge on the sphere is zero
(b) constant different from zero (d) electric potential inside the sphere is zero
(c) proportional to the distance from the curve 100. Positive electric flux indicates that electric lines of force are
(d) None of the above directed
90. In the
r figure
r the net electric flux through the area A is (a) outwards (b) inwards
f = E × A when the system is in air. On immersing the system (c) either (a) or (b) (d) None of these
in water the net electric flux through the area 101. A point charge +Q is positioned at the center of the base of
a square pyramid as shown. The flux through one of the
four identical upper faces of the pyramid is
Q
Q (a)
16e 0
Q
A (b)
4e 0
Q
(c) +Q
8e 0
(a) becomes zero (b) remains same
(c) increases (d) decreases (d) None of these
91. A charge q is placed at the centre of the open end of a 102. Electric flux over a surface in an electric field may be
cylindrical vessel. The flux of the electric field through the (a) positive (b) negative
surface of the vessel is (c) zero (d) All of these
(a) zero q 103. If the electric flux entering and leaving an enclosed surface
(b) q/eo respectively is f1 and f2, the electric charge inside the surface
(c) q/2eo will be
(a) (f2 + f2) × eo (b) (f2 – f2) × eo
(d) 2q/eo
(c) (f1 + f2) × eo (d) (f2 – f1) × eo
92. In a region of space having a uniform electric field E, a r
hemispherical bowl of radius r is placed. The electric flux f 104. Consider an electric field E = E0 xˆ where E0 is a constant.
through the bowl is The flux through the shaded area (as shown in the figure)
due to this field is z
(a) 2pRE (b) 4pR2E (c) 2pR2E (d) pR2E
93. A cylinder of radius R and length l is placed in a uniform (a) 2E0a2 (b) 2E0 a 2
(a,0,a) (a,a,a)

electric field E parallel to the axis of the cylinder. The total


flux over the curved surface of the cylinder is E0a 2
(a) zero (b) pR2E (c) 2pR2E (d) E / pR2 (c) E0 a2 (d) (0,0,0) (0,a,0)
y
2 x
94. At the centre of a cubical box + Q charge is placed. The
105. Electric charges are distributed in a small volume. The flux
value of total flux that is coming out a wall is
of the electric field through a spherical surface of radius 1 m
(a) Q / eo (b) Q / 3eo (c) Q / 4eo (d) Q / 6eo
95. An electric dipole is put in north-south direction in a surrounding the total charge is 100 V-m. The flux over the
sphere filled with water. Which statement is correct? concentric sphere of radius 2 m will be
(a) Electric flux is coming towards sphere. (a) 25 V-m (b) 50 V-m (c) 100 V-m (d) 200 V-m
(b) Electric flux is coming out of sphere. 106. A loop of diameter d is rotated in a uniform electric field until
(c) Electric flux entering into sphere and leaving the the position of maximum electric flux is found. The flux in
sphere are same.
this position is measured to be f. What is the electric field
(d) Water does not permit electric flux to enter into sphere.
96. The surface density on the copper sphere is s. The electric strength ?
4f 2f f pfd 2
field strength on the surface of the sphere is (a) 2 (b) 2 (c) 2 (d)
(a) s (b) s/2 (c) s / 2eo (d) Q / eo pd pd pd 4
r
97. A charge Q is enclosed by a Gaussian spherical surface of
radius R. If the radius is doubled, then the outward electric
(r V
)
107 A surface has the area vector A = 2i$ + 3 $j m2 . The flux of an
flux will electric field through it if the field is E = 4$i :
m
(a) increase four times (b) be reduced to half (a) 8 V-m (b) 12 V-m (c) 20 V-m (d) zero
(c) remain the same (d) be doubled
ELECTRIC CHARGES AND FIELDS 225

108. A hollow cylinder has a charge q coulomb within it. If fis the (a) 4 ´ 103 C (b) -4 ´ 103 C
electric flux in units of voltmeter associated with the curved 3
surface B, the flux linked with the plane surface A in units of ( -4 ´ 10 )
(c) C (d) - 4 ´ 103 e 0 C
voltmeter will be e
B
q 110. The
(a) 2e (b) f r electric field in a region of space is given by,
0 3 C A E = Eoiˆ + 2Eoˆj where Eo = 100 N/C. The flux of the field
q 1æ q ö through a circular surface of radius 0.02 m parallel to the
(c) e - f (d) 2 ç e - f÷ Y-Z plane is nearly:
0 è 0 ø
109. The inward and outward electric flux for a closed surface in (a) 0.125 Nm2/C (b) 0.02 Nm2/C
units of N-m2/C are respectively 8 × 103 and 4 × 103. Then (c) 0.005 Nm2/C (d) 3.14 Nm2/C
the total charge inside the surface is [where e0 = permittivity
constant]

Exercise 2 : Exemplar & Past Year MCQs


(a) in Fig. (iv) is the largest
NCERT Exemplar MCQs
(b) in Fig. (iii) is the least
1. In figure two positive charges q2 and q3 fixed along the (c) in Fig. (ii) is same as Fig. (iii) but is smaller than Fig. (iv)
y-axis, exert a net electric force in the + x-direction on a (d) is the same for all the figures
charge q1 fixed along the x-axis. If a positive charge Q is 4. Five charges q1, q2, q3,q4, and q5 are fixed at
added at (x, 0), the force on q1 their positions as shown in Figure,
S is a Gaussian surface. The Gauss’ q1
y y
q q2
(i) (ii)
q2 q2 law is given by ò E.dS = . Which q3
s e0 q5 q4
Q of the following statements is correct?
x x
q1 q1 O (x, 0) (a) E on the LHS of the above equation will have a
contribution from q1, q5 and q1, q5 and q3 while q on
q3 q3 the RHS will have a contribution from q2 and q4 only
(a) shall increase along the positive x-axis (b) E on the LHS of the above equation will have a
(b) shall decrease along the positive x-axis contribution from all charges while q on the RHS will
(c) shall point along the negative x-axis have a contribution from q2 and q4 only
(d) shall increase but the direction changes because of (c) E on the LHS of the above equation will have a
the intersection of Q with q2 and q3 contribution from all charges while q on the RHS will
2. A point positive charge is brought near an isolated conducting have a contribution from q1, q3 and q5.
sphere (figure). The electric field is best given by (d) Both E on the LHS and q on the RHS will have
contributions from q2 and q4 only
+q +q 5. Figure shows electric field lines in which an electric dipole P
+q +q is placed as shown. Which of the following statements is
correct?

–p p +p

(a) The dipole will not experience any force


(b) The dipole will experience a force towards right
(a) (b) (c) (d)
(c) The dipole will experience a force towards left
3. The electric flux through the surface (d) The dipole will experience a force upwards
s 6. A point charge + q is placed at a distance d from an isolated
conducting plane. The field at a point P on the other side of
s s the plane is
+q (a) directed perpendicular to the plane and away from the
+q +q +q plane
(b) directed perpendicular to the plane but towards the plane
(c) directed radially away from the point charge
(i) (ii) (iii) (iv) (d) directed radially towards the point charge
EBD_7418
226 PHYSICS

7. A hemisphere is uniformely charged positively. The electric The charges begin to leak from both the spheres at a constant
field at a point on a diameter away from the centre is directed rate. As a result, the spheres approach each other with a
(a) perpendicular to the diameter velocity v. Then v varies as a function of the distance x
(b) parallel to the diameter between the spheres, as : [NEET 2016, A]
1
(c) at an angle tilted towards the diameter 1 - -1
(d) at an angle tilted away from the diameter (a) v µ x2 (b) v µ x (c) v µ x 2 (d) v µ x
15. An infinitely long solid cylinder of radius R has a uniform
Past Year MCQs volume charge density r. It has a spherical cavity of radius
r R/2 with its centre on the axis of the cylinder, as shown in
8. In a medium of dielectric constant K, the electric field is E .
the figure. The magnitude of the electric field at the point P,
If Î0 is permittivity of the free space, the electric
displacement vector is [AIIMS 2014, C] which is at a distance 2R from the axis of the cylinder, is
r r r 23rR
KE E Î0 E r given by the expression . The value of k is
(a) (b) (c) (d) K Î0 E 16ke0
Î0 K Î0 K
p [AIIMS 2016, S]
9. ˆ
Two point dipoles pk and k̂ are located at (0, 0, 0) and
2 (a) 6
(1m, 0, 2m) respectively. The resultant electric field due to
the two dipoles at the point (1m, 0, 0) is – [BITSAT 2014, C]
(b) 5
9p -7p
(a) 32p Î k̂ (b) 32p Î k̂
0 0
7p (c) 7
(c) 32p Î k̂ (d) None of these
0
10. Three charge q, Q and 4q are placed in a straight line of (d) 4
1
length l at points distant 0, and l respectively from one
2
end. In order to make the net froce on q zero, the charge Q 16. An electric dipole of moment P is placed in a uniform
must be equal to [AIIMS 2015, A] electric field E such that P points along E . If the dipole
-q is slightly rotated about an axis perpendicular to the plane
(a) –q (b) – 2q (c) (d) q containing E and P and passing through the centre of
2
11. A sphere of radius R carries charge density d proportional to the dipole, the dipole executes simple harmonic motion.
Consider I to be the moment of inertia of the dipole about
the square of the distance from the centre such that d = CR2, the axis of rotation. What is the time period of such
where C is a positive constance. At a distance R/2 from the oscillation? [AIIMS 2016, C]
centre, the magnitude electric field is [AIIMS 2015, C]
CR 3 3 CR 3
CR 3 (a) ( pE / I) (b) 2p ( I / pE)
CR
(a) (b) (c) (d)
20Î0 10 Î0 5Î0 40Î0 (c) 2p ( I / 2pE) (d) None of these
12. A long cylindrical shell carries positive surface charge s in 17. The region between two concentric spheres
the upper half and negative surface charge - s in the lower of radii 'a' and 'b', respectively (see figure),
half. The electric field lines around the cylinder will look like
figure given in : (figures are schematic and not drawn to A
have volume charge density r= , where a
scale) [JEE Main 2015, C] r Q
A is a constant and r is the distance from the b
(a) (b) centre.
At the centre of the spheres is a point charge Q. The value
of A such that the electric field in the region between the
spheres will be constant, is : [JEE Main2016, S]
(c) (d)
2Q 2Q
(a) (b)
p a 2 - b2
( pa 2
13. A solid sphere of radius R has a charge Q distributed in its
volume with a charge density r = kra, where k and a are
) Q
Q
constants and r is the distance from its centre. If the electric (c) (d)
2
2p b2 - a 2
(
field at r =
R 1
is times that at r = R, the value of a is
2pa )
2 8 18. The surface charge density of a thin charged disc of radius
[BITSAT 2015, A] R is s. The value of the electric field at the centre of the disc
(a) 3 (b) 5 (c) 2 (d) 7 s
14. Two identical charged spheres suspended from a common is . With respect to the field at the centre, the electric
2 Î0
point by two massless strings of lengths l, are initially at a field along the axis at a distance R from the centre of the disc
distance d (d << l) apart because of their mutual repulsion. [BITSAT 2016, C]
ELECTRIC CHARGES AND FIELDS 227

(a) reduces by 70.7% (b) reduces by 29.3% by six surfaces x = ± a / 2, y = ± a / 2, z = ± a / 2. The electric
(c) reduces by 9.7% (d) reduces by 14.6% flux through this cubical surface is [BITSAT 2017, S]
19. Suppose the charge of a proton and an electron differ slightly. y
One of them is – e, the other is (e + De). If the net of
electrostatic force and gravitational force between two
hydrogen atoms placed at a distance d (much greater than
atomic size) apart is zero, then De is of the order of [Given x
mass of hydrogen mh = 1.67 × 10–27 kg] [NEET 2017, A]
(a) 10 C (b) 10 C (c) 10 C (d) 10–20 C
–23 –37 –47

20. Assertion : The positive charge particle is placed in front of


a spherical uncharged conductor. The number of lines of
–2C 2C 10C 12C
forces terminating on the sphere will be more than those (a) (b) (c) (d)
e0 e0 e0 e0
emerging from it.
Reason : The surface charge density at a point on the sphere 24. An electron falls from rest through a vertical distance h in a
uniform and vertically upward directed electric field E. The
nearest to the point charge will be negative and maximum in
direction of electric field is now reversed, keeping its
magnitude compared to other points on the sphere. magnitude the same. A proton is allowed to fall from rest in
[AIIMS 2017, C] it through the same vertical distance h. The time of fall of
(a) If both Assertion and Reason are correct and Reason the electron, in comparison to the time of fall of the proton is
is the correct explanation of Assertion. [NEET 2018, C]
(a) smaller (b) 5 times greater
(b) If both Assertion and Reason are correct, but Reason
(c) equal (d) 10 times greater
is not the correct explanation of Assertion. 25. Assertion : On moving a distance two times the initial
(c) If Assertion is correct but Reason is incorrect. distance away from an infinitely long straight uniformly
(d) If both the Assertion and Reason are incorrect. charged wire the electric field reduces to one third of the
ur
21. An electric dipole has a fixed dipole moment p , which makes initial value.
angle q with respect to x-axis. When subjected to an electric Reason : The electric field is inversely proportional to the
uur ur
field E1 = Eiˆ , it experiences a torque T1 = t iˆ . When distance from an infinitely long straight uniformly charged wire.
uur [AIIMS 2018, C]
subjected to another electric field E2 = 3E1 ˆj it
uur ur (a) Assertion is correct, reason is correct; reason is a
experiences torque T2 = -T1 . The angle q is :
correct explanation for assertion.
[JEE Main 2017, A] (b) Assertion is correct, reason is correct; reason is not a
(a) 60° (b) 90° (c ) 30° (d) 45° correct explanation for assertion
22. Two concentric conducting thin spherical shells A, and B (c) Assertion is correct, reason is incorrect
having radii rA and rB ((rB > rA) are charged to QA and –QB (d) Assertion is incorrect, reason is correct.
(|QB| > |QA|). The electric field along a line passing through 26. A square surface of side L meter in the plane of the paper is
the centre is [BITSAT 2017, C] placed in a uniform electric field E (volt/m) acting along the
same plane at an angle q with the horizontal side of the
square as shown in Figure. The electric flux linked to the
surface is E [AIIMS 2018, C]
(a) EL 2
(a) (b)
q
(b) EL2 cos q
(c) EL2 sin q
(d) zero
(c) (d) 27. A charged particle q is placed at the centre O of cube of
length L (A B C D E F G H). Another same charge q is placed
at a distance L from O. Then the electric flux through ABCD is
[BITSAT 2018, A]
23. A disk of radius a / 4 having a uniformly distributed charge (a) q /4pÎ0L E F
6 C is placed in the x - y plane with its centre at (–a / 2, 0, 0). D
(b) zero O
C
A rod of length a carrying a uniformly distributed charge 8 q q
C is placed on the x-axis from x = a /4 to x = 5a / 4. Two point (c) q/2 pÎ0L H
G
charges – 7 C and 3 C are placed at (a / 4, – a /4, 0) and (– 3a (d) q/3pÎ0L A
B
L
/4, 3a / 4, 0), respectively. Consider a cubical surface formed
EBD_7418
228 PHYSICS

Exercise 3 : Try If You Can


1. A solid sphere of radius R1 and volume charge density 2
æ lö
2
æ lö h1 - h2 - g ç ÷
r (a) h1 + h2 - g ç ÷ (b)
r = 0 is enclosed by a hollow sphere of radius R2 with è vø è vø
r
1/ 2 2
negative surface charge density s, such that the total charge æ lö h1 + h2 æ lö
in the system is zero. r0 is a positive constant and r is the (c) h1 + h2 - g ç ÷ (d) -gç ÷
è vø 2 è vø
distance from the centre of the sphere. The ratio R2 / R1 is 6. Two thin flat metal plates having large surface area are
s 2s r0 r0 charged separately to acquire charge densities + s and – s.
(a) r0 (b) r0 (c) (d)
2s s The plates are then brought near to each other and held
2. A positive charge +Q is fixed at a point A. Another positively parallel to each other (Fig.) : If EA, EB and EC denote the
charged particle of mass m and charge +q is projected from electric fields at the points A, B and C respectively, then
a point B with velocity u as shown in the figure. The point B which of the following will be true :
s
is at the large distance from A and at distance d from the line (a) E A = EC =
AC. The initial velocity is parallel to the line AC. The point e0
s A B C
C is at very large distance from A. The minimum distance (in (b) E A = E B = EC =
e0
s
meter) of +q from +Q during the motion is d (1 + A) . Find (c) E A = EC = 0, EB =
the value of A. e0
B –vely charged
2s
[Take Qq = 4pe0mu2d u (d) E A = EC = 0, E B = +vely charged
e0
and d = ( 2 - 1) meter.] d
7. A particle of charge q and mass m moves rectilinearly under
(a) 3 (b) 2 the action of electric field E = A – Bx, where A and B are
A C positive constants and x is distance from the point where
(c) 4 (d) 5 +Q particle was initially at rest then the distance traveled by the
3. Two identical blocks are kept on a frictionless horizontal particle before coming to rest and acceleration of particle at
table connected by a spring of stiffness k and of original that moment are respectively :
length l0. A total charge Q is distributed on the block such 2A qA
(a) ,0 (b) 0, -
that maximum elongation of spring at equilibrium is equal to x. B m
Value of Q is 2 A qA -2 A qA
(c) ,- (d) ,-
(a) 2l 0 4pe 0 k (l 0 + x ) (b) 2 x 4 pe 0 k (l 0 + x ) B m B m
8. A system consists of a uniform charged sphere of radius R
(c) 2(l 0 + x ) 4 pe 0 kx (d) (l 0 + x ) 4 pe 0 kx and a surrounding medium filled by a charge with the volume
a
4. A wire of length L (=20 cm), is bent into a semicircular arc. density r = , where a is a positive constant and r is the
r
If the two equal halves of the arc were each to be distance from the centre of the charge. The charge of the
uniformly charged with charges ± Q, [|Q| = 103e0 Coulomb sphere for which the electric field intensity E outside the
where e0 is the permittivity (in SI units) of free space] the sphere is independent of r is –
net electric field at the centre O of the semicircular arc (a) pR2a (b) 4pR2a (c) 2pR2a (d) 3pR2a/4
would be :] 9. A small sphere carrying a charge ‘q’ is hanging in between
Y two parallel plates by a string of length L. Time period of
(a) (50 × 103 N/C) $j pendulum is T0. When parallel plates are charged, the time
(b) (50 × 103 N/C) $i period changes to T. The ratio T/T0 is equal to
(c) (25 × 103 N/C) $j L
O X O m
(d) (25 × 103 N/C) $i
5. Two small balls having the same mass and charge and
located on the same vertical at heights h1 and h2 are thrown 1/ 2 3/ 2
in the same direction along the horizontal at the same æ qE ö æ g ö
(a) ç g + m ÷ (b) ç qE ÷
velocity v. The first ball touches the ground at a distance l ç ÷ çg+ ÷
from the initial vertical. At what height H2 will the second è g ø è m ø
ball be at this instant ? The air drag and the effect of the 1/ 2
charges induced on the ground should be neglected. æ g ö
(c) ç qE ÷ (d) None of these
çg+ ÷
è m ø
ELECTRIC CHARGES AND FIELDS 229

10. Two identical thin rings, each of radius a meter, are coaxially
Q 100Q 10Q 100Q
placed at a distance R meter apart. If Q1 coulomb and Q2
coulomb are respectively the charges uniformly spread on
(a) e0 (b) e0 (c) ( pe0 ) (d) ( pe0 )
the two rings, the work done in moving a charge q coulomb 15. Let there be a spherically symmetric charge distribution with
from the centre of one ring to that of the other is æ5 rö
q(Q1 - Q 2 )( 2 - 1) charge density varying as r(r ) = r0 çè - ÷ø upto r = R ,
(a) Zero (b) 4 R
4 2 pe0 a and r(r) = 0 for r > R , where r is the distance from the origin.
The electric field at a distance r(r < R) from the origin is
q 2 (Q1 + Q 2 ) q(Q1 + Q 2 )( 2 + 1)
(c) (d) given by
4pe 0 a 4 2pe 0 a
r0 r æ 5 r ö 4pr0 r æ 5 r ö
11. Two small balls of mass M each carrying charges + Q and (a) ç - ÷ (b) ç - ÷
– Q, connected by a massless rigid non-conducting rod of 4e 0 è 3 R ø 3e 0 è 3 R ø
length L lie along x-axis as shown. A uniform electric field 4r0 r æ 5 r ö r0 r æ 5 r ö
r (c) çè - ÷ø (d) ç - ÷
E = 3kˆ + 3 ˆj has been switched on. The angular velocity 4ε 0 4 R 3ε 0 è 4 R ø
vector of the dipole when dipole moment aligns with the 16. The thickness of a flat sheet of metal foil is d, and its area is
electric field is S. A charge q is located at a distance l from the centre of the
y
sheet such that d << S << l . Determine the force F with
(a)
3 2 pQ ˆ ˆ
2ML
-j+k ( ) which the sheet is attracted to the charge q, assuming that
the straight line connecting the charge to the centre of the
sheet is perpendicular to the surface of the sheet.
(b)
ML
(
6 2Q ˆ ˆ
-j+k ) –Q +Q
x (approximately)
q 2 Sd q 2 Sd q2 Sd 2q 2 Sd
(a) (b) (c) (d)
( ) 4p e 0l 6p 2 e0 l5 3p 2 e0 l5
2 5
(c)
3 2 pQ ˆ ˆ
-j-k 8p 2 e 0 l 5
z
ML 17. A conic surface is placed in a uniform electric field E as
shown in Fig. such that the field is perpendicular to the
(d)
3 2 pQ ˆ ˆ
ML
-j+k ( ) surface on the side AB. The base of the cone is of radius R,
and the height of the cone is h. The angle of the cone is q.
12. A non-conducting sphere of radius R has a positive charge A
Q uniformly distributed over its entire volume. A smaller,
concentric, spherical volume of radius r (r < R) is scooped q
out of the sphere. The magnitude of the electric field E at a
point inside the body at a distance x from the centre is E
(r < x < R) is (K = 1)(4p Î0) = constant) : h
KQ KQ r 3 B R
(a) 2 (b)
x x 2 R3
Find the magnitude of the flux that enters the cone's curved
KQ æ x - r ö
3 3
(c) ç ÷ (d) 0 surface from the left side. Do not count the outgoing flux
x 2 çè R 3 ÷ø (q < 45°)
13. An electric dipole is placed along the x-axis at the origin O. (a) ER [h cos q + p (R / 2) sin q]
A point P is at a distance of 20 cm from this origin such that (b) ER [h sin q + pR/2 cos q]
(c) ER [ h cos q + pR sin q]
OP makes an angle p/3 with the x-axis. If the electric field at
(d) None of these
P makes an angle q with the x-axis, the value of q would be
18. The flat base of a hemisphere of radius a with no charge
p p æ 3ö inside it lies in a horizontal plane. A uniform electric field
(a) (b) + tan -1 ç ÷
3 3 ç 2 ÷ ® p
è ø E is applied at an angle with the vertical direction. The
4
2p æ 3ö electric flux through the curved surface of the hemisphere is
(c) (d) tan -1 ç ÷
3 ç 2 ÷ 45° ®
è ø E
14. Electric charge is uniformly distributed
along a long straight wire of radius 1mm.
The charge per cm length of the wire is
Q coulomb. Another cylindrical surface of
radius 50 cm and length 1m symmetrically
pa2 E pa2 E ( p + 2 ) pa 2 E
encloses the wire as shown in the figure. (a) pa2 E (b) (c) (d)
The total electric flux passing through the 2 2 2 ( 2 2 )2
cylindrical surface is
EBD_7418
230 PHYSICS

19. A circular loop of radius a, centered at origin has five charges 20. Two small balls of mass m bearing a charge q each are
connected by a nonconducting thread of length 2l. At a
equally spaced on it. All charges have a magnitude Q, except
certain instant, the middle of the thread starts moving at a
the one at (0, a) which has a magnitude q, as shown in the constant velocity v perpendicular to the direction of the
figure. The electric field at the origin will be thread at the initial instant. Determine the minimum distance
y between the balls.
1 é 4Q + q ù ˆ q–
j 2lq 2
4 pe0 êë a 2 úû
(a) 2l q 2
+Q Q+ (a) (b)
q 2 + 8pe 0 mv 2 l q 2 + 4pe 0 mv 2 l
1 éQ + q ù ˆ x
j
4 pe0 êë a 2 úû
(b)
+Q Q+ 2l q 2 3lq 2
(c) (d)
1 é 4Q æ 2p ö q ù ˆ q 2 + 3pe 0 mv 2 l q 2 + 8pe 0 mv 2 l
(c) sin ç ÷ + ú j
4pe0 êë a 2 è 5 ø a2 û
1 é 4Q æ 2p ö q ù
cos ç ÷ + ú ˆj
4pe 0 êë a 2
(d)
è 5 ø a2 û

ANSWER KEYS
Exercise 1 : Topic -wise MCQs
1 (b) 12 (d) 23 (d) 34 (b) 45 (b) 56 (b) 67 (c) 78 (b) 89 (a) 100 (a)
2 (a) 13 (b) 24 (b) 35 (b) 46 (b) 57 (a) 68 (b) 79 (b) 90 (d) 101 (c)
3 (d) 14 (b) 25 (b) 36 (c) 47 (d) 58 (c) 69 (d) 80 (c) 91 (a) 102 (d)
4 (b) 15 (a) 26 (c) 37 (d) 48 (c) 59 (c) 70 (b) 81 (a) 92 (c) 103 (d)
5 (c) 16 (b) 27 (d) 38 (d) 49 (b) 60 (b) 71 (d) 82 (c) 93 (a) 104 (c)
6 (b) 17 (b) 28 (a) 39 (b) 50 (a) 61 (a) 72 (a) 83 (c) 94 (d) 105 (c)
7 (a) 18 (b) 29 (a) 40 (d) 51 (c) 62 (b) 73 (b) 84 (b) 95 (c) 106 (a)
8 (b) 19 (b) 30 (b) 41 (a) 52 (c) 63 (d) 74 (b) 85 (a) 96 (d) 107 (a)
9 (c) 20 (d) 31 (d) 42 (c) 53 (b) 64 (a) 75 (c) 86 (a) 97 (c) 108 (d)
10 (b) 21 (c) 32 (c) 43 (d) 54 (c) 65 (a) 76 (a) 87 (a) 98 (c) 109 (d)
11 (c) 22 (a) 33 (c) 44 (d) 55 (b) 66 (a) 77 (c) 88 (c) 99 (c) 110 (a)
Exercise 2 : Exemplar & Past Year MCQs
1 (a) 4 (b) 7 (a) 10 (a) 13 (c) 16 (b) 19 (b) 22 (a) 25 (a)
2 (a) 5 (c) 8 (d) 11 (d) 14 (c) 17 (c) 20 (d) 23 (a) 26 (d)
3 (d) 6 (a) 9 (b) 12 (c) 15 (a) 18 (a) 21 (a) 24 (a) 27 (b)
Exercise 3 : Try If You Can
1 (c) 3 (c) 5 (a) 7 (c) 9 (c) 11 (b) 13 (b) 15 (a) 17 (a) 19 (b)
2 (b) 4 (d) 6 (c) 8 (c) 10 (b) 12 (c) 14 (b) 16 (a) 18 (b) 20 (a)
17Chapter
ELECTROSTATIC POTENTIAL
AND CAPACITANCE

Trend
Analysis of NEET and AIIMS (Year 2010-2018)
5

4
Number of Questions

3
AIPMT/NEET
2 AIIMS

0
2010 2011 2012 2013 2014 2015 2016 2017 2018
Year

Trend
Analysis of JEE Main and BITSAT (Year 2010-2018)
4

3
Number of Questions

2
JEE Main
BITSAT
1

0
2010 2011 2012 2013 2014 2015 2016 2017 2018
Year

Chapter Utility Score (CUS)


Exam Weightage Important Concepts Difficulty Level CUS (Out of 10)
NEET 4 Electrostatic Potential
AIIMS 3 Capacitors & Capacitance 3.5/5 8/10
JEE Main 4 Grouping of Capacitors
BITSAT 3
EBD_7418
232 PHYSICS
ELECTROSTATIC POTENTIAL AND CAPACITANCE 233
EBD_7418
234 PHYSICS

Exercise 1 : Topic-wise MCQs

Topic 1: Electrostatic Potential and Equipotential Surfaces (C) Electric potential due (3) Varies inversly
to a charge at its own of radius
1. The electric potential inside a conducting sphere location is not defined
(a) increases from centre to surface
(D) Electric potential due (4) Infinite
(b) decreases from centre to surface
to uniformly charged
(c) remains constant from centre to surface
solid non-conducting
(d) is zero at every point inside
2. It becomes possible to define potential at a point in an electric sphere
field because electric field (a) (A) ® (2); (B) ® (1); (C) ® (4); (D) ® (3)
(a) is a conservative field (b) (A) ® (1); (B) ® (3); (C) ® (4); (D) ® (2)
(b) is a non-conservative field (c) (A) ® (4); (B) ® (1); (C) ® (3); (D) ® (4)
(c) is a vector field (d) (A) ® (3); (B) ® (2); (C) ® (1); (D) ® (4)
(d) obeys principle of superposition 9. Figure below shows a hollow conducting body placed in an
3. Which of the following about potential at a point due to a electric field. Which of the quantities are zero inside the
given point charge is true ? body?
The potential at a point P due to a given point charge (a) Electric field and potential
(a) is a function of distance from the point charge.
(b) varies inversely as the square of distance from the point (b) Electric field and charge density
charge. (c) Electric potential and charge density.
(c) is a vector quantity.
(d) is directly proportional to the square of distance from (d) Electric field, potential and charge density.
the point charge. 10. The value of electric potential at any point due to any electric
4. Which of the following quantities do not depend on the choice dipole is
of zero potential or zero potential energy? ur r ur r ur r ur r
p´r p´r p.r p.r
(a) Potential at a point (a) k . (b) k . (c) k . 2 (d) k .
(b) Potential difference between two points r2 r3 r r3
(c) Potential energy of a two-charge system 11. A hollow metal sphere of radius 5 cm is charged such that
(d) None of these the potential on its surface is 10 V. The potential at a distance
5. A cube of a metal is given a positive charge Q. For this of 2 cm from the centre of the sphere is
system, which of the following statements is true?
(a) Electric potential at the surface of the cube is zero (a) zero (b) 10 V (c) 4 V (d) 10/3 V
(b) Electric potential within the cube is zero 12. In moving from A to B along an electric field line, the work
(c) Electric field is normal to the surface of the cube done by the electric field on an electron is 6.4 × 10–19 J. If f1
(d) Electric field varies within the cube and f2 are equipotential surfaces, then the potential
6. A unit charge moves on an equipotential surface from a difference VC – VA is
point A to point B, then
(a) VA – VB = + ve (b) VA – VB = 0 B C
(c) VA – VB = – ve (d) it is stationary
E
7. The electric potential at a point on the equatorial line of an f2
electric dipole is A f1
(a) directly proportional to distance
(b) inversely proportional to distance (a) –4 V (b) 4 V (c) zero (d) 6.4 V
(c) inversely proportional to square of the distance 13. A non-conducting ring of radius 0.5 m carries a total charge
(d) None of these
of 1.11× 10 –10 C distributed non-uniformly on its
8. Match the Column I and Column II.
Column-I Column -II circumference producing an electric field E everywhere in
(A) Electric potential near (1) Negative l= 0
an isolated positive space. The value of the integral ò - E.dl
charge l =¥
(B) Electric potential near (2) Positive ( l = 0 being center of the ring) in volts is
an isolated negative (a) +2 (b) –1 (c) –2 (d) zero
charge
ELECTROSTATIC POTENTIAL AND CAPACITANCE 235

14. An electron is taken from point A to point B along the path 19. Consider the following statements and select the correct
AB in a uniform electric field of intensity E = 10 Vm –1. Side statement(s).
AB = 5 m, and side BC = 3 m. Then, the amount of work I. Electric field lines are always perpendicular to
done on the electron is equipotential surface.
II. No two equipotential surfaces can intersect each other.
B
E III. Electric field lines are in the direction of tangent to an
equipotential surface.
(a) I only (b) II only
A C (c) I and II (d) I, II and III
(a) 50 eV (b) 40 eV (c) – 50 eV (d) – 40 eV 20. Figure shows two hollow charged A
conductors A and B having same B
15. Consider the following statements and select the correct positive surface charge densities. B is
option placed inside A and does not touches
I. In an external electric field, the positive and negative it. On connecting them with a
charges of a non–polar molecule are displaced in conductor
opposite directions. (a) charge will flow from A to B
II. In non –polar molecules displacement stops when (b) charge will flow from B to A
the external force on the constituent charges of the (c) charge oscillates between A and B
molecule is balanced by the restoring force. (d) no charge will flow.
III. The non–polar molecule develops an induced dipole 21. Which of the following figure shows the correct
moment. equipotential surfaces of a system of two positive charges?
(a) I and II (b) II and III
(c) I and III (d) I, II and III (a) + + (b) + +
16. A, B and C are three points in a uniform electric field. The
electric potential is

A
B ®
E + +
(c) + + (d)
C
(a) maximum at B
(b) maximum at C 22. Assertion : Electric field is discontinuous across the surface
(c) same at all the three points A, B and C of a spherical charged shell.
Reason : Electric potential is continuous across the surface
(d) maximum at A
of a spherical charged shell.
17. The figure shows the electric dipole placed along x-axis. As
(a) Assertion is correct, reason is correct; reason is a
we move from point A to point B potential changes from
correct explanation for assertion.
B +q –q A
x (b) Assertion is correct, reason is correct; reason is not a
(a) positive to negative (b) negative to positive correct explanation for assertion
(c) positive to zero (d) does not change (c) Assertion is correct, reason is incorrect
(d) Assertion is incorrect, reason is correct.
18. Charges are placed on the vertices of a square as shown.
ur 23. Two equally charged spheres of radii a and b are connected
Let E be the electric field and V the potential at the centre. together. What will be the ratio of electric field intensity on
If the charges on A and B are interchanged with those on D their surfaces?
and C respectively, then a a2 b b2
q q (a) (b) (c ) (d)
A B b b2 a a2
24. If the electrostatic potential were given by
f = f0 (x2 + y2 + z2), where f0 is constant, then the charge
density giving rise to the above potential would be.
6f
D C (a) 0 (b) – 6 f0 e 0 (c) – 2 f0 e0 (d) – 0
-q -q e0
ur
(a) E changes, V remains unchanged 25. An electric charge 10–3 m C is placed at the origin (0, 0) of
ur
(b) E remains unchanged, V changes X – Y co-ordinate system. Two points A and B are situated at
ur
(c) both E and V change
ur ∋ (
2, 2 and (2, 0) respectively. The potential difference
(d) E and V remain unchanged between the points A and B will be [in volt]
(a) 4.5 (b) 9 (c) zero (d) 2
EBD_7418
236 PHYSICS

26. Equipotential surfaces are shown in figure. Then the me w 2 R 2 me w R 3 eme w R 2


electric field strength will be (a) zero (b) (c) (d)
2e 3e 2
Y 33. A plastic disc is charged on one side with a uniform surface
10V 20V 30V charge density s and then three quadrant of the disk are
removed. The remaining quadrant is shown in figure, with
O (c m )

30º X
10 20 30 (cm) V = 0 at infinity, the potential due to the remaining quadrant
at point P is P
r
(a) 100 Vm–1 along X-axis
(b) 100 Vm–1 along Y-axis
R
(c) 200 Vm–1 at an angle 120° with X-axis
(d) 50 Vm–1 at an angle 120° with X-axis
27. Four points a, b, c and d are set at equal distance from the s s
(a) [( r 2 + R )1/ 2 - r ] (b) [R - r ]
centre of a dipole as shown in figure. The electrostatic 2 Î0 2 Î0
potential Va, Vb, Vc, and Vd would satisfy the following s
relation: a (c) [(r 2 + R 2 )1/ 2 - r ] (d) None of these
8 Î0
(a) Va > Vb > Vc > Vd +q 34. Two concentric spheres of radii R and r have similar charges
(b) Va > Vb = Vd > Vc d b with equal surface charge densities (s). What is the electric
(c) Va > Vc = Vb = Vd –q potential at their common centre ?
(d) Vb = Vd > Va > Vc s
c (a) s / e 0 (b) (R - r)
28. The potential at a point x (measured in mm) due to some e0
s
charges situated on the x-axis is given by V(x) = 20/(x2 – 4) (c) e (R + r) (d) None of these
volt. The electric field E at x = 4 m m is given by 0
35. From a point charge, there is a fixed point A. At A, there is
(a) (10/9) volt/mm and in the +ve x direction
an electric field of 500 V/m and potential difference of 3000
(b) (5/3) volt/mm and in the –ve x direction
(c) (5/3) volt/mm and in the +ve x direction V. Distance between point charge and A will be
(d) (10/9) volt/mm and in the –ve x direction (a) 6 m (b) 12 m (c) 16 m (d) 24 m
36. Two metal pieces having a potential difference of 800 V are
dv
29. The expression E = - implies, that electric field is in that 0.02 m apart horizontally. A particle of mass 1.96 × 10–15 kg is
dr suspended in equilibrium between the plates. If e is the
direction in which elementary charge, then charge on the particle is
(a) increase in potential is steepest. (a) 8 (b) 6 (c) 0.1 (d) 3
(b) decrease in potential is steepest. 37. A point charge of magnitude + 1 mC is fixed at (0, 0, 0). An
(c) change in potential is minimum. isolated uncharged spherical conductor, is fixed with its
(d) None of these center at (4, 0, 0). The potential and the induced electric
30. Four point charges –Q, –q, 2q and 2Q are placed, one at field at the centre of the sphere is :
each corner of the square. The relation between Q and q for (a) 1.8 × 105 V and – 5.625 × 106 V/m
which the potential at the centre of the square is zero is (b) 0 V and 0 V/m
1 1 (c) 2.25 × 105 V and – 5.625 × 106 V/m
(a) Q = – q (b) Q = – (c) Q = q (d) Q = (d) 2.25 × 105 V and 0 V/m
q q 38. The 1000 small droplets of water each of radius r and charge
31. A conducting sphere of radius R is given a charge Q. The Q, make a big drop of spherical shape. The potential of big
electric potential and the electric field at the centre of the drop is how many times the potential of one small droplet ?
sphere respectively are: (a) 1 (b) 10 (c) 100 (d) 1000
Q 39. The electric potential at a point (x, y) in the x – y plane is
(a) 0 and
4p e 0 R 2 given by V = – kxy. The field intensity at a distance r from
Q the origin varies as
(b) and 0
4p e 0 R 1 1
(a) r 2 (b) r (c) (d)
Q Q r r2
(c) and -
4p e 0 R 4p e 0 R 2 40. Four charges q1= 2 × 10 C, q2 = -2 ´ 10 C, q3 = -3 ´ 10-8 C,
–8 8

(d) Both are 0 and q4 = 6 ´ 10-8 C are placed at four corners of a square of
side 2 m. What is the potential at the centre of the square?
32. A conducting disc of radius R rotating about its axis with an (a) 270 V (b) 300 V (c) Zero (d) 100 V
angular velocity w. Then the potential difference between 41. In a hollow spherical shell, potential (V) changes with respect
the centre of the disc and its edge is (no magnetic field is to distance (s) from centre as
present)
ELECTROSTATIC POTENTIAL AND CAPACITANCE 237

(a) (A) ® (2); (B) ® (4); (C) ® (3); (D) ® (1)


(a) (b) (b) (A) ® (2); (B) ® (3); (C) ® (4); (D) ® (1)
(c) (A) ® (2); (B) ® (3); (C) ® (1); (D) ® (4)
V V
(d) (A) ® (1); (B) ® (3); (C) ® (4); (D) ® (2)
47. If a unit positive charge is taken from one point to another
S S over an equipotential surface, then
(a) work is done on the charge
(c) (d) (b) work is done by the charge
(c) work done is constant
V V (d) no work is done
48. Assertion : For a charged particle moving from point P to
S point Q, the net work done by an electrostatic field on the
S
particle is independent of the path connecting point P to
point Q.
Topic 2: Electric Potential Energy and Work done Reason : The net work done by a conservative force on an
in Carrying a Charge object moving along a closed loop is zero.
42. The potential energy of a system of two charges is negative (a) Assertion is correct, reason is correct; reason is a correct
when explanation for assertion.
(a) both the charges are positive (b) Assertion is correct, reason is correct; reason is not a
(b) both the charges are negative correct explanation for assertion
(c) one charge is positive and other is negative (c) Assertion is correct, reason is incorrect
(d) both the charges are separated by infinite distance (d) Assertion is incorrect, reason is correct.
A
ur 49. In the electric field of a point charge
43. An electric dipole of moment p is placed normal to the lines q, a certain charge is carried from
ur
of force of electric intensity E , then the work done in point A to B, C, D and E. Then the
deflecting it through an angle of 180° is work done is
+q
(a) pE (b) + 2pE (c) – 2pE (d) zero (a) least along the path AB
B E
44. Which of the following about potential difference between (b) least along the path AD C D
any two points is true? (c) zero along all the paths AB, AC, AD and AE
I. It depends only on the initial and final position. (d) least along AE
+q
II. It is the work done per unit positive charge in moving 50. Figure shows a system of three
from one point to other. positive charges placed at the
III. It is more for a positive charge of two units as compared vertices of an equilateral triangle.
to a positive charge of one unit. To decrease the potential energy
(a) I only (b) II only of the system,
(c) I and II (d) I, II and III ur (a) a positive charge should be +q +q
ur of moment P is placed in a uniform
45. An electric dipole placed at centroid.
electric field E . Then which of the following is/are (b) a negative charge should be placed at centroid.
correct? (c) distance between the charges should be decreased.
ur ur
I. The torque on the dipole is p ´ E . p
(d) it should be rotated by an angle of radian.
ur ur 2
II. The potential energy of the system is p . E . 51. Assertion: The electric potential at any point on the
III. The resultant force on the dipole is zero. equatorial plane of a dipole is zero.
(a) I, II and II (b) I and III Reason: The work done in bringing a unit positive charge
46. Match the entries of Column I and Column II from infinity to a point in equatorial plane is equal for the two
Column I Column II charges of the dipole.
(A) Inside a conductor (1) Potential energy = 0 (a) Assertion is correct, reason is correct; reason is a correct
placed in an external explanation for assertion.
electric field. (b) Assertion is correct, reason is correct; reason is not a
(B) At the centre of a dipole (2) Electric field = 0 correct explanation for assertion
(C) Dipole in stable (3) Electric potential = 0 (c) Assertion is correct, reason is incorrect
equilibrium (d) Assertion is incorrect, reason is correct.
(D) Electric dipole (4) Torque = 0 52. Two equipotential surfaces S1 and S2 are around a charge q.
perpendicular to A test charge is moved from S1 to S2 along the paths APB
uniform electric field. and AEC as shown in figure. The work done is
EBD_7418
238 PHYSICS

A
(a) more in case of APB 62. The energy stored in a condenser of capacity C which
P
has been raised to a potential V is given by
(b) more in case of AEC B
E 1 1
(c) same in both the cases q (a) u = CV (b) u = CV2
2 2
S C 1
(d) cannot say
S (c) u = CV (d) u =
2VC
53. Two conducting spheres of radii R1 and R2 having 63. Capacitiors are used in electrical circuits where applicances
charges Q1 and Q2 respectively are connected to each need more
other. There is (a) voltage (b) current
(a) no change in the energy of the system (c) resistance (d) power
(b) an increase in the energy of the system 64. The work done in placing a charge of 8 × 10–18 coulomb on
(c) always a decrease in the energy of the system a condenser of capacity 100 micro-farad is
(d) a decrease in the energy of the system unless (a) 3.1 × 10–26 joule (b) 4 × 10–10 joule
Q1R2 = Q2R1 –32
(c) 32 × 10 joule (d) 16 × 10–232 joule
54. A and B are two points in an electric field. If the work done 65. A sheet of aluminium foil of negligible thickness is
in carrying 4.0C of electric charge from A to B is 16.0 J, the introduced between the plates of a capacitor. The
potential difference between A and B is
capacitance of the capacitor
(a) zero (b) 2.0 V (c) 4.0 V (d) 16.0 V
(a) decreases (b) remains unchanged
55. Two points P and Q are maintained at the potentials of 10 V
(c) becomes infinite (d) increases
and – 4 V, respectively. The work done in moving 100
66. The potential gradient at which the dielectric of a
electrons from P to Q is: condenser just gets punctured is called
(a) 9.60 × 10–17J (b) –2.24 × 10–16 J (a) dielectric constant (b) dielectric strength
(c) 2.24 × 10 J–16 (d) –9.60× 10–17 J (c) dielectric resistance (d) dielectric number
56. A ball of mass 1 g carrying a charge 10–8 C moves from a 67. In a charged capacitor, the energy resides
point A at potential 600 V to a point B at zero potential. The (a) in the positive charges.
change in its K.E. is (b) in both the positive and negative charges.
(a) – 6 × 10–6 erg (b) – 6 × 10–6 J (c) in the field between the plates.
(c) 6 × 10 J–6 (d) 6 × 10–6 erg (d) around the edges of the capacitor plates.
68. Consider the situation shown in the figure. The capacitor A
Topic 3: Capacitors, Grouping of Capacitors,
has a charge q on it whereas B is uncharged. The charge
Energy Stored in a Capacitor and Van-de-Graff generator appearing on the capacitor B a long time after the switch is
closed is q
57. On decreasing the distance between the plates of a parallel +
plate capacitor, its capacitance +
+
(a) remains unaffected + S
(b) decreases +
(c) first increases then decreases. +
A B
(d) increases (a) zero (b) q/2 (c) q (d) 2q
58. Energy is stored in a capacitor in the form of 69. In the given circuit diagram, both capacitors are initially
(a) electrostatic energy (b) magnetic energy
uncharged. The capacitance C1 = 2F and C2 = 4F emf of
(c) light energy (d) heat energy
battery A and B are 2V and 4V respectively.
59. If in a parallel plate capacitor, which is connected to a battery, C2 C1
we fill dielectrics in whole space of its plates, then which of
the following increases?
S1
(a) Q and V (b) V and E (c) E and C (d) Q and C
60. When air in a capacitor is replaced by a medium of A
dielectric constant K, the capacity S2
(a) decreases K times (b) increases K times
B
(c) increases K2 times (d) remains constant Column I Column II
61. A conductor carries a certain charge. When it is connected
64
to another uncharged conductor of finite capacity, then the (A) On closing switch S1 (1)
3
energy of the combined system is
with S2 open work done
(a) more than that of the first conductor
by battery A is
(b) less than that of the first conductor
(B) Switch S1 is open and (2) 4
(c) equal to that of the first conductor
S2 is closed, work done
(d) uncertain
by battery B is
ELECTROSTATIC POTENTIAL AND CAPACITANCE 239

(C) Charge on capacitor (3) 8 73. A parallel plate capacitor is located horizontally such that
C2 is (after S1 open one of the plates is submerged in
and S2 closed) a liquid while the other is above
16 the liquid surface. When plates
(D) Charge on C1 when (4) are charged the level of liquid
3
both are closed (a) rises
(5) zero (b) falls Charged Liquid
(a) (A) ® (1); (B) ® (2); (C) ® (2); (D) ® (4) (c) remains unchanged
(b) (A) ® (4); (B) ® (3); (C) ® (3); (D) ® (1) (d) may rise or fall depending on the of charge amount
(c) (A) ® (2); (B) ® (3); (C) ® (2); (D) ® (1) 74. What is the effective capacitance between points X and Y?
(d) (A) ® (3); (B) ® (1); (C) ® (4); (D) ® (2) C1 = 8mF
(a) 8 mF
70. The resultant capacitance of n condenser of capacitances
(b) 9 mF C3 = 8m F C5 = 10m F C2 = 8m F
C1, C2 .... Cn connected in series is given by Y
X
1 1 1 (c) 10 mF A C B D
(a) Cs = + + ........ + C4 = 8m F
C1 C 2 Cn (d) 12 mF
1 1 1 75. Assertion : When a dielectric slab is gradually inserted
(b) = + ........ +
Cs C1 Cn between the plates of an isolated parallel-plate capacitor,
(c) Cs = C1 + C2 + ..... + Cn the energy of the system decreases.
(d) Cs = C1 – C2 + ..... – Cn Reason : The force between the plates decreases.
71. The energy stored in a parallel plate capacitor is given by (a) Assertion is correct, reason is correct; reason is a
Q2 correct explanation for assertion.
VE = . Now which of the following statements is not (b) Assertion is correct, reason is correct; reason is not a
2C
true? correct explanation for assertion
I. The work done in charging a capacitor is stored in the (c) Assertion is correct, reason is incorrect
form of electrostatic potential energy given by (d) Assertion is incorrect, reason is correct.
Q2 76. A parallel plate capacitor is charged by connecting it to a
expression VE = . battery. Now the distance between the plates of the capacitor
2C
II. The net charge on the capacitor is Q. is increased. Which of the following remains constant ?
III. The magnitude of the net charge on one plate of a (a) Capacitance
capacitor is Q. (b) Charge on each plate of the capacitor
(a) I only (b) II only (c) Potential difference between the plates of capacitor
(c) I and II (d) I, II and III (d) Energy stored in the capacitor
72. When a dielectric slab is inserted between the plates of one 77. Two vertical metallic plates carrying equal and opposite
of the two identical capacitors shown in the figure then charges are kept parallel to each other like a parallel plate
match the following: capacitor. A small spherical metallic ball is suspended by a
long insulated thread such that it hangs freely in the centre
of the two metallic plates. The ball, which is uncharged, is
A taken slowly towards the positively charged plate and is
V made to touch that plate. Then the ball will
B (a) stick to the positively charged plate
(b) come back to its original position and will remain
there
Column I Column II (c) oscillate between the two plates touching each plate
(A) Charge on A (1) Increases in turn
(B) Potential difference (2) Decreases (d) oscillate between the two plates without touch them
across A 78. Eight drops of mercury of equal radii possessing equal
(C) Potential difference (3) Remains constant charges combine to form a big drop. Then the capacitance
across B of bigger drop compared to each individual small drop is
(D) Charge on B (4) Cannot say (a) 8 times (b) 4 times (c) 2 times (d) 32 times
(a) (A) ® (1); (B) ® (2); (C) ® (2); (D) ® (1) 79. A parallel plate condenser is filled with two
dielectrics as shown. Area of each plate is A m2
(b) (A) ® (1); (B) ® (1); (C) ® (2); (D) ® (2)
and the separation is t m. The dielectric k
(c) (A) ® (2); (B) ® (2); (C) ® (2); (D) ® (4) 1 k2
constants are k1 and k2 respectively. Its
(d) (A) ® (1); (B) ® (2); (C) ® (2); (D) ® (3) capacitance in farad will be
EBD_7418
240 PHYSICS

eo A eo A k1 + k 2 87. A parallel plate capacitor with air between the plates has a
(a) (k1 + k2) (b) . capacitance of 8 pF. Calculate the capacitance if the distance
t t 2
between the plates is reduced by half and the space between
2e o A eo A k1 - k2 them is filled with a substance of dielectric constant. (er = 6)
(c) (k1 + k2) (d) .
t t 2 (a) 72 pF (b) 81 pF (c) 84 pF (d) 96 pF
80. The resultant capacity of n condensers of capacitances C1, 88. A parallel plate air capacitor has a capacitance of 100 mF.
The plates are at a distance d apart. If a slab of thickness t
C2 .... Cn connected in parallel is (t < d) and dielectric constant 5 is introduced between the
(a) Cp = Cl + C2 + ....... + Cn parallel plates, then the capacitance will be
(b) Cp = Cl – C2 – C3 ....... – Cn (a) 50 mF (b) 100 mF (c) 200 mF (d) 500 mF
89. A parallel plate capacitor having a separation between the
1 1 1 1 plates d, plate area A and material with dielectric constant K
(c) C = C + C + ...... + C has capacitance C0. Now one-third of the material is replaced
p 1 2 n
by another material with dielectric constant 2K, so that
1 1 1
(d) C p = + + ...... +
effectively there are two capacitors one with area A,
1
C1 C 2 Cn
3
81. Two spherical conductors A1 and 2
A dielectric constant 2K and another with area A and
A2 of radii r 1 and r2 (r2 > r1) are – – 2 3
placed concentrically in air. A1 is +
dielectric constant K. If the capacitance of this new capacitor
+
given a charge +Q while A2 is A1 r1 C
is C then is
earthed. Then the equivalent + +
C0
capacitance of the system is r2
4 2 1
– – (a) 1 (b) (c) (d)
3 3 3
4p Î0 r1r2 90. A parallel plate capacitor with air between the plates is
(a) (b) 4p Î0(r1 + r2)
r2 - r1 charged to a potential difference of 500V and then insulated.
(c) 4p Î0 r2 (d) 4p Î0 r1 A plastic plate is inserted between the plates filling the whole
82. The capacitor, whose capacitance is 6mF 6mF 3mF gap. The potential difference between the plates now
6, 6 an d 3mF respectively are becomes 75V. The dielectric constant of plastic is
connected in series with 20 volt line. (a) 10/3 (b) 5 (c) 20/3 (d) 10
Find the charge on 3mF. 91. The work done in placing a charge of 8 × 10–18 coulomb on
a condenser of capacity 100 micro-farad is
(a) 30 mc (b) 60 mF 20mF (a) 16 × 10–32 joule (b) 3.1 × 10–26 joule
(c) 15 mF (d) 90 mF –10
(c) 4 × 10 joule (d) 32 × 10–32 joule
C C
83. The effective capacitance of 92. A capacitor has two circular plates whose radius are 8cm
combination of equal capacitors C C C and distance between them is 1mm. When mica (dielectric
between points A and B shown in C
constant = 6) is placed between the plates, the capacitance
C
A B of this capacitor and the energy stored when it is given
figure is
C C C potential of 150 volt respectively are
(a) 1.06 ´10-5 F, 1.2 ´10-9 J
C (b) 1.068 ´10-9 F, 1.2 ´ 10-5 J
(a) C (b) 2 C (c) 3 C (d)
2 (c) 1.2 ´ 10-9 F, 1.068 ´ 10-5 J
84. In the circuit given below, the charge in mC, on the capacitor
(d) 1.6 ´ 10-9 F, 1.208 ´ 10-5 J
having capacitance 5 mF is 93. Three condensor each of capacitance 2F are put in series.
3mF 2mF The resultant capacitance is
e d (a) 6 F (b) 3/2 F (c) 2/3 F (d) 5 F
94. Three capacitors are connected in the arms
5mF C
f c of a triangle ABC as shown in figure 5 V is
4mF applied between A and B. The voltage
a + b 2µF 3µF
6V between B and C is
(a) 4.5 (b) 9 (c) 7 (d) 15 (a) 2 V (b) 1 V A B
(c) 3 V (d) 1.5 V 2µF
85. Capacitance (in F) of a spherical conductor with radius 1 m
is 95. To obtain 3 mF capacity from three capacitors of 2 mF each,
(a) 1.1 × 10–10 (b) 106 they will be arranged.
(c) 9 × 10 –9 (d) 10–3 (a) all the three in series
(b) all the three in parallel
86. The capacitance of a metallic sphere is 1mF , then it’s radius (c) two capacitors in series and the third in parallel with
is nearly the combinatioin of first two
(a) 1.11 m (b) 10 m (c) 9 km (d) 1.11 cm (d) two capacitors in parallel and the third in series with
the combinatioin of first two
ELECTROSTATIC POTENTIAL AND CAPACITANCE 241

96. The energy required to charge a parallel plate condenser 102. Seven capacitors each of capacitance 2mF are to be
of plate separation d and plate area of cross-section A connected in a configuration to obtain an effective
such that the uniform electric field between the plates is
æ 10 ö
E, is capacitance of ç ÷ mF . Which of the combination (s) shown
è 11ø
1
(a) Î0 E2Ad (b) Î0 E2Ad in figure will achieve the desired result ?
2
1
(c) Î0 E2/Ad (d) Î0 E2/Ad
2 (a) (b)
97. A network of six identical capacitors, each of value C is
made as shown in the figure. Equivalent capacitance between
points A and B is A
(a) C/4 (c) (d)
(b) 3C/4
103. In figure, there is a four way key at the middle. If key is
(c) 4C/3
B shown from situation BD to AD, then how much charge will
(d) 3C
flow through point O ? 3V
98. A series combination of n1 capacitors, each of capacity C1
is charged by source of potential difference 4 V. When (a) 24 mC
another parallel combination of n2 capacitors each of A
capacity C2 is charged by a source of potential difference (b) 36 mC 4 mF K B 4 mF
V, it has the same total energy stored in it as the first O
D C
combination has. The value of C2 in terms of C1 is then (c) 72 mC
n2 2 C1 n2 16 C1
(a) 16 n C1 (b) n n (c) 2 n C1 (d) n n (d) 12 mC 6V
1 1 2 1 1 2
99. Two capacitors of capacitances C1 and C2 are connected in 104. A number of capacitors each of equal capacitance C, are
series, assume that C1 < C2. The equivalent capacitance of arranged as shown in Fig. The equivalent capacitance
between A and B is
this arrangement is C, where
(a) C < C1/2 (b) C1/2 < C < C2/2 (a) n2 C
(c) C1 < C < C2 (d) C2 < C < 2C2 (b) (2n + 1) C
100. Four identical square plates of side a are arranged as shown. ( n - 1) n
(c) C
The equivalent capacity between A and C 2 n groups
B D ( n + 1) n AB
3e 0 a 2 3e 0 a 2 (d) C
(a) (b) A C 2
2d 5d 105. Two parallel plate capacitors of capacitance C and 2C are
connected in parallel and charged to a potential difference
3e 0 a 2 5e 0 a 2 d d 2d V. The battery is then disconnected, and the region between
(c) (d) .
3d 3d the plates of C is filled completely with a material of dielectric
constant K. The common potential difference across the
combination becomes
101. Find equivalent capacitance between A and B. [Assume
each conducting plate is having same dimensions and 2V V 3V 3V
(a) (b) (c) (d)
e A K+2 K+2 K+3 K+2
neglect the thickness of the plate, 0 = 7 µF where A is 106. An uncharged paralle plate capacitor having a dielectric of
d dielectric constant K is connected to a similar air cored
area of plates, A > > d] parallel plate capacitor charged to a potential V0. The two
d A share the charge, and the common potential becomes V.
d The dielectric constant K is
V0 V0 V V
d (a) - 1 (b) + 1 (c) - 1 (d) +1
V V V 0 V 0
2d 107. A capacitor of capacity C1 is charged upto V volt and
then connected to an uncharged capacitor of capacity C2.
d
Then final potential difference across each will be
B
C 2V æ C2 ö
(a) 7 µF (b) 11µF (a) (b) ç1 + ÷V
C1 + C 2 è C1 ø
(c) 12 µF (d) 14 µF
C1V æ C2 ö
(c) (d) ç1 - ÷V
C1 + C 2 è C1 ø
EBD_7418
242 PHYSICS

108. A parallel plate capacitor with air between the plates is 114. From a supply of identical capacitors rated 8 mF, 250V, the
charged to a potential difference of 500V and then insulated. minimum number of capacitors required to form a composite
A plastic plate is inserted between the plates filling the whole 16 mF, 1000V is
gap. The potential difference between the plates now (a) 2 (b) 4 (c) 16 (d) 32
becomes 75V. The dielectric constant of plastic is 115. A parallel plate air capacitor of capacitance C is connected
(a) 10/3 (b) 5 (c) 20/3 (d) 10 to a cell of emf V and then disconnected from it. A dielectric
109. Three capacitors each of 4 mF are to be connected in such a slab of dielectric constant K, which can just fill the air gap of
way that the effective capacitance is 6mF. This can be done the capacitor, is now inserted in it. Which of the following is
by connecting them : incorrect?
(a) all in series (a) The energy stored in the capacitor decreases K times.
(b) all in parallel
1 æ1 ö
(c) two in parallel and one in series (b) The chance in energy stored is CV 2 ç – 1÷ .
(d) two in series and one in parallel 2 èK ø
110. A combination of parallel plate capacitors is maintained at a (c) The charge on the capacitor is not conserved.
certain potential difference. (d) The potential difference between the plates decreases
K times.
C1 C2 C3 116. In case of a Van de Graaff generator, the breakdown field
C
of air is
A D E B
(a) 2 × 108 V m–1 (b) 3 × 106 V m–1
8
(c) 2 × 10 V m –1 (d) 3 × 104 V m–1
When a 3 mm thick slab is introduced between all the plates, 117. Van de Graaff generator is used to
in order to maintain the same potential difference, the dis- (a) store electrical energy
tance between the plates is increased by 2.4 mm. (b) build up high voltage of few million volts
Find the dielectric constant of the slab. (c) decelerate charged particle like electrons
(a) 3 (b) 4 (c) 5 (d) 6 (d) both (a) and (b)
111. A capacitor of capacitance C0 is charged to a potential V0
and then isolated. A small capacitor C is then charged from 118. Which of the following is / are true about the principle of
C0 , discharged and chaged again; the process being Van de Graaff generator?
repeated n times. Due to this, the potential of the larger (a) The action of sharp points.
capacitor is decreased to V. The value of C is (b) The charge given to a hollow conductor is tranferred
1/ n éæ V ö 1/ n ù to outer surface and is distributed uniformly over it.
æV ö
(a) C0 ç 0 ÷ (b) C0 êçè 0 ÷ø - 1ú (c) It is used for accelerating uncharged particle.
è Vø ë V û (d) Both (a) and (b)
n é n ù
éæ V ö ù æ Vö 119. In a Van de Graaff generator, a spherical metal shell is to
(c) C0 êç ÷ - 1ú (d) C0 êç ÷ + 1ú be 15 × 106 V electrode. The dielectric strength of the gas
ëè V0 ø û êëè V0 ø úû
112. For the configuration of media of permittivities e0, e and e0 surrounding the electrode is 5 × 107V m–1. The minimum
between parallel plates each of area A, as shown in Fig. the radius of the spherical shell required is
equivalent capacitance is (a) 1 m (b) 2 m (c) 1.5 m (d) 3 m
(a) e0 A/d
(b) ee0 A/d 120. A parallel-plate capacitor is filled by a dielectric whose
ee 0 A e0 e e0 permittivity varies with the applied voltage according to the
(c) law e = aV, where a = 1 V–1. The same (but containing no
d ( e + e0 )
ee0 A dielectric) capacitor charged to a voltage V0 = 156V is
d d d connected in parallel to the first “nonlinear” uncharged
(d) 2e + e d
( 0) capacitor. Determine the final voltage V across the capacitors.
113. Find the capacitance between P and Q (Fig). Each Capacitor (a) 6 V (b) 12 V (c) 8 V (d) 4 V
has cpacitance C. P
(a) 2 C
(b) 3 C
B
(c) 8 C
A O C
(d) 6 C D
ELECTROSTATIC POTENTIAL AND CAPACITANCE 243

Exercise 2 : Exemplar & Past Year MCQs

NCERT Exemplar MCQs 5. Equipotentials at a great distance from a collection of charges


whose total sum is not zero are approximately
1. A capacitor of 4 µF is connected as shown in the circuit. (a) spheres (b) planes
The internal resistance of the battery is 0.5W. The amount (c) paraboloids (d) ellipsoids
of charge on the capacitor plates will be 6. A parallel plate capacitor is made of two dielectric blocks in
4 mF 10 W
series. One of the blocks has thickness d1 and dielectric
constant K1 and the other has thickness d2 and dielectric
constant K2 as shown in figure. This arrangement can be
thought as a dielectric slab of thickness d (= d1 + d2) and
effective dielectric constant K. The K is
2.5 V
d1 K1

2W d2 K2
(a) 0 µC (b) 4 µC (c) 16 µC (d) 8 µC
2. A positively charged particle is released from rest in an uniform K1 d1 + K 2 d 2 K1d1 + K 2 d 2
(a) (b)
electric field. The electric potential energy of the charge d1 + d 2 K1 + K 2
(a) remains a constant because the electric field is uniform K1K 2 ( d1 + d 2 ) 2K1K 2
(b) increases because the charge moves along the electric field (c) (d)
( K 1d 2 + K 2 d 1 ) K1 + K 2
(c) decreases because the charge moves along the electric
field Past Year MCQs
(d) decreases because the charge moves opposite to the 7. In a region, the potential is represented by V(x, y, z)
electric field = 6x – 8xy – 8y + 6yz, where V is in volts and x, y, z are in
3. Figure shows some equipotential lines distributed in space. metres. The electric force experienced by a charge of 2 coulomb
A charged object is moved from point A to point B. situated at point (1, 1, 1) is : [AIPMT 2014, C]
(a) The work done in Fig. (i) is the greatest (a) 6 5 N (b) 30 N (c) 24 N (d) 4 35 N
(b) The work done in Fig. (ii) is least 8. What is the effective capacitance between points X and Y?
(c) The work done is the same in Fig. (i), Fig.(ii) and Fig. (iii) C1 = 6m F [AIIMS 2014, A]
(d) The work done in Fig. (iii) is greater than Fig. (ii) but (a) 24 mF
equal to that in (b) 18 mF C3 = 6m F C5 = 20m F C2 = 6m F
X Y
A C B D
(c) 12 mF C4 = 6m F
20V 40V 30V
(d) 6 mF
r
9. Assume that an electric field E = 30 x 2 iˆ exists in space.
A B A B A B Then the potential difference VA – VO, where VO is the
potential at the origin and VA the potential at x = 2 m is:
[JEE Main 2014, C]
10V 20V 30V 40V 50V 10V 30V 30V 10V 20V 40V 50V (a) 120 J/C (b) –120 J/C (c) –80 J/C (d) 80 J/C
Fig. (i) Fig. (ii) Fig. (iii) 10. If potential (in volts) in a region is expressed as V(x, y, z) = 6
xy – y + 2yz, the electric field (in N/C) at point (1, 1, 0) is :
4. The electrostatic potential on the surface of a charged [AIPMT 2015, C]
conducting sphere is 100V. Two statements are made in this (a) -(6iˆ + 5jˆ + 2k)
ˆ (b) -(2iˆ + 3jˆ + k)
ˆ
regard S1 at any point inside the sphere, electric intensity is
(c) -(6iˆ + 9jˆ + k)
ˆ (d) -(3iˆ + 5jˆ + 3k)
ˆ
zero. S2 at any point inside the sphere, the electrostatic
11. The four capacitors, each of 25 m F are connected as
potential is 100V. Which of the following is a correct shown in fig. The dc voltmeter reads 200 V. The charge
statement? on each plate of capacitor is [AIIMS 2015, A]
(a) S1 is true but S2 is false V
(b) Both S1 and S2 are false
– + – +
(c) S1 is true, S2 is also true and S1 is the cause of S2
(d) S1 is true, S2 is also true but the statements are
– + – +
independant
EBD_7418
244 PHYSICS

potential at the origin due to above system of charges will be


(a) ± 2 ´ 10 -3 C (b) ± 5 ´ 10 -3 C
[AIIMS 2016, C]
(c) ± 2 ´ 10 -2 C (d) ± 5 ´ 10 -2 C
q
12. A uniformly charged solid sphere of radius R has potential (a) zero (b)
8pe 0 x 0 log e 2
V0 (measured with respect to ¥) on its surface. For this sphere
3V 5V 3V q log e 2
the equipotential surfaces with potentials 0 , 0 , 0 (c) infinity (d)
2 4 4 4pe 0 x 0
V0
and have radius R1, R2, R3 and R4 respectively. Then 17. A parallel plate capacitor of area ‘A’ plate separation ‘d’ is
4 filled with two dielectrics as shown. What is the capacitance
(a) R1 = 0 and R2 < (R4 – R3) [JEE Main 2015, S]
of the arrangement ? [BITSAT 2016, C]
(b) 2R > R4
A/2 A/2
(c) R1 = 0 and R3 > (R4 – R2)
(d) R1 ¹ 0 and (R2 – R1) > (R4 – R3)
13. At the corners of an equilateral triangle of side a (1 metre), K d
three point charges are placed (each of 0.1 C). If this system d
K
2
is supplied energy at the rate of 1 kw, then calculate the
time required to move one of the mid-point of the line joining
the other two. [BITSAT 2015, C] 3K e 0 A 4 K e0 A
(a) (b)
A 4d 3d

( K + 1) e 0 A K ( K + 3) e 0 A
(c) (d)
2d 2 ( K + 1) d
a a
18. The diagrams below show regions of equipotentials.
[NEET 2017, C]

D 20V 40V 20V 40V

B a C
(i) A B (ii) A B
(a) 50 h (b) 60 h (c) 48 h (d) 54 h
14. A capacitor of 2mF is charged as shown in the diagram.
When the switch S is turned to position 2, the percentage of 10 V 30V
10 V 30V
its stored energy dissipated is : [NEET 2016, C]
1 2
(a) 0% 10V 30V 40V
20V
(b) 20%
V (iii) A B (iv) A B
(c) 75% 2m F 8m F
10V
(d) 80% 20V 40V 30V

15. A combination of capacitors is set up as shown in the figure.


The magnitude of the electric field, due to a point charge Q A positive charge is moved from A to B in each diagram.
(having a charge equal to the sum of the charges on the 4 (a) In all the four cases the work done is the same
mF and 9 mF capacitors), at a point distance 30 m from it, (b) Minimum work is required to move q in figure (i)
would equal : [NEET 2016, S] (c) Maximum work is required to move q in figure (ii)
3mF
(a) 420 N/C 4mF (d) Maximum work is required to move q in figure (iii)
9mF 19. A capacitor is charged by a battery. The battery is removed
(b) 480 N/C and another identical uncharged capacitor is connected in
2mF parallel. The total electrostatic energy of resulting system
(c) 240 N/C + – (a) decreases by a factor of 2 [NEET 2017, C]
(d) 360 N/C 8V (b) remains the same
(c) increases by a factor of 2
16. A charge +q is fixed at each of the points x = x0, x = 3x0, (d) increases by a factor of 4
x = 5x0, .... upto ¥ on X-axis and charge –q is fixed on each
of the points x = 2x0, x = 4x0, x = 6x0, .... upto ¥ . Here x0 is a 20. Two spherical conductors A and B of radii a and b (b>a) are
positive constant. Take the potential at a point due to a placed concentrically in air. The two are connected by a
copper wire as shown in figure. Then the equivalent
Q
charge Q at a distance r from it to be . Then the capacitance of the system is [AIIMS 2017, C]
4pe 0 r
ELECTROSTATIC POTENTIAL AND CAPACITANCE 245

ab B ab
(a) 4pe 0 b (a) 4 pe0 (b) 4 pe0 (a + b)
b-a A b-a

a (c) zero (d) infinite


(b) 4pe 0 (a + b)
25. The electrostatic force between the metal plates of an
(c) 4pe 0 b isolated parallel plate capacitor C having a charge Q and
area A, is [NEET 2018, C]
(d) 4pe 0a (a) independent of the distance between the plates
21. Assertion : Electric potential and electric potential energy (b) linearly proportional to the distance between the plates
are different quantities. (c) inversely proportional to the distance between the
Reason : For a system of positive test charge and point plates
charge electric potential energy = electric potential. (d) proportional to the square root of the distance between
[AIIMS 2017, C] the plates
(a) Assertion is correct, reason is correct; reason is a 26. Assertion : For a non-uniformly charged thin circular ring
correct explanation for assertion. with net charge is zero, the electric field at any point on axis
(b) Assertion is correct, reason is correct; reason is not a of the ring is zero.
correct explanation for assertion Reason : For a non-uniformly charged thin circular ring with
(c) Assertion is correct, reason is incorrect net charge zero, the electric potential at each point on axis
(d) Assertion is incorrect, reason is correct. of the ring is zero. [AIIMS 2018, C]
22. A capacitance of 2mF is required in an electrical circuit across (a) Assertion is correct, reason is correct; reason is a
a potential difference of 1.0 kV. A large number of 1mF correct explanation for assertion.
capacitors are available which can withstand a potential (b) Assertion is correct, reason is correct; reason is not a
difference of not more than 300 V. The minimum number of correct explanation for assertion
capacitors required to achieve this is [JEE Main 2017, A] (c) Assertion is correct, reason is incorrect
(a) 24 (b) 32 (c) 2 (d) 16 (d) Assertion is incorrect, reason is correct.
23. A long, hollow conducting cylinder is kept coaxially inside 27. Three concentric metal shells A, B and C of respective radii
another long, hollow conducting cylinder of larger radius. a, b and c (a < b < c) have surface charge densities +s, –s
Both the cylinders are initially electrically neutral and +s respectively. The potential of shell B is:
[BITSAT 2017, C] [JEE Main 2018, S]
(a) A potential difference appears between the two
cylinders when a charge density is given to the inner s é a2 - b2 ù s é a 2 - b2 ù
cylinder. (a) ê +c ú (b) ê +cú
Î0 ëê a ûú Î0 ëê b ûú
(b) A potential difference appears between two cylinders
when a charge density is given to the outer cylinder.
s é b2 - c2 ù s é b 2 - c2 ù
(c) No potential difference appears between the two (c) ê +a ú (d) ê +a ú
Î0 ëê b ûú Î0 ëê c ûú
cylinders when a uniform line charge is kept along the
axis of the cylinders. 28. A parallel plate capacitor of capacitance 90 pF is connected
(d) No potential difference appears between the two to a battery of emf 20V. If a dielectric material of dielectric
cylinders when same charge density is given to both 5
constant k = is inserted between the plates, the magnitude
the cylinders. 3
24. Two conducting shells of radius a and b are connected by of the induced charge will be: [JEE Main 2018, A]
conducting wire as shown in figure. The capacity of system (a) 1.2 n C (b) 0.3 n C (c) 2.4 n C (d) 0.9 n C
is : [BITSAT 2017, C] 29. Electric potential at any point is V = –5x + 3y + 15z , then
the magnitude of the electric field is [BITSAT 2018, C]
a b (a) (b) (c) (d) 7
3 2 4 2 5 2
EBD_7418
246 PHYSICS

Exercise 3 : Try If You Can


1. A thin spherical conducting shell of radius R has a charge q. 8. A parallel plate condenser is immersed in an oil of dielectric
Another charge Q is placed at the centre of the shell. The constant 2. The field between the plates is
R (a) increased, proportional to 2
electrostatic potential at a point P , a distance from the
2 1
centre of the shell is (b) decreased, proportional to
2
2Q 2Q 2q
(a) 4pe R (b) 4pe R - 4pe R (c) increased, proportional to – 2
o o o
1
2Q q (q + Q)2 (d) decreased, proportional to -
(c) 4pe R + 4pe R (d) 4pe R 2
o o o 9. An air capacitor of capacity C = 10 mF is connected to a
2. The electric potential V (in Volt) varies with x (in metres)
constant voltage battery of 12 volt. Now the space between
according to the relation V = (5 + 4x2). The force experienced
the plates is filled with a liquid of dielectric constant 5. The
by a negative charge of 2 × 10–6 C located at x = 0.5 m is
(additional) charge that flows now from battery to the
(a) 2 × 10–6 N (b) 4 × 10–6 N
(c) 6 × 10 N–6 (d) 8 × 10–6 N capacitor is
(a) 120 m C (b) 600 m C (c) 480 m C (d) 24 m C
3. The electric potential at point A is 1V and at another point B
10. A capacitor having capacitance
is 5V. A charge 3 µC is released from B. What will be the
1µF with air is filled with two
kinetic energy of the charge as it passes through A ? K1 = 8 K2 = 4
dielectrics as shown. How many
(a) 8 × 10–6 J (b) 12 × 10–6 J times capacitance will increase?
(c) 12 × 10–9 J (d) 4 × 10–6 J (a) 12 (b) 6 (c) 8/3 (d) 3
4. A square of side ‘a’ has charge Q at its centre and charge ‘q’ 11. A parallel plate capacitor is made of two circular plates
at one of the corners. The work required to be done in separated by a distance 5 mm and with a dielectric of dialectric
moving the charge ‘q’ from the corner to the diagonally constant 2.2 between them. When the electric field in the
opposite corner is dielectric is 3 × 104 V/m the charge density of the positive
Qq Qq 2 Qq plate will be close to:
(a) zero (b) (c) (d)
4p Î0 a 4p Î0 a 2p Î0 a (a) 6 ´10-7 C m2 (b) 3 ´10-7 C m2
5. A positive point charge q is carried from a point B to a point 4
(c) 3 ´ 10 C m 2 (d) 6 ´104 C m2
A in the electric field of a point charge + Q at O. If the 12. A capacitor of capacitance C1 = 1mF can withstand maximum
permitivity of free space is e0, the work done in the process voltage V1 = 6kV (kilo-volt) and another capacitor of
is given by capacitance C2 = 3mF can withstand maximum voltage V2 =
qQ æ1 1 ö qQ æ1 1 ö 4kV. When the two capacitors are connected in series, the
(a) ç + ÷ (b) ç - ÷ combined system can withstand a maximum voltage of
4 p eo è a b ø 4 p eo è a b ø (a) 4kV (b) 6kV (c) 8kV (d) 10kV
qQ æ 1 1 ö qQ æ 1 1 ö 13. A, B, C, D, E, F are conducting plates each of area A and any
(c) çç - ÷÷ (d) çç + ÷÷ two consecutive plates separated by a distance d. The net
4 p eo è a 2 2
b ø 4 p eo è a 2
b2 ø
energy stored in the system after the switch S is closed is
6. Two charges q1 and q2 are placed 30 cm apart, as shown in
B C D E F
the figure. A third charge q3 is moved along the arc of a circle 3e0 A 2 5e 0 A 2
A
of radius 40 cm from C to D. The change in the potential (a) V (b) V
2d 12d
q3 C q3
energy of the system is k,
4p Î0 e0 A 2 e0 A 2
where k is 40 cm (c) V (d) V
2d d 180V S
(a) 8q1 (b) 6q1
q1 q2 14. A network of four capacitors of capacity
(c) 8q2 (d) 6q2 30 cm B D equal to C1 = C, C2 = 2C, C3 = 3C and C2
A
7. To establish an instantaneous current of 2 A through a 1 mF C4 = 4C are conducted to a battery as
C3 C1
capacitor ; the potential difference across the capacitor shown in the figure. The ratio of the
charges on C2 and C4 is: C4
plates should be charged at the rate of :
(a) 2 × 104 V/s (b) 4 × 106 V/s 4 3 7 22
(a) (b) (c) (d)
6
(c) 2 × 10 V/s (d) 4 × 104 V/s 7 22 4 3 V
ELECTROSTATIC POTENTIAL AND CAPACITANCE 247

15. There exists a uniform electric field E = 4 × 105 Vm–1 directed V(z) does not depend on x and y. If this potential is generated
along negative x-axis such that electric potential at origin is by a constant charge per unit volume r0 (in units of e0)
zero. A charge of – 200 µC is placed at origin, and a charge of which is spread over a certain region, then choose the correct
+ 200 µC is placed at (3m, 0). The electrostatic potential statement.
energy of the system is (a) r0 = 20 e0 in the entire region
(a) 120 J (b) – 120 J (c) – 240 J (d) zero (b) r0 = 10 e0 for |z| £ 1 m and p0 = 0 elsewhere
16. There is a uniform electrostatic field in a region. The potential (c) r0 = 20 e0 for |z| £ 1 m and p0 = 0 elsewhere
at various points on a small sphere centred at P, in the region, (d) r0 = 40 e0 in the entire region
is found to vary between in the limits 589.0 V to 589.8 V. 19. A parallel plate capacitor C with plates of unit area and
What is the potential at a point on the sphere whose radius separation d is filled with a liquid of dielectric constant
vector makes an angle of 60° with the direction of the field ? K = 2. The level of liquid is d/3 initially. Suppose the liquid
(a) 589.5 V (b) 589.2 V (c) 589.4 V (d) 589.6 V level decreases at a constant speed v, the time constant as a
17. In the circuit shown, the effective capacitance between function of time t is –
points X and Y is 6e 0 R
6 µF 3 µF
a b
(a)
5d + 3vt
8 µF 2 µF 3 µF (15d + 9vt ) e 0 R
(b) C
x 2d 2 - 3dvt - 9v 2t 2 d R
d/3
y 1 µF
6 e0 R
1 µF (c)
5d - 3vt
(15d - 9vt ) e 0 R
8 µF (d)
(a) 3.33 µF (b) 1 µF 2d 2 - 3dvt - 9v 2t 2
(c) 0.44 µF (d) None of these 20. A beam of electron moving with velocity vo ẑ is incident on
a metallic sphere of radius R. Initially the electrons stick to
18. The potential (in volts) of a charge distribution is given by
the sphere and change it. In the steady state, the charge on
V(z) = 30 – 5z2 for |z| £ 1m
the sphere(under the assumption that the charge is
V(z) = 35 – 10 |z| for |z| ³ 1 m. uniformaly distributed on the surface)is
(a) –2p mv02 e R / e (b) – (2 p mv02 e 0 R )
(c) – mv02 e 0 R / e (d) – mv02 e 0 R
EBD_7418
248 PHYSICS

ANSWER KEYS
Exercise 1 : Topic-wise MCQs
1 (c) 13 (a) 25 (c) 37 (c) 49 (c) 61 (b) 73 (a) 85 (a) 97 (c) 109 (d)
2 (a) 14 (b) 26 (c) 38 (c) 50 (c) 62 (b) 74 (a) 86 (c) 98 (d) 110 (c)
3 (a) 15 (d) 27 (b) 39 (b) 51 (d) 63 (b) 75 (c) 87 (d) 99 (b) 111 (b)
4 (b) 16 (a) 28 (a) 40 (a) 52 (c) 64 (c) 76 (c) 88 (c) 100 (c) 112 (d)
5 (d) 17 (b) 29 (b) 41 (b) 53 (d) 65 (b) 77 (c) 89 (b) 101 (b) 113 (a)
6 (b) 18 (a) 30 (a) 42 (c) 54 (c) 66 (b) 78 (c) 90 (c) 102 (a) 114 (d)
7 (d) 19 (c) 31 (b) 43 (d) 55 (c) 67 (c) 79 (b) 91 (d) 103 (c) 115 (c)
8 (a) 20 (b) 32 (b) 44 (c) 56 (c) 68 (a) 80 (a) 92 (b) 104 (d) 116 (b)
9 (b) 21 (c) 33 (c) 45 (b) 57 (d) 69 (d) 81 (a) 93 (c) 105 (d) 117 (b)
10 (d) 22 (b) 34 (c) 46 (b) 58 (a) 70 (b) 82 (a) 94 (a) 106 (a) 118 (d)
11 (b) 23 (c) 35 (a) 47 (d) 59 (d) 71 (b) 83 (b) 95 (c) 107 (c) 119 (d)
12 (b) 24 (b) 36 (d) 48 (a) 60 (b) 72 (b) 84 (b) 96 (a) 108 (c) 120 (b)
Exercise 2 : Exemplar & Past Year MCQs
1 (d) 4 (c) 7 (d) 10 (a) 13 (a) 16 (d) 19 (a) 22 (b) 25 (a) 28 (a)
2 (c) 5 (a) 8 (d) 11 (b) 14 (d) 17 (d) 20 (c) 23 (a) 26 (d) 29 (d)
3 (c) 6 (c) 9 (c) 12 (a) 15 (a) 18 (a) 21 (c) 24 (d) 27 (b)
Exercise 3 : Try If You Can
1 (c) 3 (b) 5 (b) 7 (c) 9 (c) 11 (a) 13 (c) 15 (a) 17 (b) 19 (a)
2 (d) 4 (a) 6 (c) 8 (b) 10 (b) 12 (c) 14 (b) 16 (c) 18 (b) 20 (a)
18Chapter
CURRENT ELECTRICITY

Trend
Analysis of NEET and AIIMS (Year 2010-2018)
5

4
Number of Questions

3
AIPMT/NEET
2 AIIMS

0
2010 2011 2012 2013 2014 2015 2016 2017 2018
Year
Trend
Analysis of JEE Main and BITSAT (Year 2010-2018)
5

4
Number of Question

JEE Main
2
BITSAT

0
2010 2011 2012 2013 2014 2015 2016 2017 2018
Year

Chapter Utility Score (CUS)


Exam Weightage Important Concepts Difficulty Level CUS (Out of 10)
NEET 6 Ohm's law, Combination of
AIIMS 8 Resistors, Kirchhoff's laws 4.5/5 8.5/10
JEE Main 6 & Heating effects of current
BITSAT 7
EBD_7418
250 PHYSICS
CURRENT ELECTRICITY 251
EBD_7418
252 PHYSICS

Exercise 1 : Topic-wise MCQs


10. Consider the following statements and select the correct
Topic 1: Electric Current, Drift of Electrons, Ohm's Law, statement(s).
Resistance and Resistivity I. Current is the time rate of flow of charge through any
cross-section
1. When no current is passed through a conductor, II. For a given conductor current does not change with
(a) the free electrons do not move change in cross-sectional area
(b) the average speed of a free electron over a large period III. The net charge in a current carrying conductor is
of time is not zero infinite
(c) the average velocity of a free electron over a large period
(a) I and II (b) II and III
of time is zero
(d) the average of the velocities of all the free electrons at (c) I and III (d) I, II and III
an instant is non zero 11. The relaxation time in conductors
2. Drift velocity of electrons is due to (a) increases with the increases of temperature
(a) motion of conduction electrons due to random (b) decreases with the increases of temperature
collisions. r (c) it does not depends on temperature
(b) motion of conduction electrons due to electric field E. (d) all of sudden changes at 400 K
(c) repulsion to the conduction electrons due to inner 12. A metal wire is subjected to a constant potential
electrons of ions. difference. When the temperature of the metal wire
(d) collision of conduction electrons with each other. increases, the drift velocity of the electron in it
3. A current passes through a wire of nonuniform cross- (a) increases, thermal velocity of the electron increases
section. Which of the following quantities are independent (b) decreases, thermal velocity of the electron increases
of the cross-section?
(c) increases, thermal velocity of the electron decreases
(a) The charge crossing (b) Drift velocity
(c) Current density (d) Free-electron density (d) decreases, thermal velocity of the electron decreases
4. In the equation AB = C, A is the current density, C is the 13. If N, e, t and m are representing electron density, charge,
electric field, Then B is relaxation time and mass of an electron respectively, then
(a) resistivity (b) conductivity the resistance of wire of length l and cross-sectional area A
(c) potential difference (d) resistance is given by
5. When a potential difference V is applied across a conductor ml 2mtA Ne 2 t A Ne 2 A
at a temperature T, the drift velocity of electrons is (a) (b) (c) (d)
proportional to Ne 2 A 2 t Ne 2 l 2ml 2m tl
(a) V (b) V (c) T (d) T 14. A current passes through a resistor. If K1 and K2 represent
6. For which of the following dependence of drift velocity vd on the average kinetic energy of the conduction electrons and
electric field E, is Ohm’s law obeyed? the metal ions respectively then
(a) K1 < K2
(a) vd µ E2 (b) vd = E1/2
(b) K1 = K2
(c) vd = constant (d) vd = E
7. Constantan wire is used for making standard resistance, (c) K1 > K2
because it has (d) any of these three may occur
(a) high melting point 15. The electric resistance of a certain wire of iron is R. If its
(b) low specific resistance length and radius are both doubled, then
(c) high specific resistance (a) the resistance and the specific resistance, will both
(d) negligible temperature coefficient of resistance remain unchanged
8. A strip of copper and another of germanium are cooled from (b) the resistance will be doubled and the specific
room temperature to 80 K. The resistance of resistance will be halved
(a) each of these increases (c) the resistance will be halved and the specific resistance
(b) each of these decreases will remain unchanged
(c) copper strip increases and that of germanium decreases (d) the resistance will be halved and the specific resistance
(d) copper strip decreases and that of germanium increases will be doubled
9. The resistance of a metal increases with increasing 16. We are able to obtain fairly large currents in a conductor
temperature because because
(a) the collisions of the conducting electrons with the (a) the electron drift speed is usually very large
electrons increase (b) the number density of free electrons is very high and
(b) the collisions of the conducting electrons with the this can compensate for the low values of the
lattice consisting of the ions of the metal increase electron drift speed and the very small magnitude of
(c) the number of conduction electrons decreases the electron charge
(d) the number of conduction electrons increases
CURRENT ELECTRICITY 253

(c) the number density of free electrons as well as the 26. Assertion: For a conductor resistivity increases with
electron drift speeds are very large and these increase in temperature.
compensate for the very small magnitude of the m
electron charge Reason: Since r = 2 , when temperature increases the
ne t
(d) the very small magnitude of the electron charge has random motion of free electrons increases and vibration of
to be divided by the still smaller product of the number ions increases which decreases t.
density and drift speed to get the electric current (a) Assertion is correct, reason is correct; reason is a
17. The I-V characteristics shown in figure represents correct explanation for assertion.
(a) ohmic conductors I (b) Assertion is correct, reason is correct; reason is not a
correct explanation for assertion
(b) non-ohmic conductors (c) Assertion is correct, reason is incorrect
V
(c) insulators (d) Assertion is incorrect, reason is correct.
(d) superconductors 27. The voltage V and current I graphs for a conductor at two
different temperatures T1 and T2 are shown in the figure.
18. Assertion : Drift speed vd is the average speed between
two successive collisions. The relation between T1 and T2 is
(a) T1 > T2 V
Reason : If Dl is the average distance moved between two
T1
collision and Dt is the corresponding time, then (b) T1 < T2
Dl T2
vd = lim
(c) T1 = T2
.
Dt ® 0 Dt
(a) Assertion is correct, reason is correct; reason is a 1 I
(d) T1 = O
correct explanation for assertion. T2
(b) Assertion is correct, reason is correct; reason is not a
28. The amount of charge Q passed in time t through a cross-
correct explanation for assertion
section of a wire is Q = 5 t2 + 3 t + 1. The value of current at
(c) Assertion is correct, reason is incorrect time t = 5 s is
(d) Assertion is incorrect, reason is correct. (a) 9 A (b) 49 A
19. Two wires A and B of the same material, having radii in the (c) 53 A (d) None of these
ratio 1 : 2 and carry currents in the ratio 4 : 1. The ratio of drift 29. A conductor carries a current of 50 m A. If the area of cross-
speed of electrons in A and B is section of the conductor is 50 mm 2, then value of the current
(a) 16 : 1 (b) 1 : 16 (c) 1 : 4 (d) 4 : 1 density in Am–2 is
20. When a current I is set up in a wire of radius r, the drift (a) 0.5 (b) 1 (c) 10–3 (d) 10–6
velocity is vd. If the same current is set up through a wire of 30. The resistance of a wire is R. It is bent at the middle by 180°
radius 2 r, the drift velocity will be and both the ends are twisted together to make a shorter
(a) 4 vd (b) 2 vd (c) vd/2 (d) vd/4 wire. The resistance of the new wire is
21. A straight conductor of uniform cross-section carries a (a) 2 R (b) R/2 (c) R/4 (d) R/8
current I. If s is the specific charge of an electron, the 31. At what temperature will the resistance of a copper wire
momentum of all the free electrons per unit length of the becomes three times its value at 0°C? (Temperature
conductor, due to their drift velocity only is coefficient of resistance of copper is 4 × 10–3/°C)
(a) 550°C (b) 500°C (c) 450°C (d) 400°C
(a) I s (b) I/s (c) I/s (d) (I / s)2
32. The figure shows three conductors I, II and III of same
22. If the resistance of a conductor is 5W at 50º C & 7W at material, different lengths l, 2l and 3l and of different areas of
100º C, then mean temperature coefficient of resistance (of cross-section 3A, A and 2A respectively. Arrange them in
material) is the increasing order of current drawn from battery.
(a) 0.013/ ºC (b) 0.004/ ºC (c) 0.006/ ºC (d) 0.008/ ºC l
i1 3A i1
23. If negligibly small current is passed though a wire of length I
15 m and resistance of 5W, having uniform cross section of 2l
i2 A i2
6 × 10–7 m2, then coefficient of resistivity of material is II
(a) 1×10–7W–m (b) 2×10–7W–m 2A 3l
III
(c) 3×10–7W–m (d) 4×10–7W–m i3 i3
24. The resistance of a wire at room temperature 30°C is found ( )
to be 10 W. Now to increase the resistance by 10%, the (a) i1 < i2 < i3 (b) i3 < i2 < i1
temperature of the wire must be [ The temperature coefficient
(c) i2 < i1 < i3 (d) i2 `< i3 < i1
of resistance of the material of the wire is 0.002 per °C]
33. A wire of a certain material is stretched slowly by ten per
(a) 36°C (b) 83°C (c) 63°C (d) 33°C cent. Its new resistance and specific resistance become
25. A wire X is half the diameter and half the length of a wire Y respectively:
of similar material. The ratio of resistance of X to that of Y is (a) 1.2 times, 1.3 times (b) 1.21 times, same
(a) 8 : 1 (b) 4 : 1 (c) 2 : 1 (d) 1 : 1 (c) both remain the same (d) 1.1 times, 1.1 times
EBD_7418
254 PHYSICS

34. A 4 ohm resistance wire is bent through 180º at its mid point 45. The current in the 1W resistor shown in the circuit is
and the two halves are twisted together. Then the resistance is 2 4W
(a) 1 W (b) 2 W (c) 5 W (d) 8 W (a) A
3
35. The length of a given cylindrical wire is increased by 100%. (b) 3 A 1W 6V
Due to the consequent decrease in diameter the change in (c) 6 A
the resistance of the wire will be (d) 2 A 4W
(a) 200% (b) 100% (c) 50% (d) 300% 46. A wire of resistance 12 ohms per meter is bent to form a
36. The masses of the three wires of copper are in the ratio of complete circle of radius 10 cm. The resistance between its
1 : 3 : 5 and their lengths are in the ratio of 5 : 3 : 1. The ratio two diametrically opposite points, A and B as shown in the
of their electrical resistance is figure, is
(a) 1 : 3 : 5 (b) 5 : 3 : 1 (c) 1 : 25 : 125 (d) 125 : 45 : 3 (a) 3W (b) 6 p W A B
37. A uniform wire of length l and radius r has a resistance of
r (c) 6W (d) 0. 6 p W
100 W. It is recast into a wire of radius . The resistance of
2 47. Two wires of the same metal have same length, but their
new wire will be : cross-sections are in the ratio 3:1. They are joined in series.
(a) 1600 W (b) 400 W (c) 200 W (d) 100 W The resistance of thicker wire is 10W. The total resistance of
38. The number of free electrons per 100 mm of ordinary copper the combination will be
wire is 2 × 1021 . Average drift speed of electrons is (a) 10 W (b) 20 W (c) 40 W (d) 100 W
0.25 mm/s. The current flowing is 48. Consider the following statements and select the correct
(a) 5 A (b) 80 A (c) 8 A (d) 0.8 A option.
39. At room temperature, copper has free electron density of I. When resistances are connected in parallel the
8.4 × 1028 per m3. The copper conductor has a cross-section equivalent resistance is less than the smallest
of 10–6 m2 and carries a current of 5.4 A. The electron drift resistance.
velocity in copper is II. When resistances are connected in parallel, current
(a) 400 m/s (b) 0.4 m/s (c) 0.4 mm/s (d) 72 m/s distributes in the inverse ratio of resistances.
40. An electron beam has an aperture 1.0 mm2. A total of 6×1016 III. When resistances are connected in series maximum
electrons go through any perpendicular cross section per current flows through the resistance having least
second. Find the current density in the beam. (in A/m2) value.
(a) 9.1 × 1013 (b) 9.6 × 103 (a) I only (b) II only
(c) 6.6 × 105 (d) 8.6 × 1011 (c) I and II (d) I, II and III
49. In the network shown, each resistance is equal to R. The
Topic 2: Combination of Resistances equivalent resistance between adjacent corners A and D is
41. Two or more resistors are said to be in ...A... if one end
of all resistors is joined together and similarly the other (a) R
ends joined together, Here, A refers to 2
(b) R
(a) series (b) parallel 3
(c) either (a) or (b) (d) None of these
3
42. In the series combination of two or more than two (c) R
resistances 7
(a) the current through each resistance is same 8
(d) R
(b) the voltage through each resistance is same 15
(c) neither current nor voltage through each resistance 50. A wire of length l is connected to a battery between point A
is same and B as shown in fig (1). The same wire is bent in the form of
(d) both current and voltage through each resistance are a circle and then connected to the battery between the points
same A and B as shown in fig. (2). The current drawn from the
43. A wire has a resistance 12 W. It is bent in the form of a circle. The battery
effective resistance between two points on any diameter is A B
(a) 6 W (b) 3 W (c) 12 W (d) 24 W A l B
44. In the network shown below, the ring has zero resistance.
The equivalent resistance between the point A and B is () ()
(a) 2R (1) (2)
3R (a) increases
(b) 4R 3R (b) decreases
A
B (c) remains same
(c) 7R R (d) increases if upper part of wire between A and B is a
3R
major arc and decreases if it is minor arc
(d) 10R
CURRENT ELECTRICITY 255

51. The equivalent resistance between points A and B is 60. A cell of internal resistance r is connected across an external
(a) 2R R R resistance nr. Then the ratio of the terminal voltage to the
A B emf of the cell is
(b) (3/4) R
R R 1 1 n n -1
(c) (4/3) R (a) (b) (c) (d)
n n +1 n +1 n
(d) (3/5) R 61. If n cells each of emf e and internal resistance r are
52. Six equal resistances are connected between points P, Q connected in parallel, then the total emf and internal
and R as shown in figure. Then net resistance will be maximum resistances will be
between : P
r r
(a) e, (b) e, nr (c) ne, (d) ne, nr
n n
(a) P and R
62. Under what condition will the strength of current in a wire
r
r of resistance R be the same for connection is series and in
(b) P and Q r
r
parallel of n identical cells each of the internal resistance r?
r When
(c) Q and R (a) R = n r (b) R = r/n
Q r
R (c) R = r (d) R ® ¥, r ® 0
(d) Any two points 63. Assertion : Kirchoff’s juction rule can be applied to a
junction of several lines or a point in a line.
Topic 3: Kirchhoff 's Laws and Cells Reason : When steady current is flowing, there is no
accumulation of charges at any junction or at any point
53. Kirchoff’s first law, i.e., S i = 0 at a junction, deals with the in a line.
conservation of (a) Assertion is correct, reason is correct; reason is a
(a) charge (b) energy correct explanation for assertion.
(c) momentum (d) angular momentum (b) Assertion is correct, reason is correct; reason is not a
54. The Kirchhoff's second law (SiR = SE), where the symbols correct explanation for assertion
have their usual meanings, is based on (c) Assertion is correct, reason is incorrect
(d) Assertion is incorrect, reason is correct.
(a) conservation of momentum 64. A cell having an emf e and internal resistance r is
(b) conservation of charge connected across a variable external resistance R. As the
(c) conservation of potential resistance R is increased, the plot of potential difference
(d) conservation of energy V across R is given by
55. Emf of a cell is
(a) the maximum potential difference between the terminals e e
of a cell when no current is drawn from the cell. (a) (b)
(b) the force required to push the electrons in the circuit. V V
(c) the potential difference between the positive and
0 0
negative terminal of a cell in a closed circuit. R R
(d) less than terminal potential difference of the cell.
56. An energy source will supply a constant current into the e V
load if its internal resistance is
(a) very large as compared to the load resistance (c) V (D)
(b) equal to the resistance of the load 0 0
(c) non-zero but less than the resistance of the load R R
(d) zero 65. Two sources of equal emf are connected to an external
57. To draw a maximum current from a combination of cells, resistance R. The internal resistance of the two sources are
how should the cells be grouped? R1 and R2 (R2 > R1). If the potential difference across the
(a) Parallel source having internal resistance R2 is zero, then
(b) Series (a) R= R 2 - R1
(c) Mixed grouping (b) R= R 2 ´ (R1 + R 2 ) /(R 2 - R1 )
(d) Depends upon the relative values of internal and (c) R= R 1R 2 /( R 2 - R 1 )
external resistances (d) R= R1R 2 /(R1 - R 2 )
58. A cell of internal resistance r is connected to an external 66. In the series combination of n cells each cell having emf
resistance R. The current will be maximum in R, if
e and internal resistance r. If three cells are wrongly
(a) R = r (b) R < r (c) R > r (d) R = r/2 connected, then total emf and internal resistance of this
59. A capacitor is connected to a cell of emf E having some
combination will be
internal resistance r. The potential difference across the
(a) cell is < E (b) cell is E (a) ne, (nr – 3r) (b) (ne – 2e) nr
(c) capacitor is > E (d) capacitor is < E (c) (ne – 4e), nr (d) (ne – 6e), nr
EBD_7418
256 PHYSICS

67. The internal resistance of a 2.1 V cell which gives a current 74. A dc source of emf E1 = 100 V and internal resistance r = 0.5
of 0.2 A through a resistance of 10 W is W, a storage battery of emf E2 = 90 V and an external resistance
(a) 0.5 W (b) 0.8 W (c) 1.0 W (d) 0.2 W R are connected as shown in figure. For what value of R no
68. A primary cell has an e.m.f. of 1.5 volt. When short-circuited current will pass through the battery ?
it gives a current of 3 ampere. The internal resistance of the E1 r = 0.5 W
cell is
(a) 4.5 ohm (b) 2 ohm (c) 0.5 ohm (d) (1/4.5) ohm
69. Determine the current in 2W resistor.
2W
(a) 1 A
3W
(b) 1.5 A E2
1W

(c) 0.9 A
(d) 0.6 A 6V 2.8 W
R
70. In the circuit shown in figure, with steady current, the (a) 5.5 W (b) 3.5 W (c) 4.5 W (d) 2.5 W
potential drop across the capacitor must be 75. Three batteries of emf 1V and internal resistance 1W each
(a) V are connected as shown. Effective emf of combination
V
V between the points PQ is P
(b) R
2 (a) zero
V 1W 1W
V || (b) 1V 1V
(c) C 1V
3 (c) 2V
2V (d) (2/3) V Q 1W R
1V
(d) 2R
3 2V 76. The figure below shows currents in a part of electric circuit.
71. Assertion : Two non-ideal batteries are connected in parallel. The current i is 1 amp
The equivalent emf is smaller than either of the two emfs. 2 amp
(a) 1.7 amp 1.3 amp
Reason : The equivalent internal resistance is smaller than
(b) 3.7 amp
either of the two internal resistances. 2 amp
(a) Assertion is correct, reason is correct; reason is a (c) 1.3 amp
i
correct explanation for assertion. (d) 1 amp
(b) Assertion is correct, reason is correct; reason is not a 77. The four wires from a larger circuit intersect at junction A as
correct explanation for assertion shown. What is the magnitude and direction of the current
(c) Assertion is correct, reason is incorrect
between points A and B ?
(d) Assertion is incorrect, reason is correct.
(a) 2 A from A to B 4A
72. A 3 volt battery with negligible internal resistance is
connected in a circuit as shown in the figure. The current I, A B
(b) 2A from B to A 5A
in the circuit will be
(a) 1 A (c) 3A from A to B
6A
(d) 2A from B to A
(b) 1.5 A 3W
3W 78. A battery of emf 10V and internal resistance 30hm is
3V
(c) 2 A conncted to a resister. The current in the circuit is 0.5 amp.
(d) 1/3 A The terminal voltage of the battery when the circuit is closed
3W is
73. Which of the following is the correct equation when
(a) 10V (b) zero (c) 1.5V (d) 8.5V
kirchhoff’s loop rule is applied to the loop BCDEB in clockwise
direction? 79. Five cells each of emf E and internal resistance r send the
R3 R4 same amount of current through an external resistance R
(a) –i3 R3 – i3 R4 – i2 R2 = 0 C D
whether the cells are connected in parallel or in series. Then
(b) –i3 R3 – i3 R4 + i2 R2 = 0 i3
R2
the ratio æç ö÷ is
i2 R
B E
(c) –i3 R3 + i3 R4 + i2 R2 = 0 è rø
R1 i1
(d) –i3 R3 + i3 R4 + i2 R2 = 0 A F 1 1
(a) 2 (b) (c) (d) 1
2 5
CURRENT ELECTRICITY 257

(D) Greater the resistance (4) When resistances are


Topic 4: Heating Effect of Electric Current
greater the power connected in parallel
80. The cause of heat production in a current carrying conductor is (a) (A) ® (3) ; (B) ® (1) ; (C) ® (2) ; (D) ® (4)
(a) collisions of free electrons with one another (b) (A) ® (1) ; (B) ® (3) ; (C) ® (2) ; (D) ® (4)
(b) high drift speed of free electrons (c) (A) ® (2) ; (B) ® (1) ; (C) ® (4) ; (D) ® (2)
(c) collisions of free electrons with atoms or ions of the (d) (A) ® (4) ; (B) ® (3) ; (C) ® (1) ; (D) ® (2)
conductor 91. It takes 12 minutes to boil 1 litre of water in an electric kettle.
(d) high resistance value Due to some defect it becomes necessary to remove 20%
81. Resistance of conductor is doubled keeping the potential turns of heating coil of the kettle. After repair, how much
difference across it constant. The rate of generation of heat will time will it take to boil 1 litre of water ?
(a) become one fourth (b) be halved (a) 9.6 minute (b) 14.4 minute
(c) be doubled (d) become four times
(c) 16.8 minute (d) 18.2 minute
82. An electric bulb is rated 220 volt - 100 watt. The power
consumed by it when operated on 110 volt will be 92. In household electric circuit
(a) 75 watt (b) 40 watt (c) 25 watt (d) 50 watt I. all electric appliances drawing power are joined in
83. A current of 2 A, passing through a conductor produces 80 J of parallel.
heat in 10 seconds. The resistance of the conductor in ohm is II. a switch may be either in series or in parallel with the
(a) 0.5 (b) 2 (c) 4 (d) 20 appliance which it controls.
84. The resistance of hot tungsten filament is about 10 times III. if a switch is in parallel with an appliance, it will draw
the cold resistance. What will be the resistance of 100 W power when the switch is in the ‘off’ position (open).
and 200 V lamp when not in use ? IV. if a switch is in parallel with an appliance, the fuse will
(a) 20 W (b) 40 W (c) 200 W (d) 400 W blow (burn out) when the switch is put ‘on’ closed.
85. A 25 W and 100 W bulb are joined in series and connected Which of the above statements are correct?
to the mains. Which bulb will glow brighter? (a) I and IV (b) I, III and IV
(a) 25 W bulb (c) II, III and IV (d) I, II and IV
(b) 100 W bulb 93. A torch bulb rated as 4.5 W, 1.5 V is connected as shown in
(c) Both bulb will glow brighter
fig. The e.m.f. of the cell, needed to make the bulb glow at
(d) None will glow brighter
full intensity is
86. An electric fan and a heater are marked as 100 W, 220 V and 4.5 W,
(a) 4.5 V
1000 W, 220 V respectively. The resistance of heater is 1.5V
(a) equal to that of fan 2E/9
(b) 1.5 V
(b) lesser than that of fan
(c) greater than that of fan E/9
(c) 2.67 V
(d) zero 0.33 W
87. Water boils in the electric kettle in 15 minutes after switching E/3
on. If the length of heating wire is decreased to 2/3 of its (d) 13.5 V
initial value, then the same amount of water will boil with the E, r = 2.67 W
same supply voltage in 94. Forty electric bulbs are connected in series across a 220 V
(a) 8 minutes (b) 10 minutes supply. After one bulb is fused the remaining 39 are
(c) 12 minutes (d) 15 minutes connected again in series across the same supply. The
illumination will be
88. A wire of radius r and another wire of radius 2r, both of (a) more with 40 bulbs than with 39
same material and length are connected in series to each (b) more with 39 bulbs than with 40
other. The combination is connected across a battery. The (c) equal in both the cases
ratio of the heats produced in the two wires will be (d) in the ratio 402 : 392
(a) 4.00 (b) 2.00 (c) 0.50 (d) 0.25 95. A battery of e.m.f E and internal resistance r is connected
89. A battery of e.m.f. 10 V and internal resistance 0.5 W is to a variable resistor R as shown. Which one of the following
connected across a variable resistance R. The value of R for is true ?
which the power delivered in it is maximum is given by E r
(a) 0.5 W (b) 1.0 W (c) 2.0 W (d) 0.25 W
90. Match the Column I and Column II. R
Column I Column II
(A) Smaller the resistance (1) If the same voltager is
greater the current applied and resistance
are in series (a) Potential difference across the terminals of the
(B) Greater or smaller the (2) If the same current is battery is maximum when R = r
resistance the current passed (b) Power delivered to resistor is maximum when
is same R = 2r
(C) Greater the resistance (3) When resistances are (c) Current in the circuit is maximum when R = r
smaller the power connected in series (d) Current in the circuit is maximum when R >> r
EBD_7418
258 PHYSICS

96. A current of 3 amp flows th rough th e 2W (a) the potentiometer is cheap and easy to handle.
resistor shown in the circuit. The power dissipated in the (b) calibration in the voltmeter is sometimes wrong .
5-W resistor is: 2W (c) the potentiometer almost draws no current during
(a) 4 watt measurement.
4W (d) range of the voltmeter is not as wide as that of the
(b) 2 watt potentiometer.
(c) 1 watt 106. In a wheatstone bridge in the battery and galvanometer
are interchanged then the deflection in galvanometer will
(d) 5 watt 1W 5W (a) change in previous direction
97. You have the following appliances each of 500 watt running (b) not change
on 220 volts a.c. (1) electric iron (2) an electric lamp (3) an (c) change in opposite direction
electric room heater. The electric resistance is (d) none of these
(a) maximum for the heater 107. In meter bridge or Wheatstone bridge for measurement of
(b) maximum for the electric lamp resistance, the known and the unknown resistance are
(c) maximum for the electric iron interchanged. The error so removed is
(d) same in all the three cases (a) end correction
98. Two wires having resistances of 2 W and 4 W are (b) index error
(c) due to temperature effect
connected to same voltage. The ratio of heat dissipated at (d) random error
the resistance will be 108. Match the entries of Column I with their correct mathematical
(a) 4 : 3 (b) 1 : 2 (c) 5 : 2 (d) 2 : 1 expressions in Column II
99. A 100 watt bulb working on 200 volt has resistance R and a Column I Column II
200 watt bulb working on 100 volt has resistance S then R/S is
1 1 (A) Balanced condition (1) R1 = R3
(a) (b) (c) 8 (d) 4 R2 R4
8 4 of wheatstone bridge
100. A electric tea kettle has two heating coils. When first coil of R l1
(B) Comparison of emf of (2) =
resistance R1 is switched on, the kettle begins to boil tea in S 100 - l1
6 minutes. When second coil of resistance R2 is switched two cells field.
E1 l1
on, the boiling begins in 8 minutes. The value of R1/R2 is (C) Determination of (3) =
7 3 4 internal resistance of E l2
3 2
(a) (b) (c) (d) a cell
3 7 4 3
(D) Determination of (4) r = R æ l1 - 1ö
çl ÷
Topic 5: Wheatstone Bridge and Different Measuring Instruments unknown resistance è 2 ø
by meter bridge
101. In potentiometer a balance point is obtained, when (a) (A) ® (4); (B) ® (2); C ® (3); (D) ® (1)
(a) the e.m.f. of the battery becomes equal to the e.m.f of
the experimental cell (b) (A) ® (1); (B) ® (3); C ® (4); (D) ® (2)
(b) the p.d. of the wire between the +ve end of battery to (c) (A) ® (3); (B) ® (4); C ® (2); (D) ® (1)
jockey becomes equal to the e.m.f. of the experimental cell (d) (A) ® (4); (B) ® (3); C ® (2); (D) ® (1)
(c) the p.d. of the wire between +ve point of cell and jockey 109. Assertion : In meter bridge experiment, a high resistance
becomes equal to the e.m.f. of the battery is always connected in series with a galvanometer.
(d) the p.d. across the potentiometer wire becomes equal Reason : As resistance increase current more accurately
to the e.m.f. of the battery then ammeter.
102. Why is the Wheatstone bridge better than the other methods
(a) Assertion is correct, reason is correct; reason is a
of measuring resistances?
(a) It does not involve Ohm’s law correct explanation for assertion.
(b) It is based on Kirchoff’s law (b) Assertion is correct, reason is correct; reason is not a
(c) It has four resistor arms correct explanation for assertion
(d) It is a null method (c) Assertion is correct, reason is incorrect
103. In the experiment of potentiometer, at balance point, there is (d) Assertion is incorrect, reason is correct.
no current in the 110. AB is a wire of potentiometer with the increase in value
(a) main circuit of resistance R, the shift in the balance point J will be
(b) galvanometer circuit e
(a) towards B R
(c) potentiometer circuit
(d) both main and galvanometer circuits (b) towards A
104. Sensitivity of potentiometer can be increased by (c) remains constant A J B
(a) increasing the e.m.f of the cell (d) first towards B then
(b) increasing the length of the potentiometer back towards A G
(c) decreasing the length of the potentiometer wire
(d) None of these 111. In a Wheatstone bridge all the four arms have equal
105. For measuring voltage of any circuit, potentiometer is resistance R. If the resistance of galvanometer arm is also R,
preferred to voltmeter because the equivalent resistance of combination is
(a) 2R (b) R/4 (c) R/2 (d) R
CURRENT ELECTRICITY 259

112. The resistance of the four arms P, Q, R and S in a 119. The resistances in the two arms of the meter bridge are 5W
Wheatstone’s bridge are 10 ohm, 30 ohm, 30 ohm and 90 and RW, respectively. When the resistance R is shunted
ohm, respectively. The e.m.f. and internal resistance of the with an equal resistance, the new balance point is at 1.6 l1.
cell are 7 volt and 5 ohm respectively. If the galvanometer The resistance ‘R’ is :
resistance is 50 ohm, the current drawn from the cell will be (a) 10W 5W RW
(a) 0.2 A (b) 0.1 A (c) 2. 0 A (d) 1. 0 A (b) 15W
113. In a Wheatstone's bridge, three resistances P, Q and R (c) 20W G
connected in the three arms and the fourth arm is formed by
two resistances S1 and S2 connected in parallel. The (d) 25W A B
l1 100 – l1
condition for the bridge to be balanced will be 120. Each of the resistance in the network shown in fig. is equal
P 2R P R (S1 + S2 ) to R. The resistance between the terminals A and B is
(a) = (b) Q = S S (a) R L
Q S1 + S2 1 2
P R (S1 + S2 ) P R (b) 5 R
(c) Q = 2S S (d) =
R
1 2 Q S1 + S2 R R
(c) 3 R A
114. The current I drawn from the 5 volt source will be

R
(a) 0.33 A 10W R B
(d) 6 R K M

(b) 0.5 A 5W 10W 20W 121. The figure shows a meter bridge in which null point is
obtained at a length AD = l. When a resistance S' is connected
in parallel with resistance S the new position of null point is
(c) 0.67 A I obtained
10W
+–
(d) 0.17 A R S'
5 volt B
S
115. Assertion : The e.m.f of the driver cell in the potentiometer
d G
experiment should be greater that the e.m.f of the cell to A D C
be determined.
Reason : The fall of potential across the potentiometer
wire should not be less than the e.m.f of the cell to be ( )
determined. (a) to the left of D
(a) Assertion is correct, reason is correct; reason is a (b) to the right of D
correct explanation for assertion. (c) at the same point D
(b) Assertion is correct, reason is correct; reason is not a (d) to the left of D if S' has lesser value than S and to the
correct explanation for assertion right of D if S' has more value than S
(c) Assertion is correct, reason is incorrect 122. The figure shows a circuit diagram of a Wheatstone
(d) Assertion is incorrect, reason is correct. Bridge’ to measure the resistance G of the galvanometer.
116. In the figure in balanced condition of wheatstone bridge P R
(a) B is at higher potential. B The relation = will be satisfied only when
Q G
(b) D is at higher potential.
R2 R4
(c) Any of the two B or D A G C P Q
R1 R3
can be at higher potential S
than other arbitrarily. R G
D
(d) B and D are at same potential.
117. 125 cm of potentiometer wire balances the emf. of a cell and
100 cm of the wire is required for balance, if the poles of the (a) the galvanometer shows a deflection when switch S
cell are joined by a 2W resitor. Then the internal resistance is closed
of the cell is (b) the galvanometer show a deflection when switch S is
(a) 0.25 W (b) 0.5 W (c) 0.75 W (d) 1.25 W open
118. The resistance of an ammeter is 13 W and its scale is (c) the galvanometer shows no change in deflection
graduated for a current upto 100 amps. After an additional whether S is open or closed
shunt has been connected to this ammeter it becomes (d) the galvanometer shows no deflection
possible to measure currents upto 750 amperes by this meter. 123. In the circuit , the galvanometer G shows zero deflection. If
The value of shunt-resistance is the batteries A and B have negligible internal resistance,
(a) 2 W (b) 0.2 W (c) 2 k W (d) 20 W the value of the resistor R will be
EBD_7418
260 PHYSICS

500 W 127. A cell when balanced with potentiometer gave a balance


(a) 100 W G
length of 50 cm. 4.5 W external resistance is introduced in
(b) 200 W 2V the circuit, now it is balanced on 45 cm. The internal
12V R A
resistance of cell is
B
(c) 1000 W (a) 0.25 W (b) 0.5 W (c) 1.0 W (d) 1.5 W
128. A potentiometer consists of a wire of length 4m and
(d) 500 W
resistance 10W. It is connected to a cell of e.m.f. 3V. The
124. In a metre bridge, the balancing length from the left end potential gradient of wire is
(standard resistance of one ohm is in the right gap) is found to (a) 5V/m (b) 2V/m (c) 5V/m (d) 10V/m
be 20 cm. The value of the unknown resistance is 129. Potentiometer wire of length 1 m is connected in series with
(a) 0.8 W (b) 0.5 W (c) 0.4 W (d) 0.25 W 490W resistance and 2 V battery. If 0.2 mV/cm is the potential
125. If specific resistance of a potentiometer wire is 10–7Wm gradient, then resistance of the potentiometer wire is
current flowing through it, is 0.1 amp and cross sectional (a) 4.9 W (b) 7.9 W (c) 5.9 W (d) 6.9 W
area of wire is 10–6 m2, then potential gradient will be 130. In an experiment to measure the internal resistance of a cell,
(a) 10–2 volt/m (b) 10–4 volt/m by a potentiometer, it is found that the balance point is at a
–6
(c) 10 volt/m (d) 10–8 volt/m length of 2 m, when the cell is shunted by a 5 W resistance
126. The current in the primary circuit of a potentiometer wire is and is at a length of 3 m when the cell is shunted by a 10 W
0.5 A, r for the wire is 4 × 10–7 W–m and area of cross- resistance. The internal resistance of the cell is
section of wire is 8 × 10–6 m2. The potential gradient in the (a) 1.5 W (b) 10 W (c) 15 W (d) 1 W
wire would be
(a) 25 mV/meter (b) 2.5 mV/meter
(c) 25 V/meter (d) 10 V/meter

Exercise 2 : Exemplar & Past Year MCQs

4. Two cells of emfs approximately 5 V and 10 V are to be


NCERT Exemplar MCQs
accurately compared using a potentiometer of length 400 cm.
1. Consider a current carrying wire (current I) in the shape of a (a) The battery that runs the potentiometer should have
circle. voltage of 8V.
(a) source of emf (b) The battery of potentiometer can have a voltage of 15 V
(b) electric field produced by charges accumulated on the and R adjusted so that the potential drop across the
surface of wire wire slightly exceeds 10 V.
(c) the charges just behind a given segment of wire which (c) The first portion of 50 cm of wire itself should have a
push them just the right way by repulsion potential drop of 10 V.
(d) the charges ahead (d) Potentiometer is usually used for comparing resistances
2. Two batteries of emf e1 and e2(e2 > e1) and internal resistances and not voltages.
r1 and r2 respectively are connected in parallel as shown in 5. A metal rod of length 10 cm and a rectangular cross-section
figure. 1
e1 of 1cm × cm is connected to a battery across opposite
r1 2
faces. The resistance will be
A B
(a) maximum when the battery is connected across 1 cm ×
e2 r2
1
cm faces
(a) Two equivalent emf eeq of the two cells is between e1 2
and e2, i.e., e1 < eeq < e2 (b) maximum when the battery is connected across 10 cm × 1 cm
(b) The equivalent emf eeq is smaller than e1 faces
(c) The eeq is given by eeq = e1 + e2 always (c) maximum when the battery is connected across 10 cm
(d) eeq is independent of internal resistances r1 and r2
3. A resistance R is to be measured using a meter bridge, 1
× cm faces
student chooses the standard resistance S to be 100W. He 2
finds the null point at l1 = 2.9 cm. He is told to attempt to (d) same irrespective of the three faces
improve the accuracy. 6. Which of the following characteristics of electrons
Which of the following is a useful way? determines the current in a conductor?
(a) He should measure I1 more accurately. (a) Drift velocity alone
(b) He should change 5 to 1000W and repeat the experiment. (b) Thermal velocity alone
(c) He should change S to 3W and repeat the experiment. (c) Both drift velocity and thermal velocity
(d) He should given up hope of a more accurate (d) Neither drift nor thermal velocity
measurement with a meter bridge.
CURRENT ELECTRICITY 261

Past Year MCQs (a) If both Assertion and Reason are correct and Reason
7. Two cities are 150 km apart. Electric power is sent from one is the correct explanation of Assertion.
city to another city through copper wires. The fall of potential (b) If both Assertion and Reason are correct, but Reason
per km is 8 volt and the average resistance per km is 0.5 W. is not the correct explanation of Assertion.
The power loss in the wires is : [AIPMT 2014, A] (c) If Assertion is correct but Reason is incorrect.
(a) 19.2 W (b) 19.2 kW (c) 19.2 J (d) 12.2 kW (d) If both the Assertion and Reason are incorrect.
8. Find out the value of current through 2W resistance for the 15. When 5V potential difference is applied across a wire of
given circuit [AIIMS 2014, A] length 0.1 m, the drift speed of electrons is 2.5 × 10–4 ms–1. If
the electron density in the wire is 8 × 1028 m–3, the resistivity
of the material is close to : [JEE Main 2015, A]
5W 10 W (a) 1.6 × 10–6 Wm (b) 1.6 × 10–5 Wm
10 V 20 V (c) 1.6 × 10–8 Wm (d) 1.6 × 10–7 Wm
2W 16. A constant voltage is applied between the two ends of a
uniform metallic wire. Some heat is developed in it. The heat
(a) zero (b) 2 A
developed is doubled if [BITSAT 2015, C]
(c) 5 A (d) 4 A
(a) both the length and the radius of the wire are halved.
9. The cold junction of a thermocouple is maintained at 10ºC.
No thermo e.m.f. is developed when the hot junction is (b) both the length and the radius of the wire are doubled.
maintained at 530ºC. The neutral temperature is (c) the radius of the wire is doubled.
[AIIMS 2014, A] (d) the length of the wire is doubled.
(a) 260ºC (b) 265ºC 17. The charge flowing through a resistance R varies with time
t as Q = at – bt2, where a and b are positive constants. The
(c) 270ºC (d) 520ºC
total heat produced in R is: [NEET 2016, C, BN]
10. In a large building, there are 15 bulbs of 40 W, 5 bulbs of
100 W, 5 fans of 80 W and 1 heater of 1 kW. The voltage of a 3R a 3R a 3R a 3R
(a) (b) (c) (d)
electric mains is 220 V. The minimum capacity of the main 6b 3b 2b b
fuse of the building will be: [JEE Main 2014, A] 18. A potentiometer wire is 100 cm long and a constant potential
(a) 8 A (b) 10 A (c) 12 A (d) 14 A difference is maintained across it. Two cells are connected
11. A potentiometer wire of length L and a resistance r are in series first to support one another and then in opposite
connected in series with a battery of e.m.f. E0 and a resistance direction. The balance points are obtained at 50 cm and 10
r1. An unknown e.m.f. E is balanced at a length l of the cm from the positive end of the wire in the two cases. The
potentiometer wire. The e.m.f. E will be given by : ratio of emf's is : [NEET 2016, A]
[AIPMT 2015, C] (a) 5 : 1 (b) 5 : 4 (c) 3 : 4 (d) 3 : 2
E0 r l LE r LE 0 r 19. A current source drives a current in a coil of resistance R1
E 0 l 0
(a) (r + r ) . L (b) (c) (r + r )l (d) lr1
for a time t. The same source drives current in another coil
1 L 1 of resistance R2 for same time. If heat generated is same,
12. The equivalent resistance between A and B is find internal resistance of source. [AIIMS 2016, C]
A [AIIMS 2015, A] R1R 2
R R (a) (b) R1 + R 2
R1 + R 2
B R (c) zero (d) R 1R 2
20. Twelve resistors each of resistance 16 W are connected in
R R the circuit as shown. The net resistance between AB is
8R 5R [AIIMS 2016, S]
(a) (b)
5 8
3R 7R
(c) (d)
8 8
13. A potentiometer wire, 10 m long, has a resistance of 40W. It
is connected in series with a resistance box and a 2 V storage
cell. If the potential gradient along the wire is 0.1 m V/cm, (a) 1W (b) 2W
the resistance unplugged in the box is [AIIMS 2015, A] (c) 3W (d) 4W
r
(a) 260 W (b) 760 W 21. Assertion : The current density J at any point in ohmic
r
(c) 960 W (d) 1060 W resistor is in direction of electric field E at that point.
14. Assertion : When current through a bulb decreases by 0.5%, Reason : A point charge when released from rest in a region
the glow of bulb decreases by 1%. having only electrostatic field always moves along electric
Reason : Glow (Power) which is directly proportional to lines of force. [AIIMS 2016, C]
square of current. [AIIMS 2015, C] (a) If both Assertion and Reason are correct and Reason
is the correct explanation of Assertion.
EBD_7418
262 PHYSICS

(b) If both Assertion and Reason are correct, but Reason R R


is not the correct explanation of Assertion. (a) (b) n2R (c) (d) nR
n n2
(c) If Assertion is correct but Reason is incorrect. 28. A potentiometer is an accurate and versatile device to make
(d) If both the Assertion and Reason are incorrect. electrical measurements of E.M.F. because the method
22. Assertion : Bending a wire does not effect electrical involves [NEET 2017, C]
resistance. (a) Potential gradients
(b) A condition of no current flow through the galvanometer
Reason : Resistance of wire is proportional ot resistivity of (c) A combination of cells, galvanometer and resistances
material. [AIIMS 2016, C] (d) Cells
(a) If both Assertion and Reason are correct and Reason 29. In the circuit shown, the current in the 1W resistor is :
is the correct explanation of Assertion. [AIIMS 2017, A]
(b) If both Assertion and Reason are correct, but Reason 6V P 2W
is not the correct explanation of Assertion.
(c) If Assertion is correct but Reason is incorrect. 1W 9V
(d) If both the Assertion and Reason are incorrect.
23. In the circuit shown in the figure, find the current in 45 W.
[BITSAT 2016, S] 3W W 3W
(a) 0.13 A, from Q to P
(b) 0.13 A, from P to Q
180V 90W 45W (c) 1.3A from P to Q
90W (d) 0A
30. Assertion : Kirchoff¢s junction rule follows from
100W
conservation of charge.
100W 50W
50W Reason : Kirchoff¢s loop rule follows from conservation of
momentum. [AIIMS 2017, C]
100W 50W (a) If both Assertion and Reason are correct and Reason
(a) 4 A (b) 2.5A is the correct explanation of Assertion.
(c) 2 A (d) None of these (b) If both Assertion and Reason are correct, but Reason
24. The drift velocity of electrons in silver wire with cross- is not the correct explanation of Assertion.
sectional area 3.14 × 10–6 m2 carrying a current of 20 A is. (c) If Assertion is correct but Reason is incorrect.
Given atomic weight of Ag = 108, density of silver (d) If both the Assertion and Reason are incorrect.
= 10.5 × 103 kg/m3. [BITSAT 2016, A] 31. Which of the following statements is false ?
(a) 2.798 × 10–4 m/sec. (b) 67.98 × 10–4 m/sec. [JEE Main 2017, C]
(a) A rheostat can be used as a potential divider
(c) 0.67 × 10–4 m/sec. (d) 6.798 × 10–4 m/sec.
(b) Kirchhoff's second law represents energy
25. In the shown arrangement of the experiment of the meter conservation
bridge if AC corresponding to null deflection of
(c) Wheatstone bridge is the most sensitive when all the
galvanometer is x, what would be its value if the radius of four resistances are of the same order of magnitude
the wire AB is doubled? [BITSAT 2016, A] (d) In a balanced wheatstone bridge if the cell and the
galvanometer are exchanged, the null point is
disturbed.
R1 R2
32. In the circuit the current in each resistance is
[JEE Main 2017, A]
G
(a) 0.5A
A x C B (b) 0 A
(a) x (b) x /4 (c) 4 x (d) 2 x (c) 1 A
26. Two resistances at 0° C with temperature coefficient of (d) 0.25 A
resistance a1 and a2 joined in series act as a single resistance
in a circuit. The temperature coefficient of their single 33. In the given circuit diagram when the current reaches steady
resistance will be [BITSAT 2016, A] state in the circuit, the charge on the capacitor of capacitance
a a C will be : [JEE Main 2017, A]
(a) a1 + a 2 (b) 1 2 r2
a1 + a 2 (a) CE
(r + r2 )
a1 - a 2 a1 + a 2 r1
(c) (d) (b) CE
2 2 (r1 + r)
27. The resistance of a wire is 'R' ohm. If it is melted and stretched (c) CE
to 'n' times its original length, its new resistance will be : r
(d) CE 1
[NEET 2017, C] (r2 + r)
CURRENT ELECTRICITY 263

34. A meter bridge is set up as shown, to determine an unknown 38. A set of 'n' equal resistors, of value 'R' each,are connected in
resistance ‘X’ using a standard 10 ohm resistor. The series to a battery of emf 'E' and internal resistance 'R'. The
galvanometer shows null point when tapping-key is at current drawn is I. Now, the 'n' resistors are connected in
52 cm mark. The end-corrections are 1 cm and 2 cm parallel to the same battery. Then the current drawn from
respectively for the ends A and B. The determined value of battery becomes 10 I. The value of'n' is [NEET 2018, C]
‘X’ is [BITSAT 2017, A] (a) 10 (b) 11 (c) 9 (d) 20
39. A battery consists of a variable number 'n' of identical cells
(having internal resistance 'r' each) which are connected in
series. The terminals of the battery are short-circuited and
the current I is measured. Which of the graphs shows the
correct relationship between I and n? [NEET 2018, C]
(a) 10.2 ohm (b) 10.6 ohm
(c) 10.8 ohm (d) 11.1 ohm I I
35. In the circuit shown in figure th`e current through
3W 2W 2W
(a) (b)
O O
n n

I I
9V 8W 8W 4W
(c) (d)
O O
n n
40. Assertion : A current flows in a conductor only when there
2W 2W 2W
is an electric field within the conductor.
(a) the 3 W resistor is 0.50 A. [BITSAT 2017, A] Reason : The drift velocity of electron in presence of electric
(b) the 3 W resistor is 0.25 A. field decreases. [AIIMS 2018, C]
(c) the 4 W resistor is 0.50 A (a) Assertion is correct, reason is correct; reason is a
correct explanation for assertion.
(d) the 4 W resistor is 0.25 A.
(b) Assertion is correct, reason is correct; reason is not a
36. A wire is connected to a battery between the point M and N correct explanation for assertion
as shown in the figure (1). The same wire is bent in the form (c) Assertion is correct, reason is incorrect
of a square and then connected to the battery between the (d) Assertion is incorrect, reason is correct.
points M and N as shown in the figure (2). Which of the 41. A 5V battery with internal P2
following quantities increases ? [BITSAT 2016, A] resistance 2W and a 2V battery
M N with internal resistance 1W are
M N 5V 2V
connected to a 10W resistor as 2W 10W
1W
shown in the figure.
() () The current in the 10W resistor is
(1) (2) [AIIMS 2018, A]
(a) 0.27 A P2 to P1 (b) 0.03 A P1 to P2
(a) Heat produced in the wire and resistance offered by
the wire. (c) 0.03 A P2 to P1 (d) 0.27 A P1 to P2
(b) Resistance offered by the wire and current through the 42. Two bulbs of 500 W and 200 W are manufactured to operate
wire. on 220 V line. The ratio of heat produced in 500 W and 200
(c) Heat produced in the wire, resistance offered by the W, in two cases, when firstly they are connected in parallel
wire and current through the wire. and secondary in series will be [AIIMS 2018, A]
(d) Heat produced in the wire and current through the wire. 5 2 5 5 2 5 2 2
(a) : (b) : (c) : (d) :
37. A carbon resistor of (47 ± 4.7) kW is to be marked with rings 2 5 2 2 5 2 5 5
of different colours for its identification. The colour code 43. On interchanging the resistances, the balance point of a
sequence will be [NEET 2018, C] meter bridge shifts to the left by 10 cm. The resistance of
(a) Violet – Yellow – Orange – Silver their series combination is 1kW. How much was the resistance
on the left slot before interchanging the resistances?
(b) Yellow – Violet – Orange – Silver
[JEE Main 2018, A]
(c) Green – Orange – Violet – Gold (a) 990 W (b) 505 W (c) 550 W (d) 910 W
(d) Yellow – Green – Violet – Gold
EBD_7418
264 PHYSICS

44. Two batteries with e.m.f. 12 V and 13 V are connected in


parallel across a load resistor of 10 W . The internal resistances (a) 1/10 A (b) 3/10 A
of the two batteries are 1 W and 2 W respectively. The voltage 2V
across the load lies between: [JEE Main 2018, S] (c) 4/10 A (d) 7/10 A. A
(a) 11.6 V and 11.7 V (b) 11.5 V and 11.6 V 47. A 10V battery with internal resistance 1W and a 15V battery
(c) 11.4 V and 11.5 V (d) 11.7 V and 11.8 V with internal resistance 0.6 W are connected in parallel to a
45. In a potentiometer experiment, it is found that no current voltmeter (see figure). The reading in the voltmeter will be
passes through the galvanometer when the terminals of the close to : [BITSAT 2018, A]
cell are connected across 52 cm of the potentiometer wire. If 10V
the cell is shunted by a resistance of 5 W, a balance is found
when the cell is connected across 40 cm of the wire. Find the (a) 12.5 V 1W
internal resistance of the cell. [JEE Main 2018, S] 15V
(b) 24.5 V
(a) 1 W (b) 1.5 W (c) 2 W (d) 2.5 W
46. Seven resistances, each of value 20 W, are connected to a (c) 13.1 V 0.6W
2 V battery as shown in the figure. The ammeter reading will
be [BITSAT 2018, A] (d) 11.9 V
V

Exercise 3 : Try If You Can


1. Ten identical cells connected in series are needed to heat a A R R R
F R C
wire of length one meter and radius ‘r’ by 10ºC in time ‘t’.
How many cells will be required to heat the wire of length R R R R R
two meter of the same radius by the same temperature in
time ‘t’?
B R E R R R D
(a) 10 (b) 20 (c) 30 (d) 40
2. Two batteries of emf 4 V and 1W 4V 8V 2W (a) R (b) R ( 3 - 1) (c) 3 R (d) R ( 3 + 1)
8V with internal resistance P Q
r1 r2 6. In the diagram shown, all the wires have resistance R. The
1 W and 2 W are connected equivalent resistance between the upper and lower dots
in a circuit with a resistance
shown in the diagram is :
of 9 W as shown in figure. 9W
The current and potential R
difference between the points P and Q are
1 1
(a) A and 3 V (b) A and 4 V
3 6
1 1
(c) A and 9 V (d) A and 12V
9 12
3. A heater is operated with a power of 1000 W in a 100 V line. (a) R/8 (b) R (c) 2R/5 (d) 3R/8
It is connected in combination with a resistance of 10W and 7. All batteries are having emf 10 volt and internal resistance
a resistance R to a 100 V line as shown in figure. What negligible. All resistors are in ohms. Calculate the current in
should be the value of R so that the heater operates with a the right most 2W resistor.
power of 62.5 W ? 2
A 10W B C
Heater
10
25 25 2
(a) 10 W (b) 62.5 W (a) A (b) A 2 10 2 2
R 12 6
1 10
(c) W (d) 5 W 12 6 10
5 100 V (c) A (d) A
25 25 2 10
4. An electric heating element in vacuum is surrounded by a 8. An electrical cable of copper has just one wire of radius 9
cavity at constant temperature of 227ºC; it consumes 60W mm. Its resistance is 5 ohm. This single copper wire of the
of power to maintain a temperature of 727ºC. What is the cable is replaced by 6 different well insulated copper wires
power consumed by the element to maintain a temperature each of radius 3 mm. The total resistance of the cable will
of 1227º C? now be equal to
(a) 7.5 ohm (b) 45 ohm (c) 90 ohm (d) 270 ohm
(a) 101 W (b) 304 W (c) 90 W (d) 320 W
5. In Fig. find the value of resistor to be connected between C 9. Twelve indentical resistors each of value 1 W are connected
and D, so that the resistance of the entire circuit between A as shown. Net resistance between C and D (R) is
and B does not change wth the number of elementary sets.
CURRENT ELECTRICITY 265

7 t1 t 2 - t1 t 2 - t1 t2
C (a) (b) (c) (d) t
(a) R= W t2 t2 t1
6 1
G
15. You are given several identical resistances each of value R
4 = 10W and each capable of carrying a maximum current of
(b) R= W A E one ampere. It is required to make a suitable combination of
3 B
F these resistances of 5W which can carry a current of 4
(c) R =1 W H
ampere. The minimum number of resistances of the type R
that will be required for this job is
3 (a) 4 (b) 10 (c) 8 (d) 20
(d) R= W D
16. An Aluminium (Al) rod with area of cross-section 4 × 10–6 m2
4
has a current of 5 ampere. Flowing through it. Find the drift
10. Two rods are joined end to end, as shown. Both have a velocity of electron in the rod. Density of Al = 2.7 × 103 kg/m3
cross-sectional area of 0.01 cm2. Each is 1 meter long. One
rod is of copper with a resistivity of 1.7 × 10–6 ohm-centimeter, and Atomic wt. = 27. Assume that each Al atom provides
the other is of iron with a resistivity of 10–5 ohm-centimeter. one electron
How much voltage is required to produce a current of 1 (a) 8.6 × 10–4 m/s (b) 1.29 × 10–4 m/s
–2
(c) 2.8 × 10 m/s (d) 3.8 × 10–3 m/s
ampere in the rods?
(a) 0.117 V V 17. The current density varies with radial distance r as J = a r2,
(b) 0.00145 V in a cylindrical wire of radius R. The current passing through
the wire between radial distance R/3 and R/2 is
(c) 0.0145 V 65 p a R 4 25 p a R 4
(a) (b)
(d) 1.7 × 10–6 V 2592 72
Cu Fe 81 p a 2 R 4
65 p a 2 R3
11. Calculate equivalent resistance between A and B. (c) (d)
2938 144
F
I - I1 - I2 18. To get maximum current in a resistance of 3 ohms, one can
C E
R use n rows of m cells (connected in series) connected in
I2
b parallel. If the total number of cells is 24 and the internal
I-

resistance of a cell is 0.5 ohms then


I1
I1

R R
- I2
I-

R R (a) m = 12, n = 2 (b) m = 8, n = 3


+ I3

a c

I
R I3 R
I
(c) m = 2, n = 12 (d) m = 6, n = 4
A I1 D B 19. A battery of internal resistance 4W is connected to the
I1 + I2 - I3
network of resistances as shown. In order that the maximum
(a) 4/7 R (b) 8/7 R (c) 2/7 R (d) 1/7 R power can be delivered to the network, the value of R in W
12. A hank of uninsulated wire consisting of seven and a half should be
turns is stretched between two nails hammered into a board
R R
to which the ends of the wire are fixed. The resistance of the
circuit between the nails is determined with the help of 6R
E R R
electrical measuring instruments. Determine the proportion
in which the resistance will change if the wire is unwound 4W R 4R
so that the ends remain to be fixed to the nails.
(a) 225 (b) 15 (c) 240 (d) 250
13. A wire of length L and 3 identical cells of negligible internal 4 8
resistances are connected in series. Due to the current, the (a) (b) 2 (c) (d) 18
9 3
temperature of the wire is raised by DT in a time t. A number
N of similar cells is now connected in series with a wire of 20. The resistance of an electrical toaster has a temperature
the same material and cross-section but of length 2L. The dependence given by R(T) = R0 [1 + a(T – T0)] in its range
temperature of the wire is raised by the same amount DT in of operation. At T0 = 300K, R = 100 W and at T = 500 K, R =
the same time t then the value of N is 120 W. The toaster is connected to a voltage source at 200 V
(a) 4 (b) 6 (c) 8 (d) 9 and its temperature is raised at a constant rate from 300 to
14. Two different conductors have same resistance at 0°C. It is 500 K in 30 s. The total work done in raising the temperature
found that the resistance of the first conductor at t1°C is is :
equal to the resistance of the second conductor at t 2°C. 5 2
The ratio of the temperature coefficients of resistance of (a) 400ln kJ (b) 200 ln kJ
J
6 3
a
the conductors, 1 is
a2 æ6ö 1.5
(c) 60ln ç ÷ kJ (d) 400 ln J
kJ
è5ø 1.3
EBD_7418
266 PHYSICS

ANSWER KEYS
Exercise 1 : Topic-wise MCQs
1 (d) 14 (c) 27 (a) 40 (b) 53 (a) 66 (d) 79 (d) 92 (b) 105 (c) 118 (a)
2 (b) 15 (c) 28 (c) 41 (b) 54 (d) 67 (a) 80 (c) 93 (d) 106 (b) 119 (b)
3 (d) 16 (b) 29 (b) 42 (a) 55 (a) 68 (c) 81 (b) 94 (b) 107 (a) 120 (a)
4 (a) 17 (b) 30 (c) 43 (b) 56 (d) 69 (c) 82 (c) 95 (c) 108 (b) 121 (b)
5 (b) 18 (c) 31 (b) 44 (a) 57 (d) 70 (c) 83 (b) 96 (d) 109 (c) 122 (c)
6 (d) 19 (a) 32 (d) 45 (d) 58 (a) 71 (d) 84 (b) 97 (d) 110 (a) 123 (a)
7 (d) 20 (d) 33 (b) 46 (a) 59 (b) 72 (b) 85 (a) 98 (d) 111 (d) 124 (d)
8 (d) 21 (c) 34 (a) 47 (c) 60 (c) 73 (b) 86 (b) 99 (c) 112 (a) 125 (a)
9 (a) 22 (a) 35 (d) 48 (c) 61 (a) 74 (c) 87 (b) 100 (c) 113 (b) 126 (a)
10 (a) 23 (b) 36 (d) 49 (d) 62 (c) 75 (a) 88 (a) 101 (b) 114 (b) 127 (a)
11 (b) 24 (b) 37 (a) 50 (a) 63 (a) 76 (a) 89 (a) 102 (d) 115 (a) 128 (a)
12 (b) 25 (c) 38 (d) 51 (d) 64 (b) 77 (c) 90 (c) 103 (b) 116 (d) 129 (a)
13 (a) 26 (a) 39 (c) 52 (b) 65 (c) 78 (d) 91 (a) 104 (b) 117 (b) 130 (b)
Exercise 2 : Exemplar & Past Year MCQs
1 (b) 6 (a) 11 (a) 16 (b) 21 (c) 26 (d) 31 (d) 36 (d) 41 (c) 46 (c)
2 (a) 7 (b) 12 (b) 17 (a) 22 (a) 27 (b) 32 (b) 37 (b) 42 (a) 47 (c)
3 (c) 8 (a) 13 (b) 18 (d) 23 (c) 28 (b) 33 (a) 38 (a) 43 (c)
4 (b) 9 (c) 14 (a) 19 (d) 24 (d) 29 (a) 34 (b) 39 (a) 44 (b)
5 (a) 10 (c) 15 (b) 20 (d) 25 (a) 30 (c) 35 (d) 40 (c) 45 (b)
Exercise 3 : Try If You Can
1 (b) 3 (d) 5 (b) 7 (a) 9 (a) 11 (b) 13 (b) 15 (c) 17 (a) 19 (b)
2 (a) 4 (d) 6 (d) 8 (a) 10 (a) 12 (a) 14 (d) 16 (b) 18 (a) 20 (c)
19Chapter
MOVING CHARGES AND
MAGNETISM

Trend
Analysis of NEET and AIIMS (Year 2010-2018)
5

4
Number of Questions

3
AIPMT/NEET
2 AIIMS

0
2010 2011 2012 2013 2014 2015 2016 2017 2018
Year

Trend
Analysis of JEE Main and BITSAT (Year 2010-2018)
5

4
Number of Questions

2
JEE Main
BITSAT
1

0
2010 2011 2012 2013 2014 2015 2016 2017 2018
Year

Chapter Utility Score (CUS)


Exam Weightage Important Concepts Difficulty Level CUS (Out of 10)
NEET 4 Biot-Savart’s Law,
AIIMS 3 Ampere’s Circuital Law 4.5/5 9/10
JEE Main 3 & Moving Coil Galvanometer
BITSAT 4
EBD_7418
268 PHYSICS
MOVING CHARGES AND MAGNETISM 269
EBD_7418
270 PHYSICS

Exercise 1 : Topic-wise MCQs

× ® × × × × ®
Topic 1: Motion of Charged Particle in Magnetic Field V + V
× × × × × +
®
1. A particle of mass m and charge q enters a magnetic field V
B perpendicularly with a velocity v. The radius of the × × × ×
(c) (d)
+
circular path described by it will be × +× × × ×
®
(a) Bq/mv (b) mq/Bv (c) mB/qv (d) mv/Bq × × V× × ×
2. An electron having a charge e moves with a velocity v in × × × × ×
X-direction. An electric field acts on it in Y-direction? The
10. A charged particle enters in a magnetic field in a direction
force on the electron acts in
(a) positive direction of Y-axis perpendicular to the magnetic field. Which of the following
(b) negative direction of Y-axis graphs show the correct variation of kinetic energy of the
(c) positive direction of Z-axis particle with time t?
(d) negative direction of Z-axis
(a) (b)
3. An electric charge in uniform motion produces
(a) an electric field only K.E. K.E.

(b) a magnetic field only


(c) both electric and magnetic fields
(d) no such field at all t t
4. A charged particle moves through a magnetic field in a
direction perpendicular to it. Then the (c) (d)
(a) velocity remains unchanged K.E. K.E.
(b) speed of the particle remains unchanged
(c) direction of the particle remains unchanged
(d) acceleration remains unchanged
5. A charged particle of mass m and charge q travels on a t t
circular path of radius r that is perpendicular to a magnetic 11. The work done by a magnetic field, on a moving charge is
field B. The time taken by the particle to complete one r r
(a) zero because F acts parallel to v
revolution is r r
(b) positive because F acts perpendicular to v
2pq 2 B 2pmq 2pm 2pqB r r
(a) (b) (c) (d) (c) zero because F acts perpendicular to v
m B qB m r r
6. In cyclotron the gyro-radius is (d) negative because F acts parallel to v
(a) proportional to momentum 12. Lorentz force is
(b) proportional to energy (a) electrostatic force acting on a charged particle.
(c) inversely proportional to momentum (b) magnetic force acting on a moving charged particle.
(d) inversely proportional to energy (c) the vector sum of electrostatic and magnetic force acting
on a moving charged particle.
7. A charge q is moving with a velocity v parallel to a magnetic
(d) the vector sum of gravitational and magnetic force
field B. Force on the charge due to magnetic field is
acting on a moving charged particle.
(a) q v B (b) q B/v (c) zero (d) B v/q r
13. An electric charge +q moves with velocity v = 3iˆ + 4 ˆj + kˆ
8. Cyclotron is used to accelerate ur
(a) electrons (b) neutrons in an electromagnetic field given by E = 3i$ + $j + 2k$ and
(c) a particles (d) negative ions ur
9. A positively charged particle enters in a uniform magnetic B = iˆ + ˆj - 3kˆ The y-component of the force experienced
field with velocity perpendicular to the magnetic field. Which by + q is :
of the following figures shows the correct motion of charged (a) 11 q (b) 5 q (c) 3 q (d) 2 q
14. A beam of electrons is moving with constant velocity in a
particle?
region having simultaneous perpendicular electric and
® ®
V + × ×
+
× V × magnetic fields of strength 20 Vm–1 and 0.5 T respectively
®
× × × × at right angles to the direction of motion of the electrons.
V ® × × × Then the velocity of electrons must be
(a) (b) V
+ 1
+
× +× × + × (a) 8 m/s (b) 20 m/s (c) 40 m/s (d) m/s
® × × ® × 40
V V
× × × ×
MOVING CHARGES AND MAGNETISM 271

r
15. A charged particle moves with velocity v = aiˆ + djˆ in a 21. A proton moving with a constant velocity passes through a
r region of space without any change in its velocity. If E and
magnetic field B = Aiˆ + Djˆ . The force acting on the particle
B represent the electric and magnetic fields respectively,
has magnitude F. Then,
this region of space may not have
(a) F = 0, if aD = dA
(a) E = 0, B = 0 (b) E = 0, B ¹ 0
(b) F = 0, if aD = –dA
(c) E ¹ 0, B = 0 (d) E ¹ 0, B ¹ 0
(c) F = 0, i f aA = – dD
22. A charged particle enters into a magnetic field with a velocity
(d) F µ (a2 + b2)1/2 × (A2 + D2)1/2
vector making an angle of 30º with respect to the direction
16. If a particle of charge 10–12 coulomb moving along the
of magnetic field. The path of the particle is
xˆ - direction with a velocity 105 m/s experiences a force of (a) circular (b) helical
10–10 newton in yˆ - direction due to magnetic field, then (c) elliptical (d) straight line
the minimum magnetic field is 23. If an electron and a proton having same momenta enter
(a) 6.25 × 103 Tesla in zˆ - direction perpendicular to a magnetic field, then
(b) 10–15 Tesla in zˆ - direction (a) curved path of electron and proton will be same
(ignoring the sense of revolution)
(c) 6.25 × 10 –3 Tesla in zˆ - direction
(b) they will move undeflected
(d) 10–3 Tesla in zˆ - direction ur (c) curved path of electron is more curved than that of the
ˆ ˆ
ur field E = (2i - 3j) N / C and
17. A certain region has an electric proton
ˆ ˆ
a uniform magnetic field B = (5i + 3j + 4k) ˆ T . The force
(d) path of proton is more curved
experienced by a charge 1C moving with velocity (iˆ + 2ˆj) 24. In a region, steady and uniform electric and magnetic fields
ms–1 is are present. These two fields are parallel to each other. A
(a) (10iˆ - 7ˆj - 7k)
ˆ (b) (10iˆ + 7ˆj + 7k)
ˆ charged particle is released from rest in this region. The
path of the particle will be a
(c) (-10iˆ + 7ˆj + 7k)
ˆ (d) (10iˆ + 7ˆj - 7k)
ˆ
(a) helix (b) straight line
18. A proton and an a-particle enter a uniform magnetic field (c) ellipse (d) circle
perpendicularly with the same speed. If proton takes 25 m 25. If an electron describes half a revolution in a circle of radius
second to make 5 revolutions, then the time period for the r in a magnetic field B, the energy acquired by it is
a-particle would be
(a) 50 m sec (b) 25 m sec 1 1
(a) zero (b) mv 2 (c) mv 2 (d) pr × Bev
(c) 10 m sec (d) 5 m sec 2 4
19. The figure shows a thin metalic rod × × × × × × 26. A 10eV electron is circulating in a plane at right angles to a
whose one end is pivoted at point × × × × × × uniform field at a magnetic induction 10–4 Wb/m2 (= 1.0
O. The rod rotates about the end O × × × × × × gauss). The orbital radius of the electron is
in a plane perpendicular to the × × × O× × × (a) 12cm (b) 16cm (c) 11 cm (d) 18 cm
uniform magnetic field with angular × × × × × × 27. An electron (mass = 9 × 10–31 kg, charge = 1.6 × 10–19 C)
frequency w in clockwise direction. × × × × × × moving with a velocity of 106 m/s enters a magnetic field. If it
Which of the following is correct? describes a circle of radius 0.1m, then strength of magnetic
(a) The free electrons of the rod move towards the outer field must be
end (a) 4.5 × 10–5 T (b) 1.4 × 10–5 T
(c) 5.5 × 10 T–5 (d) 2.6 × 10–5 T
(b) The free electrons of the rod move towards the pivoted
end. 28. A charged particle with charge q enters a region of constant,
(c) The free electrons of the rod move towards the ur ur
uniform and mutually orthogonal fields E and B with a
mid-point of the rod. r ur ur
(d) The free electrons of the rod do not move towards any velocity v perpendicular to both E and B , and comes out
end of the rod as rotation of rod has no effect on motion r
without any change in magnitude or direction of v . Then
of free electrons. r ur ur 2 r ur ur 2
20. The figure shows a closed loop bent in the form of a semi- (a) v = B ´ E / E (b) v = E ´ B / B
r ur ur 2 r ur ur
circle. One bead having charge +q slides from A to B along (c) v = B ´ E / B (d) v = E ´ B / E 2
the diameter in uniform motion and other bead having the 29. A proton and a deuterium nucleus having certain kinetic
same charge slides along the arc from A to B in uniform
energies enter in a uniform magnetic field with same
circular motion. Both take some time to travel from A to B.
component of velocity in the direction of magnetic field.
When both the beads are at the mid-point of their journey,
then the forces exerted by lower bead and upper bead are Which of the following is correct?
respectively (a) Proton has greater pitch of helical motion.
(b) Deuterium nucleus has greater pitch of helical motion.
(a) gravitational and magnetic A B
(c) Both particles have same pitch of helical motion.
(b) magnetic and electric
(d) Which particle has greater pitch depends on the fact
(c) electric and gravitational
that which particle has greater component of velocity
(d) gravitational and electric.
perpendicular to magnetic field.
EBD_7418
272 PHYSICS

× × × ×
30. The figure shows a spring-block 37. Magnetic field at the centre of a circular coil of radius r,
system executing SHM in a × × × × through which a current I flows is
uniform magnetic field. The block × × × ×
(a) directly proportional to r
slides on the frictionless surface × × × × (b) inverseley proportional to I
of a weighing machine. The block is having charge +q on it. (c) directly proportional to I
Assume that the block is so heavy that any force exerted by (d) directly proprotional to I2
magnetic field cannot lift it. Select the correct option from 38. The magnetic field B at a point on one end of a solenoid
the following. having n turns per metre length and carrying a current of
(a) When the block moves to right the machine shows i ampere is given by
more reading and when to left, less reading. m0ni 1
(b) When the block moves to right the machine shows (a) (b) m ni (c) 4pm0ni (d) ni
e 2 0
less reading and when to left, more reading.
(c) The machine shows same reading which is less than 39. A helium nucleus makes a full rotation in a circle of radius
the actual weight of the block in both cases. 0.8 meter in 2 sec. The value of the magnetic field induction
(d) The machine shows same reading which is more than B in tesla at the centre of circle will be
the actual weight of the block in both cases. (a) 2 × 10–19 m0 (b) 10–19 /m0
31. What is cyclotron frequency of an electron with an energy (c) 10–19 m0 (d) 2 × 10–20/m0
of 100 e V in the magnetic field of 1 × 10–4 weber / m2 if its 40. Which one of the following is the correct expression for
velocity is perpendicular to magnetic field? magnetic field on the axis of a circular current loop with x-
(a) 0.7 MHz (b) 2.8 MHz (c) 1.4 MHz (d) 2.1 MHz axis as its axis?
32. A charged particle with velocity 2 × 103 m/s passes
r m 0 IR 2 × iˆ r m0 IR 2
undeflected through electric and magnetic field. Magnetic (a) B = (b) B = iˆ
field is 1.5 tesla. The electric field intensity would be 2( x + R )
2 2 2
2 ( x2 + R2 )
32

(a) 2 × 103 N/C (b) 1.5 × 103 N/C r m0 IR r m 0 IR 3 2 ˆ


3
(c) 3 × 10 N/C (d) 4/3 × 10–3 N/C (c) B = iˆ (d) B = i
2 ( x2 + R2 ) 2 ( x2 + R2 )
32 2

Topic 2: Biot-Savart’s Law and Ampere’s Circuital Law 41. If a long hollow copper pipe carries a direct current, the
magnetic field associated with the current will be
33. Biot-Savart law indicates that the moving electron velocity
(V) produce a magnetic field B such that (a) only inside the pipe
(a) B | | V (b) only outside the pipe
(c) neither inside nor outside the pipe
(b) B ^ V
(d) both inside and outside the pipe
(c) it obeys inverse cube law
42. Energy in a current carrying coil is stored in the form of
(d) it is along the line joining electron and point of
(a) electric field (b) magnetic field
observation
(c) dielectric strength (d) heat ur
34. Ampere’s circuital law states that
43. The correct plot of the magnitude of magnetic field B vs
(a) the surface integral of magnetic field over the open
surface is equal to µ0 times the total current passing distance r from centre of the wire is, if the radius of wire
through the surface. is R
B
(b) the surface integral of magnetic field over the open
surface is equal to µ0 times the total current passing
near the surface. (a) (b)
(c) the line integral of magnetic field along the boundary
of the open surface is equal to µ0 times the total current r r
0 R R
passing near the surface.
B B
(d) the line integral of magnetic field along the boundary
of the open surface is equal to µ0 times the total current
passing through the surface. (c) (d)
35. The magnetic field around a long straight current carrying
wire is r r
0 R 0 R
(a) spherical symmetry (b) cylindrical symmetry
44. The figure shows a closely wound A D
(c) cubical symmetry (d) unsymmetrical
coil on a square core of inside edge
36. If a copper rod carries a direct current, the magnetic field
length l. The no. of turns per unit
associated with the current will be
length of the coil is n. Each turn
(a) only inside the rod
carries current i into the plane of
(b) only outside the rod
paper and out of the plane of
(c) both inside and outside the rod B i i C
papers. ABCD is an Amperian loop
(d) neither inside nor outside the rod
MOVING CHARGES AND MAGNETISM 273

enclosing an open surface in the plane of the paper. The (c) the magnetic field is different at different points inside
current enclosed by the loop is the pipe
(a) 4 ni (b) 4 nli (c) 8 nli (d) zero (d) the magnetic field at any point inside the pipe is zero
45. Which of the following statements is/are correct? 51. The magnetic field due to a current carrying circular loop of
I. The magnetic field in the open space inside the radius 3 cm at a point on the axis at a distance of 4 cm from
toroid is constant. the centre is 54 mT. What will be its value at the centre of
II. The magnetic field in the open space exterior to the loop ?
toroid is constant.
(a) 125 mT (b) 150 mT (c) 250 mT (d) 75 mT
III. The magnetic field inside the core of toroid is
constant. 52. A long straight wire in the horizontal plane carries a
(a) I and II (b) II and III current of 75 A in north of south direction, magnitude and
(c) III only (d) I only direction of field B at a point 3 m east of the wire is
46. A current of I ampere flows in a wire (a) 4 × 10–6 T, vertical up
I
forming a circular arc of radius r metres (b) 5 × 10–6 T, vertical down
q
subtending an angle q at the centre (c) 5 × 10–6 T, vertical up
as shown. The magnetic field at the O (d) 4 × 10–6 T, vertical down
53. Two concentric coils each of radius equal to 2 p cm are placed
centre O in tesla is
at right angles to each other. 3 ampere and 4 ampere are the
m0 Iq m0 Iq m0 Iq m0 Iq currents flowing in each coil respectively. The magnetic
(a) (b) (c) (d)
4 pr 2 pr 2r 4r induction in weber/m2 at the centre of the coils will be
47. An element of 0.05 $i m is placed at the origin as shown (m0 = 4p ´ 10-7 Wb / A.m)
in figure which carries a large current of 10 A. distance of (a) 10–5 (b) 12×10–5 (c) 7 × 10–5 (d) 5 × 10–5
1 m in perpendicular direction. The value of magnetic field 54. A coil of one turn is made of a wire of certain length and
is then from the same length a coil of two turns is made. If the
(a) 4.5 × 10–8 T P ` same current is passed in both the cases, then the ratio of
the magnetic inductions at their centres will be
(b) 5.5 × 10–8 T (a) 2 : 1 (b) 1.4 (c) 4 : 1 (d) 1 : 2
(c) 5.0 × 10–8 T 55. A circular coil of wire consisting of 100 turns each of
radius 9 cm carries a current of 0.4 A. The magnitude of
(d) 7.5 × 10–8 T Dx 0.05 $i m
manetic field at the centre of the coil is
48. Assertion : Ampere¢s law used for the (a) 2.4 × 10–4 T (b) 3.5 × 10–4 T
closed loop shown in figure is written as (c) 2.79 × 10 T –4 (d) 3 × 10–4 T
r r
Ñò Bg d l = m 0 (i1 - i2 ). Right side of it does not 56. A solenoid of length 1.5 m and 4 cm diameter possesses 10
include i3, because it produces no magnetic turns per cm. A current of 5A is flowing through it, the
field at the loop. magnetic induction at axis inside the solenoid is
Reason : The line integral of magnetic field produced by i3 (m0 = 4p × 10–7 weber amp–1m–1)
over the close loop is zero.
(a) 4p × 10–5 gauss (b) 2p × 10–5 gauss
(a) Assertion is correct, reason is correct; reason is a –5
correct explanation for assertion. (c) 4p × 10 tesla (d) 2p × 10–5 tesla
(b) Assertion is correct, reason is correct; reason is not a 57. A long solenoid is formed by winding 20 turns/cm. The
correct explanation for assertion current necessary to produce a magnetic field of 20
(c) Assertion is correct, reason is incorrect millitesla inside the solenoid will be approximately
(d) Assertion is incorrect, reason is correct. m0
49. Assertion : If the current in a solenoid is reversed in direction
( = 10–7 tesla – metre/ampere)
4p
while keeping the same magnitude, the magnetic field energy (a) 8.0 A (b) 4.0 A (c) 2.0 A (d) 1.0 A
stored in the solenoid decreases. 58. A solenoid of length 0.6 m has a radius of 2 cm and is
Reason : Magnetic field energy density is proportional to made up of 600 turns If it carries a current of 4 A, then
square of current. the magnitude of the magnetic field inside the solenoid is
(a) Assertion is correct, reason is correct; reason is a (a) 6.024 × 10–3 T (b) 8.024 × 10–3 T
correct explanation for assertion. (c) 5.024 × 10–3 T (d) 7.024 × 10–3 T
59. A long solenoid carrying a current produces a magnetic
(b) Assertion is correct, reason is correct; reason is not a
field B along its axis. If the current is double and the number
correct explanation for assertion of turns per cm is halved, the new value of the magnetic
(c) Assertion is correct, reason is incorrect field is
(d) Assertion is incorrect, reason is correct. (a) 4 B (b) B/2 (c) B (d) 2 B
50. A current I flows along the length of an infinitely long, 60. If we triple the radius of a coil keeping the current through it
straight, thin walled pipe. Then unchanged, then the magnetic field at any point at a large
(a) the magnetic field at all points inside the pipe is the distance from the centre becomes approximately how much
same, but not zero times ?
(b) the magnetic field is zero only on the axis of the pipe (a) 9 (b) 8 (c) 4 (d) 3
EBD_7418
274 PHYSICS

61. The figure shows n (n being an even number) wires placed 67. Axis of a solid cylinder of infinite length and radius R lies
along the surface of a cylinder of radius r. Each wire carries along y-axis, it carries a uniformly distributed current i along
current i in the same direction. The net magnetic field on the æR Rö
axis of the cylinder is +y direction. Magnetic field at a point ç , y, ÷ is
è2 2ø
m 0i ˆ ˆ m 0i ˆ ˆ
(a) (i - k) (b) ( j - k)
4pR 2pR
m0 i m 0i ˆ ˆ
(c) ĵ (d) (i + k)
m 0 ni m 0 ni 4 pR 4 pR
(a) µ0 ni (b) (c) zero (d) 68. The magnetic induction at a point P which is at a distance of
2 pr 4 pr
62. The figure shows a system of infinite concentric circular 4 cm from a long current carrying wire is 10–3 T. The field of
current loops having radii R1, R2, R3 ® Rn. The loops carry induction at a distance 12 cm from the current will be
net current i alternately in clockwise and anticlockwise (a) 3.33 × 10–4 T (b) 1.11 × 10–4 T
(c) 3 × 10 T –3 (d) 9 × 10–3 T
direction. The magnitude of net magnetic field of the centre
of the loops is 69. Two concentric circular coils of ten turns each are situated
in the same plane. Their radii are 20 and 40 cm and they carry
m0i é 1 1 1 1 ù respectively 0.2 and 0.4 ampere current in opposite direction.
(a) ê + + + ...ú
2 ë R1 R2 R3 R4 û The magnetic field in weber/m2 at the centre is
m0 i é 1 1 1 1 ù (a) m0/80 (b) 7m0/80 (c) (5/4) m0 (d) zero
(b) - + - + ...ú
R1

R3
2 êë R1 R2 R3 R4 û R4 æ aö
70. If the magnetic field at P can be written as K tan ç ÷ , the
m0 i é 1 1 1 1 ù è 2ø
(c) 4 p ê R + R + R + R + ...ú
R5
ë 1 û K is
2 3 4 i
m0 i é 1 1 1 1 ù µ0 I µ0 I
(d) 4 p ê R - R + R - R + ...ú (a)
4 pd
(b)
2 pd P a
ë 1 2 3 4 û a
63. A current loop consists of two identical semicircular parts each µ0 I 2µ0 I d
of radius R, one lying in the x-y plane and the other in x-z plane. (c) (d)
pd pd
If the current in the loop is i., the resultant magnetic field due to
71. Two very long, straight wires carrying,
the two semicircular parts at their common centre is
m0 i m 0i currents as shown in Fig. Find location
m 0i m 0i 10.0 A
(a) (b) (c) (d) where the net magnetic field is zero.
2R 2 2R 2R 4R
64. Two long straight wires are set parallel to each
other. Each carries a current i in the same (a) y = 2x (b) y = x 10.0 A
direction and the separation between them is (c) y = – x (d) y = – (x/2)
2r. The intensity of the magnetic field midway 72. Charge q is uniformly spread on a thin ring of radius R. The
between them is ring rotates about its axis with a uniform frequency f Hz.
(a) m0i/r (b) 4m0i/r (c) zero (d) m0i/4r The magnitude of magnetic induction at the centre of the
65. Assertion : Figure shows a current ring is
carrying circular loop. The magnetic m 0 qf m0 q m0 q m 0 qf
(a) (b) (c) 2pf R (d)
field at the centre of loop is zero. 2R 2f R 2 pR
Reason : Magnetic field at the centre 73. A current i ampere flows in a circular arc of wire which
m ni subtends an angle (3 p/2) radians at its centre, whose radius
of loop is given by B = 0 . is R. The magnetic field B at the centre is
2R
(a) Assertion is correct, reason is correct; reason is a (a) m0 i/R (b) m0 i/2R (c) 2 m0 i/R (d) 3 m0 i/8R
correct explanation for assertion. 74. A cell is connected between two points of a uniformly thick
(b) Assertion is correct, reason is correct; reason is not a circular conductor and i1 and i2 are the currents flowing in
correct explanation for assertion two parts of the circular conductor of radius a. The magnetic
(c) Assertion is correct, reason is incorrect field at the centre of the loop will be
(d) Assertion is incorrect, reason is correct. m0
66. A current i ampere flows along an infinitely long straight (a) zero (b) (I1 - I 2 )
4p
thin walled tube, then the magnetic field at any point inside
the tube is m0 m0
(c) ( I1 + I 2 ) (d) (I1 + I 2 )
μ 0 2i 2a a
(a) . tesla (b) zero 75. Two straight long conductors AOB and COD are
4π r
perpendicular to each other and carry currents I1 and I2.
2i The magnitude of the magnetic induction at a point P at a
(c) infinite (d) tesla
r distance a from the point O in a direction perpendicular to
the plane ABCD is
MOVING CHARGES AND MAGNETISM 275

m0 m0 82. The figure shows two long straight current


(a) (I1 + I 2 ) (b) (I1 - I 2 )
2pa 2pa carrying wire separated by a fixed distance d.
1 The magnitude of current, flowing in each wire
m0 2 2 2 m 0 I1 I 2
(c) ( I1 + I 2 ) (d) varies with time but the magnitude of current
2pa 2 p a I1 + I 2
in each wire is equal at all times.
76. Two long parallel wires P and Q are held perpendicular to
Which of the following graphs shows the correct variation
the plane of paper with distance of 5 m between them. If P
of force per unit length f between the two wires with
and Q carry current of 2.5 amp. and 5 amp. respectively in
current i?
the same direction, then the magnetic field at a point half-
way between the wires is
(a) m0/17 (b) 3m0 / 2p f f

(c) m0/2p (d) 3m0/2p (a) (b)


Topic 3: Force, Magnetic Moment and Torque on
i i
Current Carrying Conductor
77. Two thin, long, parallel wires, separated by a distance ‘d’
carry a current of ‘i’ A in the same direction. They will
f f
(a) repel each other with a force of m0i2/(2pd) (c) (d)
(b) attract each other with a force of m0i2/(2pd)
(c) repel each other with a force of m0i2/(2pd2)
i
(d) attract each other with a force of m0i2/(2pd2) i

78. The magnetic dipole moment of a current loop is 83. Three wires are situated at the same distance.
independent of A current of 1A, 2A, 3A flows through these
(a) magnetic field in which it is lying wires in the same direction. What is ratio of
(b) number of turns F1/F2, where F1 is force on 1 and F2 on 2?
(c) area of the loop 1A 2A 3A
(a) 7/8 (b) 1
(d) current in the loop (c) 9/8 (d) None of these
79. If m is magnetic moment and B is the magnetic field, then 84. A very long straight wire carries a current I. At the instant
the torque is given by ®
ur when a charge + Q at point P has velocity v , as shown,
ur ur |m| ur ur ur ur the force on the charge is
(a) m.B (b) ur (c) m ´ B (d) | m | . | B |
|B| y
80. The magnetic moment of a circular coil carrying current is
(a) directly proportional to the length of the wire in the coil Q
(b) inversely proportional to the length of the wire in the P®
v
coil I o x
(c) directly proportional to the square of the length of
the wire in the coil
(d) inversely proportional to the square of the length of (a) along oy (b) opposite to oy
the wire in the coil y
(c) along ox (d) opposite to ox
81. A square loop of side a and i 85. P, Q and R are long straight wires in air, carrying currents as
carrying current i as shown shown. The force on Q is directed
in the figure is placed in (a) to the left P Q R
gravity free space having x (b) to the right
magnetic field B = B0 k$ . Now (c) ^ to the plane of the diagram 20A 40A 60A
match following : (d) along the current in Q
Column I Column II
(A) Torque on loop (1) is zero 86. A current carrying loop is placed in a uniform magnetic
(B) Net force on loop (2) is in direction (– k$ ) field. The torque acting on it does not depend upon
(C) Potential energy of (3) has minimum magnitudes (a) shape of the loop (b) area ot the loop
loop (c) value of the current (d) magnetic field
(D) Magnetic moment of (4) has maximum ur
loop magnitudes 87. The magnetic force F on a current carrying conductor of
(a) (A) ® (2) ; (B) ® (1) ; (C) ® (3) ; (D) ® (4) ur
length I in an exteranal magnetic field B is given by
(b) (A) ® (1,2) ; (B) ® (1) ; (C) ® (4) ; (D) ® (2) ur r ur
(c) (A) ® (4) ; (B) ® (3) ; (C) ® (2) ; (D) ® (1) I´B I ´B r ur r ur
(a) r (b) (c) I ( I ´ B) (d) I 2 I ´ B
(d) (A) ® (2) ; (B) ® (1) ; (C) ® (4) ; (D) ® (3) I I
EBD_7418
276 PHYSICS

A
88. A circular coil ABCD carrying a current 97. A circular loop of area 0.02 m2 carrying a current of 10A, is
i
i is placed in a uniform magnetic field. held with its plane perpendicular to a magnetic field
If the magnetic force on the segment induction 0.2 T. The torque acting on the loop is
r D B
AB is F , the force on the remaining (a) 0.01 Nm (b) 0.001 Nm
segment BCDA is (c) zero (d) 0.8 Nm
r r r Cr 98. Through two parallel wires A and B, 10A and 2A of currents
(a) F (b) -F (c) 3F (d) -3F
are passed respectively in opposite directions. If the wire A
89. Figure shows a spiral coil of × × × × is infinitely long and the length of the wire B is 2m, then
A
negligible mass lying in a plane × × × × force on the conductor B, which is situated at 10 cm distance
perpendicular to uniform magnetic from A, will be
field. When the key is closed the × × × ×
(a) 8 × 10–7 N (b) 8 × 10–5 N
spiral coil will × × × ×
(c) 4 × 10–7 N (d) 4 × 10–5 N
(a) contract × × × × 99. An 8 cm long wire carrying a current of 10 A is placed
(b) expand × × × × inside a solenoid perpendicular to its axis. If the magnetic
()
(c) neither contract nor expand B field inside the solenoid is 0.3 T, then magnetic force on
(d) tend to rotate about an axis passing through A the wire is
and B. (a) 0.14 N (b) 0.24 N (c) 0.34 N (d) 0.44 N
90. A square current carrying loop is suspended in a uniform 100. The orbital speed of electron orbiting around a nucleus in a
magnetic field acting in the r plane of the loop. If the force on circular orbit of radius 50 pm is 2.2 × 106 ms–1. Then the
one arm of the loop is F , the net force on the remaining magnetic dipole moment of an electron is
(a) 1.6 × 10–19 Am2 (b) 5.3 × 10–21 Am2
three arms of the loop is –25 2
r r r r (c) 8.8 × 10 Am (d) 8.8 × 10–26 Am2
(a) 3 F (b) – F (c) – 3 F (d) F
91. A conducting circular loop of radius r carries a constant Topic 4: Galvanometer and its Conversion into
current i. It is placed in a uniform magnetic field B such that Ammeter and Voltmeter
B is perpendicular to the plane of the loop. The magnetic 101. The AC voltage across a resistance can be measured
force acting on the loop is using a :
(a) i r B (b) 2 p r i B (c) zero (d) p r i B (a) hot wire voltmeter
92. Two long wires are hanging freely. They are joined first in (b) moving coil galvanometer
parallel and then in series and then are connected with a (c) potential coil galvanometer
battery. In both cases which type of force acts between the (d) moving magnet galvanometer
two wires? 102. The deflection in a moving coil galvanometer is
(a) Attraction force when in parallel and repulsion force (a) directly proportional to the torsional constant
when in series (b) directly proportional to the number of turns in the
(b) Repulsion force when in parallel and attraction force coil
when in series (c) inversely proportional to the area of the coil
(c) Repulsion force in both cases (d) inversely proportional to the current flowing
(d) Attraction force in both cases 103. In a moving coil galvanometer, the deflection of the coil
93. A current carrying conductor placed in a magnetic field q is related to the elecrical current i by the relation
experiences maximum force when angle between current and (a) i µ tan q (b) i µ q
magnetic field is 2
(c) i µ q (d) i µ q
(a) 3 p/4 (b) p/2 (c) p/4 (d) zero
94. A current of 3 A is flowing in a linear conductor having a 104. A 50 W resistance is connected to a battery of 5V. A
length of 40 cm. The conductor is placed in a magnetic field galvanometer of resistance 100 W is to be used as an ammeter
of strength 500 gauss and makes an angle of 30º with the to measure current through the resistance, for this a
direction of the field. It experiences a force of magnitude resistance rs is connected to the galvanometer. Which of
(a) 3 × 10–4 N (b) 3 × 10–2 N the following connections should be employed if the
(c) 3 × 10 N 2 (d) 3 × 104 N measured current is within 1% of the current without the
95. A current of 10 A is flowing in a wire of length 1.5 m. A ammeter in the circuit?
force of 15 N acts on it when it is placed in a uniform (a) rs = 0.5 W in series with the galvanometer
magnetic field of 2 T. The angle between the magnetic (b) rs = 1 W in series with galvanometer
field and the direction of the current is (c) rs = 1W in parallel with galvanometer
(a) 30° (b) 45° (c) 60° (d) 90° (d) rs = 0.5 W in parallel with the galvanometer.
96. If a current is passed through a spring then the spring will 105. To know the resistance G of a galvanometer by half deflection
(a) expand (b) compress method, a battery of emf VE and resistance R is used to
(a) remains same (b) none of these deflect the galvanometer by angle q. If a shunt of resistance
S is needed to get half deflection then G, R and S related by
the equation:
MOVING CHARGES AND MAGNETISM 277

(a) S (R + G) = RG (b) 2S (R + G) = RG 114. A microammeter has a resistance of 100 W and full scale
(c) 2G = S (d) 2S = G range of 50 mA. It can be used as a voltmeter or as a higher
106. A moving coil galvanometer has N number of turns in a range ammeter provided a resistance is added to it. Pick the
coil of effective area A, it carries a current I. The magnetic correct range and resistance combination
field B is radial. The torque acting on the coil is (a) 50 V range with 10 kW resistance in series
(a) NA2B2I (b) NABI2 (c) N2ABI (d) NABI
107. The current sensitivity of a galvanometer is defined as (b) 10 V range with 200 kW resistance in series
(a) the current flowing through the galvanometer when a (c) 10 mA range with 1W resistance in parallel
unit voltage is applied across its terminals.
(d) 10 mA range with 0.1 W resistance in parallel
(b) current per unit deflection.
(c) deflection per unit current. 115. A galvanometer having a coil resistance of 60 W shows full scale
(d) dflection per unit current when a unit voltage is applied deflection when a current of 1.0 amp passes through it. It can be
across its terminals converted into an ammeter to read currents upto 5.0 amp by
108. The galvanometer cannot as such be used as an ammeter (a) putting in series a resistance of 15 W
to measure the value of current in a given circuit. The (b) putting in series a resistance of 240 W
following reasons are
(c) putting in parallel a resistance of 15 W
I. galvanometer gives full scale deflection for a small
current. (d) putting in parallel a resistance of 240 W
II. galvanometer has a large resistance. 116. The resistance of an ammeter is 13 W and its scale is
III. a galvanometer can give inaccurate values. graduated for a current upto 100 amps. After an additional
The correct reasons are: shunt has been connected to this ammeter it becomes
(a) I and II (b) II and III possible to measure currents upto 750 amperes by this meter.
(c) I and III (d) I, II and III The value of shunt-resistance is
109. Assertion: To convert a galvanometer into an ammeter a (a) 2 W (b) 0.2 W (c) 2 k W (d) 20 W
small resistance is connected in parallel with it. 117. A milli voltmeter of 25 milli volt range is to be converted into
Reason: The small resistance increases the combined an ammeter of 25 ampere range. The value (in ohm) of
resistance of the combination. necessary shunt will be
(a) Assertion is correct, reason is correct; reason is a (a) 0.001 (b) 0.01 (c) 1 (d) 0.05
correct explanation for assertion. 118. A moving coil galvanometer of resistance 100W is used as
(b) Assertion is correct, reason is correct; reason is not a an ammeter using a resistance 0.1W. The maximum deflection
correct explanation for assertion current in the galvanometer is 100mA. Find the minimum
(c) Assertion is correct, reason is incorrect current in the circuit so that the ammeter shows maximum
(d) Assertion is incorrect, reason is correct. deflection
110. A galvanometer having a resistance of 80 ohms is shunted (a) 100.1 mA (b) 1000.1 mA
by a wire of resistance 2 ohms. If the total current is 1amp., (c) 10.01 mA (d) 1.01 mA
the part of it passing through the shunt will be 119. A galvanometer of resistance 5 ohms gives a full scale
(a) 0.25 amp (b) 0.8 amp (c) 0.02 amp (d) 0.5 amp deflection for a potential difference of 10 mV. To convert the
111. A moving coil galvanometer has a resistance of 900 W. In galvanometer into a voltmeter giving a full scale deflection
order to send only 10% of the main current through this for a potential difference of 1V, the size of the resistance
galvanometer, the resistance of the required shunt is that must be attached to the voltmeter is
(a) 0.9 W (b) 100 W (c) 405 W (d) 90 W (a) 0.495 ohm (b) 49.5 ohm
112. In an ammeter 0.2% of main current passes through the (c) 495 ohm (d) 4950 ohm
galvanometer. If resistance of galvanometer is G, the 120. A galvanometer of resistance 100 W gives a full scale
resistance of ammeter will be : deflection for a current of 10–5 A. To convert it into a ammeter
capable of measuring upto 1 A, we should connect a
1 499 1 500
(a) G (b) G (c) G (d) G resistance of
499 500 500 499 (a) 1 W in parallel (b) 10–3 W in parallel
113. A galvanometer of resistance, G is shunted by a resistance S 5
(c) 10 W in series (d) 100 W in series
ohm. To keep the main current in the circuit unchanged, the
resistance to be put in series with the galvanometer is
S2 SG G2 G
(a) (b) (c) (d)
(S + G) (S + G) (S + G) (S + G)
EBD_7418
278 PHYSICS

Exercise 2 : Exemplar & Past Year MCQs

NCERT Exemplar MCQs m 0 æ I1 ö m0


(a) (b) (I1 + I2 )
1. Two charged particles traverse identical helical paths in a 2pd çè I2 ÷ø 2 pd
completely opposite sense in a uniform magnetic field m0 2 m0 2 2 1/ 2
B = B0 kˆ . (c) (I1 - I22 ) (d) (I1 ´ I 2 )
2 pd 2 pd
(a) They have equal z-components of momenta 7. If we double the radius of a coil keeping the current through
(b) They must have equal charges it unchanged, then the magnetic field at any point at a large
(c) They necessarily represent a particle, anti-particle pair distance from the centre becomes approximately
(d) The charge to mass ratio satisfy [AIIMS 2014, C]
(a) double (b) three times
æeö æeö
ç ÷ +ç ÷ = 0 (c) four times (d) one-fourth
è m ø1 è m ø2 8. A conductor lies along the z-axis at –1.5 £ z < 1.5 m and
2. Biot-Savart law indicates that the moving electrons (velocity carries a fixed current of 10.0 A in -aˆ z direction (see figure).
v) produce a magnetic field B such that For a field z
(a) B is perpendicular of r
B = 3.0 ´ 10-4 e -0.2 x aˆ y T,,
1.5
I
(b) B is parallel to v
(c) it obeys inverse cube law find the power required to
(d) it is along the line joining the electron and point of move the conductor at B y
constant speed to x = 2.0 2.0
observationt. m, y = 0 m in 5 × 10–3 s. x
3. A current carrying circular loop of radius R is placed in the Assume parallel motion –1.5
x-y plane with centre at the origin. Half of the loop with x > along the x-axis. [JEE Main 2014, S]
0 is now bent so that it now lies in the y-z plane.
(a) 1.57 W (b) 2.97 W (c) 14.85 W (d) 29.7 W
(a) The magnitude of magnetic moment now diminishes
9. An infinitely long wire
(b) The magnetic moment does not change carrying current I is along
(c) The magnitude of B at (0, 0, z), z > R increases Y axis such that its one I
(d) The magnitude of B at (0, 0, z), z >> R is unchanged end is at point A (0, b) A
4. An electron is projected with uniform velocity along the while the wire extends
axis of a current carrying long solenoid. Which of the upto + ¥.
following is true? The magnitude of magnetic (0,0) (a,0)
(a) The electron will be accelerated along the axis field strength at point (a, 0). [BITSAT 2014, A]
(b) The electron path will be circular about the axis
(c) The electron will experience a force at 45° to the axis and m0I æ b ö m0I æ b ö
hence execute a helical path (a) 4pa ç1 + 2 ÷ (b ) 4pa ç1 - 2 ÷
è a + b2 ø è a + b2 ø
(d) The electron will continue to move with uniform velocity
along the axis of the solenoid m0I æ b ö
5. In a cyclotron, a charged particle (c) 4pa ç1 - 2 ÷ (d) None of these
(a) undergoes acceleration all the time è a + b2 ø
(b) speeds up between the dees because of the magnetic field 10. A proton and an alpha particle both enter a region of uniform
(c) speeds up in a dees magnetic field B, moving at right angles to field B. If the
(d) slows down within a dee and speeds up between dees radius of circular orbits for both the particles is equal and
the kinetic energy acquired by proton is 1 MeV the energy
Past Year MCQs acquired by the alpha particle will be: [AIPMT 2015, A]
6. Two identical long conducting wires AOB and COD are (a) 0.5 MeV (b) 1.5 MeV
placed at right angle to each other, with one above other (c) 1 MeV (d) 4 MeV
such that ‘O’ is their common point for the two. The wires 11. A portion of a conductive wire is bent in the form of a
carry I1 and I2 currents respectively. Point ‘P’ is lying at semicircle of radius r as shown below in fig. At the centre of
distance ‘d’ from ‘O’ along a direction perpendicular to the semicircle, the magnetic induction will be [AIIMS 2015, C]
plane containing the wires. The magnetic field at the point
‘P’ will be : [AIPMT 2014, A] i
r
i
O
MOVING CHARGES AND MAGNETISM 279

(a) zero (b) infinite Y B C

μ0 π i μ0 πi 2m0 Ii m 0 Ii
(c) . gauss (d) . tesla (a) (b) 1 L
4π r 4π r 3p 2p I

12. A rectangular loop of sides 10 cm and 5 cm carrying a cur-


rent 1 of 12 A is placed in different orientations as shown in 2m0 IiL m 0 IiL X A D
(c) (d)
the figures below : [JEE Main 2015, C] 3p 2p
L/2 L
z z 18. Assertion : Free electrons always keep on moving in a
I
B B conductor even then no magnetic force act on them in
(A)
I I
(B) I magnetic field unless a current is passed through it.
y I y
I I Reason : The average velocity of free electron is zero.
x x I [AIIMS 2016, C]
z z (a) Assertion is correct, reason is correct; reason is a
B
correct explanation for assertion.
I B
I (b) Assertion is correct, reason is correct; reason is not a
(C) I y (D) I
I
y correct explanation for assertion
I I
x x I (c) Assertion is correct, reason is incorrect
If there is a uniform magnetic field of 0.3 T in the positive z (d) Assertion is incorrect, reason is correct.
direction, in which orientations the loop would be in (i) stable 19. Two identical wires A and B, each of length 'l', carry the
equilibrium and (ii) unstable equilibrium ? same current I. Wire A is bent into a circle of radius R and
(a) (B) and (D), respectively wire B is bent to form a square of side 'a'. If BA and BB are
(b) (B) and (C), respectively the values of magnetic field at the centres of the circle and
(c) (A) and (B), respectively
B
(d) (A) and (C), respectively square respectively, then the ratio A is:
13. A charged particle moving in a uniform magnetic field and BB
losses 4% of its kinetic energy. The radius of curvature of
its path changes by [BITSAT 2015, A] [JEE Main 2016, S]
(a) 2% (b) 4% (c) 10% (d) 12%
14. Two particles X and Y having equal charges, after being p2 p2 p2 p2
(a) (b) (c) (d)
accelerated through the same potential difference, enter a 16 8 2 8 16 2
region of uniform magnetic field and describe circular paths 20. A deuteron of kinetic energy 50 keV is describing a circular
of radii R1 and R2, respectively. The ratio of masses of X orbit of radius 0.5 metre in a plane perpendicular to the
and Y is [BITSAT 2015, C] magnetic field B. The kinetic energy of the proton that
(a) (R1/R2)1/2 (b) (R2/R1) (c) (R1/R2)2 (d) (R1/R2) describes a circular orbit of radius 0.5 metre in the same
15. A long straight wire along the Z-axis carries a current I in the plane with the same B is [BITSAT 2016, A]
r (a) 25 keV (b) 50 keV
negative Z-direction. The magnetic vector field B at a point
(c) 200 keV (d) 100 keV
having coordinates (x, y) in the Z = 0 plane is
[BITSAT 2015, C] 21. A 250-turn rectangular coil of length 2.1 cm and width 1.25
cm carries a current of 85 mA and subjected to magnetic field
(a)
m 0 I yiˆ - xjˆ ( (b)
)
m 0 I xiˆ + yjˆ ( ) of strength 0.85 T. Work done for rotating the coil by 180º

(
2p x 2 + y 2 )
2p x 2 + y 2 ( ) against the torque is
(a) 4.55 mJ (b) 2.3 mJ
[NEET 2017, A]
(c) 1.15 mJ (d) 9.1 mJ

(c)
(
m 0 I xjˆ - yiˆ ) (d)
(
m 0 I xiˆ - yjˆ ) 22. An arrangement of three parallel straight wires placed

(
2p x 2 + y 2 ) (
2p x 2 + y 2 )
perpendicular to plane of paper carrying same current 'I
along the same direction is shown in fig. Magnitude of force
16. A long straight wire of radius a carries a steady current I. per unit length on the middle wire 'B' is given by
The current is uniformly distributed over its cross-section. [NEET 2017, C]
The ratio of the magnetic fields B and B', at radial distances B d C

a 2m 0 i 2 2m0 i 2 90°
and 2a respectively, from the axis of the wire is : (a) (b)
2 pd pd d
[NEET 2016, C] m0 i 2 m0i2
(a) 1/4 (b) 1/2 (c) 1 (d) 4 (c) (d)
2pd 2 pd A
17. A square loop ABCD carrying a current i, is placed near and 23. A coil of circular cross-section having 1000 turns and 4 cm2
coplanar with a long straight conductor XY carrying a face area is placed with its axis parallel to a magnetic field
current I, the net force on the loop will be :[NEET 2016, C] which decreases by 10–2 Wb m–2 in 0.01 s. The e.m.f.
induced in the coil is: [AIIMS 2017, A]
EBD_7418
280 PHYSICS

(a) 400 mV (b) 200 mV 28. Assertion: The frequency of circular motion of a charged
(c) 4 mV (d) 0.4 mV particle in cyclotron is independent of the mass of the
24. A charged particle moves through a magnetic field particle.
perpendicular to its direction. Then [BITSAT 2017, C] Reason: Greater the mass of the particle less will be the
(a) kinetic energy changes but the momentum is constant frequency of the particle. [AIIMS 2018, C]
(b) the momentum changes but the kinetic energy is (a) Assertion is correct, reason is correct; reason is a
constant correct explanation for assertion.
(c) both momentum and kinetic energy of the particle are (b) Assertion is correct, reason is correct; reason is not a
not constant correct explanation for assertion
(d) both momentum and kinetic energy of the particle are (c) Assertion is correct, reason is incorrect
constant (d) Assertion is incorrect, reason is correct.
25. Two very long, straight, parallel wires carry steady currents 29. An electron, a proton and an alpha particle having the same
I and -I respectively. The distance between the wires is d. At kinetic energy are moving in circular orbits of radii re, rp, ra
a certain instant of time, a point charge q is at a point respectively in a uniform magnetic field B. The relation
equidistant from the two wires, in the plane of the wires. Its between re, rp, ra is : [JEE Main 2018, C]
instantaneous velocity v is perpendicular to this plane. The (a) re > rp = ra (b) re < rp = ra
magnitude of the force due to the magnetic field acting on (c) re < rp < ra (d) re < ra < rp
the charge at this instant is [BITSAT 2017, C] 30. The dipole moment of a circular loop carrying a current I, is
m 0 Iqv m 0 Iqv 2m0 Iqv m and the magnetic field at the centre of the loop is B1.
(a) (b) (c) (d) 0 When the dipole moment is doubled by keeping the current
2 pd pd pd
26. Current sensitivity of a moving coil galvanometer is 5 div/ constant, the magnetic field at the centre of the loop is B2.
mA and its voltage sensitivity (angular deflection per unit B1
The ratio is: [JEE Main 2018, A]
voltage applied) is 20 div/V. The resistance of the B2
galvanometer is [NEET 2018, A] 1
(a) 40 W (b) 25 W (c) 500 W (d) 250 W (a) 2 (b) 3 (c) 2 (d)
2
27. A metallic rod of mass per unit length 0.5 kg m–1 is lying 31. A charged particle enters in a uniform magnetic field with a
horizontally on a smooth inclined plane which makes an certain velocity. The power delivered to the particle by the
angle of 30°with the horizontal. The rod is not allowed to magnetic field depends on [BITSAT 2018, C]
slide down by flowing a current through it when a magnetic (a) force exerted by magnetic field and velocity of the
field of induction 0.25 T is acting on it in the vertical direction. particle.
The current flowing in the rod to keep it stationary is (b) angular speed w and radius r of the circular path.
[NEET 2018, A] (c) angular speed w and acceleration of the particle.
(a) 7.14 A (b) 5.98 A (c) 11.32 A (d) 14.76 A (d) None of these

Exercise 3 : Try If You Can


1. A thin circular wire carrying a current I has a magnetic (a) IL/4p (b) I2L2/4p (c) IL2/4p (d) IL2/8p
moment M. The shape of the wire is changed to a square 4. A long solenoid has 200 turns per cm and carries a current i.
and it carries the same current. It will have a magnetic
The magnetic field at its centre is 6.28 × 10–2 Weber/m2.
moment
Another long solenoid has 100 turns per cm and it carries a
4 4 p
(a) M (b) M (c) M (d) M i
p 2
p 4 current . The value of the magnetic field at its centre is
3
2. You are given a closed circuit with radii a and b as shown in (a) 1.05 × 10–2 weber/m2 (b) 1.05 × 10–5 weber/m2
fig carrying current i. The magnetic dipole moment of the (c) 1.05 × 10–3 weber/m2 (d) 1.05 × 10–4 weber/m2
circuit is
5. A steady current I flows down a hollow cylindrical tube of
(a) p (a2 + b2) i radius a and is uniformly distributed around the tube. Let r
1 be the distance from the axis of symmetry of the tube to a
(b) p (a2 + b2) i b
2 given point. What is the magnitiude of the magnetic field B
2
(c) p (a – b ) i2 a
i i at a point inside the tube?
1 (a) 0 (b) 2 I/r c
(d) p (a2 – b2) i
2 (c) 2 I r/a2c (d) 4 p (r – a) I/r2
3. A wire of length L metre carrying a current I ampere is bent r
6. A circular arc QTS is kept in an external magnetic field B0 as
in the form of a circle. Its magnitude of magnetic moment shown in figure. The arc carries a current I. The magnetic
will be
MOVING CHARGES AND MAGNETISM 281

field is directed normal and into the page. The force acting 11. A long straight wire, carrying P
on the arc is current I, is bent at its midpoint R 45°
T
(a) 2IB0 Rkˆ × B× × × × × × × × × × × × to form an angle of 45°.
0 I Magnetic field at point P,
(b) IB0 Rkˆ × × × × × × × × × × ×× ×
distant R from point of bending
× × × × × × × × × × × S× ×
(c) –2 IB0 Rkˆ Q 60°
× × × × × × × × × ×R × × × ( a - c)m0 I
ˆ R is equal to then find the value of (a + b + c)
(d) – IB0 Rk bpR
7. A current I flows through a thin wire shaped as regular [where a, b and c are integers]
polygon of n sides which can be inscribed in a circle of (a) 6 (b) 5 (c) 7 (d) 4
radius R. The magnetic field induction at the center of 12. The length of a solenoid is 0.4 m and the number turns in it
polygon due to one side of the polygon is is 500. A current of 3 amp, is flowing in it. In a small coil of
µ0 I æ pö µ0 I p radius 0.01 m and number of turns 10, a current of 0.4 amp. is
(a) çè tan ÷ø (b) tan flowing. The torque necessary to keep the axis of this coil
pR n 4pR n
perpendicular to the axis of solenoid will be
µ0 I æ pö µ0 I æ pö
(c) ç tan ÷ø (d) ç cos ÷ø (a) 5.92 × 10–6 N-m (b) 5.92 × 10–4 N-m
2 pR è n 2 pR è n (c) 5.92 × 10–6 dyne-cm (d) 5.92 × 10–4 dyne-cm
8. A coil having N turns is wound tightly in the form of a spiral 13. A charged particle moving along
with inner and outer radii a and b respectively. When a current +ve x-direction with a velocity v q
I passes through the coil, the magnetic field at the center is enters a region where there is a
(a)
m o NI
(b)
2m o NI uniform magnetic field B ( - kˆ ) , x = 0
0 d x=d
b a from x = 0 to x = d. The particle gets deflected at an angle q
m o NI b m0 IN a from its initial path. The specific charge of the particle is
(c) ln (d) ln
2(b - a ) a 2 (b - a) b v cos q v tan q
(a) (b)
9. Consider a hypothetic spherical body. The body is cut into Bd Bd
two parts about the diameter. One of hemispherical portion v v sin q
(c) (d)
has mass distribution m whie the other Bd Bd
portion has identical charge distribution q. 14. A parallel plate capacitor of area 60 cm2 and separation 3 mm
w is charged initially to 90 mC. If the medium between the plate
The body is rotated about the axis with
constant speed w. Then, the ratio of gets slightly conducting and the plate loses the charge
magnetic moment to angular momentum is initially at the rate of 2.5 × 10–8 C/s, then what is the magnetic
m q field between the plates?
q q
(a) (b) > (a) 2.5 × 10–8 T (b) 2.0 × 10–7 T
2m 2m
(c) 1.63 × 10–11 T (d) Zero
q 15. A particle of mass m and charge q is thrown from origin at
(c) < (d) cannot be calculated
2m t = 0 with velocity 2iˆ + 3 ˆj + 4kˆ units in a region with uniform
10. There exist uniform magnetic and electric fields of magni -
tudes 1 T and 1 V m–1, respectively, along positive y-axis. A pm
magnetic field 2iˆ units. After time t = , an electric field
charged particle of mass 1 kg and charge 1 C is having ® qB
velocity 1 m s–1 along x-axis and is at origin at t = 0. Then, E is switched on, such that particle moves on a straight
®
the coordinates of the particles at time ps will be line with constant speed. E may be
(a) (0, 1, 2) m (b) (0, – p2, –2) m
(c) (2, p2/2, 2)m (d) (0, p2/2, 2) m (a) 5iˆ - 10 ˆj units (b) -6iˆ - 9kˆ units
(c) -6kˆ + 8 ˆj units (d) 6iˆ + 8kˆ units
EBD_7418
282 PHYSICS

ANSWER KEYS
Exercise 1 : Topic-wise MCQs
1 (d) 13 (a) 25 (a) 37 (c) 49 (a) 61 (c) 73 (d) 85 (a) 97 (c) 109 (c)
2 (b) 14 (c) 26 (b) 38 (b) 50 (d) 62 (b) 74 (a) 86 (a) 98 (b) 110 (c)
3 (c) 15 (a) 27 (c) 39 (c) 51 (c) 63 (b) 75 (c) 87 (c) 99 (b) 111 (b)
4 (b) 16 (d) 28 (b) 40 (b) 52 (c) 64 (c) 76 (c) 88 (b) 100 (c) 112 (c)
5 (c) 17 (a) 29 (b) 41 (b) 53 (d) 65 (a) 77 (b) 89 (b) 101 (b) 113 (c)
6 (a) 18 (c) 30 (b) 42 (b) 54 (b) 66 (b) 78 (a) 90 (b) 102 (b) 114 (b)
7 (c) 19 (b) 31 (b) 43 (b) 55 (c) 67 (a) 79 (c) 91 (c) 103 (b) 115 (c)
8 (c) 20 (d) 32 (c) 44 (d) 56 (d) 68 (a) 80 (c) 92 (a) 104 (d) 116 (a)
9 (c) 21 (c) 33 (b) 45 (c) 57 (a) 69 (d) 81 (b) 93 (b) 105 (a) 117 (a)
10 (d) 22 (d) 34 (d) 46 (a) 58 (c) 70 (b) 82 (c) 94 (b) 106 (d) 118 (a)
11 (c) 23 (a) 35 (b) 47 (c) 59 (c) 71 (c) 83 (a) 95 (a) 107 (c) 119 (c)
12 (c) 24 (b) 36 (c) 48 (d) 60 (c) 72 (a) 84 (a) 96 (b) 108 (a) 120 (b)
Exercise 2 : Exemplar & Past Year MCQs
1 (d) 5 (a) 9 (b) 13 (a) 17 (a) 21 (d) 25 (d) 29 (b)
2 (a) 6 (d) 10 (c) 14 (c) 18 (b) 22 (c) 26 (d) 30 (c)
3 (a) 7 (c) 11 (d) 15 (a) 19 (b) 23 (a) 27 (c) 31 (d)
4 (d) 8 (b) 12 (a) 16 (b) 20 (d) 24 (b) 28 (d)
Exercise 3 : Try If You Can
1 (d) 3 (c) 5 (a) 7 (c) 9 (a) 11 (c) 13 (d) 15 (c)
2 (b) 4 (a) 6 (a) 8 (c) 10 (d) 12 (a) 14 (d)
20Chapter
MAGNETISM AND MATTER

Trend
Analysis of NEET and AIIMS (Year 2010-2018)
5

4
Number of Questions

3
AIPMT/NEET
2 AIIMS

0
2010 2011 2012 2013 2014 2015 2016 2017 2018
Year

Trend
Analysis of JEE Main and BITSAT (Year 2010-2018)
5

4
Number of Questions

3
JEE Main
BITSAT
2

0
2010 2011 2012 2013 2014 2015 2016 2017 2018
Year

Chapter Utility Score (CUS)


Exam Weightage Important Concepts Difficulty Level CUS (Out of 10)
NEET 3 Earth’s Magnetism
AIIMS 4 & Magnetic Materials 2.5/5 5.5/10
JEE Main 2
BITSAT 2
EBD_7418
284 PHYSICS
MAGNETISM AND MATTER 285
EBD_7418
286 PHYSICS

Exercise 1 : Topic-wise MCQs

(D) External field energy (4) –m0 M/4pr3


Topic 1: Magnet & Its Properties and Magnetic Moment
(a) (A) ® (3); (B) ® (4); (C) ® (2); (D) ® (1)
1. If a bar magnet of pole strength m and magnetic moment (b) (A) ® (3); (B) ® (4); (C) ® (3); (D) ® (1)
M is cut perpendicular to its axis in two equal halves then
(c) (A) ® (4); (B) ® (3); (C) ® (2); (D) ® (1)
its new pole strength m¢ and magnetic moment M¢ are
respectively (d) (A) ® (2); (B) ® (1); (C) ® (4); (D) ® (3)
11. A steel wire of length l has a magnetic moment M. It is bent
M
(a) m¢ = m and M¢ = M (b) m¢ = m and M¢ = in L-shape (Figure). The new magnetic moment is
2 (a) M
m M M
(c) m¢ = and M¢= 2M (d) m¢ = 2m and M¢ = (b)
2 2 l
2. On cutting a solenoid in half, the field lines remain ...A..., 2 2
emerging from one face of the solenoid and entering into M
(c)
the other face. Here, A refers to 2 l
(a) irregular (b) discontinuous (d) 2M 2
(c) continuous (d) alternate 12. Magnetic dipole moment is a vector quantity directed from
3. The magnetism of magnet is due to (a) south pole to north pole
(a) pressure of big magnet inside the earth (b) north pole to south pole
(b) earth (c) east to west
(c) cosmic rays
(d) west to east
(d) the spin motion of electron
4. For bar magnet effective length (Le) related with geometrical 13. A curve between magnetic moment and temperature of
length (Lg) as magnet is
6 5 M M
(a) Le = L g (b) Le = L g
5 6
(c) Le = Lg (d) Le = 2 Lg (a) (b)
5. A bar magnet of magnetic moment M, is placed in magnetic
field of induction B. The torque exerted on it is T T
r r r r r r r r O O
(a) M.B (b) - M.B (c) M ´ B (d) -B ´ M
6. The magnetic moment of a bar magnet is thus ...A... to the M M
magnetic moment of an equivalent solenoid that produces
the same magnetic field. Here, A refers to
(a) unequal (b) different (c) (d)
(c) equal (d) same
7. The pole strength of the magnet does not depend on T T
O O
(a) area of cross-section (b) nature of material 14. Magnetic lines of force due to a bar magnet do not intersect
(c) length of the magnet (d) both (a) and (b)
because
8. A bar magnet is cut into two equal halves by a plane parallel
to the magnetic axis. Of the following physical quantities (a) a point always has a single net magnetic field
the one which remains unchanged is (b) the lines have similar charges and so repel each other
(a) pole strength (c) the lines always diverge from a single force
(b) magnetic moment (d) None of these
(c) intensity of magnetisation 15. The magnetic lines of force inside a bar magnet
(d) None of these
9. Current I is flowing in a coil of area A and number of turns is (a) are from N-pole to S-pole of magnet
N, then magnetic moment of the coil in M equal to (b) do not exist
(c) depend upon the area of cross section of bar magnet
(a) NIA (b) NI/A (c) NI/ A (d) N2 AI (d) are from S-pole of magnet
10. Match the columns I and II. 16. A circular loop carrying a current is replaced by an equivalent
Column I Column II magnetic dipole. A point on the axis of the loop is in
(A) Axial field for a short dipole (1) – M.B
(a) end-on position (b) broadside-on position
(B) Equatorial field for a short dipole (2) M × B
(C) External field torque (3) m0 2M/4pr3 (c) both (d) None of these
MAGNETISM AND MATTER 287

17. Assertion : We cannot think of magnetic field configuration (for q = 180º) is


with three poles. (a) 4.32 mJ (b) 2.16 mJ
Reason : A bar magnet does not exert a torque on itself due (c) 8.6 mJ (d) None of these
to its own field. 28. A bar magnet of length ‘l’ and magnetic dipole moment ‘M’
(a) Assertion is correct, reason is correct; reason is a is bent in the form of an arc as shown in figure. The new
correct explanation for assertion. magnetic dipole moment will be
(b) Assertion is correct, reason is correct; reason is not a 3
correct explanation for assertion (a) M
p
(c) Assertion is correct, reason is incorrect 2
(d) Assertion is incorrect, reason is correct. (b) M
p
18. A bar magnet has a length 8 cm. The magnetic field at a M
point at a distance 3 cm from the centre in the broad side-on (c)
2
position is found to be 4×10–6 T. The pole strength of the (d) M
magnet is.
29. A short bar magnet, placed with its axis at 30° with an external
(a) 6 × 10–5 Am (b) 5 × 10–5 Am
–4
magnetic field of 0.16 T, experiences a torque of magnitude
(c) 2 × 10 Am (d) 3 × 10–4 Am 0.032 J. The magnetic moment of the bar magnet is
19. The work done in turning a magnet of magnetic moment M (in units of J/T)
by an angle of 90° from the meridian, is n times the (a) 4 (b) 0.2 (c) 0.5 (d) 0.4
corresponding work done to turn it through an angle of 30. A bar magnet having centre O has a length of 4 cm. Point P 1
60°. The value of n is given by is in the broad side-on and P2 is in the end side-on position
(a) 2 (b) 1 (c) 0.5 (d) 0.25 with OP1 = OP2 = 10 metres. The ratio of magnetic intensities
H at P1 and P2 is
20. The ratio of magnetic fields due to a smaller bar magnet in
(a) H1 : H2 = 16 : 100 (b) H1 : H2 = 1 : 2
the end on position to broad side on position is
(c) H1 : H2 = 2 : 1 (d) H1 : H2 = 100 : 16
(a) 1/4 (b) 1/2 (c) 1 (d) 2
21. Let r be the distance of a point on the axis of a bar magnet 31. The force between two short bar magnets with magnetic
from its centre. The magentic field at such a point is moments M1 and M2 whose centres are r metres apart is 8 N
proportional to when their axes are in same line. if the separation is increased
to 2 r, the force between them is reduced to
1 1
(a) (b) (a) 4 N (b) 2 N (c) 1 N (d) 0.5 N
r r2 32. A thin bar magnet of length 2 l and breadth 2 b pole strength m
1 and magnetic moment M is divided into four equal parts with
(c) 3 (d) None of these length and breadth of each part being half of original magnet.
r
22. A short bar magnet of magnetic moment 0.4J T–1 is placed in Then the pole strength of each part is
(a) m (b) m/2 (c) 2 m (d) m/4
a uniform magnetic field of 0.16 T. The magnet is in stable
33. Two points A and B are situated at a distance x and 2x
equilibrium when the potential energy is respectively from the nearer pole of a magnet 2 cm long. The
(a) 0.064 (b) zero (c) – 0.082 J (d) 0.064 J ratio of magnetic field at A and B is
23. The magnetic moment of a magnet is 0.1 amp × m2. It is (a) 4 : 1 exactly (b) 4 : 1 approximately
suspended in a magnetic field of intensity 3 × 10–4 weber/m2. (c) 8 : 1 approximately (d) 1 : 1 approximately
The couple acting upon it when deflected by 30º from the 34. The force between magnetic poles when their pole strength
magnetic field is and the distance between them both doubled is
(a) 1 × 10–5 N m (b) 1.5 × 10–5 N m (a) twice the previous value
–5
(c) 2 × 10 N m (d) 2.5 × 10–5 N m (b) half the previous value
24. Time periods of vibation of two bar magnets in sum and (c) four times the previous value
difference positions are 4 sec and 6 sec respectively. The (d) not change
ratio of their magnetic moments M1 / M2 is 35. If a magnetic dipole of moment M situated in the direction of
(a) 6 : 4 (b) 30 : 16 (c) 2.6 : 1 (d) 1. 5 : 1 a magnetic field B is rotated by 180°, then the amount of
25. A magnet of magnetic moment 20 C.G.S. units is freely work done is
suspended in a uniform magnetic field of intensity 0.3 MB
C.G.S. units. The amount of work done in deflecting it by (a) MB (b) 2MB (c) (d) MB
an angle of 30° in C.G.S. units is 2
(a) 6 (b) 3 3 (c) 3(2 - 3) (d) 3 36. The moment of a magnet (15 cm × 2 cm × 1 cm) is 1.2 A-m2.
26. A magnet of length 0.1 m and pole strength 10–4 A.m. What is its intensity of magnetisation?
is kept in a magnetic field of 30 Wb / m2 at an angle 30°. (a) 4 × 104 A m–1 (b) 2 × 104 A m–1
The couple acting on it is ........ × 10–4 Nm. (c) 104 A m–1 (d) None of these
(a) 7.5 (b) 3.0 (c) 1.5 (d) 6.0 37. A permanent magnet in the shape of a thin cylinder of
27. The magnetic dipole moment of a coil is 5.4 × 10–6 joule/ length 10 cm has magnetisation (M) = 106 A m–1. Its
tesla and it is lined up with an external magnetic field whose magnetization current IM is
strength is 0.80 T. Then the work done in rotating the coil
(a) 105 A (b) 106 A (c) 107 A (d) 108 A
EBD_7418
288 PHYSICS

38. Assume that each iron atom has a permanent magnetic dip are respectively
moment equal to 2 Bohr magnetons (1 Bohr magneton (a) zero, maximum (b) maximum, minimum
= 9.27×10 –24 A-m 2). The density of atoms in iron is (c) maximum, maximum (d) minimum, minimum
8.52 × 1028 atoms/m3. Find the maximum magnetic moment 49. The magnetic compass is not useful for navigation near the
magnetic poles, since
per unit volume. (a) R = 0 (b) V = 0 (c) H = 0 (d) q = 0º
(a) 1.5 × 102 A|m (b) 1.58 × 106 A|m 50. If m0 is absolute permeability of vacuum and mr is relative
5
(c) 1.2 × 10 A|m (d) 1.3 × 106 A|m magnetic permeability of another medium, then permeability
m of the medium is
Topic 2: Earth’s Magnetism, Magnetic (a) m0 mr (b) m0/mr (c) mr/m0 (d) 1/m0 mr
Materials and their Properties 51. Metals getting magnetised by orientation of atomic magnetic
moments in external magnetic field are called
39. The strength of the earth’s magnetic field is (a) diamagnetic (b) paramagnetic
(a) constant everywhere (c) ferromagnetic (d) antimagnetic
(b) zero everywhere 52. The magnetic susceptibility for diamagnetic materials is
(c) having very high value (a) small and negative (b) small and positive
(d) vary from place to place on the earth’s surface (c) large and positive (d) large and negative
40. Which of the following is responsible for the earth’s 53. Among which of the following the magnetic susceptibility
magnetic field? does not depend on the temperature?
(a) Convective currents in earth’s core. (a) Dia-magnetism (b) Para-magnetism
(b) Diversive current in earth’s core. (c) Ferro-magnetism (d) Ferrite
(c) Rotational motion of earth. 54. Curie temperature is the temperature above which
(d) Translational motion of earth. (a) a ferromagnetic material becomes paramagenetic
41. Which of the following is not correct about relative (b) a paramagnetic material becomes diamagnetic
magnetic permeability (mr)? (c) a ferromagnetic material becomes diamagnetic
(a) It is a dimensionsless pure ratio. (d) a paramagnetic meterial becomes ferromagnetic
(b) For vacuum medium its value is one. 55. A material is placed in a magnetic field and it is thrown out
(c) For ferromagnetic materials mr > > 1 of it. Then the material is
(d) For paramagnetic materials mr > 1. (a) paramagnetic (b) diamagnetic
42. The most appropriate magnetization M versus magnetising (c) ferromagnetic (d) non-magnetic
field H curve for a paramagnetic substance is 56. A temporary magnet is made of
C (a) cast iron (b) steel
(a) A M
(c) soft iron (d) stainless steel
+ A
(b) B 57. Identify the correctly matched pair
0 H (a) Diamagnetic - Gadolinium
(c) C – B (b) Soft ferromagnetic - Alnico
(d) D D (c) Hard ferromagnetic - Copper
43. The lines of force due to earth’s horizontal magnetic field are (d) Paramagnetic - Sodium
(a) parallel and straight (b) concentric circles 58. Relative permittivity and permeability of a material are er and µr,
(c) elliptical (d) curved lines respectively. Which of the following values of these quantities
44. The earth’s magnetic field always has a vertical component are allowed for a diamagnetic material?
except at the (a) e r = 1.5, m r = 0.5 (b) e r = 0.5, m r = 0.5
(a) magnetic equator (b) magnetic poles (c) e r = 1.5, m r = 1.5 (d) e r = 0.5, m r = 1.5
59. Magnetic permeability is maximum for
(c) geographic north pole (d) latitude 45º
(a) diamagnetic substance
45. At magnetic poles, the angle of dip is
(b) paramagnetic substance
(a) 45º (b) 30º (c) zero (d) 90º
(c) ferromagnetic substance
46. One can define ...A... of a place as the vertical plane which
(d) All of the above
passes through the imaginary line joining the magnetic
North and the South–poles. 60. When a piece of a ferromagnetic substance is put in a uniform
magnetic field, the flux density inside it is four times the flux
Here, A refers to
density away from the piece. The magnetic permeability of
(a) geographic meridian (b) magnetic meridian the material is
(c) magnetic declination (d) magnetic inclination (a) 1 (b) 2 (c) 3 (d) 4
47. The ratio of intensity of magnetisation and magnetising field 61. Materials suitable for permanent magnet, must have
is called which of the following properties?
(a) permeability (b) magnetic intensity (a) High retentivity, low coercivity and high permeability.
(c) magnetic intensity (d) magnetic susceptibility (b) Low retentivity, low coercivity and low permeability.
48. At the magnetic north pole of the earth, the value of the (c) Low retentivity, high coercivity and low permeability.
horizontal component of earth’s magnetic field and angle of (d) High retentivity, high coercivity and high permeability.
MAGNETISM AND MATTER 289

62. When the temperature of a magnetic material decreases, the 71. If a paramagnetic liquid is placed in a watch glass, resting
magnetization on the pole pieces, the liquid accumulates where the field is
(a) decreases in a diamagnetic material (a) zero (b) weak
(b) decreases in a paramagnetic material (c) strong (d) None of these
(c) decreases in a ferromagnetic material
72. Which of the following statements is/are correct?
(d) remains the same in a diamagnetic material
63. A ferromagnetic material is heated above its curie I. The Earth behaves as a magnet with the magnetic
temperature. Which one is a correct statement? field pointing approximately from the geographic
(a) Ferromagnetic domains are perfectly arranged South to the North.
(b) Ferromagnetic domains become random II. When a bar magnet is freely suspended, it points in
(c) Ferromagnetic domains are not influenced the North–South direction. The tip which points to
(d) Ferromagnetic material changes into diamagnetic the geographic North is called the North –pole and
material the tip which points to georaphic South is called the
64. The magnetic susceptibility is given by South–pole of magnet.
1 B M net M III. There is a repulsive force when North–poles (or
(a) c = (b) c = (c) c = (d) c = z
H H V H South–poles) of two magnets are brought close
65. According to Curie’s law, together. Conversely, there is an attractive force
1 between the North–pole of one magnet and the
(a) c µ ( T - Tc ) (b) c µ T - T
c South–pole of other.
1 IV. Iron, Cobalt, nickel etc. are diamagnetic substances
(c) c µ (d) c µ T (a) I and II (b) I, II and IV
T
66. Susceptibility of ferromagnetic substance is (c) III and IV (d) I, II and III
(a) > 1 (b) < 1 (c) 0 (d) 1 73. A short bar magnet is placed in the magnetic meridian of
67. If the susceptibility of dia, para and ferromagnetic materials the earth with north pole pointing north. Neutral points
are cd, cp, cf respectively, then are found at a distance of 30 cm from the magnet on the
East – West line, drawn through the middle point of the
(a) cd < cp < cf (b) cd < cf < cp
magnet. The magnetic moment of the magnet in Am2 is
(c) cf < cd < cp (d) cf < cp < cd
m
68. The magnetic susceptibility of a paramagnetic substances close to: (Given 0 = 10–7 in SI units and BH =Horizontal
at –73°C is 0.0060, then its value at –173°C will be 4p
(a) 0.0030 (b) 0.0120 (c) 0.0180 (d) 0.0045 component of earth’s magnetic field = 3.6 × 10–5 tesla)
69. Match the column I & II. (a) 14.6 (b) 19.4 (c) 9.7 (d) 4.9
Column I Column II 74. A bar magnet 8 cms long is placed in the magnetic merdian
(A) Horizontal component (1) BE sin q with the N-pole pointing towards geographical north. Two
BV netural points separated by a distance of 6 cms are obtained
(B) Vertical component (2) on the equatorial axis of the magnet. If horizontal component
BH
(C) tan q (3) BE cos q of earth’s field = 3.2 × 10–5 T, then pole strength of magnet is
(D) Tangent law (4) B = BH tan q (a) 5 ab-amp × cm (b) 10 ab-amp × cm
(a) A ® (3); B ® (2); C ® (1); D ® (4) (c) 2.5 ab-amp × cm (d) 20 ab-amp × cm
(b) A ® (3); B ® (1); C ® (2); D ® (4) 75. At a certain place, the angle of dip is 30º and the horizontal
(c) A ® (2); B ® (3); C ® (1); D ® (4) component of earth’s magnetic field is 0.50 oerested. The
(d) A ® (1); B ® (3); C ® (2); D ® (4) earth’s total magnetic field (in oerested) is
70. The graph between c and 1/T for paramagnetic material will 1 1
be represented by (a) 3 (b) 1 (c) (d)
3 2
Y Y 76. The B – H curve (i) and (ii) shown in fig associated with
(a) (i) diamagnetic and (ii) paramagnetic
(i)
substance (ii)
(b) (i) paramagnetic and (ii) ferromagnetic H
substance
(a) (b) (c) (i) soft iron and (ii) steel
(d) (i) steel and (ii) soft iron
X X
O 1/T O 1/T 77. Assertion : To protect any instrument from external magnetic
Y Y field, it is put inside an iron body.
Reason : Iron has high permeability.
(a) Assertion is correct, reason is correct; reason is a
(c) (d) correct explanation for assertion.
(b) Assertion is correct, reason is correct; reason is not a
X X
O 1/T O 1/T
EBD_7418
290 PHYSICS

correct explanation for assertion æ H ö


æH ö
(c) Assertion is correct, reason is incorrect (a) tan -1 ç 0 ÷ (b) tan -1 ç ÷
è H ø è H0 ø
(d) Assertion is incorrect, reason is correct.
78. Two magnets are held together and are allowed to oscillate
in the earth’s magnetic field with like poles together. 12 æ H ö æH ö
(c) cos ec -1 ç ÷ (d) cot -1 ç 0 ÷
oscillations per minute are made but for unlike poles together è H0 ø è H ø
only 4 oscillations per minute are executed. The ratio of
their magnetic moments is 90. The earth’s magnetic field lines resemble that of a dipole at
(a) 3 : 1 (b) 1 : 3 (c) 3 : 5 (d) 5 : 4 the centre of the earth. If the magnetic moment of this dipole
79. The horizontal component of the earth’s magnetic field is is close to 8 × 1022 Am2, the value of earth’s magnetic field
3.6 × 10–5 tesla where the dip angle is 60°. The magnitude of near the equator is close to (radius of the earth = 6.4 × 106 m)
the earth’s magnetic field is (a) 0.6 Gauss (b) 1.2 Gauss
(a) 2.8 × 10–4 tesla (b) 2.1 × 10–4 tesla
–5 (c) 1.8 Gauss (d) 0.32 Gauss
(c) 7.2 × 10 tesla (d) 3.6 × 10–5 tesla
80. –5
A torque of 10 Nm is required to hold a magnet at 90° with 91. Find the per cent increase in the magnetic field when the
the horizontal component H of the earth’s magnetic field. space within a current carrying toroid is filled with aluminium.
The torque to hold it at 30° will be [susceptibility of aluminium is 2.1 × 10–5]
(a) 5 × 10–6 Nm (b) data is insufficient (a) 2.1 × 10–3 (b) 1.1 × 10–5
1 (c) 8.1 × 10 –2 (d) 6.6 × 10–3
(c) ´ 10 -5 Nm (d) 5 3 ´ 10 -6 Nm
3 92. A domain in a ferromagnetic substance is in the form of a
81. The susceptibility of annealed iron at saturation is 5500. cube of side length 1 mm. If it contains 8 × 1010 atoms and
Find the permeability of annealed iron at saturation. each atomic dipole has a dipole moment of 9 × 10–24 A m2,
(a) 6.9 × 10–3 (b) 5.1 × 10–2 then the magnetization of the domain is
(c) 5 × 10 2 (d) 3.2 × 10–5
(a) 7.2 × 105 A m–1 (b) 7.2 × 103 A m–1
82. A magnetising field of 2 × 103 A m–1 produces a magnetic
flux density of 8p T in an iron rod. The relative (c) 7.2 × 109 A m–1 (d) 7.2 × 1012 A m–1
permeability of the rod will be 93. A bar magnet has coercivity 4 × 103 Am–1. It is desired to
(a) 102 (b) 1 (c) 104 (d) 103 demagnetise it by inserting it inside a solenoid 12 cm long
83. If relative permeability of iron is 2000. Its absolute and having 60 turns. The current that should be sent through
permeability in S.I. units is the solenoid is
(a) 8p × 10–4 (b) 8p × 10–3 (a) 2 A (b) 4 A (c) 6 A (d) 8 A
(c) 800/p (d) 8p × 109/p
84. The relative permeability of a medium is 0.075. What is its Topic 3: Magnetic Equipments
magnetic susceptibility?
(a) 0.925 (b) – 0.925 (c) 1.075 (d) –1.075 94. Magnetic moments of two bar magnets may be compared
85. At a temperatur of 30°C, the susceptibility of a ferromagnetic with the help of
material is found to be c. Its susceptibility at 333°C is (a) deflection magnetometer
(a) c (b) 0.5c (c) 2c (d) 11.1c (b) vibration magnetometer
86. A short magnet of length 4 cm is kept at a distance of 20 cm (c) both (a) and (b)
to the east of a compass box such that is axis is perpendicular
(d) None of these
to the magnetic meridian. If the deflection produced is 45°,
95. A dip circle is taken to geomagnetic equator. The needle is
find the pole strength (H = 30 Am–1)
allowed to move in a vertical plane perpendicular to the
(a) 17.7 Am (b) 44.2 Am (c) 27.7 Am (d) 37.7 Am
mangetic meridian. the needle will stay in
87. At a certain place, horizontal component is 3 times the (a) horizontal direction only
vertical component. The angle of dip at this place is (b) vertical direction only
(a) 0 (b) p/3 (c) p/6 (d) p/8 (c) any direction except vertical and horizontal
88. At a certain place, the angle of dip is 30º and the horizontal (d) any direction it is released
component of earth’s magnetic field is 0.50 oerested. The
96. Two short magnets with their axes horizontal and
earth’s total magnetic field (in oerested) is
perpendicular to the magnetic maridian are placed with
1 1
(a) (b) 1 (c) (d) their centres 40 cm east and 50 cm west of magnetic
3 3 2 needle. If the needle remains undeflected, the ratio of their
89. A current carrying coil is placed with its axis perpendicular magnetic moments M1 : M2 is
to N-S direction. Let horizontal component of earth’s
(a) 4 : 5 (b) 16 : 25 (c) 64 : 125 (d) 2 : 5
magnetic field be H0 and magnetic field inside the loop be H.
97. Tangent galvanometer is an instrument is used to measure
If a magnet is suspended inside the loop, it makes angle q
with H. Then q = (a) emf (b) magnetic field
(c) current (d) None of these
MAGNETISM AND MATTER 291

98. Two tangent galvanometers having coils of the same radius tan q '
are connected in series. A current flowing in them produces an angle of dip q'. Then is
tan q
deflections of 60º and 45º respectively. The ratio of the number 1 1 1
of turns in the coils is (a) (b) (c) (d) cos x
cos x sin x tan x
3 +1 3 +1 3 103. The magnetic needle of a tangent galvanometer is deflected
(a) 4/3 (b) (c) (d)
1 3 -1 1 at an angle 30° due to a magnet. The horizontal component
99. The period of oscillation of a magnet in a vibration of earth¢s magnetic field 0.34 × 10–4 T is along the plane of
magnetometer is 2 sec. The period of oscillation of a magnet the coil. The magnetic intensity is
whose magnetic moment is four times that of the first magnet
is (a) 1.96 × 10–4 T (b) 1.96 × 10–5 T
(c) 1.96 × 10 T 4 (d) 1.96 × 105 T
(a) 1 sec (b) 5 sec (c) 8 sec (d) 0.5 sec
100. Two short magnets have equal pole strengths but one is 104. A compass needle made of pure iron (with density 7900 kg/
twice as long as the other. The shorter magnet is placed 20 m3) has a length L of 3.0 cm, a width of 1.00 mm, and a
cms in tan A position from the compass needle. The longer thickness of 0.50 mm. The magnitude of the magnetic dipole
magnet must be placed on the other side of the magnetometer moment of an iron atom is MFe = 2.1 × 10–23 J/T. If the
for no deflection at a distance equal to magnetisation of the needle is equivalent to the alignment
(a) 20 cms (b) 20 (2)1/3 cms
2/3
of 10% of the atoms in the needle, what r
is the magnitude of
(c) 20 (2) cms (d) 20 (2)3/3 cms
the needle¢s magnetic dipole moment M ?
101. A compass needle placed at a distance r from a short magnet
in Tan A position shows a deflection of 60º. If the distance is (a) 2.7 × 10–1 J/T (b) 2.7 × 10–2 J/T
–3
(c) 2.7 × 10 J/T (d) 2.7 × 10–4 J/T
increased to r (3)1/3, then deflection of compass needle is
105. A short magnet oscillates in an oscillation magnetometer
( 0 °
) ( 1 °
) ( 2 °
) (
(a) 30 ´ 3 3 (b) 60 ´ 3 3 (c) 60 ´ 3 3 (d) 60 ´ 33
3 °
) with a time period of 0.10s where the earth¢s horizontal
magnetic field is 24 mT. A downward current of 18 A is
102. A dip needle lies initially in the magnetic meridian when it established in a vertical wire placed 20 cm east of the magnet.
shows an angle of dip q at a place. The dip circle is rotated Find the new time period.
through an angle x in the horizontal plane and then it shows (a) 0.076 s (b) 0.5 s (c) 0.1 s (d) 0.2 s

Exercise 2 : Exemplar & Past Year MCQs


NCERT Exemplar MCQs (d) domains are all perfectly aligned
4. Consider the two idealised systems (i) a parallel plate
1. A toroid of n turns, mean radius R and cross-sectional radius capacitor with large plates and small separation and (ii) a
a carries current I. It is placed on a horizontal table taken as long solenoid of length L >> R, radius of cross-section. In
xy-plane. Its magnetic moment m (i) E is ideally treated as a constant between plates and zero
(a) is non-zero and points in the z-direction by symmetry outside. In (ii) magnetic field is constant inside the solenoid
(b) points along the axis of the toroid (m = mf) and zero outside. These idealised assumptions, however,
1 contradict fundamental laws as below
(c) is zero, otherwise there would be a field falling as 3 at (a) case (i) contradicts Gauss’ law for electrostatic fields
r
large distances outside the toroid (b) case (ii) contradicts Gauss’ law for magnetic fields
(d) is pointing radially outwards (c) case (i) agrees with Ñò E.dl = 0.
2. The magnetic field of the earth can be modelled by that of a (d) case (ii) contradicts Ñò H.dl = I en
point dipole placed at the centre of the earth. The dipole 5. A paramagnetic sample shows a net magnetisation of 8
axis makes an angle of 11.3° with the axis of the earth. At Am–1 when placed in an external magnetic field of 0.6 T at a
Mumbai, declination is nearly zero. Then, temperature of 4 K. When the same sample is placed in an
(a) the declination varies between 11.3° W to 11.3° E external magnetic field of 0.2 T at a temperature of 16 K, the
(b) the least declination is 0° magnetisation will be
(c) the plane defined by dipole axis and the earth axis 32 2
passes through Greenwich (a) Am -1 (b) Am -1 (c) 6 Am–1 (d) 2.4 Am–1
(d) declination averaged over the earth must be always 3 3
negative Past Year MCQs
3. In a permanent magnet at room temperature.
(a) magnetic moment of each molecule is zero 6. Following figures show the arrangement of bar magnets in
(b) the individual molecules have non-zero magnetic different configurations. Each magnet has magnet ic dipole
moment which are all perfectly aligned r
moment m . Which configuration has highest net magnetic
(c) domains are partially aligned dipole moment? [AIPMT 2014, C]
EBD_7418
292 PHYSICS

N (a) Assertion is correct, reason is correct; reason is a


N S correct explanation for assertion.
A. B. S N
(b) Assertion is correct, reason is correct; reason is not a
S S N correct explanation for assertion
(c) Assertion is correct, reason is incorrect
N
N (d) Assertion is incorrect, reason is correct.
C. 30º D. 15. Hysteresis loops for two magnetic materials A and B are
60º
S N given below :
S N B B
(a) A (b) B (c) C (d) D
7. Magnetic lines of force due to a bar magnet do not intersect
because [AIIMS 2014, C] H H
(a) a point always has a single net magnetic field
(b) the lines have similar charges and so repel each other (A) (B)
(c) the lines always diverge from a single force
(d) the lines need magnetic lenses to be made to interest These materials are used to make magnets for elecric
8. Assertion : The ferromagnetic substance do not obey Curie’s generators, transformer core and electromagnet core. Then
law. it is proper to use : [JEE Main 2016, C]
Reason : At Curie point a ferromagnetic substance start (a) A for transformers and B for electric generators.
behaving as a paramagnetic substance. [AIIMS 2014, C] (b) B for electromagnets and transformers.
(a) Assertion is correct, reason is correct; reason is a (c) A for electric generators and trasformers.
correct explanation for assertion. (d) A for electromagnets and B for electric generators.
(b) Assertion is correct, reason is correct; reason is not a 16. If q1 and q2 be the apparent angles of dip observed in two
correct explanation for assertion vertical planes at right angles to each other, then the true
(c) Assertion is correct, reason is incorrect angle of dip q is given by : [NEET 2017, S]
(d) Assertion is incorrect, reason is correct. (a) tan q = tan q1 + tan q2 (b) cot q = cot2q1 – cot2q2
2 2 2 2
9. The coercivity of a small magnet where the ferromagnet (c) tan2q = tan2q1 – tan2q2 (d) cot2q = cot2q1 + cot2q2
gets demagnetized is 3 × 103 Am–1. The current required to
17. A magnetic needle of magnetic moment 6.7 × 10–2 Am2 and
be passed in a solenoid of length 10 cm and number of turns
moment of inertia 7.5 × 10–6 kg m2 is performing simple
100, so that the magnet gets demagnetized when inside the
harmonic oscillations in a magnetic field of 0.01 T. Time
solenoid, is: [JEE Main 2014, A]
taken for 10 complete oscillations is : [NEET 2017, S]
(a) 30 mA (b) 60 mA (c) 3 A (d) 6 A
10. At a temperatur of 30°C, the susceptibility of a ferromagnetic (a) 6.98 s (b) 8.76 s (c) 6.65 s (d) 8. 89 s
material is found to be c. Its susceptibility at 333°C is 18. Assertion : The poles of magnet can not be separated by
[AIIMS 2015, A] breaking into two pieces.
(a) c (b) 0.5c (c) 2c (d) 11.1c Reason : The magnetic moment will be reduced to half when
11. Assertion : A paramagnetic sample display greater a magnet is broken into two equal pieces. [AIIMS 2017, C]
magnetisation (for the same magnetic field) when cooled. (a) Assertion is correct, reason is correct; reason is a
Reason : The magnetisation does not depend on temperature. correct explanation for assertion.
[AIIMS 2015, C] (b) Assertion is correct, reason is correct; reason is not a
(a) Assertion is correct, reason is correct; reason is a correct explanation for assertion
correct explanation for assertion. (c) Assertion is correct, reason is incorrect
(b) Assertion is correct, reason is correct; reason is not a (d) Assertion is incorrect, reason is correct.
correct explanation for assertion 19. The basic magnetization curve for a ferromagnetic material
is shown in figure. Then, the value of relative permeability
(c) Assertion is correct, reason is incorrect
is highest for the point [BITSAT 2017, C]
(d) Assertion is incorrect, reason is correct.
12. The magnetic susceptibility is negative for :
1.5
[NEET 2016, C] R S
(a) diamagnetic material only
(b) paramagnetic material only 1.0
B(Tesla)

Q
(c) ferromagnetic material only
(d) paramagnetic and ferromagnetic materials 0.5
13. Two points A and B are situated at a distance x and 2x P
respectively from the nearer pole of a magnet 2 cm long. The
ratio of magnetic field at A and B is [AIIMS 2016, A]
(a) 4 : 1 exactly (b) 4 : 1 approximately O0 1 2 3 4 5 6 7
3
(c) 8 : 1 approximately (d) 1 : 1 approximately H (× 10 A/m)
14. Assertion : Electromagnetic are made of soft iron. (a) P (b) Q (c) R (d) S
Reason : Coercivity of soft iron is small. [AIIMS 2016, C]
MAGNETISM AND MATTER 293

20. A thin diamagnetic rod is placed vertically between the poles Reason : At a high temperature magnet losses its magnetism.
of an electromagnet. When the current in the electromagnet [AIIMS 2018, C]
is switched on, then the diamagnetic rod is pushed up, out (a) Assertion is correct, reason is correct; reason is a
of the horizontal magnetic field. Hence the rod gains correct explanation for assertion.
gravitational potential energy. The work required to do this (b) Assertion is correct, reason is correct; reason is not a
comes from [NEET 2018, C] correct explanation for assertion
(a) the current source (c) Assertion is correct, reason is incorrect
(b) the magnetic field (d) Assertion is incorrect, reason is correct.
(c) the induced electric field due to the changing magnetic 23. The variation of magnetic susceptibility (c) with temperature
field for a diamagnetic substance is best represented by
(d) the lattice structure of the material of the rod
[BITSAT 2018, C]
21. The given figure represents a material which is
[AIIMS 2018, C]

(a) O T (b)
O T

(a) paramagnetic (b) diamagnetic


(c) ferromagnetic (d) none of these
22. Assertion : The earth¢s magnetic field is due to iron present
in its core. (c) (d)

T O T
O

Exercise 3 : Try If You Can


1. Two short bar magnets P and Q are arranged such that their (a) 1 (b) 2 (c) 3 (d) 4
centres are on the X-axis and are separated by a large 5. A coil in the shape of an equilateral triangle of side l is
distance. The magnetic axes of P and Q are along X and Y suspended®between the pole pieces of a permanent magnet
axes respectively. At a point R, midway between their centres,
such that B is in the plane of the coil. If due to a current i in
if B is the magnitude of induction due to Q, the magnitude of
the triangle a torque t acts on it, the side l of the triangle is
total induction at R due to the both magnets is 1
1
5 2 æ t ö2 æ t ö2
(a) 3B (b) 5B (c) B (d) B (a) ç ÷ (b) 2çç ÷÷
2 3 è B.i ø è 3B.i ø
2. A magnetic dipole is acted upon by two magnetic fields
which are inclined to each other at an angle of 75°. One of 2 æ t ö 1 t
(c) ç ÷ (d)
the fields has a magnitude of 15 mT. The dipole attains stable 3 è B.i ø 3 .i
B
equilibrium at an angle of 30° with this field. The magntidue 6. A dip circle is so set that its needle moves freely in the
of the other field (in mT) is close to magnetic meridian. In this position, the angle of dip is 40º.
(a) 1 (b) 11 (c) 36 (d) 1060 Now the dip circle is rotated so that the plane in which the
3. A 25 cm long solenoid has radius 2 cm and 500 total number needle moves makes an angle of 30º with the magnetic
of turns. It carries a current of 15 A. If it is equivalent
r to a meridian. In this position, the needle will dip by an angle
magnet of the same size and magnetization M (magnetic (a) 40º (b) 30º
uur
moment/volume), then M is : (c) more than 40º (d) less than 40º
(a) 30000p Am–1 (b) 3p Am–1 7. A magnetic dipole is under the influence of two magnetic
(c) 30000 Am–1 (d) 300 Am–1 fields The angle between the field directions is 60° and one
4. The mid points of two small magnetic dipoles of length d in of the fields has a magnitude of 1.2 × 10–2 T. If the dipole
end-on positions, are separated by a distance x, (x > > d). comes to stable equilibrium at an angle of 15° with this field,
The force between them is proportional to x–n where n is: what is the magnitude of other field ?
(a) 4.4 × 10–3 tesla (b) 5.2 × 10–3 tesla
(c) 3.4 × 10–3 tesla (d) 7.8 × 10–3 tesla
N
S
S 8. A long straight horizontal cable carries a current of 2.5 A in
N
the direction 10° south of west to 10° north of east. The
x magnetic meridian of the place happens to be 10° west of the
geographic meridian. The earth’s magnetic field at the location
EBD_7418
294 PHYSICS

is 0.33 Gauss, and the angle of dip is zero. Locate the line of 12. The mass of a specimen of a Y
Soft iron
neutral points. (Ignore the thickness of the cable). ferromagnetic material is 0.6 kg and I
(a) 1.5 cm (b) 2.5 cm (c) 3.5 cm (d) 2.0 cm its density is 7.8 × 103 kg/m3. If the Steel

9. Two tangent galvanometers A and B have coils of radii 8 cm area of hysteresis loop of alternating X' H
X

and 16 cm respectively and resistance 8 W each. They are magnetising field of frequency 50Hz
connected in parallel with a cell of emf 4 V and negligible is 0.722 MKS units then the
internal resistance. The deflections produced in the tangent hysteresis loss per second will be Y'

galvanometers A and B are 30° and 60° respectively. If A has (a) 277.7 × 10-5 joule (b) 277.7 × 10-6 joule
(c) 277.7 × 10-4 joule (d) 27.77 × 10-4 joule
2 truns, then B must have 13. The time period of oscillation of a magnet in a vibration
(a) 18 turns (b) 12 turns (c) 6 turns (d) 2 turns magnetometer is 1.5 sec. The time period of oscillation of
10. A vibration magnetometer consists of two identical bar another magnet similar in size, shape and mass but having
magnets placed one over the other such that they are 1/4 magnetic moment than that of the 1st magnet oscillating
at the same place will be
perpendicular and bisect each other. The time period of
(a) 0.75 sec (b) 1.5 sec (c) 3.0 sec (d) 6.0 sec
oscillation in a horizontal magnetic field is 25/4 seconds. One 14. A circular coil of 16 turns and radius 10cm carries a current
of the magnets is removed and if the other magnet oscillates of 0.75 A and rest with its plane normal to an external magnetic
in the same field, then the time period in seconds is field of 5.0 × 10–2 T. The coil is free to rotate about its stable
(a) 21/4 (b) 21/2 (c) 2 (d) 23/4 equilibrium position with a frequency of 2.0 s–1 Compute the
moment of inertia of the coil about its axis of rotation.
11. A very small magnet is placed in the magnetic meridian with (a) 3.4 × 10–5 kg m2 (b) 1.2 × 10–4 kg m2
its south pole pointing north. The null point is obtained 20 –4
(c) 2.6 × 10 kg m 2 (d) 4.7 × 10–5 kg m2
cm away from the centre of the magnet. If the earth¢s 15. A uniform magnetic needle is suspended from its centre
magnetic field (horizontal component) at this point be 0.3 by a thread. Its upper end is now loaded with a mass of 50
gauss, the magnetic moment of the magnet is milligram, when the needle becomes horizontal. If the strength
(a) 8.0 × 102 e.m.u. (b) 1.2 × 103 e.m.u. of each pole is 98.1 ab amp-cm and g = 981 cm/s2, then find
3
the vertical component of earth’s magnetic induction.
(c) 2.4 × 10 e.m.u. (d) 3.6 × 103 e.m.u. (a) 0.25 gauss (b) 0.30 gauss
(b) 0.45 gauss (d) 0.50 gauss

ANSWER KEYS
Exercise 1 : Topic-wise MCQs
1 (b) 12 (a) 23 (b) 34 (d) 45 (d) 56 (a) 67 (a) 78 (d) 89 (a) 100 (b)
2 (c) 13 (c) 24 (c) 35 (b) 46 (b) 57 (d) 68 (b) 79 (c) 90 (a) 101 (a)
3 (d) 14 (a) 25 (c) 36 (a) 47 (d) 58 (a) 69 (b) 80 (a) 91 (a) 102 (a)
4 (b) 15 (d) 26 (c) 37 (a) 48 (a) 59 (c) 70 (d) 81 (a) 92 (a) 103 (b)
5 (c) 16 (a) 27 (c) 38 (b) 49 (c) 60 (d) 71 (c) 82 (c) 93 (d) 104 (c)
6 (c) 17 (b) 28 (a) 39 (d) 50 (a) 61 (d) 72 (d) 83 (a) 94 (c) 105 (a)
7 (c) 18 (a) 29 (d) 40 (a) 51 (b) 62 (d) 73 (c) 84 (b) 95 (d)
8 (c) 19 (a) 30 (b) 41 (d) 52 (a) 63 (b) 74 (a) 85 (b) 96 (c)
9 (a) 20 (d) 31 (d) 42 (a) 53 (a) 64 (d) 75 (c) 86 (d) 97 (c)
10 (a) 21 (d) 32 (b) 43 (b) 54 (a) 65 (c) 76 (c) 87 (c) 98 (d)
11 (b) 22 (a) 33 (c) 44 (a) 55 (b) 66 (a) 77 (a) 88 (c) 99 (a)
Exercise 2 : Exemplar & Past Year MCQs
1 (c) 4 (b) 7 (a) 10 (b) 13 (c) 16 (d) 19 (b) 22 (d)
2 (a) 5 (b) 8 (c) 11 (d) 14 (b) 17 (c) 20 (a) 23 (b)
3 (d) 6 (c) 9 (c) 12 (a) 15 (b) 18 (b) 21 (b)
Exercise 3 : Try If You Can
1 (b) 3 (c) 5 (b) 7 (a) 9 (b) 11 (b) 13 (c) 15 (a)
2 (b) 4 (d) 6 (d) 8 (a) 10 (c) 12 (a) 14 (b)
21Chapter
ELECTROMAGNETIC
INDUCTION

Trend
Analysis of NEET and AIIMS (Year 2010-2018)
5

4
Number of Questions

3
AIPMT/NEET
2 AIIMS

0
2010 2011 2012 2013 2014 2015 2016 2017 2018
Year

Trend
Analysis of JEE Main and BITSAT (Year 2010-2018)
5

4
Number of Questions

3
JEE Main
2 BITSAT

0
2010 2011 2012 2013 2014 2015 2016 2017 2018
Year

Chapter Utility Score (CUS)


Exam Weightage Important Concepts Difficulty Level CUS (Out of 10)
NEET 4 Faraday’s Laws of EMI,
AIIMS 3 Lenz’s law, Motional EMI 4/5 7.5/10
JEE Main 2 & A C Generator
BITSAT 4
EBD_7418
296 PHYSICS
ELECTROMAGNETIC INDUCTION 297
EBD_7418
298 PHYSICS

Exercise 1 : Topic-wise MCQs

(a) is clockwise
Topic 1: Magnetic Flux, Faraday’s and Lenz’s Law
(b) is zero
1. Whenever the magnetic flux linked with an electric circuit (c) is counter clockwise I
changes, an emf is induced in the circuit. This is called (d) has a direction that depends on the
(a) electromagnetic induction ratio of the loop radii.
(b) lenz’s law 9. Two identical coaxial circular loops carry a current i each
(c) hysteresis loss circulating in the same direction. If the loops approach each
(d) kirchhoff’s laws other, you will observe that the current in
2. According to Faraday’s law of electromagnetic induction (a) each increases
(a) electric field is produced by time varying magnetic (b) each decreases
(c) each remains the same
flux.
(d) one increases whereas that in the other decreases
(b) magnetic field is produced by time varying electric flux. 10. In electromagnetic induction, the induced charge is
(c) magnetic field is associated with a moving charge. independent of
(d) None of these (a) change of flux (b) time
3. Lenz’s law is a consequence of the law of conservation of (c) resistance of the coil (d) None of these
(a) charge (b) mass 11. Flux of magnetic field through an area bounded by a closed
(c) energy (d) momentum conducting loop can be changed by changing
4. A magnet is moved towards a coil (i) quickly (ii) slowly, then (a) area of the loop
ur
the induced e.m.f. is (b) magnetic field B
(a) larger in case (i) ur
(c) angle between area vector and B
(b) smaller in case (i)
(d) All of the above
(c) equal in both the cases
12. Match the column-I and column-II
(d) larger or smaller depending upon the radius of the coil Column I Column II
5. A current carrying infinitely long wire is kept along the (A) Ring uniformly (1) Constant electrostatic
diameter of a circular wire loop, without touching it, the charged field out of system
correct statement(s) is(are) (B) Rotating ring (2) Magnetic field strength
I. The emf induced in the loop is zero if the current is uniformly charged rotating with angular
constant. velocity w
II. The emf induced in the loop is finite if the current is (C) Constant current (3) Electric field (induced)
constant. in ring i
III. The emf induced in the loop is zero if the current
(D) i = i0coswt (4) Magnetic dipole moment
decreases at a steady rate.
(a) A ® 2 ; B ® 2, 3 ; C ® 1, 4, 3 ; D ® 3
(a) I only (b) II only
(b) A ® 3, 4 ; B ® 1 ; C ® 2, 3 ; D ® 2
(c) I and II (d) I, II and III
(c) A ® 1 ; B ® 1, 2, 4 ; C ® 2, 4 ; D ® 3
6. An induced e.m.f. is produced when a magnet is plunged
(d) A ® 2 ; B ® 4, 2, 1 ; C ® 2, 1 ; D ® 4, 2
into a coil. The strength of the induced e.m.f. is independent
13. A conducting loop is placed in a uniform magnetic field
of
(a) the strength of the magnet with its plane perpendicular to the field. An e.m.f. is induced
(b) number of turns of coil in the loop, if
(c) the resistivity of the wire of the coil (a) it is translated
(d) speed with which the magnet is moved (b) it is rotated about its axis
7. A coil of insulated wire is connected to a battery. If it is (c) both (a) and (b)
taken to galvanometer, its pointer is deflected, because (d) it is rotated about its diameter
(a) the induced current is produced 14. The expression for the induced e.m.f. contains a negative
(b) the coil acts like a magnet é df ù
sign êe = - ú . What is the significance of the negative
(c) the number of turns in the coil of the galvanometer are ë dt û
changed sign?
(d) None of these (a) The induced e.m.f. is produced only when the magnetic
8. Two different wire loops are concentric and lie in the flux decreases.
same plane. The current in the outer loop (I) is clockwise (b) The induced e.m.f. opposes the change in the magnetic
and increases with time. The induced current in the inner flux.
loop (c) The induced e.m.f. is opposite to the direction of the flux.
(d) None of the above.
ELECTROMAGNETIC INDUCTION 299

15. Two coils, A and B, are lined such that emf e is induced in B (c) Assertion is correct, reason is incorrect
when the current in A is changing at the rate I. If current i is (d) Assertion is incorrect, reason is correct.
now made to flow in B, the flux linked with A will be 21. A coil having an area A0 is placed in a magnetic field which
(a) (e/I)i (b) ei I (c) (eI)i (d) i I/e changes from B0 to 4 B0 in time interval t. The e.m.f. induced
16. Whenever the magnetic flux linked with a coil changes, an in the coil will be
induced e.m.f.is produced in the circuit. The e.m.f. lasts (a) 3A0 B0/t (b) 4A0 B0/t (c) 3B0 /A0 t (d) 4A0 /B0 t
I. for a short time 22. A conducting wire frame is placed in a
II. for a long time magnetic field which is directed into the
III. so long as the change in flux takes place paper. The magnetic field is increasing
The correct statement(s) is/are at a constant rate. The directions of
(a) I and II (b) II and III induced current in wires AB and CD are
(c) I and III (d) III only (a) B to A and D to C (b) A to B and C to D
17. In a coil of resistance 10 W, the i(amp) (c) A to B and D to C (d) B to A and C to D
induced current developed by 23. The magnetic flux through a circuit of resistance R changes
changing magnetic flux through 4 by an amount Df in a time Dt. Then the total quantity of
it, is shown in figure as a electric charge Q that passes any point in the circuit during
function of time. The the time Dt is represented by
magnitude of change in flux Df 1 Df
through the coil in weber is 0 0.1
t(s) (a) Q = R. (b) Q = .
Dt R Dt
(a) 8 (b) 2 (c) 6 (d) 4 Df Df
18. Fig shown below represents an area A = 0.5 m2 situated in a (c) Q = (d) Q =
R Dt
uniform magnetic field B = 2.0 weber/m2 and making an angle 24. The north pole of a bar magnet is moved towards a coil
of 60º with respect to magnetic field. along the axis passing through the centre of the coil and
perpendicular to the plane of the coil. The direction of the
B
induced current in the coil when viewed in the direction
of the motion of the magnet is
60
(a) clockwise
(b) anti-clockwise
(c) no current in the coil
The value of the magnetic flux through the area would be (d) either clockwise or anti-clockwise
equal to 25. If a current increases from zero to one ampere in 0.1 second
(a) 2.0 weber (b) in a coil of 5 mH, then the magnitude of the induced e.m.f.
3 weber
will be
(c) 3 / 2 weber (d) 0.5 weber (a) 0.005 volt (b) 0.5 volt
19. In the figure the flux through the loop perpendicular to the (c) 0.05 volt (d) 5 volt
plane of the coil and directed into the paper is varying 26. A coil has 200 turns and area of 70 cm2. The magnetic field
according to the relation f = 6t2 + 7t + 1 where f is in perpendicular to the plane of the coil is 0.3 Wb/m2 and take 0.1
milliweber and t is in second. The magnitude of the emf sec to rotate through 180º.The value of the induced e.m.f. will
induced in the loop at t = 2 s and the direction of induce be
Ä Ä Ä Ä Ä
current through R are (a) 8.4 V (b) 84 V (c) 42 V (d) 4.2 V
Ä Ä Ä Ä Ä
(a) 39 mV; right to left Ä Ä Ä Ä Ä
27. A coil of resistance 400W is placed in a magnetic field. If the
(b) 39 mV; left to right Ä Ä Ä Ä Ä magnetic flux f (wb) linked with the coil varies with time t
(c) 31 mV; right to left Ä Ä Ä Ä Ä (sec) as f = 50t2 + 4. The current in the coil at t = 2 sec is
(d) 31 mV; left to right (a) 0.5 A (b) 0.1 A (c) 2 A (d) 1 A
R
20. Assertion : Figure shows a horizontal 28. A coil having n turns and resistance R W is connected with
solenoid connected to a battery and a a galvanometer of resistance 4R W. This combination is
switch. A copper ring is placed on a moved in time t seconds from a magnetic field W1 weber to
smooth surface, the axis of the ring W2 weber. The induced current in the circuit is
being horizontal. As the switch is closed, the ring will move ( W1 - W2 ) n ( W2 - W1 )
away from the solenoid. (a) - (b) - 5 Rt
Rnt
df ( W2 - W1 )
Reason : Induced emf in the ring, e = - . n ( W2 - W1 )
dt (c) - (d) -
5 Rnt Rt
(a) Assertion is correct, reason is correct; reason is a
correct explanation for assertion. 29. Assertion : An emf can be induced by moving a conductor
(b) Assertion is correct, reason is correct; reason is not a in a magnetic field.
correct explanation for assertion Reason : An emf can be induced by changing the magnetic
field.
EBD_7418
300 PHYSICS

(a) Assertion is correct, reason is correct; reason is a 39. A rectangular coil of 20 turns and area of cross-section 25
correct explanation for assertion. sq. cm has a resistance of 100W. If a magnetic field which is
(b) Assertion is correct, reason is correct; reason is not a perpendicular to the plane of coil changes at a rate of 1000
correct explanation for assertion tesla per second, the current in the coil is
(c) Assertion is correct, reason is incorrect (a) 1 A (b) 50 A (c) 0.5 A (d) 5 A
(d) Assertion is incorrect, reason is correct. 40. A horizontal telegraph wire 0.5 km long running
30. Magnetic flux f in weber in a closed circuit of resistance east and west in a part of a circuit whose resistance is 2.5 W.
10W varies with time f (sec) as f = 6t2 – 5t + 1. The magnitude The wire falls to g = 10.0 m/s2 and B = 2 × 10–5 weber/m2,
of induced current at t = 0.25s is then the current induced in the circuit is
(a) 0.2 A (b) 0.6 A (c) 1.2 A (d) 0.8 A (a) 0.7 amp (b) 0.04 amp
31. A conducting circular loop is placed in a uniform magnetic (c) 0.02 amp (d) 0.01 amp
field, B = 0.025 T with its plane perpendicular to the loop. 41. A solenoid has 2000 turns wound over a length of
The radius of the loop is made to shrink at a constant rate of 0.3 m. Its cross-sectional area is 1.2 × 10–3 m2. Around its
1 mm s–1. The induced e.m.f. when the radius is 2 cm, is central section a coil of 300 turns is wound. If an initial
p current of 2 A flowing in the solenoid is reversed in 0.25 s,
(a) 2pmV (b) pmV (c) mV (d) 2mV the emf induced in the coil will be
2
32. The north pole of a long horizontal bar magnet is being (a) 2.4 × 10–4 V (b) 2.4 × 10–2 V
brought closer to a vertical conducting plane along the (c) 4.8 × 10–4 V (d) 4.8 × 10–2 V
perpendicular direction. The direction of the induced current
in the conducting plane will be Topic 2: Motional & Static EMI, Applications of EMI
(a) horizontal (b) vertical 42. A straight conductor of length 2m moves at a speed of
(c) clockwise (d) anticlockwise 20 m/s. When the conductor makes an angle of 30° with the
33. A coil of 50 turns is pulled in 0.02 s between the poles direction of magnetic field of induction of 0.1 wbm2 then
of a magnet, where its area includes 31 × 10–6 Wb to induced emf
1 × 10–6Wb. The average e.m.f. is (a) 4V (b) 3V (c) 1V (d) 2V
(a) 7.5 × 10–2 V (b) 7.5 × 10–3 V 43. A wire of length 1m is perpendicular to x-y plane. It is moved
(d) 7.5 × 10–4V r
(c) zero with velocity v = (3iˆ + 3jˆ + 2k)
ˆ m / s through a region of
34. A long solenoid having 200 turns per cm carries a current of r
uniform induction B = (iˆ + 2j) ˆ T . The potential difference
1.5 amp. At the centre of it is placed a coil of 100 turns of
between the ends of the wire is
cross-sectional area 3.14 × 10–4 m2 having its axis parallel to
(a) 1V (b) 1.5V (c) 2.5V (d) 3V
the field produced by the solenoid. When the direction of 44. A square coil of side 25cm having 1000 turns is rotated with
current in the solenoid is reversed within 0.05 sec, the induced a uniform speed in a magnetic field about an axis
e.m.f. in the coil is
perpendicular to the direction of the field. At an instant t,
(a) 0.48 V (b) 0.048 V (c) 0.0048 V (d) 48 V
the emf induced in the coil is e = 200 sin 100pt. The magnetic
35. A 100 turns coil of area of cross section 200 cm2 having 2 W
induction is
resistance is held perpendicular to a magnetic field of 0.1 T.
(a) 0.50 T (b) 0.02 T (c) 0.01 T (d) 0.1 T
If it is removed from the magnetic field in one second, the
45. A 10-meter wire is kept in east-west direction. It is falling
induced charge produced in it is
down with a speed of 5.0 meter/second, perpendicular to
(a) 0.2 C (b) 2 C (c) 0.1 C (d) 1 C
the horizontal component of earth's magnetic field of 0.30
36. A thin circular ring of area A is held perpendicular to a
×10-4 weber/meter2. The momentary potential difference
uniform magnetic field of induction B. A small cut is made in
induced between the ends of the wire will be
the ring and a galvanometer is connected across the ends
(a) 0.0015 V (b) 0.015 V (c) 0.15 V (d) 1.5 V
such that the total resistance of the circuit is R. When the
46. When current in a coil changes from 5 A to 2 A in 0.1 s,
ring is suddenly squeezed to zero area, the charge flowing
average voltage of 50 V is produced. The self - inductance
through the galvanometer is
of the coil is :
BR AB B2 A (a) 6 H (b) 0.67 H (c) 3 H (d) 1.67 H
(a) (b) (c) ABR (d)
A R R2 47. A coil of circular cross-section having 1000 turns and 4 cm2
37. A metal disc of radius 100 cm is rotated at a constant angular face area is placed with its axis parallel to a magnetic field
speed of 60 rad/s in a plane at right angles to an external which decreases by 10–2 Wb m–2 in 0.01 s. The e.m.f.
field of magnetic induction 0.05 Wb/m2. The emf induced induced in the coil is:
between the centre and a point on the rim will be (a) 400 mV (b) 200 mV (c) 4mV (d) 0.4 mV
(a) 3 V (b) 1.5 V (c) 6 V (d) 9 V 48. When current i passes through an inductor of self
38. A rectangular coil of 100 turns and size 0.1 m × 0.05 m is inductance L, energy stored in it is 1/2. L i2. This is stored in
placed perpendicular to a magnetic field of 0.1 T. The induced the
e.m.f. when the field drops to 0.05 T in 0.05s is (a) current (b) voltage
(a) 0.5 V (b) 1.0 V (c) 1.5 V (d) 2.0 V (c) magnetic field (d) electric field
ELECTROMAGNETIC INDUCTION 301

49. If N is the number of turns in a coil, the value of self I I


inductance varies as
(a) N0 (b) N (c) N2 (d) N–2
(c) (d)
50. Two coils of inductances L1 and L2 are linked such that
their mutual inducatance is M. Then t t
(a) M = L1 + L2 57. A rod PQ of length L moves with a uniform velocity v parallel
1 to a long straight wire carrying a current i, the end P
(b) M = (L1 + L 2 ) remaining at a distance r from the wire. The emf induced
2
(c) the maximum value of M is (L1 + L2) across the rod is
(d) the minimum value of M is L1 L 2 m0iv2 æ r + L ö m0i2 v2 æ r 2 + L ö
(a) ln ç ÷ (b) ln ç ÷
51. Two conducting circular loops of radii R1 and R2 are placed 2p è r ø 2p ç r ÷
è ø
in the same plane with their centres coinciding. If
m0iv æ r + L ö m 0 iv æ r 2
+ L2ö
R1>>R2, the mutual inductance M between them will be (c) ln ç (d) ln ç ÷
÷ 2p çè L2 ÷ø
directly proportional to 2p è r ø
(a) R1/R2 (b) R2/R1 (c) R12 / R2 (d) R22 / R1 58. A conductor AB of length l moves in x – y plane with
r r
52. The mutual inductance between two planar concentric rings ( )
velocity v = v0 ˆi - ˆj . A magnetic field B = B0 ˆi + ˆj ( )
of radii r1 and r 2 (r1 > r2) placed in air is given by exists in the region. The iduced emf is
m0 pr22 m0 pr12 (a) zero (b) B0lv0 (c) B0lv0 (d) 2B0 lv0
(a) (b) 59. A rectangular loop is being pulled at a constant speed v,
2r1 2r2
2 through a region of certain thickness d, in which a uniform
m0 p(r1 + r2 ) m0 p(r1 + r2 )2 magnetic field B is set up.
(c) (d)
2r1 2r2 The graph between position x of the
53. A coil of N = 100 tuns carries a current I = 5A and creates a right hand edge of the loop and the
magnetic flux f = 10–5 Tm2 per turn. The value of its induced emf E will be
inductance L will be
(a) 0.05 mH (b) 0.10 mH (c) 0.15 mH (d) 0.20 mH
54. A coil is wound on a frame of rectangular cross-section. If (a) (b)
all the linear dimensions of the frame are increased by a
factor 2 and the number of turns per unit length of the coil
remains the same, self-inductance of the coil increases by a
factor of
(a) 4 (b) 8 (c) 12 (d) 16
55. A wire of length 1 m is moving at a speed of 2ms –1
perpendicular to its length in a homogeneous magnetic field (c) (d)
of 0.5 T. The ends of the wire are joined to a circuit of
resistance 6W. The rate at which work is being done to keep
the wire moving at constant speed is 60. A sliding wire of length 0.25 m and having a resistance of 0.5
1 1 1 W moves along conducting guiding rails AB and CD with a
(a) W (b) W (c) W (d) 1W uniform speed of 4 m/s. A magnetic field of 0.5 T exists
12 6 3
56. An equilateral triangular loop ABC made of uniform thin normal to the plane of ABCD directed into the page. The
wires is being pulled out of a region with a uniform speed v, guides are short -circuited with resistances of 4 and 2 W as
r shown. The current through the sliding wire is :
where a uniform magnetic field B perpendicular to the plane
of the loop exists. At time t = 0, the point A is at the edge of x x x x x
A M B
the magnetic field. The induced current (I) vs time (t) graph
0.5W
will be as x x x x
A v 0.25 m
× ×××××××××××× 4W 2W
× ×××××××××××× 4m/s
× ×××××××××××× x x x x
× ××××××××××××
× ××××××××××××
× × × ×B× × × × ×C× × × × C N D
× ×××××××××××× x x x x x
I I
0.5 T
x x x x x

(a) (b) (a) 0.27 A (b) 0.37 A (c) 1.0 A (d) 0.72 A
t t 61. A six pole generator with fixed field excitation develops an
e.m.f. of 100 V when operating at 1500 r.p.m. At what speed
must it rotate to develop 120V?
EBD_7418
302 PHYSICS

(a) 1200 r.p.m (b) 1800 r.p.m 70. Consider the situation shown. The wire AB is sliding on
(c) 1500 r.p.m (d) 400 r.p.m fixed rails with a constant velocity. If the wire AB is replaced
62. If the magnetic flux linked with a coil through which a current by semi-circular wire, the magnitude of induced e.m.f. will
of x A is set up is y Wb, then the coefficient of self Ä Ä Ä A Ä Ä
inductance of the coil is (a) increase Ä Ä Ä Ä Ä
x
(a) (x – y) henry (b) henry (b) decrease Ä Ä vÄ Ä Ä
y R
y (c) remain the same Ä Ä Ä Ä Ä
(c) henry (d) x y henry
x (d) increase or decrease Ä Ä Ä B Ä Ä
63. The self inductance associated with a coil is independent depending on whether
of the semi-circle buldges
(a) current (b) time towards the resistance or
(c) induced voltage (d) resistance of coil away from it.
64. The plane in which eddy currents are produced in a 71. The mutual inductance of a pair of coils, each of N turns, is
conductor is inclined to the plane of the magnetic field at M henry. If a current of I ampere in one of the coils is
an angle equal to brought to zero in t second, the emf induced per turn in the
(a) 45° (b) 0° (c) 180° (d) 90° other coil, in volt, will be
65. Eddy currents are produced when
MI NMI MN MI
(a) A metal is kept in varying magnetic field (a) (b) (c) (d)
(b) A metal is kept in the steady magnetic field t t It Nt
72. When the current in a coil changes from 2 amp. to 4 amp. in
(c) A circular coil is placed in a magnetic field
0.05 sec., an e.m.f. of 8 volt is induced in the coil. The
(d) Through a circular coil, current is passed
coefficient of self inductance of the coil is
66. When strength of eddy currents is reduced, as dissipation
(a) 0.1 henry (b) 0.2 henry
of electrical energy into heat depends on the ...A... of the
(c) 0.4 henry (d) 0.8 henry
strength of electrical energy into heat depends on the
73. Two coils of self inductances 2 mH and 8 mH are placed so
...A... of the strength of electric current heat loss is
close together that the effective flux in one coil is completely
substantially ...B ... .
linked with the other. The mutual inductance between these
Here, A and B refer to coils is
(a) cube, increase (b) inverse, increased (a) 6 mH (b) 4 mH (c) 16 mH (d) 10 mH
(c) inverse, decreased (d) square, reduced 74. The mutual inductance of a pair of coils is 0.75 H. If current
67. Certain galvanometers have a fixed core made of non- in the primary coil changes from 0.5 A to zero in 0.01 s, find
magnetic metallic material, when the coil oscillates, ...A... average induced e.m.f. in secondary coil.
generated in the core ...B... the motion and bring the coil
(a) 25.5 V (b) 12.5 V (c) 22.5 V (d) 37.5 V
to rest ...C...
75. A wire of length 50 cm moves with a velocity of 300 m/min,
Here, A, B and C refer to
perpendicular to a magnetic field. If the e.m.f. induced in
(a) induced emf, support, long time
the wire is 2 V, the magnitude of the field in tesla is
(b) induced current, support, long time (a) 2 (b) 5 (c) 0.8 (d) 2.5
(c) mechanical energy, oppose, long time 76. A circular wire of radius r rotates about its own axis with
(d) eddy currents, oppose, quickly angular speed w in a magnetic field B perpendicular to its
68. When the plane of the armature of a generator is parallel plane, then the induced e.m.f. is
to the field. in which it is rotating 1
(a) both the flux linked and induced emf in the coil are zero. (a) Brw 2 (b) Brw2 (c) 2Brw2 (d) zero
2
(b) the flux linked with it is zero, while induced emf is 77. The coefficient of self inductance of a solenoid is 0.18 mH.
maximum. If a core of soft iron of relative permeability 900 is inserted,
(c) flux linked is maximum while induced emf is zero. then the coefficient of self inductance will become nearly.
(d) both the flux and emf have their respective maximum (a) 5.4 mH (b) 162 mH
values. (c) 0.006 mH (d) 0.0002 mH
69. Assertion : Figure shows a metallic 78. The inductance of a closed-packed coil of 400 turns is 8
conductor moving in magnetic field. The mH. A current of 5 mA is passed through it. The magnetic
induced emf across its ends is zero. flux through each turn of the coil is
Reason : The induced emf across the ends of a conductor is 1 1
given by e = Bvlsinq. (a) m0 Wb (b) m0 Wb
(a) Assertion is correct, reason is correct; reason is a 4p 2p
1
correct explanation for assertion. (c) m 0 Wb (d) 0.4 µ0Wb
(b) Assertion is correct, reason is correct; reason is not a 3p
correct explanation for assertion 79. The figure shows a wire sliding on two parallel conducting
(c) Assertion is correct, reason is incorrect rails placed at a separation I. A magnetic field B exists in
(d) Assertion is incorrect, reason is correct. a direction perpendicular to the plane of the rails. The
ELECTROMAGNETIC INDUCTION 303

force required to keep the wire moving at a constant 88. A coil is wound on a frame of rectangular cross-section. If
velocity v will be all the linear dimensions of the frame are increased by a
(a) evB × × × × × factor 2 and the number of turns per unit length of the coil
m0 Bv × × × × × remains the same, self-inductance of the coil increases by a
(b) × × ×I × × factor of
4 pI V
(c) BIv × × × × × (a) 4 (b) 8 (c) 12 (d) 16
× × × × × 89. A small square loop of wire of side l is placed inside a large
(d) zero
square loop of side L (L >> l). The loop are coplanar and
80. Assertion : When number of turns in a coil is doubled, their centres coincide. The mutual inductance of the system
coefficient of self-inductance of the coil becomes 4 times. is proportional is
Reason : This is because L µ N2. l l 2 L L2
(a) Assertion is correct, reason is correct; reason is a (a) (b) (c) (d)
L L l l
correct explanation for assertion. 90. Two coaxial solenoids are made by winding thin insulated
(b) Assertion is correct, reason is correct; reason is not a wire over a pipe of cross-sectional area A = 10 cm2 and
correct explanation for assertion length = 20 cm. If one of the solenoid has 300 turns and the
(c) Assertion is correct, reason is incorrect other 400 turns, their mutual inductance is
(d) Assertion is incorrect, reason is correct. (m0 = 4p × 10 –7 Tm A–1)
81. A metallic square loop ABCD is moving in its own plane (a) 2.4p × 10–5 H (b) 4.8p × 10–4 H
with velocity v in a unifrom magnetic field perpendicular (c) 4.8p × 10–5 H (d) 2.4p × 10–4 H
to its plane as shown in figure. An 91. Charge Q is uniformly distributed on a thin insulating ring
electric field is induced A B of mass m which is initially at rest. To what angular velocity
(a) in AD, but not in BC will the ring be accelerated when a magnetic field B,
v
(b) in BC, but not in AD perpendicular to the plane of the ring, is switched on?
(c) neither in AD nor in BC D C
QB 3QB QB QB
(d) in both AD and BC (a) (b) (c) (d)
82. A conductor of length 0.4 m is moving with a speed of 2m 2m m 4m
92. An L-shaped conductor rod is moving in transverse magnetic
7 m/s perpendicular to a magnetic field of intensity
field as shown in the figure. Potential difference between
0.9 Wb/m2. The induced e.m.f. across the conductor is ends of the rod is maximum if the rod is moving with velocity
(a) 1.26 V (b) 2.52V (c) 5.04 V (d) 25.2 V
(a) 4iˆ - 6 ˆj m/s A
83. A circular coil and a bar magnet placed nearby are made to y
move in the same direction. If the coil covers a distance of 1 (b) -4iˆ + 6 ˆj m/s 3m x
m in 0.5. sec and the magnet a distance of 2 m in 1 sec, the
(c) 3iˆ + 2 ˆj m/s B
induced e.m.f. produced in the coil is
(a) zero (b) 0.5 V (c) 1 V (d) 2 V. (d) 13 iˆ m/s 2m
84. A circular coil of radius 6 cm and 20 turns rotates about 93. A square frame of side 10 cm and a long straight wire carrying
its vertical diameter with an angular speed of 40 rad current 1 A are in the plate of the paper. Starting from close
s–1 in a uniform horizontal magnetic field of magnitude to the wire, the frame moves towards the right with a
2 × 10–2 T. If the coil form a closed loop of resistance 8W, constant speed of 10 ms–1 (see figure).
then the average power loss due to joule heating is The e.m.f induced at the
(a) 2.07 × 10–3 W (b) 1.23 × 10–3 W time the left arm of the frame I = 1A

(c) 3.14 × 10 W –3 (d) 1.80 × 10–3 W is at x = 10 cm from the wire


is
85. A 100 millihenry coil carries a current of 1 ampere. Energy x
stored in its magnetic field is (a) 2 mV
(a) 0.5 J (b) 1 J (c) 0.05 J (d) 0.1 J (b) 1 mV v

86. Two solenoids of same cross-sectional area have their (c) 0.75 mV
lengths and number of turns in ratio of 1 : 2 both. The ratio (d) 0.5 mV 10 cm
of self-inductance of two solenoids is 94. A copper disc of radius 0.1 m rotated about its centre with
(a) 1 : 1 (b) 1 : 2 (c) 2 : 1 (d) 1 : 4 10 revolutions per second in a uniform magnetic field of 0.1
87. Two coils have a mutual inductance 0.005H. The current tesla with its plane perpendicular to the field. The e.m.f.
changes in first coil according to equation I = I0 sin wt induced across the radius of disc is
where I0 = 10A and w = 100p radian/sec. The max. value of p 2p
e.m.f. in second coil is (a) volt (b) volt
10 10
(a) 2p (b) 5p (c) p (d) 4p (c) p × 10–2 volt (d) 2p × 10–2 volt
EBD_7418
304 PHYSICS

95. A generator has an e.m.f. of 440 Volt and internal resistance 12c 2 rg B ×
of 4000 hm. Its terminals are connected to a load of 4000 (a) ×
2 ×
ohm. The voltage across the load is eB s ×
×
×
(a) 220 volt (b) 440 volt (c) 200 volt (d) 400 volt 16c 2 rg M × × N
(b)
96. A generator of 220 V having internal resistance r = 10W and eB 2 s
external resistance R = 100W. What is the power developed 16c 2 r2 g
(c) a a
in the external circuit?
eBs
(a) 484 W (b) 400 W (c) 441 W (d) 369 W
16c r g
97. A rectangular loop PQRS, is pulled with constant speed (d) Q a P
into a uniform transverse magnetic field by a force F (as e2 B s v
shown). E.m.f. induced in side PS and potential difference 99. In th e space shown a non-uniform magnetic field
r
between points P and S respectively are (Resistance of the B = B0 (1 + x ) ( - kˆ ) tesla is present. A closed loop of small
loop = r) resistance placed in the x-y plane is given velocity V0. The
Fr P Q force due to magnetic field on the loop is
(a) zero, × × × × × ×
Bl (a) zero y V0
(b) zero, zero l F
(b) along +x direction × × × × ×
Fr × × q
(c) zero, (c) along –x direction
6 Bl R × × × × ×x ×
S
Fr Fr
,
B (d) along +y direction × × × × × ×
(d) 2l
6 Bl 6 Bl
98. In the given figure MNPQ which falls through the magnetic 100. A conducting ring of radius l m kept in a uniform magnetic
field B of 0.01 T, rotates uniformly with an angular velocity
field has conductivity s and mass density r. The frame’ss
100 rad s–1 with its axis of rotation perpendicular to B. The
terminal velocity assuming it to be small enough so that it
maximum induced emf in it is
reaches its final velocity before leaving the region occupied
(a) 1.5pV (b) pV (c) 2pV (d) 0.5pV
by the magnetic field is

Exercise 2 : Exemplar & Past Year MCQs


NCERT Exemplar MCQs 4. There are two coils A and B as shown in figure a current
1. A square of side L metres lies in the xy-plane in a region, starts flowing in B as shown, when A is moved towards B
where the magnetic field is given by B = B0 (2iˆ + 3jˆ + 4k)
ˆ T,, and stops when A stops moving. The current in A is counter
clockwise. B is kept stationary when A moves. We can infer
where B0 is constant. The magnitude of flux passing through
that
the square is
(a) there is a constant current in the clockwise direction in
(a) 2B0L2Wb (b) 3B0L2Wb A
(c) 4B0L2Wb (d) 29B0 L2 Wb (b) there is a varying current in A
2. A loop, made of straight edges has six corners at A (0, 0, 0), (c) there is no current in A
B (L, 0, 0), C(L, L, 0), D (0, L, 0), E(0, L, L) and F (0, 0, L). A (d) there is a constant current in the counter clockwise
direction in A
magnetic field B = B0 ( ˆi + kˆ ) T is present in the region. The
A B
flux passing through the loop ABCDEFA (in that order) is
v
(a) B0L2 Wb (b) 2B0L2Wb
(c) Ö2B0L Wb2 (d) 4B0L2Wb
3. A cylindrical bar magnet is rotated about its axis. A wire is 5. Same as problem 4 except the coil A is made to rotate about
connected from the axis and is made to touch the cylindrical a vertical axis (figure). No current flows in B if A is at rest.
surface through a contact. Then,
The current in coil A, when the current in B (at t = 0) is
(a) a direct current flows in the ammeter A
(b) no current flows through the ammeter A counter-clockwise and the coil A is as shown at this instant,
t = 0, is A
(c) an alternating sinusoidal current flows through the w B
2p (a) constant current clockwise
ammeter A with a time period T = (b) varying current clockwise
w
(d) a time varying non-sinusoidal current flows through (c) varying current counter clockwise
the ammeter A. (d) constant current counter clockwise
ELECTROMAGNETIC INDUCTION 305

6. The self inductance L of a solenoid of length l and area of 14. In an inductor of self-inductance L = 2 mH, current changes
cross-section A, with a fixed number of turns N increases as with time according to relation i = t2e–t. At what time emf is
(a) l and A increase zero? [AIIMS 2016, C]
(b) l decreases and A increases (a) 4s (b) 3s (c) 2s (d) 1s
(c) l increases and A decreases 15. A conducting circular loop is placed in a uniform magnetic
(d) both l and A decrease
field of 0.04 T with its plane perpendicular to the magnetic
Past Year MCQs field. The radius of the loop starts shrinking at 2 mm/s. The
induced emf in the loop when the radius is 2 cm is
7. A thin semicircular conducting ring B
(PQR) of radius ‘r’ is falling with its
Q [BITSAT 2016, A]
plane vertical in a horizontal magnetic (a) 4.8 p mV (b) 0.8 p mV (c) 1.6 p mV (d) 3.2 p mV
r
field B, as shown in figure. The P R 16. A long solenoid of diameter 0.1 m has 2 × 104 turns per
potential difference developed across meter. At the centre of the solenoid, a coil of 100 turns and
the ring when its speed is v, is : [AIPMT 2014, C] radius 0.01 m is placed with its axis coinciding with the
(a) Zero solenoid axis. The current in the solenoid reduces at a
(b) Bvpr2/2 and P is at higher potential constant rate to 0A from 4 A in 0.05 s. If the resistance of the
(c) prBv and R is at higher potential coil is 10p2W. the total charge flowing through the coil during
(d) 2rBv and R is at higher potential this time is: [NEET 2017, A]
8. The flux linked with a coil at any instant 't' is given by (a) 16 mC (b) 32 mC (c) 16 p mC (d) 32 p mC
f = 10t2 – 50t + 250. The induced emf at t = 3s is 17. A conducting rod AB moves parallel to X-axis in a uniform
[AIIMS 2014, C] magnetic field, pointing in the positive X-direction. The end
(a) –190 V (b) –10 V (c) 10 V (d) 190 V A of the rod gets [AIIMS 2017, C]
9. Assertion : Lenz's law violates the principle of conservation Y
of energy.
Reason : Induced emf always opposes the change in magnetic B
flux responsible for its production. [AIIMS 2014, C]
(a) Assertion is correct, reason is correct; reason is a
correct explanation for assertion. V
(b) Assertion is correct, reason is correct; reason is not a
correct explanation for assertion A
(c) Assertion is correct, reason is incorrect O X
(d) Assertion is incorrect, reason is correct. (a) positively charged
10. An electron moves on a straight line path XY as shown. (b) negatively charged
The abcd is a coil adjacent to the path of electron. (c) neutral
What will be the direction of current if any, induced in the (d) first positively charged and then negatively charged
coil? [AIPMT 2015, C] 18. In a coil of resistance 100 W, a current
(a) adcb a is induced by changing the
(b) The current will reverse its magnetic flux through it as shown in
direction as the electron goes b d the figure. The magnitude of change
past the coil in flux through the coil is
(c) No current induced c [JEE Main 2017, S]
(d) abcd X electron Y
(a) 250 Wb (b) 275 Wb (c) 200 Wb (d) 225 Wb
11. In generator, having a coil with N turns, all of the same area 19. A coil is suspended in a uniform magnetic field, with the
A and total resistance R, rotates with frequency w in a plane of the coil parallel to the magnetic lines of force. When
magnetic field B. The maximum value of emf generated in a current is passed through the coil it starts oscillating; It is
the coil is [AIIMS 2015, C] very difficult to stop. But if an aluminium plate is placed
(a) N.A.B.R.w (b) N.A.B. near to the coil, it stops. This is due to : [BITSAT 2017, C]
(c) N.A.B.R. (d) N.A.B.w
(a) developement of air current when the plate is placed
12. A conducting square loop is placed in a magnetic field B with
its plane perpendicular to the field. The sides of the loop start (b) induction of electrical charge on the plate
shrinking at a constant rate a. The induced emf in the loop at (c) shielding of magnetic lines of force as aluminium is a
an instant when its side is ‘a’ is [BITSAT 2015, A] paramagnetic material.
(a) 2aaB (b) a2aB (c) 2a2aB (d) aaB (d) electromagnetic induction in the aluminium plate giving
13. A long solenoid has 1000 turns. When a current of 4A flows rise to electromagnetic damping.
through it, the magnetic flux linked with each turn of the 20. The magnetic potential energy stored in a certain inductor
solenoid is 4 × 10–3 Wb. The self inductance of the solenoid is 25 mJ, when the current in the inductor is 60 mA. This
is : [NEET 2016, A] inductor is of inductance [NEET 2018, A]
(a) 4H (b) 3H (c) 2H (d) 1H (a) 0.138 H (b) 138.88 H (c) 13.89 H (d) 1.389 H
EBD_7418
306 PHYSICS

21. Assertion : In the phenomenon of mutual induction, self (c) Assertion is correct, reason is incorrect
induction of each of the coil persists. (d) Assertion is incorrect, reason is correct.
Reason : Self induction arises due to change in current in 22. A copper rod of length l rotates about its end with
the coil itself. In mutual induction current changes in both angular velocity w in uniform magnetic field B. The emf
the individual coil. [AIIMS 2018, C] developed between the ends of the rod if the field is
(a) Assertion is correct, reason is correct; reason is a normal to the plane of rotation is [BITSAT 2018, C]
correct explanation for assertion. 1 1
(b) Assertion is correct, reason is correct; reason is not a (a) Bwl2 (b) Bwl2 (c) 2 Bwl2 (d) Bwl2
2 4
correct explanation for assertion

Exercise 3 : Try If You Can


æ xö
1. The magnetic field in a region is given by B = B0 ç 1 + ÷ kˆ. A BwR 2 M
è aø (a) R
square loop of edge-length d is placed with its edges along 2
the x and y-axes. The loop is moved with a constant velocity
v = v0 iˆ . The emf induced in the loop is : pR 2 R
(b) Bw
(a) zero (b) v0B0d 4
v 0 B0 d 3 v 0 B0 d 2
æ 2 pR 2 ö N O
(c) (d) (c) Bw ç 2 R – ÷ B
a2 a è 4 ø
2. A wooden stick of length 3l is rotated about an end with (d) zero
constant angular velocity w in a uniform magnetic field B 5. A conducting metal circular–wire–loop of radius r is placed
perpendicular to the plane of motion. If the upper one third perpendicular to a magnetic field which varies with time as
of its length is coated with copper, the potential difference -t
across the whole length of the stick is B = B0 e t , where B0 and t are constants, at time t = 0. If
9Bwl 2 × × × × × × × × × × the resistance of the loop is R then the heat generated in the
(a)
2 × × × × × × ×copper × × × loop after a long time (t ® ¥) is :
2 × × × × × × ×coat× × ×
4 Bwl × × × × × × × × × × p 2 r 4 B04 p 2 r 4 B02
(b) (a) (b)
2 × × ×3 × × × × × × × 2tR 2tR
2 × × × × × × × × × ×
5Bwl
(c) × × × × × × × × × ×
p 2 r 4 B02 R p 2 r 4 B02
2 × × × × × × × × × × (c) (d)
Bwl 2 × × × × × × × × × × t tR
(d) × × × × × × × × × ×
2 6. A uniform magnetic field of induction B
3. Three solenoid coils of same dimension, same number of is confined to a cylindrical region of
turns and same number of layers of windings are taken. Coil radius R. The magnetic field is increasing
1 with inductance L1 was wound using a Mn wire of dB
at a constant rate of (tesla/second). B
resistance 11 W/m, coil 2 with inductance L2 was wound dt R
An electron of charge e, placed at the
using the similar wire but the direction of winding was point P on the periphery of the field
reversed in each layer; coil 3 with inductance L3 was wound experiences an acceleration. P
using a superconducting wire. The self inductance of the 1 eR dB 1 eR dB
coils L1, L2, L3 are (a) towards left (b) towards right
2 m dt 2 m dt
(a) L1 = L2 = L3 (b) L1 = L2; L3 = 0 eR dB
(c) L1 = L3; L2 = 0 (d) L1 > L2 > L3 (c) towards left (d) zero
m dt
4. The given assembly made of a conducting wire is rotated 7. Two conducting circular loops of radii x1 and x2 are placed
with a constant angular velocity w about aurvertical axis MO in the same plane with their centres coinciding. What is the
as shown in the figure. The magnetic field B exists vertically mutual inductance between them assuming x2 <<< x1?
upwards as shown in the figure. Find the potential difference (a) m0 r x22/2x1 (b) m0 r x1/x22
between points M and N, |Vm – VN| (only the magnitude) (c) m0 r x2/x12 (d) 2m0 r x1/x22
ELECTROMAGNETIC INDUCTION 307

8. In a uniform magnetic field of induction B a wire in the form 12. One conducting U tube can A
B
of a semicircle of radius r rotates about the diameter of the slide inside another as shown
circle with an angular frequency w. The axis of rotation is in figure, maintaining electrical v
v X
perpendicular to the field. If the total resistance of the circuit contacts between the tubes.
is R, the mean power generated per period of rotation is The magnetic field B is C
( B pr w )2 ( B pr 2 w ) 2 perpendicular to the plane of the figure . If each tube moves
(a) (b) towards the other at a constant speed v, then the emf
2R 8R
induced in the circuit in terms of B, l and v where l is the
B pr 2 w ( B pr w 2 ) 2
(c) (d) width of each tube, will be
2R 8R (a) – Blv (b) Blv (c) 2 Blv (d) zero
9. A square loop of side a is rotating about its diagonal with
r 13. A rectangular coil of single turn, having area A, rotates in a
angular velocity w in a perpendicular magnetic field B . It uniform magnetic field B with an angular velocity w about
has 10 turns. The emf induced is an axis perpendicular to the field. If initially the plane of the
(a) B a2 w sin wt coil is perpendicular to the field, then the average induced
(b) B a2 w cos wt × × × ×B emf when it has rotated through 90° is
wBA wBA wBA 2wBA
(c) 5 2 B a2 (a) (b) (c) (d)
a p 2 p 4 p p
(d) 10 B a2 w sin wt x x x x x
10. The two rails of a railway track, insulated from each other 14. In fig, CODFC is a semicircular w
Cx x x x x
and the ground, are connected to millivoltmeter. What is the loop of a conducting wire of B
reading of the millivoltmeter when a train passes at a speed resistance R and radius r. It is x x x x x
of 180 km/hr along the track, given that the vertical placed in a uniform magnetic F O
x x x x x
component of earth’s magnetic field is 0.2 × 10–4 wb/m2 and field B, which is directed into r
rails are separated by 1 metre the page (perpendicular to the x x x x x
D
(a) 10–2 volt (b) 10mV (c) 1 volt (d) 1mV plane of the loop). x x x x x
11. A conducting square frame of side ‘a’ and a long staight The loop is rotated with a constant angular speed w about
wire carrying current I are located in the same plane as shown an axis passing through the centre O, and perpendicular to
in the figure. The frame moves to the right with a constant the page. Then the induced current in the wire loop is
velocity ‘V’. The emf induced in the frame will be proportional (a) zero (b) Br2 w/R
to X 2
(c) Br w/2R (d) Bpr2 w/R
1
(a) 15. A circular loop of radius 0.3 cm lies parallel to a much bigger
(2x – a)2 circular loop of radius 20 cm. The centre of the small loop is
1 l
(b) on the axis of the bigger loop. The distance between their
(2x + a)2 V centres is 15 cm. If a current of 2.0 A flows through the
1 smaller loop, then the flux linked with bigger loop is
(c) (2x – a)(2x + a)
1
(a) 9.2 × 10–11 weber (b) 6 × 10–11 weber
(d) a (c) 3.3 × 10–11 weber (d) 6.6 × 10–9 weber
x2
EBD_7418
308 PHYSICS

ANSW ER KEYS
Exercise 1 : Topic-wis e MCQs
1 (a) 11 (d) 21 (a) 31 (b) 41 (b) 51 (d) 61 (b) 71 (a) 81 (c) 91 (a)
2 (a) 12 (c) 22 (a) 32 (d) 42 (d) 52 (a) 62 (c) 72 (b) 82 (b) 92 (c)
3 (c) 13 (d) 23 (c) 33 (a) 43 (d) 53 (d) 63 (d) 73 (b) 83 (a) 93 (b)
4 (a) 14 (b) 24 (b) 34 (b) 44 (c) 54 (b) 64 (d) 74 (d) 84 (a) 94 (c)
5 (a) 15 (a) 25 (c) 35 (c) 45 (a) 55 (b) 65 (a) 75 (c) 85 (c) 95 (d)
6 (c) 16 (d) 26 (a) 36 (b) 46 (d) 56 (b) 66 (d) 76 (d) 86 (b) 96 (b)
7 (a) 17 (b) 27 (a) 37 (b) 47 (a) 57 (c) 67 (d) 77 (b) 87 (b) 97 (c)
8 (c) 18 (d) 28 (b) 38 (a) 48 (c) 58 (a) 68 (b) 78 (a) 88 (b) 98 (b)
9 (b) 19 (d) 29 (b) 39 (c) 49 (c) 59 (b) 69 (a) 79 (d) 89 (b) 99 (c)
10 (b) 20 (a) 30 (a) 40 (c) 50 (d) 60 (a) 70 (c) 80 (b) 90 (d) 100 (b)
Exercis e 2 : Exemplar & Pas t Year MCQs
1 (c) 4 (d) 7 (d) 10 (b) 13 (d) 16 (b) 19 (d) 22 (b)
2 (b) 5 (a) 8 (b) 11 (d) 14 (c) 17 (a) 20 (c)
3 (b) 6 (b) 9 (a) 12 (a) 15 (d) 18 (a) 21 (a)
Exercis e 3 : Try If You Can
1 (d) 3 (b) 5 (b) 7 (a) 9 (d) 11 (c) 13 (d) 15 (a)
2 (c) 4 (a) 6 (a) 8 (b) 10 (d) 12 (c) 14 (c)
22Chapter
ALTERNATING CURRENT

Trend
Analysis of NEET and AIIMS (Year 2010-2018)
5

4
Number of Questions

3
AIPMT/NEET
2 AIIMS

0
2010 2011 2012 2013 2014 2015 2016 2017 2018
Year

Trend
Analysis of JEE Main and BITSAT (Year 2010-2018)
5

4
Number of Questions

3
JEE Main

2 BITSAT

0
2010 2011 2012 2013 2014 2015 2016 2017 2018
Year

Chapter Utility Score (CUS)


Exam Weightage Important Concepts Difficulty Level CUS (Out of 10)
NEET 4 A.C. Circuits,
AIIMS 6 LCR Series & Parallel, 3.5/5 7.5/10
JEE Main 4 LC Oscillations & Transformer
BITSAT 4
EBD_7418
310 PHYSICS
ALTERNATING CURRENT 311
EBD_7418
312 PHYSICS

Exercise 1 : Topic-wise MCQs

Topic 1: Alternating Current, Voltage and Power 1 2


(a) I p 2 R cos q (b) Ip R
2
1. Alternating current cannot be measured by D.C. ammeter,
because 4 2 1 2
(c) Ip R (d) Ip R
(a) A. C. is virtual p p2
(b) A. C. changes its direction 11. The r.m.s. value of potential difference V shown in the figure is
(c) A. C. cannot pass through D.C. ammeter V
(d) average value of A. C for complete cycle is zero V0
I0
2. The alternating current of equivalent value of is O t
2 T/2 T
(a) peak current (b) r.m.s. current (a) V0 (b) V0 / 2 (c) V0/2 (d) V0 / 3
(c) D.C. current (d) all of these 12. Assertion : Average value of ac over a complete cycle is
3. The peak value of the a.c. current flowing through a resistor always zero.
is given by Reason: Average value of ac is always defined over half
(a) I0 = e0/R (b) I = e/R cycle.
(c) I0 = e0 (d) I0 = R/e0 (a) Assertion is correct, reason is correct; reason is a
4. A.C. power is transmitted from a power house at a high correct explanation for assertion.
voltage as
(b) Assertion is correct, reason is correct; reason is not a
(a) the rate of transmission is faster at high voltages
correct explanation for assertion
(b) it is more economical due to less power loss
(c) power cannot be transmitted at low voltages (c) Assertion is correct, reason is incorrect
(d) a precaution against theft of transmission lines (d) Assertion is incorrect, reason is correct.
5. The ratio of mean value over half cycle to r.m.s. value of 13. Determine the rms value of the emf given by
A.C. is E (in volt) = 8 sin ( w t) + 6sin (2 w t)
(a) 2 : p (b) 2 2 : p (c) 2 :p (d) 2 :1 (a) 5 2 V (b) 7 2 V (c) 10 V (d) 10 2 V
6. The average power dissipated in a pure inductance is 14. The r.m.s value of an a.c. of 50 Hz is 10 amp. The time taken
by the alternating current in reaching from zero to maximum
1 2
(a) LΙ (b) L I2 (c) L I2/4 (d) zero value and the peak value of current will be
2 (a) 2 × 10–2 sec and 14.14 amp
7. If the frequency of an A.C. is made 4 times of its initial value, (b) 1 × 10–2 sec and 7.07 amp
the inductive reactance will
(c) 5 × 10–3 sec and 7.07 amp
(a) be 4 times (b) be 2 times
(c) be half (d) remain the same (d) 5 × 10–3 sec and 14.14 amp
8. When an ac voltage of 220 V is applied to the capacitor 15. The instantaneous voltage through a device of impedance
C, then 20 W is e = 80 sin 100 pt. The effective value of the current is
(a) the maximum voltage between plates is 220 V. (a) 3 A (b) 2.828 A (c) 1.732 A (d) 4 A
(b) the current is in phase with the applied voltage. 16. In an ac circuit an alternating voltage e = 200 2 sin 100 t
(c) the charge on the plate is not in phase with the volts is connected to a capacitor of capacity 1 mF. The r.m.s.
applied votage. value of the current in the circuit is
(d) power delivered to the capacitor per cycle is zero. (a) 10 mA (b) 100 mA (c) 200 mA (d) 20 mA
9. In the case of an inductor 17. An alternating voltage V = V0 sin wt is applied across a
p circuit. As a result, a current I = I0 sin (wt – p/2) flows in it.
(a) voltage lags the current by
2 The power consumed per cycle is
p (a) zero (b) 0.5 V0I0
(b) voltage leads the current by (c) 0.707 V0I0 (d) 1.414 V0I0
2 y
18. Determine the rms value of a
p
(c) voltage leads the current by semi-circular current wave
3
p which has a maximum value of
(d) voltage leads the current by a. –a +a
4 0
x
10. The sinusoidal A.C. current flows through a resistor of (a) (1 2 ) a (b) (3 / 2) a
resistance R. If the peak current is Ip, then power dissipated
is (c) ( 2 / 3) a (d) (1/ 3) a
ALTERNATING CURRENT 313

19. The heat produced in a given resistance in a given time by 27. Current in an ac circuit is given by i = 3 sin wt + 4 cos wt then
the sinusoidal current I0sin wt will be the same as that of a (a) rms value of current is 5 A
steady current of magnitude nearly (b) mean value of this current in one half period will be 6/p
(a) 0.71 I0 (b) 1.412 I0 (c) I0 (d) I0 (c) if voltage applied is V = Vm sin wt then the circuit must
20. An alternating current is given by be containing resistance and capacitance.
i = i1 coswt + i2 sinwt The rms current is given by (d) if voltage applied is V = Vm sin wt, the circuit may
i1 + i 2 2 2 contain resistance and inductance.
i1 + i 2 i12 + i 22 i1 + i 2
(a) (b) (d) (c) 28. An ac voltage is represented by
2 2 2 2
E = 220 2 cos ( 50p ) t
21. If a direct current of value ampere is superimposed on an
alternative current I = b sin wt flowing through a wire, what How many times will the current become zero in 1 s?
is the effective value of the resulting current in the circuit? (a) 50 times (b) 100 times
dc (c) 30 times (d) 25 times
+
I a I b ac =? 29. A group of electric lamps having a total power rating of 1000
watt is supplied by an AC voltage E = 200 sin
t
(310 t + 60º). Then the rms value of the circuit current is
1/ 2 (a) 10 A (b) 10 2 A
é 2 1 2ù
(a) êë a - 2 b úû (b) [a2 + b2]1/2
(c) 20 A (d) 20 2 A
1/ 2 1/2 30. A sinusoidal voltage V(t) = 100 sin (500t) is applied across a
é a2 ù é b2ù pure inductance of L = 0.02 H. The current through the coil is:
(c) ê + b 2 ú (d) ê a 2 + ú
ë 2 û ë 2 û (a) 10 cos (500 t) (b) – 10 cos (500t)
22. The voltage of an ac supply varies with time (t) as V = 120 (c) 10 sin (500t) (d) – 10 sin (500t)
sin 100 pt cos 100 pt. The maximum voltage and frequency 31. The equation of alternating current is :
respectively are
I = 50 2 sin 400pt amp. Then the frequency and root mean
120 square of current are respectively
(a) 120 volt, 100 Hz volt, 100 Hz (b)
2 (a) 200 Hz, 50 amp (b) 400p Hz, 50 2 amp
(c) 60 volt, 200 Hz (d) 60 volt, 100 Hz (c) 200 Hz, 50 2 amp (d) 50 Hz, 200 amp
23. An alternating e.m.f. of angular frequency w is applied across 32. Figure shows one cycle of an alternating current with the
an inductance. The instantaneous power developed in the segments AB, BC, CD, DE being symmetrical and parabolic.
circuit has an angular frequency The root mean square value of this current over one cycle
w w is x mA, find x.
(a) (b) (c) w (d) 2w
4 2 i
24. The voltage time (V-t) graph for triangular wave having peak B
Ö5 mA
value V0 is as shown in figure.The rms value of V in time
V0
interval from t = 0 to T/4 is then find the value of x. 2s 3s 4s
x A 1s C t(s)
0 +V
(a) 5
(b) 4 T/2 t –Ö5 mA
0 D
T/4 T
(c) 7
–V0 (a) 1 mA (b) 2 mA (c) 3 mA (d) 4 mA
(d) 3
25. A resistance of 20 ohm is connected to a source of an 33. Using an A.C. voltmeter the potential difference in the
alternating potential V = 200 cos(100 pt). The time taken by electrical line in a house is read to be 234 volt. If the line
the current to change from its peak value to rms value a is frequency is known to be 50 cycles/second, the equation for
(a) 2.5 × 10–3 s (b) 25 × 10–3 s the line voltage is
(c) 0.25 s (d) 0.20 s (a) V = 165 sin (100 p t) (b) V = 331 sin (100 p t)
26. The rms value of the function shown in figure if it is given (c) V = 220 sin (100 p t) (d) V = 440 sin (100 p t)
that for 0 < t < 0.1, y = 10 (1 – e–100t) and for
0.1 < t < 0.2, y = 10e–50(t–0.1) is Topic 2: AC Circuit, LCR Circuit, Quality and Power Factor
y
34. An A.C. source is connected to a resistive circuit. Which of
the following is true?
10 (a) Current leads ahead of voltage in phase
(b) Current lags behind voltage in phase
(c) Current and voltage are in same phase
0 0.1 0.2 0.3 0.4 (d) Any of the above may be true depending upon the
t (Second) value of resistance.
(a) 6.2 (b) 5.3 (c) 4.1 (d) 6.9
EBD_7418
314 PHYSICS

35. With increase in frequency of an A.C. supply, the inductive (a) R + f (b) R+ 2pfC
reactance 1
(a) decreases (c) R+ (d) R 2 + X 2C
(b) increases directly with frequency 2 pfC
45. In an a.c. circuit with phase voltage V and current I, the
(c) increases as square of frequency
(d) decreases inversely with frequency power dissipated is
36. The capacitive reactance in an A.C. circuit is VΙ VΙ
(a) effective resistance due to capacity (a) (b) (c) V Ι (d) VI cos q
2 2
(b) effective wattage 46. The power factor of an AC circuit having resistance (R) and
(c) effective voltage inductance (L) connected in series and an angular velocity
(d) None of these w is
37. In which of the following circuits the maximum power (a) R/wL (b) R/(R2w2L2)
dissipation is observed?
(c) wL/R (d) R/(R2w2L2)½
(a) Pure capacitive circuit (b) Pure inductive circuit
47. Which of the following graphs represents the correct
(c) Pure resistive circuit (d) None of these
variation of inductive reactance XL with frequency u?
38. In an L.C.R. series a.c. circuit, the current
(a) is always in phase with the voltage XL XL
(b) always lags the generator voltage
(c) always leads the generator voltage
(d) None of these (a) (b)
39. If an LCR series circuit is connected to an ac source, then at
u u
resonance the voltage across
(a) R is zero XL XL
(b) R equals the applied voltage
(c) C is zero (c) (d)
(d) L equals the applied voltage
40. Of the following about capacitive reactance which is
correct? u u
(a) The reactance of the capacitor is directly proportional 48. For the circuit shown in the fig., the current through the
to its ability to store charge inductor is 0.9 A while the current through the condenser is
(b) Capacitive reactance is inversely proportional to the 0.4 A. Then
frequency of the current (a) current drawn from source I = 1.13 A C
(c) Capacitive reactance is measured in farad
(d) The reactance of a capacitor in an A.C. circuit is similar (b) w = 1/(1.5 L C)
to the resistance of a capacitor in a D.C. circuit L
41. In a series resonant circuit, having L,C and R as its elements, (c) I = 0.5 A
the resonant current is i. The power dissipated in circuit at ~
resonance is (d) I = 0.6 A V = V0 sin wt

i2R 49. A coil of inductive reactance 31 W has a resistance of 8 W. It


(a) (b) zero is placed in series with a condenser of capacitative reactance
(wL -1/ wC)
25W. The combination is connected to an a.c. source of 110
(c) i2 wL (d) i2 R volt. The power factor of the circuit is
Whereas w is angular resonant frequency
(a) 0.64 (b) 0.80 (c) 0.33 (d) 0.56
42. At resonant frequency the current amplitude in series
50. A capacitor of 10 µF and an inductor of 1 H are joined in
LCR circuit is
series. An ac of 50 Hz is applied to this combination. What
(a) maximum (b) minimum
is the impedance of the combination?
(c) zero (d) infinity
43. The power factor in a circuit connected to an A.C. The value 5 ( p 2 - 5) 102 ( p 2 - 10 )
(a) W (b)
of power factor is p p
10 ( p 2 - 5) 5 (10 - p 2 )
(a) unity when the circuit contains an ideal inductance
only (c) W (d) W
(b) unity when the circuit contains an ideal resistance only p p
(c) zero when the circuit contains an ideal resistance only 51. In series L-C-R circuit, the voltages across R, L and C are
(d) unity when the circuit contains an ideal capacitance VR, VL and VC respectively. Then the voltage of applied a.c.
only source must be
44. An A. C. of frequency f is flowing in a circuit containing a (a) VR + VL + VC
resistance R and capacitance C in series. The impedance of
(b) [( VR ) 2 + ( VL - VC ) 2 ]
the circuit is equal to
ALTERNATING CURRENT 315

(c) VR + VC – VL 61. The current in resistance R at resonance is


(d) [(VR + VL)2 + (VC)2]1/2 R
52. In an a.c. circuit the voltage applied is E = E0 sin wt.
æ pö
The resulting current in the circuit is I < I0 sin ççwt , ÷÷÷ .
çè 2ø C
L
The power consumption in the circuit is given by

(a) P = 2E0 I0 (b) P < 0 0


E I ~
V = V0 sin wt
2 (a) zero (b) minimum but finite
E 0 I0 (c) maximum but finite (d) infinite
(c) P = zero (d) P <
2 62. Which one of the following curves represents the variation
53. In an LCR series resonant circuit, the capacitance is changed of impedance (Z) with frequency f in series LCR circuit?
from C to 4C. For the same resonant frequency, the Z Z
inductance should be changed from L to
L L (a) (b)
(a) 2 L (b) (c) 4 L (d)
2 4
54. In series combination of R, L and C with an A.C. source at
resonance, if R = 20 ohm, then impedence Z of the f f
combination is Z Z
(a) 20 ohm (b) zero (c) 10 ohm (d) 400 ohm (c) (d)
55. Resonance frequency of LCR series a.c. circuit is f0. Now
the capacitance is made 4 times, then the new resonance
frequency will become
(a) f0/4 (b) 2f0 (c) f0 (d) f0/2. f f
56. In an A.C. circuit, the current flowing in inductance is 63. An alternating voltage of frequency w is induced in electric
I = 5 sin (100 t – p/2) amperes and the potential difference is circuit consisting of an inductance L and capacitance C,
V = 200 sin (100 t) volts. The power consumption is equal to
connected in parallel. Then across the inductance coil the
(a) 1000 watt (b) 40 watt
I. current is maximum when w2 = 1/(L C)
(c) 20 watt (d) Zero
II. current is minimum when w2 = 1/(L C)
57. In an LCR series a.c. circuit, the voltage across each of the
III. voltage is minimum when w2 = 1/(L C)
components, L, C and R is 50V. The voltage across the LC
IV. voltage is maximum when w2 = 1/(L C)
combination will be
Which of the above statements are correct?
(a) 100 V (b)
50 2 V (a) I and III (b) I and IV (c) II and III (d) II and IV
(c) 50 V (d) 0 V 64. Which of the following graphs represents the correct
58. A resistance 'R' draws power 'P' when connected to an AC variation of capacitive reactance XC with frequency u?
source. If an inductance is now placed in series with the XC X C
resistance, such that the impedance of the circuit becomes
'Z', the power drawn will be (a) (b)
2
R æRö æRö
(a) P (b) P ç ÷ (c) P (d) P ç ÷ u u
Z èZø èZø XC XC
59. If resistance of 100W, and inductance of 0.5 henry and
capacitance of 10 × 106 farad are connected in series through
50 Hz A.C. supply, then impedance is (c) (d)
(a) 1.8765 W (b) 18.76 W (c) 187.6 W (d) 101.3 W
u u
60. Assertion : When the frequency of the AC source in an
65. when 100 volts dc is Supplied across a Solenoid, a current
LCR circuit equals the resonant frequency, the reactance of
the circuit is zero, and so there is no current through the of 1.0 anperes flows in it when 100 volts ac is applied across
inductor or the capacitor. the same coil, the current drops to 0.5 ampere If the
Reason : The net current in the inductor and capacitor is frequency of ac Source is 50 Hz, then the impedance and
zero. inductance of the Solenoid are
(a) Assertion is correct, reason is correct; reason is a (a) 200 p and 0.55 henry
correct explanation for assertion. (b) 100 p and 0.86 henry
(b) Assertion is correct, reason is correct; reason is not a (c) 200 p and 1.0 henry
correct explanation for assertion
(d) 100 p and 0.93 henry
(c) Assertion is correct, reason is incorrect
(d) Assertion is incorrect, reason is correct.
EBD_7418
316 PHYSICS

66. Two coils A and B are connected in series across a 240 V, 50 (C) Power developed (3) Remains same
Hz supply. The resistance of A is 5W and the inductance of in resistor
B is 0.02 H. The power consumed is 3 kW and the power (D) Impedance (4) May increase or
factor is 0.75. The impedance of the circuit is decrease
(a) 0.144 W (b) 1.44 W (c) 14.4 W (d) 144 W (a) (A) ® (1, 2); (B) ® (2); (C) ® (3, 4); (D) ® (1)
67. A coil of 40 henry inductance is connected in series with (b) (A) ® (1); (B) ® (2); (C) ® (1); (D) ® (2)
a resistance of 8 ohm and the combination is joined to the (c) (A) ® (1); (B) ® (3); (C) ® (1); (D) ® (1, 2)
terminals of a 2 volt battery. The time constant of the circuit (d) (A) ® (2); (B) ® (3); (C) ® (4); (D) ® (1)
is 75. Assertion : In series LCR resonance circuit, the impedance
(a) 20 seconds (b) 5 seconds is equal to the ohmic resistance.
(c) 1/5 seconds (d) 40 seconds Reason: At resonance, the inductive reactance exceeds the
68. For a series RLC circuit R = XL = 2XC. The impedance of the capacitive reactance.
(a) Assertion is correct, reason is correct; reason is a
circuit and phase difference between V and I respectively
correct explanation for assertion.
will be
(b) Assertion is correct, reason is correct; reason is not a
5R 5R correct explanation for assertion
(a) , tan -1 (2) (b) , tan -1 (1/ 2)
2 2 (c) Assertion is correct, reason is incorrect
(c) 5X C , tan -1 (2) (d) 5R, tan -1 (1/ 2) (d) Assertion is incorrect, reason is correct.
76. The impedance in a circuit containing a resistance of 1 W
69. A current of 4A flows in a coil when connected to a 12V dc and an inductance of 0.1 H in series, for AC of 50 Hz, is
source. If the same coil is connected to a 12V, 50 rad/s a.c.
source, a current of 2.4A flows in the circuit. Determine the (a) 100 10 W (b) 10 10 W
inductance of the coil. (c) 100W (d) 10W
(a) 0.08 H (b) 0.04 H (c) 0.02 H (d) 1 H 77. An ac voltage is applied to a resistance R and an inductor L
70. In the circuit diagram shown, XC = 100 W, XL = 200 W & in series. If R and the inductive reactance are both equal to
R = 100 W. The effective current through the source is 3W, the phase difference between the applied voltage and
the current in the circuit is
2 x A then find value of x. C
200V ~ (a) p/6 (b) p/4 (c) p/2 (d) zero
(a) 4 (b) 3 R
L 78. An inductance of negligible resistance whose reactance is 22
(c) 2 (d) 5 W at 200 Hz is connected to 200 volts, 50 Hz power line. The
71. An e.m.f. of 15 volt is applied in a circuit containing 5 henry value of inductance is
inductance and 10 ohm resistance. The ratio of the currents (a) 0.0175 henry (b) 0.175 henry
at time t= ¥ and at t = 1 second is (c) 1.75 henry (d) 17.5 henry
e1/ 2 e2 79. An inductive circuit contains resistance of 10 ohms and an
(a) 1/ 2 -1 (b) 2 (c) 1–e–1 (d) e–1 inductance of 2 henry. If an A.C. voltage of 120 Volts and
e e -1
72. In an L-C-R series circuit connected to an AC source, V = V0 frequency 60 Hz is applied to this circuit, the current would
be nearly
æ pö
sin çè 100pt + 6 ÷ø . Given VR = 40V, VL = 40V and VC = 10V. (a) 0.32 A (b) 0.16 A (c) 0.48 A (d) 0.80 A
80. An inductive coil has a resistance of 100 W. When an a.c.
Resistance R = 4W. signal of requency 1000 Hz is fed to the coil, the applied
Peak value of current in the circuit is voltage leads the current by 45°. What is the inductance of
the coil ?
(a) 10 2A (b) 15 2A (c) 20 2A (d) 25 2A (a) 10 mH (b) 12 mH (c) 16 mH (d) 20mH.
73. Which of the following statements is/are correct? 81. In an LR circuit f = 50 Hz, L = 2H, E = 5 volts, R = 1 W then
I. In LCR series ac circuit, as the frequency of the source energy stored in inductor is
increases, the impedence of the circuit first decreases (a) 50 J (b) 25 J
and then increases. (c) 100 J (d) None of these
II. If the net reactance of an LCR series ac circuit is same 82. An alternating voltage of 220 V, 50 Hz frequency is applied
as its resistance, then the current lags behind the
across a capacitor of capacitance 2 µF. The impedence of
voltage by 45°.
the circuit is
III. Below resonance, voltage leads the current while above
it, current leads the voltage. p 1000 5000
(a) (b) (c) 500 p (d)
(a) I only (b) II only (c) I and III (d) I and II 5000 p p
74. In a series LCR circuit, the e.m.f. leads current. Now the 83. In an experiment, 200 V A.C. is applied at the ends of an
driving frequency is decreased slightly. Match columns I LCR circuit. The circuit consists of an inductive reactance
and II. (XL ) = 50 W, capacitive reactance (XC ) = 50 W and ohmic
Column I Column II resistance (R) = 10 W. The impedance of the circuit is
(A) Current amplitude (1) Increases (a) 10W (b) 20W (c) 30W (d) 40W
(B) Phase constant (2) Decreases
ALTERNATING CURRENT 317

84. In a series resonant LCR circuit, the voltage across R is 100 1


volts and R = 1 kW with C = 2mF. The resonant frequency w e= sin(100pt + p / 3) Volt
2
is 200 rad/s. At resonance the voltage across L is
The average power in Watt consumed in the circuit is
(a) 2.5 × 10–2 V (b) 40 V
(c) 250 V (d) 4 × 10–3 V 1 3 1 1
(a) (b) (c) (d)
85. In an a.c. circuit V and I are given by 4 4 2 8
V = 100 sin (100 t) volts 90. The plot given below is of the average power delivered to
I = 100 sin (100 t + p/3) mA an LRC circuit versus frequency. The quality factor of the
the power dissipated in the circuit is circuit is:

average power (microwatts)


(a) 104 watt (b) 10 watt 1.0
(c) 2.5 watt (d) 5.0 watt
86. In the given circuit, the current drawn from the source is
0.5
V = 100 x sin(100pt )

X C = 20W
X L =10W
R = 20W

0.0
~
3 4 5 6 7
frequency (kHz)
(a) 5.0 (b) 2.0 (c) 6 (d) 0.4
(a) 20 A (b) 10 A (c) 5 A (d) 5 2 A 91. Two coils A and B are connected in series across a 240 V, 50
87. In the given circuit the reading of voltmeter V1 and V2 are Hz supply. The resistance of A is 5W and the inductance of
B is 0.02 H. The power consumed is 3 kW and the power
300 volt each. The reading of the voltmeter V3 and ammeter
factor is 0.75. The impedance of the circuit is
A are respectively
(a) 0.144 W (b) 1.44 W (c) 14.4 W (d) 144 W
L C R = 100 W 92. In given circuit capacitor R
S
initially uncharged. Now at
V1 V2 V3 t = 0 switch S is closed then
A R
current given by source at e C
~ any time t is
220 V, 50 Hz
(a) 150 V and 2.2 A (b) 220 V and 2.0 A æ – 2t ö æ – 2t ö
2ç (b) e çè1+ e CR ÷ø
(c) 220 V and 2.0 A (d) 100 V and 2.0 A (a) è1– e CR ÷ø
88. In an LCR series circuit connected to an ac source, the R 2R
e ç æ – 2t ö æ – 2t ö
supply voltage is V = V0 sin æç100pt + p ö÷ . V L = 40 V, (c) è1– e CR ÷ø (d) 2e çè 1– e CR ÷ø
è 6ø 2R R
VR = 40V, Z = 5W and R = 4W. Then match the column I 93. In the circuit shown in Figure, how does the potential
and II. difference between the points A and B depend on the value
of the variable resistor (R) ?
VL VC VR
C
(a) V/2
r
L R (b) V
C V ~ A B
(c) V/3
r R
Column I Column II (d) None of these
(A) Peak current (in A) (1) 10 2 94. Find the current passing through battery immediately after
(B) V0 (in volts) (2) 50 2 key (K) is closed. It is given that initially all the capacitors
(C) Effective value of (3) 50 are uncharged. (given that R = 6 W and C = 4µF)
C
applied voltage (in volts) R
(D) XC (in W) (4) 1 K R
(a) (A) ® (1); (B) ® (2); (C) ® (1); (D) ® (4) R C
(b) (A) ® (2); (B) ® (3); (C) ® (1); (D) ® (4)
(c) (A) ® (4); (B) ® (3); (C) ® (2); (D) ® (1) E = 5V C
(d) (A) ® (4); (B) ® (1); (C) ® (3); (D) ® (2) L
89. The instantaneous values of alternating current and voltages R C
in a circuit are given as
R
1
i= sin(100pt ) A (a) 1 A (b) 5 A (c) 3 A (d) 2 A
2
EBD_7418
318 PHYSICS

95. In an electrical circuit R, L, C and an a.c. voltage source are 101. An inductor 20 × 10–3 Henry, a capacitor 100µF and a resis-
all connected in series. When L is removed from the circuit, tor 50W are connected in series across a source of EMF V =
the phase difference between the voltage the current in the 10 sin 314t. If resistance is removed from the circuit and the
circuit is p/3. If instead, C is removed from the circuit, the value of inductance is doubled, then the variation of cur-
phase difference is again p/3. The power factor of the circuit rent with time in the new circuit is –
is
(a) 0.52 cos 314 t (b) 0.52 sin 314 t
(a) 1/2 (b) 1/ 2 (c) 1 (d) 3/2 (c) 0.52 sin (314 t + p/3) (d) None of these
96. In figure given below if ZL = Zc and reading of ammeter is
1A. Find value of source voltage V. Topic 3: Transformer and LC Oscillations
ZL ZC
(a) 80 Volt 102. A transformer is employed to
(a) convert A.C. into D.C.
L=i/p H (b) convert D.C. into A.C.
(b) 60 Volt
(c) obtain a suitable A.C. voltage
(c) 100 Volt R = 80W (d) obtain a suitable D.C. voltage
~ A 103. The transformer voltage induced in the secondary coil of a
(d) None of these V, 30 Hz transformer is mainly due to
97. In the circuit shown, assuming all ammeters to be ideal, if (a) a varying electric field
readings of the hot wire ammeters A1 and A2 are i1 and i2 (b) a varying magnetic field
respectively then reading of the hot wire ammeter A3 is (c) the vibrations of the primary coil
R (d) the iron core of the transformer
A1
104. The loss of energy in the form of heat in the iron core of
A3 a transformer is
L (a) iron loss (b) copper loss
A2 (c) mechanical loss (d) None of these
105. Quantity that remains unchanged in a transformer is
(a) voltage (b) current
~ (c) frequency (d) None of these
AC source 106. A charged 30 mF capacitor is connected to a 27 mH
(a) equal to (i1 + i2) (b) greater than (i1 + i2) inductor. The angular frequency of free oscillations of the
(c) less than (i1 + i2) (d) equal to 2 (i1 – i2) | circuit is
98. In the circuit shown, the symbols have their usual meanings. (a) 1.1 × 103 rad s–1 (b) 2.1 × 103 rad s–1
The cell has emf E. X is initially joined to Y for a long time. 3
(c) 3.1 × 10 rad s –1 (d) 4.1 × 103 rad s–1
Then, X is joined to Z. The maximum charge on C at any later 107. A step up transformer operates on a 230 V line and supplies
time will be L
a current of 2 ampere. The ratio of primary and secondary
E ER winding is 1:25 . The current in primary is
(a) (b) R + – Y (a) 25 A (b) 50 A (c) 15 A (d) 12.5 A
R LC 2 LC
108. In a transformers, number of turns in primary coil are 140
E LC E LC X and that in secondary coil are 280. If current in primary coil
(c) (d)
2R R C Z is 4A, then that in secondary coil is
99. LC circuit contains a 20 mH inductor and a 50 µF capacitor (a) 4 A (b) 2 A (c) 6 A (d) 10 A
with an initial charge of 10 mC. The resistance of the circuit 109. A current flowing in a step down transformer (220V to 22V)
is negligible. Let the instant the circuit is closed at t = 0. At having impedence 220 p , is
what time is the energy stored completely magnetic ? (a) 0.1 A (b) 2.2 A (c) 0.0001A (d) 2.1 A
(a) t = 0 (b) t = 1.57 ms 110. A transformer is used to light a 140 W, 24 V bulb from a
240 V a.c. mains. The current in the main cable is 0.7 A. The
(c) t = 3.14 ms (d) t = 6.28 ms
efficiency of the transformer is
100. When an AC source of emf e = E0 sin(100t) is connected
(a) 63.8 % (b) 83.3 % (c) 16.7 % (d) 36.2 %
across a circuit, the phase difference between the emf e and
111. Eddy currents in the core of transformer can't be developed
the current i in the circuit is observed to be p 4 , as shown by
in the diagram. If the circuit consists possibly only of R-C (a) increasing the number of turns in secondary coil
or R-L or L-C in series, find the relationship between the (b) taking laminated transformer
two elements (c) making step down transformer
(a) R = 1k W , C = 10 µF i e (d) using a weak a.c. at high potential
112. The transformation ratio in the step-up transformer is
(b) R = 1k W , C = 1 µF (a) one (b) greater than one
(c) R = 1k W , L = 10 H (c) less than one
(d) the ratio greater or less than one depends on the other
(d) R = 1k W , L = 1 H factor
ALTERNATING CURRENT 319

113. A transformer has an efficiency of 80%. It works at 4 kW 123. The primary winding of a transformer has 100 turns and its
and 100 V. If secondary voltage is 240 V, the current in primary secondary winding has 200 turns. The primary is connected
coil is to an A.C. supply of 120 V and the current flowing in it is 10
(a) 0.4 A (b) 4 A (c) 10 A (d) 40 A A. The voltage and the current in the secondary are
114. The primary of a transformer has 400 turns while the (a) 240 V, 5 A (b) 240 V, 10 A
secondary has 2000 turns. If the power output from the (c) 60 V, 20 A (d) 120 V, 20 A
secondary at 1000 V is 12 kW, what is the primary voltage? 124. A choke coil and capacitor are connected in resies and the
(a) 200 V (b) 300 V (c) 400 V (d) 500 V current through the combination is maximum for AC of
frequency n. If they are connected in parallel, at what
115. A transformer connected to 220 V mains is used to light a
frequency the current through the combination is minimum?
lamp of rating 100 W and 110 V. If the primary current is (a) n (b) n/2 (c) 2n (d) 5n
0.5 A, the efficiency of the transformer is (approximately) 125. At t < 0, the capacitor is charged and the switch is opened.
(a) 60% (b) 35% (c) 50% (d) 90% At t = 0 the switch is closed. The shortest time T at which
116. A fully charged capacitor C with initial charge Q0 is the charge on the capacitor will be zero is given by
connected to a coil of self inductance L at t = 0. The time L Q0 C
at which the energy is stored equally between the electric
and the magnetic field is
p
(a) LC (b) 2p LC (c) LC (d) p LC
4 S
117. Assertion : The power is produced when a transformer steps 3 p
up the voltage. (a) p LC (b) p LC (c) LC (d) 2p LC
2 2
Reason : In an ideal transformer VI = constant. 126. Figure shows an iron-cored transformer assumed to be
(a) Assertion is correct, reason is correct; reason is a 100% efficient. The ratio olf the secondary turns to the
correct explanation for assertion
primary turns is 1 : 20.
(b) Assertion is correct, reason is correct; reason is not a
correct explanation for assertion
Primary
(c) Assertion is correct, reason is incorrect coil
(d) Assertion is incorrect, reason is correct. 6.0 W
Secondary
118. A capacitor in an LC oscillator has a maximum potential coil
difference of 17 V and a maximum energy of 160 mJ. When
the capacitor has a potential difference of 5V and an energy A 240 V ac supply is connected to the primary coil and a 6
of 10mJ, what is the energy stored in the magnetic field ? W resistor is corrected to the secondary coil. What is the
(a) 10 mJ (b) 150 mJ (c) 160 mJ (d) 170 mJ current in the primary coil?
119. In an oscillating LC circuit with L = 50 mH and (a) 0.10 A (b) 0.14 A (c) 2 A (d) 40 A
C = 4.0 mF, the current is initially a maximum. How long will 127. The primary and secondary coil of a transformer have 50
it take before the capacitor is fully discharged for the first and 1500 turns respectively. If the magnetic flux f linked
time : with the primary coil is given by f = f0 + 4t, where f is in
(a) 7 × 10–4 s (b) 14 × 10–4 s webers, t is time in seconds and f0 is a constant, the output
(c) 28 × 10 s –4 (d) none voltage across the secondary coil is
120. In an oscillation of L-C circuit, the maximum charge on the (a) 120 volts(b) 220 volts(c) 30 volts (d) 90 volts
capacitor is Q. The charge on the capacitor, when the energy 128. A step down transformer is connected to 2400 volts line and
is stored equally between the electric and magnetic field is 80 amperes of current is found to flow in output load. The
Q Q Q Q ratio of the turns in primary and secondary coil is 20 : 1. If
(a) (b) (c) (d)
2 2 3 3 transformer efficiency is 100%, then the current flowing in
121. Figure shows three oscillating the primary coil will be
LC circuit with identical (a) 1600 amp (b) 20 amp (c) 4 amp (d) 1.5 amp
inductors and capacitors. If t1, 129. A transformer having efficiency of 90% is working on 200V
t2, t3 are the time taken by the and 3kW power supply. If the current in the secondary coil
circuits I, II, III for fully is 6A, the voltage across the secondary coil and the current
discharge, then
in the primary coil respectively are :
(a) t1 > t2 > t3 (b) t1 < t2 < t3
(a) 300 V, 15A (b) 450 V, 15A
(c) t2 < t1 < t3 (d) t3 = t1t2 (c) 450V, 13.5A (d) 600V, 15A
122. A 220 volts input is supplied to a transformer. The output 130. An ideal efficient transformer has a primary power input of
circuit draws a current of 2.0 ampere at 440 volts. If the 10 kW. The secondary current when the transformer is on
efficiency of the transformer is 80%, the current drawn by load is 25 A. If the primary: secondary turns ratio is 8 : 1,
the primary windings of the transformer is then the potential difference applied to the primary coil is
(a) 3.6 ampere (b) 2.8 ampere 104 ´ 82 104 ´ 8 104 104
(c) 2.5 ampere (d) 5.0 ampere (a) V (b) V (c) V (d) V
25 25 25 ´ 8 25 ´ 82
EBD_7418
320 PHYSICS

Exercise 2 : Exemplar & Past Year MCQs


NCERT Exemplar MCQs 9. An inductance L having a resistance R is connected to an
alternating source of angular frequency w. The Quality factor
1. If the rms current in a 50 Hz AC circuit is 5 A, the value of the
Q of inductance is [AIIMS 2014, C]
current 1/300 s after its value becomes zero is
(a) 5Ö2A (b) 5Ö3/2 A (c) 5/6 A (d) 5/Ö2A (a) R/ wL (b) (wL/R)2 (c) (R /wL)½ (d) wL/R
2. An alternating current generator has an internal reactance 10. In the circuit shown here, the point ‘C’ is kept connected to
Rg and an internal reactance Xg. It is used to supply power point ‘A’ till the current flowing through the circuit becomes
to a passive load consisting of a resistance Rg and a reactance constant. Afterward, suddenly, point ‘C’ is disconnected
XL. For maximum power to be delivered from the generator from point ‘A’ and connected to point ‘B’ at time t = 0. Ratio
to the load, the value of XL is equal to of the voltage across resistance and the inductor at t = L/R
(a) zero (b) Xg (c) –Xg (d) Rg will be equal to: [JEE Main 2014, A]
3. When a voltage measuring device is connected to AC mains, A C R
e
the meter shows the steady input voltage of 220 V. This (a) (b) 1
means 1- e L
(a) input voltage cannot be AC voltage, but a DC voltage
1- e B
(b) maximum input voltage is 220 V (c) –1 (d)
(c) the meter reads not v but < v2 > and is calibrated to e
read < v 2 > 11. An inductor 20 × 10–3 Henry, a capacitor 100µF and a resis-
(d) The pointer of the meter is stuck by some mechanical tor 50W are connected in series across a source of EMF V =
defect 10 sin 314t. If resistance is removed from the
4. To reduce the resonant frequency in an L-C-R series circuit circuit and the value of inductance is doubled, then the
with a generator variation of current with time in the new circuit is –
(a) the generator frequency should be reduced [BITSAT 2014, A]
(b) another capacitor should be added in parallel to the (a) 0.52 cos 314 t (b) 0.52 sin 314 t
first
(c) 0.52 sin (314 t + p/3) (d) None of these
(c) the iron core of the inductor should be removed 12. A series R-C circuit is connected to an alternating voltage
(d) dielectric in the capacitor should be removed source. Consider two situations:
5. Which of the following combinations should be selected (a) When capacitor is air filled.
for better tuning of an L-C-R circuit used for communication? (b) When capacitor is mica filled.
(a) R = 20 W, L =1.5 H, C = 35µF Current through resistor is i and voltage across capacitor is
(b) R=25W, L = 2.5 H, C = 45 µF V then : [AIPMT 2015, C]
(c) R =15W, L = 3.5 H, C = 30 µF (a) Va > Vb (b) ia > ib (c) Va = Vb (d) Va < Vb
(d) R = 25W, L = 1.5 H, C = 45 µF 13. A step down transformer is connected to 2400 volts line and
6. An inductor of reactance 1W and a resistor of 2W are 80 amperes of current is found to flow in output load. The
connected in series to the terminals of a 6V (rms) AC source.
ratio of the turns in primary and secondary coil is 20 : 1. If
The power dissipated in the circuit is
transformer efficiency is 100%, then the current flowing in
(a) 8 W (b) 12 W (c) 14.4 W (d) 18 W
7. The output of a step-down transformer is measured to be 24 the primary coil will be [AIIMS 2015, A]
V when connected to a 12 W light bulb. The value of the (a) 1600 amp (b) 20 amp
peak current is (c) 4 amp (d) 1.5 amp
(a) 1 / 2 A (b) 2A (c) 2 A (d) 2 2 A 14. The tuning circuit of a radio receiver has a resistance of
50W, an inductor of 10 mH and a variable capacitor. A 1 MHz
Past Year MCQs radio wave produces a potential difference of 0.1 mV. The
8. A transformer having efficiency of 90% is working on 200V values of the capacitor to produce resonance is (Take
and 3kW power supply. If the current in the secondary coil p2 = 10) [AIIMS 2015, A]
is 6A, the voltage across the secondary coil and the current (a) 2.5 pF (b) 5.0 pF (c) 25 pF (d) 50 pF
in the primary coil respectively are : [AIPMT 2014, A] 15. An inductor (L = 0.03 H) and a resistor (R = 0.15 kW) are
(a) 300 V, 15A (b) 450 V, 15A connected in series to a battery of 15V EMF in a circuit
(c) 450V, 13.5A (d) 600V, 15A shown below. The key K1 has been kept closed for a long
time. Then at t = 0, K1 is opened and key K2 is closed
ALTERNATING CURRENT 321

simultaneously. At t = l ms, the current in the circuit will be (c) Assertion is correct, reason is incorrect
: (e5 @ 150) [JEE Main 2015, A] (d) Assertion is incorrect, reason is correct.
0.03 H 0.15 kW 23. An arc lamp requires a direct current of 10 A at 80 V to
(a) 6.7 mA (b) 0.67 mA
function. If it is connected to a 220 V (rms), 50 Hz AC supply,
(c) 100 mA (d) 67 mA K2 the series inductor needed for it to work is close to :
[JEE Main 2016, A]
K1 (a) 0.044 H (b) 0.065 H (c) 80 H (d) 0.08 H
15V
16. A direct current of 5A is superposed on an alternating 24. An inductor of inductance L = 400 mH and resistors of
current I = 10 sin wt flowing through the wire. The effective resistance R1 = 2W and R2 = 2W are connected to a battery
of emf 12 V as shown in the figure. The internal resistance of
value of the resulting current will be [BITSAT 2015, A]
the battery is negligible. The switch S is closed at t = 0. The
(a) (15/2)A (b) 5 3A (c) 5 5A (d) 15 A potential drop across L as a function of time is
17. In the circuit shown below, the ac source has voltage V = 20 [BITSAT 2016, S]
cos(wt) volt with w = 2000 rad/s. The amplitude of the current 12 -3t E
will be nearest to [BITSAT 2015, A] (a) e V L
t
R1
(a) 2A
(b) 3.3A (b) ( )
6 1 - e-t / 0.2 V
R2
(c) 2 / 5A (c) 12e–5t V
(d) 6e–5t V S
(d) 5A
25. A resistor of resistance R, capacitor of capacitance C and
18. A small signal voltage V(t) = V0 sin wt is applied across an inductor of inductance L are connected in parallel to AC
ideal capacitor C : [NEET 2016, C]
power source of voltage e0 sin wt . The maximum current
(a) Current I (t), lags voltage V(t) by 90°.
through the resistance is half of the maximum current through
(b) Over a full cycle the capacitor C does not consume
the power source. Then value of R is [BITSAT 2016, A]
any energy from the voltage source.
(c) Current I (t) is in phase with voltage V(t). 3 1
(a) (b) 3 – wL
(d) Current I (t) leads voltage V(t) by 180°. 1 wC
wC –
19. An inductor 20 mH, a capacitor 50 mF and a resistor 40W are wL
connected in series across a source of emf V = 10 sin 340 t. 1
The power loss in A.C. circuit is : [NEET 2016, A] (c) 5 – wL (d) None of these
wC
(a) 0.51 W (b) 0.67 W (c) 0.76 W (d) 0.89 W 26. Figure shows a circuit that contains three identical resistors
20. A coil has resistance 30 ohm and inductive reactance 20 ohm with resistance R = 9.0 W each, two identical inductors with
at 50 Hz frequency. If an ac source, of 200 volt, 100 Hz, is inductance L = 2.0 mH each, and an ideal battery with emf e
connected across the coil, the current in the coil will be = 18 V. The current 'i' through the battery just after the switch
[AIIMS 2016, A] closed is [NEET 2017, C]
20
(a) 4.0 A (b) 8.0 A (c) A (d) 2.0 A L R R
13 +
e

21. An ideal coil of 10H is connected in series with a resistance
R L
of 5W and a battery of 5V. After 2 sec the connection is C

made, the current flowing in ampere in the circuit is


[AIIMS 2016, A] (a) 0.2 A (b) 2 A (c) 0 (d) 2 mA
(a) (1 – e–1) (b) (1 – e) 27. An inductor, a resistor and a capacitor are joined in series
(c) e (d) e–1 with an AC source. As the frequency of the source is slightly
22. Assertion : In the purely resistive element of a series LCR, increased from a very low value, the reactance of the
AC circuit the maximum value of rms current increases with [AIIMS 2017, C]
increase in the angular frequency of the applied emf. (a) inductor increases
Reason : (b) resistor increases
(c) capacitor increases
2
e max æ 1 ö (d) circuit increases
Imax = , z = R 2 + ç wL - ÷ ,
z è wC ø 28. Assertion : A laminated core is used in transformers to
where Imax is the peak current in a cycle. [AIIMS 2016, C] increase eddy currents.
(a) Assertion is correct, reason is correct; reason is a Reason : The efficiency of a transformer increases with
correct explanation for assertion increase in eddy currents. [AIIMS 2017, C]
(b) Assertion is correct, reason is correct; reason is not a (a) Assertion is correct, reason is correct; reason is a
correct explanation for assertion correct explanation for assertion
EBD_7418
322 PHYSICS

(b) Assertion is correct, reason is correct; reason is not a 33. In an a.c. circuit, the instantaneous e.m.f. and current are
correct explanation for assertion given by e = 100 sin 30 t
(c) Assertion is correct, reason is incorrect æ pö
(d) Assertion is incorrect, reason is correct. i = 20 sin ç 30 t - ÷ [JEE Main 2018, C]
è 4ø
29. A coil of inductance 300 mH and resistance 2W is connected
to a source of voltage 2 V. The current reaches half of its In one cycle of a.c., the average power consumed by the
steady state value in [BITSAT 2017, A] circuit and the wattless current are, respectively:
(a) 0.1 s (b) 0.05 s (c) 0.3 s (d) 0.15 s 1000
30. An inductor 20 mH, a capacitor 100 mF and a resistor 50W (a) 50W, 10A (b) W, 10A
2
are connected in series across a source of emf, V = 10 sin 50
314 t. The power loss in the circuit is [NEET 2018, A] (c) W, 0 (d) 50W, 0
(a) 0.79 W (b) 0.43 W (c) 1.13 W (d) 2.74 W 2
31. In a series RC circuit having battery of 12V, capacitor is 34. For an RLC circuit driven with voltage of amplitude vm and
changed from 0 to 6V in 0.1s find value of resistance R. 1
frequency w0 = the current exhibits resonance. The
[AIIMS 2018, A] LC
0.1 0.2 quality factor, Q is given by: [JEE Main 2018, C]
(a) Cln ( 2) (b) Cln ( 2) w0 L w0 R R CR
(a) (b) (c) (w C) (d) w
0.1 0.2 R L 0 0
(c) Cln ( 4) (d) Cln 4
( ) 35. A resistor and an inductor are connected to an ac supply of
32. In a series LR circuit (L = 3H, R = 1.5W) and DC voltage = 1V. 120 V and 50 Hz. The current in the circuit is 3 A. If the
Find current at t = 2 seconds. power consumed in the circuit is 108 W, then the resistance
[AIIMS 2018, A] in the circuit is [BITSAT 2018, A]
(a) 0.4A (b) 0.6A (c) 0.8A (d) 0.9A (a) 12 W (b) 40 W
(c) ( 52 ´ 25)W (d) 360 W

Exercise 3 : Try If You Can


1. A sinusoidal voltage of peak value 283 V and angular 4. In an LCR circuit shown in the following figure, what will be
frequency 320/s is applied to a series LCR circuit. Given that the readings of the voltmeter across the resistor and
R = 5 W, L= 25 mH and C = 1000 mF. The total impedance, and ammeter if an a.c. source of 220V and 100 Hz is connected to
phase difference between the voltage across the source it as shown? L C 100 W
and the current will respectively be : (a) 800 V, 8 A

(a) 10 W and tan–1 æç ö÷


5 (b) 110 V, 1.1 A
(b) 7 W and 45° V V V A
è 3ø (c) 300 V, 3 A 300V 300 V VR

æ8ö
(c) 10 W and tan –1 ç ÷ (d) 7 W and tan–1 æç ö÷
5 (d) 220V, 2.2 A
220 V, 100 Hz
è3ø è 3ø 5. Consider the RLC circuit shown below connected to an AC
2. When the rms voltages VL, VC and VR are measured source of constant peak voltage V0 and variable frequency
respectively across the inductor L, the capacitor C and the w0. The value of L is 20 mH. For a certain value w0 = w1, rms
resistor R in a series LCR circuit connected to an AC source, voltage across L, C, R are shown in the diagram. At w0 = w2,
it is found that the ratio VL : VC : VR = 1 : 2 : 3. If the rms voltage it is found that rms voltage across resistance is 50V. Then
of the AC sources is 100 V, the VR is close to: the value of w2 is C R
(a) 50 V (b) 70 V (c) 90 V (d) 100 V
3
3. A series LR circuit is connected to an ac source of frequency (a) w1 100V 60V 30V
w and the inductive reactance is equal to 2R. A capacitance 5
of capacitive reactance equal to R is added in series with 5
L and R. The ratio of the new power factor to the old one (b) w1
3
is :
2 2 3 5 (c) 50 Hz
(a) (b) (c) (d)
3 5 2 2 (d) cannot be calculated from given data
ALTERNATING CURRENT 323

6. In an alternating current circuit in which an inductance and


capacitance are joined in series, current is found to be
maximum when the value of inductance is 0.5 henry and the
value of capacitance is 8µF. The angular frequency of applied
alternating voltage will be
(a) 5000 rad/sec (b) 4000 rad/sec
(c) 2 × 105 rad/sec (d) 500 rad/sec
7. In a series L-C-R circuit, C = 10–11 Farad, L = 10–5 Henry and
R = 100 Ohm, when a constant D.C. voltage E is applied to
the circuit, the capacitor acquires a charge 10–9 C. The D.C.
2 4
source is replaced by a sinusoidal voltage source in which (a) (r – r ) (b) (r – r )
the peak voltage E0 is equal to the constant D.C. voltage E. 3 1 2 3 1 2
At resonance the peak value of the charge acquired by the
3 3
capacitor will be : (c) (r – r ) (d) (r – r )
(a) 10–15 C (b) 10–6 C (c) 10–10 C (d) 10–8 C 4 2 1 5 2 1
8. A small circular loop of wire of radius a is located at the 12. In given RC circuit, capacitance of capacitor C1= 3mF and
centre of a much larger circular wire loop of radius b. The C2 = 1mF. It is given that time constant of circuit between A
two loops are in the same plane. The outer loop of radius b and B is 3 millisecond. Value of R will be
carries an alternating current I = Io cos (wt). The emf induced (a) 1W A
in the smaller inner loop is nearly : C2 R
(b) 10W C1
2 2
pm0 Io a pm0 Io a C1
(a) . w sin (wt) (b) . w cos (wt) C2 B
2 b 2 b (c) 100W C2

a2 pm0 I o b 2 (d) 1000W 1 C R


(c) pm0 Io w sin (wt) (d) w cos ( wt)
b a 13. An ac source of angular frequency w is fed across a resistor
9. In the circuit shown below, the key K is closed at t = 0. The r and a capacitor C in series. The current registered is I. If
current through the battery is now the frequency of source is changed to w/3 (but main-
V K
taining the same voltage), the current in the circuit is found
to be halved. Calculate the ratio of reactance to resistance
L R1
at the original frequency w

R2
3 2 1 4
(a) (b) (c) (d)
5 5 5 5
VR1R2 V 14. An ac generator G with an adjustable frequency of oscillation
(a) at t = 0 and R at t = ¥
R12 + R22 2
is used in the circuit, as shown.
L1=1.6mH R=100W
V V ( R1 + R2 )
(b) R2 at t = 0 and R1 R2 at t = ¥

VR1R2 G ~ C1=3µF
V C2= C3=
(c) R2 at t = 0 and at t = ¥ 4.5µF 2.5µF
R12 + R22
V ( R1 + R2 ) V L2=2.4mH
(d) R1 R2 at t = 0 and R at t = ¥ Current drawn from the ac source will be maximum if its angular
2
10. In a series LCR circuit R = 200W and the voltage and the frequency is
frequency of the main supply is 220V and 50 Hz respectively. (a) 105 rad/s (b) 104 rad/s
On taking out the capacitance from the circuit the current (c) 5000 rad/s (d) 500 rad/s
lags behind the voltage by 30°. On taking out the inductor 15. A coil has an inductance of 0.7 henry and is joined in series
from the circuit the current leads the voltage by 30°. The with a resistance of 220 W. When the alternating emf of 220
power dissipated in the LCR circuit is V at 50 Hz is applied to it then the phase through which
(a) 305 W (b) 210 W (c) Zero W (d) 242 W current lags behind the applied emf and the wattless
11. In the figure both cells A and B are of equal emf. Find R for component of current in the circuit will be respectively
which potential difference across battery A will be zero, (a) 30°, 1 A (b) 45°, 0.5 A
long time after the switch is closed. Internal resistance of (c) 60°, 1.5 A (d) 30°, 1.5 A
batteries A and B are r1 and r2 respectively (r1 > r2).
EBD_7418
324 PHYSICS

ANSWER KEYS
Exercise 1 : Topic-wise MCQs
1 (d) 14 (d) 27 (c) 40 (b) 53 (d) 66 (c) 79 (b) 92 (b) 105 (c) 118 (b)
2 (b) 15 (b) 28 (a) 41 (d) 54 (a) 67 (b) 80 (c) 93 (a) 106 (a) 119 (a)
3 (a) 16 (d) 29 (d) 42 (a) 55 (d) 68 (b) 81 (d) 94 (a) 107 (b) 120 (b)
4 (b) 17 (a) 30 (b) 43 (b) 56 (d) 69 (a) 82 (d) 95 (c) 108 (b) 121 (c)
5 (b) 18 (d) 31 (a) 44 (d) 57 (d) 70 (d) 83 (a) 96 (c) 109 (a) 122 (d)
6 (d) 19 (a) 32 (a) 45 (d) 58 (d) 71 (b) 84 (c) 97 (c) 110 (b) 123 (a)
7 (a) 20 (c) 33 (b) 46 (b) 59 (c) 72 (a) 85 (c) 98 (d) 111 (b) 124 (a)
8 (d) 21 (d) 34 (c) 47 (b) 60 (d) 73 (d) 86 (d) 99 (b) 112 (b) 125 (c)
9 (b) 22 (d) 35 (b) 48 (c) 61 (c) 74 (b) 87 (b) 100 (a) 113 (d) 126 (a)
10 (b) 23 (d) 36 (a) 49 (b) 62 (c) 75 (c) 88 (a) 101 (a) 114 (a) 127 (a)
11 (b) 24 (d) 37 (c) 50 (b) 63 (d) 76 (b) 89 (d) 102 (c) 115 (d) 128 (c)
12 (b) 25 (a) 38 (d) 51 (b) 64 (c) 77 (b) 90 (b) 103 (b) 116 (a) 129 (b)
13 (a) 26 (d) 39 (b) 52 (c) 65 ( a) 78 (a) 91 (c) 104 (a) 117 (a) 130 (b)
Exercise 2 : Exemplar & Past Year MCQs
1 (b) 5 (c) 9 (d) 13 (c) 17 (a) 21 (a) 25 (a) 29 (a) 33 (b)
2 (c) 6 (c) 10 (c) 14 (a) 18 (b) 22 (c) 26 (b) 30 (a) 34 (a)
3 (c) 7 (a) 11 (a) 15 (b) 19 (a) 23 (b) 27 (a) 31 (a) 35 (a)
4 (b) 8 (b) 12 (a) 16 (b) 20 (a) 24 (c) 28 (d) 32 (a)
Exercise 3 : Try If You Can
1 (b) 3 (d) 5 (a) 7 (d) 9 (c) 11 (b) 13 (a) 15 (b)
2 (c) 4 (d) 6 (d) 8 (a) 10 (d) 12 (d) 14 (c)
23Chapter
ELECTROMAGNETIC WAVES

Trend
Analysis of NEET and AIIMS (Year 2010-2018)
5

4
Number of Questions

3
AIPMT/NEET
2 AIIMS

0
2010 2011 2012 2013 2014 2015 2016 2017 2018

Year

Trend
Analysis of JEE Main and BITSAT (Year 2010-2018)
5

4
Number of Questions

3
JEE Main
2 BITSAT

0
2010 2011 2012 2013 2014 2015 2016 2017 2018

Year

Chapter Utility Score (CUS)


Exam Weightage Important Concepts Difficulty Level CUS (Out of 10)
NEET 3 Electromagnetic Waves-Their
AIIMS 4 Characteristics & Applications 2/5 4/10
JEE Main 2 and Spectrum of EM Waves
BITSAT »0
EBD_7418
326 PHYSICS
ELECTROMAGNETIC WAVES 327

Exercise 1 : Topic-wise MCQs

Topic 1: Electromagnetic Waves, Conduction 10. If the frequency of EM radiations is halved then the energy
of EM radiation will become
and Displacement Current
(a) double (b) remains unchanged
1. The electromagnetic waves do not transport (c) becomes half (d) becomes one fourth
(a) energy (b) charge 11. Select the wrong statement. EM waves
(c) momentum (d) information (a) are transverse in nature.
(b) travel in free space at a speed of light.
2. In an electromagnetic
r wave, the direction of the magnetic
(c) are produced by accelerating charges.
induction B is r (d) travel in all media with same speed.
(a) parallel to the electric field E
r 12. Which of the following has/have zero average value in a
(b) perpendicular to the electric field Er plane electromagnetic wave?
(c) antiparallel to the Poynting vector S (a) Both magnetic and electric field
(d) random (b) Electric field only
3. The polarisation of electromagnetic wave is in (c) Magnetic energy
(a) the directions of electric and magnetic field (d) Electric energy
(b) the directions of electric field 13. The electromagnetic waves
(c) the direction of magnetic field (a) travel with the speed of sound
(d) can not be polarized (b) travel with the same speed in all media
4. The current in the electric circuit which arises due to flow (c) travel in free space with the speed of light
of electrons in the connecting wires of the circuit in a
(d) do not travel through a medium
defined closed path is called
14. An electromagnetic wave propagating along north has its
(a) alternating current (b) direct current
electric field vector upwards. Its magnetic field vector
(c) conduction current (d) displacement current
point towards
5. If a variable frequency ac source connected to a capacitor
(a) north (b) east
then with decrease in frequency, the displacement current
(c) west (d) downwards
will
15. According to Maxwell’s hypothesis, a changing electric
(a) increase
field gives rise to
(b) decrease
(a) an e.m.f
(c) remain constant
(b) electric displacement current
(d) first decrease then increase
(c) magnetic field
6. An instantaneous displacement current of 1 A can be set up
(d) pressure gradient
by changing the potential difference across the parallel plates
16. A plane electromagnetic wave is incident on a material
dV
capacitor (1.0 mF) at the rate of . surface. If the wave delivers momentum p and energy E, then
dt
dV (a) p = 0, E = 0 (b) p ¹ 0, E ¹ 0
here is (c) p ¹ 0, E = 0 (d) p = 0, E ¹ 0
dt
(a) 106 Vs–1 (b) 103 Vs–1 (c) 102 Vs–1 (d) 101 Vs–1 17. The oscillating electric and magnetic vectors of an
7. Displacement current goes through the gap between the electromagnetic wave are oriented along
plates of a capacitor when the charge on the capacitor (a) the same direction but differ in phase by 90°
(a) is changing with time (b) decreases (b) the same direction and are in phase
(c) does not change (d) decreases to zero (c) mutually perpendicular directions and are in phase
8. An electron oscillating with a frequency of 3 × 106 Hz, would (d) mutually perpendicular directions and differ in phase by
generate 90°
(a) X-rays (b) ultraviolet rays 18. When electromagnetic waves enter the ionised layer of
(c) radio waves (d) microwaves ionosphere, then the relative permittivity i.e. dielectric
9. In an electromagnetic wave constant of the ionised layer
(a) power is transmitted along the magnetic field
(a) does not change
(b) power is transmitted along the electric field
(b) appears to increase
(c) power is equally transferred along the electric and
magnetic fields (c) appears to decrease
(d) power is transmitted in a direction perpendicular to (d) sometimes appears to increase and sometimes to
both the fields decrease
EBD_7418
328 PHYSICS

19. Displacement current is 27. A beam has intensity 2.5 × 1014 Wm–2. The ratio of electric
(a) continuous when electric field is changing in the and magnetic fields in the beam is
circuit (a) 2.98 × 108 NC–1T–1 (b) 8.32 × 105 NCT–1
(c) 6.22 × 10 NC T7 –1 –1 (d) 2.88 × 106 NCT–1
(b) continuous when magnetic field is changing in the
circuit 28. An electromagnetic wave passes through space and its
(c) continuous in both types of fields equation is given by E = E0 sin (wt – kx) where E is electric
(d) continuous through wires and resistance only field. Energy density of electromagnetic wave in space is
20. Maxwell’s modified form of Ampere’s circuital law is 1 1
® ® ® ® (a) e 0 E 20 (b) e 0 E 20 (c) e 0 E 02 (d) 2e 0 E 02
2 4
(a) ò B . ds = 0 (b) ò B . dl = m 0I 29. A plane electromagnetic wave is incident on a plane surface
s s of area A, normally and is perfectly reflected. If energy E
® ® ® ® d fE
1 dq
(c) ò B . dl = m 0 I + (d) ò B . dl = m 0 I + m 0 e o strikes the surface in time t then average pressure exerted
e o dt s
dt on the surface is (c = speed of light)
s
21. The electric field of an electromagnetic wave travelling through
(a) zero (b) E/Atc (c) 2E/Atc (d) E/c
vaccum is given by the equation E = E0 sin (kx – wt). The
30. The pressure exerted by an electromagnetic wave of
quantity that is independent of wavelength is
intensity I (watts/m2) on a nonreflecting surface is [c is the
k velocity of light]
(a) kw (b) (c) k2 w (d) w
w (a) Ic (b) Ic2 (c) I/c (d) I/c2
22. When an electromagnetic waves enter the ionised layer of 31. An electromagnetic wave with frequency w and wavelength
ionosphere, the motion of electron cloud produces a space l travels in the + y direction. Its magnetic field is along + x-
current and the electric field has its own capacitative axis. The vector equation for the associated electric field (of
displacement current, then amplitude E0) is
(a) the space current is in phase of displacement current ® æ 2p ö
(b) the space current lags behind the displacement current (a) E = - E0 cos ç wt + y ÷ xˆ
è l ø
by a phase 180°. ® 2p ö
æ
(c) the space current lags behind the displacement current (b) E = E0 cos ç wt - y ÷ xˆ
è l ø
by a phase 90°. ® æ 2p ö
(d) the space current leads the displacement current by a (c) E = E0 cos ç wt - y÷ zˆ
è l ø
phase 90°.
® æ 2p ö
23. The figure shows graphs of the electric E = - E0 cos ç wt + y÷ zˆ
(d) è
field magnitude E versus time t for four l ø
uniform electric fields, all contained within 32. An electromagnetic wave going through vacuum is
identical circular regions. Which of them described by E = E0sin(kx – wt); B = B0 sin (kx – wt). Which
is according to the magnitudes of the of the following equations is true?
magnetic field? (a) E0 k = B0w (b) E0 w = B0 k
(a) A (b) B (c) C (d) D (c) E 0 B0 = wk (d) None of these
24. Assertion : The velocity of electromagnetic waves depends 33. A plane electromagnetic wave travels in free space along
on electric and magnetic properties of the medium. x-axis. At a particular point in space, the electric field along
Reason : Velocity of electromagnetic waves in free space y-axis is 9.3V m–1. The magnetic induction (B) along z-axis is
is constant. (a) 3.1 × 10–8 T (b) 3 × 10–5 T
(a) Assertion is correct, reason is correct; reason is a correct (c) 3 × 10 T –6 (d) 9.3 × 10–6 T
explanation for assertion. 34. The electric and the magnetic field associated with an E.M.
(b) Assertion is correct, reason is correct; reason is not a wave, propagating along the +z-axis, can be represented by
r r r r r
correct explanation for assertion (a) éë E = E 0i,ˆ B = B ˆjù (b) éë E = E 0 k, B = B0ˆi ùû
0 û
(c) Assertion is correct, reason is incorrect r r r r
(c) éë E = E 0 ˆj, B = B0iˆùû (d) éë E = E 0 ˆj,B = B0 kˆ ùû
(d) Assertion is incorrect, reason is correct.
25. The magnetic field in a travelling electromagnetic wave has a 35. In an apparatus, the electric field was found to oscillate with
peak value of 20 nT. The peak value of electric field strength an amplitude of 18 V/m. The magnitude of the oscillating
is magnetic field will be
(a) 3 V/m (b) 6 V/m (c) 9 V/m (d) 12 V/m (a) 4 × 10–6 T (b) 6 × 10–8 T
(c) 9 × 10 T –9 (d) 11 × 10–11 T
26. In a plane electromagnetic wave propagating in space has
an electric field of amplitude 9 × 103 V/m, then the amplitude 36. A wave is propagating in a medium of electric dielectric
of the magnetic field is constant 2 and relative magnetic permeability 50. The wave
(a) 2.7 × 1012 T (b) 9.0 × 10–3 T impedance of such a medium is
(c) 3.0 × 10 T –4 (d) 3.0 × 10–5 T (a) 5 W (b) 376.6 W (c) 1883 W (d) 3776 W
ELECTROMAGNETIC WAVES 329

37. If a source is transmitting electromagnetic wave of frequency 45. Match the columns I and II.
8.2 × 106 Hz, then wavelength of the electromagnetic waves Column I Column II
transmitted from the source will be 1
(a) 36.6 m (b) 40.5 m (c) 42.3 m (d) 50.9 m (A) Energy associated with 1.
me
38. In a plane em wave, the electric field oscillates sinusoidally an electromagnetic wave
at a frequency 2 × 1010 Hz and amplitude 48 Vm–1, then the
amplitude of oscillating magnetic field is u
(B) Radiation pressure 2.
(a) 16 × 10–8 T (b) 12 × 10–6 T c
(c) 11 × 10 T–5 (d) 6.6 × 10–3 T
1 1 B2
39. When an electromagnetic wave enters an ionised layer of (C) Speed of EM wave in 3. e0 E 2 +
earth’s atmosphere present in ionosphere 2 2 m0
(a) the electron cloud will not oscillate in the electric field a medium
of the wave dfE
(b) the electron cloud will oscillate in the electric field of (D) Displacement current 4. ID = e 0
dt
wave in the phase of sinusoidal electromagnetic wave (a) (A) ® (3) ; (B) ® (2) ; (C) ® (1) ; (D) ® (4)
(c) the electron cloud will oscillate in the electric field of (b) (A) ® (2) ; (B) ® (2) ; (C) ® (4) ; (D) ® (3)
wave in the opposite phase of sinusoidal (c) (A) ® (4) ; (B) ® (3) ; (C) ® (2) ; (D) ® (1)
electromagnetic wave (d) (A) ® (2) ; (B) ® (1) ; (C) ® (4) ; (D) ® (3)
(d) the electron cloud will oscillate in the electric field of 46. The amplitude of an electromagnetic wave in vacuum is
wave with a phase retardation of 90° for a sinusoidal doubled with no other changes made to the wave. As a
result of this doubling of the amplitude, which of the
electromagnetic wave.
following statements are incorrect?
40. In an electromagnetic wave, the electric and magnetic fields I. The speed of wave propagation chages only
are 100 V m–1 and 0.265 A m–1. The maximum energy flow is II. The frequency of the wave changes only
(a) 26.5 W/m2 (b) 36.5 W/m2 III. The wavelength of the wave changes only
(c) 46.7 W/m 2 (d) 765 W/m2 (a) I and II (b) II and III
41. A new system of unit is evolved in which the values of µ0 (c) I and III (d) I, II and III
and Î0 are 2 and 8 respectively. Then the speed of light in 47. Figure shows a parallel plate capacitor and the current in the
this system will be connecting wires that is discharging the capacitor.
(a) 0.25 (b) 0.5 (c) 0.75 (d) 1
42. An electromagnetic wave of intensity I falls on a surface
kept in vacuum and exerts radiation pressure P on it.
Which of the following statements are true?
I. Radiation pressure is I/c if the wave is totally
absorbed.
II. Radiation pressure is I/c if the wave is totally (a) The displacement current is leftward.
reflected.
(b) The displacement current is rightward
III. Radiation pressure is 2 I/c if the wave is totally r
(c) The electric field E is rightward
reflected.
(d) The magnetic field at point P is out the page.
(a) I and II (b) I and III
48. An electromagnetic wave of frequency u = 3 MHz passes
(c) III only (d) I, II and III
from vacuum into a dielectric medium with permittivity
43. The figure here gives the electric field
e = 4. Then
of an electromagnetic wave at a (a) wavelength and frequency both become half.
certain point and a certain instant. (b) wavelength is doubled and frequency remains
The wave is transporting energy in unchanged.
the negative z-direction. The (c) wavelength and frequncy both remain unchanged.
direction of the magnetic field of the (d) wavelength is halved and frequency remains
wave at that point and instant is unchanged.
(a) + ve x-direction (b) –ve x-direction 49. The transmitting antenna of a radiostation is mounted
(c) +ve z-direction (d) –ve y-direction vertically. At a point 10 km due north of the transmitter the
44. In order to establish an instantaneous displacemet current peak electric field is 10–3 Vm–1. The magnitude of the radiated
of 5 mA in the space between the plates of 5mF parallel magnetic field is .
plate capacitor, the potential difference need to apply is (a) 3.33 × 10–10 T (b) 3.33 × 10–12 T
–3
(c) 10 T (d) 3 × 105 T
(a) 100 Vs–1 (b) 200 Vs–1 (c) 300 Vs–1 (d) 1000 Vs–1
EBD_7418
330 PHYSICS

50. The electric field component of a monochromatic radiation 59. The magnetic field of a beam emerging from a filter facing a
ur by
is given floodlight is given by
E = 2 E0 $i cos
ur kz cos wt B0 = 12 × 10–8 sin(1.20×107z – 3.60 × 1015t) T.
Its magnetic field B is then given by :
What is the average intensity of the beam?
2Eo $ 2E
(a) j sin kz cos wt (b) - o $j sin kz sin wt (a) 1.72 × 102 W/m2 (b) 1.72 W/m2
2
c c (c) 2.31 W/m (d) 2.31 × 102 W/m2
2Eo $ 2Eo $ 60. A parallel plate capacitor with a and seperation d, is
(c) j sin kz sin wt (d) j cos kz cos wt
c c charged by a constant current I. Consider a plane surface
51. For plane electromagnetic waves propagating in the a
z-direction, which one of the following combination gives of area parallel to the plates and drawn symmetrically
ur ur 2
the correct possible direction for E and B field respectively? between the plates. What is the displacement current
through this area?
(a) (2$i + 3$j) and ($i + 2$j) (b) (-2$i - 3$j) and (3i$ - 2 $j)
(a) I/2 (b) I/3 (c) I/5 (d) I/7
(c) (3$i + 4 $j) and (4$i - 3$j) (d) ($i + 2$j) and (2i$ - $j) 61. Suppose that the electric field amplitude of an em wave is
52. A plane electromagnetic wave propagating in the X-direction Eo = 120 N/C and that its frequency is u =50 Hz, then
has wavelength of 6.0 mm. The electric field is in the (a) Bo= 200 T (b) w = r × 108 rad/s
Y-direction and its maximum magnitude is 33 Vm–1. The
equation for the electric field as a function of x and t is ( )
(c) k = p 4 rad / m (d) l = 92 m
62. Find the average energy density corresponding to
æ xö é æ x öù maximum electric field, if magnetic field in a plane
(a) 11sin p ç t – ÷ (b) 33sin êp´1011 ç t – ÷ú electromagnetic wave is given by
è cø ë è c øû
B = 200 × 10–6 sin[(4 × 1015)(t – x/c)]
æ xö é 11 æ x öù (a) 1.6 J m–3 (b) 0.16 J m–3
(c) 33sin p ç t – ÷ (d) 11sin êp´ 10 ç t – ÷ú (c) 0.016 J m–3 (d) 0.0016 J m–3
è cø ë è c øû
53. A circular ring of radius r is placed in a homogeneous 63. The amplitude of the electric field if the intensity of a plane
magnetic field perpendicular to the plane of the ring. The electromagnetic wave is given as 2.0 Wm –2 is
field B changes with time according to equation B = kt, (a) 38.8 NC–1 (b) 48.3 NC–1
where, k is a constant and t is the time). The electric field in (c) 50.2 NC–1 (d) 68.8 NC–1
the ring is:
kr kr kr k Topic 2: Electromagnetic Spectrum
(a) (b) (c) (d)
4 3 2 2r 64. The range of wavelength of the UV rays is
54. The magnetic field in the plane electromagnetic field is given (a) 10 Å to 100 Å (b) 109 Å to 1010 Å
by: By = 2 × 10–7 sin (0.5 × 103 z + 1.5 × 1011 t) T (c) 8,000 Å to 10,000Å (d) 10,000 Å to 15,000 Å
The expression for the electric field may be given by 65. Which of the following electromagnetic waves has the
(a) Ey = 2 × 10–7 sin(0.5 × 103 z + 1.5 × 1011 t) V/m longest wavelength?
(b) Ex = 2 × 10–7 sin(0.5 × 103 z + 1.5 × 1011 t) V/m (a) uv-rays (b) Visible light
(c) Ey = 60 sin(0.5 × 103 z + 1.5 × 1011 t) V/m (c) Radio waves (d) Microwaves
(d) Ex = 60 sin(0.5 × 103 z + 1.5 × 1011 t) V/m 66. Which of the following is of shortest wavelength?
55. The magnetic field between the plates of radius 12 cm (a) X-rays (b) g-rays
separated by distance of 4 mm of a parallel plate capacitor of
capacitance 100 pF along the axis of plates having (c) Microwaves (d) Radio waves
conduction current of 0.15 A is 67. The range of wavelength of visible light is
(a) zero (b) 1.5 T (c) 15 T (d) 0.15 T (a) 50 Å to 500 Å (b) 4000 Å to 8000 Å
56. An electric charge oscillating with a frequency of 1 kilo (c) 9000 Å to 25,000 Å (d) 15,000 Å to 25,000 Å
cycles/sec can radiate electromagnetic waves of wavelength 68. An accelerated electron would produce
(a) 800 km (b) 400 km (c) 300 km (d) 100 km (a) g-rays (b) b-rays
57. The energy contained in a cylinder of cross-section 10cm 2 (c) a-rays (d) e.m. waves
and length 50cm along the x-axis if electric field in an 69. Which of the following is the infrared wavelength?
electromagnetic wave is given by (a) 10–4 cm (b) 10–5 cm
E = 50 sin w(t – x/c) is –6
(c) 10 cm (d) 10–7 cm
(a) 1.2 × 10–10 J (b) 5.5 × 10–12 J 70. The wavelength of X-ray is of the order of
–10
(c) 2.3 × 10 J (d) 3.6 × 10–12 J (a) 1 metre (b) 1 cm
58. The charge on a parallel plate capacitor varies as q = q0 cos (c) 1 micron (d) 1 angstrom
2pvt. The plates are very large and close together (area = A, 71. Radio waves do not penetrate in the band of
separation = d). Neglecting the edge effects, the (a) ionosphere (b) mesosphere
displacement current through the capacitor is (c) troposphere (d) stratosphere
(a) 2pnq0 sin 2pnt (b) 3/5pnq0 sin pnt 72. What is the cause of “Green house effect”?
(c) 4pnq0 sin 2pnt (d) None of these (a) Infrared rays (b) Ultraviolet rays
(c) X-rays (d) Radio waves
ELECTROMAGNETIC WAVES 331

73. If vs, vx and vm are the speed of soft gamma rays, X-rays and (a) increases as we move from g-rays to radio waves
microwaves respectively in vacuum, then (b) decreases as we move from g-rays to radio waves
(a) vs > vx > vm (b) vs < vx < vm (c) is same for all of them
(c) vs > vx < vm (d) vs = vx = vm (d) None of these
74. The waves which are electromagnetic in nature are 86. It is possible to take pictures of those objects which are not
(a) sound waves and light waves fully visible to the eye using camera films sensitive to
(b) water waves and radio waves (a) ultraviolet rays (b) infrared rays
(c) light waves and X-rays (c) microwaves (d) radiowaves
87. The waves used in telecommunication are
(d) sound waves and water waves
(a) IR (b) UV
75. Assertion : The basic difference between various types of
(c) Microwave (d) Cosmic rays
electromagnetic waves lies in their wavelength or
88. The electromagnetic radiation used in food processing
frequencies. sterilizing agent is
Reason : Electromagnetic waves travel through vacuum (a) microwaves (b) UV rays
with the same speed. (c) gamma rays (d) radio waves
(a) Assertion is correct, reason is correct; reason is a correct 89. Various electromagnetic waves are given in column I and
explanation for assertion. various frequency ranges in column II. Match the two
(b) Assertion is correct, reason is correct; reason is not a columns.
correct explanation for assertion Column I Column II
(c) Assertion is correct, reason is incorrect (A) Radio waves (1) 1 × 1016 to 3 × 1021 Hz
(d) Assertion is incorrect, reason is correct. (B) g-rays (2) 1 × 109 to 3 × 1011 HZ
76. The ozone layer in the atmosphere absorbs (C) Microwaves (3) 3 × 1018 to 5 × 1022 Hz
(a) only the radiowaves (D) X-rays (4) 5 × 105 to 109 Hz.
(b) only the visible light (a) (A) ® (2) ; (B) ® (5) ; (C) ® (3) ; (D) ® (4)
(c) only the g-rays (b) (A) ® (2) ; (B) ® (2) ; (C) ® (4) ; (D) ® (3)
(d) X-rays and ultraviolet rays (c) (A) ® (4) ; (B) ® (3) ; (C) ® (2) ; (D) ® (1)
77. Which one of the following has the maximum energy? (d) (A) ® (2) ; (B) ® (1) ; (C) ® (4) ; (D) ® (3)
(a) Radio waves (b) Infrared rays 90. In electromagnetic spectrum, the frequencies g-rays, X-rays
(c) Ultraviolet rays (d) Micro waves and ultraviolet rays are denoted by n1, n2 and n3 respectively
78. Which one of the following has the shortest wavelength? then
(a) n1 > n2 > n3 (b) n1 < n2 < n3
(a) Infrared rays (b) Ultraviolet rays
(c) n1 > n2 < n3 (d) n1 < n2 > n3
(c) Microwaves (d) Gamma rays
79. The velocity of all radio waves in free space is 3 × 108 m/s. 91. Photons of an electromagnetic radiation has an energy
11 keV each. To which region of electromagnetic spectrum
The frequency of a radio wave of wavelength 150m is
does it belong?
(a) 20 kHz (b) 2 kHz (c) 2 MHz (d) 1 MHz
(a) X-ray region (b) Ultra violet region
80. Microwaves are electromagnetic waves with frequency in (c) Infrared region (d) Visible region
the range of 92. X-rays, gamma rays and microwaves travelling in vaccum
(a) Micro hertz (b) Giga hertz have
(c) Mega hertz (d) Hertz (a) same wavelength but different velocities
81. Ozone layer above earth’s atmosphere will not (b) same frequency but different velocities
(a) prevent infrared radiations from sun reaching earth. (c) same velocity but different wavelength
(b) prevent infra red radiations originated from earth from (d) same velocity and same frequency
escaping earth’s atmosphere. 93. Radio waves diffract around building, although light
waves do not. The reason is that radio waves
(c) prevent ultraviolet rays from sun. (a) travel with speed larger than c
(d) reflect back radio waves. (b) have much larger wavelength than light
82. The frequency of electromagnetic wave, which best suited (c) are not electromagnetic waves
to observe a particle of radii 3 × 10–4 cm is of the order of (d) None of these
(a) 1015 (b) 1014 (c) 1013 (d) 1012 94. Assertion : Infrared radiation plays an important role in
83. If l = 10 Å then it corresponds to maintaining the average temperature of earth.
Reason : Infrared radiations are sometimes referred to as
(a) infrared (b) microwaves
heat waves.
(c) ultraviolet (d) X-rays (a) Assertion is correct, reason is correct; reason is a correct
84. 10 cm is a wavelength corresponding to the spectrum of explanation for assertion.
(a) infrared rays (b) ultra-violet rays (b) Assertion is correct, reason is correct; reason is not a
(c) microwaves (d) g-rays correct explanation for assertion
85. The speed of electromagnetic wave in vacuum depends (c) Assertion is correct, reason is incorrect
upon the source of radiation. It (d) Assertion is incorrect, reason is correct.
EBD_7418
332 PHYSICS

95. A radar sends the waves towards a distant object and 99. Radio waves and visible light in vacuum have
receives the signal reflected by object. These waves are (a) same velocity but different wavelength
(a) sound waves (b) light waves (b) same frequency
(c) radio waves (d) microwaves (c) different velocity
96. It is possible to take pictures of those objects which are not (d) same wavelength
fully visible to the eye using camera films sensitive to 100. We consider the radiation emitted by the human body.
(a) ultraviolet rays (b) infrared rays Which one of the following statements is true?
(c) microwaves (d) radiowaves (a) The radiation emitted is in the infrared region.
97. Which of the following wave can not be produced by charges (b) The radiation is emitted only during the day.
accelerating in AC circuits having an inductor and capacitor? (c) The radiation is emitted during the summers and
(a) Radio wave (b) Microwave absorbed during winters.
(c) Infrared (d) None of these (d) The radiation is emitted lies in the ultraviolet region
98. The ozone layer converts the ultraviolet radiation to [which and hence is not visible.
is used to heat the atmosphere and the earth's surface.]
(a) Infrared (b) Visible ray
(c) g-ray (d) None of these

Exercise 2 : Exemplar & Past Year MCQs


NCERT Exemplar MCQs 6. The ratio of contributions made by the electric field and
magnetic field components to the intensity of an EM wave is
1. One requires 11 eV of energy to dissociate a carbon monoxide
(a) c : 1 (b) c2 : 1 (c) 1 : 1 (d) c :1
molecule into carbon and oxygen atoms. The minimum
7. An EM wave radiates outwards from a dipole antenna, with
frequency of the appropriate electromagnetic radiation to
E0 as the amplitude of its electric field vector. The electric
achieve the dissociation lies in
field E0 which transports significant energy from the source
(a) visible region (b) infrared region
falls off as
(c) ultraviolet region (d) microwave region
2. A linearly polarised electromagnetic wave given as 1 1
(a) 3 (b)
E = E0 ˆi cos ( kz - wt ) is incident normally on a perfectly r r2
reflecting infinite wall at z = a. Assuming that the material of 1
(c) (d) remains constant
the wall is optically inactive, the reflected wave will be given r
as 8. An electromagnetic wave travels in vacuum along z-direction
(a) E = E ˆi ( kz - wt ) (b) E = E iˆ cos ( kz + wt ) E = ( E1iˆ + E 2 ˆj) cos ( kz - wt ) . Choose the correct options
r 0 r 0
from the following
(c) Er = -E 0 iˆ cos ( kz + wt ) (d) Er = E 0ˆisin ( kz - wt ) (a) The associated magnetic field is given as
B = ( E1iˆ - E 2 ˆj) cos ( kz - wt )
3. Light with an energy flux of 20 W/cm2 falls on a non-reflecting 1
surface at normal incidence. If the surface has an area of 30 c
cm2, the total momentum delivered (for complete absorption) (b) The associated magnetic field is given as
B = ( E1iˆ - E 2 ˆj) cos ( kz - wt )
during 30 min is 1
(a) 36 × 10–5 kg-m/s (b) 36 × 10–4 kg-m/s c
(c) The given electromagnetic field is circularly polarised
(c) 108 × 104 kg-m/s (d) 1.08 × 107 kg-m/s
(d) The given electromagnetic wave is plane polarised
4. The electric field intensity produced by the radiations coming
from 100 W bulb at a 3 m distance is E. The electric field Past Year MCQs
intensity produced by the radiations coming from 50 W bulb
at the same distance is 9. Light with an energy flux of 25 × 104 Wm– 2 falls on a perfectly
E E reflecting surface at normal incidence. If the surface area is
(a) (b) 2E (c) (d) 2E 15 cm2, the average force exerted on the surface is:
2 2
5. If E and B represent electric and magnetic field vectors of the [AIPMT 2014, A]
(a) 1.25 × 10 N– 6 (b) 2.50 × 10– 6 N
electromagnetic wave, the direction of propagation of
electromagnetic wave is along (c) 1.20 × 10 N– 6 (d) 3.0 × 10– 6 N
(a) E (b) B (c) B × E (d) E × B
ELECTROMAGNETIC WAVES 333

10. Which of the following are not electromagnetic waves? (b) Assertion is correct, reason is correct; reason is not a
[AIIMS 2014, C] correct explanation for assertion
(a) cosmic rays (b) g-rays (c) Assertion is correct, reason is incorrect
(c) b-rays (d) X-rays. (d) Assertion is incorrect, reason is correct.
11. Assertion : Environmental damage has increased the amount 18. Arrange the following electromagnetic radiations per quantum
of ozone in the atmosphere. in the order of increasing energy : [JEE Main 2016, C]
Reason : Increase of ozone increases the amount of A : Blue light B : Yellow light
ultraviolet radiation on earth. [AIIMS 2014, C] C : X-ray D : Radiowave.
(a) Assertion is correct, reason is correct; reason is a (a) C, A, B, D (b) B, A, D, C
correct explanation for assertion. (c) D, B, A, C (d) A, B, D, C
(b) Assertion is correct, reason is correct; reason is not a 19. In an electromagnetic wave in free space the root mean
correct explanation for assertion square value of the electric field is Erms = 6V/m. The peak
(c) Assertion is correct, reason is incorrect value of the magnetic field is :- [NEET 2017, A]
(d) Assertion is incorrect, reason is correct. (a) 2.83 × 10–8 T (b) 0.70 × 10–8 T
12. During the propagation of electromagnetic waves in a (c) 4.23 × 10–8 T (d) 1.41 × 10–8 T
medium: [JEE Main 2014, C] 20. Electromagnetic wave consists of periodically oscillating
(a) Electric energy density is double of the magnetic electric and magnetic vectors
energy density. (a) in mutually perpendicular planes but vibrating with
(b) Electric energy density is half of the magnetic energy a phase difference of p
density. (b) in mutually perpendicular planes but vibrating with
(c) Electric energy density is equal to the magnetic energy p
a phase difference of
density. 2
(d) Both electric and magnetic energy densities are zero. (c) in randomly oriented planes but vibrating in phase
13. Match List - I (Electromagnetic wave type) with List - II (Its (d) in mutually perpendicular planes but vibrating in
association/application) and select the correct option from phase [AIIMS 2017, C]
the choices given below the lists: [JEE main 2014, C] 21. Assertion : Microwaves are better carrier of signals than
List 1 List 2 optical waves.
Reason : Microwaves move faster than optical waves.
1. Infrared waves (i) To treat muscular strain [AIIMS 2017, C]
2. Radio waves (ii) For broadcasting (a) Assertion is correct, reason is correct; reason is a
3. X-rays (iii) To detect fracture of bones correct explanation for assertion.
4. Ultraviolet rays (iv) Absorbed by the ozone (b) Assertion is correct, reason is correct; reason is not a
layer of the atmosphere correct explanation for assertion
1 2 3 4 (c) Assertion is correct, reason is incorrect
(a) (iv) (iii) (ii) (i) (d) Assertion is incorrect, reason is correct.
(b) (i) (ii) (iv) (iii) 22. An em wave is propagating in a medium with a velocity
(c) (iii) (ii) (i) (iv) r
(d) (i) (ii) (iii) (iv) V = Viˆ . The instantaneous oscillating electric field of this
14. The energy of the em waves is of the oder of 15 keV. To which em wave is along +y axis. Then the direction of oscillating
part of the spectrum does it belong? [AIPMT 2015, A] magnetic field of the em wave will be along [NEET 2018, C]
(a) Infra-red rays (b) Ultraviolet rays (a) –z direction (b) +z direction
(c) g-rays (d) X-rays (c) –x direction (d) –y direction
15. Light wave isr travelling along y-direction. If the 23. In which one of the following regions of the electromagnetic
spectrum will the vibrational motion of molecules give rise to
corresponding r E vector at any time is along the x-axis, the absorption? [AIIMS 2018, C]
direction of B vector at that time is along [AIIMS 2015, C] (a) Ultraviolet (b) Microwaves
y
(a) y-axis (c) Infrared (d) Radio waves
(b) x-axis 24. An EM wave from air enters a medium. The electric fields
x ur é ù
are E1 = E01 xµcos ê2pv æç - tö÷ ú in air and
(c) + z-axis z
(d) – z-axis ë è c ø û
z which one can be used to produce ur
16. Out of the following options µ
E 2 = E 02 x cos [ k(2z - ct)] in medium, where the wave
a propagating electromagnetic wave ? [NEET 2016, C]
number k and frequency v refer to their values in air. The
(a) A charge moving at constant velocity
(b) A stationary charge medium is nonmagnetic. If Îr1 and Îr2 refer to relative
(c) A chargeless particle permittivities of air and medium respectively, which of the
(d) An accelerating charge following options is correct? [JEE Main 2018, S]
17. Assertion : Radio waves can be polarised. Îr Îr Îr 1 Îr 1
1 =4 1 1 1
Reason : Sound waves in air are longitudinal in nature. (a) Î (b) Î = 2 (c) Î = (d) =
r r r 4 Îr 2
[AIIMS 2016, C] 2 2 2 2
(a) Assertion is correct, reason is correct; reason is a
correct explanation for assertion.
EBD_7418
334 PHYSICS

Exercise 3 : Try If You Can


1. An electromagnetic wave of frequency 1 × 1014 hertz is 9. The rms value of the electric field of the light coming from
propagating along z-axis. The amplitude of electric field is 4 the Sun is 720 N/C. The average total energy density of the
V/m. If e0 = 8.8 × 10–12 C2/N-m2, then average energy density electromagnetic wave is
(a) 4.58 × 10–6 J/m3 (b) 6.37 × 10–9 J/m3
of electric field will be: –12 3
(c) 81.35 × 10 J/m (d) 3.3 × 10–3 J/m3
(a) 35.2 × 10–10 J/m3 (b) 35.2 × 10–11 J/m3 10. A point source of electromagnetic radiation has an average
–12
(c) 35.2 × 10 J/m 3 (d) 35.2 × 10–13 J/m3 power output of 1500 W. The maximum value of electric field
2. A point source of electromagnetic radiation has an average at a distance of 3m from this sources in Vm–1 is
power output of 800 W. The maximum value of electric field 500 250
at a distance 4.0 m from the source is (a) 500 (b) 100 (c) (d)
3 3
(a) 64.7 V/m (b) 57.8 V/m 11. The electric field part of an electromagnetic wave in a medium
(c) 56.72 V/m (d) 54.77 V/m is represented by Ex= 0;
3. A plane electromagnetic wave in a non-magnetic dielectric
ur ur E y = 2.5
N éæ
cos êç 2π ×106
rad ö æ
÷ t – ç π ×10
–2 rad ö ù ;
medium is given by E = E 0 (4 ´ 10 -7 x - 50t ) with distance C ëè mø è ÷x
s ø úû
being in meter and time in seconds. The dielectric constant Ez = 0. The wave is :
of the medium is : (a) moving along x direction with frequency 106 Hz and
(a) 2.4 (b) 5.8 (c) 8.2 (d) 4.8 wave length 100 m.
4. A plane electromagnetic wave travels in free space along
r (b) moving along x direction with frequency 106 Hz and
X-direction. If the value of B (in tesla) at a particular point in wave length 200 m.
r
space and time is 1.2 × 10–8 k̂. The value of E (in Vm–1) at (c) moving along – x direction with frequency 106 Hz and
that point is wave length 200 m.
(a) 1.2 ˆj (b) 3.6 kˆ (c) 1.2 kˆ (d) 3.6 ˆj (d) moving along y direction with frequency 2p × 106 Hz
5. The electric field associated with an e.m. wave in vacuum is and wave length 200 m.
r 12. In order to establish an instantaneous displacemet current
given by E = iˆ 40 cos (kz – 6 × 108t), where E, z and t are in
of 1 mA in the space between the plates of 2mF parallel
volt/m, meter and seconds respectively. The value of wave
plate capacitor, the potential difference need to apply is
vector k is
(a) 2 m–1 (b) 0.5 m–1 (c) 6 m–1 (d) 3 m–1 (a) 100 Vs–1 (b) 200 Vs–1 (c) 300 Vs–1 (d) 500 Vs–1
6. An electromagnetic wave 13. The magnetic component of a polarised wave of light is
r r in vacuum has the electric and
magnetic field E and B , which are always perpendicular Bx= (4.0 × 10–6T) sin[(1.57 × 107 m–1)y + wt]. The intensity
r to
each other. The direction of polarization is given by X and of light is
r
that of wave propagation by k . Then (a) 1.9 kW/m2 (b) 3.8 kW/m2
r r r r r r r r r r (c) 5.7 kW/m 2 (d) 7.6 kW/m2
(a) X || B and k || B ´ E (b) X || E and k || E ´ B
r r r r r r 14. The average electric field of electromagnetic waves in certain
r r r r
(c) X || B and k || E ´ B (d) X || E and k || B ´ E region of free space is 9 × 10–4 NC–1. Then the average
7. The charge on a parallel plate capacitor varies as q = q0 magnetic field in the same region is of the order of
cos 2put. The plates are very large and close together (a) 27 × 10–4 T (b) 3 × 10–12 T
(area = A, separation = d). The displacement current
æ 1ö –12
through the capacitor is (c) èç ø÷ ´ 10 T (d) 3 × 1012 T
(a) q0 2pu sinput (b) – q0 2pu sin2put 3
15. A lamp emits monochromatic green light uniformly in all
(c) q0 2p sinput (d) q0 pu sin2put
directions. The lamp is 3% efficient in converting electrical
8. Find the energy contained in a cylinder of cross-section
power to electromagnetic waves and consumes 100 W of
10 cm2 and length 50 cm along x-axis, if E = 50 sin w (t – x/C)
power. The amplitude of the electric field associated with
be the electric field in an electromagnetic wave
the electromagnetic radiation at a distance of 5 m from the
(a) 5.5 × 10–12 J (b) 1.5 × 10–11 J
–10 lamp will be nearly:
(c) 6.2 × 10 J (d) 1.1 × 10–15 J
(a) 1.34 V/m (b) 2.68 V/m
(c) 4.02 V/m (d) 5.36 V/m
ELECTROMAGNETIC WAVES 335

ANSW ER KEYS
Exercise 1 : Topic-wis e MCQs
1 (b) 11 (d) 21 (b) 31 (c) 41 (a) 51 (b) 61 (b) 71 (a) 81 (d) 91 (a)
2 (b) 12 (a) 22 (b) 32 (a) 42 (b) 52 (b) 62 (c) 72 (a) 82 (a) 92 (c)
3 (b) 13 (c) 23 (c) 33 (a) 43 (a) 53 (c) 63 (a) 73 (d) 83 (d) 93 (b)
4 (c) 14 (b) 24 (b) 34 (a) 44 (d) 54 (d) 64 (b) 74 (c) 84 (c) 94 (b)
5 (b) 15 (c) 25 (b) 35 (b) 45 (a) 55 (a) 65 (b) 75 (a) 85 (c) 95 (d)
6 (a) 16 (b) 26 (d) 36 (c) 46 (d) 56 (c) 66 (b) 76 (d) 86 (b) 96 (b)
7 (a) 17 (c) 27 (a) 37 (a) 47 (a) 57 (b) 67 (b) 77 (c) 87 (c) 97 (c)
8 (c) 18 (c) 28 (a) 38 (a) 48 (d) 58 (a) 68 (d) 78 (d) 88 (b) 98 (a)
9 (d) 19 (a) 29 (c) 39 (d) 49 (b) 59 (b) 69 (a) 79 (c) 89 (c) 99 (a)
10 (c) 20 (d) 30 (c) 40 (a) 50 (c) 60 (a) 70 (d) 80 (b) 90 (a) 100 (a)
Exercis e 2 : Exemplar & Pas t Year MCQs
1 (c) 4 (c) 7 (c) 10 (c) 13 (d) 16 (d) 19 (a) 22 (b)
2 (b) 5 (d) 8 (d) 11 (a) 14 (d) 17 (c) 20 (d) 23 (b)
3 (b) 6 (c) 9 (b) 12 (c) 15 (c) 18 (c) 21 (d) 24 (c)
Exercis e 3 : Try If You Can
1 (c) 3 (b) 5 (a) 7 (b) 9 (a) 11 (b) 13 (a) 15 (b)
2 (d) 4 (d) 6 (b) 8 (a) 10 (b) 12 (d) 14 (b)
EBD_7418
336 PHYSICS
24Chapter
RAY OPTICS AND
OPTICAL INSTRUMENTS

Trend
Analysis of NEET and AIIMS (Year 2010-2018)
5

4
Number of Questions

3
AIPMT/NEET
2 AIIMS

0
2010 2011 2012 2013 2014 2015 2016 2017 2018
Year

Trend
Analysis of JEE Main and BITSAT (Year 2010-2018)
5

4
Number of Questions

3 JEE Main
BITSAT

0
2010 2011 2012 2013 2014 2015 2016 2017 2018
Year

Chapter Utility Score (CUS)


Exam Weightage Important Concepts Difficulty Level CUS (Out of 10)
NEET 6 Refraction at curved surfaces,
AIIMS 5 Lens Maker’s formula, Total 3.5/5 7.5/10
JEE Main 5 Internal Reflection, Dispersion
BITSAT 6 by prism & Optical Instruments
EBD_7418
338 PHYSICS
RAY OPTICS AND OPTICAL INSTRUMENTS 339
EBD_7418
340 PHYSICS

Exercise 1 : Topic-wise MCQs


(c) Assertion is correct, reason is incorrect
Topic 1: Plane Mirror, Spherical Mirror and (d) Assertion is incorrect, reason is correct.
Reflection of Light 7. A person is six feet tall. How tall must a plane mirror be if he
is able to see his entire length?
1. The light reflected by a plane mirror may form a real image (a) 3 ft (b) 4.5 ft (c) 7.5 ft (d) 6 ft
(a) if the rays incident on the mirror are diverging 8. An object is placed 40 cm from a concave mirror of focal
(b) if the rays incident on the mirror are converging length 20 cm. The image formed is
(c) if the object is placed very close to the mirror (a) real, inverted and same in size
(d) under no circumstances (b) real, inverted and smaller
2. In image formation from spherical mirrors, only paraxial rays (c) virtual, erect and larger
are considered because they (d) virtual, erect and smaller
(a) are easy to handle geometrically 9. Which of the following is incorrect statement?
(b) contain most of the intensity of the incident light (a) the magnification produced by a convex mirror is
(c) form nearly a point image of a point source always less than one
(d) show minimum dispersion effect (b) a virtual, erect, same-sized image can be obtained using
3. For reflection through spherical surfaces, the normal at a plane mirror
the point of incidence is (c) a virtual, erect, magnifield image can be formed using
(a) perpendicular to the principle axis and passes through a concave mirror
the centre of curvature (d) a real, inverted, same-sized image can be formed using
(b) perpendicular to the focal plane and passes through a convex mirror.
the pole. 10. Match the Column I and Column II
(c) perpendicular to the tangent plane at pole and passes Column I Column II
through the focus. (A) An object is placed at focus (1) Magnification is –¥
(d) perpendicular to the tangent plane at the point of
before aconvex mirror
incidence and passes through the centre of curvature.
(B) An object is placed at centre (2) Magnification is 0.5
4. A convex mirror is used to form the image of an object. of curvature before a concave
Then which of the following statements is/are true? mirror
I. The image lies between the pole and the focus (C) An object is placed at focus (3) Magnification is +1
II. The image is diminished in size before a concave mirror
III. The image is real (D) An object is placed at centre (4) Magnification is –1
(a) I only (b) II only of curvature before a convex (5) Magnification is 0.33
(c) I and III (d) I and II mirror
5. Figure shows two rays A and B being reflected by a mirror (a) (A) ® (2); (B) ® (3); (C) ® (4); (D) ® (5)
and going as A' and B'. The mirror (b) (A) ® (5); (B) ® (4); (C) ® (3); (D) ® (1)
(c) (A) ® (2); (B) ® (4); (C) ® (1); (D) ® (5)
A (d) (A) ® (3); (B) ® (5); (C) ® (2); (D) ® (4)
11. A rod of length 10 cm lies along the principal axis of a
concave mirror of focal length 10 cm in such a way that its
B A end closer to the pole is 20 cm away from the mirror. The
length of the image is
B (a) 10 cm (b) 15 cm (c) 2.5 cm (d) 5 cm
(a) is plane
(b) is convex 1
12. A ray of light travelling in the direction (iˆ + 3 ˆj ) is
(c) is concave 2
(d) may be any spherical mirror incident on a plane mirror. After reflection, it travels along
6. Assertion : The focal length of the convex mirror will increase, 1
the direction (iˆ - 3 ˆj ) . The angle of incidence is
if the mirror is placed in water. 2
Reason : The focal length of a convex mirror of radius R is (a) 30° (b) 45° (c) 60° (d) 75°
equal to , f = R/2. 13. Two plane mirrors are inclined to each other at a certain
(a) Assertion is correct, reason is correct; reason is a angle. A ray of light first incident on one of them at an
correct explanation for assertion. inclination of 10° with the mirror retraces its path after five
(b) Assertion is correct, reason is correct; reason is not a reflections. The angle between the mirrors is :
correct explanation for assertion (a) 12° (b) 22° (c) 30° (d) 20°
RAY OPTICS AND OPTICAL INSTRUMENTS 341

14. A car is fitted with a convex side-view mirror of focal length æu- f ö
1/ 2
æu - f ö
2

20 cm. A second car 2.8 m behind the first car is overtaking (a) lç (b) l ç
è f ÷ø è f ÷ø
the first car at a relative speed of 15 m/s. The speed of the 1/ 2 2
æ ö
image of the second car as seen in the mirror of the first one is : (c) l ç f ÷ (d) l æç f ö÷
1 1 èu- f ø èu- f ø
(a) m/s (b) 10 m/s (c) 15 m/s (d) m/s
15 10 23. A point source has been placed as shown in the figure.
15. The width of man’s face is 10cm. The distance between the
What is the length on the screen that will receive reflected
eyes of the man is 4cm. Then the minimum width of plane
light from the mirror?

Screen
mirror to see his full face, is Source
(a) 5 cm (b) 4 cm (c) 3 cm (d) 10 cm (a) 2H
16. A ray of light is incident at 50° on the middle of one of the H
(b) 3H
two mirrors arranged at an angle of 60° between them. The
ray then touches the second mirror, get reflected back to the (c) H H H 2H
first mirror, making an angle of incidence of (d) None of these
(a) 50° (b) 60° (c) 70° (d) 80°
24. An object kept on the principle axis is moving in the same
17. An object is moving with speed v0 towards a spherical mirror
directions as that of mirror as shown in figure. Speed of object
with radius of curvature R, along the central axis of the
and mirror is 10m/s and 40/12 m/s. Radius of the curvature of
mirror. The speed of the image with respect to the mirror is
the mirror is 20 cm. What should be the distance of object
(U is the distance of the object from mirror at any given time t)
2 from the mirror at this instant so that the image is statonary?
æ R ö 2 æ R ö
(a) + ç v (b) - ÷ v0
è U - 2 R ÷ø 0 çè
R - 2U ø (a) 25 cm
2
æ R ö æ R ö 2 (b) 45 cm
(c) - ç ÷ v (d) + ç v
è 2U - 2 R ø 0 è 2U - 2 ÷ø 0 (c) 37.5 cm
O
18. A person walks at a velocity v in a straight line forming an
angle a with the plane of a mirror. Determine the velocity (d) 15 cm
vrel at which he approaches his image, assuming that the 25. You are asked to design a shaving mirror assuming that a
object and its image are symmetric relative to the plane of person keeps it 10 cm from his face and views the magnified
the mirror. image of the face at the closest comfortable distance of 25
(a) 2v sin a (b) 2v cos a (c) v sin a (d) v cos a cm. The radius of curvature of the mirror would then be :
19. A concave mirror is placed on a horizontal table, with its axis (a) 60 cm (b) –24 cm (c) – 60 cm (d) 24 cm
directed vertically upwards. Let O be the pole of the mirror 26. In figure find the total magnification after two successive
and C its centre of curvature. A point object is placed at C. reflections first on M1 and then on M2.
It has a real image, also located at C. If the mirror is now f = 10 cm f = 20 cm
filled with water, the image will be. (a) + 1
(a) real, and will remain at C.
(b) real, and located at a point between C and ¥. (b) –2
(c) virtual, and located at a point between C and O.
(c) +2 M2 M1
(d) real, and located at a point between C and O
r (d) –1 10 cm 30 cm
20. A point object is moving with velocity u = 2iˆ + ˆj - kˆ m/s
27. A boy of height h is walking away from a street lamp with a
in front of a stationary plane mirror. The magnitude of relative
constant speed v. The height of the street lamp is 3h. The
velocity of the image with respect to object is maximum if
the normal of the plane mirror will be along rate at which the length of the boy's shadow is increasing
when he is at a distance of 10 h from the base of the street
(a) 2iˆ + ˆj + kˆ (b) -2iˆ + ˆj - kˆ
lamp is :
(c) 2iˆ + ˆj - kˆ (d) 2iˆ - ˆj - kˆ (a) 2v (b) v (c) v/2 (d) v/3
21. A cube of side 2 m is placed in front of a concave mirror of
focal length 1m with its face P at a distance of 3 m and face Topic 2: Refraction of Light at Plane Surface
Q at a distance of 5 m from the mirror. The distance between and Total Internal Reflection
the image of face P and Q is
28. When light is refracted into a medium,
(a) 1 m
(a) its wavelength and frequency both increase
(b) 0.5 m
(b) its wavelength increases but frequency remains
(c) 0.5 m
unchanged
(d) 0.25 m
(c) its wavelength decreases but frequency remains
22. A short linear object of length l lies along the axis of a
unchanged
concave mirror of focal length f at a distance u from the pole
(d) its wavelength and frequency both decrease
of the mirror. The size of the image is approximately equal to
EBD_7418
342 PHYSICS

29. Total internal reflection can take place only if 38. A ray of light passes from vacuum into a medium of refractive
(a) light goes from optically rarer medium (smaller index µ, the angle of incidence is found to be twice the angle
refractive index) to optically denser medium of refraction. Then the angle of incidence is
(b) light goes from optically denser medium to rarer (a) 2 cos -1 æç µ ö÷ (b) sin –1(m)
medium 2
è ø
(c) the refractive indices of the two media are close to
æ µ ö æµö
different (c) sin -1 ç ÷ (d) cos -1 ç ÷
è 2ø è2ø
(d) the refractive indices of the two media are widely
39. When light passes from denser to rarer medium, then which
different of the following combination is correct ?
30. When the angle of incidence of a light ray is greater than
the critical angle it gets sin i m 2 sin i m1
(a) = ,r > i (b) = ,i = r
(a) critically refracted (b) totally reflected sin r m1 sin r m 2
(c) total internally reflected (d) totally refracted sin i m1 sin i
31. Critical angle of light passing from glass to water is minimum (c) = ,i > r (d) =2,i = r
for sin r m 2 sin r
(a) red colour (b) green colour 40. A vessel of depth x is half filled with oil of refractive index m1
(c) yellow colour (d) violet colour and the other half is filled with water of refractive index m2.
32. Light travels through a glass plate of thickness t and The apparent depth of the vessel when viewed from above
is
refractive index m. If c is the speed of light in vacuum, the
time taken by light to travel this thickness of glass is x (m1 + m 2 ) xm1m 2
(a) (b)
t 2m1m2 2(m1 + m2 )
(a) mtc (b) tc (c) (d) mt
m m c c xm1m2 2 x (m1 + m2 )
(c) (d)
33. One face of a rectangular glass plate 6 cm thick is silvered. An (m1 + m 2 ) m1m2
object held 8 cm in front of the first face, forms an image 12 cm 41. A ray of light travelling inside a rectangular glass block of
behind the silvered face. The refractive index of the glass is
(a) 0.4 (b) 0.8 (c) 1.2 (d) 1.6 refractive index 2 is incident on the glass-air surface at
34. Light travels in two media A and B with speeds 1.8 × an angle of incidence of 45º. The refractive index of air is
one. Under these conditions the ray will
108 m s–1 and 2.4 × 108 m s–1 respectively. Then the critical (a) emerge into the air without any deviation
angle between them is (b) be reflected back into the glass
æ 2ö -1 æ 3 ö
(c) be absorbed
(a) sin -1 ç ÷ (b) tan ç ÷ (d) emerge into the air with an angle of refraction equal
è 3ø è 4ø to 90º
æ 2ö æ 3ö 42. A ray of light travelling in a transparent medium of
(c) tan -1 ç ÷ (d) sin -1 ç ÷ refractive index m, falls on a surface separating the
è 3ø è 4ø
35. A green light is incident from the water to the air - water medium from air at an angle of incidence of 45°. For which
of the following value of m the ray can undergo total
interface at the critical angle (q). Select the correct statement.
internal reflection?
(a) The entire spectrum of visible light will come out of
(a) m = 1.33 (b) m = 1.40 (c) m = 1.50 (d) m = 1.25
the water at an angle of 90º to the normal.
43. A light ray falls on a rectangular glass slab as shown. The
(b) The spectrum of visible light whose frequency is less index of refraction of the glass, if total internal reflection is
than that of green light will come out to the air medium. to occur at the vertical face, is 45º
(c) The spectrum of visible light whose frequency is more
than that of green light will come out to the air medium. (a) 3/ 2 (b)
( 3 + 1)
(d) The entire spectrum of visible light will come out of 2
the water at various angles to the normal.
(c)
(2 +1 ) (d) 5 / 2
Glass

36. A glass slab of thickness 4 cm contains the same number of 2


waves as 5 cm of water when both are traversed by the same 44. Two transparent slabs of thickness (t1, t2) and refractive
monochromatic light. If the refractive index of water is 4/3, index (m1, m2) are arranged one over another. An object is
what is that of glass? seen through this combination. Find equivalent refractive
(a) 5/3 (b) 5/4 (c) 16/15 (d) 1.5 index of the system which will allow the image to be formed
37. A ray of light is travelling from glass to air. (Refractive at the same place.
index of glass = 1.5). The angle of incidence is 50°.
The deviation of the ray is ( t1 + t 2 ) m1m 2 t1 + t 2
(a) (b)
(a) 0° (b) 80° t1m 2 + t 2 m1 m1 + m 2
é sin 50° ù -1 é sin 50° ù t1t 2m1m2 t1m1
(c) 50° - sin -1 ê ú (d) sin ê ú - 50° (c) (d) t m
ë 1.5 û ë 1.5 û (t1 + t 2 ) 2 2
RAY OPTICS AND OPTICAL INSTRUMENTS 343

45. A ray of light passes 2d sin q


through four transparent
µ1 µ2 µ3 D (a)
n 2 - sin 2 q
media with refractive indices µ4 q
µ1, µ2, µ3 and µ4 as shown 2d sin q
(b) y0
in the figure. The surfaces C 1
B sin 2 q -
of all media are parallel. If the A n2 q
emergent ray CD is parallel 2
2d n - sin q 2

to the incident ray AB, we must have (c)


sin q
(d) None of these d
(a) µ1 = µ2 (b) µ2 = µ3 (c) µ3 = µ4 (d) µ4 = µ1
46. An electromagnetic radiation of frequency n, wavelength 51. A rectangular glass slab ABCD of refractive index n1 is
l, travelling with velocity v in air, enters a glass slab of immersed in water of refractive index n2(n1 > n2). A ray of
refractive index m. The frequency, wavelength and velocity light is incident at the surface AB of the slab as shown. The
of light in the glass slab will be respectively maximum value of the angle of incidence amax such that the
n l v l v ray comes out only from the other surface CD is given by
(a) , and (b) n, and
m m m m m én æ æ n ööù A D
v 2n l (a) sin-1 ê 1 cos ç sin-1 ç 2 ÷ ÷ ú n1 n2
ëê n2 è è n1 ø ø ûú
(c) n, 2 l and
a max
(d) , and v
m m m B C
é æ æ 1 öö ù
47. A diverging beam of light from a point source S having (b) sin-1 ên1 cos ç sin-1 ç ÷ ÷ ú
divergence angle a, falls symmetrically on a glass slab as ëê è è n2 ø ø ûú
shown. The angles of incidence of the two extreme rays are
equal. If the thickness of the glass slab is t and the refractive æn ö æn ö
(c) sin -1 ç 1 ÷ (d) sin -1 ç 2 ÷
index n, then the divergence angle of the emergent beam is è n2 ø è n1 ø
S
(a) zero (b) a 52. An observer can see through a pin-hole the top end of a
i a i
thin rod of height h, placed as shown in the figure. The
beaker height is 3h and its radius h. When the beaker is
-1 æ 1 ö æ 1ö
(c) sin ç ÷ (d) 2sin -1 ç ÷ m t filled with a liquid up to a height 2h, he can see the lower
è nø è nø
end of the rod. Then the refractive index of the liquid is
48. In a vessel, as shown in Fig. point P is just visible when no 5
liquid is filled in vessel from point D. When liquid is filled in (a)
2
the vessel completely, point Q is visible without moving
the vessel. Find the refractive index of the liquid. 5
D (b)
2
3h
14 85 3
(a) (b) Q
3 5 B C (c) h
2
2R R R
(c) 2 (d) 3 P EA 2h
3
R R (d)
2
49. In a thick glass slab of thickness l and refractive index n1 a
cuboidal cavity of thickness m is carved as shown in the 53. A printed page is pressed by a glass of water. The refractive
figure and is filled with liquid of R.I. n2 (n1 > n2). The ratio of index of the glass and water is 1.5 and 1.33, respectively. If
the thickness of the bottom of glass is 1 cm and depth of
l / m , so that shift produced by this slab is zero when an
observer A observes an object B with paraxial rays is water is 5 cm, how much the page will appear to be shifted if
A viewed from the top?
air
n1 - n2 n - n2 n1 (a) 1.033 cm (b) 3.581 cm (c) 1.574 cm (d) 1.90 cm
(a) (b) 1 (1)
n2 - 1 n2 ( n1 - 1)
n2 (2) m Topic 3: Refraction at Curved Surface, Lenses and
n1 - n2 n1 - n2
(c) (d) Power of Lens
n1 - 1 n1 ( n2 - 1)
air 54. A convex lens is immersed in a liquid of refractive index
B
50. A ray of light incident from air on a glass plate of refractive greater than that of glass. It will behave as a
index n is partly reflected and partly refracted at the two (a) convergent lens (b) divergent lens
surfaces of the glass. The displacement y0 in the figure is (c) plane glass (d) homogeneous liquid
EBD_7418
344 PHYSICS

55. A point object is placed at the centre of a glass sphere of 65. The rays of different colours fail to converge at a point
radius 6 cm and refractive index 1.5. The distance of the after going through a converging lens. This defect is called
virtual image from the surface of the sphere is (a) spherical aberration (b) distortion
(a) 2 cm (b) 4 cm (c) 6 cm (d) 12 cm (c) coma (d) chromatic aberration
56. A planoconvex lens of focal length 16 cm, is to be made of 66. What causes chromatic aberration?
glass of refractive index 1.5. The radius of curvature of the (a) Marginal rays
curved surface should be (b) Central rays
(a) 8 cm (b) 12 cm (c) 16 cm (d) 24 cm (c) Difference in radii of curvature of its surfaces
57. Two thin lenses are in contact and the focal length of the (d) Variation of focal length of lens with colour
combination is 80 cm. If the focal length of one lens is 20 cm,
then the power of the other lens will be 67. A convex lens is made of 3 layers of glass of 3 different
(a) 1.66 D (b) 4.00 D (c) – 100 D (d) – 3.75 D materials as in the figure. A point object is placed on its axis.
58. A plano-convex lens is made of material of refractive index The number of images of the object are
1.6. The radius of curvature of the curved surface is 60 cm. (a) 3
The focal length of the lens is (b) 4
(a) 50 cm (b) 100 cm (c) 200 cm (d) 400 cm (c) 1
59. A parallel beam of light is incident on the surface of a (d) 2
transparent hemisphere of radius R and refractive index 2.0
68. Two point source S1 and S2 are 24 cm apart. Where should
as shown in figure. The position of the image formed by
a convex lens of focal length 9 cm be placed in between
refraction at the first surface is :
them so that the images of both sources are formed at the
(a) R/2 same place?
(b) R (a) 6 cm from S1 (b) 15 cm from S1
(c) 2R (c) 10 cm from S1 (d) 12 cm from S1
(d) 3R 69. A parallel beam of light is incident on a converging lens
60. A lens made of glass whose index of refraction is 1.60 has a parallel to its principal axis. As one moves away from the
focal length of + 20 cm in air. Its focal length in water, whose lens on the other side on its principal axis, the intensity
refractive index is 1.33, will be of light
(a) three times longer than in air (a) remains constant
(b) two times longer than in air (b) continuously increases
(c) same as in air (c) continuously decreases
(d) None of these (d) first increases then decreases
61. A convex lens is in contact with concave lens. The magnitude 70. The given lens is broken into four parts rearranged as
of the ratio of their powers is 2/3. Their equivalent focal shown. If the initial focal length is f, then after rearrangment
length is 30 cm. What are their individual focal lengths (in cm)? the equivalent focal length is
(a) –15, 10 (b) –10, 15 (c) 75, 50 (d) –75, 50 (a) f
62. A bi-convex lens made of glass (refractive index 1.5) is put 1 2
in a liquid of refractive index 1.7. Its focal length will (b) f / 2 3 4
(a) decrease and change sign (b) f / 4
(b) increase and change sign
(c) decrease and remain of the same sign (d) 4 f in air

(d) increase and remain of the same sign 71. The focal length of a converging lens are fV and fR for
63. If two + 5 diopter lenses are mounted at some distance apart, violet and red light respectively. Then
the equivalent power will always be negative if the distance is (a) fV > fR
(a) greater than 40 cm (b) equal to 40 cm (b) fV = fR
(c) equal to 10 cm (d) less than 10 cm (c) fV < fR
64. A transparent sphere of radius R has a cavity of radius R/2 (d) any of the three is possible depending on the value of
as shown in figure. Find the refractive index of the sphere if the average refractive index m
a parallel beam of light falling on left surface focuses at 72. The layered lens as shown is made of
point P two types of transparent materials-
one indicated by horizontal lines and
3+ 5 3- 5 the other by vertical lines. The number
(a) m= (b) m =
2 2 P of images formed of an object will be
O
(a) 1 (b) 2 (c) 3 (d) 6
1+ 5
(c) m = 3+ 5 (d) m =
2
RAY OPTICS AND OPTICAL INSTRUMENTS 345

73. Assertion : When a convex lens (µg= 3/2) of focal length f is 81. Figure shows a concavo - convex lens µ2. What is the
4 condition on the reflective indices so that the lens is
dipped in water, its focal length becomes f . diverging?
3 2R
(a) 2µ3 < µ1 + µ2 m1 m3
Reason : The focal length of convex lens in water becomes 4f.
(a) Assertion is correct, reason is correct; reason is a (b) 2µ3 > µ1 + µ2
correct explanation for assertion. m2
(c) µ3 > 2 (µ1 – µ2)
(b) Assertion is correct, reason is correct; reason is not a (d) none of these R
correct explanation for assertion
82. Column I Column II
(c) Assertion is correct, reason is incorrect (A) Lens of power + 2.0 D (1) Convex lens of focal
(d) Assertion is incorrect, reason is correct. length 200 cm.
74. Two identical thin plano-convex glass lenses (refractive (B) Lenses of combination (2) Concave lens of focal
index 1.5) each having radius of curvature of 20 cm are placed of power +0.25 D and length 40 cm
with their convex surfaces in contact at the centre. The +0.25 D
intervening space is filled with oil of refractive index 1.7. (C) Lens of power –2.0 D (3) Convex lens of focal
The focal length of the combination is length 50 cm
(a) –25 cm (b) –50 cm (c) 50 cm (d) –20 cm (D) Lenses combination of (4) Concave lens of focal
75. A plano convex lens of refractive index 1.5 and radius of power –60 D and +3.5 D length 50 cm
curvature 30 cm. Is silvered at the curved surface. Now this (a) (A) ® (1); (B) ® (2); (C) ® (3); (D) ® (4)
(b) (A) ® (3); (B) ® (1); (C) ® (4); (D) ® (2)
lens has been used to form the image of an object. At what
(c) (A) ® (4); (B) ® (3); (C) ® (1); (D) ® (2)
distance from this lens an object be placed in order to have
(d) (A) ® (3); (B) ® (4); (C) ® (2); (D) ® (1)
a real image of size of the object
83. A man’s near point is 0.5 m and far point is 3 m. Power of
(a) 60 cm (b) 30 cm (c) 20 cm (d) 80 cm spectacle lenses required for (i) reading purposes, (ii) seeing
76. A point object is placed at a distance of 20 cm from a thin distant objects, respectively, are
plano-convex lens of focal length 15 cm, if the plane surface (a) –2 D and + 3 D (b) +2 D and –3 D
is silvered. The image will form at : (c) +2 D and –0.33 D (d) –2 D and + 0.33 D
(a) 60 cm left of AB 84. A double convex lens of focal length 6 cm is made of glass
(b) 30 cm left of AB of refractive index 1.5. The radius of curvature of one surface
(c) 12 cm left of AB is double that of other surface. The value of small radius of
(d) 60 cm right of AB curvature is
(a) 6 cm (b) 4.5 cm (c) 9 cm (d) 4 cm
77. The dispersive power of material of a lens of focal length 20
85. A planoconcave lens is placed on a paper on which a flower
cm is 0.08. What is the longitudinal chromatic aberration of
is drawn. How far above its actual position does the flower
the lens ? appear to be? Radius of
(a) 0.08 cm (b) 0.08/20 cm (c) 1.6 cm (d) 0.16 cm curvature =20cm
(a) 10 cm
78. An achromatic convergent doublet of two lenses in contact Air
(b) 15 cm
has a power of + 2D. The convex lens has power + 5D. What µ=3/2
t=20cm
is the ratio of dispersive powers of convergent and divergent (c) 50 cm
lenses ? (d) None of these
Paper
(a) 2 : 5 (b) 3 : 5 (c) 5 : 2 (d) 5 : 3 86. The position of final image formed by the given lens
79. A convex lens of focal length 12 cm is placed in contact with combination from the third lens will be at a distance of
a plane mirror. If the object is placed at 20 cm from the lens, (f1 = + 10 cm, f2 = – 10 cm and f3 = + 30 cm).
the position of final image is (a) 15 cm
O
(a) 30 cm above lens (b) infinity
(b) 30 cm below lens (c) 45 cm
(c) 20 cm above lens (d) 30 cm 30 cm 5 cm 10 cm
(d) 8.6 cm below lens 87. A luminous object is placed at a distance of 30 cm from the
80. A thin convergent glass lens (mg = 1.5) has a power of convex lens of focal length 20 cm. On the other side of the
+ 5.0 D. When this lens is immersed in a liquid of refractive lens, at what distance from the lens a convex mirror of radius
index m, it acts as a divergent lens of focal length 100 cm. of curvature 10 cm be placed in order to have an upright
The value of m must be image of the object coincident with it?
(a) 4/3 (b) 5/3 (c) 5/4 (d) 6/5 (a) 12 cm (b) 30 cm (c) 50 cm (d) 60 cm
EBD_7418
346 PHYSICS

97. An isosceles prism of angle 120° has a refractive index 1.44.


Topic 4: Prism and Dispersion of Light
Two parallel monochromatic rays enter the prism parallel to
88. The angular dispersion produced by a prism each other in air as shown. The rays emerge from the opposite
(a) increases if the average refractive index increases faces
(b) increases if the average refractive index decreases (a) are parallel to each other
(c) remains constant whether the average refractive index (b) are diverging
increases or decreases (c) make an angle 2 [sin–1 (0.72) – 30°]
120°
(d) has no relation with average refractive index. with each other
–1
(d) make an angle 2 sin (0.72)
89. If a glass prism is dipped in water, its dispersive power
(a) increases with each other
(b) decreases 98. The graph between angle of deviation (d) and angle of
(c) does not change incidence (i) for a triangular prism is represented by
(d) may increase or decrease depending on whether the (a) (b) d
d
angle of the prism is less than or greater than 60º
90. The refracting angle of a prism is ‘A’, and refractive index of
the material of the prism is cot(A/2). The angle of minimum
deviation is : o o
i i
(a) 180° – 2A (b) 90° – A (c) (d)
(c) 180° + 2A (d) 180° – 3A d d
91. Yellow light is refracted through a prism producing minimum
deviation. If i1 and i2 denote the angle of incidence and
emergence for this prism, then o o
(a) i1 = i2 (b) i1 > i2 i i
(c) i1 < i2 (d) i1 + i2 = 90º 99. Refractive index of a prism is 7 / 3 and the angle of prism
92. By properly combining two prisms made of different is 60°. The minimum angle of incidence of a ray that will be
materials, it is not possible to have transmitted through the prism is
(a) dispersion without average deviation (a) 30° (b) 45° (c) 15° (d) 50°
(b) deviation without dispersion 100. Light of wavelength 4000 Å is incident at small angle on a
(c) both dispersion and average deviation prism of apex angle 4°. The prism has nv = 1.5 and nr = 1.48.
(d) neither dispersion nor average deviation The angle of dispersion produced by the prism in this light is
93. The angle of prism is 60° and angle of deviation is 30°. (a) 0.2° (b) 0.08°
In the position of minimum deviation, the values of angle (c) 0.192° (d) None of these
of incidence and angle of emergence are: 101. A thin prism P1 with angle 4° and made from glass of
(a) i = 45°; e = 50° (b) i = 30°; e = 45° refractive index 1.54 is combined with another prism P2 made
(c) i = 45°; e = 45° (d) i = 30°; e = 30° of glass of refractive index 1.72 to produce dispersion
94. When the incidence angle is equal to the angle of without deviation. The angle of prism P2 is
emergence of light from the prism the refracted ray inside (a) 5.33° (b) 4° (c) 2.6° (d) 3°
the prism 102. A triangular prism of glass is inside water. A ray, incident
(a) becomes parallel to the right face of prism normally, on one of the faces, is totally reflected from face
(b) becomes perpendicular to the base of prism BC. Then the minimum refractive index of glass is –
(c) becomes parallel to the base of prism
3 5
(d) becomes perpendicular to the left face of prism (a) (b) A C
95. If D is the deviation of a normally falling light beam on a thin 2 3
prism of angle A and d is the dispersive power of the same
prism then 2 2 4 2
(a) D is independent of A. (c) (d)
5 3 B
(b) D is independent of refractive Index.
(c) d is independent of refractive index. 103. A ray of light is incident normally on one refracting surface
(d) d is independent of A. of an equilateral prism. If the refractive index of the material
96. A given ray of light suffers minimum deviation in an of the prism is 1.5, then
equilateral prism P. Additional prism Q and R of identical (a) the emergent ray is deviated by 30°
shape and of the same material as P are now added as shown (b) the emergent ray is deviated by 60°
in the figure. The ray will now suffer (c) the emergent ray just graces the second reflecting
(a) greater deviation surface
Q (d) the ray undergoes total internal reflection at second
(b) no deviation
refracting surface
(c) same deviation as before R (e) the ray emerges normally from the second refracting
P
(d) total internal reflection surface
RAY OPTICS AND OPTICAL INSTRUMENTS 347

104. Dispersive power of a prism to depends on: 112. A ray of light PQ is incident at angle i on a prism face. (See
(a) angle of prism (b) material of prism figure) After 2 refractions it leaves the prism at a grazing
(c) incident angle (d) refraction angle angle. B
105. In Fig. ABC is the cross section of a right - angled prism If m is the refractive index and B is the
and ACDE is the cross section of a glass slab. The value of refractive index and B is the refracting
q so that incident normally on the face AB does not cross angle of the prism, the refracting index i Q
the face AC is (given sin–1 (3/5) = 37°). (m) is: P
A E
(a) q £ 37° 1
(b) q < 37° é ì sin i + cos B ü 2 ù 2 cos i sin B

n = 6/5
n = 3/2
(a) ê1 + í (b)
(c) q £ 53° ý ú cos B
êë î sin B þ úû
(d) q < 53° q
B C D
106. A prism has a refracting angle of 60º. When placed in the (c)
(cos i + cos B )2 (d
sin i cos B
position of minimum deviation, it produces a deviation of sin B sin B
30º. The angle of incidence is
113. A thin prism of angle 15º made of glass of refractive index
(a) 30º (b) 45º (c) 15º (d) 60º
µ1 = 1.5 is combined with another prism of glass of refractive
107. A ray of light passes through an equilateral prism such that
the angle of incidence is equal to the angle of emergence index µ2 = 1.75. The combination of the prism produces
and the latter is equal to 3/4th of the angle of prism. The dispersion without deviation. The angle of the second prism
angle of deviation is should be
(a) 45º (b) 39º (c) 20º (d) 30º (a) 7° (b) 10° (c) 12° (d) 5°
108. The refractive index of a glass is 1.520 for red light and 1.525 114. Find the refractive index of the material of the prism, if the
for blue light. Let D1 and D2 be angles of minimum deviation angle of minimum deviation from the prism is 37° and angle
for red and blue light respectively in a prism of this glass. of prism is 53° [take sin 26.5° = 1/2.234]
Then, (a) 1.58 (b) 1.99 (c) 2.88 (d) 3.1
(a) D1 < D2
(b) D1 = D2 Topic 5: Optical Instruments
(c) D1 can be less than or greater than D2 depending 115. The image formed by an objective of a compound
upon the angle of prism microscope is
(d) D1 > D2 (a) real and diminished (b) real and enlarged
109. r and r' denote the angles inside an equilateral prism, as (c) virtual and enlarged (d) virtual and diminished
usual, in degrees. 116. An astronomical telescope has a large aperture to
Consider that during some time interval from (a) reduce spherical aberration
t = 0 to t = t, r' varies with time as r' = 10 + t2. r r' (b) have high resolution
During this time r will vary as (assume that (c) increases span of observation
r and r' are in degree) (d) have low dispersion
(a) 50 – t2 (b) 50 + t2 (c) 60 – t2 (d) 60 + t2 117. To increase the angular magnification of a simple
110. A black spot is present just inside one of the face of an microscope, one should increase
equilateral prism. A man places his eye directly at the opposite (a) the focal length of the lens
corner. He sees two images of the spot at an angular (b) the power of the lens
separation of 60°. Then the minimum value of refractive index (c) the aperture of the lens
of the prism is : (d) the object size
118. A normal eye is not able to see objects closer than 25 cm
3 because
(a) m= (b) m=2 (a) the focal length of the eye is 25 cm
2
2 (b) the distance of the retina from the eye-lens is 25 cm
3
(c) m= (d) m= (c) the eye is not able to decrease the distance between
2 3
Black spot the eye-lens and the retina beyond a limit
111. ACB is right - angle prism with other angles as 60° and 30°. (d) the eye is not able to decrease the focal length beyond
Refractive index of the prism is 1.5. AB has thin layer of a limit
liquid on it as shown. Light falls normally on the face AC. 119. Resolving power of a telescope increases with
For total internal reflections, maximum refractive index of (a) increase in focal length of eye-piece
the liquid is (b) increase in focal length of objective
(a) 1.4 (c) increase in aperture of eye piece
A 60° 30° B
(b) 1.3 60° (d) increase in apeture of objective
(c) 1.2 120. A normal eye has retina 2cm behind the eye lens. What is
the power of the eye-lens when the eye is fully relaxed ?
(d) 1.6 C (a) 50 D (b) 54 D (c) 40 D (d) 45 D
EBD_7418
348 PHYSICS

121. The focal length of the objective of a telescope is 60 cm. To 126. The focal lengths of objective and eye lens of an
obtain a magnification of 20, the focal length of the eye astronomical telelscope are respectively 2 meter and 5 cm.
piece should be Final image is formed at (i) least distance of distinct vision
(a) 2 cm (b) 3 cm (c) 4 cm (d) 5 cm (ii) infinity Magnifying power in two cases will be
122. If the focal length of objective lens is increased then (a) – 48, – 40 (b) – 40, – 48
magnifying power of : (c) – 40, + 48 (d) – 48, + 40
(a) microscope will increase but that of telescope 127. In a compound microscope, the focal length of objective
decrease. lens is 1.2 cm and focal length of eye piece is 3.0 cm. When
(b) microscope and telescope both will increase. object is kept at 1.25 cm in front of objective, final image is
(c) microscope and telescope both will decrease formed at infinity. Magnifying power of the compound
(d) microscope will decrease but that of telescope microscope should be:
increase. (a) 200 (b) 100 (c) 400 (d) 150
123. A telescope has an objective lens of focal length 150 cm 128. The magnifying power of a telescope is 9. When it is
and an eyepiece of focal length 5 cm. If a 50 m tall tower at adjusted for parallel rays, the distance between the objective
a distance of 1 km is observed through this telescope in and the eye piece is found to be 20 cm. The focal length of
normal setting, the angle formed by the image of the tower lenses are
is q, then q is close to : (a) 18 cm, 2 cm (b) 11 cm, 9 cm
(a) 6.1 rad (b) 3.2 rad (c) 1.5 rad (d) 0.2 rad (c) 10 cm, 10 cm (d) 15 cm, 5 cm
124. The diameter of the objective lens of microscope makes an 129. The focal lengths of objective lens and eye lens of a Galilean
angle b at the focus of the microscope. Further, the medium telescope are respectively 30 cm and 3.0 cm. telescope
between the object and the lens is an oil of refractive index produces virtual, erect image of an object situated far away
n. Then the resolving power of the microscope from it at least distance of distinct vision from the eye lens.
(a) increases with decreasing value of n In this condition, the magnifying power of the Galilean
(b) increases with decreasing value of b telescope should be:
(c) increases with increasing value of n sin 2b (a) + 11.2 (b) – 11.2 (c) – 8.8 (d) + 8.8
1 130. A telescope has an objective lens of 10 cm diameter and is
(d) increases with increasing value of situated at a distance of one kilometer from two objects.
n sin 2b
125. The focal length of the objective and the eyepiece of a The minimum distance between these two objects, which
telescope are 50 cm and 5 cm respectively. If the telescope is can be r esolved by the telescope, when the mean
focussed for distinct vision on a scale distant 2 m from its wavelength of light is 5000 Å, is of the order of
objective, then its magnifying power will be : (a) 5 cm (b) 0.5 m (c) 5 m (d) 5mm
(a) – 4 (b) – 8 (c) + 8 (d) – 2

Exercise 2 : Exemplar & Past Year MCQs


4. A passenger in an aeroplane shall
NCERT Exemplar MCQs (a) never see a rainbow
(b) may see a primary and a secondary rainbow as
1. A ray of light incident at an angle q on a refracting face of a concentric circles
prism emerges from the other face normally. If the angle of (c) may see a primary and a secondary rainbow as
the prism is 5° and the prism is made of a material of refractive concentric arcs
index 1.5, the angle of incidence is (d) shall never see a secondary rainbow
(a) 7.5° (b) 5° (c) 15° (d) 2.5° 5. You are given four sources of light each one providing a
2. A short pulse of white light is incident from air to a glass light of a single colour - red, blue, green and yellow. Suppose
slab at normal incidence. After travelling through the slab, the angle of refraction for a beam of yellow light
the first colour to emerge is corresponding to a particular angle of incidence at the interface
(a) blue (b) green (c) violet (d) red of two media is 90°. Which of the following statements is
correct if the source of yellow light is replaced with that of
3. An object approaches a convergent lens from the left of the
other lights without changing the angle of incidence?
lens with a uniform speed 5 m/s and stops at the focus. The
(a) The beam of red light would undergo total internal
image
reflection
(a) moves away from the lens with an uniform speed 5 m/s (b) The beam of red light would bend towards normal while
(b) moves away from the lens with an uniform acceleration it gets refracted through the second medium
(c) moves away from the lens with a non-uniform (c) The beam of blue light would undergo total internal
acceleration reflection
(d) moves towards the lens with a non-uniform acceleration (d) The beam of green light would bend away from the
normal as it gets refracted through the second medium
RAY OPTICS AND OPTICAL INSTRUMENTS 349

6. The radius of curvature of the curved surface of a plano- 11. There are certain material developed in laboratories which
convex lens is 20 cm. If the refractive index of the material of have a negative refractive index figure. A ray incident from
the lens be 1.5, it will air (Medium 1) into such a medium (Medium 2) shall follow
(a) act as a convex lens only for the objects that lie on its a path given by
curved side
(b) act as a concave lens for the objects that lie on its i 1
curved side
(a)
(c) act as a convex lens irrespective of the side on which
the object lies r 2
(d) act as a concave lens irrespective of side on which the
object lies
7. The phenomena involved in the reflection of radiowaves by i
1
ionosphere is similar to
(a) reflection of light by a plane mirror (b)
(b) total internal reflection of light in air during a mirage r
2
(c) dispersion of light by water molecules during the
formation of a rainbow
(d) scattering of light by the particles of air i r 1
8. The direction of ray of light incident on a concave mirror is
(c)
shown by PQ while directions in which the ray would travel
after reflection is shown by four rays marked 1, 2, 3 and 4 2
(figure). Which of the four rays correctly shows the direction
of reflected ray? 1 1
(a) 1 2
Q
4 (d)
2
(b) 2
Past Year MCQs
(c) 3 C F

12. The angle of a prism is ‘A’. One of its refracting surfaces is


(d) 4 3 P
silvered. Light rays falling at an angle of incidence 2A on
9. The optical density of turpentine is higher than tnat of water the first surface returns back through the same path after
while its mass density is lower. Figure shows a layer of suffering reflection at the silvered surface. The refractive
turpentine floating over water in a container. For which one index m, of the prism is : [AIPMT 2014, C]
of the four rays incident on turpentine in figure, the path
shown is correct? 1 2 3 4 1
(a) 2 sin A (b) 2 cos A (c) cos A (d) tan A
(a) 1 2
(b) 2
Air 13. An equilateral prism is placed on a horizontal surface. A ray
T PQ is incident onto it. For minimum deviation
(c) 3 [AIIMS 2014, C]
(d) 4 W G (a) PQ is horizontal R
10. A car is moving with at a constant speed of (b) QR is horizontal Q S
(c) RS is horizontal P
60 km h–1 on a straight road. Looking at the rear view mirror,
the driver finds that the car following him is at a distance of (d) Any one will be horizontal
100 m and is approaching with a speed of 5 kmh –1. 14. A light ray is incident perpendicularly to one face of a 90°
In order to keep track of the car in the rear, the driver begins prism and is totally internally reflected at the glass-air
to glance alternatively at the rear and side mirror of his car interface. If the angle of reflection is 45°, we conclude that
after every 2 s till the other car overtakes. If the two cars the refractive index [AIIMS 2014,C]
were maintaining their speeds, which of the following
1
statement (s) is/are correct? (a) n>
(a) The speed of the car in the rear is 65 km h –1 2
(b) In the side mirror, the car in the rear would appear to
approach with a speed of 5 kmh –1 to the driver of the (b) n> 2
leading car
(c) In the rear view mirror, the speed of the approaching
1
car would appear to decrease as the distance between (c) n<
the cars decreases 2
(d) In the side mirror, the speed of the approaching car would
appear to increase as the distance between the cars n< 2
(d)
decreases
EBD_7418
350 PHYSICS

15. Assertion: Critical angle is minimum for violet colour. R


(a) M
1 2 30°
-1 æ

Reason : Because critical angle qc = sin ç ÷ and m µ . R
èmø l (b)
3
[AIIMS 2014, C] P Q C
(a) Assertion is correct, reason is correct; reason is a 2 R-R
(c)
correct explanation for assertion. 2
(b) Assertion is correct, reason is correct; reason is not a (d) æ 1 ö
R ç1 - ÷
correct explanation for assertion è 3ø
(c) Assertion is correct, reason is incorrect 21. A planoconvex lens fits exactly into a planoconcave lens.
(d) Assertion is incorrect, reason is correct. Their plane surface are parallel to each other. If the lenses
æ 3ö are made of different materials of refractive indices µ1 & µ2
16. A thin convex lens made from crown glass ç m = ÷ has and R is the radius of curvature of the curved surface of the
è 2ø
focal length f. When it is measured in two different liquids lenses, then focal length of combination [BITSAT 2015, A]
4 5 R 2R
having refractive indices and , it has the focal lengths (a) (b)
3 3 µ1 – µ2 µ1 – µ2
f1 and f2 respectively. The correct relation between the focal R R
lengths is: [JEE Main 2014, A] (c) (d)
(a) f1 = f2 < f 2 ( µ1 – µ2 ) 2 – ( µ1 + µ2 )
(b) f1 > f and f2 becomes negative 22. The angle of incidence for a ray of light at a refracting surface
(c) f2 > f and f1 becomes negative of a prism is 45° . The angle of prism is 60°. If the ray suffers
(d) f1 and f2 both become negative minimum deviation through the prism, the angle of minimum
17. A beam of light consisting of red, green and blue colours is deviation and refractive index of the material of the prism
incident on a right angled prism. The refractive index of the respectively, are : [NEET 2016, A]
material of the prism for the above red, green and blue 1 1
wavelengths are 1.39, 1.44 and 1.47, respectively. (a) 45°, (b) 30°, 2 (c) 45°, 2 (d) 30°,
[AIPMT 2015, C] 2 2
23. A astronomical telescope has objective and eyepiece of focal
lengths 40 cm and 4 cm respectively. To view an object 200
Blue cm away from the objective, the lenses must be separated
Green by a distance : [NEET 2016, A]
Red (a) 37.3 cm (b) 46.0 cm (c) 50.0 cm (d) 54.0 cm
45° 24. A glass prism of refractive index 1.5 is immersed in water
The prism will: (refractive index 4/3). A light beam incident normally on the
(a) separate all the three colours from one another face AB is totally reflected to reach on the face BC if
(b) not separate the three colours at all [AIIMS 2016, S]
(c) separate the red colour part from the green and blue 8 A B
colours (a) sin q ³
9 q
(d) separate the blue colour part from the red and green
2 8
colours (b) < sin q <
18. A man 160 cm high stands in front of a plane mirror. His eyes 3 9
are at a height of 150 cm from the floor. Then the minimum 2
(c) sin q £
length of the plane mirror for him to see his full length image 3
is [AIIMS 2015, A] (d) None of these
(a) 85 cm (b) 170 cm (c) 80 cm (d) 340 cm 25. A ray of light is incident at an angle of incidence, i, on one
19. An achromatic convergent lens of focal length 20 cms is face of prism of angle A (assumed to be small) and emerges
made of two lenses (in contact) of materials having dispersive normally from the opposite face. If the refractive index of
the prism is m, the angle of incidence i, is nearly equal to
powers in the ratio of 1 : 2 and having focal lengths f1 and f2.
[AIIMS 2016, C]
Which of the following is true? [AIIMS 2015, A]
mA A A
(a) f1 = 10 cms, f2 = –20 cms (a) mA (b) (c) (d)
2 m 2m
(b) f1 = 20 cms, f2 = 10cms
26. When light falls on a given plate at angle of incidence of
(c) f1 = –10 cms, f2 = –20 cms 60º, the reflected and refracted rays are found to be normal
(d) f1 = 20 cms, f2 = –20 cms to each other. The refractive index of the matertial of the
20. A ray parallel to principal axis is incident at 30° from normal plate is then [AIIMS 2016, A]
on concave mirror having radius of curvature R. The point (a) 0.866 (b) 1.5 (c) 1.732 (d) 2
on principal axis where rays are focussed is Q such that PQ 27. In an experiment for determination of refractive index of glass
is [BITSAT 2015, S] of a prism by i – d, plot it was found thata ray incident at
RAY OPTICS AND OPTICAL INSTRUMENTS 351

angle 35°, suffers a deviation of 40° and that it emerges at (c) The spectrum of visible light whose frequency is more
angle 79°. In that case which of the following is closest to than that of green light will come out to the air medium.
the maximum possible value of the refractive index? (d) The entire spectrum of visible light will come out of
[JEE Main 2016, C] the water at various angles to the normal.
(a) 1.7 (b) 1.8 (c) 1.5 (d) 1.6 35. Assertion : Plane mirror may form real image.
28. An observer looks at a distant tree of height 10 m with a Reason : Plane mirror forms virtual image, if object is real.
telescope of magnifying power of 20. To the observer the
[AIIMS 2017, C]
tree appears : [JEE Main 2016, C]
(a) Assertion is correct, reason is correct; reason is a
(a) 20 times taller (b) 20 times nearer
correct explanation for assertion.
(c) 10 times taller (d) 10 times nearer
29. A biconvex lens has a radius of curvature of magnitude 20 (b) Assertion is correct, reason is correct; reason is not a
cm. Which one of the following options best describe the correct explanation for assertion
image formed of an object of height 2 cm placed 30 cm from (c) Assertion is correct, reason is incorrect
the lens? [BITSAT 2016, A] (d) Assertion is incorrect, reason is correct.
(a) Virtual, upright, height = 1 cm 36. Two plano-concave lenses (1 and 2) of glass
(b) Virtual, upright, height = 0.5 cm of refractive index 1.5 have radii of curvature
(c) Real, inverted, height = 4 cm 25 cm and 20 cm. They are placed in contact
(d) Real, inverted, height = 1cm with their curved surface towards each other 1 2
30. For the angle of minimum deviation of a prism to be equal to and the space between them is filled with
its refracting angle, the prism must be made of a material liquid of refractive index 4/3. Then the combination is
whose refractive index : [BITSAT 2016, C] [BITSAT 2017, C]
(a) convex lens of focal length 70 cm
(a) lies between 2 and 1 (b) lies between 2 and 2 (b) concave lens of focal length 70 cm
(c) is less than 1 (d) is greater than 2 (c) concave lens of focal length 66.6 cm
31. A lens having focal length f and aperture of diameter d forms (d) convex lens of focal length 66.6 cm
d 37. On a hypotenuse of a right prism (30° – 60° – 90°) of refractive
an image of intensity I. Aperture of diameter in central index 1.50, a drop of liquid is placed as shown in figure.
2
region of lens is covered by a black paper. Focal length of Light is allowed to fall normally on the short face of the
lens and intensity of image now will be respectively: prism. In order that the ray of light may get totally reflected,
[BITSAT 2016, C] the maximum value of refractive index is : [BITSAT 2017, A]
I 3f I
(a) f and (b) and
4 4 2
3I f I
(c) f and (d) and
4 2 2
32. A beam of light from a source L is incident normally on a
plane mirror fixed at a certain distance x from the source.
The beam is reflected back as a spot on a scale placed just
above the source I. When the mirror is rotated through a (a) 1.30 (b) 1.47 (c) 1.20 (d) 1.25
small angle q, the spot of the light is found to move through 38. A telescope has an objective lens of 10 cm diameter and is
a distance y on the scale. The angle q is given by situated at a distance of one kilometer from two objects.
[NEET 2017, S] The minimum distance between these two objects, which
x x
can be r esolved by the telescope, when the mean
y y
(a) (b) 2y (c) y (d) wavelength of light is 5000 Å, is of the order of
x 2x [BITSAT 2017, A]
33. A thin prism having refracting angle 10° is made of glass of (a) 5 cm (b) 0.5 m (c) 5 m (d) 5mm
refractive index 1.42. This prism is combined with another 39. An astronomical refracting telescope will have large angular
thin prism of glass of refractive index 1.7. This combination magnification and high angular resolution, when it has an
produces dispersion without deviation. The refracting angle objective lens of [NEET 2018, C]
of second prism should be [NEET 2017, A] (a) small focal length and large diameter
(a) 6° (b) 8° (c) 10° (d) 4° (b) large focal length and small diameter
34. A green light is incident from the water to the air - water (c) small focal length and small diameter
interface at the critical angle (q). Select the correct statement. (d) large focal length and large diameter
[AIIMS 2017, C] 40. The refractive index of the material of a prism is 2 and the
(a) The entire spectrum of visible light will come out of angle of the prism is 30°.One of the two refracting surfaces
the water at an angle of 90º to the normal. of the prism is made a mirror inwards, by silver coating. A
(b) The spectrum of visible light whose frequency is less beam of monochromatic light entering the prism from the
than that of green light will come out to the air medium.
EBD_7418
352 PHYSICS

other face will retrace its path (after reflection from the travels from vacuum to a medium of absolute refractive index
silvered surface) if its angle of incidence on the prism is µ. The ratio of wavelength of the incident and refracted
[NEET 2018, A] wave is [AIIMS 2018, A]
(a) 60° (b) 45° (c) Zero (d) 30° (a) µ2 : 1 (b) 1 : 1 (c) µ : 1 (d) 1 : µ
41. An object is placed at a distance of 40 cm from a concave 43. A point object O is placed in front of a glass rod having
mirror of focal length 15 cm.If the object is displaced through spherical end of radius of curvature 30 cm. The image would
a distance of 20 cm towards the mirror, the displacement of be formed at [BITSAT 2018, S]
the image will be [NEET 2018, A]
(a) 30 cm away from the mirror
(b) 36 cm away from the mirror
(c) 36 cm towards the mirror
(d) 30 cm towards the mirror (a) 30 cm left (b) infinity
42. The wavelength of a monochromatic light in vacuum is l. It (c) 1 cm to the right (d) 18 cm to the left

Exercise 3 : Try If You Can


1. A ball is dropped from a height of 20 m above the surface of (a) 15 ´ 3 ´ 5 cm (b) 15 ´ 3 7 cm
water in a lake. The refractive index of water is 4.3. A fish
inside the lake, in the line of fall of the ball, is looking at the 15 ´ 7 15 ´ 3
ball. At an instant, when the ball is 12.8 m above the water (c) cm (d) cm
3 7
surface, the fish sees the speed of ball as [Take g = 10 m/s2.]
6. A convex lens, of focal length 30 cm, a concave lens of focal
(a) 9 m/s (b) 12 m/s (c) 16 m/s (d) 21.33 m/s
length 120 cm, and a plane mirror are arranged as shown.
2. The size of the image of an object, which is at infinity, as For an object kept at a distance of 60 cm from the convex
formed by a convex lens of focal length 30 cm is 2 cm. If a lens, the final image, formed by the combination, is a real
concave lens of focal length 20 cm is placed between the image, at a distance of :
convex lens and the image at a distance of 26 cm from the
convex lens, calculate the new size of the image. (a) 60 cm from
(a) 3 cm (b) 2.5 cm (c) 4 cm (d) 4.3 cm the convex lens
3. A container is filled with (b) 60 cm from
water (m = 1.33) upto a |Focal length| |Focal length|
the concave lens = 30 cm = 120 cm
height of 33.25 cm. A 60cm 20cm
concave mirror is placed 15 15 cm (c) 70 cm from
cm above the water level the convex lens
70 cm
and the image of an object (d) 70 cm from the concave lens
placed at the bottom is m=1.33 25 cm
formed 25 cm below the 33.25 cm 7. A cube of side 2 m is placed in front of a concave mirror of
water level. Focal length focal length 1 m with its face A at a distance of 3 m and face
I
of the mirror is O B at a distance o f 5 m form the mirror. The distance between
(a) 15 cm (b) 20 cm (c) –18.31 cm (d) 10 cm the images of faces A and B and heights of images of A and
4. The focal lengths of the objective and the eye piece of a B are respectively,
compound microscope are 2.0 cm and 3.0 cm, respectively. (a) 1 m, 0.5 m, 0.25 m 2m
The distance between the objective and the eye piece is (b) 0.5 m, 1 m, 0.25 ,
B A
15.0 cm. The final image formed by the eye piece is at infinity. 2m
The two lenses are thin. The distance in cm of the object (c) 0.5 m, 0.25 m, 1 m 3m
and the image produced by the objective, measured from
(d) 0.25 m, 1 m, 0.5 m
the objective lens, are respectively
(a) 2.4 and 12.0 (b) 2.4 and 15.0 8. A point object is kept in front of a plane mirror. The plane
(c) 2.0 and 12.0 (d) 2.0 and 3.0 mirror is doing SHM of amplitude 2 cm. The plane mirror
5. A diver looking up through the water sees the outside world moves along the x - axis which is normal to the mirror. The
contained in a circular horizon. The refractive index of water amplitude of the mirror is such that the object is always in
4 front of the mirror. The amplityde of SHM of the image is
is , and the diver’s eyes are 15 cm below the surface of
3 (a) 0 (b) 2 cm (c) 4 cm (d) 1 cm
water. Then the radius of the circle is:
RAY OPTICS AND OPTICAL INSTRUMENTS 353

9. Consider the situation shown in fig. Water (µ = 4/3) is filled distance over which he can see the image of the light source
in a beaker upto a height of 10 cm. A plane mirror is fixed at in the mirror is B

a height of 5 cm from the surface of water. Distance of (a) d/2 d

image from the mirror after reflection from it of an object O (b) d


L
at the bottom of the beaker is 5 cm (c) 2d
2L
(d) 3d
(a) 15 cm (b) 12.5 cm 15. An object 2.4 m in front of a lens forms a sharp image on a
10 cm
film 12 cm behind the lens. A glass plate 1 cm thick, of
(c) 7.5 cm (d) 10 cm refractive index 1.50 is interposed between lens and film
10. Point object O is placed on the principal axis of a convex with its plane faces parallel to film. At what distance (from
lens of focal length 20 cm at a distance of 40 cm to the left lens) should object shifted to be in sharp focus of film?
of it. The diameter of the lens is 10 cm. If the eye is placed (a) 7.2 m (b) 2.4 m (c) 3.2 m (d) 5.6 m
60 cm to the right of the lens at a distance h below the 16. Two rays are incident on a spherical mirror of radius R = 5
principal axis, then the maximum value of h to see the image cm. parallel to its optical axis at distances h1 = 0.5 cm. and
will be h2 = 3 cm. Determine the distance Dx (approximately)
(a) 0 (b) 5 cm (c) 2.5 cm (d) 10 cm between the points at which these rays intersect the optical
11. A plane mirror is kept parallel to y-axis. A point object is axis after being reflected at the mirror.
r
approaching the mirror with velocity u = (10i$ + 10j)
$ m/s. The (a) 0.2 cm. (b) 1.5 cm. (c) 0.6 cm. (d) 1.0 cm.
magnitude of relative velocity of objective w.r.t image is 17. When an object is placed at a distance of 25 cm from a
y mirror, the magnification is m1. The object is moved 15cm
equal to
Plane further away with respect to the earlier position, and the
(a) 20 2m / s u mirror magnification becomes m2. If m1/m2 = 4, the focal length of
(b) 20 m/s the mirror is :
x
O (a) 10 cm (b) 30 cm (c) 15 cm (d) 20 cm
(c) 10 2m / s 18. Two plane mirrors A and B are aligned parallel to each other,
(d) 10 m/s as shown in the figure. A light ray is incident at an angle 30°
12. A small coin is resting on the bottom of a beaker filled with at a point just inside one end of A. The plane of incidence
liquid. A ray of light from the coin travels upto the surface of coincides with the plane of the figure. The maximum number
the liquid and moves along its surface. How fast is the light of times the ray undergoes reflections (including the first
travelling in the liquid? 3 cm one) before it emerges out is B
2 3m
(a) 2.4 × 108 m/s (a) 28

(b) 3.0 × 108 m/s (b) 30 0.2m 30°


4 cm
(c) 1.2 × 108 m/s (c) 32

coin (d) 34 A
(d) 1.8 × 108 m/s 19. A spherical surface of radius of curvature R separates air
13. A combination of two thin lenses with focal lengths f1 and (refractive index 1.0) from glass (refractive index 1.5). The
f2 respectively forms an image of a distance object at centre of curvature is in the glass. A point object P placed in
distance 60 cm when lenses are in contact. The position of air is found to have a real image Q in the glass. The line PQ
the image shift by 30 cm towards the combination when cuts the surface at a point O, and PO = OQ. The distance PO
lenses are separated by 10 cm. The values of f1 and f2 will be: is equal to
(a) 30 cm, – 60 cm (b) 20 cm, – 30 cm (a) 5R (b) 3R (c) 2R (d) 1.5R
(c) 15 cm, – 12 cm (d) 12 cm, – 15 cm 20. A 10D lens is used as a magnifier. Where should the object
14. A point source of light B is placed at a distance L in front of be placed to obtain maximum angular magnification for a
the centre of a mirror of width 'd' hung vertically on a wall. normal eye ?
A man walks in front of the mirror along a line parallel to the
(a) –7.1 cm (b) –8.3 cm (c) 6.2 cm (d) 9.3 cm
mirror at a distance 2L from it as shown in fig. The greatest
EBD_7418
354 PHYSICS

ANSWER KEYS
Exercise 1 : Topic-wise MCQs
1 (b) 14 (a) 27 (c) 40 (a) 53 (c) 66 (d) 79 (d) 92 (d) 105 (b) 118 (d)
2 (c) 15 (c) 28 (b) 41 (d) 54 (b) 67 (c) 80 (b) 93 (c) 106 (b) 119 (d)
3 (d) 16 (c) 29 (b) 42 (c) 55 (c) 68 (a) 81 (b) 94 (c) 107 (d) 120 (a)
4 (d) 17 (b) 30 (c) 43 (a) 56 (a) 69 (a) 82 (b) 95 (d) 108 (a) 121 (b)
5 (a) 18 (a) 31 (d) 44 (a) 57 (d) 70 (b) 83 (c) 96 (c) 109 (a) 122 (d)
6 (d) 19 (d) 32 (d) 45 (d) 58 (b) 71 (c) 84 (b) 97 (c) 110 (d) 123 (c)
7 (a) 20 (c) 33 (c) 46 (b) 59 (c) 72 (b) 85 (a) 98 (c) 111 (b) 124 (c)
8 (a) 21 (d) 34 (d) 47 (b) 60 (a) 73 (d) 86 (d) 99 (a) 112 (a) 125 (d)
9 (d) 22 (d) 35 (b) 48 (b) 61 (a) 74 (b) 87 (c) 100 (d) 113 (b) 126 (a)
10 (c) 23 (a) 36 (a) 49 (b) 62 (b) 75 (c) 88 (a) 101 (d) 114 (a) 127 (a)
11 (d) 24 (a) 37 (b) 50 (a) 63 (a) 76 (c) 89 (b) 102 (d) 115 (b) 128 (a)
12 (a) 25 (c) 38 (a) 51 (a) 64 (a) 77 (c) 90 (a) 103 (d) 116 (b) 129 (d)
13 (d) 26 (c) 39 (a) 52 (b) 65 (d) 78 (b) 91 (a) 104 (b) 117 (b) 130 (d)
Exercise 2 : Exemplar & Past Year MCQs
1 (a) 6 (c) 11 (a) 16 (b) 21 (a) 26 (c) 31 (c) 36 (c) 41 (b)
2 (d) 7 (b) 12 (b) 17 (c) 22 (b) 27 (c) 32 (d) 37 (a) 42 (c)
3 (c) 8 (b) 13 (b) 18 (c) 23 (d) 28 (b) 33 (a) 38 (d) 43 (a)
4 (b) 9 (b) 14 (b) 19 (a) 24 (a) 29 (c) 34 (b) 39 (d)
5 (c) 10 (d) 15 (b) 20 (d) 25 (a) 30 (b) 35 (b) 40 (b)
Exercise 3 : Try If You Can
1 (c) 3 (c) 5 (d) 7 (d) 9 (b) 11 (b) 13 (b) 15 (d) 17 (d) 19 (a)
2 (b) 4 (a) 6 (a) 8 (c) 10 (c) 12 (d) 14 (d) 16 (c) 18 (b) 20 (a)
25Chapter
WAVE OPTICS

Trend
Analysis of NEET and AIIMS (Year 2010-2018)
5

4
Number of Questions

3
AIPMT/NEET
2 AIIMS

0
2010 2011 2012 2013 2014 2015 2016 2017 2018
Year

Trend
Analysis of JEE Main and BITSAT (Year 2010-2018)

4
Number of Questions

3
JEE Main

2 BITSAT

0
2010 2011 2012 2013 2014 2015 2016 2017 2018
Year

Chapter Utility Score (CUS)


Exam Weightage Important Concepts Difficulty Level CUS (Out of 10)
NEET 6 Huygens’ Principle,
AIIMS 5 Interference of Light, 4/5 8/10
JEE Main 4 Young’s Experiment and
BITSAT 4 Diffraction.
EBD_7418
356 PHYSICS
WAVE OPTICS 357
EBD_7418
358 PHYSICS

Exercise 1 : Topic-wise MCQs

Topic 1: Wavefront, Interference of Light, Coherent and 12. Which of the following is correct for light diverging from
a point source?
Incoherent Sources of Light (a) The intensity decreases in proportion for the
1. Wavefront is the locus of all points, where the particles distance squared.
of the medium vibrate with the same (b) The wavefront is parabolic.
(a) phase (b) amplitude (c) The intensity at the wavelingth does depend of the
(c) frequency (d) period distance.
2. A plane wave passes through a convex lens. The (d) None of these.
geometrical shape of the wavefront that emerges is 13. Figure shows wavefront P passing R
(a) plane (b) diverging spherical through two systems A and B and
(c) converging spherical (d) None of these emerging as Q and then as R. The system
A and B could, respectively, be Q B
3. Huygen’s concept of secondary wave
(a) a prism and a convergent lens P A
(a) allows us to find the focal length of a thick lens
(b) is a geometrical method to find a wavefront (b) a convergent lens and a prism
(c) is used to determine the velocity of light (c) a divergent lens and a prism
(d) is used to explain polarisation (d) a convergent lens and a divergent lens P B
4. The phenomena which is not explained by Huygen’s 14. Two coherent light sources each of
construction of wavefront is wavelength l are separated by a
(a) reflection (b) diffraction distance 3l. The maximum number of
(c) refraction (d) origin of spectra minima formed on line AB which runs
5. When light suffers reflection at the interface between water from –¥ to +¥ is
and glass, the change of phase in the reflected wave is (a) 2 (b) 4 Q
A
(a) zero (b) p (c) p/2 (d) 2p (c) 6 (d) 8
6. Two sources of light are said to be coherent, when they 15. Shape of wavefront in case of
give light waves of same I. light diverging from a point source is spherical
(a) amplitude and phase II. light emerging out of a convex lens when a point
(b) wavelength and constant phase difference source is placed at its focus is plane
(c) intensity and wavelength III. the portion of the wavefront of light from a distant
(d) phase and speed star intercepted by the Earth is plane.
7. The colour of bright fringe nearest to central achromatic Which of the above statements are correct?
fringe in the interference pattern with white light will be (a) I and II (b) II and III
(a) violet (b) red (c) green (d) yellow (c) I and III (d) I, II and III
8. A wavefr ont AB passing A D 16. The principle of superposition is basic to the phenomenon
through a system C emerges as
C of
DE. The system C could be
(a) Polarisation (b) interference
E (c) reflection (d) refraction
(a) a slit (b) Ba biprism
(c) a prism (d) a glass slab 17. Two coherent monochromatic light beams of intensities I
9. The interfering fringes formed by a thin oil film on water and 4I are superposed. The maximum and minimum possible
are seen in yellow light of sodium lamp. We find the fringes intensities in the resulting beam are
(a) coloured (a) 5I and I (b) 5 I and 3I (c) 9 I and I (d) 9 I and 3I
(b) black and white 18. Irreducible phase difference in any wave of 5000 Å from a
(c) yellow and black source of light is
(d) coloured without yellow (a) p (b) 12p
10. Two coherent sources of intensity ratio 1 : 4 produce an (c) 12p × 106 (d) p × 106
interference pattern. The fringe visibility will be 19. Spherical wavefronts, emanating from a point source,
strike a plane reflecting surface. What will happen to
(a) 1 (b) 0.8 (c) 0.4 (d) 0.6
these wavefronts, immediately after reflection?
11. Two beams of light of intensity I1 and I2 interfere to give an
(a) They will remain spherical with the same curvature,
interference pattern. If the ratio of maximum intensity to that
both in magnitude and sign.
of minimum intensity is 25/9, then I1/I2 is
(b) They will become plane wave fronts.
(a) 5/3 (b) 4 (c) 81/625 (d) 16
WAVE OPTICS 359

(c) They will remain spherical, with the same curvature, (c) equally spaced
but sign of curvature reversed. (d) having fringe spacing which increases as we go
(d) They will remain spherical, but with different outwards
curvature, both in magnitude and sign. 27. Two light waves superimposing at the mid-point of the
20. Two coherent point sources S1 and S2 are separated by a screen are coming from coherent sources of light with
small distance 'd' as shown in figure. The fringes obtained phase difference 3p rad. Their amplitudes are 1 cm each.
on the screen will be The resultant amplitude at the given point will be.
(a) 5 cm (b) 3 cm (c) 2 cm (d) zero
28. The path difference between two wavefronts emitted by
d
coherent sources of wavelength 5460Å is 2.1 micron. The
S1 S2 Screen phase difference between the wavefronts at that point is
7.692 7.692
D (a) 7.692 (b) 7.692 p (c) (d)
p 3p
29. A thin film of soap solution (µs = 1.4) lies on the top of a
(a) points (b) straight lines glass plate (µg = 1.5). When visible light is incident almost
(c) semi-circles (d) concentric circles normal to the plate, two adjacent reflection maxima are
21. Assertion : No interference pattern is detected when two observed at two wavelengths 420 and 630 nm. The minimum
coherent sources are infinitely close to each other. thickness of the soap solution is
Reason : The fringe width is inversely proportional to the (a) 420 nm (b) 450 nm (c) 630 nm (d) 1260 nm
distance between the two sources.
30. Two coherent plane light waves of equal amplitude makes
(a) Assertion is correct, reason is correct; reason is a
a small angle a (< < 1) with each other. They fall almost
correct explanation for assertion.
normally on a screen. If l is the wavelength of light waves,
(b) Assertion is correct, reason is correct; reason is not
the fringe width Dx of interference patterns of the two sets
a correct explanation for assertion
(c) Assertion is correct, reason is incorrect of waves on the screen is
(d) Assertion is incorrect, reason is correct. 2l l l l
(a) (b) (c) (d)
22. A point p is situated 90.50 and 90.58 cm away from two a a ( 2a ) a
coherent sources. The nature of illumination of the point 31. In the adjacent diagram, CP represents a wavefront and
‘p’ if the wavelength of light is 4000 Å, is AO & BP, the corresponding two rays. Find the condition
(a) bright (b) dark on q for constructive interference at P between the ray BP
(c) neither bright nor dark (d) none of these and reflected ray OP.
23. The path difference between two interfering waves at a point Q O R

on screen is 171.5 times the wavelength. If the path difference


is 0.01029 cm. Find the wavelength.
C
(a) 6000 × 10–10 cm (b) 6000 Å d
(c) 6000 × 10–8 mm (d) None of these
24. Two coherent sources separated by
A
distance d are radiating in phase having
P
wavelength l. A detector moves in a big
circle around the two sources in the d B
plane of the two sources. The angular (a) cos q = 3l/2d (b) cos q = l/4d
S1 S2
position of n = 4 interference maxima is
(c) sec q – cos q = l/d (d) sec q – cos q = 4l/d
given as
32. Light from two coherent sources of the same amplitude
nl 4l d l A and wavelength l illuminates the screen. The intensity
(a) sin -1 (b) cos -1 (c) tan -1 (d) cos -1
d d 4l 4d of the central maximum is I0. If the sources were
25. Find the minimum thickness of a film which will strongly incoherent, the intensity at the same point will be
reflect the light of wavelength 598 nm. The refractive index I0
of the material of the film is 1.25. (a) 4I0 (b) 2I0 (c) I0 (d)
2
(a) 118 nm (b) 120 nm (c) 218 m (d) 225 mm
26. A thin slice is cut out of a glass cylinder Topic 2: Young’s Double Slit Experiment
along a plane parallel to its axis. The
33. The fringe width in a Young’s double slit experiment can
slice is placed on a flat glass plate as
be increased if we decrease
shown in Figure. The observed
(a) width of slits
interference fringes from this combination shall be
(a) straight (b) separation of slits
(b) circular (c) wavelength of light used
(d) distance between slits and screen
EBD_7418
360 PHYSICS

34. If Young’s double slit experiment is performed in water produces second dark fringe at a distance of 1 mm from central
keeping the rest of the set-up same, the fringes will fringe, the wavelength of monochromatic light used would be
(a) increase in width (b) decrease in width (a) 60 × 10–4 cm (b) 10 × 10–4 cm
–5
(c) 10 × 10 cm (d) 6 × 10–5 cm
(c) remain unchanged (d) not be formed
35. In Young’s double slit experiment, the minimum amplitude 45. The ratio of intensities at two points P and Q on a screen
is obtained when the phase difference of super-imposing in young’s double slit experiment when waves from
waves is (where n = 1, 2, 3, ...) sources S1 and S2 have phase difference of (a) 0° and
(a) zero (b) (2 n – 1) p (b) p/2 respectively, is
(c) n p (d) (n + 1) p (a) 1 : 4 (b) 4 : 1 (c) 1 : 2 (d) 2 : 1
36. In Young’s double slit experiment, one slit is covered with 46. In YDSE, how many maximas can be obtained on a screen
red filter and another slit is covered by green filter, then including central maxima in both sides of the central fringe
interference pattern will be if l = 3000Å, d = 5000Å
(a) red (b) green (a) 2 (b) 5 (c) 3 (d) 1
47. Which of the following statement (s) is/are true about
(c) yellow (d) invisible
interference pattern due to double slits?
37. In Young's double slit experiment with sodium vapour lamp
I. The interference pattern has equal number of bright
of wavelength 589 nm and the slits 0.589 mm apart, the half
and dark band width .
angular width of the central maximum is II. The pattern is obtained by superposition of two
(a) sin–1 (0.01) (b) sin –1 (0.0001) waves originating from the two narrow slits.
–1
(c) sin (0.001) (d) sin–1 (0.1) III. We get a minima at an angle of l/a where ‘a’ is the
38. In a Young’s double-slit experiment, let b be the fringe distance between two slits
width, and let I0 be the intensity at the central bright fringe. (a) I only (b) II only
At a distance x from the central bright fringe, the intensity (c) I and II only (d) I, II and III
will be 48. Distance between screen and source is decreased by 25%.
æxö æxö Then the percentage change in fringe width is
(a) I 0 cos çç ÷÷ (b) I 0 cos 2 çç ÷÷ (a) 20% (b) 31% (c) 75% (d) 25%
èbø èbø
49. Monochromatic light of wavelength 400 nm and 560 nm
æ px ö æI ö æ px ö are incident simultaneously and normally on double slits
(c) I 0 cos 2 çç ÷÷ (d) ç 0 ÷ cos 2 çç ÷÷
è b ø è 4ø è b ø apparatus whose slits separation is 0.1 mm and screen
39. The maximum number of possible interference maxima for distance is 1m. Distance between areas of total darkness
slit-separation equal to twice the wavelength in Young’s will be
double-slit experiment is (a) 4mm (b) 5.6 mm (c) 14mm (d) 28mm
(a) three (b) five (c) infinite (d) zero 50. In a Young¢s double slit experiment, the separation of the
40. If the intensities of the two interfering beams in Young’s two slits is doubled. To keep the same spacing of fringes,
double-slit experiment are I1 and I2, then the contrast the distance D of the screen from the slits should be made
D D
between the maximum and minimum intensities is good when (a) (b) (c) 2D (d) 4D
(a) | I1 and I2 | is large (b) | I1 and I2 | is small 2 2
51. The Young’s double slit experiment is performed with blue
(c) either I1 or I2 is zero (d) I1 = I2
and with green light of wavelengths 4360Å and 5460Å
41. In the Young’s Double slit experiment, when we place a
converging lens after the slits and place the screen at the respectively. If x is the distance of 4th maxima from the central
focus of the lens, it one, then
(a) introduces an extra path difference in the parallel beam. (a) x (blue) = x (green) (b) x (blue) > x (green)
(b) introduces no path difference in the parallel beam. x (blue) 5460
(c) x (blue) < x (green) (d) =
(c) introduces an extra phase difference in the parallel x (green) 4360
beam. 52. The maximum intensity of fringes in Young¢s experiment is
(d) introduces an extra fringe in the diffraction pattern. I. If one of the slit is closed, then the intensity at that place
42. In Young's double slit interference experiment, the distance becomes Io. Which of the following relation is true ?
between two sources is 0.1/p mm.The distance of the screen (a) I = Io
from the source is 25 cm. Wavelength of light used is 5000 (b) I = 2Io
Å. Then the angular position of the first dark fringe is (c) I = 4Io
(a) 0.10º (b) 0.15º (c) 0.30º (d) 0.45º (d) there is no relation between I and Io
43. In Young’s double slit experiment intensity at a point is 53. In Young's double slit experiment, the fringes are displaced
(1/4) of the maximum intensity. Angular position of this by a distance x when a glass plate of refractive index 1.5 is
point is introduced in the path of one of the beams. When this
(a) sin–1(l/d) (b) sin –1(l/2d) plate is replaced by another plate of the same thickness,
–1
(c) sin (l/3d) (d) sin –1(l/4d) the shift of fringes is (3/2) x. The refractive index of the
44. In Young’s experiment, two coherent sources are placed 0.90 second plate is
mm apart and fringe are observed one metre away. If it (a) 1.75 (b) 1.50 (c) 1.25 (d) 1.00
WAVE OPTICS 361

54. Light of wavelength 6.5 × 10–7 m is made incident on two from S to A remains unchanged. As a result of this the
slits 1 mm apart. The distance between third dark fringe fringe pattern moves somewhat
and fifth bright fringe on a screen distant 1 m from the slits (a) upwards
will be (b) downwards
(a) 0.325 mm (b) 0.65 mm (c) towards left horizontally
(c) 1.625 mm (d) 3.25 mm (d) towards right horizontally
55. In Young’s double slit expt. the distance between two 63. In Young's double slit experiment shown in figure S1 and S2
sources is 0.1 mm. The distance of the screen from the are coherent sources and S is the screen having a hole at a
source is 20 cm. Wavelength of light used is 5460 Å. The point 1.0mm away from the central line. White light (400 to
angular position of the first dark fringe is 700nm) is sent through the slits. Which wavelength passing
(a) 0.08º (b) 0.16º (c) 0.20º (d) 0.32º through the hole has strong intensity?
56. The separation between successive fringes in a double slit Centre of
arrangement is x. If the whole arrangement is dipped under S1
screen
water what will be the new fringe separation? [The 0.5mm
S2 Screen
wavelenght of light being used is 5000 Å] 1.0mm
(a) 1.5 x (b) x (c) 0.75 x (d) 2 x hole
57. With a monochromatic light, the fringe-width obtained in a 50cm S
Young’s double slit experiment is 0.133 cm. The whole set- (a) 400 nm (b) 700 nm
up is immersed in water of refractive index 1.33, then the (c) 500 nm (d) 667 nm
new fringe-width is 64. Assertion : In YDSE, if a thin film is introduced in front of
(a) 0.133 cm (b) 0.1 cm the upper slit, then the fringe pattern shifts in the downward
1.33 direction.
(c) 1.33 × 1.33 cm (d) cm Reason : In YDSE if the slit widths are unequal, the minima
2
58. In a double slit experiment, the two slits are 1 mm apart and will be completely dark.
the screen is placed 1 m away. A monochromatic light (a) Assertion is correct, reason is correct; reason is a
wavelength 500 nm is used. What will be the width of each correct explanation for assertion.
slit for obtaining ten maxima of double slit within the central (b) Assertion is correct, reason is correct; reason is not
maxima of single slit pattern ? a correct explanation for assertion
(c) Assertion is correct, reason is incorrect
(a) 0.1 mm (b) 0.5 mm (c) 0.02 mm (d) 0.2 mm (d) Assertion is incorrect, reason is correct.
59. In young’s double-slit experiment, the intensity of light at a
65. The figure shows a schematic diagram for Young’s double
point on the screen where the path difference is l is I, l
slit experiment. Given d << l, d << D, l / D << 1. Which
being the wavelength of light used. The intensity at a point
l of the following is a right statement about the wavelength
where the path difference is will be of light used?
4
I I
(a) (b) (c) I (d) zero S1
4 2
d d
60. In Young’s expt., the distance between two slits is and S0
3 O
the distance between the screen and the slits is 3 D. The
1 S2
number of fringes in m on the screen, formed by
3 l D
monochromatic light of wavelength 3l, will be (a) Larger the wavelength, larger will be the fringe width.
d d d d (b) If white light is used, violet colour forms its first maxima
(a) (b) (c) (d)
9Dl 27 D l 81 D l Dl closest to the central maxima.
61. In a two-slit experiment, with monochromatic light, fringes (c) The central maxima of all wavelength coincide.
are obtained on a screen placed at some distance from the (d) All of the above
slits. If the screen is moved by 5 × 10–2 m towards the slits, 66. White light used to illuminate the two slits in Young’s double
the change in fringe width is 10–3 m. Then the wavelength slit experiment. The separation between the slits is d and
of light used is (given that distance between the slits is the distance between the screen and the slit is D (>>d). At
0.03 mm) a point on the screen in front of one of the slits, certain
(a) 4500 Å (b) 5000 Å (c) 5500 Å (d) 6000 Å. wavelengths are missing. The missing wavelengths are
62. In Young’s double slit experiment, the source S and two
slits A and B are lying in a horizontal plane. The slit A is d2 (2n + 1)d 2
(a) l = (b) l =
above slit B. the fringes are obtained on a vertical screen K. (2n + 1)D D
The optical path from S to B is increased by putting a d2 (n + 1)D
transparent material of higher refractive index. The path (c) l= (d) l=
(n + 1)D d2
EBD_7418
362 PHYSICS

67. A double slit, S 1 – S 2 is 72. A Young’s double slit experiment is conducted in olive oil
illuminated by a light source of refractive index m and a glass plate of thickness

////////////////////////////////////////////////
S emitting light of wavelength 1.8×10–5 m and refractive index 1.51 is placed in path of
l. The slits are separated by a one slit. If the magnitude of the optical path difference at
distance d. A plane mirror is
S2
centre of screen will be 3.6×10–7 m, then find m
placed at a distance D in front S d (a) 1.48 (b) 1.33 (c) 1.28 (d) 1.11
of the slits and a screen is 73. A double slit arrangement produces fringes for l = 5890 Å
S1
placed at a distance 2D that are 0.4° apart. What is the angular width if the entire
2D D
behind arrangement is immersed in water ? (µw = 4/3)
the slits. The screen receives light reflected only by the (a) 0.3° (b) 2.3° (c) 0.8° (d) 1.3°
plane mirror. The fringe-width of the interference pattern 74. Young’s double slit experiment is made in a liquid. The 10th
bright fringe lies in liquid where 6th dark fringe lies in
on the screen is
vacuum. The refractive index of the liquid is approximately
Dl 2Dl 3Dl 4Dl (a) 1.8 (b) 1.5 (c) 1.3 (d) 1.6
(a) (b) (c) (d)
d d d d 75. A monochromatic light is used in Young's double slit
68. In figure, Young’s double experiment when one of the slits is covered by a transparent
slit experiment Q is the sheet of thickness 1.8 mm, made of material of refractive
position of the first bright Q
S1
index m1 number of fringes which shift is 18, when another
fringe on the right side of B sheet of thickness 3.6 mm, made of material of refractive
O. P is the 11th fringe on index m2 is used, number of fringes which shift is 9. Relation
O
the other side, as S2 between m1 and m2 is given by
measured from Q. If l = (a) 4m2 – m1 = 3 (b) 4m1 – m2 = 3
P (c) 3m2 – m1 = 4 (d) 2m1 – m2 = 4
6000 Å, then S1B will be
equal to 76. In a Young’s double slit experiment, the separation between
(a) 6 × 10–6 m (b) 6.6 × 10–6 m the slits = 2.0 mm, the wavelength of the light = 600 nm and
(c) 3.318 × 10 m –7 (d) 3.144 × 10–7 m the distance of the screen from the slits = 2.0 m. If the
intensity at the centre of the central maximum is 0.20 W/
69. Interference fringes were produced in Young's double slit
m2, what will be the intensity (in W/m2) at a point 0.5 cm
experiment using light of wavelength 5000 Å. When a film
away from this centre along the width of the fringes?
of material 2.5 × 10–3 cm thick was placed over one of the (a) 1.12 (b) 0.05 (c) 2.33 (d) 0.001
slits, the fringe pattern shifted by a distance equal to 77. In young’s double slit experiment the y-coordinates of
20 fringe width. The refractive index of the material of the central maximum and 10th maxima are 2 cm and 5 cm
film is respectively. When the YDSE apparatus is immersed in a
(a) 1.25 (b) 1.33 (c) 1.4 (d) 1.5 liquid of refractive index 1.5, the corresponding
70. The intensity of a point source of light, S, placed at a y-coordinates will be
distance d in front of a screen A, is I0 at the center of the (a) 2 cm, 7.5 cm (b) 3 cm, 6 cm
screen. Find the light intensity at the center of the screen if 4 10
a completely reflecting plane mirror M is placed at a (c) 2 cm, 4 cm (d) cm, cm
3 3
distance d behind the source, as shown in figure. 78. In a Young’s double slit experiment, if the incident light
M A consists of two wavelengths l1 and l2, the slit separation
is d, and the distance between the slit and the screen is D,
s the maxima due to the two wavelengths will coincide at a
d × d distance from the central maxima, given by :
l1 l2
(a)
2 Dd
27 I 0 25 I0 17 I0 10 I0
(a) (b) (c) (d) (b) (l1 - l2) . 2d
9 9 9 9 D
71. Consider the YDSE arrangement shown in figure. If d = 10 l
then position of 8th maxima is (c) LCM of l1 . D and l2 . D
d d
l 1D l2D
(d) HCF of and
y d d
d 79. In a Young’s double slit experiment with light of wavelength
l, fringe pattern on the screen has fringe width b. When
two thin transparent glass (refractive index m) plates of
D >>d thickness t1 and t2 (t1 > t2) are placed in the path of the two
D D 4 4D beams respectively, the fringe pattern will shift by a
(a) y = (b) y = (c) y = D (d) y= distance
10 3 5 3
WAVE OPTICS 363

b ( m - 1) æ t1 ö 88. When ordinary light is made incident on a quarter wave


mb t1
plate, the emergent light is
(a) l çè t ÷ø
2
(b) l t2 (a) linearly polarised (b) circulary polarised
b ( m - 1) l (c) unpolarised (d) elliptically polarised
(c) ( t1 - t2 ) (d) ( m - 1) ( t1 + t2 ) 89. In the propagation of light waves, the angle between the
l b
80. In Young's double slit experiment, one of the slit is wider plane of vibration and plane of polarisaiton is
than other, so that amplitude of the light from one slit is (a) 0º (b) 90º (c) 45º (d) 80º
double of that from other slit. If Im be the maximum intensity, 90. The phenomenon of diffraction can be treated as
the resultant intensity I when they interfere at phase interference phenomenon if the number of coherent
difference f is given by sources is
(a) one (b) two (c) zero (d) infinity
Im Im æ 2 fö 91. The figure shows
(a) (4 + 5 cos f) (b) ç 1 + 2 cos ÷
9 3 è 2ø Fraunhoffer’s diffraction due 1
Im æ 2 fö Im æ 2 fö to a single slit. If first minimum 2
(c) ç 1 + 4 cos ÷ (d) ç 1 + 8 cos ÷ is obtained in the direction
5 è 2ø 9 è 2ø 3
81. In a Young’s double slit experiment the intensity at a point shown, then the path
difference between rays 1
l and 3 is
where the path difference is (l being the wavelength of
6 (a) 0 (b) l/4 (c) l/2 (d) l
I 92. From Brewster’s law of polarisation, it follows that the
light used) is I. If I0 denotes the maximum intensity, is anlge of polarisaiton depends upon
I0
equal to (a) the wavelength of light
(b) plane of polarisation’s orientation
3 1 3 1
(a) (b) (c) (d) (c) plane of vibration’s orientation
4 2 2 2 (d) None of these
82. In Young’s double slit experiment, the slits are 2 mm apart and 93. A slit of width a is illuminated by red light of wavelength
are illuminated by photons of two wavelengths l1 = 12000Å 6500 Å. If the first minimum falls at q = 30°, the value of a is
and l2 = 10000Å. At what minimum distance from the (a) 6.5 × 10–4 mm (b) 1.3 micron
common central bright fringe on the screen 2 m from the slit (c) 3250 Å (d) 2.6 × 10–4 cm
will a bright fringe from one interference pattern coincide 94. The angle of incidence at which reflected light is totally
with a bright fringe from the other ? polarized for reflection from air to glass (refractive index n), is
(a) 6 mm (b) 4 mm (c) 3 mm (d) 8mm (a) tan–1(1/n) (b) sin–1(1/n)
(c) sin–1(n) (d) tan–1(n)
Topic 3: Diffraction, Polarisation of Light and Resolving Power 95. When an unpolarized light of intensity I0 is incident on a
83. If the width of the slit in single slit diffraction experiment polarizing sheet, the intensity of the light which does not
is doubled, then the central maximum of diffraction get transmitted is
pattern becomes 1 1
(a) I0 (b) I0 (c) I 0 (d) zero
(a) broader and brighter (b) sharper and brighter 4 2
(c) sharper and fainter (d) broader adn fainter. 96. Consider Fraunhoffer diffraction pattern obtained with a
84. When monochromatic light is replaced by white light in single slit illuminated at normal incidence. At the angular
Fresnel’s biprism arrangement, the central fringe is position of the first diffraction minimum the phase difference
(a) coloured (b) white (in radians) between the wavelets from the opposite edges
(c) dark (d) None of these
of the slit is
85. Light transmitted by nicol prism is (a) p/4 (b) p/2 (c) 2p (d) p
(a) unpolarised (b) plane polarised 97. The diffraction effects in a microscopic specimen become
(c) circularly polarised (d) elliptically polarised important when the separation between two points is
86. Optically active substances are those substances which (a) much greater than the wavelength of light used.
(a) produces polarised light (b) much less than the wavelength of light used.
(b) produces double refraction (c) comparable to the wavelength of light used.
(c) rotate the plane of polarisation of polarised light (d) independent of the wavelength of light used.
(d) converts a plane polarised light into circularly polarised 98. When unpolarised light is incident on a plane glass plate at
light. Brewster’s angle, then which of the following statements is
87. The condition for observing Fraunhoffer diffraction from a correct?
single slit is that the light wavefront incident on the slit (a) Reflected and refracted rays are completely polarised
should be with their planes of polarization parallel to each other
(a) spherical (b) cylindrical (b) Reflected and refracted rays are completely polarised
(c) plane (d) elliptical with their planes of polarization perpendicular to each
other
EBD_7418
364 PHYSICS

(c) Reflected light is plane polarised but transmitted light IV. The bands are of decreasing width.
is partially polarised (a) II and III (b) I and II
(d) Reflected light is partially polarised but refracted light (c) I, III and IV (d) I and III
is plane polarised 107. Yellow light is used in a single slit diffraction experiment
99. Abeam of unpolarised light of intensity I0 is passed through with slit width of 0.6 mm. If yellow light is replaced by
a polaroid A and then through another polaroid B which is X– rays, then the observed pattern will reveal,
oriented so that its principal plane makes an angle of 45° (a) that the central maximum is narrower
relative to that of A. The intensity of the emergent light is (b) more number of fringes
(c) less number of fringes
(a) I0 (b) I0/2 (c) I0/4 (d) I0/8
2
(d) no diffraction pattern
100. An unpolarised beam of intensity 2a passes through a 108. The angle substanded by the first diffraction minimum for
thin polaroid. Assuming zero absorption in the polaroid, a point source viewed in the hydrogen line at 1420 MHz
the intensity of emergent plane polarised light will be with a radio telescope having an aperture of 25 m is :
(a) 0.8° (b) 0.64° (c) 1.2° (d) 2.2°
a2
(a) 2a2 (b) a2 (c) 2 a2 (d) 109. When the diffraction pattern from a certain slit illuminated
2 with laser light (l = 6330 Å) is projected on a screen 150 cm
101. Which of the following diagrams represent the variation of from the slit, the second minima on each side are separated
electric field vector with time for a circularly polarised light ? by 8 cm. This tells us that :
(a) (b) (a) the slit is approximately 0.005 cm wide
(b) the slit is approximately 0.05 cm wide
|E | |E | (c) a / l is approximately 7.5 (a is the slit width)
(d) a / l is approximately 750
t t 110. A parallel beam of light of wavelength l is incident normally on
(c) (a) (d) (b) a narrow slit. A diffraction pattern is formed on a screen placed
|E | perpendicular to the direction of the incident beam. At the
|E | second minimum of the diffraction pattern, the phase difference
between the rays coming from the two edges of slit is
t (a) pl (b) 2p (c) 3p (d) 4p
t (d)
102. Aperture of the human eye is 2 mm. Assuming the mean
(c) 111. Unpolarised light of intensity 32 W m–2 passes through
wavelength of light to be 5000 Å, the angular resolution three polarizers such that the transmission axis of the last
limit of the eye is nearly polarizer is crossed with that of the first. The intensity of
(a) 2 minute (b) 1 minute (c) 0.5 minute (d)1.5 minute final emerging light is 3 W m–2. The intensity of light
103. If we observe the single slit Fraunhofer diffraction with transmitted by first polarizer will be
wavelength l and slit width b, the width of the central (a) 32 W m–2 (b) 16 W m–2
(c) 8 W m –2 (d) 4 W m–2
maxima is 2q. On decreasing the slit width for the same l
(a) q increases 112. Light of wavelength 6328 Å is incident normally on a slit
having a width of 0.2 mm. The angular width of the central
(b) q remains unchanged
maximum measured from minimum to minimum of diffraction
(c) q decreases
pattern on a screen 9.0 metres away will be about
(d) q increases or decreases depending on the intensity
(a) 0.36 degree (b) 0.18 degree
of light (c) 0.72 degree (d) 0.09 degree
104. When the angle of incidence is 60° on the surface of a 113. If the polarizing angle of a piece of glass for green light is
glass slab, it is found that the reflected ray is completely 54.74°, then the angle of minimum deviation for an
polarised. The velocity of light in glass is equilateral prism made of same glass is
[Given, tan 54.74° = 1.414]
(a) 2 ´ 108 ms -1 (b) 3 ´ 108 ms -1
(a) 45° (b) 54.74° (c) 60° (d) 30°
(c) 2 ´ 108 ms -1 (d) 3 ´ 108 ms -1 114. The width of a slit is 0.012 mm. Monochromatic light is
incident on it. The angular position of first bright line is
105. Two nicols are oriented with their principal planes making
5.2º. The wavelength of incident light is
an angle of 60º. Then the percentage of incident unpolarised
light which passes through the system is [sin 5.2º = 0.0906].
(a) 6040 Å (b) 4026 Å
(a) 100 (b) 50 (c) 12.5 (d) 37.5
(c) 5890 Å (d) 7248 Å
106. Which of the following statements are true about the
diffraction pattern? 115. In Fresnel’s biprism expt., a mica sheet of refractive index
I. It has a central bright maxima of twice the width of 1.5 and thickness 6 × 10–6 m is placed in the path of one of
other maxima. interfering beams as a result of which the central fringe
II. The first null occurs at an angle l/2a. gets shifted through 5 fringe widths. The wavelength of
III. The intensity of maxima falls as we move away from light used is
the central maxima. (a) 6000 Å (b) 8000 Å
(c) 4000 Å (d) 2000 Å
WAVE OPTICS 365

Exercise 2 : Exemplar & Past Year MCQs


NCERT Exemplar MCQs 4. In a Young’s double-slit experiment, the source is white
light. One of the holes is covered by a red filter and another
1. Consider a light beam incident from air to a glass slab at by a blue filter. In this case,
Brewster’s angle as shown in figure. (a) there shall be alternate interference patterns of red and
A polaroid is placed in the path of the emergent ray at point blue
P and rotated about an axis passing through the centre and (b) there shall be an interference pattern for red distinct
perpendicular to the plane of the polaroid. from that for blue
(c) there shall be no interference fringes
(d) there shall be an interference pattern for red mixing with
one for blue
5. Figure shows a standard two slit arrangement witn slits S1,
S2, P1, P2 are the two minima points on either side of P (figure).

Screen
S1 P1

P S
P
(a) For a particular orientation, there shall be darkness as S3
observed through the polaroid
S2 P2 S4 Second
(b) The intensity of light as seen through the polaroid shall screen
be independent of the rotation
(c) The intensity of light as seen through the polaroid shall At P2 on the screen, there is a hole and behind P2 is a
go through a minimum but not zero for two orientations second 2-slit arrangement with slits S3, S4 and a second
of the polaroid screen behind them.
(d) The intensity of light as seen through the polaroid shall (a) There would be no interference pattern on the second
go through a minimum for four orientations of the screen but it would be lighted
polaroid (b) The second screen would be totally dark
2. Consider sunlight incident on a slit of width 104 Å. The (c) There would be a single bright point on the second
image seen through the slit shall screen
(a) be a fine sharp slit white in colour at the centre (d) There would be a regular two slit pattern on the second
(b) a bright slit white at the centre diffusing to zero screen
intensities at the edges
Past Year MCQs
(c) a bright slit white at the centre diffusing to regions of
different colours 6. A beam of light of l = 600 nm from a distant source falls on
(d) only be a diffused slit white in colour a single slit 1 mm wide and the resulting diffraction pattern
3. Consider a ray of light incident from air onto a slab of glass is observed on a screen 2 m away. The distance between
first dark fringe on either side of the central bright fringe is:
(refractive index n) of width d, at an angle q. The phase
[AIPMT 2014, A]
difference between the ray reflected bv the top surface of
(a) 1.2 cm (b) 1.2 mm (c) 2.4 cm (d) 2.4 mm
the glass and the bottom surface is
7. In the Young’s double-slit experiment, the intensity of light
2pd æ 1 2 ö
12
at a point on the screen where the path difference is l is K,
(a) ç1 - 2 sin q ÷ +p (l being the wave length of light used). The intensity at a
l è n ø
point where the path difference is l/4, will be:
12 [AIPMT 2014, A]
4 pd æ 1 2 ö
(b) ç1 - 2 sin q ÷ (a) K (b) K/4 (c) K/2 (d) Zero
l è n ø
8. In Young’s double slit experiment, we get 10 fringes in the
12 field of view of monochromatic light of wavelength 4000Å.
4pd æ 1 2 ö p
(c) ç1 - sin q ÷ + If we use monochromatic light of wavelength 5000Å, then
l è n2 ø 2 the number of fringes obtained in the same field of view is
12 [AIIMS 2014, A]
4pd æ 1 2 ö
(d) ç1 - sin q ÷ + 2p (a) 8 (b) 10 (c) 40 (d) 50
l è n2 ø
EBD_7418
366 PHYSICS

9. The condition for obtaining secondary maxima in the (a) bends downwards
diffraction pattern due to single slit is [AIIMS 2014, C] (b) bends upwards
l (c) becomes narrower
(a) a sin q = nl (b) a sin q = ( 2n - 1) (d) goes horizontally without any deflection
2
17. Calculate the wavelength of light used in an interference
nl
(c) a sin q = ( 2n - 1) l (d) a sin q = experiment from the following data : Fringe width = 0.03
2 cm. Distance between the slits and eyepiece through which
10. Two beams, A and B, of plane polarized light with mutually the interference pattern is observed is 1m. Distance
perpendicular planes of polarization are seen through a
between the images of the virtual source when a convex
polaroid. From the position when the beam A has maximum
intensity (and beam B has zero intensity), a rotation of lens of focal length 16 cm is used at a distance of 80 cm
polaroid through 30° makes the two beams appear equally from the eyepiece is 0.8 cm. [BITSAT 2015, S]
bright. If the initial intensities of the two beams are IA and (a) 0.0006 Å (b) 0.0006 m (c) 600 cm (d) 6000 Å
18. In a YDSE, the light of wavelength l = 5000 Å is used,
IA
IB respectively, then equals: [JEE Main 2014, A] which emerges in phase from two slits a distance d = 3 ×
IB 10–7m apart. A transparent sheet of thickness t = 1.5 × 10–
3 1 7m refractive index m = 1.17 is placed over one of the slits.
(a) 3 (b) (c) 1 (d)
2 3 what is the new angular position of the central maxima of
11. A parallel beam of monochromatic light is incident on a the interference pattern, from the centre of the screen?
narrow rectangular slit of width 1mm. When the diffraction [BITSAT 2015, S]
pattern is seen on a screen placed at a distance of 2m. the
Find the value of y.
width of principal maxima is found to be 2.5 mm. The wave
length of light is- [BITSAT 2014, A] D ( m - 1) t p
(a) 4.9°and
(a) 6250 Å (b) 6200 Å 2d
(c) 5890 Å (d) 6000 Å D ( m - 1) t y
12. At the first minimum adjacent to the central maximum of a (b) 4.9°and
d S1
single-slit diffraction pattern, the phase difference between q
the Huygen's wavelet from the edge of the slit and the D ( m + 1) t d
(c) 3.9°and q d sinq
wavelet from the midpoint of the slit is : [AIPMT 2015, C] d
S2
p p p 2D ( m + 1) t
(a) rad (b)p rad (c) rad (d) rad (d) 2.9°and
2 8 4 d
13. The Fraunhoffer ‘diffraction’ pattern of a single slit is formed 19. The intensity at the maximum in a Young's double slit
in the focal plane of a lens of focal length 1 m. The width of
experiment is I0. Distance between two slits is d = 5l, where
slit is 0.3 mm. If third minimum is formed at a distance of 5
l is the wavelength of light used in the experiment. What will
mm from central maximum, then wavelength of light will be
[AIIMS 2015, A] be the intensity in front of one of the slits on the screen
(a) 5000 Å (b) 2500 Å (c) 7500 Å (d) 8500 Å placed at a distance D = 10 d ? [NEET 2016, A]
14. A parallel beam of monochromatic unpolarised light is I0 3 I0
incident on a transparent dielectric plate of refractive index (a) I0 (b) (c) I0 (d)
4 4 2
1 20. In a diffraction pattern due to a single slit of width 'a', the
. The reflected beam is completely polarised. Then the first minimum is observed at an angle 30° when light of
3 wavelength 5000 Å is incident on the slit. The first secondary
angle of incidence is [AIIMS 2015, A]
(a) 30º (b) 60º (c) 45º (d) 75º maximum is observed at an angle of : [NEET 2016, A]
15. Assertion : In Young’s double slit experiment if wavelength -1 æ ö
1 -1 æ ö
2 -1 æ ö
1 -1 æ 3 ö
of incident monochromatic light is just doubled, number of (a) sin çè ÷ø (b) sin çè ÷ø (c) sin çè ÷ø (d) sin çè ÷ø
4 3 2 4
bright fringe on the screen will increase. 21. A single slit Fraunhoffer diffraction pattern is formed with
Reason : Maximum number of bright fringe on the screen white light. For what wavelength of light the third
is directly proportional to the wavelength of light used. secondary maximum in the diffraction pattern coincides
[AIIMS 2015, C] with the second secondary maximum in the pattern for red
(a) Assertion is correct, reason is correct; reason is a light of wavelength 6500 Å? [AIIMS 2016, A]
correct explanation for assertion.
(a) 4400 Å (b) 4100 Å (c) 4642.8 Å (d) 9100 Å
(b) Assertion is correct, reason is correct; reason is not a
correct explanation for assertion 22. The box of a pin hole camera, of length L, has a hole of
(c) Assertion is correct, reason is incorrect radius a. It is assumed that when the hole is illuminated by
(d) Assertion is incorrect, reason is correct. a parallel beam of light of wavelength l the spread of the
16. On a hot summer night, the refractive index of air is smallest spot (obtained on the opposite wall of the camera) is the
near the ground and increases with height from the ground. sum of its geometrical spread and the spread due to
When a light beam is directed horizontally, the Huygens' diffraction. The spot would then have its minimum size
principle leads us to conclude that as it travels, the light (say bmin) when : [JEE Main 2016, C]
beam : [JEE Main 2015, C]
WAVE OPTICS 367

(a) a = lL and bmin = 4lL (c) Assertion is correct, reason is incorrect


(d) Assertion is incorrect, reason is correct.
l2 29. In a Young's double slit experiment, slits are separated by
(b) a= and b min = 4lL
L 0.5 mm, and the screen is placed 150 cm away. A beam of
light consisting of two wavelengths, 650 nm and 520 nm, is
l2 æ 2l2 ö
(c) a = and bmin = çç ÷
÷
used to obtain interference fringes on the screen. The least
L è L ø distance from the common central maximum to the point
æ 2l2 ö where the bright fringes due to both the wavelengths
(d) a = ll and b min = çç ÷ coincide is : [JEE Main 2017, A]
L ÷ (a) 9.75 mm (b) 15. 6 mm (c) 1.56 mm (d) 7.8 mm
è ø
23. A ray of light of intensity I is incident on a parallel glass slab 30. There are two sources kept at distances 2 l. A large screen
at point A as shown in diagram. It undergoes partial reflection is perpendicular to line joining the sources. Number of
maximas on the screen in this case is (l = wavelength of
and refraction. At each reflection, 25% of incident energy is
light) [BITSAT 2017, C]
reflected. The rays AB and A'B' undergo interference. The
ratio of I max and Imin is: [BITSAT 2016, A] ¥
B B' S1 S2
A' 2l
A ¥
(a) 1 (b) 3 (c) 5 (d) 7
31. In Young's double slit experiment the separation d between
C C' the slits is 2 mm, the wavelength l of the light used is 5896
Å and distance D between the screen and slits is100 cm. It
(a) 49 : 1 (b) 7 : 1 (c) 4 : 1 (d) 8 : 1 is found that the angular width of the fringes is 0.20°. To
increase the fringe angular width to 0.21° (with same l and
24. Young's double slit experment is first performed in air and
D) the separation between the slits needs to be changed to
then in a medium other than air. It is found that 8th bright [NEET 2018, A]
fringe in the medium lies where 5th dark fringe lies in air. The (a) 1.8 mm (b) 1.9 mm (c) 1.7 mm (d) 2.1 mm
refractive index of the medium is nearly [NEET 2017, A] 32. Unpolarised light is incident from air on a plane surface of
(a) 1.59 (b) 1.69 (c) 1.78 (d) 1.25 a material of refractive index 'm'. At a particular angle of
25. Two Polaroids P1 and P2 are placed with their axis incidence 'i', it is found that the reflected and refracted rays
perpendicular to each other. Unpolarised light I0 is incident are perpendicular to each other. Which of the following
on P1. A third polaroid P3 is kept in between P1 and P2 such options is correct for this situation? [NEET 2018, C]
that its axis makes an angle 45° with that of P1 . The intensity (a) Reflected light is polarised with its electric vector
of transmitted light through P2 is [NEET 2017, C] parallel to the plane of incidence
(b) Reflected light is polarised with its electric vector
I0 I0 I0 I0 perpendicular to the plane of incidence
(a) (b) (c) (d)
4 8 16 2
æ1ö æ1ö
26. The ratio of resolving powers of an optical microscope for (c) i = tan –1 ç ÷ (d) i = sin –1 ç ÷
two wavelengths l1 = 4000 Å and l2 = 6000 Å is m
è ø èmø
[NEET 2017, C] 33. In the figure shown S is the source of white light kept at a
(a) 9 : 4 (b) 3 : 2 (c) 16 : 81 (d) 8 : 27 distance X0 from the plane of the slits. The Source moves
27. A diffraction pattern is obtained by using beam of red with a constant speed u towards the slits on the line
light what will happen, if red light is replced by the blue perpendicular to the plane of the slits and passing through
light? [AIIMS 2017, C] the slit S1. Find the instanteneous velocity (magnitude and
(a) Bands disappear. direction) of the central maxima at time t having range 0 £
(b) Bands become broader and farther apart. x -d
(c) No change will take place. t << 0 . Assume that D >> d. [AIIMS 2018, S]
u
(d) Diffraction bands become narrow and crowded
Ddu
together.
(a) (downwards)
28. Assertion : Diffraction takes place for all types of waves 2(x 0 - ut) 2 S
X0
S
Sc
1
mechanical or non-mechanical, transverse or longitudinal. o
2 Ddu u
Reason : Diffraction's effect are perceptible only if (b) (downwards)
wavelength of wave is comparable to dimensions of (x 0 - ut) 2 d
diffracting device. [AIIMS 2017, C]
Ddu
(a) Assertion is correct, reason is correct; reason is a (c) (upwards)
correct explanation for assertion. 2(x 0 - ut)2
D
(b) Assertion is correct, reason is correct; reason is not 2du
a correct explanation for assertion (d) (up words)
(x 0 - ut)2
EBD_7418
368 PHYSICS

34. In a single slit diffraction the distance between slit & screen 37. The angular width of the central maximum in a single slit
is 1 m. The size of the slit is 0.7 mm & second maximum is diffraction pattern is 60°. The width of the slit is 1 mm. The
formed at the distance of 2 mm from the centre of the screen, slit is illuminated by monochromatic plane waves. If another
then find out the wavelength of light. [AIIMS 2018, A] slit of same width is made near it, Young's fringes can be
(a) 5600 Å (b) 6600 Å observed on a screen placed at a distance 50 cm from the
(c) 7600 Å (d) 9600 Å slits. If the observed fringe width is 1 cm, what is slit
separation distance? (i.e. distance between the centres of
35. Light is incident on a polarizer with intensity I0. A second
each slit.) [JEE Main 2018, S]
prism called analyzer is kept at a angle of 15° from the first
polarizer then the intensity of final emergent light will be: (a) 25 mm (b) 50 mm (c) 75 mm (d) 100 mm
[AIIMS 2018, A] 38. The figure shows the interference pattern obtained in a
(a) 0.46 I0d (b) 0.36 I0d double-slit experiment using light of wavelength 600nm.
1, 2, 3, 4 and 5 are marked on five fringes.
(c) 0.56 I0d (d) 0.96 I0d
[BITSAT 2018, C]
36. Unpolarized light of intensity I passes through an ideal
polarizer A. Another indentical polarizer B is placed behind The third order bright fringe is
I (a) 2 (b) 3 (c) 4 (d) 5
A. The intensity of light beyond B is found to be . Now 39. In Young’s double slit experiment, l = 500nm, d = 1mm,
2
another identical polarizer C is placed between A and B. D = 1m. Minimum distance from the central maximum for
I which intensity is half of the maximum intensity is
The intensity beyond B is now found to be . The angle [BITSAT 2018, A]
8
between polarizer A and C is: [JEE Main 2018, S] (a) 2.5 × 10–4 m (b) 1.25 × 10–4 m
(a) 0° (b) 30° (c) 45° (d) 60° (c) 0.625 × 10–4 m (d) 0.3125 × 10–4 m

Exercise 3 : Try If You Can


1. A mixture of light, consisting of wavelength 590 nm and an of P is ahead of the phase of Q by 90°. A, B and C are three
unknown wavelength, illuminates Young's double slit and distant points of observation equidistant from the mid-point
gives rise to two overlapping interference patterns on the of PQ. The intensity of radiations at A, B, C will bear the ratio
screen. The central maximum of both lights coincide. Further, (a) 0 : 1 : 4 (b) 4 : 1 : 0 (c) 0 : 1 : 2 (d) 2 : 1 : 0
it is observed that the third bright fringe of known light 6. A beam of light consisting of two wavelength 6500Å &
coincides with the 4th bright fringe of the unknown light. 5200Å is used to obtain interference fringes in a Young's
From this data, the wavelength of the unknown light is double slit experiment. The distance between the slits is 2.0
(a) 393.4 nm (b) 885.0 nm mm and the distance between the plane of the slits and the
(c) 442.5 nm (d) 776.8 nm screen is 120 cm. What is the least distance from the central
2. In a two slit experiment with monochromatic light fringes maximum where the bright fringes due to both the wavelengths
are obtained on a screen placed at some distance from the coincide?
sits. If the screen is moved by 5 ×10–2 m towards the slits, (a) 0.156 cm (b) 0.152 cm (c) 0.17 (d) 0.16 cm.
the change in fringe width is 3 ×10–5 m/s. If separation 7. A broad source of light (I =
between the slits is 10–3 m, the wavelength of light used is 680 nm) illuminates normally incident light
(a) 6000 Å (b) 5000 Å (c) 3000 Å (d) 4500 Å two glass plates 120 mm
3. In Young’s double slit experiment carried out with light of long that touch at one end t 0.034mm
wavelength (l) = 5000Å, the distance between the slits is and are
0.2 mm and the screen is at 200 cm from the slits. The central separated by a wire 0.034 mm 120mm
maximum is at x = 0. The third maxima (taking the central in diameter at the other end. The total number of bright
maximum as zeroth maximum) will be at x equal to fringes that appear over the 120 mm distance is –
(a) 1.67 cm (b) 1.5 cm (c) 0.5 cm (d) 5.0 cm (a) 50 (b) 100 (c) 200 (d) 400
8. Two polaroids are placed in the path of unpolarized beam
4. White light is used in Y.D.S. experiment. Find the minimum
of intensity I0 such that no light is emitted from the second
order of the violet fringe (l = 400 nm) which overlaps with polaroid. If a third polaroid whose polarization axis makes
a red fringe (l = 700 nm) an angle q with the polarization axis of first polaroid, is
(a) 7 (b) 5 (c) 3 (d) 2 placed between these polaroids then the intensity of light
5. Figure here shows P and Q as emerging from the last polaroid will be
two equally intense coherent æ I0 ö 2 æ I0 ö 2
sources emitting radiations of (a) ç ÷ sin 2q (b) ç ÷ sin 2q
è 8 ø è 4 ø
wavelength 20 m. The
æ I0 ö 4
separation PQ is 5.0 m and phase (c) ç ÷ cos q (d) I 0 cos 4 q
è 2 ø
WAVE OPTICS 369

9. In Young's double slit experiment the two slits are illuminated


by light of wavelenght 5890Å and the distance between the P1
fringes obtained on the screen is 0.2°. If the whole apparatus S2
is immersed in water then the angular fringe width will be, if a d b
the refractive index of water is 4/3. O
(a) 0.30º (b) 0.15º (c) 15º (d) 30º D(>> d)
10. Three identical coherent point screen S1
sources S1, S2 and S3 are P P2
placed on a line perpendicular D >> d
x a –b a +b a +b a –b
to the screen as shown in the (a) cot cot (b) tan cot
figure. The wavelength of light 2 2 2 2
emitted by source is d = 3l, the S1 S2 S3 a +b a –b a –b a +b
(c) sin cos (d) tan cot
distance of S2 from the screen d d 2 2 2 2
is D(>>> l). Find the minimum 13. Two beams of light having intensities I and 4I interfere to
distance x of a point P on the screen at which complete produce a fringe pattern on a screen. The phase difference
darkness is obtained. p
between the beams is at point A and p at point B. Then
2
2 2D 17 D 4 2D the difference between the resulting intensities at A and B is
(a) (b) (c) 4 5 D (d)
7 8 7 (a) 2 I (b) 4 I (c) 5 I (d) 7 I
11. A YDSE is conducted in water (µ1) 14. There are two plane mirrors.
S1
as shown in figure. A glass plate water µ1 They are mutually inclined as q
of thickness t and refractive index shown in figure. S is a source S
O
µ2 is placed in the path of S2. The S of monochromatic light of q
optical path difference at O is S2 µ2 wavelength l. The reflected Screen
t
beam interfere and fringe a 4a

pattern is obtained on the screen. If q is small, the fringe


(a) (m2 – 1)t (b) (m1 – 1)t Screen
width will be :
æm
2 ö (a) l/q (b) 3l/2q
(c) ç m - 1÷ t (d) (m2 – m1)t (c) 2l/3q (d) none of these
è 1 ø
15. The central fringe of the interference pattern produced by
12. In YDSE distance between the S1 and S2 is d. P1 and P2 are light of wavelength 6000Å is found to shift to the position
two points equidistance from O at an angular position b as of 4th bright fringe after a glass plate of refractive index 1.5
shown. A parallel beam of monochromatic light is incident is introduced in front of one of slits in Young's experiment.
at an angle a on the slits. Then the ratio of path difference The thickness of the glass plate will be
at P1 and P2 is: (a) 4.8 µm (b) 8.23 µm (c) 14.98 µm (d) 3.78 µm
EBD_7418
370 PHYSICS

ANSWER KEYS
Exercise 1 : Topic-wise MCQs
1 (a) 13 (b) 25 (a) 37 (c) 49 (d) 61 (d) 73 (a) 85 (b) 97 (c) 109 (a)
2 (c) 14 (c) 26 (a) 38 (c) 50 (c) 62 (b) 74 (a) 86 (c) 98 (c) 110 (d)
3 (b) 15 (b) 27 (d) 39 (b) 51 (c) 63 (c) 75 (a) 87 (c) 99 (c) 111 (b)
4 (d) 16 (b) 28 (b) 40 (d) 52 (c) 64 (d) 76 (b) 88 (d) 100 (b) 112 (a)
5 (b) 17 (c) 29 (b) 41 (b) 53 (a) 65 (d) 77 (c) 89 (b) 101 (a) 113 (d)
6 (b) 18 (a) 30 (c) 42 (d) 54 (c) 66 (a) 78 (c) 90 (d) 102 (b) 114 (d)
7 (a) 19 (c) 31 (b) 43 (c) 55 (b) 67 (d) 79 (c) 91 (c) 103 (a) 115 (a)
8 (c) 20 (d) 32 (d) 44 (d) 56 (c) 68 (a) 80 (d) 92 (a) 104 (b)
9 (a) 21 (a) 33 (b) 45 (d) 57 (b) 69 (c) 81 (a) 93 (b) 105 (c)
10 (b) 22 (a) 34 (b) 46 (c) 58 (d) 70 (d) 82 (a) 94 (d) 106 (d)
11 (d) 23 (b) 35 (b) 47 (c) 59 (b) 71 (d) 83 (b) 95 (b) 107 (d)
12 (a) 24 (b) 36 (d) 48 (d) 60 (c) 72 (a) 84 (b) 96 (c) 108 (c)
Exercise 2 : Exemplar & Past Year MCQs
1 (c) 5 (d) 9 (b) 13 (a) 17 (d) 21 (c) 25 (b) 29 (d) 33 (a) 37 (a)
2 (a) 6 (d) 10 (d) 14 (a) 18 (b) 22 (a) 26 (b) 30 (b) 34 (a) 38 (a)
3 (a) 7 (c) 11 (a) 15 (d) 19 (d) 23 (a) 27 (d) 31 (b) 35 (a) 39 (c)
4 (c) 8 (a) 12 (b) 16 (b) 20 (d) 24 (c) 28 (b) 32 (b) 36 (c)
Exercise 3 : Try If You Can
1 (c) 3 (b) 5 (c) 7 (b) 9 (b) 11 (d) 13 (b) 15 (a)
2 (a) 4 (a) 6 (a) 8 (a) 10 (c) 12 (d) 14 (b)
26Chapter
DUAL NATURE OF
RADIATION AND MATTER

Trend
Analysis of NEET and AIIMS (Year 2010-2018)
5

4
Number of Questions

3
AIPMT/NEET
2 AIIMS

0
2010 2011 2012 2013 2014 2015 2016 2017 2018
Year

Trend
Analysis of JEE Main and BITSAT (Year 2010-2018)
5

4
Number of Questions

3
JEE Main
2 BITSAT

0
2010 2011 2012 2013 2014 2015 2016 2017 2018
Year

Chapter Utility Score (CUS)


Exam Weightage Important Concepts Difficulty Level CUS (Out of 10)
NEET 5 Photoelectric Effect, X- Rays
AIIMS 4 & de - Broglie Wavelength 3.5/5 7.5/10
JEE Main 3 of Matter Waves
BITSAT 3
EBD_7418
372 PHYSICS
DUAL NATURE OF RADIATION AND MATTER 373
EBD_7418
374 PHYSICS

Exercise 1 : Topic-wise MCQs

9. Cathode rays and canal rays produced in a certain discharge


Topic 1: Matter Waves, Cathode and Positive Rays
tube are deflected in the same direction if
1. A strong argument for the particle nature of cathode rays is (a) a magnetic field is applied normally
that they (b) an electric field is applied normally
(a) produce fluoroscence (c) an electrin field is applied tangentially
(b) travel through vacuum
(d) a magnetic field is applied tangentially
(c) get deflected by electric and magnetic fields
(d) cast shadow 10. If E1, E2, E3 are the respective kinetic energies of an electron,
2. In an electron gun the control grid is given a negative an alpha-particle and a proton, each having the same
potential relative to cathode in order to de-Broglie wavelength, then
(a) decelerate electrons (a) E1 > E3 > E2 (b) E2 > E3 > E1
(b) repel electrons and thus to control the number of
electrons passing through it (c) E1 > E2 > E3 (d) E1 = E2 = E3
(c) to select electrons of same velocity and to converge 11. If the momentum of electron is changed by P, then the de
them along the axis. Broglie wavelength associated with it changes by 0.5%.
(d) to decrease the kinetic energy of electrons The initial momentum of electron will be
3. When the speed of electrons increase, then the value of its
P
specific charge (a) 200 P (b) 400 P (c) (d) 100 P
(a) increases 200
(b) decreases 12. Cathode rays are made to pass between the poles of a magnet
perpendicular to axis, the effect of the magnetic field is
(c) ramains unchanged
(a) to increase the velocity of rays
(d) increases upto some velocity and then begins to decrease (b) to deflect them towards the north pole
4. The specific charge for cathode rays is (c) to deflect them towards the south pole
(a) constant (d) to deflect them upwards above the plane of paper
(b) variable
(c) depend upon the material of the cathode 13. If the kinetic energy of a free electron doubles, it’s de-Broglie
(d) depend upon the nature of gas in the discharge tube wavelength changes by the factor
5. A particle with rest mass m0 is moving with speed of light c. 1 1
The de-Broglie wavelength associated with it will be (a) 2 (b) (c) 2 (d)
2 2
(a) ¥ (b) zero (c) m0 c/h (d) hn/m0c 14. If the kinetic energy of the particle is increased to 16 times
its previous value, the percentage change in the de-Broglie
6. A steel ball of mass m is moving with a kinetic energy K. The wavelength of the particle is :
de-Broglie wavelength associated with the ball is (a) 25 (b) 75 (c) 60 (d) 50
15. A aprticle of mass m is projected from ground with velocity
h h u making angle q with the vertical. The de Broglie
(a) (b)
2mK 2mK wavelength of the particle at the highest point is
(a) ¥ (b) h/mu sin q
h (c) h/mu cos q (d) h/mu
(c) (d) None of these 16. A proton and a-particle are accelerated through the same
2mK
potential difference. The ratio of their de-Broglie wavelength
7. If a photon and an electron have same de-Broglie
will be
wavelength, then
(a) both have same kinetic energy (a) 1 : 1 (b) 1 : 2 (c) 2 : 1 (d) 2 2 : 1
(b) proton has more K.E. than electron 17. A free particle with initial kinetic energy E, de-Broglie
(c) electron has more K.E. than proton wavelength l, enters a region wherein it has a potential
(d) both have same velocity energy V, what is the new de-Broglie wavelength ?
8. For an electron accelerated from rest through a potential (a) l (1 + E/V) (b) l (1 – V/E)
V, the de Broglie wavelength associated will be (c) l (1 + V/E)0.5 (d) l / (1 – V/E)0.5
18. An electron of mass m and charge e initially at rest gets
1.772 1.227
(a) nm (b) mm accelerated by a constant electric field E. The rate of change
V V of de-Broglie wavelength of this electron at time t ignoring
1.227 1.772 relativistic effects is
(c) nm (d) µm -h - mh -h
V V -eht
(a) 2 (b) (c) 2 (d)
e Et E e Et eE
DUAL NATURE OF RADIATION AND MATTER 375

19. The de-Broglie wavelength of a proton (mass = 1.6 × 29. The photoelectric current does not depend upon the
10–27 kg) accelerated through a potential difference of 1 kV is (i) frequency of incident light
(a) 600 A (b) 0.9 × 10–12m (ii) work function of the metal
(c) 7 Å (d) 0.9 nm. (iii) stopping potential
20. An a-particle and a singly ionized 4Be8 atom are accelerated (iv) intensity of incident light
through the same potential difference. What is the ratio of
(a) (i) and (iv) only (b) (ii) and (iii) only
the de-Broglie wave lengths in the two cases ?
(c) (iii) only (d) (ii) only
(a) 1 : 2 (b) 1 : 1 (c) 2 : 1 (d) 4 : 1
30. The stopping potential is directly related to
21. The magnitude of the de-Broglie wavelength (l) of electron
(a) the work function of the metal
(e), proton (p), neutron (n) and a-particle (a) all having the (b) intensity of incident radiation
same energy of 1 MeV, in the increasing order will follow (c) the saturation current for the given frequency
the sequence (d) the kinetic energy gained by the photoelectrons
(a) le, lp, ln, la (b) le, ln, lp, la 31. When light is incident on a metal surface the maximum
(c) la, ln, lp, le (d) lp, le, la, ln kinetic energy of emitted electrons
(a) vary with intensity of light
Topic 2: Photon, Photoelectric Effect, X-rays and Davison- (b) vary with frequency of light
Germer Experiment (c) vary with speed of light
22. Photoelectric effect is the phenomenon in which (d) vary irregularly
(a) photons come out of a metal when it is hit by a beam of 32. A photoelectric cell is a device which
electrons. (a) converts light into electricity
(b) photons come out of the nucleus of an atom under the (b) converts electricity into light
action of an electric field. (c) stores light
(c) electrons come out of a metal with a constant velocity (d) stores electricity
(d) which depends on the frequency and intensity of 33. The maximum energy of electrons released in a photocell is
incident light wave. independent of
23. The photoelectrons emitted from a metal surface are such (a) the frequency of the incident light
that their velocity (b) the intensity of the incident light
(a) is zero for all (c) the nature of the cathode
(b) is same for all (d) All of the above
(c) lies between zero and infinity 34. Light of a particular frequency n is incident on a metal
(d) lies between zero and a finite maximum surface. When the intensity of incident radiation is
24. In which of the following, emission of electrons does not increased, the photoelectric current
take place? (a) decreases
(a) Thermionic emission (b) X-rays emission (b) increases
(c) Photoelectric emission (d) Secondary emission (c) remains unchanged
25. Photoelectric emmision occurs only when the incident light (d) sometimes increases and sometimes decreases
has more than a certain minimum 35. If the energy of a photon is 10 eV, then its momentum is
(a) power (b) wavelength
(c) intensity (d) frequency (a) 5.33 × 10–23 kg m/s (b) 5.33 × 10–25 kg m/s
26. The energy of a photon of green light of wavelength 5000Å (c) 5.33 × 10–29 kg m/s (d) 5.33 × 10–27 kg m/s
is 36. The work function of a metal is independent of
(i) nature of the surface of the metal
(a) 3.459 × 10–19 joule (b) 3.973 × 10–19 joule
(ii) dimensions of the metal
(c) 4.132 × 10–19 joule (d) 8453 × 10–19 joule (iii) properties of the metal
27. Which of the following when falls on a metal will emit (iv) abundance of the metal
photoelectrons ? (a) (i) only (b) (i) and (iii)
(a) UV radiations (b) Infrared radiation (c) (ii) and (iii) (d) (ii) and (iv)
(c ) Radio waves (d) Microwaves 37. The work function of a substance is 4.0 eV. The longest
28. The work-function of a metal is wavelength of light that can cause photoelectron emission
(a) the minimum current required to take out electron from this substance is approximately
from the metal surface
(a) 310 nm (b) 400 nm (c) 540 nm (d) 220 nm
(b) the maximum frequency required to take out electron
38. A monochromatic source of light operating at 200 W emits
from the metal surface
4 × 1020 photons per second. Find the wavelength of light.
(c) the minimum amount of energy required to take out
(a) 400 nm (b) 200 nm
the electron from the metal surface –10
(d) None of these (c) 4 × 10 Å (d) None of these
EBD_7418
376 PHYSICS

39. Sodium and copper have work functions 2.3 eV and 4.5 eV 48. In photoelectric emission process from a metal of work
respectively. Then the ratio of the wavelengths is nearest to function 1.8 eV, the kinetic energy of most energetic
(a) 1 : 2 (b) 4 : 1 (c) 2 : 1 (d) 1 : 4 electrons is 0.5 eV. The corresponding stopping potential is
40. The work functions of Silver and Sodium are 4.6 and 2.3 eV, (a) 1.8 V (b) 1.2 V (c) 0.5 V (d) 2.3 V
respectively. The ratio of the slope of the stopping potential 49. The threshold frequency for a photosensitive metal is
3.3 × 1014 Hz. If light of frequency 8.2 × 1014 Hz is incident
versus frequency plot for Silver to that of Sodium is
on this metal, the cut-off voltage for the photoelectric
(a) 1 (b) 2 (c) 4 (d) zero
emission is nearly
41. Which one of the following graphs represents the variation (a) 2 V (b) 3 V (c) 5 V (d) 1 V
of maximum kinetic energy (EK) of the emitted electrons 50. According to Einstein’s photoelectric equation, the plot of
with frequency u in photoelectric effect correctly ? the kinetic energy of the emitted photo electrons from a
EK EK metal Vs the frequency of the incident radiation gives as
straight the whose slope
(a) depends both on the intensity of the radiation and the
(a) (b) metal used
(b) depends on the intensity of the radiation
u u
(c) depends on the nature of the metal used
EK EK (d) is the same for the all metals and independent of the
intensity of the radiation
51. From the graph it is clear that
(c) (d)
u u Stopping Metal A
u0 potential
42. 4eV is the energy of incident photon and the work function (V0) Metal B
v > v0
is 2eV. The stopping potential will be v0 v’0 v > v’0

(a) 2V (b) 4V (c ) 6 V (d) 2 2 V 0 Frequency of incident radiation (v)

43. The photoelectric work function for a metal surface is I. the stopping potential varies linearly with the
4.125 eV. The cut-off wavelength for this surface is frequency of incident radiation for the given metal.
(a) 4125 Å (b) 3000 Å (c) 6000 Å (d) 2062 Å II. the work function of metal A is greater than that for
44. For intensity I of a light of wavelenght 5000Å the metal B.
photoelectron saturation current is 0.40 µA and stopping III. the stopping potential is zero below the minimum cut
potential is 1.36 V, the work function of metal is off frequency.
(a) 2.47 eV (b) 1.36 eV IV. the stopping potential is independent of the intensity
of incident radiation.
(c) 1.10 eV (d) 0.43 eV (a) I and III only (b) I, III and IV
45. A photon of 1.7 × 10–13 joule is absorbed by a material (c) II and IV only (d) I, II and IV
under special circumstances. The correct statement is 52. The frequency and work function of an incident photon are
(a) Electrons of the atom of absorbed material will go the v and f0. If v0 is the threshold frequency then necessary
higher energy states condition for the emission of photoelectron is
(b) Electron and positron pair will be created
v
(c) Only positron will be produced (a) v < v0 (b) v = 0
(d) Photoelectric effect will occur and electron will be 2
produced (c) v ³ v0 (d) None of these
46. Einstein’s photoelectric equation states that 53. The work function of aluminium is 4.2 eV. If two photons,
each of energy 3.5 eV strike an electron of aluminium, then
hn = W0 + Ek.
emission of electrons
In this equatin, Ek refers to the
(a) will be possible
(a) kinetic energy of all the emitted electrons
(b) will not be possible
(b) mean kinetic energy of the emitted electrons (c) Data is incomplete
(c) maximum kinetic energy of the emitted electrons (d) Depends upon the density of the surface
(d) minimum kinetic energy of the emitted electrons
54. Assertion : The kinetic energy of photoelectrons emitted
47. Two radiations of photons energies 1 eV and 2.5 eV,
from metal surface does not depend on the intensity of
successively illuminate a photosensitive metallic surface of
incident photon.
work function 0.5 eV. The ratio of the maximum speeds of
the emitted electrons is Reason : The ejection of electrons from metallic surface is
(a) 1 : 4 (b) 1 : 2 (c) 1 : 1 (d) 1 : 5 not possible with frequency of incident photons below the
threshold frequency.
DUAL NATURE OF RADIATION AND MATTER 377

(a) Assertion is correct, reason is correct; reason is a correct (c) work function of the emitter
explanation for assertion. (d) Plank’s constant
(b) Assertion is correct, reason is correct; reason is not a 61. The cathode of a photoelectric cell is changed such that the
correct explanation for assertion work function changes from W1 to W2 (W2 > W1). If the
(c) Assertion is correct, reason is incorrect current before and after changes are I1 and I2, all other
(d) Assertion is incorrect, reason is correct. conditions remaining unchanged, then (assuming hn > W2)
55. For a given photosensitive material and frequency (> (a) I1 = I2 (b) I1 < I2
threshold frequency) of incident radiation, the (c) I1 > I2 (d) I1 < I2 < 2 I1
photoelectric current varies with the intensity of incident 62. A small photocell is placed at a distance of 4 m from a
light as photosensitive surface. When light falls on the surface the
current is 5 mA. If the distance of cell is decreased to 1 m,
Current Current
the current will become
æ 5ö
(a) 1.25 mA (b) ç ÷ mA (c) 20 mA (d) 80 mA
è 16 ø
(a) (b) 63. A and B are two metals with threshold frequencies
1.8 × 1014 Hz and 2.2 × 1014 Hz. Two identical photons of
Intensity Intensity energy 0.825 eV each are incident on them. Then
photoelectrons are emitted in (Take h = 6.6 × 10–34 Js)
Current (a) B alone (b) A alone
Current
(c) neither A nor B (d) both A and B.
64. In a photoelectric effect measurement, the stopping potential
for a given metal is found to be V0 volt when radiation of
(c) (d)
wavelength l0 is used. If radiation of wavelength 2 l0 is
used with the same metal then the stopping potential (in
Intensity
Intensity volt) will be
56. Light of wavelength 3500Å is incident on two metals A and V0 hc hc
B whose work functions are 3.2 eV and 1.9 eV respectively. (a) (b) 2 V0 (c) V0 + (d) V0 -
2 2el 0 2 el0
Which metal will emit photoelectrons?
65. In a photoelectric experiment, anode potential (v) is plotted
(a) A (b) B
(c) Both A and B (d) Neither A nor B against plate current (I)
57. Light of wavelength lA and lB falls on two identical metal
plates A and B respectively. The maximum kinetic energy of I
photoelectrons is KA and KB respectively, then which one
of the following relations is true (lA = 2 lB)
C B
K
(a) K A < B (b) 2 KA = KB A
2
(c) KA = 2 KB (d) KA > 2 KB
58. In photoelectric effect, stopping potential for a light of V
frequency n 1 is V1. If light is replaced by another having a (a) A and B will have different intensities while B and C
frequency n 2 then its stopping potential will be will have different frequencies
h h (b) B and C will have different intensities while A and C
(a) V1 - ( n 2 - n1 ) (b) V1 + ( n 2 + n1 )
e e will have different frequencies
h h (c) A and B will have different intensities while A and C
(c) V1 + ( n 2 - 2n1 ) (d) V1 + ( n 2 - n1 ) will have equal frequencies
e e
59. In a photoelectric effect experiment, for radiation with (d) A and B will have equal intensities while B and C will
frequency u0 with hu0 = 8eV, electrons are emitted with have different frequencies
energy 2 eV. What is the energy of the electrons emitted for 66. The photoelectric threshold of Tungsten is 2300Å. The energy
incoming radiation of frequency 1.25 u0 ? of the electrons ejected from the surface by ultraviolet light
(a) 1 eV (b) 3.25 eV (c) 4 eV (d) 9.25 eV. of wavelength 1800Å is
60. The maximum kinetic energy (Emax) of photoelectrons (a) 0.15 eV (b) 1.5 eV (c) 15 eV (d) 150 eV
emitted in a photoelectric cell varies with frequency (n) as 67. For photoelectric emission from certain metal the cut-off
shown in the graph. The slope of the graph is equal to frequency is n. If radiation of frequency 2n impinges on the
E max metal plate, the maximum possible velocity of the emitted
(a) charge of the electron
electron will be (m is the electron mass)
e (a) hn / m (b) 2 hn / m
(b) of the electron
m
(c) 2 hn / m (d) hn / ( 2m )
n0 n
EBD_7418
378 PHYSICS

68. In a photoelectric experiment the stopping potential for the IV. distance of surce from emitter is kept constant.
incident light of wavelength 4000Å is 2 volt. If the wavelength Which of the above statements are correct ?
be changed to 3000 Å, the stopping potential will be (a) I and II (b) II and III
(a) 2 V (b) zero (c) III and IV (d) I, II and III
(c) less than 2 V (d) more than 2 V 79. All electrons ejected from a surface by incident light of
69. A photocell is illuminated by a small bright source placed 1 m wavelength 200nm can be stopped before travelling 1m in
away. When the same source of light is placed 2 m away, the the direction of uniform electric field of 4N/C. The work
number of electrons emitted by photocathode are reduced by function of the surface is
a factor of (a) 4 eV (b) 6.2 eV (c) 2 eV (d) 2.2 eV
(a) 1/8 (b) 1/16 (c) 1/2 (d) 1/4 80. The maximum velocity of an electron emitted by light of
70. Light from a hydrogen discharge tube is incident on wavelength l incident on the surface of a metal of work-
the cathode of a photoelectric cell, the work function of function f is
the cathode surface is 4.2 eV. In order to reduce the
photocurrent to zero the voltage of the anode relative to the 2( hc + lf ) 2(hc + lf )
(a) (b)
cathode must be made ml ml
(a) – 4.2 V (b) – 9.4 V (c) – 17.8 V (d) + 9.4 V
71. In Davison-Germer experiment, an electron beam is incident 2( hc - lf ) 2(hl - f)
(c) (d)
on a crystal. The reflected beam consists of ml m
(a) photons (b) protons 81. The potential difference that must be applied to stop the
(c) x-rays (d) electrons fastest photoelectrons emitted by a nickel surface, having
72. In the Davisson and Germer experiment, the velocity of work function 5.01 eV, when ultraviolet light of 200 nm falls
electrons emitted from the electron gun can be increased by on it, must be
(a) increasing the potential difference between the anode (a) 2.4 V (b) – 1.2 V (c) – 2.4 V (d) 1.2 V
and filament 82. The glancing angle in a X-ray diffraction is 30º and the
(b) increasing the filament current wavelength of X-rays used is 20 nm. The interplanar spacing
(c) decreasing the filament current of the crystal dffracting these X-rays will be
(d) decreasing the potential difference between the anode (a) 40 nm (b) 20 nm (c) 15 nm (d) 10 nm
and filament 83. The maximum distance between interatomic lattice planes is
73. Radiations of two photon’s energy, twice and ten times the 15 Å. The maximum wavelength of X-rays which are
work function of metal are incident on the metal surface diffracted by this crystal will be
successsively. The ratio of maximum velocities of (a) 15 Å (b) 20 Å (c) 30 Å (d) 45 Å
photoelectrons emitted in two cases is
84. When X-rays of wavelength 0.5 Å would be transmitted by
(a) 1 : 2 (b) 1 : 3 (c) 1 : 4 (d) 1 : 1
an aluminium tube of thickness 7 mm, its intensity remains
74. When the minimum wavelength of X-rays is 2Å then the
one-fourth. The absorption coefficient of aluminium for these
applied potential difference between cathode and
anticathode will be X-rays is
(a) 6.2 kV (b) 2.48 kV (c) 24.8 kV (d) 62 kV (a) 0.188 mm–1 (b) 0.189 mm–1
75. When the X-ray tube is operated at 1kV, then X-rays of (c) 0.198 mm–1 (d) None of these
minimum wavelength 6.22 Å are produced. If the tube is 85. In the photoelectric experiment, if we use a monochromatic
operated at 10 kV, then the minimum wavelength of x-rays
will be light, the I-V curve is as shown. If work function of the metal
(a) 0.622 Å (b) 6.22 Å (c) 3.11 Å (d) zero is 2eV, estimate the power of light used. (Assume
76. The X-rays of wavelength 0.5 Å are scattered by a target. efficiency of photo emission = 10–3%, i.e., number of
What will be the energy of incident X-rays, if these are photoelectrons emitted are I
scattered at an angle of 72º ? 10–3% of number of photons 10µA
(a) 12.41 keV (b) 6.2 keV incident on metal)
(c) 18.6 keV (d) 24.82 keV (a) 2W
77. The potential difference applied to an X-ray tube is 5kV and (b) 5W
the current through it is 3.2mA. Then the number of electrons (c) 7W
striking the target per second is (d) 10W –5 volt
V

(a) 2 ´ 1016 (b) 5 ´ 10 6 (c) 1´ 1017 (d) 4 ´ 1015 86. An atom emits a photon of wavelength l = 600 nm by
transition from an excited state of life time 8 × 10–9 s. If Dv
78. To observe the effect of intensity of light on photocurrent,
represents the minimum uncertainty in the frequency of the
I. collector is maintained at positive potential with Dv
respect to emitter. photon, the fractional width of the spectral line is of
v
II. frequency of incident light is kept fixed. the order of
III. accelerating potential is fixed. (a) 10–4 (b) 10–6 (c) 10 –8 (d) 10–10
DUAL NATURE OF RADIATION AND MATTER 379

87. A 200 W sodium street lamp emits yellow light of wavelength 94. A 5 watt source emits monochromatic light of wavelength
0.6 µm. Assuming it to be 25% efficient in converting 5000 Å. When placed 0.5 m away, it liberates photoelectrons
electrical energy to light, the number of photons of yellow from a photosensitive metallic surface. When the source is
light it emits per second is moved to a distance of 1.0 m, the number of photoelectrons
(a) 1.5 × 1020 (b) 6 × 1018 liberated will be reduced by a factor of
(c) 62 × 1020 (d) 3 × 1019 (a) 8 (b) 16 (c) 2 (d) 4
88. At t = 0, light of intensity 1012 photons/s–m2 of energy 95. To decrease the cut-off wavelength of continuous X-rays
6eV per photon start falling on a plate with work function by 25%, the potential difference across X-ray tube
2.5 eV. If area of the plate is 2 ×10–4 m2 and for every 105 100
photons one photoelectron is emitted, charge on the plate (a) must be increased by %
3
at t = 25s is
(a) 8 × 10 –15 C (b) 4 × 10 –14 C 100
(b) must be decreased by %
(c) 12 × 10 –14 C (d) 16 × 10 –14 C 3
89. Electromagnetic radiation falls on a metallic body whose (c) must be increased by 25%
work function is 2eV. For a particular radiation of frequency
(d) must be decreased by 25%
v, the maximum kinetic energy of the photoelectron is found
to be 4 eV. What would be the maximum kinetic energy of 96. A perfectly relflecting solid sphere of radius x is kept in the
5v path of a paralled beam of light of large aperture. If the beam
photoelectron for the radiation of frequency ? carries an intensity I, find the force exerted by the beam on
3
8 10 20 the sphere.
(a) eV (b) 8 eV (c) eV (d) eV
3 3 3 p xi2
90. Photons with energy 5 eV are incident on a cathode C, on a p x 2i px3i 2 3 p x 2i
(a) (b) (c) 2 (d)
photoelectric cell. The maximum energy of the emitted C 2C 2C C
photoelectrons is 2 eV. When photons of energy 6 eV are 97. An X-ray tube with Cu target is operated at 25 kV. The
incident on C, no photoelectrons will reach the anode A if glancing anglefor aNaCl. Crystal for theCu ka line is 15.8°.
the stopping potential of A relative to C is Find the wavelength of this line.
(a) 3 V (b) –3 V (c) –1 V (d) 4 V (d for NaCl = 2.82 Å, h = 6.62 × 10–27 erg-sec)
91. Light coming from a discharge tube filled with hydrogen falls (a) 3.06 Å (b) 1.53 Å
on the cathode of the photoelectric cell. The work function of (c) 0.75 Å (d) None of these
the surface of cathode is 4eV. Which of the following values 98. When the X-ray tube is operated at 1kV, then X-rays of
of the anode voltage (in volts) with respect to the cathode minimum wavelength 6.22 Å are produced. If the tube is
will likely to make the photo current zero? operated at 10 kV, then the minimum wavelength of x-rays
(a) – 4 (b) – 6 (c) – 8 (d) – 10 will be
92. A 200 W sodium street lamp emits yellow light of wavelength (a) 0.622 Å (b) 6.22 Å (c) 3.11 Å (d) zero
0.6 µm. Assuming it to be 25% efficient in converting 99. The short wavelength limit of continuous X-radiation emitted
electrical energy to light, the number of photons of yellow by an X-ray tube operating at 30 kV is 0.414 Å. Calculate
light it emits per second is Planck's constant.
(a) 1.5 × 1020 (b) 6 × 1018 (a) 6.22 × 10–34 erg-sec (b) 6.624 × 10–24 erg-sec
(c) 62 × 1020 (d) 3 × 1019
(c) 6.624 × 10–27 J-sec (d) 6.624 × 10–34 J-sec
93. A horizontal cesium plate (work function = 1.9 eV) is moved
vertically downward at a constant speed V in a room full of 100. A source S1 is producing, 1015 photons per second of
radiation of wavelength 250 nm and above. The minimum wavelength 5000 Å. Another source S2 is producing
value of v so that the vertically upward component of 1.02×1015 photons per second of wavelength 5100Å Then,
velocity is nonpositive for each photoelectron. (power of S2) to the (power of S1) is equal to :
(a) 1.04 × 106 ms–1 (b) 2.03 × 107 ms–1 (a) 1.00 (b) 1.02 (c) 1.04 (d) 0.98
(c) 4.03 × 108 ms–1 (d) 5.11 × 109 ms–1
EBD_7418
380 PHYSICS

Exercise 2 : Exemplar & Past Year MCQs


(a) remains constant
NCERT Exemplar MCQs
(b) increases with time
1. A particle is dropped from a height H. The de-Broglie (c) decreases with me
wavelength of the particle as a function of height is (d) increases and decreases periodically
proportional to 7. An electron (mass m) with an initial velocity v = v0 i (v0 > 0) is
(a) H (b) H1/2 (c) H0 (d) H–1/2 in an electric field E = – E0 î (E0 = constant > 0). It's
2. The wavelength of a photon needed to remove a proton de-Broglie wavelength at time t is given by [NEET 2018, S]
from a nucleus which is bound to the nuclear with 1 MeV l0 æ eE t ö
energy is nearly (a) (b) ç 1 + 0 ÷
æ eE 0 t ö è mv 0 ø
(a) 1.2 nm (b) 1.2 × 10–3 nm ç1 + m v ÷
–6
(c) 1.2 × 10 nm (d) 1.2 × 10 nm è 0ø
(c) l0 (d) l0t
3. Consider a beam of electrons (each electron with energy E0)
incident on a metal surface kept in an evacuated chamber. 8. An electron (mass m) with an initial velocity v = v0 iˆ is in
Then, an electric field E = E ˆj . If l = h /mv, it's de-Broglie
0 0
(a) no electrons will be emitted as only photons can emit wavelength at time t is given by
electrons e 2 E 20 t 2
(a) l0 (b) l0 1 +
(b) electrons can be emitted but all with an energy, E0 m 2 v02
(c) electrons can be emitted with any energy, with a l0 l0
maximum of E0 – f (f is the work function)
e2 E 20 t 2 æ e 2 E 02 t 2 ö
(d) electrons can be emitted with any energy, with a (c) 1+ (d) ç1 + 2 2 ÷
m 2 v02 ç m v0 ÷
maximum of E0 è ø
4. Consider figure given below. Suppose the voltage applied Past Year MCQs
to A is increased. The diffracted beam will have the maximum
at value of q that 9. When the energy of the incident radiation is increased by
20%, the kinetic energy of the photoelectrons emitted from
(a) will be larger than the earlier value a metal surface increased from 0.5 eV to 0.8 eV. The work
(b) will be the same as the earlier value function of the metal is : [AIPMT 2014, A]
(c) will be less than the earlier value (a) 0.65 eV (b) 1.0 eV (c) 1.3 eV (d) 1.5 eV
(d) will depend on the target 10. A 15.0 eV photon collides with and ionizes a hydrogen atom.
– + If the atom was originally in the ground state (ionization
HT
potential =13.6 eV), what is the kinetic energy of the ejected
electron? [AIIMS 2014, A]
(a) 1.4 eV (b) 13.6 eV
Nickel (c) 15.0 eV (d) 28.6 eV
A Electron beam target
11. Light of wavelength 500 nm is incident on a metal with work
Electron
function 2.28 eV. The de Broglie wavelength of the emitted
LT gun electron is: [AIPMT 2015, A]
Diffracted
electron
Vaccum
(a) < 2.8 × 10 m -9 (b) ³ 2.8 × 10 m-9
Movable chamber
collector beam
(c) £ 2.8 × 10 m -12 (d) < 2.8 × 10-10 m
To galvanometer
12. A photoelectric surface is illuminated successively by
5. A proton, a neutron, an electron and an a-particle have l
same energy. Then, their de-Broglie wavelengths compare as monochromatic light of wavelength l and . If the maximum
2
(a) lp = ln > le > la (b) la < lp = ln > le kinetic energy of the emitted photoelectrons in the second
(c) le < lp = ln > la (d) le = lp = ln = la case is 3 times that in the first case, the work function of the
r surface of the material is :
6. An electron is moving with an initial velocity v = v0 i and
r (h = Planck's constant, c = speed of light) [AIPMT 2015, A]
is in a magnetic field B = B0 j . Then, it's de-Broglie
hc 2hc hc hc
wavelength (a) (b) (c) (d)
l l 3l 2l
DUAL NATURE OF RADIATION AND MATTER 381

13. Assertion : Photoelectric saturation current increases with 19. Radiation of wavelength l, is incident on a photocell. The
the increase in frequency of incident light. fastest emitted electron has speed v. If the wavelength is
Reason : Energy of incident photons increases with increase 3l
in frequency and as a result photoelectric current increases. changed to , the speed of the fastest emitted electron
4
[AIIMS 2015, C] will be: [JEE Main 2016, A]
(a) Assertion is correct, reason is correct; reason is a correct
1 1 1 1
explanation for assertion.
æ 4 ö2 æ 3 ö2 æ 4 ö2 æ 4 ö2
(b) Assertion is correct, reason is correct; reason is not a (a) = v çç ÷÷ (b) = v çç ÷÷ (c) > v çç ÷÷ (d) < v çç ÷÷
correct explanation for assertion è3 ø è4 ø è3ø è3ø
(c) Assertion is correct, reason is incorrect 20. Find the number of photon emitted per second by a 25 watt
(d) Assertion is incorrect, reason is correct. source of monochromatic light of wavelength 6600 Å. What
14. Assertion : In process of photoelectric emission, all emitted is the photoelectric current assuming 3% efficiency for
electrons do not have same kinetic energy. photoelectric effect ? [BITSAT 2016, S]
Reason : If radiation falling on photosensitive surface of a 25 25
metal consists of different wavelength then energy acquired (a) ´ 1019 J, 0.4amp (b) ´ 1019 J, 6.2 amp
3 4
by electrons absorbing photons of different wavelengths 25
shall be different. [AIIMS 2015, C] (c) ´ 1019 J, 0.8 amp (d) None of these
2
(a) Assertion is correct, reason is correct; reason is a correct
21. The photoelectric threshold wavelength of silver is 3250 ×
explanation for assertion.
10–10m. The velocity of the electron ejected from a silver
(b) Assertion is correct, reason is correct; reason is not a
surface by ultraviolet light of wavelength 2536 × 10–10 m is
correct explanation for assertion
(Given h = 4.14 × 10–15 eVs and c = 3 × 108 ms–1)
(c) Assertion is correct, reason is incorrect
(d) Assertion is incorrect, reason is correct. [NEET 2017, S]
(a) » 0.6 × 10 ms 6 –1 (b) » 61 × 103 ms–1
15. The beam of light has three wavelengths 4144Å,4972Å and
6216 Å with a total intensity of 3.6 × 10–3 Wm2 equally (c) » 0.3 × 106 ms–1 (d) » 6 × 105 ms–1
distributed amongst the three wavelengths. The beam falls 22. The de-Broglie wavelength of a neutron in thermal
normally on the area 1 cm2 of a clean metallic surface of equilibrium with heavy water at a temperature T (Kelvin)
work function 2.3 eV. Assume that there is no loss of light and mass m, is : [NEET 2017, C]
by reflection and that each energetically capable photon
ejects one electron. Calculate the number of photoelectrons h 2h 2h h
(a) (b) (c) (d)
liberated in 2s. [BITSAT 2015, S] 3mkT 3mkT mkT mkT
(a) 2 × 109 (b) 1.075 × 1012 23. The anode voltage of a photocell is kept fixed. The
(c) 9 × 108 (d) 3.75 × 106 wavelength l of the light falling on the cathode is gradually
16. When 0.50 Å X-rays strike a material, the photoelectrons changed. The plate current I of the photocell varies as
from the k shell are observed to move in a circle of radius 23 follows [AIIMS 2017, C]
mm in a magnetic field of 2 × 10 –2 tesla acting
I I
perpendicularly to the direction of emission of
photoelectrons. What is the binding energy of k-shell
electrons? [BITSAT 2015, S] (a) (b)
(a) 3.5 keV (b) 6.2 keV (c) 2.9 keV (d) 5.5 keV O l O l
17. An electron of mass m and a photon have same energy E.
The ratio of de-Broglie wavelengths associated with them I I
is : [NEET 2016, C]
1 1 1 (c) (d)
1 æ E ö2 1
æ E ö2 1 æ 2m ö 2
O
(a) ç ÷ (b) çè ÷ø (c) c(2mE) 2 (d) ç ÷ l O l
c è 2m ø 2m xc è E ø
24. A particle A of mass m and initial velocity v collides with a
18. When a metallic surface is illuminated with radiation of m
wavelength l, the stopping potential is V. If the same surface particle B of mass which is at rest. The collision is head
2
is illuminated with radiation of wavelength 2l, the stopping on, and elastic. The ratio of the de-Broglie wavelengths lA
V to lB after the collision is [JEE Main 2017, S]
potential is . The threshold wavelength for the metallic
4 (a) 2/3 (b) 1/2 (c) 1/3 (d) 2
surface is : [NEET 2016, A]
25. An electron beam is accelerated by a potential difference V
5
(a) 4l (b) 5l (c) l (d) 3l to hit a metallic target to produce X-rays. It produces
2 continuous as well as characteristic X-rays.If lmin is the
EBD_7418
382 PHYSICS

smallest possible wavelength of X-ray in the spectrum, the (a) 1 : 2 (b) 1 : 4 (c) 2 : 1 (d) 4 : 1
variation of log lmin with log V is correctly represented in : 28. Assertion : Threshold frequency depends on metal is
[JEE Main 2017, C] Reason : Threshold wavelength µ Threshold frequency
[AIIMS 2018, C]
(a) Assertion is correct, reason is correct; reason is a correct
(a) (b)
explanation for assertion.
(b) Assertion is correct, reason is correct; reason is not a
correct explanation for assertion
(c) (d) (c) Assertion is correct, reason is incorrect
(d) Assertion is incorrect, reason is correct.
26. When a metal surface is illuminated by light of wavelengths 29. In a photoelectric effect experiment, for radiation with
400 nm and 250 nm, the maximum velocities of the frequency u0 with hu0 = 8eV, electrons are emitted with
photoelectrons ejected are v and 2v respectively. The work energy 2 eV. What is the energy of the electrons emitted for
function of the metal is (h - Planck's constant, c = velocity of incoming radiation of frequency 1.25 u0 ? [AIIMS 2018, A]
light in air) [BITSAT 2017, A] (a) 1 eV (b) 3.25 eV (c) 4 eV (d) 9.25 eV.
(a) 2 hc ×106 J (b) 1.5 hc × 106 J
30. In an electron gun, the potential difference between the
(c) hc × 106 J (d) 0.5 hc × 106 J filament and plate is 3000 V. What will be the velocity of
27. When the light of frequency 2n0 (where n0 is threshold electron emitting from the gun? [BITSAT 2018, S]
frequency), is incident on a metalplate, the maximum velocity
of electronsemitted is v1. When the frequency of the incident (a) 3 × 108 m/s (b) 3.18 × 107 m/s
radiation is increased to 5n0, the maximum velocity of (c) 3.52 × 107 m/s (d) 3.26 × 107 m/s
electrons emitted from the same plate is v2. The ratio of v1 to
v2 is [NEET 2018, A]

Exercise 3 : Try If You Can


1. A photosensitive metallic surface has work function, hn0. (a) |Py| d > h
If photons of energy 2 hn0 fall on this surface, the Y
electrons come out with a maximum velocity of 4 × 106 m/
s. When the photon energy is increased to 5 hn0, then (b) |Py| d < h
maximum velocity of photoelectrons will be X
(a) 2 × 107 m/s (b) 2 × 106 m/s (c) |Py| d ; h
6
(c) 8 × 10 m/s (d) 8 × 105 m/s
2. Two identical photocathodes receive light of frequencies f1
and f2. If the velocites of the photo electrons (of mass m ) (d) |Py| d > >h
coming out are respectively v1 and v2, then 5. When photons of wavelength l1 are incident on an isolated
1/ 2 sphere, the corresponding stopping potential is found to
2h é 2h ù be V. When photons of wavelength l2 are used, the
(a) v12 - v2 2 = ( f1 - f 2 ) (b) v1 + v2 = ê ( f1 + f 2 ) ú
m ë m û corresponding stopping potential was thrice that of the
1/ 2 above value. If light of wavelength l3 is used then find the
2 2 2h é 2h ù stopping potential for this case :
(c) v1 + v2 = ( f1 + f 2 ) (d) v1 - v2 = ê ( f1 - f 2 ) ú
m ëm û
hc é 1 1 1ù hc é 1 1 1ù
3. A metal surface is illuminated by light of two different (a) ê + - ú (b) ê + - ú
wavelengths 248 nm and 310 nm. The maximum speeds of e ë l 3 l 2 l1 û e ë l3 2l 2 l1 û
the photoelectrons corresponding to these wavelengths are
u1 and u2, respectively. If the ratio u1 : u2 = 2 : 1 and hc = hc é 1 3 1 ù hc é 1 1 3 ù
(c) ê - + ú (d) ê + - ú
1240 eV nm, the work function of the metal is nearly e ë l3 2l 2 2l1 û e ë l3 2l 2 2l1 û
(a) 3.7 eV (b) 3.2 eV (c) 2.8 eV (d) 2.5 eV 6. A beam of light has two wavelengths of 4972Å and 6216Å
4. A parallel beam of electrons travelling in with a total intensity of 3.6 × 10–3 Wm–2 equally distributed
x-direction falls on a slit of width d (see figure). If after among the two wavelengths. The beam falls normally on an
passing the slit, an electron acquires momentum py in the
area of 1 cm2 of a clean metallic surface of work function
y-direction then for a majority of electrons passing through
2.3 eV. Assume that there is no loss of light by reflection
the slit (h is Planck’s constant) :
DUAL NATURE OF RADIATION AND MATTER 383

and that each capable photon ejects one electron. The K

number of photoelectrons liberated in 2s is approximately:


K
(a) 6 × 1011 (b) 9 × 1011 (c) 11 × 1011 (d) 15 × 1011
7. A copper ball of radius 1 cm and work function 4.47eV is K
irradiated with ultraviolet radiation of wavelength 2500 Å. Fixed
The effect of irradiation results in the emission of electrons
from the ball. Further the ball will acquire charge and due to Smooth surface
this there will be a finite value of the potential on the ball.
The charge acquired by the ball is : (a) IA / Kc (b) 2Ia/3Kc (c) 3Ia/Kc (d) 4Ia/3Kc
(a) 5.5×10–13C (b) 7.5 × 10–13C 12. A photon has same wavelength as the de Broglie
(c) 4.5 × 10–12C (d) 2.5 × 10–11C wavelength of electrons. Given C = speed of light, v = speed
8. Light of wavelength 0.6 µm from a sodium lamp falls on a of electron. Which of the following relation is correct? [Here
photocell and causes the emission of photoelectrons for Ee = kinetic energy of electron, Eph = energy of photon, Pe
which the stopping potential is 0.5 V. With light of = momentum of electron and Pph = momentum of photon]
wavelength 0.4 µm from a mercury vapor lamp, the stopping (a) Ee/Eph = 2 C/v (b) Ee/Eph = v/2C
potential is 1.5 V. Then, the work function [in electron volts] (c) Pe/Pph = 2C/v (d) Pe / Pph = C/v
of the photocell surface is 13. A sensor is exposed for time t to a lamp of power P placed
(a) 0.75 eV (b) 1.5 eV (c) 3 eV (d) 2.5 eV at a distance l. The sensor has an opening that is 4d in
9. An electron is accelerated through a potential difference of
diameter. Assuming all energy of the lamp is given off as
V volt. It has a wavelength l associated with it. Through
light, the number of photons entering the sensor if the
what potential difference an electron must be accelerated wavelength of light is l is
so that its de Broglie wavelength is the same as that of a (a) N = Pld2t/hcl2 (b) N = 4Pld2t / hcl2
proton? Take mass of proton to be 1837 times larger than (c) N = Pld t / 4hcl
2 2
(d) N = Pld2t / 16 hcl2
the mass of electron. 14. In a photoelectric experiment, with light of wavelength l,
(a) V volt (b) 1837 V volt the fastest electron has speed v. If the exciting wavelength
(c) V/1837 volt (d) 1837 V volt is changed to 5l/4, the speed of the fastest emitted electron
10. A homogeneous ball (mass = m) of ideal black material at will become
rest is illuminated with a radiation having a set of photons 5 5
(wavelength = l), each with the same momentum and the (a) v (b) v
4 3
same energy. The rate at which photons fall on the ball is n. 5 5
The linear acceleration of the ball is. (c) less than v (d) greater than v
3 3
(a) ml/nh (b) nh/ml 15. Light of wavelength l from a small 0.5 mW He – Ne laser
(c) nh/ (2p) (ml) (d) 2pml/nh source, used in the school laboratory, shines from a
11. Light of intensity I is incident perpendicularly on a perfectly spacecraft of mass 1000 kg. Estimate the time needed for
reflecting plate of area A kept in a gravity free space. If the the spacecraft to reach a velocity of 1.0 km s–1 from rest.
photons strike the plate symmetrically and initially the The momentum l of a photon of wavelength l is given by
spring was at its natural length, find the maximum p = h/l, where h is Planck's constant.
compression in the springs. (a) 6 × 1018 (b) 3 × 1017 (c) 6 × 1017 (d) 2 × 1015
EBD_7418
384 PHYSICS

ANSW ER KEYS
Exercise 1 : Topic-wis e MCQs
1 (c) 11 (a) 21 (c) 31 (b) 41 (d) 51 (b) 61 (a) 71 (d) 81 (d) 91 (d)
2 (b) 12 (d) 22 (d) 32 (a) 42 (a) 52 (c) 62 (d) 72 (a) 82 (b) 92 (a)
3 (b) 13 (d) 23 (d) 33 (c) 43 (b) 53 (b) 63 (b) 73 (b) 83 (c) 93 (a)
4 (a) 14 (b) 24 (b) 34 (b) 44 (c) 54 (b) 64 (d) 74 (a) 84 (c) 94 (d)
5 (b) 15 (b) 25 (d) 35 (d) 45 (b) 55 (a) 65 (a) 75 (a) 85 (c) 95 (a)
6 (c) 16 (d) 26 (b) 36 (d) 46 (c) 56 (c) 66 (a) 76 (d) 86 (b) 96 (a)
7 (c) 17 (d) 27 (a) 37 (a) 47 (b) 57 (a) 67 (b) 77 (a) 87 (a) 97 (b)
8 (c) 18 (a) 28 (c) 38 (a) 48 (c) 58 (d) 68 (d) 78 (d) 88 (a) 98 (a)
9 (a) 19 (b) 29 (c) 39 (c) 49 (a) 59 (c) 69 (d) 79 (d) 89 (b) 99 (d)
10 (a) 20 (b) 30 (d) 40 (a) 50 (d) 60 (d) 70 (b) 80 (c) 90 (b) 100 (a)
Exercis e 2 : Exemplar & Pas t Year MCQs
1 (d) 4 (c) 7 (a) 10 (a) 13 (d) 16 (b) 19 (c) 22 (a) 25 (c) 28 (c)
2 (b) 5 (b) 8 (c) 11 (b) 14 (a) 17 (a) 20 (a) 23 (d) 26 (a) 29 (c)
3 (d) 6 (a) 9 (b) 12 (d) 15 (b) 18 (d) 21 (a, d) 24 (d) 27 (a) 30 (d)
Exercis e 3 : Try If You Can
1 (c) 3 (a) 5 (c) 7 (a) 9 (c) 11 (d) 13 (a) 15 (c)
2 (a) 4 (a) 6 (b) 8 (b) 10 (b) 12 (b) 14 (d)
27Chapter
ATOMS

Trend
Analysis of NEET and AIIMS (Year 2010-2018)
5

4
Number of Questions

3
AIPMT/NEET
2 AIIMS

0
2010 2011 2012 2013 2014 2015 2016 2017 2018
Year

Trend
of JEE Main and BITSAT (Year 2010-2018)
Analysis
5

4
Number of Questions

3
JEE Main
2 BITSAT

0
2010 2011 2012 2013 2014 2015 2016 2017 2018

Year

Chapter Utility Score (CUS)


Exam Weightage Important Concepts Difficulty Level CUS (Out of 10)
NEET 6 Rutherford & Bohr’s Model
AIIMS 5 and Spectra of Hydrogen 3/5 6/10
JEE Main 4 Atom
BITSAT 3
EBD_7418
386 PHYSICS
ATOMS 387
EBD_7418
388 PHYSICS

Exercise 1 : Topic-wise MCQs

Topic 1: Atomic Structure and Rutherford's Nuclear Model head on. The correct relation between the distance of closest
approach D and T is
1. According to classical theory, the path of an electron in 1 1
Rutherford atomic model is (a) D µ (b) D µ T (c) D µ T 2 (d) D µ 2
T T
(a) spiral (b) circular 12. In a Rutherford scattering experiment when a projectile of
(c) parabolic (d) straight line charge Z1 and mass M1approaches a target nucleus of
2. Rutherford’s a-particle experiment showed that the atoms
charge Z2 and mass M2, the distance of closest approach
have
(a) Proton (b) Nucleus is r0. The energy of the projectile is
(c) Neutron (d) Electrons (a) directly proportional to Z1 Z2
3. Rutherford’s atomic model was unstable because (b) inversely proportional to Z1
(a) nuclei will break down (c) directly proportional to mass M1
(b) electrons do not remain in orbit (d) directly proportional to M1 × M2
(c) orbiting electrons radiate energy 13. The diagram shows the path of four a-particles of the same
(d) electrons are repelled by the nucleus energy being scattered by the nucleus of an atom
4. As one considers orbits with higher values of n in a simulateneously which of those is not physically possible?
hydrogen atom, the electric potential energy of the atom
(a) decreases (b) increases 1.
(c) remains the same (d) does not increase
2.
5. The significant result deduced from the Rutherford's 3.
scattering experiment is that 4.
(a) whole of the positive charge is concentrated at the
centre of atom (a) 3 and 4 (b) 2 and 3 (c) 1 and 4 (d) 4 only
(b) there are neutrons inside the nucleus 14. Assertion : Electrons in the atom are held due to coulomb
(c) a-particles are helium nuclei forces.
(d) electrons are embedded in the atom
Reason : The atom is stable only because the centripetal
(e) electrons are revolving around the nucleus
6. When an a-particle of mass 'm' moving with velocity 'v' force due to Coulomb's law is balanced by the centrifugal
bombards on a heavy nucleus of charge 'Ze', its distance of force.
closest approach from the nucleus depends on v as : (a) If both Assertion and Reason are correct and Reason
1 1 1 is the correct explanation of Assertion.
(a) (b) (c) (d) v (b) If both Assertion and Reason are correct, but Reason
v v v2
7. In Rutherford's a -particle scattering experiment, what will be is not the correct explanation of Assertion.
correct angle for a scattering for an impact parameter b = 0 ? (c) If Assertion is correct but Reason is incorrect.
(a) 90º (b) 270º (c) 0º (d) 180º (d) If both the Assertion and Reason are incorrect.
8. Which of the following parameters is the same for all 15. The observations of Geiger–Marsden experiment are
hydrogen-like atoms and ions in their ground states? I. many of a particles pass straight through the gold foil.
(a) Radius of the orbit II. only about 0.14% of a-particles scatter by more than 1°
(b) Speed of the electron III. about 1 in 8000 of a-particles is deflected more than 90°.
(c) Energy of the atom IV. very few particles are reflected back.
(d) Orbital angular momentum of the electron (a) I, II and IV (b) I, II and III
9. An a-particle of energy 5 MeV is scattered through 180º by a (c) II, III and IV (d) I, II III and IV
fixed uranium nucleus. The distance of closest approach is of 16. In an atom, the two electrons move round the nucleus in
the order of circular orbits of radii R and 4R. The ratio of the time taken
(a) 10–12 cm (b) 10–10 cm by them to complete one revolution is
(c) 10 cm–20 (d) 10–15 cm (a) 1/4 (b) 4/1 (c) 8/1 (d) 1/8
10. The distance of closest approach of a certain nucleus is 7.2 17. An a particle passes rapidly through the exact centre of a
fm and it has a charge of 1.28 × 10–17 C. The number of hydrogen molecule, moving on a line perpendicular to the
neutrons inside the nucleus of an atom is internuclear axis. The distance between the nuclei is b.
(a) 136 (b) 142 (c) 140 (d) 132 Where on its path does the a particle experience the greatest
11. If the collision between the incident a-particle whose kinetic force? (Assume that the nuclei do not move much during
energy is T and electric charge 2e and the nucleus were the passage of the a particle. Also neglect the electric field
ATOMS 389

of the electrons in the molecule.) 25. Which of the following in a hydrogen atom is independent
b of the principal quantum number n? (The symbols have
(a) their usual meanings).
2 +e (a) nn (b) Er (c) En (d) nr
b 26. If the angular momentum of an electron in an orbit is J then
(b) the K.E. of the electron in that orbit is
2 2 +2e b
J2 Jv J2 J2
b a -particle (a) 2 (b) (c) (d)
(c) +e 2mr r 2m 2p
2 27. If the frequency of revolution of electron in an orbit in H
H2 molecule atom is n then the equivalent current is
(d) None of these 2p re en
18. An a-particle of 10 MeV collides head-on with a copper (a) (b) 2p r (c) e2 pn (d) en
n
nucleus (Z = 29) and is deflected back. Then, the minimum 28. Ionization energy of a hydrogen-like ion A is greater than
distance of approach between the centres of the two is: that of another hydrogen-like ion B. If r, u, E and L represent
(a) 8.4 × 10–15 cm (b) 8.4 × 10–15 m the radius of the orbit, speed of the electron, energy of the
(c) 4.2 × 10–15 m (d) 4.2 × 10–15 cm atom and orbital angular momentum of the electron
respectively then in ground state
19. In Rutherford’s experiment, the number of a-particles
(a) rA > rB (b) uA > uB
scattered through an angle of 60o by a silverfoil is 200 per
(c) EA > EB (d) LA > LB
minute. When the silver foil is replaced by a copper foil of
the same thickness, the number of a-particles scattered 29. In which of the following series, does the 121.5 nm line of
the spectrum of the hydrogen atom lie ?
through an angle of 60o per minute is:
(a) Lyman series (b) Balmer series
2
200 ´ ZCu æZ ö (c) Paschen series (d) Brackett series.
(a) (b) 200 ´ ç Cu ÷ 30. Hydrogen atom excites energy level from fundamental state
ZAg ç ZAg ÷
è ø to n = 3. Number of spectral lines according to Bohr, is
2 (a) 4 (b) 3 (c) 1 (d) 2
ZAg æ ZAg ö 31. The linear speed of an electron, in Bohr’s orbit is given by
(c) 200 ´ (d) 200 ´ çç ÷÷
ZCu è ZCu ø e2 e2 2 Î0 nh
(a) (b) (c) (d) 2Î0 h
h 2 Î0 nh e
Topic 2: Bohr's Model and the Spectra of the Hydrogen Atom 32. Which of the following series in the spectrum of hydrogen
20. According to Bohr’s model of hydrogen atom atom lies in the visible region of the electromagnetic
(a) the linear velocity of the electron is quantised spectrum?
(b) the angular velocity of the electron is quantised (a) Paschen series (b) Balmer series
(c) the linear momentum of the electron is quantised (c) Lyman series (d) Brackett series
(d) the angular momentum of the electron is quantised 33. The Balmer series for the H-atom can be observed
21. As the quantum number increases, the difference of energy (a) if we measure the frequencies of light emitted when
between consecutive energy levels an excited atom falls to the ground state
(a) remain the same (b) if we measure the frequencies of light emitted due
(b) increases to transitions between excited states and the first
(c) decreases excited state
(d) sometimes increases and sometimes decreases. (c) in any transition in a H-atom
22. When hydrogen atom is in its first excited level, it’s radius is (d) None of these
(a) four times, it ground state radius 34. In a hydrogen atom, which of the following electronic
(b) twice times, it ground state radius transitions would involve the maximum energy change
(c) same times, it ground state radius (a) n = 2 to n = 1 (b) n = 3 to n = 1
(d) half times, it ground state radius. (c) n = 4 to n = 2 (d) n = 3 to n = 2
23. When an electron jumps from the fourth orbit to the 35. Angular speed of an electron in a Bohr’s orbit is given by
second orbit, one gets the pme 4 4 Î20 n 3h 3
(a) second line of Paschen series (a) w= (b) w =
(b) second line of Balmer series 2 Î20 n 3h 3 me4
(c) first line of Pfund series me4
(d) second line of Lyman series (c) w= (d) all of these
24. In terms of Bohr radius r0, the radius of the second Bohr orbit 4 Î02 n 3 h 3
of a hydrogen atom is given by 36. If ‘r’ is the radius of the lowest orbit of Bohr’s model of
H-atom, then the radius of nth orbit is
(a) 4 r0 (b) 8 r0 (c) 2 r0 (d) 2 r0
(a) r n2 (b) 2r (c) n2/r (d) r n
EBD_7418
390 PHYSICS

37. The speed of electron in first Bohr orbit is c/137. The speed 48. Which of the plots shown in the figure represents speed
of electron in second Bohr orbit will be (v) of the electron in a hydrogen atom as a function of the
2c 4c c c principal quantum number (n)?
(a) (b) (c) (d) A C
137 137 274 548
38. The Lyman transitions involve (a) B
v
(a) largest changes of energy (b) D D
(b) smallest changes of energy
(c) largest changes of potential energy (c) C
B
(d) smallest changes of potential energy o
(d) A 1 2 3 4
n
39. According to the Bohr theory of H-atom, the speed of the
49. An electron makes a transition from outer orbit (n = 4) to
electron, its energy and the radius of its orbit varies with
the inner orbit (p = 2) of a hydrogen atom. The wave number
the principal quantum number n, respectively, as
of the emitted radiations is
1 2 1 1 2 2R 3R 4R 5R
(a) ,n , (b) n, 2 , n (a) (b) (c) (d)
n 2
n n 16 16 16 16
1 1 1 1 2 50. In Bohr model of hydrogen atom, let P.E. represents
(c) n, 2 , 2 (d) , ,n potential energy and T.E. represents the total energy. In
n n n n2
40. The transition from the state n = 4 to n = 3 in a hydrogen going to a higher level.
like atom results in ultraviolet radiation. Infrared radiation (a) P. E. decreases, T.E. increases
will be obtained in the transition from (b) P. E. increases, T.E. decreases
(c) P. E. decreases, T.E. decreases
(a) 2 ® 1 (b) 3 ® 2 (c) 4 ® 2 (d) 5 ® 4
(d) P. E. increases, T.E. increases
41. In Balmer series of emission spectrum of hydrogen, first 51. The shortest wavelength in Balmer’s series for Hydrogen
four lines with different wavelength Ha Hb Hg and Hd are
atom is ...A... and this is obtained by substituting ...B ...
obtained. Which line has maximum frequency out of in Balmer’s formula. Here, A and B refer to
these?
(a) 656.3 nm, n = 3 (b) 486.1 nm, n = 4
(a) Ha (b) Hb (c) Hg (d) Hd (c) 410.2 nm, n = 5 (d) 364.6 nm, n = ¥
42. In a hypothetical Bohr hydrogen atom, the mass of the 52. Assertion : Hydrogen atom consists of only one electron
electron is doubled. The energy E¢0 and radius r¢0 of the but its emission spectrum has many lines.
first orbit will be (r0 is the Bohr radius) Reason : Only Lyman series is found in the absorption
(a) –11.2 eV (b) –6.8 eV (c) –13.6 eV (d) –27.2 eV sepectrum of hydrogen atom whereas in the emission
43. If the radius of hydrogen atom in its ground state is spectrum, all the series are found.
5.3 × 10–11 m. After collision with an electron it is found to (a) If both Assertion and Reason are correct and Reason
have a radius of 21.2 × 10–11 m. The principle quantum is the correct explanation of Assertion.
number of the final orbit is (b) If both Assertion and Reason are correct, but Reason
(a) n = 4 (b) n = 3 (c) n = 2 (d) n = 16 is not the correct explanation of Assertion.
(c) If Assertion is correct but Reason is incorrect.
44. In the hydrogen atom, an electron makes a transition from
n = 2 to n = 1. The magnetic field produced by the circulating (d) If both the Assertion and Reason are incorrect.
electron at the nucleus 53. The diagram shows the energy levels for an electron in a
(a) decreases 16 times (b) increases 4 times certain atom. Which transition shown represents the
(c) decreases 4 times (d) increases 32 times emission of a photon with the most energy?
n=4
45. The ionization energy of hydrogen atom is 13.6 eV. Following n=3
Bohr’s theory, the energy corresponding to a transition
between 3rd and 4th orbit is n=2
(a) 3.40 eV (b) 1.51 eV (c) 0.85 eV (d) 0.66 eV
46. The largest wavelength in the ultraviolet region of the n =1
hydrogen spectrum is 122 nm. The smallest wavelength in I II III IV
the infrared region of the hydrogen spectrum (to the nearest (a) 4 (b) 3 (c) 2 (d) 1
integer) is 54. In a hydrogen atom following the Bohr’s postulates the
(a) 802 nm (b) 823 nm (c) 1882 nm (d) 1648 nm product of linear momentum and angular momentum is
47. A stationary hydrogen atom of mass M emits a photon proportional to (n)x where ‘n’ is the orbit number. Then ‘x’ is
corresponding to the first line of Lyman series. If R is the (a) 0 (b) 2 (c) –2 (d) 1
Rydberg's constant, the velocity that the atom acquires is 55. The energy of a hydrogen atom in the ground state is
– 13.6 eV. The energy of a He+ ion in the first excited state
3 Rh Rh Rh Rh will be
(a) (b) (c) (d)
4 M 4M 2M M (a) –13.6 eV (b) – 27.2 eV (c) – 54.4 eV (d) – 6.8 eV
ATOMS 391

56. Out of the following which one is not a possible energy for 71. The energy of electron in the nth orbit of hydrogen atom is
a photon to be emitted by hydrogen atom according to -13.6
Bohr’s atomic model? expressed as E n = eV. The shortest and longest
(a) 1.9 eV (b) 11.1 eV (c) 13.6 eV (d) 0.65 eV n2
57. If 13.6 eV energy is required to ionize the hydrogen atom, wavelength of Lyman series will be
then the energy required to remove an electron from n = 2 is (a) 910 Å, 1213 Å (b) 5463 Å, 7858 Å
(a) 10.2 eV (b) 0 eV (c) 3.4 eV (d) 6.8 eV. (c) 1315 Å, 1530 Å (d) None of these
58. Energy required for the electron excitation in Li++ from the 72. The longest wavelength of the Balmer series is 6563 Å. The
first to the third Bohr orbit is Rydberg’s constant is
(a) 36.3 eV (b) 108.8 eV (c) 122.4 eV (d) 12.1 eV (a) 1.09 × 105 m–1 (b) 1.09 × 106 m–1
59. The ratio of the energies of the hydrogen atom in its first to (c) 1.09 × 107 m–1 (d) 1.09 × 108 m–1
second excited states is
73. The first line of the Lyman series in a hydrogen spectrum
(a) 1/4 (b) 4/9 (c) 9/4 (d) 4 has a wavelength of 1210 Å. The corresponding line of
60. Excitation energy of a hydrogen like ion in its excitation a hydrogen-like atom of Z = 11 is equal to
state is 40.8 eV. Energy needed to remove the electron from
(a) 4000 Å (b) 100 Å (c) 40 Å (d) 10 Å
the ion in ground state is
(a) 54.4 eV (b) 13.6 eV (c) 40.8 eV (d) 27.2 eV 74. Four lowest energy levels of H-atom are shown in the figure.
61. A hydrogen atom in its ground state absorbs 10.2 eV of The number of possible emission lines would be
energy. The orbital angular momentum is increased by n=4
n=3
(a) 1.05 × 10–34 J-s (b) 3.16 × 10–34 J-s
(c) 2.11 × 10 J-s–34 (d) 4.22 × 10–34 J-s n=2
62. Hydrogen atom is excited from ground state to another
state with principal quantum number equal to 4. Then the n =1
number of spectral lines in the emission spectra will be (a) 3 (b) 4 (c) 5 (d) 6
(a) 2 (b) 3 (c) 5 (d) 6 75. Electron in hydrogen atom first jumps from third excited
63. The extreme wavelengths of Paschen series are state to second excited state and then from second excited
(a) 0.365 mm and 0.565 mm (b) 0.818 mm and 1.89 mm to thefirst excited state. Theratio of thewavelength l1 : l2
(c) 1.45 mm and 4.04 mm (d) 2.27 mm and 7.43 mm emitted in the two cases is
64. The first line of Balmer series has wavelength 6563 Å. What (a) 7/5 (b) 27/20 (c) 27/5 (d) 20/7
will be the wavelength of the first member of Lyman series
76. An electron of a stationary hydrogen atom passes from the
(a) 1215.4 Å (b) 2500 Å (c) 7500 Å (d) 600 Å
fifth energy level to the ground level. The velocity that the
65. Taking Rydberg’s constant RH = 1.097 × 107m, first and
atom acquired as a result of photon emission will be
second wavelength of Balmer series in hydrogen spectrum
is 24 hR 25hR 25m 24m
(a) (b) (c) (d)
(a) 2000 Å, 3000 Å (b) 1575 Å, 2960 Å 25m 24m 24hR 25hR
(c) 6529 Å, 4280 Å (d) 6563 Å, 4861 Å 77. If the atom 100Fm257 follows the Bohr model and the radius
66. The third line of Balmer series of an ion equivalnet to of 100Fm257 is n times the Bohr radius, then find n.
hydrogen atom has wavelength of 108.5 nm. The ground
state energy of an electron of this ion will be (a) 100 (b) 200 (c) 4 (d) 1/4
th
78. If in hydrogen atom, radius of n Bohr orbit is rn, frequency
(a) 3.4 eV (b) 13.6 eV (c) 54.4 eV (d) 122.4 eV
67. The wavelength of the first spectral line in the Balmer series of revolution of electron in n th orbit is fn, choose the correct
of hydrogen atom is 6561 A°. The wavelength of the second option.
spectral line in the Balmer series of singly-ionized helium rn
atom is ær ö
lo g ç n ÷
(a) 1215 A° (b) 1640 A° (c) 2430 A° (d) 4687 A° è r1 ø
68. According to Bohr’s theory, the wave number of last line
(a) (b)
of Balmer series is (Given R = 1.1 × 107 m–1)
(a) 5.5 × 105 m–1 (b) 4.4 × 107 m–1 O n O log n
6 –1
(c) 2.75 × 10 m (d) 2.75 × 108 m–1
æf ö
69. If the wavelength of the first line of the Balmer series of log ç n ÷
è f1 ø
hydrogen is 6561 Å, the wavelength of the second line
of the series should be (c) (d) Both (a) and (b)
(a) 13122 Å (b) 3280 Å (c) 4860 Å (d) 2187 Å
O log n
70. The energy of an excited state of H atom is –0.85 eV. What
will be the quantum number of the orbit, if the ground state 79. An electron jumps from the 4th orbit to the 2nd orbit
energy for hydrogen is –13.6 eV? of hydrogen atom. Given the Rydberg’s constant R = 105
(a) 4 (b) 3 (c) 2 (d) 1 cm–1. The frequency in Hz of the emitted radiation is
EBD_7418
392 PHYSICS

3 3 86. In the Bohr 's model of a hydrogen atom, the centripetal


(a) ´ 105 (b) ´ 1015 force is furnished by the coulomb attraction between the
16 16 proton and the electron. If a0 is the radius of the ground
9 3 state orbit, m is the mass and e is charge on the electron
(c) ´ 1015 (d) ´ 1015
16 4 and Î0 is the vacuum permittivity, the speed of the
80. In an inelastic collision an electron excites as hydrogen electron is
atom from its ground state to a M-shell state. A second e
electron collides instantaneously with the excited hydrogen (a) Zero (b) Î a m
0 0
atom in the M-state and ionizes it. At least how much energy
e 4 pÎ0 a 0 m
the second electron transfers to the atom in the M-state?
(c) 4pÎ0 a 0 m (d)
(a) + 3.4 eV (b) + 1.51 eV(c) – 3.4 eV (d) – 1.51 eV e
81. If one were to apply Bohr model to a particle of mass ‘m’ 87. If u1 is the frequency of the series limit of Lyman series,
and charge ‘q’ moving in a plane under the influence of a u2 is the frequency of the first line of Lyman series and
magnetic field ‘B’, the energy of the charged particle in the u3 is the frequency of the series limit of the Balmer series
nth level will be : then
(a) u1 - u2 = u3 (b) u1 = u2 - u3
æ hqB ö æ hqB ö
(a) n çè ÷ (b) n çè ÷ 1 1 1 1 1 1
2pm ø 8pm ø (c) = + (d) = +
u2 u1 u3 u1 u2 u3
æ hqB ö æ hqB ö 88. The radiation corresponding to 3 ® 2 transition of
(c) n çè ÷ø (d) n çè ÷
4pm pm ø hydrogen atom falls on a metal surface to produce
82. The wavelength of radiation is l0 when an electron jumps photoelectrons. These electrons are made to enter a
from third to second orbit of hydrogen atom. For the electron magnetic field of 3 × 10–4 T. If the radius of the largest
to jump from the fourth to the second orbit of the hydrogen circular path followed by these electrons is 10.0 mm, the
atom, the wavelength of radiation emitted will be work function of the metal is close to:
16 20 27 25 (a) 1.8 eV (b) 1.1 eV (c) 0.8 eV (d) 1.6 eV
(a) l 0 (b) l 0 (c) l 0 (d) l0 89. A hydrogen atom and a doubly ionized lithium atom are
25 27 20 16
83. In Bohr’s model of the hydrogen atom, let R, V, T and E both in the second excited state. If LH and LLi respectively
represent the radius of the orbit, speed of the electron, time represent their electronic angular momenta and EH and ELi
period of revolution of electron and the total energy of the their energies, then
electron respectively. The quantity proportional to the (a) LH > LLi and | EH | > | ELi |
quantum number n is
(b) LH = LLi and | EH | < | ELi |
(a) VR (b) E (c) r (d) T
84. Maximum energy evolved during which of the following (c) LH = LLi and | EH | > | ELi |
transition? (d) LH < LLi and | EH | < | ELi |
(a) n = 1 to n = 2 (b) n = 2 to n = 1 90. Suppose potential energy between electron and proton at
(c) n = 2 to n = 6 (d) n = 6 to n = 2 separation r is given by U = K ln (r), where K is a constant.
85. The energy levels of the hydrogen spectrum is shown in For such a hypothetical hydrogen atom, the ratio of energy
figure. There are some transitions A, B, C, D and E. difference between energy levels (n = 1 and n = 2) and (n =
Transition A, B and C respectively represent 2 and n = 4) is
n=¥ – 0.00 eV (a) 1 (b) 2 (c) 3 (d) 4
n=6 – 0.36 eV 91. An unknown hot gas emits radiation of wavelengths 46.0
n=5 – 0.54 eV nm, 82.8 nm and 103.5 nm only. Assume that the atoms have
n=4 – 0.85 eV
n=3
C
– 1.51 eV only two excited states and the difference between consecutive
B D
n=2 – 3.39 eV energy levels decreases as energy is increased. Taking the
A E energy of the highest energy state to be zero, find the
energies of the ground state and the first excited state.
n=1 – 13.5 eV
(a) –27 eV, – 12 eV (b) –6 eV, – 3 eV
(a) first member of Lyman series, third spectral line of
Balmer series and the second spectral line of Paschen (c) –11 eV, – 8 eV (d) –9 eV, – 3 eV
series 92. The difference between the longest wavelength line of the
(b) ionization potential of hydrogen, second spectral line Balmer series and shortest wavelength line of the Lyman
of Balmer series, third spectral line of Paschen series series for a hydrogenic atom (atomic number Z) equal to Dl
. The value of the Rydberg constant for the given atom is :
(c) series limit of Lyman series, third spectral line of Balmer
series and second spectral line of Paschen series
5 1 5 Z2
(a) 2 (b)
(d) series limit of Lyman series, second spectral line of 31 Dl.Z 36 Dl
Balmer series and third spectral line of Paschen series 31 1
(c) (d) none of these
5 Dl.Z 2
ATOMS 393

93. When an electron in a hydrogen atom makes a transition of n in the intermediate state.
from 2nd excited stated to ground state it emits a photon of (a) 12.1 eV, n = 3 (b) 6.1 eV, n = 4
frequency f. The frequency of photon emitted when an (c) 2.1 e V, n = 3 (d) 1.1 eV, n = 5
electron of Li++ makes a transition from Ist excited state to 98. Assume in specific conditions only those transitions are
ground state is allowed to hydrogen atoms in which the principal quantum
243 141 81 63 number n changes by 2. Find the smallest wavelength
(a) f (b) f (b) f (d) f
32 32 32 32 emitted by hydrogen and wavelenght emitted by hydrogen
94. The photon radiated from hydrogen corresponding to 2nd in the visible range (380 nm to 780 nm). respectively (in nm)
line of Lyman series is absorbed by a hydrogen like atom X (a) 103, 487 (b) 123, 532 (c) 211, 410 (c) 320, 435
in 2nd excited state. As a result the hydrogen like atom X
makes a transition to nth orbit. Then – 99. Ultraviolet light of wavelengths l1 and l2 when allowed to
(a) X = He+, n = 4 (b) X = Li++, n = 6 fall on hydrogen atoms in their ground state is found to
+
(c) X = He , n = 6 (d) X = Li++, n = 9 liberate electrons with kinetic energy K.E 1 and K.E 2
95. When a gas of hydrogen-like ions is prepared in a particular respectively. Find the value of Planck’s constant.
excited state X. It emits photons having wavelength equal
to the wavelength of the first line of the Lyman series (KE 2 - KE1 )(l1 + l 2 )
(a) h=
together with photons of five other wavelengths Identify C(l1 – l 2 )
the gas and find the principal quantum number of the state
X respectively. (KE1 - KE 2 )(l 2 – l1 )
(b) h=
(a) He+, 4 (b) Li++, 3 C l1l 2
+
(c) H , 2 (d) None of these
96. A Spectroscopic instrument can resolve two nearby (KE1 - KE 2 )l1l 2
(c) h=
wavelength l and l + Dl if l/Dl is smaller than 8000. This C(l 2 - l1 )
is used to study the spectral lines of the Balmer series of (d) None of these
hydrogen. Approximately how many lines will be resolved 100. The ionization energy of a hydrogen like Bohr atom is 4
by the instrument ? Rydbergs. If l1 is the wavelength of the radiation emitted
(a) 60 (b) 43 (c) 38 (d) 21 when the electron jumps from the first excited state to the
97. A hydrogen atom in state n = 6 makes two successive ground state,
transitions and reaches the ground state. In the first (a) l1 = 300 A° (b) l1 = 2.5 × 10–11 m
transition a photon of 1.13 eV is emitted. Find the energy of (c) l1 = 100 A° (d) l1= 1.5 × 10–11 m
the photon emitted in the second transition and the value

Exercise 2 : Exemplar & Past Year MCQs


(b) Bohr – quantisation applies only two electron
NCERT Exemplar MCQs (c) the frame in which the electron is at rest is not inertial
1. Taking the Bohr radius as a0 = 53 pm, the radius of Li++ ion (d) the motion of the proton would not be in circular orbits,
in its ground state, on the basis of Bohr's model, will be even approximately.
about As the mass of an electron is negligible as compaterd to
(a) 53 pm (b) 27 pm Proton. So the centripetal force cannot Provide the
(c) 18 pm (d) 13 pm electronstatic force Fi = mv2/r
2. The binding energy of a H – atom, considering an electron 3. The simple Bohr model cannot be directly applied to
moving around a fixed nuclei (proton), is calculate the energy levels of an atom with many electrons.
This is because
me 4
B=- ( m = electron mass ) (a) of the electrons not being subject to a central force
8n 2 e 02 h 2 (b) of the electrons colliding with each other
If one decides to work in a frame of reference where the (c) of screening effects
electron is at rest, the proton would be moving around it. (d) the force between the nucleus and an electron will no
By similar arguments, the binding energy would be longer be given by Coulomb's law
4. For the ground state, the electron in the H – atom has an
me 4
B=– ( m = proton mass ) angular momentum = h, according to the simple Bohr model.
8n 2 e02 h 2 Angular momentum is a vector and hence there will be
This last expression is not correct, because infinitely many orbits with the vector pointing in all possible
(a) n would not be integral directions. In actuality, this is not true,
EBD_7418
394 PHYSICS

(a) because Bohr model gives in correct values of angular 11. As an electron makes a transition from an excited state to
momentum the ground state of a hydrogen - like atom/ion :
(b) because only one of these would have a minimum [JEE Main 2015, C]
energy (a) kinetic energy decreases, potential energy increases
(c) angular momentum must be in the direction of spin of but total energy remains same
electron (b) kinetic energy and total energy decrease but potential
(d) because electrons go around only in horizontal orbits energy increases
5. O2 molecule consists of two oxygen atoms. In the molecule, (c) its kinetic energy increases but potential energy and
nuclear force between the nuclei of the two atoms total energy decrease
(a) is not important because nuclear forces are short – (d) kinetic energy, potential energy and total energy
ranged decrease
(b) is as important as electrostatic force for binding the 12. One of the lines in the emission spectrum of Li2+ has the
same wavelength as that of the 2nd line of Balmer series in
two atoms
hydrogen spectrum. The electronic transition
(c) cancels the repulsive electrostatic force between the
corresponding to this line is n = 12 ® n = x. Find the value
nuclei
of x. [BITSAT 2015, A]
(d) is not important because oxygen nucleus have equal (a) 8 (b) 6 (c) 7 (d) 5
number of neutrons and protons 13. When an a-particle of mass 'm' moving with velocity 'v'
6. Two H atoms in the ground state collide inelastically. The bombards on a heavy nucleus of charge 'Ze', its distance of
maximum amount by which their combined kinetic energy closest approach from the nucleus depends on m as :
is reduced, is [NEET 2016, C]
(a) 10.20 eV (b) 20.40 eV 1 1 1
(c) 13.6 eV (d) 27.2 eV (a) (b) (c) (d) m
m m m2
Past Year MCQs 14. Given the value of Rydberg constant is 10 m–1, the wave
7
number of the last line of the Balmer series in hydrogen
7. Hydrogen atom in ground state is excited by a spectrum will be : [NEET 2016, A]
monochromatic radiation of l = 975 Å. Number of spectral (a) 0.025 × 104 m–1 (b) 0.5 × 107 m–1
lines in the resulting spectrum emitted will be (c) 0.25 × 107 m–1 (d) 2.5 × 107 m–1
[AIPMT 2014, A] 15. The wavelength of the first line of Lyman series for
(a) 3 (b) 2 (c) 6 (d) 10 hydrogen atom is equal to that of the second line of Balmer
8. Hydrogen (1H1), Deuterium (1H2), singly ionised Helium series for a hydrogen like ion. The atomic number Z of
hydrogen like ion is [AIIMS 2016, A]
(2He4)+ and doubly ionised lithium (3Li6)++ all have one
electron around the nucleus. Consider an electron transition (a) 3 (b) 4
from n = 2 to n = 1. If the wavelengths of emitted radiation (c) 1 (d) 2
are l1, l2, l3 and l4 respectively then approximately which 16. Assertion : In Lyman series, the ratio of minimum and
one of the following is correct? [JEE Main 2014, A] 3
maximum wavelength is .
(a) 4l1 = 2l 2 = 2l3 = l 4 (b) l1 = 2l 2 = 2l3 = l 4 4
Reason : Lyman series constitute spectral lines
(c) l1 = l 2 = 4l3 = 9l 4 (d) l1 = 2l 2 = 3l3 = 4l 4 corresponding to transition from higher energy to ground
9. In the spectrum of hydrogen, the ratio of the longest state of hydrogen atom. [AIIMS 2016, C]
wavelength in the Lyman series to the longest wavelength (a) If both Assertion and Reason are correct and Reason
in the Balmer series is [AIPMT 2015, A] is the correct explanation of Assertion.
(a) 9/4 (b) 27/5 (c) 5/27 (d) 4/9 (b) If both Assertion and Reason are correct, but Reason
10. Assertion : Between any two given energy levels, the is not the correct explanation of Assertion.
number of absorption transitions is always less than the
(c) If Assertion is correct but Reason is incorrect.
number of emission transitions.
(d) If both the Assertion and Reason are incorrect.
Reason : Absorption transitions start from the lowest energy
level only and may end at any higher energy level. But 17. If the series limit wavelength of Lyman series for the
emission transitions may start from any higher energy level hydrogen atom is 912 Å, then the series limit wavelength
and end at any energy level below it. [AIIMS 2015, C] for Balmer series of hydrogen atoms is [BITSAT 2016, A]
(a) 912 Å (b) 912 × 2 Å
(a) If both Assertion and Reason are correct and Reason 912
is the correct explanation of Assertion. (c) 912 × 4 Å (d) Å
2
(b) If both Assertion and Reason are correct, but Reason 18. The ratio of wavelengths of the last line of Balmer series
is not the correct explanation of Assertion. and the last line of Lyman series is :- [NEET 2017, A]
(c) If Assertion is correct but Reason is incorrect. (a) 1 (b) 4 (c) 0.5 (d) 2
(d) If both the Assertion and Reason are incorrect.
ATOMS 395

19. Which one did Rutherford consider to be supported by the atom, the kinetic energy of ejected electron from H atom
results of experiments in which a-particles were scattered Z2
by gold foil? [AIIMS 2017, C] would be (eV) – [Given E n = - 2 (13.6 eV) ]
n
(a) The nucleus of an atom is held together by forces which
are much stronger than electrical or gravitational [AIIMS 2018, A]
forces. (a) 4.155 eV (b) 8.310 eV
(b) The force of repulsion between an atomic nucleus (c) 2.515 eV (d) 5.550 eV
and an a-particle varies with distance according to
25. Assertion : The force of repulsion between atomic nucleus
inverse square law.
and a-particle varies with distance according to inverse
(c) a-particles are nuclei of Helium atoms.
square law. [AIIMS 2018, C]
(d) Atoms can exist with a series of discrete energy levels
Reason : Rutherford did a-particle scattering experiment.
20. As an electron makes a transition from an excited state to
(a) If both Assertion and Reason are correct and Reason
the ground state of a hydrogen - like atom/ion
is the correct explanation of Assertion.
[AIIMS 2017, C]
(b) If both Assertion and Reason are correct, but Reason
(a) kinetic energy decreases, potential energy increases
is not the correct explanation of Assertion.
but total energy remains same
(b) kinetic energy and total energy decrease but potential (c) If Assertion is correct but Reason is incorrect.
energy increases (d) If both the Assertion and Reason are incorrect.
(c) its kinetic energy increases but potential energy and 26. An electron from various excited states of hydrogen atom
total energy decrease emit radiation to come to the ground state. Let ln, lg be the
(d) kinetic energy, potential energy and total energy de Broglie wavelength of the electron in the n th state and
decrease the ground state respectively. Let Ln be the wavelength of
21. Assertion : Bohr had to postulate that the electrons in the emitted photon in the transition from the n th state to
stationary orbits around the nucleus do not radiate. the ground state. For large n, (A, B are constants)
Reason : According to classical physics all moving [JEE Main 2018, S]
electrons radiate. [AIIMS 2017, C] B
(a) L n » A+ (b) Ln » A + B ln
(a) If both Assertion and Reason are correct and Reason l 2n
is the correct explanation of Assertion.
(b) If both Assertion and Reason are correct, but Reason (c) L 2n » A + B l 2n (d) L 2n » l
is not the correct explanation of Assertion. 27. If the series limit frequency of the Lyman series is v1, then
(c) If Assertion is correct but Reason is incorrect. the series limit frequency of the P-fund series is :
(d) If both the Assertion and Reason are incorrect. [JEE Main 2018, A]
22. Radius of Li++ ion in its first excited state is: (a) 25 nL (b) 16 nL (c) nL/16 (d) nL/25
(assuming Bohr's model to be valid) [BITSAT 2017, A] 28. Consider a 3-level system with energies E1, E2 and E3 in
(a) 72 pm (b) 125 pm ascending order. l1, l2 and l3 are the wavelengths of
(c) 120.50 pm (d) 1125 pm radiation corresponding to the transitions E2 ® E 1 ,
23. The ratio of kinetic energy to the total energy of an electron E3 ® E2 and E3 ® E1 respectively. The wavelengths are
in a Bohr orbit of the hydrogen atom, is [NEET 2018, C] related by [BITSAT 2018, C]
(a) 1 : 1 (b) 1 : –1 (c) 1 : –2 (d) 2 : –1 (a) l1 = l 2 l3 /(l3 - l 2 ) (b) l 2 = l3 - l1
24. If radiation corresponding to first line of “Balmer series” of
He+ ion knocked out electron from 1st excited state of H (c) l 2 = l1l3 /(l1 + l3 ) (d) l3 = l1l2 /(l1 + l 2 )
EBD_7418
396 PHYSICS

Exercise 3 : Try If You Can


1. In Hydrogen spectrum, the wavelength of Ha line is 656 the radius of the nth orbit in terms of the Bohr radius
nm, whereas in the spectrum of a distant galaxy, Ha line
wavelength is 706 nm. Estimated speed of the galaxy with æ e h2 ö
ç a0 = 0 ÷ is :
respect to earth is ç mp e 2 ÷ø
è
(a) 2 × 108 m/s (b) 2 × 107m/s
6 (a) rn = a0n – b (b) rn = a0n2 + b
(c) 2 × 10 m/s (d) 2 × 105 m/s
(c) rn = a0n – b
2 (d) rn = a0n + b
2. Electrons in a certain energy level n = n1, can emit 3 spectral
lines. When they are in another energy level, n = n 2. They 10. The ionization energy of the electron in the hydrogen atom
can emit 6 spectral lines. The orbital speed of the electrons in its ground state is 13.6 eV. The atoms are excited to
in the two orbits are in the ratio of higher energy levels to emit radiations of 6 wavelengths.
(a) 4 : 3 (b) 3 : 4 (c) 2 : 1 (d) 1 : 2 Maximum wavelength of emitted radiation corresponds to
3. The electron in hydrogen atom jumps from the ground the transition between
state to the higher energy state where its velocity is (a) n = 3 to n = 1 states (b) n = 2 to n = 1 states
reduced to one - third of its initial value. If the radius of the (c) n = 4 to n = 3 states (d) n = 3 to n = 2 states
orbit in the ground state is r, the radius of new orbit will be 11. If potential energy between a proton and an electron is
(a) 3r (b) 9r (c) r/3 (d) r/9 given by |U| = ke2/2R3, where e is the charge of electron
4. In a hypothetical system, a particle of mass m and charge and R is the radius of atom, then radius of Bohr's orbit is
–3q is moving around a very heavy particle charge q. given by (h = Planck's constant, k = constant)
Assume that Bohr's model is applicable to this system,
ke2 m 6p 2 ke2 m
then velocity of mass m in the first orbit is (a) (b)
3q 2 3q 2 3q 3q h2 n2 h2
(a) (b) (c) 2pe h (d) 4pe h 2p ke m2
4p 2 ke2 m
2e 0 h 4e 0 h 0 0
(c) (d)
5. An electron, in a hydrogen-like atom, is in an excited state. n h2 n2h 2
It has a total energy of –3.4 eV. The kinetic energy and the 12. Suppose an electron is attracted towards the origin by a
de-Broglie wavelength of the electron are respectively k
force where ‘k’ is a constant and ‘r’ is the distance of the
(a) +3.4 eV, 0.66 × 10–9 m (b) –3.4 eV, 1.99 × 10–9 m r
(c) 2.8 eV, 2.38 × 10 m–10 (d) 1.1 eV, 1.28 × 10–9 m electron from the origin. By applying Bohr model to this
6. The wavelength of the first spectral line in the Balmer series system, the radius of the nth orbital of the electron is found
of hydrogen atom is 6561 Å. If the wavelength of the spectral to be ‘rn’ and the kinetic energy of the electron to be ‘Tn’.
line in the Balmer series of singly-ionized helium atom is Then which of the following is true?
1215Å when electron jumps from n2 to n1, then n2 and n1 are 1 2
(a) 4, 2 (b) 5, 3 (c) 6, 3 (d) 6, 2 (a) Tn µ , rn µ n (b) Tn independent of n, rn µ n
n2
7. Orbits of a particle moving in a circle are such that the
1 1 2
perimeter of the orbit equals an integer number of de-Broglie (c) Tn µ , rn µ n (d) Tn µ , rn µ n
wavelengths of the particle. For a charged particle moving n n
in a plane perpendicular to a magnetic field, the radius of 13. The wavelength Ka of X-rays for two metals ‘A’ and ‘B’ are
4 1
the nth orbital will therefore be proportional to : and respectively, where ‘R’ is Rydbergg
(a) n 2 (b) n (c) n 1/2 (d) n 1/4 1875R 675R
constant. Find the number of elements lying between A
8. In the Bohr model an electron moves in a circular orbit
and B according to their atomic numbers
around the proton. Considering the orbiting electron to be
(a) 3 (b) 1 (c) 4 (d) 5
a circular current loop, the magnetic moment of the hydrogen
14. An electron in the hydrogen atom jumps from excited state
atom, when the electron is in nth excited state, is : n to the ground state. The wavelength so emitted illuminates
æ e n2 h ö æ e ö nh a photosensitive material having work function 2.75 eV. If
(a) ç ÷ (b) ç ÷ the stopping potential of the photoelectron is 10 V, the
ç 2m 2p ÷ è m ø 2p
è ø value of n is
2 (a) 3 (b) 4 (c) 5 (d) 2
æ e ö nh æ e ön h
(c) ç ÷ (d) ç ÷ 15. An electron in hydrogen atom makes a transition n1 ® n2
è 2m ø 2p è m ø 2p
9. In the Bohr’s model of hydrogen-like atom the force between where n1 and n2 are principal quantum numbers of the two
states. Assuming Bohr’s model to be valid the time period
the nucleus and the electron is modified as
of the electron in the initial state is eight times that in the
e2 æ 1 b ö final state. The possible values of n1 and n2 are
F= ç + ÷ , where b is a constant. For this atom, (a) n1 = 4 and n2 = 2 (b) n1 = 6 and n2 = 2
4pe0 è r 2 r 3 ø
(c) n1 = 8 and n2 = 1 (d) n1 = 8 and n2 = 2
ATOMS 397

ANSWER KEYS
Exercise 1 : Topic-wise MCQs
1 (a) 11 (a) 21 (c) 31 (b) 41 (d) 51 (d) 61 (a) 71 (a) 81 (c) 91 (a)
2 (b) 12 (a) 22 (a) 32 (b) 42 (d) 52 (b) 62 (d) 72 (c) 82 (b) 92 (c)
3 (b) 13 (d) 23 (b) 33 (b) 43 (c) 53 (b) 63 (b) 73 (d) 83 (a) 93 (a)
4 (b) 14 (c) 24 (a) 34 (b) 44 (d) 54 (a) 64 (a) 74 (d) 84 (b) 94 (d)
5 (a) 15 (d) 25 (b) 35 (a) 45 (d) 55 (a) 65 (d) 75 (c) 85 (c) 95 (a)
6 (c) 16 (d) 26 (a) 36 (a) 46 (b) 56 (b) 66 (c) 76 (a) 86 (c) 96 (c)
7 (d) 17 (b) 27 (d) 37 (c) 47 (a) 57 (c) 67 (a) 77 (d) 87 (a) 97 (a)
8 (d) 18 (b) 28 (b) 38 (a) 48 (a) 58 (b) 68 (c) 78 (d) 88 (b) 98 (a)
9 (a) 19 (b) 29 (a) 39 (d) 49 (b) 59 (c) 69 (c) 79 (c) 89 (b) 99 (c)
10 (a) 20 (d) 30 (b) 40 (d) 50 (d) 60 (a) 70 (a) 80 (b) 90 (a) 100 (a)
Exercise 2 : Exemplar & Past Year MCQs
1 (c) 4 (a) 7 (c) 10 (a) 13 (a) 16 (b) 19 (b) 22 (a) 25 (b) 28 (d)
2 (c) 5 (a) 8 (c) 11 (c) 14 (c) 17 (c) 20 (c) 23 (b) 26 (a)
3 (a) 6 (a) 9 (c) 12 (b) 15 (d) 18 (b) 21 (b) 24 (a) 27 (d)
Exercise 3 : Try If You Can
1 (b) 3 (b) 5 (a) 7 (c) 9 (c) 11 (b) 13 (c) 15 (a)
2 (a) 4 (a) 6 (a) 8 (c) 10 (c) 12 (b) 14 (b)
EBD_7418
398 PHYSICS
28Chapter
NUCLEI

Trend
Analysis of NEET and AIIMS (Year 2010-2018)
5

4
Number of Questions

3
AIPMT/NEET

2 AIIMS

0
2010 2011 2012 2013 2014 2015 2016 2017 2018
Year
Trend
Analysis of JEE Main and BITSAT (Year 2010-2018)
5

4
Number of Questions

3 JEE Main
BITSAT
2

0
2010 2011 2012 2013 2014 2015 2016 2017 2018

Year

Chapter Utility Score (CUS)


Exam Weightage Important Concepts Difficulty Level CUS (Out of 10)
NEET 3 Nuclear Binding Energy,
AIIMS 4 Radioactivity, Nuclear 3.5/5 7/10
JEE Main 2 Fission & Fusion
BITSAT 4
EBD_7418
400 PHYSICS
NUCLEI 401
EBD_7418
402 PHYSICS

Exercise 1 : Topic-wise MCQs


(a) If both Assertion and Reason are correct and Reason
Topic 1: Composition and Size of Nucleus
is the correct explanation of Assertion.
1. The constituents of nucleus are (b) If both Assertion and Reason are correct, but Reason
(a) electrons and protons is not the correct explanation of Assertion.
(b) protons and neutrons (c) If Assertion is correct but Reason is incorrect.
(c) neutrons and electrons (d) If both the Assertion and Reason are incorrect.
(d) electrons, protons and neutrons 13. The electrons cannot exist inside the nucleus because
2. The radius of a nucleus is (a) de-Broglie wavelength associated with electron in
(a) directly proportional to its mass number b -decay is much less than the size of nucleus
(b) inversely proportional to its atomic weight (b) de-Broglie wavelength associated with electron in
(c) directly proportional to the cube root of its mass b -decay is much greater than the size of nucleus
number (c) de-Broglie wavelength associated with electron in
(d) None of these
b -decay is equal to the size of nucleus
3. A nuclei having same number of neutron but different
(d) negative charge cannot exist in the nucleus
number of protons / atomic number are called
14. If Avogadro number is 6 × 1023, then number of protons,
(a) isobars (b) isomers neutrons and electrons in 14 gms of 6C14 are respecitvely
(c) isotones (d) isotopes (a) 36 × 1023, 48 × 1023, 36 × 1023
4. The radius of nucleus is of the order of (b) 36 × 1023, 36 × 1023, 36 × 1023
(a) 10–10 m (b) 10–6 m (c) 10–15 m (d) 10–13 m (c) 48 × 1023, 36 × 1023, 48 × 1023
5. The ratio of volumes of nuclei (assumed to be in spherical (d) 48 × 1023, 48 × 1023, 36 × 1023
shape) with respective mass numbers 8 and 64 is 15. Which of the following statements are correct ?
(a) 0.5 (b) 2 (c) 0.125 (d) 0.25
I. Nuclear density is a constant for all matter.
6. The nuclei of which one of the following pairs of nuclei
are isotones? II. Nuclear density is around 2.3 × 1017 kg/m3.
(a) 34Se74 , 31Ga 71 (b) 38Sr 84 , 38Sr 86 II. Nuclear density is very large compared to ordinary
(c) 42Mo92 , 40Zr 92 (d) 20Ca 40 , 16S32 matter.
7. If the radius of a nucleus 256X is 8 fermi, then the radius of IV. Mass of ordinary matter is mainly due to nucleus.
4He nucleus will be (a) I, II and III (b) II and III
(a) 16 fermi (b) 2 fermi (c) 32 fermi (d) 4 fermi (c) I and II (d) I, II, III and IV
8. The set which represents the isotope, isobar and isotone 16. A nucleus splits into two nuclear parts which have their
respectively is velocity ratio equal to 2 : 1. What will be the ratio of their
(a) (1H2, 1H3 ), (79Au197, 80Hg198 ) and (2He3, 1H2 ) nuclear radius?
(b) (2He3, 1H1 ), (79Au197, 80Hg198 ) and (1H1, 1H3 ) (a) 21/3 : 1 (b) 1 : 21/3
1/2
(c) 3 : 1 (d) 1 : 31/2
(c) (2He3, 1H3 ), (1H2, 1H3 ) and (79Au197, 80Hg198 )
(d) (1H2, 1H3 ), (2He3, 1H3 ) and (79Au197, 80Hg198 ) 17. If radius of the 27 nucleus is estimated to be 3.6 fermi
13 Al
9. Nucleus of an atom whose atomic mass is 24 consists of
then the radius of 125 nucleus be nearly
(a) 11 electrons, 11 protons and 13 neutrons 52 Te
(b) 11 electrons, 13 protons and 11 neutrons (a) 8 fermi (b) 6 fermi (c) 5 fermi (d) 4 fermi
(c) 11 protons and 13 neutrons 18. Atomic weight of boron is 10.81 and it has two isotopes
(d) 11 protons and 13 electrons 10 11 10 11
5B and 5B . Then ratio of 5B : 5B in nature would be
10. The mass number of He is 4 and that for sulphur is 32. The (a) 19 : 81 (b) 10 : 11
radius of sulphur nuclei is larger than that of helium by (c) 15 : 16 (d) 81 : 19
(a) 8 (b) 4 (c) 2 (d) 8 19. The radius of germanium (Ge) nuclide is measured to be
11. A nuclear transformation is denoted by X (n, a ) 73 Li . Which twice the radius of 94 Be . The number of nucleons in Ge are
(a) 74 (b) 75 (c) 72 (d) 73
of the following is the nucleus of element X ?
20. The nuclear radius of 8O16 is 3 × 10–15 m. If an atomic mass
(a) 105 B (b) 12 C 6 (c) 114 Be (d) 95 B unit is 1.67 × 10–27 kg, then the nuclear density is
12. Assertion : Neutrons penetrate matter more readily as approximately
compared to protons.
(a) 2.35 × 1017 g cm–3 (b) 2.35 × 1017 kg m–3
Reason : Neutrons are slightly more massive than protons.
(c) 2.35 × 1017 gm–3 (d) 2.35 × 1017 kg mm–3
NUCLEI 403

and N neutrons. The mass M(N, Z) of the nucleus is given


Topic 2: Mass-Energy Equivalence and Nuclear Reactions
by (c is the velocity of light)
21. When the number of nucleons in nuclei increases, the (a) M(N, Z) = NMn + ZMp + B/c2
binding energy per nucleon (b) M(N, Z) = NMn + ZMp – Bc2
(a) increases continuously with mass number (c) M(N, Z) = NMn + ZMp + Bc2
(b) decreases continuously with mass number (d) M(N, Z) = NMn + ZMp – B/c2
(c) remains constant with mass number 32. Calculate the binding energy of a deuteron atom, which
(d) first increases and then decreases with increase of consist of a proton and a neutron, given that the atomic
mass number mass of the deutron is 2.014102 u.
22. The mass of an atomic nucleus is less than the sum of (a) 0.002388 MeV (b) 2.014102 MeV
the masses of its constituents. This mass defect is (c) 2.16490 MeV (d) 2.224 MeV
converted into 33. From the following equations, pick out the possible nuclear
reactions.
(a) heat energy
(a) 6C13 + 1H1 ® 6C14 + 4.3 MeV
(b) light energy
(b) 6C12 + 1H1 ® 9N14 + 2 MeV
(c) electrical energy
(c) 7N14 + 1H1 ® 8O15 + 7.3 MeV
(d) energy which binds nucleons together
(d) 92U235 + 0n1 ® 54X140 + 38Si94 + 20n1 + g + 200 MeV
23. Nuclear forces are
34. On an average, the number of neutrons and the energy of a
(a) spin dependent and have no non-central part neutron released per fission of a uranium atom are
(b) spin dependent and have a non-central part respectively
(c) spin independent and have no non-central part (a) 2.5 and 2 keV (b) 3 and l keV
(d) spin independent and have a non-central part (c) 2.5 and 2 MeV (d) 2 and 2 keV
24. Fusion reaction occurs at temperatures of the order of 35. Complete the equation for the following fission process :
(a) 103 K (b) 107 K (c) 10 K (d) 104 K 235
25. Control rods used in nuclear reactors are made of 92 U + 0n1 ®38 Sr 90 + ...
(a) 143
+ 3 0n1 (b) 54X145 + 3 0n1
(a) stainless steel (b) graphite 54X
(c) 142 1 (d) 54X142 + 0n1
(c) cadmium (d) plutonium 57X + 3 0n
26. If 1 mg of U235 is completely annihilated, the energy liberated 36. A moderator is used in nuclear reactors in order to
(a) slow down the speed of the nuetrons
is
(b) acceleerate the neutrons
(a) 9 × 1010 J (b) 9 × 1019 J
18
(c) increase the number of neutrons
(c) 9 × 10 J (d) 9 × 1017 J (d) decrease the number of neutrons
27. Two nucleons are at a separation of 1 fermi. The net force 37. Fusion reactions take place at high temperature because
between them is F1 if both are neutrons, F2 if both are protons (a) atoms are ionised at high temperature
and F3 if one is proton and the other is a neutron. Then (b) molecules break up at high temperature
(a) F1 > F2 > F3 (b) F1 = F3 > F2 (c) nuclei break up at high temperature
(c) F2 > F1 > F3 (d) F1 = F2 > F3 (d) kinetic enrgy is high enough to overcome repulsion
28. Which of the following statements is true? between nuclei
(a) 78Pt192 has 78 neutrons 38. For a nuclear fusion process, suitable nuclei are
(b) 84Po214 ® 82Pb210 + b– (a) any nuclei
(c) 92U238 ® 90Th234 + 2He4 (b) heavy nuclei
(d) 90Th234 ® 91Pa234 + 2He4 (c) lighter nuclei
29. Which of the following statement is not true regarding (d) nuclei lying in the middle of periodic table
Einsteins mass energy relation? 39. Binding energy per nucleon plot against the mass number
(a) Mass disappears to reappear as energy. for stable nuclei is shown in the figure. Which curve is
(b) Energy disappears to reappear as mass. correct?
(c) Mass and energy are two different forms of the same (a) A
entity. (b) B
(d) Mass and energy can never be related to each other. (c) C
30. The curve of binding energy per nucleon as a function of (d) D
atomic mass number has a sharp peak for helium nucleus. 40. Assertion : It is not possible to use 35Cl as the fuel for
This implies that helium fusion energy.
(a) can easily be broken up Reason : The binding energy of 35Cl is to small.
(b) is very stable (a) If both Assertion and Reason are correct and Reason
(c) can be used as fissionable meterial is the correct explanation of Assertion.
(d) is radioactive (b) If both Assertion and Reason are correct, but Reason
31. Mp denotes the mass of a proton and Mn that of a neutron. is not the correct explanation of Assertion.
A given nucleus, of binding energy B, contains Z protons (c) If Assertion is correct but Reason is incorrect.
(d) If both the Assertion and Reason are incorrect.
EBD_7418
404 PHYSICS

41. If MO is the mass of an oxygen isotope 8 O17 ,MP and MN 49. Assertion : The binding energy per nucleon, for nuclei
are the masses of a proton and a neutron respectively, the with atomic mass number A > 100, decrease with A.
nuclear binding energy of the isotope is Reason : The forces are weak for heavier nuclei.
(a) (MO –17MN)c2 (b) (MO – 8MP)c2 (a) If both Assertion and Reason are correct and Reason
(c) (MO– 8MP –9MN)c (d) MOc2
2 is the correct explanation of Assertion.
(b) If both Assertion and Reason are correct, but Reason
42. If the total binding energies of 2 4 56 235
1 H, 2 He, 26 Fe & 92 U is not the correct explanation of Assertion.
nuclei are 2.22, 28.3, 492 and 1786 MeV respectively, identify (c) If Assertion is correct but Reason is incorrect.
the most stable nucleus of the following. (d) If both the Assertion and Reason are incorrect.
(a) 56 (b) 12 H 235
(c) 92 (d) 42 He 50. The binding energies per nucleon for a deuteron and an
26 Fe U
43. The masses of neutron and proton are 1.0087 a.m.u. and a-particle are x1 and x2 respectively. What will be the energy
1.0073 a.m.u. respectively. If the neutrons and protons Q released in the reaction 1H2 + 1H2 ® 2He4 + Q
combine to form a helium nucleus (alpha particles) of mass (a) 4(x1 + x2) (b) 4(x2 – x1)
4.0015 a.m.u the binding energy of the helium nucleus will (c) 2(x1 + x2) (d) 2(x2 – x1)
be (1 a.m.u. = 931 MeV) 51. In a fission reaction
(a) 28.4 MeV (b) 20.8 MeV 236
92 U ®117 X +117 Y + n + n
(c) 27.3 MeV (d) 14.2 MeV
44. Column – I Column – II the binding energy per nucleon of X & Y is 8.5 MeV.
(A) Hydrogen bomb (1) Fission Whereas of 236U is 7.6 MeV. The total energy liberated will
(B) Atom bomb (2) Fusion be about
(C) Binding energy (3) Critical mass (a) 2000 MeV (b) 200 MeV
(D) Nuclear reactor (4) Mass defect (c) 2 MeV (d) 200 KeV
(a) (A) ® (3); (B) ® (2); (C) ® (1); (D) ® (4) 52. If a star can convert all the He nuclei completely into oxygen
(b) (A) ® (2); (B) ® (1); (C) ® (4); (D) ® (3) nuclei. The energy released per oxygen nuclei is [Mass of
(c) (A) ® (3); (B) ® (1); (C) ® (2); (D) ® (4) He nucleus is 4.0026 amu and mass of Oxygen nucleus is
15.9994 amu]
(d) (A) ® (4); (B) ® (2); (C) ® (3); (D) ® (1)
(a) 7.6 MeV (b) 56.12 MeV
45. The mass defect in a particular nuclear reaction is 0.3 grams.
The amount of energy liberated in kilowatt hour is (c) 10.24 MeV (d) 23.9 MeV
(Velocity of light = 3 × 108 m/s) 53. If 200 MeV energy is released in the fission of a single U235
(a) 1.5 × 106 (b) 2.5 × 106 nucleus, the number of fissions required per second to
6 produce 1 kilowatt power shall be (Given 1eV = 1.6 × 10–19 J)
(c) 3 × 10 (d) 7.5 × 106
46. For binding energy per nucleon versus mass number (a) 3.125 × 1013 (b) 3.125 × 1014
(c) 3.125 × 10 15 (d) 3.125 × 1016
curve, which of the following are correct ?
I. Binding energy per nucleon Ebn is independent of 54. A certain mass of Hydrogen is changed to Helium by the
mass number range 30 < A < 170. process of fusion. The mass defect in fusion reaction is
II. Binding energy is lower for both light nucei (A < 30) 0.02866 a.m.u. The energy liberated per a.m.u. is
and heavy nuclei (A < 170). (Given : 1 a.m.u = 931 MeV)
III. Binding energy is maximum of about 8.75 MeV for (a) 26.7 MeV (b) 6.675 MeV
A = 56. (c) 13.35 MeV (d) 2.67 MeV
IV. In region 0 < A < 80, binding energy increases with
mass number.
55. The binding energy per nucleon of deuteron ( H) and
2
1
(a) I, II, III and IV
(c) II, III and IV
(b) I, II and IV
(d) I, II and III
helium nucleus ( He ) is 1.1 MeV and 7 MeV respectively..
4
2
If two deuteron nuclei react to form a single helium nucleus,
47. The mass of a 37 Li nucleus is 0.042 u less than the sum of then the energy released is
the masses of all its nucleons. The binding energy per (a) 23.6 MeV (b) 26.9 MeV
nucleon of 37 Li nucleus is nearly (c) 13.9 MeV (d) 19.2 MeV

(a) 46 MeV (b) 5.6 MeV 56. If the binding energy per nucleon in 73 Li and 42 He nuclei
(c) 3.9 MeV (d) 23 MeV are 5.60 MeV and 7.06 MeV respectively, then in the reaction
48. Energy released in the fission of a single 235 nucleus is
92 U
p + 73 Li ¾¾
® 2 42 He
200 MeV. The fission rate of a 235
92 U filled reactor operating energy of proton must be
at a power level of 5 W is
(a) 28.24 MeV (b) 17.28 MeV
(a) 1.56 × 10–10 s–1 (b) 1.56 × 1011 s–1
(c) 1.46 MeV (d) 39.2 MeV
(c) 1.56 × 10–16 s–1 (d) 1.56 × 10–17 s–1
NUCLEI 405

57. Assume that a neutron breaks into a proton and an electron. 67. A nuclear reaction is given by
The energy released during this process is : (mass of neutron A
ZX ® Z+1Y A + -1 e 0 + n , represents
= 1.6725 × 10–27 kg, mass of proton = 1.6725 × 10–27 kg,
mass of electron = 9 × 10–31 kg). (a) fission (b) b-decay
(a) 0.511 MeV (b) 7.10 MeV (c) s -decay (d) fusion
68. Neutron decay in free space is given as follows
(c) 6.30 MeV (d) 5.4 MeV
1
58. A heavy nucleus having mass number 200 gets 0n ®1 H1 + -1 e 0 + [ ]
disintegrated into two small fragments of mass number 80 Then the parenthesis [ ] represents a
and 120. If binding energy per nucleon for parent atom is (a) neutrino (b) photon
6.5 MeV and for daughter nuclei is 7 MeV and 8 MeV
(c) antineutrino (d) graviton
respectively, then the energy released in the decay is 69. A radioactive substance has a half life of four months. Three
X × 110 MeV. Find the value of X fourth of the substance will decay in
(a) 3 (b) 4 (c) 2 (d) 1 (a) three months (b) four months
(c) eight months (d) twelve months
Topic 3: Radioactivity 70. They decay constant of radium is 4.28 × 10–4 per year. Its
59. Radioactivity is half life will be
(a) irreversible process (a) 2000 years (b) 1240 years
(c) 63 years (d) 1620 years
(b) self disintegration process
71. A radioactive element forms its own isotope after 3
(c) spontaneous consecutive disintegrations. The particles emitted are
(d) all of the above (a) 3 b–particles
60. Beta rays emitted by a radioactive material are (b) 2 b–particles and 1 a–particle
(a) electromagnetic radiations (c) 3 b–particles and 1 a–particle
(b) the electrons orbiting around the nucleus (d) 2 a–particles and 1 b–particle.
(c) charged particles emitted by nucleus 72. N atoms of a radioactive substance emit na-particles per
(d) neutral particles second. The half life of the radioactive substance is
61. Which of the following is not a mode of a radioactive decay ? n N
(a) sec (b) sec
(a) Positron emission (b) Electron capture N n
(c) Fusion (d) Alpha decay 0.693N 0.693n
(c) sec (d) sec
62. The half-life period and the mean life period of a radioactive n N
element are denoted respectively by Th and Tm. Then 73. In one half-life time duration
I. activity of a sample reduced to half of its initial value.
(a) Th = Tm (b) Th > T m
II. total number of nuclei present are reduced to half of
(c) Th < Tm (d) Th ³ Tm its initial value.
63. The same radioactive nucleus may emit III. number of radio active nuclei present is reduced to
(a) all the three a, b and g one after another half of its initial value.
(b) all the three a, b and g radiations simultaneously IV. mass of sample is reduced to half of its initial value.
(c) only a and b simultaneously Out of these, correct statements are
(d) only one a, b and g at a time (a) I and II (b) I and II
64. A radioactive material undergoes decay by ejecting (c) II and IV (d) II and III
electrons. The electron ejected in this process is 74. Match Column I of the nuclear processes with Column II
(a) the electron from the decay of a neutron
containing parent nucleus and one of the end products of
(b) the electron present in the nucleus
(c) the resulting from the conversion of g photon each process.
(d) an orbital electron Column I Column II
65. The rate of disintegration at a given instant, is directly 15 15
proportional to the number of atoms present at that
(A) Alpha decay (1) 8 O®7 N + ...
instant. This is the statement of (B) b+ decay 238 234
(2) 92 U ® 90 Th + ...
(a) law of radioactive decay
(b) half life 185 184
(C) Fission (3) 83 Bi ® 82 Pb + ...
(c) law of radioactive transformation
(d) group displacement law 239
® 140
(D) Proton emission (4) 94 Pu 57 La + ...
66. Half life of radioactive element depends upon
(a) amount of element present (a) (A) ® (1); (B) ® (3); (C) ® (2); (D) ® (4)
(b) temperature (b) (A) ® (4); (B) ® (3); (C) ® (2); (D) ® (1)
(c) pressure (c) (A) ® (2); (B) ® (1); (C) ® (4); (D) ® (3)
(d) nature of element (d) (A) ® (3); (B) ® (1); (C) ® (2); (D) ® (4)
EBD_7418
406 PHYSICS

75. Radioactive element decays to form a stable nuclide, then (c) If Assertion is correct but Reason is incorrect.
the rate of decay of reactant is (d) If both the Assertion and Reason are incorrect.
N N 83. When 3 Li7 nuclei are bombarded by protons, and the
resultant nuclei are 4Be8, the emitted particles will be
(a) (b) (a) alpha particles (b) beta particles
(c) gamma photons (d) neutrons
t t
N N 84. A nuclear transformation is denoted by X (n, a ) 73 Li .
Which of the following is the nucleus of element X ?
(c) (d) (a) 10 (b) 12 C 6 (c) 11 (d) 95 B
5 B 4 Be
t t 85. If N0 is the original mass of the substance of half-life period
76. The count rate from a radioactive sample falls from 4.0× 106 t1/2 = 5 years, then the amount of substance left after 15
per second to 1 × 106 per second in 20 hour. What will be years is
(a) N0/8 (b) N0/16 (c) N0/2 (d) N0/4
the count rate, 100 hour after the beginning?
86. If T is the half life of a radioactive material, then the fraction
(a) 1.1 × 102 s–1 (b) 2.1 × 10–3s–1
(c) 3.91 × 103s–1 (d) 2.8 × 10–2s–1 T
that would remain after a time is
77. The ratio of 235U to 238U in natural uranium deposits 2
1 3 1 2 -1
today is 0.0072. What was this ratio 2.0 × 109 year ago ? The (a) (b) (c) (d)
half-lives of the two isotopes are 7.04 × 108 year and 44.7 × 2 4 2 2
108 year, respectively. 7 66
87. Starting with a sample of pure Cu ,
of it decays into
(a) 4.8% (b) 6.9% (c) 1.2% (d) 3.8% 8
78. Calculate the Q-value in the following decays : Zn in 15 minutes. The corresponding half life is
(a) 19O ® 19F + e + n (a) 15 minutes (b) 10 minutes
The atomic masses needed are as follows : 1
(c) 7 minutes (d) 5 minutes
19
O = 19.003576u 19 F = 18.998403u , 2
(a) 4.816 MeV (b) 3.254 MeV 88. The energy spectrum of b-particles [number N(E) as a
(c) 5.382 Mev (d) 8.332 MeV function of b-energy E] emitted from a radioactive source
79. Three a–particles and b–particle decaying takes place in is
series from an isotope 238
88 Ra . Finally the isotope obtained
will be
220 222 226 215 (a) N(E) (b) N(E)
(a) 84 X (b) 86 X (c) 83 X (d) 83 X

80. A radioactive reaction is 92 U 238 ® 82Pb206 . How many a E E


E0 E0
and b particles are emitted
(a) 10 a, 6b (b) 4 protons, 8 neutrons
(c) 6 electrons, 8 protons (d) 6b, 8a
81. An element A decays into an element C by a two - step (c) N(E) (d) N(E)
process :
E E
A ® B + He 42 and B ® C + 2e 0 -1 E0 E0
Then 89. Assertion : A free neutron decays to a proton but a free
(a) A and C are isotopes proton does not decay to a neutron. This is because neutron
(b) A and C are isobars is an uncharged particle and proton is a charged particle.
(c) B and C are isotopes Reason : Neutron has larger rest mass than the proton.
(d) A and B are isobars (a) If both Assertion and Reason are correct and Reason
82. Assertion : Radioactivity of 108 undecayed radioactive is the correct explanation of Assertion.
nuclei of half life of 50 days is equal to that of 1.2 × 108 (b) If both Assertion and Reason are correct, but Reason
number of undecayed nuclei of some other material with is not the correct explanation of Assertion.
half life of 60 days. (c) If Assertion is correct but Reason is incorrect.
Reason : Radioactivity is proportional to half-life. (d) If both the Assertion and Reason are incorrect.
(a) If both Assertion and Reason are correct and Reason 90. Actinium 231, 231 AC89, emit in succession two b particles,
is the correct explanation of Assertion. four a-particles, one b and one a plus several g rays. What
(b) If both Assertion and Reason are correct, but Reason is the resultant isotope?
is not the correct explanation of Assertion. (a) 221 Au79 (b) 211 Au 79 (c) 221 Pb 82 (d) 211 Pb 82
NUCLEI 407

236
91. The compound unstable nucleus 92 (a) 0.922 per minutes (b) 0.691 per minutes
U oftendecays in
accordance with the following reaction (c) 0.461 per minutes (d) 0.230 per minutes
236 140 100. The activity of a freshly prepared radioactive sample is
92 U ®54 Xe +94
38 Sr + other particles 1010 disintegrations per second, whose mean life is 109 s.
In the nuclear reaction presented above, the "other particle" The mass of an atom of this radioisotope is 10–25 kg. The
might be mass (in mg) of the radioactive sample is
(a) an alpha particle
(a) 1 (b) 3 (c) 5 (d) 6
(b) two protons
(c) one proton and neutron 101. The half life of the radioactive substance is 40 days. The
(d) two neutrons substance will disintegrate completely in
92. A radioactive nucleus undergoes a -emission to form a (a) 40 days (b) 400 days
stable element. What will be the recoil velocity of the (c) 4000 days (d) infinite time
daughter nucleus if v is the velocity of a emission? 102. A radioactive substance contains 10000 nuclei and its half-
4v 2v life period is 20 days. The number of nuclei present at the
(a) (b) end of 10 days is
A-4 A-4
(a) 7070 (b) 9000 (c) 8000 (d) 7500
4v 2v
(c ) (d) 103. The half-life of radioactive Radon is 3.8 days. The time at
A+4 A+4
93. A radioactive element forms its own isotope after three the end of which (1/20)th of the Radon sample will remain
consecutive disintegrations. The particles emitted are undecayed is (given log10e = 0.4343)
(a) 3 b–particles (a) 13.8 days (b) 16.5 days
(b) 2 b–particles and 1 a–particle (c) 33 days (d) 76 days
(c) 3 b–particles and 1 a–particle 104. A freshly prepared radioactive source of half life 2 hr emits
(d) 2 a–particles and 1 b–particle. radiation of intensity which is 64 times the permissible safe
TA level. The minimum time after which it would be possible to
94. The ratio of half-life times of two elements A and B is T . work safely with this source is
B
(a) 6 hr (b) 12 hr (c) 24 hr (d) 128 hr
lA
The ratio of respective decay constant , is 105. At time t = 0, N1 nuclei of decay constant l1 and N2 nuclei
lB
of decay constant l 2 are mixed. The decay rate of mixture is
(a) TB / TA (b) TA / TB
TA + TB TA - TB (a) - N1 N 2 e -(l1 + l 2 ) t
(c) (d)
TA TA æN ö
95. A radioactive nucleus of mass M emits a photon of (b) - çç 1 ÷÷e -(l1 + l 2 ) t
frequency n and the nucleus recoils. The recoil energy will be è N2 ø
t t
(a) Mc2 – hn (b) h2n2 / 2Mc2 (c) - ( N1l1e -l1 + N1l 2 e -l 2 )
(c) zero (d) hn
96. When a U 238 nucleus originally at rest, decays by emitting (d) - N1l1N 2 l 2 e (- l1 +l 2 ) t
an alpha particle having a speed ‘u’, the recoil speed of the
106. An archaeologist analyses the wood in a prehistoric
residual nucleus is
structure and finds that C14 (Half life = 5700 years) to C12
4u
(a) 4 u (b) - 4u (c) (d) - 4u is only one-fourth of that found in the cells of buried plants.
238 234 234 238 The age of the wood is about
221 (a) 5700 years (b) 2850 years
97. 87 Ra is a radioactive substance having half life of 4 days.
(c) 11,400 years (d) 22,800 years
Find the probability that a nucleus undergoes decay after
107. A radioactive element X converts into another stable
two half lives element Y. Half life of X is 2 hrs. Initially only X is present.
1 3 1 After time t, the ratio of atoms of X and Y is found to be 1 :
(a) 1 (b) (c) (d)
2 4 4 4, then t in hours is
98. A radioactive sample at any instant has its disintegration (a) 2 (b) 4
rate 5000 disintegrations per minute. After 5 minutes, the rate (c) between 4 and 6 (d) 6
is 1250 disintegrations per minute. Then, the decay constant 108. After 150 days, the activity of a radioactive sample is 5000
(per minute) is dps. The activity becomes 2500 dps after another 75 days.
(a) 0.4 ln 2 (b) 0.2 ln 2 (c) 0.1 ln 2 (d) 0.8 ln 2 The initial activity of the sample is
99. The activity of a radioactive sample is measured as 9750 (a) 20000 dps (b) 40000 dps
counts per minute at t = 0 & 975 counts per minute at T = 5
(c) 7500 dps (d) 10000 dps
minutes. The decay constant is approximately.
EBD_7418
408 PHYSICS

109. A nucleus with Z= 92 emits the following in a sequence: 115. The radioactivity of a sample is R1 at a time T1 and R2 at a
time T2. If the half-life of the specimen is T, the number of
a, b - , b - a, a, a, a, a, b - , b- , a, b + , b + , a atoms that have disintegrated in the time (T1 – T2) is
Then Z of the resulting nucleus is proportional to
(a) 76 (b) 78 (c) 82 (d) 74 (a) (R1T1 – R2T2) (b) (R1 – R2)
110. A radioactive nucleus (initial mass number A and atomic (c) (R1 – R2)/T (d) (R1 – R2) T
number Z emits 3 a - particles and 2 positrons. The ratio of 116. In the uranium radioactive series, the initial nucleus is 92U238
number of neutrons to that of protons in the final nucleus and that the final nucleus is 82Pb206. When uranium nucleus
will be decays to lead, the number of a particles and b particles
A- Z -8 A- Z -4 emitted are
(a) (b)
Z -4 Z -8 (a) 8a, 6 b (b) 6a, 7b (c) 6a, 8b (d) 4a, 3b
117. Radium 226Ra, spontaneously decays to radon with the
A - Z - 12 A- Z -4
(c) (d) emission of an a-particle and a g ray. If the speed of the a
Z -4 Z -2 particle upon emission from an initially stationary radium
111. Half life of a radioactive substance is 20 minute. Difference nucleus is 1.5 ×107 m/s, what is the recoil speed of the
between points of time when it is 33% disintegrated and resultant radon nucleus? Assume the momentum of g ray is
67% disintegrated is approximately negligible compared to that of a particle.
(a) 40 minute (b) 10 minute
(a) 2..0 × 105 m/s (b) 2.7 × 105 m/s
(c) 15 minute (d) 20 minute
(c) 3.5 × 105 m/s (d) 1.5 × 107 m/s
112. At any instant, the ratio of the amount of radioactive
substances is 2 : 1. If their half lives be respectively 12 and 118. Mean life (t) of a radioactive substance is x times of its half
16 hours, then after two days, what will be the ratio of the life (t). Here x is
substances ? (a) 1.4 (b) 2.5 (c) 3.2 (d) 4.8
(a) 1 : 1 (b) 2 : 1 (c) 1 : 2 (d) 1 : 4 119. In a certain hypothetical radioactive decay process, species
113. Half lives for a and b emission of a radioactive material are A decays into species B and species B decays into species
16 years and 48 years respectively. When material decays C according to the reactions :
giving a and b emission simultaneously, time in which 3/ A ¾¾ ® 2B + particles + energy
4th material decays is B ¾¾ ® 3C + particles + energy
(a) 29 years (b) 24 years The decay constant for species is l1 = 1 sec–1 and that for
(c) 64 years (d) 12 years the species B is l2 = 100 sec–1. Initially 104 moles of species
of A were present while there was none of B and C. It was
114. A radioactive nucleus undergoes a series of decay
found that species B reaches its maximum number at a time
according to the scheme t0 = 2 ln (10) sec. Calculate the value of maximum number of
a b a g moles of B.
A ¾¾
® A1 ¾¾
® A 2 ¾¾
® A 3 ¾¾
® A4
(a) 1 (b) 2 (c) 5 (d) 4
If the mass number and atomic number of ‘A’ are 180 and 120. Half lives of two isotopes X and Y of a material are known
72 respectively, then what are these numbers for A4 to be 2 × 109 years and 4 × 109 years respectively. If a planet
(a) 172 and 69 (b) 174 and 70 was formed with equal number of these isotopes, then the
(c) 176 and 69 (d) 176 and 70 current age of planet, given that currently the material has
20% of X and 80% of Y by number, will be –
(a) 2 × 109 years (b) 4 × 109 years
9
(c) 6 × 10 years (d) 8 × 109 years
NUCLEI 409

Exercise 2 : Exemplar & Past Year MCQs


(a) Triton energy is less than that of a He3 nucleus
NCERT Exemplar MCQs
(b) The electron created in the beta decay process cannot
1. Suppose we consider a large number of containers each remain in the nucleus
containing initially 10000 atoms of a radioactive material (c) both the neutrons in triton have to decay simultaneously
with a half life of 1 yr. After 1 year, resulting in a nucleus with 3 protons, which is not a
(a) all the containers will have 5000 atoms of the material He3 nucleus.
(d) free neutrons decay due to external perturbations
(b) all the contains will conatin the same number of atoms which is absent in triton nucleus
of the material but that number will only be 6. Heavy stable nuclei have more neutrons than protons. this
approximately 5000 is because of the fact that
(c) the containers will in general have different numbers (a) neutrons are heavier than protons
of the atoms of the material but their average will be (b) electrostatic force between protons are repulsive
close to 5000 (c) neutrons decay into protons through beta decay
(d) none of the containers can have more than 5000 atoms. (d) nuclear force between neutrons are weaker than that
between protons
2. The gravitational force between a H-atom and another
7. In a nuclear reactor, moderators slow down the neutrons
particle of mass m will be given by Newton's law which come out in a fission process. the moderator used
M.m where r is in km and have light nuclei. Heavy nuclei will not serve the purpose,
F=G , because
r2
(a) they will break up
(a) M = mproton + melectron (b) elastic collision of neutrons with heavy nuclei will
B not slow them down
(b) M = mproton + melectron - ( B = 13.6 eV ) (c) the net weight of the reactor would be unbearably high
c2
(d) substances with heavy nuclei do not occur in liquid
(c) M is not related to the mass of the hydrogen atom
or gaseous state at room temperature
(d) M = mproton + melectron
Past Year MCQs
V
- 2 ( V = magnitude of the potential energy of 8. The Binding energy per nucleon of 73 Li and 4
c 2 He nuclei
electron in the H-atom. are 5.60 MeV and 7.06 MeV, respectively.
3. When a nucleus in an atom undergoes a radioactive decay, In the nuclear reaction 73 Li + 11H ® 42 He + Q , the value of
the electronic energy levels of the atom energy Q released is : [AIPMT 2014, A]
(a) do not change for any type of radioactivity (a) 19.6 MeV (b) – 2.4 MeV
(b) change for a and b-radioactivity but not for g- (c) 8.4 MeV (d) 17.3 MeV
radioactivity 9. A radio isotope ‘X’ with a half life 1.4 × 109 years decays to
(c) change for a-radioactivity but not for others ‘Y’ which is stable. A sample of the rock from a cave was
found to contain ‘X’ and ‘Y’ in the ratio 1 : 7. The age of the
(d) change for b-radioactivity but not for others rock is : [AIPMT 2014, A]
4. Mx and My denote the atomic masses of the parent and the (a) 1.96 × 109 years (b) 3.92 × 109 years
daughter nuclei respectively in radioactive decay. The Q – (c) 4.20 × 109 years (d) 8.40 × 109 years
value for a b– decay is Q1 and that for a b+ decay is Q2. If me 10. A radioactive nuclide is produced at the constant rate of n
denotes the mass of an electron, then which of the following per second (say, by bombarding a target with neutrons).
statements is correct? The expected number N of nuclei in existence t seconds
(a) Q1 = (Mx – My) c2 and Q2 = [Mx – My – 2me] c2 after the number is N0 is given by [AIIMS 2014, C]
(b) Q1 = (Mx – My) c2 and Q2 = (Mx – My) c2 (a) N = N0e–lt
(c) Q1 = (Mx – My – 2me) c2 and Q2 = (Mx – My + 2ce) c2 n
(b) N= + N 0e - lt
(d) Q1 = (Mx – My + 2me) c2 and Q2 = (Mx – My + 2 me)c2 l
5. Tritium is an isotope of hydrogen whose nucleus triton (c) N = n + æç N 0 - n ö÷ e - l t
contains neutrons and 1 proton. Free neutrons decay into l è lø
n æ nö
p + e + n. If one of the neutrons is Triton decays, it would (d) N = + ç N 0 + ÷ e - lt
transform into He3 nucleus. This does not happen. This is l è lø
because Where l is the decay constant of the sample
EBD_7418
410 PHYSICS

11. Assertion : The ionising power of b-particle is less 17. Half-lives of two radioactive elements A and B are 20 minutes
compared to a-particles but their penetrating power is more. and 40 minutes, respectively. Initially, the samples have
Reason : The mass of b-particle is less than the mass of a- equal number of nuclei. After 80 minutes, the ratio of
decayed number of A and B nuclei will be :
particle. [AIIMS 2014, C]
[JEE Main 2016, A]
(a) If both Assertion and Reason are correct and Reason (a) 1 : 4 (b) 5 : 4 (c) 1 : 16 (d) 4 : 1
is the correct explanation of Assertion. 18. A radioactive element X converts into another stable
(b) If both Assertion and Reason are correct, but Reason element Y. Half life of X is 2 hrs. Initially only X is present.
is not the correct explanation of Assertion. After time t, the ratio of atoms of X and Y is found to be 1 :
(c) If Assertion is correct but Reason is incorrect. 4, then t in hours is [BITSAT 2016, A]
(d) If both the Assertion and Reason are incorrect. (a) 2 (b) 4
12. A nucleus of uranium decays at rest into nuclei of thorium (c) between 4 and 6 (d) 6
and helium. Then : [AIPMT 2015, C] 19. The mass of 7N15 is 15.00011 amu, mass of 8O16 is 15.99492
(a) the helium nucleus has less momentum than the amu and mP = 1.00783 amu. Determine binding energy of
thorium nucleus. last proton of 8O16. [BITSAT 2016, A]
(b) the helium nucleus has more momentum than the (a) 2.13 MeV (b) 0.13 MeV
thorium nucleus. (c) 10 MeV (d) 12.13 MeV
(c) the helium nucleus has less kinetic energy than the 20. Radioactive material 'A' has decay constant '8 l' and material
thorium nucleus. 'B' has decay constant 'l'. Initially they have same number
(d) the helium nucleus has more kinetic energy than the of nuclei. After what time, the ratio of number of nuclei of
thorium nucleus. 1
é1ù material 'B' to that 'A' will be ? [NEET 2017, A]
e
13. The fossil bone has a 14C : 12C ratio, which is ê ú of that
ë 16 û 1 1 1 1
14 (a) (b) (c) (d)
in a living animal bone. If the half-life of C is 5730 years, 7l 8l 9l l
then the age of the fossil bone is [AIIMS 2015, A] 21. The activity of a radioactive sample is measured as N0
(a) 11460 years (b) 17190 years counts per minute at t = 0 and N0/e counts per minute at t =
(c) 22920 years (d) 45840 years 5 minutes. The time (in minutes) at which the activity
14. Assertion : Energy is released when heavy nuclei undergo reduces to half its value is [AIIMS 2017, A]
fission or light nuclei undergo fusion and 5
Reason : For heavy nuclei, binding energy per nucleon (a) log e 2 / 5 (b) log 2
e
increases with increasing Z while for light nuclei it decreases (c) 5 log 102 (d) 5 loge 2
with increasing Z. [AIIMS 2015, C] 22. Assertion : Energy is released when heavy nuclei undergo
(a) If both Assertion and Reason are correct and Reason fission or light nuclei undergo fusion and
is the correct explanation of Assertion. Reason : For heavy nuclei, binding energy per nucleon
(b) If both Assertion and Reason are correct, but Reason increases with increasing Z while for light nuclei it decreases
is not the correct explanation of Assertion. with increasing Z. [AIIMS 2017, C]
(c) If Assertion is correct but Reason is incorrect. (a) If both Assertion and Reason are correct and Reason
is the correct explanation of Assertion.
(d) If both the Assertion and Reason are incorrect.
(b) If both Assertion and Reason are correct, but Reason
15. In an ore containing uranium, the ratio of U238 to Pb206 is 3. is not the correct explanation of Assertion.
Calculate the age of the ore, assuming that all the lead (c) If Assertion is correct but Reason is incorrect.
present in the ore is the final stable product of U238. Take (d) If both the Assertion and Reason are incorrect.
the half-life of U238 to be 4.5 × 109 yr. [BITSAT 2015, A] 23. A radioactive nucleus A with a half life T, decays into a
(a) 1.6 × 193 yr (b) 1.5 × 104 yr nucleus B. At t = 0, there is no nucleus B. At sometime t, the
(c) 1.867 × 10 yr 9 (d) 2 × 105 yr ratio of the number of B to that of A is 0.3. Then, t is given by
[JEE Main 2017, S]
16. Binding energy per nucleon versus mass number curve for
T
nuclei is shown in the figure. W, X, Y and Z are four nuclei (a) t = T log (1.3) (b) t = log(1.3)
indicated on the curve. The process that would release
energy is [AIIMS 2016, C] log 2 log1.3
(c) t =T (d) t = T
(a) Y ® 2Z log1.3 log 2
(b) W ®X+Z 24. After two hours, one-sixteenth of the starting amount of a
(c) certain radioactive isotope remained undecayed. The half
W ® 2Y
life of the isotope is [BITSAT 2017, A]
(d) X®Y+Z (a) 15 minutes (b) 30 minutes
(c) 45 minutes (d) 4 hour
NUCLEI 411

25. When 92U235 undergoes fission, 0.1% of its original mass 27. The luminous dials watches are usually made by mixing a
is changed into energy. How much energy is released if 1 zinc sulphide phosphor with an a- particle emitter. The
mass of radium (mass number 226, half - life 1620 years)
kg of 92U235 undergoes fission [BITSAT 2017, A]
that is needed to produce an average of 10 a-particles per
(a) 9 × 1010 J (b) 9 × 1011 J second for this purpose is [AIIMS 2018, A]
(c) 9 × 1012 J (d) 9 × 1013 J (a) 2.77 mg (b) 2.77 g
(c) 2.77 × 10–23 g (d) 2.77 × 10–13 kg
26. For a radioactive material, half-life is 10 minutes. If initially 28. A radioactive substance with decay constant of 0.5s–1 is
there are 600 number of nuclei, the time taken (in minutes) being produced at a constant rate of 50 nuclei per second.
for the disintegration of 450 nuclei is [NEET 2018, A] If there are no nuclei present initially, the time (in second)
(a) 20 (b) 10 (c) 15 (d) 30 after which 25 nuclei will be present is [BITSAT 2018, A]
(a) 1 (b) ln 2 (c) ln (4/3) (d) 2 ln (4/3)

Exercise 3 : Try If You Can


1. Two radioactive substances A and B have decay constants R are (tQ, lQ) and (tR, lR) respectively. The plot implies
5l and l respectively. At t = 0 they have the same number that (the symbol “»” means “approximately equal to ”)
of nuclei. The ratio of number of nuclei of A to those of B 6
will be (1/e)2 after a time interval 10
5 Q
(a) 4 l (b) 2 l (c) 1/ 2 l (d) 1/4 l 10

Counts/year
2. A nuclear reactor generates power at 50% efficiency by R
4
fission of 92U235 into two equal fragments of 46Pd116 with 10
the emission of two gamma rays of 5.2 MeV each and three
3
neutrons. The average binding energies per particle of 10
235 and Pd116 are 7.2 MeV and 8.2 MeV respectively. 0 10 20 30 40
92U 46
Calculate the energy released in one fission event. Time (years)
(a) 10 MeV (b) 25 MeV (a) t Q » t R an d l Q > l R
(c) 100 MeV (d) 200 MeV
3. A star initially has 1040 deuterons. It produces energy via (b) t Q > t R an d l Q < l R
the processes 12 H +12 H ®13 He + p and 12 H +13 ®24 He + n
If the average power radiated by the star is 1016 W, the (c) t Q » t R an d l Q » l R
deuteron supply of the star is exhausted in a time of the
order of (d) t Q < t R an d l Q > l R
[ Given : M(2H) = 2.014 u, M (n) = 1.008 u, M(p) = 1.008 u, 7. The proton -proton mechanism that accounts for energy
and M (4He) = 4.001 u] production in the sun releases 26.7 MeV energy for each event.
(a) 106 s (b) 108 s (c) 1012 s (d) 1016 s In this process, protons fuse to form an alpha particle (4He).
4. A sample of radioactive material decays simultaneously by dm
At what rate is hydrogen being consumed in the core of
two processes A and B with half lives 1/2 and 1/4 hr. dt
respectively. For first half hour it decays with the process the sun by the p-p cycle? Power of sun is 3.90 × 1026 W.
A, next one hour with the process B and for further half an (a) 1.1 × 1011 kg/s (b) 2.3 × 1010 kg/s
9
(c) 3.2 × 10 kg/s (d) 6.2 × 1011 kg/s
hour with both A and B. If originally there were N0 nuclei,
the number of nuclei after 2 hour of such decay is 8. Find the kinetic energy of the a-particle emitted in the decay
x 238
æ 1ö Pu ®234 U + a
N 0 ç ÷ then find the value of x.
è 2ø The atomic masses needed are as follows :
(a) 5 (b) 4 (c) 3 (d) 8 238 234
Pu = 238.04955u U = 234.04095u
5. A radioactive source, in the form of a metallic sphere of radius
10–2 m emits b-particles at the rate of 5 × 1010 particles per
4
He = 4.002603u [Neglect any residual nucleus]
second. The source is electrically insulated. How long will (a) 5.58 MeV (b) 2.38 eV (c) 6 eV (d) 9.8 MeV
it take for its potential to be raised by 2V, assuming that 9. In a nuclear reactor U235 undergoes fission liberating 200
40% of the emitted b-particles escapes the sources. MeV of energy. The reactor has 10% efficiency and
(a) 2.32 × 10–3s (b) 1.1 × 10–2 s produces 1000 MW power. If the reactor is to function for
(c) 3.2 × 10 s –3 (d) 6.94 × 10–4s 10 year, find the total mass of the uranium required.
6. The activity of two radioactive nuclei Q and R is plotted on (a) 1.1 × 103 kg (b) 2.1 × 103 kg
a semi-log graph. The half-life and decay constant of Q and (c) 2 kg (d) 3.8 × 104 kg
EBD_7418
412 PHYSICS

10. A neutron of energy 1 MeV and mass 1.6 × 10–27 kg passes 13. The half life of a radioactive isotope ‘X’ is
a proton at such a distance that the angular momentum of 20 years. It decays to another element ‘Y’ which is stable.
the neutron relative to the proton approximately equals The two elements ‘X’ and ‘Y’ were found to be in the ratio
10–33 J s. The distance of closest approach neglecting the of 1 : 7 in a sample of a the given rock. The age of the rock
interaction between particles is is estimated to be
(a) 0.44 nm (b) 0.44 mm (c) 0.44 Å (d) 0.44 fm (a) 60 years (b) 80 years
11. in a sample of rock, the ratio of 206 Pb to 238U nuclei is found (c) 100 years (d) 40 years
to be 0.5. The age of the rock is (given half – life of U238 is
14. The half life of a radioactive nucleus is 50 days. The time
4.5 × 109 years) 2
(a) 2.25 × 109 year (b) 4.5× 109 ln 3 year interval (t2 – t1 ) between the time t2when of it has
3
æ 3ö 1
1n ç ÷ decayed and the time t1 when of it had decayed is
è 2ø 9 æ 3ö 3
(c) 4.5 ´ 109 year (d) 2.25 ´ 10 1n çè 2 ÷ø year (a) 30 days (b) 50 days (c) 60 days (d) 15 days
1n 2
12. The half life of a radioactive isotope 'X' is 50 years. It decays 15. Two radioactive nuclei P and Q, in a given sample decay
to another element 'Y' which is stable. The two elements 'X' into a stable nucleus R. At time t = 0, number of P species
and 'Y' were found to be in the ratio of 1 : 15 in a sample of are 4 N0 and that of Q are N0. Half-life of P (for conversion
a given rock. The age of the rock was estimated to be to R) is 1 minute where as that of Q is 2 minutes. Initially
(a) 150 years (b) 200 years there are no nuclei of R present in the sample. When number
(c) 250 years (d) 100 years of nuclei of P and Q are equal, the number of nuclei of R
present in the sample would be
9N 0 5N0
(a) 3N0 (b) (c) (d) 2N0
2 2

ANSWER KEYS
Exercise 1 : Topic-wise MCQs
1 (b) 13 (b) 25 (c) 37 (d) 49 (c) 61 (c) 73 (b) 85 (a) 97 (b) 109 (b)
2 (c) 14 (a) 26 (a) 38 (c) 50 (b) 62 (c) 74 (c) 86 (c) 98 (a) 110 (b)
3 (c) 15 (d) 27 (b) 39 (c) 51 (b) 63 (d) 75 (c) 87 (d) 99 (c) 111 (d)
4 (c) 16 (b) 28 (c) 40 (c) 52 (c) 64 (a) 76 (c) 88 (c) 100 (a) 112 (a)
5 (c) 17 (b) 29 (d) 41 (c) 53 (a) 65 (a) 77 (d) 89 (d) 101 (d) 113 (b)
6 (a) 18 (a) 30 (b) 42 (a) 54 (b) 66 (d) 78 (a) 90 (d) 102 (a) 114 (a)
7 (b) 19 (c) 31 (d) 43 (a) 55 (a) 67 (b) 79 (c) 91 (d) 103 (b) 115 (d)
8 (d) 20 (b) 32 (d) 44 (b) 56 (b) 68 (c) 80 (d) 92 (a) 104 (b) 116 (a)
9 (c) 21 (d) 33 (c) 45 (d) 57 (a) 69 (c) 81 (d) 93 (b) 105 (d) 117 (b)
10 (c) 22 (d) 34 (c) 46 (d) 58 (c) 70 (d) 82 (c) 94 (a) 106 (c) 118 (a)
11 (a) 23 (b) 35 (a) 47 (b) 59 (d) 71 (b) 83 (c) 95 (b) 107 (c) 119 (b)
12 (b) 24 (b) 36 (c) 48 (b) 60 (c) 72 (c) 84 (a) 96 (a) 108 (a) 120 (d)
Exercise 2 : Exemplar & Past Year MCQs
1 (c) 4 (a) 7 (b) 10 (c) 13 (c) 16 (c) 19 (d) 22 (d) 25 (d) 28 (d)
2 (b) 5 (a) 8 (d) 11 (b) 14 (c) 17 (b) 20 (a) 23 (d) 26 (a)
3 (b) 6 (b) 9 (c) 12 (d) 15 (c) 18 (c) 21 (d) 24 (b) 27 (d)
Exercise 3 : Try If You Can
1 (c) 3 (c) 5 (d) 7 (d) 9 (d) 11 (c) 13 (a) 15 (b)
2 (d) 4 (d) 6 (d) 8 (a) 10 (d) 12 (b) 14 (b)
29
SEMICONDUCTOR
ELECTRONICS : MATERIALS,
Chapter DEVICES AND SIMPLE CIRCUITS

Trend
Analysis of NEET and AIIMS (Year 2010-2018)
5

4
Number of Questions

3
AIPMT/NEET
2 AIIMS

0
2010 2011 2012 2013 2014 2015 2016 2017 2018
Year

Trend
Analysis of JEE Main and BITSAT (Year 2010-2018)
5

4 JEE Main
BITSAT
Number of Questions

0
2010 2011 2012 2013 2014 2015 2016 2017 2018

Year

Chapter Utility Score (CUS)


Exam Weightage Important Concepts Difficulty Level CUS (Out of 10)
NEET 6 P-N Junction Diode,
AIIMS 4 Junction Transistors & 3/5 6/10
JEE Main 4 Logic Gates
BITSAT 3
EBD_7418
414 PHYSICS
SEMICONDUCTOR ELECTRONICS : MATERIALS, DEVICES AND SIMPLE CIRCUITS 415
EBD_7418
416 PHYSICS

Exercise 1 : Topic-wise MCQs


12. The drift current in a p-n junction is from the
Topic 1: Semiconductor and P-N Junction Diode
(a) n-side to the p-side
1. In a semiconductor (b) p-side to the n-side
(a) there are no free electrons at 0 K (c) n-side to the p-side if the junction is forward-biased
(b) there are no free electrons at any temperature and in the opposite direction if it is reverse biased
(c) the number of free electrons increases with pressure (d) p-side to the n-side if the junction is forward-biased
(d) the number of fre electrons is more than that in a and in the opposite direction if it is reverse-biased
conductor 13. If a small amount of antimony is added to germanium crystal
2. The electrical conductivity of pure germanium can be (a) it becomes a p–type semiconductor
increased by (b) the antimony becomes an acceptor atom
(a) increasing the temperature (c) there will be more free electrons than holes in the
(b) doping acceptor impurities semiconductor
(c) doping donor impurities (d) its resistance is increased
(d) All of the above 14. The relation between number of free electrons (n) in a
3. The resistivity of a semiconductor at room temperature is in semiconductor and temperature (T) is given by
between
(a) n µ T (b) n µ T2
(a) 10–2 to 10–5 W cm (b) 10–3 to 106 W cm
(c) 106 to 108 W cm (d) 1010 to 1012 W cm (c) n µ T (d) n µ T3/2
4. Electric conduction in a semiconductor takes place due to 15. If the two ends of a p-n junction are joined by a wire
(a) electrons only (a) there will not be a steady current in the circuit
(b) holes only (b) there will be a steady current from the n-side to the p-
(c) both electrons and holes side
(d) neither electrons nor holes (c) there will be a steady current from the p-side to the n-
5. A p-type semiconductor is side
(a) positively charged (d) there may or may not be a current depending upon the
(b) negatively charged resistance of the connecting wire
(c) uncharged 16. The average value of output direct current in a full wave
(d) uncharged at 0K but charged at higher temperatures rectifier is
6. Let nh and ne be the number of holes and conduction (a) I0/p (b) I0/2 (c) p I0/2 (d) 2 I0/p
electrons in an extrinsic semiconductor. Then
17. In bridge rectifier circuit, (see fig.), the input signal should
(a) nh > ne (b) nh = ne (c) nh < ne (d) nh ¹ ne
7. The impurity atoms with which pure silicon may be doped be connected between A
to make it a p-type semiconductor are those of (a) A and D
(a) phosphorus (b) boron
(b) B and C D B
(c) antimony (d) nitrogen
8. When an impurity is doped into an intrinsic semiconductor, (c) A and C
the conductivity of the semiconductor (d) B and D
(a) increases (b) decreases C
(c) remains the same (d) becomes zero 18. A semiconductor device is connected in a series circuit with
9. The ratio of forward biased to reverse biased resistance for a battery and a resistance. A current is found to pass through
pn junction diode is the circuit. If the polarity of the battery is reversed, the
(a) 10–1 : 1 (b) 10–2 : 1 (c) 104 : 1 (d) 10–4 : 1 current drops to almost zero. The device may be a/an
10. The average value of output direct current in a half wave (a) intrinsic semiconductor
rectifier is (b) p-type semiconductor
(a) I0/p (b) I0/2 (c) p I0/2 (d) 2 I0/p (c) n-type semiconductor
11. A strip of copper and another of germanium are cooled from (d) p-n junction diode
room temperature to 80K. The resistance of 19. On doping germanium with donor atoms of density
(a) each of these decreases 1017 cm–3 its conductivity in mho/cm will be
(b) copper strip increases and that of germanium decreases [Given : me = 3800 cm2/V–s and ni = 2.5 × 1013 cm–13]
(c) copper strip decreases and that of germanium increases
(a) 30.4 (b) 60.8 (c) 91.2 (d) 121.6
(d) each of these increases
SEMICONDUCTOR ELECTRONICS : MATERIALS, DEVICES AND SIMPLE CIRCUITS 417

20. What is the conductivity of a semiconductor if electron (a) Assertion is correct, reason is correct; reason is a
density = 5 × 1012/cm3 and hole density = 8 × 1013/cm3 correct explanation for assertion.
(µe = 2.3 m2 V–1 s–1, µh = 0.01 m2V–1 s–1) (b) Assertion is correct, reason is correct; reason is not
a correct explanation for assertion
(a) 5.634 W–1 m–1 (b) 1.968 W–1 m–1
(c) Assertion is correct, reason is incorrect
(c) 3.421 W–1 m–1 (d) 8.964 W–1 m–1
21. The current through an ideal PN-junction shown in the (d) Assertion is incorrect, reason is correct.
following circuit diagram will be 27. When the forward bias voltage of a diode is changed from
0.6 V to 0.7 V, the current changes from 5 mA to 15 mA.
Then its forward bias resistance is
(a) 0.01 W (b) 0.1 W (c) 10 W (d) 100 W
28. If the lattice constant of this semiconductor is decreased,
then which of the following is correct?
(a) zero (b) 1 mA (c) 10 mA (d) 30 mA conduction
Ec
22. In a p-n junction having depletion layer of thickness band width
10–6 m the potential across it is 0.1 V. The electric field is band gap Eg
(a) 107 V/m (b) 10–6 V/m valence
5
(c) 10 V/m (d) 10–5 V/m Ev
band width
23. In a reverse biased diode when the applied voltage changes
(a) All Ec, Eg, Ev increase
by 1 V, the current is found to change by 0.5 µA. The reverse
bias resistance of the diode is (b) Ec and Ev increase, but Eg decreases
(a) 2 × 105 W (b) 2 × 106 W (c) Ec and Ev decrease, but Eg increases
(c) 200 W (d) 2 W. (d) All Ec, Eg, Ev decrease
24. For a junction diode the ratio of forward current (If) and 29. Pure Si at 500K has equal number of electron (n e) and hole
reverse current (Ir) is (nh) concentrations of 1.5 × 1016 m–3. Doping by indium
[e = electronic charge, increases n h to 4.5 × 1022 m–3. The doped semiconductor is
V = voltage applied across junction, of
k = Boltzmann constant, (a) n–type with electron concentration
T = temperature in kelvin] ne = 5 × 1022 m–3
(a) e –V/kT (b) eV/kT (b) p–type with electron concentration
(c) (e –eV/kT + 1) (d) (eeV/kT – 1) ne = 2.5 ×1010 m–3
25. If in a p-n junction diode, a square input signal of 10 V is (c) n–type with electron concentration
applied as shown ne = 2.5 × 1023 m–3
(d) p–type having electron concentration
5V
ne = 5 × 109 m–3
RL 30. The peak voltage in the output of a half-wave diode rectifier
fed with a sinusoidal signal without filter is 10V. The d.c.
-5V component of the output voltage is
Then the output signal across RL will be (a) 20/p V (b) 10/Ö2 V (c) 10/p V (d) 10V
10 V 31. In the half wave rectifier circuit operating from 50 Hz mains
+5V frequency, the fundamental frequency in the ripple
(a) (b) would be
(a) 25 Hz (b) 50 Hz (c) 70.7 Hz (d) 100 Hz
32. A d.c. battery of V volt is connected to a series
combination of a resistor R and an ideal diode D as shown
(c) (d)
-5V in the figure below. The potential difference across R
-10 V
will be
26. Assertion : In semiconductors, thermal collisions are R D
(a) 2V when diode is forward biased
respossible for taking a valence electron to the conduction
band. (b) zero when diode is forward biased
Reason : The number of conduction electrons go on (c) V when diode is reverse biased
increasing with time as thermal collisions continuously take (d) V when diode is forward biased V
place.
EBD_7418
418 PHYSICS

33. Of the diodes shown in the following diagrams, which 39. If a battery of 10V is connected across a plate of the
one is reverse biased ? semiconductor of area 4 × 10–4 m2 and of thickness 2.2 ×10–3
+10 V m, then the current flowing through the plate is
R (a) 3.8 mA (b) 1.8 mA (c) 2.8 mA (d) 4.8 mA
(a) (b)
+5 V –12 V R 40. Assuming the diodes to be of silicon with forward resis-
–5 V tance zero, the current I in the following circuit is
+5 V
I 2 kW
R
(c) R
(d)
E = 20V
–10 V

34. If the forward bias on p-n junction is increased from zero to


0.045 V, then no current flows in the circuit. The contact
potential of junction i.e. VB is (a) 0 (b) 9.65 mA (c) 10 mA (d) 10.36 mA
(a) zero (b) 0.045 V 41. A pure semiconductor has equal electron and hole
(c) more than 0.045 V (d) less than 0.045 V concentration of 1016 m–3. Doping by indium increases nh
35. Assertion : A pure semiconductor has negative temperature to 5 × 1022 m –3 . Then, the value of n e in the doped
coefficient of resistance. semiconductor is
Reason : In a semiconductor on raising the temperature, (a) 106/m3 (b) 1022/m3
more charge carriers are released, conductance increases 6
(c) 2 × 10 /m 3 (d) 2 × 109/m3
and resistance decreases. 42. In fig., the input is across the terminals A and C and the
(a) Assertion is correct, reason is correct; reason is a output is across B and D. Then the output is
correct explanation for assertion. B C
(b) Assertion is correct, reason is correct; reason is not
a correct explanation for assertion
(c) Assertion is correct, reason is incorrect
(d) Assertion is incorrect, reason is correct.
A D
36. In the energy band diagram of
a material shown below, the Ec (a) zero (b) same as the input
open circles and filled circles (c) half wave rectified (d) full wave rectified
denote holes and electrons Eg
43. An electric field is applied to a semiconductor. Let the
respectively. The material is
number of charge carriers be n and the average drift speed
(a) an insulator be v. If the temperature is increased
(b) a metal Ev (a) both n and v will increase
(c) an n-type semiconductor (b) n will increase but v will decrease
(d) a p-type semiconductor (c) v will increase but n will decrease
(d) both n and v will decrease
37. The grid voltage of any triode valve is changed from –1
44. Currents flowing in each of the circuits A and B respec-
volt to –3 volt and the mutual conductance is 3 × 10–4 mho.
tively are
The change in plate circuit current will be
(a) 0.8 mA (b) 0.6 mA (c) 0.4 mA (d) 1 mA 4W 4W

38. A d.c. battery of V volt is connected to a series combination


of a resistor R and an ideal diode D as shown in the figure 4W 4W
below. The potential difference across R will be
8V 8V
R D
+ – + –
(Circuit A) (Circuit B)
(a) 1 A, 2A (b) 2A, 1A
(c) 4A, 2A (d) 2A, 4A
V
45. The ratio of electron and hole currents in a semiconductor
(a) 2V when diode is forward biased is 7/4 and the ratio of drift velocities of electrons and holes
(b) zero when diode is forward biased is 5/4, then the ratio of concentrations of electrons and
(c) V when diode is reverse biased holes will be
(d) V when diode is forward biased (a) 5/7 (b) 7/5 (c) 25/49 (d) 49/25
SEMICONDUCTOR ELECTRONICS : MATERIALS, DEVICES AND SIMPLE CIRCUITS 419

46. A zener diode, having breakdown voltage equal to 15V, is 55. A half-wave rectifier is being used to rectify an alternating
used in a voltage regulator circuit shown in figure. The voltage of frequency 50 Hz. The number of pulses of
current through the diode is rectified current obtained in one second is
(a) 50 (b) 25 (c) 100 (d) 2000
250W
Topic 2: Junction Transistor
20V 15V 1kW
56. The transistor are usually made of
(a) metal oxides with high temperature coefficient of
(a) 10 mA (b) 15 mA (c) 20 mA (d) 5 mA resistivity
47. In a p-type semiconductor the acceptor level is situated 60 (b) metals with high temperature coefficient of resistivity
meV above the valence band. The maximum wavelength of (c) metals with low temperature coefficient of resistivity
light required to produce a hole will be
(d) semiconducting materials having low temperature
(a) 0.207 × 10–5 m (b) 2.07 × 10–5 m
–5 coefficient of resistivity
(c) 20.7 × 10 m (d) 2075 × 10–5 m
48. The concentration of hole - electron pairs in pure silicon at 57. To use a transistor as an amplifier, emitter-base junction
T = 300 K is 7 × 1015 per cubic meter. Antimony is doped is kept in ...X... and base-collector junction is kept in ...Y...
into silicon in a proportion of 1 atom in 107 Si atoms. Here, X and Y refer to
Assuming that half of the impurity atoms contribute electron (a) forward bias, forward bias
in the conduction band, calculate the factor by which the (b) reverse bias, reverse bias
number of charge carriers increases due to doping. The
(c) reverse bias, forward bias
number of silicon atoms per cubic meter is 5 × 1028
(a) 2.8 × 105 (b) 3.1 × 102 (d) forward bias, reverse bias
(c) 4.2 × 10 5 (d) 1.8 × 105 58. A transistor has three impurity regions. All the three regions
49. If gm = 2 × 10 ohm ; m = 42; resistance of load, R = 50 kilo
–3 –1 have different doping levels. In order of increasing doping
ohm, then the voltage amplification obtained from this will level, the regions are
be (a) emitter, base and collector
(a) 30.42 (b) 29.57 (c) 28.18 (d) 27.15 (b) collector, base and emitter
50. The intrinsic conductivity of germanium at 27°C is 2.13 mho (c) base, emitter and collector
m–1 and mobilities of electrons and holes are 0.38 and 0.18 (d) base, collector and emitter
m2V–1s–1 respectively. The density of charge carriers is 59. When npn transistor is used as an amplifier
(a) 2.37 × 1019 m–3 (b) 3.28 × 1019 m–3 (a) electrons move from collector to base
(c) 7.83 × 10 m 19 –3 (d) 8.47 × 1019 m–3 (b) holes move from emitter to base
51. A diode having potential difference 0.5 V across its junction (c) electrons move from base to collector
which does not depend on current, is connected in series (d) holes move from base to emitter
with resistance of 20 W across source. If 0.1 A current 60. Current gain in common emitter configuration is more than
passes through resistance then what is the voltage of the 1 because
source?
(a) 1.5 V (b) 2.0 V (c) 2.5 V (d) 5 V (a) Ic < Ib (b) Ic < Ie (c) Ic > Ie (d) Ic > Ib
52. A zener diode of voltage VZ (= 6V) is used to maintain a 61. In a common base amplifier, the phase difference between
constant voltage across a load resistance RL (= 1000 W) by the input signal voltage and output voltage is
using a series resistance Rs (= 100W). If the e.m.f. of source
is E (= 9 V), what is the power being dissipated in Zener p p
(a) p (b) (c) (d) 0
diode ? 4 2
(a) 0.144 watt (b) 0.324 watt 62. For a transistor amplifier in common emitter configuration
(c) 0.244 watt (d) 0.544 watt for load impedance of 1k W (hfe = 50 and h oe = 25ms) the
53. In general maximum rectification efficiency for a half wave current gain is
recifier is (a) – 24.8 (b) – 15.7 (c) – 5.2 (d) – 48.78
(a) 40.6% (b) 59.8% 63. In a common base mode of a transistor, the collector current
(c) 73.1% (d) 85.2% is 5.488 mA for an emitter current of 5.60 mA. The value of
54. A sinusoidal voltage of amplitude 25 volt and frequency
50Hz is applied to a half wave rectifier using P-n junction the base current amplification factor (b) will be
diode. No filter is used and the load resistor is 1000W. The (a) 49 (b) 50 (c) 51 (d) 48
forward resistance Rf of ideal diode is 10W. The percentage 64. A transistor has a base current of 1 mA and emitter current
rectifier efficiency is 90 mA. The collector current will be
(a) 40% (b) 20% (c) 30% (d) 15%
(a) 90 mA (b) 1 mA (c) 89 mA (d) 91 mA
EBD_7418
420 PHYSICS

65. In a common emitter transistor amplifier b = 60, Ro = 5000 W 75. A transistor is operated in common emitter configuration at
and internal resistance of a transistor is 500 W. The voltage VC = 2V such that a change in the base current from 100 mA
amplification of amplifier will be to 300 mA produces a change in the collector current from
(a) 500 (b) 460 (c) 600 (d) 560 10mA to 20 mA. The current gain is
66. For a common base amplifier, the values of resistance gain (a) 50 (b) 75 (c) 100 (d) 25
and voltage gain are 3000 and 2800 respectively. The current 76. In the following common emitter configuration an NPN
gain will be transistor with current gain b = 100 is used. The output
(a) 1.1 (b) 0.98 (c) 0.93 (d) 0.83 voltage of the amplifier will be
67. What is the voltage gain in a common emitter amplifier,
where input resistance is 3 W and load resistance 24 W, b
= 0.6 ?
(a) 8 . 4 (b) 4 . 8 (c) 2 . 4 (d) 480
68. The current gain of a transistor in common base mode is
0.995. The current gain of the same transistor in common (a) 10 mV (b) 0.1 V (c) 1.0 V (d) 10 V
emitter mode is 77. In a transistor, the change in base current from 100 µA to
(a) 197 (b) 201 (c) 198 (d) 199 125 µA causes a change in collector current from 5 mA to
69. An NPN-transistor circuit is arranged as shown in figure. 7.5 mA, keeping collector-to-emitter voltage constant at 10
It is V. What is the current gain of the transistor?
(a) 200 (b) 100 (c) 50 (d) 25
78. The input resistance of a silicon transistor is
100 W. Base current is changed by 40 mA which results in a
change in collector current by 2 mA. This transistor is used
as a common emitter amplifier with a load resistance of 4
(a) a common base amplifier circuit KW. The voltage gain of the amplifier is
(b) a common emitter amplifier circuit (a) 2000 (b) 3000 (c) 4000 (d) 1000
(c) a common collector amplifier circuit 79. In a common emitter (CE) amplifier having a voltage gain G,
(d) neither of the above the transistor used has transconductance 0.03 mho and
70. In a npn transistor 1010 electrons enter the emitter in current gain 25. If the above transistor is replaced with
10–6 s. 4% of the electrons are lost in the base. The current another one with transconductance 0.02 mho and current
transfer ratio will be gain 20, the voltage gain will be
(a) 0.98 (b) 0.97 (c) 0.96 (d) 0.94 1 5 2
71. In common emitter amplifier, the current gain is 62. The (a) 1.5 G (b) G (c) G (d) G
3 4 3
collector resistance and input resistance are 5 kW an 500W 80. A npn transistor is connected in common emitter
respectively. If the input voltage is 0.01 V, the output voltage configuration in a given amplifier. A load resistance of 800
is W is connected in the collector circuit and the voltage drop
(a) 0.62 V (b) 6.2 V (c) 62 V (d) 620 V across it is 0.8 V. If the current amplification factor is 0.96
72. A common emitter amplifier has a voltage gain of 50, an and the input resistance of the circuit is 192W, the voltage
input impedance of 100W and an output impedance of gain and the power gain of the amplifier will respectively be :
200W. The power gain of the amplifier is (a) 4, 3.84 (b) 3.69, 3.84(c) 4, 4 (d) 4, 3.69
(a) 500 (b) 1000 (c) 1250 (d) 50 81. A transistor connected in common emitter configuration
73. Transfer characteristics [output voltage (V0 ) vs input has input resistance RB = 2 kW and load resistance of 5 kW.
voltage (V1)] for a base biased transistor in CE configuration If b = 60 and an input signal 12 mV is applied, calculate the
voltage gain, the power gain and the value of output voltage
is as shown in the figure. For using transistor as a switch,
it is used RC
(a) in region (III) V0 I II RB VCC =10V
III
(b) both in region (I) and (III)
VCE =5V
(c) in region (II)
(d) in region (I) Vi
74. A transistor has b = 40. A change in base current of 100 m A, (a) Av = 150, Vout = 1.8V, and power gain = 9000
produces change in collector current
(b) Av = 20, Vout = 1V, and power gain = 2000
(a) 40 × 100 microampere (b) (100 – 40) microampere (c) Av = 150, Vout = 1.5V, and power gain = 8500
(c) (100 + 40) microampere (d) 100/40 microampere
(d) Av = 20, Vout = 1.5V, and power gain = 2000
SEMICONDUCTOR ELECTRONICS : MATERIALS, DEVICES AND SIMPLE CIRCUITS 421

82. A transistor is connected in common -emitter configuration. 90. The figure shows a logic circuit with two inputs A and B
The collector supply is 8V and the voltage drop across a and the output C. The voltage wave forms across A, B and
resistor of 800 W in the collector is 0.5 V. If the current gain C are as given. The logic gate circuit is:
factor a is 0.96. Find the base current.
(a) 26 m A (b) 16 m A (c) 10 m A (d) 5 m A
83. A pnp transistor is used in common-emitter mode in an A
amplifier circuit. A change of 40 µA in the base current
brings a change of 2 mA in collector current and 0.04 V in
base emitter voltage. Which of the following is the correct B
value of the input resistor Rin?
(a) Rin = 9 kW (b) Rin = 75 kW
C
(c) Rin = 1 kW (d) None of these t1 t2 t3 t4 t5 t6
84. A npn transistor in a common emitter mode is used as a (a) OR gate (b) NOR gate
simple voltage amplifier with a collector current of 4mA. (c) AND gate (d) NAND gate
the terminal of a 8V battery is connected to the collector 91. Which of the following gates will have an output of 1?
through a load resistance RL and to the base through a
resistance RB. The collector emitter voltage VCE = 4V, base- 1 0
emitter voltage VBE = 0.6 and the base current amplification 1 1
(A) (B)
factor bdc = 100, calculate the value of RB.
(a) RB = 15 kW (b) RB = 200 kW 0 0
(c) RB = 1 kW (d) RB = 185 kW 1 0
(C) (D)
Topic 3: Digital Electronics and Logic Gates (a) D (b) A (c) B (d) C
92. The following circut represents
85. In Boolean algebra, Y = A + B implies that
(a) output Y exists when both inputs A and B exist A
(b) output Y exists when either input A exists or input B
exists or both inputs A and B exist Y
(c) output Y exists when either input A exists or input B
exists but not when both inputs A and B exist B
(d) output Y exists when both inputs A and B exists but (a) OR gate (b) XOR gate
not when either input A or B exist
(c) AND gate (d) NAND gate
86. NAND and NOR gates are called universal gates primarily
because they 93. In the circuit below, A and B represent two inputs and C
represents the output.
(a) are available universally
(b) can be combined to produce OR, AND and NOT gates A
(c) are widely used in Integrated circuit packages
C
(d) are easiest to manufacture
87. The gate for which output is high if atleast one input is B
low?
(a) NAND (b) NOR (c) AND (d) OR
88. What is the value of A.C + A.B.C where A, B and C are The circuit represents
inputs ? (a) NOR gate (b) AND gate
(c) NAND gate (d) OR gate
(a) A.C (b) A.B (c) A (d) B
94. The following configuration of gate is equivalent to
89. The truth-table given below is for which gate?
A OR
A B C B
0 0 1 Y
0 1 1 AND
1 0 1 NAND
1 1 0
(a) NAND gate (b) XOR gate
(a) XOR (b) OR (c) AND (d) NAND (c) OR gate (d) NOR gate
EBD_7418
422 PHYSICS

95. In the following circuit, the output Y for all possible 99. The diagram of a logic circuit is given below. The output
inputs A and B is expressed by the truth table. F of the circuit is represented by
A
A W
Y
B X
B
F
(a) A B Y (b) A B Y W
0 0 1 0 0 1 Y
0 1 1 0 1 0
1 0 1 1 0 0 (a) W . (X + Y) (b) W . (X . Y)
1 1 0 1 1 0 (c) W + (X . Y) (d) W + (X + Y)
100. The logic circuit shown below has the input waveforms ‘A’
A B Y and ‘B’ as shown. Pick out the correct output waveform.
(c) (d) A B Y
0 0 0 0 0 0 A
0 1 1 0 1 0
1 0 1 1 0 0 Y
1 1 1 1 1 1 B
96. The combination of gates shown below yields
Input A

A
Input B
X
Output is :
B

(a) OR gate (b) NOT gate (a)


(c) XOR gate (d) NAND gate
97. For the given combination of gates, if the logic states of
inputs A, B, C are as follows A = B = C = 0 and A = B = 1, C
(b)
= 0 then the logic states of output D are
(a) 0, 0
(b) 0, 1
(c) 1, 0 (c)
(d) 1, 1
98. Which logic gate with inputs A and B performs the same
operation as that performed by the following circuit?
(d)
A
(a) NAND gate B

(b) OR gate
V
(c) NOR gate
Lamp
(d) AND gate
SEMICONDUCTOR ELECTRONICS : MATERIALS, DEVICES AND SIMPLE CIRCUITS 423

Exercise 2 : Exemplar & Past Year MCQs


5. Hole in semiconductor is
NCERT Exemplar MCQs
(a) an anti – particle of electron
1. The conductivity of a semiconductor increases with (b) a vacancy created when an electron leaves a covalent
increase in temperature because bond
(a) number density of free current carries increases (c) absence of free electrons
(b) relaxation time increases (d) an artificially created particle
(c) both number density of carries and relaxation time 6. The output of the given circuit in figure given below,
increase
(d) number density of carries increases, relaxation time
decreases but effect of decrease in relaxation time is
much less than increase in number density vm sin wt
2. In figure given below V0 is the potential barrier across a
p–n junction, when no battary is connected across the
(a) would be zero at all times
junction
(b) would be like a half wave rectifier with positive cycles
1 in output
2 (c) would be like a half wave rectifier with negative cycles
3 in output
V0 (d) would be like that of a full wave rectifier
7. In the circuit shown in figure given below, if the diode
forward voltage drop is 0.3 V, the voltage difference between
A and B is
(a) 1 and 3 both correspond to forward bias of junction
(b) 3 corresponds to forward bias of junction and 1 A A
0.2mA 0.2mA
corresponds to reverse bias of junctions
5K r1 5K
(c) 1 corresponds to forward bias and 3 corresponds to
reverse bias of junction
(d) 3 and 1 both correspond to reverse bias of junction
0.3V
3. In figure given below, assuming the diodes to be ideal
(a) D1 is forward biased and D2 is reverse biased and hence 5K r2 5kW
current flows from A to B
(b) D2 is forward biased and D1 is reverse biased and B B
hence no current flows from B to A and vice–versa (a) (b)
(c) D1 and D2 are both forward biased and hence current (a) 1.3 V (b) 2.3 V
flows from A to B (c) 0 (d) 0.5 V
(d) D1 and D2 are both reverse biased and hence no current
flows from A to B and vice – versa
8.
..
Truth table for the given circuit is
A C

. R
D1

. .
A E
–10V

.
D2 B D

B
(a) A B E (b) A B E
0 0 1 0 0 1
4. A 220 V AC supply is connected between points A and B
(figure). What will be the potential difference V across the 0 1 0 0 1 0
capacitor? 1 0 1 1 0 0
A 1 1 0 1 1 0

(c) A B E (d) A B E
200 AC C V 0 0 0
0 0 0
0 1 1 0 1 1
B 1 0 0 1 0 1
(a) 220 V (b) 110 V (c) 0 V (d) 220 2V 1 1 1 1 1 0
EBD_7418
424 PHYSICS

Past Year MCQs 15. The input signal given to a CE amplifier having a voltage
æ pö
9. The given graph represents V - I characteristic for a gain of 150 is Vi = 2 cos çè15t + ÷ø . The corresponding
semiconductor device. 3
output signal will be : [AIPMT 2015, A]
I
æ 2p ö æ 5p ö
A
(a) 75cos ç15t + ÷ (b) 2cos ç15t + ÷
V
è 3ø è 6ø
B æ 4p ö æ pö
(c) 300cos ç15t + ÷ (d) 300cos ç15t + ÷
Which of the following statement is correct ? è 3 ø è 3ø
[AIPMT 2014, C] 16. Carbon, Silicon and Germanium atoms have four valence
(a) It is V - I characteristic for solar cell where, point A electrons each. Their valence and conduction bands are
represents open circuit voltage and point B short separated by energy band gaps represented by (Eg)C, (Eg)Si
circuit current. and (E g)Ge respectively. Which one of the following
(b) It is a for a solar cell and point A and B represent open
relationship is true in their case? [AIIMS 2015, C]
circuit voltage and current, respectively.
(c) It is for a photodiode and points A and B represent (a) (Eg)C > (Eg)Si (b) (Eg)C < (Eg)Si
open circuit voltage and current, respectively. (c) (Eg)C = (Eg)Si (d) (Eg)C < (Eg)Ge
(d) It is for a LED and points A and B represent open
circuit voltage and short circuit current, respectively. 17. Assertion : If the temperature of a semiconductor is
10. The barrier potential of a p-n junction depends on: increased then it’s resistance decreases.
(A) type of semi conductor material [AIPMT 2014, C] Reason : The energy gap between conduction band and
(B) amount of doping valence band is very small. [AIIMS 2015, C]
(C) temperature (a) If both Assertion and Reason are correct and Reason
Which one of the following is correct ?
is the correct explanation of Assertion.
(a) (A) and (B) only (b) (B) only
(c) (B) and (C) only (d) (A), (B) and (C) (b) If both Assertion and Reason are correct, but Reason
is not the correct explanation of Assertion.
11. An oscillator is nothing but an amplifier with
(c) If Assertion is correct but Reason is incorrect.
[AIIMS 2014, C]
(d) If both the Assertion and Reason are incorrect.
(a) positive feedback (b) large gain
18. In CE transistor amplifier, the audio signal voltage across
(c) no feedback (d) negative feedback
12. Following diagram performs the logic function of the collector resistance of 2 kW is 2 V. If the base resistance
[AIIMS 2014, A] is 1kW and the current amplification of the transistor is
A 100, the input signal voltage is [BITSAT 2015, A]
Y (a) 2 mV (b) 3mV (c) 10mV (d) 0.1 mV
B
19. Consider the junction diode as ideal. The value of current
(a) XOR gate (b) AND gate flowing through AB is : [NEET 2016, A]
(c) NAND gate (d) OR gate A 1kW B
13. The forward biased diode connection is: [JEE Main 2014, C] +4V –6V
(a)
+2V –2V (a) 0 A (b) 10–2 A (c) 10–1 A (d) 10–3 A
–3V –3V
20. A npn transistor is connected in common emitter
(b) configuration in a given amplifier. A load resistance of 800
2V 4V W is connected in the collector circuit and the voltage drop
(c)
across it is 0.8 V. If the current amplification factor is 0.96
–2V +2V
(d) and the input resistance of the circuit is 192W, the voltage
14. In the given figure, a diode D is connected to an external gain and the power gain of the amplifier will respectively be
resistance R = 100 W and an e.m.f. of 3.5 V. If the barrier [NEET 2016, S]
potential developed across the diode is 0.5 V ,the current in (a) 4, 3.84 (b) 3.69, 3.84 (c) 4, 4 (d) 4, 3.69
the circuit will be: [AIPMT 2015, A] 21. To get output 1 for the following circuit, the correct choice
(a) 40 mA D for the input is [NEET 2016, A]
R = 100W A
(b) 20 mA
B
Y
(c) 35 mA C
(a) A = 0, B = 1, C = 0 (b) A = 1, B = 0, C = 0
(d) 30 mA (c) A = 1, B = 1, C = 0 (d) A = 1, B = 0, C = 1
3.5V
SEMICONDUCTOR ELECTRONICS : MATERIALS, DEVICES AND SIMPLE CIRCUITS 425

22. Assertion : A transistor amplifier in common emitter 26. The given electrical network is equivalent to :
configuration has a low input impedence. [NEET 2017, A]
Reason : The base to emitter region is forward biased. A Y
[AIIMS 2016, C] B
(a) If both Assertion and Reason are correct and Reason (a) OR gate (b) NOR gate
is the correct explanation of Assertion. (c) NOT gate (d) AND gate
(b) If both Assertion and Reason are correct, but Reason
27. In a common emitter transistor amplifier the audio signal
is not the correct explanation of Assertion.
voltage across the collector is 3V. The resistance of collector
(c) If Assertion is correct but Reason is incorrect.
is 3 kW. If current gain is 100 and the base resistance is 2
(d) If both the Assertion and Reason are incorrect.
kW, the voltage and power gain of the amplifier is
23. Identify the semiconductor devices whose characteristics [NEET 2017, A]
are given below, in the order (i), (ii), (iii), (iv) : (a) 15 and 200 (b) 150 and 15000
[JEE Main 2016, C] (c) 20 and 2000 (d) 200 and 1000
I
I
28. Which one of the following represents forward bias diode ?
[NEET 2017, C]
V –4V R –3V
V (a)
–2V R +2V
(b)
(i)
(ii)
3V R 5V
I dark Resistance (c)
0V R –2V
(d)
V
Intensity 29. Two ideal diodes are connected to a battery as shown in
Illuminated of light
the circuit. The current supplied by the battery is
(iii) (iv) [AIIMS 2017, A]
D1 10W
(a) Solar cell, Light dependent resistance, Zener diode, A B
simple diode
D2 20W
(b) Zener diode, Solar cell, simple diode, Light dependent C D
resistance
(c) Simple diode, Zener diode, Solar cell, Light dependent 5V
E F
resistance
(a) 0.75 A (b) 0.5 A
(d) Zener diode, Simple diode, Light dependent resistance,
Solar cell (c) 0.25 A (d) zero
24. The temperature dependence of resistances of Cu and 30. Truth table for system of four NAND gates as shown in
undoped Si in the temperature range 300-400 K, is best figure is [AIIMS 2017, A]
described by : [JEE main 2016, C] A
(a) Linear increase for Cu, exponential decrease of Si.
(b) Linear decrease for Cu, linear decrease for Si. Y
(c) Linear increase for Cu, linear increase for Si.
(d) Linear increase for Cu, exponential increase for Si.
25. If a, b, c, d are inputs to a gate and x is its output, then, as per B
the following time graph, the gate is : [JEE Main 2016, C] A B Y A B Y
d 0 0 0 0 0 0
(a) 0 1 1 (b) 0 1 0
c 1 0 1 1 0 1
1 1 0 1 1 1
b
A B Y A B Y
a 0 0 1 0 0 1
x (c) 0 1 1 (d) 0 1 0
1 0 0 1 0 1
(a) OR (b) NAND (c) NOT (d) AND 1 1 1
1 1 0
EBD_7418
426 PHYSICS

31. Assertion : Diode lasers are used as optical sources in (a) IB = 40 mA, IC = 10 mA, b = 250
optical communication. (b) IB = 25 mA, IC = 5 mA, b = 200
Reason : Diode lasers consume less energy.
(c) IB = 40 mA, IC = 5 mA, b = 125
[AIIMS 2017, C]
(a) If both Assertion and Reason are correct and Reason (d) IB = 20 mA, IC = 5 mA, b = 250
is the correct explanation of Assertion. 35. In a p-n junction diode, change in temperature due to heating
(b) If both Assertion and Reason are correct, but Reason [NEET 2018, C]
is not the correct explanation of Assertion. (a) Affects only reverse resistance
(c) If Assertion is correct but Reason is incorrect. (b) Affects only forward resistance
(d) If both the Assertion and Reason are incorrect. (c) Affects the overall V-I characteristics of p-n junction
32. In the circuit given below, V(t) is the sinusoidal voltage (d) Does not affect resistance of p-n junction
source, voltage drop VAB(t) across the resistance R is 36. Suppose a ‘n’-type wafer is creatd by doping Si crystal
[BITSAT 2017, C] having 5 ´ 1028 atoms/m3 with 1ppm concentration of As.
On the surface 200 ppm Boron is added to create ‘P’ region
in this wafer. Considering ni = 1.5 ´ 1016 m–3 calculate the
density of the minority charge carriers in the p regions.
[AIIMS 2018, S]
(a) 2.25 × 107/m3 (b) 1.12 × 103/m3
(a) is half wave rectified (c) 3.11 × 106/m3 (d) 2.11 × 105/m3
(b) is full wave rectified 37. A silicon diode has a threshold voltage of 0.7 V. If an input
(c) has the same peak value in the positive and negative voltage given by 2 sin (pt) is supplied to a half wave rectifier
half cycles circuit using this diode, the rectified output has a peak
(d) has different peak values during positive and negative value of [AIIMS 2018, A]
half cycle (a) 2 V (b) 1.4 V (c) 1.3 V (d) 0.7 V
33. In the combination of the following gates the output Y can
be written in terms of inputs A and B as [NEET 2018, A] 38. The reading of the ammeter for a silicon diode in the given
A circuit is : [JEE Main 2018, A]
(a) 0 200W
B
Y
(b) 15 mA
(c) 11.5 mA
(d) 13.5 mA
(a) A×B (b) A×B+ A×B 3V
(c) A+B (d) A × B + A × B 39. What is the voltage gain in a common emitter amplifier,
where input resistance is 3 W and load resistance 24 W,
34. In the circuit shown in the figure, the input voltage Vi is 20 b = 0.6 ?
V, VBE = 0 and VCE = 0. The values of IB, IC and b are given [BITSAT 2018, A]
by [NEET 2018, S] (a) 8 . 4 (b) 4 . 8 (c) 2 . 4 (d) 480
20 V

Rc 4 kW
C
RB
Vi
500 kW B
E
SEMICONDUCTOR ELECTRONICS : MATERIALS, DEVICES AND SIMPLE CIRCUITS 427

Exercise 3 : Try If You Can


1. A potential barrier of 0.3 V exists across a p-n junction. positive terminal is connected to A, is 50W
An electron with speed 5 × 105 m/s approaches this p-n (a) 0.1 A
50W
junction from n-side, what will be its speed on entering the (b) 0.2 A
p-side? (c) 0.3A
(a) 3 × 105 m/s (d) 0.4 A A B
(b) 3.8 × 105 m/s
7. The correct option for getting X = 1 from the given circuit is:
(c) 5 × 105 m/s
(a) A = B = C = 1 A AB
(d) 2.6 × 103 m/s
(b) A = B = 1 & C = 0 B
2. The current-voltage characteristic of an ideal p-n junction AB(BC)
X
diode is given by i = i0 (eev / kT - 1) (c) A = C = 1 & B = 0
where the drift current i0 equal 10 µA. Take the temperature (d) A = 0 & B = C = 1
BC
C
T to be 300 K.
8. A transistor is operated in common emitter configuration at
Find the voltage V0 for which eeV / KT = 100 . One can
neglect the term 1 for voltages greater than this value VC = 2V such that a change in the base current from 100 mA
to 300 mA produces a change in the collector current from
(a) 0.12 V (b) 0.5 V (c) 0.25 V (d) 1.5 V 10mA to 20 mA. The current gain is
3. A sinusoidal voltage of peak value 200 volt is connected to (a) 50 (b) 75 (c) 100 (d) 25
a diode and resistor R in the circuit shown so that halfwave 9. A Zener diode is connected to a battery and a load as show
rectification occurs. If the forward resistance of the diode below:
is negligible compared to R, the r.m.s. voltage (in volt) across
4 kW A IL
R is approximately
(a) 200 (b) 100 IZ
I
E0 = 200 R
200 Volt 10 V = VZ RL = 2kW
(c) (d) 280 60 V
2
4. What is the conductivity of a semiconductor sample having B
electron concentration of 5 × 1018 m–3, hole concentration The currents, I, IZ and IL are respectively.
of 5 × 1019 m–3, electron mobility of 2.0 m2 V–1 s–1 and
(a) 15 mA, 5 mA, 10 mA (b) 15 mA, 7.5 mA, 7.5 mA
hole mobility of 0.01 m2 V–1 s–1 ?
(Take charge of electron as 1.6 × 10–19 C) (c) 12.5 mA, 5 mA, 7.5 mA (d) 12.5 mA, 7.5 mA, 5 mA
(a) 1.68 (W – m)–1 (b) 1.83 (W – m)–1 10. A pnp transistor is used in common-emitter mode in an
(c) 0.59 (W – m) –1 (d) 1.20 (W – m)–1 amplifier circuit. A change of 40 µA in the base current
5. Two junction diodes one of Germanium (Ge) and other of brings a change of 2 mA in collector current and 0.04 V in
silicon (Si) are connected as shown in figure to a battery of base emitter voltage.
emf 12 V and a load resistance 10 k W . The germanium If a load of 6 kW is used, then also find the voltage gain of
diode conducts at 0.3 V and silicon diode at 0.7 V. When a the amplifier.
current flows in the circuit, the potential of terminal Y will be (a) 100 (b) 50 (c) 300 (d) 900
Ge
11. A 2V battery is connected across AB as shown in the
figure. The value of the current supplied by the battery
Y when in one case battery’s positive terminal is connected
Si to A and in other case when positive terminal of battery
12 V 10 kW is connected to B will respectively be:
(a) 0.4 A and 0.2 A D1 5W

(b) 0.2 A and 0.4 A


(a) 12 V (b) 11 V (c) 11.3 V (d) 11.7 V D2 10 W
6. A 5-V battery is connected across the points A and B as (c) 0.1 A and 0.2 A
shown in the figure given below. Assuming that the
resistance of each diode is zero in forward bias, and infinity (d) 0.2 A and 0.1 A
in reverse bias, the current supplied by the battery when its A B
EBD_7418
428 PHYSICS

12. Figure shows a circuit in which three identical diodes are 14. The number density no of conduction electrons in pure
used. Each diode has forward resistance of 20 W and infinite silicon at room temperature is about 1016m–3. Assume that,
backward resistance. Resistors R1 = R2 = R3 = 50 W. Battery by doping the silicon lattice with phosphorus, we want to
voltage is 6 V. The current through R3 is : increase this number by a factor of a million (106). What
æ np ö
fraction of ç n ÷ silicon atoms must we replace with
D1 R1 è Si ø
phosphorus atoms ? (Recall that at room temperature,
thermal agitation is so effective that essentially every
D2 D3 R2
phosphorus atom donates its “extra” electron to the
conduction band) given density of silicon = 2.33 ×103.
+ –
6V R3 1 2 8 1
(a) (b) (c) (d)
(a) 50 mA (b) 100 mA (c) 60 mA (d) 25 mA 2 ´ 105 3 ´ 105 6 ´ 10 4 5 ´ 106
13. A Si diode (p-n junction) is connected to a resistor and a
15. A npn transistor in a common emitter mode is used as a
biasing battery of variable voltage VB. Assume that the
simple voltage amplifier with a collector current of 4mA.
diode requires a minimum current of 1 mA to be above the
the terminal of a 8V battery is connected to the collector
knee point 0.7 V of its V-I characteristic curve. Also assume
through a load resistance RL and to the base through a
that the voltage V across the diode is independent of current
above the knee (cut-off) point R
resistance RB. The collector emitter voltage VCE = 4V, base-
I f V B = 5V, t h e n t h e emitter voltage VBE = 0.6 and the base current amplification
maximum value of R so factor bdc = 100, calculate the value of RL respectively
that the voltage V is (a) 1 kW (b) 9 kW
above the knee point (c) 15 kW (d) 12 kW
+ –
voltage, should be
VB

(a) 0.7 kW (b) 4.3 kW (c) 5 kW (d) 5.7 kW

ANSWER KEYS
Exercise 1 : Topic-wise MCQs
1 (a) 11 (c) 21 (a) 31 (b) 41 (d) 51 (c) 61 (d) 71 (b) 81 (a) 91 (d)
2 (d) 12 (a) 22 (c) 32 (d) 42 (d) 52 (a) 62 (d) 72 (c) 82 (a) 92 (b)
3 (b) 13 (c) 23 (b) 33 (d) 43 (a) 53 (a) 63 (a) 73 (b) 83 (c) 93 (d)
4 (c) 14 (d) 24 (d) 34 (c) 44 (c) 54 (a) 64 (c) 74 (a) 84 (d) 94 (b)
5 (c) 15 (a) 25 (a) 35 (a) 45 (b) 55 (b) 65 (c) 75 (a) 85 (a) 95 (c)
6 (d) 16 (d) 26 (c) 36 (d) 46 (d) 56 (a) 66 (c) 76 (c) 86 (b) 96 (a)
7 (b) 17 (d) 27 (c) 37 (b) 47 (b) 57 (d) 67 (b) 77 (b) 87 (d) 97 (d)
8 (a) 18 (d) 28 (c) 38 (d) 48 (d) 58 (d) 68 (d) 78 (a) 88 (b) 98 (b)
9 (d) 19 (b) 29 (d) 39 (c) 49 (b) 59 (d) 69 (b) 79 (d) 89 (d) 99 (c)
10 (a) 20 (b) 30 (c) 40 (c) 50 (a) 60 (d) 70 (c) 80 (a) 90 (a) 100 (a)
Exercise 2 : Exemplar & Past Year MCQs
1 (d) 5 (b) 9 (a) 13 (a) 17 (a) 21 (d) 25 (a) 29 (b) 33 (c) 37 (c)
2 (b) 6 (c) 10 (d) 14 (d) 18 (c) 22 (a) 26 (b) 30 (a) 34 (c) 38 (c)
3 (b) 7 (b) 11 (a) 15 (c) 19 (b) 23 (c) 27 (b) 31 (c) 35 (c) 39 (b)
4 (d) 8 (c) 12 (b) 16 (a) 20 (a) 24 (a) 28 (d) 32 (d) 36 (a)
Exercise 3 : Try If You Can
1 (b) 3 (b) 5 (d) 7 (b) 9 (d) 11 (a) 13 (b) 15 (a)
2 (a) 4 (a) 6 (a) 8 (a) 10 (c) 12 (a) 14 (d)
30Chapter
COMMUNICATION SYSTEMS

Trend
Analysis of JEE Main and BITSAT (Year 2010-2018)
5

4
Number of Questions

3 JEE Main
BITSAT
2

0
2010 2011 2012 2013 2014 2015 2016 2017 2018
Year

Chapter Utility Score (CUS)


Exam Weightage Important Concepts Difficulty Level CUS (Out of 10)
JEE Main 3 Bandwidth of Signals,
Modulation & Demodulation 2.5/5 4/10
Fission & Fusion
EBD_7418
430 PHYSICS
COMMUNICATION SYSTEMS 431
EBD_7418
432 PHYSICS

Exercise 1 : Topic-wise MCQs

Topic 1: Communication System 11. Long range propagation is not possible by space wave
propagation because
1. Communication is the process of (a) height of troposhere is quite small
(a) keeping in touch (b) height of troposhere is large
(b) exchange information (c) troposphere absorbs transmitted wave
(c) broad casting (d) None of these.
(d) entertainment by electronics 12. Critical frequency that gets reflected back from ionosphere is
2. Which of the following is the element of a communication (a) same for all layers of the ionosphere
system? (b) different for different layers of the ionosphere
(a) Transmitter (b) channel (c) not dependent on layers of the ionosphere
(c) Receiver (d) All of the above (d) None of these
3. A transducer used at the transmitting end, serves the 13. Encoding of signal is required for
purpose of converting (a) modulation at transmitting end
(a) electrical signal to sound form (b) modulation at receiving end
(b) sound signal to electrical form (c) demodulation at receiving end
(c) electrical signal to magnetic form
(d) demodulation at transmitting end
(d) sound signal to magnetic form
        14. Communication on ground is through electromagnetic
4. The term channel is used to indicate
(a) the amplitude range allocated to a given source waves of wavelength
(b) the frequency range allocated to a given source (a) larger than 600 m
(c) the voltage-range allocated to a given source (b) between 200 and 600 m
(d) All of the above (c) between 1 and 5 m
5. Buffer amplifier is used at the transmitting end to (d) between 10 – 3 and 0.1
(a) feed carrier frequency to master oscillator 15. During ground wave propagation the transmitted waves
(b) amplify carrier frequency gets attenuated because
(c) mix modulating signal with carrier frequency (a) earth surface absorbs the waves
(d) isolate master oscillator from other stages of (b) frequency of the waves are too low
transmitter.
(c) energy content of these waves are high
6. The purpose of a detector at the receiving end is
(a) to amplify signal (d) earth surface offers resistance.
(b) to reduce its frequency level 16. Wave obtained on superimposition of audible frequency
(c) to modulate signal e.m. wave is known as
(d) to demodulate signal (a) carrier wave (b) high frequency wave
7. Modem is a short form of (c) modulating wave (d) modulated wave
(a) modulator-demodulator 17. The AM wave is equivalent to the summation of
(b) multiplexer-demultiplexer (a) two sinusoidal waves
(c) multivibrator-degenerator
(b) three sinusoidal waves
(d) None of these
8. Ground wave propagation is possible for (c) four sinusoidal waves
(a) low radio frequency over a short range (d) None of these
(b) high radio frequency over a short range 18. Frequencies of sideband is
(c) high radio frequency over a long range (a) 2.50005 × 106 Hz, 2.49995 × 106 Hz
(d) low radio frequency over a short range. (b) 2.505× 106 Hz, 2.495 × 106 Hz
9. Long range transmission of TV-signal is done by (c) 2.505× 106 kHz, 2.495 × 106 kHz
(a) space-wave (b) sky waves (d) 2.505 MHz, 2.495 kHz
(c) ground wave (d) artificial satellite. 19. The process of superimposing signal frequency (i.e., audio
10. Field strength of tropospheric TV signal is proportional to wave) on the carrier wave is known as
1 1 (a) Transmission (b) Reception
(a) (b) l (c) (d) l2 (c) Modulation (d) Detection
l l2
COMMUNICATION SYSTEMS 433

20. As the height of satellite orbit gets lower, the speed of the 31. The function of an amplitude limitter in an FM-receiver is
satellite (a) to reduce the amplitude of the signal to suit IF amplifier
(a) increases (b) decreases (b) to amplify low frequency signal
(c) remain same (d) both (a) and (b) (c) to eliminate any change in amplitude of receiver FM
21. Which of the following is drawback of amplitude signal
modulation? (d) None of these
32. Depth of modulation in terms of Emax and Emin is
(a) low efficiency (b) noise reception
(a) ma = Emax + Emin/Emin
(c) operating range is small (d) all of these (b) ma = Emax – Emin/Emax
22. What is the modulation index of an over modulated wave (c) ma = Emax – Emin/Emax + Emin
(a) 1 (b) Zero (c) < 1 (d) > 1 (d) ma = Emax + Emin/Emax – Emin
23. The maximum line-of-sight distance dM between two 33. In an ionized medium, the phase velocity vp, group velocity
antennas having heights hT and hR above the earth is vg and the speed of light are related as
(a) vp > vg > c (b) vp = vg = c
(a) R(h T + h R ) (b) 2R /(h T + h R )
(c) vp < vg < c (d) vp > c, vg< c .
34. For a radio wave reaching the ionised medium
(c) Rh T + 2Rh R (d) 2Rh T + 2Rh R
(a) will bend away from normal
24. In AM waves, the amplitude of each side band frequency is (b) will bend towards normal
mE c (c) will bend follow a straight-path
(a) Ec (b) mEc (c) (d) 2mE (d) None of these.
2
25. For good demodulation of AM signal of carrier frequency f, 35. Audio signal cannot be transmitted because
the value of RC should be (a) the signal has more noise
(b) the signal cannot be amplified for distance
1 1 communication
(a) RC = (b) RC <
f f (c) the transmitting antenna length is very small to design
(d) the transmitting antenna length is very large and
1 1 impracticable
(c) RC ³ (d) RC >>
f f 36. The service area of space wave communication increases by
(a) increasing the height of transmitting antenna
26. If a number of sine waves with modulation indices n1, n2,
n3..... modulate a carrier wave, then the total modulation (b) decreasing the height of receiving antenna
index (n) of the wave is (c) increasing the height of both transmitting and receiving
antenna
(a) n1 + n2 ... + 2(n1 + n2....) (b) n1 - n 2 + n 3 ...... (d) decreasing the distance between transmitting and
receiving antenna
(c) n12 + n 22 + n 32 ...... (d) n1 + n 2 + ...... 37. 100% modulation in FM means
27. For a single side band transmission a balanced modulator (a) actual frequency deviation > maximum allowed
is used to frequency deviation
(a) increase power of carrier wave (b) actual frequency deviation = maximum allowed
(b) increase amplitude of carrier wave frequency deviation
(c) suppress audio signal (c) actual frequency deviation ³ maximum allowed
(d) suppress carrier component frequency deviation
28. In FM, when frequency deviation is doubled, then (d) actual frequency deviation < maximum allowed
(a) modulation is halved frequency deviation
(b) carrier swing is halved 38. Pre-emphasis in FM system is done to
(c) modulation is doubled (a) compress modulating signal
(d) modulation index is decreased (b) expand modulating signal
29. Television signals are (c) amplify lower frequency component of the modulating
(a) frequency modulated signal
(b) amplitude modulated (d) amplify higher frequency component of the modulating
(c) both frequency and amplitude modulated signal
(d) phase modulated 39. The ratio of Emax – Emin to Emax + Emin is known as
30. In PCM if the transmission path is very long (a) range of modulating signal
(a) pulse spacing is reduced (b) amplitude variation of modulating signal
(b) pulse amplitude is increased (c) depth of modulation
(c) pulse width is increased (d) None of these.
(d) repeater stations are used.
EBD_7418
434 PHYSICS

40. In an AM wave, the information is contained within 47. Of the following which is preferred modulation scheme for
(a) r.f. carrier wave digital communication?
(b) only lower and upper side frequencies (a) Pulse Code Modulation (PCM)
(c) both r.f. carrier and side frequencies (b) Pulse Amplitude Modulation (PAM)
(d) None of these (c) Pulse Position Modulation (PPM)
41. The fundamental radio antenna is a metal rod which has a (d) Pulse Width Modulation (PWM)
length equal to 48. Assertion : The loss of strength of a signal while
(a) l in free space at the frequency of operation propagating through a medium is known as attenuation.
(b) l/2 in free space at the frequency of operation Reason : Transmitter helps to avoid attenuation.
(c) l/4 in free space at the frequency of operation (a) If both Assertion and Reason are correct and Reason
(d) 3l/4 in free space at the frequency of operation is the correct explanation of Assertion.
42. In statellite communication
(b) If both Assertion and Reason are correct, but Reason
I. the frequency used lies between 5 MHz and 1 MHz.
is not the correct explanation of Assertion.
II. the uplink and downlink frequencies are different.
III. the orbit of geostationary satellite lies in the (c) If Assertion is correct but Reason is incorrect.
equatorial plane at inclination of 0. (d) If both the Assertion and Reason are incorrect.
Which of the above statement(s) is/are correct? 49. A broadcast radio transmitter radiates 12 kW when
(a) II and III (b) I and II percentage of modulation is 50%, then the unmodulated
(c) Only I (d) I, II, III and IV carrier power is
43. Amplitude modulated waves (a) 5.67 kW (b) 7.15 kW (c) 9.6 kW (d) 12 kW
I. contain frequencies (wc –wm), wc and (wc + wm) 50. A transmitter radiates 10 kW of power with the carrier
II. can be produced by application of the message unmodulated and 11.8 kW with the carrier sinusoidally
signal and the carrier wave to a non-linear device modulated. The modulation factor is
followed by a band pass filter. (a) 56% (b) 60 % (c) 72 % (d) 84%
Which of the above statements is/are correct? 51. What will be the image frequency of an FM radio receiver
(a) I only (b) II only that is tuned to 98.6 MHz broadcast station?
(c) I and II (d) None of these (a) 111.8 MHz (b) 108 MHz
44. Match the Columns I and II. (c) 121.6 MHz (d) 132 MHz
Column I Column II 52. A 1 kW carrier is modulated to a depth of 80%. The total
(A) Attenuation (1) The process of power in the modulated wave is
increasing the amplitude (a) 1.32 (b) 1.56 (c) 1.84 (d) 1.96
(B) Amplification (2) The loss of strength of 53. The frequency deviation in a FM transmission is 18.75 KHz.
a signal If it broadcasts in 88-108 MHz band, then the percent
(C) Bandwidth (3) The process of retrieval modulation is
of information from the (a) 10 % (b) 25 % (c) 50 % (d) 75 %
carrier wave
54. A 10 kW carrier is sinusoidally modulated by two carriers
(D) Demodulation (4) The frequency range
over which an corresponding to a modulation index of 30% and 40%
equipment operates respectively then total power radiated by the modulator is
(a) (A) ® (2); (B) ® (1); (C) ® (4); (D) ® (3) (a) 10.25 kW (b) 11.25 kW
(b) (A) ® (4); (B) ® (2); (C) ® (1); (D) ® (3) (c) 12.75 kW (d) 17 kW
(c) (A) ® (3); (B) ® (1); (C) ® (4); (D) ® (2) 55. An FM signal has a resting frequency of 105 MHz and
(d) (A) ® (1); (B) ® (3); (C) ® (4); (D) ® (2) highest frequency of 105.03 MHz when modulated by a
45. A radio station has two channels. One is AM at 1020 kHz and signal of frequency 5 kHz. The carrier swing is
the other FM at 89.5 MHz. For good results you will use (a) 25 kHz (b) 54 kHz (c) 60 kHz (d) 75 kHz
(a) longer antenna for the AM channel and shorter for 56. Assertion : The information contained in our original low
the FM frequency baseband signal is to be translated into high
(b) shorter antenna for the AM channel and longer for or radio frequencies before transmission.
the FM Reason : For transmitting a signal, the antenna should
(c) same length antenna will work for both
have a size comparable to the wavelength of the signal.
(d) information given is not enough to say which one to
use for which (a) If both Assertion and Reason are correct and Reason
is the correct explanation of Assertion.
46. In frequency modulation
(b) If both Assertion and Reason are correct, but Reason
(a) the amplitude of modulated wave varies as frequency
is not the correct explanation of Assertion.
of carrier wave
(b) the frequency of modulated wave varies as amplitude (c) If Assertion is correct but Reason is incorrect.
of modulating wave (d) If both the Assertion and Reason are incorrect.
(c) the frequency of modulated wave varies as frequency 57. For an AM wave, the maximum voltage was found to be 10
of modulating wave V and minimum voltage was 4 V. The modulation index of
(d) the frequency of modulated wave varies as frequency the wave is
of carrier wave (a) 0.33 (b) 0.43 (c) 0.56 (d) 0.64
COMMUNICATION SYSTEMS 435

58. For an AM-system the total power of modulated signal is 67. There is a need of translating the information contained
600 W and that of carrier is 400 W, the modulation index is in our original low frequency baseband signal into ...X...
(a) 0.25 (b) 0.36 (c) 0.54 (d) 1 or ...X... frequencies before transmission . Here, X and Y
59. The rms value of a carrier voltage is 100 volts. Compute its refere to
rms value when it has been amplitude modulated by a (a) low, radio (b) high, radio
sinusoidal audio voltage to a depth of 30%. (c) low, audio (d) high, video
(a) 94 V (b) 104 .5V (c) 114.4 V (d) 124 V 68. A geosynchronous satellite is
60. Consider the following amplitude modulated (AM) signal , (a) located at a height of 34860 km to ensure global
where fm < B xAM (t) = 10 (1 + 0.5 sin 2pfmt) cos 2pfct coverage
The average side-band power for the AM signal given above is (b) appears stationary over a place on earth’s magnetic
(a) 25 (b) 12.5 (c) 6.25 (d) 3.125 pole
61. The maximum and minimum amplitude of an AM wave are (c) not really stationary at all, but orbits the earth within
90 mV and 30 mV respectively. The depth of modulation is 24 hours
(a) 0.6 (b) 0.5 (c) 0.4 (d) 0.3 (d) always at fixed location in space and simply spins
62. For 100% modulation (AM), the useful part of the total about its own axis
power radiated is 69. For an A.M. wave, the maximum and minimum amplitude is
1 1 found to be 20V and 4v. Find the modulation index(m).
(a) of the total power (b) of the total power (a) 3/5 (b) 5/3 (c) 2/3 (d) 3/2
2 3
1 2 70. The maximum and minimum amplitude of an AM wave are
(c) of the total power (d) of the total power 90 mV and 30 mV respectively. The depth of modulation is
4 3
(a) 0.6 (b) 0.5 (c) 0.4 (d) 0.3
63. Assertion : When the height of a TV transmission tower 71. Identify the incorrect statement from the following.
is increased by three times, the range covered is doubled. (a) AM detection is carried out using a rectifier and an
Reason : The range covered is proportional to the height envelop detector
of the TV transmission tower. (b) Pulse position denotes the time of rise or fall of the
(a) If both Assertion and Reason are correct and Reason pulse amplitude
is the correct explanation of Assertion. (c) Modulation index m is kept ³ 1, to avoid distortion
(b) If both Assertion and Reason are correct, but Reason (d) Facsimile (FAX) scans the contents of the document
is not the correct explanation of Assertion. to create electronic signals
(c) If Assertion is correct but Reason is incorrect. 72. Which one of the following statement is wrong?
(d) If both the Assertion and Reason are incorrect. (a) Radio waves in the frequency range 30 MHz to 60
64. In space communication, the sound waves can be sent from MHz are called sky waves.
one place to another (b) Radio horizon of the transmitting antenna for space
(a) through space waves is dT = (2Rh T ), (R= radius of earth,
(b) through wires hT = height of transmitting antenna).
(c) by superimposing it on undamped electromagnetic (c) Within the skip distance neither the ground waves
waves nor the sky waves are received.
(d) by superimposing it on damped electromagnetic (d) The principle of fibre optical communication is total
waves internal reflection.
65. Intensity of electric field obtained at receiver antenna for a 73. If the heights of transmitting and the receiving antennas
space wave propagation is are each equal to h, the maximum line-of-sight distance
(a) directly proportional to the perpendicular-distance between them is (R is the radius of earth)
from transmitter to antenna (a) 2Rh (b) 4Rh (c) 6Rh (d) 8Rh
(b) inversely proportional to the perpendicular-distance 74. For sky wave propagation of a 10 MHz signal, what should
from transmitter to antenna be the minimum electron density in ionosphere
(c) directly proportional to the square perpendicular- (a) ~ 1.2 × 1012 m–3 (b) ~ 106 m–3
distance from transmitter to antenna (c) ~ 1014 m–3 (d) ~ 1022 m–3
(d) inversely proportional to the square perpendicular- 75. The electron density of a layer of ionosphere at a height
distance from transmitter to antenna 150 km from the earth's surface is 9 × 109 per m3. For the sky
66. If the output of the information source is a non-electrical transmission from this layer up to a range of 250 km,
signal like a voice signal, a ...A... converts it to ...B.. form The critical frequency of the layer is
before giving it as an input to the ...C... . Here, A, B and (a) 2 Hz (b) 2.7 Hz (c) 2.78 kHz (d) 2.7 MHz
C refer to 76. A radio station has two channels. One is AM at 1020 kHz
(a) receiver, electrical, channel and the other FM at 89.5 MHz. For good results you will use
(b) channel, magnetic, transducer (a) longer antenna for the AM channel and shorter for
(c) transducer, electrical, channel the FM
(d) transducer, electrical, transmitter (b) shorter antenna for the AM channel and longer for
the FM
EBD_7418
436 PHYSICS

(c) same length antenna will work for both 87. An audio signal represented as 25 sin 2p (2000 t) amplitude
(d) information given is not enough to say which one to modulated by a carrier wave : 60 sin 2p (100, 000)t. The
use for which
modulation index of the modulated signal is
77. The area of the region covered by the TV broadcast by a
TV tower of 100 m height is (Radius of the earth = 6.4 × 106 m) (a) 25% (b) 41.6 % (c) 50 % (d) 75 %
(a) 12.8p × 108 km2 (b) 1.28p × 103 km2 88. Calculate the power developed by an amplitude modulated
(c) 0.64p × 10 km 3 2 (d) 1.28 × 103 km2 wave in a load resistance of 100 W, if the peak voltage of
78. When radio waves passes through ionosphere, phase carrier wave is 100 V and modulation index is 0.4.
difference between space current and capacitive (a) 50 watt (b) 54 watt (c) 104 watt (d) 4 watt
displacement current is 89. In an FM system a 7 kHz signal modulates 108 MHz carrier
(a) 0 rad (b) (3p /2) rad(c) (p/2) rad (d) p rad so that frequency deviation is 50 kHz. The carrier swing is
79. An AM wave is expressed as e = 10 (1 + 0.6 cos 2000 p t) cos (a) 7.1 (b) 2.8 (c) 5.8 (d) 3.2
2 × 108 pt volts, the minimum and maximum value of 90. If Ec = 20 sin 105pt and Em = 10 sin 400pt are carrier and
modulated carrier wave are respectively. modulating signals, the modulation index is:
(a) 10 V and 20 V (b) 4V and 8V
(c) 16 V and 4V (d) 8 V and 20 V (a) 56% (b) 30% (c) 50% (d) 48%
80. An AM wave varies from 10V to 4V. Its percentage 91. A tuned amplifier circuit is used to generate a carrier
modulation is frequecy of 2 MHz for the amplitude modulation. The value
(a) 36% (b) 42.8% (c) 54% (d) 68 %
of LC is:
81. In a radar system, peak transmitted power is increased a
factor of 81 and the antenna diameter is increased by a 1 1
factor of 3, then the maximum-range will increase by a factor of (a) 6 (b)
3p´ 10 2p´106
(a) 81 (b) 27 (c) 9 (d) 3 1
82. Long range radio transmission is possible when the radio 1
(c) 4p´106 (d)
waves are reflected from the ionosphere. For this to happen 2 ´ 106
the frequency of the radio waves must be in the range: 92. The maximum range for the tropospheric transmission of
(a) 80 - 150 MHz (b) 8 - 25 MHz radio wave of wavelength 3m using the transmitting antenna
(c) 1 - 3 MHz (d) 150 - 1500 kHz and receiving antenna of hights 100m and 60m respectively is
83. A diode detector is used to detect an amplitude modulated (a) 8m (b) 800m (c) 8km (d) 80km
wave of 60% modulation by using a condenser of capacity 93. The velocity of all radio waves in free space is 3 × 108 m/s.
250 picofarad in parallel with a load resistance 100 kilo ohm. What is the frequency of a radio wave of wavelength 150m.
Find the maximum modulated frequency which could be (a) 5 MHz (b) 2 MHz (c) 0.5 MHz (d) 0 MHz
detected by it. 94. The area of the region covered by the TV broadcast by a
D TV tower of 100 m height is (Radius of the earth = 6.4 × 106 m).
(a) 1.28 p × 103 km2 (b) 2 p × 104 km2
(c) 3.1 × 104 km2 (d) 6 p × 104 km2
Signal C R
95. For transmission of e.m.wave of audible frequency, these
waves are superimposed with waves of
(a) 10.62 MHz (b) 10.62 kHz (a) frequency less than 20 Hz
(c) 5.31 MHz (d) 5.31 kHz (b) frequency less than 10 KHz.
84. An AM- signal is given as (c) frequency in the audible range.
xAM (t) = 100 [p(t) + 0.5g(t)] cos wct (d) radio-frequency.
in interval 0 £ t £ 1. One set of possible values of the 96. Determine the modulation index (mf) and carrier swing (S)
modulating signal and modulation index would be of an FM carrier having a frequency deviation of 25 kHz
(a) t, 0.5 (b) t, 1.0 (c) t, 1.5 (d) t2, 2.0 and a modulating signal of 5 kHz.
85. 12 signals each band limited to 5 kHz are to be transmitted (a) mf = 3 (b) mf = 5
by frequency-division multiplexer. If AM-SSB modulation (c) mf = 8 (d) mf = 9
guard band of 1 kHz is used then the bandwidth of 97. 1000 KHz carrier wave is amplitude modulated by the signal
multiplexed signal is frequency 200-4000 Hz. The channel width of this case is:
(a) 101 kHz (b) 99 kHz (c) 84 kHz (d) 71 kHz (a) 8 KHz (b) 4 KHz (c) 7.6 KHz (d) 3.8 KHz
86. An audio frequency of 10 kHz is transmitted by SSB after 98. Maximum usable frequency (MUF) in F-region layer is x,
AM with carrier waves of frequency 1MHz. The frequency when the critical frequency is 60 MHz and the angle of
of current in output load is incidence is 70°. Then calculate x.
(a) 1010 kHz or 990 kHz (b) 1010 MHz or1010 MHz (a) 122 MHz (b) 216 Hz
(c) 110 kHz or 990 kHz (d) 110 MHz or 990 MHz (c) 175 MHz (d) 75 Hz
COMMUNICATION SYSTEMS 437

99. A carrier frequency of 1 MHz and peak value of 10 V is (a) 0.014 N/C, 36 m (b) 0.14 N/C, 36 m
amplitude modulated with a signal frequency of 10 kHz (c) 0.14 N/C, 360 m (d) 0.014 N/C, 360 m
with peak value of 0.5 V. Then, the modulation index and
106. A 10 kW transmitter emits radio waves of wavelength 500
the side band frequencies respectively are
m. The number of photons emitted per second by the
(a) 0.05 and 1 ± 0.010 MHz (b) 0.5 and 1 ± 0.010 MHz
transmitter is of the order of
(c) 0.5 and 1 ± 0.005 MHz (d) 0.05 and 1 ± 0.005 MHz
100. The antenna current of an AM transmitter is 8 A when only (a) 1037 (b) 1031 (c) 1025 (d) 1043
the carrier is sent, but it increases to 893 A when the carrier 107. The electron density of a layer of ionosphere at a height
is sent, but it increases to 893 A when the carrier is 150 km from the earth's surface is 9 × 109 per m3. For the sky
modulated by a single sine wave. Find the percentage transmission from this layer up to a range of 250 km,
modulation. The critical frequency of the layer is
(a) 60.1% (b) 70.1% (c) 80.1% (d) 50.1% (a) 2 Hz (b) 2.7 Hz
101. In an amplitude modulated wave for audio frequency of
(c) 2.78 kHz (d) 2.7 MHz
500 cycle/second, the appropriate carrier frquency will be
(a) 50 cycles/s (b) 100 cycles/s 108. When radio waves passes through ionosphere, phase
(c) 500 cycles/s (d) 50, 000 cycles/s difference between space cur rent and capacitive
displacement current is
102. To cover a population of 20 lakh, a transmission tower
should have a height: (radius of the earth = 6400 km, (a) 0 rad (b) (3p /2) rad
population per square km = 1000) (c) (p / 2) rad (d) p rad
(a) 25 m (b) 50 m (c) 75 m (d) 100 m 109. What should be the maximum acceptance angle at the aircore
103. An antenna has a radiation resistance of 68 W, a load interface of an optical fibre if n1 and n2 are the refractive
resistance of 10 W, and power gain of 16. The directive indices of the core and the cladding, respectively?
gain of the antenna is
(a) 15 (b) 16.02 (c) 17.08 (d) 18.35 (a) sin–1 (n2 /n1 ) (b) sin –1 n12 – n22
104. The carrier frequency generated by a tank circuit containing
1nF capacitor and 10 mH inductor is é –1 n2 ù é –1 n1 ù
(a) 3002 kHz (b) 1592 kHz
(c) ê tan n ú (d) ê tan n ú
ë 1û ë 2û
(c) 529 kHz (b) 38 kHz 110. A transmitting antenna of height h and the receiving
105. A radio transmitter transmits at 830 kHz. At a certain antenna of height 45 m are separated by a distance of 40 km
distance from the transmitter magnetic field has amplitude for satisfactory communication in line of sight mode. Then
4.82 × 10–11T. The electric field and the wavelength are the value of h is (given radius of earth is 6400 km)
respectively (a) 10 m (b) 20 m (c) 50 m (d) 80 m

Exercise 2 : Exemplar & Past Year MCQs

NCERT Exemplar MCQs 2. A 100 m long antenna is mounted on a 500m tall building.
The complex can become a transmission tower for waves
1. Three waves A, B and C of frequencies 1600 kHz, 5MHz
with l
and 60 MHz, respectively are to be transmitted from one
(a) ~ 400 m (b) ~ 25 m
place to another. Which of the following is the most
(c) ~ 150 m (d) ~ 2400 m
appropriate mode of communication?
3. A 1 kW signal is transmitted using a communication channel
(a) A is transmitted via space wave while B and C are
which provides attenuation at the rate of – 2dB per km. If
transmitted via sky wave
the communication channel has a total length of 5 km, the
(b) A is transmitted via ground wave, B via sky wave and
C via space wave æ P0 ö
power of the signals received is [gain in dB = 10 log ç ÷ ]
(c) B and C are transmitted via ground wave while A is è Pi ø
transmitted via sky wave
(d) B is transmitted via ground wave while A and C are (a) 900 W (b) 100 W (c) 990 W (d) 1010 W
transmitted via space wave
EBD_7418
438 PHYSICS

4. A speech signal of 3 kHz is used to modulate a carrier signal (a) ABCDE (b) BADEC
of frequency 1 MHz, using amplitude modulation. The
(c) BDACE (d) BEADC
freqeuncies of the side bands will be
9. Identify the mathematical expression for amplitude
(a) 1.003 MHz and 0.997 MHz
modulated wave
(b) 3001 kHz and 2997 kHz
(a) Ac sin [{wc + k1Vm (t)} t + f]
(c) 1003 kHz and 1000 kHz
(b) Ac sin {wct + f + k2Vm (t)}
(d) 1 MHz and 0.997 MHz
(c) {Ac + k2Vm (t)} sin (wct + f)
5. A message signal of frequency wm is superposed on a
(d) Ac Vm (t) sin (wct + f)
carrier wave of frequency wc to get Amplitude Modulated
Wave (AM). The frequency of the AM wave will be Past Year MCQs
wc + wm wc - wm 10. A signal of 5 kHz frequency is amplitude modulated on a
(a) wm (b) wc (c) (d)
2 2 carrier wave of frequency 2 MHz. The frequencies of the
6. I – V characteristics of four devices are shown in figure. resultant signal is/are : [JEE Main 2015, A]
(a) 2005 kHz, 2000 kHz and 1995 kHz
I I (b) 2000 kHz and 1995 kHz
(i) (ii)
(c) 2 MHz only
V V
(d) 2005 kHz and 1995 kHz
11. Choose the correct statement : [JEE Main 2016, C]
I I
(iii) (iv) (a) In frequency modulation the amplitude of the high
frequency carrier wave is made to vary in proportion
V V to the amplitude of the audio signal.
Identify devices that can be used for modulation
(b) In frequency modulation the amplitude of the high
(a) (i) and (iii) frequency carrier wave is made to vary in proportion
(b) only (iii) to the frequency of the audio signal.
(c) (ii) and some regions of (iv) (c) In amplitude modulation the amplitude of the high
frequency carrier wave is made to vary in proportion
(d) All the devices can be used
to the amplitude of the audio signal.
7. A male voice after modulation–transmission sounds like
(d) In amplitude modulation the frequency of the high
that of a female to the receiver. The problem is due to
frequency carrier wave is made to vary in proportion
(a) poor selection of modulation index (selected 0 < m < 1) to the amplitude of the audio signal.
(b) poor bandwidth selection of amplifiers 12. In amplitude modulation, sinusoidal carrier frequency used
is denoted by wc and the signal frequency is denoted by wm.
(c) poor selection of carrier frequency
The bandwidth (Dwm) of the signal is such that Dwm < wc.
(d) loss of energy in transmission Which of the following frequencies is not contained in the
8. A basic communication system consists of modulated wave? [JEE Main 2017, C]

A. transmitter (a) wm + wc (b) wc – wm (c) wm (d) wc

B. information source 13. A telephonic communication service is working at carrier


frequency of 10 GHz. Only10% of it is utilized for
C. user of information transmission. How many telephonic channels can be
D. channel transmitted simultaneously if each channel requires a
bandwidth of 5 kHz? [JEE Main 2018, S]
E. receiver
(a) 2 × 103 (b) 2 × 104 (c) 2 × 105 (d) 2 × 106
Choose the correct sequence in which these are arranged
in a basic communication system.
COMMUNICATION SYSTEMS 439

Exercise 3 : Try If You Can


1. Sinusoidal carrier voltage of frequency 1.5 MHz and 2
l
æ1ö æ1ö
amplitude 50 V is amplitude modulated by sinusoidal voltage (a) l, Peff = K ç ÷ (b) , P = Kç ÷
èlø 8 eff èlø
of frequency 10 kHz producing 50% modulation. The lower
1
and upper side-band frequencies in kHz are l 3 l æ 1 ö2
, Peff = K æç ö÷
1
(a) 1490, 1510 (b) 1510, 1490 (c) (d) , Peff = K ç ÷
16 èlø 5 èlø
1 1 1 1 9. If a carrier wave c(t) = A sin wct is amplitude modulated by
(c) , (d) , a modulator signal m(t) = A sin wmt then the equation of
1490 1510 1510 1490
2. In optical communication system operating at 1200 nm, only modulated signal [Cm(t)] and its modulation index are
2% of the source frequency is available for TV transmission respectively
having a bandwidth of 5 MHz. The number of TV channels (a) Cm (t) = A (1 + sin wm t) sin wc t and 2
that can be transmitted is (b) Cm (t) = A (1 + sin wm t) sin wm t and 1
(c) Cm (t) = A (1 + sin wm t) sin wc t and 1
(a) 2 million (b) 10 million
(d) Cm (t) = A (1 + sin wc t) sin wm t and 2
(c) 0.1 million (d) 1 million
3. A carrier frequency of 10 kV at 1 MHz is amplitude modulated 10. A radar has a power of 1kW and is operating at a frequency
of 10 GHz. It is located on a mountain top of height 500 m.
by a 1 kHz signal of 6 kV peak voltage. When the modulation
The maximum distance upto which it can detect object
pattern is observed on a calibrated CRO, the voltage
located on the surface of the earth is (Radius of earth
indicated by the CRO will be
(a) 16 (b) 32 (c) 864 (d) 84 = 6.4 × 106m)
4. A super heterodyne receiver is designed to receive (a) 80 km (b) 16 km (c) 40 km (d) 64 km
transmitted signals between 5 and 10 MHz. The tuning 11. Given the electric field of a complete amplitude modulated
range of the local oscillate for IF frequency 600 kHz for wave as
high-side tuning would be
æ ö
ˆ c 1 + Em cos w m t cos w c t .
®
(a) 4.6 to 9.6 MHz (b) 5.6 to 10.6 MHz E = iE çè Ec ÷ø
(c) 4.6 to 10.6 MHz (d) 5.6 to 9.6 MHz
5. A transmitting antenna at the top of a tower has height 32 Where the subscript c stands for the carrier wave and m for
m and height of the receiving antenna is 50 m. What is the the modulating signal. The frequencies present in the
maximum distance between them for satisfactory modulated wave are
communication in line of sight (LOS) mode?
(a) 55.4 km (b) 45.5 km (c) 54.5 km (d) 455 km (a) w c and wc2 + w m
2

6. The maximum range for the tropospheric transmission of


(b) wc , wc + wm and wc - wm
radio wave of wavelength 3m using the transmitting antenna
and receiving antenna of heights 100 m and (c) wc and w m
60 m respectively is
(a) 8 m (b) 800 m (c) 8 km (d) 80 km (d) wc and wc wm
7. The height of a television tower is 100 m. If radius of earth 12. A device with input x(t) and outputy(t) is characterized by:
is 6.4 × 106 and average-population density surrounding y(t) = x2 (t).
the tower is 1000 per km2, then the population covered by
An FM signal with frequency deviation of 90 kHz and
the television transmission is
modulating signal bandwidth of 5 kHz is applid to this
(a) 2.06 × 106 (b) 4.02 × 106
9
device. The bandwidth of the output signal is
(c) 5.18 × 10 (d) 6.04 × 109
(a) 370 kHz (b) 190 kHz (c) 380 kHz (d) 95 kHz.
8. A signal is to be transmitted through a wave of wavelength
l, using a linear antenna. The length 1 of the antenna and 13. A modulated signal Cm(t) has the form Cm(t) = 30 sin 300pt
effective power radiated Peff will be given respectively as : + 10 (cos 200pt – cos 400pt). The carrier frequency fc, the
(K is a constant of proportionality) modulating frequency (message frequency) fw and the
modulation indix m are respectively given by :
EBD_7418
440 PHYSICS

1 (c) Modulation index=0.8, side frequency bands are at


(a) fc = 200 Hz; fw = 50 Hz; m =
2 1180 kHz and 1220 kHz
2
(b) fc = 150 Hz; fw = 50 Hz; m = (d) Modulation index=0.2, side frequency bands are at
3
1 1220 kHz and 1180 kHz
(c) fc = 150 Hz; fw = 30 Hz; m =
3 15. An audio signal consists of two distinct sounds: one a
1 human speech signal in the frequency band of 200 Hz to
(d) fc = 200 Hz; fw = 30 Hz; m =
2
2700 Hz, while the other is a high frequency music signal in
14. A signal of frequency 20 kHz and peak voltage of 5 Volt is
the frequency band of 10200 Hz to 15200 Hz. The ratio of
used to modulate a carrier wave of frequency 1.2 MHz and
the AM signal bandwidth required to send both the signals
peak voltage 25 Volts. Choose the correct statement.
together to the AM signal bandwidth requried to send just
(a) Modulation index = 5, side frequency bands are at
the human speech is :
1400 kHz and 1000 kHz
(a) 2 (b) 5 (c) 6 (d) 3
(b) Modulation index = 5, side frequency bands are at
21.2 kHz and 18.8 kHz

ANSWER KEYS
Exercise 1 : Topic-wise MCQs
1 (b) 12 (b) 23 (d) 34 (a) 45 (b) 56 (a) 67 (b) 78 (a) 89 (a) 100 (b)
2 (d) 13 (a) 24 (c) 35 (d) 46 (b) 57 (b) 68 (c) 79 (c) 90 (c) 101 (d)
3 (b) 14 (d) 25 (d) 36 (c) 47 (a) 58 (d) 69 (c) 80 (b) 91 (c) 102 (b)
4 (b) 15 (d) 26 (c) 37 (b) 48 (c) 59 (b) 70 (b) 81 (c) 92 (c) 103 (d)
5 (d) 16 (d) 27 (d) 38 (d) 49 (c) 60 (c) 71 (c) 82 (b) 93 (b) 104 (b)
6 (d) 17 (b) 28 (c) 39 (c) 50 (b) 61 (b) 72 (a) 83 (b) 94 (a) 105 (d)
7 (a) 18 (a) 29 (c) 40 (b) 51 (c) 62 (b) 73 (d) 84 (a) 95 (d) 106 (b)
8 (a) 19 (c) 30 (d) 41 (c) 52 (a) 63 (c) 74 (a) 85 (d) 96 (b) 107 (d)
9 (d) 20 (b) 31 (c) 42 (a) 53 (b) 64 (c) 75 (d) 86 (a) 97 (a) 108 (a)
10 (a) 21 (d) 32 (c) 43 (c) 54 (b) 65 (d) 76 (b) 87 (b) 98 (c) 109 (b)
11 (a) 22 (d) 33 (d) 44 (a) 55 (c) 66 (d) 77 (b) 88 (b) 99 (a) 110 (b)
Exercise 2 : Exemplar & Past Year MCQs
1 (b) 3 (b) 5 (b) 7 (b) 9 (c) 11 (c) 13 (c)
2 (d) 4 (a) 6 (c) 8 (b) 10 (a) 12 (c)
Exercise 3 : Try If You Can
1 (a) 3 (b) 5 (b) 7 (b) 9 (c) 11 (b) 13 (b) 15 (c)
2 (d) 4 (b) 6 (c) 8 (a) 10 (a) 12 (c) 14 (d)
1. PHYSICAL WORLD
Exercise - 1 20. (b) Name of the Achievement in
Physicist Year
1. (d) The macroscopic domain includes phenomena at the unification
laboratory, terrestrial and astronomical scales. Isaac Newton 1687 Unified celestial and
2. (c) 3. (d) terrestrial mechanics,
4. (b) Science is exploring, experimenting and predicting showed that the same laws
from what we see around us. of motion and the law of
5. (d) gravitation apply to both the
6. (c) The concept of energy is central to Physics and the domains.
expressions for energy can be written for every physical Michael 1830 Showed phenomena of
system. When all forms of energy e.g., Heat, mechanical Faraday electromagnetism.
energy, electrical energy etc., are counted, it turns out
that energy is conserved. The general law of J.C. Maxwell 1873 Unified electricity,
conservation of energy is true for all forces and for magnetism and optics,
any kind of transformation between different forms of showed that light is an
energy. electromagnetic wave.
7. (c) 8. (a) Cario Rubia 1984 Verified experimentally the
9. (a) Optics deals with the phenomena involving light. The predictions of the theory of
working of telescopes and microscopes, colours electromagnetic force.
exhibited by thin films, etc., are topics in optics.
The microscopic domain of Physics deals with the 21. (a) 22. (b) 23. (a)
constitution and structure of matter at the minute 24. (b) The alpha particle scattering experiment of Rutherford
scales of atoms and nuclei (and even lower scales of gave the nuclear model of the atom as shown in figure
length) and their interaction with different probes such Flash of
Lead
as electrons, photons and other elementary particles. block light
10. (a) Johannes Kepler examined the extensive data on
planetary motion collected by Tycho Brahe the
planetary circular orbits in heliocentric theory (Sun a Fluorescent
at the centre of the solar system) imagined by A q
screen
Nicolas Copernicus had to be replaced by elliptical
orbits to fit the data better. Scattering
11. (d) B angle
12. (c)
Polonium Gold foil
13. (a) sample
14. (d)
15. (b) 25. (c) 26. (b)
16. (a) 27. (a) It is mainly the electromagnetic force that governs the
structure of atoms and molecules, the dynamics of
chemical reactions and the mechanical, thermal and
Name of the Major contribution Country of other properties of materials.
physicist /discovery origin 28. (a) When we hold a book in our hand , we are balancing the
Galileo Galilei Law of inertia Italy gravitational force on the book due to the huge mass of
JC Bose Ultra short radio India the Earth by the ‘normal force’ provided by our hand.
waves
29. (d)
30. (d) In a nucleus, strong n uclear force (strongest
JJ Thomson Electron Uk fundamental force) binds protons and neutrons. The
Albert Einstein Explanation of Germeny silicon ‘chip’ triggered the computer revolution the last
photoelectric effect; three decades of the twentieth century.
Theory of relativity A most significant area to which Physics has and will
contribute is the development of alternative energy
17. (b) resources. The fossil fuels of the planet are dwindling
fact and there is an urgent need to discover new and
18. (c)
affordable sources of energy.
19. (d) Calorimeter measures heat. 31. (d) Gravitational force-weakest force strong nuclear force-
Strongest force.
EBD_7418
2. UNITS AND MEASUREMENTS

31. (a)
Exercise - 1
32. (d) Unitless quantity.
1. (b) Unit of magnetic flux is weber (Wb) 33. (d)
2. (d) 3. (b) 34. (b) The work done = force × displacement
4. (c) Picoprefix used for 10–12. \ unit, u1 = Fs
5. (a) 6. (c) and u2 = 4F × 4s = 16u.
7. (d) Joule second is the unit of angular momentum. g
8. (d) Temperature is one of the basic physical quantities. 35. (b) In CGS system, d = 4 3
cm
9. (c) The unit of mass is 100g and unit of length is 10 cm, so
10. (b)(A)®(2); (B)®(4); C®(3); (D)®(1)
11. (c)1 light year = speed of light in vacuum × no. of seconds æ 100g ö æ 4 ö
4ç çè ÷
in one year = (3 × 108) × (365 × 24 × 60 × 60) è 100 ÷ø 100 ø (100g)
= 9.467 × 1015 m. density = 3
= 3 3
æ 10 ö æ 1 ö (10cm)
12. (d) F = 6phvr çè cm÷ø çè ÷ø
10
10
\ Unit of coefficient of viscosity in S.I. system kg/m-s.
13. (a) E = sAT4 4 100g
E Watt = ´ (10)3 · = 40 unit
E is energy dissipated per second. s = = 2 4 100 (10cm) 3
4
AT m K 36. (d)
14. (d) 1 astronomical unit = 1.496 × 1011 m Densityof Substance
l RA 37. (d) Relative density =
15. (c) R = r \ r= Density of water
A l Hence no dimension.
16. (a) M = Pole strength × length distance displacement
= amp – metre × metre = amp – metre2 38. (d) speed = and velocity =
time time
dyne 70 ´ 10 -5 N
17. (b) S = 70 = = 70 ´ 10 -3 N / m. 39. (b)
cm -2
10 m
Energy ML2 T -2
18. (c) We know that Q = n1u1 = n2u2 are the two units of 40. (b) =
measurement of the quantity Q and n1, n2 are their
Volume L3
respective numerical values. From relation Q1= n1u1 = [ML–1T–2] = Pressure
= n2u2, nu = constant Þ n µ 1/u i.e., smaller the unit of q
measurement, greater is its numerical value. 41. (d) Angular velocity w = = [M0L0T–1]
t
19. (b) Change in length
20. (b) (A)®(3); (B)®(2); C®(4); (D)®(1) 42. (a) Strain = Original length
21. (c) Unit of energy will be kg-m2/sec2
Hence no dimension.
PV J
22. (a) R = = = J K -1 mol -1 . 43. (d) Both energy and work have same unit.
nT mol K \ energy/work is a pure number.
23. (b) SI is based on seven fundamental units.
44. (c) (A)®(3); (B)®(1); C®(4); (D)®(2)
24. (a) 1 newton = 105 dyne
25. (d) Q ML2 T -2
45. (d) s= = = [L2 T -2 K -1 ]
N 10 5 dynes mq MK
26. (b) = = 10 dynes / cm 2 = 10 b 46. (b) [Torque] = [Force] [distance]
2 4 2
m 10 cm
a = MLT–2. L = ML2T–2
27. (b) As 2 = P
V dyne 47. (a) Dimension of magnetic flux
\ a = PV 2 = (cm 3 ) 2 = dyne ´ cm 4
cm 2 = Dimension of voltage × Dimension of time
28. (b) Trigonometric ratio are a number and hence = [ML2T–3A–1] [T] = [ML2T–2A–1]
dimensionless Work
29. (c) [x] = [bt2]. Hence [b] = [x/t2] = km/s2 Q Voltage =
Charge
q2 48. (c) We know that F = q v B
30. (d) e o = 2
(r )4 pF F MLT -2
Þ unit of eo is (coulomb)2/ newton-metre2 \ B= = = MT -1C -1
qv C ´ LT -1
1 2 a
49. (a) Energy stored in an inductor, U = LI 64. (b) Dimensionally v = Þ a = LT -1 ´ T = L
2 t
[ML2 T -2 ] 3 LT -1 -4
Þ L=
2U
\ [L] = = [ML2 T -2 A -2 ] also dimensionally v = bt Þ b = 3 = LT
[A]2 T
I2 65. (c) 66. (b)
Energy Q2
50. (b) Power = 67. (c) We know that is energy of capacitor, so it represent
time 2C
51. (d) the dimension of energy = [ML2 T -2 ] .
drift velocity Vd (ms -1 ) m 2 s -3
Bx Bx B
52. (b) Mobility m = = = 68. (b) From K = 2 2
= 2 =
electric field E (Vm -1 ) V x +A x x
\ B = K × x = K.E. × distance = work × distance.
æ joule(J) ö
çèQ Volt = V = coulomb(C) ÷ø MLT –2
69. (d) Pressure = = [ ML–1T –2 ]
L2
m 2 s -1C m 2 s -1 As Þ a = 1, b = – 1, c = – 2.
= = [Coulomb,c = As]
J kg m 2 s -2 70. (a) Let m = KFa Lb Tc
= kg–1 s2 A = M–1 T2 A Substituting the dimensions of
53. (a) From stokes law [F] = [MLT–2], [L] = [L] and [T] = [T] and comparing
F both side, we get m = FL–1T2
F = 6phrv Þ h = GMm
6 prv F=
71. (a)
MLT-2 R2
\h = Þ h = [ML-1T -1 ]
[L][LT-1 ] FR 2
\ G= Þ G = [M–1L3T–2]
54. (d) Mass × acceleration (ma) = F (force) Mm
72. (c) [at] = [F] amd [bt2] = [F]
2pIw
Planck's constant nh Þ [a] = MLT–3 and [b] = MLT–4
55. (b) = n [As = Iw ] 73. (b) Here, b and x2 = L2 have same dime nsions
Moment of inertia I 2p
x2 L2
2pI(2pf ) æç 4p 2 ö÷ Also, a = = = M -1T 1
=
nI
=
ç n
.f = [T -1 ]
÷
E ´t (
M L2T -2 T )
è ø a×b= [M–1 L2T1]
56. (a)
57. (d) Energy incident per unit area per second 74. (b) 75. (b) 76. (d)
77. (d) Dimensionally e0L = C where C = capacitance
Energy ML2 T -2 Now the given expression reduces to CDV/Dt.
= = = MT–3
area×second L2 T Dimensionally CDV = q where q is charge.
58. (b) In S = A (t + B) + Ct2; B is added to time t. Therefore, Again the given expression reduces to q/Dt.
dimensions of B are those of time. q
Dimensionally = I where I is current
59. (d) Let us write the dimension of various quantities on Dt
two sides of the given relation. 78. (a) Use principle of homogeneity.
L.H.S. = T = [T] 79. (d) Dimension of A ¹ dimension of (C)
Hence A – C is not possible.
LT –2 80. (c) Let µ0 related with e, m, c and h as follows.
R.H.S. = 2p g / l = = [T –1 ]
L m0 = keambcchd
[\ 2p has no dimension]. As dimensions of L.H.S is [MLT–2A–2] = [AT]a [M]b [LT–1]c [ML2T–1]d
= [M b + d Lc + 2d Ta – c – d Aa]
not equal to dimensions of R.H.S. Therefore
according to principle of homogeneity the relation On comparing both sides we get
a=–2 ...(i)
T = 2p g / l is not valid. b+d=1 ...(ii)
60. (a) c + 2d = 1 ...(iii)
61. (d) (A)®(2); B®(1); C®(3); (D)®(4) a – c – d = –2 ...(iv)
By equation (i), (ii), (iii) & (iv) we get,
h ML2 T -1
62. (c) = = ML2 T -3 A - 2 = Resistance (ohm) a = – 2, b = 0, c = – 1, d = 1
2 2
e (AT )
1 2 é h ù
2 -2
63. (a) Energy stored in an inductor = L i = [ ML T ] \ [m 0 ] = ê 2 ú
2 ë ce û
EBD_7418
444 PHYSICS

81. (a) Absolute error, Dan = an– amean value of 1division of main scale
82. (c) 107. (c) L.C. =
number of division on main scale
83. (d) Taking large no. of observations and their mean value,
we can eliminate random errors. 1 1
84. (d) =
mm = cm
N 10N
85. (a) Least count is the smallest value that can be 108. (a) 109. (b)
measured by the measuring instrument. DM
86. (d) 87. (b) 88. (d) 110. (a) Fractional error = as DM A = DM B
M
89. (d) If z = AB then
DM A DM B
Dz DA DB as M B > M A Þ M > M
= + A B
z A B 1
90. (c) 111. (b) The value of 1 division of main scale = = 0.1 cm
10
91. (c) Least count of a screw gauge 8 ´ 0.1
Pitch The value of 1 division of vernier scale = = 0.08 cm
10
= Thus L.C. = 0.1 – 0.08 = 0.02 cm
Number of circular scale divisions
1 mm 112. (b)
= = 0.02 mm 113. (a) The mean value of refractive index,
50
Therefore the pitch and no. of circular scale divisions 1.34 + 1.38 + 1.32 + 1.36
m= = 1.35 and
4
are 1mm and 50 respectively.
| (1.35 - 1.34) | + | (1.35 - 1.38) | + | (1.35 - 1.32) | + | (1.35 - 1.36) |
92. (d) Dm =
4
93. (a) L + B = 2.331 + 2.1 @ 4.4 cm = 0.02
Since minimum significant figure is 2. Dm 0.02
Thus ´ 100 = ´ 100 = 1.48
94. (a) 95. (a) m 1.35
96. (c) In multiplication or division the final result should 114. (a) X = Ma Lb T c ;
return as many significant figures as there are in the
original number with the least significant figures. DX æ a DM b DL c DT ö
´ 100 = ç + + ÷ ´ 100
4 3 X è M L T ø
97. (c) V= pr ;
3 = (a a + b b + c g )%
DV æ Dr ö a 3b2 DP Da Db
´ 100 = 3ç ÷ ´ 100 = 3 ´ 1 % = 3 % 115. (d) P = , × 100% = 3 × 100% + 2 ×
V è r ø cd P a b
98. (d) (A) ® (2); (B) ® (1); C ® (3); (D) ® (4) Dc Dd
100% + × 100% + × 100%.
99. (a) Number of significant figures in 23.023 = 5 c d
Number of significant figures in 0.0003 = 1 = 3 × 1% + 2 × 2% + 3% + 4% = 14%

Number of significant figures in 2.1 × 10–3 = 2 116. (c) l1 - l 2 = (3.323 ± 0.001) - (3.321 ± 0.001)

100. (b) The significant number in the potential, V = iR; should = (0.002 ± 0.002)cm
be the minimum of either i or R. So corresponding to 117. (c)
i = 3.23 A, we have only three significant numbers in 1
118. (b) Thickness of wall = (4.23 - 3.87) ± (0.01 + 0.01)
V = 35.02935 V. Thus the result is V = 35.0 V. 2
101. (b) 102. (c) 103. (c) = (0.18 ± 0.02)cm
104. (c) Maximum absolute error is Da + Db. Therefore the 9
119. (c) 10 VD = 9MD, 1VD = MD
absolute error 10
percentage error = ´ 100
actual value Vernier constant = 1 MD – 1 VD
M M æ 9ö 1 1 1
105. (d) Density, r = = 2 = ç1- ÷ MD = MD = ´ = 0.05 mm
V pr l è 10ø 10 10 2

Dr é DM 2 Dr Dl ù DQ 2DI DR Dt
\ ´ 100 = ê M + r + l ú ´ 100
120. (d) ´ 100 = ´ 100 + ´100 + ´100
r ë û Q I R t
= 2 ´ 2% +1% +1% = 6%.
é 0.003 0.005 0.06 ù
= ê 0.3 + 2 0.5 + 6 ú ´ 100 = 4 Pitch
ë û 121. (a) Least count =
no. of division in circular scale
0.5 0.05
106. (b) Least count = = 0.01 mm = = 2.5 ´ 10 -4
50 200
UNITS AND MEASUREMENTS 445

122. (c) Given, P = a1/2 b2 c2 d–4, The significant figure in given numbers 4.237 g and 2.5
Maximum relative error, cm3 are four and two respectivey so, density should
DP 1 Da Db Dc Dd be reported to two significant figures.
= +2 +3 +4
P 2 a b c d 4.237 g
1 Density = 3
= 1.6948 = 1.7 gcm-3
= ´ 2 + 2 ´ 1 + 3 ´ 3 + 4 ´ 5 = 32% 2.5 cm
2 \ As rounding off the number upto 2 significant
DP DF Dl
123. (c) ´ 100 = ´ 100 + 2 ´ 100 = 6% + 2 × 3% figures, we get density = 1.7.
P F l 4. (d) Rounding off 2.745 upto 3 significant figures here IVth
124. (b) digit is 5 and its preceding is even, so no change in 4.
125. (a) From Ohm’s law,
Thus answer would be 2.74. Rounding off 2.735 upto 3
V
Þ R = Þ ln R = ln V – ln I significant figures, here IV digit is 5 and its preceding
I digit is 3 (odd). So 3 is increased by 1 answer become
DR DV DI
Þ = + = 3% + 3% = 6% would be 2.74.
R V I 5. (a) If Dx is error in a physical quantity, then relative error
126. (a)
127. (a) Since percentage increase in length = 2 % Dx
is calculated as .
Hence, percentage increase in area of square sheet x
= 2 × 2% = 4% Given that,
Dx æ Dx 2 Dy 1 Dz ö Length l = (16.2 ± 0.1) cm
128. (b) ´ 100 = ç + + ´ 100
x è x y 3 z ÷ø Breadth b = (10.1 ± 0.1) cm
Q Dl = 0.1 cm, Db = 0.1 cm
1
= 2 + 2 ´ 1 + ´ 3 = 5% Area (A) = l × b = 16.2 × 10.1 = 163.62 cm2
3 In significant figure rounding off to three significant
0.1 digits, area A = 164 cm2
129. (b) Least count = = 0.01 cm
10 DA Dl Db 1.01 0.1 1.620.1 2.63
= + == + =
d1 = 0.5 + 8 × 0.01 + 0.03 = 0.61 cm A l b 16.2 16.2 10.1
10.1 163.62
d2 = 0.5 + 4 × 0.01 + 0.03 = 0.57 cm 2.63 2.63
d3 = 0.5 + 6 × 0.01 + 0.03 = 0.59 cm So, DA = A ´ = 164 ´ = 2.636 cm2
163.62 163.62
0.61 + 0.57 + 0.59 Now rounding off up to one significant figure DA = 3 cm2.
Mean diameter =
3 So, Area A = A ± DA = (164 ± 3) cm2.
= 0.59 cm 6. (c) Tension = force = [MLT–2]
130. (d) As we know, time period of a simple pendulum force [MLT -2 ]
Surface tension = =
L 4p2 L length [L]
T = 2p Þg= 2
g T = [ML0T–2]
The maximum percentage error in g 7. (a) By applying the Rule of significant figure in multiplication
Dg DL æ DT ö and addition.
´ 100 = ´ 100 + 2 ç ´ 100 ÷ As given that,
g L è T ø
A = 2.5 ms–1 ± 0.5 ms–1, B = 0.10 s ± 0.01 s
= 2% + 2(3%) = 8%
x = AB = (2.5)(0.10) = 0.25 m (consider only significant
Exercise - 2 figure value)
Dx DA DB
1. (b) In 0.06900, the two zeroes before six are not siginificant = +
and two zeroes on right side of 9 are significant figures. x A B
Hence, number of significant figures are four (6900). Dx 0.5 0.01 0.05 + 0.025 0.075
= + = =
2. (b) In addition the result will be in least number of places x 2.5 0.10 0.25 0.25
after decimal and minimum number of significant figure. Dx = 0.075 = 0.08 m
The sum of the given numbers can be calculated as (rounding off to two significant figures.)
663.821. The number with least decimal places is 227.2 AB = (0.25 ± 0.08) m
is correct to only one decimal place but in addition of 8. (d) As given that,
numbers, the final result should be rounded off to one A = 1.0 m ± 0.2 m, B = 2.0 m ± 0.2 m
decimal place i.e., 664.
3. (c) As we know that in multiplication or division, the final So, X = AB = (1.0)(2.0) = 1.414 m
result should retain as many significant figures as are Rounding off upto two significant digit
there in the original number with the least significant X = 1.4 m = (r) (Let)
figures.
EBD_7418
446 PHYSICS

Dx 1 é DA DB ù 1 é 0.2 0.2 ù 13. (d) Force = mass × acceleration


= + = +
x 2 êë A B úû 2 êë 1.0 2.0 úû Þ [Mass]
0.6 x 0.6 ´ 1.4 é force ù
Þ Dx = = = 0.212 =ê
2 ´ 2.0 2 ´ 2.0 ë acceleration úû
Rounding off upto one significant digit é force ù –1
= ê ú = [F V T]
Dx = 0.2 m = Dr (Let) ë velocity / time û
So, correct value of
æ1ö
14. (c) Here ç ÷e 0 E 2 represents energy per unit volume
AB = r + Dr = (1.4 ± 0.2) m è2ø
9. (a) For the most precise measurement, the unit must be
é 2 -2 ù
least and number of digits including zeroes after Energy ë ML T û
decimal must be zero. [ 0 ] ë û volume
e é E 2ù
= = = ML–1T–2
é L3 ù
Now, take first option, ë û
As here 5.00 mm has the smallest unit and the error in 15. (c) 16. (b) 17. (d)
5.00 mm is least (commonly taken as 0.01 mm if not 18. (b) Let surface tension
specified), hence, 5.00 mm is most precise. s = Ea Vb Tc
10. (a) Now, checking the errors with each options one by b
one, MLT –2 æ Lö
= (ML2 T –2 )a ç ÷ (T)C
|Dl1| = |5 – 4.9| = 0.1 cm L è Tø
|Dl2| = |5 – 4.805| = 0.195 cm Equating the dimension of LHS and RHS
|Dl3| = |5.25 – 5| = 0.25 cm ML0T–2 = MaL2a + b T–2a – b + c
|Dl4| = |5.4 – 5| = 0.4 cm Þ a = 1, 2a + b = 0, –2a – b + c = –2
Error Dl1 is least or minimum. Þ a = 1, b = – 2, c = – 2
So, 4.9 cm is most precise. Hence, the dimensions of surface tension are [E V–2 T–2]
11. (c) It is given that Young's modulus (Y) is, 19. (d) Applying dimensional method :
Y = 1.9 × 1011 N/m2
vc = hxryrz
1N = 105 dyne
[M0LT–1] = [ML–1T–1]x [ML–3T0]y [M0LT0]z
So, Y = 1.9 × 1011 × 105 dyne/m2
Convert meter to centimeter Equating powers both sides
Q 1m = 100 cm x + y = 0; –x = –1 \ x = 1
Y = 1.9 × 1011 × 105 dyne/(100)2 cm2 1 + y = 0 \ y = –1
= 1.9 × 1016 – 4 dyne/cm2 –x – 3y + z = 1
Y = 1.9 × 1012 dyne/cm2 –1 – 3(–1) + z = 1
12. (d) Given that fundamental quantities are momentum (p), –1 + 3 + z = 1
area (A) and time (T). \ z = –1
Let us consider the dimensional formula for
0.2
E µ [ p a AbT c ] 20. (c) ´100 = 0.8
25
E = kp a AbT c ] 2 l
where k is dimensionless constant of proportionality. 21. (d) As, g = 4 p
T2
Dimensions of energy [E] = [ML2T–2] and Dimension
Dg Dl DT
of momentum p = mv = [MLT–1] So, ´ 100 = ´ 100 + 2 ´ 100
Dimension of Area [A] = [L2] g l T
Dimension of Time [T] = [T] 0.1 1
= ´ 100 + 2 ´ ´ 100 = 2.72 ; 3%
Dimension of energy [E] = [K] [p]a[A]b[T]c 20 90
Putting all the dimensions, value eV W PV
ML2T–2 = [MLT–1]a [L2]b [T]c 22. (c) = = =R
= MaL2b + aT–a + c
T T T
By principle of homogeneity of dimensions, R
and = Boltzmann constant.
1 N
a = 1, 2b +a = 2 Þ 2b + 1 = 2 Þ b =
2 23. (d) Let unit ‘u’ related with e, a0, h and c as follows :
- a + c = -2 [u] = [e]a [a0]b [h]c [C]d
c = –2 + a = –2 + 1 = –1
So, Dimensional formula (of energy) Using dimensional method,
E = [pA1/2T–1] [M–1L–2T+4A+2]
UNITS AND MEASUREMENTS 447

= [A1T1]a[L]b[ML2T–1]c[LT–1]d DT Dr Dh
[M–1L–2T+4A+2] = [Mc Lb+2c+d Ta–c–d Aa] Relative error in surface tension, = + +0
T r h
a = 2, b = 1, c = – 1, d = – 1 (Q g, 2 and 10 are constant)
3

Percentage error
e 2 a0
\u=
hc DT æ 10 –2 ´ 0.01 10 –2 ´ 0.01ö
100 ´ =ç + ÷ 100
24. (b) Significant figure refers to the accuracy of measurement T è 1.25 ´ 10 –2 1.45 ´ 10 –2 ø
and accuracy of measurment also depends upon the = (0.8 + 0.689) = (1.489) = 1.489% @ 1.5%
least count of measuring instrument.
dimensions of X
0.5 32. (c) Dimensions of Y =
25. (d) L.C. = = 0.01 mm dimensions of Z2
50
Zero error = 5 × 0.01 = 0.05 mm (Negative) M -1 L-2T 4 A2
=
Reading = (0.5 + 25 × 0.01) + 0.05 = 0.80 mm ( MT -2 A-1 )2
| DT1 | + | DT2 | + | DT3 | + | DT4 | 2 + 1 + 3 + 0 = [M–3 L–2 T8 A4]
26. (c) DT = = = 1.5
4 4 33. (c) Diameter of the ball
As the resolution of measuring clock is 1.5 therefore
= MSR + CSR × (least count) – zero error
the mean time should be 92 ± 1.5
= 0.5 cm + 25 × 0.001 – (–0.004)
1/ 2
p éFù p2F = 0.5 + 0.025 + 0.004 = 0.529 cm
27. (c) u= Þ m =
2l êë m úû 4l 2 u 2 34. (b) v = k l a rb g c
Now, dimensional formula of R.H.S.
[M 0 LT-1 ] = La (ML-3 ) b (LT -2 ) c
MLT -2 = M b La - 3b + c T -2c
= 2
= ML–1T0.
æ1ö \ b = 0; a - 3b + c = 1
L2 ç ÷
èTø 1
-2c = -1 Þ c = 1 / 2 \ a=
So, dimensions of m will be ML–1T0. 2
28. (d) Let dimensions of length is related as, v µ l1 / 2 r0 g1 / 2 or v 2 µ l g
z
x y
é e2 ù Mass (M) M
L = [c] [G] ê ú 35. (c) Density (d) = =
êë 4pe 0 úû Volume (V) L3
L = [LT–1]x [M–1L3T–2]y[ML3T–2]z Dd DM 3DL
\ Error in density, = + = 1.5% + 3(1%) = 4.5%
[L] = [Lx + 3y + 3z M –y + z T–x – 2y – 2z] d M L
2 2 -2
C LT
Comparing both sides 36. (d) = = [ L]
–y + z = 0 Þ y = z ...(i) g LT -2
x + 3y + 3z = 1 ...(ii) Exercise - 3
–x – 4z = 0 (Q y = z) ...(iii)
From (i), (ii) & (iii) æ ö Di
1. (c) We have e = L çè Dt ÷ø , L ® inductance
1
z = y = , x = –2
2 ML2T -3A -1 ML2 ML2
e
1/2 \ L= = (A / T) = =
é e2 ù æ Di ö (AT) 2 Q2
-2
Hence, L = c êG × ú çè ÷ø
Dt
ëê 4pe 0 ûú 1.53 ´ 0.9995 1.529235
2. (a) = = 0.9605747 = 0.961
F F 1.592 1.592
29. (d) As, pressure P = = 2 (Rounding off to three significant digits)
A L
3. (c) Volume flow rate µ PxRylzrahb
DF DL
% Error = ´100 + 2 ´100
F L L3
= [M x L- x T -2x ] [Ly ] [Lz ] [Ma L-3a ] [Mb L-b T -b ]
= 4 + 2 × 2 = 8% T
30. (a) x + a + b = 0 and 2x + b = 1
–x + y + z – 3a – b = 3
rhg
31. (d) Surface tension, T = ´ 103 Solving above equations with the help of options
2
x = 1, y = 4, z = – 1, a = 0, b = –1
EBD_7418
448 PHYSICS

k q.t 3 0.5 0.2


4. (b) = dimensionless = ´ 100% + ´ 100% = 16.25%
ma 8 2
R = 4W ± 16.25%
ML2T -2 K -1.K .T 3 13. (a) Given:
= M 0 L0 T 0
M .a Q = x2/5y–1t–1/2 z3
a = L2T DQ 2 Dx Dy
\ ´ 100 = ´ 100 + ´ 100
Dimension of P = Dimension of b Q 5 x y
a
[b] 1 Dt Dz
Þ ML–1T–2 = Þ [b] = MLT–1 = [Linear momentum] + ´ 100 + 3 ´ 100
L2T 2 t z
5. (c) 2 1
= ´ 2.5 + 2 + ´ 1 + 3 ´ 0.5 = 5%
6. (c) Use the concept that the expression (a/pb ) and 5 2
(az/KBq) are dimensionless. 14. (a) f µ makb
T–1 = MaMbT–2b
é a ù é az ù éa Kb q ù
Þê ú = ê ú Þ [b ] = ê ú a + b = 0 ; – 2b = – 1 Þ b = 1/2, a = – 1/2
ë pb û êë Kbq úû ë paz û
k 0.50 85 1 72
ML2T -2q-1.q f µ ; = Þ f = = 0.46 s–1
= = L 2 M 0T 0 m f 72 2 85
ML-1T -2 .L 15. (b) According to question
7. (c)
Ey µ JxBZ
8. (b) In new system, 1g* = 10g, 1cm* = 5cm.
\ Constant of proportionality
I = 6 × 100 × g* (20 cm*)2 = 6 × 100 × 400 g× cm*2
= 2.4 × 105 g* cm*2 Ey C m3
K= = =
BZ J x J x As
ATML2T -1
9. (c) mn = = L2 A 16. (c) Least count of measuring device = 0.1
M Using F = ma ; m = (1 ± 0.1) kg
I 17. (c) From Kepler's law, time period of a satellite,
10. (a) I = I 0 sin 2 q Þ q = sin -1
I0
r3 4p 2 3
1 I0 dI T = 2p T2 = r
dq = Gm GM
2 I0 - I I0 I Relative uncertainty in the mass of the earth
dq 1 dI DM DT
=
q 2 I ( I0 - I ) sin -1 I / I 0 =2 = 2 ´ 10-2 (Q 4p & G constant and
M T
4
\ % error = 3 ´ 10 -2 % Dr
p relative uncertainty in radius
negligible)
r
p T 18. (d) Maximum percentage error in A
11. (a) Given f = . Taking log of both sides
2l m = 3(% error in P) + 2(% error in Q)
æ pö 1 1
log f = log ç ÷ - log l + log T - log m 1
è 2ø 2 2 + (% error in R) + 1(% error in S)
2
Differentiating partially on both sides,
df dl 1 dT 1 dm 1
=0- + - = 3 ´ 0.5 + 2 ´ 1 + ´ 3 + 1 ´ 1.5
f l 2 T 2 m 2
= 1.5 + 2 + 1.5 + 1.5 = 6.5%
df æ dl ö æ 1 dT ö æ 1 dm ö
or ´ 100 = ç - ´ 100÷ + ç ´ 100÷ - ç ´ 100÷ 19. (a) L.C of screw guage
f è l ø è2 T ø è2 m ø
= Pitch (p)/Number of divisions on circular scale (n)
1 1 20. (b) Plank length is a unit of length, lp = 1.616229 × 10–35 m
= ( -1) + ( -2) - (4) = –1 – 1 – 2 = – 4%
2 2
8 hG
12. (a) Actual R = = 4 lp =
2 c3
% error in R = % error in V + % error in I
3. MOTION IN A STRAIGHT LINE
19. (d) In (a), at the same time particle has two positions which
Exercise - 1 is not possible. In (b), particle has two velocities at
1. (c) If a body is moving along a straight line path with the same time. In (c), speed is negative which is not
constant velocity then distance travelled = displacement possible.
i.e., D = S 20. (a) Instantaneous speed is the distance being covered
2. (a) When location of a particle has changed, it must have by the particle per unit time at the given instant. It is
covered some distance and undergone some equal to the magnitude of the instantaneous velocity
displacement. at the given instant.
3. (c) 21. (c) 22. (c) 23. (a)
| displacement | v1t / 3 + v 2 ( 2t / 3) v1 2v 2
4. (b) | Average velocity | = 24. (a) vav = = +
time t 3 3
2r 1 displacement
= = 2 ´ = 2 m/s. 25. (c) Average velocity =
t 1 time interval
Displacement A particle moving in a given direction with non-zero
5. (d) £1 velocity cannot have zero speed.
distance
6. (b) In general, average speed is not equal to magnitude of
7. (d) Distance covered by a particle is zero only when it is average velocity. However, it can be so if the motion
at rest. Therefore, its displacement must be zero. is along a straight line without change in direction.
8. (d) Motion of a body along a straight line is one dimensional 26. (c) 27. (c) 28. (c)
motion. 29. (a) Since displacement is zero.
9. ( b) 30. (d) Let the total distance be d. Then for first half distance,
10. (c) As x – t graph is a straight line in either case, velocity d
of both is uniform. As the slope of x – t graph for P is time = , next distance. = v1t and last half distance
2v 0
greater, therefore, velocity of P is greater than that of Q. = v2t
s d
11. (d) When s µ t, so = constant . d t=
t \ v1t + v2 t = ;
2 2(v1 + v2 )
12. (c) The velocity-time graph for a uniform motion is a
straight. line parallel to time axis. Its slope is zero. Now average speed
13. (b) d
t=
|Average velocity| |displacement| d d d
= + +
|Average speed| | distance | 2 v 0 2( v1 + v 2 ) 2( v1 + v 2 )
because displacement will either be equal or less
than distance. It can never be greater than distance 2v0 (v1 + v2 )
=
travelled. (v1 + v 2 ) + 2v0
14. (c) The slope of the tangent drawn on position-time graph 31. (d) The nature of the path is decided by the direction
at any instant gives instantaneous velocity. of velocity, and the direction of acceleration. The
S trajectory can be a straight line, circle or a parabola
depending on these factors.
A
AB 32. (b) For a particle moving with uniform acceleration the
Position C B AC = = Vinst displacement-time graph is a parabola.
BC
O Time S
15. (d) vA = tan 30º and v B = tan 60º

vA tan 30º 1/ 3 1
\ = = =
vB tan 60º 3 3 t
16. (a) 33. (c) Slope of velocity-time graph shows acceleration.
34. (c) Because acceleration is a vector quantity.
17. (b) The speed in general ³ the magnitude of velocity 35. (d)
18. (a) Total time of motion is 3 min 20 sec = 20 sec. 36. (a) When the body is projected vertically upward then
As time period of circular motion is 40 sec so in 20 sec at the highest point its velocity is zero but
athlete will complete 5 revolution i.e., he will be acceleration is not equal to zero (g = 9.8m/s2).
at starting point i.e., displacemet = zero. 37. (b) 38. (d)
EBD_7418
450 PHYSICS

39. (d) Velocity time curve will be 52. (a) Velocity at time t is tan 45° = 1. Velocity at time (t = 1)
a straight line as shown: v
is tan 60° = 3 . Acceleration is change in velocity in
40. (b) At the highest point v = 0.
41. (c) one second = 3 - 1 .
42. (a) Let s be the distance travelled o t 53. (c) Let a be the constant acceleration of the particle. Then
by the vehicle before it stops. 1 1
s = u t + a t 2 or s1 = 0 + ´ a ´ (10 ) 2 = 50 a
Final velocity u = 0, initial velocity = u 2 2
Using equation of motion v2 – u2 = 2aS é 1 2ù
02 – u2 = 2aS and s 2 = ê0 + a(20) ú - 50a = 150a
ë 2 û
u2
Stopping distance, S = – \ s 2 = 3s1
2a 54. (c) Distance travelled in the nth second is given by
43. (c) 44. (c)
a
u2 (20)2 t n = u + (2n - 1)
45. (a) v2 – u2 = 2as Þ a = = = 1 m/s2 2
2as 2 ´ 200 4
46. (a) In uniform circular motion, there is acceleration of put u = 0, a = ms -2 , n = 3
constant magnitude. 3
a 4 4 10
47. (c) Sn = u + (2n – 1) \ d=0+ (2 × 3 –1) = ´ 5 = m
2 3´ 2 6 3
55. (c) Initial velocity of car (u) = 0
a 2
or, S = (2 ´ 2 – 1) Þ a = m/s2 Final velocity of car (v) = 144 km/hr = 40 m/s
2 3 Time taken = 20 s
1 We know that, v = u + at
48. (c) During OA, acceleration = tan 30º = m/s2 40 = a × 20 Þ a = 2 m/s2
3
During AB, acceleration = –tan60º = - 3 m/s2 v2 – u 2
Also, v2 – u2 = 2as Þ s =
1/ 3 1 2a
Required ratio = =
(40) 2 – (0) 2 1600
3 3 Þ s= = = 400 m.
49. (b) For first part of penetration, by equation of motion 2´ 2 4
2
3u 2 56. (b) Differentiated twice.
æuö 2
ç ÷ – (u) = 2aS or a = – ...(i) 57. (c) On differentiating, acceleration = 0.2t Þ a = f ( t )
è2ø 8S
For latter part of penetration 58. (d) v
2
æuö u2 vmax A
(0) 2 – ç ÷ = 2aS¢, S¢= -
è2ø 8a
2
u æ 8S ö a b
S¢ = – ç
8 è –3u 2 ÷ø
(Using (i))
S 40 A1 B
S¢ = or S¢ = cm t
3 3 O t1 t2 t0
50. (c) The distance covered in n th second is
1 In fig., AA1 = vmax. = at1 = bt2
S n = u + ( 2 n - 1)a
2 v max v max
where u is initial velocity & a is acceleration But t = t1 + t 2 = +
a b
19 a
then 26 = u + ....(i) æ 1 1ö æa+bö
2 = v max çç + ÷÷ = v max çç ÷÷
è a b ø è ab ø
21a
28 = u + ....(ii)
2 æ ab ö
or, v max = t çç ÷÷
23 a èa +bø
30 = u + ....(iii)
2 59. (d) 60. (c)
61. (d) From displacement-time graph, it is clear that in equal
25 a intervals of time displacements are not equal infact,
32 = u + ....(iv)
2 decreases and after 40s displacement constant i.e. the
From eqs. (i) and (ii) we get u = 7m/sec, a=2m/sec2 particle stops.
\ The body starts with initial velocity u =7m/sec 62. (d) Equation of motion can be applied if the acceleration
and moves with uniform acceleration a = 2m/sec2 is in opposite direction to that of velocity and
uniform motion mean the acceleration is zero.
51. (a) v = 3x + 16 Þ v 2 = 3x + 16 Þ v 2 - 16 = 3x 63. (c) When particle comes to rest,
Comparing with v 2 - u 2 = 2aS, we get, u = 4 units, 2a dx d
= 3 or a = 1.5 units V=0= = (40 + 12t – t3)
dt dt
MOTION IN A STRAIGHT LINE 451

Þ 12 – 3t2 = 0
1 2 1
12 S1 = at = ´ 6 ´ 52 = 75 m
Þ t2 = = 4 \ t = 2 sec 2 2
3
Distance travelled in first 5 sec is 75m.
Therefore distance travelled by particle before coming
Distance travelled with uniform speed of 30 m/s is S2
to rest, 395 = S1 + S2 + S3 Þ 395 = 75 + S2 + 45 Þ S2 = 275 m
x = 40 + 12t – t3 = 40 + 12 × 2 – (2)3 = 56m
1 dx = -1 275
Time taken to travel 275 m = = 9.2 sec
64. (a) x = \ v= 30
t+5 dt (t + 5) 2
For retarding motion, we have
d2x 2 02 – 302 = 2 (– a) × 45, we get a = 10 m/s2
\ a= 2 = = 2x3
dt (t + 5)3 1 2 1 2
1 3 S = ut + at Þ 45 = 30t + (–10)t Þ 45 = 30t – 5t2
1 1 2 2
Now µv \2 µ v2 µ a
(t + 5) (t + 5)3 On solving we get, t = 3 sec
Total time taken = 5 + 9.2 + 3 = 17.2 sec.
change in velocity Dv 1 2
65. (c) Average acceleration = = 71. (d) Distance from A to B = S = ft 1
time interval t 2
Distance from B to C = (ft 1 ) t
uur uur v2
ˆ v = 5jˆ
v1 = 5i, 2 N u2 (ft ) 2
Distance from C to D = = 1
D v = (v 2 - v 1 ) D v = v 2 + (- v1 ) 2a 2( f / 2)
= ft 12 = 2S
= v12 + v 22 + 2 v1v 2 cos 90 C f /2 D
90°
E
A f B
W
- v1 v1 t
= 5 2 + 52 + 0 t1 2t 1
[As | v1 | = | v 2 | = 5 m/s] S 15 S
= 5 2 m/s Þ S + f t1t + 2S = 15 S

5 2 1 Þ f t1t = 12 S ............. (i)


Dv 5
Avg. acc. = = = m / s 2 Þ tan q = = -1 1 2
t 10 2 -5 f t1 = S ............ (ii)
which means q is in the second quadrant. (towards north- 2
t
west) Dividing (i) by (ii), we get t1 =
66. (a) Differentiate two times and put x = 0. 6
2
dv 1 ætö f t2
dv
= -2.5 v Þ Þ S= fç ÷ =
67. (a) = – 2.5 dt 2 è 6ø 72
dt v
Integrating, 72. (d) Let u be the initial velocity that have to find and a be
0
the uniform acceleration of the particle.
0 t é v +½ ù For t = 3s, distance travelled S = 12 m and
= -2.5 [ t ]0
t
ò v -½ dv = -2.5ò dt Þ ê ú
êë (½) úû 6.25
for t = 3 + 3 = 6 s distance travelled S¢ = 12 + 30 = 42 m
6.25 0
From, S = ut + 1/2 at2
Þ – 2(6.25)½ = – 2.5t Þ t = 2 sec 1
1 12 = u × 3 + ´ a ´ 32
68. (a) x = x0 + (ut + at 2 ) At t = 0, x = – 2, \ – 2 = x0 + 0 2
2
or x0 = – 2 or 24 = 6u + 9a ...(i)
1 1
Thus, 0 = -2 + (u ´ 1 + ´ a ´ 12 )...(i) Similarly, 42 = u × 6 + ´ a ´ 62
2 2
1 2 or 42 = 6u + 18a ...(ii)
and 6 = -2 + (u ´ 2 + ´ a ´ 2 ) ...(ii) On solving, we get u = 1 m s–1
2
After solving equations, we get u = 0, a = 4 m/s2.
total length X+X X
Now for t = 3, 73. (a) Time = = = s = 1s
1
relative velocity 20 + 20 40
x = -2 + (u ´ 3 + ´ 4 ´ 32 ) = 16 m. 74. (d) All options are correct :
2
Clearly it represents motion with constant acceleration. (i) When two bodies A & B move in opposite
69. (d) directions then relative velocity between A & B either
70. (d) Given : u = 0, t = 5 sec, v = 108 km/hr = 30m/s VAB or VBA both are greater than VA & VB.
By eqn of motion v = u + at (ii) When two bodies A & B move in same direction
v 30 then VAB = VA - VB Þ VAB < VA
or a= = = 6 m / s 2 [Q u = 0] VBA = VB - VA Þ VBA < VB
t 5
EBD_7418
452 PHYSICS

75. (d) 84. (d) Speed to cover 1200 m by scootarist


76. (c) Velocity of A is same as that of B in magnitude but vr × 60 = 1200 Þ vr = 20
opposite in direction. speed to overtake bus
77. (d) So by figure the velocity of parrot v = vr + 10 = 30 m/s
w.r. t. train is = 5–(–10) = 15m/sec 85. (b) Let the initial velocity of ball be u \ Time of rise
so time taken to cross the train is u2
u
length of train t1 = and height reached =
=
150 g +a 2( g + a)
= = 10 sec
relative velocity 15 Time of fall t2 is given by
1 u2
North ( g – a )t22 =
2 2( g + a )
train
u u g+a
10m/sec t2 = =
West East ( g + a)( g - a) ( g + a) g –a
parrot
5m/sec 1 1
\ t2 > t1 because <
g +a g -a
South
86. (d)
8+8 87. (d) Because acceleration due to gravity is constant so
78. (a) Velocity of boat = = 8 km h -1 the slope of line will be constant i.e. velocity time
2 curve for a body projected vertically upwards is
Velocity of water = 4 km h -1 straight line.
88. (b)
8 8 8 89. (b) Free fall of an object (in vacuum) is a case of motion
t= + = h = 160 minute
8-4 8+ 4 3 with uniform acceleration.
79. (b) The relative vel. of rain w.r.t. car is inclined to the 90. (d) As ball returns to starting point so displacement is
vertical in the backward direction. Therefore, it will zero.
strike the front screen. 91. (a) We should know the displacement in this time.
r
80. (a) In a, b, c and d time taken are respectively r r at 2
S = ut + (we take upward as positive)
2d 2d d d 2du 2du 2
, , - = 2 , 2
v 2 2 u + v u - v u - v u - v2
2 4
v -u S = 30 ´ 4 - 10 ´ 4 ´ = 40 m.
[u = stream speed] 2
The average velocity will be 10 m/sec.
81. (d) By definition of relative velocity
r r r r r r 2
v1 = v0 + v2 Þ v0 + v2 + ( - v1 ) = 0 92. (d) At highest point v = 0 and Hmax= u
Þ v0, v1 and v2 will be sides of a triangle and we know that 2g
the sum of any two sides is greater than third side of the 93. (c) (A)®( 2); (B)®(1); C®(3); (D)®(4)
triangle. 94. (d)
82. (b) Relative speed of each train with respect to each other 1 2
95. (a) h = ut1 - gt1
be, n = 10 + 15 = 25 m/s 2
Here distance covered by each train = sum of their 1
Also h = ut2 - gt22
lengths = 50 + 50 = 100 m 2
After simplify above equations, we get
100
\ Required time = = 4 sec . 1
h = gt1t2 .
25 2
ur $ 96. (c) If h is the height of the building, then
( )
(a) V A = 10 –i$
N(j)
83. v2A = v 2 + 2 gh and vB2 = ( - v ) 2 + 2 gh .
Clearly vA = vB.
ur
V B = 10 ( $j) 97. (b) Velocity when the engine is switched off
w 10 km/h A 100 km
ur 45° v = 19.6 ´ 5 = 98 m s -1
V BA = 10 $j + 10 iˆ 2
100 km O h max = h1 + h 2 where h1 = 1 at 2 & h 2 = v
= 10 2 km / h 2 2a
B 1 98 ´ 98
S h max = ´ 19.6 ´ 5 ´ 5 + = 735 m
2 2 ´ 9.8
Distance OB = 100 cos 45° = 50 2 km
98. (b) Height attained by balls in 2 sec is
Time taken to each the shortest distance between
1
OB 50 2 = ´ 9 . 8 ´ 4 = 19 . 6 m
A and B = uuuur = = 5h 2
VBA 10 2 the same distance will be covered in 2 second (for
descent)
MOTION IN A STRAIGHT LINE 453

Time interval of throwing balls, remaining same. So, 2S


t1 = A
for two balls remaining in air, the time of ascent or g
descent must be greater than 2 seconds. Hence speed
4S 4S 2S S
of balls must be greater than 19.6 m/sec. t1 + t 2 = , t2 = -
99. (a) Clearly distance moved by 1st ball in 18s = distance g g g B
moved by 2nd ball in 12s. 2S
6S S
Now, distance moved in 18 s by 1st ball t1 + t 2 + t 3 = 3S
1 g C
= × 10 × 182 = 90 × 18 = 1620 m
2 6S 4S
Distance moved in 12 s by 2nd ball S
t3 = -
1 g g
= ut + gt2 \ 1620 = 12 v + 5 × 144 D
2
t1 : t 2 : t 3 : : 1 : ( 2 - 1 ) : ( 3 - 2 )
Þ v = 135 – 60 = 75 ms –1 1 1
109. (a) h = ut + gt 2 Þ h = gt 2 [Qu = 0] \ t = 2h / g
1 2 1 2 2
100. (a) Q h= gt \ h1 = g(5)2 = 125 110. (c) 111. (b) 112. (b)
2 2 113. (d) Since S = ut + ½ gt2
1 2 where u is initial velocity & a is acceleration.
h1 + h2 = g(10) = 500 Þ h2 = 375
2 In this case u = 0 & a = g
1 so distance travelled in 4 sec is,
\ h1 + h2 + h3 = g(15)2 = 1125 S = ½ × 10 × 16 = 80m
2
1 2
Þ h3 = 625 114. (a) s = ut + at
2
h2 h3 –65 = 12t – 5t2 on solving we get, t = 5s
h2 = 3h1 , h3 = 5h1 or h1 = = 115. (d) For A to B
3 5
101. (a) 1 2
102. (c) S = gt ...(i)
2
2
For A to C
T 1 æTö h 1 2
In sec, the distance travelled = g ç ÷ = 2S = gt ¢ ...(ii)
3 2 è3ø 9 2
t 1
h 8h Dividing (i) by (ii) we get =
\ Position of the ball from the ground = h -= m t¢ 2
9 9 116. (d) Let the two balls Pand Q meet at height x m from the
103. (a) Before hitting the ground, the velocity n is given by ground after time t s from the start.
v 2 = 2gd We have to find distance, BC = (100 – x)
ædö Q C
Further, v '2 = 2g ´ ç ÷ = gd ;
è2ø (100 – x)m
ævö 100 m
\ ç ÷ = 2 or v = v ' 2 –1 B
è v'ø 25 m s
As the direction is reversed and speed is decreased xm
and hence graph (a) represents these conditions P
correctly. A
For ball P
2h S = x m, u = 25 m s–1, a = –g
104. (b) Time taken by the stone to reach the water level t 1 =
g 1
Time taken by sound to come to the mouth of the well, From S = ut + at 2
2
h 2h h 1 2
t2 = \ Total time t1 + t 2 = + x = 25t – gt .......... (i)
v g v 2
For ball Q
105. (c) 106. (b) S = (100 – x) m, u = 0, a = g
1 2 1 2
107. (a) S = ut + at \ 100 – x = 0 + gt .......... (ii)
2 2
1 2 52 Adding eqns. (i) and (ii), we get
-76 = 4 ´ 6 - ´ 10 ´ ( 6 ) Þ u = m/s 100 = 25t or t = 4 s
2 3
From eqn. (i),
1 2 1
108. (b) S = AB = g t 1 Þ 2S = AC = g (t1 + t 2 )2 1
2 2 x = 25 × 4 – ´ 9.8 ´ (4) 2 = 21.6 m
1 2
2
and 3S = AD = g ( t 1 + t 2 + t 3 ) Hence distance from the top of the tower
2 = (100 – x) m = (100 – 21.6 m) = 78.4 m
EBD_7418
454 PHYSICS

117. (d) For constant acceleration and zero initial velocity 100
and max. height is H2 then, H 2 = .
hµt 2 2g
It implies that H2 = 4H1
h1 t12 h2
125. (c) Given, H = 10 m, u = 5 m/s, g = 10 m/s2
= 2 Þ t2 = t1 = 2 ´ t1 = 2 ´ 2s
h2 t2 h1
118. (b) When a body falls through a height h, it acquires a
Speed on reaching ground v = u 2 + 2 gh
Now, v = u + at
velocity 2gh .
119. (c) u = 12m/s, g = 9.8 m/sec2, t = 10 sec Þ u 2 + 2 gh = -u + gt
1 2
Displacement = ut + gt u
2 Time taken to reach highest point is t = ,
1 g
= 12 ´ 10 + ´ 9.8 ´ 100 = 610m
120. (b)
2 u + u 2 + 2 gH nu
Þ t= = (from question)
121. (c) As we know, distance traversed in nth second g g
1
Sn = u + a (2 n - 1) Þ 2gH = n(n –2)u2
2
Here, u = 0, a = g 2gH 2 ´ 10 + 10
1 Þ n (n–2) = 2 = =8 Þn=4
\ Sn = g (2n - 1) u 5´ 5
2
Distance traversed in 1st second i.e., n = 1
1 1 Exercise - 2
S1 = g (2 ´ 1 - 1) = g
2 2
Distance traversed in 2nd second i.e., n = 2 1. (b) If we draw a line parallel to time axis from the point (A)
1 3 on graph at t = 0 sec. This line can intersect graph at B.
S2 = g (2 ´ 2 - 1) = g
2 2 In graph (b) for one value of displacement there are
Distance traversed in 3rd second i.e., n = 3 two different points of time. so, for one time, the
1 5
S3 = g (2 ´ 3 - 1) = g average velocity is positive and for other time is
2 2 equivalent negative. x
1 3 5
\ S1 : S2 : S3 = g : g : g = 1: 3 : 5 As there are opposite velocities
2 2 2
122. (b) Let the body fall through the height of tower in in the inteval 0 to T hence A B
a average velocity can vanish in
t seconds. From, D n = u + (2n – 1) we have, total (b). Thiscan be seen in the
2 t
distance travelled in last 2 second of fall is O
figure given below. T
D = D t + D(t -1) Here, OA = BT (same displacement) for two different
é g ù é g ù points of time.
= ê0 + (2t - 1) ú + ê0 + {2(t - 1) - 1} ú 2. (a) As the lift is moving downward directions so
ë 2 û ë 2 û displacement is negative (zero). We have to see
g g g whether the motion is accelerating or retarding.
= (2 t - 1) + (2 t - 3) = (4 t - 4)
2 2 2 Due to downward motion displacement is negative the
10 lift reaches 4th floor is about to stop hence, motion is
= ´ 4( t - 1)
2 retarding (–a) downward in nature hence, x < 0; a > 0.
or, 40 = 20 (t – 1) or t = 2 + 1 = 3s As displacement is
Distance travelled in t second is 8th floor
in negative direction,
1 1 x<0
s = ut + at 2 = 0 + ´10 ´ 32 = 45m x < 0 velocity will also be
2 2 6th floor
123. (b) The stone rises up till its vertical velocity is zero and negative i.e., v < 0 but net
again reached the top of the tower with a speed u 4th floor
acceleration is +ve a > 0, x<0 O
(downward). The speed of the stone at the base is 3u.
u Ground floor
that can be shown in the graph.
4u 2 – 3. (b) In one dimensional motion, for the maximum and
Hence (3u)2 = (-u)2 + 2gh or h = minimum displacement we must have the magnitude
g
+ and direction of maximum velocity.
124. (a) From third equation of motion v 2 = u 2 + 2ah As maximum velocity in positive direction is v0, hence
In first case initial velocity u1 = 5 m/sec v, g, h maximum velocity in opposite direction is also –v0.
final velocity v1 = 0, a = – g Maximum displacement in one direction = v0T
25 Maximum displacement in opposite directions = –v0T.
and max. height obtained is H1, then, H1 =
2g Hence, -v0T < x < v0T
In second case u2 = 10 m/sec, v2 = 0, a = –g
MOTION IN A STRAIGHT LINE 455

4. (c) Time taken to travel first half distance, 7. (b) Vi = 0, Vf = Vmax


l/2 l 11
t1 = = (Q L1 = l / 2) DV = area under the curve = 10 ´ = 55
v1 2v1 2
Time taken to travel second half distance, or Vf - Vi = 55 m / s since Vi = 0
l Vf = 55 m / s = Vmax
t2 = (Q L2 = l / 2)
2v2 8. (a) According to question,
V (x) = bx–2n
So, total time taken to travel full distance
= t1 + t2 dv
So, = – 2 nb x– 2n – 1
dx
l l l é 1 1 ù l év + v ù
= + = ê + ú = ê 2 1ú Acceleration of the particle as function of x,
2v1 2v2 2 ë v1 v2 û 2 ë v1v2 û

l 2v1v2
a=v
dv
dx
{
= bx–2n b (–2 n) x
–2n –1
}= – 2nb2x–4n–1
Þ vav. = = 9. (c) In uniform motion the object moves with uniform velocity,
l é1 1 ù v1 + v2
ê + ú the magnitude of its velocity at different instance i.e., at
2 ë v1 v2 û t = 0, t =1, sec, t = 2sec ..... will always be constant. Thus
5. (b) As given that, x = (t – 2)2 velocity-time graph for an object in uniform motion along
a straight path is a straight line parallel to time axis.
dx d 10. (b) y1 = 10t – 5t2 ; y2 = 40t – 5t2
velocity v = = (t - 2) 2 = 2 (t – 2) m/s
dt dt for y1 = – 240m, t = 8s
\ y2 – y1 = 30t for t < 8s.
dv d
a= = [2(t - 2)] = 2 [1 – 0] = 2 m/s2 = 2 ms–2 1
dt dt for t > 8s, y2 – y1 = 240 – 40t – gt2
2
at t = 0; v0 = 2 (0 – 2) = –4 m/s
t = 2 s; v2 = 2 (2 – 2) = 0 m/s 11. (c) Let the body is projected vertically upwards from A
t = 4 s; v4 = 2 (4 – 2) = 4 m/s with a speed u0.
1 2
V Using equation, s = ut + at
2
1 2
4 m/s B For case (1) – h = u0t1 – gt1 ...(1)
2
O A D
t
1 2
2 4 (Time) For case (2) – h = u0t2 – gt1 ...(2)
2
–4 m/s C Subtracting eq (2) from (1), we get

v-t graph is shown in diagram.


1
(
0 = u 0 ( t 2 + t1 ) + g t 22 - t12
2
)
Distance travelled 1
= area between time axis of the graph Þ u 0 = g ( t1 - t 2 ) ...(3)
2
= area OAC + are ABD Putting the value of u0 in eq (2), we get
1 1 æ1ö æ1ö
= OA ´ OC + AD ´ BD = 8 m -h = - ç ÷ g ( t1 - t 2 ) t 2 - ç ÷ gt 22
2 2 è2ø è2ø
If displacement occurs 1
Þ h = g ( t1t 2 ) ...(4)
1 1 2
= - ´ OA ´ OC + ´ AD ´ BD = 0 For case 3, u0 = 0, t = ?
2 2 æ1ö
6. (c) Speed on reaching ground u -h = 0 ´ t - ç ÷ gt 2 Þ h = æç 1 ö÷ gt 2
è2ø è2ø
v= u 2 + 2 gh Comparing eqs. (4) and (5), we get
H 1 2 1
Now, v = u + at gt = gt1t 2 \ t = t1t 2
2 2
Þ u 2 + 2 gh = -u + gt 12. (c) Given : Velocity
u dx
Time taken to reach highest point is t = , V = At + Bt2 Þ = At + Bt2
g dt
By integrating we get distance travelled
x 2
u + u 2 + 2 gH
ò ( At + Bt ) dt
nu
ò
2
Þ t= = (from question) Þ dx =
g g 0 1
Þ 2gH = n(n –2)u2 Distance travelled by the particle between 1s and 2s
x = ( 2 -1 ) + ( 2 - 1 ) =
A 2 2 B 3 3 3A 7B
+
2 3 2 3
EBD_7418
456 PHYSICS

1 2 1 it retarded. V = 0 + 12 – 3t2 = 0
13. (c) y1 = gn , y 2 = g ( n - N ) 2
2 2 3t2 = 12 Þ t = 2 sec
1
\ y1 - y 2 = g[n 2 - (n - N) 2 ] dv
2 Now the retardation, a = = 0 – 6t
g dt
Þ 1 = (2n - N)N [Q y1 - y 2 = 1] a [t = 2] = – 12 m/s2 retardation = 12 m/s2
2
1 N 19. (c) Height of tap = 5m and (g) = 10 m/sec2.
Þ n= + For the first drop,
gN 2
1 1
d 5 = ut + gt 2 = (0 ´ t ) + ´10t 2 = 5t2 or t2 = 1 or t =
14. (b) Velocity of preeti w.r.t. elevator v1= 2 2
t1 1.
d It means that the third drop leaves after one second
Velocity of elevator w.r.t. ground v 2 = then of the first drop. Or, each drop leaves after every 0.5
t2
velocity of preeti w.r.t. ground sec.
v = v1 + v2 Distance covered by the second drop in 0.5 sec
d d d 1 1 1 1 2 1
= + Þ = + = ut + gt = (0 ´ 0.5) + ´ 10 = (0.5) 2 = 1.25m .
t t1 t 2 t t1 t 2 2 2
t1t 2 Therefore, distance of the second drop above the
\ t= (time taken by preeti to walk up on ground = 5 – 1.25 = 3.75 m.1
(t1 + t 2 ) 20. (b) Graphs in option (c) position-time
distance
the moving escalator) and option (a) velocity-position are
1 2 corresponding to velocity-time
15. (a) s = ut + at
2 graph option (d) and its distance- time
–65 = 12t – 5t2 on solving we get, t = 5s time graph is as given below.
16. (a) For a body thrown vertically upwards acceleration Hence distance-time graph option (b) is incorrect.
remains constant (a = – g) and velocity at anytime t is 21. (a)
given by V = u – gt 22. (b) Time taken by same ball to return to the hands of
During rise velocity decreases linearly and during fall
velocity increases linearly and direction is opposite 2u 2 ´ 20
juggler = = = 4 s. So he is throwing the
to each other. g 10
Hence graph (a) correctly depicts velocity versus
balls after each 1 s. Let at some instant he is throwing
time.
ball number 4. Before 1 s of it he throws ball. So height
t=0 a t = 1 –a t = 2 of ball 3 :
A B
17. (b) v=0
–1
v=0 1
v = 6 ms
h3 = 20 × 1 – 10(1)2 = 15 m
C 2
t = 3 –1 –a Before 2s, he throws ball 2. So height of ball 2 :
v = –6 ms 1
v –u 6–0 h2 = 20 × 2 – 10(2)2 = 20 m
Acceleration, a = = 2
t 1 Before 3 s, he throws ball 1. So height of ball 1 :
= 6 ms–2
For t = 0 to t = 1 s, 1
h1 = 20 × 3 – 10(3)2 = 15 m
1 2 2
S1 = ´ 6(1) = 3 m ...(i)
2
For t = 1 s = to t = 2 s, Exercise - 3
1 2
S2 = 6.1 – ´ 6(1) = 3m ...(ii) 1. (d) B (v = 0)
2
For t = 2 s to t = 3 s, 4 sec 4 sec
1 2
S3 = 0 – ´ 6(1) = –3 m (iii) (2 sec) A C
2
Total displacement S = S1 + S2 + S3 = 3 m
3 (t = 0)
Average velocity = = 1 ms–1 D
3
Total distance travelled = 9 m As the time taken from D to A = 2 sec.
and D ® A ® B ® C = 10 sec (given).
9
Average speed = = 3 ms–1 As ball goes from B ® C (u = 0, t = 4 sec)
3
18. (d) x = 8 + 12t – t3 vc = 0 + 4g.
The final velocity of the particle will be zero, because 1
As it moves from C to D, s = ut + gt 2
2
MOTION IN A STRAIGHT LINE 457

1 25p 3p 28p
s = 4g ´ 2 + g ´ 4 = 10 g. V7 = + = = 7p = 22m / s
2 4 4 4
æ tö dv æ tö
2. (c) Here, f = f 0 çè1 - ÷ø or, = f 0 ç1 - ÷
è Tø d 2x
T dt 7. (a) a= = -s w2 sin wt .
2
dt
or, dv = f 0 æç1 - ö÷ dt
t
è Tø
On integrating, dx = s w 2 cos w t = s w cos w t
é æ t öù æ t2 ö dt w
\ v = ò dv = ò ê f0 ç1 - ÷ údt or, v = f 0 ç t - ÷ + C Again on integrating, we get
ë è T øû è 2T ø
where C is the constant of integration. sin w t
x = sw = s sin w t
At t = 0, v = 0. w
æ 0ö æ t2 ö 8. (c) Q t = x + 3 Þ x = t – 3 Þ x = (t – 3)2
\ 0 = f0 ç 0 - ÷ + C Þ C = 0 \ v = f0 ç t - ÷
è 2T ø è 2T ø dx
v= = 2(t – 3) = 0 Þ t = 3
dt
If f = 0, then 0 = f0 æç1 - ö÷ Þ t = T
t
è Tø \ x = (3 – 3)2 Þ x = 0.
9. (b) Distance along a line i.e., displacement
Hence, particle's velocity in the time interval
(s) = t3 (Q s µ t 3 given).
t = 0 and t = T is given by
By double differentiation of displacement, we get
t =T T acceleration.
é æ t öù
vx = ò dv = ò ê f 0 çè1 - T ÷ø údt ds dt 3
= 3t 2 and a =
dv d 3t 2
t =0 t =0 ë û V= = = = 6t
dt dt dt dt
éæ
T a = 6t or a µ t
t2 ö ù
= f 0 êç t - ÷ ú = 1 f 0T . Hence graph (b) is correct.
êè 2T ø ú 10. (a)
ë û0 2
40km
3. (a) 11. (c)
4. (c) According to question, object is moving with
constant negative acceleration i.e., a = – constant (C) 40
vL = 8 km/h, s = v0 × t Þ t = = 5h .
8
vdv Total distance flown by the bird = 10 × 5 = 50 km.
= - C Þ vdv = – Cdx
dx dy
12. (c) v = = b + 2 c t - 4 d t3
v2 v2 k dt
= - Cx + k Þ x = - + .
2 2C C v0 = b + 2c(0) - 4d (0)3 = b . (Q for initial velocity, t = 0)
Hence, graph (c) represents correctly. dv
a Now a = = 2 c - 12 d t 2
5. (a) Sn = (2n – 1); dt
2 \ a 0 = 2c - 12 d (0) 2 = 2 c , (at t = 0)
a a S 2n - 1 13. (d) Given x = ae–at + bebt
Sn + 1 = [2(n + 1) – 1] = (2n + 1) \ n =
2 2 S n +1 2n + 1 dx
Velocity, v = = –aae–at + bbebt
dt
3p i.e., go on increasing with time.
6. (b) V7 = V5 + ´2
8 u +u u + u2 u + u3
To calculate V5 i.e. velocity at end of 5 sec. 14. (b) V1 = 0 1 , V2 = 1 , V3 = 2
2 2 2
dv u 2 = u 0 + a(t1 + t 2 ) , u3 = u1 + a(t 2 + t 3 )
a = (25 - t 2 )1/ 2 Þ = (25 - t 2 )1/ 2
dt V1 - V2 u 0 - u 2 -a(t1 + t 2 )
= =
ì t = 5sin q V2 - V3 u1 - u3 -a(t 2 + t 3 )
ï dt 15. (b) The length of side CA at any time t is = 20 – 5t.
= 5cos q
ò dv = ò 5 (25 - 25sin q) cos qdq ïítdq= 0, q = 0
2 1/ 2
The length of side CB at any time t is = 10 – 5t.
ï At the instant A, B and C are collinear B
p
p/2 ï t = 5, q = (20 – 5t) + (10 – 5t) = 20. 20 m
25 p î 2
= ò 5 ´ 5cos qdq = 4
2
Solving we get t = 1. A
0 Therefore, length of CA at 5 m/s
t = 1 is 20 – 5 = 15 m. 5 m/s C(t = 0)
EBD_7418
458 PHYSICS

4. MOTION IN A PLANE

15. (d) Addition and subtraction of scalars make sense only


Exercise - 1
for quantities with same units, however multiplication
1. (c) A vector quantity is defined as the quantity which has and division of scalars of different unit is possible.
magnitude and direction and for which all the 16. (d)
mathematical operations are possible only through 17. (b)
r
vector laws of algebra. 18. (b) Position vector, r = ( a cos q)iˆ + ( a sin q) ˆj
r r
2. (a) Resultant vector of two vectors A & B inclined at r
an angle q, is given by | r |= a 2 cos 2 q + a 2 sin 2 q = a
19. (d)
R= A 2 + B2 + 2ABcos q \ if q = 0°; cos 0° = 1 ur ur ur
20. (c) By the triangle law of vector addition C + A = B .
r r
\ R= A 2 + B2 + 2AB = (A + B)2 21. (c) Using the law of vector addition, (d + e ) is as shown
R =A+ B in the fig.
This is the maximum resultant possible.
d
3. (c) The resultant of any three vectors will be cancel out
by fourth vector.
4. (c) The dot product should be zero.
A +B 3î + 5k̂
5. (c) Unit vector = =
|A +B | 32 + 52 f
6. (d) e
r r r
7. (c) | A ´ B | =1´ 1´ sin q and Â.B̂ = 1.1 ´ cos q \ d +e = f
22. (c)
| Â ´ B̂ |
\ = tan q r r
Â.B̂ 23. (b) | A + B | = A 2 + B2 + 2AB cos120° (q = 120°)
8. (d) ( 0.4) 2 + (0.7) 2 + c 2 = 1
-1
A 2 + B 2 + 2 AB æ ö æ cos120° = - ö
1
c 2 = 1 - (0.16 + 0.49) = è 2ø è 2ø
Þ c 2 = 1 - 0.65 = 0.35 Þ c = 0.35
9. (d) All the three unit vectors have the magnitude as unity = A 2 + B2 - A(A) = B2 = B (\ A = B)
ˆi = ˆj = kˆ = 1 r
\ y 24. (c) A = Axiˆ + Ayjˆ + Azkˆ
r
10. (c) | iˆ + ˆj | = (1)2 + (1)2 = 2 | A | = Ax 2 + Ay 2 + Az 2
ĵ ˆi + jˆ
| ˆi | = 1 \ Even if one component is non – zero the sum
Ax2 + Ay2 + Az2 can’t be zero.
cos b =
1 B r
\ b = 45° x
\ for | A | = 0, Ax = Ay = Az = 0.
2 iˆ
11. (c) 25. (d) A ® (1) ; B ® (2) ; C ® (4) ; D ® (3)
ur
12. (c) The resultant of A ´ 0 is a vector of zero magnitude. ur ur (iˆ + ˆj ) + (iˆ - ˆj ) ˆ
The product of a vector with a scalar gives a vector. (A) ( A + B) / 2 = =i
2
13. (a) In a clockwise system,
$i ´ $j = k,
$ $j ´ k$ = $i and k$ ´ $i = $j ur ur (iˆ + ˆj ) - (iˆ - ˆj ) ˆ
i$ (B) ( A - B) / 2 = = j
2
And $i ´ $i = $j ´ $j = k$ ´ k$ = 0
$i . $i = $j . $j = k$ . k$ = 1 k$ j$ ur ur
(C) ( A.B) / 2 =
(iˆ + ˆj ).(iˆ - ˆj ) 1 - 1
= =0
2 2
$i . $j = $j .k$ = k$ . $i = 0 ur ur
( A ´ B) (iˆ + ˆj ) ´ (iˆ - ˆj ) 0 - kˆ - kˆ + 0
(D) = = = - kˆ
Therefore, the right option is $j ´ k$ = $i 2 2 2
14. (b)
MOTION IN A PLANE 459

A uur uur 2 51 uur uur 7 51 uur uur 4 51


C a.r = - , b.r = - and c . r =
A 89 5 89 89

A+ ( B + C )
+B
C r ˆ
26. (a) B+ B 37. (d) r = (a cos wt )î + (a sin wt ) j
r d(r ) d
v= = {(a cos wt )î + (a sin wt )ˆj}
C dt dt
It illustrates the associative law of addition. = (- aw sin w t )î + (aw cos w t ) ĵ
ur ur ur
27. (b) Given A and B lie in a plane and vector C lies
= w[(-a sin wt )î + (a cos wt )ˆj]
outside this plane.
ur ur r r
r .v = 0
Resultant vector of A and B lies in the same plane
ur ur \ velocity is perpendicular to the displacement.
as that vectors A and B . 38. (b) y = bx2. Differentiating w.r.t to t an both sides, we get
ur ur ur
Resultant vector of A , B and C in non. dy dx
= b2x Þ vy = 2bxvx
coplanar vector therefore, their resultant can never dx dt
be zero. Again differentiating w.r.t to t on both sides we get
28. (c) 29. (b) dv y dx dv
30. (b) For two vectors to be perpendicular to each other = 2bv x + 2bx x = 2bv 2x + 0
dt dt dt
® ®
A × B =0 dv x
[ = 0, because the particle has constant
Ù Ù Ù Ù Ù Ù dt
( 2 i + 3 j+ 8k ) · ( 4 j- 4 i + a k ) = 0 acceleration along y-direction]
dv y
4 1 Now, = a = 2bv 2x ;
–8 + 12 + 8a = 0 or a=- =- dt
8 2
31. (a) (A)®(4); (B)®(1); C®(2); (D)®(3) a a
r v 2x = Þ vx =
r r ar . b 2b 2b
32. (a) Component of a along b = r r
|b| 39. (c) r = 2t 2 ˆi + 3tjˆ + 4kˆ
r
î ĵ k̂ r dr d
1 uur uur 1 \ v= ˆ = 4tiˆ + 3jˆ
= = (2t 2 iˆ + 3tjˆ + 4k)
33. (a) Area of triangle = | A ´ B | = 2 -3 4 dt dt
2 2 r
1 0 -1 r dv d
1 a= = (4tiˆ + 3j) ˆ = 4iˆ
= |[î (3 - 0) - ĵ (-2 - 4) + k̂ (+3)]| dt dt
2 r
\ a = 4ms -2 along x-direction
1 1
= | 3î + 6ˆj + 3k̂ | = 9 + 36 + 9 6t 4t
2 2 40. (a) ax = ,a y =
= 13.5 units 3 3
î ĵ k̂ so u x = òot a x dt = t 2 Þ (u x ) t=3 = 9m/sec
34. (b) A ´ B = 4 6 0 = î (0 - 0) - ĵ (0 - 0) + k̂ (12 - 12) = 0
2t 2 Þ ( u )
2 3 0
r and u y = òot a y dt = y t = 3 = 6 m / sec
r r 3
35. (d) Q arlies in the plane of b and c
r
\ a = b + lc r (because ux & uy = 0 at t = 0 sec)
r
Þ aiˆ + 2 ˆj + bkˆ = iˆ + ˆj + l ( ˆj + kˆ) 41. (a) u = 3iˆ + 4 ˆj Þ ux = 3 and uy = 4
Þ a = 1, 2 = 1+ l, b = l Þ a = 1, b = 1 r
uur a = 0.4iˆ + 0.3 ˆj Þ ax = 0.4 and ay = 0.3
36. (d) Let r be the required vector. Since it makes equal
uur uur uur So, Vx = ux + axt = 3 + 0.4 × 10 = 3 + 4 = 7
angles with a , b and c therefore,
rr rr rr Vy = uy + ayt = 4 + 0.3 × 10 = 4 + 3 = 7
a.r b.r c.r r
r r = r r = r r r
|a||r | |b|| r | |c|| r | V = 7iˆ + 7 ˆj Þ | V | = 72 + 7 2 = 7 2 units
uur uur uur uur uur uur 42. (a) 43. (d)
Þ a . r = b . r = c . r [Q | a | = 1, | b | = 1, | c | = 1]
44. (a) According to the question, at any instant t,
We check each of the given options.
uur x = 4t2, y = 3t2
51 ˆ
If r = (8i + 4 ˆj - 3kˆ), then dx d
89 \ vx = = (4t 2 ) = 8t
dt dt
EBD_7418
460 PHYSICS

dy d \Ut=0 = u x2 + y 2y = 12 + 12 = 2 m/s.
and vy = = (3t 2 ) = 6t
dt dt
The speed of the particle at instant t. d 2x d 2y
ax = 2 = 0; dy = =–2
dt dt 2
v= v x2 + v 2y 2
= (8t ) + (6t ) = 10t 2
For time of flight,
45. (d) Along same straight line, velocity & acceleration can y = 0 or 0 = t – t2 \ t = 1s.
be in the same direction, opposite to each other or
perpendicular as in circular motion with uniform speed. 1
For maximum height,t = s.
Thus q can be anywhere between 0 & 180°. 2
46. (c) Only in case of parabolic motion, the direction and 1 æ 1ö 1
2
magnitude of the velocity changes, acceleration \ H =t – t2 = -ç ÷ = m.
2 è 2ø 4
remains same. Morever, in case of uniform circular 58. (b) Two bodies will collide at the highest point if both
motion, the direction changes. cover the same vertical height in the same time.
47. (d) The equation of motion for projectile is V12 sin 2 30° V22 V2 1
So = Þ V = sin 30° = 2
1 2 2g 2g 1
x = x0 + Uxt + at
2 x 1
\ The shape of the trajectory depends on the initial \ V2 = V
2 1
position, initial velocity and acceleration. 59. (a) (A)®(1,2); (B)®(1); C®(2); (D)®(4)
48. (c) Vy = u sin q – gtm = 0
Range of the ball in absence of the wall
u y sin q
\ tm = (time to reach the maximum height) u 2 sin 2q 20 2 sin150°
g = = m = 20m
g 10
2(u sin q)
Total time of flight Tf = When d < 20m, ball will hit the wall, when d = 25m, ball will
g
fall 5m short of the wall.
\ Tf = 2tm
When d < 20m, ball will hit the ground, at a distance, x = 20m
u 2 sin 2q – d in front of the wall.
49. (d) Horizontal range =
g 60. (b) At point B the direction of velocity component of the
For maximum range q = 45° projectile along Y - axis reverses.
ur
Hence, V B = 2$i - 3 $j
u 2 sin 90° u 2
\ Rmax = = (Q sin 90° = 1) 61. (a) 62. (a)
g g
50. (b) If air resistance is ignored, then there is no acceleration u 2 sin 2q
63. (a) R = If q = 45° + a
in horizontal direction in projectile motion. Hence the g
particle move with constant velocity in horizontal
u 2 sin 2(45° + a) u 2 sin(90° + a) u 2 cos a
direction. then R1 = = =
g g g
51. (b) In both the cases, the initial velocity in the vertical
If q = 45° – a
downward direction is zero. So they will hit the ground
simultaneously. u 2 sin 2 (45° - a ) u 2 sin(90° - a)
then R2 = =
g g
2u sin (q - a)
52. (d) T=
g cos a u 2 cos a
= \ R1 = R2
53. (d) Velocity is minimum at the highest point. g
54. (b) Only horizontal component of velocity (u cos q) 64. (a)
p v2 65. (a)
55. (c) If the angle of projection is , then range = 0 sin (p/2) 66. (c) The time taken to reach the ground depends on the
4 g
height from which the projectile is fired horizontally.
v02 Here height is same for both the bullets and hence
Þ ( R)max = [Q {sin (p/2)}max = 1] they will reach the ground simultaneously.
g
56. (d) At the highest point of trajectory, the acceleration is equal H1 u 2 sin 2 q / 2g
67. (d) = 2 = tan 2 q
to g. H 2 u sin 2 (90º - q) / 2g
57. (c) (A)®(2); (B)®(3); C®(1); (D)®(4) 2 u sin q
68. (d) Time of flight =
dx dx
g
Ux = =1 and Uy = = 1 – 2t 2 ´ 9.8 ´ sin 30º 1
dt dt = = 2 ´ = 1 sec .
9. 8 2
MOTION IN A PLANE 461

(u sin q) 2 u 2 sin 2 q æ ds ö
69. (c) We know that, y m = H = = 77. (b) Resistance = kv ç = k ÷
2g 2g è dt ø
DH 2 Du Du Equations of motion are
\ = . Given = 2%
H u u d2x dx
DH 2
= -k ............. (1)
\ = 2 ´ 2 = 4% dt dt
H
1 gx 2 d2 y dy
= -k -g ............. (2)
70. (c) y = x tan q - dt 2 dt
2 u 2 cos 2 q
Integrating (1) and (2) and using the initial conditions,
10 ´ 50 ´ 50
y = 50 tan 60° - =5 m we get
2 ´ 25 ´ 25 ´ cos 2 60° dx
71. (a) For maximum range, the angle of projection, q = 45° = u cos a.e - kt ............. (3)
dt
u 2 sin 2θ ( 20 ) sin ( 2 ´ 45° ) 400 ´1
2

\R = = = = 40m. dy
g 10 10 and k + g = (ku sin a + g ).e - kt
dt
72. (c) dy 1
i.e., = [(ku sin a + g ).e - kt - g ] ......... (4)
2u sin θ dt k
73. (a) t1 = and
g dy dy / dt [(ku sin a + g ).e - kt - g ]
= = .... (5)
2 u sin (90 - q) 2 u cos q dx dx / dt ku cos a.e - kt
t2 = =
g g Direction of projection was a with the horizontal, when
the direction of motion again makes the angle a with
4 u 2 cos q sin q 2 é u 2 sin 2 q ù 2 the horizontal, it really makes the angle (p – a) with the
\ t1 t 2 = = ê ú = R,
g2 g ëê g úû g horizontal in the sense of the direction of projection. If
where R is the range. this happens after the time t, we have from (5),
Hence t 1 t 2 µ R ( ku sin a + g ).e- kt - g
tan ( p - a ) =
74. (d) Vh = V cos q ku cos a.e - kt
where h is the maximum height (ku sin a + g ) - ge - kt
i.e., - tan a =
æ u2 sin 2 q ö ku cos a
Vh × h = (u cos q) çç ÷÷ i.e., -ku sin a = ku sin a + g - g .e- kt
è 2g ø
u3 sin 2 q cos q 3 u3 2ku
= = or e kt = 1 + sin a
2g 16g g
1 æ 2 ku ö
u 2 sin 2 30° or t = log ç1 + sin a ÷
75. (c) max height, HA = A k è g ø
2g 78. (b) P be the point where the tangent is parallel to the
u 2B sin 2 q inclined plane. If PN = z be perpendicular from P on the
max height, HB = inclined plane and PM the vertical altitude of P then
2g
As we know, HA = HB evidently for all points on the path, P is the point where
z is the greatest and consequently PM is greatest.
u 2A sin 2 30° u 2B sin 2 q sin 2 q u2
= Þ 2
= A2
2g 2g sin 30° u B P
2
æu ö b
sin 2 q = ç A ÷ sin 2 30° u
è uB ø
z
1 A
Þ sin 2 q =
=
36
b M N
1 æ1ö
sin q = Þ q = sin -1 ç ÷
6 è6ø Now for the point P, velocity perpendicular to the
76. (b) As we know, R = u cosq × t inclined plane is zero. Now the velocity and acceleration
Given, R = 300 m, t = 6 s perp. to the plane at O is u sin (a – b) and g cos b and
R 300 this velocity becomes zero at P.
\ u cos q = = = 50ms –1 \ 0 = u2 sin2 (a – b) – 2g cos b.z
t 6
EBD_7418
462 PHYSICS

u 2 sin 2 (a - b) 81. (d) For complementary angles of projection (45° + a) and


z= (45° – a) with same initial velocity u, range R is same.
2 g cos b
p b p b q1 + q2 = (45° + a) + (45° – a) = 90°
For max. range a = + or a - b = - 82. (b) Comparing the given equation with
4 2 4 2
u2 æ p bö gx 2
sin 2 ç - ÷ y = x tan q - 2 , we get
Hence, z = 2u cos 2 q
2 g cos b è 4 2ø
u2 é æp öù tan q = 3
= ê1 - cos çè - b÷ø ú 83. (b) Horizontal distance covered should be same for the
4 g cos b ë 2 û
time of collision.
u2 1
= (1 - sin b) or PM = z sec b 400 cos q = 200 or cos q = or q = 60°
4 g cos b 2
u2 u2 1 84. (b) Comparing the given equation with the equation of
= (1 - sin b) = = (max. range) trajectory of a projectile,
2
4 g cos b 4 g (1 + sin b) 4
Þ Maximum range = 4 × PM gx 2
y = x tan q -
79. (b) Let the two bodies meet each other at a height h after 2u 2 cos 2 q
time T of the projection of second body. Then before 1
we get, tan q = Þ q = 30°
meeting, the first body was in motion for time (t + T) 3
whereas the second body was in motion for time T. 20 40
and 2u 2 cos 2 q = 20 Þ u 2 = =
The distance moved by the first body in time (t + T) 2cos 2 q 3
2
1 u 40 4
= u (t + T) – g (t + T)2. Now, R max = = = m
2 g 3 ´ 10 3
And the distance moved by the second body in time T 85. (c) the horizontal velocity of the stone will be the same as
1 2 that of the train. In this way, the horizontal motion will
= vT – gT = h (supposed above). ........ (1) be uniform. The vertical motion will be controlled by
2
the force of gravity. Hence it is accelerated motion.
Q The two bodies meet each other,
The resultant motion is a parabolic trajectory.
\ They are equidistant from the point of projection.
86. (b) 87. (c)
1 1
Hence, u (t + T) – g (t + T)2 = vT – gT2
2 2 88. (c) The bullet performs a horizontal journey of 100 cm with
or gt2 + 2t (gT – u) + 2 (v – u) T = 0......... (2) constant velocity of 1500 m/s. The bullet also performs
1 a vertical journey of h with zero initial velocity and
Also from (1) we get, h = vT – gT2 downward acceleration g.
2 Distance
dh \ For horizontal journey, time (t) =
\ = v - gT Velocity
dT
100 1
\ h increases as T increases \t= = sec … (1)
1500 15
\ T is minimum when h is minimum i.e., when
The bullet performs vertical journey for this time.
dh 1 2
= 0, i.e. when v – gT = 0 or T = v/g. For vertical journey, h = ut + gt
dT 2
2
Substituting this value of T in (2), we get 1 æ 1 ö or, 20
h = 0 + ´10 ´ ç ÷ h= cm = 2.2cm
gt2 + 2t (v – u) + 2 (v – u) (v/g) = 0 2 è 15 ø 9
89. (c) Velocity of projectile u = 147 ms–1
2 g (u - v ) + 4 g 2 (u - v )2 + 8vg 2 (u - v ) angle of projection a = 60°
or t =
2g 2 Let, the time taken by the projectile from O to A be t
where direction b = 45°. As horizontal component of
u - v + u2 - v2
or t = velocity remains constant during the projectile motion.
g Þ v cos 45° = u cos 60° v sin 45°
neglecting the negative sign which gives negative v
1 1
value of t. Þ v× = 147 ´
45°
2 2 2 u sin 60° u
u sin 2q A v cos 45°
80. (b) Horizontal range R = .
g Þ v = 147 ms -1
60°
Range is same for angle of projection q and (90° – q). 2
O u cos 60° B
For Vertical motion,
MOTION IN A PLANE 463

vy = uy – gt Where v is the speed of an object and R is the radius of


Þ v sin 45° = 45sin 60° – 9.8 t the circle. It is always directed towards the centre of the
147 1 3 circle. Since v and R are constants for a given uniform
Þ ´ = 147 ´ - 9.8 t circular motions, therefore the magnitude of centripetal
2 2 2
acceleration is also constant. However, the direction of
147 centripetal acceleration changes continuously. Therefore,
Þ 9.8 t = ( 3 - 1) Þ t = 5.49 s
2 a centripetal acceleration is not a constant vector.
1 v1 r1w 1
90. (a) We know, R = 4 H cot q Þ cot q = 100. (b) = = [v = rw]
2 Ö5 v2 r2w 2
From triangle we can say that
2 1
2 r ur r
sin q = , cos q = 101. (d) v = w´ r
q r
5 5 As linear velocity vector v is along the tangent to the
1 ur
2v 2 sin q cos q circular path and angular velocity vector w is
\ Range of projectile R = r ur
g perpendicular to v , so w is along the axis of rotation.
2v 2 2 1 4v 2 102. (c) When a particle moves on a circular path with a
= ´ ´ =
g 5 5 5g constant speed, then its motion is said to be a
uniform circular motion in a plane. This motion has
u 2sin 2q
91. (a) Horizontal range = so g µ u2 radial acceleration whose magnitude remains constant
g but whose direction changes continuously, So ar ¹ 0
g planet (u planet )2 and at = 0.
or = If the circular motion of the particle is not uniform
g earth (u earth ) 2 but accelerated then along with the radial acceleration
æ3ö
2 it will have tangential acceleration also and both
2
Therefore gplanet = ç ÷ (9.8 m / s ) these acceleration will be mutually perpendicular.
è5ø
= 3.5 m/s2 Vr at
92. (d)
r r r ar
93. (c) v BA = vB - vA = 80 - 65 = 15 km/hr
[Q both are moving in the same direction] r r
ar
r r r o o
94. (b) v br = vb + vr vr

Þ v br = v b2 + v r2
vb So, ar ¹ 0 and at = 0.
vbr
Þ 10 = 8 2
+ v 2r When, ar = 0 and at = 0 motion is accelerated translatory.
Also, when ar = 0 and then motion is uniform translatory.
Þ v r = 6 km/hr 103. (c) In circular motion with constant speed, acceleration is
vr
vr 1 always inward, its magnitude is constant but direction
95. (c) sin 30 ° = =
vm 2 changes, hence acceleration changes, so does velocity.
v m 0.5 vm 30° K.E. is constant.
Þ vr = = = 0.25 m/s. 90°
2 2
a
96. (c) Relative velocity = (3iˆ + 4j)
ˆ - (-3iˆ - 4ˆj) = 6iˆ + 8jˆ .
v
97. (a) Speed along the shortest path
1 5 km/hr 104. (c) Since the circular motion is uniform, therfore there is
= = 4 km/hr 4 km/hr
15 / 60 no change of angular velocity. Thus angular
acceleration is zero.
Speed of water v = 5 2 - 4 2 = 3 km/hr
r r 105. (c) In uniform circular motion, the body move with vT
98. (a) v SB = v $j = v s + 3$i (tangential velocity) & aC. If aC = 0 then it implies that
r 100 the body is no longer bound to rotate in circle & so no
v S = v $j – 3$i and v = = 2m/s
50 change in the direction of velocity. Hence it move
r tangentially to the circle outward with velocity vT.
\ v S = v 2 + (3)2 = 22 + 9 = 13 m/s
Drift = 50 × 3 = 150 m vT

v2 O aC
99. (d) Centripetal acceleration, ac =
R
EBD_7418
464 PHYSICS

106. (b) When a body moves on a curved path with a con- Vw


stant speed, it experiences the centripetal acceleration
which along the radius. Since velocity acts along the q
tangent therefore acceleration is perpendicular to the r
Vw
direction of velocity and hence motion. \ tan q = r
Vr
107. (d) The swimmer will cross
straight if the resultant R Vr
velocity of river flow and B C
swimmer acts perpendicular to 112. (c) According to pythagorus theorem
the direction of river flow. It
Vrm = Vr 2 + Vm 2

1 km
will be so if the swimmer moves v
VS
making an angle a with the
upstream. i.e. goes along OB. O Vr A

v2
108. (c) a= = 1 cm/s. Centripetal acceleration is directed
r VW Vm
towards the centre. Its magnitude = 1. Unit vector at O
the mid point on the path between P and Q is
q
-(xˆ + y)
ˆ / 2.

B Vrm
109. (b) Boat 2 v Boat 1 Vr
v
river
Vr,g=5m/s
If component of velocities of boat relative to river is v 2 (250) 2
113. (a) ac = = = 62.5 m/s2
same normal to river flow (as shown in figure) both r 103
boats reach other bank simultaneously. 62.5
Þ ac /g = = 6.38
1km 9.8
110. (b) v= = 4 km h -1 , v b = 5 km h -1 v1 v sin θ v 2
1
h 114. (a) ω = = = sin θ
4 r a/sinq a
vw 1 æ 72v 2 ö 2
115. (b) As when they collide vt + ç t - pR = vt
2 è 25pR ÷ø
5 pR
v \ t=
vb 6v
vt
Now, angle covered by A = p +
R
11p
v w = v b 2 - v 2 = 25 - 16 = 9 = 3 km h -1 Put t , \ angle covered by A =
6
111. (a) Man should hold the umbrella in the direction of the
æ 2p ö p 2
relative velocity of the rain. If Vr ® velocity of rain, 116. (d) Dv = 2v = 2wr = 2 ç ÷ ´ 1 = cm/s
è 60 ø 30
VW ® velocity of wind and Vrw ® relative velocity
of rain w.r.t. wind 117. (c) Clearly Y
N r
a = ac cos q(-iˆ) + ac sin q(- ˆj ) P( R, q )
R
q
-v 2 v2 X
= cos q iˆ - sin q ˆj O
R R
VW –Vw C
W E
O p
q 118. (c) Given, w = 2 rad s–1, r = 2 m, t = s
Vr 2
p
Angular displacement, q = wt = 2 ´ = p rad
B D 2
Vr Linear velocity, v = r × w = 2 × 2 = 4 m s–1
S q æpö
\ change in velocity, Dv = 2v sin = 2 × 4 × sin ç ÷
2 è2ø
= 8 m s–1
MOTION IN A PLANE 465

119. (b) Circumference of circle is 2pr = 40m 3. (b) From the diagram, u = aiˆ + bjˆ
Total distance travelled in two revolution is 80m. Initial
As u is in the first quadrant, so both components a
velocity u = 0, final veloctiy v = 80 m/sec
and b will be positive.
so from v2 = u2+2as Þ (80)2 = 02 + 2 × 80 × a
Þ a = 40m/sec2 For v = piˆ + qjˆ, as it is in positive x-direction and
120. (a) ar = w2 R located downward so x-component p will be positive
ar = (2p2)2R = 4p222R and y-component q will be negative.
Hence, a, b and p are positive but q is negative.
æ 22 ö
2 é 22 ù 4. (b) Let r makes an angle q with positive x-axis component
= 4 p 2 çè ÷ (1)
44 ø êëQ v = 44 úû of r along x-axis.
dv rx =| r | cos q
at = =0
dt (rx )maximum =| r | (cos q) maximum
anet = ar = p2 ms–2 and direction along the radius towards
the centre. =| r | cos 0° =| r |
121. (b) (Q cos q is maximum of q = 0°)
wt q = 0°.
122. (d) PQ = R sin ´2 Hence the vector r has maximum value along positive
2
x-axis.
wt
= 2R sin . 5. (c) Consider, projectile is fired at an angle q.
2
According to question, Y
123. (a) As time periods are same and so
q = 15° and R = 50 m
a1 w 2 r1 r1 2 1
= = = = . u 2 sin 2q u
a2 w 2 r2 r2 4 2 Range, R =
124. (b) From the geometry of the figure, the g
q
angle traverses about A and C are q u 2 sin(2 ´ 15°) X
R = 50 m = R
and 2q respectively. So g
1
q 2q 50 ´ g = u sin 30° = u 2 ´
2
Þ 50 × g × 2 = u2
wA = and w C = = 2w A . 2
t t
u 2 = 50 ´ 9.8 ´ 2 = 100 ´ 9.8 = 980
125. (c) Given : r = 30 cm = 0.3 cm = 0.3 m and V = 2t
Radial acceleration at t = 3 sec u = 980 = 31.304 m/s = 14 5
(Q g = 9.8 m/s2 )
v2 4t 2 4 ´ (3) 2
ar = = = = 120 m/s 2 u 2 sin 2 ´ 45° u 2
r 0.3 0.3 Now, q = 45°; R = = = 100 m
and tangential acceleration g g
6. (b) As we know that,
dv
at = = 2 m/s 2
dt æ Dp ö
Impulse, I = F Dt = ç ÷ Dt = Dp
è Dt ø
Exercise - 2 where F is force, Dt is time duration and Dp is change
in momentum. As Dp is a vector quantity, hence impulse
1. (b) Given, A = iˆ + ˆj is also a vector quantity. Sometimes area can also be
B = iˆ - ˆj treated as vector.
As we know that 7. (d) As speed is a scalar quantity, hence it will be related
r r with path length (scalar quantity) only.
A × B =| A || B | cos q
total distance travelled
(iˆ + ˆj ) × (iˆ - ˆj ) = ( 12 + 12 )( 12 + 12 ) cos q Hence, Speed v0 =
time taken
So, total distance travelled = Path length
(i + j )(i - j ) = 2 ´ 2 cos q
where q is the angle between A and B = (speed) × time taken
Hence, path length which is scalar and traversed in
1- 0 + 0 -1 equal intervals.
cos q = =0
2 2 8. (c) As given that in two dimensional motion the
\ q = 90° instanteous speed v0 is positive constant and we know
2. (d) A scalar quantity does not depend on direction so it that acceleration is rate of change of velocity or
has the same value for observers with different instantaneous speed and hence it will also be in the
orientations of the axes. plane of motion.
EBD_7418
466 PHYSICS

r
r Dr (displacement) N($j)
9. (d) vav =
Dt (time taken)
(13 - 2)iˆ + (14 - 3)ˆj 11 ˆ ˆ
= = (i + j)
5-0 5
w 10 km/h A 100 km
10. (b) Given, u1 = u2 = u, q1 = 60º, q2 = 30º
45°
In Ist case, we know that range
u 2 sin 2(60°) u 2 sin 120° u 2 sin( 90° + 30°) 100 km O
R1 = = = V BA = 10 2 km / h
g g g
B
u 2 (cos 30 ° ) 3u 2 S
= = Time taken to reach the shortest distance between
g 2g
OB 50 2
In IInd case when q 2 = 30° , then A and B = uuuur = = 5h
VBA 10 2
u 2 sin 60° u 2 3 15. (a)
R2 = = Þ R1 = R2
g 2g
Boat 2 v Boat 1
(we get same value of ranges).
v
River
11. (c) Initially u = cos a ˆi + u sin a ˆj . At highest point
Vr,g =5m/s
v = u cos a ˆi \ difference is u cos a . c is correct
If component of velocities of boat relative to river is
12. (b) Two vectors are same normal to river flow (as shown in figure) both
r ˆ ˆ boats reach other bank simultaneously.
A = cos wti + sin wtj 16. (c) From question,
r wt wt Horizontal velocity (initial),
B = cos ˆi + sin ˆj 40
2 2 ux = = 20m/s
r r r r 2
For two vectors A and B to be orthogonal A.B = 0 1
Vertical velocity (initial), 50 = uy t + gt2
2
r r wt wt
A.B = 0 = cos wt.cos + sin wt.sin 1
2 2 Þ uy × 2 + (–10) ×4
2
or, 50 = 2uy – 20
æ wt ö æ wt ö 70
= cos ç wt - ÷ = cos ç ÷ or, uy = = 35m / s
è 2 ø è 2 ø 2
u y 35 7
wt p p \ tan q = = =
So, = \ t= u x 20 4
2 2 w 7
13. (b) Here,x = 4sin(2pt) ...(i) Þ Angle q = tan–1
4
y = 4cos(2pt) ...(ii) 17. (c) Given: Position vector
r = cos wt + sin wt ŷ
Squaring and adding equations (i) and (ii) r x̂
x2 + y2 = 42 Þ R = 4 r
\ Velocity, v = – wsin wt x̂ + wcos wt ŷ
Motion of the particle is circular motion, acceleration and acceleration,
r r
ur V2 a = –w2 cos wt x̂ – w2sin wt ŷ = – w2 r
vector is along – R and its magnitude = r . r = 0 hence rr ^ vr and
R r v
r
Velocity of particle, V = wR = (2p) (4) = 8p a is directed towards the origin.
r r r r
ur ur 18. (b) A + B = A - B
14. ( )
(a) V A = 10 –i$ ; V B = 10 $j () Squaring on both sides
ur r r2 r r2
V BA = 10 $j + 10 ˆi = 10 2 km / h A+B = A-B
r r r r r r
Þ A · A + 2A · B + B · B
Distance OB = 100 cos 45° = 50 2 km
MOTION IN A PLANE 467

r r r r r r 2 2
= A · A – 2A · B + B · B æ 1 ö æ 1 ö 1
r r sin 2 q = ç ÷ ç ÷ =
Þ 4A · B = 0 Þ 4AB cos q = 0 è 2ø è 2ø 4
Þ cos q = 0 Þ q = 90° 1 æ1ö
19. (c) Yes, the person can catch the ball when horizontal sin q = Þ q = sin -1 ç ÷ = 30°
2 è2ø
velocity is equal to the horizontal component of ball’s
velocity, the motion of ball will be only in vertical 24. (c) R 2 = [A 2 + B 2 + 2 AB cos q ]
v
direction w.r.t person for that 0 = v 0 cos θ or θ = 60° R 2 = R 2 + R 2 + 2 R 2 cos q
2
2 2
u sin q - R 2 = 2 R 2 cos q or cos q = -1 / 2 or q = 2p / 3
20. (a) H1 =
2g r r r
25. (c) Moment of force, t = r ´ F
u 2 sin 2 (90° - q) u 2 cos 2 q Y
and H 2 = =
2g 2g
r
u 2 sin 2 q u 2 cos2 q (u 2 sin 2q)2 R2 r r
F
H1H 2 = ´ = = A r – r0
2g 2g 16 g 2 16 P
r
r0 r
\ R = 4 H1 H 2 r
21. (b) Given: O X
x = 5t – 2t2 y = 10t r r r r
t = (r – r0 ) ´ F
dx dy r r
vx = = 5 – 4t vy = = 10 r – r0 = (2iˆ + 0 ˆj – 3kˆ) – (2iˆ – 2 ˆj – 2kˆ)
dt dt
dv x dv y = 0iˆ + 2 ˆj – kˆ
ax = =–4 ay = =0 r
dt dt t = (0iˆ + 2 ˆj – kˆ) ´ (4iˆ + 5 ˆj – 6kˆ)
r r
a = a xi + a y j a = -4i m / s2 iˆ ˆj kˆ
r
Hence, acceleration of particle at (t = 2 s) = –4m/s2 t = 0 2 –1 = –7iˆ – 4 ˆj – 8kˆ
22. (d) From the figure it is clear that range is required 4 5 –6
u 2 sin 2q (10)2 sin(2 ´ 30°) gx 2
R= = =5 3 26. (d) y = x tan q –
g 10
u 2u 2 cos 2 q
A
OC x
30° 27. (b) t = =
Range R u cos q u cos q
AC = x tan q u sin q

BC = distance travelled by bullet u


10m

10m

B
Tower in time t, vertically.
y
1 2
y = u sin q t – gt
2 q
23. (c) For projectile A O u cos q C
1
AB = x tan q – (u sin qt – gt2)
u 2A 2
sin 45° 2
Maximum height, HA = x 1
2g = x tan q – (usinq × – gt2)
u cosq 2
For projectile B
Þ distance trevelled by monkey
u 2B sin 2 q 1 2 1 2
Maximum height, HB = = x tan q – x tan q + gt = gt
2g 2 2
. (\ bullet will always hit the monkey)
As we know, HA = HB
u 2A sin 2 45° u 2B sin 2 q Exercise - 3
= r r r
2g 2g 1. (c) Given, A = B - C
r r r
sin 2 q u 2A 2 \ C = B-A
= 2 æu ö 2 r r
2 Þ sin q = ç A ÷ sin 45° If q is the angle between A and B , then
sin 45° u B2 u
è Bø
EBD_7418
468 PHYSICS

B2 +A 2 – C2 (v0 /ay ) av0


C2 = B 2 + A2 - 2 AB cos q \ cosq = Now ax = a sinq = av0 × 2
=
2
2AB æv ö æ ay ö
1+ ç 0 ÷ 1+ ç ÷
2. (c) 3. (a) è ay ø è v0 ø
4. (b) For any point P (x, y), 1 ay
we have 2
at = a cosq = av0 × = a v0
2
gx æv ö ( ay )2 + v02
y = x tanq – 1+ ç 0 ÷
2u 2 cos 2 q è ay ø
a2 y
=
2
= x tanq éê1 - gx ù
ú
æ ay ö
1+ ç ÷
2 2
ë 2u cos q tan q û è v0 ø
é gx ù ur ur
é xù
= x tanq ê1 - u 2 (2sin q cos q) ú = x tanq ê1 - ú 7. (a) Let v1 and are the
v2
ë û ë Rû velocities of any two particles and
éR - xù yR q is the angle between them. As each
= x tanq ê Þ tanq =
ë R úû x( R - x ) particle has same speed, so v1 = v2
From figure; =v
y y y ( R - x ) + xy yR The relative velocity of particle 2
tana + tanb = + = x( R - x ) = x( R - x ) w.r.t. 1 is given by
x R- x
ur ur ur
Hence, tanq = tana + tanb v 21 = v 2 – v1
5. (c) x + u2cos q2t = u1 cos q1 t
or v21 = v 2 + v 2 - 2v v cos q = 2v 2 (1 - cos q)
x
\ t = u cos q - u cos q ...(i) q
1 1 2 2 = 2v 2 ´ 2 sin 2 q / 2 = 2 v sin .
2
Also u1 sinq1 = u2 sin q2 ...(ii) As the velocities of the particles are randomly distributed,
After solving above equations, we get so q varies from 0 to 2p. The magnitude of average velocity
x sin q 2 when averaged over all such pairs. Thus
t= . 2p
u1 sin(q 2 - q1)
6. (b) Since velocity in vertical direction is constant, ò v21d q 4v
dv y v 21 = 02p =
\ ay = dt = 0 π
ò dq
The acceleration in horizontal direction, 0
dv d (av0t ) 8. (a) Consider the motion of the particle along the direction
ax = x = = av0
dt dt (x-axis) and perpendicular direction of OB (y-axis). The ini-
tial velocities and accelerations along these directions are
a = ax 2 + a y 2 = (av0 )2 + 0 = av0 shown in the figure. The displacement along y-axis in time T
The total acceleration is av0 and directed along horizontal becomes zero.
direction. By using second equation of motion along y-axis, we have
Let q is the angle that the resultant velocity makes with 1 1
y = uy t – ayt2 or 0 = u sinq T – (g cosq) T2
horizontal, then 2 2
Normal acceleration an = a sinq and tangential acceleration
which gives,
ay 2
at = a cosq , we have x = 2u sin q 2u tan q
2v0 T= =
g cos q g
2xv0
or y = In this duration the
a
Differentiating both sides displacement along x-axis
of equation (iii) w.r.t. x, 2u 2 tan q é cos2 q + sin 2 q ù
we get = ê ú
g êë cos q úû
a dy
1= × 2y × 2u 2 tan q sec q
2v0 dx =
g
dy v0
or = = tanq 9. (a) BQ is the vertical target, OB = d. P is the position of the
dx ay
shot at any instant, OP is the line of sight cutting BQ at Q.
We have to find the velocity of Q.
Let BQ = z and Ð QOB = f
PM
Then z = d tan f = d.
OM
MOTION IN A PLANE 469

Q 1
Þ xy = uyt + a yt 2
P 2
z 1
1 g cos 30° t2
20 cos 30° = u cos 60° t +
y u sin at - gt 2 f 2
2
= d. = d. u2 3 3 3u 2
B
u cos a.t
O d M 5u 2
x 20 3 = + g. Þ 20 =
1 g 2 g2 2g
u sin a - gt
2 æ 1 g ö
= d. = d ç tan a - .t ÷ u = 80 m/s = 4 5 m/s.
u cos a è 2 u cos a ø 14. (b) Let at any instant points
1 gd A and B are at the positions
\ z=- = constant since u cos a = const.
2 u cos a shown in figure. The point A
1 moves towards B with
10. (a) 150 = 100 t + 10 t2 ; t = 10 sec velocity v. At the same time B
2
400 m/s
T 37° 3
move away from A with the
500 m/s speed u cos a where a is the
100 m/s 3 inclination of the line
AB with x-axis. The distance between them decreases at the
400 4000
x= t= m rate of (v – u cos a).
3 3 The initial moment the separation between them is l. This
x separation reduced to zero when A and B meet. Suppose A
2 and B meet after time T, then
v T T T
11. (c) Centripetal acceleration
r
. It is perpendicular to
ò 0 (v - u cos a)dt = l or ò0 v dt - ò0 u cos a dt = l
the rate of increase in speed, i.e., acceleration, which is T
equal to g according to the question. It is tangential to or vT - u ò cos a dt = l .... (i) [Here a is not constant]
0
circular path. Hence, net acceleration of the particle Along x-axis, the distance described by B in time T is uT.
éæ v2 ö 2
1/ 2
ù The velocity of A parallel to x-axis is v cos a. Therefore,
= êç ÷ + g 2 ú T
êè r ø
ë
ú
û
distance describes by A in time T is ò0 v cos a dt .
When point A and B meet
12. (c) If t is the time of flight, then
T
1
0 = v t – (g cos q)t2 Þ t =
2
2v
g
g cosq. ò0 v cos a dt = uT .... (ii)

2 From equation (ii), we have


1 2 1 æ 2v ö 2v 2 T T uT
R = 0 + ( g sin q)t = g sin q çè g cos q ÷ø =
2 2 g
tan q sec q v ò0 cos a dt = uT or ò0 cos a dt = v
.... (iii)
13. (c) X-direction Substituting the value in equation (i), we get
x
Þ vx = ux + axt
u æ uT ö 2 2
vT - u ç ÷ = l or (v - u ) T = l or T = vl
0 = u cos 30° – g sin 30° .t .
è v ø v v2 - u 2
g cos 30° g sin 30°
Þ u 3 = gt ... (i) g 15. (b) x = 3t1 = 5t2 Þ t1 = x/3 and t2 = x/5
Y-direction 1 2 4 gx 2
30° Now h1 = 4t1 – gt1 = x – .
y 2 3 18
1 2 4x gx 2
and h2 = 4t2 – gt2 = – 10
2 5
Clearly, h2 < h1.
EBD_7418
470 PHYSICS

5. LAWS OF MOTION
16. (d) Here m = 0.5 kg ; u = – 10 m/s ;
Exercise - 1 t = 1/50 s ; v = + 15 ms–1
1. (c) Newton's first law of motion defines the inertia of Force = m (v– u)/t = 0.5 (10 + 15) × 50 = 625 N
body. It states that every body has a tendency to 17. (b) If a large force F acts for a short time dt the impulse
remain in its state (either rest or motion) due to its imparted I is
inertia. dp
2. (d) Inertia is defined as the ability of a body to oppose any I = F.dt = .dt
dt
change in its state of rest or of uniform motion. I = dp = change in momentum
3. (c) Newton's first law of motion is related to the physical 18. (a) 19. (c)
independence of force. 20. (c) Swimming is a result of pushing water in the
4. (b) The frame of reference which are at rest or in uniform opposite direction of the motion.
motion are called inertial frames while frames which 21. (a)
are accelerated with respect to each other are non– 22. (c) Hot gases with high velocity react against the rocket
inertial frames. Spinning or rotating frames are and push it up.
accelerated frame, hence these are non-inertial frames. 23. (b) A breeze causes branches of tree to swing. In general
5. (a) r r force is required to put a stationary object in motion.
6. (a) Impulse = change in momentum = m v 2 - m v1 24. (c) The gun applied a force F12 on the bullet in forward
2 2 2 direction & according to Newton’s third law bullet
8 + (-6) + (-10)
7. (b) m= = 10 2kg applies a reaction force on gun F21 in backward
1 direction. But the recoil speed of gun is very low in
8. (c) Mass (m) = 0.3 kg Þ F = m.a = – 15 x comparison to bullet due to large mass.
15 -150 25. (b) 26. (d)
a=– x= x = - 50 x 27. (a) It works on the principle of conservation of linear
0.3 3
momentum.
a = – 50 × 0.2 = 10 m / s 2 28. (a) If m1, m2 are masses and u1, u2 are velocity then by
F 5 ´ 10 4
5 conservation of momentum m 1 u1 + m 2u2 = 0 or
9. (c) F = ma Þ a = = = ´ 10-3 ms -2 | m1u1 | = | m2 u 2 |
m 3 ´ 107 3 29. (c) In series each spring will have same force.
Also, v2 – u2 = 2as Here it is 4 kg-wt.
5 30. (a) y
Þ v2 – 02 = 2 ´ ´ 10 -3 ´ 3 = 10-2 Þ v = 0.1 ms–1
3
10. (c) From Newton’s second law if SFi = 0 then the body is 2 kg m2
in translational equilibrium. 8 m/sec
11. (a) Since force at a point at any instant is related to the Presultant
acceleration at that point, at that instant and
acceleration is determined only by the instantaneous 12 m/sec
m1
force and not by any history of the motion of the c x
particle. Therefore, the moment the stone is thrown /se 1 kg
4m
out of an accelerated train, it has no horizontal force 3
and acceleration, if air resistance is neglected. m
12. (a) No force is required for an object moving in straight line
with constant velocity or for non acceleration motion. Presultant = 122 + 162 = 144 + 256 = 20
udM dM mg 600 ´ 10
13. (c) Thrust = = mg Þ = = = 6 kg s–1 20
dt dt u 1000 m3v3 = 20 or, m3 = = 5 kg
4
14. (b) Here u = 10 ms–1, v = –2 ms–1,
t = 4 s, a = ? 31. (d) According to law of conservation of momentum the
v - u -2 - 10 third piece has momentum
Using a = = = -3 m / s 2 y
t 4
\ Force, F = ma = 10×(–3) = –30 N = 1´ –(3iˆ + 4j)
ˆ kg ms–1
15. (b) Thrust on the satellite,
-vdM DP 1 + 4ˆj
F= = -v(av) = -av 2 Þ Average force = x
dt Dt 1 + 3iˆ
F - av 2 4j)
Acceleration = = +
3i
M M –(3iˆ + 4ˆj)kg ms –1 ( –(
= 1×
–4
10 s
LAWS OF MOTION 471

32. (b) From law of conservation of momentum F1 + F2 + F3 = 0.


MV = m1v1 + m2v2 ur uur uur F1
Here, M = 100 kg, v = 104 m s–1 R = F1 + F2 R
m1 = 10 kg, v1 = 0; m2 = 90 kg, v2 = ? uur uuur
F3 = - R (In eqbm)
100 ´ 104
\ 100 × 104 = 10 × 0 + 90 × v2 \ v 2 = uur uur uur F2
90 \ F3 = - (F1 + F2 )
v2 = 11.11 × 103 m s–1. F3
33. (c) Change in momentum = mv sin q – (– mv sin q) uur uur uur
\ F1 + F2 + F3 = 0
2vsin q
= 2 mv sin q = mg ´ 46. (d) If rope of lift breaks suddenly, then acceleration
g
becomes equal to g so that tension T = m(g – g) = 0.
= weight of the ball × total time of flight
34. (a) N = m a sin q + mg cos q ......(1) 47. (a) 49
Mass = = 5 kg
also m g sin q = m a cos q ......(2) 9.8
m
from (2) a = g tan q ac When lift is moving downward
o sq
sin 2 q N Apparent weight = 5(9.8 – 5) = 5 × 4.8 = 24 N
\ N = mg + mg cos q , ma
cos q q 48. (b) (A) ® (2) ; (B) ® (1) ; (C) ® (4) ; (D) ® (3)
cos q
mg m
mg q
cos mg g
or N = m a sin
A
300N
cos q q
120°
150°
35. (d) When a rain drop falls down with the constant speed, C
B
its weight is balanced by the upward viscous drag of TBC TBC

air and the force of buoyancy. Thus the net force acting
w1 w2
on it is zero.
36. (b) Opposite force causes retardation. Applying Lami's equation, we have
37. (d) By Newton’s second law of motion TBC W2 300
= =
F = n(mv) = nmv sin150° sin120° sin 90°
38. (a) As the ball, m = 10 g = 0.01 kg rebounds after striking TAB W1 TBC
the wall and = =
sin 90° sin150° sin120°
\ Change in momentum = mv – (–mv) = 2 mv After simplifying, we get
Inpulse = Change in momentum = 2mv TAB = 173 N, TBC = 150 N, W1 = 87 N, W2 = 260 N.
Impulse 0.54 N s
\n= = = 27 m s-1 49. (c) a=1
2m 2 × 0.01 kg
39. (a)
40. (a) As we know, |impulse| = |change in momentum|
= |p2 – p1|
= |0 – mv1| = |0 – 3 × 2| = 6 Ns m = 1000 kg
2 2
v –u
41. (b) As we know, a =
2S
400 Total mass = (60 + 940) kg = 1000 kg
= = 8 m/s2. So force required = F = ma
50 Let T be the tension in the supporting cable, then
= 800 × 8 = 6400 N T – 1000g = 1000 × 1
42. (b) In eqbm T1cos q T cos q Þ T = 1000 × 11 = 11000 N
T1 1
q
50. (b) When force is applied on m1
= T2 = 60N. …(1)
T1 sin q
then T = m2a and when force is applied on m2, then
T2
T1sin q = 60 N …(2) 2m T = m1a. Thus value of T is different for each case. And
T2
it depends on whether the force is applied on m1, or m2.
\ tan q = 1 Þ q = 45°. 51. (c) Common acceleration of system is
W = 60N
43. (c) When an elevator cabin falls down, it is accelerated down F
with respect to earth i.e. man standing on earth. a=
m1 + m2 + m3
44. (a)
m 3F
45. (c) Equilibrium under three concurrent forces F1, F2and F3 \ Force on m3 is F3 = m3 × a = m + m + m
requires that vector sum of the three forces is zero. 1 2 3
EBD_7418
472 PHYSICS

F force increases so much, it overcomes the static friction


52. (c) a = m + m So the acceleration is same whether the and the body starts moving.
1 2 68. (d)
force is applied on m1 or m2.
53. (d) Acceleration of the system 69. (c) Limiting friction is the maximum static friction beyond
T
F - 4 g - 2 g 120 - 40 - 20 which the object starts moving. It decreases a little bit
a= = before the object comes into motion. Thus limiting
4+2 6 friction is less than the kinetic friction.
2 kg a 70. (a)
= 10 ms-2 71. (d) In case (a) In case (b)
From figure N mN N
F2
T – 2g = 2a 2g
FBD of block
T = 2 (a + g) = 2 (10 + 10) F1
= 40 N q mg q mg
mg cos q mN q sin mg c os q s in
54. (b) 2 T cos 60º = mg or T = mg = 2×10 = 20 N. q q q
10.2
55. (b) F = (m + m + m) ´ a \a = m / s2 mg mg
6
10.2 mg sin q = F1 – mN
\ T2 = ma = 2 ´ = 3.4N
6 N = mg cos q \ mg sin q + m mg cos q = F1
F
T2 In second case (b)
C
T2
B
T1 T1
A mN + F2 = mg sin q Þ m mg cos q – F2 = mg sin q
or F2 = mg sin q – mmg cos q but F1 = 2F2
56. (d )
57. (a) For the bag accelerating down therefore mg sin q + m mg cos q
mg – T = ma = 2(mg sin q – m mg cos q)
mg sin q = 3 m mg cos q
49 or tan q = 3m or q = tan–1 (3m)
\ T = m( g – a ) = (10 – 5) T
10 72. (a) mBg = ms mAg {Q mAg = ms mAg}
a Þ mB = ms mA or mB = 0.2 × 2 = 0.4 kg
= 24.5 N mg 73. (d) Frictional force on the box f = mmg
\ Acceleration in the box a = mg = 5 ms–2
58. (b) R = mg – ma = 0.5 × 10 – 0.5 × 2 = 5 – 1 = 4 N
m2 3 2
2 v2 = u2 + 2as Þ 0 = 22 + 2 × (5) s Þ s = – w.r.t. belt
59. (b) a = m + m g = 7 + 3 ´ 10 = 3m / s 0° 5
1 2 s3 co Þ distance = 0.4 m
60. (c) ma cos 30° = mg sin 30° ma
g 74. (b)
\ a= 30° 60°
3 30° ma (pseudo force)
sin
mg M a
S/2
h
30° 90° oot S/2 sin q
Sm
61. (c) Friction does not depend on area in cantact. q
force of friction S/2
62. (a) Coefficient of static friction = ugh S/2 sin q
normal reaction Ro
Therefore, coefficient of static friction depends upon q
the normal reaction.
63. (d) Coefficient of friction is independent of applied force. For upper half of inclined plane
64. (b) When a box is in stationary position with respect to v2 = u2 + 2a S/2 = 2 (g sin q) S/2 = gS sin q
train moving with acceleration, then relative motion is For lower half of inclined plane
opposed by the static friction. 0 = u2 + 2 g (sin q – m cos q) S/2
65. (c) Þ – gS sin q = gS ( sinq – m cos q)
66. (d) When brakes are on, the wheels of the cycle will slide on 2 sin q
the road instead of rolling there. It means the sliding Þ 2 sin q = m cos q Þ m= = 2 tan q
friction will come into play instead of rolling friction. cos q
The value of sliding friction is more than that of rolling 75. (d) For the block to remain stationary with the wall
f = mN
friction. f=W
67. (d) The static friction comes into play, the moment there is
an applied force. As the applied force increases, static mN = W 10N 10N 10N
friction also increases, remaining equal and opposite
to the applied force upto a certain limit. But if the applied 0.2 × 10 = W Þ W = 2 N W
LAWS OF MOTION 473

76. (d) Friction is the retarding force for the block 84. (a)
F = ma = mR = mmg F
Therefore, from the first equation of motion 85. (b) As we know, coefficient of friction m =
N
v = u – at ma a
V Þm= = (a = 7.35 m s–2 given)
0 = V – mg × t Þ =t mg g
mg 7.35
\m= = 0.75
77. (d) Limiting friction = 0.5 ´ 2 ´10 = 10N 9.8
The applied force is less than force of friction, therefore 86. (c) Minimum force on A
the force of friction is equal to the applied force. = frictional force between the surfaces
6 = 12 N
78. (a) a = mg = [using v = u + at] Therefore maximum acceleration
10
12 N
6 6 amax = = 3m/s 2
Þ m= = = 0.06 4 kg
10 ´ g 10 ´ 10 Hence maximum force,
Fmax = total mass × amax
fs = 9 × 3 = 27 N
79. (c) N 87. (c) As m would slip in vertically downward direction, then
mg = µN
mg mg 100
Þ N= = = 200 Newton
m 0.5
Same normal force would accelerated M,
200
mg sinq = f s ( for body to be at rest) thus aM = = 4 m/s 2
50
Þ m ´ 10 ´ sin 30 ° = 10 Taking m + M as system
F = (m + M) 4 = 240 N
Þ m = 2.0 kg
88. (a)
N N
80. (c) N1 F1 N2
f
2
F

q q
f2 sin si n
mg q mg q
q q
mg sin q mg sin q mg mg cosq mg mg cosq
f1 mg cos q mg cos q
mg q mg Smooth surface Rough surface
q
1
For the upward motion of the body For smooth surface, s = g sin q t12 ...(i)
2
mg sin q + f1 = F1 For rough surface, a = g (sin q – m cosq) t22
or, F1 = mg sin q + mmg cos q 1
For the downward motion of the body, \ s = g(sin q- m cos q)t22 ...(ii)
2
mg sin q – f 2 = F2 From (i) and (ii),
or F2 = mg sin q – mmg cos q 1 1
\ s = gsin q t12 = g(sin q - m cos q)t22
F1 sin q + m cos q 2 2
\ = 2 2
Given, q = 45° \ t1 = (1 - m)t2
F2 sin q - m cos q
tan q + m 2m + m 3m Also, given that, t2 = nt1 \ t12 = (1 - m)n 2t12
Þ = = =3
tan q - m 2m - m m 1 æ 1 ö
=1- m \m = ç1 - 2 ÷
81. (d) When the body has maximum speed then n2 è n ø
m = 0.3x = tan 45° \ x = 3.33 m 89. (a) For no slipping between m and M,
F - mR 100 - 0.5 ´ (10 ´ 10) F £ (M + m) g/3
82. (c) a = = = 5 ms - 2 F £ 40 N
m 10 m=0.5
83. (b) The magnitude of the For no toppling of m block
F £ (M + m) g/4
frictional force f has to
F £ 30 N
balance the weight 0.98 5N \ Fmin = 30N
N acting downwards. 5N
5N 90. (a) From figure, the equations of motion are
Therefore the frictional force S F1 = T – µkW1 = ma1
= 0.98 N S F2 = µkW1 = 4ma2
0.1×9.8
=0.98 N S F3 = W1 – T = ma1
EBD_7418
474 PHYSICS

a1
2N1 sin q = 4 mg
2mg
1 \ N1 = ...(i)
sin q
f1 =µk W1 T Now for the equilibrium of lower cylinder,
W1 R1 =W1
N 2 = N1 sinq + mg ...(ii)
and f = N1 cosq
a2
T
or mN2 = N1 cosq ...(iii)
f2 =µk W2 After solving above equations, we get
2 3 1
m= .
a3 = a1
3 2
94. (a) The forces acting on the block are shown. Since the
W1 block is not moving forward for the maximum force F
4W1 R = 5W
2 1 applied, therefore
Solve the first and third equations simultaneously to get a1 = F cos 60° = f = µN ... (i) (Horizontal Direction)
(g/2) (1 – µk), from the second equation, a2 = (g/4)µk. Then Note : For maximum force F, the frictional force is the
the displacements of blocks 1 and 2 are given by limiting friction = µN]
1 2 g 2 g 2 and F sin 60° + mg = N... (ii)
x= at , i.e., x1 = (1 - m k ) t , x2 = 8 m k t From (i) and (ii)
2 4
At the instant that one-fourth of block 1 remains on N
block 2, x2 + l = x1 + (l/16), where l is the length of
g g l F cos 60o
block 2. Therefore, m k t 2 + l = (1 - m k ) t 2 +
8 4 16 mg 60
o
f
2 15l
or t =
2 g (2 - 3m k ) F
Fsin60o
æ ö g 15l k 15m
and x2 = ç m k ÷ = F cos 60° = µ [F sin 60° + mg]
è 8 ø 2 g (2 - 3m k ) 16 (2 - 3m k )
1 m mg
l=
7.47 Þ F=
cos 60° - m sin 60°
91. (c) Acceleration of mass at distance x
1
a = g (sin q – µ0 x cos q) ´ 3 ´ 10
2 3 5
Speed is maximum, when a = 0 = = = 20 N
g (sin q – µ0 x cos q) = 0 1 1 3 1
- ´
2 2 3 2 4
tan q
x= 95. (a) Here f is the angle of repose.
m0
92. (c) T' ' sin 45° = 0.2 g \ µ = tan f
andT ' cos 45°=T
T¢ s in f
T T µN mg
f
q
0.2g The retardation of the block
\ T = 0.2 g F mN + mg sin f
a= =
= 0.2 × 9.8 = 1.96 N. m m
flim = mN = 0.3 × 1g = 2.94 N
= tan f ´ mg cos f + mg sin f = 2g sin f
As external force is less than limiting friction, so m
friction = 1.96 N. Let s is the distance travelled, then by third equation of
r 1 motion
93. (c) sin q = = ,cos q = 8 / 9 2
3r 3 v2 = u0 – 2 a s
If mass of smaller cylinder is m, then mass of bigger 2 u02
or 0 = u0 – 2 × 2g sin f × s or s =
one will be 4m. For the equilibrium of upper cylinder, 4 g sin f
96. (c) When force is applied on a moving body in a direction
N1 N1 perpendicular to the direction of motion, then it takes a
circular path. Thus the direction of motion changes
N2 N
1 without changes in the speed.
q
97. (a) Optimum speed is given by V0 = (Rg tanq)1/2 on a
f
q
mg
banked road, the normal reaction’s component is
enough to provide the necessary centripetal force to a
car driven at optimum speed.
LAWS OF MOTION 475

98. (d)
99. (b) Due to centrifugal force, the inner wheel will be left up N Ncosq Train
when car is taking a circular turn. Due to this, the reaction q q
on outer wheel is more than that on inner wheel. sin A
100. (b) The cyclist bends while taking turn in order to provide N
h
O q
necessary centripetal force. mg B
inside
101. (a) N cos q

trrack
Rail
N outside
q
f cos q mv 2
N sin q N sin q = ...................(ii)
r
f
q mg v2
& tan q = ....................(iii)
f sin q rg
where q is angle of banking of rail track, N is normal
Clearly form the figure, N sinq and f cosq reaction exerted by rail track on rail.
contribute to the centripetal force. It is clear from the equation (i) & (ii) that N cosq balance
2 the weight of the train & N sinq provide the necessary
\ N sin q + f cos q = mv centripetal force to turn.
R If width of track is l (OB) & h (AB) be height of outside
102. (c) It is a case of uniform circular motion in which of track from the inside, then
velocity and acceleration vectors change due to h v2 v2l
tan q = = or h = .................(iv)
change in direction. As the magnitude of velocity l rg rg
remains constant, the kinetic energy is constant. So it is clear from the above analysis that if we increase
v2 202 1 the height of track from inside by h metre then resultant
103. (d) Angle of banking is tan q = = =
rg 40 3 ´10 3 force on rail is provided by railway track & whose
104. (a) The velocity should be such that the centripetal direction is inwards.
acceleration is equal to the acceleration due to gravity 109. (c) Force exerted by a car when passes through a bridge
v2 Mv 2
= g or v = g R . F = Mg - .
R r
105. (d) tan q = v2 / rg, tan q = H / 1.5, r = 200 m, b = 1.5 m 110. (c) In uniform circular motion, the direction of motion
v = 36 km/hour = 36 × (5/18) = 10 m/s. changes, therefore velocity changes.
Putting these values, we get H = 0.075 m. As P = mv therefore momentum of a body also
changes uniform circular motion.
106. (b) 111. (b) For negotiating a circular curve on a levelled road, the
107. (a) The car over turn, when reaction on inner wheel of car maximum velocity of the car is vmax = mrg
is zero, i.e., first the inner wheel of car leaves the ground
(where G is C.G of car, h is height of C.G from the ground, Here m = 0.6, r = 150 m, g = 9.8
f1 & f2 are frictional force exerted by ground on inner & \ vmax = 0.6´150´9.8 ; 30m / s
outer wheel respectively). The max. speed for no over 112. (c) The tension T1 at the topmost point is given by
turning is m v12
T1 = - mg
20
G Centrifugal force acting outward while weight acting
N1 N2 downward.
m v 22
f1 B h The tension T2 at the lowest point T2 = + mg
A 20
gra inner wheel f2 mg outer wheel Centrifugal force and weight (both) acting downward
v max = of car of car m v 22 - m v12
h T2 - T1 = + 2mg
2a 20
v1 = v 2 - 2 g h or v 2 - v12 = 2 g ( 40) = 80 g
2 2 2

\ T2 - T1 = 80 m g + 2 m g = 6 m g
where r is radius of the path followed by car for turn & 20
113. (d) Given; speed = 10 m/s; radius r = 10 m
2a is distance between two wheels of car (i.e., AB)
Angle made by the wire with the vertical
108. (a) If the outside rail is h units higher than inside of rail
v2 102 p
track as shown in figure then tan q = = = 1 Þ q = 45° =
rg 10 ´ 10 4
N cos q = mg....................(i) 114. (b) The centrifugal force on the particle
2 2
= mass × acceleration of the frame = m ´ w 0 r = mw 0 r
EBD_7418
476 PHYSICS

115. (b) m mg = m v 2 / r or v = m g r The situation is shown in (figure) where a body A is in


unfirom translatory motion.
or v = ( 0 .25 ´ 9 .8 ´ 20 ) = 7 m / s 2. (b) According to question we have to apply Newton's
116. (a) r = 30 m and m = 0.4. second law of motion, in terms of force and change in
vmax = mrg = 0.4 ´ 30 ´ 9.8 = 10.84m/s momentum.

117. (b) v = gr = 10 ´ 40 = 20 m s -1 dp
We know that F =
dt
118. (c) Tension at the highest point As given that the meter scale is moving with uniform
mv2 velocity, hence
Ttop = – mg = 2mg (\ vtop = 3gr )
r Force (F) = m × 0 = 0
Tension at the lowest point No change in its velocity i.e., acceleration of it zero by
Tbottom = 2mg + 6mg = 8mg Newton's second law.
Ttop 2mg 1 Hence, net or resultant force must act on body is zero.
\ =
Tbottom 8mg 4
= . r r r so,
t = r ´ F,
119. (b) As all part of the scale is moving with uniform velocity
120. (d) The inclination of person from vertical is given by and total force is zero, hence, torque will also be zero.
v2 (10) 2 1 3. (c) As given that,
tan q = = = \ q = tan -1 (1/ 5)
rg 50 ´ 10 5 Mass of the ball = 150 g = 0.15 kg
121. (c ) r
122. (a) Given : Mass (m) = 0.4 kg u = (3iˆ + 4 ˆj ) m/s
r
Its frequency (n) = 2 rev/sec v = -(3iˆ + 4 ˆj ) m/s
Radius (r) =1.2 m. We know that linear velocity of the (Dp) Change in momentum
body (v) = wr = (2pn)r = Final momentum – Initial momentum
= 2 × 3.14 × 1.2 × 2 = 15.08 m/s. r r
= mv - mu
Therefore, tension in the string when the body is at r r
= m(v - u ) = (0.15)[ -(3iˆ + 4 ˆj ) - (3iˆ + 4 ˆj )]
the top of the circle (T)
mv2 0.4 ´ (15.08) 2 = (0.15)[-6iˆ - 8 ˆj ]
= - mg = - (0.4 ´ 9.8)
r 2 = -[0.15 ´ 6iˆ + 0.15 ´ 8 ˆj]
= 45.78 - 3.92 = 41.56 N
= -[0.9iˆ + 1.20 ˆj ]
123. (a) Since water does not fall down, therefore the velocity
of revolution should be just sufficient to provide Dp = -[0.9iˆ + 1.2 ˆj ]
centripetal acceleration at the top of vertical circle. So, Hence verifies option (c).
v = (g r ) = {10 ´ (1.6)} = (16) = 4 m/sec.
4. (c) From previous solution
124. (d) The particle is moving in circular path Dp = -(0.9iˆ + 1.2 ˆj) = -0.9iˆ - 1.2 ˆj
From the figure, mg = R sin q … (i) R Rsinq
2 2
2 q
Magnitude of | Dp |= (-0.9) + (-1.2)
mv
= R cos q … (ii) Rcosq = 0.81 + 1.44
r
From equations (i) and (ii) we get mg
h = 2.25 = 1.5 kg-m s–1
rg r q Verifies the option (c).
tan q = 2 but tan q = 5. (d) By Newton's second law :
v h
v 2
(0.5) 2 r dp
\h = = = 0.025m = 2.5cm Fext =
g 10 dt
r
v2 As Fext in law of conservation of momentum is zero.
125. (a) tan q = If Fext = 0, dp = 0 Þ p = constant
rg
2250 Hence, momentum of a system will remain conserve if
tan12° = (150)2 r ´ 10 Þ r= = 10.6 km
0.2125 external force on the system is zero.
In case of collision between particle equal and opposite
EXERCISE - 2 forces will act on individual particle by Newton's third
1. (c) In a uniform translatory motion if all the parts of the law.
r r r
body moves with (same velocity in same straight line, So, F12 = - F21 (Q Fext = 0)
so the velocity is constant. Total force on the system will be zero.
r r
= - 21 or dpr12 = -dpr 21
v dp12 dp
A v
v dt dt
r r
Þ (dp12 + dp21 = 0)
LAWS OF MOTION 477

So prove the law of conservation of momentum and r


Velocity acquired along east (v ) = viˆ
verifies the option (d). Time (t) = 2 s.
6. (c) Consider the adjacent diagram. We know that, v = u + at
N
Þ viˆ = 0 + a ´ 2
r v
Þ a = iˆ
2
P2 (Force, by engine is internal force)
B A r r mv ˆ
F = ma = i
P1 2
mv
( P2– P1) O Hence, force acting on the car is towards east due
B 2
W E mv ˆ
to force of friction is i which moves the car in
2
eastward direction. Hence, force by engine is internal
S force.
The force on player is due to rate of change of 10. (a) Let upthrust of air be Fa then
momentum. The direction of force acting on player will For downward motion of balloon
be the same as the direction of change in momentum. Fa = mg – ma
mg – Fa = ma
Let OA = P1 i.e., Initial momentum of player northward For upward motion
AB = P2 i.e., Final momentum of player towards west. Fa – (m – Dm)g = (m – Dm)a
Clearly, OB = OA + OB 2ma
Change in momentum Therefore Dm =
g+a
= P2 – P1 11. (c) Change in momentum,
= AB – OA = AB + (–OA)
= Clearly resultant AR will be along south-west. Dp = ò Fdt
So, it will be also the direction of force on player. = Area of F-t graph
7. (a) As given that, mass = 2 kg = ar of D – ar of + ar of
p = 3 m/s, q = 4 m/s2, r = 5 m/s3 1
= ´ 2 ´ 6 - 3 ´ 2 + 4 ´ 3 = 12 N-s
As given equation is 2
12. (c) Acceleration
x(t ) = pt + qt 2 + rt 3
Net force in the direction of motion
ds (t ) =
v= = p + 2qt + 3rt 2 Total mass of system
dt
m g - m(m 2 + m3 )g g
dv d 2 x (t ) = 1 = (1 - 2m )
a= = = 0 + 2q + 6rt m1 + m 2 + m3 3
dt dt 2 (Q m1 = m2 = m3 = m given)
é d 2 x (t ) ù 13. (a)
ê 2 ú
= 2q + 12r
14. (a) At limiting equilibrium,
ë dt û ( t = 2)
m = tanq
= 2q + 12r = 2 × 4 + 12 × 5 = 8 + 60 = 68 m/s2
r
Force acting on body ( F ) = ma m
= 2 × 68 = 136 N. y
8. (b) As given that mass = m = 5 kg q
r
Acting force = F = (-3iˆ + 4 ˆj ) N
r
Initial velocity at t = 0, u = (6iˆ - 12 ˆj ) m/s
r
F æ 3iˆ 4 ˆj ö
Retardation, aˆ = = ç - + ÷ m/s 2 dy x 2
m è 5 5 ø tanq = m = = (from question)
As the final velocity along Y-component only. So its x- dx 2
component must be zero. Q Coefficient of friction m = 0.5
From v x = u x + ax t , for X-component only,, x2
\ 0.5 = Þx=+1
-3iˆ 3iˆ 5´ 6 2
0 = 6iˆ + t Þ (t ) = 6iˆ Þ t = = 10 s
5 5 3 x3 1
t = 10 sec. Hence verifies the option (b). Now, y = = m
6 6
9. (b) As given that mass of the car = m 15. (d) According to question, two stones experience same
As car starts from rest, u = 0 centripetal force
i.e. FC1 = FC2
EBD_7418
478 PHYSICS

mv12 2mv 22 Adding eqs. (1) and (2), we get


or, = or, V12 = 4V22 (m1 + m2) a = m2g – m2 (g/2) = m2 g/2
r (r / 2)
So, V1 = 2V2 i.e., n = 2 m2 g
\ a = 2 (m + m )
16. (a) Coefficient of static friction, 1 2
f1 f2
1
ms = tan 30° = = 0.577 @ 0.6
3 N
1 2 1 1 22. (a) F
A B
S = ut + at Þ 4 = a(4)2 Þ a = = 0.5
2 2 2
[Q s = 4m and t = 4s given]
f1
0.9 20N 100N
a = gsinq – mk(g) cosq Þ mk = = 0.5 Assuming both the
f
3
F blocks are stationary
17. (a) Acceleration of system a = net N= F
M total
14 14
= = = 2 m / s2 f1 = 20 N
4 + 2 +1 7
f2 = 100 + 20 = 120N
14N A B 120N
4kg C Considering the two blocks as one system and due to
2kg 1kg
equilibrium f = 120N
The contact force between A and B 23. (a) Consider FBD of structure.
= (mB + mC) × a = (2 + 1) × 2 = 6N B
BH
18. (b) For the motion of both the blocks L
m1a = T – mkm1g
A BV
m2g – T = m2a 200N
a AH
T AV
mk m1g m1
Applying equilibrium equations,
mk Av + Bv = 200 N … (i)
AH = BH … (ii)
m2 From FBD of block B,
m g – m k m1g a BV
a= 2
m1 + m 2
m2g
æ m 2 g – m k m1g ö B
BH 60°
m2g – T = (m2) ç m + m ÷
è 1 2 ø NB
solving we get tension in the string
m m g (1 + m k ) g 60°
T= 1 2 FB
m1 + m 2 300N
BH + FB cos 60° – NB sin 60° = 0
mv - (- mv ) 2mv 2 ´ 0.5 ´ 2
19. (a) F = = = = 2 × 103 N NB cos 60° – BV – 300 + FB sin 60° = 0
t t -3
10 FB = 0.25 NB
1 2 BH – 0.74 NB = 0 … (iii)
20. (c) So by second equation of motion, we get S = ut + at
2 – BV + 0.71 NB = 300 … (iv)
here S = l, u = 0, a = g sinq FBD of block A
2l 2h 1 2h æ hö AV
t= = = çèQ sin q = ÷ø
a 2
g sin q sin q g l
q A AH
si n
g
g cosq

h FA
g
q
400N
NA
21. (a) Let a be the acceleration of mass m 2 in the downward FA – AH = 0
direction. Then NA – AV = 400 … (v)
T – m2 (g/2) = m1 a ....(i) FA = µA NA
and m2 g – T = m2 a ....(ii) \ µANA – AH = 0 … (vi)
LAWS OF MOTION 479

On solving above equations, we get According to question, t 2 = nt1


NA = 650 N, FA = 260N, FA = µANA
2d 2d
260 n =
\ µA = = 0.4 g sin q g sin q - mg cos q
250
24. (d) For the blocks A and B FBD as shown below m, applicable here, is coefficient of kinetic friction as
mg mg N the block moves over the inclined plane.
F
1
A n=
B 1- mk
æ 1 ö
F N N¢ ççQ cos 45° = sin 45° = ÷÷
è 2ø
Equations of motion 1 1
n2 = or 1 - m k = 2
F 1- mk n
a A = ( in - x direction )
M 1
or m k = 1-
F n2
a B = ( in + x direction )
M
Relative acceleration, of A w.r.t. B, 2 1
F F 28. (d) m AW = ; m BW = .
a A,B = a A - a B = - - 3 3
m M
æ M + m ö
= -F ç ÷ (along – x direction) T T
è Mm ø
Initial relative velocity of A w.r.t. B, m B NB
2mg
uAB = v0 2m
A 2
using equation v2 = u2 + 2as mg
Mmv 20 mg 2 2mg 2mg
2F ( m + M ) S mg
0= - v 20 Þ S= 2 45° 45°
Mm 2F ( m + M ) 2
i.e., Distance moved by A relative to B Diagram shows the various forces acting on the masses
Mmv 02 and their resolution in the direction of motion. Let us
SAB =
2F ( m + M ) consider the two masses to be a system. The forces
25. (b) On a banked road, trying to move the system such that A moves upwards
2 and B moves downwards
Vmax æ m + tan q ö
=ç s ÷ =
2mg mg mg
- =
Rg è 1 - ms tan q ø 2 2 2
Maximum safe velocity of a car on the banked road The forces trying to stop this motion (i.e., maximum
frictional force)
é m + tan q ù
Rg ê s
Vmax = ú = fA + fB = m A N A + m B N B
ë1 - ms tan q û
2 mg 1 2mg 4 mg
26. (d) To complete the loop a body must enter a vertical loop =´ + ´ =
3 2 3 2 3 2
of radius R with the minimum velocity v = 5gR . Since the stopping force is more therefore the mass
g sin q - mg cos q system will not move
27. (b) Þ acceleration of the system is zero.
d q 29. (c) Acceleration of body along AB is g cos q
in
gs d 1 2
45° 45° Distance travelled in time t sec = AB = ( g cos q)t
smooth 2
rough From DABC, AB = 2R cos q
When surface is When surface is 1
smooth rough Thus, 2 R cos q = g cos qt 2
2
1
d= (g sin q)t 12 , 4R R
2 Þ t2 = Þt = 2
g g
1 2
d = (g sin q - mg cos q) t 2 30. (d) Net force on particle in uniform circular motion is cen-
2
æ mv2 ö
2d 2d tripetal force ç l ÷ which is provided by tension in
t1 = , t2 = è ø
g sin q g sin q - mg cos q
string so the net force will be equal to tension i.e., T.
EBD_7418
480 PHYSICS

31. (c) Frictional force is always opposite to the direction of Force and impulse are in the same direction the force
motion on wall due to the ball is normal to the wall along
positive x–direction in both (a) & (b) case.
N 38. (b) The apple will fall slightly away from the hand of his
brother in the direction of motion of the train due to
W E inertia of motion. When train is just going to stop,
the boy and his brother slows down with train but
the apple which is in free fall continue to move with
S the same speed and therefore, falls slightly away
F from the hand in the direction of motion of the train.
32. (b) As we know, coefficient of friction m = 39. (b)
N
40. (b) Given : m1 = 5kg; m2 = 10 kg; m = 0.15
ma a FBD for m1, m1g – T = m1a Þ 50 – T = 5 × a
Þm= = (a = 7.35 m s–2 given)
mg g and, T – 0.15 (m + 10) g N
= (10 + m)a m
7.35 T
\m= = 0.75 For rest a = 0
m2
9.8 m(m+m2)g (m+m2)g T
33. (d) The man can exert force on block by pulling the rope. or, 50 = 0.15 (m + 10) 10
3 m1
The tension in rope will make the man move. Hence Þ 5 = (m + 10)
Assertion is incorrect. 20 m1g = 50N
34. (a) Before cutting the string 100
= m + 10 ; close to option (b) \ m = 23.3kg
kx = T + 3 mg ...(i) 3
T = mg ...(ii) 41. (a) After the stone is thrown out of the moving train, the
Þ kx = 4mg m only force acting on it is the force of gravity i.e. its weight.
\ F = mg = 0.05 × 10 = 0.5 N.
42. (c) Impulse experienced by the body
After cutting the string T = 0
= MV – (–MV) = 2MV.
4mg - 3mg 4mg mg
aA =
3m
EXERCISE - 3
g
aA =
3 1. (a) T = T1 + T2 = m1 (g + a) + m2g
mg = 10 (10 + 2) + 8 (10) = 120 + 80 = 200N
and a B = =g 2. (a) Let a be the acceleration of the rope and M be its total
m
35. (c) Coefficient of friction or sliding friction has no mass. Then
dimension. l a a
F
T F
f L L-l T l
f = msN Þ ms =
N M
T= ( L - l) a ... (i)
36. (c) Let the mass of block is m. It will remains stationary if L
forces acting on it are in equilibrium. i.e., ma cos q = mg sin M
and F – T = ´l a ... (ii)
q Þ a = g tan q L
Dividing (i) and (ii)
ma cos q
F -T l
ma = Þ F (L – l) – T (L – l) = Tl
q T L-l
mg sin q
æ ö l
Þ F (L – l) = T (L – l + l) = T × L Þ T = F ç1 - ÷
a L è ø
mg m m
q 3. (c) g –T = a ....(i)
2 2
Here ma = Pseudo force on block, mg = weight. ma
37. (d) Case (a) T cos60° = ...(ii)
cos 60°
(Px)i = mu Py(initial) = 0 Solving (i) and (ii)
(Px)f = f = –mu Py(final) = 0 2g
Impulse = DP = –2mu (along x –axis) acceleration of ring =
9
Impulse = 0 along y–axis
parallaly in case (b) 4. (c)
(Px)i = mu cos30° (Py)i = –mu sin30°
(Px)f = f = –mu cos30° (Py)f = –mu sin30° T
\ Impulse = –2mu cos 30°(along x-axis)
ma m a
Impulse = 0 (along y–axis)
a
mg
LAWS OF MOTION 481

(Force diagram in the frame of the car) At equilibrium, mass does not move so,
Applying Newton’s law perpendicular to string 3mg sinq = m3mg cosq
a [ μ min < tan θ
mg sin q = ma cos q Þ tan q = 9. (c) If x is the extension, then
g
Applying Newton’s law along string mg cosq

Þ T - m g 2 + a 2 = ma or T = m g 2 + a 2 + ma
5. (d) Case I : q
T – N = 40 a T (mg cos q) sin q = kx
and 20g – T = 20 a
T mg sin 2q
Also N = 20 a \x=
After simplifying, we get T A 2k
N
20g 10. (b) y
g
a=
4
g a x
Acceleration of block B, = 2a = .
2 2
Case II : T x2 = 4a (a) [since y = a]
Þ x2 = 4a2 Þ x = ± 2a
T The slope of curve at point (2a, a)
2x 2
A m = tan q = = ´ 2a = 1
4a 4 a
20g
g
so, tangential acceleration = g sin q =
T =40 a 2
and 20 g – T =20 a 11. (c) 12. (a)
After simplifying above equation, we get 13. (b) mg = mN =m ma
a =g/3 g
g/2 2 3
\ a= m .
Ratio = = .
g /3 2 2 B T2 T1 T1
14. (a)
6. (d) Perpendicular to surface, their acceleration components l 2l
must be equal. T1 = mw2(3l)
aA cos q = aB sin q and T2 – T1 = mw2l
21 a A = 2a B or T2 = T1 + mw2l = mw2(3l) + mw2l
aAcos q aA = 4mw2l
0.5 T2 4
2 \ T1
= .
aA q 3
90° – q
21
15. (c) N = Mg – f sin q ...(i)
and f =mN ...(ii)
N F sinq
aAsin q
Fcosq
7. (c) a = b + c f F
Net acceleration of A = a 2 + c 2 + 2ac cos( p - q) mg
Now
= (b + c ) 2 + c 2 - 2(b + c ).c.cos q (F + F cos q) – f = Ma ...(iii)
8. (b) Let m be the minimum coefficient of friction After solving above equations, we get
a = 12 m/s2.
m3mgcosq

3mgsinq 3g
EBD_7418
482 PHYSICS

6. WORK, ENERGY AND POWER

Exercise - 1 = area of D + area of rectangle + area of D


5´ 3 5´ 3
1. (b) When force retards motion i.e., F –(ve) so, work = + 10 ´ 3 + = 45 J
done –(ve) 2 2
2. (c) When a man pushes a wall and fails to displace it, 18. (b) In close loop, s = 0, and so W = Fs = 0.
then displacement of wall = 0 dx
19. (a) x = 3t –4t2 + t3 = 3 - 8t + 3t 2
\ Work done by man = F × 0 = 0 dt
Therefore, man does no work at all.
3. (a) When a person carrying load on his head moves over d 2x
a horizontal road, work done against gravitational force Acceleration = = -8 + 6 t
dt 2
is zero.
4. (d) W = FS cos q Acceleration after 4 sec = –8 + 6 × 4 = 16
\ If F = 0; W= 0 Displacement in 4 sec = 3 ×4 – 4 × 42 + 43 = 12 m
If S = 0; W= 0 \ Work = Force × displacement
& if q = 90°; cos 90° = 0 = Mass × acc. × disp. = 3 × 10–3 × 16 × 12 = 576 mJ
\ W = 0.
5. (c) Displacement of the particle when it takes a complete 20. (c) As the cord is trying to hold the motion of the block,
round the circular path is zero. work done by the cord is negative.
\ Work done = force × displacement æ gö - 3M gd
W=F×0=0 W = – M (g – a) d = - M ç g - ÷ d =
Therefore, work done by the force is zero. è 4ø 4
6. (b) (A) ® (1); (B) ® (1); (C) ® (3); (D) ® (3) 21. (a) In this case, s = 0, and so W = 0.
22. (d) Though an equal and opposite force acts on the road
7. (b) W = 1 kl 22 - 1 kl 12 = 1 k (l 22 - l 12 )
2 2 2 2 but since road does not undergo any displacement,
d s 2 hence no work is done on the road.
8. (b) Acceleration = 2 =
dt 3 23. (c) Motion without slipping implies pure rolling. During
2 pure rolling work done by friction force is zero.
Force acting on the body = 3 ´ = 2 newton
3 1 3 2 1 3 2
1 4 24. (a) W1 = ´ 5 ´ 10 (0.05) Þ W2 = ´ 5 ´ 10 (0.10)
Displacement in 2 secs = ´ 2 ´ 2 = m 2 2
3 3 1
4 8 \ DW = ´ 5 ´103 ´ 0.15 ´ 0.05 = 18.75J.
Work done = 2 ´ = J 2
3 3 r
x1 x1 x 25. (d) Given : F = 3i$ + $j
é1 ù 1 ur r
$ $ uur
9. (b) W= ò ò
F dx = c x dx = ê c x 2 ú
ë2 û0 ( ) $ $
r1 = 2i + k , r2 = 4i + 3j - k ( )
0 0
r uur ur r
1 2 1 2
= c ( x1 - 0) = c x 1 (
$ $
r = r2 - r1 = 4i + 3j - k – 2i + k
$ $
) ( )
2 2 r
10. ( a) or r = 2i$ + 3j$ – 2k$
11. (b) (A)®(3); (B)®(1); C®(2,3); (D)®(1) rr
So work done by the given force w = f .r
12. ( c) 13. ( a)
rr
14. (c) W = F.s = (5 î + 2 ˆj).(2 î + 3 ĵ) = 10 + 6 = 16 J. ( )( )
= 3i$ + $j . 2i$ + 3j$ - 2k$ = 6 + 3 = 9J
15. (b) F(N) 26. (c) here, m = 4 , g = 4 × 10–3 kg
x = 4t 2 + t
A B dx d2 x
3 = 8t + 1 =8
\
2 dt dt 2
1 C d 2 x æ dx ö
x(m) Work done, W = ò fdx = ò m ç ÷ dt
0 2 dt 2 è dt ø
1 2 3 4 5 6 7
= ò (4 ´ 10-3 )(8)(8 t + 1) dt
Work done = area under F-x graph
0
1 2
2 é 2 ù
-3 8t
= area of trapezium OABC = ( 3 + 6)( 3) = 13.5 J = 32 ´ 10 -3 ò (8t + 1)dt = 32 ´ 10 ê + tú
2 ê 2 úû 0
16. (a) W = F s cos 90º = zero 0 ë
17. (c) W = area of F – x graph = 32 × 10–3 [4(2)2 + 2 – 0] = 576 mJ
WORK, ENERGY AND POWER 483

27. (c) W = F s cos q = 10 × 2 cos 60º = 10 J. Therefore, increase in kinetic energy is 2.25 K – K
1
28. (b) Workdone, W = k x 2 - x1
2
2
(
2
) 44. (c)
= 1.25 K or 125%.
Weight Mg moves the centre of gravity of the spring
1 1 (0 + l)
= k é( 0.15) - (0.05) 2 ù = ´ 800 ´ 0.02 = 8J
2
through a distance = l/2
2 ë û 2 2
q =
W
=
25 1
= , q = 60º.
\ Mechanical energy stored = Work done = Mg l/2.
29. (c) W = F s cos q, cos 45. (a) U1 = mgh1 and U2 = mgh2
F s 5 ´ 10 2
30. (a) Work done by the net force = change in kinetic energy U - U2
% energy lost = 1 ´100
of the particle. U1
This is according to work energy theorem. mgh1 - mgh 2 æ h - h2 ö
31. (a) Since momentum of both bodies are equal = ´ 100 = ç 1 ÷ ´100
mgh1 è h1 ø
M u 2 - 1.5
So p1= p2 Þ 1 = 2 Þ u 2 > u1 (let M1>M2) = ´ 100 = 25%
M 2 u1 2
46. (d) Work done on the body is gain in the kinetic energy.
E k1 P12 / 2M1 M2 Þ E < E Acceleration of the body is a = V/T.
so = = k1 k2 V
P22 / 2M 2
E k2 M1 Velocity acquired in time t is v = at = t
T
It means that light body has greater kinetic energy, if V2t 2
2
K.E. acquired µ v . That is work done µ
they have equal momentum.
T2
32. (d) Let the velocity of the particle be v m/s. 3/2
47. (b) At x = 0, vi = 5 ( 0) = 0
Momentum of the particle (p) = mv
and x = 2 m, v f = 5 ( 2 )
3/2
Kinetic energy of the particle = 5 8 m/s

( ) (
é
) 2ù
2 2 1 1 2
1 1 (mv ) p
(E) = mv 2 = . Þ E= W = DK = m v 2f - vi2 = ´ 0.5 ê 5 8 - 0 ú
2 2 m 2m 2 2 ë û
= 50 J
1 2
33. (b) K. E = mv It is always positive P2 1
2 48. (b) E = \Eµ [If P = constant]
34. (b) 2m m
35. (a) We define the potential energy V(x) of the spring to be i.e. the lightest particle will possess maximum kinetic
zero when block and spring system is in the equilibrium energy and in the given option mass of electron is
position. minimum.
36. (a) Conservative force is negative gradient of potential 49. (d) When work is done upon a system by a conservative
–dV(x) force then its potential energy increases.
F(x) = 50. (a)
dx 51. (c) If there were no friction, moving vehicles could not be
1 stopped by looking the brakes. Vehicles are stopped
37. (a) For a given spring, u = kx 2 by air friction only. So, this statement is correct.
2
1 2
Kx 1
u2 2
\ u = 1
2
=
(3x )2 = 9 : 1 52. (d) K.E. = mv
2
2
1 Kx 2 x2
2 1 Further, v 2 = u 2 + 2as = 0 + 2ad = 2ad = 2(F / m) d
38. (a)
39. (b) The principle of conservation of total mechanical 1
Hence, K.E. = m ´ 2(F / m ) d = Fd
energy can be stated as, the total mechanical energy 2
of a system is conserved if the forces, doing work on or, K.E. acquired = Work done = F × d = constant.
it, are conservative. i.e., it is independent of mass m.
40. (c) Only momentum is conserved. Some kinetic energy 53. (c) Let m = mass of boy, M = mass of man
is lost when bullet penetrates the block. v = velocity of boy, V = velocity of man
41. (a) If an object of mass m is released from rest from top of 1 1 é1 ù
a smooth inclined plane, its speed at the bottom is MV 2 = ê mv 2 ú ...(i)
2 2 ë2 û
2gh , independent of angle q and mass.
1 é1 ù
42. (b) Energy required = mgh M (V + 1) = 1 ê mv 2 ú
2
In both cases, h is the same. Hence energy both is the ...(ii)
2 ë2 û
same.
43. (c) Initial momentum (p1) = p; Final momentum (p2) = 1.5 p M 1
Putting m = and solving V =
and initial kinetic energy (K1) = K. 2 2 -1
p2 2
54. (b) v2 = u2 + 2gh = (10) + 2 ´10 ´19.5 = 490
Kinetic energy ( K ) = µ p2
2m K.E. at the ground
2 2
K1 æ p1 ö æ p ö 1 1 2 1 5 49
or, = =ç = or, K2 = 2.25 K. = mv = ´ ´ 490 = J
K 2 çè p2 ÷ø è 1.5 p ÷
ø 2.25 2 2 1000 40
EBD_7418
484 PHYSICS

5 æ - 50 ö 1 1
P.E. = mgh = ´ 10 ´ ç ÷=- J \ K.E. = mv 2
1000 è 100 ø 40 2
1
\ Change in energy =
49 æ 1 ö 50
-ç- ÷ = = 1.25J If mass is 40 kg then, K.E. = ´ 40 ´ (10) 2 = 2000 J
40 è 40 ø 40 2
If mass is 100 kg then,
dU 1
55. (d) |F | = , which is greatest in the reagion CD. K.E. = ´ 100 ´ (10) 2 = 5000 J
dx
56. (d) 2
dk
p 2 Þ E1 = p12 66. (d) Given, = constant
57. (c) E= Þ E 2 = E1 ´ 4 dt
2m E2 p 22
Þ kµt Þ vµ t
\ E 2 - E1 = 3E1
dk
Also, P = Fv = = constant
1 2 1 2 dt
58. (c) As mA v A = mB v B
2 2 1 1
Þ Fµ Þ F µ
vA mB v t
= ;
vB mA p2
67. (d) Kinetic energy of a body, K =
PB mB vB mB mA mB 1 2m
= = = = As p1 = p2 (Given)
PA mA vA mA mB mA 3 K m 5
x x
\ 1 = 2 =
1 K 2 m1 4
59. (a) U=-
ò 0
Fdx = -
ò
0
kxdx = - kx 2 .
2 68. (d) Here, F = 100 N, d = 5 m,
It is correctly drawn in (a) frictional force fr = 40 N
60. (d) At the lowest point, h = 0 \ P.E. = 0 (gravitational \ F – fr = ma
P.E.). There is no work done on the bob by the tension 100 – 40 = ma
as it is perpendicular to the displacement. Now kinetic energy gained is = ma × d
\ Potential energy is associated only to the gravitational = 60 × 5 = 300 J
force. 69. (a) This condition is applicable for simple harmonic
61. (b) Total energy at the time of projection motion. As particle moves form mean position to
1 1 extreme position its potential energy increases
= m v 2 = ´ 0.1(20) 2 = 20J 1 2
2 2 according to expression U = kx and according
Half way up, P.E. becomes half the P.E. at the top i.e. 2
20 kinetic energy decreases.
P.E. = = 10J \ K.E. = 20 – 10 = 10J. 70. (d) For any uniform rod, the mass is supposed to be
2 concentrated at its centre.
62. (b) At th e top of flight, horizontal component of \ height of the mass from ground is, h = (l/2) sin 30º
velocity = u cos 45º = u / 2 \ Potential energy of the rod
l
1 æ u ö 1 æ mu 2 ö 1
2 = m ´ g ´ sin 30º l
\ K.E. = m = ç ÷ = K. 2 60º
2 çè 2 ÷ø2 çè 2 ÷ø 2
l 1 mgl
m
=m´g´ ´ = 30º
1- 0 1 2 2 4
63. (c) The uniform acceleration is a = = ms -2
15 15
2 m
Let v be the velocity at kinetic energy J
9 l/2
h
1 2 2 2 -1
30º
therefore ´1 ´ v = or v = ms
2 9 3 1 1
mv2 = kx 2 Þ mv = kx or m × (1.5) = 50 × (0.15)
71. (a) 2 2 2 2
Using v = u + at
2 2
2 1 \ m = 0.5 kg
= 0 + ´ t Þ t = 10s
3 15 72. (b) Let the blow compress the spring by x before stopping.
64. (c) v2 = - m1 v1 = -3 ´ 16 = -8 m / s Kinetic energy of the block = (P.E of compressed spring)
m2 6 + work done against function.
1 1
1
E2 =
1
m 2 v 22 = ´ 6(-8) 2 = 192 J ´ 2 ´ (4) 2 = ´ 10, 000 ´ x 2 + ( +15) ´ x
2 2 2 2
65. (d) The average speed of the athelete 10,000 x2 + 30x – 32 = 0
100 Þ 5000x 2 + 15x - 16 = 0
v= = 10m / s
10
WORK, ENERGY AND POWER 485

1 1 æVö
15 ± (15)2 - 4 ´ (5000)(-16) V¢ = k (10) 2 = ´ ç ÷ (10) 2 = 25V
\ x =- 2 2 è2ø
2 ´ 5000 dU
= 0.055m = 5.5cm. 79. (d) F = - = b - 2ay
73. (d) FP = – kI xP dy
80. (b) Applying momentum conservation
FP k P x P
FQ = – kQxQ or F = k x m1u1 + m2u2 = m1v1 + m2v2
Q Q Q 0.1u + m(0) = 0.1(0) + m(3)
xP kQ 0.1u = 3m
FP = FQ (Given) \ x =k
Q P 1 1
0.1u 2 = m(3) 2
1 2 2 2
Energy stored in a spring is U = kx
2 Solving we get, u = 3
2 2 2
UP kPxP k k Q 1 2 1 æ xö 1
\ U = = P´ kx = K ç ÷ + (0.1)32
2 k 2
Q kQ xQ Q kP 2 2 è2ø 2
UP kQ 1 3 2 3 1
= = æ kP ö Þ kx = 0.9 Þ ´ kx 2 = 0.9
UQ k P 2 çQ k Q = ÷ 4 2 2
è 2 ø
1 2
UQ E \ Kx = 0.6 J (total initial energy of the spring)
or UP = = (Q UQ = E) 2
2 2 81. (a) Loss in K.E = Area under the curve
74. (c) From Hooke’s law
F µ x Þ F = kx., where k is spring constant mv2 = æç 1 mv 2 ö÷ 2 = 2K = 2aS
82. (d) Centripetal force =
Since force is same in stretching for both spring so R è2 øR R R
F=k1x1 = k2x2 Þ x1<x2 because k1>k2 83. (c) U = (1/ 2)Mv 2
1 84. (b) As we know, dU = F.dr
so work done in case of first spring is W1 = k1x12
2 r
ar 3
and work done in case of second spring is U = ò ar 2 dr = ...(i)
1 W x 3
0
W2 = k 2 x 22 so 1 = 1 Þ W1 < W2
2 W2 x 2 mv 2 2 2 3
It means that more work is done in case of second
As, = ar 2 = m v = mar
r
spring (work done on spring is equal to stored elastic 1 3
potential energy of the spring) or, 2m(KE) = ar ...(ii)
2
75. (a) Stored elastic potential energy of spring =½kx2 where
Total energy = Potential energy + kinetic energy
x is compression or elongation of spring from its natural
Now, from eqn (i) and (ii)
length. In this position the spring can do work on the
block tied to it, which is equal to ½kx2, so both option ar 3 ar 3 5 3
(a) & b are correct. Total energy = K.E. + P.E. = + = ar
3 2 6
1 85. (c) Let E be the total energy then
76. (d) P.E. = kx2
2 mgh
P.E 2 5
ÞE=
\ If x = 4x, then P.E = k(16x2) = 16 æ kx 2 ö
1 1 = = mgh
K .E E - mgh 3 2
2 è2 ø
When velocity is double then inital energy becomes
77. (b) 4E.
mgh mgh
100 So, = NL =
30 4 E - mgh 10 mgh - mgh
20
P.E 1
mgH 1 On solving we get = .
mv 2 + mgh K .E 9
2
Using conservation of energy, l
3
m 1
æ1 ö
m (10 × 100) = m ç v 2 + 10 ´ 20 ÷
86. (a) U1 = ò – l gxdx = – 18 mgl;
0
è 2 ø
1 2 l
or v = 800 or v = 1600 = 40 m/s m 1
2 U2 = ò – gxdx = – mgl
l 2
1 1 2V V 0
78. (d) V = k ( x ) 2 = k (2) 2 or k = =
2 2 4 2
EBD_7418
486 PHYSICS

® ®
4 96. (d) The power of body is given by = F . v as the body is
los s in P.E. = U1 – U 2 = mgl
9 moving in circular path, centripetal force and velocity
are at 90°, or power = 0.
4 8
= ´ 0.1 ´ 10 ´ 2 = J = Final K.E. 97. (b) The work is done against gravity so it is equal to the
9 9
change in potential energy. W = Ep = mgh
87. (a)
88. (d) Power is defined as the rate of doing work. For the For a fixed height, work is proportional to weight lifted.
automobile, the power output is the amount of work Since Johnny weighs twice as much as Jane he works
done (overcoming friction) divided by the length of time twice as hard to get up the hill.
in which the work was done. Power is work done per unit time. For Johnny
89. (b) u = 0; v = u + aT; v = aT this is W/Dt. Jane did half the work in half the
2 time,
Instantaneous power = F × v = m. a. at = m. a . t
(1/2 W)/(1/2 Dt) = W/Dt which is the same power
v2 delivered by Johnny.
\ Instantaneous power = m t
T2 dv
90. (b) Constant power of car P0 = F.V = ma.v 98. (a) Power, P = F.v = m .v
dt
dv v
P0 = m .v = constant
dt As P is constant,
dt
P0 dt = mvdv Integrating Þ v µ dt Þ v µ t
mv 2 2P0 t work done
P0 .t = v= 99. (c) Power =
2 m time
Q P0 , m and 2 are constant \ vµ t mgh
91. (d) Therefore power of A, PA = t
A
W mgh
92. (b) P= . Here, P = 2kW = 2000 W.. and power of B, PB = t
t B
W = Mgh = M × 10 × 10 = 100 M and t = 60 s.
PA t B 4
This gives, M = 1200 kg \ = = = 2 :1
Its volume = 1200 litre as 1 litre of water contains 1 kg PB t A 2
of its mass. 100. (c) Volume of water to raise = 22380 l = 22380×10–3m3
93. (a) mgh V rgh V rgh
P= = Þt=
94. (b) We know that F × v = Power t t P
\ F ´ v = c where c = constant 22380 ´10-3 ´ 103 ´ 10 ´10
t= = 15 min
dv æ mdv ö 10 ´ 746
\m ´v = c çè\ F = ma = ÷
dt dt ø 101. (d) Power = total work done
v t
1 time
\ mò vdv = c ò dt \ mv 2 = ct 1
2 Mv 2
0 0 2 1 æ M ö
= = (mv 2 )n çQ = mn ÷
2c 1 t 2 è t ø
\ v= ´t 2
m é 1 ù
= kn êQK.E. K = mv2 ú
dx 2c 1 2 dx ë 2 û
\ = ´t where v =
dt m dt 102. (b) P = 10 kW = 10000 W
x t Fuel consumption = 1g/s
2c 1
Calorific value = 2 kcal/g
\ ò dx = m
´ ò t 2 dt
\ Energy produced = 2 kcal/s
0 0
3 Input power
2c 2t 2 3 = 2 kcal/s = 2000 cal/s = 2000 × 4.18 J/s = 8.4 k W
x= ´ Þ xµt 2
m 3 \ This claim is invalid.
95. (b) Power exerted by a force is given by
P = F.v 103. (b) Power =
Work done 2
1
(
m v2 - u 2 )
=
When the body is just above the earth’s surface, its Time t
velocity is greatest. At this instant, gravitational force
is also maximum. Hence, the power exerted by the
P= ´
6 é
1 2.05 ´10 ´ ë( 25 ) - 5 û
2
( )
2 ù

gravitational force is greatest at the instant just before


2 5 ´ 60
the body hits the earth.
P = 2.05 × 106 W = 2.05 MW
WORK, ENERGY AND POWER 487

v1
104. (d) m = 10 ´ 0.8kg = 8kg
A
height of iron chain = 5m y
mgh 8 ´ 10 ´ 5 A u1 B f
P= = W = 40W 117. (c)
t 10 m m q
105. (c) The law of conservation of momentum is true in all type x
of collisions, but kinetic energy is conserved only in
elastic collision. The kinetic energy is not conserved in B
v2
inelastic collision but the total energy is conserved in all (a) By law of conservation of momentum
type of collisions. (i) along x axis
106. (c) In a perfectly inelastic collision, the two bodies move
mu1 + 0 = mv1 cos f + mv 2 cos q .........(i)
together as one body.
(ii) along y axis
107. (b) In an inelastic collision, momentum remains conserved,
but K.E is changed. 0 = mv1 sin f - mv 2 sin f . ........(ii)
– (v1 - v2 ) -velocity of separation (b) By law of conservation of energy
108. (a) Since e = =
(u 2 - u1 ) velocity of approach 1 1 1 2 2 2
mu12 = mv12 + mv22 Þ u1 = v1 + v 2 .......(iii)
(i) For perfectly elastic collision e =1 2 2 2
(ii) For perfectly inelastic collision e = 0 From eq (i) & (ii) we get
(iii) For other collision 0 < e < 1 u12 = v12 + v22 + 2v1v 2 cos(j + q) …… (iv)
109. (b) For a perfectly inelastic collision, e = 0.
110. (c) Now use eq.(iii) in eq.(iv), we get
111. (a) During elastic collision between two equal masses, cos( f + q) = 0 Þ q + f = 90º
the velocities get exchanged. Hence energy transfer
ie, after collision they will move in mutually
is maximum when m1 = m2.
112. (a) As the floor exerts a force on the ball along the normal, perpendicular directions.
& no force parallel to the surface, therefore the velocity 118. (d) The billiard balls in an elastic collision are in a
component along the parallel to the floor remains deformed state. Their total energy is partly kinetic
constant. Hence V sin q = V1 sin q1. and partly potential. So K.E. is less than the total
113. (b) energy. The energy spent against friction is dissipated
as heat which is not available for doing work.
114. (b) The law of conservation of energy is valid at any
119. (a) As u2 = 0 and m1 = m2, therefore from
instant & in all circumstances.
115. (b) According to principle of conservation of energy m1 u1 + m2 u2 = m1 v1 + m2 v2 we get u1 = v1 + v2
Potential energy = kinetic energy v - v1 v 2 - v1 1 - v1 / v 2
Also, e = 2 = = ,
u1 v 2 + v1 1 + v1 / v 2
1
Þ mgh = mv2 Þ v = 2gh v 1- e
2 which gives 1 =
v2 1 + e
If h 1 and h2 are initial and final heights, then
120. (c) Apply conservation of momentum,
Þ v1 = 2gh 1 , v 2 = 2gh 2 m1v1 = (m1 + m2)v
m1 v1
Loss in velocity, Dv = v1 – v 2 = 2gh 1 – 2gh 2 v=
(m1 + m 2 )
\ fractional loss in velocity Here v1 = 36 km/hr = 10 m/s,
m1 = 2 kg, m2 = 3 kg
Dv 2gh1 – 2gh 2 h2 10 ´ 2
= = =1 – v= = 4 m/s
v1 2gh1 h1 5
1 2
1.8 2 K.E. (initial) = ´ 2 ´ (10) = 100 J
=1– = 1 – 0.36 = 1 – 0.6 = 0.4 = 2
5 5
116. (a) In an elastic collision 1 2
K.E. (Final) = ´ (3 + 2) ´ (4) = 40 J
2
(m1 - m 2 ) Loss in K.E. = 100 – 40 = 60 J
V1 = u Alternatively use the formula
m1 + m 2 1
1 m1m2
-DE k = ( u1 - u 2 )2
2m1u1 2 ( m1 + m 2 )
V2 =
m1 + m 2 121. (a) Clearly v1 = 2 ms –1, v2 = 0
m1 = m (say), m2 = 2m
2 m1v1
\ if m1 = m2, then V1 = 0; and V2 = = u1 v1' = ?, v'2 = ?
2m1
EBD_7418
488 PHYSICS

v1 '- v2 ' Here, m1 = 2.0 kg, m2 = M


e= v -v ....(i) 3 4 ´ 2M 2
2 1 \ = Þ M = kg or 6kg
4 (2 + M) 2 3
By conservation of momentum,
1 2 1 20
2m = mv1' + 2mv2' ... (ii) 129. (a) Initial, K.E. = mv = ´ ´ 600 × 600 = 3600 J
2 2 1000
v2 '- v1 ' Change in K.E. = P.E.
From (i), 0.5 = 1
2 m (v 2 - v' 2 ) = mgh
\ v 2' = 1 + v1' 2
1 20
From (ii), 2 = v1'+ 2 + 2 v1' Þ 3600 - ´ ´ v12 = 4 × 10 × 80
2 1000
Þ v1 = 0 and v2 = 1 ms–1 Þ v1 = 200 m / s
1 2 1 2
122. (b) Mv = k L 130. (c) Initial kinetic energy of the system
2 2
1 1 1
M K.Ei = mu 2 + M(0) 2 = ´ 0.5 ´ 2 ´ 2 + 0 = 1J
k 2 2 2
Þv= .L
M For collision, applying conservation of linear
momentum
k
Momentum = M × v = M × .L = kM . L m × u = (m + M) × v
M
123. (a) \ 0.5 ´ 2 = (0.5 + 1) ´ v
124. (c) 2
Þ v= m/s
125. (b) m1v1 + m2 v2 = ( m1 + m2 ) vsys. 3
20 ´ 10 + 5 ´ 0 = ( 20 + 5) vsys Þ vsys = 8m / s Final kinetic energy of the system is
1 1 2 2 1
1 K.E f = (m + M)v 2 = (0.5 + 1) ´ ´ = J
K. E. of composite mass = ( 20 + 5 ) ´ ( 8 ) = 800 J
2
2 2 2 3 3 3
126. (a) As the two masses stick together after collision, hence æ 1ö
\ Energy loss during collision = ç1 - ÷ J = 0.67J
it is inelastic collision. Therefore, only momentum is è 3ø
conserved.
2v Exercise - 2
1. (b) When electron and proton are moving under influence
of their mutual forces, then according to the flemings
m v x left hand rule, the direction of force acting on a charge
3m particle is perpendicular to the direction of motion.
r
\ mviˆ + 3m(2v)ˆj = (4m)v In magnetic field, work-done = F. s. cosq
r v 6 = F . s. cos 90° = 0.
v = ˆi + vjˆ
4 4 So magnetic forces do not work on moving charge
vˆ 3 ˆ particle.
= i + vj
4 2 2. (c) Forces between two protons is same as that of between
1 2 proton and a positron.
127. (d) K. E. of colliding body before collision = mv As positron is much lighter than proton, it moves away
2
After collision its velocity becomes through much larger distance compared to proton.
( m - m2 ) v = m v = v
v' = 1 Work done = Force × Distance
( m1 + m2 ) 3m 3 As forces are same in case of proton and positron but
1 mv 2 distance moved by positron is larger, hence, work done
\ K. E. after collision = on positron will be more than proton.
2 9 1 2
mv 3. (d) When the man squatting on the ground he is tilted
K .E.before
Ratio of kinetic energy = = 2 2 = 9 :1 somewhat, hence he also has to apply frictional force
K .E.after 1mv besides his weight.
2´9 R (reactional force) = friction force (f) + mg
128. (d) For the object of mass 2.0 kg. i.e. R > mg
Dk k – k / 4 3 When the man does not squat and gets straight up in
= =
k k 4 that case friction ( f ) » 0
Kinetic energy transferred R (Reactional force) » mg
Dk 4m1m 2 Hence, the reaction force (R) is larger when squatting
=
k (m1 + m 2 ) 2 and become equal to mg when no squatting.
WORK, ENERGY AND POWER 489

4. (c) According to the question, work done by the frictional dv dv d


force on the cycle is : a0 = = v × = ax3/2 (ax3/2 )
= 200 × 10 = –2000 J dt dx dx
As the road is not moving, hence work done by the 3 3
= ax 3/ 2 ´ a ´ ´ x1/ 2 = a 2 x 2
cycle on the road is zero. 2 2
5. (c) As the body is falling freely under gravity and no 3 2 2
external force act on body in vaccum so law of Now, force = ma0 = m a x
2
conservation, the potential energy decreases and x =2
kinetic energy increases because total mechanical From (i), work done = ò Fdx
x =0
energy (PE + KE) of the body and earth system will be 2
remain constant. 2é3 ù 3 æ x3 ö 1 2
6. (c) According to the question, consider the two bodies as = ò ê ma 2 x 2 ú dx = ma 2=´ çç ´ (0.5)
÷÷ = ´ (25)
ma ´ 8 = 50 J
0 ë2 û 2 3
è ø0 2
system, the total external force on the system will be zero.
11. (b) As given that power = constant
Hence, in an inelastic collision KE does not conserved As we know that power (P)
but total linear momentum of the system remain r uur
conserved. dW F × dx F dx
P= = =
7. (c) As the (inclined surface) are frictionless, hence, dt dt dt
mechanical energy will be conserved. As both the tracks As the body is moving unidirectionally.
having common height, h (and no external force acts Hence, F × dx = Fdx cos 0° = Fdx
on system). Fdx
KE & PE of stone I at top = KE + PE at bottom of I. P= = constant (Q P = constant by question)
dt
From conservation of mechanical energy,
1 L2 µ T 3 Þ L µ T3/ 2 Þ Displacement (d ) µ t 3/ 2
0 + mv12 = mgh + 0 Verifies the graph (b).
2
12. (d) 13. (c)
Þ v1 = 2 gh similarly v2 = 2 gh 14. (a) As given that, mass (m) = 5 kg, n = 300 revolution
Hence, speed is same for both stones. Radius (R) = 1 m; t = 60 sec
For stone I, acceleration along inclined plane a1 = g
sin q1 æ 2pn ö
w =ç ÷ = (300 ´ =
2 ´ p) rad /rad/s
60s = 10 p rad/s
Similarly, for stone II a2 = g sin q2 è t ø
sin q1 < sin q2 Thus, q2 > q1 hence a2 > a1. linear speed (v) = wR = (10p × 1) Þ v = 10p m/s
a2 is greater than a1 and both length for track II is also
1 2 1
less hence, stone II reaches earlier than stone I. KE = mv = ´ 5 ´ (10p)2 = 250p 2 J
8. (b) Total Mechanical energy is E = PE + KE at any instant. 2 2
When particle is at x = xm i.e., at extreme position, So, verifies the option (a).
partical returns back and its velocity become zero for 15. (b) P.E. is maximum when drop start falling at
an instant. Hence, at x = xm; x = 0, K.E. = 0. t = 0 as it fall is P.E. decrease gradually to zero. So, it
From Eq. (i), rejects the graph (a), (c) and (d).
1 K.E. at t = 0 is zero as drop falls with zero velocity, its
E = PE + 0 = PE = V(xm) = kxm2
2 velocity increases (gradually), hence, first KE also
but at mean position at origin V(xm) = 0. increases. After sometime speed (velocity) is constant
9. (b) If two bodies of equal masses collides elastically, their this is called terminal velocity, so, KE also become
velocities are interchanged. æ 3ö
When ball 1 collides with ball-2, then velocity of ball-1, constant. It happens when it falls ç ÷ height or
v1 becomes zero and velocity of ball-2, v2 becomes v, è 4ø
æ 4ö
i.e., similarly then its own all momentum is mV. remains at ç ÷ from ground, then PE decreases
So, v1 = 0 Þ v2 = v, P1 = 0, P2 = mV è 4ø
continuously as the drop is falling continuously.
Now ball 2 collides to ball 3 and its transfer it's
The variation in PE and KE is best represented by (b).
momentum is mV to ball 3 and itself comes in rest.
16. (d) As given that, h = 1.5 m, v = 1 m/s, m = 10 kg, g = 10 ms–2
So, v2 = 0 Þ v3 = v, P2 = 0, P3 = mV By the law of conservation of mechanical energy as no
So, ball 1 and ball 2, become in rest and ball 3 move force acts on shotput after thrown.
with velocity v in forward direction. (PE)i + (KE)i = (PE)f + (KE)f
10. (b) As we know that, 1
mghi + mvi2 = 0 + (KE) f
x2
r uur x2 r uur 2
W.D. = ò F × dx = ò ma0 ×dx 1 2
x1 x1 (KE)f = mghi + mvi
2
As given that, m = 0.5 kg, a = 5 m–1/2 s–1, Total energy when it reaches ground, so
v = ax 3/ 2 1
We also know that Acceleration, (KE) f = 10 ´ 10 ´ 1.5 + ´ 10 ´ (1) 2
2
E = 150 + 5 = 155 J.
EBD_7418
490 PHYSICS

17. (b) First velocity of the iron sphere 102


V = 2 gh after sometime its velocity becomes constant, = = 1.70 watt
60
called terminal velocity. Hence, according first KE
increases and then becomes constant due to resistance 24. (a) When ball collides with the ground it loses its 50% of
of sphere and water which is represented by (b). energy
18. (c) As given that, 1
150 KEf 1 mVf2 M
m = 150 g = kg = 0.15 kg = Þ 2 1
1000 \ =
KEi 2 1
Dt = time of contact = 0.001 s mVi2 2
126×1000 2
u = 126 km/h = m/s = 35 m/s V 1
60 ´ 60 or f =
v = -126km/h Vi 2
5
= -126 – × 2gh 1 M
8 or, =
5 V02 + 2gh 2
= -126 ´ = -35 m/s
18
So, final velocity is acc. to initial force applied by or, 4gh = V02 + 2gh
batsman.
So, change in momentum of the ball \ V0 = 20ms–1
3 21 dw
Dp = m(v - u) = (-35 - 35) =- kg-m/s 25. (d) As we know power P =
20 2 dt
As we know that, force 1
Dp -21/ 2 Þ w = Pt = mV2
F= = N = –1.05 × 10 4 N 2
Dt 0.001 2Pt
Hence negative sign shown that direction of force will So, v =
m
be opposite to initial velocity which taken positive
direction. Hence verify the option (c). dV 2P 1
Hence, acceleration a = = .
19. (b) By conservation of linear momentum dt m 2 t
Therefore, force on the particle at time ‘t’
v
2mv1 = 2mv Þ v1 = 2Km 2 1 Km mK –1/2
2 = ma = . = = t
v m 2 t 2t 2
m 1 2
26. (b) As we know work done in stretching spring w = kx
2
m where k = spring constant
v x = extension
Case (a) If extension (x) is same,
2m 1
W = K x 2 . So, WP > WQ (Q KP > KQ)
v1 2
As two masses of each of mass m move perpendicular F2
to each other. Case (b) If spring force (F) is same W =
Total KE generated 2K
So, WQ > WP
1 2 1 2 1 2 mv2 3 27. (d) From, F = ma
= mv + mv + (2m)v1 = mv 2 + = mv 2
2 2 2 2 2
æ dp ö F 0.1x dV
p2 \
dE
= 2 ç ÷ = 2 ´ 5% = 10% a= = = 0.01x = V
20. (d) As E = m 10 dx
2m E è pø
v2 30
21. (c) Work done in stretching the rubber-band by a distance So, x
ò vdV = ò dx
dx is v1 20 100
dW = F dx = (ax + bx2)dx
V2 30
Integrating both sides, V2 x2 30 ´ 30 20 ´ 20
– = = – = 4.5 – 2 = 2.5
L L 2 200 200 200
aL2 bL3 V1 20
W = ò axdx + ò bx 2 dx = +
2 3
0 0
22. (c)
rr r
1
2
( )
m V22 – V12 = 10 ´ 2.5 J = – 25J
23. (d) Power F.V = PAV = rghAV
1 2 1 2 1
é F ù Final K.E. = mv2 = mv1 – 25 = ´ 10 ´ 10 ´ 10 – 25
êëQ P = A and P = rgh úû
2 2 2
= 500 – 25 = 475 J
= 13.6 × 103 × 10 × 150 × 10–3 × 0.5 × 10–3/60
WORK, ENERGY AND POWER 491

r
28. (d) Here, M1 = M2 and u2 = 0 33. (d) Given force F = 2tiˆ + 3t 2 ˆj
According to Newton's second law of motion,
V r
u1 = V, V1 = ; V2 = ? dv
3 m = 2tiˆ + 3t 2 ˆj (m = 1 kg)
V1=V/3 dt r
t
v
r 2 ˆ 3ˆ
dv = ( 2ti + 3t j) dt Þ v = t i + t j
r
ò ˆ 2ˆ

u1=V u2=0 M1 q
Þ
ò 0
0
M1 M2 r r
M2 f Power P = F·v (2t iˆ + 3t 2 ˆj) · (t 2 ˆi + t 3 ˆj) = (2t3 + 3t5)W
10
34. (a) Given: Mass of particle, M = 10g = kg
V2=? 1000
From figure, along x-axis, radius of circle R = 6.4 cm
M1u1 + M2u2 = M1V1 cosq + M2V2 cosf ...(i) Kinetic energy E of particle = 8 × 10–4J
acceleration at = ?
Along y-axis 1 1 æ 10 ö 2
0 = M1V1 sinq – M2Vs sinf ...(ii) mv 2 = E Þ ç ÷ v = 8 × 10–4
2 2 è 1000 ø
By law of conservation of kinetic energy Þ v2 = 16 × 10–2
Þ v = 4 × 10–1 = 0.4 m/s
1 1 1 1 Now, using
M1u12 + M 2 u 22 = M1V12 + M 2 V22 ...(iii)
2 2 2 2 v2 = u2 + 2ats(s = 4pR)
Putting M1 = M2 and u2 = 0 in equations (i), (ii) and (iii) æ 22 6.4 ö
(0.4)2 = 02 + 2at ç 4 ´ ´ ÷
we get è 7 100 ø
p 2 2 2 7 ´100
q + f = = 90° and u1 = V1 + V2 Þ at = (0.4)2 × = 0.1 m/s2
2 8 ´ 22 ´ 6.4
2 35. (a) Loss in P.E. = Work done against friction from P ® Q
æVö 2 é Vù
V2 = ç ÷ + V2 êQ u1 = V and V1 = ú + work done against friction from Q ® R
è3ø ë 3û
mgh = m(mg cos q) PQ + mmg (QR)
2
æVö V2 h = m cos q × PQ + m(QR)
or, V2 – ç ÷ = V22 Þ V 2 - = V22 2
è3ø 9 2 = 2 3 m + mx --- (i) [sin 30° = ]
PQ
8 2 2 2 Also work done P ® Q = work done Q ® R
or V22 = V Þ V2 = V
9 3 \ 2 3 m = mx \ x » 3.5m
ur
29. (d) For collision V B/A should be along
r From (i) 2 = 2 3 m + 2 3 m = 4 3 m
B ® A ( rA/B ) 2 1
ur ur r r m= = = 0.29
V 2 - V1 r1 - r2 4 3 2 ´ 1.732
So, V - V = r - r W mgh ´ 1000 10 ´ 9.8 ´ 1 ´ 1000
2 1 1 2 36. (b) n = = =
input input input
V1 V2
98000
Input = = 49 × 104J
0.2
49 ´ 104
A B Fat used = = 12.89 × 10–3kg.
A B 3.8 ´ 107
37. (b)
30. (a) Loss in K.E = Area under the curve y axis
u1 v1
31. (a) u2 = 0 A
32. (b) In the string elastic force is conservative in nature.
\ W = – DU m1
q
Work done by elastic force of string, x axis
W = – (UF – Ui) = Ui – UF. m1 m2 f
A
W=
1 2 k
2 2
2 1
2
1
2
(
kx - ( x + y ) = kx 2 - k x 2 + y 2 + 2xy ) B
B m2
v2
1 2 1 2 1 2 1
= kx - ky - kx - k ( 2xy )
2 2 2 2 Before collision After collision
1 2 Given m1 = m2
= -kxy - ky
2 We will apply the principle of conservation of
Therefore, the work done against elastic force momentum in the mutually perpendicular dirn.
ky
Wexternal = - W = ( 2x + y ) Along x-axis, m1u1 = m1v1 cosq + m2 v2 cosf
2
EBD_7418
492 PHYSICS

or u1 = v1 cosq + v2 cosf ...(i) 1 1 V


Along y-axis, 0 = m1v1 sinq - m2 v2 sinf mV f2 = mV02 Þ V f = 0 = 5 m/s
2 8 2
or 0 = v1 sinq – v2 cosf ...(ii)
Again for elastic collision, kinetic energy is conserved æ dV ö 2 dV
F = mç ÷ = -kV \ (10–2) = –kV2
è dt ø dt
1 1 1
Þ mu 12 = mv12 + mv 2 2 5 10
2 2 2 dV 1 1
or u12 =v12 + v22 ...(iii) ò V2 = -100 K ò dt Þ - = 100 K (10)
5 10
10 0
Squaring and adding (i) & (ii), we get or, K = 10–4 kgm–1
u12 = v12 (cos2q + sin2q) + v22 (cos2f + sin2f) Y
+ 2v1v2 cosq cosf – 2v1v2 sinq sinf
pf = 3 m V
or u12 = v12 + v22 + 2v1v2 cos (q + f) ...(iv)
Using (iii) & (iv), we get m
2v pi 45°
p p 43. (a) X
cos (q + f) = 0 = cos Þ q+f =
2 2
Note : This is a standard case of oblique collision. v
38. (c) From work-energy theorem, 2m
Wg + Wa = DK.E Initial momentum of the system
1 2
or, mgh + Wa = mv - 0 pi = m ´ 2 viˆ + 2m ´ vjˆ
2
1
10 -3 ´ 10 ´ 103 + Wa = ´ 10 -3 ´ (50) 2 = (m ´ 2v)2 + (2m ´ v )2 = 2 2 mv (magnitude)
2
Þ Wa = –8.75 J Final momentum of the system = 3mV
which is the work done due to air resistance By the law of conservation of momentum
Work done due to gravity = mgh
2 2v
= 10–3 × 10 × 103 = 10 J 2 2mv = 3mV Þ = Vcombined
r 3
39. (a) F = –5iˆ + 9cos 60°iˆ + 9sin 60° ˆj - 3 ˆj Loss in energy
iˆ æ 9 3 ö 1 1 1
9
= –5iˆ + iˆ +
9 3ˆ
j - 3 ˆj = - + çè - 3÷ ˆj DE = m1V12 + m2V22 - (m1 + m2 )Vcombined
2

2 2 2 2 ø 2 2 2
4 5
r DE = 3mv2 - mv2 = mv2 = 55.55%
s = –3iˆ . 3 3
44. (c) As track is frictionless, so total mechanical energy
r ˆ
é i æ9 3 ö ù
W = F .sr = ê - 2 + çè - 3÷ ˆj ú .( -3iˆ) = 1.5 J.
ø û
will remain constant
ë 2

1 2 h
40. (d) Potential energy U = kx i.e. U µ x 2 B
2
This is a equation of parabola, so graph between U vL
and x is a parabola not a straight line. A
1 2
dV i.e., 0 + mgh = mvL + 0
41. (c) Using, F = ma = m 2
dt vL2
2 2
Using v – u = 2gh, h = (Q u = 0)
dV 2g
6t = 1. [Q m = 1 kg given]
dt For completing the vertical circle, vL ³ 5gR
v
ét 2 ù
1 5gR 5 5
= R= D
ò dV = ò 6t dt Þ V = 6 ê ú = 3 ms
2
–1 [t = 1 sec] or, h =
2g 2 4
0 ë û0 v=0 v¢
v v=0
From work-energy theorem,
45. (b) m m
1
( 1
)
W = DKE = m V 2 - u 2 = ´ 1 ´ 9 = 4.5 J
2 2
4m
Before Collision
4m
After Collision
42. (a) Let Vf is the final speed of the body. According to law of conservation of linear momentum,
From questions, v
mv + 4m × 0 = 4 mv¢ + 0 Þ v¢ =
4
WORK, ENERGY AND POWER 493

Coefficient of restitution, 1 1
Relative velocity of separation mv2 - mv12
8
e = Relative velocity of approach Pd = 2 2 = = 0.89
1 9
mv 2
v 2
4 1 Now, For collision of neutron with carbon nucleus
= or, e = = 0.25 v v1 v2
v 4
46. (b) m 12m m 12m
¶u K Applying Conservation of momentum
47. (c) F =- rˆ = 3 rˆ
¶r r mv + 0 = mv1+ 12mv2 ....(iii)
Since particle is moving in circular path v = v2 – v1 ....(iv)
mv2 K K From eqn (iii) and eqn (iv)
F= = Þ mv2 =
r r 3
r2 11
v1 = - v
1 K 13
\ K.E. = mv2 = 2 1 1 æ 11 ö
2
2 2r mv2 - m ç v ÷
Total energy = P.E. + K.E. 2 2 è 13 ø 48
Pc = = » 0.28
K K K 1 2 169
= - 2 + 2 = Zero (Q P.E. = - 2 given) mv
2r 2r 2r 2
51. (b)
48. (b) Before Collision After Collision
V0 V1 V2 Exercise - 3
m m Þ m m
1. (c) Work done by friction
Stationary
1 2 1 2 3æ1 ö ur uur x dx
mv + mv = ç mv02 ÷ = ò F × ds = ò µmg cos q = µmg x = 20 J
2 1 2 2 2è2 ø cos q
0
3
Þ v12 + v 22 = v20 ....(i) N
2 q
From momentum conservation
mv0 = m(v1 + v2) ....(ii) f q
Squarring both sides,
dx ds dy
(v1 + v2)2 = v02 Þ v12 + v22 + 2v1v2 = v02 mg cos q = q
ds
v2 dx
2v1v2 = - 0
2 2. (a) Let u be the initial velocity of the bullet of mass m.
3 v2
(v1 - v2 )2 = v21 + v22 - 2v1v2 = v20 + 0 After passing through a plank of width x, its velocity
2 2 decreases to v.
Solving we get relative velocity between the two particles 4 4 u(n - 1)
\ u–v= or, v = u - =
v1 - v2 = 2v0 n n n
If F be the retarding force applied by each plank, then using
49. (a) Change in momentum work – energy theorem,
P ˆ P ˆ P ˆ P ˆ 2
DP = J+ J+ i- i 1 2 1 2 1 2 1 2 ( n - 1)
2 2 2 2 Fx = mu – mv = mu – mu
2P ˆ 2P ˆ 2 2 2 2 n2
DP = J = IH molecule Þ Iwall = - J
é 2ù
2 2 1 2 ê 1 - ( n - 1) ú
= 2 mu
F 2P ê n2 ú
Pressure, P = = n (Q n = no.of particles) ë û
A A
1 æ 2 n - 1 ö
2 ´ 3.32 ´ 10 -27 ´ 103 ´ 1023 Fx = mu 2 ç ÷
= =2.35 × 103N/m2 2 è n2 ø
2 ´ 10 -4 Let P be the number of planks required to stop the bullet.
50. (a) For collision of neutron with deuterium: Total distance travelled by the bullet before coming to rest
v v1 v2 = Px
m 2m m 2m Using work-energy theorem again,
Applying conservation of momentum : 1
mv + 0 = mv1 + 2mv2 .....(i) F ( Px ) = mu 2 - 0
2
v2 – v1 = v .....(ii) é1
or, P ( Fx ) = P ê mu 2
( 2n - 1) ù 1 2
Q Collision is elastic, e = 1 ú = mu
v ë2 n2 û 2
From eqn (i) and eqn (ii) v1 = - n2
3 P=
\
2n - 1
EBD_7418
494 PHYSICS

3. (a) By using work-energy theorem, Wall = D K, we have Pin = 10 × 103 W.


Work done by F + work done by Mg = 0 Pout 3000
F (AB) – Mg (AC) = 0 \ h= ´100 =
3
´100 = 30%
Pin 10 ´10
æ AC ö é l - l 2 ù output energy
F = Mg ç ÷= ê ú 10. (b) Efficiency =
è AB ø ë l 2 û input energy

= Mg( 2 – 1). 75 g ´ 3 75 ´ 10 ´ 3
i.e. Efficiency = = = 0.75 = 75%
250 ´ 12 250 ´ 12
4. (b) The centripetal acceleration 11. (b) Constant power of car P0 = F × V = ma.v
2 dv
v P0 = m × v
ac = k2 r t2 or = k 2 rt 2 \ v = krt dt
r mv 2
dv P0dt = mvdv. Integrating P0 × t =
So, tangential acceleration, at = = kr 2
dt 2P0 t
Work is done by tangential force. v=
Power = Ft×v×cos 0° = (mat)(krt) = (mkr)(krt) = mk2r2t m
5. (c) Initial KE. = 0, Initial P.E. = 0 Q P0, m and 2 are constant \ v µ t
When the rope is just pulled off the table, 12. (a) Masses of the pieces are 1, 1, 3 kg. Hence
1 (1 × 21)2 + (1 × 21)2 = (3 × V)2
final K.E. = (ll)v 2 , final P.E. = (ll)gl/2
2 That is, V = 7 2 m/s
time taken = t = l/v
13. (b) From conservation law of momentum, before collision
net change in energy and after collision linear momentum (p) will be same. That
Average power =
time is,
1 2
llv + llgl / 2 initial momentum = final momentum.
2 1 llvg
= = lv 3 + Þ 0 = m1v1 – m2v2 Þ m1v1 = m2v2
l/v 2 2 p1 = p2
m p2
6. (d) dU = (dm) g(–y) = - ( Rd q) gR (1 - cos q) Now, E =
l 2m
y
E1 p12 2m2 E m
mgR 2 \ = ´ Þ 1 = 2 [ p1 = p2]
=- (1 - cos q) E2 2m1 P2 2 E2 m1
l q
dq 14. (b) Initial length of the spring,
l
q=
R 2 l/R
mgR l i = (2 R)2 + (1.5 R) 2 = 2.5 R
\ U = ò dU = -
l ò (1 - cos q)d q
\ xi = 2.5R – 2R = 0.5R.
0 0
1 2 1 2
mgR 2 mgR 2 æ l lö Now kxi + mg (1.5 R ) = mv
= - (q - sin q)l0 / R = ç sin - ÷ . 2 2
l l è R Rø
1 1 é 4mg ù
7. (b) The required work done by man = kinetic energy of or k (0.5R )2 + mg (1.5R ) = mv 2 ê k=
man + kinetic energy of boat 2 2 ë R úû
\ v = 2 gR
1 p2 1 p2
= + (where p = Mv) 15. (c) Let xA and xB be the position of ends A and B at time t
2M 2 m
from the block, then stretched length of the spring will be
1 æ M 2 M 2 ö 2 = 1 æ M + M ö v2
2
\ W = ç + ç ÷ l2 = xA – xB
÷v
2 çè M m ÷ø 2 çè m ÷ø and so the stretch
8. (b) For equilibrium Dl = l2 – l1 = (xA – xB) – l1 (l1 natural length of the spring)
dU -2 A B 2A 1 1
= 0 Þ 3 + 2 =0 r= So, U = k Dl2 = k[( x A – xB ) – l1 ]2
dr r r B 2 2
for stable equilibrium dU 1 æ dx dx ö
P= = k × 2 ( x A – x B – l1 ) ç A – B ÷
d 2U dt 2 è dt dt ø
should be positive for the value of r.
dr 2 P
P = F (vA – vB) Þ F =
d 2U 6A 2B 2A v A – vB
here = - is +ve value for r =
2 4 3
dr r r B F P 20
Dl = = =
mgh 9000 ´10 ´10 k (v A – vB )k (4 – 2) ´ 100
9. (d) Pout = = = 3000 W
t 5 ´ 60 Dl = 0.1 m = 10 cm
7. SYSTEM OF PARTICLES AND ROTATIONAL MOTION
s1
Exercise - 1
s1
1. (b) Centre of mass epends on the distribution of mass in
the body.
C
2. (d) By definition, position vector of centre of mass of two
particle system is such that the product of total mass
of the system and position vector of centre of mass is s2
equal to the sum of products of masses of two particles s2
and their respective position vectors i.e. (I) (ii)
r r r
(m1 + m2 ) R = m1 r1 + m2 r2 (i) Trajectories of two stars. S1 (dotted line) and S2 (solid
line) forming a binary system with their centre of mass
3. (d) The position of centre of mass of a system depends C in uniform motion
upon mass, position and symmetry of the body.
(ii) The same binary system, with the centre of mass C at rest.
RCM = å i i
mr So, to understand the motion of a complicated system,
å mi L
we can separate the motion of the system into two
parts. So, the combination of the motion of the CM and
L ( dm) x ò0 (lxdx) x = 2L motion about the CM could described the motion of
4. (a) Xcm = ò0 L = L
3 the system.
ò ( dm) ò (lxdx)
0 0
20. (a) If centre of mass y
5. (a) 6. (a)
7. (c) When the system is released, heavier of system lies at
r m1 m2 x
mass move downward and the lighter origin then r cm = 0 r1 r2
one upward. Thus, centre of mass ur r
r m1 r1 + m2 r 2 ur r
will move towards the heavier mass r cm = \ m1 r1 + m2 r 2 = 0 or m1r1 = m2r2
with acceleration m1 + m2
T
a T a md
æ 3m – m ö g m 21. (c) m1d = m2 d2 Þ d 2 = 1
a =ç ÷g = 2 3m m2
è 3m + m ø mg
3mg 22. (a) X cm = m1 x1 + m2 x2 + m3 x3
8. (b) linear velocities (v) for particles at different distances m1 + m2 + m3
(r) from the axis of rotation are different. 300 ´ (0) + 500(40) + 400 ´ 70
X cm =
1 ´ 0 + 1 ´ PQ + 1 ´ PR PQ + PR 300 + 500 + 400
9. (b) xcm = = and ycm = 0
1+ 1 +1 3 500 ´ 40 + 400 ´ 70
® ® ® ® ® ® X cm = = 40 cm
® m r + m 2 r2 m( x + y) x + y 1200
10. (b) r = 1 1 = = . 23. (a) Does not shift as no external force acts. The centre of
m1 + m 2 2m 2 mass of the system continues its original path. It is
11. (c)
12. (b) The motion of a rigid body which is not pivoted or fixed only the internal forces which comes into play while
in some way is either a pure translation or a combination breaking.
d r
of translation and rotation. The motion of a rigid body
which is pivoted or fixed in some way is rotation.
24. (b) External force on the system Fext = M
rdt cm
v ( )
13. (c) If system is isolated i.e. Fext = 0 then v cm constant
14. (c) In rotation of a rigid body about a fixed axis, every If initially the velocity of centre of mass is zero then
particle of the body moves in a circle, which lies in a it will remain zero.
plane perpendicular to the axis and has its centre on 25. (c) 26. (d) 27. (a) 28. (d)
the axis. 29. (b) The coordinates of C.M of three particle are
15. (a) m1 x1 + m 2 x 2 + m 3 x 3 m y + m 2 y 2 + m 3 y3
x= &y = 1 1
16. (c) Centre of mass does not necessarily lie only where m1 + m 2 + m 3 m1 + m 2 + m 3
there is mass. It can lie outside the body as well. For here m1 = m2 = m3= m
e.g. Centre of mass of circular ring lies in the centre of
the ring where there is no mass. ( x1 + x 2 + x 3 ) m = 2 , y=
( y1 + y 2 + y3 ) m = 2
so x =
17. (c) 18. (a) m+ m+ m m+m+m
19. (a) When no external force acts on the binary star, its CM r
so coordinates of rC.M. of three particle are (2,2)
will move like a free particle [Fig. (a)]. From the CM r m1v1 + m2 v2 2 ´ 2 + 4 ´10
30. (b) vcm = = = 7.3m / s
frame, the two stars will seems to move in a circle about m1 + m2 2+4
the CM with diametrically opposite positions. 31. (b)
EBD_7418
496 PHYSICS

37. (b)
æ xö 1 æ yö
(rx ) ç ÷ + ry ç ÷ 38. (a) For just complete rotation
x è 2ø 2 è 2ø
32. (c) Centre of mass xcm = v < Rg at top point
2 r( x + y )
A The rotational speed of the drum
1 y y2
Þ ∗ < v g 10
2 x x2 Þω< < <
x R R 1.25
BC y
[ < The maximum rotational speed of the drum in
AB x revolutions per minute
1∗ 3 C 60 10
< < 1.37 B (0,0) y ω(rpm) < < 27 .
2 2p 1.25
33. (b) Let the mass per unit area be s.
Then the mass of 2R
the complete disc
= s [p(2R)2 ] = 4psR 2 O R
39. (d)

g
The mass of the removed disc = s (pR 2 ) = psR 2
Let us consider the above situation to be a complete 3g
angular acceleration a =
disc of radius 2R on which a disc of radius R of negative 2L
mass is superimposed. Let O be the origin. Then the
above figure can be redrawn keeping in mind the 40. (a)
concept of centre of mass as :
1
4p s R 2 ¬¾®
R
41. (b) q = w0 t + at 2 Þ q = 100 rad
· · 2
O psR 2 100
\ Number of revolutions = = 16 (approx.)

Xc.m =
( 4psR ) ´ 0 + ( -psR ) R
2 2 2p
æ 4500 - 1200 ö
4psR 2 - psR 2 2p ç ÷
2 p(n 2 - n1 ) è 60 ø rad/s 2
42. (d) a= =
-psR 2 ´ R R 1 t 10
\ x c.m = \ x c.m = - Þa=
3psR 2 3 3 3300
2p
= 60 ´ 360 degree a = 1980 degree/s 2
m1x1 + m 2 x 2 + m 3 x 3 1´ 0 + 1´ 3 + 1´ 0 10 2p s 2
34. (d) X CM = = =1
m1 + m 2 + m3 1 + 1+ 1 43. (c) w = w0 - 2aqÞ 0 = 4p2 n 2 - 2aq
2 2
2
m1 y1 + m 2 y2 + m3 y3 æ 1200 ö
4p 2 ç
YCM = è 60 ÷ø
m1 + m 2 + m3 q= = 200 p 2 rad
2´4
1 ´ 0 + 1´ 0 + 1´ 4 4
= = \ 2pn = 200p 2 Þ n = 100p = 314 revolutions
1 + 1+ 1 3
æ 4ö 44. (d ) 45. (a)
Therefore the coordinates of centre of mass are ç1, ÷ . 46. (c) The velocity of top point of the wheel is twice that
è 3ø
35. (d) Centre of mass is at rest of centre of mass. And the speed of centre of mass
\ VCM = 0 is same for both the wheels. [v = wr]
36. (b) Centre of mass of the rod is given by: 47. (c) Given, no. of rotation n = 1800 rpm = 1800 rps
L
Time, t = 2 minutes = 120s
bx2 2p ´ 1800
ò (ax + ) dx aL2 bL2 L æ a + b ö \ Initial angular speed w0= rad s –1
L + ç ÷
xcm = 0 = 2 3 = è 2 3ø 60
L bL b = 60p rad s–1
bx aL + a+
ò (a + L )dx 2 2
Final angular speed (as wheel comes to rest)
w=0
0
w0 – w 60p - 0 p
a b \ Angular retardation = = = rad s –2
+ t 120 2
7L 2 3 48. (b) Only the transverse component contributes to the
Now = Þ b = 2a
12 b torque.
a+
2
SYSTEM OF PARTICLES AND ROTATIONAL MOTION 497

49. (b) A couple can produce rotational motion only and not
linear motion. 63. (a) From the relation, angular momentum, L = mvr
50. (a) From L = I w, we find that angular momentum is directly L
proportional to the moment of inertia. v=
mr
51. (c) Turning moment of force = Force × distance from the Centripetal force acting on the particle
axis of rotation. 2
Thus a small force is required to produce a given turning æ Lö
mç ÷
moment, when distance is large. That is why handle of mv 2 è mr ø L2
F= = = 3
a screw is made wider. r r mr
51. (c) Torque = Force × perpendicular distance of line of action T
mr 2
of force from the axis of rotation (d). 64. (b) Tr = a1 ....... (1)
Hence for a given applied force, torque or true 2 r a1 a a
b
tendency of rotation will be high for large value of mr 2 r
Tr = a ....... (2)
d. If distance d is smaller, then greater force is 2 T acm
a1 = a ....... (3)
required to cause the same torque, hence it is harder
Acceleration of point b = acceleration of point a
to open or shut down the door by applying a force
ra1 = acm – ra ....... (4)
near the hinge.
Hence, 2ra = acm
52. (d) 53. (a) 65. (d) 66. (a)
® ® ® 67. (d) t × Dt = L0 {Q since Lf = 0}
54. (d) As angular momentum, L = r ´ p , therefore, direction
or t × 30 = 2p
® p
of L is along a line perpendicular to the plane of t= N-m
15
rotation. 68. (c) Planar, perpendicular and perpendicular. The moment
55. (c) of inertia of a planar body about an axis perpendicular
56. (a) Angular momentum of particle is given by : to its plane is equal to the sum of its moments of inertia
L = mr2w = 2pmr2f [Q W = 2p f ] about two perpendicular axes concurrent with
perpendicular axis and lying in the plane of the body.
If frequency is halved then, 69. (c) M.I. of a uniform circular
w L disc of radius ‘R’ and
L¢ = mr 2 = pmr 2f \L¢ = mass ‘M’ about an axis
2 2 passing through C.M.
57. (d) and normal to the disc is
58. (c) An ice-skater stretches out arms and legs during 1 2
performance to take advantage of principle of I C.M. = MR
2
conservation of angular momentum. As on doing so, From parallel
their moment of inertia increases or decreases axis theorem
respectively and hence the angular velocity of spin
motion decreases or increases accordingly.
1 3
59. (a) When angular acceleration (a) is zero then torque on I T = I C.M . + MR 2 = MR 2 + MR 2 = MR 2
the wheel becomes zero. 2 2
70. (c)
dq
q(t) = 2t3 – 6t2 Þ = 6t 2 - 12t 71. (a)
dt 72. (d) Moment of inertia of a hollow cylinder of mass M and
d 2q radius r about its own axis is Mr2.
Þa= = 12t - 12 = 0 \ t = 1 sec. 73. (a)
dt 2
60. (b) Moment of inertia of a ring about its axis and
61. (d) From Newton's second law for rotational motion, perpendicular to its plane = Mr2
r Moment of inertia of disc about its axis and
r dL r r
t = , if L = constant then t = 0 1
dt
r r r perpendicular to its plane = Mr 2 = 0.5 Mr2
So, t = r ´ F = 0 2
Moment of inertia of a solid sphere about one of its
( 2iˆ - 6ˆj -12kˆ ) ´ (aˆi + 3jˆ + 6k)ˆ = 0 2
diameter = Mr 2 = 0.4 Mr2
Solving we get a = –1 5
62. (a) Angular momentum, o 0.6m Moment of inertia of a spherical shell about one of
L0 = mvr sin 90°
2
= 2 × 0.6 × 12 × 1 × 1 its diameter =Mr 2 = 0.66 Mr2
0.8m 1m 3
[As V = rw, Sin 90° = 1]
Therefore, the moment of inertia of the ring is highest
So, L0 = 14.4 kgm2/s
O
EBD_7418
498 PHYSICS

74. (a) Radius of gyration of a body depends on the axis of 87. (d) Angular momentum will be conserved
rotation. I1w
75. (d) I1w = I1w' + I2w' Þ w¢ = I + I
1 2
76. (d) Ratio of M.I is
y1 y¢1
MAr 2 I
2
= A
M B ( 2r ) IB 88. (d) Circular disc (1)
IA 1
= [Q M A = M B ] or, I A = I B
IB 4 4 MR 2 MR 2 5
77. (b) Because the entire mass of a ring is at its periphery i.e. Iy = \ I¢y = + MR 2 = MR 2
1 4 1 4 4
at maximum distance from the centre and I = Mr 2
78. (a) y2 y¢2
79. (d) Ixy, moment of inertia of a ring about its tangent in the
3 Circular ring (2)
plane of ring I x1 y = MR 2
2
Moment of inertia about a tangent perpendicular to
the plane of ring Ixy = 2MR2 MR 2
3 Iy =
3 2 2 3 2 2
\ Ixy = (2MR ) = MR or Ixy = I x1y1
4 2 4 MR 2 3
80. (a) According to parallel axis theorem of the moment of \ I ¢y = + MR 2 = MR 2
2 2 2
Inertia
I = Icm + md2 I ¢y = MK12 , I ¢y2 = MK 22
1
d is maximum for point B so Imax about B. I ¢y
K12
81. (d) IAX = m(AB)2 + m(OC)2 = ml2 + m (l cos 60º)2 \ = 1
Þ K1 : K 2 = 5 : 6
= ml2 + ml2/4 = 5/4 ml2 K 22 I ¢y
2
X 89. (b) By theorem of parallel axes,
O Cm I = Icm + Md2
60º I = I0 + M (L/2)2 = I0 + ML2/4
l l
90. (c) l
D C
n
60º O
2
Am l Bm l/
82. (b) The distribution of mass about axis EF is minimum so A B
radius of gyration is minimum and therefore moment of
inertia is minimum about EF.
n'
83. (c) As the polar ice melts, water so formed flows
towards the equator. The moment of inertia of the Inn' = M.I due to the point mass at B +
earth increases. To conserve angular momentum, M.I due to the point mass at D + M.I due to the point
angular velocity decreases. This increases the length mass at C.
(T = 2 p/w) of the day. æ l ö
2
84. (b) A raw egg behaves like a spherical shell and a half = 2 ´ mçç ÷÷ + m( 2l) 2 = ml 2 + 2 ml 2 = 3ml 2
boiled egg behaves like a solid sphere è 2ø
1 Ma 2
Ir 2 / 3 m r 2 5 91. (d) Inn' = M(a 2 + a 2 ) =
\ = = >1 12 6
n m
Is 2 / 5 m r 2 3
85. (c) In a hollow sphere, the mass is distributed away from
A
the axis of rotation. So, its moment of inertia is D
greater than that of a solid sphere.
86. (d) t × Dt = Iw
or t × 60 = 2 × 2 × 60p/60 Þ t = 0.21N - m
60 ö
(Q f = 60rpm \ w = 2pf = 2p ´ ÷ B
60 ø C

n' m'
SYSTEM OF PARTICLES AND ROTATIONAL MOTION 499

DB 2a a 96. (d) According to law of conservation of an gular


Also, DO = = = momentum,
2 2 2
According to parallel axis theorem I1w1 + I 2w 2 = ( I1 + I 2 ) w
2 Substituting the values of w1 = 2 rad s–1
æ a ö Ma 2 Ma 2 w2 = 5 rad s–1
Imm ' = Inn ' + M ç ÷ = +
è 2ø 6 2 I2 = I ´ 10–3 kg m2
Ma 2 + 3Ma 2 2 I1 ´ 2 + 1 ´ 10 , ´ 5 = (I1 + 1 ´ 10–3) ´ 4
–3
= = Ma 2 Þ 2I1 + 5 ´ 10–3 = 4I1 + 4 ´ 10–3
6 3
92. (a) Moment of inertia of the whole system about the axis Þ 2I1 = 1 ´ 10–3
of rotation will be equal to the sum of the moments of 1 ´ 10 –3
inertia of all the particles. Þ I1 = = 0.5 ´ 10 –3 kg m 2
2
Y
2
L
97. (d) K= Þ L2 = 2KI Þ L = 2 KI
(0, 3, 0) 2I
3 kg
X L1 K1 I1 K I 1
(0, 0, 9) (2, 0, 0) = × = × = Þ L1 : L2= 1 : 2
L2 K 2 I2 K 2I 2
(–2,–2, 0) 1 kg 2 kg
4 kg 98. (b) Use theorem of parallel axes.
mæ lö2
\ I = I 1 + I2 + I3 + I4 ç ÷
2 è 2ø m æ l ö 2 ml 2
= 0 + 0 + 27 + 16 = 43 kg m2 99. (a) I= + ç ÷ =
3 2 è 2ø 6
93. (d) For circular disc 1
M
mass = M, radius R1 = R 100. (a) r=
4 R2
moment of inertia I1 = I0 p (R 32 - R13 )
For circular disc 2, of same thickness t, 3
r 2 2 R1
mass = M, density = Ishell = M 2 R 22 - M1R12 .... (1)
2 5 5
r
then pR 2 2 t ´ = pR 12 t ´ r = M 4 MR 3 MR 3
2 M 2 = r ´ pR 32 = 3 2 3 ; M1 = 3 1 3
R22 = 2R12 Þ R2 = 2R1 = 2R 3 (R 2 - R1 ) R 2 - R1
As we know, moment of inertia I µ (Radius)2 Putting values of M1 and M2 in eq. (1),
2
I0 æ R ö
2 2M (R 52 - R15 )
I1 æ R1 ö Ishell =
\ =ç ÷ Þ I = çç 2 R ÷÷ Þ I 2 = 2I0 5 (R 32 - R13 )
I2 è R 2 ø 2 è ø
94. (d) For a thin uniform 101. (c) Moment of inertia about z-axis,
square sheet Iz = mr2 (about centre of mass)
Applying parallel axes theorem,
ma 2 Iz = Icm + mk2
m
I1 = I2 = I3 = I1 2
12 æ2 ö (2/p) r
Icm = Iz – m ç r÷ z
I2 è2p ø r

4 I3 2 m4r 2 æ 4 ö
2 p R3 = mr - 2 = mr ç1 - 2 ÷
M p è p ø
95. (a) Here a = R Now, = 3
i.e., k = 4
3 M¢ a3
102. (b) I = 2 ´ 5 ´ (0.2)2 + 2 ´ 2 ´ (0.4)2 = 1kg ´ m 2
4 3
pR 103. (d) Here a = 2 revolutions/s2 = 4p rad/s2 (given)
3
= 3 3
= p. a
æ 2 ö 2 1 1
I cylinder = MR 2 = (50)(0.5)2
ç R÷ 2 2
è 3 ø
2M 25
M¢ = Kg-m2
=
3p 4
Moment of inertia of the cube about the given axis, As t = Ia so TR = Ia
M¢a 2 æ 25 ö
I= (4 p )
6 Ia çè 4 ÷ø
2 ÞT= = N = 50 pN = 157 N
2M æ 2 ö R (0.5)
´ç R÷
3p è 3 ø 4MR 2
= =
6 9 3p
EBD_7418
500 PHYSICS

104. (c) From figure, 1 æ K2 ö


ma = F – f ....(i) mv2 ç 2 ÷
a Rotational KE 2 K2
èR ø
110. (b) = = .
Total KE 1 2
æ K2 ö K 2 + R 2
F mv ç1 + 2 ÷
Mass = m 2 è R ø
O
g sin q g sin q 5
111. (a) a= = = g sin q
K2 2 7
1+ 1+
f 5
R2
mR 2 112. (a) 113. (c)
And, torque t = Ia Þ a = fR 114. (c) After collision velocity of COM of A becomes zero and
2
that of B becomes equal to initial velocity of COM of A.
mR 2 a é a ù ma But angular velocity of A remains unchanged as the
= fR êQ a = ú Þ = f ...(ii)
2 R ë R û 2 two spheres are smooth.
Put this value in equation (i), K2 2
115. (a) For solid sphere =
ma = F –
ma
or F =
3ma R2 5
2 2 K2 1
For disc & solid cylinder 2 =
R 2
1 2 æ k2 ö Since acceleration of a body, which is rolling on an
mv ç 2 ÷
2 çR ÷ inclined plane at angle q with horizontal is
KErot è ø K2 2 g sin q
105. (d) KEtrans = 1 2 = 2 = 0.4 = a= ....................(i)
R 5
mv 1+ K2 / R 2
2 It is clear from eq.(i) that
2 2 a solid sphere > a disc = a solid cylinder
Þ k2 = R Therefore it is a solid sphere. hence solid sphere take least time in reaching the bottom
5
of the inclined plane.
1 2 1 2 116. (b) By energy conservation
106. (b) mgh = mv + lw
2 2 ( K.E ) i + ( P.E ) i = ( K.E ) f + ( P.E ) f
2 ( K.E ) i = 0, ( P.E ) i = mgh , ( P.E ) f = 0
1 1 æ MR 2 ö æ V ö
= mv2 + ç ÷ç ÷ (K.E)f = ½Iω 2 + ½mv cm 2
2 2è 2 ø è Rø Where I (moment of inertia) = ½mR2
1 1 3 (for solid cylinder)
= mv 2 + mv 2 = mv 2 æ v2 ö
2 4 4 so mgh = ½(½mR 2 ) ç cm ÷ + ½mv 2cm Þ vcm = 4gh / 3
2
1 2 1 4 è R ø
Rotation K.E. = mv = ´ mgh F mg sin a
4 4 3 117. (c) m = R = mg cos a = tan a
mgh
= = 2 ´ 9.8 ´ (3 / 3) = 19.6J v12 w2 R12
3 118. (c) a1 = = = w2 R 1
107. (a) R1 R1
108. (a) For disc rolling without slipping on inclined plane, a2
R2
acceleration v 22
a2 = = w2 R 2 v 2 = wR 2
g sin q g sin q 2 R2 R1
a= = 2
= g sin q a1 v = wR
2 3 1 1
K (R/ 2) Taking particle masses
1+ 2 1+
R R 2 equal
109. (c) The centre of mass of the whole carpet is originally at F1 ma1 a 1 R1
= = =
a height r above the floor. When the carpet unrolls F2 ma 2 a 2 R 2
r
itself and has a radius , the centre of mass is at a 119. (a) Let friction force = f
2 20N
F + f = ma … (i)
r
height . The mass left over unrolled is (F – f) R = Ia …(ii)
2 From eqns. (i) and (ii),
mp(r / 2) 2 m Ia f
= = 2F = ma +
pr 2 4 R
The decrease in potential energy a Ia
Use a = (for pure rolling) 2F = ma + 2
æmö æ r ö 7 R R
= mgr - ç ÷ g ç ÷ = mgr 0.02a
è 4 ø è 2ø 8 40 = 4a + 40
; a= = 6.7 m / s 2
(0.1) 2 6
SYSTEM OF PARTICLES AND ROTATIONAL MOTION 501

120. (d) Velocity at the bottom and top of the circle is 5gr and 2
1 2 æv ö 1 1
= ´ 2 mR 2 ´ ç cm ÷ + mv2cm + mv2cm
gr . Therefore (1/2)M(5gr) = MgH and (1/2) M (gr) = 2 5 è R ø 2 2
Mgh. 4 2 mgh 1 2
121. (a) If spool is not to translate Þ mgh = mv cm Þ = mv cm
7 7 5
F cos q = f ....... (i) After collision, only translational kinetic energy is
If spool is not to rotate transferred to ball B.
Fr = f R ........ (ii) So just after collision, rotational kinetic energy of ball
From eqs. (i) and (ii) we get static friction 1 mgh
Fsin A = mv 2cm =
fR 5 7
cos q = f F
r Exercise - 2
r 1. (d) A bangle is in the form of a ring as
or cos q = r
R Fcos shown in figure. The centre of
C
æ rö mass of the ring lies at its centre, Centre
or q = cos -1 ç ÷ f which is outside the ring or bangle.
è Rø
static friction 2. (c) Centre of mass of a system lies towards the part of the
122. (a) For rolling, vA = 2m/s system, having bigger mass. In the above diagram,
4m/s lower part is heavier, hence CM of the system lies below
or 4 – 1.w = 2 the horizontal diameter at C point.
2m/s
or w = 2 rad/sec (clockwise) 3. (b) The initial velocity is vi = veˆ y and after reflection from
the wall, the final velocity is v f = -veˆ y . The trajectory
1 2 1 2 is at constant distance a on z axis and as particle moves
123. (b) mgR = mv + I w
2 2 along y axis, its y component changes .
v So position vector (moving along y-axis),
w= ; N r
r r = yeˆy + aeˆz
1 2 1 2 2 v2 Hence, the change in angular momentum is
mgR = mv + . mr ´ 2 r
2 2 5 r r ´ m(v f - v j ) = 2mvaeˆx .
mg dw
7 2 10 gR 4. (d) Angular acceleration a =
gR = mv Þ v = dt
10 7 where w is angular velocity of the disc and is uniform
mv 2 17 mg or constant.
N - mg = ÞN= dw
R 7 a= =0
mg sin q dt
124. (d) As we know, acceleration, a =
I Hence, angular acceleration is zero.
m+
r2 5. (b) According to the question, when the small piece Q
M c .g. sin qc M c .g. sin qc removed it is stick at P through axis of rotation passes,
For cylinder, a c = 2 =
1 McR M R2 but axis of rotation does not passes through Q and It is
Mc + 2 Mc + c 2 glued to the centre of the plate, the mass comes closer
2 R 2R
2 to the z-axis, hence, moment of inertia decreases.
or, ac = g sin qc
3 6. (c) Let us consider the diagram of problem 5, there is a line
For sphere, shown in the figure drawn along the diagonal. First,
M g sin qs Ms g sin qs centre of mass of the system was on the dotted line
as = s = and was shifted towards Q from the centre (1st
Is 2 MR 2
Ms + 2 Ms + quadrant).
r 5 R2
5 y y
or, a s = g sin qs given, ac = as
7 5 Q Q hole
g
2 5 sin qc 7 15 x x
i.e., g sin qc = g sin qs \ = =
3 7 sin qs 2 14 x
g
3
125. (a) Just before collision between two balls When the mass Q is decrease, it will be on the same
Potential energy lost by A = kinetic energy gained by ball line but shifted towards other side of Q on the line
A. joining QPor away from the centre so, new CM will lies
h 1 1 in IIIrd quadrant in x-y plane.
mg = Icm w 2 + mvcm 2
2 2 2
EBD_7418
502 PHYSICS

7. (a) As given that density :


v R = v 2 + v 2 + 2 v 2 cos q = 2 v 2 (1 + cos q)
2
r( x) = a(1 + bx ) q
= 2 v cos
where a and b are constants and 0 £ x £ 1 2
At b = 0, r( x) = a = constant 12. (a) The tendency of rotation will be about the point C.
In that case, centre of mass will be at F A
x = 0.5 m (i.e., mid-point of the rod)
Putting, b = 0 in options. a a
a 3
3 2 1 4 2 2
(a) ´ = = 0.5 m (b) ´ ¹ 0.5 m
4 3 2 3 3
B a C
3 3 4 3
(c) ´ ¹ 0.5 m (d) ´ ¹ 0.5 m mg
4 2 3 2 For minimum force, the torque of F about C has to be equal
So, only (a) option gives 0.5. to the torque of mg about C.
8. (a) As there is external torque acting on the system, angular
momentum should be conserved. æ 3ö æaö mg
Hence Iw = constant ...(i) \ F çç a 2 ÷÷ = mg ç 2 ÷ Þ F =
è ø è ø 3
where, I is moment of inertia of the system and w is
13. (c) Moment of inertia of shell 1 along diameter
angular velocity of the system.
From Eq. (i), 2 2
Idiameter = m r
I1w1 = I 2 w2 where w1 and w2 are angular velocities 3
before and after jumping. Moment of inertia of shell 2 = m. i of shell 3
2 2 5 2
(Q I = mr 2 ) So, given that, 2
= Itangential = mr + mr = mr
3 3
m1 = 2M, m2 = M, w1 = w, w2 = ?
X
r1 = r2 = R m1r12 w1 = m2 r22w2
2MR 2 w = MR2 w2 1
as mass reduced to half, hence, moment of inertia also 2 3
reduced to half.
w2 = 2w X¢
9. (a) For solid sphere rolling without slipping on inclined So, I of the system along x x1
plane, acceleration = Idiameter + (Itangential) × 2
g sin q
a1 = 2 æ5 ö 2
K2 or, Itotal = mr 2 + ç mr 2 MR
÷ ´ 2 = 4mr
2
1+ 2 3 è 3 ø
R
For solid sphere slipping on inclined plane without 14. (c) By torque balancing about B
rolling, acceleration a2 = g sin q NA NB
a 1 1 5 NA (d) = W (d – x)
ratio = 1 = = = A B
a2 K 2 2 7
1+ 2 1 + 5 W(d – x)
R NA = x
d
d–x
10. (d) Here a = 2 revolutions/s2 = 4p rad/s2 (given) d
1 2 1 2 25
Icylinder = MR = (50)(0.5) = Kg-m2 W
2 2 4 15. (b) Applying angular momentum conservation
As t = Ia so TR = Ia
æ 25 ö
(4 p )
Ia çè 4 ÷ø V0
ÞT= = N = 50 pN = 157 N m
R (0.5)
11. (b) v
qv
q
æ R0 ö
mV0R0 = (m) (V1) ç ÷ \ v1 = 2V0
è 2 ø

1 2
Therefore, new KE = m (2V0)2 = 2mv0
2
SYSTEM OF PARTICLES AND ROTATIONAL MOTION 503

16. (d) From Newton's second law for rotational motion, K2 K2


r
r dL r r 22. (b) Time of descent µ ; Order of value of
t = , if L = constant then t = 0 R2 R2
dt K2 K2
r r r for disc;
1
== 0.5; for sphere;
2
= = 0.4
So, t = r ´ F = 0 2 2 2 5
R R
( 2iˆ - 6ˆj -12kˆ ) ´ (aˆi + 3jˆ + 6k)ˆ = 0 23.
(sphere) < (disc) \ Sphere reaches first
(a) Given: Radius of disc, R = 50 cm
Solving we get a = –1 angular acceleration a = 2.0 rads–2; time t = 2s
Particle at periphery (assume) will have both radial (one)
17. (c) Work required to set the rod rotating with angular and tangential acceleration
1 2 at = Ra = 0.5 × 2 = 1 m/s2
velocity w0; K.E. = Iw From equation, w = w0 + at
2
w = 0 + 2 × 2 = 4 rad/sec
Work is minimum when I is minimum. ac = w2R = (4)2 × 0.5 = 16 × 0.5 = 8m/s2
I is minimum about the centre of mass Net acceleration,
So, (m1) (x) = (m2) (L – x)
atotal = a 2t + a 2c = 12 + 82 » 8 m/s2
m2L 24. (b) Moment of inertia of complete disc about point 'O'.
or, m1x = m2L – m2x \ x = m + m M
1 2 MR 2
18. (d) Given : Speed V = 54 kmh–1 = 15 ms–1 ITotal disc =
2
Moment of inertia, I = 3 kgm2
Time t = 15s
Mass of removed disc O R
M R
wi = V = 15 = 100 wf = 0 MRemoved = (Mass µ area)
4
r 0.45 3
wf = wi + at Moment of inertia of removed disc about point 'O'.
IRemoved (about same perpendicular axis)
100 100
0= + (– a) (15) Þ a = = Icm + mx2
3 45
2 2
Average torque transmitted by brakes to the wheel M ( R / 2) MæRö 3MR 2
= + ç ÷ =
100 4 2 4è2ø 32
t = (I) (a) = 3 × = 6.66 kgm2s–2 Therefore the moment of inertia of the remaining part
45
of the disc about a perpendicular axis passing through
2 the centre,
19. (a) Here a = R
3 IRemaing disc = ITotal – IRemoved
4 4 3 MR 2 3 13
M
pR 3 pR a = - MR 2 = MR 2
3 3 2 32 32
Now, = = 3 = p.
M¢ a 3 3
2 25. (d) For a thin uniform square sheet
æ 2 ö
ç R÷ ma 2
è 3 ø I1 = I2 = I3 =
2M 12
M¢ =
3p
Moment of inertia of the cube about the given axis,
2
2M æ 2 ö I1
´ç R÷ 2
M¢a 2 3p è 3 ø = 4MR
I= =
6 6 9 3p I2
20. (d) Applying conservation of angular mumentum about I3
the axis r
8w M 9 R 2 8w M 2 8w r r
MR2 × w = MR2 × + × × + r × 26. (c) L = m( r ´ v )
9 8 25 9 8 9 r
L = m éêv0 cos qt iˆ + (v0 sin qt - gt 2 ) ˆj ùú
4R 1
Þ r=
5 y a ë 2 û
ò ydm
r h ´ éë v0 cos q iˆ + (v0 sin q - gt ) ˆj ùû
21. (d) ycm =
ò dm é 1 ùˆ
h R = mv0 cos qt ê - gt ú k
2 ë 2 û
ò pr dy r ´ y
3h 1
= - mgv0 t cos qkˆ
2
=0 =
1 2 4 2
pR hr
3
EBD_7418
504 PHYSICS

27. (a) We know that L = mvr^ 30 ´ (0.4)


In none of the cases, the perpendicular a= or, a = 25rad / s 2
æ R ö 3 ´ (0.4) 2
distance r^ is çè - a÷
2 ø 33. (b) In pure rolling, mechanical energy remains conserved.
28. (c) As shown in the diagram, B D Therefore, when heights of inclines are equal, speed
of sphere will be same in both the case. But as
the normal reaction of AB acceleration down the plane, a µ sin q. Therefore,
on roller will shift towards acceleration and time of descent will be different.
O. O 34. (d) Radius of gyration of body is not a constant
This will lead to tending quantity. Its value changes with the change in
of the system of cones to location of the axis of rotation. Radius of gyration
A
turn left. C of a body about a given axis is given as
29. (c) According to problem disc is melted and recasted into a r12 + r22 + .....rn2
solid sphere so their volume will be same. K=
n
2 4 3 35. (c) Torque at angle q
VDisc = VSphere Þ pRDisc t = pRSphere
3
w,a
2 æ RDisc ö 4 3 é RDisc ù l
Þ pRDisc ç 6 ÷ = 3 pRSphere êt = 6 , given ú t = Mg sin q.
è ø ë û 2
R Also t = la Q
Q
3 3
Þ RDisc = 8 RSphere Þ RSphere = Disc l
2 \ l a = Mg sin q
Moment of inertia of disc 2
1 Ml 2 l é Ml 2 ù
I Disc = MRDisc 2
= I (given) .a = Mg sin q êQ I rod = ú
2 3 2 ë 3 û
\ M (RDisc) = 2I2
la sin q 3 g sin q
Þ =g \ a=
1 2 3 2 2l
Moment of inertia of sphere ISphere = MRSphere 36. (c) As we know, moment of inertia of a solid cylinder about
2
2 an axis which is perpendicular bisector
2 æ RDisc ö M 2I I
= M = ( RDisc )2 = = mR 2 ml 2
5 çè 2 ÷ø 10 10 5 I= +
30. (d) Centre of mass may or may not coincide with centre of 4 12
gravity. Net torque of gravitational pull is zero about mé l2 ù l
I = ê R2 + ú
centre of mass. 4ë 3û
t g = Sti = Sri ´ mig = 0 Let V = Volume of cylinder = pR2l
m é V l2 ù dI m é -V 2l ù
Load = ê + ú Þ = + =0
Mechanical advantage , M. A.= 4 ë pl 3 û dl 4 êë pl 2 3 úû
Effort
If M.A. > 1 Þ Load > Effort V 2l 2pl 3
= Þ V =
31. (a) Here, Iw1 + Iw2 = 2Iw pl 2 3 3
w1 + w2 2 2pl 3 l2 3 l 3
Þ w= pR l = Þ 2 = or, =
2 3 R 2 R 2
1 2 1 2 x r
(K.E.)i = Iw1 + Iw2 37. (a) =
2 2 sin 2q sin(90 - q)
2
1 2 æ w + w2 ö Þ x = 2r sin q
(K.E.)f = ´ 2Iw = I ç 1
2 è 2 ÷ø
dx dq
1 2 \ = 2r cos q ´
Loss in K.E. = (K.E) f - (K.E)i = I(w1 - w2 ) dt dt
4
32. (b) Given, mass of cylinder m = 3kg dq dx / dt v0 v
= = = 0
R = 40 cm = 0.4 m dt 2r cos q 2r cos 60° r
F = 30 N ; a = ? 40 38. (b) In rolling motion, rotational kinetic energy.
As we know, torque t = Ia cm 1 2
F × R = MR2a Kt = mv
2
F´ R 1 2 1 2
a= And, Kt + Kr = mv + l w
2 2
MR 2 2
F = 30 N 1 2 1 æ 2 2 öæ v ö 7 2
= mv + ç mr ÷ç ÷ = mv
2 2è5 øè r ø 10
SYSTEM OF PARTICLES AND ROTATIONAL MOTION 505

1 2 9MR 2 9MR 2 (9 - 1)MR 2


Kt
mv
5 = - = = 4 MR 2
\ = 2 = 2 18 2
Kt + Kr 7
mv 2 7
mw2 R = Force µ n
1 mv2
43. (c) (Force = )
10 R R
39. (a) Work done required to bring them rest 1 1
Þ w2 µ n +1 Þ w µ n +1
DW = DKE (work-energy theorem) R
R 2
1 2æ 1 2ö n +1
DW = I w ç DkErot = I w ÷ 2p
2 è 2 ø Time period T = Þ Time period, T µ R 2
or, DW µ I (for same w) w
44. (a)
2 2
I solid sphere = MR
5 Exercise - 3
1 2
IDisk = MR , IRing = MR2 m ´ l + 2m ´ 2l + 3m ´ 3l + ... + nm ´ nl
2 1. (a) xcm =
\ WC > WB > WA m + 2m + 3m + .. + nm

40. (b) (1 + 4 + 9 + .. + n 2 )l ln(n + 1)(2n + 1) / 6 (2n + 1)l


= = =
41. (d) Using parallel axes theorem, moment of inertia about ‘O’ 1 + 2 + .. + n n(n + 1) / 2 3
Io = Icm + md2 2. (b) Vertical line from hinge A must pass through C.M. of
2R
7MR 2
55MR 2 2R 2R rod system. A
= + 6(M ´ (2R)2 ) = O 2 /3
2 2 q
Again, moment of inertia about point P, 2R
OP l / 2 P

Ip = Io + md 2 2R tan q = =
2R AP 2l / 3 B /2
D
55MR 2 181 O
= + 7M(3R) 2 = MR 2
2 2
42. (a) Let s be the mass per unit area. 3 æ 3ö
tan q = Þ q = tan -1 ç ÷
The total mass of the disc R/ 3 4 è 4ø C
O'
= s × p R2 = 9M 3. (b) Friction force will act towards right.
The mass of the circular disc cut 4. (b) In fig, C1 and C2 are IC (instantaneous centre of
2R /3
2 rotation) of the two cylinders. The cylinder can be
æ Rö O
= s´pç ÷ considered as rotating about C1 and C2. In the absence of
è 3ø slipping between the plank and the cylinders, points A1
p R2 and A2 have the same velocity.
= s´ =M 2v v
9 Angular velocity of the larger cylinder is =
Let us consider the above system as a complete disc of 4R 2R
mass 9M and a negative mass M super imposed on it. A2 A1
2v
Moment of inertia (I 1 ) of the complete disc
2R R
1 2 v
= 9MR about an axis passing through O and
2
perpendicular to the plane of the disc.
M.I. of the cut out portion about an axis passing through C1
O' and perpendicular to the plane of disc C2
2
1 æ Rö æ v ö
´M ´ç ÷ v CM = ( 2R ) ç =v
=
2 è 3ø è 2R ÷ø
\ M.I. (I2) of the cut out portion about an axis 1 2
5. (c) Kinetic energy (rotational) KR = Iw
passing through O and perpendicular to the plane of 2
disc 1
é1 Kinetic energy (translational) KT = Mv 2 (v = Rw)
æ 2R ö ù
2 2
æ Rö 2
= ê ´M ´ç ÷ + M ´ç ÷ ú M.I.(initial) Iring = MR2; winitial =w
êë 2 è 3ø è 3 ø ú
û
Mw
[Using perpendicular axis theorem] M.I.(new) I¢(system) = MR 2 + 2mR 2 Þ w¢(system) =
\ The total M.I. of the system about an axis passing M + 2m
through O and perpendicular to the plane of the disc is Mm
Solving we get loss in K.E. = w2 R 2
I = I1 + I2 (M + 2m)
1 é1 æ Rö
2
æ 2R ö ù
2
6. (c) Consider a square lamina of mass (dm).
2
= 9 MR - ê ´ M ´ ç ÷ + M ´ ç ÷ ú Moment of inertia from this about the shown
2 è 3ø è 3 ø ú
ëê 2 û
EBD_7418
506 PHYSICS
a
11. (b) v = Kr .t
dL
2 a L = mvr = m Kr 3 t Þ = m Kr 3
diagonal is (dm) a . dt
12 12. (d) Apply conservation of angular momentum about the
Now consider an element of the cube of thickness dx and a hinge, we get
m
distance x from the face diagonal. mvR = R 2w + m ( Rw ) R
2
a2 2v 2 ´ 5 10
Now dI = (dm) + (dm)x2 Rw = = = m /s
12 3 3 3
(Q Parallel axis theorem) 13. (b) Consider a small element of disc of thickness dx at a
radius x.
Mdx é a 2 2
ù
2pxdx
Þ dI = ê +x ú dx Now dN = µmg.
a ëê 12 ûú a x p R2
a dx
M a æ a2 ö M é a 2 x x3 ù 2µ mgx
ò 0 çè 12 + x ÷ø dx = a êê 12 + 3 úú
As I = 2 = dx x
a ë û0 R2
2µmgx 2 dx
é 1 1 ù 5 Þ d t = xdN =
= Ma 2 ê + ú = Ma 2 R2
ë12 3 û 12
2µ mg R 2 2 1
7. (b) Moment of interia of semicircular portions about x and Þ òdt = 2 ò x dx = µmgR = I a = mR 2 a
y axes are same. But moment of inertia of straight portion R 0 3 2
about x-axis is zero. 4 µg
I Þ a = , If t be the time for complete stop.
\ Ix < I y or x < 1 3 R
Iy w 3w R
8. (a) at = w Þ t = =
a 4µ g
9. (a) y = x + 4 line has been shown in the figure when x = 0, 14. (a) When the thread is pulled, the bobbin rolls to the right.
y = 4. So, OP = 4. Resultant velocity of A
Y
The slope of the line can be point B along the thread
mv
R a
obtained by comparing with the is v = v0 sin a - wr ,
4 v
equation of the straight line + where v0 sin a is the
x v0
P y =
y = mx + c componen t of B
R
m = tan q = 1 Þ q = 45° translational velocity
Q q X along the thread and wr
ÐOQP = ÐOPQ = 45° O
linear velocity due to
If we draw a line perpendicular to this line, rotation. As the bobbin C
length of the perpendicular = OR rolls without slipping,
OR v0 = wR . Solving the obtained equations, we get
In DOPR, = sin 45°
OP vR
1 4
Þ OR = OP sin 45° = 4 ´ = =2 2 v0 =
2 2 R sin a - r
Angular momentum of particle going along this line = r × 15. (d) mg sin q – f = macm f
mv 2 2
f R = MR a mg sin
= 2 2 ´ 5 ´ 3 2 = 60 units 5 mg cos
(torque = Ia about COM)
10. (b) The angular momentum (L) is conserved, since text is
2mg sin q
zero. acm = Ra Þ f =
Here, Ii = I0 If = 2I0 7
16. (a) The given body can be considered as a right circular
wi = w0 wf = w (say)
cone ABC from which a cone ADE and a cylindrical PQRS
w0 have been cut out as shown.
Then I0w0 = 2I0w Þ w =
2 Let the x-axis be along the base and y-axis as the axis of
1 2 symmetry. For the cone ABC of height hy is :
Ki = I0 w 0 = K A
2 h2 h h - 20 h
1 æ w0 ö
2
1 w2 = or = h2
Kf = (2 I ) ´ ç ÷ = ´ 2I0 ´ 0 r1 r 5 10 R
2
0
è 2 ø 2 4 Þ 2h - 40 = h Þ h = 40cm. D S r1 E

( )
ì1 ü 1 K h
= í I 0 w20 ý ´ = 1 2 h1
2 Volume V1 = pr h r3
î þ 2 2 3
1 4000 Q
= p(10)2 ´ 40 = p cm3
3 3 B P r C
SYSTEM OF PARTICLES AND ROTATIONAL MOTION 507

Position of CG on y-axis, 18. (d) Moment of inertia about the diameter of the circular loop
1
1 (ring) = MR 2.
y1 = ´ 40 = 10 cm. 2
4 Using parallel axis theorem
For cone ADE, Volume The moment of inertia of the loop about XX' axis is
1 2 1 500
V2 = pr1 h2 = p(5)2 ´ 20 = p cm3 MR 2 3
3 3 3 IXX' = + MR 2 = MR 2
2 2
Position of CG on y-axis,
Where M = mass of the loop and R = radius of the loop
1
y2 = 20 + × 20 = 25cm. L
4 Here M = Lr and R = ;
For cylindrical hole 2p
2
Volume, V3 = pr32 h1 = p æç ö÷ ´ 20 = 125 p cm3
5 3 æ Lö
2
3L3r
è 2ø \ IXX' = ( Lr) ç ÷ =
2 è 2p ø 8p 2
h 20
Position of CG on y-axis, y3 = 1 = = 10 cm. 19. (c) The moment of inertia of the system about axis of
2 2
rotation O is
The given body has a volume = V1 – V2 – V3 0.3 kg
I = I1 + I2
\ Position of CG of given body on y-axis
= 0.3x2 + 0.7 (1.4 – x)2 x
S Vy V1 y1 - V2 y 2 - V3 y3
y= = = 0.3x2 + 0.7 (1.96 + x2 – 2.8x) O
SV V1 - V2 - V3
= x2 + 1.372 – 1.96x
4000 500 The work done in rotating the rod is converted 1.4 – x
p ´ 10 - p ´ 25 - 125p ´ 10
3 3 into its rotational kinetic energy.
= 4000 500 1 2 1 2 2
p- p - 125p \ W = I w = [ x + 1.372 - 1.96 x] w 0.7 kg
3 3 2 2
For work done to be minimum
40000 - 12500 - 3750 23750 dW 1.96
y= = = 7.6c m = 0 Þ 2x – 1.96 = 0 Þ x = = 0.98 m
4000 - 500 - 375 3125 dx 2
17. (b) Moment of Inertia depend upon
mass and distribution of masses as I = A
20. (d) The sphere is on the verge
Smr2. of topping when a line of action
Further, as the distance of masses is
of weight passes through the
more , more is the moment of Inertia. 4 cm
If we choose BC as axis. Distance is edge.
maximum. Hence, Moment of Inertia is
B C (R - h)
3 cm cos q = Þ h = R - R cos q
maximum. R
\ I2 > I1 , I2 > I3
EBD_7418
508 PHYSICS

8. GRAVITATION
13. (a) Time period of satellite is given by :
EXERCISE - 1
1. (d) Kepler's third law states that the square of time period circumference of an orbit 2pR 2p R 2 pR 3/2
= = = =
of revolution of a planet around the sun is directly Velocity in orbit v0 GM m GM m
proportional to the cube of semi-major axis of its R
elliptical orbit.
2. (b) In planetary motion, there is no external torque. Hence 4p2 R 3
Squaring both sides, we get T 2 =
GM m
dL r r
from the equation = t ext . , if text. =0 Þ L = constant 14. (d) From Kepler's law of periods,
dt
3. (b) 3/2 3/ 2
T2 = T1 æç R2 ö÷ = 365 æ R / 2 ö
4. (d) Angular momentum of the planet about S is conserved.
è R1 ø ç R ÷
è ø
So, mvr = constant.
v is maximum when r is minimum. 1
= 365 × = 129 days.
So, v is maximum at point P4. Hence K.E. is maximum at 2 2
P4.
L dA T12 r13
5. (d) = (L = angular momentum) 15. (b) = = (4)3
2m dt T22 r23
mvmax rmin dA v 2 dA / dt
= ; max = r = 40 \ T2 = T1 /8.
2m dt min 16. (c) It is applicable to both small & big bodies.
dA L dA
6. (c) = Þ µ vr µ wr 2 17. (b) Various regions of spherical shell attract the point mass
dt 2m dt
Tp 1 inside it in various directions. These forces cancel each
7. (c) = other completely. Therefore the gravitational force on
Te 3
the particle is zero.
2 3
æ Tp ö æ rp ö 3 3 18. (d) G is a universal gravitational constant as the value of G
çT ÷ =çr ÷ Þ re = 9rp
è eø è eø is same for all pairs of bodies situated anywhere in the
universe.
8. (a) By law of conservation of angular momentum,
19. (a) The gravitational force of attraction on a body of
mvr = constant
mass m is given by
vmin ´ rmax = vmax ´ rmin
GMm 1
F= Therefore, F µ
60 ´1.6 ´10 12
60 R 2 R2
\ vmax = = = 12 m / s
8 ´1012 5 The radius of earth is maximum at equator, therefore,
gravitational force of attraction is least at equator.
9. (d) Time period does not depend upon the mass of satellite 20. (c) Gravitational force is independent of medium. Hence,
r1 + r2 this will remain same.
10. (b) T2 a r3, where r = mean radius =
2 m( M - m)
11. (c) 21. (b) F = G
12. (b) According to Kepler’s law, the areal velocity of a planet r2
around the sun always remains constant. For maximum value of F, dF/dm = 0, and so
SCD : A1– t1 (areal velocity constant) m = M/2.
SAB : A2 – t2 Gme Gmm
22. (d) =
A1 A 2 A x 2 ( D - x )2
= , t =t . 1, (given A1 = 2A2)
t1 t 2 1 2 A2
G (81m) m
2A 2 or =
= t2 . \ t1 = 2t2 x 2 ( D - x )2 x D
A2
9D
\ x= .
10
GRAVITATION 509

ur ur ur ur ur ur
23. (b) F = F1 + F 2 + F 3 + F 4 + F 5 Gm(2 m) ˆ
FGA = j
m m 1
Gm(2 m) ˆ
FGB = ( - i cos30° - ˆjsin 30°)
1
Gm(2 m) ˆ
m F2 F3 m FGC = (i cos30° - ˆjsin 30°)
1
\ Resultant force on (2m) is FR = FGA + FGB + FGC
F1 F4
= 2Gm 2 ˆj + 2Gm 2ˆi( - cos 30° + cos30° )
m F5 m
ur ur ur ur + 2Gm 2 ˆj(- sin 30° - sin 30°)
F 2 = F 5 and F 2 = F 4
= 2Gm 2 ˆj.2Gm 2 ˆj æç - 2 ´ ö÷
ur ur 1
F1 = F 3 + 2 F2 cos 30° + 2F1 cos 60° è 2 ø
Gm 2 Gm2 Gm2 = 2Gm 2 ˆj - 2Gm 2 ˆj = 0 .
F3 = ; F2 = ; F1 =
4a 2 3a 2 a2 32. (c) The gravitational force of attraction between two
2
Gm æ 5 1 ö Gm æ 5 1 ö identical spheres of radius r is
F= 2 ç + ÷ = mw2a Þ w = ç + ÷
a è4 3ø 4 4
a3 è 4 3ø G pr3r ´ pr 3r
Gm1m 2
4 3a3
F= = 3 3
T = 2p r2 (2r)2
Gm (5 3 + 4)
4 r r
24. (a) F =
G m1m2
=
G m1m2
. = p 2 p2 r 4 m1 m2
( r1 + r2 ) 2 9
r2
25. (b) Gravitational force is always attractive and a long range ie. Fµr 4
force. It is independent of the presence of other bodies. 33. (a) Here, centripetal force will be given by the gravitational
26. (c) As m2 attracts m1 towards itself, \ force is along r 3. force between the two particles.
27. (d) Gm2
A x C (60–x) B = mw 2 R
( 2R )2 R
4kg 1kg 9kg
60cm Gm m m
Þ 3
= w2
4 ´1 9 ´1 4R
\G 2 =G
x (60 – x) 2 Gm
Þ w=
2 x 4 R3
or = Þ x = 24 cm If the velocity of the two particles with respect to the
3 (60 – x)
28. (a) centre of gravity is v then v = wR
Gm Gm
v= ´R=
é G(2m1 )(2m 2 ) Gm1m 2 ù 4R 3
4R
ê - ú
êë (3r)2 r 2 úû 34. (b) Acceleration due to gravity (g) is given by
29. (c) % change = ´ 100
Gm1m 2 GM 1
r2
g= 2 Þ gµ
4 R R2
-1 As one moves from the equator to the poles, the radius
9 ´ 100 = -56%,
= of the earth decreases, hence g increases.
1
–ve sign indicates that force of attraction decreases. 35. (c) Acceleration due to gravity g varies with dpth as
GM (3m) 3GMm æ ö
30. (c) F =
d2
=
d2
. g¢ = g ç1 - d ÷
è RE ø
31. (c) y 36. (a) The weight (= mg) of the body at the centre of the earth
Am is zero, because the value of g at centre is zero.
37. (b) Because of smaller value of acceleration due to gravity
on moon, the value of escape velocity is small there.
G 2m x The molecules of atmospheric gases have thermal
30°
speeds much greater than the escape velocity. So all
m m
C the molecules have escaped and there is no
B
atmosphere.
z
EBD_7418
510 PHYSICS

38. (b) Acceleration due to gravity with height h varies as 4


1 g = rpG. R where r ® average density
g µ 2 3
r æ 3g ö
(when r = R + h). Thus variation of g and r is a parabolic r =ç ÷
curve. è 4 pGR ø
39. (c) Þ r is directly proportional to g.
40. (c) Applying conservation of total mechanical energy
1 2 53. (b) At the equator, g' = g – Rw 2
principle mv = mg A h A = mg B hB As w increases, g' decreases and hence weight
2 decreases.
Þ g A hA = g B hB At the pole, g' = g.
So weight remain unchanged.
æg ö 54. (d)
Þ hB = ç A ÷ hA = 9 × 2 = 18 m
è gB ø 55. (b) K.E. of the person = P.E. at the maximum height
1 2 GM
u 2 sin 2q mv = m 2 h.
41. (d) Range of projectile R = 2 R
g
1 GM M¢
if u and q are constant then R µ \ m 2 .h = mG 2 h¢
g R R¢
Rm ge Rm 1 R 4 R 2
= Þ = Þ Rm = e Þ Rm = 5Re Given r = p ¢, =
Re gm Re 0.2 0.2 5 R¢ 3
3
2 2 4 pR r 4 pR ¢3r¢
g' æ R ö 1 æ R ö \ mG . .h = mG . .h¢
42. (b) =ç ÷ Þ =ç ÷ 3 R2 3 R ¢2
g è R+hø 100 è R + h ø
2 4
Þ h = 9R Þ R ¢. r¢1.5 = R ¢r¢h¢ \ h¢ = 0.8 m.
43. (c) g' = g – w'2R, for g' to be zero, 3 5
w' = g / R ; 17w GM
56. (c) g = 2 . Since M and r are constant, so g = 9.8 m/s2
44. (b) Value of g is larger at poles than the equator. r
45. (c) Because of the centrifugal force, earth stretches out 57. (a) g = g p - Rw2 cos 2 l
and so bulges at the equator.
1
46. (c) x = mg = GMm 1
k Þ x µ 2 Þ x2 = g p - w2 R cos 2 60° = g p - Rw2
k R2 4
R 58. (b)
2
æ 6400 ö 59. (a) We know that,
=ç x = 0.79cm
è 7200 ÷ø 1 æ4
Gç pR 3 ÷r
ö
GM è3 ø 4
g' R2 g= 2 = 2 = pGRr
47. (d) We know that g = R R 3
( R + h) 2 g¢ R ' 0.2R
2 = = = 0.2 \ g¢ = 0.2g
g /9 é R ù R 1 g R R
\ =ê ú \ = \ h = 2R
g ëR + hû R+h 3 60. (d)
48. (a) To counter balance the effect of gravity.
61. (a)
GM 4
49. (a) g = also M = d ´ pR 3
(a) As we know, g¢ = g æç1 -
2h ö æ dö
R 2 3 62. ÷ = g ç1 - ÷
è R ø è Rø
4
\ g = dpR at the surface of planet \
2h
=
d
\h=
d
or d = 2h.
3 R R 2
4 4 \ d = 2 × 50 = 100 km.
g p = (2d )pR ¢ , g e = (d )pR
3 3 63. (d) Both decreases but variation are different.
ge = gp Þ dR = 2d R' Þ R' = R/2
50. (d) We know that -dV
64. (b) I= . If I = 0 then V = constant
æ4 ö dr
GMG ç pR3 ÷ r 4
3 65. (a)
g= 2 = è 2 ø = pGRr
R R 3 r g
r gr
g' R ' 3R 66. (a) g'= – for r £ R and g' = (1 + r / R)2 for r ³ R
= = =3 \ g' = 3g R
g R R so option (a) is correct.
51. (c) 67. (a) Potential at the given point = Potential at the point due
4
G ´ r ´ pR 3 to the shell + Potential due to the particle
GM Gr´ V 3
52. (d) g = = Þg= GM 2GM 3GM
R2 R2 R 2
= - - =-
a a a
GRAVITATION 511

68. (b) Gravitational potential energy (GPE) on the surface of GM GM


earth, 76. (b) vg = – for r £ R and vg = – , for r > R, and so
R r
GMm option (b) is correct.
E1 = - 77. (b)
R
78. (d) E earth = E moon R/4 Moon
GMm GMm
GPE at 3R, E2 = - =-
( R + 3R ) 4R GM GM / 81
\ Change in GPE Þ =
2
x (60R - x) 2
GMm GMm 3GMm 60 R
= E2 - E1 = - + = 1 1
4R R 4R Þ x = 9 (60R - x) x

3g R 2 m æQ g = GM ö 3 R Earth
= ç ÷ = mg R
4R è R2 ø 4 Þ x = 54 R from centre of earth.
69. (a) (A) ® (2) ; (B) ® (5) ; (C) ® (3) ; (D) ® (4) 79. (d)
GM 80. (c) Work done in raising the body
Inside a shell V = - = constant and E = 0
R 2R 2R 2R
gR 2 -1
mdx = mgR 2 . éê ùú
GMm
Outside the shell, V = -
GM GM
and E = 2
= ò x2
dx = ò x2 ë x ûR
r r R R
-1 1 ù -1 + 2 ù 1
As r increases, V increases and E decreases. = mgR 2 éê + = mgR 2 éê = mgR
ë 2R R úû ë 2R úû 2
70. (b) Mass of the satellite = m and height of rock from earth 81. (a) The gravitational potential at the centre of uniform
6 spherical shell is equal to the gravitational potential at
(h) = R e 2 = 3.2 ´ 10 m the surface of shell i.e.,
We know that gravitational potential energy of the rock -GM
rotating at height V= , where a is radius of spherical shell
a
GMem 2 Now, if the shell shrinks then its radius decrease then
h =- = - mgRe density increases, but mass is constant. so from above
Re + h 3
expression if a decreases, then V increases.
(where, GMe = gRe2 and h = Re) a
A B
71. (b) There is no gravitational field in the shell. 82. (c)
72. (d) At a point inside a spherical shell, the value of
gravitational intensity, I = 0. a a
If V = 0 then gravitational field is necessarily zero. O
73. (c) Gravitational potential energy associated with two
particles of masses m1 & m2 separated by distance r is D a C
given by
a 2
G m1m 2 1 OA = OB = OC = OD = = a
V= - \ if r ® ¥. \ = 0. 2 2
r ¥ Total gravitational potential at the centre of the square
Þ V = 0.
-Gm ´ 4 -4Gm -4 2Gm
74. (c) Kinetic energy = 1 mv 2 is always positive but =
OA
= =
a
2 a/ 2
potential energy and total energy are negative for a 83. (d) The sense of synchronous satellite must be same as
satellite. the sense of rotation of earth i.e., from west to east.
75. (b) (A) ® (2) ; (B) ® (2) ; (C) ® (4) ; (D) ® (3)
84. (b)
GM
g= 2
, 1
R 85. (b) orbital speed v µ ; As Jupiter is farther than earth
g g r
At height h = R: g' = =
h 2 from sun, so its orbital speed is less than orbital speed
1+
R of earth.
1 86. (d) Total energy of satellite is half the potential energy i.e.,
i.e., g' decreases by a factor .
2
2 U
E=
Similarly, at height h = R/2, g ¢ = g . 2
3
R æ h ö æ 1ö g 87. (c) Geo-stationary satellites are always remain about the
At depth h = : g ' = g çè1 - ÷ø = çè1 - ÷ø = same path on equater, i.e., it has a period of exactly
2 R 2 2
one day (86400 sec)
3
Similarly at h = R/4, g ¢ = g
4
EBD_7418
512 PHYSICS

æ 3 ö
GM m GM (2m ) GMm
-
So orbit radius ç T = 2p r ÷ comes out to be 102. (c) Fmin = r 2
(2r ) 2
=
2r 2
çç GM ÷÷
è ø 3 GMm Fmin
GMm GM (2m ) 1
42400 km, which is nearly equal to the circumference of and Fmax = 2 += 2 2 .\F = . 2
r (2r ) r max 3
earth. So height of Geostationary satellite from the earth
surface is 42,400 – 6400 = 36,000 km. 103. (b) When gravitational force becomes zero, centripetal force
88. (d) on satellite becomes zero so satellite will move
89. (b) tangentially to the original orbit with same velocity.
104. (c) According to Kepler’s law of period T2 µ R3
2GM
90. (a) Escape velocity = v =
R T12 R13 (6 R )3 24 ´ 24
= = =8 Þ =8
2GM T22 3
R2 (3 R ) 3
T22
Þ v2 = .......(i)
R
v2 = (2GM)R–1 24 ´ 24
Þ T22 = = 72 = 36 × 2 Þ T2 = 6 2
Differentiating both sides, we get, 8
dv 2GM dv GM 105. (d) Potential energy on earth surface is – mgR while in free
2v =– Þv = ......(ii) space it is zero. So, to free the spaceship, minimum
dR R 2 dR R2
required energy is
1 dv 1
Dividing (ii) by (i), =– K = mgR = 103 × 10 × 6400 × 103 J = 6.4 × 1010 J
v dR 2R
106. (a) K.E. of satellite moving in an orbit around the earth is
dv 1
Þ × 100 = × 4% = 2% 2
v 2 1 1 æ GM ö÷ GMm
K= mv 2 = mç =
\ If the radius decreases by 4%, escape velocity 2 2 çè r ÷ø 2r
will increase by 2%.
P.E. of satellite and earth system is
l
91. (b) Since T = 2π GMm
g GMm K 2r = 1
U= Þ = GMm
but inside the satellite g = 0 r U 2
So T = ¥ r
92. (c) 107. (a) The orbital velocity of satellite near the earth surface is
93. (d) Because, the period of satellite is equal to period of
rotation of earth about its own axis & it seems to be at v0 = gR e
one point about the equator and so is able to transmit g = 9.8 m/sec2, Re = 6.4 × 106 metre
the signals from one part to other.
94. (b) When satellite is orbiting close to the surface of earth, Þ v0 = 7.92 × 103m/sec = 7.92 km/sec » 8km/sec
108. (b)
GM
orbital velocity, vo = where M and R are the
R 2GM p
mass and radius of earth. (ve ) p Rp Mp
GM Re
95. (b) Since v 0 (orbital velocity ) = 109. (c) = = ´ =
(ve )e 2GM e Me Rp
r
v 01 r2 Re
So = Þ v 01 < v 02 Q r1 > r2
v 02 r1
10M e Re
96. (a) vesc = 2 gR , where R is radius of the planet. = ´ = 10
Me R e /10
Hence escape velocity is independent of m.
\ (ve)p = 10 × (ve)e = 10 × 11 = 110 km/s
97. (c) Escape speed, Ve = R 8 pGs \ Ve µ R. 110. (b ) 111.(c)
3
98. (c) 112. (b) We know that v e = 2 v0 , where v 0 is orbital
99. (c) Since total energy of a body is positive or zero, when it is velocity.
at infinity from the earth therefore, the satellite will escape
to infinity if its total energy becomes zero or positive. 1
K.E. in the orbit, E = M v 02
100. (a) A geostationary satellite moves in a circular orbit in 2
the equatorial plane at an approx. distance of 4.22 × 104
1 1
km from zero the earth's centre. K.E. to escape E = M v e 2 = M (2 v 0 2 )
2 2
GM 1
101. (d) Orbital velocity v0 = = M v 02 ´ 2 = 2 E
r 2
GRAVITATION 513

113. (a)
114. (b) Orbital velocity of a satellite in a circular orbit of radius
EXERCISE - 2
a is given by 1. (d) Let the density of earth as a sphere is uniform, then it
can be treated as point mass placed at its centre then
GM 1 v a1 acceleration due to gravity g = 0, at the centre. But if the
v= Þ va Þ 2 =
a a v1 a2 density of earth is considered as a sphere of non-uniform
then value of 'g' will be different at different points
4R
\ v2 = v1 = 2 v1 = 6V æ 4 ö
R
çQ g = prGR ÷ . So g cannot be zero at any point.
115. (a) given R = 3200 km, R1 = 6400 kg, m = 100 kg è 3 ø
-GMm 2. (c) Force of attraction between any two objects obeys the
Gravitational potential energy = inverse square law.
r
As observed from the earth, the sun appears to move
and orbital velocity, v0 = GM / R + h in an approximate circular orbit. The gravitational force
1 GMm 1 GM GMm of attraction between the earth and the sun always
Ef = mv02 - = m - follows inverse square law.
2 3R 2 3R 3R Due to relative motion between the earth and mercury,
GMm æ 1 ö - GMm the orbit of mercury, as observed from the earth will
= ç - 1÷ = not be approximately circular, since the major
3R è 2 ø 6R
gravitational force on mercury is due to the sun is very
-GMm 5GMm large than due to earth and due to the relative motion
Ei = + K also Ei = E f Þ K = to sun and earth with mercury.
R 6R
3. (a) As we know that, the torque on earth due to
5GM ´ 100 GM GM
\k = ´ 10 –3 = = gravitational attractive force on earth is zero.
6 ´ 3200 6 ´ 32 ´ 200 38400 As the earth is revolving around the sun in a circular
116. (a) If h is the height attained, then motion due to gravitational attraction. The force of
é1 GMm ù GMm attraction will be of radial nature i.e., angle between
0.8 ´ ê mv 2 - =-
ë2 R úû R+h position vector r and force F is also, zero.
After substituting the given values, we get So, torque = t = r ´ F = rF sin 0° = 0
h = 1655 km 4. (c) As the total (P.E.) of the earth satellite orbiting in orbit
117. (c) Angular speed of ear th = angular speed of
æ -GM ö
geostationary satellite. is negative ç ÷ , where r is radius of the satellite
If w is double, time period become half using è 2r ø
T2 µ r¢3 and M is mass of the earth.
r Due to the viscous force acting on satellite, energy
r ¢ = 1/ 3 decreases continuously and radius of the orbit or
4 height decreases gradually.
R 5. (b) The major force acting on moon is due to gravitational
118. (a) In DAOB : cos 60° = Þ OB = 2R
OB force of attraction by sun and earth and moon is not
A always in line of joining sun and earth.
Here gravitational force will
provide the required R As observed from the sun, two types of forces are
60° 30°
centripetal force. O
B acting on the moon one is due to gravitational attraction
Earth between the sun and the moon and the other is due to
GMm
Hence, = m(OB)w 2 gravitational attraction between the earth and the moon.
(OB) 2 So these two force have different lines of action and it
will not be strictly elliptical because total force on the
GM GM GM
Þ w= 3
= 3
Þw= moon is not central.
(OB) (2R) 8R 3 6. (d) Asteroids are also being acted upon by central
119. (a) gravitational forces, hence Asteroid will move in
2p 2p circular orbits like planets and obey Kepler's laws.
120. (c) Given, 8 = = 7. (d) Gravitational mass of proton is equivalent to its inertial
w1 + w 2 2 p 2p
+ mass and is independent of presence of neighbouring
T1 T2 heavy objects so verifies the option (d).
T1 = 24 hours for earth.
GMm
Þ T2 = 12 hours (T2 being the time period of satellite, 8. (c) Force of Gravitation, Fg =
it will remain same as the distance from the centre of r2
the earth remains constant). Let AB = r
So, force on B due to A
2p 2p
Þ T= = = 24 hours G (2 Mm)
w 2 - w1 2p 2p = FBA = towards BA.
- ( AB) 2
T2 T1
EBD_7418
514 PHYSICS

G (2 Mm ) (ii) In the second case Vµ = + 10 unit


= = 2 Fg
r2 Vr – 10 = – 5
and force on B due to C or Vr = + 5 unit
GMm 16. (d)
= FBC = towards BC
( BC ) 2 17. (b) The escape velocity from earth is given by
As, (BC) = 2AB
ve = 2g Re … (i)
GMm GMm GMm Fg
Þ FBC = = = = The orbital velocity of a satellite revolving around earth
(2 AB) 2 4( AB )2 4r 2 4 is given by
As FBA > FBC,
GM e
hence, m will move towards (BA) i.e., (2M). v0 =
9. (c) From question, ( Re + h )
Escape velocity where, Me = mass of earth, Re = radius of earth, h =
2GM 2GM height of satellite from surface of earth.
= = c = speed of light Þ R = 2
R c By the relation GMe = g Re2
2 ´ 6.6 ´ 10 - 11 ´ 5.98 ´ 10 24 g Re2
= m = 10– 2 m So, v0 = … (ii)
(3 ´ 108 )2 ( Re + h )
10. (b) At a height h above the surface of earth the gravitational Dividing equation (i) by (ii), we get
potential energy of the particle of mass m is 2( Re + h )
ve
GM e m =
Uh = - v0 ( Re )
Re + h v
Given, v0 = e
Where Me & Re are the mass & radius of earth 2
respectively. 2 ve 2( Re + h )
In this question, since h = Re =
ve Re
GM e m - mgRe Squaring on both side, we get
So U h = Re = - = 2( Re + h )
2 Re 2 4= or Re + h = 2Re
11. (d) 12. (b) Re
i.e., h = Re
13. (c) The gravitational force on the satellite will be aiming 18. (b) As we know, escape velocity,
towards the centre of the earth so acceleration of the satellite
will also be aiming towards the centre of the earth. 2GM 2G æ 4 3 ö
Ve = = · ç pR r ÷ µ R r
14. (d) Given: Height of the satellite from the earth's surface h R R è3 ø
= 0.25 × 106m Ve R e re V R re
\ = Þ e = e
Radius of the earth R = 6.38 × 106m Vp R p rp Vp 2R e 2r e
Acceleration due to gravity g = 9.8 m/s2
Orbital velocity, V0 = ? Ve
\ Ratio =1 : 2 2
Vp
GM GM R 2
V0 = = . 19. (a) As we know, gravitational potential (v) and acceleration
(R + h) R 2 (R + h) due to gravity (g) with height
9.8 ´ 6.38 ´ 6.38 é GM ù -GM
= –5.4 × 107
êQ 2 = g ú
= = 7.76 km/s V= …(1)
6.63 ´10 6
ë R û R +h
GM
15. (b) The gravitational potential V at a point distant ‘r’ from and g = =6 …(2)
a body of mass m is equal to the amount of work done ( R + h )2
in moving a unit mass from infinity to that point. Dividing (1) by (2)
r ur r -GM
Vr - Vµ = - ò E.dr = -GM (1/ r - 1/ µ ) -5.4 ´107 5.4 ´ 107
R +h = =6
µ Þ
GM 6 (R + h )
-GM æ r -dV ö
( R + h )2
= çè As E = ÷
r dr ø Þ R + h = 9000 km so, h = 2600 km
(i) In the first case 20. (c) T2 µ R3 (According to Kepler’s law)
- GM T12 µ (1013)3 and T22 µ (1012)3
when Vµ = 0, Vr = = -5 unit
r
GRAVITATION 515

28. (a) As we know,


T12 T1
\ = (10)3 or = 10 10 - GMm
T2 2 T2 Gravitational potential energy =
r
21. (b) For h << R, the orbital velocity is gR and orbital velocity, v0 = GM / (R + h)
Escape velocity =2gR GM GM
= =
\ The minimum increase in its orbital velocity (R + 2R) 3R
= 2gR – gR =
gR ( 2 – 1) 1 GMm 1 GM GMm
Ef = mv02 - = m -
22. (b) True weight at equator, W = mg 2 3R 2 3R 3R
Observed weight at equator, GMm æ 1 ö -GMm
= ç - 1÷ =
2
W ' = mg ' = mg 3R è 2 ø 6R
3 -GMm
At equator, latitude l = 0 Ei = +K
R
Using the formula,
Ei = E f
mg ' = mg - mRw2 cos 2 l 5GMm
Therefore minimum required energy, K =
6R
2
mg = mg – mR w2 cos20 = mg – mR w2 29. (a) U = mV = kmr.
3 dU
1/2 Force, F = - = - km
æ1 gö dr
\ w=ç
è 3 R ÷ø Now ,
mv 2
= km Þ v µ r
1/2

GM r
23. (a) As we know, orbital speed, Vorb = 2pr 2pr
r \T= = 1/2 Þ T µ r1/2
2pr 2 pr v cr
Time period T = = r
vorb GM 30. (a) Speed of the planet will be maximum when its distance
Squarring both sides, from the sun is minimum as mvr = constant.
2 B
æ 2 pr r ö 4p 2 3 T 2 4p 2 Perihelion VC
T2 = ç ÷ = .r Þ = =K
è GM ø GM r 3 GM A C
S
Þ GMK = 4p2. VA Aphelion
24. (c) Above earth surface. Below earth surface Point A is perihelion and C is aphelion.
æ 2h ö æ d ö Clearly, VA > VB > VC
gh = g ç1 - R ÷ gd = g ç1 - ÷
è eø è Re ø So, KA > KB > KC
According to question, gh = gd 31. (c) If universal gravitational constant becomes ten times,
æ 2h ö æ d ö then G¢ = 10 G
g ç1 - ÷ = g ç1 - ÷ GM
è Re ø è Re ø Acceleration due to gravity, g = 2
Clearly, R
d = 2h = 2 km So, acceleration due to gravity increases.
25. (d) Variation of g with altitude is, 32. (d) Gravitational field, I = ( 5iˆ + 12 ˆj ) N/kg
é 2h ù dv
g h = g ê1 - ú ; I =-
ë Rû dr
variation of g with depth is, éx y ù

é dù
g d = g ê1 - ú êë 0
ò
v = - ê I x dx +
ê ò
0
I y dy ú
ú
úû
ë Rû
Equating gh and gd, we get d = 2h = – éë I x .x + I y .y ùû
26. (b) Space rocket are usually launched from west to east to
take the advantage of rotation of earth. = – éë5 ( 7 - 0 ) + 12 ( -3 - 0 ) ùû
Also g' = g – w2R cos2 l, at equator l = 0, and so cos l = - éë35 + ( -36 ) ùû = 1 J / kg
= 1, and g' is least.
27. (a) Both the astronauts are in the condition of i.e., change in gravitational potential 1 J/kg.
weightlessness. Gravitational force between them pulls Hence change in gravitational potential energy 1 J
towards each other. Hence Astronauts move towards 33. (c) x=0 1 2 4 8
each other under mutual gravitional force.
EBD_7418
516 PHYSICS

GM æ1 1 1 1 ö 45
V=– = – G ç + + + + ......¥÷ Þ xM = R = 7.5 R
r è1 2 4 8 ø 6
æ 1 ö
4. (c) During total eclipse:
= – Gç = – 2G Total attraction due to sun and moon,
è 1 - 1/ 2 ÷ø
GM s M e GM m M e
F1 = +
r12 r22
EXERCISE - 3 When moon goes on the opposite side of earth.
Effective force of attraction,
Gm ( mdx )
1. (b) Q dF = GM s M e GM m M e
x2 F2 = -
r12 r22
dx
x 2GM m M e
Change in force, DF = F1 - F2 =
r22
x Change in acceleration of earth
DF 2GM m
Da = =
dx Me r22
F = Gm ò ( A + Bx )
a +L

x 2

Fav GM s
Average force on earth, Fav = = 2 % age
é æ1 1 ö ù Me r1
F = Gm ê A ç - ÷ø + BL ú change in acceleration
ë è a a + L û
2. (a) Da 2GM m r12
= ´ 100 = ´ ´ 100
3. (c) The gravitational force acting on both the masses is aav r22 GM s
the same. We know that 2
Force = mass × acceleration. æ r ö Mm
= 2ç 1÷ ´ 100
9R è r2 ø M s
5. (c) We know, g' = g – w2R cos2 q
xM x5M 3g
R 2R = g - w2R
4
12R 3
1 Given, g ' = g
For same force, acceleration µ 4
g
mass w 2R =
a5 M M 1 4
\ = = .....(i)
aM 5M 5 g 10
w= = = 0.6 × 10–3 rad/s
Let t be the time taken for the two masses to collide and 4R 4 ´ 6400 ´ 103
x5M, xM be the distance travelled by the mass 5M and
M respectively. 4 g1 Rr
For mass 5M 6. (d) g= prGR \ = 1 1
u = 0, S = x5M, t = t, a = a5M 3 g 2 R2 r2
1 1 GM
S = ut + at 2 \ x5 M = a5M t 2 7. (c) g = = GMR–2,
2 2 R2
....(ii) Dg DR
For mass M \ g ´ 100 = – 2 ´ 100 = – 2 × (–2%) = 4%
u = 0, s = xM, t = t, a = aM R
1 2 1 1 DK DI
Q s = ut + at Þ xM = aM t
2
… (iii) AlsoK = I w 2 , \ = ,
2 2 2 K I
Dividing (ii) by (iii) DI 2 DR
1 As I = KR2, so =
x5 M a5 M t 2 I R
2 a 1 DK æ DR ö
= = 5M = ´ 100÷ = 2 (–2%) = – 4%.
xM 1 aM 5 [From (i)] \ ´ 100 = 2 ç
è R ø
aM t 2 K
2 8. (d) Let mass per unit y
\ 5x5M = xM ....(iv)
m
From the figure it is clear that light of wire, l =
x5M + xM= 9R ....(v) l dm
Where O is the point where the two spheres collide. l d
From (iv) and (v) and pr = l, r = . E
xM p O r B x
+ xM = 9 R \ 6 xM = 45R
5 Gdm
mass of element, dm = lrdq then dE =
r2
GRAVITATION 517

p p
Glrdq ˆ gR 2 gR 2
(i cos q + ˆjsin q) Hence, r 3/2 = or r 3 = 2
ò dE = ò r 2 w w
0 0 or, r = (gR2/w2)1/3
p p mv 2 K
Gl é ˆ ù 12. (b) = 5/ 2 ,
E= ê ò i cos q + ò ˆjsin q ú R R
r êë 0 0 úû 2pR 2pR
\ v = CR -3/ 4 Þ T = v = -3/ 4
CR
2Gl ˆ 2GM ˆ 2Gmp ˆ \ T µ R7/2.
2
= j= j= j (along y-axis) 13. (a) Orbital velocity for close to earth orbits
r lr l2
9. (a) We know that intensity is negative gradient of = gR . Escape velocity required = 2 gR .
potential, ve - v0
% Increase required = ´ 100
i.e., I = – (dV/dr) and as here I = – (K/r), so v0
dV K gR ( 2 - 1)
dr r ò
= , i.e., dV = K =
gR
´100 = 41%
r r
or V - V0 = K log so V = K log + V0 14. (b) Gravitational force will be due to M1 only.
r0 r0
x x -3 K GMm dE GMm 1 dr
10. (d) V = - ò E dx = ò¥ Kx dx = . 15. (c) E=- Þ - =
¥
2x2 2r dt 2r r 2 dt
2 pr 2pr 2pr 3/2 2p t
GMm dr
R
GMm é 1 1 ù
11. (a) T= = = =
v0 2
( gR / r )1/2
gR 2 w ò dt = - ò
2C r r 2
; t= -
2C êë R r úû
0
EBD_7418
518 PHYSICS

9. MECHANICAL PROPERTIES OF SOLIDS

Exercise - 1 FA
20. (a) Strain =
g
F l & F = mg
1. (a) Y = . or F = YA
A Dl DL l A = pr
2

Hence force constant = F = YA 21. (c) The given graph does not obey Hooke's law. and there
Dl l is no well defined plastic region. So the graph represents
2. (c) Young's modulus of steel is highest. elastomers.
F l 1
3. (c) g= ´ Þg µ 22. (c) d
pr 2 Dl r2
4. (b) For a beam, the depression at the centre is given by,
5. (c) For a perfectly rigid body strain produced is zero for
æ fL ö
the given force applied, so d= ç ÷
Y = stress/strain = ¥ è 4Ybd3 ø
6. (b) The ratio of stress to strain is always constant. If stress [f, L, b, d are constants for a particular beam]
is increased, strain will also increase so that their ratio 1
remains constant. i.e. d µ
Y
7. (c) From the figure, it is clear that 23. (d) (A) ® (3); B ® (1, 4) ; (C) ® (2) ; (D) ® (2)
Fcom < Fext. 24. (a) From the graph l = 10–4m, F = 20 N
x
A = 10–6m2, L = 1m
FL 20 ´ 1
\ Y= =
Al 10-6 ´ 10-4

Fcom Fext
F = 20 ´ 1010 = 2 ´ 1011 N/m 2
25. (c) Since OE is a straight line so, stress µ strain.
F \ Hooke's law is obeyed in the region OE of the graph.
8. (b) strain = =10–8
pr 2 g Fl YADl
26. (d) Y = ;Þ F=
9. (c) 10. (b) 11. (d) 12. (b) ADl l
F1 A1 l 2 r2 l
13. (a) stress =
F = ´ = 12 ´ 2 = 4 ´ 1 = 1 : 1.
A F2 A 2 l1 r2 l1 1 4
1 27. (c) According to Hooke’s law
stress µ
A F Dl
Stress µ strain i.e., µ
14. (a) For most material h » g 3 A l
15. (d) The hammering increases elasticity while annealing 1
Þ For same F & l, Dl µ
decreases it. The increase in temperature increases the A
elasticity while decrease in temperature decreases it. FL 1
The impurity in the substance increases the elasticity. 28. (d) l = Þ l µ 2 (F, L and Y are same)
AY r
16. (b) Yield point is the point, beyond which the wire starts 2 2
showing increase in strain without any increase in stress. l A æ rB ö æ r ö 1
17. (d) Elastic after effect is defined as the temporary delay in = ç ÷ = ç B ÷ = Þ l A = 4l B
lB è rA ø 2
è Bør 4
regaining the original configuration by an elastic body
after the removal of a deforming force. lA
18. (b) Hydraulic stress is relevant to volumetric strain, DV/V, or l B =
but there is no change in shape. 4
29. (b) Initial length (circumference) of the ring = 2pr
normal stress F/A Final length (circumference) of the ring = 2pR
19. (c) Young's modulus = =
longitudinal strain DL / L Change in length = 2pR – 2pr
F´ L Mg ´ L 2p(R–r) R - r
= = strain = =
A ´ DL pr 2 ´ DL 2pr r
MECHANICAL PROPERTIES OF SOLIDS 519

F/A F/A d1 3 p r 4 3p r4 3
Young's modulus E = = Now = 4 = =
l / L (R - r ) / r d2 b 2 2
(p r ) p
æ R-r ö (Q b2 = pr2 i.e., they have same cross-sectional area)
\ F = AE ç ÷ 38. (c) If l is the original length of wire, then change in length
è r ø of first wire,
30. (a)
Dl1 = (l1 – l)
31. (b)
Change in length of second wire,
32. (b) Stress = 1 kg wt/mm2 = 9.8 N/mm2
Dl2 = (l2 – l)
= 9.8 × 106 N/m2.
Dl T l T l
Y = 1 ´ 1011 N / m 2 , Now, Y = 1 ´ = 2´
´100 = ? A Dl 1 A Dl 2 T
l
Stress Stress T1 T T1 T
Y= = or = 2 or = 2
Strain Dl / l D l1 Dl 2 l1 - l l 2 - l
Dl Stress 9.8 ´ 106 W
\ = = or T1l2 – T1l = T2l1 –lT2
l Y 1´ 1011
Dl T l -T l
´100 = 9.8 ´ 10 -11 ´ 100 ´ 106 or l= 2 1 1 2
l T2 - T1
= 9.8 × 10–3 = 0.0098% T1 T2
33. (d) 39. (d) For stress to be equal, A = A
34. (a) Given, the breaking strength of cable fu = 7 × 107 N/m2 1 2
The force carried by the cable, T1 A1 1
F = m( g + a ) = 2000(9.8 + 1.5) = 22600 N \ =
T2 = A2 2
F 22600
The area of cross-section, A = f = 40. (c) Maximum possible strain = 0.2/100
u 7 ´ 107
= 3.28 × 10–4 m2. F 10 4 ´ 100
\ A= = = 7 .1 ´ 10 - 4 m2
S 106 100 Y ´ strain 9
(7 ´ 10 ) ´ 0 .2
35. (a) L = = = = 34 m
dg 3 ´ 103 ´ 10 3 41. (a)
36. (c) Young’s modulus of rubber, Yrubber 42. (c) Volume i.e. (A × l) must be constant
F l Dl 43. (c) The deformation of a body on application of a force
= ´ Þ F = YA.
A Dl l depends on the nature of the material and the magnitude
On putting the values from question, of the applied force.
5 ´ 108 ´ 25 ´ 10 -6 ´ 5 ´ 10 -2 44. (a) Ratio of radii r1:r2 = 1:2
F= = 6250 N
10 ´ 10 - 2 Ratio of area, A1:A2 = pr12 : pr22
kinetic energy = potential energy of rubber
A1 : A2 = 1 : 4
1 1
mv 2 = F Dl Now, Stress1 : Stress2 = 4 : 1
2 2 So, Strain1 : Strain2 = 4 : 1
FDl 6250 ´ 5 ´ 10-2 l1
v= = = 62500 = 250 m/s 4
m 5 ´10-3 \ l = Þ 4l2 = l1 = 8
2 1
W l3 \ l2 = 2 mm
37. (a) d = , where W = load, l = length of beam and I is
3Y I Increase in length of B is 2 mm.
geometrical moment of inertia for rectangular beam, 45. (b) Using Hooke’s law, F = kx we can write
bd3 4 = k (a - l 0 ) … (i)
I= where b = breadth and d = depth
12 and 5 = k (b - l 0 ) … (ii)
b4
For square beam b = d \ I1 = If l be the length under tension 9N, then
12
9 = k (l - l 0 ) … (iii)
æ p r4 ö After solving above equations, we get
For a beam of circular cross-section, I2 = ç ÷
è 4 ø l = (5b - 4a) .
3 3 (b) Let l 0 be the unstretched length and l 3 be the length
\ d1 = W l ´ 12 = 4 W l
46.
3Y b 4 Y b4 under a tension of 9N. Then
(for sq. cross section)
4l 0 5l 0 9l 0
W l3 4 W l3 Y= = =
and d 2 = = A (l1 - l 0 ) A (l 2 - l 0 ) A (l 3 - l 0 )
3Y (p r 4 / 4) 3Y (p r 4 )
(for circular cross-section) These give
EBD_7418
520 PHYSICS

4 5 Stress F
= Þ l 0 = 5l1 - 4l 2 Þ Strain = =
l1 - l 0 l 2 - l 0 Y AY
105 105 1
4 9 = = =
Further, l - l = l - l
2 -4 11 628
pr Y 3.14 ´ 10 ´ 2 ´ 10
1 0 2 0
1
Substituting the value of l 0 and solving, Therefore % strain = ´ 100 = 0.16%
628
we get l 3 = 5l 2 - 4l1 56. (a) Case (i)
47. (c) Young’s modulus of elasticity is At equilibrium, T = W
F/A FL W/A
Y= \ DL = Y= .....(1)
DL / L AY l/L
L Case (ii)
So, D L µ At equilibrium T = W
A
W/A W/A T T
D L 2 L 2 A1 2 2 \Y = ÞY =
\ = ´ = ´ =4 l/2 l/ L
D L1 L1 A 2 1 1
L/2
DL2 = 4 × DL1 = 4 × 1 = 4 cm Þ Elongation is the same.
48. (c) W W
F ( L / 2) ( A L r g )( L / 2)
dF 57. (b) Dl = =
49. (d) = 0 at l/2 AY AY
dx
æ 1ö
50. (a) To find the minimum diameter, and hence minimum = ç ÷ r g L2 / Y
cross-sectional area, we assume that the force F = 400 è 2ø
N brings us to the elastic limit. Then from the stress, 58. (d) The three elastic moduli viz young's modulus, shear
F/A = 379 × 106 Pa, we get modulus and bulk modulus are used to describe the
400N elastic behavior of objects as they respond to
A= 6
= 1.0554 ´ 10-6 m 2 . deforming forces acting on them.
379 ´ 10 Pa
59. (b) Young's modulus Y = 2G(1 – s)
pD 2 G = modulus of rigidity and s = poissons's ratio
Then, A =
4 60. (c) 61. (c)
-6 2 62. (b) The atoms when brought from infinity are attracted
4 A 4 (1.0554 ´ 10 m )
D2 = = = 1.344 × 10–6 due to interatomic electrostatic force of attraction. At
p p point B, the potential energy is minimum and force of
-6 2 attraction is maximum. But
and D = 1.344 ´ 10 m = 1.16 × 10–3 m = 1.16 mm P.E.
51. (a) if we bring atoms closer than
52. (a) x = B, force of repulsion
between two nucleii starts x
Force C
53. (b) Breaking stress = and P.E. increases.
area
A B
The breaking force will be its own weight. 1
F = mg = Vrg = area × lrg 63. (d) Compressibility =
Bulk modulus
area ´ l ´ rg 64. (c) 65. (d)
Breaking stress = 6 ´ 10 6 =
area p h r4
66. (c) Couple per unit angle of twist, C =
2l
6 ´ 106
or l = = 200 m.
3 ´ 103 ´ 10 p h r 4q
\ Couple t = C q =
2l
Y × strain × A 2×1011 ×10-3 ´ 3 ´10 -6
54. (d) m = = = 100 kg Here h, l, C & t are same. So, r 4q = constant
2 2
q1 æç r24 ö÷
55. (a) Given: F = 100 kN = 105 N \ =
q 2 çè r14 ÷ø
Y = 2 × 1011 Nm–2
l0 = 1.0 m 67. (a) (A)®(3), (B)®(4), (C)®(2), (D)®(1)
radius r = 10 mm = 10– 2 m Fl 500 ´ 8
(A) Dl copper = AY = = 0.8 mm
Stress 0.5 ´ 10-4 ´ 1011
From formula, Y =
Strain
MECHANICAL PROPERTIES OF SOLIDS 521

Fl 500 ´ 4 kp
(B) Dl steel = AY = = 0.2 mm
81. (c) Here, kQ =
0.5 ´ 10 -4 ´ 2 ´ 1011 2
(C) Dl = Dl copper + Dl steel = 1.0 mm According to Hooke's law
\ Fp = –kp xp
é e 2Ycopper e 2Y ù
(D) U = ê + steel ú ´ Vol = 0.25 J
Fp k p xp
ë û
FQ = –kQxQ Þ F = k
2 2
68. (b) 69. (a) 70. (b) Q Q xQ

P 100 Fp = FQ [Given]
71. (c) Bulk modulus K = = = 106 atm x p kQ
DV 0.01 / 100
\ x =k ...(i)
V Q p
= 10 N/m = 10 dyne/cm2
11 2 12
1 2
Energy stored in a spring is U = kx
dV dL 2
72. (d) If side of the cube is L. then V = L3 Þ =3
V L Up k p x 2p
k p kQ2 1 é kp ù
\ % change in volume = 3 × (% change in length) = = ´ 2 = êQ kQ = ú
= 3 × 1% = 3% \ 2
U Q kQ xQ kQ k p 2 2û
ë
DV UQ E
\ Bulk strain = 0.03
V Þ Up = = [\ U Q = E ]
2 2
F 4 ´ 9.8
73. (b) K= = = 19.6 ´ 102 DP 100 ´ 105
x 2 ´ 10 -2 82. (d) B=- = = 50 ´ 109 N/m2s
DV / V (0.02 /100)
1 2 2
Work done = 19.6 ´10 ´ (0.05) = 2.45 J
2 ph( r24 - r14 ) ph r 4
83. (a) C1 = , C2 =
74. (a)
1 1
Work done = ´ Stress ´ Strain = ´ Y ´ (Strain)2 . 2l 2l
2 2 Initial volume = Final volume
Since, elasticity of steel is more than copper, hence
\ p[r22 - r12 ]lr = pr 2 lr
more work has to be done in order to stretch the steel.
F2 Þ r 2 = r22 - r12 Þ r 2 = (r2 + r1 )(r2 - r1 )
75. (c) We have, U =
2k Þ r 2 = (8.02 + 7.98)(8.02 - 7.98)
Yl Yl d2
where k = = \ Uµ Þ r 2 = 16 ´ 0.04 = 0.64 cm Þ r = 0.8 cm
A 1 l
pd2
4 C1 r24 - r14 [8.02]4 - [7.98]4
1 \ = =
76. (d) P .E . = ´ stress ´ strain ´ volume C2 r4 [0.8]4
2
1 1
or (P.E./ volume) = ´ (Y ´ strain) (strain) 84. (a) Compressibility =
2 Bulk modulus
1 2 1 2 As bulk modulus is least for ethanol (0.9) and maximum for
= Y (strain) = Y X
2 2 mercury (25) among ehtanol, mercury and water. Hence
Dr / r 1 Dr 1 Dl
77. (d) = 0.5 = , = DV
Dl / l 2 r 2 l compression in volume
Dl V
78. (a) 10
Y = 2× 10 Nm ; –2 = 0.01 Ethanol > Water > Mercury
l
Work done per unit volume dV dL
2 85. (b) = (1 + 2s)
1 1 1 æ Dl ö V L
= × stress × strain = ´ Y ´ strain2 = Y ç ÷
2 2 2 è l ø dV é 1ù
2 = 2 ´ 2 ´ 10-3 = 4 ´ 10 -3 êQ s = 0.5 = ú
1 æ 1 ö V ë 2û
\ U = ´ 2 ´ 1010 ´ ç 6
÷ = U = 10 Jm
–3
–1
2 è 100 ø \ Percentage change in volume = 4 × 10 = 0.4%
1 1 86. (b) T1 – T2 = m w02 (2l)
79. (b) W = ´ F ´ l = mgl
2 2 T2 = m w02 (3l) \ T1 = 5 m w02 l.
1
= ´ 10 ´ 10 ´ 1 ´ 10 -3 = 0.05 J
2 Energy ( A ) ( stress )2A volume A
80. (d)
1
Elastic energy = ´ F ´ x Energy ( B) = ( stress ) 2B volume B
2
F = 200 N, x = 1 mm = 10–3 m (5) 2 æ A ´ 2l ö 50
1 = ´ç ÷=
\ E = ´ 200 ´1´10 -3 = 0.1 J (3)2 è A ´ l ø 9
2
EBD_7418
522 PHYSICS

DY 1 ´ 10 -3 So, if temperature increass DT increases, hence Young's


87. (c) e = =
modulus of elasticity (Y) decreases.
l 4
2
e Y æ 10 -3 ö
2
2 ´ 1011 4. (c) When a spring is stretched by applying a load to its
u= =ç ÷ ´ = 0.075 J. free end. Clearly the length and shape of the spring
2 è 4 ø 2
changes. So strain produced when change in length
1 corresponds to longitudinal strain and change in shape
88. (a) U / volume = Y ´ strain 2 = 3600 J m–3
2 corresponds to shearing strain.
[Strain = 0.06 × 10–2] 5. (b) As we know that,
89. (c) D The Young's modulus
Stress F / A F L
Y= = = ´
F Strain DL / L A DL
F L 4 FL
= ´ =
p( D / 2) DL pD 2 DL
2

h
4 FL 4 FL
D2 = ÞD=
pDLY pDLY
Shearing strain is created along the side surface of the L 1
If F and are constants. So, D µ
punched disk. Note that the forces exerted on the disk are DL Y
exerted along the circumference of the disk, and the total Hence, we can find ratio as
force exerted on its center only.
Let us assume that the shearing stress along the side surface Dcopper Yiron
=
of the disk is uniform, then Diron Ycopper
F = ò dFmax = ò smax dA = smax ò dA 6. (a) Consider the given diagram
surface surface surface L L
æ Dö A C B
= s max .2p çè ÷ø h 90°–q 90°–q
2
8 æ1 -2 ö -2 x
= 3.5 ´10 ´ ç ´ 10 ÷ ´ 0.3 ´10 ´ 2p
è2 ø
q q
= 3.297 ´ 10 4 = h 3.3 ´ 10 4 N
1 O
90. (c) Dw = [F2 l 2 – F1 l1 ]
2 m
1 So, change in length
=
2 DL = ( A0 + B0 ) - ( AC + CB) = 2BO – 2AC
[4 ´ 10 ´ 1 ´ 10–3 – 2 ´ 10 ´ 0.6 ´ 10 –3 = 14 ´ 10 –3 J] = 2 [BO – AC] (\ AO = BO, AC = CB)
Exercise - 2 éæ x 2 ö1/ 2 ù
= 2[( x2 + L2 )1/ 2 - L] = 2 L êç1 + ÷ - 1ú
1. (b) As liquid is ideal so no. frictional force exists hence, êçè L2 ÷ø ú
tangential forces are zero so there is no stress ë û
developed. é 1x 2 ù x 2
2. (d) As we know that, DL » 2L ê1 + 2
- 1ú = [Q x << L]
Breaking force ë 2L û L
Breaking stress = ..(i)
Area of cross-section DL x 2 / L x 2
Strain = = = 2
When length of the wire changes (or by reducing half) 2L 2L 2L
area of cross-section remains same. 7. (c) Let us consider the free body diagram of the rectangular
Hence, breaking force will be same because breaking frame
stress does not depend on length. Tsinq
Tsinq
3. (d) As we know that, length of a wire when the temperature T
increased
Lt = L0 (1 + aDT ) q q
where DT is change in the temperature. Tcosq Tcosq
L0 is original length, m
a is coefficient of linear expansion and
Lt is length at temperature T.
Now, DL = Lt - L0 = L0aDT Stress
Now, Young's modulus (Y) = mg
FL0 FL0 1 Strain 1 Net forces acting on frame will be zero.
= = µ As, Y µ
A ´ DL AL0 aDT DT DT So, Balancing vertical forces
MECHANICAL PROPERTIES OF SOLIDS 523

2T sinq – mg = 0 [T is tension in the string] DR P


2T sinq = mg ...(i) Þ =
R 3B
Total horizontal force = T cos q - T cos q = 0 250 ´ 9.8
F/A -6
Now from Eq. (i), T =
mg
i.e. Tmax =
mg 14. (c) Y= = 50 ´ 10
2 sin q 2sin qmin Dl / l 0.5 ´ 10-3
2
1
As mg is constant then T µ 250 ´ 9.8 2
sin q = ´ Þ 19.6 ´ 1010 N / m 2
50 ´ 10 - 6 0.5 ´ 10 -3
sin qmin = 0 Þ qmin = 0 15. (b) The incompressibility of solids is primarily due to the
No option matches with q = 0° tight coupling between the neighbouring atoms.
Molecules in gases are very poorly coupled to their
mg
Tmin = (since, sinqmax = 1) neighbours.
2sin qmax
16. (a) Wire 1: F
sin qmax = 1 Þ q = 90° A, 3l
So tension is all three cases are different rejects option (a) æ F ö
For minimum tension q must be 90° i.e. sin q = 1 Dl = ç ÷ 3l ...(i)
è AY ø
Hence, tension is the least for the case (b). Wire 2: F¢
8. (d) A mass M is attached at the centre 3A, l
or midpoint of rod of rubber and æ F¢ ö
steel. As the mass is attached to Dl = ç ÷l ...(ii)
è 3 AY ø
both the rods, both rod will be From equation (i) & (ii),
elongated as shown in figures M M æ F ö æ F¢ ö
but due to different elastic Dl = ç ÷ 3l = ç ÷ l or, F¢ = 9 F
è AY ø è 3 AY ø
properties of material rubber 17. (a)
changes shape also. (Rubber) (Steel) volumetric stress
As the Young's modulus of rigidity for steel, is larger 18. (c) Bulk modulus, K = volumetricstrain
DL mg
than rubber, so strain for rubber is larger than K=
L æ dV ö Þ dV = mg ....(i)
steel for same stress. aç ÷ V Ka
F è V ø
A Þ Dl = Fl 4 3
9. (b) As Y = Dl volume of sphere , V = pR
AY 3
l dV 3dr
Fractional change in volume = ...(ii)
V Fl 2 V r
But V = Al so A = Þ Dl = µ l2 3dr mg
l VY Using eq. (i) & (ii) =
Hence graph of Dl versus l2 will give a straight line. r Ka
10. (a) dr mg
\ = (fractional decrement in radius)
W l r 3Ka
11. (a) Young's modulus Y = .
A Dl
W1 W2 A, l, Dl l, A, Dl
Exercise - 3
= [ Q A, l, Dl same Y2 Y1
Y1 Y2 1. (c) Given that F/A = 4.8 × 107 Nm–2
for both brass and steel] Brass Steel \ F = 4.8 ×107 ×A or
mv2
W1 Y1 = 4.8 ´ 10 7 ´ 10 - 6 = 48
= = 2 [Ysteel/Ybrass = 2 given] r
W2 Y2
m r 2 w2 2 48
12. (a) As stress µ strain2 hence graph (a) correctly dipicts. or = 48 or w =
r mr
13. (c) Bulk modulus is given by
æ 48 ö
P DV P w= ç ÷ = 16 = 4 rad / sec
B= or = è 10 ´ 0.3 ø
æ DV ö V B
ç ÷ Weight of load + Weight of puller
è V ø 2. (d) = Compressive stress
area
DR P DR
3 = (here, = fractional decreases in radius) 20A ´ d ´ 10 + 5 ´ 105
R B R Þ = 1.6 ´ 106
area
Where d is the density.
EBD_7418
524 PHYSICS

20A ´ 2.5 ´ 103 ´ 10 + 5 ´ 105 F Dl Fl


= 1.6 ´ 106 8. (a) Young's modulus Y = / Þ Y< 2
A A l pr Χl
ie, 5× 105 = 1.1× 106A Given, radius r = 5mm, force F = 50pk N,
or A = 0.45m2 l F l
l1 r1 Y1 < 0.01 mm \ Y = 2 = 2 × 1014 N / m2.
3. (d) Given, l = a, r = b , Y = c Χl pr Dl
2 2 2 9. (c) Consider a small element dx of radius r,
Let Young’s modulus of steel be 2R R
r< x∗R
Y1, and that of brass be Y2 L x
T At equilibrium change in
F1l1 T dx r
\ Y1 = A Dl …(i) Brass
Steel
length of the wire
1 1
1
Mg dx L
F2l2
and Y2 = A Dl …(ii)
ò dL < ò é 2R ù 2
2 2 0 pê x ∗ Rú y 3R
2g êë L úû Mg
Dividing equation (i) by
T 2g Taking limit from 0 to L
equation (ii), we get
Y1 F1 A2 l1Dl2 é ù
= ê ú
Y2 F2 A1l2 Dl1 …(iii) ê ú
Mg ê 1 L ú MgL
ΧL < ê, ´ ú<
Force on steel wire from free body diagram p y ê é 2Rx ù L 2R ú 3p R 2 y
T = F1 = (2 g ) Newton ê ê ∗ Rú ú
ê êë L úû 0 ú
ë û
Force on brass wire from free body diagram
The equilibrium extended length of wire = L + DL
F2 = T1 = T + 2 g = 4 g Newton
MgL æ 1 Mg ö÷
< L çç1 ∗
Now putting the value of F1, F2 in equation (iii), we get
< L∗ ÷
3p R 2 Y èç 3 p YR 2 ø÷
Y1 æ 2 g ö æ pr22 ö é l1 ù æ Dl2 ö 1 æ 1 ö æ Dl2 ö
= ç ÷ .ê ú . = .a 10. (b) 11. ( b)
Y2 çè 4 g ÷ø è pr12 ø ë l2 û çè Dl1 ÷ø 2 çè b2 ÷ø çè Dl1 ÷ø
12. (b) The change in length of rod due to increase in
Dl1 a Y temperature in absence of walls is
= 2´ 2 = a ´1 = a Dl = l µ DT = 1000 × 10–4 × 20mm = 2mm.
Dl 2 2b Y1 2b 2 c 2b 2 c
4. (b) But the rod can expand upto 1001mm only.
5. (d) Yc ´ ( DLc / Lc ) = Ys ´ (DLs / Ls ) At that temperature its natural length = 1002 mm.
\ Compression = 1mm
æ 1´ 10 -3 ö æ DL ö Dl 1
Þ 1´1011 ´ çç ÷ = 2 ´1011 ´ ç s ÷ \ Mechanical stress = Y = 1011 ´ = 108 N / m 2
è 1 ø
÷ è 0.5 ø l 1000
13. (b) We know that Y = F L/p r2 l or r2 = F L/(Y p l)
0.5 ´10-3
\ DLs = = 0.25 mm \ R 2B = F L /(YB p l) and R S2 = F L /(YS p l)
2
Therefore, total extension of the composite wire R 2B YS 2 ´ 1010
or = = =2
= DLc + DLs R S2 YB 1010
= 1 mm + 0.25 mm = 1.25 mm
æ Fö or R 2B = 2 R S2 or R B = 2 R S
6. (c) Given , y = 2 ´ 1011 Nm-2 ; Stress ç ÷ = 5 ´ 107 Nm -2
è Aø
DV = 0.02% = 2 × 10–4 m3 \ RS = R B / 2
stress æ Dl ö g 14. (d) F – T = 3a; T = 2a Þ T = 2.5 × 109 × 4 × 10–8
g= Þ strain ç ÷ = … (i) T = 100 N, T = 2a Þ 100 = 2a; a = 50 N
strain è l 0 ø stress
F = 5 × 50 = 250 N
DV = 2prl 0 Dr - pr 2 Dl … (ii) 15. (c) Using the usual expression for the Young’s modulus,
From eqns (i) and (ii) putting the value of Dl, l0 and DV the force constant for the wire can be written as k
and solving we get
Dr F YA
= 0.25 ´10-4 = = where the symbols have their usual
r Dl L
F Dl Dl meanings. Now the two wires together will have an
7. (b) F = – K Dl and g = = F = gA ´ é k1k2 ù
A l l
effective force constant ê k + k ú . Substituting the
æ gpd ö
2
2 ë 1 2û
=ç ÷ Dl Þ k µ gd / l corresponding lengths and the Young’ss moduli we
è 4l ø get the answer.
10. MECHANICAL PROPERTIES OF FLUIDS

Exercise - 1 Total weight of iceberg


= weight of water displaced by iceberg.
1. (c) Liquid pressure depends upon the height of liquid
column and is independent of the shape of liquid V.r ice g = Vi .rwater .g Þ Vi = rice = 0.92
V rwater 1.03
surface and the area of liquid surface. The liquid at rest
exerts equal pressure in all directions. Thus the fraction of total volume of iceberg above the
2. (d) Hydraulic machines & lifts are based on sea level
F F é æ 0.92 ö ù
P1 = P2 ; 1 = 2 ê V – çè 1.03 ÷ø V ú
A1 A 2 æ V - Vi ö
=ç ´ 100% = ë û
´ 100%
3. (a) Archimedes's principle states that when a body is è V ÷ø V
immersed in a liquid, its loses its weight and the loss in 0.11
æ 0.92 ö
weight of body is equal to the weight of the liquid = ç1 - ÷ ´ 100% = 1.03 ´ 100% » 11%
displaced by the immersed part of the body. è 1.03 ø
14. (b) The volume of liquid displaced by floating ice
40
4. (c) Volume of ball = = 50 cm3 M
0.8 VD =
Downthrust on water = 50 g. sL
Therefore reading is 650 g. M
Volume of water formed by melting ice, VF =
5. (b) sw
6. (d) Pressure applied to enclosed fluid is transmitted equally
M M
in all direction according to Pascal law. If sL > sw, then < i.e., VD < VF
7. (c) P = hrg i.e. pressure does not depend upon the area s L sw
of bottom surface. i.e., volume of liquid displaced by floating ice will be
8. (d) From the figure it is clear that liquid 1 floats on liquid 2. lesser than water formed and so the level if liquid will
rise.
The lighter liquid floats over heavier liquid. Therefore
15. (d) Weight of submerged part of the block
we can conclude that r1 < r2
Also r3 < r2 otherwise the ball would have sink to the 1
W = v (Density of water) g ...(i)
bottom of the jar. 3
Also r3 > r1 otherwise the ball would have floated in 2
Excess weight, = weight of water having volume of
liquid 1. From the above discussion we conclude that 3
the block.
r1 < r3 < r2.
2
9. (b) The candle floats on the water with half its length above W ' = v (Density of water) g ...(ii)
and below water level. Let its length be 10 cm. with 5 3
Dividing (ii) by (i),
cm. below the surface and 5 cm. above it. If its length is
W' 2/3
reduced to 8 cm, it will have 4 cm. above water surface. = \ W ' = 2W Þ W ' = 2 ´ 6 = 12 kg
So we see tip going down by 1 cm. So rate of fall of tip W 1/ 3
16. (c)
= 1 cm/hour.
10. (c) Specific gravity of a body is defined as ratio of weight
of body in air to the loss of weight of body in water at Oil
4°C.
Vsg s Mercury
= = = Relative density of the body..
Vswg sw
11. (c)
As the sphere floats in the liquid. Therefore its weight
12. (c) In case of iron needle, the weight of water displaced by
will be equal to the upthrust force on it
the needle is much less than the weight of the needle,
hence it sinks but in case of a large iron ship the weight 4 3
Weight of sphere = pR rg … (i)
of water displaced by the ship is higher than the weight 3
of the ship, hence it floats in water. Upthrust due to oil and mercury
13. (d) Let Vi be the volume of the iceberg inside sea water 2 2
= pR3 ´ s oil g + pR3s Hg g … (ii)
and V is the total volume of iceberg. 3 3
EBD_7418
526 PHYSICS

Equating (i) and (ii) 26. (c) Weight of the bowl = mg


4 éæ D ö æ d ö ù
4 3 2 2 3 3
pR rg = pR3 0.8 g + pR3 + 13.6 g = V r g = p êç ÷ - ç ÷ ú r g
3 3 3 3 êè 2 ø è 2 ø ú
Þ 2r = 0.8 + 13.6 = 14.4 Þ r = 7.2 ë û
17. (c) Since pressure is transmitted undiminished throughout Where D = Outer diameter
the fluid (Pascal's law) d = Inner diameter, r = Density of bowl
A p (5 ´ 5) Weight of the liquid diplaced by the bowl
F1 = 1 F2 = (1350 ´ 9.81) » 1.5 × 103 N 3
A2 p (15 ´ 15) 4 æDö
= V sg = p ç ÷ sg
The air pressure that will produce this force is 3 è2ø
where s is the density of the liquid
F1 1.5 ´ 103
P= = = 1.9 ´ 105 Pa For the floation
A1 p (5 ´ 10-2 m) 2
4 éæ D ö æ d ö ù
3 3 3
4 æDö
18. (a) p ç ÷ sg = p êç ÷ - ç ÷ ú pg
3 è2ø 3 êè 2 ø è 2 ø ú
19. (d) The pressure outside the submarine is ë û
P = Pa + rgh 3 é 3 3ù
æ1ö æ1ö æd ö
Pressure inside the submarine is Pa. Þ ç ÷ ´1.2 ´103 = êç ÷ - ç ÷ ú 2 ´ 104
Net pressure acting on the window is è2ø êëè 2 ø è 2 ø úû
Pg = P – Pa = rgh By solving we get d = 0.98 m.
= 10–3 kgm–3 × 10 ms–2 × 2000 m = 2 × 107 Pa 27. (a) Let L = PQ = length of rod
Area of window is L
\ SP = SQ =
A = 50 cm × 50 cm = 2500 × 10–4 m2 2
Force on the window is Weight of rod, W = Alrg. acting
F = PgA = 2 × 107 Pa × 2500 × 10–4 m2 At point S
= 5 × 106 N Q

R
20. (b)
R FB S L
r concrete h = L/2
P W
q
And force of buoyancy,
(mconcrete + msawdust)g = Vrwg FB = Alr0g. [l = PR]
4 4 4 Which acts at mid-point of PR.
2.4 ´ p ( R 3 - r 3 ) + 0.3 ´ pr 3 = pR3 ´ 1 or R3 = 3 r 3
3 3 3 2 For rotational equilibrium.
4 3
rconcrete p ( R - r )3 æ 3 3 3ö
r -r ÷ l L
mconcrete
3 2.4 çè 2 ø Alr0 g ´ cos q = ALrg ´ cos q
\ msawdust = = =4 2 2
4 3 0.3 r 3
rsawdust pr l 2
r l r
3
21. (d) Due to acceleration down the plane, both body B and Þ 2 =r ÞL= r
L 0 0
liquid will received the same component of acceleration
down the plane. h L 1 r0
From figure, sin q = = =
22. (a) 23. (b) 4 2
pr rwg
l 2l 2 r
3
24. (c) T1 = T2 , Mg + 2T1 28. (a) Pressure difference between lungs and atmosphere
= (760 –750) mm of Hg
4
= pr3rw g T1 = 250 2N = 10 mm of Hg = 1 cm of Hg
3
45° 45°
Let the boy can suck water from depth h. Then
T1 T2 Pressure difference = hrwater g = 1 cm of Hg
Mg or, h × 1g cm–3 × 980 cm s–2
= 1 cm × 13.6g cm–3 × 980 cm s–2
25. (a) Total cross-sectional area of the thigh bones \ h = 13.6 cm
A = 2 (10 × 10–4) = 2 × 10–3 m2 The boy can suck water from the depth of 13.6 cm
Force acting on the bones = mg = 40 × 10 = 400 N 29. (d) As the block moves up with the fall of coil, l decreases,
F 400 similarly h will also decrease because when the coin is
\ Pav = A = = 2 × 105 N/m2 in water, it displaces water equal to its own volume only.
2 ´ 10-3
MECHANICAL PROPERTIES OF FLUIDS 527

Dp Dp on both the bricks. Hence statement 1 is true, statement


30. (b) Bulk modulus, B = -V0 Þ DV = -V0
2 is a correct explanation for statement 1.
DV B
é Dp ù 36. (a)
Þ V = V0 ê1 -
ë B úû
-1
é Dp ù é Dp ù
\ Density, r = r0 ê1 - ú = r0 ê1 +
ë B û ë B úû
where, Dp = p - p0 = hr0 g
= pressure difference between depth and surface of
ocean
é r0 gy ù
\ r = r0 ê1 + B ú (As h = y)
ë û
31. (a) Let r be the density of the material. r0 be the density
of water when the sphere has just started sinking, the weight Suppose Pgas is the pressure of the gas on the oil. As
of the sphere = weight of water displaced (approx). the points A and B are at the same level in the mercury
4 ( 3 3) 4 columns, so
Þ p R - r rg = pR 3r0 g
3 3 PA = PB
( R 3 - r3 ) r0 r (7)
1/ 3
or Pgas + roil ghoil = Pa + r Hg g hHg
Þ ( R 3 - r 3 ) r = R 3r0 Þ = Þ =
R3 r R 2 or Pgas + 820 ´ 9.8 ´ (1 + 1.50) =
32. (c) Let us consider a small dotted segment of thickness dx Pa + 13.6 ´ 103 ´ 9.8 ´ (1.5 + 0.75)
for observation. 3
A (dx) or Pgas + 20.09 ´ 10 = Pa + 299.88 ´ 103
Since, this segment is accelerated towards D
= 299.88 ´ 103 – 20.09 ´ 103
right, a net force is acting in this segment \ Pgas – Pa
towards right from the liquid towards the or [Pgas]gauge = 279.8 ´ 103 N/m2
left of ABCD. According to Newton's third B C = 2.8 ´ 105 N/m2
law, the segment ABCD will also apply a force on the previous Absolute pressure of gas
section creating a pressure on it which makes the liquid rise. [Pgas] absolute = [Pgas]gauge + Pa
33. (b) Let V be the volume of iceberg and let x be the fraction = 2.8 ´ 105 + 1.01 ´ 105
of volume above water. = 3.81 ´ 105 N/m2
Using law of floatation, weight of floating body = weight of 37. (a) Mass of liquid in horizontal portion of U-tube = Adr
liquid displaced by part of the floating body inside the liquid. Pseudo force on this mass = Adra
Therefore, Vrice g = (1 – x) Vrwaterg. Force due to pressure difference in the two limbs
Using the value of rice and rwater, we get x = (13/103). = (h1rg – h2rg) A
34. (b) V is the vertex of the cone and VB is the generator in Equating both the forces
contact with the horizontal table. (h1 – h2) rgA = Adra
A L
A is the highest point of the Adra ad
cone and the horizontal Þ (h1 – h 2) = rgA = g
plane passing through A is a
38. (b) Beyond the critical speed, the flow of fluids becomes
the level of the free- O turbulent as the flow loses its steadiness.
surface. Le t a be t h e 39. (b)
semi-vertical angle of the 40. (d) Reynold’s number N for turbulent motion is more than
cone, then Ð AOL = a. vρr
Let h be the height of the V a 3000 and N = i.e., N µ vρ
N B η
cone, 41. (c) In Bernoulli’s theorem only law of conservation of
then the radius of the base of the cone = h tan a. energy is obeyed.
Also the depth of O, the C.G. of the base of the cone below 42. (b)
the free surface 43. (d)
= OL = AO cos a = h tan a cos a = h sin a 44. (d) For streamline flow, all are correct.
Thrust on the base of the cone 45. (c)
= wzs = w.OL.p (h tan a)2 46. (a)
é1 3 2 ù
= p w h sin a h2 tan2 a = 3 sin a ê ph tan a.w ú 47. (a) Air flows just above the roof and there is no air flow
ë 3 û
just below the roof inside the room. Therefore v1 = 0
= 3 sin a [weight of the liquid contained in the cone]
35. (a) Since the net buoyant force on the brick completely and v2 = v. Applying Bernaulli’s theorem at the points
submerged in water is independent of its depth below inside and outside the roof, we obtain.
the water surface, the man will have to exert same force (1/2) rv12 + rgh1 + P1 = (1/2) rv22 + rgh2 + P2.
Since h1 = h2 = h, v1 = 0 and v2 = v1
EBD_7418
528 PHYSICS

P1 = P2 + 1/2 rv2 Þ P1 – P2 = DP = 1/2 rv2. Fv B=Vr 2 g


\ V r1 g = V r2 g + kvt2
Since the area of the roof is A, the aerodynamic lift
exerted on it = F = (DP) A Þ F = 1/2 rAv2
where r = density of air = 1.3 kg/m3
A = 20 m2, v = 29.44 m/sec. Vg (r1 - r 2 )
\ vt =
Þ F = {1/2 × 1.3 × 20 × (29.44)2} N = 1.127 × 104 N. k
48. (b) 63. (b) W=V r 1 g
49. (b) According to Bernoulli’s theorem, vf 2 2
v = W
3 f 3 eff
P +12 2
r v = P0 + 0
P
1 2
So, DP = rv Weff = vf
2
1 P0
F = DPA = rv2A
2 Weff
Weff
1 When the When the ball attains
= × 1.2 × 40 × 40 × 250 = 2.4 × 105 N (upwards) ball is released terminal velocity When the ball attains
2 2/3 of terminal velocity
50. (a) When the ball is just released, the net force on ball is
51. (c) Weff (= mg – buoyant force)
mg 1 2 The terminal velocity vf of the ball is attained when net
52. (b) hrg + = rv
A 2 force on the ball is zero.
1 1
53. (c) Pa + r1v1 + 0 = Pa + r2v22 + (r1gh1 + r2gh2)
2
\ Viscous force 6phr vf = Weff
2 2
2
When the ball acquires rd of its maximum velocity
r2 3
As v2 << v1, \ v1 = 2 g (h1 + h2 ). 2
r1 vf the viscous force is = Weff
3
54. (a) From the equation of continuity,
2 1
A æ Aö Hence net force is Weff - Weff = Weff
Rate = r = Vp + ç A - ÷ VQ 3 3
3 è 3ø \ required acceleration is a/3
64. (d) T + mg = Fb Fb
3r 3 ´ 0.1
\ VP + 2VQ = = = 30m / s \ T = Fb – mg
A 10-2
= Vrwg – mg
As VP = 20 m/s, so VQ = 5 m/s.
m
55. (a) = r g - mg
56. (a) As the temperature rises the atoms of the liquid become
( w / h) w
r T
mg
more mobile and the coefficient of viscosity falls. = (h – 1)mg.
57. (b) When terminal velocity is reached then body moves
with constant velocity hence, accelesation is zero. 65. (c)
66. (d) Using the formula of the terminal velocity of a body
58. (c)
falling through a viscous medium,
59. (d)
60. (a) For low velocity, the water layer in contact with the 2r 2 (r - s)g 2r 2 (r - s)g
V= Þ h=
river bed can be assumed to be stationary. 9h 9v
Since the velocity of water layers, increase from 0 to 2 Where r is the density of material of body and s is the
m/s over a vertical height of 2m so, the velocity gradient
density of medium.
Dv 2 - 0 In case of the air bubble r = 1 and s = 1.47 × 103 kg/ms
= = = 1 s -1
Dy 2 and the air bubble rises up.
æ Dv ö 2r 2 sg
From viscous drag F = hA ç ÷ (numerically), h=
è Dy ø 9V
we have F = 0.90 × 10–2 × 8 × 104 × 1 = 720 dyne 2 ´ (10 -2 ) 2 ´ 1.47 ´ 103 ´ 9.8
2 ´ 1.47 ´ 9.8 ´ 10
= =
To maintain, uniform speed, an external force equal to 9 ´ 0.21 ´ 10 –2 9 ´ 0.21
the viscous drag F should be applied. So, the required 3 4
= 1.52 × 10 decapoise = 1.52 × 10 Poise
force is 720 dyne. 67. (c)
61. (c) 68. (a) dv = 8 cm/s and dx = 0.1 cm
62. (a) The condition for terminal speed (vt) is
dv 8
Weight = Buoyant force + Viscous force Velocity gradient = = = 80 / s.
dx 0.1
MECHANICAL PROPERTIES OF FLUIDS 529

r - r0 shearing stress
69. (b) vµ Coefficient of viscosity, h =
h strain rate
\ Shearing stress = h × strain rate
v2 r - r02 h1 7.8 –1.2 8.5 ´ 10 -4 ´ 10 -2 5
\ v = r-r ´ h = ´ = 10 ´ = 10–2 Nm–2
1 01 2 7.8 –1 13.2 5
= 6.25 ×10–4 cms–1 79. (d) Surface tension of a liquid is due to force of attraction
70. (a) between like molecules of a liquid i.e., cohesive force
between the molecules.
71. (c) Velocity of efflux v = 2gh = 2 ´10 ´ 0.2 = 2 ms -1
80. (c) Excess pressure inside the soap bubble
4
72. (d) From stoke's law, F = 6phR1v, and V = pR 3 p=
4s
\ pµ
1
3 r r
æ 4 3 ö 81. (a) Surface tension = workdone per unit area in increasing
F' = 6phR2v, ç volume 8 V = p(2R) ÷
è 3 ø the surface area of a liquid under isothermal condition.
= 6ph(2R)v = 2F 82. (b) Over a small temperature ranges, S.T. of water decreases
73. (c) According to equation of continuity linearly with rise of temperature.
A1V1 = A2V2 + A3V3 83. (c) Angle of contact is acute.
84. (c) T ´ 2pr + mg = Fb
Þ 4 × 0.2 = 2 × 0.2 +0.4 × V3 Þ V3 = 1 m/s.
4 é4 3ù Fb
1 2 1 2
74. (a) P1 + rv1 = P2 + rv 2 or T ´ 2 pr + r pr 3 g = ê 3 pr ú sg
3 ê ú
2 2 ë 2 û
\ v 22 - v12 = 2gh ...(1) 3T T × 2p r
According to the equation of continuty \ r= g (2r - s)
A1v1 = A 2 v2 ...(2) 85. (d) T1 + T cos (p – q) = T2 mg
From equation (2),
T
A1 v 2 6 6
= = or, v 2 = v1
A 2 v1 10 10 p–q q
Putting this value of v2 in equation (1)
2
T1 T2
æ6 ö
ç v1 ÷ - ( v1 ) = 2 ´10 ´ 5
2 3
è 10 ø T2 – T1 T –T
\ cos (p – q) = \ cos q = 1 2
éQ g = 10m / s 2 = 103 cm / s 2 ù T T
ê ú 86. (a) The surface tension of liquid at critical temperature is
ëêand h = 5 cm ûú zero.
10 1
Solving we get v1 = 87. (c) Excess of pressure in a soap bubble, P = 4T/r i.e., P µ
8 r
Therefore the rate at which water flows through the therefore pressure in a smaller bubble is more than that
6 ´ 10 of a bigger bubble. When two bubbles of different radii
tube = A1v1 = A 2 v 2 = = 7.5cc / s are in communication, then the air flows from higher
8
75. (c) pressure to lower pressure i.e. from smaller bubble into
larger one.
2 r 2 (r - r0 )g
76. (b) Terminal velocity, v 0 = 88. (a) When two drops merge together to form one drop, the
9h surface area of drop will decrease, due to which the
2 ´ (2 ´10 -3 ) 2 ´ (8 - 1.3) ´103 ´ 9.8 energy of bigger drop is less than the sum of the energy
= = 0.07 ms–1 of two smaller drops. Due to it, the energy is released.
9 ´ 0.83
77. (c) If r is the density of the ball and r¢ that of the another 89. (c) 90. (b) 91. (c) 92. (d) 93. (a)
ball, m for the balls are the same, but r¢= 2r 94. (d) Surface tension of oils and paints is kept low so that it
can spread over larger area.
\ mg = 6prhv (by Stoke's law) 95. (a) When a highly soluble salt (like sodium chloride) is
v dissolved in water, the surface tension of water
or, 6prhv = 6p2rhv¢ So, v¢ = increases.
2
78. (b) h = 10–2 poise 96. (b) When a sparingly soluble salt (like detergent) added
18000 to water, the surface tension of water decreases.
v = 18 km/h = = 5 m/s 97. (b) The surface tension of oil is less than that of water, so
3600
l =5m the oil spreads as a thin layer.
v 98. (b) Kerosene oil rises up in wick of a lantern because of
Strain rate = capillary action. If the surface tension of oil is zero,
l
then it will not rise, so oil rises up in a wick of a lantern
due to surface tension.
EBD_7418
530 PHYSICS

2 T cos q This excess pressure is provided by charge on bubble.


99. (a) h= ; The liquid will rise i.e., h is positive
rr g 4T s 2 4T Q2 é Q ù
= ; r = s=
if cosq is +ve; It is so if q < 90º or q is acute. r 2e 0 16p 2 r 4 ´ 2e 0 êë ú
4pr 2 û
100. (a) With the increase in temperature, the surface tension
of liquid decreases and angle of contact also decreases. Q = 8pr 2rTe 0
101. (b) Surface tension depends on the temperature like 111. (c) Water fills the tube entirely in gravity less condition
viscosity, surface tension decreases with rise in i.e., 20 cm.
temperature. 2T
2 ´ 0.07 112. (d) For rise in capillary, the formula is h =
2T rρg
102. (b) = = 103 N/m 2
r 0.14 ´ 10 -3
2T
2T So, for first capillary tube h1 =
Pressure applied, = Pa - = 105 – 103 = 99 × 103 N/m2 r1ρg
r 2T
103. (c) For second, h 2 =
r2 ρg
104. (d) Surface tension = 0.075 N/m; diameter = 30 cm = 0.30 m
\ Force = 0.075 × 0.30 = 0.0225 N = 2.25 × 10–2N. h1 r2 3 r é r ù
= Þ = 1 r2 = 1 ú
105. (c) Work done = Surface tension × increase in area of the h2 r1 h 2 3×r1 êë 3û
film h2 = 9mm
113. (d) In the satellite , the weight of the liquid column is zero.
W = S × DA
So the liquid will rise up to the top of the tube.
Increase in area = Final area – initial area 114. (a) Weight of the liquid column = T cosq × 2pr.
= 10 × (0.5 + 0.1) – 10 × 0.5 = 1 cm2 For water q = 0°. Here weight of liquid column
\ W = 72 × 2 × 1 = 144 erg W = 7.5 × 10–4 N and T = 6 × 10–2 N/m. Then
[Q There are 2 free surfaces; \ DA = 2 × 1]. circumference, 2pr = W/T = 1.25 × 10–2m
106. (c) Work done, W = S [2 × (Change in area)]
4T
[ Q there are two free surface] 115. (b) Pressure inside tube = P = P0 +
r
W
Surface tension, S = \ P2 < P1 (since r 2 > r1)
2 ´ (change in area)
Hence pressure on side 1 will be greater than side 2. So
3 ´ 10 -4 air from end 1 flows towards end 2
= = 3 × 10–2 N/m
2 ´10(11 - 6) ´ (10 -2 ) 2
107. (a) W = TDA = 4pR2T(n1/3 – 1)
= 4 × 3.14 × (10–2)2 × 460 × 10–3 [(106)1/3 –1] = 0.057
108. (c) Work done = Change in surface energy
2 1
Þ W = 2T × 4p ( R22 - R12 )
= 2 × 0.03 × 4p[(5)2 – (3)2] × 10–4J = 0.4 p mJ Subhemispherical Hemispherical
4S
radius = r2 radius = r1
109. (d) Excess pressure inside the soap bubble = F 6.28 ´ 10-4
r T= = = 5 ´10 –2 N / m
116. (b)
4S 2pr 2 ´ 3.14 ´ 2 ´10 -3
So the pressure inside the soap bubble = Patm +
r 2T 2 ´ 6 ´ 10 –2
From ideal gas equation PV = nRT 117. (b) h= = = 2.4 ´ 10-2 m = 2.4cm
rdg 5 ´10-4 ´ 103 ´ 10
æ 4S ö 4 3
çè 8 + r ÷ø 3 p(rA ) 2s cos q
PAV A nA n 118. (d) Since h =
= Þ
A
= A h rg
PBVB nB æ 4S ö 4 3 nB if q is obtuse, cos q is negative. Hence h is negative
çè 8 + r ÷ø 3 p(rB ) and water is depressed in the tube.
B
Substituting S = 0.04 N/m, rA = 2cm, rB = 4 cm. 119. (b) For capillary rise, according, to Zurin’s law
nA 1 nB h1r1 = h2r2
= \ = 6. 6 × 1 = h2 × 2 Þ h2 = 3 cm
nB 6 nA
4T 2s cos q hr
110. (a) Inside pressure must be greater than outside pres- 120. (c) h = Þsµ
r rrg cos q
sure in bubble. s w h w r w cos q m 10 ´ 1 cos 135°
Þ = ´ = ´
pa s m cos q w h m r m cos 0° - 3.1 ´ 13.6
pa 10 ´ (-0.707 ) 1
= »
- 3.1 ´13.6 6
MECHANICAL PROPERTIES OF FLUIDS 531

121. (c) Diameter at 2nd section (d2) = 3.75.


10 According to equation of continuity,
122. (c) m × 10 = 2 × 3 × 10 –2 × for cross-sections A1 and A2.
100
or m = 6 × 10–4 kg = 6 × 10–4 × 103 g = 0.6 g A1 v1 = A2 v2
2
123. (d) Freq = mg + 2 [T (2 p R)] [ T = 75 ´ 10–3 N/m ] v1 A2 p(r22 ) æ r2 ö
= = =ç ÷
= 0.1 + 2 [75 ´ 10–3 (0.2)] = 0.130 N v2 A1 p(r12 ) è r1 ø
124. (b) None of the given option is correct. 2
When the bubble gets detached, æ d2 ö
ç ÷ æ d2 ö
2
Bouyant force = force due to surface tension è 2 ø
= =ç ÷ é d2 d1 ù
æ d1 ö
2
è d1 ø êQ r2 = 2 , r1 = 2 ú
ç ÷ ë û
è 2ø
2
æ 3.75 ö 9
=ç ÷ =
R è 2.5 ø 4
q \ v1 : v2 = 9 : 4
5. (c) We observed that meniscus
rq of liquid is convex shape as convex
T×dl
shown in figure which is
Force due to excess pressure = upthrust 140°
possible if only, the angle of
2T contact is obtuse. Hence, the
Excess pressure in air bubble =
R combination will be of case of mercury
3 mercury-glass (140°). Hence
2T 4p R
(p r 2 ) = rw g verifies the option (c).
R 3T
6. (c) As surface area decreases so energy released.
2 R 4 rw g 2 2rw g
Þ r2 = Þ r=R = 4pR2T[n1/3 – 1] where R = n 1/3r
3T 3T
125. (d) At equilibrium, weight of the given block is balanced 3 é1 1 ù é1 1 ù
= 4pR T ê - ú = 3VT ê - ú
by force due to surface tension, i.e., ër R û ër R û
2L. S = W 7. (a) Velocity of water from hole
-2
or S = W = 1.5 ´ 10 N
= 0.025 Nm -1
A = v1 = 2gh
2L 2 ´ 0.3 m Velocity of water from hole B
= v2 = 2 g ( H 0 - h)
Exercise - 2
Time of reaching the ground from hole B
1. (c) When the pebble is dropped from the top of cylinder = t1 = 2( H 0 - h ) / g
filled with viscous oil and pebble falls under gravity
Time of reaching the ground from hole A
with constant acceleration, but as it is dropped it enter
in oil. So dragging or viscous force is = t 2 = 2h / g
F = 6phrv 8. (c)
where r is radius of the pebble, v is instantaneous speed, 9. (a) (54–x )
h is coefficient of viscosity. 54 cm P
8 cm x
As the force is variable, hence acceleration is also
variable so v-t graph will not be straight line due to
viscosity of oil. First velocity increases and then
becomes constant known as terminal velocity. Hg

2. (d) In a streamline flow the velocity of fluid particles


remaines constant across any cross-sectional area, then Length of the air column above mercury in the tube is,
a point on the area cannot have different velocities at P + x = P0 Þ P = (76 – x)
the same time, hence two streamlines flow layers do Þ 8 × A × 76 = (76 – x) × A × (54 – x) \ x = 38
not cross each other. Thus, length of air column = 54 – 38 = 16 cm.
3. (b) In streamline flow, the speed of liquid of each particle 2T cos q
at a point in a particular cross-section is constant, 10. (a) Using h = ,
rdg
between two cross-section of a tube of flow because d2
h1 T1
AV = constant (law of continuity). we get = × d (other quantities cancel)
4. (a) As given that, h2 T2 1
Diameter at 1st section (d1) = 2.5. 60 0.6 9
= × = ; h : h = 9 : 10
50 0.8 10 1 2
EBD_7418
532 PHYSICS

11. (a) Water rises upto the top of capillary tube and stays L
there without overflowing. Þ kx 0 = Mg - s Ag
2
12. (a) Inflow rate of volume of the liquid = Outflow rate of
sLAg
volume of the liquid Mg -
Þ x0 = 2 = Mg æ1 - LAs ö
pR 2 V VR 2 ç ÷
pR2V = npr2(v) Þ v = = 2 k k è 2M ø
npr 2 nr Hence, extension of the spring when it is in equilibrium
13. (b)
Mg æ LAs ö
P PV is, x 0 = ç1 - ÷
14. (c) K = \ DV = k è 2M ø
DV / V K
2T cos q 1 h r 2
P = hrg = 200 × 103 × 10 N/m2 19. (c) h= Þhµ Þ 2 = 1 =
K = 22000 atm = 22000 × 105 N/m2 rrg r h1 r2 3
V = 1m3 æ 3 ö
çQ r1 = r, r2 = r + 50% of r = r ÷
200 ´ 103 ´10 ´ 1 è 2 ø
DV = = 9.1´ 10- 4 m3 2
22000 ´10 5 2 æ3 ö æ2 ö
New mass m2 = pr2 h 2r = p ç r1÷ ç h1 ÷ r
dv è2 ø è3 ø
15. (a) Newton’s law of viscosity, F = hA

F æ dv ö
dy
æ 4 y 3y2 ö
3 2
( 3
= pr1 h1 r = m
2 2
)
Stress = = h ç ÷ = hk ç 2 - 3 ÷ 20. (c) Here, h oil ´ roil ´ g = h water ´ rwater ´ g
A è dy ø èa a ø
r0g × 140 × 10–3 = rwg × 130 × 10–3
æ 4 3 ö hk
At y = a, stress = hk ç - ÷ = 130
è a aø a roil = ´ 103 » 928kg / m3 [Q rw = 1 kgm–3]
16. (b) Water wets glass and so the angle of contact is zero. 140
For full rise, neglecting the small mass in the meniscus T T
21. (d) Here excess pressure, Pexcess = +
2T r1 r2
2prT =pr 2 hrg Þ h = [Qwater wets glass, q=0°]
rrg T ær = R ö
2 ´ 0.07 Pexcess = Qç1 ÷
= R è r2 = O ø
0.25 ´10-3 ´1000 ´ 9.8
22. (a)
As the tube is only 2 cm above the water and so, water
23. (a) W = TDA = 4pR2T(n1/3 – 1)
will rise by 2 cm and meet the tube at an angle such
that, = 4 × 3.14 × (10–2)2 × 460 × 10–3 [(106)1/3 –1] = 0.057
2prT cos q = pr2h¢rg æ 2T cos q ö
Þ 2T cos q = h¢rrg 24. (b) M = ( pr 2 h )r = pr 2 ç r
è rrg ÷ø
h¢r rg
Þ cos q =
2T 2 æ 2T cos q ö
and M¢ = p (2r ) çè r ´ 2 r ´ g ÷ø r
2 ´10 -2 ´ 0.25 ´ 10-3 ´1000 ´ 9.8
=
2 ´ 0.07 = 2M.
The liquid will meet the tube at an angle, q @ 70° 25. (d) Power = rate of production of heat = F.V
17. (d) As we know, d = 6phr VT × VT = 6phrVT2
Pressure P = Vdg r (1 – p)L
Here, L A d g (Q F = 6phVT r stoke’s formula)
= (pL)A(nr)g + (1 – p)L A r g VT µ r2
nr pL
Þ d = (1 – p)r + pn r = [1 + (n – 1)p]r
18. (c) From figure, kx0 + FB = Mg 2 r 2 (r – s)
Q VT = g
9 h

KX0 \ Power µ r5
26. (b) Increment in area of soap film = A2 – A1
FB = 2 × [(10 × 0.6) – (10 × 0.5)] × 10–4 = 2 × 10–4 m2
Work done = T× DA
Mg = 7.2 × 10–2 × 2 × 10–4 = 1.44 × 10–5J
L 27. (b) Due to increase in velocity, pressure will be low above
kx 0 + s Ag = Mg [Q mass = density × volume]
2 the surface of water.
MECHANICAL PROPERTIES OF FLUIDS 533

Exercise - 3 or h = 3 = 1.732m
3. (c) Vertical distance covered by water before striking
1. (a) Let r be the radius of the base of the hemisphere or
ground = (H – h). Time taken is, t = 2 ( H - h ) / g :
cone then the height of the base = 2r (given)
Horizontal velocity of water coming out of hole at P,
i.e. VK = 2r or VL = LK = r
Also from similar triangles VLF and VKB, we have u = 2 gh
V
LF VL r 1 1 1
r
\ Horizontal range = ut = 2gh ´ 2(H - h) / g
= = = or LF = KB =
KB VK 2r 2 2 3 L
\ The volume of the frustum E F = 2 h (H - h )
p 4. (a) For 1 m length of horizontal tube
ABFE = h (r12 + r22 + r1r2 )
3 Mass of water M = density × volume
A B
1 é æ 1 öù 7 = 103 × area × length
2 K
æ1 ö
= pr ê r 2 + ç r ÷ + r ç r ÷ ú = pr 3
3 êë è 2 ø è 2 ø ú 12
û
= 103 × 10–2 × 1 = 10 kg
Dp
2 Therefore minimum force = (rate of change of
And the volume of the hemisphere = pr 3 C Dt
3 momentum)
\ Weight of the liquid contained in the vessel = (volume = 10 × 1.5 = 15 N
of the frustum + volume of the hemisphere) g r 5. (a) Range is same for both holes
æ7 2 ö 5 \ 2 (H - h1 ) h1 = 2 (H - h2 ) h2
= ç pr 3 + pr 3 ÷ gr = pr 3 gr , where r is the density
è 12 3 ø 4 Squaring both sides,
of the liquid. 4 (H – h1) h1 = 4 (H – h2) h2
Now the resultant vertical thrust on the vessel, which
is partly pressed upwards and partly downwards Hh1 – h12 = Hh2 – h22
= weight of the liquid contained On solving we get,
5 3 15 æ 2 ö 15 H = h1 + h2
= pr g r = ç pr 3 g r÷ = (the weight of the liquid
4 8 è3 ø 8
h1
that the hemisphere can hold) Hence, the ratio of depends on H.
h2
2. (d) Consider the forces per unit width of the gate. For the
6. (a) 7. (d)
level of water, h metre above the hinge,
8. (a) The initial velocity of the water with respect to the
Force on horizontal part OA equals
1000 × 9.81 × h × (1 × 1) = 9810h N vertically upwards. blade is v = 2 gh - wR . Therefore, a mass of water
This force acts at a distance of 1/2 m from the hinge.
m = rA ( 2 gh - wR ) impinges on the blade in a unit
Force on the vertical part of the gate equals
of time. After the impact, the velocity of the water with
h h2 reference to the blade is zero, and for this reason the
1000 × 9.81 × × (h × 1) = 9810 .
2 2 change in the momentum of the water in a unit time is
mv. According to Newton’s second law, the sought
h
This force acts at from the hinge. force is
3
Tipping would occur when the overturning clockwise F = rA ( 2 gh - wR )2 .
moment just exceeds the resisting anticlockwise
dv
moment about the hinge O. 9. (c) Viscous force F = -h (2pr l) ,
dr
where l = length of the cylinder
dr æ 2 phl ö
= -ç dv
h 9810h²/2 1m
r è F ÷ø
h/3 R2 0
dr 2 phl æ R ö 2 phl
O 0.5m A
ò r
=-ò
F
dv Þ ln ç 2 ÷ =
è R1 ø F
v0 .... (i)
R1 v0
9810h Suppose velocity of liquid at a distance r is v,
h2 h 1
9810 ´ ´ = 9810 ´ h ´ r
dr
v
2 ph l r 2 ph l
2 3 2 ò r
=-ò
F
dv Þ ln
R1
=
F
( v0 - v ) ... (ii)
R1 v0
EBD_7418
534 PHYSICS

12. (b) 13. (b)


æ R ö 2phl
ln ç 2 ÷ = v x
è r ø F 14. (b) b

On solving eqs. (i) and (ii),


h
ln ( R2 / r ) ln ( r / R2 )
v = v0 Þ v = v0
ln ( R2 / R1 ) ln ( R1 / R2 )
10. (a) (2pr1 + 2pr2)s = mg

é 8.7 8. 5 ù Let the width of each plate is b and due to surface


ê2 p ´ 2 + 2p ´ 2 ús = 3.97 ´ 980 tension liquid will rise upto height h then upward
ë û
force due to surface tension.
Þ s = 72 dyne cm–1 = 2Tbcosq ...(i)
11. (d) When drops combine to form a single drop of radius R. Weight of the liquid rises in between the plates
é1 1 ù = Vdg = (bxh)dg ...(ii)
Then energy released, E = 4pTR 3 ê - ú
ër R û Equating (i) and (ii) we get, 2T cosq = xhdg
If this energy is converted into kinetic energy then 2T cos q
\ h=
1 2 é1 1 ù xdg
mv = 4pR 3T ê - ú 4T ö V
2 ër R û æ 4T ö æ
15. (a) çè P0 + ÷ø V = çè P + ÷
r r / 2ø 8
1 é4 3 ù 2 é1 1 ù
´ ê pR rú v = 4pR 3T ê - ú
Þ P = 8 æç P0 + ö÷ .
3T
2 ë3 û ër R û è r ø
6T é 1 1 ù
v= -
r êë r R úû
11. THERMAL PROPERTIES OF MATTER

Exercise - 1 12. (b) The volume of cavity inside the solid ball increases
when it is heated.
1. (a) Let the readings of two thermometers coincide at C = F Reading on any scale – LFP
=x 13. (c) = constant for all scales
UFP - LFP
C F - 32 x x - 32 340 - 273 ° y - (-160)
As = \ = =
67 y + 160
5 9 5 9 Þ =
373 - 273 -50 - (-160) 100 110
or 9x = 5x – 160 x = – 40°C
\ y = – 86.3° Y
2. (c) Solids, liquids and gases all expand on being heated,
as a result density (= mass/volume) decreases. 14. (c)
3. (a) Temperature on celsius scale (TC), kelvin scale (TK) 15. (c) Due to volume expansion of both mercury and flask,
the change in volume of mercury relative to flask is
and Fahrenheit scale (TF) are related as
TC - 0 TF - 32 TK - 273.15 given by DV = V0 éë g L - g g ùû Dq = V éë g L - 3a g ùû Dq
= =
100 180 100 = 50 [ 180 × 10–6 – 3×9×10–6] (38 – 18) = 0.153 cc
4. (a) Among glass, wood and metals, metals expand more
for same rise in temperature. L1 1 + a(Dq)1
16. (c) L = L0 (1 + aDq) Þ =
5. (a) V + DV = (L + DL)3 = (L + aLDT)3 L 2 1 + a (Dq)2
= L3 + (1 + 3aDT + 3a2DT2 +a3DT3)
2
Þ a and a3 terms are neglected. 10 1 + 11´ 10 –6 ´ 20
Þ = Þ L2 = 9.99989
\ V (1 + gDT) = V (1 + 3aDT) L2 1 + 11´10–6 ´ 19
1 + gDT = 1 + 3aDT Þ Length is shorten by
\ g = 3a. 10 – 9.99989 = 0.00011 = 11 × 10–5 cm.
6. (c) 17. (d) Water can be boiled, even at low temperature on
Temp.(°F)
releasing the excess pressure, water refreezes into ice
called regelation.
18. (a) g real = g app. + g vessel

Temp.°C So ( g app. + g vessel )glass = ( g app + g vessel )steel


7. (c) Þ 153 ´ 10
-6
+ ( g vessel )glass = 144 ´ 10-6 + ( g vessel )steel
8. (c) As coefficient of cubical expansion of liquid equals
Further,
coefficient of cubical expansion of vessel, the level
of liquid will not change on heating. ( g vessel )steel = 3a = 3 ´ (12 ´ 10 -6 ) = 36 ´10 -6 / °C
9. (c) Initial diameter of tyre = (1000 – 6) mm = 994 mm, so -6 -6 -6
Þ 153 ´10 + ( g vessel )glass = 144 ´10 + 36 ´ 10
994
initial radius of tyre R = = 497 mm
2 Þ ( g vessel )glass = 3a = 27 ´10-6 / °C
6
and change in diameter DD = 6 mm so DR = = 3 mm
2 Þ a = 9 ´ 10 -6 / °C
After increasing temperature by Dq tyre will fit onto 19. (a) From question,
wheel Rise in temperature Dt = 40°C
Increment in the length (circumference) of the iron tyre Dr
Fractional change in the density =?
g g r0
DL = L ´ a ´ Dq = L ´ ´ Dq [As a = ]
3 3 Coefficient of volume expansion
ægö 3 DR 3´ 3 g = 5 × 10–4K–1 r = r0 (1 –gDt)
2pDR = 2pR ç ÷ Dq Þ Dq = = Dr
è3ø g R 3.6 ´ 10 –5 ´ 497 Þ = gDT = (5 × 10–4) (40) = 0.02
Þ Dq ; 500°C r0
10. (a) 20. (b) If the temperature of surrounding increases by DT , the
11. (a) The area of circular hole increases when we heat the new length of rod becomes
metal sheet & expansion of metal sheet will be l ¢ = l (1 + aDT )
independent of shape & size of the hole.
EBD_7418
536 PHYSICS

Due tochange in length, moment ofinertia ofrod alsochanges g¢real = g¢ vessel


and it is about an end P and is given as Change in volume in liquid relative to vessel
M l ¢2
l P¢ = DVapp = Vg 'app Dq = V(g 'real - g 'vessel )
3
As no external force or torque is acting on rod thus its angula TH - TL
26. (b) We know that Q =
rmomentum remains constant and during heating, thus we R
have l l
I P w = I P¢ w ' Also thermal resistance, R = KA = 2
Kpr
Heat flow will be maximum when thermal resistance is
[If w¢ is the final angular velocity of rod after heating].
minimum. From given option
M l2 M l 2 (1 + aDT ) 2 2l l
or w= w¢ (i)
0 0
r = 2r0, l = 2l0 \ R = Kp ( 2 r ) 2 = 2 Kpr 2
3 3 0 0
or w¢ = w(1 - 2aDT ) l0 l0
(ii) r = 2r 0, l = l0 \ R= 2
= 2
[using binomial expansion for small a]. Kp ( 2 r0 ) 4 Kpr0
Thus percentage change in angular velocity of rod due to 2l 0 2l 0
(iii) r = r0, l = 2l0 \ R= 2
= 2
heating can be given as Kp r0 Kpr0
w - w¢ l0 l
Dw = ´ 100% = 2aDT ´100% (iv) r = r0, l = l0 \ R= 2
= 02
w Kp r0 Kpr0
DL 27. (b) Let the final temperature be T.
21. (c) a = ; It found greatest for rod (c).
L DT Then 200 × 1 × (T – 20) + 20 × (T – 20)
(45 – 25)g (45 – 27)g = 440 (92 – T)
22. (c) V1 = and V2 =
d1 d2 Solving it, we get T = 68ºC.
V1 20 d 2 28. (a) Suppose, height of liquid in each arm before rising the
\ ´
V2 = 18 d1 temperature is l.
V1 20 é d1 ù
or = ´ê ú t1
V1 (1 + rmetal ´ 10) 18 ë (1 + rl ´ 10) d1 û
g t2 t1 t2
After simplifying, we get a = metal = 2.6 ´ 10 -3 / °C l1
3 l l l2
23. (b) For different temperature scales, we have
x - L.F.P With temperature rise height of liquid in each arm
= constant
U.F.P. – L.F.P. increases i.e. l1 > l and l2 > l
Where L.F.P Þ Lower Fixed point l1 l2
U.F.P. Þ Upper Fixed Point Also l = =
1 + g t1 1 + g t2
where x is the measurement at that scale. Here, if C and W be l1 – l2
the measurements on Celsius and W scale then, Þ l1 + g l1t2 = l2 + g l2 t1 Þ g = .
l2 t1 – l1t2
C–0 W – 39 1
= (\ C = 39°C) 29. (c) Time lost/gained per day = aDq´ 86400 second
100 – 0 239 – 39 2
39 ´ 200 1
Þ W= + 39 = 78 + 39 = 117° W 12 = a (40 – q) ´ 86400 .... (i)
100 2
V2 - V1 l 2 - l1 1
24. (a) g = V (T - T ) ; a = l (T - T ) 4 = a (q – 20) ´ 86400 ....(ii)
1 2 1 1 2 1 2
T1 = 0°C, T2 = 100°C 40 – q
On dividing we get, 3 =
V - V1 l -l q – 20
g= 2 ;a = 2 1 3q – 60 = 40 – q
100V1 100l1 4q = 100 Þ q = 25°C
l 1 = 10 cm; l 2 – l 1 = 0.08 mm = 0.008 cm 30. (b) gr = ga + gv where gr = coefficient of real expansion,
0.008 ga = coefficient of apparent expansion and gv = coefficient of
a= = 8 ´ 10-6 / °C; g = 3a = 24 × 10–6 / °C
10 ´ 100 expansion of vessel.
V - 1000 For copper: gr = C + 3aCu = C + 3A
\ 24 × 10–6 = 2
1000 ´ 100 For silver, gr = S + 3aAg
V2 – 1000 = 24 × 10–6 × 105 = 2.4 \ V2 = 1002.4 cc C - S + 3A
25. (b) When there is no change in liquid level in vessel then Þ C + 3A = S + 3aAg Þ aAg =
3
THERMAL PROPERTIES OF MATTER 537

L T 48. (b) As 1g of steam at 100°C melts 8g of ice at 0°C.


dL
31. (b) As,
L0
= a(T)dT ò dL = L0 ò a(T)dT 10 g of steam will melt 8× 10 g of ice at 0°C
L0 T0
Water in calorimeter = 500 + 80 + 10g = 590g
T é T ù 49. (b) Water and ice have different specific heats. Specific
L – L0 = L0 ò a(T) dT L = L0 ê1 + ò a(T) dT ú heat of water is 1 Jg–1°C–1 while that of water ice is
ê T ú
T0 ë 0 û 0.5 Jg–1°C–1.
50. (d) The thermal resistance is given by
32. (d) l = 5m t1 = 0°C x 4x x 2 x 3x
+ = + =
l 2 = 5.01m t2 = 100°C KA 2KA KA KA KA
dQ DT (T2 - T1 ) KA 1 ì A(T2 - T1 ) K ü
l 2 - l1 5.01 - 5 \ = = = í ý
a= = = 2 ´10-5 / °C . dt 3x 3x 3î x þ
l1 (t 2 - t1 ) 5 ´ 100
KA
1 1
33. (c) Dt = a DT ´ t \f =
2 3
1
\ 5 = a (T - 15 ) ´ 86400 51. (a)
2
52. (a) Initially, on heating temperature rises from – 100°C to
1
and 10 = a (30 - T ) ´ 86400 0°C. then ice melts and temperature does not rise. After
2 the whole ice has melted, temperature beigins to rise
34. (d) Water equivalent is the quantity of water whose
until it reaches 100°C. then it becomes constant, as at
thermal capacity is same as the heat capacity of the
the boiling point will not rise.
body.
53. (c) The heat lost in condensation = x × 540 cal.
D Q1 D Q2 T -T T -T \
35. (c) = Þ K1A1 1 2 = K2A2 1 2 x ´ 540 = y ´ 80 + y ´ 1 ´ (100 – 0)
Dt Dt L1 L2 x 1
A1 K 2 or y
= .
3
(L1 = L2) \ A = K
2 1 1æ1 2ö
36. (b) 37. (c) 38. (c) 39. (c) 54. (a) Heat produced = msDT = ç mv ÷ø
2è2
40. (b) Work done to raise the temperature of 100 gm water
v 2 ( 200)
2
4 ´ 104
through 10°C is Þ DT = = = = 80°C
W = JQ = 4.2 × (100 × 10–3× 1000 ×10) =4200 J 4s 4 ´ 125 4 ´ 125
41. (a) The latent heat of vaporization is always greater than 55. (a) The amount of heat required to increase the temperature
latent heat of fusion because in liquid to vapour of copper by 21ºC is
phase change there is a large increase in volume. Q=m DT = 100 ´ 10 –3 ´ 400 ´ 21 J
Hence more heat is required as compared to solid to CuSCu
liquid phase change. The amount of heat required to increase the temperature
42. (a) of water by DT10C is
S 1 DQ Q1 = m DT 1 = 50 ´ 10 –3 ´ 4200 ´ DT1
43. (b) Specific heat capacity is given by C = = . w Sw
m m DT According to question, Q = Q1
It is defined as the amount of heat per unit mass
\ 100 × 10–3 × 400 × 21 = 50 × 10–3 × 4200 × DT1
absorbed or rejected by the substance to change its
temperature by one unit. It depends on the nature of 2 ´ 21´ 4
Þ DT1 = = 4º C
the substance and its temperature. It is independent of 42
mass of substance. 1æ1 2ö
44. (d) 56. (a) W = JQ Þ ç Mv ÷ = J(m.c.Dq)
2è2 ø
45. (d) H = ncpDt
1
46. (c) Water has highest specific heat capacity and hence Þ ´ 1´ (50) 2 = 4.2[200 ´ 0.105 ´ Dq] Þ Dq = 7.1°C
it is used as a coolant in car radiators as well as 4
heater in hot water bags. 57. (c) Specific heat of water = 4200 J/kg-K
47. (c) Temperature difference between the end points A and Latent heat of fusion = 3.36 × 105 J/kg
D = 200 – 20 = 180°C Latent heat of vapourisation = 22.68 × 105 J/kg
As the resistances for the three parts are equal, the x × 10–3 × 22.68 × 105 J = y × 10–3 × 3.36 × 105 J
temperature difference must be distribuited equally in + y × 10–3 × 4200 × 100
the three parts (= 180/3 = 60°C) x 7.56 1
\ Temperature of B = 200°C – 60° = 140°C. \ = =
y 22.68 3
EBD_7418
538 PHYSICS

58. (a) Dq = 0.0023h = 0.0023 ´ 100 = 0.23°C Q – max. intensity is at red Þ lm is maximum Þ temp
59. (c) Here, specific heat of ice, Sice = 0.5 cal g–1°C–1 minimum i.e., Tp > TR > TQ
Specific heat of water, Swater = 1 cal g–1°C–1 69. (c) As the temperature difference DT = 10°C as well as the
thermal resistance is same for both the cases, so thermal
Latent heat of fusion of ice Llice = 80 cal g–1
current or rate of heat flow will also be same for both
Here ice will absorb heat while hot water will release it. the cases.
Let T be the final temperature of the mixture. 70. (d)
Assuming water equivalent of calorimeter to be 71. (a) Suppose temperature difference between A and B is
neglected.
100°C and q A > q B
Heat given by water, Q1 = mwaterSwaterDT
= 19 × 1 × (30 – T) = 570 – 19T … (i) C
Heat absorbed by ice.
H/2 H/2
Q2 = mice × Sice × [0 – (–20)] + mice × Lf ice
+ mice × Swater × (T – 0) A B
= 5 × 0.5 × 20 + 5 × 80 + 5 × 1 × T = 450 + 5T H
H
According to principle of calorimetry, Q1 = Q2
H/2 H/2
120
i.e., 570 – 19T = 450 + 5T Þ T = = 5° C
24
60. (a) D
Heat current will flow from A to B via path ACB and
61. (c) Let x kg of ice can melt
ADB. Since all the rod are identical, so, (Dq)AC = (Dq)AD
Using law of Calorimetry,
Heat lost by copper = Heat gained by ice Dq
(Because heat current H = ; here R = same for all)
2 × 400 × (500 – 0) = x × 3.5 × 105 R
2 ´ 400 ´ 500 8 Þ q A - qC = q A - q D Þ qC = q D
or x = = kg
3.5 ´ 105 7 i.e., temperature difference between C and D will be
62. (d) Let K1 and K2 be the thermal conductivities of larger zero.
and smaller sphere. 72. (d) Consider a shell of thickness (dr) and of radii (r) and
For bigger sphere, the temperature of inner and outer surfaces of this shell
4 be T, (T – dT)
p(3r)3 rL
K1 4p (3r) 2 ´ 100 3 dQ
= = rate of flow of heat through it
t/4 30 ´ 60 dt
For smaller sphere,
KA[(T - dT ) - T ] - KAdT
4 3 = =
pr rL dr dr
K 2 ´ 4pr 2 ´ 100 3 K1 1
= \ K =2 dT
t 20 ´ 60 2 = -4pKr
2
(Q A = 4pr 2 )
63. (c) Since specific heat = 0.6 kcal/g × °C = 0.6 cal/g × °C dr
From graph it is clear that in a minute, the temperature To measure the radial rate of T - dT
is raised from 0°C to 50°C. heat flow, integration dr
Þ Heat required for a minute = 50 × 0.6 × 50 = 1500 cal. technique is used, since the ·
T1
area of the surface through r1 r
Also from graph, Boiling point of wax is 200°C.
64. (a) 65. (b) 66. (c) which heat will flow is not
67. (a) By the law of conservation of energy, energy given by constant. T2
r2
heater must be equal to the sum of energy gained by r2 T2
æ dQ ö 1
water and energy lost from the lid. Then, ç
è dt ÷ø ò r 2 dr = -4pK ò dT
Pt = ms Dq + energy lost r1 T1
2000t = 3 × 4.2 × 103 × (97 – 27) + 130t
dQ é 1 1 ù
Þ t = 472 sec ê - ú = -4pK [T2 - T 1 ]
dt ë r1 r2 û
68. (a) From Wein’s displacement law, lm × T = constant
P – max. intensity is at violet Þ lm is minimum Þ temp dQ -4pKr1r2 (T2 - T1 ) dQ r r
maximum or = \ µ 1 2
dt (r2 - r1 ) dt (r2 - r1 )
R – max. intensity is at green Þ lm is moderate Þ temp
moderate
THERMAL PROPERTIES OF MATTER 539

mL 4 3 æ4 ö
1/3
mW qW – i i Þ pr = a 3 Þ a = r ç p ÷
SW 3 è3 ø
73. (d) qmix =
mi + mW Substituting the value of a in equation (i), we get
100 ´ 80 Qsphere 4pr 2 4pr 2 æpö
1/3
300 ´ 25 – = = = ç ÷ :1
= 1 = –1.25°C Qcube 6a 2
ìïæ 4 ö1/3 üï
2
è6ø
100 + 300 6 íç p ÷ r ý
Which is not possible. Hence qmix = 0°C îïè 3 ø þï
100°C (steam) TE + TD
82. (b) TB = Þ 2TB = TE + TD ...(i)
2
74. (a) 0.1 m A = 0.36 m2 T +T
TD = E C Þ TE = 2TD – TC ...(ii)
2
ice 0ºC
2TB = 2TD – TC + TD Þ TC = 3TD – 2TB
Rate of heat given by steam = Rate of heat taken by ice
83. (d) From Wien’s law, lmT = constant, where T is the
where K = Thermal conductivity of the slab
temperature of black body and lm is the wavelength
m = Mass of the ice corresponding to maximum energy of emission. Energy
L = Latent heat of melting/fusion distribution of black body radiation is given below:
A = Area of the slab i. U1 and U2 are not zero because a black body emits
dQ KA(100 - 0) dL radiations of nearly all wavelengths.
= =m , ii. Since U1 corresponding to lower wavelength, U3 corre-
dt 1 dt
sponds to higher wavelength and U2 corresponds to
K ´ 100 ´ 0.36 4.8 ´ 3.36 ´ 105 medium wave length, hence U2 > U1.
=
0.1 60 ´ 60 84. (d) Thermal resistance R =l/KA
K =1.24 J/m/s/°C Where l =20cm. & A(cylindrical rod)
75. (a) = pr2 = 40pcm2
76. (a) OA refers to change of state from ice to water 20 cal
So R = = 0.318
without change of temperature. 0.5 ´ 40p sec ´º C
æ dθ ö 85. (c) According to Stefan’s Law, the rate of loss of heat is
77. (c) Q = -KA ç ÷ ´ t
è dx ø Q
= sA ( T14 - T24 ) ´ e
78. (c) According to question only one-quarter of the heat t
produced by falling piece of ice is absorbed in the here s = 5.67 × 10–8J/m2 × sec.K2,
melting of ice. T1= 527 + 273 = 800K,
mgh 4L 4 ´ 3.4 ´105 T2 = 27 + 273 = 300K & A = 200×10–4m2
i.e., = mL Þ h = = = 136 km .
4 g 10 So. Q = 5.67 ´ 10 -8 ´ 2 ´ 10 -2
t
79. (d) Rate of transmission of heat
[(800)4 - (300)4 ] ´ 0.4
Temperature difference dQ d q @ 182 joule
= \ =
Thermal Resistance dt R l 2l
dQ (q - q2 ) q1 - q q - q2 q1 - q 86. (d) tµ , t'µ
Here, = = Þ = A A/2
dt R2 R1 R2 R1 3
87. (d) E = s ´ area ´ T 4 ; T increases by a factor .
Þ R1q - R1q2 = R2 q1 - R2q 2
1
R2 q1 + R1q2 Area increases by a factor .
Þ q( R1 + R2 ) = R2 q1 + R1q2 \ q= 4
R1 + R2 88. (c) Let T be temperature of the junction TA T TB
90 – V V - 30 Here, K A = 2 K B , T - TB = 50 K
80. (d) = Þ V = 50°C
2 RT RT At the steady state, HA = HB A B
81. (c) Q = s A t (T4 – T04) K A(TA - T) K B A(T - TB )
\ A =
If T, T0, s and t are same for both bodies, then L L
L L
Qsphere Asphere 4pr 2 2K B (TA - T) = K B (T - TB )
= = ...(i) T - TB 50 K
Qcube A cube 6a 2 TA - T = = = 25 K
But according to problem, 2 2
Volume of sphere = Volume of cube
EBD_7418
540 PHYSICS

89. (c) Heat gain = heat lost 94. (b) We have q – qs = (q0 – qs) e–kt ......(1)
CA 3 where q0 = initial temperature of body = 40°C
C A (16 - 12) = CB (19 - 16) Þ =
CB 4 q = temperature of body after time t
CB 5 qs = temperature of surrounding
and CB (23 - 19) = Cc (28 - 23) Þ C = 4 Since body cools from 40°C to 38°C in 10 min, we have
C
38° – 30° = (40° – 30°) e–10k ......... (2)
C A 15
Þ = ...(i) Let after 10 min, the body temp. be q.
CC 16
q – 30° = (38° – 30°) e–10k ......... (3)
If q is the temperature when A and C are mixed then,
Dividing equ. (2) by equ. (3) gives,
C 28 - q
C A (q - 12) = CC (28 - q) Þ A = ...(ii) 8° 10°
CC q - 12 = Þ q - 30° = 6.4° Þ q = 36.4°
On solving equations (i) and (ii) q = 20.2°C q - 30° 8°
95. (a)
90. (c) Power radiated by the sun at t°C
50 – 49.9 æ 50 + 49.9 ö
96. (b) = Kç – 30÷ ....(i)
= s(t + 273)4 4pr 2 5 è 2 ø
Power received by a unit surface 40 – 39.9 é 40 + 39.9 ù
=Kê – 30 ú ....(ii)
s(t + 273)4 4pr 2 r 2s(t + 273)4 t ë 2 û
= = From equations (i) and (ii), we get t » 10 s.
4pR2 R2
91. (c) From the figure it is clear that emission takes place 97. (c) From Newtwon's law of cooling
from the surface at temperature T2 (circular cross section). dQ dT
Heat conduction and radiation through lateral surface is = -KA
dt dx
zero.
Area of cross-section A and thickness dx is the same.
Heat conducted through rod is0
Also dQ = mCdq
KA(T1 - T2 ) Dt
Q= Thus in first case
l
Energy emitted by surface of rod in same time Dt, is m ´ C ´ (61° – 59°) -KA éæ 61° + 51° ö ù
= ê çè ÷ø - 30°ú (i)
4 4 4 dx ë 2 û
E = e s A (T2 - Ts ) Dt
In second case,
Since rod is at thermal equilibrium therefore E = Q
m ´ C ´ (51° - 49°) - KA éæ 51° + 49° ö ù
KA(T1 - T2 )Dt = êç ÷ø - 30° ú (ii)
hence, = e s A (T24 - Ts4 ) Dt t dx ëè 2 û
l
Dividing equation (i) by equation (ii)
e s (T24 - Ts4 ) l
Þ T1 – T2 = t 30
K = or t = 6 minutes.
Here T2 - Ts = DT and Ts >> DT 4 20
98. (a) According to Newton's law of cooling,
esl é
T1 - ( DT + Ts ) = ( DT + Ts )4 - Ts 4 ùû q1 - q 2 éq + q ù
K ë = K ê 1 2 - q0 ú
t ë 2 û
esl éæ DT ö4 ù where q0 is the surrounding temperature.
T1 - ( DT + Ts ) = ´ Ts 4 êç1 + - 1ú
K ëêè Ts ÷ø ûú 60 - 40 æ 60 + 40 ö
\ = Kç - 10÷
esl é 4D T ù 7 è 2 ø
T1 - ( DT + Ts ) = ´ Ts 4 ê1 + - 1ú
K ë Ts û 20 1
Þ = 40K Þ K =
4es l 3 7 14
or T1 – (Ts + DT) = Ts DT
K 40 - 28 é 40 + 28 ù 12
\ = Kê - 10ú Þ = 24K
æ 4 e s l Ts3 ö t ë 2 û t
or T1 – Ts = ç + 1÷ DT 12 12 ´ 14
ç K ÷
è ø or t = = = 7 min
æ 4 esl Ts3 ö 24K 24
\ The proportionality constant = çç 1 + ÷ - d q æ q1 + q2 ö
K ÷ 99. (c) Rate of colling = µç - q0 ÷
è ø dt è 2 ø
92. (b) In second case average temperature will be less hence
93. (b) When hot water temperature (T) and surrounding rate of colling will be less. Therefore time taken will be
temparature (T0) readings are noted, and log(T – T0) more than 4 minutes.
is plotted versus time, we get a straight line having
a negative slope; as a proof of Newton’s law of
cooling.
THERMAL PROPERTIES OF MATTER 541

100. (b) Apply Newton's law of cooling, No external torque is acting on the system so angular
DT momentum should be conserved.
= - k (Tav - Tq ) L = Angular momentum = Iw = constant
t
20 I1w1 = I 2 w2
= - k (70 - 30) .......... (1)
10 w2 I1
60 - T æ 60 + T ö So, w = I < 1
= -k ç - 30÷ .......... (2) 1 2 Rod
10 è 2 ø w2 < w1 (Q Due to expansion of the rod I2 > I1)
Divide eq. (2) by eq. (1)
So, angular veloctiy decreases.
60 - T T / 2 T 3. (b) In the given graph shows lowest fixed point for scale A
= Þ 120 - 2T = Þ T = 48°C
20 40 2 is 30° and lowest point for scale B is 0°. Upper fixed
101. (a) According to Newton’s law of cooling if temperature point for the scale A is 180° and upper fixed point for
difference between body & surrounding is large, then scale B is 100°. Hence, formula is
rate of cooling is also fast hence curve A shows correct
behaviour. t A - (LEP) A t B - (LEP) B
=
102. (c) If a body cools from temperature q1 to q2 in time t, (UFP) A - (LFP) A (UFP) B - (LFP) B
when the surrounding temperature is q0, by Newton,s where , LFP - Lower fixed point, UFP - Upper fixed
law of cooling, point.
q1 - q 2 éq + q ù (+A) B
= c ê 1 2 - q0 ú , where c = constant. 180
(+ B)
t ë t û C
103. (d)

Temperature (°A)
dQ DtA = 150°
104. (b) For q-t plot, rate of cooling = = slope of the curve.
dt 90°–q
dQ q
AT P, = |tan(180°– f2)| = tan f2 = k(q2 – q1)
dt O
where k = constant. 30°
DtB = 100°
At Q,
dQ O Temperature (°B) 100
=| tan(180° - j1 ) |= tan j1 = k(q1 - q 0 ) t A - 30 t -0 t A - 30 tB
dt So, = B Þ =
tan f 2 q 2 - q 0 180 - 30 100 - 0 150 100
\ = 4. (a) As we know that, the Buoyant force (F) on a body
tan f1 q1 - q 0
volume (V) and density of (r), when immersed in liquid
105. (a) Initially at t = 0
of density (rl) is = V¢rlg
Rate of cooling (R) µ Fall in temperature of body where V ' = volume of displaced liquid by dipped body
(q – q0) (V).
R 1 q1 – q0 100 – 40 3 Let volume of the sphere is V and r is its density, then
Þ = = = we can write buoyant force
R 2 q 2 – q0 80 – 40 2
F = VrsG & F µ rl (for liquid)
Exercise - 2 F4°C r4°C
= >1 (Q r4°C > r0°C) \ F4°C > F0°C
1. (d) If strips of aluminium and steel are fixed together on F0°C r0°C
metallic strip and both are heated then (aAl > asteel) Hence, buoyancy will be less in water at 0°C than that
aluminium will exapnd more because the metallic strip in water at 4°C.
with higher coefficient of linear expansion (aAl) will 5. (a) As the temperature increased the length (L) of the
expand more Thus it should have larger radius of pendulum increases due to expansion is linear.
curvature. Hence, aluminium will be on convex side. So, Time period of pendulum
Aluminium L L
T = 2p
g
Tµ L Pendulum
Hence on increasing temperature, time period (T) also
q Steel increases.
6. (a) As we know that when the temperature increases
O vibration of molecules about their mean position
2. (b) On heating a uniform metallic rod its length will increase increases hence the kinetic energy associated with
so moment of inertia of rod increased from I1 to I2. random motion of molecules increases.
EBD_7418
542 PHYSICS

7. (d) Let the radius of the sphere is R. 70 - 60 é 70 + 60 ù


As the temperature increases Þ = kê - q0 ú Þ 2 = k [65 – q ] ...(i)
radius of the sphere increases 5 ë 2 û 0

as shown. R 60 - 54 é 60 + 54 ù 6
=kê - q0 ú Þ = k [57 – q ] ...(ii)
4 3 5 ë 2 û 5 0
Original volume V = pR dV
3 By dividing (i) by (ii) we have
Coefficient of linear expansion = a 10 65 - q0
Coefficient of volume expansion = 3a = Þ q0 = 45º
6 57 - q0
DV
As we know that Y = 11. (d) E = σAT 4
V Dt
By putting the value of V, increase in the volume A α R 2 \ Eα R 2 T 4
E R 2 T4
DV \ 2 = 2 2
Þ DV = 3VaDt = 4pR3aDt \ 3a = E1 R12 T14
V Dt
8. (c) Loss of heat temperature on cooling temperature put R 2 = 2R, R1 = R
increase depend on material of object surface area E 2 (2R) 2 (2T) 4
exposed to surrounding and temperature difference T2 = 2T, T1 = T Þ = = 64
E1 R 2T 4
between body and surrounding.
12. (c) g = a1 + a 2 + a 3
Let us consider the diagram where all the three objects
are heated to same temperature T. As we know that = 13 ´ 10 - 7 + 231 ´ 10 - 7 + 231 ´ 10 - 7
density = 475 ´ 10 - 7
mass 13. (c) Rate of heat flow is given by,
r= KA(q1 - q 2 )
volume Q=
where r is same for all the three objects hence, volume l
will also be same. Where, K = coefficient of thermal conductivity
m l = length of rod and A = area of cross-section of rod
T m If the junction temperature is T, then 100°C
T
QCopper = QBrass + QSteel Copper

m T
0.92 ´ 4(100 - T ) B Brass
T Steel
46 0°C 0°C
Sphere Cube Plate
As thickness of the plate is least so, surface area of the 0.26 ´ 4 ´ (T - 0) 0.12 ´ 4 ´ (T - 0)
plate is maximum. = +
13 12
We know that, according to Stefan's law of heat loss Þ 200 – 2T = 2T + T Þ T = 40°C
H µ AT4 0.92 ´ 4 ´ 60
where, A is surface area of object and T is temperature. \ QCopper = = 4.8 cal/s
46
So, Hsphere : Hcube : Hplate
14. (a) Coefficient of volume expansion
= Asphere : Acube : Aplate
g = 5 × 10–4K–1; r = r0 (1 –gDt)
So area of circular plate is maximum.
Dr
For sphere, as the sphere is having minimum surface Þ = gDT = (5 × 10–4) (40) = 0.02
area. r0
Hence, the sphere cools slowest and circular plate will 15. (a) reff = r1r2
cool faster.
9. (d) According to the principle of calorimetry. dQ KA(T2 - T 1 ) Kpr1r2 (T2 - T1 )
= =
Heat lost = Heat gained dt L L
mLv + mswDq = mwswDq 16. (a) When same stress is applied at two different
temperatures, the increase in length is more at higher
Þ m × 540 + m × 1 × (100 – 80) temperature. Thus T1 > T2.
= 20 × 1 × (80 – 10) Þ m = 2.5 g 17. (d) The entropy change of the body in the two cases is
Therefore total mass of water at 80°C same as entropy is a state function.
= (20 + 2.5) g = 22.5 g 18. (d) From question, (l2 – l1) is maintained same at all
10. (a) Let the temperature of surroundings be q0 temperatures hence change in length for both rods
By Newton's law of cooling should be same
q1 - q 2 é q + q2 ù i.e.,Dl1 = Dl2
= kê 1 - q0 ú As we know, coefficient of linear expansion,
t ë 2 û
THERMAL PROPERTIES OF MATTER 543

Dl K1 8
a= Þ l1a1DT = l2a2DT Þ l1a1 = l2a2 \ K = 25
l 0 DT 2
19. (d) According to wein's displacement law, maximum amount 23. (d) Heat current H = H1 + H2
b K1A(T1 - T2 ) K 2 A(T1 - T2 )
of emitted radiation corresponding to lm = = +
T d d
2.88 ´106 nmK K EQ 2A(T1 - T2 ) A(T1 - T2 )
lm = = 500 nm = [K1 + K 2 ]
5760K d d
­ U2 Hence equivalent thermal conductivities for two rods
U
K1 + K 2
Emitted of equal area is given by K EQ =
(radiation) wave length K 2
250 nm 1000 nm
500 nm 24. (c) Given r1 = 12 cm , r2 = 6 cm
From the graph U1 < U2 > U3 T1 = 500 K and T2 = 2 × 500 = 1000 K; P1 = 450 watt
20. (c) According to question only one-quarter of the heat
produced by falling piece of ice is absorbed in the Rate of power loss P µ r2T4

mgh P1 r12 T14 r 2T 4


= Þ P2 = P1 22 24 = 1800 watt
melting of ice. i.e., = mL P2 r22 T24 r1 T1
4
25. (c) As we know, Bulk modulus
4L 4 ´ 3.4 ´ 105
Þ h= = = 136 km. DP DV P
g 10 K= Þ =
æ -DV ö V K
21. (a) Let m g of steam get condensed into water (By heat a
çè ÷ø
loss). This happens in following two steps. V
DV
100°C V = V0 (1 + gDt) Þ V = gDt
0
100°C Water P P P
Steam (H1= m × 540) \ = gDt Þ Dt = =
K gK 3aK
dQ DT
26. (b) = KA
[H2= m ×1× (100× 90)] dt L
æ dQ ö 3KA
For the first rod, ç ÷ = (100 – q)
è dt ø1 L
æ dQ ö A
90°C Similarly, ç ÷ = 2K (q – 50)
Water è dt ø 2 L
æ dQ ö A
Heat gained by water (20°C) to raise it's temperature ç dt ÷ = K L (q – 20)
upto 90°C = 22 × 1× (90 – 20) è ø3
Hence , in equilibrium, heat lost = Heat gain æ dQ ö æ dQ ö æ dQ ö
Now, ç ÷ =ç ÷ +ç ÷
Þ m × 540 + m × 1 × (100 – 90) è dt ø1 è dt ø2 è dt ø3
= 22 × 1 × (90 – 20) Þ 3 (100 – q) = 2 (q – 50) + (q – 20)
Þ m = 2.8 gm Þ q = 70°
The net mass of the water present in the mixture = 22 27. (d) water expands on both sides of 4 °C.
+2.8 =24.8 gm. 28. (d) According to principle of calorimetry,
22. (d) Radius of small sphere = r Heat lost = Heat gain
Thickness of small sphere = t 100 × 0.1(T – 75) = 100 × 0.1 × 45 + 170 × 1 × 45
Radius of bigger sphere = 2r T × 10 – 750 = 450 + 7650
Thickness of bigger sphere = t/4 T × 10 = 1200 + 7650 = 8850 Þ T = 885°C
Mass of ice melted = (volume of sphere) × (density of 29. (d) The change of state from liquid to vapour (for gas) is
ice) called vapourisation. It is observed that when liquid is
Let K1 and K2 be the thermal conductivities of larger heated, the temperature remains constant untill the
and smaller sphere. entire amount of the liquid is converted into vapour.
For bigger sphere,
4 3 The temperature at which the liquid and the vapour
K1 4p (2r)2 ´ 100 3 p(2r) rL states of the substance coexists is called its boiling
= point.
t/4 25 ´ 60
For smaller sphere, 30. (d) From Wien’s law
4 3 lmax T = constant
pr rL
K 2 ´ 4pr 2 ´ 100 3 i.e., lmax T1 = lmax T2
= 1 2
t 16 ´ 60
EBD_7418
544 PHYSICS

L
3l0 6. (a)
Þ l0 T = T¢
4 100°C Copper Steel 0°C
4
Þ T¢ = T L/2 L/2
3 Let conductivity of steel Ksteel = k then from question
Power radiated P µ T4 Conductivity of copper Kcopper = 9k
qcopper = 100°C
P2 ¢ 4 4
So, = n = æç T ö÷ = æç 4 ö÷ = 256 qsteel = 0°C
P1 èT ø è 3ø 81
L
31. (b) At C, solid completely will convert into liquid. lsteel = lcopper =
2
32. (c) Heat is carried away from a fire sideways mainly by
From formula temperature of junction;
radiations. Above the fire, heat is carried by both
radiation and by convection of air. The latter process K copper qcopper lsteel + K steel qsteel lcopper
q=
carries much more heat. K copper lsteel + Ksteel lcopper
33. (a) Heat gained by the water = (Heat supplied by the coil)
L L 900
– (Heat dissipated to environment) 9k ´ 100 ´ + k ´0´ kL
Þ mc Dq = PCoil t – PLoss t 2 2 2
= L L = = 90°C
10kL
Þ 2 × 4.2 × 103 × (77 – 27) = 1000 t – 160 t 9k ´ + k ´
2 2 2
4.2 ´ 105 7. (b) Heat supplied = Heat used in converting m grams of
Þ t= = 500 s = 8 min 20 s
840 ice from –5°C to 0°C + Heat used in converting 1 gram
of ice at 0°C to water at 0°C
Exercise - 3
2100 1 ´ 3.36 ´ 105
1. (a) M1g = Mg – V 1rl1 g Þ 420 = m ´ ´5+
1000 1000
or M1g = Mg – V1r1g … (i)
Þ 420 = m ´ 10.5 + 336
r1
and M 2 g = Mg - V1[1 + g g (t2 - t1)] g … (ii)
84
[1 + g l (t2 - t1 )] \m = = 8 grams
After simplifying, we get 10.5 2T T' 3T
æ M 2 - M1 ö 1 8. (c) Under steady conditions, the
gl = g g + ç M - M ÷ heat gained per second by a
è 2 ø (t 2 - t1 )
plate is equal to the heat
2. (a) A A
released per second by the
3. (a) W0 – W1 = V ´ d l ´ g ... (i) plate.
1 2 3
W0 – W2 = V '´ d 'l ´ g ... (ii) Heat gained Heat gained
Also, V' = V (1 + b DT) ... (iii) [by (2) from (1)] +
Second Second
and dl = dl ' (1 + g l DT ) ... (iv) Heat gained
From (ii), (iii) and (iv) [(by ((2) from (3)] = (by 2)
Second
V (1 + b DT ) ´ d l \ sA(2T)4 + sA(3T)4 = s(2A) (T ¢)4
W0 – W2 = ´g ... (v)
1 + g l DT 1/ 4
é 97 ù
Dividing (i) and (v), we get \ T ¢= ê ú T
W0 - W1 Vd l g (1 + g l D T ) ë2û
= 9. (d) Suppose that the temperature of the water in the first
W0 - W2 V (1 + bDT ) d l g vessel is q1(t) and that of the second is q2(t), then
W0 - W1 1 + g l DT W0 - W1 1 + g l (t2 - t1 ) dq KA
Þ = Þ = ms 1 = - (q1 - q 2 ) ... (i)
W0 - W2 1 + bDT W0 - W2 1 + b (t2 - t1 ) dt L
Þ (W0 – W1) [1 + b (t2 – t1) = (W0 – W2) [1 + g l (t2 - t1 )] dq KA
and ms 2 = (q1 - q 2 ) ... (ii)
W2 - W1 b (W0 - W1 ) dt L
Þ gl = + From (i) and (ii), we get
(W0 - W2 ) (t2 - t1 ) (W0 - W2 )
d q -2 KA
4. (a) The rate of heat loss per unit area due to radiation = q, where q = q1 - q 2 .
dt msL
= Îs (T4–T04) 1
= 0.6 × 5.67 × 10–8 [(400)4–(300)4] = 595 Jm–2s–1. The time, in which the temperature difference reduces to
e
5. (c) of its initial value, is given by
msL msL
Dt = Þ K=
2 KA 2 ADt
THERMAL PROPERTIES OF MATTER 545

10. (a) Ti + Tc
15. (d) Since Tn = = Neutral temperature
Dq Aes(T 4 - T04 ) 2
11. (c) Rate of cooling = Ti + 0 Ti
t mc Tn = = [Tc = 0°C = temperature of cold junction]
m 2 2
Þtµ [ Dq, t , s, (T 4 - T04 ) are constant] 16. (a) The heat flow rate is given by
A
dQ kA(q1 - q)
m Volume a 3 t a1 =
Þtµ µ µ ÞtµaÞ 1 = dt x
A Area a2 t2 a2
x dQ x dQ
100 1 Þ q -q = Þ q = q1 -
1 kA dt
Þ = Þ t2 = 200sec kA dt
t2 2
where q1 is the temperature of hot end and q is temperature at a
12. (a)
distance x from hot end.
13. (a) For equilibrium in case 1 at 0° C The above equation can be graphically represented by option (a) .
K1Vd2g
Upthrust = Wt. of body 17. (c) More the surface area, more will be the rate of cooling and
d1 hence lesser will be the temperature. Greater the number of
\ K1Vd2g = Vd1g V
surrounding surfaces of the objects having equal area and equal
d1 volume, lesser will be the surface area. Among all the given objects,
Þ K1 = ... (i)
d2 d2 Vd1g sphere has least and pyramid has largest surface area. Hence the
o temperature of sphere is highest while that of the pyramid is
For equilibrium in case 2 at 60° C At 0 C
lowest.
Note : When the temperature is increased the density will 18. (d) Heat current:
decrease. K2Vd' '2g
dT
\ d1' = d1 (1 + gFe × 60) d'1 i = - kA
V' dx
and d2' = d2 (1 + gHg × 60)
idx = –kA dT
Again upthrust = Wt. of body
V'd'1g l T2
\ K2V'd2'g = V'd1'g d'2
i ò dx = - Aa ò T dT
At 60oC
é d2 ù d1
0 T1
\ K2 ê ú=
êë 1 + g Hg ´ 60 úû 1 + g Fe ´ 60 (T22 - T12 ) Aa(T12 - T22 )
il = - Aa Þ i=
é 1 + g Fe ´ 60 ù d 2 2l
K 1 + g Fe ´ 60
\ K2 ê ú= 1 Þ 1 =
êë1 + g Hg ´ 60 úû d2 K 2 1 + g Hg ´ 60 é - dT ù A 4pr 2
19. (d) =
êë dr úû mc é
ëT - T0 ù
û = 4 ëéT - T0 ûù
14. (c) The given arrangement of rods r´ pr 3c
3
can be redrawn as follows
2K1K2 é - dT ù 1
K= \ êë dr úû µ
K1+K2 rrc
H1 H1 20. (c) Power radiated by the sun at t°C
K1 K2
l l
= s(t + 273)4 4pr 2
H H2 H Power received by a unit surface

K3 s(t + 273)4 4pr 2 r 2s(t + 273)4


= =
It is given that H1 = H2 4 pR 2 R2
KA(q1 - q2 ) K3 A(q1 - q2 ) K KK
Þ = Þ K3 = = 1 2
2l l 2 K1 + K2
EBD_7418
546 PHYSICS

12. THERMODYNAMICS

Exercise - 1 25. (a) In isothermal process temperature remains constnat.


i.e., DT = 0. Hence according to
1. (d) Heat always refers to energy transmitted from one body Q
to another because of temperature difference. C= Þ Ciso = ¥
2. (c) mDT
3. (d) Pressure, volume, temperature and mass are all 26. (b) In adiabatic process, no heat is taken or given by the
macroscopic variables which can be measured. system i.e., DQ = 0 Þ DU = -DW
4. (c) Heat and work are not state variables. They are
energy, transfer to a system which change the If DW is negative (work done on system), then DU
internal energy of a system, which is a state variable. increases & temperature increases and vice-versa. So
5. (c) Internal energy of an ideal gas depends only on the work done in adiabatic change in particular gas (ideal
temperature. gas) depends on change in temperature.
6. (a) Work is energy transfer brought about by moving 27. (a)
piston of a cylinder containing the gas, by raising or
28. (a) 29. (b)
lowering some weight connected to it.
7. (c) 8. (c) 9. (b) 30. (c) In an isochoric process, no work is done on or by
10. (a) Zeroth law defines temperature and first law defines the gas. V is constant
internal energy. 31. (a)
11. (d) Zeroth law of thermodynamics tells about thermal 32. (b) Q = mCDT = 1.5 × 0.12 × 4200 × (400 – 25)
equilibrium.
12. (d) Thermodynamics concerned with DH, DU and and DW. = 2.83 × 105 J
13. (b) W = P(DV) = P(VgDT)
14. (b) DQ = DU + DW = 105 × (5 × 10–2)3 × 3.5 × 10–5 × 375 = 0.164 J
15. (c) Thus Q = DU + W
16. (b) DQ = DU + DW or 2.83 × 105 = DU + 0.164 ; DU = 282 kJ
Þ DW = DQ – DU = 110 – 40 = 70 J 33. (b) W = P(DV) = m(Cp – Cv)DT
17. (b) According to first law of thermodynamics = 4(0.219 – 0.157) × 4200 × (120 – 20) ; 104 × 103J
Q = DU + W 34. (d) W = P(DV) = 1 × 105 × (3.34 – 2 × 10–3) = 340 × 103J.
Given : Q = 2 kcal = 2000 × 4.2 = 8400 J 35. (a) Every point on this isothermal curve represents the
W = 400 J condition of a system.
\ DU = Q – W = 8400 – 400 = 8000 J 36. (a) 37. (a)
18. (a) 38. (a) For adiabatic process Q = 0.
19. (d) According to first law of thermodynamics By first law of thermodynamics,
DQ = DU + DW Q = DE + W
DU = DQ – DW Þ DEint = – W.
DQ = 35J, DW = – 15J 39. (b)
\ DU = 35J – (– 15J) = 50J
Slope of adiabatic curve (dP / dV )adi
20. (a) As we know, 40. (c) = = +g
Slope of isothermal curve (dP / dV )iso
DQ = D u + D w (Ist law of thermodynamics)
Þ DQ = D u + P D v æ C ö
or 150 = Du + 100 (1 - 2 ) = Du - 100 So slope to adiabatic curve is g ç = P ÷ times of
\ Du = 150 + 100 = 250J è CV ø
21. (d) dU = dQ - dW = (8 ´105 - 6.5 ´105 ) = 1.5 ´ 105 J isothermal curve, as clear also from figure.
5 5
dW = dQ - dU == 10 - 1.5 ´ 10 = - 0.5 ´ 10 J
5 P
– ve sign indicates that work done on the gas is
Isothermal
0.5 ´105 J . curve
22. (a) Cp – Cv = work done
23. (d)
24. (c) For an adiabatic process of an ideal gas.

g C V
PV = const where g = P
CV Adiabatic curve
THERMODYNAMICS 547

41. (c) T1 = 273 + 27 = 300K Þ Slope of curve for isothermal change < slope of
T2 = 273 + 927 = 1200K curve for adiabatic change.
For adiabatic process, P1–g Tg = constant So, curve B shows isothermal change and curve A
shows adiabatic change.
Þ P11–g T1g = P21–g T2g
51. (b) In the first process W is + ve P
1-g g 1-1.4 1.4 3
æP ö æ T2 ö æ P1 ö æ 1200 ö as DV is positive, in the
Þç 1÷ =ç ÷ Þç ÷ =ç P1
è T2 ø è T1 ø è P2 ø è 300 ÷ø second process W is – ve as
DV is – ve and area under P2
æ 1.4 ö æ7ö
ç ÷ ç ÷ the curve of second process
= P1 (27) = 2 × 128 = 256 atm
P2 = P1 4è 0.4 ø
= P1 4è 2 ø is more V1 V2
42. (b) Initial temperature (T1) = 18°C = 291 K \ Net Work < 0 and also P3 > P1.
V 52. (d) Isochoric proceess dV = 0
Let Initial volume (V1) = V Final volume (V2) = W= 0 proceess 1
8
According to adiabatic process, TVg – 1 = constant Isobaric : W = P DV = nRDT
nRDT
According to question, T1V1g -1 = T2V2g -1 Adiabatic | W | = 0<g–1<1
g -1
7 As workdone in case of adiabatic process is more so
-1
Þ T2 = 293(8) 5 = 293 × 2.297 = 668.4K
process 3 is adiabatic and process 2 is isobaric.
é Cp 7ù 53. (c) Heat absorbed in a thermodynamic process is given
ê For diatomic gas g = = ú by DQ = DU + DW.
ë Cv 5 û
Here DU is same for all the three processes as it depends
43. (a) DU = n Cv DT =2 × 103 × 20 × 273 = 10.9 MJ. only on initial and final states.
44. (b) For path iaf, But DWI = + Ve, DWII = 0, DWIII = – ve
Q = 50 cal \ DQI > DQII
W = 20 cal
By first law of thermodynamics, 54. (c) – 20 = DU + 50 Þ DU = – 70.
From 2 ® 1, DU = 70 kJ.
DU = Q - W = 50 – 20 = 30 cal.
a Now 10 =70 + W Þ W = – 60 kJ.
For path ibf f
1
Q' = 36 cal 55. (a) As P µ 1.5 , So PV1.5 = constant
W' = ? V
or, W' = Q' – DU' i \ g = 1.5 (Q Process is adiabatic)
b
Since, the change in internal energy does not depend Cp Cp
As we know, =g \ = 1.5
on the path, therefore Cv Cv
DU ' = 30 cal 56. (a) U = a + bPV ......(1)
\ W' = Q' – DU ' = 36 – 30 = 6 cal. In adiabatic change,
45. (c) nR nR
46. (b) A ® (3); B ® (1); C ® (2); D ® (4) dU = – dW = (T2 - T1 ) = (d T )
g -1 g -1
47. (c)
nR
48. (d) Work done by the system in the cycle
= Area under P-V curve and V-axis
ÞU =
ò dU = g - 1 ò dT
1 æ nR ö PV
or U =ç T +a = + a ......(2)
= (2P0 - P0 )(2V0 - V0 ) +
2 è g - 1ø÷ g -1
é æ 1ö ù where a is the constant of integration.
ê - çè 2 ÷ø (3P0 - 2P0 )(2V0 - V0 ) ú Comparing (1) and (2), we get
ë û
1 b +1
P0 V0 P0 V0 b= Þg = .
= - =0 g -1 b
2 2
49. (c) P1V1g = P2V2g (Adiabatic change) 57. (c) For adiabatic process, dU = – 100 J
g
which remains same for other processes also.
g
æV ö æ V ö Let C be the heat capacity of 2nd process then
P2 = P1 ç 1 ÷ = P1 ç 1 ÷ = P2(3)g – (C) 5 = dU + dW
è V2 ø è V1 / 3 ø = – 100 + 25 = – 75
50. (c) Curve A, B shows expansion. For expansion of a gas, \ C = 15 J/K
Wisothermal > Wadiabatic 58. (c) As DU = 0 in a cyclic process,
Pisothermal > Padiabatic DQ = DW = area of circle = pr2
Tisothermal > Tadiabatic
or DW = 102 pJ
EBD_7418
548 PHYSICS

59. (d) Under adiabatic change = –3 × 102 × 3.19 = –957 joules


1- g 1- g [–ve sign shows external work done on the gas]
T2 æ P1 ö g g 68. (d)
=ç ÷ or T2 = T1 (P1 / P2 )
T1 çè P2 ÷ø 69. (b) Work done in expansion is positive and in compression
1- ( 7 / 5 ) it is negative.
\ T2 = 300 ( 4 / 1) (7 / 5 )
; g = 1.4 = 7 / 5 for air 70. (c) Work done = Area under curve ACBDA
-2 / 7 71. (c) Q = mL =1 × L = L; W = P(V2 – V1)
or T2 = 300 (4)
Now Q = DU + W
60. (d) From C to D, V is constant. So process is isochoric. or L = DU + P(V2 – V1)
From D to A, the curve represents constant
temperature. So the process is isothermal. \ DU = L – P(V2 – V1)
From A to B, pressure is constant . So, the process is
isobaric. 72. (b) A ® (3); B ® (1); C ® (2); D ® (2)
BC represents constant entropy. Va a Vd V Vb
61. (a) From first law of thermodynamics, 73. (a) We know that V = Þ =
b Vc Vd Vc
DH = Du + w 74. (d) QAB = DUAB + WAB
In adiabatic process DH = 0 WAB = 0
\ Du = – w
f f
62. (a) The opening of bottle is the rapid or adiabatic process. DUAB = nRDT Þ ( DPV )
2 2
In the process temperature falls.
63. (c) 5
DUAB = (DPV) Þ QAB = 2.5 P0V0
5 2
64. (a) T1 = T, W = 6R joules, g = Process BC:
3
PV - P V nRT - nR(T1 - T2 ) QBC = DUBC + WBC = 0 + 2P0V0 ln 2 = 1.4 P0V0
1 nRT2
W= 1 1 2 2 = = Qnet = QAB + QBC = 3.9 P0V0
g -1 g -1 g -1
pr1r2 p ´1´1
R (T - T2 ) 75. (c) W = = = p /2 J
n = 1, T1 = T Þ = 6R Þ T2 = ( T–4)K 2 2
5 / 3 -1 76. (c) Slope of adiabatic curve
pr 2 p (6) 2 = g × slope of isothermal curve
65. (b) WABC = = = 18p
2 2 = 1.4 × (–400) = – 560 MPa/m3
p ´ 32 p ´ 32 77. (d) Isobaric compression is represented by curve AO
WDEF = + + (15 - 12) ´ 18 = 6.75p + 54 Work done = area under AD
2 4
= 2 × 102 × (3 – 1)
WDEF > WABC
= 4 × 102 = 400 J.
66. (c) Given that dQ = – 30 J and dW = – 10 J
Ei = 40 J and let final internal energy = Ef 78. (b) dW = P D V = 1 .01 ´ 10 5 [1671 - 1] ´ 10 - 6 Joule
So, dQ = E f - E i + dW -30 = E f - 40 - 10 1.01´167
= cal. = 40cal. nearly
Ef = 20 J 4.2
Δ Q = mL = 1 ´ 540,
67. (a) For an adiabatic change PVg = constant
P1V1g = P2V2g ΔQ = ΔW + ΔU or D U = 540 - 40 = 500 cal.
As molar specific heat of gas at constant volume 79. (d) The process is equivalent to TV 1/2 = C
3 Compare with TV g–1 = C Þ x = 3/2
Cv = R 3 1
2 R R R R
ÞC= + = + = R – 2R = – R
3 5 g – 1 1 – X 2 / 3 1 – (3 / 2) 2 2
CP = CV + R = R + R = R ; 80. (b) In VT graph
2 2
CP (5 / 2) R 5 ab-process : Isobaric, temperature increases.
g= = = bc process : Adiabatic, pressure decreases.
CV (3 / 2) R 3
\ From eqn. (1) cd process : Isobaric, volume decreases.
g 5/3
da process : Adiabatic, pressure increases.
æV ö æ 6ö The above processes correctly represented in P-V
P2 = ç 1 ÷ P1 = ç ÷ ´ 105 N / m 2
è V2 ø è 2ø diagram (b).
5 P
= (3) × 10 = 6.19 × 105 N/m2
5/3 5
81. (c) W = W1 + W2 5×10
Work done
= 4 × 1 × 105 + ½ ×4 × 105 × 4
1 5 W1
= [6.19 ´ 105 ´ 2 ´ 10-3 - 10-5 ´ 6 ´ 10-3 ] = 12 × 105 Joule 1×10
1 - (5 / 3) W2
é 2 ´ 10 ´ 3
2 ù V
1 5
= -ê (6.19 - 3) ú
ëê 2 ûú
THERMODYNAMICS 549

82. (b) In the first-case adiabatic change, must be low.


DQ = 0, DW = –35 J T1
From 1st law of thermodynamics, 97. (d) h = 1- T
2
DQ = DU + DW, T1 = –23°C = 250 K, T2 = 100°C = 373K
or 0 = DU – 35
250 373 - 250
\ DU = 35 J h = 1- =
373 373
In the second case
DQ = 12 cal = 12 × 4.2 J = 50.4 J 300 100 1
98. (a) h = 1- = =
DW = DQ – DU = 50.4 – 35 = 15.4 J 400 400 4
1
83. (d) For path ab : ( DU )ab = 7000 J h= ´ 100 = 25%
4
By using DU = mCV DT Hence, it is not possible to have efficiency more than 25%.
5 99. (b)
7000 = m ´ R ´ 700 Þ m = 0.48
2 100. (c) Absolute zero temperature is practically not reachable.
For path ca :
( DQ)ca = ( DU )ca + ( DW )ca ...(i) 101. (c)
102. (c) The working of an air coditioner is similar to the
Q ( DU )ab + (DU )bc + ( DU )ca = 0
working of a refrigerator. An air conditioner removes
Q 7000 + 0 + (DU )ca = 0 ...(ii) heat from the room, does some work and rejects the
Þ (DU )ca = -7000 J heat to the surroundings. As air conditioner is put in
Also ( DW )ca = P1 (V1 - V2 ) = mR (T1 - T2 ) the middle of the room then due to continuous,
= 0.48 × 8.31 × (300 – 1000) = – 2792.16 J ...(iii) external work the room will become slightly warmer.
On solving equations (i), (ii) and (iii)
æ T ö
( DQ )ca = -7000 - 2792.16 = -9792.16 J » -9800 J 103. (d) Efficiency, h = ç 1 – 2 ÷ ´ 100
7R è T1 ø
84. (d) Q = nCP × 30 = n ´ ´ 30
(T –100 – T2 + 100)
2 (T2 – 100)
5R h¢ = 1 – ´ 100 = 1 ´ 100
and Q = n ´ ´ DT (T1 – 100) T1 –100
2
7R 5R æ T –T ö
\ n´ ´ 30 = n ´ ´ DT or DT = 42 K. h¢ = ç 1 2 ÷ ´100.
2 2 è T1 –100 ø
85. (a) First operation in carnot cycle is isothermal expansion Comparing with h we get, the efficiency increases.
86. (a) External amount of work must be done in order to flow 104. (d) The coefficient of performance of a refrigerator is given
heat from lower temperature to higher temperature. by
This is according to second law of thermodynamics. Q Q2
a= 2 =
87. (b) W Q1 - Q2
88. (c) For process to be reversible it must be quasi-static. Substituting the given values, we get
For quasi static process all changes take place
1 Q2
infinitely slowly. Isothermal process occur very slowly =
so it is quasi-static and hence it is reversible. 3 200 - Q2
89. (d) In reversible cyclic Process Þ 200 – Q2 = 3Q2 Þ 4Q2 = 200
DU = 0 200
or Q2 = J = 50J
90. (b) 4
91. (a) Slow isothermal expansion or compression of an ideal \ W = Q1 – Q2 = 200 J – 50 J = 150 J
gas is reversible process, while the other given ( 627 + 273 ) - ( 273 + 27 )
105. (b) h =
processes are irreversible in nature. 627 + 273
92. (b) Heat engine is device by which a system is made to 900 - 300 600 2
= = =
undergo cyclic process that result in conversion of 900 900 3
heat into work. 2
P work = (h) × Heat = ´ 3 ´ 10 6 ´ 4 .2 J = 8.4 × 106 J
When gas (system) in heat 3
1 2
engine undergoes process 106. (b) The efficiency of cycle is
1 ® 2 ® 3 ® 4 ® 1, then 3 T
4 h =1– 2
work done by gas = area enclosed T1
V
by figure formed by joining 1, 2 , 3 , 4. For adiabatic process
T Vg–1 = constant
Work is positive if arrows move clockwise. 7
93. (a) 94. (c) 95. (a) For diatomic gas g =
5
T2 T1V1g–1 = T2V2g–1
96. (d) h =1- So for h be high T1 must be high and T2 g –1
T1 æV ö
T1 = T2 ç 2 ÷
è V1 ø
EBD_7418
550 PHYSICS

7
–1 T2
T1 = T2 (32) 5 = T2 (25 ) 2 / 5 = T2 ´ 4 or = 0.8 ...(i)
T1
When T2 is reduced by 50 K, its efficiency becomes 0.4
T1 = 4T2 \ h = æç1 – 1 ö÷ = 3 = 0.75 T – 50
è 4ø 4 \ 0.4 = 1 – 2
T 1 T2 T2 5 T1
107. (d) h1 = 1 – 2 Þ = 1– Þ = … (i)
T1 6 T1 T1 6 T2 – 50
or = 0.6 ...(ii)
T2 – 62 1 T1
h2 = 1 – Þ = 1 – T2 – 62 …(ii) Dividing eqn. (i) by (ii)
T1 3 T1
T2 0.8 4
On solving Eqs. (i) and (ii) = =
T1 = 372 K and T2 = 310 K T2 – 50 0.6 3 Þ 3T2 = 4T2 – 200 or T2 = 200 K
108. (b) Here, Coefficient of performance (b) = 5 T – 50 200 – 50
T1 = 27°C = (27 + 273)K = 300 K From eqn. (ii), T1 = 2 = = 250K
0.6 0.6
T2 T2 116. (a)
T
As, b = Þ5=
117. (d) Efficiency of engine A, h1 = 1 - ,
T1 - T2 300 – T2 T1
or 1500 - 5T2 = T2 or 6T2 = 1500 T
Efficiency of engine B, h2 = 1 - 2
1500 T
\ T2 = = 250 K Here, h1 = h2
6 T T2
109. (d) T \ =
T1 T Þ T = T1T2
118. (b) Q1 = 1000 J, Q2 = 600 J T1 = 127°C = 400 K
2T0
Q1 Efficiency of carnot engine,
Q3
W Q2 - Q1
T0
Q2 h= ´100% or, h = ´100%
Q1 Q1
S
S0 2S0 1000 - 600
or, h = ´ 100% = 40%
1 3 1000
Q1 = T0S0 + T0S0 = T0S0 Q 2 T2
2 2 Now, for carnot cycle =
Q1 T1
Q2 = T0(2S0 – S0) = T0S0 and Q 3 = 0 T
600 600 ´ 400
= 2 Þ T2 = = 240 K
W Q1 - Q 2 Q TS 1 1000 400 1000
h= = = 1 - 2 = 1- 0 0 =
Q1 Q1 Q1 3 3 T2Þ= -33°C
= 240 – 273
T0S0
2 119. (a) WAB = 0, WBC = PDV = nRDT = –nRT0
T2 50 500 Vf
110. (c) h =1- or = 1-
T1 100 T1 WCA = nRT ln = nR ( 2T0 ) ln 2
Vi
Þ T1 = 1000 K
T æ nRg ö
60 QBC = nC p DT = ç T0
è g – 1÷ø
2
Also, = 1- Þ T2 = 400 K
100 1000
111. (b) When milk cools, its energy content decreases. W é 2ln 2 – 1 ù
Efficiency, h= =ê ú
T1 - T2 Q ë g / ( g – 1) û
112. (b) is maximum in case (b).
T1 W T
120. (a) As we know h = = 1– 2
113. (d) Change in entropy is given by Q1 T1
dQ DQ mLf 300 K 3
dS = or DS = = Þ h = 1– =
T T 273 1200K 4
1000 ´ 80 3 W 4 4
DS = = 293cal / K. = Þ Q1 = W ´ Þ Q1 = 12.6 ´106 ´
273 4 Q1 3 3
Q1= 16.8 × 106J.
Woutput w 1
114. (d) Efficiency h = = = 121. (a)
Heat input 3w 3 122. (d) T2 = 7°C = (7 + 273) = 280 K
Q2 1 Q 2 T2 T 50 50 1
h = 1- = \ 2 = h = 1- Þ 2 =1- h =1- = =
T1 T1 100 100 2
Q1 3 Q1 3
115. (b) When efficiency of carnot engine, h = 0.2 \ T1 = 2 × T2 = 2 × 280 = 560 K
Efficiency of a Carnot engine, New efficiency, h¢ = 70%
T T T2
h = 1 – 2 or, 0.2 = 1 – 2 70 30 3
T1 T1 \ T1 = 1 – h¢ =
1- = =
100 100 10
THERMODYNAMICS 551

10 2800 4. (b) According to the given p-V diagram.


\ T1¢ = ´ 280 = = 933.3 K
3 3 Work done in the process ABCD
\ Increase in the temperature of high temp. = (AB) × BC = (3V0 – V0) × (2p0 – p0)
reservoir = 933.3 – 560 = 373.3 K = 380 K = 2V0 × p0 = 2p0V0
123. (b) 124. (c)
Here the direction of arrow is anti-clockwise, so work
125. (d) ds = nCvdT + PdV = 0 done is negative.
dT Hence, work done by the gas = –2p0V0
nR + ( p0 - aV ) = 0
dV
pV = nRT That shows external work done on the system.
5. (a) Let us consider the p-V diagram for container A
p0V - aV 2 = nRT
(isothermal) and for container B (adiabatic).
dT
p0 - 2aV = nR
dV 2 2
-( p0 - aV )(g - 1) = p0 - 2aV p p
- p0 (g - 1) + a(g - 1)V = p0 - 2aV
p0V = aV (g + 1) p0 1 p0 1
p0 g V0 2V0 V0 2V0
V = V V
a( g + 1) Container A Container B
(Isothermal) (Adiabatic)
5
p0 ´ In both process compression of the gas.
V = 3 = 5 p0
For isothermal process (gas A) during 1® 2
æ 5 ö 8a
a ç + 1÷
è3 ø
p1V1 = p2V2 (QV1 = 2V0 , V2 = V0 )
p0 (2V0 ) = p2 (V0 )
Exercise - 2 p2 = 2 p0
1. (c) For the straight line in the graph denoted by 4, that For adiabatic process, (gas B) during (1® 2)
shows pressure is constant, so curve 4 represents an p1V1g = p2V2g (Q V1 = 2V0 , V2 = V0 )
isobaric process.
4 p0 (2V0 )g = p2 (V0 ) g
p 3 g
æ 2V ö
2 p2 = ç 0 ÷ p0 = (2) g p0
1 è V0 ø
V
So, ratio of final pressure
For the straight line in graph denoted by 1, that shows æ ( p ) ö (2) g p0
volume is constant, so curve 1 represents isochoric = ç 2 B÷ = = 2g -1
è ( p1 ) A ø 2 p0
process. Out of curves 3 and 2, curve 2 is steeper.
where, g is ratio of specific heat capacities for the gas.
Hence, curve 2 is adiabatic and curve 3 is isothermal.
Hence, verifies the option (a).
2. (a) As we know that amount of sweat evaporated/minute 6. (b) Consider the equilibrium temperature of the system is
Sweat produced/ minute T.
=
Number of calories required for evaporation/kg Let us consider, T1, T2 < T < T3.
Amount of heat produced per minute in jogging As given that, there is no net loss to the surroundings.
=
Latent heat (in cal/kg) Heat lost by M3 = Heat gained by M1
580 × 10 3 calories are needed to convert + Heat gained by M2
1 kg H2O into stream. M 3s (T3 - T ) = M1s(T - T1 ) + M 2 s (T - T2 )
1 cal. will produce sweat = 1 kg/ 580 × 103 M 3 sT3 - M 3 sT = M 1 sT - M 1 sT1
14.5 × 103 cal will produce (sweat) + M 2 sT - M 2 sT2
14.5 ´ 103 145 (where, s is specified heat of the copper material)
= kg= kg/m = 0.025 kg.
580 ´ 103 580 T [ M1 + M 2 + M 3 ] = M 3T3 + M1T1 + M 2T2
3. (c) According to given P–V diagram that
M 1T1 + M 2T2 + M 3T3
pV = constant T=
So we can say that the gas is going through an M1 + M 2 + M 3
isothermal process. 7. (c)
If pressure (P) increase at constant temperature volume 8. (b) Change in internal energy do not depend upon the
V decreases, the graph (iii) shows that pressure (P) is path followed by the process. It only depends on initial
smaller at point 2 and larger at point 1 point so the gas and final states i.e.,
expands and pressure decreases. Hence verifies option
(c). DU1 = DU2
EBD_7418
552 PHYSICS

9. (c) = 2 × 4.2 × 1000 – 500


10. (d) In cyclic process, change in total internal energy is = 8400 –500
zero. DUcyclic = 0 = 7900 J
5R 23. (a) Here, T1 = 500 K, T2 = 375 K
DUBC = nCv DT = 1 ´ DT Q1 = 25 × 105 J
2
Where, Cv = molar specific heat at constant volume. T2 375
For BC, DT = –200 K \ DUBC = –500R \ h = 1- =1- = 0.25
T1 500
11. (d) W = hQ = 0.25 × 25 × 105 = 6.25 × 105 J
12. (d) Since area under (P – V) curve the curve is maximum 24. (d) According to first law of thermodynamics, DQ = DU +
for adiabatic process so, work done (W = PdV) on the DW = DU +PDV. If heat is supplied in such a manner
gas will be maximum for adiabatic process. that volume does not change DV = 0,i.e., isochoric
13. (d) Coefficient of performance, process, then whole of the heat energy supplied to the
T2 system will increase internal energy only. But, in any
Cop =
T1 - T2 other process it is not possible.
273 - 20 253 1518 Also heat may be adsorbed or evolved when state of
5= = \ T1 = = 303.6 thermal equilibrium changes.
T1 - (273 - 20) T1 - 253 5
25. (d) The change in internal energy DU is same in all
or, T1 = 303.6 – 273 = 30.6 @ 31°C process.
14. (a) 15. (c)
QACB = DU + WACB,
16. (b) Wext = negative of area P
QADB = DU ,
with volume-axis
Adiabatic
QAEB = DU + WAEB
W(adiabatic) > W(isothermal) Here WACB is positive and WAEB is negative.
Isothermal Hence QACB > QADB > QAEB.
O 26. (a) Process AB is isobasic and BC is isothermal, CD
V0 2V0 V
17. (c) Coefficient of performance of a refrigerator, isochoric and DA isothermic compression.
27. (b) Using first law of thermodynamics equation,
Q2 T2 DQ = DU + DW
b = W = T -T (Where Q2 is heat removed)
1 2 Þ 54 × 4.18
Given: T2 = 4°C = 4 + 273 = 277 k = DU + 1.013 × 105(167.1 × 10–6 – 0)
T1 = 30°C = 30 + 273 = 303 k (Q DW = PDV)
600 ´ 4.2 277 Þ DU = 208.7 J
\ b= = Þ W = 236.5 joule
W 303 - 277 5
28. (a) Gas is monatomic, so Cp = R
W 236.5 joule 2
Power P = = = 236.5 watt. Given process is isobaric
t 1sec
18. (b) In cyclic process ABCA \ dQ = n Cp dT
Qcycle = Wcycle æ5 ö
QAB + QBC + QCA = ar. of DABC Þ dQ = n ç R ÷ dT
è2 ø
1 dW = P dV = n RdT
+ 400 + 100 + QC®A = (2 × 10–3) (4 × 104) dW nRdT 2
2 \ Required ratio = = =
dQ æ5 ö 5
Þ QC ® A = – 460 J n ç R ÷ dT
Þ QA ® C = + 460 J è 2 ø
19. (c) 29. (a) Efficiency of ideal heat engine,
20. (a) Process I volume is constant hence, it is isochoric æ T ö
h = ç1 – 2 ÷
In process IV, pressure is constant hence, it is isobaric
1 è T1 ø
21. (a) Given, efficiency of engine, h = Sink temperature, T2 = 100°C = 100 + 273 = 373 K
10 Source temperature, T1 = 0°C = 0 + 273 = 273 K
work done on system W = 10J
Coefficient of performance of refrigerator æ T2 ö
Percentage efficiency, %h = ç 1– T ÷ ´ 100
1 9 è 1ø
Q 2 1 - h 1 - 10 10 æ 273 ö æ 100 ö
b= = = 1 = 1 =9 = ç 1– ÷ ´100 = ç ÷ ´100 = 26.8%
W h è 373 ø è 373 ø
10 10 æV ö
Energy absorbed from reservoir 30. (a) W = nRT ln ç 2 ÷
è V1 ø
Q2 = bW = 9 × 10 = 90 J
22. (c) According to first law of thermodynamics æV ö
Q = DU + W W = R0T0 ln ç 2 ÷
è V1 ø
DU = Q – W
THERMODYNAMICS 553

æ1ö 7. (c)
W = 1015 ×10 × 10–3 ln ç ÷ Þ W = – 2.70 × 103 J
è 15 ø 8. (c) dU = nCv dT or 80 = 5 ´ C v (120 - 100)
31. (a) DQ = Du + DW Cv = 4.0 joule/K
m = 1 gm 9. (d) For isothermal process :
Lv = 540 cal/gm PV = Pi .2V
DQ = 1 × .540 = 540 P = 2Pi ...(i)
540 = Du + PDv For adiabatic process
540 = Du + 105 × (1671 – 1) × 10–6 PVg = Pa (2V)g (Q for monatomic gas g= 5 3 )
540 = Du + 167 5 5
Du = 540 – 167 = 373 cal or, [From (i)]
32. (d) 33. (a) 2Pi V 3 = Pa (2V) 3
-2
Exercise - 3 P 2 Pa
Þ a = 5 Þ =2 3
Pi Pi
1. (b) Qa = DU + 0 = DU
23
and Qb = DU + PDV 10. (d) DH = mL = 5 × 336 × 103 = Qsink
As Qb > Qa, \ Change in entropy is greater in case (b). Qsink T T
< sink Þ Qsource < source ≥ Qsink
2. (d) Here TVg –1 = constant Qsource Tsource Tsink
5 Energy consumed by freezer
As g = , hence TV2/3 = constant æ Tsource ö÷
3 ç ,1÷÷
[ w output < Qsource , Qsink < Qsink çç
Now T1L12/3 = T2 L2/3
2 (Q V µ L); çè Tsink ø÷
T1 æ L2 ö
2/3 Given: Tsource < 27↓C ∗ 273 < 300K,
Hence, =ç ÷ Tsink < 0↓C ∗ 273 < 273 k
T2 è L1 ø
3 æ 300 ö
3. (a) Change in internal energy for cyclic process (DU) = 0.
Woutput = 5≥336≥10 èççç ,1÷÷÷ < 1.67≥105 J
For process a ® b, (P-constant) 273 ø
Wa®b = P DV = nR DT = - 400 R 11. (d) Given P = aV
mV
For process b ® c, (T-constant)
Wb®c = -2R (300) ln 2
Work done, w =
ò PdV
V
For process c ® d, (P-constant) mV 2
aV
Wc®d = +400R
For process d ® a, (T-constant)
= ò aVdV =
2
(m 2 - 1) .
V
Wd®a = +2R (500) ln 2 12. (a) 13. (c)
14. (b) Q = Q1 + Q2 + Q3 + Q4
Net work (DW) = Wa ®b + Wb®c + Wc®d + Wd®a
= 6000 – 5500 – 3000 + 3500
DW = 400R ln 2 = + 1000 J
\ dQ = dU + dW, first law of thermodynamics W = W1 + W2 + W3 + W4
\ dQ = 400 R ln 2. = 2500 – 1000 – 1200 + x
4. (c) The temperature remains unchanged therefore = + 300 + x
U f = U i . Also, DQ = DW . In cyclic process,
DU = 0
In the first step which is isochoric, DW = 0 . Now, Q = DU + W
P or 1000 = 0 + (300 + x)
In second step, pressure = . Volume V is increased \ x = 700 J
n
from V on nV. W 1000
h = = = 10.5%
P æ n - 1ö Q1 + Q4 6000 + 3500
\ W = (nV - V) = PV ç = RT(1 - n -1 )
n è n ÷ø
5. (a) Initial and final condition is same for all process 15. (c) Let pA , p B be the initial pressures in A and B
DU1 = DU2 = DU3 respectively. When the gases double their volumes at
from first law of thermodynamics DQ = DU + DW pA p
Work done constant temperature, their pressures fall to and B
2 2
DW1 > DW2 > DW3 (Area of P.V. graph) pA
\ for A, p A - = Dp or pA = 2Dp
So DQ1 > DQ2 > DQ3 2
AT - BT 2 2 pB
6. (b) p = or V = AT - BT for B, p B -
2
= 1.5Dp or pB = 3Dp
V p
pA 2 mA m
AdT - B ´ 2TdT \ = Also, p AV = RT and p BV = B RT
\ dV = , since p is constant pB 3 M M
P
or pdv = (A – BT) dT pA mA 2
T2 T2
\ = = or 3mA = 2mB.
3
p B m B
W=
ò pdV = ò ( A - 2BT )dT
T1 T1
Þ A(T2 - T1 ) - B(T22 - T12 )
EBD_7418
554 PHYSICS

13. KINETIC THEORY

24. (a) Boyle temperature is defined as


Exercise - 1
a 27 æ 8a ö 27
1. (c) Boyle’s and Charle’s law follow kinetic theory of gases. TB = = ç ÷ = T = 3.375 Tc
Rb 8 è 27Rb ø 8 c
2. (b) At low pressure and high temperature the molecules = 3.375 × 100 = 337.5ºC
are far apart and molecular interactions are negligible. m
Without interactions the gas behaves like an ideal one. 25. (d) PV = RT
3. (a) 4. (a) M
5. (c) Perfect gas equation is PV = mRT. 6
Initially, PV = R ´ 500
m is the number of moles, R = NAkB M
M N P (6 - x)
m= = Finally, V = R ´ 300 (if x g gas leaks out)
M0 NA 2 M
6 5
\ PV = kBNT or P = kBnT Hence, 2 = ´ \ x = 1 gram
6. (c) Molecules of ideal gas behaves like perfectly elastic 6-x 3
26. (b)
rigid sphere. P1V1 P2 V2
7. (d) 8. (a) 9. (a) 27. (d) =
10. (b) According to ideal gas equation T1 T2
PV = nRT PV T 4 ´1500 ´ 270
\ V2 = 1 1 ´ 2 = = 2700 m3
m r r M r 1 T1 P2 300 ´ 2
PV = RT, P = RT or = or µ 28. (a) Given, V1 = V; V2 = 2V; T1 = 27° + 273 = 300 K
M M P RT P T
From charle’s law
r
Here, represent the slope of graph V1 V2 V 2V
P = or, =
T1 T2 300 T2
Hence T2 > T1
\ T2 = 600 K = 600 – 273 = 327°C
11. (c) 12. (d) 13. (a) 29. (a) From gas Law, PV = NRT Þ N = PV/RT
14. (b) According to boyle's law, at constant temperature.
1 1 ´ 105 ´ 1
Pµ = = 2.4 × 1025
or P1V1 = P2 V2 1.38 ´ 10 -23 ´ 300
V
15. (c) 16. (a) 30. (a) Avogadro’s Law, at S.T.P. 22.4L of any gas contains
17. (c) V and T will be same for both gases. 1 mol molecule
P1V = m1RT and P2V = m2RT 1
31. (b) Boyle’s Law states P µ for constant T..
5 æm ö 5 V
(P1/P2) = \ ç 1÷= V P 2P
3 è m2 ø 3 So, 1 = 2 = = 2:1
V2 P1 P
N1 N2 32. (b) Let n1 and n2 be the number of moles of each gas.
By definition, m1 = and m2 = Then
NA NA
PV PV
N1 m1 5 n1 = and n2 =
\ = = RT RT
N 2 m2 3 When the two gases are mixed, total number of moles,
18. (a) n = n 1 + n2
19. (b) According to ideal gas law
P 'V PV PV
P1V1 P2 V2 PV Þ = +
= or T2 = T1 2 2 RT RT RT
T1 T2 P1V1 (where P¢ is the pressure of the mixture.)
Þ P¢ = 2P
æ P öæ V ö
T ç ÷ç ÷ T 33. (d) VA = 2VB ; TA = 2TB ; PA = 2PB
\ T = è 2 øè 2 ø T2 = PAVA PV
2
PV 4 = B B = nAR = nBR
TA TB
20. (a) 21. (d)
22. (d) Apply Boyle’s law, at constant temperature hA PV T
\ = A A B
1 hB PBVB TA

V (2 PB ) (2VB ) (TB )
23. (c) = =2
PB VB (2TB )
KINETIC THEORY 555

34. (c) Lighter the molecule, higher the average speed. \ Velocity after collision is (–vx, vy and vz).
35. (c) In equilibrium, the average kinetic energy of The change in momentum is
molecules of different gases will be equal. That is – mvx – mvx = – 2mvx
53. (a) The average kinetic energy per molecule of any ideal
1 1 æ3 ö
m1v1-2 = m2 v2-2 = ç k BT ÷
gas is always equal to æç ö÷ kBT. It depends only on
2 2 è 2 ø 3
36. (b) Average kinetic energy of gas molecues depends è 2ø
on the temperature of the gas as the temperature and is independent of the mass and
1 2 3 nature of the gas.
mv = k BT 54. (b) As the temperature increases, the average velocity
2 2 increases. So, the collisions are faster.
37. (c) 38. (a)
55. (b) 1 mnc 2 and P ' = 1 æ m ö ´ n ´ (2c ) 2 = 2 P .
P0 = ç ÷
3rv 3 V 3è 2 ø 0
39. (c) v rms =
mass of the gas 56. (b) In all the three directions x, y and z gas possess equal
energies.
40. (a)
3
57. (d) 1 eV = kT
gRT 2
41. (a) The speed of sound in a gas is given by v = 3
M -23
or 1.6 ´ 10-19 = ´ 1.38 ´ 10 T
2
vO2 g O2 M He 1.4 4 \ T = 7.7 × 103 K.
\ = ´ = ´ = 0.3237
vHe M O2 g He 32 1.67 3
58. (a) k BT = K av where Kav is the average kinetic energy
vO 2 2
460
\ vHe = = = 1421 m / s of the proton.
0.3237 0.3237
2K av 2 ´ 4.14 ´ 10 –14 J
42. (a) (A) ® (3) ; (B) ® (4) ; (C) ® (1) ; (D) ® (2) \ = = = 2 ´ 109 K.
3k B –23
3 ´ 1.38 ´ 10 JK –1
43. (c) Internal energy can be increased when molecules of
gas will get greater velocity w.r.t. container.
44. (c) The centre of mass of the gas molecules moves with 3RTO2 3R(300)
59. (c) (v rms ) O 2 = (v rms ) H 2 \ =
uniform speed along with lorry. As there is no change in M O2 M H2
relative motion, the translational kinetic energy and hence
the temperature of the gas molecules will remain same. MO2 32
45. (d) Pressure exerted by a gas is given by TO 2 = 300 ´ = 300 ´ = 4800 K
M H2 2
1 mn 2
P= v
3 V 60. (a) The motion of the container is known as the ordered
1 –2 motion of the gas and zigzag motion of gas molecules
or P = rv \ Pµr within the container is called disordered motion
3 When the container suddenly stops, ordered kinetic
Therefore, pressure exerted by a gas is directly proportional energy gets converted into disordered kinetic energy
to the density of the gas. which is turn increases the temperature of the gas.
3RT Ordered motion
46. (a) Vrms = M is least for hydrogen among the
M Disordered motion
hydrogen, oxygen, nitrogen and carbon dioxide.
47. (d) 48. (c)
mass 1
3 61. (a) Volume = = m3
49. (d) Translational kinetic enegy E = kBNT density 4
2
2 5 5 1
\ PV = E [Q k B NT = PV] K.E = PV = ´ 8 ´ 104 ´ = 5 ´ 10 4 J
3 2 2 4
50. (a) Based on Maxwell's velocity distribution curve. 62. (a) When temperature is same according to kinetic theory
51. (a) Mean kinetic energy per molecule will be equal for of gases, kinetic energy of molecules will be same.
both the gases because it depends only upon the 2
1 æ1ö 1
3 K.E. = ´ 32 ´ ç ÷ = ´ 2 ´ v2
temperature. (E = k B T) 2 è2ø 2
2
52. (d) Since it hits the plane wall parallel to y z – plane and it RMS velocity of hydrogen molecules = 2 km/sec.
rebounds with same velocity, its y and z components 63. (a)
of velocity do not change, but the x-component
reverses the sign.
EBD_7418
556 PHYSICS

5 5
3R ´ 289 æ 3RT ö U = k B T ´ N A = RT (Q R = k B N A )
64. (c) v oxg . = ç vrms =
M ÷ø
2 2
32 è
78. (a) Work done is to be done in expanding the gas at
constant pressure.
3R ´ 400
vH = so vH = 2230.59 m/sec
2 PV 1.013 ´105 ´ 22.4 ´10-3
79. (a) R= =
65. (b) T 273
66. (a)
67. (b) Root-mean square-velocity is given by R 1.013 ´105 ´ 22.4
CP - CV = = = 1.972 kcal/kmolK
J 273 ´ 4200
3RT æTö 80. (b) Law of equipartition of energy states that the
v rms = i.e., v rms µ ç ÷
M èMø energy for each degree of freedom in thermal
1
é TO M H2 ù 1 equilibrium is kBT. Thus each vibrational mode
(v rms )O 2 2
\ = ê 2´ ú == 2 gives 2 degrees of freedom (kinetic and potential
( v rms )H 2 êë TH 2 M O 2 úû energy modes) corresponding to the energy
1 1930 1
\ ( v rms )O 2 = ( v rms )H 2 ´ = = 965 m/s 2 ´ kBT = kBT
2 2 2
68. (a) 81. (c) A fly moving in a room has three degrees of freedom,
3 3 because it is free to move in space.
69. (d) E = ´ 300 ; E ' = R(600) = 2E = 2 × 6.21 × 10–21
2 2 82. (b) Helium is monoatomic and hydrogen is diatomic.
= 12.42 × 10–21 J. Helium has smaller number of degrees of freedom
3R ´ 300 3R ´ 600
than hydrogen. So CP / CV for helium is more than
vrms = ; v 'rms = = 2 vrms that for hydrogen.
M M
= 682.44 m/s 83. (a) 84. (c)
85. (c) According to law of equipartition of energy, the energy
CP 2
70. (d) = g =1+ 1
CV n per degree of freedom is kT .
2
n1 + n2 n1 n For a polyatomic gas with n degrees of freedom, the
71. (b) = + 2
g –1 g1 - 1 g 2 – 1
1
2 1 1 mean energy per molecule = nkT
or g – 1 = 5 + 7 2
–1 –1 86. (c) No. of degree of freedom = 3 K – N
3 5 where K is no. of atom and N is the number of relations
3 between atoms. For triatomic gas,
\ g= .
2 K = 3, N = 3
72. (b) Mean free path in a gas is 100 times the interatomic 87. (d) Both are diatomic gases and Cp – Cv = R for all gases.
distance. 88. (c) P-V diagram of the gas is a straight line passing
1 through origin. Hence P µ V or PV–1 = constant
73. (a) Mean free path, l =
Molar heat capacity in the process PVx = constant is
where, 2 pd 2 n
n = number of molecules per unit volume, R R
C= + ; Here g = 1.4 (For diatomic gas)
d = diameter of the molecules g -1 1 - x
74. (b)
R R
75. (c) Coefficient of volume expansion at constant pressure ÞC= + Þ C = 3R
1.4 - 1 1 + 1
1 89. (c) Given
is for all gases. The average transnational K.E. is
273 CP - CV = 5000 J / mole °C .......(i)
same for molecules of all gases and for each molecules
CP
3 = 1.6 .......(ii)
it is kT CV
2
From Equation (i) & (ii),
kT
Mean free path l = (as P decreases, l C C 5000 5000
2 pd 2 P Þ P - V = Þ 1.6 - 1 =
increases) CV CV CV CV
Q 5000
76. (c) C= ; If DT = 0 , C = ¥ and if Q = 0 , then C = 0 Þ CV = = 8.33 ´ 103
m DT 0.6
77. (c) A rigid diatomic molecule has 5 degrees of freedom 4
total internal energy of one mole of rigid diatomic gas is Hence CP = 1.6,CV = 1.6 ´ 8.33 ´ 10C3P = 1.33 ´10
KINETIC THEORY 557

90. (d) 7
10R + R C p 47R 18
2 1 = 4 = 47R \ = ´ = 1.62
91. (c) Moles of He = = 9 18 Cv 18 29R
4 2
2
1
Molecules = ´ 6.02 ´ 10 23 = 3.01 × 1023 99. (a) As degree of freedom is defined as the number of
2 independent variables required to define body’s
As there are 3 degrees of freedom corresponding to 1 motion completely. Here f = 2(1 Translational + 1
molecule of a monatomic gas. Rotational).
\ Total degrees of freedom = 3 × 3.01 × 1023 100. (a) For an ideal gas CP – CV = R
= 9.03 × 1023 If CP – CV = 1.06 R
92. (b) then gas will be real gas. Thus pressure is high and
93. (b) For a monoatomic gas, the average energy of a temperatue is low for real gas.
3 EXERCISE - 2
molecule at temperature T is k B T .
2
1. (b) As the relative velocity of molecule with respect to
3 the walls of container does not change in rocket, due
\ Internal energy U = RT
2 to the mass of a molecule is negligible with respect to
dU 3 the mass of whole system and system of gas moves
CV = = R as a whole and (g = 0) on molecule energy where.
dT 2
For an ideal gas, CP – CV = R Hence pressure of the gas inside the vessel, as
observed by us, on the ground remain the same.
5 C 5 2. (d) Pressure on the wall due to force exerted by molecule
\ CP = R and g = P = on walls due to its rate of transfer of momentum to wall.
2 CV 3
94. (d) For a diatomic gas, In an ideal gas, when a molecule collides elastically
Molar heat capacity at constant pressure, with a wall, the momentum transferred to each molecule
7 will be twice the magnitude of its normal momentum is
CP = R 2 mv. For the wall EFGH, absorbs those molecules.
2 Which strike to it so rate of change in momentum to it
Molar heat capacity at constant volume, become only mv so the pressure of EFGH would half
7 of ABCD.
5 R 3. (b) Boyle's law is applicable at constant temperature and
CP 2 7
CV = R \ = = temperature remains constant in isothermal process.
2 CV 5 R 5 For ideal gas, pV = nRT = constant
2 1
No. of degree of freedom = 3 (3) – 3 = 6 So, pV = constant (at constant temperature) p µ
V
95. (b) According to Mayer's relationship So, this process can be called as isothermal process.
Cp – Cv = R 4. (c) Let us consider the given diagram where an ideal gas
Cp Cv R is contained in a cylinder, having a piston of mass M.
\ - = Here M = 28. The pressure on gas does not change.
M M M M
96. (c) pa
97. (a) Cp – Cv = R Þ Cp = Cv + R
Cp Cv + R C R
Q g= = = v + pa Mg/A
Cv Cv Cv Cv p
R R R
Þg=1+ Þ = g – 1 Þ Cv = A p
Cv Cv g -1
98. (a) For mixture of gas, The pressure inside the gas will be
n1Cv1 + n 2 Cv 2 p = pa + Mg /A
Cv = where, pa = atmospheric pressure
n1 + n 2 A = area of cross-section of the piston.
3 1 5 5 Mg = weight of piston
4 ´ R + ´ R 6R + R Hence, p = constant.
= 2 2 2 = 4 = 29R ´ 23 = 29R
9 As the piston and cylinder is frictionless so the
æ 1 ö 9´ 4 18
ç4+ ÷ 2
equation for ideal gas
è 2ø pV = nRT, volume (V) increases at constant pressure.
5R 1 7R as p, R, n are constant so,
n1Cp1 + n 2 Cp 2 4 ´ 2 + 2 ´ 2
and Cp = = V µT
(n1 + n 2 ) æ 1ö so on increasing temperature of system its volume
ç4+ ÷ increased but p will remain constant.
è 2ø
EBD_7418
558 PHYSICS

5. (a) As we know that an ideal gas equation, æ T ö ( p )(V )(1.1 T )


as the pressure and quantity of gas in system are p2 = ( p1V1 ) ç 2 ÷ =
constant è V2T1 ø (1.05)V (T )
[p1 = p, V2 = 1.05 V
æ nR ö
pV = nRT Þ V = ç ÷T and T2 = 1.1 T]
è p ø æ 1.1 ö
T V æ 1ö = p ´ç ÷ = p (1.0476)
V µ as n, R are constant µ ç ÷ è 1.05 ø
p T è pø So, final pressure p2 will lies between p and 1.1p.
Slope of the V – T graph, 1
9. (b) Mean free path lm =
dV nR 2p d 2 n
m= = [m = slope of V – t graph]
dT p where d = diameter of molecule and d = 2r
dV 1 1 1
µ or m µ [Q nR = constant] \ lm µ 2
dT p p r
10. (a) The mean free path of a gas molecule is the averge
1
So, p µ distance between two successive collisions. It is
m represented by l.
p1 m2 1 kT m
hence, = <1 l= and l =
p2 m1 2
2 ps P 2 × ps2 d
where, m1 is slope of the graph corresponding to p1
and similarly m2 is slope corresponding to p2. So slope Here, s = 0 diameter of molecule and
of p1 is smaller than p2. Hence, (p1 > p2). k = Boltzmann’s constant.
6. (d) Pressure exerted by gas is due to rate of change of Þ l µ 1 / d, l µ T and l µ 1 / P.
momentum (p) imparted by particles to wall. 11. (d) Molar mass of the gas = 4g/mol
Speed of sound
When the molecules breaks into atoms, the number of
moles would become twice. gRT g ´ 3.3 ´ 273
V= Þ 952 =
From ideal gas equation, m 4 ´ 10-3
pV = nRT 16 8
Þ g = 1.6 = =
where, p = Pressure of gas, n = Number of moles 10 5
R = Gas constant, T = temperature CP 8
As gases breaks number of moles becomes twice of Also, g = C = 5
V
initial, so n2 = 2n1 8´ 5
So, p µ nT So, CP = = 8JK–1mol–1 [CV = 5.0 JK–1 given]
5
12. (a) From PV = nRT
p nT (2n1 )(3000) r M r M
or 2 = 2 2 = = 20 So, p2 = 20p1
PA = A A and PB = B B
p1 n1T1 n1 (300) RT RT
Hence, final pressure of the gas would be 20 times to From question,
the initial pressure. PA rA M A M 3 M 3
= = 2 A = So, A =
7. (b) For a function f1(v) the number of molecules (n) which PB rB M B MB 2 MB 4
will have their speeds between v and v + dv.
For each function f1(v) and f2(v) number of molecules 1
13. (a) vrms µ Þ (v rms)1 < (vrms)2 < (vrms)3 also in
remain same 1 mole each but due to mass difference M
their speed will be different. mixture temperature of each gas will be same, hence
Hence both gases of each function f1(v) and f2(v) will kinetic energy also remains same.
obey the Maxwell's distribution law separately. 1
8. (d) As we know that an ideal gas equation, 14. (a) t=
æ ö 3RT
N
pV = nRT 2p d 2 ç ÷
where, n = Number of moles, p = Pressure, èVø M
g–1
As, TV = K
V = Volume, R = Gas constant,
T = Temperature So, t µ Vg + 1/2
pV g +1
n= Therefore, q =
RT 2
If n, R are constant for the system or as number of 15. (d) Let there are n 1 moles of hydrogen and n 2 moles of
moles of the gas remains fixed, hence, we can write helium in the given mixture. As Pv = nRT
pV p1V1 p2V2 Then the pressure of the mixture
= constant or =
RT1 RT2 n RT n 2 RT RT
T P= 1 + = ( n1 + n 2 )
V V V
KINETIC THEORY 559

(8.3 ´ 300 ) 22. (d) (crms )H 2 =1.84 km/s, (crms )O2 = ?


Þ 2 ´101.3 ´103 = ( n1 + n 2 ) ´
-3
20 ´ 10 M H2 = 2, M O2 = 32
2 ´ 101.3 ´ 10 ´ 20 ´10 -3
3
or, ( n1 + n 2 ) = Þ Rms velocity,,
(8.3)( 300 )
3RT
or, n1 + n2 = 1.62 ... (1) crms =
The mass of the mixture is (in grams) M
n1 × 2 + n2 × 4 = 5 cH 2 M O2
Þ (n1 + 2n 2) = 2.5 ... (2) \ =
cO 2 M H2
Solving the eqns. (1) and (2), we get
n1 = 0.74 and n 2 = 0.88 1.84 32
Þ = =4
m H 0.74 ´ 2 1.48 2 CO2 2
Hence, m = 0.88 ´ 4 = 3.52 = 5
He 1.84
16. (b) Here v1 = 200 m/s; Þ C O2 = = 0.46 km/s
4
temperature T1 = 27°C = 27 + 273 = 300 k 23. (b) When a person is driving a car then the temperature
temperature T2 = 127° C = 127 + 273 = 400 k, V = ? of air inside the tyre is increased because of motion.
R.M.S. Velocity, V µ T From the Gay Lussac’s law,
PµT
v 400 200 ´ 2 400 Hence, when temperature increases the pressure also
Þ = Þ v= m/s = m/s
200 300 3 3 increase.
17. (c) As no heat is lost, 24. (b) Given: Temperature Ti = 17 + 273 = 290 K
Loss of kinetic energy = gain of internal energy of gas Temperature Tf = 27 + 273 = 300 K
Atmospheric pressure, P0 = 1 × 105 Pa
Volume of room, V0 = 30 m3
1 2 Difference in number of molecules, Nf – Ni = ?
mv = nCV DT
2 The number of molecules
1 2 m R PV P0V0 æ 1 1 ö
Þ mv = × DT Þ N= (N0) \ Nf – Ni = ç - ÷N
2 M g –1 RT R è T f Ti ø 0
1 ´ 105 ´ 30 æ 1 1 ö
mv 2 (g –1) = ´ 6.023 ´ 10 23 ç - ÷ = – 2.5 × 1025
Þ DT = K 8.314 è 300 290 ø
2R
25. (a) As we know, Cp – Cv = R where Cp and Cv are molar
18. (c) The number 6.02 × 1023 is Avogadro’s number and
one mole of a substance contains Avogadro’s number specific heat capacities
of molecules. R
or, Cp – Cv =
19. (d) For a polytropic process M
R R R
C = Cv + \ C - Cv = For hydrogen (M = 2) Cp – Cv = a =
1- n 1- n 2
R R R
\ 1- n = \ 1- =n For nitrogen (M = 28) Cp – Cv = b =
C - Cv C - Cv 28
a
C - Cv - R C - C v - Cp + Cv \ = 14 or, a = 14b
\ n= = b
C - Cv C - Cv
é 500 + 600 + 700 + 800 + 900 ù
C - Cp 26. (b) vav = ê úû = 700 m/s
= (Q C p - C v = R ) ë 5
C - Cv
20. (b) and
21. (c) Internal energy of the system is given by
f 5002 + 600 2 + 700 2 + 8002 + 900 2
vrms = = 714 m/s
U = nRT 5
2
Degree of freedom; Fdiatomic = 5; fmonoatomic = 3 Thus vrms is greater than average speed by 14 m/s.
and, number of moles
n(O2) = 2; n(Ar) = 4 27. (b) Let at temperature T rms speed of oxygen molecules
become just sufficient for escaping from the Earth’s
5 3
Utotal = (2)RT + (4)RT = 11RT
T atmosphere
2 2 Vescape = 11200 m/s
EBD_7418
560 PHYSICS

3k BT é\ ù
Also, Vrms = Vescape = mO2 = 11200 m/s x1T1 283 ê
Þ x2 Þ = x T1 = 283K úú
T2 383 ê
Putting value of KB and mO2 we get, êëT2 = 383K úû
T = 8.360 × 104 K 3. (b)
28. (a) CP – CV = R is true for any gas. 4. (c) According to Dalton’s law of partial pressures, we
have P = P1 + P2 + P3
29. (c) By law of equipartition of energy, the energy for each 5. (b)
1 V
degree of freedom in thermal equilibrium is k B T . 6. (d) Using Charle's law, we have = constant
2 T
Each quadratic term form in the total energy expression
l l
of a molecules is to be counted as a degree of freedom. +5 -5
Thus each vibrational mode gives 2 degree of freedom Þ2 = 2
i.e, kinetic and potential energy modes, corresponding 373 273
As the piston moves 5 cm, the length of one side will
to the energy 2 æç k B T ö÷ = k B T .
1
æl ö
be ç + 5÷ and other side æç - 5ö÷ . On solving this
l
è2 ø
30. (c) In an adiabatic process è2 ø è2 ø
TVg–1 = Constant equaiton, we get l = 64.6 cm.
or, T1V1g–1 = T2V2g–1 7. (c)
5 8. (a) Pressure of the gas will not be affected by motion of
For monoatomic gas g = the system, hence by
3
(300)V2/3 = T2(2V)2/3
3P 3P 1
300 vrms = Þc2 = Þ P = rc 2
Þ T2 = r r 3
(2) 2/3
T2 = 189 K (final temperature) 9. (b)
f 12 + 22 + ...... + N 2 N(N + 1) (2N + 1)
Change in internal energy DU = n R DT 10. (d) Vrms = =
2 N 6N
æ öæ ö
3 25 1 + 2 + .....N N(N + 1)
= 2 ç ÷ç ÷ (-111) = -2.7 kJ
è 2 øè 3 ø Vmean = =
N 2N
31. (b)
3
32. (b) Let ‘n’ be the degree of freedom 11. (c) For a monatomic gas C v = R
2
æn ö So correct graph is
Cp çè + 1÷ø R 5 æ 2ö
2 æ 2ö
g= = = ç 1 + ÷ = 1.66 = 3 = ç 1 + 3 ÷ ­ 3 / 2R
Cv æ nö è nø è ø Cv
çè ÷ø R
2
Þ n = 3 Þ gas must be monoatomic. T®
12. (d)
EXERCISE - 3 R
13. (a) C = C v mix + .....(1)
P V P ¢ (90 / 100 ) V 1- n
1. (d) =
T T 3R 5R
n1C v1 + n 2 C v2 1´ +2´
P ¢ 100 10 Cvmix = = 2 2 = 13R
or = = 1+
P 90 90 n1 + n 2 1+ 2 6
P ¢ - P 10 13R R 1
or = = 11 .1 % From (1), 3R = + Þn=-
P 90 6 1- n 5
2. (d) Let the mass of the gas be m.
At a fixed temperature and pressure, volume is fixed. 14. (c) The average speed is given by
m 8kT kT
Density of the gas, r = v = = 1 .6
V πm m
r m m
Now = = here (M) atomic = 1.0gm, (m) atomic = 4.0gm,
P PV nRT hydrogen helium
m
Þ = x (By question)
nRT so v( H ) atomic = 2v(He) atomic
Þ xT = constant Þ x1T1 = x2T2
KINETIC THEORY 561

1 1 3
15. (a) P = r u 2 = ´ ( 6 ´ 10 - 2 ) ´ (500) 2 = n RT
3 3 2
= 5 ´ 10 3 N / m 2 Total energy when n moles of diatomic gas con-
verted into monoatomic (Uf)
3P 3 ´ 105
16. (a) c= = = 500 ms -1 3 5 1 5
r 1.2 = 2n RT + (N - n) RT = nRT + NRT
2 2 2 2
17. (b) From graph, T2V = const. .....(1) Now, change in total kinetic energy of the gas
As we know that TVg–1 = const
1
1 DU = Q = nRT
g-1
2
Þ VT = cons. ....(2) 19. (d) Using equipartition of energy, we have
On comparing (1) and (2), we get 6
Þ g = 3/2 KT = mCT
2
3P Pg
Also v rms = and vsound = 3 ´1.38 ´10 –23 ´ 6.02 ´1023
r r C= = 925 J/kgK
27 ´10–3
v rms 3 20. (b) We know that
Þ = = 2
vsound g PAVA = nRT, PBVB = nBRT and
18. (a) Energy associated with N moles of diatomic gas, Pf (VA + VB) = (nA + nB) RT
Pf (VA + VB) = PAVA + PBVB
5
Ui = N RT
2 æ P V + PBV B ö 1.4 ´ 0.1 + 0.7 ´ 0.15
\ Pf = ç A A = MPa
Energy associated with n moles of monoatomic gas è V A + V B ÷ø 0.1 + 0.15
= 0.98MPa
EBD_7418
562 PHYSICS

14. OSCILLATIONS

15. (c) When a particle is dropped from


Exercise - 1
a height h above the centre of
1. (c) phase at time, t = 2 pnt + a tunnel. h
2. (c) Displacement, y = r sin wt (i) It will oscillate, through the
dy earth to a height h on both sides.
V= = r w cos wt (ii) The motion of particle is periodic.
dt
(iii) The motion of particle will not be S.H.M.
dV
a= = – w2 r sin wt 16. (b)
dt 17. (a) At point 2, the acceleration of the particle is maximum,
a = – w2y \ a µ y which is at the extreme position. At extreme position,
3. (d) As seen from figure after one
the velocity of the particle will be zero.
time period the bob return to its
18. (c) Max. force = mass × max. acceleration
equilibrium position, so
= m 4 p2 n2 a = 1 × 4 ×p2 × (60)2 × 0.02 = 288 p2
diplacement of the particle is zero, 1/ 2
but distance covered by the é v2 ù
particle in one time period is 4A 19. (b) v = ω[A - x ] \Þ x = ê A2 -
2 2 1/ 2
ú
êë ω2 úû
(where A is amplitude of bob, A A
v Aω
when it does S.H.M.) Given that v = max = .
2 2
4. (d) x (t) = A cos (wt + f) é 2 A 2 w2 ù
1/ 2
3
dx so, ê A - ú = .A
= – wA sin (wt + f) 2 2
V(t) = ëê 4 w ûú
dt
20. (c)
dv
a(f )= = – w2A cos (wt + f) 21. (d) v2 = w2(A2 – x2) … (i)
dt and a2 =(w2x2) = w4x2 … (ii)
5. (b) Elasticity brings the particle towards mean position
From above equations, we have
and inertia needed to cross mean position.
6. (a) The motion of the particle is periodic, (not oscillatory), a2
v =-
2 + w2 A2 Þ y = mx + c
beacuse it returns to its starting point after a fixed time. w2
7. (c) The motion of particle is circular or elliptic, when two It represents straight line with negative slope.
S.H.M. which are perpendicular to each other 22. (d) Initially t = 0
superimpose on the particle. The particle moves on a x = a cospt = – a cos 0° = a
(i) Ellipse if amplitudes of two S.H.M. are different Finally at t = 3
(ii) Circle, if amplitudes of two S.H.M. are same. x = a cos 3p = – 3a
8. (d) Total displacement = 2a
9. (c) The motion of a planet around the sun is periodic 23. (c) Given t = 1s
motion but not a simple harmonic motions. æ pö
10. (b) Every simple harmonic motion (S.H.M.) is necessarily \ x = 5 cos ç 2p + ÷
è 4ø
periodic, but a periodic motion may or may not be simple
harmonic motion. p 5
= 5 cos = m
11. (d) Let y = A sin wt 4 2
dy 5
vinst = = Aw cos wt = Aw sin(wt + p / 2) i.e., displacement at t = 1s is m
dt 2
2 24. (a) Here,
Acceleration = - Aw 2 sin wt = Aw sin(p + wt)
x = x0 cos (wt – p / 4 )
p \ Velocity,
\f = = 0.5p
2 pö
dx æ
12. (b) v= = - x 0 w sin ç w t - ÷
dt è 4ø
13. (d)
14. (a) t = 0, v maximum. The motion begins from mean position. Acceleration,
So it represents S.H.M. dv æ pö
a= = - x 0 w2 cos ç wt - ÷
dt è 4ø
OSCILLATIONS 563

é æ p öù æf -f ö
= x 0 w2 cos ê p + ç wt - ÷ ú \ 2A sin ç 2 1 ÷ = A.
ë è 4 øû è 2 ø
= x0 w2 Þ f2 – f1 = p/3.
3p 31. (c) The force at the extreme position is, F = mw2A.
cos æç wt + ö÷ ...(1) 32. (d) At mean position velocity is maximum
è 4 ø
Acceleration, a = A cos (wt + d) ...(2) vmax 16
i.e., vmax = wa Þ w = = =4
Comparing the two equations, we get a 4
3p
A = x0w2 and d = \ v = w a 2 - y 2 Þ 8 3 = 4 42 - y 2
4
25. (a) With respect to an observer, the force on the particle Þ 192 = 16(16 - y 2 ) Þ 12 = 16 - y 2 Þ y = 2cm.
F = –k[x + (v0 – v0)t] = – kx, so it represents SHM.
26. (a) For x = (– A), we have dy1 æ pö
33. (b) v1 = = 0.1´ 100p cos ç100pt + ÷
–A =A sin(w × 0 + f0) dt è 3ø
p
or f0 = – . dy 2 æ pö
2 v2 = = - 0.1p sin pt = 0.1p cosç pt + ÷
So for x < (–A), f0 < (–p/2). dt è 2ø
27. (a) Velocity in SHM is given by
p p 2 p - 3p p
v = w a 2 - y2 \ Phase diff. = f1 - f 2 = - = =–
3 2 6 6
At y = 4 cm = 0.04 m, v = 3 m / s 34. (b) Equation of S.H.M. is given by
\ 3 = w a 2 - ( 0.04) 2 ....(1) x = A sin (wt + d)
At y = 3 cm = 0.03 m, v = 4 m/s (wt + d) is called phase.
A
\ 4 = w a 2 - ( 0.03) 2 ....(2) When x = , then
2
Dividing (2) by (1), we get a = 0.05 = 5 cm 1
28. (b) Maximum velocity in SHM, vmax = aw sin (wt + d) =
2
Maximum acceleration in SHM, Amax = aw2 p 1
A
where a and w are maximum amplitude and angular Þ wt + d =
6
frequency. p A
or f1 = 2 x = A/2
A 6
Given that, max = 10 i.e., w = 10 s–1 For second particle, x=0
v max
p 5p
Displacement is given by f2 = p - =
x = a sin (wt + p/4) 6 6
4p 2p
at t = 0, x = 5 \ f = f2 - f1 = =
6 3
5 = a sin 45° Þ a = 5 2 1
35. (d) E = mw2 a 2 Þ E µ a 2
Maximum acceleration Amax = aw2 = 500 2 m/s2 2
36. (a) P.E. of body in S.H.M. at an instant,
æ pö 1 1
29. (c) Given, x = 10 sin ç 2t – ÷ U = m w 2 y 2 = ky 2
è 6ø 2 2
A = 10 and w = 2 Hz If the displacement, y = (a – x) then
1 1
\ u = w A 2 – d 2 = 2 (10)2 – (6)2 U = k(a - x)2 = k(x - a) 2
2 2
–1
= 2 100 – 36 = 2 ´ 8 = 16m s 37. (a) If displacement of particle is y, then
30. (a) Let x1 = A sin(wt + f1) and x2 = A sin (wt + f2) 1 2 2 2 1
x2 – x1 = A[sin (wt + f2) – sin(wt – f1)] KE = mw (a - y ) & P.E. = mw2 y 2
2 2
æ 2wt + f1 + f 2 ö æ f 2 - f1 ö 1 1 2 2 1 2 2
= 2A cos ç ÷ø sin çè ÷ If KE = PE mw2 y 2 = mw a - mw y
è 1 2 ø 2 2 2
The resultant motion can be treated as a simple harmonic a
2y2 = a2 \ y =
æf -f ö 2
motion with amplitude 2A sin ç 2 1 ÷ . 38. (d) Total mechanical energy is constant throughout the
è 2 ø
Given, maximum distance between the particles 1
\ Amplitude of resultant S.H.M. motion and equals mw2 A2 .
2
= X0 + A – X0 = A
EBD_7418
564 PHYSICS

1 2 1 1
39. (c) Kinetic energy, K = mv = mw2 A2 sin 2 (wt + f) In SHM, E = kA 2
2 2 2
1 2 2
Potential energy, U = m w A cos 2 (wt + f) 1
2 \ 4 = ´ 800 ´ A 2
1 2
2 2
\ Total mech. energy = mw A 8 1
2 A2 = = , A = 0.1 m
40. (a) The total energy of S.H.M = Kinetic energy of particle 800 100
+ potential energy of particle Maximum acceleration, amax = w2A
The variation of total energy of the particle in SHM k æ k ö
with time is shown in a graph = A çèQ w = ÷
m mø
Energy 800 Nm-1
Zero slope = ´ 0.1 m = 20ms -2
A Total energy 4 kg
Kinetic energy
48. (b) The kinetic energy of a particle executing S.H.M. is
Potential energy
given by
T/4 2T/4 3T/4
1
1 1 K = ma2 w2 sin2wt
41. (a) K. E. = mw 2 (a 2 – y 2 ) and P. E. = mw2 y 2 2
2 2 where, m = mass of particle
1 2 2 a = amplitude
At mean position, y = 0 \ K. E. = mw a
2 w = angular frequency
a 1 a2 1 2 2 t = time
At , y = ; P. E. = mw2 = mw a Now, average K.E. = < K >
2 2 4 8
(1 / 2)mw 2 a 2 1 1
\ Ratio = = < mw2 a2 sin2 wt > = mw2a2 <sin2 wt >
(1 / 8)mw 2 a 2 2 2
1 æ 1ö 1 2 2 æ 2 1ö
=
K.E. at mean position 4
= = mw2a2 çè ÷ø = mw a çè Q < sin q > = ÷ø
PE at a / 2 1 2 2 4 2
42. (a) K.E. of a body undergoing SHM is given by,
1 1 1 2
2 2 2
K .E. = ma w cos wt and T .E. = ma w
2 2 = ma (2pn )2 (Q w = 2pn)
2 2 4
Given K.E. = 0.75 T.E.
or, < K > = p2 ma 2 n 2
2 p
Þ 0.75 = cos wt Þ wt = 49. (a) In x = A cos wt, the particle starts oscillating from extreme
6 position. So at t = 0, its potential energy is maximum.
p p´2 1 1
Þt= Þt= Þt= s 50. (b) In SHM, Total energy, E total = mw 2 A2
6´w 6 ´ 2p 6 2
1 1 1
43. (b) K = mw 2 ( A2 - x 2 ) \ K max = mw 2 A2 , at x = 0. and, Kinetic energy, E K = mw 2 ( A2 – x 2 )
2 2 2
1 where x is the distance from the mean position.
44. (a) Eav = U av = mw 2 A2
4 At x = 0.707A
45. (a) KE and PE completes two vibration in a time during
1 1
which SHM completes one vibration. Thus frequency E K = mw 2 ( A2 – (0.707 A2 ) = mw 2 (0.5 A2 )
of PE or KE is double than that of SHM. 2 2
As per question, Etotal = 100 J
46. (a) As we know,
æ1 2 2ö
1 \ E K = 0.5 ç mw A ÷ = 0.5 ´ 100J = 50J
kinetic energy = mw2 (A 2 – x 2 ) è2 ø
2
1 A
2 2 51. (c) Given, x =
Potential energy = mw x 2
2
\ from x = A sin wt
1
mw2 (A 2 – x 2 ) A2 – x 2 1 Þ wt = 30°
2 1
\ = Þ =
KE
( )
1 2 4 2
2 2
mw x
4 x And, = cot 2 wt = 3 =3
2 PE
2 T
4 52. (d) = 0.5 sec Þ T = 1 sec
4A2 – 4x2 = x2 Þ x 2 = A 2 Þ x = A. 2
5 5 1
47. (d) Here, m = 4 kg; k = 800 Nm–1; E = 4 J n = = 1 Hz
T
OSCILLATIONS 565

53. (a) At extreme position


p æ g ö
l = ç 2 ÷ T2
y = A sin (wt + ) = A cos wt è 4p ø
2
Now y = A/2 then A/2 = A cos wt Þ wt = cos–1 (1/2) \ l µ T2
2p p
t = Þt =T/6 70. (a) (A) For equilibrium, F = 8 – 2x = 0
T 3 or x=4m
54. (a) 55. (c)
56. (c) When a simple pendulum oscillates in air continously Equilibrium
then its time period remains same whereas amplitude (B) From figure, position
decreases. O A
4
57. (a) 6
58. (b) T = mg cos q
At the mean position, q = 0° and cos 0° = 1 A = 2m
So, the value of tension is greatest. (C) Time taken from x = 2 to 4
59. (a)
T p
1 or A to O is , which differes in phase by .
mw2 r12 æ ö2 2 4 2
E1 2 r æ5ö
60. (c) = =ç 1 ÷ = ç ÷ =1:4 6
E2 1
2
r
mw2 r22 è 2 ø è 10 ø (D) Energy of SHM,E = ò Fdx
4
61. (b) 6
62. (c) For spring mass m system, the time period
of the oscillation of mas m is defined as
= ò (8 - 2 x)dx = 8x - x
4
26
4
= 4 J.

T = 2p m / k = 2p y / g L + y0 71. (a)
ky
72. (d) According to equation of SHM
where m = mass of particle x = A sinwt
k = spring constant A
y = extension in spring Here, x = and t = 1
mg
2
L = natural length of spring A
If g is changed, then y also changes so that y/g is \ = A sin w ´ 1
2
constant, so the time period T of spring mass is = A sin w
independent from the variation in g. Hence n 1 p
(frequency) will also not change. or, sin w = , w =
2 6
63. (b) 2p p
64. (b) Let k be the force constant of spring of length l2. Since \ = or T = 12 s
T 6
l1 = n l2, where n is an integer, so the spring is made of 73. (a) Maximum velocity,
(n + 1) equal parts in length each of length l2.
v max = aw
1 (n + 1)
\ = or k = (n + 1) K 2p
K k v max = a ´
T
The spring of length l1 (= n l2) will be equivalent to n
springs connected in series where spring constant 2pa 2 ´ 3.14 ´ 7 ´10 -3
ÞT= = » 0.01 s
k v max 4.4
k¢ = = (n + 1) K / n & spring constant of length l2 is
n 74. (b) Using v 2 = ω 2 (a 2 - y 2 ) we have
K(n+1).
65. (a) keq = k + 2 k = 3 k 10 2 = w 2 (a 2 - 4 2 ) and 8 2 = w 2 (a 2 - 5 2 ) ;
66. (c) v max = a ω ; v¢max = 2a w = 2 vmax = 2v so 102 – 82 = w2 (52 – 42) = (3w2) or 6 = 3 w or w = 2
(since vmax = v) or T = 2 p/w = 2 p/2 = p s.
67. (b) When some mercury is drained off, the centre of grav- 75. (b) Here, x = 2 × 10–2 cos p t
ity of the bob moves down and so length of the pendu- Speed is given by
lum increases, which result increase in time period. dx
v= = –2 × 10–2 p sin p t
68. (b) pmax = 2mE max dt
For the first time, the speed to be maximum,
l sin p t = 1
l p
69. (d) T = 2p or, sin p t = sin
g 2
p 1
T Þ pt = or t = = 0.5 sec.
2 2
EBD_7418
566 PHYSICS

m m V12 – V22
t 1 = 2p , t 2 = 2p Þ w=
76. (b) k1 k2 x 22 – x12

k1k 2 x 22 – x12
when springs are in series then, k eff = Þ T = 2p
kl + k 2 V12 – V22
m( k l + k 2 ) 1 2 1 2 mv02
\ T = 2p 82. (b) kx0 = mv0 or x0 =
k1k 2 2 2 k
é T ù
m m t 22 t2 Time period, T¢ = êë 2 4 + 2t úû
\ T = 2p + = 2p + 1
k 2 k1 ( 2 p) 2 ( 2 p) 2 é 2p m l ù
= ê +2 0ú
2
ÞT = t12 + t 22 ë 2 k v0 û

77. (c) (n + 1)TS = nTL 1 2


Energy of oscillation = mv0 .
2
(n + 1) 1 = n 1.44
83. (d) y = 5sin(p t + 4p ), comparing it with standard equation
Þ (n + 1) = 1.2n Þ n = 1 = 5 æ 2p t ö
0.2 y = a sin(wt + f ) = a sin ç +f ÷
78. (a) Force on a spring F = k.x è T ø
2p t
1 2 F .x F F F 2 a = 5m and = p t Þ T = 2sec .
W= kx = = . = T
2 2 2 k 2k 84. (b) Two springs of force constants k1 and k1 are in parallel.
W1 k2 k B Hence
Now, = k ¢ = k1 + k1 = 2 k1
W2 = k1 k A
The third spring k2 is in series with spring of force
5T constant k¢.
79. (d) (n + 1)T = ´n -1
4 1 é 1 1 ù é 1 1 ù
n=4 \ =ê + ú or k = ê + ú
k ë 2k1 k 2 û ë 2 k1 k 2 û
Time t = (n + 1) T = 5T
dU
l 85. (b) U = U 0 + ax 2 Þ F = - = -2ax
80. (d) T = 2p dx
g
F 2a 2a 2p m
a= =- x Þ w2 = ÞT= = 2p
1 ælö m m m w 2a
log T = log( 2p) + logçç ÷÷
2 ègø 86. (a)
1 1 87. (a) For the block is about to slip, mg = w 2 a
Þ log T = log(2π) + log(l) - log(g)
2 2
mmgg 1 mg
DT 1 Dl ÞÞpwν == Þν=
Differentiating, = 0 + ´ -0 aa 2p a
T 2 l
88. (d) Oscillations along spring length are independent of
DT 1 Dl gravitation.
Þ ´ 100 = ´ ´ 100
T 2 l 89. (d) The second pendulum placed in a space laboratory
1 orbiting around the earth is in a weightlessness state.
= ´ 21 = 10.5 » 10% Hence g = 0 so T = ¥
2
81. (a) As we know, for particle undergoing SHM, 90. (a) The restoring force
V =wA – X 2 2 F = – ( PA) y
y
= –[rg (2 y sin 30°)] A
V12 = w2 (A 2 – x12 ) 30° 30°
= rgA( - y )
V22 2
= w (A – 2
x 22 )
Substructing we get, F rgA m
\ a = = Thus T = 2p rgA
m m
V12 2 V22
+ x1 = + x 22 l l
w2 w2 91. (a) t = 2p ; t 0 = 2p
g eff g
V12 – V22
Þ = x 22 – x12 Bob in air
w2
OSCILLATIONS 567

2p
1000 Vg As we know, w =
T
2p 2p
\ Time period, T = = = p 2s
4 w 2
´ 1000 Vg
3 97. (b)

Bob in water l
98. (c) As we know, time period, T = 2p
æ4 ö 1000 g
Net force = ç - 1÷ ´ 1000 Vg = Vg
When additional mass M is added then
è3 ø 3
1000Vg g é 4 ù l + Dl
g eff = = ê Mass = ´ 1000 ´ V ú TM = 2p
4 4 ë 3 û
3 ´ ´ 1000V g
3
l l + Dl æ TM ö
2
l + Dl
\ t = 2p TM =
g / 4 \ t = 2t 0 l or çè ÷ø =
T T l
92. (d) If it gives correct time at equator, it will give correct 2 é Mgl ù
æ TM ö Mg
time at poles also because the time period of spring- or, ç =1+ êQ Dl = Ay ú
è T ÷ø Ay ë û
mass system is independent of g.
1 éæ TM ö
93. (c) As we know, 2 ù
A
F = ma Þ a µ F or, a µ sin t \ = êç ÷ - 1ú
y êëè T ø úû Mg
0 t
dv
Þ
dt
µ sin t Þ ò dV µ ò sin t dt 99. (c) fA =
1 g f
fB = A =
1 g
0 0 2p L A and 2 2p LB
V µ – cos t + 1
x t fA 1 g L LB L
= ´ 2p B Þ 2 = Þ 4= B ,
ò dx = ò (- cos t + 1) dt Þ x = sin t – 1 sin 2t \
fA/ 2 2p LA g LA LA
0 0 2
regardless of mass.
94. (c) h = Length of block immerged in water 100. (b) y = kt2
mg = FB
dy d2y
rAl g = rB Ahg \ = 2 kt or = 2k
dt dt 2
650 × A × 54 × 10–2g = 900 × A × hg
Þ h = 0.39 m = 39 cm. or ag = 2m / s 2 (Q k = 1 m/s2 given)
2
95. (a) Under the action of first force, F1 = mw1 y l
We know that T = 2p
g
Under the action of second force, F2 = mw 22 y
Under the action of resultant force, T12 g2 T12 12 6
\ = Þ 2 = =
F1 + F2 = mw 2 y Þ mw 2 y = mw12 y + mw 22 y T22 g1 T2 10 5
2
æ 2p ö æ 2p ö
2 2 [Q g1 = 10 m/s2 and g2 = g + 2 = 12 m/s2]
æ 2p ö
Þ w 2 = w12 + w 22 Þ ç ÷ = ç ÷ + ç ÷ 101. (a) Suppose the liquid in left side limb is displaced slightly by
è Tø è 1ø
T è T2 ø y, the liquid in right limb will increase by y/2. The restoring force
2 2 F = –PA A 2A A 2A
æ 4 ö æ 3ö
T12T22 çè ÷ø × çè ÷ø æ 3y ö
5 5 12 = –rg ç ÷ ´ 2 A = 3rgA( - y ) .
ÞT= = = . è 2ø y/2
T12 + T22 æ 4ö æ 3ö
2 2
25 y
çè ÷ø + çè ÷ø F m
5 5 a= = 3rgA(- y ) / m
m
96. (a) The differential equation of simple harmonic motion is
2
On comparing with, a = -w y , we get
d2y d2y
+ 2 y = 0 or = –2 y ...(i) 3rgA m
dt 2 dt 2 w= and T = 2p
m 3rgA
Standard equation of simple harmonic motion is
102. (a) The restoring torque (for small q)
d2y
= –w2 y ...(ii) Lq
dt 2 2 q
Comparing eq. (i) and (ii), L/2 L/2
w2 = 2 or w = 2
EBD_7418
568 PHYSICS

110. (a) In case of sustained force oscillations the amplitude


é kLq L ù 2
trest = - ê ´ ú ´ 2 = kL ( -q) of oscillations decreases linearly.
ë 2 2û 2
111. (b)
t rest kL2 / 2 6k 112. (c)
\ a= = ( -q) = ( -q)
I ML2 /12 M 113. (a) Displacement amplitude of an oscillator depends on
M the angular frequency of the driving force.
\ T = 2p .
6k 114. (c) Damping can never be zero in reality, so amplitude can
103. (b) Springs on the left of the block are in series, hence their never be infinity.
equivalent spring constant is 115. (d) F = – bV, b depends on all the three i.e, shape and size
(2k)(2k) of he block and viscosity of the medium.
K1 = =k 116. (b) Amplitude of a damped oscillator at any instant t is
2k + 2k
Springs on the right of the block are in parallel, hence their equivalent given by
spring constant is A = A0e–bt/2m
k2 = k + 2k = 3k where A0 is the original amplitude
Now again both K1 and K2 are in parallel From question,
\ Keq = k1 + k2 = k + 3k = 4k
A A
Hence, frequency is When t = 2 s, A = 0 \ 0 = A0e–2b/2m
3 3
1 Keq 1 4k 1
f = = or, = e–b/m … (i)
2p M 2p M 3
l A A
104. (c) t0 = 2p g . When t = 6 s, A = 0 \ 0 = A0e–6b/2m
n n
The restoring force in a liquid q 1
or, = e–3b/m = (e–b/m)3
F = -(mg - V re g )sin q l n
Vr e g
3
æ m ö 1 æ 1ö
mg - ´ 1000 g æ x ö x or, =ç ÷ \ n = 33 (Using eq. (i))
= -ç æ4 ö ÷ç ÷ n è 3ø
ç çè ´ 1000÷ø ÷ è lø
è 3 ø q
æ 3g ö æ - x ö g æ - x ö mg 117. (d)
or a = çè g - ÷ø çè ÷ø = 4 çè ÷ø 118. (b) At resonance, amplitude of oscillation is maximum
4 l l
Þ 2w2 – 36w + 9 is minimum
l
\ t = 2p ( g / 4) = 2t0 . Þ 4w – 36 = 0 (derivative is zero) Þ w = 9
bt
105. (a) In equilibrium position –
m 119. (d) As we know, E = E0 e m
mg = FB = r0 (1 + ay0 ) g Þ 4 = 1 + ay0
4r0 –
b15

b15
1
3 15 = 45e m Þ = e m
y0 = = 1.5 3
2 [As no. of oscillations = 15 so t = 15sec]
Now displace it downward by Dy
m
Taking log on both sides
\ F = mg - r0 [1 + a ( y0 + Dy)] g b 1
4r0 = ln3
mg m 15
= mg - [1 + ay0 + a Dy ] 120. (d) 121. (d)
4 122. (c) At maximum energy of the particle, velocity resonance
mga æ ag ö takes place, which occurs when frequency of external
F=- Dy ; Acceleration = - ç ÷ Dy
4 è 4ø periodic force is equal to natural frequency of
2p 4 2p undamped vibrations, i.e. w2 = w0.
\ T= = 2p = sec. Further amplitude resonance takes place at a frequency
w ag 5
of external force which is less than the frequency of
106. (c) In forced oscillations, the body oscillates at the angular
frequency of the driving force. undamped natural vibrations, i.e. w1 ¹ w0 .
107. (b) The resonance wave becomes very sharp when 123. (b) x = A sin(wt + f)
damping force is small. F0 F0
108. (d) where A = =
109. (c) A particle oscillating under a force F = - kx - bu is 2
m (w 0 - w )2 2 m (w 0 2 - w 2 )
a damped oscillator. the first term -kx represents Here damping effect is considered to be zero
the restoring force and second term - bu represents
1
the damping force. \x µ
2 2
m (w 0 - w )
OSCILLATIONS 569

124. (b) Vm = V0/(aw2 – bw + c) Compare it by standard equation


If there is a single resonant frequency, then this y = a cos(wt + f)
equation should be satisfied for only one that particular So, w¢ = 2w
resonant frequency, hence aw2 – bw + c = 0 will have
2p 2p p
equal roots therefore; = 2w Þ T ' = =
D = 0 Þ (–b)2 – 4ac = 0 Þ b2 = 4ac T' 2w w
125. (d) For damped harmonic motion, p
ma = – kx – mbv Hence, the motion is SHM with period .
or ma + mbv + kx = 0 w
Solution to above equation is 2. (b) A motion will be harmonic if a µ displacement and a
-
bt 2 simple harmonic motion is always periodic but all simple
k b
x = A0 e 2 sin wt; with w2 =
- harmonic motion are periodic but all periodic are not
m 4 harmonic.
where amplitude drops exponentially with time
bt
As given equation of motion is
-
i.e., A t = A0 e 2 y = sin3 wt
Average time t is that duration when amplitude drops = (3 sin wt – 4 sin 3wt)/4
by 63%, i.e., becomes A0/e. [Q sin 3q = 3 sin q – 4 sin3 q]
bt Differentiating both side w.r.t. t
A -
Thus, A t = 0 =A0e 2 dy é d d ù
e So, v = = ê (3sin wt ) - (4 sin 3wt ) ú / 4
2 dt ë dt dt û
bt
or = 1 or t =
2 b 3 é (3sin q - sin 3q) ù
or sin q = ê ú
ë 4 û
Exercise - 2
dy
4 = 3w cos wt - 4 ´ [3w cos 3wt ]
æp ö dt
1. (b) As given that, y = 3cos ç - 2wt ÷ Again, differentiating both side w.r.t. t
è4 ø
Velocity of the particle d2y
4´ = -3 w 2 sin wt + 36 w sin 3wt
dy d é æp öù dt 2
v= = 3cos ç - 2wt ÷ ú
dt dt êë è4 øû d 2 y -3 w 2 sin wt + 36 w sin 3wt
a= =
é æp öù dt 2 4
= 3(-2w) ê - sin ç - 2wt ÷ú
ë è4 øû æ d2y ö
So, çç 2 ÷÷ is not directly proportional to y.
æp ö è dt ø
= 6w sin ç - 2wt ÷
è 4 ø So motion is not harmonic.
So, acceleration, y(t) = sin 3 wt,
dv d é æp öù 3 é 2p ù
a= = ê6w sin ç - 2wt ÷ ú y(t + T) = sin 3(wt + T) = sin ê (t + T ) ú
dt dt ë è4 øû ëT û
æp ö = sin3 (2p + wt) = sin3 wt
= (6w) ´ (-2w) cos ç - 2wt ÷
è 4 ø 3. (d) For motion to be SHM acceleration of the particle must
be opposite of restoring force and proportional to
é æ p ö ù
= -4w2 ê3cos ç - 2wt ÷ú negative of displacement. So, F = ma = m(–2x)
ë è4 øû i.e., F = –2mx, so F µ – x
a = –4w2y
In simple harmonic motion acceleration (or force) is Hence, ax = –2x
directly proportional to the negative of displacement We should be clear that x has to be linear.
of particle 4. (c) Consider a U-tube filled in which a liquid column
Þ as acceleration, a µ – y oscillates. When liquid column lifted upto height y from
Hence, due to negative sign motion is simple harmonic A to B in arm Q. The liquid level in arm P becomes at C',
motion (SHM.) so the difference between the height of two columns
A simple harmonic motion is always periodic. So motion are = AB + A'C = y + y = 2y. P Q
is periodic simple harmonic. In this case, restoring force acts on B' B
From the given equation, the liquid due to gravity. Acceleration y
A' A
æp ö of the liquid column, can be calculated h C
C'
y = 3cos ç - 2wt ÷ in terms of restoring force.
è4 ø
Restoring force, Equilibrium level
EBD_7418
570 PHYSICS

f = Weight of liquid column of height 2y 7. (b) When pendulum vibrate with Elastic support
transverse vibration th en G G
f = -( A ´ 2 y ´r) ´ g = -2 Argy [Q h = rv]
l
As restoring force at A opposite to gravitational force T = 2p e
g B
as liquid is lifted against wt(mg)mk = 2Arg
l = length of pendulum. A
Then f µ – y so motion is SHM. C
D
Time period
Through the elastic rigid support the disturbance is
m A ´ (2h) ´ r h h transferred to all the pendulum A and C are having
T = 2p = 2p = 2p ; T = 2p length and hence same frequency. They will be in
k 2 Arg g g
resonance, because their time period of oscillation.
Time period is independent of the density of the liquid
So, a periodic force of period (T) produces resonance in
and motion is harmonic.
A and C and they will vibrate with maximum amplitude
5. (c) The resultant-displacement can be find by adding x
as in resonance.
and y-components.
8. (a) As the particle (P) is executing circular motion with
According to variation of x and y, trajectory will be radius B.
predicted, so resultant displacement is y' = (x + y) Let particle P is at Q at instant any (t), foot of
As given that, perpendicular on x axis is at R vector OQ makes Ðq ,
x = a cos wt ...(i) with its zero position not P displacement of particles
y = a sin wt ...(ii) for O to R. y
p(t=0)
So, y¢ = (a cos wt + a sin wt) Consider angular velocity of the
r Q
Þ y¢ = a(cos wt + sin wt) particle executing circular motion q q
is w and when it is at Q makes O R
90–q
x
é cos wt sin wt ù and angle q as shown in the B
y' = a 2 ê + ú
ë 2 2 û diagram.
Clearly, q = wt
y ' = a 2(cos wt cos 45° + sin wt sin 45°)
Now, we can write
= a 2 cos(wt - 45°) OR = OQ cos (90 – q) (Q OR = X)
So, the displacement is not straight line and not x = OQ sin q = OQ sin wt
parabola also.
= r sin wt [Q OQ = r]
Now, squaring and adding eqs. (i) and (ii),
x 2 + y 2 = a2 [Q cos2 wt + sin2 wt = 1] 2p æ 2p ö
\ x = B sin t = B sin ç t ÷
This is the equation of a circle, so motion in circular T è 30 ø
(independent of time).
Clearly, the locus is a circle of constant radius a. æ 2p ö
x = B sin ç t ÷
6. (d) As given that, the displacement is è 30 ø
y = a sin wt + b cos wt Hence, this equation represents SHM.
Let a = A sin f ...(i) 9. (c) According to the question,
and b = A sin f ...(ii)
Squaring and adding (i) and (ii) x = a cos (at)2
is a cosine function, so it is an oscillatory motion.
a 2 + b2 = A2 sin 2 f + A2 cos 2 f
Now, at, t = (t + T). The equation of motion of particle
A = a 2 + b2 (amplitude)
x(t + T ) = a cos[a(t + T )]2 [Q x(t ) = a cos(at )2 ]
y = A sin f× sin wt + A cos f cos wt
y = A cos(wt – f) = a cos[at 2 + aT 2 + 2atT ] ¹ x (t )
where, T is supposed as period of the function w(t).
dy
= - A w sin(wt - f) Hence, it is oscillatory but not periodic.
dt
10. (a) Let us consider a equation of an SHM is represented
d2y by y = a sin wt
= - Aw 2 cos (wt - f) = - w 2 y
dt 2
dy
v= = aw cos wt
d2y dt
Thus, µ (- y ) so, motion is SHM.
dt 2 (v )max = aw = 30 cm/sec (given) ...(i)
Hence, it is an equation of SHM with amplitude
dv
Acceleration (a) = = - aw 2 sin wt
A= a2 + b2 . dt
OSCILLATIONS 571

Work done = Uf – Ui = 18 × 107 J


amax = w 2 a = 60 (given) ...(ii)
Also, energy supplied per sec = 1000 J (given)
Eqs. (i) and (ii), we get
Time required to move one of the mid-point of the line
w(wa) = 60 Þ w(30) = 60 joining the other two
w = 2 rad/s
18 ´ 107
2p t= = 18 ´ 10 4 s = 50 h
= 2 rad/s 1000
T 17. (c) As, we know, in SHM
T = p sec Maximum acceleration of the particle, a = Aw2
11. (c) Displacement, x = A cos (wt) (given) a
dx Maximum velocity, b = Aw Þ w=
Velocity, v = = - Aw sin (wt) b
dt
dv 2p 2pb é 2p ù
Acceleration, a = = - Agw 2 cos ( wt) Þ T= = Qw = ú
dt w a ê
ë Tû
Hence graph (c) correctly dipicts the variation of a with t.
18. (b) y = a sin (w t + f) ; when y = a/2,
æ pt ö a
12. (b) y = 2 sin ç + f ÷ then = a sin ( w t + f )
è2 ø 2
1 p 5p
dy p æ pt ö or sin ( w t + f) = = sin or sin
velocity of particle = 2 ´ cos ç + f ÷ 2 6 6
dt 2 è 2 ø So phase of two particles is p/6 and 5 p/6 radians
Hence phase difference = (5 p/6) – p/6 = 2 p/3
d2y p2 æ pt ö
acceleration =- sinç + f ÷ 1 1
dt 2 è2 ø 19. (d) K.E = k ( A2 - d 2 ) and P.E. = kd 2
2
2 2
p At mean position d = 0. At extrement positions d = A.
Thus a max =
2 20. (a) The displacement of the particle is given by:
13. (b)
14. (b) At the middle point velocity of the particle under SHM x = A sin(-2wt ) + B sin 2 wt
is maximum but acceleration is zero since displacement B
is zero. So Assertion is true. = - A sin 2wt +
(1 - cos 2wt )
2
We know that x = a sin wt ...(1)
B B
Where x is displacement and a is amplitude. = -( A sin 2wt + cos 2wt ) +
2 2
dx
Velocity = = aw cos wt This motion represents SHM with an amplitude:
dt
æp ö B2 B
= aw cos( - wt ) = aw sin ç - (- wt )÷ A2 + , and mean position .
è 2 ø 4 2
21. (b) According to energy conservation principle,
æ pö If, x1 is maximum elongation in the spring when the
= aw sinç wt + ÷ ...(2)
è 2ø particle is in its lowest extreme position. Then,
From equation (i) and (ii) it is clear that
1
p mgh = kx12 - mgx1
Velocity is ahead of displacement (x) by angle. 2
2
15. (d) 1 2
Þ kx1 - mgx1 - mgh = 0
16. (a) Initial potential energy of the system 2
1 é q2 q2 q2 ù 1 æ 3q 2 ö 2 2mg 2mg
= 4pe ê + + ú= ç ÷ or, x1 - x1 - .h = 0
0 êë a a a ûú 4pe0 èç a ø÷ k k

æ ( 0.1)2 ö 2mg éæ 2mg ö 2 2mg ù


9 ´ 10 9ç
3´ ÷ = 27 ´107 J ± êç ÷ + 4´ hú
= ç 1 ÷ k
ëêè
k ø k
ûú
è ø \ x1 =
Let charge at A is moved to mid-point O, Then final 2
potential energy of thhe system Amplitude A = X1 – X0 (elongation in spring for
equilibrium position)
1 é 2q 2 q2 ù
Uf = ê + ú mg æ 2hk ö
4pe0 êë ( a / 2 ) a úû A= ç1 + ÷
k è mg ø
1 æ q2 ö
= 5´ ç ÷ = 45 ´ 107 J
4pe0 çè a 2 ÷ø
EBD_7418
572 PHYSICS

m T1 M1 1.5 1
22. (d) T = 2 p \ T = Þ A= ´1 = m = 0.05m
K 2 M2 600 20
A = 5 cm.
M2 2M Time period of oscillation,
T2 = T1 M1 = T 1 M m1 + m 2 1.5 2p p
T = 2p = 2p = = s
T2 = T1 2 = 2 T (where T1 =T) k 600 20 10
27. (a) The displacement of a particle in S.H.M. is given by :
23. (d) As it starts from rest, we have,
y = a sin (wt + f)
x = A cos wt. At t = 0, x = A
When t = t, x = A – a and dy
velocity = = wa cos (wt + f)
when t = 2t, x = A – 3a dt
Þ A – a = A ´ cos ´ wt and ....... (i) The velocity is maximum when the particle passes
A – 3a = A ´ cos ´ 2wt ....... (ii) through the mean position i.e.
As, cos2wt = 2cos2wt – 1, æ dy ö
çè ÷ø
A - 3a æ A-a ö
2 dt max = w a
Þ = 2ç ÷ -1
A è A ø The kinetic energy at this instant is given by
A - 3a 2 A2 + 2a 2 - 4 Aa - A2 2
\ = 1 æ dy ö 1
A A2 m çè ÷ø = mw2 a2 = 8 × 10–3 joule
\ A2 – 3aA = A2 + 2a2 – 4Aa
2 2 dt max 2
\ a = 2aA Þ A = 2a
Now, A – a = A ´ cos ´ wt ..... [From (i)] 1
1 2p p or × (0.1) w2 × (0.1)2 = 8 × 10–3
Þ cos ´ wt = \ t = Þ T = 6t 2
2 T 3 Solving we getw = ± 4
24. (b)
1 Substituting the values of a, w and f in the equation of
25. (c) Time lost/gained per day = µ Dq ´ 86400 second S.H.M., we get
2
1 y = 0.1 sin (± 4t + p /4) metre.
12 = a (40 – q) ´ 86400 .... (i)
2 28. (b) Given, Amplitude A = 3 cm
1 When particle is at x = 2 cm
4 = a (q – 20) ´ 86400 ....(ii)
2 According to question, magnitude of velocity
40 – q
On dividing we get, 3 =
q – 20 = acceleration Þ w A 2 - x 2 = xw2
Þ 3q – 60 = 40 – q Þ q = 25°C
26. (a) Applying linear momentum conservation, æ 2p ö 4p 4p
(3)2 - (2)2 = 2 ç ÷ Þ 5 = Þ T=
0.5 × 3 = (1 + 0.5) v or v = 1 m/s è T ø T 5
By conservation of energy, 29. (b) Let l be the complete length of the spring.
mean position
l l l
Length when cut in ratio, 1 : 2 : 3 are , and
6 3 2
///////////////

1
Spring constant (k) µ
3m/s length ( l)
0.5kg 1 kg
Spring constant for given segments
//////////////////////////////////////////////////// k1 = 6k, k2 = 3k and k3 = 2k
When they are connected in series
1 1 1 1 1 6
After collision = + + Þ =
k ' 6k 3k 2k k ' 6k
1 1 1.5
(1 + 0.5) v 2 = kA 2 Þ A = ´v \ Force constant k' = k
2 2 k
And when they are connected in parallel
mean position k" = 6k + 3k + 2k Þ k" = 11k
k' 1
Then the ratios =
///////////////

k " 11
bt
0.5kg 1 kg -
2m
30. (c) Q A = A0 e (where, A0 = maximum amplitude)
////////////////////////////////////////////////////
According to the questions, after 5 seconds,
OSCILLATIONS 573

b(5) This equation always fits to the differential equation


-
0.9A 0 = A 0 e 2m … (i)
d2 x 2
After 10 more seconds, = -w2 x or m d x = -mw 2 x
2
dt dt
b(15)
-
…(ii) Þ F = – mw x2 …… (ii)
A = A0 e 2m
Equations (i) and (ii) give
From equations (i) and (ii)
A = 0.729 A0 -3k | x |2 = - mw 2 x
\ a = 0.729
3kx 3ka
31. (b) For a particle executing SHM Þ w= = [sin(wt)]1/ 2
At mean position; t = 0, wt = 0, y = 0, V = Vmax = aw m m
1
\ K.E. = KEmax = mw2a2 1
2 Þ wµ a Þ Tµ
T p a
At extreme position : t = , wt = , y = A, V = 36. (b)
4 2
Vmin = 0 \ K.E. = KEmin = 0 37. (b) As we know, frequency in SHM
1 1 1 k
KE = mw2(a2 – y2) = mw2a2cos2wt f= = 1012
2 2 2p m
Hence graph (b) correctly depicts kinetic energy time where m = mass of one atom
graph.
108
32. (a) t=t ´ 10-3 kg
wt
45°
t=0
(
Mass of one atom of silver, = 6.02 ´1023
)
x
1 k
´ 6.02 ´ 1023 = 1012
2p 108 ´ 10-3
p Solving we get, spring constant, K = 7.1N/m
x = a cos æç wt + ö÷ 38. (b) Phase change p in 50 oscillations.
è 4ø
Phase change 2p in 100 oscillations.
æ 2pt p ö So frequency different ~ 1 in 100.
or x = a cos ç + ÷
è 4 4ø
Exercise - 3
33. (b) From question, acceleration, a = 20 m/s 2, and
displacement, y = 5m 1. (b) Distance covered by lift is given by
|a| = w2y y = t2
\ Acceleration of lift upwards
Þ 20 = w2(5)
Þ w = 2 rad/s d2 y d g
= 2
=(2t) = 2 m / s 2 =
Time period of pendulum, dt dt 5
2p 2p l l 5
T= = = ps T ' = 2p = 2p = T.
w 2 g 6 6
g+ g
34. (b) As the particle starts from rest, i.e., from extreme position 5 5
x = A sin (wt – f) 2. (b) Let equation of simple harmonic motion is
p x = A sin ( wt + d )
At t = 0; x = A Þf=
It is given, A = 0.5 m and
2
A 2p 2 p -1 p -1
A - = A cos wt w= = s = s
5 T 4 2
4 4 At t = 0.5 s, x = 0.3m
= cos wt Þ wt = cos-1 so 0.3 = 0.5 sin (wt + d)
5 5
T æ4ö æp 1 ö 3
t= cos -1 ç ÷ Þ sin ç ´ + d ÷ =
2p è5ø è2 2 ø 5
35. (a) V(x) = k | x |3 p
Þ + d = 37º Þ d = 37º - 45º = -8º
dV(x) 4
since, F = - = -3k | x |2 …… (i) So, equation of motion is
dx
x = a sin (wt) é pt ù
x = ( 0.5 m ) sin ê - 8º ú
ë2 û
EBD_7418
574 PHYSICS

3. (b) Let the line joining AB represents axis 'r'. By the 8. (c)
conditions given 'r' coordinate of the particle at time t 9. (b) Washer contact with piston Þ N = 0
is Given Amplitude A = 7 cm = 0.07 m.
2p 2p amax = g = w2A
w= = =p
T 2 The frequency of piston
r = 2 2 cos wt ω g 1 1000 1
f< < < < 1.9 Hz.
r 2π A 2π 7 2p
x = r cos 45º = 2 cos pt
2
m 2p m
ax = -w2 x = -p2 2 cos pt 10. (c) T = 2p Þ = 2p
K w K
Fx = max = -4p2 cos pt
1 1
Equilibrium point A or, = Þ K = 625 Nm–1.
25 K
4. (d)
A O The maximum force exerted by the system on the floor
2 = Mg + Kx + mg = 4 × 10 + 625 × 1.6 × 10–2 + 1 × 10
(at time t = 0) = 40 + 10 + 10 = 60 N
Angle covered to meet m 4 p2m 4 p2 m
p 120° 11. (b) t 1 = 2p or t12 = or k1 = 2
q = 60° = rad. k1 k1 t1
3
If they cross each other at 60°
4 p2m 4p 2 m
time t then Similarly, k 2 = and (k1 + k 2 ) =
q p T
t 22 t 02
t< < T<
2p 3≥ 2p 6 4 p2m 4 p 2m 4 p2m 1 1 1
5. (b) Let T1, T2 be the time period of shorter length and \ = + or = +
longer length pendulums respectively. As per question, t 02 t12 t 22 t 02
t 22 t12
n T1 = (n – 1) T2 ; 12. (b) The change in time period compared to the undamped
oscillator increases by 8%.
0 .5 20
so n 2 p = ( n - 1) 2 p 13. (d) When the spring undergoes displacement in the
g g downward direction it completes one half oscillation
or n = (n - 1) 40 » (n - 1) 6 while it completes another half oscillation in the upward
Hence n = 6/5 » 1 direction. The total time period is

M+m m m
M T =p +p
6. (b) T = 2p and T ¢ = 2 p k 2k
k k
14. (a)
5 M M+m 15. (d) As we know, x = x0 e–bt/2m
Þ 2p = 2p k
3 k k 40b
-
From question, 8 = 10e 2m ....(i)
25 M M + m m 16 M
Þ = Þ = -
bt
9 k k M 9 Similarly, 5 = 10e 2m ....(ii)
7. (b) The equivalent situation is a series combination of two Solving eqns (i) and (ii) we get
springs of spring constants k and 2k.
If k' is the equivalent spring constant, then t @ 142 s

(k )(2 k ) 2 k 3m
k' = = Þ T = 2p
3k 3 2k
15. WAVES

17. (d) yz A sin (kx – wt) for the wave progressing along the x-
Exercise - 1 axis and for the reflected wave,
1. (a) Frequency of wave is a function of the source of waves. y ¢= A sin (kx + wt).
Therefore, it remains unchanged. But the position of the rigid wall is at x = 0.
\ For the given wave, its reflected wave
2. (d) Longitudinal waves like sound require material
y¢ = – A sin (kx + wt).
medium.
g RT
3. (c) Sound can travel longitudinally as well as transversly 18. (a) Velocity of sound =
in solids. M
When water vapour are present in air, average molecular
4. (c) Intensity = energy/sec/area = power/area
From a point source, energy spreads over the surface weight of air decreases and hence velocity increases.
of a sphere of radius r. 19. (c) The equation of progressive wave propagating in the
positive direction of X-axis is
P P 1
\ Intensity = A = µ 2 2p
4pr 2
r y = a sin ( nt - x ) or
But Intensity = (Amplitude)2 l
y = a sin (wt – kx)
2 1
\ ( Amplitude ) µ 2 20. (a) Standing waves are produced when two waves
r propagate in opposite direction
At a distance 2 r, amplitude becomes A/2 As z 1 & z 2 are propagating in +ve x-axis &
5. (d) –ve x-axis
6. (b) y = A sin 2 (kx - wt ) so, z1 + z2 will represent a standing wave.
A 21. (b) Speed of wave is given by
Þ y = [1 - cos 2(kx - wt )] v = nl
2
w v
w ' = 2w, 2pn ' = 2w, n ' =
p \ Frequency of wave n =
l
7. (b) On reflection at a denser medium, change of
phase = p (radian) Therefore, the frequency of wave varies directly with
speed of wave.
Speed of body Frequencies, nu ³ na ³ ni
8. (a) = mach number
Speed of sound Therefore speeds of waves
9. (a) Velocity of sound in a gas is vu ³ va ³ vi
dq
v = g P / r and from P = 1 r v 2r.m.s. 22. (b) 2Tsin = dm × w2 R
3 2
dq m
g v 2T = Rdq × w2 R
v r.m.s. = 3P / r \ = 2 l
vr.m.s. 3 dq/2
10. (b) The position of such a wave changes in two mw2 R 2 T T
T=
dimensional plane with time. l
T T wL
gP E \ V= = = wR 2 \ V =
11. (b) v= = m m/l 2p
r r
23. (a) In a standing wave energy of one region is always
12. (c) confined in that region. All particles cross their mean
13. (d) The displacement of the points B and F are equal in position together.
magnitude and sign. So these points are in same phase. 24. (a) Here, y1 = a sin (wt + kx + 0.57)
14. (b) According to Laplace when sound propagate in a and y2 = a cos (wt + kx)
gaseous medium the compressions and rarefactions
are formed periodically at a very high speed. Some ép ù
= a sin ê + (wt + kx) ú
heat is produced at the places of compressions and ë2 û
some heat is lost at the places of rarefactions, but Phase difference, Df = f2– f1
there is no transfer of heat between the compressions p
and rarefactions in the gaseous medium. = - 0.57
2
15. (d) 16. (d)
3.14
= - 0.57 = 1.57 – 0.57 = 1 radian
2
EBD_7418
576 PHYSICS

25. (c) y = A sin (wt – kx) 1 T 1


Particle velocity, 36. (d) T= s; t = = s
500 2 1000
dy 37. (a) Frequency (n) = 4.2 MHz = 4.2 × 106 Hz and speed of
vp = = A wcos (wt – kx)
dt sound (v) = 1.7 km/s = 1.7 × 103 m/s. Wave length of
\ vp max = A w sound in tissue
w v 1.7 ´ 10 3
wave velocity = (l ) = = = 4 ´ 10 - 4 m .
k n 4.2 ´ 10 6
w 38. (b) Standard equation
\ Aw =
k æw ö
1 2p y(x, t) = A cos ç x - wt ÷
i. e., A = But k = èV ø
k l
From any of the displacement equation
\ l = 2pA
Say y1
26. (a)
27. (d) The speed of sound in liquid, w
= 0.50 p and w = 100 p
V
k 2 ´109 1 100p
v= = = ´ 106 \ = 0.5p \ V = 100 p = 200 m/s
r 8000 4 V 0.5p
1 39. (b) y(x, t) = f (x – vt)
v= ´ 103 = 500m s –1 .
2 4 ´ 10-3
28. (a) The intensity of wave, I = 2p2f 2A2rv, so I µ f 2 and y = (x, 0) =
I µ A2. 8 - x2
29. (d) For a travelling wave in the x-direction
4 ´ 1 + 1 ´ 16 4 ´ 10 -3
30. (b) If rH = 1, then r mix = ( 4 + 1) =4 y(x, t) =
8 - (x - 5t)2
r r r
v mix rH 1 1 40. (d) y(r , t ) = A sin (wt – k . r )
= = =
vH r mix 4 2
$ 2p k $ $ )
k= (cos a$i + cos b$j + cos g $
k) = (i + j + k )
v H 1224 l 3
v mix = = = 612 m / s
2 2 f1 – f2 = 2 (3) = 6 rad.
-1 41. (c) 42. (b)
31. (a) Vmax = aw = 0.1´ 2p´ 300 = 60p cm s
43. (a) Frequency does not depend upon radius. As length is
doubled, fundamental frequency becomes half.
V1 T
32. (d) = 1 44. (a)
V2 T2 45. (a) u µ T
l D x 2 p (0.4)
33. (c) From Dx = Df , l = 2 p = = 0. 5 m 2p
2p Df 1 .6 p 46. (b) Time period, T = ; from given eqn.
w
v 330 w = 3.0 s–1
n= = = 660 Hz
l 0 .5 2p
34. (d) As Y = A sin (wt – kx + f) or T = = 2.09 s
3
2p 1 47. (d)
w = 2pf = =2 [Q f = ]
p p
2p 2p
k= = =1 [Q l = 2 p] 48. (a)
l 2p N N N N
\ Y = 1 sin (2t – x + f)[Q A = 1 m]
35. (b) From equation, w = 100 Total no. of nodes = 4
2p 100 49. (d) Third overtone has a frequency 7 n, which means
\ = 100 Þ n =
T 2p 7l
L= = three full loops + one half loop, which would
2p 2p 4
= 20 Þ l =
l 20 make four nodes and four antinodes.
2 p 100 6
v = ln = ´ = 5m / s 50. (a) No. of loops in longitudinal mode = = 3
20 2 p 2
WAVES 577

51. (a) Total length of string l = l1 + l2 + l3 56. (a) According to Hooke's law FR µ x
(As string is divided into three segments) [Restoring force FR = T, tension of spring]
T
1 æ 1 Tö Velocity of sound by a stretched string v =
But frequency µ çQ f =
ç ÷ m
length è 2l m ÷ø where m is the mass per unit length
v T
1 1 1 1 Hence v µ T or, =
so = + + v' T'
n n1 n 2 n 3
T' 1.5x
lA 1 nA 2 or v' = v =v = 1.22 v
= = T x
52. (c) lB 2 Þ nB 1
57. (c) Speed of a transverse wave in a stretched string,
T
A= l A = 2L n=
m
Speed of transverse wave on a stretched string does
not depend upon the frequency of the wave.
v v l c 3v 2 3
58. (b) 3 ´ = 4´ or = ´ =
B= 4l c 2l0 l0 4 4v 8

l B = 4L 1 T 1
59. (b) Frequency n = Q Vµ
2L m l
1 T n 2 l1 T2
53. (d) n= \ = 1 1 1
2l m n1 l 2 T1 \ l1 : l2 : l3 =: : = 6 : 3: 2
1 2 3
æ 44 ö w2 (2p / 0.004) 2
T1 + ç T 60. (d) T = mv 2 = m 2 = 0.04 = 6.25 N
l1 è 100 ÷ø 1 100 12 k (2p / 0.50)2
= = ´ = 2 :1 61. (d) Two waves moving in uniform string with uniform tension
é 40 ù T1 60 10
l – l
êë 1 100 1 úû shall have same speed and may be moving in opposite
directions. Hence both waves may have velocities in
54. (d) In case of closed organ pipe frequency, opposite direction. Hence statement-1 is false.
v 62. (b) Fundamental frequency,
fn = (2n + 1)
4l
v 1 T 1 T é T mù
for n = 0, f0 = 100 Hz f = = = êQ v = and m = ú
n = 1, f1 = 300 Hz 2l 2l m 2l Ar ë m lû
n = 2, f2 = 500 Hz Tl T Y Dl 1 gDl
n = 3, f3 = 700 Hz Also, Y = Þ = Þ f = ....(i)
ADl A l 2l lr
n = 4, f4 = 900 Hz
Dl
n = 5, f5 = 1100 Hz Putting the value of l, , r and g in eqn. (i) we get,
l
n = 6, f6 = 1300 Hz 2 103
Hence possible natural oscillation whose frequencies f = ´
7 3
< 1250 Hz = 6(n = 0, 1, 2, 3, 4, 5)
or, f » 178.2 Hz
55. (b) Fundamental frequency of closed organ pipe
63. (d) Load supported by sonometer wire = 4 kg
V Tension in sonometer wire = 4 g
Vc = 4l If m = mass per unit length
c
Fundamental frequency of open organ pipe then frequency u = 1 T
2l m
V 1 4g
V0 = 2l Þ 416 =
0 2l m
When length is doubled, i.e., l¢ = 2l
Second overtone frequency of open organ pipe
Let new load = L
3V As, u¢ = u
= 2l
0 1 Lg 1 4g
\ =
From question, 2l ¢ m 2l m
V 3V 1 Lg 1 4g
= Þ =
4lc 2l0 4l m 2l m
Þ l0 = 6lc = 6 × 20 = 120 cm
Þ L = 2 ´ 2 Þ L = 16 kg
EBD_7418
578 PHYSICS

64. (b) After 2 s, the each wave travels a distance = 2 × 2 = 4 m. 1


1 T
The wave shape is shown in figure. 72. (a) From formula, f = Þ µl
x m f
Thus energy is purely kinetic.
1 1 1
\ l1 : l2 : l3 = : : = f2 f3 : f1 f3 : f1 f2
f1 f 2 f3
65. (a) L0 = 60 cm n 0 = 256 Hz. [Given: f1 : f2 : f3 = 1 : 3 : 5]
1 T 1 = 15 : 5 : 3
n= \ nµ Therefore the positions of two bridges below the wire are
2L m L
15 ´ 100 15 ´100 + 5 ´100
n1 L0 L0 60 cm and cm
= Þ n1 = n 0 = 256 ´ = 1024 Hz. 15 + 5 + 3 15 + 5 + 3
n 0 L1 L1 15
i.e., 1500 cm, 2000 cm
nv v 23 23
66. (a) Given = 315 and (n + 1) = 420
2l 2l 73. (d) For producing beats, their must be small difference in
n + 1 420 frequency.
Þ = Þn =3 74. (c)
n 315
v v 1
Hence 3 ´ = 315 Þ = 105 Hz 75. (a) Persistence of hearing is s.
2l 2l 10
The lowest resonant frequency is when n = 1 76. (d) The time for which sound continues to be heard after
Therefore lowest resonant frequency = 105 Hz. the source has stopped producing sound is called
v reverberation time. Sabine formula for reverberation
67. (a) For open pipe, n = , where n0 is the fundamental
2l time of a hall is
frequency of open pipe. 0.16V
T=
v 330 11 Sas
\ l= = = where V is volume of the hall in m3 and Sas is total
2 n 2 ´ 300 20
As freq. of 1st overtone of open pipe = freq. of 1st absorption. In particular, reverberation time of a hall
overtone of closed pipe is adjusted by providing a few open windows,
v v 3l 3 11 covering the walls with absoring materials.
\ 2 2 l = 3 4 l¢ Þ l' = = ´ = 41 .25 cm
77. (d)
4 4 20
A1 + A2 A x -1 78. (c) The intensity of wave
68. (c) =x ; 2 = ; Energy µ A2 I = 2p2a2n2rv
A1 - A2 A1 x + 1
æ x - 1ö
2 \ I µ a2 µ n2
Þ ç ÷ Therefore the intensity of a wave depends upon
è x + 1ø
both, its frequency and amplitude.
vA D B 1
69. (a) = = 79. (c) The contrast will be maximum, when I1 = I2 i.e.
v B DA 2 a = b. In that event, Imin = (a – b)2 = 0, where a and b are
70. (a) T / l = constant; Tension decreases by a factor the amplitudes of interfering waves.
(8 – 1) / 8, length decreases by square root of this i.e. 0.77. 80. (b) Maximum number of beats
71. (a) For fundamental mode, = (n + 1) – (n – 1) = 2
81. (d) As number of beats/sec = diff. in frequencies has to be
1 T less than 10, therefore 0 < (n1 –n2) < 10
f=
2l m 82. (d) y = A sin (kx – kct) + A sin (kx + kct)
Taking logarithm on both sides, we get æ kx - kct + kx + kct ö æ kx - kct - kx - kct ö
= 2 A sin ç 2 ÷ × cos ç 2 ÷
æ Tö è ø è ø
log f = log æç ö÷ + log ç
1
÷ = 2 A sin (kct). cos kx.
è 2l ø è mø 2p 2p
æ1 ö 1 æTö Thus = k, \ l =
l k
= log èç ø÷ + log ç ÷
2l 2 èmø l p
The distance between adjacent nodes = =
æ ö 1
1 2 k
or log f = log ç ÷ + [log T - log m] 83. (d) Figure(a) represents a harmonic wave of frequency
è 2l ø 2
Differentiating both sides, we get 7.0 Hz, figure (b) represents a harmonic wave of
frequency 5.0 Hz. Therefore beat frequency
df 1 dT
= (as l and m are constants) vs = 7 – 5 = 2.0 Hz.
f 2 T
dT df 84. (d) Frequency of sound produced by siren, f = 800 Hz
Þ = 2´ Speed of observer, u = 2 m/s
T f
Here df = 6 Velocity of sound, v = 320 m/s
f = 600 Hz No. of beats heard per second = ?
dT 2 ´ 6
\ = = 0.02
T 600
WAVES 579

No. of extra waves received by the observer per second 92. (b) A tuning fork produces 4 beats/sec with another tuning
= + 4l fork of frequency 288 cps. From this information we
\ No. of beats/ sec can conclude that the frequency of unknown fork is
288 + 4 cps or 288 – 4 cps i.e. 292 cps or 284 cps. When
2 æ 2ö 4
= -ç- ÷ = a little wax is placed on the unknown fork, it produces
l è lø l 2 beats/sec. When a little wax is placed on the unknown
2´ 2 2´2´800 æ Vö fork, its frequency decreases and simultaneously the
= 320 = = 10 çQ l = ÷ beat frequency decreases confirming that the frequency
320 è f ø
800 of the unknown fork is 292 cps.
85. (b) From the equation y (x, t) = 2a sin kx cos wt the 93. (c)
position of nodes (where amplitude is zero) are given 94. (d) The frequency of the piano string = 512 ± 4 = 516 or
by sin kx = 0 or kx = np where n = 0, 1, 2, 3, ... 508. When the tension is increased, beat frequency
86. (d) Here, A1 = A, A2 = A, f = 120° decreases to 2, it means that frequency of the string is
The amplitude of the resultant wave is 508 as frequency of string increases with tension.
95. (b) The path difference,
A R = A12 + A 22 + 2A1A 2 cos f
Dx = 2 × 65 – 2 × 60 = 60 m.

65
= A 2 + A 2 + 2AA cos 120° For constructive interference

65

m
25 m
Dx = 10 = nl
æ 1ö P 60m 60m Q
= A2 + A2 - A2 çQ cos120° = - ÷ 10
è 2ø \ l= ; For n = 1, 2, 3, ......; l = 10, 5, 2.5....
n
= AR = A Ceiling here acts as free boundary, so do not add phase
87. (b) Equation of progressive wave is given by difference due to reflection.
Y = A sin2pf t 96. (c)
Given Y1 = 4sin500 pt and Y2 = 2sin506pt. 97. (c) The apparent frequency remains the same because
Comparing the given equations with equation of distance between the source and observer does not
progressive wave, we get change.
2f1 = 500, Þ f1 = 250 98. (c) These apparent change in frequency due to motion
2f2 = 506 Þ f2 = 253 of source and observer relative to the medium along
Beats = f2– f1 = 253 – 250 = 3 beats/sec the line of sight is called Doppler's effect.
= 3 × 60 = 180 beats/minute. 99. (a) 100. (d) 101. (d)
88. (a) 102. (c) The apparent frequency heard by the stationary
89. (b) l1 = 50 cm. l 2 = 51 cm. observer
v1 T2 273 + 20 æ v ö æ 1 ö
vµ T Þ = = Þ v2 = 319.23. f¢ = f ç =f
v2 T1 273 ÷
è v - vs ø ç vs ÷
çè 1 - ÷ø
v 2 319.23 v
n1 = = = 640 Hz. -1
l1 0.50 æ v ö
= f ç1 - s ÷
v 319.23 è vø
n2 = 2 = = 625.94 = 626 Hz.
l 2 51 ´ 10-2 æ v ö
No. of beats = n 2 - n1 = 14 Hz = f ç1 + s ÷ (By expanding binomially)
è vø
f0v 103. (c) Change in frequency has nothing to do with distance
90. (c) f 2 = v + v
0 between source and listener.
The wave which reaches wall f1 is reflected. 104. (c) Frequency is not affected by blowing of wind so long
fv as source and listener are stationary.
f1 = 0
v - v0 105. (c) Let f ' be the frequency of sound heard by cliff.
The reflected frequency is f1 as the wall is at rest.
νf
fv fv 2 f vv \f¢= ......(1)
Beats = f1 - f 2 = 0 - 0 = 2 0 02 v - vc
v - v0 v + v0 v - v0
Now for the reflected wave cliff. acts as a source
91. (d) Path difference,
f ¢ (v + v c ) ......(2)
2 \2 f ¢ =
Dx = S2 P - S1P = (2 10) + 32 - 4 2 + 32 = 7 - 5 = 2m v
For constructive interference, (v + v c ) f v
Dx = nl, where n = 1, 2, 3, ... 2f = Þ 2v – 2 vc = v + vc or = v c
v - vc 3
nv 1 ´ 340 2 ´ 340 3 ´ 340
Þ f= = , , ,... 106. (c) Change in apparent frequency due to relative motion
Dx 2 2 2 between source and listener is Doppler effect.
= 170 Hz, 340 Hz, 510 Hz .....
EBD_7418
580 PHYSICS

A - (3) æ n - n0 ö n + n0
115. (b) 176 ç ÷ = 165
Intensity of sound varies with time in case of beats. è n - 22 ø n
B - (1) Here v = 330 m/s, after simplifying , we get
Sound waves in air are longitudinal in nature. v0 = 22 m/s
C - (4)
æ v + vD ö
Light waves are transverse in nature. 116. (d) n' =nç ÷
D - (2) è v – vS ø
107. (d) In doppler effect for sound wave effect due to observer Here, n = 600 Hz, n D = 15 m/s
and source motion are different. vs = 20 m/s, v = 340 m/s
108. (d) 109. (b) æ 355 ö
110. (c) Velocity of source \ v ' = 600 ç ÷ » 666 Hz
è 320 ø
vs = r w = 0.50 ´ 20 = 10 ms -1
v 340 ´ 385 117. (a) Here, n ' = 9 n
n¢ = n= = 374 Hz 8
v + vs 340 + 10
111. (c) The motorist receives two sound waves: direct one Source and observer are moving in opposite direction,
and that reflected from the wall. therefore, apparent frequency

v + vm (v + u) 9 340 + u
f n' = n ´ Þ n = n´
f¢ = (v - u) 8 340 - u
v + vb
For reflected sound waves: 340
Þ 17u = 340 ´ 1 Þ u = = 20 m/sec.
Frequency of sound wave reflected from the wall is 17
v 340 340
v 118. (a) l = = = 2 m, n ' = ´ 170
f² = ´ f n 170 340 - 17
v - vb
n ' = 178.9 Hz
vm vb
v 340
Now l ' = = = 1.9
Band master Wall n ' 178.9
Motorist
Frequency of the reflected waves as received by the Þ l - l ' = 2 - 1.9 = 0.1
moving motorist is 119 (c)
v + vm v + vm S
f²¢ = ´ f² = ´f S
v v - vb
120. (a)
Therefore, the beat frequency is vS vS
Receding train
v + vm v + vm 2v b (v + v m ) Approaching train
f ²¢ – f ¢ = ´f - ´f = f
v - vb v + vb v 2 - v 2b Frequency of sound heard by the man from
approaching train.
é v ù
112. (c) n' = n ê ú æ v ö æ 320 ö
ë v - vs û na = n ç = 240 ç = 243 Hz
÷
è v - vs ø è 320 - 4 ÷ø
é 300 ù Frequency of sound heard by the man from receding
Þ 10000 = 9500 ê ú
ë 300 - v û æ v ö æ 320 ö
train nr = n ç = 240 ç = 237 Hz
Þ 300 – v = 300 × 0.95 ÷
è v + vs ø è 320 + 4 ÷ø
Þ v = 300 – 0.05 = 15 ms–1
Hence, number of beats heard by man per sec
113. (a)
114. (a) Frequency heard by observer directly coming from = na – nr = 243 – 237 = 6
355 – 5
source = ´ 180 = 175 Hz.
355 + 5 Exercise - 2
f2 ® frequency heard by observer after reflection
1. (b) As the water waves are produced by a motor boat on
é 355 ù é 355 – 5 ù surfaces of water as well as inside the water are both
=ê ´ ´ 180 = 180 Hz
ë 355 – 5 úû êë 355 úû longitudinal and transverse, because the waves,
f2 – f1 = 5 Hz produce transverse as well as lateral vibrations in the
particles of the medium.
2. (c) Let the frequency in the first medium is v and in the
WAVES 581

second medium is v'. shape. Thus each, element of the medium is subjected
As we know that, to shearing stress. Solids and strings have shear
When waves passes from one medium to another its modulus, so sustain shearing stress.
frequency does not change but its velocity and Fluids have no shape of their own, they yield to
wavelength changes. shearing stress. Hence, transverse waves are possible
v v' in solids and strings but not in fluids.
Hence, v = v ' Þ = 7. (d)
l l'
(frequency are same for both medium) (a) The density of medium changes due to compression
and rarefraction. At compressed regions density is
æv'ö maximum and at rarefactions density is minimum.
l ' = ç ÷l
èvø (b) As density is changing very rapidly so temperature of
l and l', are wavelengths and v and v' speeds in first medium increases. Hence Boyle's law is not obeyed.
and second medium respectively. (c) Bulk modulus of air remains same or constant.
æ 2v ö (d) The time of compressions and rarefraction is very small
So, l ' = ç ÷ l = 2l
è v ø i.e., we can assume adiabatic process, i.e. no transfer
3. (c) The speed of sound (longitudinal) wave in air is of heat from surroundings.
pp 8. (b) As given that
v= . Amplitude of reflected wave
r
The density of water vapours is small (rises up) than
2 æ2 ö
Ar = ´ Ai = ç ´ 0.6 ÷ = 0.4 units
the air, so on increasing humidity the density of medium 3 è3 ø
increase the speed of sound in air. Given incident wave
For air l and p are constants, æ xö
yi = 0.6sin 2p ç t - ÷
1 è 2ø
vµ , where r is density of air..
r As we know that the reflected wave equation at denser
medium where phase difference is p
v2 r2
= æ x ö
v1 r1 yr = Ar sin 2p ç t + + p ÷
è 2 ø
where r1 is density of dry air and r2 is density of The positive sign is due to reversal of direction of
moist air. propagation
v2 æ xö
As r2 < r1 = v > 1 Þ v2 > v1 So, yr = -0.4sin 2p ç t + ÷ [Q sin(p + q) = - sin q]
1 è 2ø
Hence, speed of sound wave in air increases with 9. (b) As given that, Mass M = 2.5 kg
increase in humidity. (Mass per unit length)
4. (c) As we know that,
M 2.5 kg 125
vt = v0 (1 + 0.61t) m= = = = 0.125 kg/m
l 20 10
Speed of sound wave in a medium v µ T
T 200
(where T is temperature of the medium) Speed v = =
m 0.125
Clearly, when temperature increase then the speed also
[speed of transverse waves in any string]
increase as frequency does not change during
propogation of wave by formula. Distance l = v ´ t
As, v = nl 200
Þ 20 = ´t
where n is frequency and l is wavelength. 0.125
Frequency (n) remains fixed, 125 25 ´ 5
t = 20 ´ = 20 ´
v µ l or l µ v 2 ´ 105 2 ´105
So velocity v and wavelength (l) both increases.
1
5. (b) Only energy is transmitted from one point to another = 20 ´ 25 ´
and during propagation of any longitudinal waves in a 0.4 ´ 105
medium transmission of energy through the medium 1 20 ´ 5
= 20 ´ 5 =
without matter being transmitted. 4 ´ 10 4
2 ´102
6. (c) Mechanical transverse wave can propagates through 1
a solid medium, the constituent of the medium oscillate t= = 0.5 sec.
2
perpendicular to wave motion causing change in
EBD_7418
582 PHYSICS

10. (c) When observer is at rest and source of sound is moving 17. (a) Here, original frequency of sound, f0 = 100 Hz
towards observer then observed frequency n'. Speed of source Vs = 19.4 cos 60° = 9.7
Let the original frequency of the source is n0. From Doppler's formula 19.4
Let the speed of sound wave in the medium is v.
æ V - V0 ö
As observer is stationary f = f0 ç ÷ = 100
è V - Vs ø
v
v0 æ 9.7 ö 60°
= 100 ç 1 + ÷ = 103Hz S O
è 330 ø 19.4 cos 60° = 9.7
Apparent frequency f = 103 Hz
18. (c) In a stretched string all multiples of frequencies can be
Observer obtained i.e., if fundamental frequency is n then higher
frequencies will be 2n, 3n, 4n ...
æ v ö
Apparent frequency ( n ') = ç ÷ n0 75 cm
è v - vs ø
(when train is approaching)
æ v ö So, the difference between any two successive
=ç ÷ n0 ( n ' > n0 ) frequencies will be 'n'
è v - vs ø According to question, n = 420 – 315 = 105 Hz
When the train is moving away from the observer. So the lowest frequency of the string is 105 Hz.
19. (c) Velocity of wave v = nl
æ v ö
Apparent frequency n '' = ç ÷ n0 (n '' < n0 ) v
è v + vs ø where n = frequency of wave Þ n =
l
Hence, frequencies in both cases are same and n' > n''.
v2 396
so graph (c) verifies. n2 = = = 396 Hz
l 2 100 ´ 10-2
11. (c) According to Doppler's effect
Apparent frequency no. of beats = n1 –n2 = 4
20. (d) Two waves moving in uniform string with uniform
æ v + v0 ö æ 343 + 10 ö tension shall have same speed and may be moving in
n¢ = n ç ÷ = 1392 ç ÷ . = 1412 Hz
è v + vs ø è 343 + 5 ø opposite directions. Hence both waves may have
velocities in opposite direction. Hence Assertion is
v v incorrect.
12. (c) n 1 = 256 = =
4 l 1 4 ´ 25 . 4
é v ù 320
\ v = 256 × 101.6 cm/s 21. (d) f1 = f ê ú= f´ Hz
ë v - vs û 300
v 256 ´ 101.6
n2 = = = 254 Hz é v ù 320
4l2 4 ´ 25.6 f2 = f ê ú= f ´ Hz
ë v + vsû 340
No. of beats/sec = n 1 – n2 = 256 – 254 = 2
2p 2p æ f2 ö æ 300 ö
13. (b) w= and k = çè f - 1÷ø ´ 100 = çè - 1÷ ´ 100 ; 12%
0.01 0.30 1 340 ø
w 2p 0.30 22. (a) Let v = speed of sound and vS = speed of tuning
v= = ´ = 30 m s -1 forks. Apparent frequency of fork moving towards the
k 0.01 2p
observer is
14. (d) Reason is correct, Assertion is incorrect. In doppler
for sound wave effect due to observer and source æ v ö
n1 = ç ÷n
motion are different. è v - vs ø
15. (c) Length of pipe = 85 cm = 0.85m Apparent frequency of the fork moving away from the
Frequency of oscillations of air column in closed organ observer is
pipe is given by, æ v ö
n2 = ç ÷n
(2n - 1)u (2n - 1) ´ 340
£ 1250 è v + vs ø
f = £ 1250 Þ
4L 0.85 ´ 4 If f is the number of beats heard per second.
Þ 2n – 1 < 12.5 » 6 then f = n1 – n2
16. (a) æ v ö æ v ö
Þ f = ç v - v ÷n -ç v + v ÷n
è sø è sø
WAVES 583

v ( v + vs ) - v ( v - vs ) Given that the two frequencies are equal.


Þf= (n)
v 2 - vs2 1 TAB 1 TCD
\ =
2vvs n 2l m l m
Þ 2
= f Þ 2 æç vs ö÷ = n = f {if v << v}
v - vs2 è v ø s
TAB
Þ = TCD Þ TAB = 4TCD ....(i)
fv 4
Þ vs =
2n For rotational equilibrium of massless rod, taking
putting v = 340 m/s, f = 3, n = 340 Hz we get, torque about point O,
340 ´ 3 TAB ´ x = TCD (L - x ) ....(ii)
vs = = 1.5m / s
3 ´ 340 For translational equilibrium,
23. (b) As we know, frequency TAB + TCD = mg ....(iii)
f µ mg or f µ g mg
On solving, (i) & (iii) we get, TCD = ;
In water, f w = 0.8fair 5
g¢ r 4mg
( 0.8)2 = 0.64 Þ 1 - w = 0.64 \ TAB =
g rm 5
Substituting these values in (ii) we get
rw
Þ r = 0.36 ...(1) 4mg
´x =
mg
(L - x) Þ 4x = L - x Þ x =
L
m 5 5 5
g¢ 2 27. (a) L1 = 50 cm, L2 = 50.5 cm
In liquid, = ( 0.6 ) = 0.36
g as L2 > L1, so n2 < n1
r r For open pipe,
1 - 1 = 0.36 l = 0.64 ...(2)
rm rm v væ 1 1 ö
From eq. (1) and (2) n= Þ n1 – n2 = 3 beats/s \ 2 ç L - L ÷ = 3
2L è 1 2ø
rl 0.64
= \rl = 1.77 v æ 1 1 ö
rn 0.36 \ -
-2 çè 50 50.5 ÷ø = 6
24. (c) For a closed organ pipe first minimum resonating length 10
l 6 ´ 50 ´ 50.5 ´ 10 -2
L1 = = 50 cm \ v= = 303 m/s
4 0.5
3l
\ Next or second resonating length, L2 = = 150 cm c
4 v
S
25. (c) According to Doppler's effect in sound
28. (a) D D

Observer source 15 m/s


c S v
Largest frequency (f1) Lowest frequency (f2)
v Largest frequency will be detected when the source
Apparent frequency, n' = v - v n 0 approaches detector along the line joining and the
s
smallest frequency will be detected when the source
=
330
( 800 ) = 330 ´ 800 = 838 Hz recedes the detector along the line joining them
330 - 15 315
The frequency of sound observer hears in the echo æ c ö
f
reflected from the cliff is 838 Hz. f1 çè c - v ÷ø c+v
= =
1 TAB f2 æ c ö c- v
26. (a) Frequency of 1st harmonic of AB = çè ÷ø f
c+v
2l m
29. (c) Laplace assumed adiabatic process during sound
1 TCD propagation.
Frequency of 2nd harmonic of CD = 30. (a) We know that velocity in string is given by
l m
T
A C v= ...(I)
m
m mass of string
l AB l CD = l where m = =
=l l length of string
2 TAB TCD m
The tension T = ´ x ´ g ...(II)
O l
B D
x L–x

m
L
EBD_7418
584 PHYSICS

v
From (1) and (2) 36. (a) As f = ; and so with increase in temperature v
l 2l
increases more than l.
dx T
= gx gRT
dt x
Also v=
M
l l
x -1/2 dx = g dt \ ò x -1/2dx - g ò dt 5 F 5 9g
0 0
37. (a) f = 2l m = 2l m
.........(i)

l 20
2 l = g´t \ t =2 =2 =2 2 5l / 2
g 10
31. (b) The fundamental frequency in 3 Mg
v f and f = 2l m
............(ii)
case (a) is f =
2l
The fundamental frequency in l From above equations, we get M = 25 kg.
l
case (b) is 38. (c) The frequency of tuning fork, f = 392 Hz.
v u (a) (b) 1
f'= = =f Also 392 = F /m ..........(i)
4(l / 2) 2l 2 ´ 50
32. (c) For a string vibrating in its nth overtone (n + 1)th
After decreasing the length by 2%, we have
harmonic)
1
///////////////////
///////////////////

æ (n + 1) px ö f ¢ = 2 ( 49 ) F / m ..........(ii)
y = 2A sin çè ÷ø cos wt
L ? From above equations,
l f ¢ = 400 Hz.
For x = ] 2A = a & n = 3;
3 \ Beats frequency = 8 Hz.
é æ 4p l ö ù 39. (b) Two successive resonance are produced at 20 cm and
y = êa sin ç . ÷ ú cos wt 73 cm of column length
ë è l 3ø û
l
4p æ 3ö \ = (73 – 20) × 10–2 m
= a sin cos wt = -a ç ÷ cos wt 2
3 è 2 ø Þ l = 2 × (73 – 20) × 10–2
Velocity of sound, v = nl
l 3a
i.e. at x = , the amplitude is = 2 × 320 [73 – 20] × 10–2 = 339.2 ms–1
3 2
33. (a) Difference in two successive frequencies of closed pipe 40. (a) For closed organ pipe, third harmonic
2v (2 N –1)V 3V
v n= = (Q N = 2)
= 260 – 220 = 40 Hz Þ = 20Hz 4l 4l
4l 4l
For open organ pipe, fundamental frequenty
Which is the fundamental frequency of system of closed
organ pipe. NV V
n= = (Q N = 1)
34. (c) As we known from Doppler's Effect 2l 2l ¢
é v + v0 ù 3V V
é 340 + 16.5 ù According to question, =
fapprent = f0 ê v - v ú = 400 ê ú 4l 2l ¢
ë sû ë 340 - 22 û 4l 2l 2 ´ 20
Þ l¢ = = = = 13.33 cm
fapprent = 448 Hz 3´ 2 3 3
vs = 22 m/s v0= 16.5 m/s 150 l
A B 41. (d) - l = (2n + 1) ....... (1)
f0= 400 Hz 100 4
35. (c) Frequency of the echo detected by the driver of the (n = 0, 1, 2, ....) v = f l ....... (2)
train is Using eq. (1) and eq. (2) & v = 340 m/s, f = 340 Hz
(According to Doppler effect in sound) 1 3 5 1
We get, l = , , \ min. l = m = 25 cm.
æ v + uö 4 4 4 4
f '=ç f
è v - u ÷ø 42. (c) R = 32 + 42 = 5 mm
where f = original frequency of source of sound
f ¢ = Apparent frequency of source because of the
relative motion between source and observer.
æ 330 + 220 ö
f '= ç 1000 = 5000 Hz
è 330 - 220 ÷ø
WAVES 585

4 mm 2. (b)
3. (b) Total length of sonometer wire, l = 110 cm = 1.1 m
4 mm R Length of wire is in ratio, 6 : 3 : 2 i.e. 60 cm, 30 cm, 20 cm.
Tension in the wire, T = 400 N
7 mm 10mm 3mm Mass per unit length, m = 0.01 kg
Minimum common frequency = ?
As we know,
Y 9.27 ´ 1010 1 T 1000
43. (a) In solids, Velocity of wave V = = Frequency, n = = Hz
r 2l m 11
2.7 ´ 103
1000 1000
v = 5.85 × 103 m/sec Similarly, n1 = Hz ; n2 = Hz
Since rod is clamped at middle fundamental wave shape 6 3
is as follow 1000
n3 = Hz
l A A 2
= L Þ l = 2L N Hence common frequency = 1000 Hz
2 l/2 4. (d) Two lowest frequencies to which tube will resonates
l = 1.2m (Q L = 60 cm = 0.6m (given) are 272 Hz and 544 Hz.
Using v = fl 5. (b) For first resonant length
v 5.85 ´103 v v
Þ f= = = 4.88 × 103 Hz ; 5 KHz n= = (in winter)
l 1.2 4l1 4 ´ 18
44. (b) l1 = 50 cm. l 2 = 51 cm. For second resonant length
v1 T2 273 + 20 3v ' 3v '
vµ T Þ = = Þ v2 = 319.23. n' = = (in summer)
v2 T1 273 4l 2 4x
v 2 319.23 v 3v'
n1 = = = 640 Hz. \ =
l1 0.50 4 ×18 4× x
v 319.23 v' v'
n2 = 2 = = 625.94 = 626 Hz. \ x = 3 ´ 18 ´ \ x = 54 ´ cm
l 2 51 ´ 10-2 v v
No. of beats = n 2 - n1 = 14 Hz v' > v because velocity of light is greater in summer as

45. (d) 1 2 3 10 compared to winter (v µ T ) \ x > 54cm

6. (c) f = 500 Hz
A 4 m/s C B
Listener
4 4 Case 1 : When source is moving towards stationary
nFirst nLast
listener; apparent frequency
Using nLast = nFirst + (N – 1)x
æ v ö
where N = Number of tuning forks in series h ' = hç ÷ = 500 æç 340 ö÷ = 506 Hz
x = beat frequency between two successive forks è v - vs ø è 336 ø
Þ 2n = n + (10 – 1) × 4 Case 2 : When source is moving away from the
stationary listener
Þ n = 36 Hz
æ v ö æ 340 ö
Exercise - 3 h" = h ç ÷ = 500 ç ÷ = 494 Hz
è v + v s ø è 344 ø
1. (a) Equation of a wave moving in positive x-axis is given In case 1 number of beats heard is 6 and in case 2
as y = A sin (wt– f) or vP =Aw cos (wt–f) number of beats heard is 18 therefore frequency of the
Here y = 5 cm, A = 10 cm, source at B = 512 Hz
\5 = 10 sin (wt – f) Þ wt– f = 30º 7. (a) y(x, t) = 0.005 cos (ax - bt) (Given)
Substituting this value in the equation of velocity we Comparing it with the standard equation of wave
get vP = 0.10 × w cos 30º
y(x, t) = a cos (kx - wt) we get
n 0.10
N ow v = n l \ v = = = 0.2 k=a and w = b
l 0.5
\ w = 2pv = 2p × 0.2 = 0.4π 2p 2p
But k = and w =
Þ vP= 0.1× 0.4p ×
3
=
3
π. l T
2 50
It has to be in positive y direction.
EBD_7418
586 PHYSICS

2p 2p
Þ = a and =b l1 T
l T Þ = 1
Given that l = 0.08 m and T = 2.0s l2 T2
2p 2p
\ a= = 25p and b = =p l2 m1 + m2
0.08 2 Þ =
8. (b) As passenger is a part of moving train, there is no l1 m2
relative motion between source and listener. Therefore, 13. (b) Frequency received by guard is
n' = n = 200 Hz vs
q2 P v0
5 q1
é v + v0 ù é v + v /15 ù v (v + v 0 cos q1 ) q1
n' = nê ú = 600 ê
n = n0 0 3 q2
9. (a) ú (v + v s cos q 2 ) R S
ë v - vs û ë v - v /10 û O 4

é16 10 ù 400 ´ 16 v = 330 m/s


= 600 ê ´ ú = » 710 Q

ë15 9 û 9 Here, v0 = vs = v/3, cos q1 = 3/5, cos q2 = 4/5.


10. (b) Effective value of velocity of source,
æ v 3ö
100 100 3 çv + ´ ÷
´ = 20 m s -1 3 5 ø æ 6 ´ 15 ö n = 18n 0
vs = cos q =
3
100 m s–1 \ n = n0 è =ç ÷ 0 = 1800 Hz
3 3 5 3 æ v 4 ö è 5 19 ø 19
çv+ ´ ÷
v q è 3 5ø
n' = n
v - vs 1.2
4 14. (d) m= = 0.6 kg / m
340 2
5
Þ n' = ´ 640 Hz f = 5 Hz, l = 2l = 4m
340 - 20 v = nl = 5 × 4 = 20 m/s
= 680 Hz T
Using v = Þ T = 202 ´ 0.6 = 240 N
11. (c) If x be the distance of epicentre from the seismograph, m
then
15. (b) Here the observer is at rest with respect to the medium
x x x x
– = 4 × 60 or 4.5 - 8 = 4 × 60 and the source is going away from the observer. The
vs vp apparent frequency heard by the observer is, therefore,
on simplifying, we get
v 340
x = 2500 km n¢ = n= × 500 Hz Þ n¢ = 486 Hz
12. (b) From figure, tension T1 = m2g v + vs 340 + 10
T2 = (m1 + m2)g The frequency received by the wall is
Rigid support
As we know v 340
n¢¢ = n= × 500 = 515 Hz
Velocity µ T So, T2 v - vs 340 - 10
lµ T The wall reflects this sound without changing the
L m1 frequency. Thus, the frequency of the reflected wave
as heard by the ground observer is 515 Hz.
T1
m2
16. ELECTRIC CHARGES AND FIELDS

Exercise - 1 = 4.5 × 10–3 N


2q q –3q
1. (b) When we say that a body is charged, we always mean 18. (b) P A B
that the body is having excess of electrons (negatively l d
charged) or is of deficient of electrons (positively charged). Let a charge 2q be placed at P, at a distance I from
2. (a) Valence electrons are outermost electrons these can A where charge q is placed, as shown in figure.
get transferred on rubbing. The charge 2q will not experience any force, when
3. (d)
force, when force of repulsion on it due to q is
4. (b) 5. (c)
balanced by force of attraction on it due to –3q at B
kq × 2q kq × 4 q 3kq 2 where AB = d
6. (b) TAB = + =
d2 (2d ) 2 d
(2q)(q ) (2q )(-3q)
k × 4 q × 2q kq × 4q 9kq 2 or =
TBC = + = 4pe0 l 2
4pe 0 (l + d )2
d2 (2 d )2 d2

1C (l + d)2 = 3l2 or 2l2 – 2ld – d2 = 0


7. (a) Number of protons = » 6 ´ 1018
1.66 ´10 -19 C 2d ± 4d 2 + 2d 2 d 3d
\ l= = ±
8. (b) 9. (c) 10. (b) 4 2 2
11. (c) The weight mg of the charge hold in air is in equillibrium l=
with net electrostatic force exerted by the four charges 19. (b) Here q1 = q2 = 3.2 × 10–7 C, r = 60 cm = 0.6 m
situated at the corners. The net electrostatic force is 1 q1q2
given by the vector sum of the individual forces exerted Electrostatic force, F =
4pe0 r 2
by the charges at the corners. This is principle of
superposition. 9 ´109 (3.2 ´10-7 ) 2
Kq1q 2 = = 2.56 × 10–3 N
12. (d) From Coulomb’s law F = i.e., F µ 1 which is (0.6) 2
r2 r2 20. (d) Let q charge is situated at the mid position of the line
correctly shown by graph (d). AB. The distance between AB is x. A and B be the
13. (b) Force on any charge due to a number of other positions of charges Q and Q respectively.
charges is the vector sum of all the forces on that
charge due to the other charges, taken one at a time. A C B
Q Q
The individual force are unaffected due to the x q x
presence of other charges. This is the principle of 2 2
superposition of charges. x
14. (b) On touching the metal knob with a positively charged x x
rod some electrons from the gold leaves get transferred Let AC = , BC =
2 2
to the rod making gold leaves positively charged and The force on A due to charge q at C,
they get separated. When a negatively charged rod is ® 1 Q.q
touched with metal knob some negative charge flows F CA = . along AC
to the gold leaves lessening the positive charge there 4pe 0 ( x / 2) 2
and the separation between the leaves decreases. The force on A due to charge Q at B
15. (a) ® 1 Q2 ®
F AB = . 2 along BA
16. (b) Electrostatic force is given by 4pe0 x
1 q1q 2 The system is in equilibrium, then two oppositely
F=
4pe 0 r 2 directed force must be equal, i.e., total force on A is
Here, charge and distance are same. So, force between equal to zero.
two protons will be same ® ® ® ®
F CA + F AB = 0 Þ F CA = - F AB
17. (b) Here, q1 = 1 × 10–7C, q2 and 2 × 10–7 C,
r = 20 cm = 20 × 10–2 m 1 4Q.q –1 Q 2 Q
. 2 = . Þq=-
4 pe 0 x 4 pe 0 x 2 4
q1 q2 9 ´109 ´1´10-7 ´ 2 ´ 10-7
F= 2 =
4pe0 r (20 ´ 10-2 )2
EBD_7418
588 PHYSICS

21. (c) Let n be the number of electrons missing. 29. (a)

1 q2
F= × 2
4pe 0 d 2 Þ q = 4pe 0 d F = ne

4pe 0 Fd 2
\ n= kq 2
e2 Fe r/2 2
22. (a) 23. (d) From figure, tan q = mg Þ y = r
24. (b) If +ve charge is displaced along x-axis, then net force mg
will always act in a direction opposite to that of kq 2
[Q F = 2 from coulomb’s law]
displacement and the test charge will always come back r
to its original position. y r' 1 r
25. (b) Force on charge q1 due to q2 is Þ r3 µ y Þ r'3 µ Þ = 1/3 Þ r' = 3
2 r 2 2
qq
F12 = k 1 22 30. (b) Here, r1 = iˆ + ˆj + kˆ
b
Force on charge q1 due to q3 is r2 = 2iˆ + 3jˆ + kˆ
qq r r r
F13 = k 1 3 \ r = r2 - r1 = (2iˆ + 3jˆ + k)
ˆ - (iˆ + ˆj + k)
ˆ = ˆi + 2jˆ
a2
r
The X - component of the | r |= (1) 2 + (2) 2 = 5
force (Fx) on
By Coulomb's law
q1 is F12 + F13 sin q
1 q1q2 9 ´109 ´ 3 ´10-6 ´ 3´10-6
q1q2 q1q2 F= =
\ Fx = k +k sin q 4pe0 r 2 ( 5)2
2
b a2
81
\ Fx µ
q2
+
q3
sin q = ´ 10-3 N Nearest answer is 16 × 10–3 N.
5
b2 a 2

s Q1Q2
26. (c) T sin q = . q ... (i) P 31. (d) Q1 + Q2 = Q … (i) and F = k … (ii)
e0 K T cos q r2
q T kQ1 (Q - Q1 )
T cos q = mg .. (ii)
F = Eq =
s
q From (i) and (ii) F =
Dividing (i) by (ii), e0K r2
sq T sin q dF Q
tan q = For F to be maximum dQ = 0 Þ Q1 = Q2 = 2
e 0 K . mg 1
\ s µ tan q mg 32. (c)
27. (d) In equilibrium, Fe = T sin q 33. (c) The charge on disc A is 10–6 mC. The charge on disc
mg = T cos q B is 10 × 10–6 mC. The total charge on both = 11 mC.
F q2 q When touched, this charge will be distributed equally
tan q = e =
mg 4p Î0 x 2 ´ mg
l i.e. 5.5 mC on each disc.
Tcos q 34. (b) Initial tension : T1 = mg
q
x/2 Tsin q Final tension : T2 cos q = mg T2
also tan q » sin = q Fe mg
T1
l x q or T2 = q
2 2
cos q q2 q1 F
x q 2q l Obviously, T2 > mg. mg
Hence, 2l = 2
Þ x3 = mg
4p Î0 x ´ mg 4p Î0 mg Here we have assumed q to be small so that F is almost
1/3 horizontal.
æ q 2l ö 1/3 -12
\ x=ç ÷ Þ x µl 35. (b) Fnet = 2|F31|cos a = 2 ´ 1 ´ 2 ´ 4 ´ 10 ´ 4 = 0.46 N
è 2p Î0 mg ø 4 pe 0 ( 0.5 )2 5
y
Charge q1= 2.0 µ C
28. (a) Surface charge density (s) = +
Surface area
0.50 m
– Q + 2Q = Q 0.30 m
-2Q a
Q = 4.0 µ C
So sinner = –2Q O +
a (F1 on Q)x
x
4pb2 a b 0.30 m
0.40 m
0.50 m
Q c +
and sOuter = +2Q q2= 2.0 µ C
4pc 2 (F1 on Q)y
ELECTRIC CHARGES AND FIELDS 589

distance between ther two forces = qaEsinq or


r r
1 q2 t = pE sin q or t = p ´ E (vector form)
36. (c) In vacuum, F = …(i)
4pe0 r 2 47. (d)
Suppose, force between the chrages is same when 48. (c) Figure shows the electric fields due to the sheets 1, 2
and 3 at point P. The direction of electric fields are
charges are r¢ distance apart in dielectric. according to the charge on the sheets (away from
1 q2 positively charge sheet and towards the negatively
\ F' = …(ii) charged sheet and perpendicular). 1
4pe0 kr '2 Z = 3a P -s
E1
From (i) and (ii), kr'2 = r2 or, r = kr ' The total electric field E2
In the given situation, force between the charges would ur ur ur ur E3 2
be E = E1 + E 2 + E 3 Z=a -2s
1 q2 4 q2 4F
F' = = = = E1 ( -k$ ) + E2 ( -k$ ) + E3 ( - k$ ) 3
4pe0 æ r rö
2 9 4pe0 r 2 9 -s
Z = –a
ç + 4 ÷ é s 2s s ù $ 2s $
è2 2ø
r r = ê + + ú (-k ) = - e k
37. (d) They will not experience any force if | FG |=| Fe | ë 2e 0 2e 0 2e 0 û 0

m2 1 q2 q 2kp kp
Þ G -2 2
=
. -
Þ = 4pe 0G 49. (b) We have Ea = and Ee = 3 ; \ Ea = 2Ee
(16 ´ 10 ) 4pe 0 (16 ´ 10 )
2 2 m 3
r r
38. (d) Here, D = 2r = 4.4 m, or = r = 2.2 m 50. (a) 51. (c)
s = 60 mC m–2 52. (c)
Charge on the sphere, q = s × 4pr2
+q
22
= 60 × 10–6
×4× × (2.2)2 = 3.7 × 10–3 C F1
7
E1
39. (b) Electric field is directly proportional to the magnitude
of charge and inversely proportional to the square of F2
the distance from the charge. Therefore charge +Q E2
produce a comparatively stronger electric field than +q –q
which get at cancelled with each other at a point closer
The electric field will be different at the location of force
to +q.
on the two charges. Therefore the two charges will be
40. (d) 41. (a) 42. (c) 43. (d) unequal. This will result in a force as well as torque.
44. (d) For distances far away from centre of dipole 53. (b)
1 2p 54. (c) Electric field near the conductor surface is given by
E axis = Ea =
4pe0 r 3 s
and it is perpendicular to surface.
1 p e0
Eequa = E e = - 3 55. (b) (i) Electrostatic field is zero inside a charged
4pe 0 r
conductor or neutral conductor.
d 1 d 1 p (ii) Electrostatic field at the surface of a charged
( Ea ) = 2p ( r -3 ) = -6 × ... (i)
dr 4pe0 dr 4pe 0 r 4 conductor must be normal to the surface at every
point.
d 1 d 1 p
( Ee ) = p ( r -3 ) = -3 ... (ii)
(iii) There is no net charge at any point inside the
dr 4pe 0 dr 4pe0 r 4 conductor and any excess charge must reside at
From equations (i) and (ii) the magnitude of change in the surface.
electric field w.r.t. distance is more in case of axis of 56. (b) The electric field around a charge propagates with the
dipole as compared to equatorial plane. speed of light away from the charge. Therefore the
45. (b) The direction of electric field A distance OP
required time = = .
at equatorial point A or B speed c
will be in opposite direction, 57. (a)
as that of direction of dipole 58. (c) The electric field will increase if positive charge is
moment. B brought in an electric field.
r P 59. (c) Here, l = 2.4 m, r = 4.6 mm = 4.6 × 10–3 m
46. (b) Given : Dipole®moment of the dipole = p and uniform q = – 4.2 × 10–7 C
electric field = E . We know that dipole moment (p) = q.a
(where q is the charge and a is dipole length). And when q
® Linear charge density, l =
a dipole p is placed in uniform electric l
® of dipole moment
field E , then Torque (t) = Either force × perpendicular
EBD_7418
590 PHYSICS

=
-4.2 ´ 10-7
2.4
= –1.75 ×10–7 C m–1 68. (b) ( æ
è ) Nö æ 1ö
t = 2.57 ´ 10-17 Cm ç 3.0 ´ 104 ÷ ç ÷
C ø è 2ø
l = 3.855 ´ 10 -13 Nm.
Electric field, E =
2pe0 r r r r
69. (d) Torque, t = p ´ E = pE sin q
-1.75 ´ 10 -7 4 = p × 2 × 105 × sin 30°
=
2 ´ 3.14 ´ 8.854 ´ 10 -12 ´ 4.6 ´ 10 -3 4
or, p = 5
= 4 ´ 10-5 Cm
= –6.7 × 105 N C–1 2 ´ 10 ´ sin 30°
q q Dipole moment, p = q × l
60. (b) E= Þ Ar = Þ q = 4pe o Ar3
2
4p Î0 r 2 p 4 ´10-5
4 pe o r q= = = 2 ´ 10-3 C = 2mC
61. (a) – eE = mg l 0.02
70. (b) Since t = pE sin q on decreasing the distance between
uur 9.1 ´ 10 -31 ´ 10 -11 the two charges, and on decreasing angle q between
E =- = -5.6 ´10 N / C the dipole and electric field, sin q decreases therefore
1.6 ´ 10 -19 torque decreases.
kQ E ´ r 2 3 ´ 106 ´ (2.5)2 71. (d) The rate of decrease of electric field is different in the
62. (b) E= ÞQ= = two cases. In case of a point charge, it decreases as
r2 k 9 ´ 109 1/r2 but in the case of electric dipole it decreases
= 2 ´ 10-3 C more rapidly, as E µ 1/r3.
63. (d) Unit positive charge at O will be repelled equally by nkQR nkQ(2R)
72. (a) E1 = 2 2 3/2
, E2 = 2
three charges at ther three corners of triangle. By (R + R ) (R + (2R)2 )3/2
symmetry, resultant E at O would be zero.
64. (a) Charge resides on the outer ++ +++ E1 5 5
++ + \= =
surface of a conductin g +
+ + E2 4 2
hollow sphere of radius R. We + + +q +q
R +
consider a spherical surface of ++ +
73. (b) Force of interaction
O S p1 p2
radius r < R. + +
1 3p1p2
+ r +
By Gauss theorem + E + F= . –q –q
+ + 4p Î0 x 4 x
+ +
++
+ + +
+ 74. (b) Let us consider a spherical shell of thickness dx and
rr 1 1
ò
2
E.ds = ´ charge enclosed or E ´ 4pr = ´0 radius x. The volume of this spherical shell = 4px 2 dx .
s e0 e0 The charge enclosed within shell
ÞE= 0 é Q.x ù R
= ê 4ú
[4px2dx]
i.e electric field inside a hollow sphere is zero. ë pR û
65. (a) Here, q = 1 C, e0 = 8.85 × 10–12 C2N–1m–2 4Q
Number of lines of force = Electric force = 4
x 3dx x dx
R
q 1 The charge enclosed in a
= = = 1.13 × 1011 sphere of radius r1 is
e 0 8.85 ´10-12
66. (a) Given : Length of the dipole (2l) = 10cm = 0.1m or l = r1 r1
4Q 4Q é x 4 ù Q 4
ò
3
0.05 m = x dx = ê ú = 4 r1
Charge on the dipole (q) = 500 mC = 500 × 10–6 C and R4 4 4
R ëê ûú0
R
0
distance of the point on the axis from the mid-point of \ The electric field at point P inside the sphere at a
the dipole (r) = 20 + 5 = 25 cm = 0.25 m. distance r1 from the centre of the sphere is
We know that the electric field intensity due to dipole é Q 4ù
r
on the given point (E) 1 êë R 4 1 úû 1 Q 2
E= = r1
1 2(q.2l )r 4p Î0 r12 4p Î0 R 4
= ´
4pe 0 (r 2 - l 2 ) 2 1 2 1 æ Eq ö 2
75. (c) x= at = ç ÷ t and x = 1 æç Eq ö÷ t ¢2 \ t ¢ = 2 t .
2(500 ´10 -6 ´ 0.1) ´ 0.25 2 2è m ø 2 è 2m ø
= 9 ´109 ´
[(0.25) 2 - (0.05) 2 ]2 76. (a) The charge density nearby A is greatest and at C, it is least.
77. (c) Char ges (q) = 2 × 10 –6 C, Distan ce (d)
= 6.25 ´ 107 N / C (k = 1 for air) = 3 cm = 3 × 10 –2 m and electric field (E)
67. (c) Intensity of electric field due to a Dipole = 2 × 105 N/C. Torque (t) = q.d.
E = (2 × 10–6) × (3 × 10–2) × (2 × 105) = 12 × 10–3 N–m .
p 1 rr
E= 3 3cos 2
q + 1 Þ Eµ 3 78. (b) w = F.s = Eqi.(ˆ -aiˆ - bj)
ˆ = - Eqa
4pe 0 r r
ELECTRIC CHARGES AND FIELDS 591

79. (b) In equilibrium, charges is not well defined at the location of the charges.
T cos q = mg ....(1) But the Gaussian surface can pass through a
q continuous charge distribution.
T sin q = qE ....(2) T 86. (a) Gauss's law is valid for any closed surface, no matter
mg q T cosq what its shape or size.
From (1), T = qE 87. (a) According to Gauss’s law total electric flux through a
cos q T sinq
1
qE mg closed surface is times the total charge inside that
From (2), T = e0
sin q surface.
r 1 q
80. (c) E= rˆ q
4pe0 r 2 Electric flux, fE =
r e0
r (1.2 - 0) iˆ + ( -1.6 - 0) ˆj 1 2e
rˆ =
r
=
2 2
= (1.2iˆ - 1.6 ˆj ) Charge on a-particle = 2e \ fE =
(1.2) + (1.6) 2
r r e0
r ( -8 ´ 10-9 ) é 1 ˆ ù 88. (c) Ñò E.dA = 0, represents charge inside close surface is
9
E = 9 ´ 10 ´ 2
´ ê (1.2i - 1.6 ˆj ) ú = -10.8iˆ + 14.4 ˆj zero. Electric field as any point on the surface may be
2 ë 2 û
81. (a) The field at O due to small element dx is zero.
1 ldx O 89. (a) Electric charge resides only on the surface of a
dE = × spherical shell. According to Gauss's theorem the
4 pe0 x 2 dx x total electric flux over a closed surface is equal to the
Hence, due to one wire, 1
¥ times the total charge enclosed by the closed
1 ldx 1 l
e0
E1 = ò
a
×
4 pe0 x 2
E1 = × towards left.
4 pe0 a surface. r
Electric field at O due to other wire, 90. r r field E decreases inside water, therefore
(d) Since electric
1 l flux f = Ε.A also decreases.
E2 = × towards left
4 pe0 a 91. (a) The flux is zero according to Gauss’ Law because it is a
1 l l open surface which enclosed a charge q.
\ Net field at O is E = 2 ´ =
4 pe0 a 2pe0 a
92. (c) f = E (ds) cos q = E (2p r 2 ) cos 0º = 2p r 2 E .
1 é 2p 2p ù 93. (a) For the curved surface, q = 90º
82. (c) E= -
4 p Î0 êê ( r - l 2 )3 ( r + l 2 )3 úú
ë û \ f = E ds cos90º = 0 .
1 3Q 94. (d) According to Gauss’ Law
= . P P
Q enclosed by closed surface
4p Î0 r 4
83. (c) The dipoles can be resolved along x and y axes as
r
ò E.ds = eo
= flux
y
shown in figure. so total flux = Q/eo
Since cube has six face, so flux coming out through
The formulae for electric p/ 2 one wall or one face is Q/6eo.
p/ 2
field at axial and equatorial
95. (c) If electric dipole, the flux coming out from positive
points of a dipole are : x
p/ 2 p/ 2 charge is equal to the flux coming in at negative
2Kp charge i.e. total charge on sphere = 0. From Gauss
E (axial) = along the dipole r r
r3 law, total flux passing through the sphere = 0.
Kp 96. (d) According to Gauss's theorem,
E (equatorial) = 3 opposite to the direction of dipole.
q é
Here Ñ
r
ò ds = 4pR ùû
2
Electric field at centre due to horizontal components of E Ñò ds =
Î0 ë
dipoles will cancel out.
Total electric field at centre due to vertical components will q / 4 pR 2
\E = [Q q / 4pR 2 = s]
Î0
Kp / 2
be E = ´ 2 in vertically downward direction. or E = s / e o
r3 Q in
\ E = - Kp 2 ˆj 97. (c) By Gauss’s theorem, f =
Î0
r3 Thus, the net flux depends only on the charge enclosed
84. (b) F = qE = mg (q = 6e = 6 × 1.6 × 10–19) by the surface. Hence, there will be no effect on the net
Density (d)
flux if the radius of the surface is doubled.
mass m m q
= or r3 =
volume 4 4 L/2
pr 3 pd
3 3 q
Putting the value of v and m (= 2E/g) and solving we L L/2
get r = 7.8 × 10–7 m 98. (c)
85. (a) Gaussian surface cannot pass through any discrete L L
charge because electric field due to a system of discrete L
EBD_7418
592 PHYSICS

The given square of side L may be considered as one


of the faces of a cube with edge L. Then given charge +q2
q will be considered to be placed at the centre of the
cube. Then according to Gauss's theorem, the q1 x
magnitude of the electric flux through the faces (six) of +q1
the cube is given by
f = q/e0 +q3
Hence, electric flux through one face of the cube for The attractive force between these charges states that
the given square will be
q1 is a negative charge (since, q2 and q3 are positive).
1 q
f' = f = Then the force acting between q 1 and charge Q
6 6e 0 (positive) is also know as attractive force and then the
99. (c) By Gauss Law net force on q1 by q2, q3 and Q are along the same
100. (a) direction as shown in the figure.
Q
101. (c) Flux going in pyramid = . +q2
2e 0
which is divided equally among all 4 faces. +Q
x
Q –q1 (x, 0)
\ Flux through one face =
8e 0
+q3
102. (d) 103. (d)
104. (c) The figure shows that the force on q1 shall increase
105. (c) Flux does not depend on the size and shape of the along the positive x-axis due to the positive charge Q.
close surface, and so, it remains same. 2. (a) If a positive point charge is brought near an isolated
pd 2 4f
106. (a) f = EA cos 0° = E ´ , \ E= . conducting sphere without touching the sphere, then
2
r r 4 pd the free electrons in the sphere are attracted towards
107. (a) f = E. A = 4iˆ.(2iˆ + 3 ˆj ) = 8 V-m the positive Charge and electric field passes through a
q
108. (d) Since f total = f A + f B + fC = , charged body. This leaves an excess of positive charge
e0 on the (right) surface of sphere due to the induction
where q is the total charge.
As shown in the figure, flux associated with the curved process.
surface B is f = fB Both type of charges are bound in the (isolated
Let us assume flux linked with the plane surfaces A and conducting) sphere and cannot escape. They,
C be therefore, reside on the surface.
fA = fC = f' Thus, the left surface of sphere has an excess of
Therefore, negative charge and the right surface of sphere has an
q 1æ q ö
= 2f '+ f B = 2f '+ f Þ f ' = ç - f÷ excess of positive charge as shown in figure.
e0 2 è e0 ø
1 attracted negative
109. (d) By Gauss's law f = (Qenclosed )
e0 charge
+
Þ Qenclosed = fe 0 = (-8 ´ 103 + 4 ´ 103 ) e 0 +
+
= -4 ´103 e0 coulomb. +q +
+
®
110. (a) E = E0 iˆ + 2E0ˆj An electric field lines start from positive charge and
®
Given, E0 = 100N / c So, E = 100iˆ + 200ˆj ends at negative charge.
Radius of circular surface = 0.02 m Also, electric field line emerges from a positive charge,
2 22 in case of single charge and ends at infinity shown in
Area = pr = ´ 0.02 ´ 0.02 figure (a).
7
-3 ˆ 2
3. (d) By Gauss’s law : The total of the electric flux out of a
= 1.25 ´10 i m [Loop is parallel to Y-Z plane] closed surface is equal to the charge enclosed devided
Now, flux (f) = EA cosq Q
by the permittivity i.e., f = .
( ) -3
= 100iˆ + 200ˆj .1.25 ´ 10 ˆi cos q° [q = 0°] e0
Thus, electric flux through a surface doesn’t depend
= 125 × 10–3 Nm2/C = 0.125 Nm2/C on the shape, size or area of a surface but it depends
EXERCISE - 2 on the number of charges enclosed by the surface. So
all the given figures have same electric flux as all of
1. (a) The force on q1 depend on the force acting between them also has same single positive charge.
q1 and q2 and q1 and q3 so that the net force acting on 4. (b) Gauss's law states that total electric flux of an enclosed
q1 by q2 and q1 by q3 is along the + x-direction, so the q
surface is given by, Ñò E.dS = , includes the sum of
force acting between q1, q2 and q1, q3 is attractive s
e 0
force as shown in figure : all charges enclosed by the surface.
ELECTRIC CHARGES AND FIELDS 593

The charges may be located anywhere inside the æ 4pk ö R ( a +3)


surface, and out side the surface. Then, the electric or E ´ 4pR = ç Î ÷ (a + 3)
2
field on the left side of equation is due to all the charges, è 0 ø
both inside and outside S.
kR( a +1)
So, E on LHS of the above equation will have a \ E1 =
contribution from all charges while q on the RHS will Î0 (a + 3)
have a contribution from q2 and q4 only. a +1
5. (c) The electric field lines, are directed away from æRö
R kç ÷
positively charged source and directed toward For r = . E = è 2 ø
negatively charged source. In electric field force are 2 2
Î0 (a + 3)
directly proportional to the electric field strength hence,
E1
higher the electric field strength greater the force and Given, E2 =
vice-versa. 8
The space between the electric field lines is increasing, a +1
æRö
from left to right so strength of electric field decreases kç ÷ 1 kR( a +1)
with the increase in the space between electric field or è2ø =
8 Î0 (a + 3)
lines. Then the force on charges also decreases from Î0 (a + 3)
left to right. 1 1
Thus, the force on charge – q is greater than force on \ =
charge + q in turn dipole will experience a force towards 2 a+1 8
left. or a = 2.
6. (a) When a positive point charge +q is placed near an Fe O
isolated conducting plane, some negative charge 14. (c) From figure tan q = ;q
mg
developes on the surface of the plane towards the q
l
charge and an equal positive charge developes on kq 2 x
opposite side of the plane. This is called induction =
process and the electric field on a isolated conducting x mg 2l
2

plane at point is directly projected in a plane or x3 µ q2 …(1) q q


perpendicular to the field and away from the plane. or x3/2 µ q …(2) x
7. (a) Consider a point on diameter away from the centre of Differentiating eq. (1) w.r.t. time
hemisphere uniformly positively charged, then the dx dq dq
electric field is perpendicular to the diameter and the 3x2 µ 2q but is constant
dt dt dt
component of electric intensity parallel to the diameter so x2(v) µ q Replace q from eq. (2)
cancel out. x2(v) µ x3/2 or v µ x–1/2
15. (a) We suppose that the cavity is filled up by a positive as
8. (d) Electric displacement vector, D = eE
well as negative volume charge of r. So the electric
As, e = e 0 K \ D = e0 K E field now produced at P is the superposition of two
9. (b) electric fields.
(i) The electric field created due to the infinitely long
10. (a) (Fnet )q = 0
solid cylinder is
Qq 4q 2
Þ k +k =0 rR
ælö
2
l2 E1 = directed towards the +Y direction
ç ÷ 4e 0
è2ø (ii) The electric field created due to the spherical
l/2 l/2 negative charge density
q Q 4q
rR
1 E2 = directed towards the –Y direction.
where k = 96e 0
4pe0
\ The net electric field is
Þ 4Qq + 4q2 = 0
Þ Q=–q é 23rR ù
1
E = E1 - E 2 = ê ú
11. (d) ë 16e0 û
6
12. (c) Field lines originate perpendicular from positive charge
16. (b) The dipole experiences a torque pE sin q tending to
and terminate perpendicular at negative charge. Further
bring itself back in the direction of field.
this system can be treated as an electric dipole.
13. (c) Using Gauss's law, we have Therefore, on being released (i.e. rotated) the dipole
R
oscillates about an axis through its centre of mass and
r r 1 1 perpendicular to the field. If I is the moment of inertia
Ñò E .dA = Î0 ò (rdv) = Î0 ò0 kr ´ 4pr dr
a 2
of the dipole about the axis of rotation, then the
equation of motion is
EBD_7418
594 PHYSICS

I.d2q/dt2 = – pE sin q 21. (a) T = PE sinq Torque experienced by the dipole in an


ur ur ur
For small amplitude sin q » q electric field, T = P ´ E
ur ur r
Thus d 2 q / dt 2 = -(pE / I).q = -w 2 q p = p cosq iˆ + psinq ĵ Þ E1 = Ei
ur ur ur
where w = ( pE / I) . T 1 = p ´ E1 = (p cos q iˆ + p sin q ĵ ) × E ( iˆ )
t k̂ = pE sinq (– k̂ ) ...(i)
This is a S.H.M., whose period of oscillation is ur
E 2 = 3 E1 ˆj
T = 2p / w = 2p ( I / pE) . ur
17. (c) Applying Gauss's law T 2 = p cos qiˆ + p sin qˆj ) ´ 3 E1 ˆj
uur uur Q Q + 2par 2 - 2pAa 2 tkˆ = 3 pE1 cos qkˆ ...(ii)
Ñò S E. ds = Î0 \ E × 4pr2 = Î0 From eqns. (i) and (ii), pE sinq = 3 pE cosq
dr tanq = 3 \ q = 60°
r= r dr 22. (a) For, r < rA , E = 0
dv
Qa Gaussiam 1 QA
Q = r4pr2 surface r = rA , E= .
. b 4 p Î0 rA2
A
ò r 4pr
2
Q= dr =2pA [r2 – a2] 1 QA
a rA < r < rB , E=
4 p Î0 r 2
1 é Q - 2pAa 2 ù
E= ê + 2pA ú 1 æ QA - QB ö
4p Î0 êë r 2
úû r = rB , E= .
4p Î0 çè rB2 ÷ø
For E to be independent of 'r'
Q 1 æ QB - QA ö
Q – 2pAa2 = 0 \ A= = - 4p Î ç rB2 ø
÷
2pa 2 0è

18. (a) Electric field intensity at the centre of the disc. These values are correctly represent in option (a).
s
E= (given) –QB
2 Î0
Electric field along the axis at any distance x from the
centre of the disc QA
rA

s æ x ö
E' = ç1 - ÷ rB
2 Î0 ç ÷
è x + R2
2
ø
From question, x = R (radius of disc)
æ ö 23. (a) Total flux through the cubical surface,
\ E ' = s ç1 - R æ ö
÷ = s ç 2R - R ÷
2 Î0 çè R + R2
2 ÷ 2Î ç
ø 0è 2R ÷ø qin é 3 + 2 + (-7) ù 2C
f= = úC = -
Î0 êë Î0 û Î0
4 24. (a) As we know, F = qE = ma
=E
14
qE 1qE 2 2hm
\ % reduction in the value of electric field Þ a= ; h= t \ t=
m 2m qE
æ 4 ö i.e., time t µ m as ‘q’ is same for electron and proton.
ç E - E ÷ ´ 100 1000
14 ø
=è = % = 70.7% Since, electron has smaller mass so it will take smaller
E 14 time.
19. (b) According to question, the net electrostatic force (FE) 25. (a) Since for an infinitely long straight uniformly charged
= gravitational force (FG) l
wire, E = on moving a distance two times the
2 2 2pe0 r
1 De Gm
FE = FG or = initial distance away from wire, the distance from wire
4pe0 d 2 d2 becomes 3r. Therefore final value of electric field
G æ 1 = k = 9 ´109 ö l E
Þ De = m ç ÷ E¢ = = .
K è 4pe0 ø 2pe 0 ( 3r ) 3
26. (d) Electric flux, f = EA cos q , where q
6.67 ´ 10-11
= 1.67 × 10–27 De » 1.436 × 10–37 C = angle between E and normal to the surface.
9 ´ 109
p
20. (d) No. of lines entering the surface = No. of lines leaving Here q = Þ f=0
the surface. 2
ELECTRIC CHARGES AND FIELDS 595

27. (b) The flux for both the charges exactly cancels the effect acting on the system is the force of gravity. It is only
of each other. this force that will determine the motion of the centre
of mass of the system. Since the masses of the balls are
EXERCISE - 3 equal, the initial position of the centre of mass is at a
height (h1 + h2)/2, and its initial velocity v is horizontal.
R1 R1 Then the centre of mass will move along a parabola
r0
ò r ´ 4pr dr = 4p ò r ´ r dr = 2pr0R12
2 2
1. (c) q1 = characterized by the following equation :
0 0 2
and q2 = –s × 4pR22 h1 + h2 æ g ö æ x ö
h= -ç ÷ç ÷ ......... (i)
Given q1 + q2 = 0 or 2pr0R12 - s ´ 4pR22 = 0 2 è2øèvø
where x is the horizontal coordinate of the centre of
R2 r0 mass, and h is its vertical coordinate. At the moment
\ = .
R1 2s the first ball touches the ground at a distance x = l, the
2. (b) The path of the particle will be as shown in figure. At height H of the centre of mass, according to expression
the point of minimum distance (D) the velocity of the (1), is
particle will be ^ to its position vector w.r.t. +Q. h1 + h2 æ g ö æ l ö .
2
1 1 kQq H= -ç ÷ ç ÷
mu 2 + 0 = mv 2 + ............. (1) 2 è 2øèvø
2 2 rmin
Since the masses of the balls are equal, the second ball
v D must be at a height H2 = 2H at this instant.
2
B ælö
u H 2 = h1 + h2 - g ç ÷
èvø
r min s –s
d
6. (c)
A A. .B .C
+Q C
Q Torque on q about Q is zero hence angular
momentum about Q will be conserved. s s s s s
Þ mvrmin = mud ............. (2) EA = - =0 EC = 0, EB = 2e + 2e = e .
2e 0 2e0 0 0 0
By eq. (2) in eq. (1)
2 7. (c)
1 1 æ ud ö kQq
mu 2 = m ç
Ñò E dS =
+ Q + Q¢
2 2 è rmin ÷ø rmin 8. (c) p
e0
where Q' is

1 2
æ d 2 ö mu 2 d the charge outside the sphere
Þ 2 mu ç1 - 2 ÷ = r {Q kQq = mu2d } r r
è rmin ø a
min
ò ò r ´ 4pr dr
2 Q
Q1¢ = dV =
Þ 2 2
rmin - 2rmin d - d = 0 R R

ær 2öræ r 2 R2 ö
2d ± 4d 2 + 4d 2 = 4pa ç ÷ = 4pa ç -
Þ rmin = = d (1 ± 2) P
2 è 2 øR è 2 2 ÷ø R
Q distance cannot be negative
= 2pa (r 2 - R 2 )
\ rmin = d (1 + 2)
3. (c) For maximum elongation charges on the blocks must Q + 2pa (r 2 - R 2 )
E p ´ 4pr 2 =
be equal to Q/2 on each block. e0
QQ Q a aR 2
1 Ep = +
-
\ 2 2 = kx Q = 2(l 0 + x) 4pe0kx . 4 pr 2 e 0 2 e 0 2 r 2 e 0
4pe0 (l 0 + x) 2
Q aR 2
4. (d) Given: Length of wire L = 20 cm E is independent of r if - = 0 Þ Q = 2pR2a
charge Q = 103e0 4pr 2 e0 2r 2 e 0
We know, electric field at the centre of the semicircular arc L
æ 2Q ö 9. (c) T0 = 2p
2K l 2K ç ÷ g
E= or, E = è pr ø é Asl = 2Q ù When the plates are charged, the net acceleration is,
r r ê pr úû
ë g' = g +a
2
4 KQ 4 KQp 4pKQ
= = = = 25 ´ 103 N / Ci$ qE æ qE ö
pr 2
pL 2 2 g' = g + çè a = ÷
L m mø
5. (a) Let us consider the two charged balls to be a single T æ g ö
1/ 2
mechanical system. The coulomb interaction between L =
\ T = p \
T0 çç g + qE ÷÷
2
the balls is internal, and hence it does not affect the qE
g+
motion of the centre of mass. The only external force m è m ø
EBD_7418
596 PHYSICS

10. (b) æ5 xö 2
r dq = r.4px 2 dx = r0 ç 4 - R ÷ .4px dx
11. (b) P = (QL ) iˆ. è ø
r r \ Total charge in the spherical region from centre to r
The angle between P and E ; qi = 90°, and q4 = 0. (r < R ) is
Now W = DK dx
r x
1 2 æ5 xö
– PE (cos 90° – cos 0°) = Iw q = ò dq = 4 pr0 ò ç - ÷ x 2 dx
2 è4 Rø
0
1 æ ML2 ö 2 12 2Q
or + PE = ç ÷ w or w = é 5 r3 1 r 4 ù
2è 2 ø
- . ú = pr0r 3 æç - ö÷
ML 5 r
r = 4 pr0ê .
Angular velocity vector, w = w( rˆ) êë 4 3 R 4 úû è3 Rø
12 2Q ( - ˆj + kˆ )
1 q
= =
6 2Q ˆ ˆ
(- j + k ) \ Electric field at r, E = . 2
ML 2 ML
4p Î0 r
3 r0 r æ 5 r ö
12. (c) Charge of the remaining part after scooped out of the 1 pr0 r æ 5 r ö
= . ç - ÷= ç - ÷
sphere 4pÎ0 r 2 è 3 R ø 4 Î0 è 3 R ø
Q 4 16. (a)
= Q- ´ pR 3 x P
4 3 3 17. (a) Flux entering the cone from side AB will ultimately
pR
3 r also pass through areas A1 and A2. So
æ
3
r3 ö Q R - r
= Q ç1 - 3 ÷ =
(3
) R
f = EA1 cos q + EA2 cos (90 – q)
è R ø R3
æ1 p ö
Now charge of the part r < ..... < x = E ç 2 Rh cos q + R2 sin q ÷
è2 2 ø
æ r3 ö
Q ç1 - 3 ÷
è R ø 4 Q = ER[h cos q + p(R/2) sin q]
= ´ p( x 2 - r 3 ) = 3 ( x3 - r 3 ) = Q1
4 3
Ñò E × dS = E Ñò dS
p ( R3 - r 3 ) R
18. (b) We know that, f = cos 45°
5
Q1 K In case of hemisphere
Now field at P = K 2 = 2 fcurved = fcircular
x x 1 E pa2
Therefore, fcurved = Epa 2 × =
Q 3 3 KQ x3 - r 3 ö
æ 2 2
( x - r ) = ç ÷
R3 x2 è R3 ø 19. (b) Field due to charge Q along x direction will be
counter balanced and nutralised but in y-direction
13. (b) From figure, q = p + a , where Q p Q p Q p
3 =- 2
sin Jˆ - 2
sin Jˆ + 2
sin3 Jˆ
4pe 0 a 10 4 pe 0 a 10 4 pe0 a 10
E 2 æ p sin q ö æ 4p Î0 r 3 ö 1 Q 3p q
tan a = =ç ÷ç ÷ = tan q + sin Jˆ + Jˆ
E1 è 4 p Î0 r3 ø è 2p cos q ø 2 4 pe 0 a 2 10 4 pe 0 a 2
1 é æ 3p pö ùˆ
æ pö 1 p 3 = ê 2Q ç sin - sin 10 ÷ø + q ú J
tan ç q - ÷ = tan = 4pe 0a 2 ë è 10 û
è 3 ø 2 3 2 E E
a 1 1 1
p æ 3ö E2 p/3 = [ 2Q ´ 0.5 + q ] Jˆ = [ Q + q ] Jˆ
q- = tan -1 ç ÷ 4pe 0 a 2 4pe 0 a 2
or 3 P
è 2 ø 20. (a) Let us go over to the inertial reference frame fixed to
the moving centre of the thread. Then the balls have
æ 3ö p/3 the same velocity v at the initial instant. The energy
or a = tan -1 ç ÷
ç 2 ÷ –q O
+q
stored initially in the system is
è ø
æ 3ö q2 2mv 2
p W1 = +
or q = + tan -1ç ÷ 4pe0 × 2l 2
3 ç 2 ÷
è ø At the moment of the closest approach, the energy of
14. (b) The change inside cylinder is, qin = Q × 100. the system is
qin 100 Q q2
Now f = = . W2 = .
Î0 Î0 4pe0 d
15. (a) Let us consider a spherical shell of radius x and Using the energy conservation law, we find that
thickness dx.
Charge on this shell 2lq 2
d=
q + 8pe0 mv 2l
2
17. ELECTROSTATIC POTENTIAL AND CAPACITANCE

15. (d) In an external electric field, the positive and negative


Exercise - 1
charges of a non-polar molecule are displaced in
1. (c) Electric potential inside a conductor is constant and it opposite directions. The displacement stops when
is equal to that on the surface of the conductor. the external force on the constituent charges of the
2. (a) Potential energy is defined only in case of conservative molecule is balanced by the restoring force (due to
forces like electrostatic force (and due to which internal fields in the molecule). The non-polar molecule
electrostatic field is a conservative field). It is not thus develops an induced dipole moment. The
defined for non-conservative forces like friction. dielectric is said to be polarised by the external field.
16. (a) Potential at B, VB is maximum
1 q
3. (a) Since V = , for a given point charge, q is VB > VC > VA
4pe 0 r
constant, therefore V depends only on r. Hence V is a As in the direction of electric field potential decreases.
function of distance. 17. (b) The potential due to dipole at any arbitrary point P is
4. (b) given by
5. (d) Surface of metallic cube is an equipotential surface. P
Therefore, electric field is normal to the surface of the
cube. r
6. (b)
7. (d) B +q q –q A
8. (a) (A) ® (2); (B) ® (1); (C) ® (4); (D) ® (3) O
9. (b) Electric field is always zero inside a conductor. 1 P cos q
V=
If there is any excess of charge on a hollow conductor 4pe 0 r2
it always resides on the outer surface of conductor. For A, q = 180°
Therefore inside a hollow conductor there is no charge
and hence charge density is zero. 1 P cos180 ° 1 P
rr \ VA = =- = a negative
p.r k pr cos q 4 pe 0 r2 4 pe 0 r 2
10. (d) V =k = quantity.
r3 r3
p cos q For B, q = 0°.
=k .
r2 1 P cos 0° 1 P
11. (b) Potential at any point inside the sphere = potential at \ VB = = = a positive quantity..
the surface of the sphere = 10V. 4pe0 r2 4pe0 r 2
Therefore as we move from A to B potential change
12. (b) Wel. = q (Vi – Vf)
from negative to positive.
or 6.4 × 10–19 = – 1.6 × 10–19 (VA – VB) 18. (a) As shown in the figure, the resultant electric fields
or VA – VB = – 4V before and after interchanging the charges will have
or VA – VC = –4 V (Q VB = VC) the same magnitude, but opposite directions.
or VC– VA = 4 V + Also, the potential will be same in both cases as it is a
q+
+
l = 0 ur uur kq + R
scalar quantity.
+
13. (a) òl = ¥ - E . d l = V0 - V¥ = -0 q q -q -q
+
+
r +
A B A B
+ O ®
+
+
E
+
9 ´ 109 ´ 1.11 ´ 10 -10
+ +
+
+
= »2V
++
+
0.5
+
non uniform charge
distribution on the ring
14. (b) WAB. = WAC + WCB ®
E
WCB should be zero, because in moving from C to B, we D C D C
always move perpendicular to field. Hence, force applied -q -q q q
by field and displacement will be at 90°. 19. (c) Electric field lines are always perpendicular to
WAC = – e (VC – VA) equipotential surface so, they cannot be in a direction
of tangent to an equipotential surface.
VC – VA = –E × AC = – 10 × 4 = –40
20. (b) Irrespective of the charges on the inner and outer
\ WAC = – e × (–40) = 40e conductors, the inner conductor is always at a higher
So WAB = 40e J = 40 e V potential as long as the charge on inner conductor is
EBD_7418
598 PHYSICS

not zero. Therefore charge flows from B to A. When \ Potential difference between the points A and B is
the whole charge of B flows to A and charge on B given by
becomes zero then A and B are at same potential. 1 Q 1 Q
21. (c) Equipotential surfaces are VA – VB = . - .
4p Î0 OA 4p Î0 OB
normal to the electric field
lines. The following figure Q æ 1 1 ö Q æ 1 1ö
= çè - ÷ø = ç - ÷
shows the equipotential 4p Î0 OA OB 4p Î0 è 2 2 ø
surfaces along with electric 10-3 ´10-6
field lines for a system of two positive charges. = ´ 0 = 0.
4p Î0
22. (b) ur r 10V
26. (c) Using dV = – E. d r 20V
Þ DV = – E Dr cos q 4E
q
a b 30º 30º
23. (c) -DV Dr
Þ E=
Dr cos q
-(20 - 10) -10
Let charge on each sphere = q Þ E= = -2
-2
when they are connected together their potential will 10 ´ 10 cos120° 10 ´10 (– sin 30°)
be equal . = 200 V / m
Now let charge on a = q1 and on b = 2q – q1 Direction of E be perpendicular to the equipotential
1 q1 1 2q - q1 surface i.e. at 120° with X-axis.
Þ Va = Vb or = 27. (b)
4peo a 4pe o b
q1 a 20
Þ = 28. (a) Here, V(x) = 2 volt
2q - q1 b x -4
1. q1 dV d æ 20 ö
We know that E = - =- ç ÷
Ea 4peo a 2 æ q1 ö b 2 dx dx è x 2 - 4 ø
= =ç ÷
Eb 1 q 2 è 2q - q1 ø a 2 40x
or, E= +
4peo b2 (x - 4) 2
2

a b2 b At x = 4 mm,
= . = =b: a 40 ´ 4 160 10
b a2 a E= + 2 2 = + 144 = + 9 volt / mm.
24. (b) E = -Ñf = -f 2[ xiˆ + yiˆ + zxˆ ] (4 - 4) r
0
Positive sign indicates that E is in +ve x-direction.
= e0Ñ.E = –2e0 f0Ñ.( xiˆ + yiˆ + zxˆ) 29. (b) As we move towards a positive charge distribution
n = –6f0 e 0 dV
25. (c) Y V increases i.e., is positive. The increase in
dr
potential is steepest when we move exactly towards
A(Ö2,Ö2) charge distribution. But E is in a direction exactly away
® from charge distribution, therefore E is in exactly
r1
opposite direction in which increase in potential is
O X dV
steepest. Hence E = - .
(0,0) ®
r2 B (2,0) dr
( )
The distance of point A 2, 2 from the origin,
30. (a) Let the side length of square be 'a' then potential at
centre O is
ur 2 2
OA = | r1 | = ( 2) + ( 2) –Q –q

= 4 = 2 units.
O
The distance of point B(2, 0) from the origin,
ur 2 2
OB = | r2 | = (2) + (0) = 2 units. 2Q 2q
1 Q
Now, potential at A, VA = . k ( -Q) k (-q ) k (2q) k (2Q)
4p Î0 (OA) V= + + + = 0 (Given)
æ a ö a a a
1 Q ç ÷
Potential at B, VB = . è 2ø 2 2 2
4p Î0 (OB)
= – Q – q + 2q + 2Q = 0 = Q + q = 0 Þ Q = – q
ELECTROSTATIC POTENTIAL AND CAPACITANCE 599

31. (b) Due to conducting sphere


At centre, electric field E = 0 40. (a) AC = BD = ( 2 )2 + ( 2 )2 =2m
Q -8
q 4 = 6 ´ 10 C
And electric potential V = -8
4p Î0 R D C q 3 = -3 ´ 10 C
32. (b) e E = mew2r
R
mew2 e
Þ ò E dr = e ò rdr r O
0

me w 2 R 2 A B
Þ V = q 1 = 2 ´ 10 -8 C q 2 = -2 ´ 10 -8 C
2e
33. (c) The potential at P due to whole disc is 2
\ DO = OB = AO = OC = = 1m
s é 2 2 2
V = ë R + r - r ùû . q
2 Î0 \ Potential at the centre O, V = k
Now potential due to quarter disc, r
é 2 ´ 10-8 -2 ´ 10-8 -3 ´ 10-8 6 ´ 10 -8 ù
V s é 2 2
V = = ë R + r - r ùû . V = kê + + + ú
4 8 Î0 ëê 1 1 1 1 ûú
34. (c) Charge on the outer sphere = q1 = 4pR2s V = k ´ 3 ´ 10 -8 = 9 ´ 10 9 ´ 3 ´ 10 -8 volt
Charge on the inner sphere = q2 = 4pr2s = 27 × 10 = 270 V
R 41. (b) In shell, q charge is uniformly distributed over its
1 q1 1 q2
v= + r surface, it behaves as a conductor. +
4p Î0 R 4p Î0 r +
q + q
1 é 4pR 2 s 4pr 2 s ù 4ps s V= potential at surface =
= ê + ú = (R + r) = (R + r) 4pe 0 R R +
4p Î0 +
ëê R r ûú 4p Î0 Î0 q + +
and inside V =
35. (a) E = 500 V/m V = 3000 V. 4pe 0 R +
V Because of this it behaves as an equipotential surface.
We know that electric field (E) = 500 =
d 42. (c) The potential energy is negative whenever there is
3000 attraction. Since a positive and negative charge attract
or d = = 6m each other therefore their energy is negative. When
500
V both the charges are separated by infinite distance,
36. (d) In equilibrium, F = q E = ( n e) = mg
d they do not attract each other and their energy is zero.
mg d 1.96 ´ 10 -15 ´ 9.8 ´ 0.02 43. (d) W = PE(cos 90° – cos 270°) = 0.
n= = =3
eV 1.6 ´ 10 -19 ´ 800 W
44. (c) Since V = , more work will be done for a positive
37. (c) q = 1µC = 1 × 10–6C, r = 4 cm = 4 ×10–2 m Q
kq 9 ´ 109 ´10-6 charge of two units as compared to positive charge of
Potential V = = = 2.25 × 105 V.
r 4 ´ 10-2 W
kq one unit, but the ratio is same. Therefore potential
Induced electric field E = – 2 Q
r
difference is same.
9 ´109 ´1´10-6 ur
= = –5.625 × 106 V/m
16 ´10-4 45. (b) In a uniform electric field E , dipole experiences a
38. (c) Volume of big drop = 1000 × volume of each small drop r r ur ur
torque t given by t = p ´ E but experiences no
4 3 4
pR = 1000 ´ pr 3 Þ R = 10r force. The potential energy of the dipole in a uniform
3 3 ur ur
ur
kq kq electric field E is U = – p . E
Q V= and V ' = ´1000
r R 46. (b) A ® (2); B ® (3); C ® (4); D ® (1)
Total charge on one small droplet is q and on the big 47. (d) On the equipotential surface, electric field is normal
drop is 1000q. to the charged surface (where potential exists) so
V ' 1000r 1000 that no work will be done.
Þ = = = 100 \ V ' = 100V
V R 10 48. (a)
r ¶v ¶v ˆ
j \ | Er |= k ( x 2 + y 2 ) = kr
49. (c) ABCDE is an equipotential surface, on equipotential
39. (b) E = iˆ + surface no work is done in shifting a charge from one
¶x ¶y
Given v = –kxy Eµr place to another.
r
ˆ
\ E = kyi + kxj ˆ
EBD_7418
600 PHYSICS

50. (c) Potential energy decreases whenever there is attraction. 72. (b) A ® (1); B ® (1); C ® (2); D ® (2)
A negative charge placed at centroid causes attraction. If V is the potential applied across the capacitor then
51. (d) Two equipotential surfaces are not necessarily parallel
V
to each other. p.d. across each capacitor will be . When
2
52. (c) B and C are at the same potential, therefore potential
difference between A and B and that between A and C (A) dielectric is inserted in capacitor B, then
is same in both the cases. Hence work done is same in (B) V1 + V2 = V
both the cases.
(C) and CV1 = kCV2
Q Q
53. (d) When 1 - 2 ; current will flow in connecting wire (D) On solving above equations, we get
R1 R2
so that energy decreases in the form of heat through æ kV ö and æ V ö.
V1=ç V2 = ç
the connecting wire. è k + 1÷ø è k + 1÷ø
54. (c) Since WA ® B = q(VB –VA)
Clearly potential of A increases and that of B decreases.
16 Initial charges on the capacitors are :
Þ VB – VA = = 4V
4 CV CV
q1 = , q2 =
WPQ = q (VQ - VP ) = (– 100 × 1.6 × 10–19) (– 4 – 10) 2 2
55. (c)
' kCV ' CV
= +2.24 × 10–16J charges : q1 = CV1 = and q2 = .
k +1 k +1
56. (c) As work is done by the field, K.E. of the body increases Charge on capacitor A will increase, and on B will decrease.
by 73. (a) The molecules of liquid will convert into induced
K.E. = W = q (VA - VB ) = 10 -8 (600 - 0) = 6 ´10 -6 J dipole, get oriented along the electric field produced
between the plates and rise due to force of attraction.
e0 A 74. (a)
57. (d) Since capacitance C = , as d decreases capacitance
d
increases. q2 q2
58. (a) 59. (d) 75. (c) C¢ = kC, and U ¢ = = . With the introduction of
2C ¢ 2 kC
60. (b) Cmedium = K × Cair
dielectric, energy of the system decreases. As charge
61. (b) Energy will be lost during transfer of charge (heating
on the capacitor remains same, and so force between
effect).
them remains same.
v 1 76. (c) As the capacitor remains connected to the battery, the
62. (b) u = ò CV dV = CV 2
0 2 potential difference provided by the battery remains
63. (b) constant.
1 q2 (8 ´ 10-18 )2 77. (c) The ball on touching plate A will
64. (c) Work done = = = 32 × 10–32 J get positively charged. It will be B
2 C 2 ´ 100 ´ 10-6 A

65. (b) 66. (b) 67. (c) repelled by A and get attracted + –
68. (a) When S is closed, there will be no shifting of negative towards B. After touching B it will + + +
+ + –
+ +
charge from plate A to B as the charge – q is held by the get negatively charged. It will + –

charge + q. Neither there will be any shifting of charge now be repelled by B and + –
get attracted towards A. + –
from B to A.
+ –
69. (d) (A) ® (3); (B) ® (1); (C) ® (4); (D) ® (2) Thus it will remain oscillating
and at the extreme position touch the plates.
æ 1 2ö 2 78. (c) Volume of 8 small drops = Volume of big drop
W.d. by battery A, = 2 çè 2 C1V1 ÷ø = 2 ´ 2 = 8 J
4
8 ´ pR3 = 4 pR3 Þ R = 2r
é 1 2ù 3 3
W.d. by battery B, = 2 ê 2 CV2 ú
ë û
As capacity is proportional to r, hence capacity
é1 4 ´ 2 ù 64 becomes 2 times.
= 2ê ´ ´ 42 ú = J
ë 2 4 + 2 û 3 79. (b) The two capacitors are in parallel so
æ 4 ´ 2ö 16 eo A
q2 = CV = ç ´4=
è 4 + 2 ÷ø 3 C= (k1 + k2)
t´2
q1 = C1V1 = 2 ´ 2 = 4 80. (a) 81. (a)
70. (b)
1 1 1 1
71. (b) There is equal and opposite charge on the plates of a 82. (a) In series = + + and charge on each
parallel plate capacitor. Therefore there is no net charge C C1 C 2 C3
on capacitor. capacitor is same.
ELECTROSTATIC POTENTIAL AND CAPACITANCE 601

C C
U=
1 Q 2 1 8 ´ 10
= ´
( )
-18 2
= 32 ´ 10 –32 J
2 C 2 100 ´ 10 -6
83. (b) C C
A B 92. (b) Area of plate = pr2 = p × (8 × 10–2)2 = 0.0201 m2
C C and d = 1mm = 1 × 10–3 m
Capacity of capacitor
C C e e A 8.85 ´ 10-12 ´ 6 ´ 0.0201
The figure sh ows two independent balanced C= 0 r = = 1.068 × 10–9 F
d 1´ 10-3
wheatstone Brides connected in parallel each having a Potential difference, V = 150 volt
capacitance C. So, Cnet = CAB = 2C.
Energy stored,
84. (b) Potential difference across the branch de is 6 V. Net
1 1
capacitance of de branch is 2.1 µF U = CV2 = × (1.068 × 10–9) × (150)2 = 1.2 × 10–5 J
So, q = CV = 2.1 × 6 µC = 12.6 µ C 2 2
93. (c) Capacitance are in series
Potential across 3 µF capacitance is 1 1 1 1 2
12.6 = + + Þc= F
V= = 4.2 volt c 2 2 2 3
3 94. (a) The equivalent circuit diagram as shown in the figure.
Potential across 2 and 5 combination in parallel is Q2 2µF C Q2 3µF
6 – 4.2 = 1.8 V
So, q' = (1.8) (5) = 9 µC A B
85. (a) Capacitance of spherical conductor = 4pe0a Q Q1 2µF Q
where a is radius of conductor.
1 1 5V
Therefore, C = ´ 1 = ´ 10 -9 The equivalent capacitance between A and B is
9 9
9 ´ 10 2µF ´ 3µF 16
-9
= 0.11´10 F = 1.1 ´ 10-10 F Ceq = + 2µF = µF
2µF + 3µF 5
86. (c) Total charge of the given circuit is
87. (d) Capacity of parallel plate capacitor
16
ee A Q= µF ´ 5V = 16µC
C= r 0 (For air er = i) 5
d
e0 A Q1 = (2µF) ´ 5V = 10µC
So, = 8 ´10-12 \ Q2 = Q – Q1 = 16 µC – 10 µC = 6 µC
d
\ Voltage between B and C is
d Q 6µC
If d ® and er ® 6 then new capacitance VBC = 2 = = 2V
2 3µF 3µF
e0 A e0 A
C' = 6 × = 12 = 12 × 8 pF = 96 pF 2´2
d/2 d 95. (c) C = + 2 = 3 mF
88. (c) Capacitance will increase but not 5 times (because 2+ 2
dielectric is not filled completely). Hence, new 96. (a) Energy required to charge the capacitor is
capacitance may be 200 mF. W = U = QV
k Î0 A e0 A 2 e 0 Ad 2
89. (b) C0 = Þ U = CV2 = .V = .V = e E2Ad
d d d2 0

4 k Î0 A é Vù.
k Î0 2 2k Î0 A 4 k Î0 A C 4 êQ E = d ú
C= + = \ = 3 d = ë û
3d 3d 3 d C0 k Î0 A 3 97. (c) The network is equivalent to A C C B
d Therefore equivalent capacitance
q = [2C series C] // [C series 2C] C
90. (c) V0 = C
C0 æ 2C × C ö 4C C C
2
= çè ÷ =
q V C0 C 0 500 20 2C + C ø 3
V= Þ = Þ = = 98. (d) Equivalent capacitance of n2 number of capacitors each
C V0 C C 75 3
of capacitance C2 in parallel = n2C2
20 Equivalent capacitance of n 1 number of capacitors each of
By definition, C = kC0 Þ k =
3 capacitances C1 in series.
91. (d) The work done is stored as the potential energy. The C
potential energy stored in a capacitor is given by Capacitance of each is C1 = 1
n1
EBD_7418
602 PHYSICS

According to question, total energy stored in both the C1V1 + C2 V2


combinations are same Vcommon =
C1 + C2
1æC ö 1 16C1
i.e., ç 1 ÷ ( 4V ) = ( n2 C2 ) V 2
2
\ C2 = CV0 + KC ´ O V
2 è n1 ø 2 n1 n2 \ Vcommon = or K = 0 - 1
C + KC V
99. (b) C1 < C2
C1 C2 1 C1V + C2 ´ 0 C1
1 107. (c) Common potential V’ = = C +C V
\ < and > C1 + C2
C1 + C2 2 C 1 + C 2 2 1 2

CC C2 C q q V C0 C0 500 20
C = 1 2 = C1 . > 1 108. (c) V0 = ; V= Þ = Þ = =
C1 + C2 C1 + C2 2 C0 C V0 C C 75 3
C2 20
Similarly, C < By definition, C = kC0 Þ k =
2 3
100. (c) Equivalent circuit is shown a2 109. (d) To get effective capacitance of 6 mF two capacitors of
C1 = Ceq
3e0 a 2 C = C a 4 mF each connected in sereies and one of 4 mF capacitor in
2
d
\ Ceq = 1 eq
d 1 parallel with them.
5d B 4mF 4mF
Two capacitors in series
A D C 1 1 1 1 1 1
\ = + = + =
a2 C C1 C 2 4 4 2
C1 = Ceq
d 1 capacitor in parallel 4mF
C
\ Ceq < C3 ∗ C < 4 ∗ 2 < 6 mF
4C
´C 110. (c) Before introducing a slab capacitance of plates
101. (b) Ceq = 3 +C C C e A
7C C1 = 0
A B 3
3
C/2 If a slab of dielectric constant K is introduced between plates
11C
= then
7 e A
Ke A
C = 0 then C1' = 0 C1'
C d 2.4
102. (a) The equivalent capacitance C1 and C1' are in series hence,
e A e A
1 1 1 1 = 11 k 0 . 0
= + + e0A 3 2.4
Ceq 2 2 2 ´ 5 10 = Slab
3 e0 A e 0 A
k +
10 3 2.4
Þ Ceq = µF 3 k = 2.4 k + 3 0.6 k = 3
11
30
Hence, the dielectric constant of slap is given by, k = =5
103. (c) When key is on the position BD, the situation is shown 6
in the figure. 111. (b) Potential of larger capacitor after the first charging is
3V 24 C
– 24 C C0 V0
V1 =
12 C
O
6V
( C + C0 )
– 12 C After second charging, potential is
When key is on the position AD, the situation is shown in 2
C0 V1 æ C0 ö
the figure. V2 = =ç V
– 24 C 3V ( C + C0 ) è C + C0 ÷ø 0
24 C
O
After nth charging, potential is
– 12 C n
æ C0 ö
6V 12 C Vn = ç V0
The charge flows to each capacitor is 36 mC, and so total è C + C0 ÷ø
charge flows through point O is 72 mC. éæ V ö 1/ n ù
( n + 1) n But Vn = V So C = C0 êç 0 ÷ - 1ú
104. (d) Ceq = C + 2C + 3C + ...... + nC = C ëè V ø û
2 e0
Q1 + Q 2 CV + 2VC 3V 112. (d) Ceq =
105. (d) VC = = = d d d
C1 + C2 KC + 2C K+2 + +
106. (a) C1 = C, V1 = V0, C2 = KC, V2 = 0 and Vcommon = V K1 K 2 K 3
We know that Here, K1 = K3 = 1, K2 = e/e0
ELECTROSTATIC POTENTIAL AND CAPACITANCE 603

113. (a) A, B, C and D are equipotential points (see fig.)


Exercise - 2
A 1. (d) As capacitor offers infinite resistance in dc-circuit.
P B O
C So, current flows through 2W resistance from left to
D right, given by
4C V 2.5V 2.5
I= = = =1A
R+r 2 + 0.5 2.5
P O = So, the potential difference across 2W resistance
4C 4C 2C V = IR = 1 × 2 = 2 volt.
114. (d) Let ‘n’ such capacitors are in series and such ‘m’ such Since, capacitor is in parallel with 2W resistance, so it
branch are in parallel. also has 2V potential difference across it.
\ 250 × n = 1000 \ n = 4 … (i) As current does not flow through capacitor branch so
8 16 ´ n no potential drop will be accross 10W resistance. The
Also ´ m = 16 Þ m = =8 … (ii) charge on capacitor
n 8 q = CV = (4 µF) × 2V = 8 µC
\ No. of capacitor = 8 ´ 4 = 32 2. (c) The direction of electric field is always perpendicular
Q to the direction of electric field and equipotential
115. (c) Capacitance of the capacitor, C = surface maintained at high electrostatic potential to
V other equipotential surface maintained at low
After inserting the dielectric, new capacitance electrostatic potential.
C¢ = K.C The positively charged particle experiences the
New potential difference electrostatic force in the direction of electric field i.e.,
V 1 Q2 from high electrostatic potential to low electrostatic
V¢ = ; ui = CV2 = (Q Q = CV) potential. Thus, the work done by the electric field on
K 2 2C the positive charge, so electrostatic potential energy
Q 2 Q 2 C2 V 2 æ ui ö of the positive charge decreases because speed of
uf = = = =ç ÷ charged particle moves in the direction of field due to
2f 2kc 2KC è k ø r
force qE.
1 ì1 ü 3. (c) The work done (in displacing a charge particle) by a
Du = uf – ui = CV2 í – 1ý
2 îk þ electric force is given by W12 = q(V2 – V1). Here initial
As the capacitor is isolated, so change will remain and final potentials are same in all three cases are
conserved p.d. between two plates of the capacitor equal (20V) and same charge is moving from A to B, so
Q V work done is (DVq) same in all three cases.
L= = 4. (c) As we know that the relation between electric field
KC K dV
intensity E and electric potential V is E = -
116. (b) 117. (b) 118. (d) dr
dV
119. (d) : Here, V = 15 × 106 V dielectric strength Electric field intensity E = 0 then =0
dr
= 5 × 107 V m–1 This imply that V = constant
Maximum electric field, E = 10% of dielectirc stength Thus, E = 0 inside the charged conducting sphere then
10 the constant electrostatic potential 100V at every where
\ E= ´ 5 × 107 = 5 × 106 V m–1 inside the sphere and it verifies the shielding effect
100
also.
V V 15 ´ 106 5. (a) Here we have to findout the shape of equipotential
As E = \r= = =3m
r E 5 ´ 106 surface, these surface are perpendicular to the field
120. (b) The capacitance of the nonlinear capacitor is lines, so there must be electric field which can not be
C = e C0 = aVC0, without charge.
where C0 is the capacitance of the capacitor without a So, the collection of charges, whose total sum is not
dielectric. The charge on the nonlinear capacitor is zero, with regard to great distance can be considered
qn = CV = aC0V2, while the charge on the normal capacitor as a point charge. The equipotentials due to point charge
is q0 = C0V. It follows from the charge conservation law are spherical in shape as electric potential due to point
qn + q0 = C0V0 charge q is given by
q
4aV0 + 1 - 1 V= Ke
that the required voltage is V = = 12V . r
2a This suggest that electric potentials due to point charge
is same for all equidistant points. The locus of these
equidistant points which are at same potential, form
spherical surface.
EBD_7418
604 PHYSICS

The lines of field from point charge are radial. So the C1 C2


equipotential surface perpendicular to field lines from As C = C
3 4
a sphere. Hence no charge will flow through 20mF
6. (c) The capacitance of parallel plate capacitor filled with
C1 C2 C'
dielectric of thickness d1 and dielectric constant K1 is
KeA
C1 = 1 o X Y ÞX Y
d1
Capacitance of parallel plate capacitor filled with
dielectric of thickness d2 and dielectric constant K2 is C3 C4 C''
K 2 eo A C1 and C2 are in series, also C3 and C4 are in series.
C2 = Hence C' = 3 mF, C'' = 3 mF
d2
Since both capacitors are in series combination, then C' and C'' are in parallel hence net capacitance
the equivalent capacitance is = C' + C'' = 3 + 3 = 6 mF
1 1 1 9. (c) Potential difference between any two points in an elec-
= +
C C1 C 2 tric field is given by,
VA
K1e 0 A K 2 e 0 A ur uur 2
dV = – E.dx Þ ò dV = - ò 30 x 2 dx
C1C2 d1 d2 K1K 2 e 0 A VO 0
or C = = Ke A K e A = ... (i)
C1 + C2 1 0 + 2 0 K 1d 2 + K 2 d1 V A - VO = -[10 x 3 ]02 = -80 J / C
d1 d2 10. (a) Potential in a region V = 6xy – y + 2yz
So multiply the numerator and denominator of equation As we know the relation between electric potential and
(i) with (d1 + d2)
ur -dV
K1K 2 e 0 A (d + d )
´ 1 2
electric field is E =
C= dx
( K1d 2 + K 2d1 ) ( d1 + d 2 )
ur æ ¶V ˆ ¶V ˆ ¶V ˆ ö
K1K 2 ( d1 + d 2 ) e0 A E = ç ¶x i + ¶y j + ¶z k ÷
= ´ ... (ii) è ø
( K1d 2 + K 2d1 ) ( d1 + d 2 ) ur
Ke 0 A E = ë (6yi + (6x - 1 + 2z) j + (2y)kˆ ùû
é ˆ ˆ
Equivalent capacitances is C = ... (iii) ®
(d1 + d 2 ) E (1,1,0) = -(6iˆ + 5jˆ + 2k)
ˆ
Comparing, (ii) and (iii), the dielectric constant of new
11. (b) Charge on each plate of each capacitor
capacitor
K K (d + d ) Q = ±CV = ±25 ´ 106 ´ 200 = ±5 ´ 10 -3 C
K= 1 2 1 2
K1d 2 + K 2 d1 Kq
r ¶V $ ¶V $ ¶V $ 12. (a) We know, V0 = = Vsurface
7. (d) E = - i- j- k R
¶x ¶y ¶z Kq
Now, Vi = (3R 2 – r 2 ) [For r < R]
3
= -[(6 - 8y)iˆ + (-8x - 8 + 6z)ˆj + (6y)k]
ˆ 2R
3
r At the centre of sphare r = 0. Here V = V 0
At (1, 1, 1), E = 2i$ + 10j$ - 6k$ 2
r 5 Kq Kq 2 2
Now, = (3R – r )
Þ (E) = 22 + 102 + 62 = 140 = 2 35 4 R 2R 3
r
\ F = qE = 2 ´ 2 35 = 4 35 R 3 Kq Kq 1 Kq Kq
R2 = ; = ; =
C1 = 6mF 2 4 R R3 4 R R4
8. (d)
R4 = 4R
C3 C5 6mF Also, R1 = 0 and R2 < (R4 – R3)
X A Y
6mF 20mF C2 13. (a) Initial potential energy of the system
C4 é q2 q2 q2 ù
1 1 æ 3q 2 ö
6mF = 4pe ê + + ú= ç ÷
Equivalent circuit 0 êë a a a ûú 4pe0 èç a ø÷
æ ( 0.1)2 ö
9ç ÷ = 27 ´107 J
6mF C2
= 9 ´ 10 3´
C1 6mF ç 1 ÷
m
6F è ø
X C5 20mF
m
6F Y Let charge at A is moved to mid-point O, Then final
potential energy of the system
C3 6mF
6mF C4 1 é 2q 2 q2 ù
Uf = ê + ú
4pe 0 ëê ( a / 2 ) a ûú
ELECTROSTATIC POTENTIAL AND CAPACITANCE 605

19. (a)
1 æ q2 ö 7
= 5´ ç ÷ = 45 ´10 J 20. (c) All the charge given to inner sphere will pass on to the
4pe0 çè a 2 ÷ø
outer one. So capacitance that of outer one is 4p Î0 b .
Work done = Uf – Ui = 18 × 107 J
21. (c) Potential and potential energy are different quantities
Also, energy supplied per sec = 1000 J (given)
and cannot be equated.
Time required to move one of the mid-point of the line
22. (b) To get a capacitance of 2 mF arrangement of capacitors
joining the other two
of capacitance 1mF as shown in figure 8 capacitors of
18 ´ 107
t= = 18 ´ 10 4 s = 50 h 1mF in parallel with four such branches in series i.e., 32
1000 such capacitors are required.
14 (d)
3µF C1 = 4µF 12µF = CP
m m m m
15. (a) 4µF

9µF
Þ
2µF
8v 8v 1 1 1 1 1
= + + + \ Ceq = 2 mF
é æ 12 ö ù Ceq 8 8 8 8
Charge on C1 is q1 = ê ç
è ÷ø ´ 8 ú ´ 4 = 24mc
ë 4 + 12 û 23. (a) When charge is given to inner cylinder, an electric field
4 will be in between the cylinders. So there is potential
The voltage across CP is VP = ×8 = 2v difference between the cylinders.
4 + 12
\ Voltage across 9mF is also 2V q
\ Charge on 9mF capacitor = 9 × 2 = 18mC 24. (d) V = 0, and so C = ® ¥.
V
\ Total charge on 4 mF and 9mF = 42mc 25. (a) Electrostatic force between the metal plates
-6
KQ 42 ´ 10 Q2
\E = = 9 × 109 × = 420 NC–1 Fplate =
r 2 30 ´ 30 2 Ae 0
16. (d) Potential at origin
For isolated capacitor Q = constant
= (V1 + V3 + V5 + .....) – (V2 + V4 + V6 + .....)
Clearly, F is independent of the distance between plates.
q é 1 1 1 ù 26. (d) For a non-uniformly charged thin circular ring with net
Þ ê - + .....¥ú
4pe 0 ë x 0 2x 0 3x 0 û zero charge, electric potential at each point on its axis
is zero. Hence electric field at each point on its axis
q é 1 1 1 ù
Þ 1 - + - .....¥ ú must be perpendicular to the axis. Therefore Assertion
4 pe 0 x 0 êë 2 3 4 û
is false and Reason is true.
q q KQ
Þ log e (1 + 1) Þ log e 2 27. (b) Potential outside the shell, Voutside =
4pe 0 x 0 4pe 0 x 0 r
( A / 2)e 0 Ae 0 Ae Ae where r is distance of point from the centre of shell
17. (d) c1 = = , c2 = K 0 , c3 = K 0
d/2 d d 2d KQ
Potential inside the shell, Vinside =
R
s
where ‘R” is radius of the shell –s
C
c1 B
c3 Kq A Kq B KqC s
K d VB = + + a A
d K c2 rb rb rc
2 b c

1 é s4pa 2 s4pb 2 s4pc 2 ù


\ c1 ´ c 2 ( 3 + K ) K Ae 0 VB = ê - + ú
c eq. = + c3 = 4p Î0 êë b b c úû
c1 + c 2 2 d ( K + 1)
sé a 2 - b2 ù
(Q C1 and C2 are in series and resultant of these VB = ê + cú
Î0êë b úû
two in parallel with C3)
28. (a) Charge on Capacitor, Qi = CV
18. (a) As the regions are of equipotentials, so Work done After inserting dielectric of dielectric constant = K
W = qDV Qf = (kC) V
DV is same in all the cases hence work - done will also Induced charges on dielectric
be same in all the cases.
EBD_7418
606 PHYSICS

Qind = Qf - Qi = KCV - CV 1 æ q1q2 q2 q3 ö


-
4p Î0 çè 0.1 0.5 ÷ø
Ufinal – Uinitial =
æ5 ö
= (K - 1)CV = ç - 1÷ ´ 90pF ´ 2V = 1.2nC 1 q3
= 4p Î [10q2 q3 - 2q2 q3 ] = 4p Î (8q2 )
è3 ø

(
0
)
rv – r 0
29. (d) Ex = – = –5x + 5y + 15z = 5 Q It V I 2
rx rx 7. (c) As, C = = Þ = = = 2 × 106 V/s
V V t C 1 ´ 10 -6
rv
E y = = –3, E z = – 15 8. (b) In oil, C becomes twice, V becomes half. Therefore,
ry E = V/d becomes half.
Now, E = E xy + E zy + E zz = 7 9. (c) q1 = C1V = 10 ´12 = 120 m C
q 2 = C 2 V = KC1 ´ V = 5 ´10 ´ 12 = 600 m C
Exercise - 3 Additional charge that flows
= q 2 - q1 = 600 - 120 = 480 m C.
1. (c) Electric potential due to charge Q placed at the centre
of the spherical shell at point P is e A
10. (b) Using the formula, C = 0
R/2 d
1 Q 1 2Q Q P Capacitance of the 1st part (K1 = 8)
V1 = =
4pe o R / 2 4pe o R R q 8e 0 A / 2 e A
C1 = = 4 0 =4×C
Electric potential due to charge q on the surface of the d d
spherical shell at any point inside the shell is é e0 A ù
1 q êQ Original capacitance = C = d = 1ú
V2 = ë û
pe
4 o R = 4 µF
\ The net electric potential at point P is Capacitance of the 2nd part
1 2Q 1 q 4 Î0 ( A / 2 ) Î0 A
V = V1 + V2 = + C2 = =2 = 2 C = 2 mF
4pe o R 4pe o R d d
They are in parallel .
dV
2. (d) V = 5 + 4x2 \ = 8x ...(1) \ Total capacitance = 4 + 2 = 6µF
dx So, capacitance will increase 6 times
Force on a charge is
11. (a) Electric field in presence of dielectric between the two
æ dV ö plates of a parallel plate capaciator is given by,
F = qE = q ç -
è dx ÷ø = q(–8x); from (1) s
E=
= –2 × 10–6 × (–8 × 0.5) = 8 × 10–6 N K e0
3. (b) When the charge is released to move freely, the work Then, charge density
done by electric field is equal to change in kinetic energy s = K e0 E
\ WEF = D KE = 2.2 × 8.85 × 10–12 × 3 × 104 » 6 × 10–7 C/m2
– q DV = D KE 12. (c) As Q = CV, (Q1)max = 10–6 × 6 × 103 = 6mC
KE = – 3 × 10–6 (1 – 5) = 12 × 10–6 J
Aq While (Q2)max = 3 × 10–6 × 4 × 103 = 12mC
1 Q However in series charge is same so maximum charge
4. (a) Here, VA = VB = . a Ö2
4pe 0 a / 2 Q on C2 will also be 6 mC (and not 12 mC) and potential
a
difference across it V2 = 6mC/3 mF = 2kV and as in
Hence, VA – VB = 0 a Ö2 series V = V1 + V2 so Vmax = 6kV + 2kV = 8kV
Work done, W = q(VA – VB) = 0 B
e0 A
5. (b) WBA = q (VA - VB ) 13. (c) Ceff = since effective capacitance between
d
é Q Q ù = q Q é1 - 1ù plates A and E is zero.
A B
=q ê - ú 4 p e 0 êë a b úû C D
ë 4 p e0 a 4 p e 0 b û
E F
6. (c) We know that potential energy of discrete system of 1 2 e0 A 2
charges is given by \ U = CV = V
2 2d 180V S
1 æ q1q2 q2 q3 q3 q1 ö
U = + +
4 p Î0 çè r12 r23 r31 ÷ø
According to question,
1 æ q1q2 q2 q3 q3 q1 ö
Uinitial = + +
4p Î0 çè 0.3 0.5 0.4 ÷ø 14. (b)
1 æ q1q2 q2 q3 q3 q1 ö
+ +
4p Î0 çè 0.3 0.4 ÷ø
Ufinal =
0.1
ELECTROSTATIC POTENTIAL AND CAPACITANCE 607

Equivalent capacitance for three capacitors Dotted circuit makes balanced wheatstone bridge, The
(C1, C2 & C3) in series is given by circuit can be simplified as follows :
1 1 1 1 C C + C3C1 + C1C2 2 µF
= + + = 2 3
Ceq. C1 C2 C3 C1C2C3
8 µF
C1C2C3
Þ Ceq. = x
C1C2 + C2 C3 + C3C1 2 µF
y 1 µF 1 µF
C (2C )(3C ) 6
Þ Ceq. = C (2C ) + (2C )(3C ) + (3C ) C = 11 C
8 µF
Þ Charge on capacitors (C1, C2 & C3) in series
8 µF
6C
= CeqV = V 2/3F 2/3F
11
Charge on capacitor C4 = C4V = 4C V
6C
Charge on C2 11 V
8 µF
6 1 3
=
Charge on C4 4CV 11 4 22
= ´ = Þ Þ
8 µF 4/3 µF 8 µF 1 µF
15. (a) Potential energy of the system
1 q1q 2 18. (b)
U = q1V1 + q 2 V2 + 19. (a) Let the level of liquid at an instant of time 't' be x. Then
4pe 0 r12 x t
dx
Now, V1 [electric potential at origin] = 0 v=– Þ dx = –vdt Þ ò dx = – v ò dt
dt
V2 [electric potential at (3m, 0)] = 4 × 105 × 3 = 12 × 105 d /3 0
Þ U = (+200) × 10–6 × 12 × 105 + 9 × 109 d d

®
(200 ´ 10-6 ) ´ (-200 ´ 10-6 ) Þx – =– vt Þ x = – vt d
× = 240 – 120 = 120 J 3 3 x

®
®
®
3 Also the capacitance can be
16. (c) Potential gradient is given by, considered as an equivalent of
DV = E.d two capacitances in series such
0.8 = Ed (max) that
DV = Ed cos q = 0.8 × cos 60 = 0.4 1 1 1
= +
Hence, maximum potential at a point on the sphere = 589.4 Ceq C1 C 2
V 1 1 1 d- x x
17. (b) 6 µF 3 µF Þ = + = +
a Ceq Îo A Îo AK Îo A Îo AK
b
2 µF 3 µF
d –x x
8 µF Î0 AK
\ Ceq = But A = 1, K = 2 and x
x Kd + x(1 - K)
y 1 µF
d
1 µF = – vt
3
Î0 ´1 ´ 2 6 Î0
\ Ceq = =
8 µF é d ù 5d + 3vt
2d + ê - vt ú (1 - 2 )
ë3 û
2 µF 6R Î0
\ Time constant t = RCeq =
5d + 3vt
8 µF 2 µF 3 µF 20. (a) Let V be the steady state potential of sphere, then
x 1 mv02
eV = mv02 Þ V =
2 2e
y 1 µF 1 µF
\ Q = CV = 4 pe 0 R ´
mv02
= –
2pe 0 R mv02 ( )
2e e
8 µF negative sign has been included because of electron.
EBD_7418
608 PHYSICS

18. CURRENT ELECTRICITY


21. (c)
Exercise - 1 22. (a) [Hint Þ Rt = Ro (1 + a t)]
1. (d) Average of the velocities of all free electrons at an 5W = R0 (1 + a × 50) and 7W = R0 (1 + a × 100)
instant is non-zero, if not current is passed through a 5 1 + 50a 2
conductor. or = or a = = 0.0133/ °C
2. (b) Motion of conduction electrons due to random 7 1 + 100a 150
collisions has no preffered direction and average to R .A
23. (b) (Hint Þ r = = Coefficient of resistivity)
zero. Drift velocity is caused due to motion of l
r 24. (b) Rt = R0 (1 + at)
conduction electrons due to applied electric field E . Initially, R0 (1 + 30a) = 10 W
3. (d) Finally, R0 (1 + at) = 11 W
4. (a) J = sE Þ Jr = E
J is current density, E is electric field 11 1 + at
\ =
so B = r = resistivity. 10 1 + 30a
5. (b) 6. (d) 7. (d) 8. (d) or, 10 + (10 × 0.002 × t) = 11 + 330 × 0.002
9. (a) The conduction electrons collides with each other 1.66
more. the specific resistance of a conductor increases or, 0.02t = 1 + 0.66 = 1.066 or t = = 83°C.
0.02
with temperature according to the reaction rT rl l
= r0eEg/kgT where r0 is the specific resistance at 0° 25. (c) R = 2
or R µ .
( p D / 4) D2
C, Eg = energy of the gap between the valence and
Rx l D y2 ly / 2 D y2 2
the conduction band, kB is the Boltzmann constant = x2 ´ = ´ =
and T, the temperature of the resistor. R y Dx ly (D y / 2)2 l y 1
10. (a) 26. (a) When temperature increases the random motion of
11. (b) Because as temperature increases, the resistivity electrons and vibration of ions increases which results
increases and hence the relaxation time decreases for in more frequent collisions of electrons with the ions.
æ 1ö Due to this the average time between the successive
conductors ç t µ ÷ collisions, denoted by t, decreases which increases r.
è r ø
12. (b) When the temperature increases, resistance increases. V
As the e.m.f. applied is the same, the current density T1
decreases the drift velocity decreases. But the rms
velocity of the electron due to thermal motion is T2
27. (a)
proportional to T . Therefore, the Thermal velocity
increases. O I
13. (a) 14. (c) The slope of V – I graph gives the resistance of a
rl conductor at a given temperature.
15. (c) R = 1 , now l2 = 2l1 From the graph, it follows that resistance of a
A1
A 2 = p(r2)2 = p (2r1)2 = 4p r12 = 4A1 conductor at temperature T1 is greater than at
temperature T2 As the resistance of a conductor is
r(2 l1 ) r l R more at higher temperature and less at lower
\ R2 = = =
4A1 2A 2 temperature, hence T1 > T2 .
\ Resistance is halved, but specific resistance remains dQ
the same. 28. (c) I= = 10t + 3
dt
16. (b)
At t = 5s, I = 10 × 5 + 3 = 53 A
17. (b) The figure is showing I – V characteristics of non
29. (b) Current density J = I/A
ohmic or non linear conductors.
18. (c) Drift speed is the average speed between two = 50 ´16 -6 / 50 ´10 -6 = 1 Am -2
successive collision. l
19. (a) Current flowing through the conductor, 30. (c) Resistance of wire (R) = r
A
I = n e v A. Hence If wire is bent in the middle then
2
4 nevd1 p(1) vd 4 ´ 1 16 l
l¢ = , A¢ = 2 A
= or 1 = = . 2
1 nevd p(2) 2 vd 2 1 1
2 l
r
l¢ rl R
20. (d) I = n A e vd or vd µ 1/ p r 2 \ New, R¢ = r = 2 = = .
A¢ 2A 4A 4
CURRENT ELECTRICITY 609

31. (b) R t = R 0 (1 + at) at t°C Rt = 3R0 rl 2 m


36. (d) R = 2
. But m = pr2 ld \ pr =
pr ld
a = 4 ´ 10 -3 / °C
rl12 d rl22 d rl32 d
3R0 = R0 (1 + 4 × 10–3 × t) \ R1 = , R2 = ; R3 =
\ 3 – 1 = 4 × 10–3t m1 m2 m3
2 l12 l2 2 l32
\ t= = 500°C R1 : R2 : R3 = : : = 125 : 45 : 3
4 ´ 10-3 m1 m2 m3
l 37. (a)
32. (d) As we know, resistance R = r . The resistance of 38. (d) I = n e A vd = 2 ´ 10 21 ´ 1.6 ´ 10 -19 ´ 10 ´ 0.25 ´ 10 -3
A
conductor l is given by 8
= 2 × 1.6 × 0.25 = = 0. 8 A
l R æ l ö 10
RI = r = ç where R = r ÷ I
3A 3 è A ø 39. (c) v d = Here, I = 5.4A, n = 8.4 × 1028, per m3
2l neA
Similarly, RII = r = 2 R A = 10–6m2, e = 1.6 × 10–19C
A 5.4
\ vd = = 0.4 mm/s
3l 3 8.4 ´10 ´1.6 ´ 10 –19 ´ 10 –6
28
and RIII = r = R
2A 2 40. (b) J = I / A = 10 Q/ tA = ne tA
-4

From this we conclude that RII > RIII > RI. Since in
parallel combination of resistances current distributes 9.6 ´ 103
in inverse ratio of resistances, therefore i2 < i3 < i1. = 6.7 ´10-4 C m -2 / s = 6.7 ´10-4 Am-2
l 41. (b) Two or more resistors are said to be in parallel, if one
33. (b) Resistance of a wire is given by R = r end of all resistors is joined together and similarly
a
If the length is increased by 10% then new the other ends joined together.
1 11 I1 R1
length l' = l + = l
10 10 I
In that case, area of cross-section of wire would
decrease by 10% A I B
\ New area of cross-section
A 9 I2 R2
A' = A - = A
10 10 Two resistors R1 and R2 connected in parallel.
1 42. (a) In series combination, current across its circuit
l components is always constant and in parallel
l' 10
\ R' = r =r combination the voltage across the circuit
A' 9 components in constant.
A
10 43. (b) Resistance of the wire of a semicircle = 12/2 = 6W
11 l For equivalent resistance between two points on any
R' = r R ' = 1.21R diameter, 6W and 6W are in parallel.
9 R 44. (a) As the ring has no resistance, the three resistances of
Thus the new resistance increases by 1.21 times. The
3R each are in parallel.
specific resistance (resistivity) remains unchanged as
it depends on the nature of the material of the wire. 1 1 1 1 1
Þ = + + = Þ R¢ = R
r l ¢ r ( l / 2) 1 R ¢ 3R 3R 3R R
34. (a) R = r l / A; R ¢ = = = R. \ between point A and B equivalent resistance
A¢ 2A 4
35. (d) The total volume remains the same before and after = R+ R = 2R.
stretching. 45. (d) Two 4W resistors are in parallel combination. Their
equivalent resistance 4W
Therefore A ´ l = A ' ´ l '
4 ´ 4 16
Here l ' = 2l = = = 2W 4W
4+4 8
A´l A´l A \ Total resistance of the
\ A' = = =
l' 2l 2 network = 2 + 1 = 3W
Percentage change in resistance 6 1W 6v
\ Current through 1W resistor = = 2A
æ l' l ö 3
rç - ÷
R f - Ri A' A ø
= ´100 = è ´100 pR
Ri l 46. (a) 6W
r
A
A B
é æ 2l A ö ù A B
= êç ´ ÷ - 1ú ´ 100 = 300% 6W
êëè 2 l ø úû
A

pR
EBD_7418
610 PHYSICS

The resistance of length 2pR is 12W. Hence the r 3


resistance of length pR is 6W. Thus two resistances of ´ r
r ær ö 3 2 3
6W can be represented as shown in fig. 2. rPR = 11ç + r÷ = = r
6´6 3 è2 ø r 3 11
= 3W + r
\ Equivalent resistance R = 3 2
12
47. (c) Length of each wire = l; Area of thick wire Hence, it is clear that rPQ is maximum.
(A1) = 3A; Area of thin wire (A2) = A and resistance of 53. (a)
l 1 54. (d) Kirchhoff's first law is based on conservation of charge
thick wire (R1) = 10 W. Resistance ( R ) = r µ (if l and Kirchhoff's second law is based on conservation
A A
is constant) of energy.
R A A 1 55. (a)
\ 1 = 2 = =
R2 A1 3 A 3 E
56. (d) I = , Internal resistance (r) is zero,
or, R2 = 3R1 = 3 × 10 = 30 W R+r
The equivalent resistance of these two resistors in E
series I= = constant.
R
= R1 + R2 = 30 + 10 = 40W. 57. (d)
48. (c) When resistances are connected in series the same 58. (a)
current flows through each resistance.
59. (b) In the given case cell is in open circuit (i = 0) so voltage
49. (d) The equivalent circuit is as shown in figure.
across the cell is equal to its e.m.f.
The resistance of arm AOD (= R + R) is in parallel to the
60. (c) Internal resistance = r, External resistance = nr.
resistance R of arm AD.
Let terminal voltage = V
Their effective resistance Er
then V = E - Ir Þ V = E -
2R ´ R 2 (n + 1)r
R1 = = R
2R ´ R 3
nE V n
The resistance of arms V= Þ =
AB, BC and CD is n +1 E n +1
61. (a) In the parallel combination,
2 8
R2 = R + R + R = R e eq e1 e 2 e
3 3 = + + .... + n
req r1 r2 rn
The resistance R1 and R2 are in parallel. The effective
resistance between A and D is 1 1 1 1
= + + .... +
2 8 req r1 r2 rn
R´ R
R1 ´ R 2 3 3 = 8 R. (Q e1= e2 = e3 = ..... = en = e and r1 = r2= r3 = ... r)
R3 = =
R1 + R 2 2 8 15 eeq e e e e
R+ R
3 3 \ r = r + r + .... + r = n r .... (i)
eq
50. (a) In figure (2) the upper and lower part of the wire between
A and B are in parallel. Therefore the equivalent e 1 1 1 n
= + + .... + = r = r/n ...(ii)
resistance of these two parts is less than the resistance req r r r r eq
of each of the upper and lower part. Since the resistance From (i) and (ii)
between A and B decreases in fig. (2) therefore the e e e
current drawn from the battery increases. eeq = n ´ req = n ´ ´ = e
req r r
51. (d)
62. (c) 63. (a)
52. (b) Resistance between P and Q
64. (b) Current in the circuit,
5
r´ r e
ær rö 6 = 5r I=
rPQ = r11 ç + ÷ = R+r
è 3 2 ø r + 5 r 11
6 Potential difference across R,
Resistance between Q and R
V = IR = æç e ö÷ R = e
r 4 è R+rø r
´ r 1+
r r 4
rQR = 11(r + ) = 2 3 = r R
2 3 r 4 11 When R = 0, V = 0
+ r
2 3 R = ¥, V = e
Resistance between P and R
CURRENT ELECTRICITY 611

2e 0 + V + IR = VD
65. (c) I =
R + R1 + R 2 R1 R2 V 4V
I Þ 0+V+ ´ R = VD Þ VD =
3R 3
4V V
\ potential different across capacitor = -V =
3 3
71. (d) The equivalent emf of the two batteries in parallel,
æer +r r ö
R e = ç 1 2 2 1 ÷ . e may be; e1 £ e £ e2.
Pot. difference across second cell = V = e - IR 2 = 0 è r1 + r2 ø
2e æ rr ö
e = .R2 = 0 Internal resistance, r = ç 1 2 ÷ . This value is smaller
R + R1 + R 2 è r1 + r2 ø
than either of r1 and r2.
R + R1 + R 2 - 2R 2 = 0 72. (b)
R + R1 - R 2 = 0 \ R = R 2 - R1 3W 3W
66. (d) Since due to wrong connection of each cell the total 6W
2W
emf reduced to 2e then for wrong connection of three
cells the total emf will reduced to (ne – 6e) whereas 3W = 3W =
the total or equivalent resistance of cell combination 3V 3V
3V
will be nr.
3
67. (a) Given : emf e = 2.1 V Þ I = = 1. 5 A
I = 0.2 A, R = 10W 2
Internal resistance r = ? 73. (b) If we apply Kirchhoff’s loop rule to the loop BCDEB in
From formula. clockwise direction the changes in potential across R3
e – Ir = V = IR and R4 are negative. Therefore i3 R3 and i3 R4 should
2.1 – 0.2r = 0.2 × 10 have negative sign. But for this clockwise direction we
2.1 – 0.2 r = 2 or 0.2 r = 0.1 are moving in a direction opposite to i2 across R2.
Current flows from higher potential to lower potential
0.1
Þ r= = 0.5 W but we are moving from lower potential to higher
0.2 potential i.e., potential is increasing. So the change in
68. (c) r = E / I = 1.5 / 3 = 0.5 ohm.
69. (c) At steady state the capacitor will be fully charged and potential is positive. Therefore i2 R2 has positive sign.
thus there will be no current in the 1W resistance. So 100 90 R + r 10
74. (c) = Þ =
the effective circuit becomes R+r R R 9
2W
0.5 10 0.5 1
I1
Þ 1+ = Þ = Þ R = 4.5 W
R 9 R 9
A B 2V
I2 2W
3W 75. (a)

6V 2.8W 1V
Net current from the 6V battery,
Q 1W P
6 6 3
I= = = = 1.5A E r - E 2 r1 2-2
æ 2 ´ 3 ö 2.8 1.2 + 2.8 2 E net = 1 2 or E net = =0
ç ÷+ 2 +1
è 2+3ø 1 r1 + r2
Between A and B, voltage is same in both resistances, 76. (a) According to Kirchhoff’s first law
At junction A, iAB = 2 + 2 = 4 A
2I1 = 3I 2 where I1 + I 2 = I = 1.5 At junction B, iAB = iBC – 1 = 3A
Þ 2I1 = 3(1.5 - I1 ) Þ I1 = 0.9A 2A
70. (c) Applying Kirchhoff’s 1A
1.3 A
law in BCDEFAB we get, B
V R A
F E C
V I 2A
I=
3R t
Let A be at 0 V. Then V C At junction C, i = iBC – 1.3 = 3 – 1.3 = 1.7 amp
potential at G is V. A D 77. (c) Kirchhoff’s junction rule states that the algebraic sum
0V G of all currents into and out of any branch point is zero :
Applying Krichhoff’s SI = 0. By convention, the sign of current entering a
I
law for AFED, we get junction is positive and current leaving a junction is
negative.
B C
2V 2R
EBD_7418
612 PHYSICS

4A + 5 A – 6A + IAB = 0, therefore IAB = – 3A. The wire l


between points A and B carries a current of 3A away 88. (a) H = I2 Rt. Here R1 = r and
from the junction. p r2
78. (d) l H1
R 2 =r That is, R1 = 4R2. Hence, =4
79. (d) Given : Number of cells, n = 5, emf of each cell = E
p ( 2r )
2 H2
Internal resistance of each cell = r
In series, current through resistance R 89. (a) According to maximum power theorem, the power in
nE 5E the circuit is maximum if the value of external resistance
I= =
nr + R 5r + R is equal to the internal resistance of battery.
In parallel, current through resistance R 90. (c)
E nE 5E 91. (a) As voltage of appliance remains constant, the amount
I¢ = = =
r r + nR r + 5R of heat produced is given by,
+R
n V2
According to question, I = I' H= t … (i)
5E 5E R
\ = Þ 5r + R = r + 5R when resistance is reduced by 20%, new resistance
5r + 5 R r + 5 R
R is R' = R – 0.2 R = 0.8 R
or R = r \ = 1
r V2
80. (c) When current flows through a conductor electrons start H' = t' … (ii)
0.8R
moving in the opposite direction of current and collide
Equating Eqs. (i) and (ii), we get
with the metal atoms or ions in the conductor.
81. (b) The rate of generation of heat, for a given potential V2 V2
t= t'
difference is, P = V2/R R 0.8R
82. (c) The resistance of the bulb is Þ t ' = 0.8t = 0.8 ´ 12 = 9.6 min
V 2 (220) 2 92. (b)
R= =
P 100 (1.5) 2
93. (d) Resistance of bulb R b = = 0.5 W
The power consumed when operated at 110 V is 4.5
(110)2 100 E E
P= = = 25 W Current drawn from battery = =
2
(220) /100 4 2.67 + 0.33 3
H 80 2 E 2E
83. (b) H = I 2 Rt or R = = = 2W Share of bulb = ´ =
3 3 9
( I 2t ) (22 ´ 10) 2
æ 2E ö
V2 \ ç ÷ ´ 0.5 = 4.5 or E = 13.5 V..
84. (b) P = Vi = è 9 ø
R 94. (b) Since, the voltage is same for the two combinations,
V 2 200 ´ 200 1
Rhot = = = 400 W therefore H µ . Hence, the combination of 39 bulbs
P 100 R
400 will glow more.
Rcold = = 40 W
10 E E
95. (c) I= =
V2 V2 R + r ( R - r )2 + 2 R r
85. (a) Power, P = ÞR =
R P I is maximum when R = r
1
i.e., R µ P = I 2 R , when I is max, P is also max.
P
i..e., the resistance of 25 W bulb is greater than 100 W Pmax = I 2max R .
bulb. 96. (d) Clearly, 2W , 4W and ( 1 + 5) W resistors are in parallel.
Heat produced by the bulbs, H = I2Rt. Hence, potential difference is same across each of them.
Since the bulbs are connected in series, current remains
same. \ I1 × 2 = I 2 × 4 = I 3 × 6
I1 2W
i.e., H µ R
i.e., The heat produced by 25 W bulb is greater and Given I1 = 3A \ I1 × 2 = I3 × 6
hence it will glow brighter. Given I1 = 3A. 4W
86. (b) As R µ V2/P or R µ 1/P, so resistance of heater is less \ I1 × 2 = I3 × 6 provides I2
than that of fan.
V2 V2 I ´ 2 3´ 2 I
87. (b) H = ´ 15 ´ 60 = ´t I3 = 1 = = 1A. I3 1W
6 6 5W
R (2/3)R
Now, the potential across the
2
or t = × 15 × 60 = 600 s = 10 m inutes. 5W resistor is
3 V = I3 × 5 = 1 × 5 = 5V.
CURRENT ELECTRICITY 613

\ the power dissipated in the 5W resistor


P R S1S 2
V 2 52 113. (b) = where S = S + S
P= = = 5watt. Q S 1 2
R 5 114. (b) The network of resistors is a balanced wheatstone
97. (d) Resistance, R = V2/P = (220)2 /P. Since P of each bridge. The equivalent current is
appliance is the same, hence R is same for all the three 30W
appliances.
98. (d) When voltage applied is same, then the amount of
heat dissipated is given by, 15W
H = V2 / R 1
Þ Hµ
2 R
V
H= t
R 5V
H 4 2 15 ´ 30 V 5
Ratio of heat dissipated = 2W = = Req = = 10 W Þ I = = = 0.5 A
H 4W 2 1 15 + 30 R 10
99. (c) P = V2 / R 115. (a) If either e.m.f. of the driver cell or potential difference
2 4 across the whole potentiometer wire is lesser than
Þ R = V / P = 4 ´ 10 /100 = 400 W. the e.m.f. of then experimental cell, then balance
S = 104 / 200 = 0.5 ´ 10 2 = 50 W. point will not obtained.
116. (d) In balance condition, since no current flows through
R/S=8
the galvanometer therefore B and D are at the same
V2 potential.
100. (c) Heat supplied = ´t
R l 1 – l2
117. (b) r= ´ RW
t t 6 8 R1 3 l2
Þ 1 = 2 Þ = Þ =
R1 R 2 R 1 R 2 R2 4 Here, l1 = 125 cm, l 2 = 100 cm, R = 2W.
101. (b) 102. (d) 103. (b) \ r = 0.5W
118. (a) We know
104. (b) 105. (c)
I G
106. (b) The deflection in galvanometer will not be changed = 1+
due to interchange of battery and the galvanometer. IS S
107. (a) In meter bridge experiment, it is assumed that the 750 13
= 1+
resistance of the L shaped plate is negligible, but 100 S
actually it is not so. The error created due to this is S Þ 2W
called end error. To remove this the resistance box 119. (b) This is a balanced wheatstone bridge condition,
and the unknown resistance must be interchanged 5 l1 5 1.6l1
and then the mean reading must be taken. = and =
R 100 - l1 R / 2 100 - 1.6l1
108. (b) 109. (c) Þ R = 15 W
110. (a) Due to increases in resistance R the current through
the wire will decrease and hence the potential 120. (a)
gradient also decreases, which results in increase in 121. (b) The working principle of meter bridge is
balancing length. So. J will shift towards B. R l
111. (d) = … (i)
S 100 - l
112. (a) Given : V = 7 V
r = 5W When S' is connected in parallel with S we obtain
equivalent resistance Seq of S and S' which is less than
P Q S. Thus if the value of denominator of L.H.S. of eq. (i)
decreases then value of denominator of R.H.S. of eq.
(i) also decreases. For this to happen the null point
shifts to the right of D.
122. (c)
5W
123. (a) 500W
A
7V
i
40 ´ 120 2V
Req = W R
40 + 120 12V

V 7
I= = = 0.2 A.
R 40 ´ 120 10 1
5+
40 + 120 12 – 2 = (500W) i Þ i = =
500 50
EBD_7418
614 PHYSICS

12 1 choice of S, R is unknown resistance :


Again, i = =
500 + R 50 R Rl1
Since, =
Þ 500 + R = 600 Þ R = 100 W S R (100 - l1 )
P l l1
or P = l ´ Q = ´ 1 = 0.25W.
= 20 R é l ù
124. (d)
Q (100 - l ) 100 - l 80 S
= 100 - l or R = S ê 1 ú
1 ë100 - l1 û
VA - VB i ´ r 0.1 ´ 10-7
125. (a) Potential gradient = = = é 2.9 ù
l 10-6
R = Sê
A ë 97.1 úû
= 10 -2 V / m So, here, R : S = 2.9 : 97.1 implies that the S is nearly 33
V ærö times to that of R. In orded to make this ratio 1 : 1 it is
126. (a) Potential gradient of wire = = ç ÷´ I
l èAø necessary to reduce the value of S nearly
1
times
where l & A are the length and cross-section of wire 33
-7
i.e., nearly 3 W,
so V = 4 ´ 10 ´ 0.5 = 25mV / meter 4. (b) The potential drop across wires of potentiometer
l 8 ´ 10-6 should be more than emfs of primary cells. Here, values
127. (a) of emfs of two cells are given as 5V and 10V, so the
Pot.Difference VA - VB potential drop along the potentiometer wire must be
128. (a) Potential gradient = = more than 10V. So battery should be of 15V and about
length of wire l
4V potential is droped by using variable resistance.
129. (a) Pot. gradient = 0.2mV/cm
l
0.2 ´ 10 - 3 5. (a) As we known that the resistance of wire is R = r
= = 2 ´ 10 - 2 V / m A
10 - 2 For maximum value of R, l must be higher and A should
be lower and it is possible only when the battery is
Emf of cell = 2×10–2×1m = 2 ´ 10 - 2 V = 0.02 V æ1ö
As per the condition of potentiometer connected across area of cross section = 1cm ´ ç ÷ cm.
0.02 (R + 490) = 2 (R) or 1.98 R = 9.8 è2ø
6. (a) We know that the relationship between current and
9.8
Þ R= = 4.9 W drift speed is
1.98
I = ne Avd
130. (b) In case of internal resistance measurement by
Where, I is the current and Vd is the drift velocity.
potentiometer,
So, I µ Vd
V1 l1 {E R 1 /(R1 + r )} R 1 (R 2 + r)
= = = Hence, only drift velocity determines the current in a
V2 l 2 {E R 2 /(R 2 + r )} R 2 (R1 + r) conductor.
Here l 1 = 2 m, l 2 = 3 m, R1 = 5 W and R 2 = 10 W 7. (b) Total resistance R = (0.5 W/km) × (150 km) = 75 W
2 5 (10 + r ) Total voltage drop = (8 V/km) × (150 km) = 1200 V
\ = or 20 + 4 r = 30 + 3 r or r = 10 W
3 10 (5 + r ) (DV)2 (1200) 2
Power loss = = W = 19.2 kW
R 75
EXERCISE - 2
8. (a) The current in 2W resistor will be zero because it is not
1. (b) As we know, electric current per unit area a part of any closed loop.
I
I/A, is called current density j i.e., j =
A
The SI units of the current density are A/m2. 5W 10 W
10 V 20 V
The current density is also directed along E and is also
2W
a vector and the relationship is
j = sE
9. (c) Neutral temperature,
Current density changes due to electric field produced
by charges accumulated on the surface of wire. qi + q 0 530 + 10
qn = = = 270º C.
2. (a) As we know the equivalent emf (eeq) in the parallel 2 2
combination 10. (c) Total power consumed by electrical appliances in the
e r +e r
eeq = 2 1 1 2 building, Ptotal = 2500 W
r1 + r2
So according to formula the equivalent emf eeq of the Watt = Volt × ampere
two cells in parallel combination is between e1 and e2. Þ 2500 = V × I Þ 2500 = 220 I Þ I » 12 A
Thus (e1 < eeq < e2).
(Minimum capacity of main fuse)
3. (c) Adjusting the blance point near the middle of the
bridge, i.e.. when l1 is close to 50 cm. requires a suitable
CURRENT ELECTRICITY 615

V Further, as heat generated is same, so


11. (a) EMF, E = Kl where K = potential gradient
L I12R1t = I22R2t
V iR æ E 0 r ö l 2
æ V ö
2
K= = =ç æ V ö
L L è r + r1 ÷ø L or çç ÷÷ R1 = çç ÷÷ R 2
è R + R1 ø è R + R2 ø
E 0 rl Þ R1(R + R2)2 = R2(R + R1)2
So, E = Kl =
(r + r1 )L Þ R2R1 + R1R22 + 2RR1R2
12. (b) The equivalent circuit can be redrawn as = R2R2 + R12R2 + 2RR1R2?
R R
R R Þ R2(R1 – R2) = R1R2(R1 – R2)
A· · B º ·
R R 2R Þ R= R 1R 2
R R
R R 20. (d)
5R / 8
º º º R
2R / 3
R 5R / 3
13. (b) Potential gradient along wire R
potential difference along wire
=
length of wire R
I ´ 40 R R
or, 0.1 ´10 -3 = V / cm R
R
R R
1000 R
1
or, Current in wire, I= A
400 R
2 1 B
or, = or R = 800 - 40 = 760 W A
40 + R 400
14. (a) Glow = Power (P) = I2R R/3
dP æ dI ö
\ = 2 ç ÷ = 2 ´ 0.5 = 1%
P è I ø
l V R/3 R/3
15. (b) V = IR = (neAvd )r \ r=
A nel Vd
5
r= A R/3 B
8 ´ 1028 ´ 1.6 ´ 10-19 ´ 2.5 ´ 10-4 ´ 0.1
3R R
= 1.6 × 10–5Wm ´ 2
3 3 = R = 4W
16. (b) The heat produced is given by Rnet between AB = 3R R
4R
æ 2ö +
V2 rl \ H = V 2 ç pr ÷ r r
3 3
r
H= and R = 2 ç rl ÷
è ø 21. (c) From relation J = s E , the current density J at any
R pr r
æ pV 2 ö r 2 point in ohmic resistor is in direction of electric field E
or H = ç at that point. In space having non-uniform electric field,
ç r ÷÷ l charges released from rest may not move along ELOF.
è ø
Thus heat (H) is doubled if both length (l) and radius Hence Assertion is correct while Reason is incorrect.
(r) are doubled. I
22. (a) Resistance wire R = r , where r is resistivity of
17. (a) A
18. (d) When two cells are connected in series i.e., E 1 + E2 the material which does not depend on the geometry of
balance point is at 50 cm. And when two cells are wire. Since when wire is bent resistivity, length and
connected in opposite direction i.e., E1 – E2 the balance area of cross-section do not change, therefore
resistance of wire also remain same.
point is at 10 cm. According to principle of potential 23. (c)
E1 + E 2 50 E1 3
= Þ =
E1 - E 2 10 E2 2 180V 45W
90W
19. (d) Let internal resistance of source = R
Current in coil of resistance 90W
V
R1 = I1 =
R + R1
Current in coil of resistance 40W 50W
V
R 2 = I2 =
R + R2
EBD_7418
616 PHYSICS

28. (b) Reading of potentiometer is accurate because during


i
taking reading it does not draw any current from the
180V 90W 45W circuit.
29. (a) From KVL
i
– 6 + 3I1 + 1 (Ii – I2) = 0
45W
180 6V P 2W
i= = 2A
90
6.023 ´ 1026 9V
24. (d) Number of electrons per kg of silver = 1W
108
Number of electrons per unit volume of silver
3W q 4W
26
6.023 ´ 10 6 = 3 I 1 + I1 – I2
n= ´ 10.5 ´ 103
108 4I1 – I2 = 6 ...(1)
I – 9 + 2I2 – (I1 – I2) + 3I2 = 0
vd =
neA – I1 + 6I2 = 9 ...(2)
20 On solving (1) and (2)
= 26 3 -19 -6
´ 108
6.023 ´ 10 ´ 10.5 ´ 10 ´ 1.6 ´ 10 ´ 3.14 ´ 10 I1 = 0.13A
= 6.798 × 10–4 m/sec. Direction Q to P, since I1 > I2.
30. (c) Kirchoff 's loop rule follows from conservation of
25. (a) energy.
31. (d)
32. (b) The potential difference in each loop is zero.
R1 G R2
\ No current will flow or current in each resistance is
x Zero.
A R3 C R4 B 33. (a) In steady state, flow of current through capacitor will
be zero.
R1 R3 x Current through the circuit,
At null point = =
R2 R4 100 - x E
i=
If radius of the wire is doubled, then the resistance of r + r2
AC will change and also the resistance of CB will Potential difference through capacitor
R1 R Q æ E ö r2
change. But since does not change so, 3 should Vc = = E - ir = E - ç r \ Q = CE
R2 R4 C ÷
è r + r2 ø r + r2
also not change at null point. Therefore the point C
34. (b) At Null point
does not change.
26. (d) R1 = R0(1 + a1t) + R0(1 + a2t)
X 10W
æ a + a2 ö
= 2 R0 ç1 + 1 t÷
è 2 ø
Comparing with R = R0 (1+ at)
a1 + a 2 A B
a= 52 cm 48 cm
2
X 10
rl =
27. (b) We know that, R = l1 l 2
A
Here l1 = 52 + End correction = 52 + 1 = 53 cm
rl 2
or R = Þ R µ l2
Volume l 2 = 48 + End correction = 48 + 2 = 50 cm
According to question l2 = nl1
X 10 53
R2 n 2l12 R2 \ 53 = 50 \X = = 10.6W
= 2 or,, = n 2 Þ R2 = n2R1 5
R1 l1 R1 35. (d) The net resistance of the circuit is 9W as shown in the
following figures.
CURRENT ELECTRICITY 617

3W 2W 2W 3W 2W
R
Rparallel =
9V 8W 8W 4W 8W 8W 8W n
9V
E
I= ...(i)
2W 2W 2W 2W 2W nR + R
3W 3W 2W E
10 I = R ...(ii)
8W 8W 8W 4W
+R
9V 9V n
Dividing eq. (ii) by (i),
2W 2W 2W
(n + 1) R
3W 10 = 1
æ ö
ç + 1÷ R
4W 9V 9W èn ø
9V
Solving we get, n = 10
2W 39. (a) Short circuited current,
V 9V ne e
I= = = 1.0 A I= =
R 9W nr r
The flow of current in the circuit is as follows.
1ampere 3W 0.5A I
0.5A
9V 9W 8W 8W
9V

2W O
n
1A 2W 2W 0.25A 3W 2W 0.25A So, I is independent of n and I is constant.
3W 0.25A 40. (c) Before the presence of electric field, the free electrons
9V
8W 8W 4W
9V 8W 8W 8W move randomly in the conductor, so their drift velocity
is zero and therefore there is no current in the conductor.
2W 2W 2W 2W 2W In the presence of electric field, each electron in the
The current divides into two equal parts if passes through conductor experience a force in a direction opposite to
two equal resistances in parallel. the electric field. Now the free electrons are accelerated
Thus current through 4W resistor is 0.25 A. from negative and to the positive end of the conductor
36. (d) When the wire is bent in the form of a square and and hence a current starts to flow from the conductor.
connected between M and N as shown in fig. (2), the 41. (c) Applying Kirchoff's loop law in AB P2P1A , we get
effective resistance between M and N decreases to -2i + 5 - 10 i1 = 0 .....(i)
one fourth of the value in fig. (1). The current increases
B i P2 i–i1 C
four times the initial value according to the relation
i1
V = IR. Since H = I2 Rt, the decrease in the value of
resistance is more than compensated by the increases 10W 2V
5V 1W
in the value of current. Hence heat produced increases.
2W
Percentage loss in energy during the collision ; 56%
37. (b) Colour code for carbon resistor A P1 D
0 Black
Again applying Kirchoff's loop law in P2 CDP1P2 we
1 Brown Tolerance : ± 5% Gold
2 Red ± 10% Silver get, 10 i1 + 2 – i + i1= 0 ....(ii)
3 Orange ± 20% No colour é 5 - 10i1 ù
4 Yellow From (i) and (ii) 11 i1 + 2 - ê ú=0
ë 2 û
5 Green
1
6 Blue Þ i1 = A from P2 to P1
7 Violet 32
8 Grey H P t P 500 5
42. (a) In parallel, 1 = 1 = 1 = =
9 White H 2 P2 t P2 200 2
(47 ± 4.7) kW = 47 × 103 ± 10% I2 R t R
H V 2 /P
\ Yellow - Violet - Orange - Silver In series, 1 = 2 1 = 1 = 2 1
38. (a) In series grouping equivalent resistance Rseries = nR H 2 I R 2 t R 2 V /P2
In parallel grouping equivalent resistance P2 200 2
= = = .
P1 500 5
EBD_7418
618 PHYSICS

43. (c) R1 + R2 = 1000 R1 R2 = 1000 – R1 same as the terminal voltage of either cell.
Since the current is drawn from the cell of 15 V
G 5
Þ R2 = 1000 – R1 \ V1 = E1 – Ir1 = 15 – ´ 0.6 = 13.1 V
1.6
On balancing condition (l ) 100 – l EXERCISE - 3
R1(100 – l) = (1000 – R1)l ...(i)
1. (b) Resistance is directly proportionl to length of the wire.
On Interchanging resistance balance point shifts left As length is doubled so mass is doubled and resistance
by 10 cm R2=1000 – R1 is doubled.
R1 We have
On balancing condition
(1000 – R1) (110 – l) G (10E )2 (nE )2 t
= t mS DT , Now= (2m)S DT
= R1 (l – 10) R 2R
or, R1 (l – 10) (l – 10) (100 – l + 10) n2 E 2t 102 E 2t
=(110 – l) Þ =2 Þ n = 20
= (1000 – R ) (110 – l)
1
...(ii) 2R R
Dividing eqn (i) by (ii) 8-4 4 1
2. (a) I= = = A;
1 + 2 + 9 12 3
100 - l l 1
= VP - VQ = 4 - ´ 3 = 3 volt
l - 10 110 - l 3
Þ (100 – l) (110 – l) = l(l – 10) V 2 100 ´ 100
3. (d) Resistance of heater R h = = = 10W
Þ 11000 = 200l or, l = 55 P 1000
Putting the value of ‘l’ in eqn (i) 10 ´ R
\ total resistance of circuit = 10 +
R1 (100 – 55) = (1000 – R1) 55 10 + R
Þ R1 (45) = (1000 – R1) 55 100 + 20R
=
Þ 20 R1 = 11000 \ R1 = 550KW 10 + R
44. (b) Using Kirchhoff’s law at P we get R 100 R
Current = I. = ´ 5´ R
V - 12 V - 13 V - 0 12V 1W 10 + R 100 + 20 R 10 +R =
+ + =0 T U 5 +R
1 2 10 10 + R
v–12 2W
[Let potential at P, Q, U = 0 S P 2
and at R = V] æ 5R ö
v–13 \ power P = I2 Rh Þ 62.5 = ç ÷ ´10 \ R = 5W
V V V 12 è5+ R ø
Þ + + = R Q
1 2 10 1 V 10W 0 H
4. (d) P= = s (T14 - T2 4 )
13 0 t
+ +
2 10 T1 = 727 + 273 = 1000 K, T2 = 227 + 273 = 500 K,
10 + 5 + 1 24 + 13 37 P = 60 watt
Þ V= =
10 2 2 In the second case, T1' = 1227 + 273 = 1500 K,
37 ´10 370 T2 ' = 500 K, P' = ?
ÞV= = = 11.56 volt
16 ´ 2 32
45. (b) Using formula, internal resistance,
P ¢ (1500) 4 - (500) 4 (500) 4 [3 4 -1] 80
æ l -l ö = =
r = ç 1 2 ÷ s = æç
52 - 40 ö P (1000) 4 - (500) 4 (500) 4 [2 4 -1] Þ 15
÷ ´ 5 = 1.5W
è l2 ø è 40 ø
46. (c) 80
47. (c) As the two cells oppose each other hence, the Þ P' = 60 ´ = 320 watt.
15
effective emf in closed circuit is 15 – 10 = 5 V and
5. (b) Let the resistor to be connected across CD be x. Then the
net resistance is 1 + 0.6 = 1.6 W (because in the
equivalent resistance across EF should be x and also across
closed circuit the internal resistance of two cells are
AB should be x. So we get A R
in series. F

effective emf 5 ( 2R + x ) R x
Current in the circuit, I = = A =x R x
total resistance 1.6 3R + x
E
The potential difference across voltmeter will be B R
solve to get x = ( 3 - 1) R
CURRENT ELECTRICITY 619

6. (d) Points 1, 2, 3, ........ are equipotential æ lö


çèQ R = r ÷ø
A
From ohm’s law V = RI
R/8 = (1.7 × 10–6 × 10–2 + 10–5 × 10–2) ¸
0.01 × 10–4 volt
R/8 = 0.117 volt (Q I = 1A)
= 11. (b) After distribution of current using kirchhoff’s loop rule
R/8 for
Loop a – (I – I1) R – I2 R + I1 R = 0 or 2I1 – I2 = I
Loop b – (I – I1 – I2) R + I3R + I2R = 0 or I1 + 2I2 + I3 = I
and 1¢, 2¢, 3¢, ........ are also equipotential. Loop c –I3R – (I – I1 – I2 + I3) R + (I1 + I2 – I3) R = 0
7. (a) The simplified circuit is or 2I1 + 2I2 – 3I3 = I
Solving for I1, I2 and I3 we get
10V 4 B x(assume)
4 I I
10 x–10 I1 = I, I2 = , I3 =
2 7 7 7
10 2 4
Here I2 = I3 so, no distribution of current takes place at
D so it is a pseudo junction. The equivalent circuit
10V 20V becomes
10 C R E
20
P
R R
O(assume) R
R
We have to find I.
Let potential of point P be 0. Potential at other points R
D
R
are shown in the figure apply Kirchoff’s current law at A B
B where potential is assume to be x volt.
2
x - 10 x - 10 x - 20 (x - 10) - 0 Resistance of triangle DCE is R parallel to (R + R) = R
+ + + =0 3
4 2 4 2 2 8
Þ x – 10 + 2x – 20 + x – 20 + 2x – 20 = 0 Resistance of network other than AB = R + R + R = R
3 3
35
35 20 - Equivalent resistance between AB
Þ 6x = 70 Þ x = volt \ I = 3 = 25 A
3 8R
4 12 2R ´
8. (a) Let the resistance of single copper wire be R1. If r is R eq = 3 = 8R
the specific resistance of copper wire, then 8R 7
2R +
3
r´ l r´l
R1 = = 2 ...(1) 12. (a) Let the resistance of half the turn be R. Then in the
A1 p r1 former case, we have fifteen resistors of resistance R
When the wire is replaced by six wires, let the resistance connected in parallel, the total resistance being R/15.
of each wire be R2. Then In the latter case, we have the same fifteen resistors
r´l r´l connected in series, the total resistance being 15R.
R2 = = ...(2) Therefore, as a result of unwinding, the resistance of the
A2 p r22 wire will increase by a factor of 225.
From eqs. (1) and (2), we get 13. (b) Let R be the resistance of wire.
2 3 2 l
R1 r2 5 (3 ´10 )
= or =
R 2 r1 2 R 2 (9 ´10 -3 ) 2 ; R 2 = 45 W
These six wires are in parallel. Hence the resistance of
the combination would be R 2 = 7.5 W V V V
9. (a) (3V )2
10. (a) Copper rod and iron rod are joined in series. Energy released in t second = ´t
R
l 9V 2
\ R = RCu + RFe = (r1 + r2) \ Q= ´ t But Q = mcDT (where m = mass of
A R
wire)
EBD_7418
620 PHYSICS

18. (a) Let, we connect 24 cells in n rows of m cells, then if I is


9V 2 the current in external circuit then
\ mcDT = ´t .... (i)
R mE
Let R' be the resistance of the second wire I= ...(1)
mr / n + R
2l
For I to be maximum, (mr + nR) should be minimum.
mr
It is minimum for R = ...(2)
n
So maximum current in external circuit is
V V n times V mE
Þ R' = 2R I= ...(3)
(Q length is twice)
2R
( NV ) 2 m
\ Energy released in t-seconds = ´t here R =3, r = 0.5 so equation (2) become =6
2R n
Also Q' = m'cDT = (2m) CDT so n = 2, m = 12
19. (b) The equivalent circuits are shown in the figure.
N 2V 2
2 mcDT = ´t .... (ii)
2R R
R 2R
Dividing (i) by (ii) R
R 6R R 6R
E 4W
mcDT 9V 2 ´ t / R 1 9´ 2
= 2 2 or, = 2R 4R
2mc DT N V t / 2 R 2 N2 R 4R

or, N 2 = 18 × 2 \ N=6 E 4W Balanced


Wheat stone
14. (d) R1 = R 0 (1 + a1t1 ) ; R 2 = R 0 (1 + a 2 t2 ) Bridge
1 2 The circuit represents balanced Wheatstone Bridge. Hence 6R W
As R1 = R2 and R10 = R 20 , resistance is ineffective
R 2R 3R
a1 t2
\ 1 + a1t1 = 1 + a2t2; a1t1 = a2t2 Þ = 2R 4R
a 2 t1 6R

15. (c) To carry a current of 4 ampere, we need four paths, each 4W


carrying a current of one ampere. Let r be the resistance E
of each path. These are connected in parallel. Hence, E 4W
their equivalent resistance will be r/4. According to the 1 1 1 (3R )(6 R)
r = + , Req = = 2R
given problem = 5 or r = 20W . Req 3R 6 R (3R) + (6R)
4 2R
For this propose two resistances should be connected.
There are four such combinations. Hence, the total 4W
number of resistance = 4 × 2 = 8.
E
d´N For Max. Power
16. (b) Electron density, n = External Resistance = Internal Resistance
M
2R = 4 W
2.7 ´ 103 ´ 6.02 ´ 10 26 R=2W
so, n = (atom/kg)
27
= 6.02 × 1028 electrons/m3 30
V2
\ Drift velocity, = 1.29 × 10–4 m/s 20. (c) Work done in 30s, W = ò R
dt
17. (a) Given ; J = ar2. 0
30 2 30
2 R/2 (200) (200)2
dt
i= ò1 J ´ 2 p r dr = òR/3 ar 2 ´ 2 p r dr or, W= ò 20t ò 20a
dt =
100
0 100(1 + a ) 0 1+ t
R/2
p a éæ R ö æ Rö ù 3 3
4 4
R/2 r4
ò
3 ê -
= 2pa r dr = 2 p a = ç ÷ çè ÷ø ú æ 1 + 20a ö
4 2 ëêè 2 ø 3 ûú ´ 30 ÷ æ 6ö
R /3 400 ´ 3 ç
÷ = 60,000 ln çè 5 ÷ø
R/3 = lnç 3
4 20a è 1 ø
p aR 65 65 p aR 4
= ´ = 1
2 81 ´ 16 2592 Q 120 = 100 éë1 + a ( 200 ) ùû \ a =
1000
MOVING CHARGES AND MAGNETISM 621

19. MOVING CHARGES AND MAGNETISM

Exercise - 1 Bmin =
F 10-10 -3 Tesla in
ẑ - direction
qv 10-12 ´ 105 = 10
=
mv2 mv r ur r ur
1. (d) Force, F = qVB = \ R= 17. (a) Lorentz force, F = q {E + (v ´ B)}
R Bq
2. (b) 3. (c) iˆ ˆj kˆ
r ur
4. (b) Magnetic force acts perpendicular to the velocity. v ´ B = 1 2 0 = 8iˆ - 4jˆ - 7kˆ
Hence speed remains constant. 5 3 4
5. (c) Equating magnetic force to centripetal force, r
mv2 F = 1 (2iˆ - 3jˆ + 8iˆ - 4jˆ - 7k)
ˆ = (10iˆ - 7ˆj - 7k)
ˆ
= qvB sin 90º 18. (c) Time taken by proton to make one revolution
r
Time to complete one revolution, 25
= = 5 m sec .
2pr 2pm 5
T= = 2 pm T m q
v qB As T = ; so 2 = 2 ´ 1
qB T1 m1 q 2
mv mv p
6. (a) Þr= r=
Bq Bq
= [ where p = mv] or T2 = T1
m 2 q1 5 ´ 4 m1 q
= ´ = 10 m sec.
Bq
m1 q 2 m1 2q
Þ r µ momentum r r r
7. (c) 8. (c) 19. (b) The application of equation FB = q (V ´ B ) on the
r r r element dl of the rod gives force on positive charge
9. (c) Force, FB = q (V ´ B )
towards the outer end. Therefore electrons will move
which gives direction of force towards centre.
towards pivoted end.
10. (d) The change in K.E. is equal to work done by net force
× ×
which is zero because the magnetic force is
× × ×
perpendicular to velocity. K.E. remains constant.

dl
®
11. (c) × × V ×
12. (c) As Lorentz force is given by × × ×
r r r r r r r O
F = q ( E + V ´ B ) = qE + q (V ´ B ) × × ×
r r r
F = FE + FB × × ×
13. (a) Lorentz force acting on the particle 20. (d) Obviously gravitational and electric force is there as
ur ur r ur both the particle have mass and charge. Since both
F = q éë E + v × B ùû
charges are in motion so they constitute currents which
é ˆi ˆj kˆ ù generate magnetic fields around them and thus exert
ê ú magnetic force on each other.
= q ê 3i$ + $j + 2k$ + 3 4 1 ú
ê ú 21. (c) 22. (b)
ê 1 1 -3 ú 23. (a) r = mv/Bq is same for both.
ë û
24. (b) The charged particle will move along the lines of electric
$ $ $
= q éë3i + j + 2k + i ( -12 - 1)
ˆ
field (and magnetic field). Magnetic field will exert no
- $j ( -9 - 1) + k ( 3 - 4 ) ùû force. The force by electric field will be along the lines
Fy = 11qj ˆ of uniform electric field. Hence the particle will move in
14. (c) The electron moves with constant velocity without a straight line.
deflection. Hence, force due to magnetic field is equal and 25. (a) Since magnetic force is always perpendicular to the
opposite to force due to electric field. velocity of electron, so it can only change the direction
E 20 of velocity of electron, but it (the magnetic force) cannot
qvB = qE Þ v = = = 40 m/s accelerate or deaccelerate the electron.
B 0.5 26. (b)
r r
15. (a) F µ ( vr ´ B) = kˆ [ aD - dA ] m v2 m v (9 ´ 10 -31 ) ´ 10 6
F 27. (c) Bqv = or B = =
16. (d) F = qvB sin q Þ B = r rq 0.1 ´ (1.6 ´ 10 -19 )
qv sin q = 5.5 × 10–5 T
EBD_7418
622 PHYSICS

r r 44. (d) Since the total current going into the surface is equal
28. (b) Here, E and B are perpendicular to each other and
r to total current coming out of the surface, therefore
the velocity v does not change; therefore current enclosed is zero.
E 45. (c)
qE = qvB Þ v =
B µ I q m Iq
r ur ur 46. (a) B = 0 ´ = 0
r
If velocity is ^ to both E and B ,
v 2r 2p 4 pr
r r m0 IdI sin q
E´B E Bsin q E B sin 90° E r
Also, = = = = |v| = v 47. (c) dB =
B 2
B 2 B 2 B 4p r 2
29. (b) Due to perpendicular component both will execute Here, dI = Dx = 0.05 m, I = 10 A, r = 1 m
2pm sinq = sin 90° = 1,
circular motion for which T = . Since q is same, 10 ´ 0.05 ´ 1
qB
therefore T µ m. Hence deuterium nucleus will travel \ dB = 10–7 ×
(1) 2
more distance. = 0.50 × 10 = 5.0 × 10–8 T
–7
30. (b) When the block moves to the right application of 48. (d) The magnetic field at any point on the closed loop is
r r r
equation FB = q (V ´ B ) gives magnetic force in due to all the three currents, but line integral of i3 over
r the closed loop will be zero.
upward direction similarly when block moves left, FB
is in downward direction. 49. (a) Reversing the direction of the current reverses the
direction of the magnetic field. However, it has no effect
Bq on the magnetic-field energy density, which is
31. (b) Cyclotron frequency (v) =
2pm proportional to the square of the magnitude of the
32. (c) E = vB = 2 × 103 × 1.5 = 3 × 103 V/m. magnetic field.
r m 0 q(vr ´ rr) r r r
50. (d) There is no current inside the pipe. Therefore
ur uur
33. (b) dB =
4p r 3
i.e. B ^r v & r both
Ñò B.dt = µo I
ur uur I=0 \ B= 0
34. (d) According to Ampere’s circuital law ò B . dl = m I
o
m 0i a 2
51. (c) B=
m 0i 1 3
35. (b) Magnetic field is given by B = i.e., B µ which 2(x 2 + a 2 ) 2
2pr r
implies that field has cylindrical symmetry. æ 2 2 2ö
3
m i m 0i a 2 ç (x + a ) ÷
36. (c) B' = 0 = 3 ç
2a a2 ÷
37. (c) Field at the center of a circular coil of radius r is 2a(x 2 + a 2 ) 2 è ø
m0 I B.(x 2 + a 2 )3/2
B= B' =
2r
38. (b) Magnetic field at a point on one end of a solenoid a3
54(53 )
1 Put x = 4 & a = 3 Þ B' = = 250mT
B = m0ni 3´3´3
2 52. (c) From Ampere circuital law
m 0 2pi ur uur
39. (c) B =
4p r
where Ñò B.dI = m0 I enc
B × 2pR = m0Ienc
2e 2 ´ 1.6 ´10 -19 m0 I enc
i= = = 1.6 ´ 10 -19 A 75
t 2 B= = 2 × 10–7 × = 5 × 10–6 T
2pR 3
m i m ´ 1.6 ´ 10 -19 The direction of field at the given point will be
\ B= 0 = 0 = m 0 ´ 10 -19 T
2r 2 ´ 0.8 vertical up determined by the screw rule or right
40. (b) 41. (b) 42. (b) hand rule.
43. (b) The magnetic field from the centre of wire of radius 53. (d) (1)
R is given by
æ m0 I ö
B = ç 2 ÷ r (r < R) Þ B µ r
è 2R ø (2)
m0 I 1
and B = (r > R) Þ B µ
2 pr r
From the above descriptions, we can say that the The magnetic field due to circular coil,
graph (b) is a correct representation.
µ0i1 m 0i1 m 0 ´ 3 ´ 10 2
B1 = 2r = =
2 (2p ´ 10-2 ) 4p
MOVING CHARGES AND MAGNETISM 623

m 0i2 m 0 ´ 4 ´ 102 directions therefore the correct net field at centre is


B2 = = given by vector sum of field produced by each loop
2(2p ´ 10 -2 ) 4p which are alternately in opposite directions.
m 63. (b) Magnetic fields due to the two parts at their common
B = B12 + B 22 = 0 . 5 × 102
4p centre are respectively, z
Þ B = 10–7 × 5 × 102 Þ B = 5 × 10–5 Wb / m2 m 0i m 0i
By = and Bz = i
54. (b) Let l be length of wire 4R 4R
l Resultant field = B y2 + Bz2
Ist case : l = 2pr Þ r =
2p 2 i
2
m0I m 0I æ m iö æ m iö
B= = = ç 0 ÷ +ç 0 ÷
2pr l è 4R ø è 4R ø y

l m 0i m i
2nd Case : l = 2(2pr ¢) Þ r ¢ = = 2. = 0 i
4p 4R 2 2 R
m 0 In 2m 0 I m0 i m0 i
B¢ = = (where n = 2) 64. (c) B= . - . =0
l l 2p r 2p r
2p 65. (a) The magnetic field of two equal halfs of the loop is
4p 2
r
æ m0I ö equal and opposite and so B = 0 .
on putting the value of B Þ B¢ = 4ç ÷ = 4B
è l ø 66. (b) Using Ampere’s law at a distance r from axis, B is same
55. (c) Here, N = 100 from symmetry.
R = 9 cm = 9 × 10–2 m, and I = 0.4 A
m0 NI 2p ´ 10-7 ´ 100 ´ 0.4
ò B.dl = mi
Now, B = = B ´ 2pr = m0i B × 2pr = m0i
2R 9 ´ 10 -2
Here i is zero, for r < R, whereas R is radius
2 ´ 3.14 ´ 0.4
= ´ 10 -3 \B=0 x
9 67. (a) The magnitude of magnetic
= 0.279 × 10–3 T = 2.79 × 10–4 T æR Rö
56. (d) B = m0nI = 4p × 10–7 × 10 × 5 = 2p × 10–5 T. field at P ç , y, ÷ is B
è2 2ø
B m Jr m i R ^
B R/Ö2 P
57. (a) B = m0ni Þ i = B= 0 = 02´ q =45°
m0 n 2 2pR 2 O
z

20 ´ 10-3 m0 i
=
= = 7.9 A = 8 A 2 2pR
4 p ´ 10 -7 ´ 20 ´ 100
(independent on y-coordinate)
600
58. (c) Here, n = = 1000 turns / m I = mA Unit vector in direction of magnetic field is
0.6
1 ˆi - kˆ
I = 0.6 m, r = 0.02 m Q = 30 i.e. I >> r B̂ = ( shown by dotted lines)
r 2
Hence, we can use long solenoid formula, them r m i
\ B = m0nI = 4 × 10–7 × 103 × 4 \ B = BBˆ = 0 (iˆ - k) ˆ
= 50.24 × 10–4 = 5.024 × 10–3 T 4pR
m I 1
æN ö 68. (a) B = 0 Þ B µ
59. (c) B = m0N0i; B1 = (m0 ) ç 0 ÷ (2 i) = m0 N 0 i = B 2 pr r
è 2 ø As the distance is increased to three times, the magnetic
Þ B1 = B 1 -3
induction reduces to one third. Hence, B = ´ 10 tesla
æ m 0 NI ö 2 3
= 3.33 × 10–4 tesla
60. (c) Baxis = çç ÷÷R
è 2x 3 ø m 0 2pni1 m 0 2pni 2 m é ni ni ù
B µ R2 69. (d) B= . - . = 0 ê 1- 2ú
4p r1 4p r2 2 ë r1 r2 û
So, when radius is tripled, magnetic field becomes 9
times. I
61. (c) Since n is an even number, we can assume the wires in 70. (b)
pairs such that the two wires forming a pair is placed
diametrically opposite to each other on the surface of P
cylinder. The fields produced on the axis by them are d a a
d sin

equal and opposite and can get cancelled with each other.
a

62. (b) Field at the centre of a circular current loop is given by


mi
B= . Since the currents are alternately in opposite Let us compute the magnetic field due to any one segment:
2R
EBD_7418
624 PHYSICS

µ0 i A = area of the loop


B= ( cos 0° + cos (180 - a ) )
4p ( d sin a ) From the above relation, we can conclude that
magnetic dipole moment of a current loop is
(1 - cos a ) = 0 tan a
µ0 I µ I
= independent of magnetic field in which it is lying.
4p ( d sin a ) 4pd 2
79. (c)
Resultant field will be
80. (c) M = NiA Þ M µ A Þ M µ r2
µ I a µ I [As l = 2pr Þ l µ r]
Bnet = 2B = 0 tan Þ K = 0
2pd 2 r 2rpd Þ M µ l2
71. (c) Along the dashed line B1 and B2 are in opposite 81. (b) (A) ® (1,2) ; (B) ® (1) ; (C) ® (4) ; (D) ® (2)
directions. If the line has slope –1.00, then r 1 = r2 and m0 i 2
B1 = B2. So, Btot = 0 82. (c) The force per unit length is given by f =
2pd
#2 i.e., f µ i2
r2 83. (a) Due to flow of current in same direction at adjacent
r1 10.0 A side, an attractive magnetic force will be produced.
#1 Ù
45° 84. (a) The direction of B is along ( - k )
10.0 A
\ The magnetic force
F = Q( v ´ B) = Q( v î ) ´ B(- k̂ ) = QvBĵ
m 0 qf
72. (a) Magnetic field at the centre of the ring is Þ along OY..
2R 85. (a)
73. (d)
74. (a) Let l1, l2 be the lengths of the two parts PRQ and PSQ of 86. (a) Because t = NiAB cosq
the conductor and r be the resistance per unit length of 87. (c)
r r r
the conductor. The resistance of the portion PRQ will be 88. (b) Here, FAB + FBCDA = 0
R1 = l1 r r r r
I2 Þ FBCDA = - FAB = - F
r
(Q FAB = F )
S O Q 89. (b) As the current flows in the spiral in clockwise direction,
R r r r
P I1 the application of equation dFB = i dl ´ B given the
force on current element dl in radially outward direction
The resistance of the portion PSQ will be R2 = l2 r hence expand.
Pot. diff. across P and Q = I1 R1 = I2 R2 90. (b) The force on the two arms parallel to the field is zero.
or I1 l1 r = I2 l2 r or I1 l1 = I2 l2 ...(1) <
Magnetic field induction at the centre O due to currents
<

<

through circular conductors PRQ and PSQ will be F


m I l sin 90º m 0 I 2 l 2 sin 90º B
= B1 – B2 = 0 1 1 2 - = 0.
4p r 4p r2 –F
75. (c) The point P is lying symmetrically w.r.t. the two long <
straight current carrying conductors. The magnetic \ Force on remaining arms = –F
fields at P due to these current carrying conductors are 91. (c) 92. (a)
mutually perpendicular. 93. (b) F = iB l sin q. This is maximum when sin q = 1
m 2i2 m 2 i1 m 4 or q = p/2.
76. (c) B = 0 - 0 = 0 (i 2 - i1 ) 94. (b) F = Il B sin q = 3 ×0.40 × (500 × 10–4) × sin 30º
4 p ( r / 2) 4 m ( r / 2) 4 p r
= 3 × 10–2 N.
m 4 m
= 0 (5 - 2.5) = 0 . F
4p 5 2p 95. (a) F = IIB sinq or sinq =
IIB
i i 15 1
F m 0 i1 i 2 m 0i 2 F sinq = = or q = 30°
77. (b) = = 10 ´1.5 ´ 2 2
l 2pd 2 pd 96. (b) 97. (c)
m 0 2I1I 2 10 -7 ´ 2 ´10 ´ 2
(attractive as current is in the same direction) 98. (b) F= ´l = ´ 2 = 8 ´10 -5 N
4p r 0.1
78. (a) Current loop acts as a magnetic dipole. Its magnetic 99. (b) F = IIB
moment is given by Here, q = 90°, I = 10 A
M = NIA I = 8 cm = 8 ×10–2 m, B = 0.3 T
where N = number of turns, I = current in a loop, \ F = 10 × 8 × 10–2 × 0.3 × sin90° = 0.24 N
MOVING CHARGES AND MAGNETISM 625

100. (c) Magnetic dipole moment Ig IS V V S


e e erv Þ < Þ < ≥
m = iA = ´ pr 2 = ´ pr 2 = . 2 G ∗S 2(R ∗ G) R ∗ GS G ∗S
T (2 pr / v) 2 G ∗S
1.6 ´ 10 -19 ´ 50 ´ 10 -12 ´ 2.2 ´ 10 6 1 S
= Þ <
2 2(R ∗ G) R(G ∗ S) ∗ GS
= 8.8 × 10–25 Am2.
Þ R(G ∗ S) ∗ GS < 2S(R ∗ G)
101. (b) To measure AC voltage across a resistance a moving
coil galvanometer is used. Þ RG ∗ RS ∗ GS < 2S(R ∗ G)
NiAB Þ RG < 2S(R ∗ G) , S(R ∗ G)
102. (b) q= ÞqµN [Number of turns]
C [ RG < S(R ∗ G)
Cq 106. (d) t = MB sin q Þ tmax = NiAB, [q = 90°]
103. (b) i= Þiµq
NAB 107. (c)
V 5 108. (a) The galvanometer cannot as such be used as an
104. (d) As we know, I = = = 0.1 ammeter to measure the value of the current in a
R 50
I' = 0.099 given circuit. This is for two reasons (i) Galvanometer
When Galvanometer is connected is a very sensitive device, it gives a full-scale
deflection for a current of the order of mA. (ii) For
100S V
R eq < 50 ∗ < measuring currents, the galvanometer has to be
100 ∗ S I connected in series and as it has a large resistance,
100S 5 this will change the value of the current in the circuit.
Þ < , 50 109. (c) An ammeter should have a low resistance which we
100 ∗ S 0.099
100S 100S get when we connect low resistance in parallel with
Þ < 50.50 , 50 Þ < 0.5 galvanometer.
100 ∗ S 100 ∗ S
110. (c) [Hint Þ S × (I – Ig) = Rg × Ig]
Þ 100S = 50 + 0.55 Þ 99.5S = 50
50 111. (b) Ig = 0.1I, Is = 0.9 I ; S = Ig R g / Is
S< < 0.5 W
99.05
= 0.1 ´ 900 / 0.9 = 100 W.
So, shunt of resistance = 0.5W is connected in parallel
with the galvanometer. 112. (c) As 0.2% of main current passes through the
V V 998
105. (a) According to Ohm's Law, I = \ Ig < galvanometer hence I current through the shunt.
R R ∗G 1000
where, Ig-Galvanometer current, G-Galvonometer 998I
resistance æ 2I ö æ 998I ö Þ S = G 1000 S
ç ÷G = ç ÷S 499
IG è 1000 ø è 1000 ø
R Total resistance of Ammeter
G G
æ G ö
ç ÷ G I 2I
SG 499 ø G
= è
1000
V R= =
S+G æ G ö 500
ç ÷+G
When shunt of resistance S is connected parallel to è 499 ø
GS 113. (c) To keep the main current in the circuit unchanged, the
the Galvanometer then G < resistance of the galvanometer should be equal to the net
G ∗S
resistance.
V
\ I< æ GS ö GS G2
GS \G = ç + S¢ Þ G - = S¢ \ S¢ =
R∗ è G + S ÷ø G+S
.
G ∗S G+S
I I
IG G G

R 2
G
S
S æ SG ö
114. (b) ig = i ç
è S + G ÷ø
Equal potential difference is given by æ S ö
or 50 ´ 10 -6 = i ç ....... (i)
I'g G < (I , I'g )S Þ I 'g (G ∗ S) < IS è S + 100 ÷ø
Also V = ig (G + R0)
or V = 50 × 10–6(100 + R0) ....... (ii)
EBD_7418
626 PHYSICS

On simplifying above equations, we get opposite sign.


V = 10 V, R0 = 200 × 10–3 W æeö æeö
115. (c) G = 60W, Ig=1.0A, I=5A. \ ç ÷ +ç ÷ =0
è m ø1 è m ø2
I Ig 60W 2. (a) By Biot-Savart law
G
Idl sin q æ I ´ dl ö
(I–Ig) dB = =ç ÷
r2 è r ø
In Biot-Savat’s law, magnetic field B||idl × r and idl due
S to flow of electron is in opposite direction of v and by
Let S be the shunt resistance connected in parallel to direction of cross product of two vectors
galvanometer B^V
Ig G = (I – Ig) S, So, the magnetic field is ^ to the direction of flow of
Ig G 1 charge.
S= = ´ 60 = 15W 3. (a) As the direction of magnetic moment of circular loop
I – Ig 5 – 1
of radius R placed in the x-y plane is along z-direction
Thus by putting 15 W in parallel, the galvanometer can be
and given by M = I (pr2), when half of the loop with x >
converted into an ammeter.
0 is now bent so that it now lies in the y-z plane, the
116. (a) We know
magnitudes of magnetic field moment of each
I G 750 13 semicircular loop of radius R lie in the x-y plane and the
= 1+ = 1+ S Þ 2W
IS S 100 S y-z plane is M' = I(pr2)/4 and the direction of magnetic
117. (a) Galvanometer is converted into ammeter, by connected field moments are along z-directlon and x-direction
a shunt, in parallel with it. respectively.
G Then resultant is :
GS VG 25 ´ 10-3 I
==
M ¢2 + M ¢2 = 2 M¢ = 2 I ( pr ) 4
2
G+ S I 25 Mnet =

GS So, Mnet < M or M diminishes.


= 0.001W S Hence, the magnitude of magnetic moment is now
G+S
diminishes.
Here S << G so S = 0.001 W
4. (d) Magnetic Lorentz force :
SG
118. (a) ig = i F = qVB sin q
S +G Magnetic Lorentz force electron is projected with
æ 0.1 ´ 100 ö uniform velocity along the axis of a current carrying
or 100 ´ 10 -6 = i ç or i = 100.1 × 10–3 A.
è 100 + 0.1 ÷ø
long solenoid F = –qvB sin 180° = 0(q = 0°) as magnetic
119. (c) Ig = 10 × 10–3/5 = 2 × 10–3 A; field and velocity are parallel and electric field is zero
V 1 (E = 0) due to this magnetic field (B) perpendicular to
R = - Rg = - 5 = 495 W.
Ig 2 ´ 10 -3 the direction of motion (V). So it will not affect the
120. (b) Here, Rg = 100 W; Ig = 10–5 A; I =1A; S = ? velocity of moving charge particle. So the electron will
continue to move with uniform velocity along the axis
Ιg R g 10-5 ´ 100
S= = -5
= 10-3 W in parallel of the solenoid
I - Ig 1 - 10 5. (a) There is crossed electric and magnetic field between
dees so the charged particle accelerates by electric
Exercise - 2
field between dees towards other dees.
1. (d) As we know that the uniqueness of helical path is So, the charged particle undergoes acceleration as
determined by its pitch (i) speeds up between the dees because of the oscillating
2p mv cos q electric field.
P(Pitch) = (ii) speed remain the same inside the dees because of the
Bq
magnetic field but direction undergoes change
Where q is angle of velocity of charge particle with x-
continuously.
axis
Hence, the charge particle accelerates inside and
For the given pitch d correspond to charge particle, we
between Dees always.
have
q 2 pv cos q 6. (d) Net magnetic field, B = B12 + B22
= = constant
m BP 2 2
æm I ö æm I ö
If motion is not helical, (q = 0) = ç 0 1 ÷ +ç 0 2 ÷
As charged particles traverse identical helical paths in è 2pd ø è 2pd ø
a completely opposite direction in a same magnetic m0 æ m 0 I1 m 0 I2 ö
field B, LHS for two particles should be same and of = I12 + I22 çQ B1 = 2 pd and B2 = 2 pd ÷
2pd è ø
MOVING CHARGES AND MAGNETISM 627

æ m 0 NI ö 2 mv2
13. (a) As we know F = qvB =
7. (c) Baxis = çç ÷÷R r
è 2x 3 ø
mv
\ r=
B µ R2 Bq
So, when radius is doubled, magnetic field becomes 1
four times. And KE = k = mv 2
2
8. (b) Work done in moving the conductor is, \ mv = 2km
2 2 -4 -0.2 x
W = ò Fdx = ò 3.0 ´ 10 e ´ 10 ´ 3dx mv 2km
0 0
-3 \ r= =
2 9 ´10 qB qB
-3
= 9 ´ 10 ò e
-0.2 x
dx = [-e -0.2´2 + 1]
0 0.2 Þ r µ k or r = c1/2 (c is a constant)
-3
9 ´10 dk1/ 2 cDk
= ´ [1 - e -0.4 ] l=3m dr
=c or =2 k
0.2 I = 10 A dr dr Dr
z
Dr cDk Dk
= 9 × 10–3 × (0.33) = 2.97 × 10–3J or = =
x r 2 kc k 2k
W Therefore percentage changes in radius of path,
Power required to move the conductor is, P =
t Dr Dk
2.97 ´ 10 -3 ´ 100 = ´100 = 2%
P= = 2.97 W r 2k
(0.2) ´ 5 ´ 10-3 14. (c) When a charge particle is allowed to move in a uniform
9. (b) magnetic field, then it describes spiral or circular path
m0 I i mv2 æ qB ö
B= [sin 90° + sin( -q)] I Centripetal force, = qvB \ v = ç R÷
4p a A R èm ø
m0 i 2qV æ qB ö éQ V = 2qV ù
= (1 - sin q) Hence, =ç ÷R ê ú
4p a m èmø ë m û
(0,0) (a,0)
1/ 2
m0 i æ b ö æ 2mV ö 1
= ç1 - 2 ÷ Þ R =ç ÷ ´
4p a è a + b2 ø è q ø B
or, m µ R2 [Q V, q and B are constant]
mv 2 2
10. (c) As we know, F = qvB = m1 æ R1 ö
R =ç
or, ÷
mV 2 2m(kE) m2 è R 2 ø
\R= =
qB qB 15. (a) The wire carries a current I in the negative z-direction.
We have to consider the magnetic vector field
q2 ur
Since R is same so, KE µ
m B at (x, y) in the z = 0 plane.

Therefore KE of a particle =
q2
=
( 2 )2 = 1 MeV y
m 4 P(x, y)
11. (d) The straight part will not contribute magnetic field at the
centre of the semicircle because every element of the r q
straight part will be 0º or 180º with the line joining the q
O× x
centre and the element
1 m0i m 0 i ur
Due to circular portion, the field is 2 2 r = 4 r Magnetic field B is perpendicular to OP.
ur
\ B = B sin qiˆ - B cos qˆj
m0 i
Hence total field at O = tesla y x m I
4r sin q = , cos q = B = 0
r r 2pr
r r ur m0 I
12. (a) For stable equilibrium M || B
r r
\ B=
2pr 2
(
yiˆ - xjˆ )
For unstable equilibrium M || (– B)
or B =
(
ur m 0 I yiˆ - xjˆ
.
)
(
2p x 2 + y 2 )
EBD_7418
628 PHYSICS

16. (b) = 2 × 250 × 85 × 10–6[1.25 × 2.1 × 10–4] × 85 × 10–2 = 9.1 mJ


17. (a) The direction of current in conductor 22. (c) Force per unit length between two parallel current car-
XY and AB is same Y FBC m 0 i1i 2
\ FAB = ilB (attractive) B C rying conductors, F =
F AB FCD 2pd
m0iI i Since same current flowing through both the wires
FAB = ( ¬) I L
p ii = i2 = i F1[due to wire A]
FBC opposite to FAD = m0iI ( ® ) X A L D
3p FAD m0i2
so F1 = = F2
2pd
Therefore the net force on the loop F2[due to wire C]
Fnet = FAB + FBC + FCD + FAD \ Magnitude of force per unit length on the middle wire 'B'
m o iI m o iI 2m o iI m0 i 2
Þ Fnet = - = Fnet = F12 + F22 =
p 3p 3p 2pd
18. (b) In the absence of the electric current, the free electrons 23. (a) Given: No. of turns N = 1000
in a conductor are in a state of random motion, like Face area, A = 4 cm2 = 4 × 10–4 m2
molecule in a gas. Their average velocity is zero. i.e. Change in magnetic field,
they do not have any net velocity in a direction. As a DB = 10–2 wbm–2
result, there is no net magnetic force on the free Time taken, t = 0.01s = 10–2 sec
electrons in the magnetic field. On passing the current, Emf induced in the coil e = ?
the free electrons acquire drift velocity in a definite Applying formula,
direction, hence magnetic force acts on them, unless
the field has no perpendicular component. -d f
Induced emf, e =
dt
DB ö
19. (b) Case (a) : = N æç ÷ A cos q
è Dt ø
µ0 I µ I 1000 ´ 10-2 ´ 4 ´ 10 -4
bA = ´ 2p = 0 ´ 2p (2pR = l) = = 400 mV
4p R 4p l / 2p 10-2
µ0 I 24. (b) When a charged particle enters a magnetic field at a
= ´ (2p)2
4p l direction perpendicular to the direction of motion, the
Case (b) : path of the motion is circular. In circular motion the
45°
µ0 I a B direction of velocity changes at every point (the
BB = 4 × [sin 45° + sin 45°] B
magnitude remains constant).
4p a / 2 a/2
µ I 2 µ I Therefore, the tangential momentum will change at
= 4´ 0 ´ ´ = 0 ´ 32 ´ 2 [4a = l] every point. But kinetic energy will remain constant as
4p l / 8 2 4 pl
20. (d) For a charged particle orbiting in a circular path in a 1
it is given by mv 2 and v 2 is the square of the
magnetic field 2
magnitude of velocity which does not change.
mv2 Bqr
= Bqv Þ v = 25. (d) Net magnetic field due to the wires will be downward
r m r
or, mv 2 = Bqvr as shown
ur below in the figure. Since angle between v
Also, and B is 180°,
1 2 1 r Bqr B 2 q 2 r 2 v Z
EK = mv = Bqvr = Bq . =
2 2 2 m 2m Y
2 2 2
B q r
For deuteron, E1 =
2 ´ 2m I -I
B2 q 2r 2 X
For proton, E2 =
2m B Wires are in X-Y plane
E1 1 d and velocity in Z-direction
50keV 1
= Þ = Þ E2 = 100keV ur r ur
E2 2 E2 2 Therefore, magnetic force F m = q(v ´ B) = 0
21. (d) Work done, W = MB(cosq1–cosq2)
26. (d) Current sensitivity of moving coil galvanometer
When it is rotated by angle 180° then
W = MB (cos0° – cos 180°) = MB (1 + 1) NBA
Is = ...(i)
W = 2MB = 2 (NIA) B C
MOVING CHARGES AND MAGNETISM 629

Voltage sensitivity of moving coil galvanometer, L


Finally for square coil side a = and
NBA 4
Vs = CR ...(ii) 2
æ Lö iL2 ...(ii)
G M ' =i ´ç ÷ =
Dividing eqn. (i) by (ii) è4ø 16
Resistance of galvanometer
Is 5 ´1 5000
RG = = = = 250 W
Vs 20 ´10 –3 20
27. (c) From figure, for equilibrium, °
30 L/4
mg sin 30° = I/B cos 30° B cos i
mg Il
B pM
Þ I= tan 30° Solving equation (i) and (ii) M ¢ =
lB ° 30° IlB 4
30
sin 1 1
0.5 ´ 9.8 g 2. (b) m = current x area = i ( pa2 + pb2)
= = 11.32 A m 30° 2 2
0.25 ´ 3
28. (d) The frequency of revolution is given by 1
= ip (a2 + b2).
qB 2
n= , hence Statement I is false and II is true. 3. (c) If r is the radius of the circle,
2 pm L
29. (b) As we know, radius of circular path in magnetic field then L = 2pr or, r =
2p
2Km Area = pr 2 = pL2 / 4p 2 = L2 / 4p
r=
qB i
B2 m 0 n 2 i 2 100 ´
= B 2 3
2Km e 4. (a) Þ =
For electron, re = ....(i) B1 m0 n1i1 6.28 ´ 10 -2 200 ´ i
eB
6.28 ´10 -2
2Kmp Þ B2 = = 1.05 ´ 10- 2 Wb / m 2
For proton, rp = ....(ii) 6
eB 5. (a) 6. (a)
2Km a 2K4m p 2Km p
For a particle, ra = = = ...(iii) µ0 I é pù
qa B 2eB eB 7. (c) B= ê 2 sin ú
4pd ë nû
\ r e < rp = ra (Q me < mp)
p r p
m0 I But cos = or r = R cos
30. (c) Magnetic field at the centre of loop, B1 = n R n
2R µ0 I é pù
B= 2 sin ú
Dipole moment of circular loop is m = IA pê
\ 4p R cos ë nû R
m1 = I.A = I.pR2 {R = Radius of the loop} n p/n p/n

If moment is doubled (keeping current constant) µ I é pù


= 0 ê tan ú r
R becomes 2R 2 pR ë nû
8. (c)
( )
2
m 2 = I.p = 2.IpR 2 = 2m1
2R q
9. (a) The ratio M/L is always
m0 I 2m
m0 I B 10. (d) The particle will move in a non-uniform helical path
B2 = \
Þ 1
= 2R = 2
2 2R( B2 ) m0 I
with increasing pitch as shown in Fig.
x
2 ( 2R )
work done
31. (d) Power =
time
As no work is done by magnetic force on the charged y
particle because magnetic force is perpendicular to 2pm
Its time period will be : T = = 2p s
velocity, hence power delivered is zero. qB
Changing the view, the particle seems to move in a circular
Exercise - 3 path in (X – Z) plane as shown in Fig.
X
1. (d) Initially for circular coil L = 2pr and M = i × pr2
2
æ Lö iL2
= i ´ pç ÷ = ...(i)
è 2p ø 4p
Z . P
O
EBD_7418
630 PHYSICS

After p seconds, the particle will be at point 'P' (after


completing half circle), hence X - coordinate of P will be 0. mv
d = R sin q = sin q
1 Eq 2 p 2 qB
y ( p) = 0 (p) + (p) = ,
2 m 2 q v sin q
z = 2r = 2mv/qB = 2 m Þ =
m Bd
æ p2 ö 14. (d) Magnetic field between the plates in this case is zero.
Hence, the coordinates of the particle are çè 0, , 2÷ø æ ® ®ö
2 ®
15. (c) F m = q çç v ´ B ÷÷
m0I m I è ø
11. (c) B = (sin 90° + sin135°) = 0 ( 2 - 1)
4p
R 4 pR = q (2iˆ + 3 ˆ
j )
+ 4kˆ ´ 2iˆ
2
= q ( -6k + 8 j )
ˆ ˆ
m N i
12. (a) Bsolenoid = m0 ns is = 0 S S , ® ®
LS F m+ F e = 0
® ®
m 0 NS iS iNp r 2 \ Fe = Fm
t = BS iNA =
LS (
= – q -6kˆ + 8 ˆj )
4p ´ 10-7 ´ 500 ´ 3 ´ 0.4 ´ 10 ´ p ´ (0.01) 2 pm T
t= After qB = , the direction of motion becomes
0.4 2
= 5.92 × 10–6 N-m. opposite to initial and so applied field may be
13. (d) sin q = (d/R)
( )
®
E = -6kˆ + 8 ˆj units
MAGNETISM AND MATTER 631

20. MAGNETISM AND MATTER

Exercise - 1 M1 T22 + T12 62 + 42 52


24. (c) = = = = ( 2.6 ) :1
1. (b) When a bar magnet cut perpendicular to its axis into M 2 T22 - T12 62 - 42 20
l 25. (c) Work done W = MBH (1 – cosq)
two equal parts then m¢ = m and length l¢ =
2 æ 3ö
¢ ¢ l M = 20 × 0.3(1 – cos 30°) = 6 çç1 - 2 ÷÷ = 3(2 - 3)
\M = ml = m =
2 2 è ø
2. (c) The field lines remain continuous, emerging from one 26. (c) t = MB sinq = m × (2l) × B sinq
face of the solenoid and entering into the other face. = 10–4 × 0.1 × 30 sin 30° = 1.5 × 10–4 Nm
3. (d) 4. (b) 5. (c) 27. (c) The potential energy of a magnetic dipole m placed in
6. (c) The magnetic moment of a bar magnet is thus equal r r
an external magnetic dipole is U = - m.B .
to the magnetic moment of an equivalent solenoid
Therefore, work done in rotating the dipole is-
that produces the same magnetic field.
W = DU = 2mB = 2 × 5.4 × 10-6 × 0.8
7. (c)
= 8.6 × 10-6 Joule.
8. (c) 9. (a) 28. (a) Magnetic dipole moment l
10. (a) (A) ® 3; (B) ® 4; (C) ® 2; (D) ® 1 M=m×l M' = m × r
11. (b) Magnetic moment, M = ml From figure
r sin 30º r sin 30º
pr 3l
M
= m , where m is the polestrength.
l= or r =
3 p r 30º 30º
l m ´ 3l 3 60º
Therefore distance between poles so, M' = m × r = = M
p p
l ml M 29. (d) Torque, t = MB sin q
= ( l 2) + ( l 2) =
2 2 So, M' = =
2 t 0.032 0.032 ´ 2
2 2 Þ M= = = = 0.4 J / T
12. (a) B sin q 0.16 ´ sin 30o 0.16 ´ 1
13. (c) Magnetism of a magnet falls with rise of temperatue 30. (b) Magnetic intensity on end side -on position is twice
and becomes practically zero above curie temperature. than broad side on position.
14. (a) 15. (d) 16. (a) 17. (b) 1
31. (d) As F µ 4 and r becomes twice, therefore, F becomes
18. (a) Magnetic field due to a bar magnet in the broad-side r
on position is given by 1 1 1
µ0 M 4
= times \ ´ 8 = 0 .5 N .
B = ; M = ml . 2 16 16
4p é 3/ 2
2 l2 ù 32. (b) As breadth of each part is half the original breadth,
ê r + ú
ë 4 û therefore, pole strength becomes half (i.e. m/2).
After substituting the values and simplifying we get 33. (c) Taking distances from the centre of the magnet,
B = 6 × 10–5 A -m B1 æ x 2 ö
3
æ 2 x +1ö
3

19. (a) Magnetic moment = M; Initial angle through which = çç ÷÷ = ç ÷ = 8 : 1, approximately.


B 2 è x1 ø è x +1 ø
magnet is turned (q1) = 90º and final angle which
magnet is turned (q2)= 60º. Work done in turning the m1m 2
34. (d) F=k
magnet through r2
90º(W1) = MB (cos 0º – cos 90º)= MB (1–0) = MB. 35. (b) Work done in rotating the magnetic dipole from
Similarly, W2 = MB (cos 0º – cos 60º) position q1= 0° to q2= 180°
Q W = MB (cosq1– cosq2)
æ 1 ö MB
= MBç1 - ÷ = . \ W = MB (cosq° – cos 180°) = 2MB
è 2ø 2
\ W1 = 2W2 or n = 2. 36. (a) Intensity of magnetisation
20. (d) 21. (d) M 1.2
Ιm = = = 4 ´ 104 A m -1
22. (a) For stable equilibrium V (15 ´ 2 ´ 1)10 -6
U = –MB = – (0.4) (0.16) = – 0.064 J 37. (a) As BI = m0MIM = m0(I + IM)
23. (b) t = MB sin q = 0.1 × 3 × 10–4 sin 30º Here, I = 0
or t = 1.5 × 10–5 N–m. Then m0MI = m0(IM)
Þ IM = MI = 106 × 0.13 10–5 A
EBD_7418
632 PHYSICS

38. (b) The total magnetic moment per unit volume. (103 to 105)
(8.52 ´1028 ) ´ (2 ´ 9.27 ´10-24 ) 1
= 68. (b) As magnetic susceptibility c m µ , therefore
1 T
= 1.58 × 106 A/m c 2 T1 c2 273 - 73 200
= Þ = = =2
39. (d) The strength of the earths magnetic field is not c1 T2 0.0060 273 - 173 100
constant. It varies from one place to other place on
c2 = 2 × 0.0060 = 0.0120
the surface of earth. Its value being of the order of
10–5 T. 69. (b) 70. (d) 71. (c)
40. (a) The earth’s core is hot and molten. Hence, convective 72. (d) Iron, cobalt and nickel are ferromagnetic substances.
current in earth’s core is responsible for it’s magnetic 73. (c) Here, r = 30cm = 0.3cm
field. m0 M
we know = BH = 3.6 ´ 10 –5
41. (d) Relative magnetic permeability 4pr 3

magnetic permeability of material (μ) 3.6 ´10–5


mr = (0.3)3
permeability of free space (μ 0 ) Þ M=
10–7
It is a dimensionless pure ratio and for paramagnetic
Hence, M = 9.7 Am2
materials mr > 1.
42. (a) For paramagnetic substance magnetization M is 6
74. (a) Here, 2l = 8 cm , l = 4 cm , d = = 3 cm .
proportional to magnetising field H, and M is positive. 2
43. (b) 44. (a) 45. (d) At neutral point,
46. (b) Magnetic meridian of a place is defined as the m0 M
H =B=
vertical plane which passes through the imaginary 4 p (d 2 + l 2 )3 / 2
line joining the magnetic North and South-poles.
M M
This pane would intersect the surface of the Earth in = 10 - 7 =
-2 3 1250
a longitude like circle. (5 ´ 10 )
47. (d) 48. (a) 49. (c)
\ M = 1250 H = 1250 ´ 3.2 ´ 10 -5 Am 2
50. (a) Relative magnetic permeability
m M 1250 ´ 3.2 ´ 10 -5
m r = o Þ m = m o ´ mr m= = A m.
m 2l 8 ´ 10 -2
51. (b) 52. (a) 1
= 0.5 Am = 0.5 ´ ab amp ´ 100 cm
53. (a) Diamagnetic substance do not obey Curie’s law and 10
cm is independent of T. = 5 ab-amp cm.
54. (a) H 0.50 0.50 ´ 2
75. (c) B= = = = 1/ 3
55. (b) 56. (a) 57. (d) cos q cos 30º 3
58. (a) mr < 1 and er > 1. 76. (c)
59. (c) 77. (a) Because of high permeability of the iron, the entire
60. (d) The magnetic permeability of the material magnetic field will pass through iron, and so rest space
B 4H becomes free from magnetic field.
µ= = =4
H H 60 60
61. (d) Materials suitable for permanent magnets should 78. (d) Here, T1 = = 5 s, T2 = = 15 s
12 4
have high retentivity, high coercivity and high
M1 T22 + T12 152 + 5 2 250 5
permeability. = = = =
62. (d) When the temperature of a magnetic material decreases, M 2 T22 - T12 15 2 - 52 200 4
the magnetization remains the same in a diamagnetic 79. (c) Horizontal component of earth’s field,
material. H = B cos q, since, q = 60° H
63. (b) Beyond curie temperature, ferormagnetic material turns q
1
into paramagnetic material, as if ferromagnetic domains 3.6 ´10-5 = B ´
become random. 2 ®
-5 B V
64. (d) 65. (c) 66. (a) Þ B = 7.2 ´ 10 Tesla
67. (a) cd < cp < cf 80. (a) The torque acting on the magnet of magnetic moment
For diamagnetic substance cd is small negative (10–5) M, when held at angle q to magnetic field B, t = MB
sin q
For paramagnetic substances cp is small and positive
t = MB sin 30° = 0.5 × 10–5 Nm.
(10–3 to 10–5)
= 5 × 10–6 Nm
For ferromagnetic substanes cf is very large
MAGNETISM AND MATTER 633

81. (a) We know that mr = 1 + x 92. (a) The volume of the cube is
= 1 + 5500 = 5501 V = (10–6 m)3 = 10–18 m3
\ µ = mrm0 = (5501) × (4p × 10–7) Net dipole moment mnet = 8 × 1010 × 9 × 10–24 A m2
= 6.9 × 10–3 = 72 × 10–14 A m2
Intensity of magnetization is magnetic moment
82. (c) Here, H = 2 × 103 A m–1, B = 8p T, m0 = 4p × 107
developed per unit volume.
m mH B m net
Since mr = = = \ magnetization, M =
m0 m0 H m0 H volume
8p 72 ´ 10 –14 A m 2
= = 72 ´ 10 4 A m –1 = 7.2 × 105 A m–1
= -7 = 104 10 –18 3
m
4p´10 ´ 2 ´103
93. (d) The bar magnet has coercivity 4 × 103 Am–1 i.e., it
83. (a) m = m0mr = (4p × 10–7) × 2000 = 8p × 10–4 S.I. units
requires a magnetic intensity H = 4 × 103 Am–1 to get
84. (b) From mr = 1 + cm; demagnetised. Let i be the current carried by solenoid
Magnetic suscaptibility, cm = mr – 1 having n number of turns per metre length, then by
cm = 0.075 – 1 = –0.925. definition H = ni. Here, H = 4 × 103 Am–1
m C N 60
85. (b) According to Curie's law, cm = 0 n= = = 500 turn metre–1
T l 0.12
where C is Curie constant, T = temperature H 4 ´103
Þ i= = = 8A
1 n 500
\ cma 94. (c) We can compare magnetic moments with the help of
T
cm1 deflection and vibration magnetometer both.
T 273 + 333 606
= 2 = = =2 95. (d)
c m2 T1 273 + 30 303 3 3

1 (
\ cm2 = cm1 / 2 = 0.5cm = 0.5 c. Q cm1 = c ) 96. (c) For null deflection
M1 æ d1 ö æ 40 ö
=ç ÷ =ç ÷ =
M 2 è d2 ø è 50 ø
64
125
86. (d) The compass box will be on the axial line of the magnet,
97. (c)
1 2M 1 2 ´ 2lm 98. (d) In series, same current flows through two tangent
Hence, . = . = H tan q
4p r 3 4p r3 galvanometers.
Given that H = horizontal component of the earth’s æ Ι ö
99. (a) T = 2p ç
magnetic field = 30 Am–1, q = 45°, è M BH ÷ø
r = 20 cm = 0.02 m,
M = 2 l m = 4 × 10–2 m æ Ι ö 1é Ι ù
T ' = 2p ç ÷ = ê 2p ú
è 4M BH ø 2 ë (M BH ) û
2 ´ 4 ´ 10 -2 m
Hence, = 30 × tan 45°=30 × 1; 1
4 p (0.2)3 = ´ 2 = 1 second.
2
30 ´ 4p ´ (0.2)3
\m = = 37.7 Am 100. (b) Here, d1 = 20 cm, M2 = 2 M1, d2 = ?
2 ´ 4 ´ 10-2
M 2 d 32 1/ 3
87. (c) tan d =
V
=
V
=
1 = = 2 Þ d2 = 2 d1 = 20 (2)1/ 3 cm
H M1 d13
3V 3
\ d = 30º = p / 6 radian tan q 2 d13 r3 1
101. (a) = 3 = =
H 0.50 0.50 ´ 2 tan q1 d 2 [r (3) ]
1/ 3 3 3
88. (c) B= = = = 1/ 3
cos q cos 30º 3 1 tan 60 3 1
tan q 2 = tan q1 = = = \ q 2 = 30º
3 3 3 3
H0 æH ö
89. (a) tan q = Þ q = tan -1 ç 0 ÷ 102. (a)
V
tan q = , tan q¢ =
V
;
tan q¢
=
1
H è H ø H H cos x tan q cos x
90. (a) Given M = 8 × 1022 Am2 103. (b) We know that
d = Re = 6.4 × 106m B
BH = tan q or B = BH tan q = 0.34 × 10 tan 30°
–4
m0 2M
Earth’s magnetic field, B = .
4p d3 = 1.96 × 10–5 T
4 p ´ 10 -7 2 ´ 8 ´ 1022 104. (c) The volume of the needle,
= ´ @ 0.6 Gauss
4p (6.4 ´ 106 )3 V = (3 × 10–2) × (1 × 10–3) × (0.5 × 10–3) = 1.5 × 10–8 m3
91. (a) The mass of the needle = rV
EBD_7418
634 PHYSICS

= 7900 × 1.5 × 10–8 So, the declination varies between 11.3° W to 11.3° E .
= 1.183 × 10–4 kg 3. (d) We know that a permanent magnet is a substance which
The number of atoms in the needle at room temperature retain ferromagnetic property for a
é1.185 ´10-4 ù long period of time. The individual atoms in a
23
= ê -3
ú ´ 6.02 ´ 10 ferromagnetic material possess a dipole moment as in a
ë 56 ´ 10 û paramagnetic material. However, they interact with one
= 1.27 × 1021 another in such a way that they spontaneously align
The needle's dipole moment themselves in a common direction over a macroscopic
1
(1.27 ´1021 ) ´ (2.1 ´ 10 –23 )
volume i.e., domain.
M=
10 Hence, in a permanent magnet at room temperature,
= 2.7 × 10–3 J/T domains are all perfectly aligned.
T 4. (b) The electric field lines, do not form a continuous path
105. (a) We know that T1 = 2p MBH … (i) while the magnetic field lines form the closed paths.
1

Where BH1 = 24 × 10–6 T q


The magnetic field produced by, wire
Gauss’s law states that, Ñò E.ds = e
s 0
for electrostatic
field. So, it does not contradict for electrostatic fields as
µ0 i –7 (18)
B= . = (2 ´10 ) ´ = 1.8 ´ 10 -6
T the electric field lines do not form closed continuous
2p r 0.20
path.
Now BH 2 = BH1 + B = 42 × 10–6 T According to Gauss’ law in magnetic field,
T2 = 2p
I
MBH 2 … (ii) Ñò E.ds = 0
s
Using equations (i) and (ii) , and substituting the values, It contradicts for magnetic field, because there is a
we get magnetic field inside the solenoid and no field outside
T2 = 0.076 s the solenoid carrying current but the magnetic field lines
form the closed path.
5. (b) According to the Curie law, the intensity of
Exercise - 2 magnetisation (I) is directly proportional to the magnetic
1. (c) Toroid is a hollow circular ring on which a large number field induction and inversely proportional to the
of turns of a wire are closely wound. Thus, in this case temperature (t) in kelvin.
magnetic field is only confined inside the body of toroid. So, I magnetisation
So no magnetic field outside the toroid and magnetic B (magnetic field induction)
µ
field only inside the toroid. t(temperature in kelvin)
In case of toroid, the magnetic field is in the form of I2 B2 t1
concentric magnetic lines of force and there is no Þ I1
= B ´t ... (i)
1 2
magnetic field outside the body of toroid. This is As given that :I1 = 8 Am–1, I2 = ?
because the loop encloses no current. Thus, the B1 = 0.6 T, t1 = 4K
magnetic moment of toroid is zero. B2 = 0.2 T, t2 = 16K
In other case, if we take r as a large distance outside
by putting the value of B1, B2, t1, t2 I1 in equation (i)
1
the toroid, then mµ 3 . Which is not possible. 0.2 4 I2
r So, ´ =
2. (a) Magnetic declination is an angle between angle of 0.6 16 8
1 2
magnetic meridian and the geographic meridian. We get, I2 = 8 ´ = A/m
As the earth’s magnetism, the magnetic field lines of 12 3
q
the earth resemble that of a hypothetical magnetic 6. (c) Net magnetic dipole moment = 2 Mcos
dipole located at the centre of the earth. 2
q
The axis of the dipole does not coincide with the axis of As value of cos is maximum in case (c) hence net
2
rotation of the earth but is presently tilted by 11.3° magnetic dipole moment is maximum for option (c).
(approx) with respect to geographical of axis earth. This 7. (a)
results into two situations as given in the figure.
8. (c) The susceptibility of ferromagnetic substance
N N decreases with the rise of temperature in a complicated
11
.3 manner. After Curies point in the susceptibility of
º
.3

S º S
11

W E W E
ferromagnetic substance varies inversely with its
N absolute tempearture. Ferromagnetic substance obey’s
11.3º N Curie’s law only above its Curie point.
S S
MAGNETISM AND MATTER 635

9. (c) Magnetic field in solenoid B = m0n i H2


or = cot 2 q1 + cot 2 q 2
B V 22
Þ m = ni (Where n = number of turns per unit length) i.e.,cot q = cot2 q1 + cot2 q2
0
17. (c) Given : Magnetic moment, M = 6.7 × 10–2 Am2
B Ni 100i
Þ = Þ 3 ´ 103 = Þ i = 3A Magnetic field, B = 0.01 T
m0 L 10 ´ 10-2 Moment of inertia, I = 7.5 × 10–6 Kgm2
m0C I 2p
10. (b) According to Curie's law, cm = Using, T = 2p = ´ 1.06 s
T MB 10
1 Time taken for 10 complete oscillations
where C is Curie constant, T = temperature \ c m a t = 10T = 2p × 1.06 = 6.6568 » 6.65 s
T
18. (b) When a magnet is cut into pieces, each piece becomes
c m1 T 273 + 333 606 ml M
= 2 = = =2 new magnet. M ¢ = = .
c m 2 T1 273 + 30 303 2 2
B
\ c m2 = c m1 / 2 = 0.5c m = 0.5 c 19. (b) B = m0 mr H Þ m r µ = slope of B-H curve
1 H
(Q cm 1
=c ) According to the given graph, slope of the graph is
highest at point Q.
11. (d) A paramagnetic sample display greater magnetisation 20. (a) Rod gains gravitational potential energy which comes
when cooled, this is because at lower temperature, the from energy of current source.
tendency to disrupt the alignment of dipoles (due to 21. (b)
magnetising field) decreases on account of reduced 22. (d) Magnetic field of earth is due to moving charged
random thermal motion. particles in the atmosphere. With increase in
12. (a) Magnetic susceptibility c for dia-magnetic materials temperature, the magnetic moment of magnet
only is negative and low |c| = –1; for paramagnetic decreases.
substances low but positive |c| = 1 and for 23. (b)
ferromagnetic substances positive and high |c| = 102.
13. (c) Taking distances from the centre of the magnet, Exercise - 3
N
3 3
B1 æ x 2 ö æ 2 x + 1 ö
=ç ÷ =ç ÷ = 8 : 1, approximately.
B2 çè x1 ÷ø è x + 1 ø 1. (b)
S P N R
14. (b) Electromagnets are magnets, which can be turnd on Q
d d
and off by switching the current on and off. R BP
As the material in electromagnets is subjected to cyclic
changes (magnification and demangetisation), the BR
BQ S
hysteresis loss of the material must be small. The
material should attain high value of I and B with low 2 2
æ m 0 2M ö æ m0 M ö
BR = BP2 + BQ çè 4 p 3 ÷ø + çè 4 p 3 ÷ø
2
value of magnetising field intensity H. As soft iron has =
small coercivity, so it is a best choice for this purpose. d d
15. (b) Graph [A] is for material used for making permanent m M ì m M ü
= 5 0 3 = 5 B , í BQ = 0 = Bý
magnets (high coercivity) 4p d î 4 p d 3
þ
Graph [B] is for making electromagnets and 2. (b) We know that, magnetic dipole moment
transformers. M = NiA cosθ i.e., M µ cosθ
16. (d) If q1 and q2 are opparent angles of dip
When two magnetic fields are inclined at an angle of
Let a be the angle which one of the plane make with 75° the equilibrium will be at 30°, so
the magnetic meridian. 1
v cos θ < cos(75°, 30°) < cos 45° <
v 2
tan q1 = Þ i.e., cos a = H tan q …(i) x 15
H cos a 1 < [ x » 11
v v 2 2
tan q2 = , Þ i.e., sin a = …(ii) r NiA
H sin a H tan q2 3. (c) M (mag. moment/volume) =
Squaring and adding (i) and (ii), we get Al
2 Ni (500)15
æV ö æ 1 1 ö = = = 30000 Am–1
cos 2 a + sin 2 a = ç ÷ ç + ÷ l 25 ´ 10 –2
è H ø è tan q1 tan 2 q2 ø
2
1
4. (d) In magnetic dipole Force µ
V2 r4
i.e., 1 = 2 éëcot q1 + cot q2 ùû
2 2
H In the given question,
Force µ x– n Hence, n = 4
EBD_7418
636 PHYSICS

5. (b) t = MB sinq A
µ0 I 2.5 5.0 ´ 10-7
t = iAB sin90º B= = 2 × 10–7 × = T
t l l 2 pR R R
\ A= According to right-hand-palm rule no. 1, the field B is
iB
Also, A = 1/2 (BC) (AD) B
l/2 l/2 directed horizontally along BH at a point below the cable,
D C
l and opposite to BH at a point above the cable. Therefore,
2
neutral points will be obtained above the cable. At these
1 1 ælö 3 2 points, will be equal and opposite to BH. Thus
But (BC)(AD) = (l ) l 2 -ç ÷ = l
2 2 è2ø 4
5.0 ´ 10-7
1 = BH = 0.33 × 10–4 .
3 2 t æ ö2 R
Þ (l ) = \ l = 2ç t ÷
4 Bi ç 3 B.i ÷
è ø 5.0 ´ 10-7
or R = = 15 × 10–3 m = 1.5 cm
0.33 ´ 10-4
6. (d) d1 = 40º , d 2 = 30º , d = ? Thus, the line of neutral points lies above and parallel to
2 2 the cable at a distance of 1.5 cm from it.
cot d = cot 2 d1 + cot 2 d 2 = cot 40º + cot 30º
2rH
cot d = 1.192 + 3 = 2.1 9. (b) For a tangent galvanometer, i = tan q
m0 N
\ d = 25º i.e. d < 40º.
Now 8 W and 8 W in paralle = 4 W.
7. (a) Given that : B1 = 1.2 × 10–2 T, orientation of dipole with emf 4 8W
the field B1, q1 = 15° B2 i= = = 1ampere
resistance 4 T.G.
60°
Hence, orientation of dipole with B2, 45° r tan q m0i
\ For T.G. = , 8W
q2 = 60° – 15° = 45° (figure) N 2H
N 15° i T.G.
S
B1 r = radius
rA tan q A rB tan q B
As the dipole is in equilibrium, therefore, the torque on \ =
NA NB 4V
the dipole due to the two fields must be equal and
rA tan 30° rB tan 60°
opposite. If M be the magnetic dipole moment of the Þ =
NA NB
dipole, then
t1 = t2 or MB1 sin q1 = MB2 sin q2 8 ´ 1 16 ´ 3
N = number of turns Þ =
B1 sin q1 1.2 ´ 10-2 sin15° 3N A NB
or, B2 = =
sin q2 sin 45° 8 16 3
-2 Þ = Þ N B = 12 turns
1.2 ´ 10 ´ 0.2588 3 ´2 NB
= = 4.4 × 10–3 Tesla
0.7071 10. (c) Initially magnetic moment of system
8. (a) The situation is shown in figure. The horizontal
component of earth’s magnetic field at the location of M 1 = M 2 + M 2 = 2M and moment of inertia
the cable (angle of dip q = 0) is I1 = I + I = 2I.
Magnetic North Finally when one of the magnet is removed then
North M2 = M and I2 = I
10° Geographic
Magnetic Meridean
I
Meridean I Cable So, T = 2p
Geographic 10° Geographic M BH
West East
Current
T1 I1 M 2 2I M
= ´ = ´
T2 I 2 M1 I 2M
South Magnetic
South 25 / 4
Þ T2 = 1/ 4 = 2sec
BH = B cos q = B cos 0 = B = 0.33 Gauss 2
S N
= 0.33 × 10–4 Tesla 11. (b)
BH is directed horizontally in the magnetic meridian. The W E
magnetic field produced by the cable at a distance of R r N
meter is given by S

BH
MAGNETISM AND MATTER 637

B = BH 14. (b)
m 0 2M 15. (a) Here, mass M = 50 milligram = 50 × 10–3 gram
or . = 0.3 mV
4p r 3
2M O
or 10 -7 ´ = 0.3
(20)3 mV
Mg
\ M = 1.2 × 103 emu. 2l
m strength of each pole, m =98.1 ab amp-cm, g = 981 cm/s2,
12. (a) WH = VAft = Aft
d V= ?
0.6 In equilibrium, mV× 2l = Mg × l
or WH = × 0.722 × 50 = 277.7 × 10-5 joule
7.8 ´ 103 Mg 50 ´ 10-3 ´ 981
BV = or BV = = 0.25 gauss
T2 M1 M1 2m 2 ´ 98.1
13. (c) = = =2
T1 M2 1
M
4 1
\ T2 = 2T1 = 3 s
EBD_7418
638 PHYSICS

21. ELECTROMAGNETIC INDUCTION

Exercise - 1 df
= 24 + 7 = 31 mv
dt
df Direction of current is from left to right according to
1. (a) Induced emf, e = –
dt Flemings right hand rule.
2. (a) Farady’s law states that time varying magnetic flux 20. (a) When switch is closed , the magnetic flux through the
can induce an e.m.f. ring will increase and so ring will move away form the
3. (c) 4. (a) solenoid so as to compensate this flux. This is according
5. (a) Emf will be induced in the circular wire loop when flux to Lenz's law.
through it changes with time. df dB A dB
Df 21. (a) Induced e.m.f. e = = = A0
e =- dt dt dt
Dt
when the current is constant, the flux changing æ 4 B 0 - B0 ö
through it will be zero. = A0 ç ÷ = 3 A 0 B0 / t
è t ø
When the current is
I 22. (a) As the magnetic field increases, its flux also increases
decreasing at a steady rate into the page and so induced current in bigger loop
then the change in the flux will be anticlockwise. i.e., from D to C in bigger loop
®
(decreasing inwards) on the B and then from B to A in smaller loop.
right half of the wire is equal Df Df
®
B 23. (c) = e = iR Þ Df = (iDt )R = QR Þ Q =
to the change in flux Dt R
(decreasing outwards) on the Circular
24. (b)
left half of the wire such that loop 25. (c) e = (5 ´10 -3 ) (1 / 0.1) = 0.05 V .
Df through the circular loop 26. (a) Change in flux = 2 B A N
is zero. 2 ´ 0.3 ´ 200 ´ 70 ´10 -4
6. (c) 7. (a) \ Induced e.m.f. =
0.1
8. (c) As I increases, f increases 27. (a) According, to Faraday’s law of induction
\ Ii is such that it opposes the increases in f. df
Hence, f decreases (By Right Hand Rule). The induced Induced e.m.f. e = - = - (100t )
dt
current will be counterclockwise. Induced current i at t = 2 sec.
9. (b) 10. (b) 11. (d) e 100 ´ 2
= =+ = + 0.5Amp
12. (c) A - 1; A charged ring can produced electric field out of R 400
the centre. df (W2 - W1 )
B - 1, 2, 4; A charged rotating ring can produce electric 28. (b) = R tot = (R + 4R)W = 5R W
field out of centre, magnetic and dipole moment. dt t
ndj - n(W2 - W1 )
C- 2, 4; Current carrying produces magnetic field at the i= = .
centre. R tot dt 5Rt
D- 3; Alternating current can produce induced electric (Q W2 & W1 are magnetic flux)
29. (b) In both the cases, the magnetic flux will change, and
field. so there is an induced current.
13. (d) 14. (b) 15. (a) 16. (d)
17. (b) The charge through the coil = area of current-time 30. (a) e =
- df -d 2
dt
=
dt
( )
6t - 5t + 1 = -12t + 5
(i – t) graph e = – 12 (0.25) + 5 = 2 volt
1 e 2
q = ´ 0.1 ´ 4 = 0.2 C i = = = 0.2A.
2 R 10
Df 31. (b) Magnetic flux linked with the loop is f = Bpr2
q= Q Change in flux (Df) = q × R df dr
R | e |= = Bp × 2 r
Df dt dt
q = 0.2 = dr
10 When r = 2 cm, = 1 mm s–1
Df = 2 weber dt
18. (d) f = BA cos q = 2.0 ´ 0.5 ´ cos 60º e = 0.025 × p × 2 × 2 × 10–2 × 10–3
2.0 ´ 0.5 = 0.100 × p × 10–5 = p × 10–6 V = pmV
= = 0.5 weber. 32. (d) According to Lenz’s law
2
df - n ( f2 - f1 )
19. (d) f = 6t 2 + 7t + 1 Þ = 12t + 7 33. (a) e=
dt t
At time, t = 2 sec.
ELECTROMAGNETIC INDUCTION 639

-50 (1 ´ 10 -6 - 31 ´ 10 -6 )
42. (d) | e | = Bl V sin q
= 43. (d) e = [Bvl]
0.02 3 3 2
= 7.5 × 10–2 V
34. (b) B = m0 ni = (4p × 10–7) (200 × 10–2) × 1.5 |e|= 1 2 0
= 3.8 × 10–2 Wb / m2 0 0 1
Magnetic flux through each turn of the coil (as length of conductor is 0iˆ + 0jˆ + 1kˆ )
f = BA = (3.8 × 10–2) (3.14 × 10–4) = 1.2 × 10–5 weber
\ | e | = 3V
When the current in the solenoid is reversed, the
44. (c) e = 200 sin100 pt
change in magnetic flux
\ e0 = 200, w = 100p
= 2 × (1.2 × 10–5) = 2.4 × 10–5 weber
Now, NABw = e0
df 2.4 ´ 10 -5
Induced e.m.f. = N = 100 ´ = 0.048 V. e0 200
dt 0.05 \ B= or B =
35. (c) Initial magnetic flux linked with the coil is NAw 1000 ´ (25 ´ 10-2 ) 2 ´ 100p
fi = BA cos q = 0.1 × 200 × 10–4 × cos0º = 2 × 10–3 Wb or B = 0.01 T
Final magnetic flux linked with the coil is ff = 0 45. (a) If a wire, l meter in length, moves perpendicular to a
NDf – N(ff – fi ) magnetic field of B weber/meter2 with a velocity of v
\ By Faraday's law, e = – =
Dt Dt meter/second, then the e.m.f. induced in the wire is
- df given by
36. (b) The individual emf produced in the coil e = V = B vl volt.
dt
|e| 1 df Here, B = 0.30 × 10-4 weber/meter 2,
\ The current induced will be i = Þi= v = 5.0 meter/second and l = 10 meter.
R R dt
dq dq 1 df 1 BA \ B = 0.30 × 10–4 × 5.0 × 10 = 0.0015 volt.
But i = Þ = Þ ò dq = ò df Þ q = 46. (d) According to Faraday’s law of electromagnetic induction,
dt dt R dt R R
37. (b) Induced emf produced between the centre and a point Ldi
on the disc is given by Induced emf, e =
dt
1
e = wBR 2 æ 5–2 ö
2 50 = L ç ÷
Putting the values, è 0.1sec ø
w = 60 rad/s, B=0.05 Wb/m 2 50 ´ 0.1 5
and R = 100 cm = 1m Þ L= = = 1.67 H
3 3
1 2 47. (a) Given: No. of turns N = 1000
We get e = ´ 60 ´ 0.05 ´ (1) = 1.5V
2 Face area, A = 4 cm2 = 4 × 10–4 m2
df d dB Change in magnetic field,
38. (a) e = = ( NBA ) = NA DB = 10–2 wbm–2
dt dt dt
Time taken, t = 0.01s = 10–2 sec
æ 0.1 - 0.05 ö
= 100 ´ 0.1 ´ 0.05 ´ ç = 0.5 V Emf induced in the coil e = ?
è 0.05 ÷ø Applying formula,
nAdB -d f
39. (c) i = e = dt 20 ´ (25 ´10 -4 ) ´1000 Induced emf, e =
= = 0.5A dt
R R 100 æ DB ö
e 1 df = Nç ÷ A cos q
40. (c) i = = è Dt ø
R R dt 1000 ´ 10-2 ´ 4 ´ 10 -4
Here df = B × A = (2 ´ 10-5 ) ´ (0.5 ´ 10+3 ´ 5) = = 400 mV
10-2
dt = time taken by the wire to fall at ground 48. (c) 49. (c) 50. (d)
= (2 h / g)1/ 2 = (10 /10)1/ 2 = 1sec. 51. (d) Mutual inductance between two coil in the same plane
with their centers coinciding is given by
1 é (2 ´ 10 -5 ) ´ (0.5 ´ 103 ´ 5) ù
\ i= ê ú = 0.02 amp. m 0 æ 2p2 R22 N1 N 2 ö
2.5 ëê 1 ûú M= ÷ henry.
4p çè R1 ø
N 2000 20000
41. (b) n= = = 52. (a) The mutual inductance between two planar concentric
l 0.3 3
m pr 2
d
x = ( NBA ) = NA
dB rings of radii r1 and r2 (r1 > r2) is given by M = 0 2
dt dt 2r1
Nf 100 ´ 10 -5
Since B = µ0nI 53. (d) Nf = Li Þ L = = = 0.20 mH
i 5
dt
Þ x = ( mNAn ) Þ x = 0.024 V 2 2
dt 54. (b) Self inductance = m 0 n AL = m 0 n (l ´ b) ´ L
EBD_7418
640 PHYSICS

So, when all linear dimensions are increased by a factor The effective circuit is shown in figure.
of 2. The new self inductance becomes L¢ = 8L.
55. (b) Rate of work = i
W æ Bvl ö 0.5W
= P = Fv; also F = Bil = B ç 4W 2W
÷l 0.5 V
t è R ø
B 2v 2l 2 (0.5) 2 ´ (2)2 ´ (1) 2 1
ÞP = = = W The equivalent resistance of the circuit
R 6 6
[Here W = Watts] 4´ 2
r= + 0.5 = 1.83 W
4+2
dA V 0.5
B. Now, i = = = 0.27 A
56. (b) e | d f / dt | dt A v
R 1.83
I= = = 61. (b) The e.m.f. induced is directly proportional to rate at
R R R h
× × × × × ×b× × × × × × × which flux is intercepted which in turn varies directly
××××××××××××× as the speed of rotation of the generator.
d æ1 ö dh ×××××××××××××
B ç h.b÷ Bb × × × ×B × × × × ×C× × × ×
dt è 2 ø dt ××××××××××××× f y
= µ µ Bbv × × × × × × × × × × × × × 62. (c) f = LI Þ L = = henry
R R ××××××××××××× I x
\ bµt Þ Iµt 63. (d) 64. (d) 65. (a)
66. (d) 2
P = I R when I becomes half, then P becomes one-
57. (c) fourth, heat per unit time depends on square of I.
P v Q When I is reduced P is substancially reduced.
r
67. (d) Electromagnetic damping certain galvanometers have
L a fixed core made of non-magnetic metallic material.
When the coil oscillates, the eddy currents generated
x in the core oppose the motion and bring the coil to
x + dx rest quickly.
68. (b) As f = NAB cos 90° = 0
Figure shows a rod PQ of length L which moves with a e = e0 sin 90° = e0 = maximum
uniform velocity v parallel to a long straight wire q is the angle between the field and normal to the
carrying a current i. Here a magnetic field is produced plane of the coil.
by the current carrying wire and the rod moves in this 69. (a) In the given case, there is no component of velocity,
field. Consider a small element of length dx of the rod perpendicular to the magnetic field and so e = Bvl sin 0°.
at a distance x and (x + dx) from the wire. 70. (c) E.m.f. will remain same because change in area per unit
The emf induced across the element time will be same in both cases.
de = B v dx ..(i)
We know that magnetic field B at a distance x from a d dI NM I
71. (a) E= ( NMI ) Þ E = NM Þ E=
wire carrying a current i is given by dt dt t
m i E MI
B= 0. ...(ii) emf induced per unit turn = =
2p x N t
From eqs. (i) and (ii), di é ( 4 - 2) ù
m i 72. (b) e =M or 8 = M ê ú
de = 0 v dx dt ë 0.05 û
...(iii)
2 px 8 ´ 0.05
The emf e induced in the entire length of the rod PQ is \ M= = 0.2 henry
2
given by
73. (b) Mutual Inductance of two coils
Qm i
ò
e = de = ò 0
P 2p x
v dx M = M1 M 2 = 2mH ´ 8mH = 4mH
dI 0.5 - 0
r+L m i m0 r + L dx m iv 74. (d) Given : M = 0.75 H and = = 50 A / s
=ò r
0
2p x
v dx =
2p
iv
rò x
= 0 [log e x ]rr + L
2x
dt 0.01
\ Average induced e.m.f. in secondary coil
m iv m iv æ r + Lö dI
= 0 [log e ( r + L) - log e r ] = 0 log e ç e = M = 0.75 ´ 50 = 37.5 V
2p 2p è r ÷ø dt
r r r 75. (c) The magnitude of induced e.m.f. is given by
58. (a) l, v, and B are coplanar.. |e| = Blv
v = 300 m/min = 5m/s
59. (b) Till front side of the loop moves into the field the emf |e| 2
induced e = Bvl across it. When rear side comes in the \ B= = = 0.8 tesla
field, the emf is induced across it. l v 0.5 ´ 5
76. (d) The e.m.f. is induced when there is change of flux. As
60. (a) The induced emf across the sliding wire in this case there is no change of flux, hence no e.m.f.
e = Bvl = 0.5 × 4 × 0.25 = 0.5 V will be induced in the wire.
ELECTROMAGNETIC INDUCTION 641

77. (b) L = m0 nI
L2 m ds
ring is dQ = Q , where r is the radius of the ring.
\ L =m ----(Q n and I are same) 2 pr
1 0
\ L2 = mrL1 = 900 × 0.18 = 162 mH The force exerted on it is dFt = dQEt, and the resulatnt
78. (a) Nf = LI torque is dt = rdFt.
LI 8 ´ 10 -3 ´ 5 ´10 -3 Thus, the total torque experienced by the ring is
\ f= =
N 400 ds Q
t = ò dt = ò rQ
2p ò
Et = E t ds
-7 m0 2pr
= 10 = Wb
4p The induced electromotive force along the ring is
79. (d) No change in flux, hence no force required. directly proportional to the rate of change in the
80. (b)
magnetic flux, we have
81. (c) The electric field/emf is induced neither in sides AD
and nor in BC. Unless the metallic square loop is dF dB
entering or leaving the magnetic field and the flux ò E t ds = dt
= pr 2
dt
linked with it is changing. As result of the torque, the ring, which has a moment
82. (b) Length of conductor (l) = 0.4 m; Speed (v) = 7 m/s and of inertia I = mr2, starts to spin with angular acceleration
magnetic field (B) = 0.9 Wb/ m 2. Induced e.m.f. a. During a time interval dt its angular velocity changes
(e) = Blv cos q = 0.9 × 0.4 × 7 × cos 0º = 2.52 V. by
1 t Q æ 2 dB ö 1 Q
83. (a) Vel. of coil = = 2m / s
0.5 dw = adt = dt = ç pr ÷ dt = dB
2 I 2p è dt ø mr 2 2m
velocity of magnet = = 2m / s. Since the magnetic field strength increases from zero
1
As they are made to move in the same direction, their to B, the final angular velocity of the ring will be
relative velocity is zero. Therefore, induced e.m.f. = 0. w = (QB /2m)
84. (a) Here, r = 6 cm = 6 × 10–2 m, N = 20, w = 40 rads–1 92. (c) For maximum potential difference the velocity must be
B = 2 × 10–2 T, R = 8W perpendicular to the line AB, and so
Maximum emf induced, e = NABw ® v
= N(pr2)Bw v = v sin qiˆ + v cos qˆj q
= 20 × p × (6 × 10–2) 2 × 10–2 × 40 = 0.18 V
Average value of emf induced over a full cycle é3 2 ù v 3
= v ê iˆ + ˆj ú = éë3iˆ + 2 ˆj ùû
eav = 0 ël l û l
Maximum value of current in the coil. q
93. (b) In the given question, 2
eI 0.18 Current flowing through the wire, I = 1A
I= = = 0.023 A
R 8 Speed of the frame, v = 10 ms–1
Average power dissipated, Side of square loop, l = 10 cm
eI 0.18 ´ 0.023 Distance of square frame from current carrying wires
P= = = 2.07 × 10–3 W
2 2 x = 10 cm.
85. (c) Energy stored U is given by We have to find, e.m.f induced e = ?
1 1
U = L i 2 = ´ (100 ´10 -3 ) (1) 2 = 0.05 J. According to Biot-Savart’s law
2 2 m Idlsin q 4p´10-7 1´ 10-1
l1 1 N1 1 2 2 B= 0 = ´ = 10–6
= From L = m0 N A a N 4p x 2 4p
( )
2
86. (b) Given l = 2 and 10 -1
2 N2 2 l l
2 2 Induced e.m.f. e = Blv = 10–6 × 10–1 × 10 = 1 mV
L1 æ N1 ö æ l1 ö (1/ 2 ) 1
we get, =ç ÷ ç ÷= = 1 1
L2 è N2 ø l
è 2ø 1/ 2 2 94. (c) e.m.f. induced = BR 2 w = B R 2 (2 p n)
2 2
M
87. (b) ε= dI=0.005×I 0 cos ωt×ω 1
dt = ´ (0.1) ´ (0.1)2 ´ 2 p ´ 10 = (0.1)2 p volts
and emax = 0.005 × I0 × w = 5p 2
95. (d) Total resistance of the circuit = 4000 + 400 = 4400 W
88. (b) 89. (b)
V 440
m0 N1N 2 A 4p´ 10-7 ´ 300 ´ 400 ´ 100 ´ 10-4 Current flowing i = = = 0.1 amp.
90. (d) M= = R 4400
l 0.2
= 2.4p × 10–4 H Voltage across load = R i = 4000 × 0.1 = 400 volt.
91. (a) In accordance with Faraday's law of electromagnetic 96. (b) V = 200V; r = 10W
induction, the changing magnetic field induces an R¢ = 10 + 100W = 110W
electric field in the ring. Let us imagine the ring to be V 220
I= = = 2A
divided into differential elements of length ds and R¢ 100
denote the tangential component of the induced P = I2R = 4 × 100 = 400 W
electric field by Et. The charge on element ds of the 97. (c) There is no charge in flux through PS and so induced
EBD_7418
642 PHYSICS

emf will be zero. But So, f = B.A= B0 ( 2iˆ + 3jˆ + 4kˆ ) .L2 kˆ
VP – VB = VQ – VR
= 4B0 L2 Wb
F
Also, F = Bil or i = 2. (b) The loop can be considered in two planes, Plane of
Bl
F r Fr
ABCDA lies x-y plane whose area vector A1 = |A| k̂ , A1
\ VP – VB = VQ – VR = irQR = ´ = = L2 k̂
Bl 6 6 Bl
98. (b) Since MN is perpendicular to the magnetic field as well whereas plane of ADEFA lies in y-z plane whose area
as velocity ‘v’, hence, the induced emf developed along vector A2 = |A| î , A2 = L2 î .
MN is given as Then the magnetic flux linked with uniform surface of
v area A in uniform magnetic field is
E = eBa , (directed from M to N)
c Y
The induced emf results due to decrease of flux as the (0,L,0) D (L,L,0)
C
frame leaves the region occupied by the magnetic field. L
E
According to Lenz’s law, the direction of induced (0,L,L) L
L X
current is clockwise. All this results in magnetic force A B (L,0,0)
acting on the loop which is equal and opposite on (0,0,0)
F
Z (0,0,L)
sides MQ and NP, hence cancel each other.
\ Net force is the magnetic force on MN given by f = B.A
I
F = B a (directed upwards) A = A1 + A2 = ( L2 kˆ + L2 iˆ )
c
and B = B ( ˆi + kˆ )
The constant terminal falling velocity is achieved when 0

F is able to completely balance the force of gravity on Now, f = B.A = B0( iˆ + kˆ ) × ( L2 kˆ + L2ˆi )
the loop. = 2 B0L2 Wb
Let A = cross-section area of the loop/frame 3. (b) Induced current flow only when circuit is complete and
Then mass of the frame is given as there is a variation about circuit this problem is associated
1 a with the phenomenon of electromagnetic induction.
m = 4a A r and R = 4
sA If there is a symmetry in magnetic field of cylindrical
bar magnet is rotated about its axis, no change in flux
E v s eBv s A linked with the circuit takes place, consequently no
\ I= = eBa A=
R c 4a 4c emf induces and hence, no current flows in the ammeter
(A).
I eB 2 a v s A
Hence F = Ba =
c 4 c2 Axis
Equating F to mg, we get A
2 N
eB 2 a v s A Þ v = 16 c r g
4a A r g =
4 c2 e B2 s Bar
w
magnet
(v is independent of geometrical dimensions of the
frame.)
99. (c) Use Lenz's law. Induced emf of the current opposes
the change in flux through it.
100. (b) Given, B = 0.01 T, A = pR2 = p × (1 m)2 = pm2 S
w = 100 rads–1 w
\ The maximum induced emf emax= BAw
= 0.01 × p × 100 V = pV 4. (d) When the coil A stops moving the current in B b ecome
ze ro, it possible only if the current in A is constant. If
Exercise - 2 the current in A would be variable, there must be an
induced emf (current) in B even if the A stops moving.
1. (c) As we know that, the magnetic flux linked with uniform So there is a constant current in same direction or
surface of area A in uniform magnetic field is counter clockwise direction in A as in B by lenz's law.
f = B.A 5. (a) By Lenz's law, at (t = 0) the current in B is counter-
clockwise and the coil A is considered above to it. The
The direction of A is perpendicular to the plane of
counterclockwise flow of the current in B is equivalent
square and square line in x-y plane in a region. to north pole of magnet and magnetic field lines are
A = L2k emanating upward to coil A.
As given that, B = B0 ( 2iˆ + 3jˆ + 4kˆ ) When coil A start rotating at t = 0, the current in A is
constant along clockwise direction by Lenz’s rule. As
ELECTROMAGNETIC INDUCTION 643

flux changes across coil A by rotating it near the N- Þ E = – B.2 (a0 – a t) (0 – a) = +2aaB
pole formed by flowing current in B, in anticlockwise. 13. (d) Here, number of turns n = 100; current through the
6. (b) The self-inductance of a long solenoid of cross- solenoid i = 4A; flux linked with each turn = 4 ×
sectional area A and length l, having n turns per unit 10–3 Wb \ Total flux linked, and total
length, filled the inside of the solenoid with a material = 1000[4 × 10–3] = 4 Wb
of relative permeability is given by ftotal = 4 Þ L i = 4 Þ L = 1 H
L = µrµ0n2 Al 14. (c) L = 2mH, i = t2e–t
\ n = N/l
di
é N 2 .A ù E = - L = - L[ - t 2 e - t + 2 te - t ]
L = µrµ0 ê dt
ú .l
ë l.l û when E = 0
–e–t t2 + 2te–t = 0
æ 1ö 2t e–t = e–t t2
L = µrµ0 [N2A/l] çè L µ A, L µ ÷ø
l t = 2 sec.
As µr and N are constant here so, to increase L for a 15. (d) Induced emf in the loop is given by
coil, area A must be increased and l must be decreased. dA
e = – B. where A is the area of the loop.
7. (d) Rate of decreasing of area of semicircular ring = dt
dA d dr
= (2r)V e = – B . (p r2) = – B p 2r
dt dt dt
From Faraday’s law of electromagnetic induction r = 2cm = 2 × 10–2 m
dr = 2 mm = 2 × 10–3 m
dq dA dt = 1s
e= - = -B = - B(2rV)
dt dt 2×10–3
e = – 0.04 × 3.14 × 2 × 2 ×10–2 × V
–5
1
= 0.32 p × 10 V
= 3.2 p ×10–6V
= 3.2 p mV
16. (b)
17. (a) According to right hand palm rule, the Lorentz force
As induced current in ring produces magnetic field in on free electrons in the conductor will be directed
towards end B. Hence, the end A gets positively
upward direction hence R is at higher potential. charged.
8. (b) f = 10t 2 - 50t + 250
18. (a) According to Faraday's law of electromagnetic
df df
e =- = - (20 t - 50) \ e t =3 = -10 V induction, e =
dt dt
9. (a) Lenz's law (that the direction of induced emf is always Also, e = iR
such as to oppose the change that cause it) is direct df
consequence of the law of conservation of energy. \ iR = Þ ò d f = R ò idt
dt
10. (b) Current will be induced, Magnitude of change in flux (df) = R × area under
when e– comes closer the induced current will be current vs time graph
anticlockwise 1 1
when e– comes farther induced current will be clockwise or, df = 100 ´ ´ ´ 10 = 250 Wb
2 2
19. (d)
20. (c) From question energy stored in inductor, U = 25 × 10–3 J
Current, I = 60 mA
e– e– 1 2
Energy stored in inductor U = LI
ur ur 2
df d(NB.A) 1 –3 2
11. (d) e = - =- 25 × 10 = ´ L ´ (60 ´ 10 )
–3
dt dt 2
d 25 ´ 2 ´ 106 ´10 –3
= - N (BA cos wt ) = NBAw sin wt L= = 13.89 H
dt 3600
Þ e max = NBAw 21. (a) 22. (b)
12. (a) At any time t, the side of the square a = (a0 – a t),
where a0 = side at t = 0. Exercise - 3
At this instant, flux through the square : 1. (d) At x = 0, B = B0, so e1 = B0v0d
f = BA cos 0° = B (a0 – a t)2
At, x = d, B = B0 æç 1 + d ö÷ , so e2 = B0 æç 1 + ö÷ v0d
df d
\ emf induced E = – è aø è aø
dt
EBD_7418
644 PHYSICS

T T
B0v0 d 2 2
Now enet = e2 - e1 =
a ò Pinst ´ dt (wBA) 2
ò sin wtdt
Pavg = 0 = 0 1 ( wBA) 2
2. (c) T R T =
(b) The self-inductance of a coil L = m0n2Al where 2 R
3. ò dt ò dt
m0 = permeability of air, 0 0
n = number of turns per unit length, é 2 2 pr 2 ù
(w B pr ) ê A = ú
A = Area of cross-section and \ Pavg = 2 ûú
l = length of the solenoid 8R ëê
9. (d) f = n BA cos q = 10 B a2 cos wt
This depends on the geometry of the inductor such as
cross-sectional area, length and number of turns and e=-
df
dt
=-
d
dt
( )
10 B a 2 cos wt = 10 B a 2 sin wt ( w ) .
not on the material, even if it is made of a super 10. (d) e = B1v = (0.2 × 10–4)(1)(180 × 5/18) = 10–3 V = 1 mV
conducting material. If the superconductor is below 11. (c) Emf induced in side 1 of frame e1 = B1Vl
the critical temperature, the current will continuously mo I
flow and the inductance may not have the property of B1 =
2p (x – a/ 2)
inductance any more. \ L1 = L2; L3= 0. Emf induced in side 2 of frame e2 = B2 Vl
df wR 2 moI
4. (a) = B. B2 =
dt 2 2p (x + a/ 2)
wR 2 x
where is area swept in unit time perpendicular
2 I
to the magnetic field. 1 2
5. (b) Electric flux is given by
a v
f = B.A x–

(Q B = B0e-t/ t )
2
f = B0 pr 2 e - t / t a
a
x+
df B0 pr 2 - t / t 2
Induced E.m.f. e = = e
dt t2 Emf induced in square frame
¥ 2 e = B1Vl – B2Vl
e p 2 r 4 B02 m0 I m0 I
Heat = ò = = lv – lv
R 2tR 2p (x – a / 2) 2p (x + a/ 2)
0
6. (a) If we consider the cylindrical surface to be a ring of 1
or, e µ
radius R, there will be an induced emf due to changing (2x – a)(2 x + a)
field. 12. (c) Relative velocity = v + v = 2v
\ emf. = B.l (2v)
r df dB 13. (d) Initially flux, f = BA cos 0 = BA
ò E.dl = - dt = -A dt After rotating through an angle 90°.
dB dB R dB Flux through the coil is zero.
Þ E (2pR) = -A = -pR 2 ÞE=- So, Df = BA
dt dt 2 dt
\ Force on the electron 2p
eR dB Angular speed = w, so, time period = =T
F = - Ee = w
2 dt T
1 eR dB is time taken to rotate 90°.
Þ acceleration = 4
2 m dt Δf BA 2BAω
As the field is increasing being directed inside the So, = =
Δt T/4 π
paper, hence there will be anticlockwise induced
14. (c) The area swept by radius OC in one half circle is pr2/2.
current (in order to oppose the cause) in the ring
The flux change in time T/2 is thus (pr2 B/2). The
(assumed). Hence there will be a force towards left on
é 2p ù
the electrons. induced emf is then e = pr2B/T = Bwr2/2 êQ T =
7. (a) ë w úû
r r The induced current is then I = e/R = Bwr2 /2R
8. (b) f = B. A ; f = BA cos wt
15. (a) As we know, Magnetic flux, f = B. A
df wBA
e=- = wBA sin wt ; i = sin wt
dt R m0 (2)(20 ´ 10 -2 ) 2
2
´ p(0.3 ´ 10 -2 ) 2
2 2
æ wBA ö 2[(0.2) + (0.15) ]
Pinst = i 2 R = ç ÷ ´ R sin 2 wt
è R ø On solving
= 9.216 × 10–11 = 9.2 × 10–11 weber
ALTERNATING CURRENT 645

22. ALTERNATING CURRENT

Exercise - 1 14. (d)


15. (b) Given equation, e = 80 sin 100pt …(i)
1. (d) Alternating current cannot be measured by D.C. Standard equation of instantaneous voltage is given
ammeter because average value of A.C. for complete by e = em sinwt …(ii)
cycle is zero. Compare (i) and (ii), we get em = 80 V
where em is the voltage amplitude.
I0
2. (b) = RMS current e
2 Current amplitude Im = m where Z = impendence
Z
eo = 80/20 = 4 A.
3. (a) Peak value, I0 =
R 4 4 2
4. (b) Ir.m.s = = = 2 2 = 2.828 A.
2 2
5. (b) We know that Ι r m s = Ι 0 / 2 and Ι m = 2 Ι 0 / p 200 2
16. (d) Vrms = = 200V
Ιm 2 2 2
\ =
Ιrms p 200
Vrms 1
6. (d) In case of pure inductance cos f = 0, so no power Irms = =
dissipates. X C 100 ´ 10-6
7. (a) = 2 × 10–2 = 20mA
8. (d) When an ac voltage of 220 V is applied to a capacitor 17. (a) The phase angle between voltage V and current I is
C, the charge on the plates is in phase with the p/2. Therefore, power factor cos f = cos (p/2) = 0. Hence
applied voltage. the power consumed is zero.
As the circuit is pure capacitive so, the current 18. (d)
developed leads the applied voltage by a phase 19. (a)
angle of 90° Hence, power delivered to the capacitor 20. (c)
per cycle is 21. (d) as current any instant in the circuit will be
P = Vrms Irms cos 90° = 0. I = Idc + Iac = a + b sin wt
p 1/ 2
é T 2 ù
ê ò0 I dt ú
9. (b) In an inductor voltage leads the current by or 1/ 2
2 é1 T 2 ù
p
so, Ieff =ê T ú = ê ò ( a + b sin wt ) dt ú
current lags the voltage by . ëT 0 û
2 êë ò0 dt úû
I
10. (b) The value of r.m.s current is I rms = P 1/ 2
2 i.e.
é1 T
ëT 0
(
Ieff = ê ò a 2 + 2ab sin wt + b 2 sin 2 wt dt ú
ù
û
)
1
so power dissipated is P = I 2rms R = I 2P R 1 T 1 T 2 1
2 but as ò0 sin wtdt = 0 and ò0 sin wt dt =
T T 2
(T / 2)V0 2 + 0 V é 1 ù
1/ 2
11. (b) Vrms = = 0 . So, Ieff = ê a 2 + b 2 ú
T 2 ë 2 û
12. (b) The means or average value of alternating current or 22. (d) V = 120sin100pt cos100pt Þ V = 60 sin200pt
e.m.f during a half cycle is given by Vmax = 60V and v = 100Hz
Im = 0.636I0 or Em = 0.636E0 23. (d) The instantaneous values of emf and current in inductive
During the next half cycle, the mean value of ac will be
equal in magnitude but opposite in direction. For this æ pö
circuit are given by E =E0sinwt and i = i0sin çè w t - ÷ø
reason the average value of ac over a complete cycle is 2
always zero. So the average value is always defined respectively.
over a half cycle of ac. æ pö
13. (a) E = 8 sin wt + 6 sin 2wt \ Pinst=E.i=E0Sinwt×i0sin çè w t - ÷ø
2
æ p pö
Þ E peak = 82 + 6 2 = 10 V = E0 i0 sin wt ç sin wtcos - cos wtsin ÷
è 2 2ø
10 = -E0i0 sin wt cos wt
E rms = =5 2 V
2
EBD_7418
646 PHYSICS

1 \ initial value is not given hence the mean value will be


= - E0i0 sin 2wt (sin 2wt = 2 sin wt cos wt) difference for various time intervals.
2
If voltage applied is V = Vm sin wt then i given by eq. (1)
Hence, angular frequency of instantaneous power is 2w indicates that it is ahead of V by d where 0 < d < 90° which
V 4V0 indicates that the circuit contains R and C.
24. (d) V = 0 t Þ V = t
T/4 T 28. (a)
ìT/4 2 ü
1/ 2 29. (d)
ï ò t dt ï 30. (b) In a pure inductive circuit current always lags behind
4V0 4V0 ï 0 ï V0 p
Þ Vrms = < V2 > = 2
<t > = í ý = the emf by .
T T ï T /4 ï 3 2
ï ò dt ï æ pö
î 0 þ If v(t) = v0 sin wt then I = I0 sin ç wt - ÷
è 2ø
25. (a) The current and potential difference are in phase with
the resistance. So, the time taken would be same as time Now, given v(t) = 100 sin (500 t)
for voltage to change from (t = 0) that is peak value to E 100
and I 0 = 0 = [Q L = 0.02 H]
rms value. wL 500 ´ 0.02
200
Time taken by voltage to achieve its rms value of . æ pö
2 I 0 = 10sin ç 500t - ÷ = –10 cos(500 t)
è 2ø
200
= 200 cos(100pt) 31. (a) 2pnt = 400pt \ n = 200
2
I 0 = 50 2 amp.
1 æpö
Þ cos(100pt) = = cos ç ÷
2 è4ø r.m.s. current = I 0 / 2 = 50 amp.
1 32. (a) RMS value over one cycle = RMS value over AB.
t= second = 2.5 × 10–3 sec.
400
ì0.1 0.2 ü 0 £ t £ 15 1
1 ï 2 ï 5ò t 4dt
26. (d) Y2 =
0.2 ïò 2
ò
í y dt + y dt ý
ïþ
î0 0.1 i(t) = 5 t2 c i rms = < i 2 > = 0
= 1 mA
1
ì 0.1 ü
ò dt
ï
1 ï0
ò
ï 100 (1 + e -200t - 2e -100t )dt ï
ï 0
ï 33. (b) V= V0 sin w t
= í 0.2
ý
0.2 ï ï Voltage in r.m.s. value
ï
ïî
ò
+ 100e -100 (t - 0.1) dt ï
ïþ V0 = 2 ´ 234 V = 331 volt
0.1
ì0.1 ü and w t = 2 p n t = 2 p ´ 50 ´ t = 100 p t

1 ï0
ï ò
ï 100 (1 + e -200t - 2e -100t )dt ï
ï
Thus, the equation of line voltage is given by
V = 331 sin (100 p t)
ï
= í ý 34. (c) When resistance is connected to A.C source, then
0.2 ï 0.2
ï current & voltage are in same phase.
ï
ïî
ò
+ 100e -100 (t - 0.1)
dt ï
ïþ 35. (b) XL = wL Þ XL µ w
0.1
36. (a) Capacitive reactance in an A.C circuit is
ìé -200t 0.1 ü
ïï ët - 0.005e + 0.02e -100t ù ï 1
û0 ï XC = ohm , where C is the capacitance of
= 500 í ý wC
0.2
ï + é -0.01e -100 (t - 0.1) ù ï capacitor & w = 2pn (n is the frequency of A.C source).
îï ë û 0.1 þï 37. (c)
= 500 {[0.1 – 0.005 e–20 + 0.02 e–10) – (0 – 0.005 38. (d) VL
+ 0.002)] + [(– 0.01e–10) – (0.01)]}= 500 × 0.095 = 47.5
\ Y = 47.5 = 6.9 VR
VC – VL

27. (c) i = 3 sin wt + 4 cos wt f I


E
I

é3 4 ù E(applied
= 5 ê sin wt + cos wt ú = 5 [sin (wt + d)] …(1) voltage)
ë5 5 û 2 T
VC - VL (if V > V )
ò i dt tan f =
VR
C L
5 T1
Þ rms value = Þ mean value = T2 VL - VC
2 = (if VL> VC)
ò dt R
where f is angle between current & applied voltage.
T1
ALTERNATING CURRENT 647

39. (b) In series RLC circuit, 55. (d) In LCR series circuit, resonance frequency f0 is given
by
Voltage, V = VR2 + (VL - VC ) 2
1 1 1
And, at resonance, VL = VC Lw = Þ w2 = \ w= = 2pf0
Cw LC LC
Hence, V = VR
1 1
1 1 \ f0 = or f 0a
40. (b) XC = Þ XC a for given C. 2p LC C
wC w
41. (d) At resonance wL= 1/wC When the capacitance of the circuit is made 4 times, its
resonant frequency become f0¢
and i = E/R , So power dissipated in circuit is P = i 2R.
42. (a) f' C f
\ 0 = or f 0' = 0
R f0 4C 2
43. (b) cos f = , where Z is the impedance & 56. (d) Power, P = Ir.m.s × Vr.m.s × cos f
Z
In the given problem, the phase difference between
Z = R 2 + (XL - XC )2 , if there is only resistance voltage and current is p/2. Hence
then Z = R Þ cosf = 1 P = Ir.m.s × Vr.m.s × cos(p/2) = 0.
44. (d) 57. (d) Since the phase difference between L & C is p
45. (d) P = V I cos q. So, power dissipation depends upon V \ net voltage difference across LC = 50 - 50 = 0
and I. Pure resistor L-R series circuit
58. (d)
46. (b)
R R L
47. (b) Inductive reactance,
XL = wL = 2puL
V V
Þ XL µ u Vs V
Hence, inductive reactance increases linearly with
frequency. Phasor diagram
48. (c) The current drawn by inductor and capacitor will be in R
opposite phase. Hence net current drawn from q
generator R
XL cos q =
= IL – IC = 0.9 – 0.4 = 0.5 amp. Z Z
49. (b)
50. (b) Here, XL = wL = 2pfL = 2p × 50 × 1 = 100 p W
Z = impedance
1 1 1 103
Xc = = = = W For pure resistor circuit, power
wC 2pfC 2p ´ 50 ´ 10 ´ 10-6 p
V2
103 é p 2 - 10 ù P= Þ V 2 = PR
So, X = X L - X C = 100p - = 10 2 ê úW R
p ë p û For L-R series circuit, power
51. (b) 2
V2 V 2 R PR æ Rö
52. (c) We know that power consumed in a.c. circuit is given P1 = cos q = . = 2 .R =Pç ÷
Z Z Z Z è Zø
by, P = Erms . Irms cos f
2
Here, E = E0 sinwt æ 1 ö
59. (c) Z = R 2 + çç w L - ÷
æ pö è w C ÷ø
I = I0 sin ç wt - ÷
è 2ø Here R = 100 W, L = 0.5 henry, C = 10 × 106 farad
which implies that the phase difference, w = 2 p p = 100 p.
p 60. (d) The currents in capacitor and in inductor are opposite
f= and so net current is zero.
2
61. (c) At resonance XL = XC Þ Z = R & current is maximum
p æ p ö
\ P = E rms .Irms .cos = 0 ç Q cos = 0÷ E
2 è 2 ø but finite, which is I max = , where E is applied
R
53. (d) For resonant frequency to remain same voltage.
LC = constant 62. (c) Impedance at resonant frequency is minimum in series
LCR circuit.
LC = constant
2
As, C ® 4C æ 1 ö
So, Z = R 2 + ç 2 pfL - ÷
L è 2 pfC ø
\ L® When frequency is increased or decreased, Z increases.
4
54. (a) 63. (d)
EBD_7418
648 PHYSICS

1 1 74. (b)
64. (c) Capacitive reactance, XC = =
wC 2puC 75. (c) In series resonance circuit, inductive reactance is equal
1 to capacitive reactance.
Þ XL µ 1
u i.e. wL =
With increases in frequency, XC decreases. wC
2
Hence, option (c) represents the corrent graph. æ 1 ö
65. ( a) \ Z = R 2 + ç wL = ÷ =R
E 2v cos f è wC ø
66. (c) P = 76. (b)
Z
(240) 2 (0.75) 77. (b) The phase difference f is given by
P = 3000 = d Þ Z = 14.4W
Z XL 3
67. (b) Time constant is L/R tan f = = =1
Given, L = 40H & R = 8W R 3
\ t = 40/8 = 5 sec. p
Þf= .
68. (b) R = XL = 2XC 4
78. (a) XL = wL = 2pnL
Z = R 2 + (X L - X C ) 2 = (2X C )2 + (2X C - X C ) 2
XL 22 ´ 7
\ L= = H = 0.0175H
2 p n 2 ´ 22 ´ 200
= 4X 2C + X 2C XL–XC 79. (b)
5R 80. (c)
= 5X C = Z
2 81. (d) L = 2H, E = 5 volts, R = 1W
X - XC 2XC - XC 6 4 447
Z 4 4 48
tan f = L =
R 2XC
R L
1 -1 æ 1 ö f
tan f = ; f = tan çè ÷ø R
2 2 E
69. (a) A coil consists of an inductance (L) and a resistance (R). 1 2 E
In dc only resistance is effective. Hence, Energy in inductor = LI I =
2 Z
V 12 5 5
R= = = 3W I= =
i 4
R 2 + ( wL ) 1 + 4 p 2 ´ 50 2 ´ 4
2
V Vrms
In ac, i rms = rms =
Z 5 5
R + w2 L2
2
= =
2 2 1 + ( 200p )
2 200p
1 æ Vrms ö 2 1 æ 12 ö
Þ L= - R = ç ÷ - (3)2
w èç i rms ø÷ 50 è 2.4 ø Energy =
1
´2´
5´5
= 6.33 × 10–5 joules
= 0.08 henry 2 200 ´ 200 p 2
70. (d) 82. (d) Impedence of a capacitor is XC = 1/wC
71. (b) i = i0 (1–e–Rt/L) 1 1 5000
XC = = -
= .
E 2pfC 2p´ 50 ´ 2 ´ 10 6 p
io = (Steady current) when t = ¥
R 83. (a) We know that impedance of the LCR circuit
E 15
i¥ = (1 - e -¥ ) = = 1.5 (Z) = {(X L - X C ) 2 + R 2 } = {(50 - 50) 2 + (10) 2 } = 10 W
R 10
i1 = 1.5(1 - e - R / L ) = 1.5(1 - e -2 ) V 100
= = 0.1 A
84. (c) Across resistor, I =
i 1 e2 R 1000
Þ ¥ = = At resonance,
i1 1 - e -2 e 2 - 1
1 1
72. (a) X L = XC = = = 2500
wC 200 ´ 2 ´ 10-6
73. (d) Option (d) is false because the reason why the voltage Voltage across L is
1 IXL = 0.1 × 2500 = 250 V
leads the current is because > Lw and if the
Cw 1
voltage lags, the inductive reactance is greater than 85. (c) P = Vr.m.s ´ I r.m.s ´ cos f = V0 I 0 cos f
2
the capacitive reactance. 1
= ´ 100 ´ (100 ´ 10 - 3 ) cos p / 3 = 2.5 W
2
ALTERNATING CURRENT 649

e æ
– 2t ö
86. (d) e –t/t
(1 + e ) = ç 1 + e CR ÷
87. (b) As VL = VC = 300 V, resonance will take place 2R 2R çè ÷ø
\ VR = 220 V 93. (a) The vector diagram of the potential differences across
the elements of the circuit shown in question is
220 presented in Figure. Note that since the same current
Current, I = = 2.2 A
100 flows through R and C, the vectors corresponding to
ur ur
\ reading of V3 = 220 V the voltages on R and on C are perpendicular, V R ^ V C .
and reading of A = 2.2 A
B
VR 40
88. (a) A-1 : irms = = = 10 A; i0 = 2 irms = 2 2 A VC
R 4 VR
B-2; \ Vrms = iZ = 10 ´ 5 = 50V ; V0 = 2 Vrms = 50 2 V VAB

C-1 : 1 A 1
Vr = 2 V Vr = V
D-4 : Now V 2 = VR2 + (VL - VC ) 2 2
The two resistors r are equal, and the current through
or 502 = 402 + (40 – VC)2 ur 1 ur
them is the same, hence | V r | = | V | . The potential
\ VC = 10V, 2
VC 10
difference between the points A and B is the magnitude
and X C = = = 1W ur
i 10 of the vector V AB . Since the angle at vertex B is 90°, a
89. (d) The average power in the circuit where cos f = power well known geometrical property tells us that
ur ur ur
factory | V AB | = | V r | = | V / 2 | , and A is the center of the circle
< P > = Vrms × Irms cos f circumscribing the triangle of vectors.
ur
180 Hence the magnitude of V AB does not depend on the
f = p/3 = phase difference = = 60 value of R, since it always equals the radius of the
3 ur
1 circumscribed circle, which is half of | V | .
1 ur
1 As R changes the vector | V AB | will rotate around A,
2 = volt 2 = æ 1ö A
Vrms = ; Irms = ç ÷ although it’s magnitude will not change. Therefore,
2 2 2 è 2ø
this circuit is suitable for changing the phase between
1 1 1 1 ur
<P>= ´ ´ = W the input voltage V and the one picked up in between
2 2 2 8 A and B.
90. (b)
Pmax 5R 6E
1.0 94. (a) Req = Þ I= = 1A
6 5R
P Pmax
=P 95. (c)
2
0.5 when L is removed from the circuit
XC p R
= tan p/3
0.0 R 3
3 4 5 6 7
w1 w0 w2 p Z1
w0 5
X C = R tan ....(1) XC
Quality factor of the circuit = = = 2.0 3
w2 - w1 2.5 when C is remove from the circuit
E 2v cos f XL p Z2
91. (c) P= = tan
Z R 3 XL
p p/3
(240) 2 (0.75) X C = R tan ...(2)
P = 3000 = d Þ Z = 14.4W 3 R
Z
EC R I net impedence Z = R 2 + ( X L - X C )2 = R
92. (b) Q= (1– e – t / t )
2 I1 I2 R
power factor cos f = =1
eC – t / t e – t / t R Z
I2 = e = e e C
2r R 96. (c) If ZL = ZC current will be same,
So, VZ L = VZC ;
Vc e / 2
I1 = = (1– e – t / t ) 1
R R
\ VL = 1 × 2p × 30 × = 60 Volt
p
e e e –t/t
I1 = – e– t / t + – e VR = 80 × 1 = 80 Volt;
R 2R 2R
V= VL2 + VR2 = (80) 2 + (60)2 = 100 Volt
EBD_7418
650 PHYSICS

i2
97. (c) i3 = i12 + i22 VP N P VS ´ N P 1000 ´ 400
= of, VP = = = 200V.
VS NS NS 2000
115. (d) Power in primary of transformer is
and i3 < (i1 + i2)
PP = Vp.IP = 220 × 0.5
E 1 LE 2 i1 = 110 W
98. (d) Current in inductor = \ its energy = But power in secondary of transformer is
R 2 R2
Same energy is later stored in capacitor Ps = 100 W
Q 2 1 LE 2
100
= ÞQ= LC
E \ h= = 0.9 = 90%
2C 2 R 2 R 110
99. (b) 1 1
116. (a) As = w2 = or w =
100. (a) Since current leads emf (as seen from the graph), therefore, LC LC
this is an R – C circuit.
XC - X L 1 Q02
tan f = Maximum energy stored in capacitor =
R 2 C
Here f = 45° Let at any instant t, the energy be stored equally
\ XC = R [XL = 0 as there is no inductor]
between electric and magnetic field. Then energy
stored in electric field at instant t is
1 1 -1
= R Þ RCw = 1 \ RC = s
1 Q 2 1 é 1 Q0 ù
2
wC 100
= ê ú
101. (a) xC – xL = 31.85 – 2(6.28) = 19.29 2 C 2 êë 2 C úû
10
Im = = 0.52 Q02 Q0
19.29 or Q2 = or Q =
Hence I = 0.52 sin (314 t + p/2) = 0.52 cos 314 t 2 2
102. (c) A transformer is employed to obtain a Suitable A. C. Q0
Þ Q0 coswt =
voltage. 2
103. (b) p p p
104. (a) : Iron loss is the energy loss in the form of heat due or wt = or t= =
4 4w 4 ´ (1/ LC )
to the formation of eddy currents in the iron core of
the transformer. p LC
=
105. (c) A transformer does not change the frequency of ac. 4
106. (a) : Here, C 30 mF = 30 × 10–6 F, 117. (a) Transformer cannot produce power, but it transfer from
L = 27 mH = 27 × 10–3 H primary to secondary.
118. (b) U e + U m = 160
1 1 1
\ w= = = \ Um = 160 – Ue
-3 -6
LC 27 ´10 ´ 30 ´10 81´10-8 = 160 – 10 = 150 µJ.
4
10 119. (a) Time period, T = 2p LC = 2 p (50 ´ 10-3 ) ´ 4 ´ 10 -6
= 1.1 × 103 rad s–1
=
9 = 28 × 10–4 s
np Ep Time taken by capacitor to charge fully,
1
107. (b) = = T
ns E s 25 t= = 7 ´ 10 -4 s .
4
\ Es = 25 Ep 120. (b) If q is the required charge, then
E S ´ IS q2 1 Q2
But Es Is = E p I p Þ I p = Þ I p = 50A =
Ep 2C 2 2C
108. (b) Q
\ q= .
109. (a) 2
110. (b) Power of source = EI = 240 × 0.7 = 166 LC
121. (c) Time period, T1 = 2p LC , T2 = 2p ,
140 2
Þ Efficiency = Þ h = 83.3%
166 T3 = 2p 2 LC .
111. (b) Clearly t2 < t1 < t3.
112. (b)
V2 I
P 4000 122. (d) = 0.8 1 Þ V2 I 2 = 0.8
113. (d) As E p Ι p = Pi \ Ιp = i = = 40 A. V1 I2 V1 I1
Ep 100
V1 = 220 V, I2 = 2.0 A, V2 = 440 V
114. (a) NP = 400, NS = 2000 and VS = 1000 V.
ALTERNATING CURRENT 651

V2 I 2 10 440 ´ 2 ´ 10 æ p 3ö
I1 = ´ = =5A çè\ sin = ÷
V1 8 220 ´ 8 3 2 ø
Es n æn ö æ 3ö
123. (a) = s or E s = E p ´ ç s ÷ I = ç 5 ÷ Amp
ç 2÷
Ep np ç np ÷ è ø
è ø
2. (c) To deliver maximum power from the generator to the
æ 200 ö
\ E s = 120 ´ ç ÷ = 240 V load, total internal reactance must be equal to conjugate
è 100 ø of total external reactance.
Ι p ns æ np ö æ 100 ö So, Xint = Xext
= or Ι s = Ι p ç ÷ \ Is = 10 ç = 5 amp
Ι s n p èn ø s
è 200 ÷ø Xg = (XL) = –XL
124. (a) Hence, XL = –Xg (Reactance in external circuit)
125. (c) The time period of LC oscillations, T = 2p LC . 3. (c) As we know that,
T
The time at which charge on the capacitor will be zero is
4
. The voltmeter in AC reads rms values of voltage
p
So t = LC . Irms = 2I0 and Vrms = 2v0
2
126. (a) The equivalent primary load is The voltmeter in AC circuit connected to AC mains
2 2 reads mean value (<v2>) and is calibrated in such a
æ N1 ö æ 20 ö
R1 = ç ÷ R2 = çè 1 ÷ø ( 6.0 ) = 2400 W way that it gives rms value of <v2>, which is multiplied
è N2 ø by form factor Ö2 to give rms value Vrms.
240 240 4. (b) As we know that,
Current in the primary coil = R = 2400 = 0.1 A
1 The resonant frequency in an L-C-R series circuit is
Vs N 1
127. (a) Since = s n0 =
Vp N p 2p LC
Where So, to reduce n0 either increase L or increase C.
Ns = No. of turns across primary coil = 50 To increase capacitance, another capacitor must be
Np = No. of turns across secondary coil = 1500 connect in parallel with the first capacitor.
df d 1500 5. (c) As we know that, Quality factor (Q) of an
and Vp = = (f + 4t) = 4 Þ Vs = ´ 4 = 120 V
dt dt 0 50 L-C-R circuit must be higher so Q is
Ι s n p 80 20
128. (c) = ; = or Ip = 4 amp. 1 L
Ι p ns Ιp 1 Q=
R C
Vs I s V (6)
129. (b) Efficiency h = Þ 0.9 = s 3 Þ Vs = 450 V where R is resistance, L is inductance and C is
VpI p 3 ´ 10
capacitance of the circuit.
3000 3000
As VpIp = 3000 so Ip = = A = 15 A So, for higher Q, L must be large, and C and R should
Vp 200 be low.
10 4 Hence, option (c) is verify.
130. (b) P = V1i1 = V2i2 10 ´ 103 = 25 ´V2 V2 =
25 6. (c) As given that,
n1 8 104
Now V1 = ´ V2 = ´ V XL = 1W, R = 2W, Erms = 6V, Pav = ?
n2 1 25
The average power dissipated in the L, R, series circuit
with AC source
Exercise - 2 Then Pav= Erms Irms cos f ... (i)

1 I0 E rms
1. (b) As given that, v = 50 Hz, Irms = 5A, t = s Irms = =
300 2 Z
I0 Z= R + X 2L =
2
4 +1 = 5
As we know that Irms =
2 6
Irms = A
I0 = Peak value = 2.Irms = 2 ´ 5 = 5 2A 5
1
at, t = sec , I = I0 sinwt = 5 2 sin 2 p ´ 50 ´ 1 cos f =
R
=
2
300 300 Z 5
p 3 By putting the value of Irms, Erms, cos f in equation (i),
I = 5 2 sin = 5 2´ = 5 3 2 Amp then,
3 2
EBD_7418
652 PHYSICS

6 2 72 If we fill a di-electric material like mica instead of air


Pav = 6 ´ ´ = then capacitance C­ Þ Vc¯ So, Va > Vb
5 5 5 5
Ιs n p 80 20
72 13. (c) = ; = or Ι p = 4 amp.
= = 14.4 watt Ιp ns Ιp 1
5
7. (a) As given that, 14. (a) L = 10 mHz = 10–2 Hz
f = 1MHz = 106 Hz
Secondary voltage (VS) is :
VS = 24 Volt 1
f=
Power associated with secondary is : 2p LC
PS = 12 Watt 2 1
f = 2
As we know that PS = VSIS 4p LC
PS 12 1 1 1
IS = = = A = 0.5 Amp ÞC = =
VS 24 2 4p f L 4 ´10 ´10-2 ´1012
22

Peak value of the current in the secondary 10 -12


= = 2.5 pF
I0 = IS 2 = 0.5 2
4
15 ´ 100
15. (b) I(0) = = 0.1A
5 é 1 ù 0.15 ´ 103
= 2 ê I0 = Ampú I (¥) = 0
10 ë 2 û –t
Vs Is V (6) I (t) = [I (0) – I (¥)] e L / R + i( ¥ )
8. (b) Efficiency h = Þ 0.9 = s
Vp Ip 3 ´ 103 –t
I(t) = 0.1 e L / R = 0.1 e L
R

Þ Vs = 450 V
0.15 ´1000
3000 3000
As VpIp = 3000 so, Ip = = A = 15A I(t) = 0.1 = 0.67mA
e 0.03
Vp 200
16. (b) Total carrent, l = (5 + 10 sin wt)
9. (d)
1/2
Potential drop across capacitor or inductor é T I2 dt ù
Q= êò ú
Potential drop across R. Þ Ieff =ê 0 ú
T
wL êë ò0 dt úû
=
R 1/2
10. (c) Applying Kirchhoff's law of voltage in closed loop é1 T 2 ù
= ê ò ( 5 + 10sin wt ) dt ú
V ëT 0 û
–VR –VC = 0 Þ R = -1
VC 1/2
A C
VR
R
é1 T
ëT 0
(
= ê ò 25 + 100sin wt + 100sin 2 wt ú
ù
û
)
1 T 1 T 2 1
L VL But, ò0 sin wt.dt = 0 and ò0 sin wt.dt =
T T 2
B 1/2
é 1 ù
So, Ieff = ê 25 + ´100ú = 5 3A
ë 2 û
11. (a) x C - x L = 31.85 - 2 (6.28) = 19.29
2
æ 1 ö
10 17. (a) Z = R 2 + ç wL - ÷
Im = = 0.52 è wC ø
19.29
Hence I = 0.52 sin (314 t + p/2) = 0.52 cos 314 t 2
æ 1 ö
12. (a) For series R – C circuit, capacitive reactance, = 102 + ç 2000 ´ 5 ´10-3 - ÷
2 è 2000 ´ 50 ´10-6 ø
Xc = 1/cw
æ 1 ö
Zc = R2 + ç ÷ = 10 W
è Cw ø
R V 20
V V C i= 0 = = 2A.
Current i = = Z 10
Zc 2 ~
æ 1 ö 18. (b) As we know, power P = Vrms · Irms cosf
R2 + ç ÷
è Cw ø
AC Source as cosf = 0 (Q f = 90°)
V 1 V \ Power consumed = 0
Vc = iXc = ´ = (in one complete cycle)
2 Cw
2 æ 1 ö (RCw)2 + 1 19. (a) Given: L = 20 mH; C = 50 mF; R = 40 W
R +ç ÷
è Cw ø V = 10 sin 340 t
ALTERNATING CURRENT 653

10 e0 / R 1
\ Vruns = Þ =
2 e
2 2
1 1 ( e 0 / R) 2 + æçè e0wC – w0L ö÷ø
XC = = = 58.8 W
wC 340 ´ 50 ´10-6
3
XL = wL = 340 × 20 × 10–3 = 6.8 W ÞR=
æ 1 ö
R 2 + ( XC - X L )
2 çè wC – ÷
Impedance, Z = wL ø
e0
2
= 40 2 + ( 58.8 - 6.8 ) = 4304 W
2 iC iR
0
æV ö 0
Power loss in A.C. circuit, P = i 2rms R = ç rms ÷ R wt
è Z ø
2
æ 10 / 2 ö 50 ´ 40
=ç ÷ ´ 40 = ; 0.51 W iL 0
è 4304 ø 4304
26. (b)
20. (a) If w = 50 × 2p then wL = 20W
If w¢ = 100 × 2p then w¢L = 40W
Current flowing in the coil is
200 200 200 At t = 0, no current flows through R1 and R3
I= = =
Z 2
R + (w¢L) 2
(30)2 + (40) 2
I = 4A.
21. (a) We have, I =
Io 1 - e L ( -
R
t
) \

(When current is in growth in LR circuit)


æ - tö
R æ - ´2 ö
5 Current through battery just after the switch closed is
E 5
= ç 1- e L ÷ = ç 1 - e 10 ÷ e 18
R çè ÷ø 5 çè ÷ø i= = = 2A
R2 9
–1
= (1 – e ) 27. (a)
28. (d) Large eddy currents are produced in non-laminated
22. (c) iron core of the transformer by the induced emf, as the
resistance of bulk iron core is very small. By using thin
23. (b) Here iron sheets as core the resistance is increased.
e e e Laminating the core substantially reduces the eddy
i= = =
R 2
+ X 2L 2
R + 4 p2 v 2 L2
R +w L 2 2 2 currents. Eddy current heats up the core of the
transformer. More the eddy currents greater is the loss
220 V 80
10 = [Q R = = = 8] of energy and the efficiency goes down.
2 2
64 + 4p (50) L I 10
29. (a) The charging of inductance given by,
On solving we get
L = 0.065 H æ - ö÷
Rt
ç
24. (c) Growth in current in LR2 branch when switch is closed i = i 0 ç1 - e L ÷
ç ÷
is given by è ø
Rt Rt
E - -
i= [1 - e - R2t / L ] i0
= i 0 (1 - e L ) Þ e L =
1
R2 2 2
R t
di E R2 - R2t / L E - L2 Taking log on both the sides,
Þ = . .e = e
dt R2 L L
Hence, potential drop across Rt
- = log 1 - log 2
æ E -R t / L ö -R t / L L
L = çè e 2 ÷ø L = Ee 2
L
2t L 300 ´10 -3
- Þ t= log 2 = ´ 0.69 Þ t = 0.1 sec.
= 12e 400´10 -3 = 12e–5tV R 2
iR0 30. (a) Power dissipated in an LCR series circuit connected
1
25. (a) = to an a.c. source of emf E
2
(iR0 ) + (ic0 – iL0 ) 2 2
EBD_7418
654 PHYSICS

2 2
Erms R Erms R Z = R 2 + (X L - X C ) 2 = 25 + (4.9) 2 = 7 W
P = Erms irms cos f = =
Z2 2 Phase difference between the voltage and current
æ 1 ö
R 2 + ç wL – ÷ X - XC
è C wø tan f = L
R
2 4.9
æ 10 ö tan f = » 1 Þ f = 45°
ç ÷ ´ 50 5
= è 2ø 2. (c) Given, VL : VC : VR = 1 : 2 : 3
2
æ 1 ö V = 100 V VR = ?
(50) + ç 314 ´ 20 ´10 –3 –
2
–6 ÷
è 314 ´ 100 ´ 10 ø
Solving we get, P = 0.79 W As we know, V = VR2 + ( VL - VC )2
31. (a) V = V0 [1 – e–t/t]
Solving we get, VR ; 90V
6 = 12 [1 – e–0.1/t]
e0.1/t = 2 3. (d) Power factor (old)
R R R
0.1 = = =
= ln(2) 2 2 2 2 5R
t R + XL R + (2R)
0.1 Power factor(new)
RC = ln 2 R R R
( ) = = =
R 2 + (X L - XC )2 R 2 + (2R - R) 2 2R
0.1
R = C ln 2 R
( ) New power factor 2R = 5
\ Old power factor =
Î R 2
32. (a) I = [1 – e–t/c]
R 5R
4. (d) In case of series RLC circuit,
L 1
Z= ,I= [1 – e–2/2] Equation of voltage is given by
R 1.5
V 2 = VR2 + (VL - VC )
2
3 2 é 1ù Here, V = 220 V; VL = VC = 300 V
Z= = 2, I = ê1 - ú = 0.4 A
1.5 3 ë eû
\ VR = V 2 = 220V
33. (b) As we know, average power Pavg = Vrms Irms cosq
V 220
æ V öæ I ö æ 100 ö æ 20 ö Current i = = = 2.2A
= ç 0 ÷ ç 0 ÷ cos q = ç ÷ç ÷ cos 45° (Q q = 45°) R 100
è 2 øè 2 ø è 2 øè 2 ø 5. (a) If voltage across resistor is 50V then this should be the
1000 resonance condition.
Pavg = watt At resonance, XL = XC
2
Wattless current I = Irms sin q 1 1
w2L = w =
I 20 w2 C ; 2 LC
= 0 sin q = sin 45° = 10A Also, At w = w1
2 2 100 60 100 60
w0 w L I= = =
34. (a) Quality factor Q = = 0 X L X C ; w1L 1/ w1C
2Dw R
35. (a) In an ac circuit, a pure indcutor does not consume any 100 5
C= 2 = 2
power. Therefore, power is consumed by the resistor w1 L ´ 60 3w1 L
only.
1 3
\ P = I v2 R w2 = = w1
5 5
or 108 = (3)2 R or R = 12 W L´
3w12 L
6. (d) Current is maximum when XL = XC
Exercise - 3 1 1 1
Þ wL = Þ w= =
wC
1. (b) Given, LC 0.5 ´ 8 ´ 10-6
V0 = 283 volt, w = 320, R = 5 W, L = 25 mH, C = 1000 1
= = 500 rad/s.
µF 2 ´10-3
xL = wL = 320 × 25 × 10–3 = 8 W 7. (d)
1 1 8. (a) For two concentric circular coil,
xC = = = 3.1 W
wC 320 ´ 1000 ´ 10 -6
Total impedance of the circuit :
ALTERNATING CURRENT 655

m pN N a 2 RC = 3 × 10–3 sec
Mutual Inductance M = 0 1 2 R × 3 × 10–6 = 3 × 10–3
2b
here, N1 = N2 = 1 R = 1000 W
2 V
m 0 pa
Hence, M = ..... (i) 13. (a) i = 2
2b æ 1 ö
R2 + ç
and given I = I0 cos wt ..... (ii) è wC ÷ø
Now according to Faraday's second law induced emf V
or I= … (i)
dI 1
R2 +
e = -M 2 2
wC
dt
From eq. (ii), a I V
and = … (ii)
2 w2C 2
-m 0 pa 2 d b 2
R +
e= (I0 cos w t) 9
2b dt
On simplifying above equations, we get
m pa 2
e= 0 I0 sin w t (w ) X L wL 3
2b = = .
R R 5
pm I a 2
e = 0 0 . w sin w t 14. (c)
2 b
9. (c) At t = 0, no current will flow through L and R1 15. (b) L = 0.7H, R = 220W, E0 = 220V, v = 50 Hz.
V This is an L-R circuit
\ Current through battery = R
2 Phase difference,
At t = ¥,
RR X L w L 2pvL 22
effective resistance, Reff = 1 2 tan f = = = [XL= 2pvL = 2 × × 50 × 0.7 =
7
R1 + R2 R R R
V 220W]
V ( R1 + R2 )
\ Current through battery = R = R1R2 220
eff = = 1 or, f = 45°
220
10. (d)
Wattless component of current
11. (b)
I0 1 E0
12. (d) Equivalent circuit can be drawn as below = I0 sin f = = ×
2 2 Z
A
A 1 220 1 220 1
1µF R = = × = = 0.5 A
R 2 X L2 +R 2 2 2
220 + 220 2 2
1µF
1µF
1µF 3µF B
1µF B
1µF R R
EBD_7418
656 PHYSICS

23. ELECTROMAGNETIC WAVES

Exercise - 1 1 c
24. (b) v= =
me mr e r
1. (b) The electromagnetic waves do not transport charge.
2. (b) In an electromagnetic wave the directions of magnetic 25. (b) From question,
B0 = 20 nT = 20 × 10–9T
induction is perpendicular to electric field. r r r
3. (b) 4. (c) E0 = B0 ´ C
5. (b) Current through capacitor, r r r
| E 0 |=| B0 | . | C |= 20 ´ 10-9 ´ 3 ´ 108 = 6 V/m.
E E (Q velocity of light in vacuum C = 3 × 108 ms–1)
I= = = wCE = 2pu CE or I µ u.
XC 1 E 0 9 ´ 10 3
wC 26. (d) B0 = = = 3 ´ 10 -5 T.
c 8
\ decrease in frequency u of ac source decreases the 3 ´ 10
27. (a)
conduction current. As displacement current is equal 28. (a) Energy density (EM waves)
to conduction current, decrease in u decreases 2
displacement current in circuit. 2 æ E0 ö 1
= e0 Erms = e 0 ç ÷ = e0 E02
è 2ø 2
dv dv i d
6. (a) id = c ´ Þ = = 106 Vs –1 E
dt dt c 29. (c) Incident momentum, p =
7. (a) Displacement current arises when electric field in a c
For perfectly reflecting surface with normal incidence
region is changing with time. It will be so if the
2E Dp 2E F 2E
charge on a capacitor is changing with time. Dp = 2p = F= = P= =
8. (c) 9. (d) 10. (c) 11. (d) c Dt ct A ctA
30. (c)
12. (a) Both magnetic and electric fields have zero average
31. (c) In an electromagnetic wave electric field and magnetic
value in a plane e.m. wave. field are perpendicular to the direction of propagation
13. (c) The electromagnetic waves of all wavelengths travel of wave. The vector equation for the electric field is
with the same speed in space which is equal to r
velocity of light. æ 2p ö
E = E0 cos ç wt - y zˆ
14. (b) è l ÷ø
15. (c) According to Maxwell, a changing electric field is a E0 2p
source of magnetic field. 32. (a) = c . also k = and w = 2pv
B0 l
16. (b) An electromagnetic wave has both energy and
momentum. These relation gives E0 k = B0w
r r
17. (c) E and B are mutually perpendicular to each other 33. (a) Velocity of light
and are in phase i.e., they become a zero and minimum E E 9.3
C= ÞB= = = 3.1 ´ 10-8 T
at the same place and at the same time. B C 3 ´ 108
18. (c) 34. (a) E.M. wave always propagates in a direction
19. (a) Displacement current is set up in a region where the perpendicular to both electric and magnetic fields. So,
electric field is changing with time. electric and magnetic fields should be along + X– and
20. (d) + Y–directions respectively. Therefore, option (a) is
2p the correct option.
21. (b) Here, k = , w = 2pu 35. (b) Here, E0 = 18 V/m; B0 = ?
l
k 2p / l 1 1 E 18
\ = = = (Q c = u l) B0 = 0 = 8
= 6 ´ 10-8 T
w 2pu pu c c 3 ´10
where c is the speed of electromagnetic wave in 36. (c)
vacuum. It is a constant whose value is 3 × 106 m s–1 c 3 ´108
37. (a) Here, l = = = 36.6 m.
22. (b) n 8.2 ´ 106
r r df 38. (a)
23. (c) Ñò B.d l b 2 - 4 ac = m 0 Î0
dt 39. (d)
40. (a) Here, amplitude of electric field, E0 = 100 V/m; amplitude
æ ö
dE æ dE ö
or B ´ 2pr = m 0 Î0 A ç ÷ \ B µ çè ÷ø of magnetic field, H0 = 0.265 A/m. We know that the
è dt ø dt
maximum rate of energy flow,
S = E0 × H0 = 100 × 0.265 = 26.5 W/m2.
ELECTROMAGNETIC WAVES 657

ur r
Ñò E × d l = –
1 1 1 d fB d
41. (a) The speed of light C = = = = 0.25 53. (c) or E(2pr) = (kt ´ pr 2 )
m0 e 0 2´8 4 dt dt
42. (b) kr
\ E=
43. (a) Direction of energy progration of EM-waves is given by 2
r r r r 54. (d) By = 2 × 10–7 sin(0.5 × 103 z + 1.5 × 1011 t) T
D = K ( E ´ B) or –kˆ = K ( E ˆj ´ B )
Clearly direction of magnetic field is along positive x- The electric vector is perpendicular to B as well as
axis. direction of propagation of electromagnetic wave.
44. (d) Therefore Ex has to be taken.
45. (a) (A) ® (3) ; (B) ® (2) ; (C) ® (1) ; (D) ® (4) Further, E0 = B0 × c
46. (d) : Velocity of electromagnetic wave = 2 × 10–7 × 3 × 108 V/m
1 E0 = 2 × 10–7 × 3 × 108 = 60 V/m
c = m e = 3 × 108 m s–1 \ The corresponding value of the electric field is
0 0
It is independent of amplitude, frequency and Ex = 60 sin(0.5 × 103 z + 1.5 × 1011 t) V/m
wavelength of electromagnetic wave. 55. (a) As B µ r, since the point is on the axis where r = 0, So
47. (a) According to conservation of charge, the displacement B= 0
current must be leftward. 56. (c)
id 1
i 57. (b) Energy density = e 0 E02 = 1.1 × 10–8 J m–3
2
Volume of cylinder = A × L = 5 × 10–4 m3
48. (d) The frequency of electromagnetic wave remains So, required energy = 1.1 × 10–8 × 5 × 10–4 = 5.5×10–12
unchanged but the wavelength of electromagnetic
J
wave changes when it passes from one medium to
another. 58. (a) As we know that,
1 As the displacement current through the capacitor is,
c= dq
m0 e0 Id = Ic = ... (i)
dt
1 1 As given that, q = q0 cos2pnt
\ cµ and v µ
e0 e Putting this value of q in Eq (i), we get
c e 4 d
= = So, Id = Ic = (q cos2pnt)
\ dt 0
v e0 1 =2
Id = Ic =–q0sin2pnt × 2pn
c ul l l Id = Ic = 2pvq0 sin 2pvt
= = = 2 or l ' =
v ul ' l ' 2 59. (b) As we know that,
E0 The standard equation of magnetic field
49. (b) B0 =
c B = B0sin(kx – wt)
E0 - Electric field, c - speed of light, B0 - Magnetic Field.
And, the given equation is
10 -3 B =12 × 10–8 sin (1.20 × 107z – 3.60 × 1015t)T.
B0 = = 3.33 ´ 10 -12 T
8
3 ´ 10 On comparing this equation with standard equation (i),
r 2 E0
50. (c) Magnetic field vector, B = ĵ sin kz sin wt we have.
c B0 = 12 × 10–8
uur ur uur ur
51. (b) As we know, E × B = 0 Q [E ^ B] So, the average intensity of the beam
uur ur
and E ´ B should be along Z direction
( )
-8 2
1 B02 1 12 ´ 10 ´ 3 ´ 108
As (–2$i – 3 $j ) ´ (3$i – 2 $j ) = 5k$ Iav = ×c = ´
2 m0 2 4p ´ 10-7
Hence option (b) is the correct answer.
= 1.72 W/m2
8
2pc 2p ´ 3 ´10 60. (a)
52. (b) w = 2pv = = = p × 1011 rad/sec
l 6 ´ 10 –3 61. (b)
The equation for the electric field, along y-axis in the 62. (c) Average energy density corresponding to maximum
electromagnetic wave is electric field,
Ey = E0 sin w æç t – ö÷
x
B20 (200 ´10 -6 ) 2
cøè u av = = @ 0.016 Jm -3
é x ù 2m 0 2 ´ 4p ´10 -7
= 33sin ê p ´ 10 ç t – ö÷ ú
11 æ
ë è c øû
EBD_7418
658 PHYSICS

1 atmosphere is absorbed by the earth’s surface and


63. (a) Intersity of a plane electromagnetic wave = e0 E02 c
2 re-radiated as infrared radiation. The radiation is
trapped by greenhouse gases such as carbon
2I
E= –1 dioxide and water vapour and they heat up and heat
e0c = 38.8 NC
their surrondings.
64. (b) 65. (b) 66. (b) 95. (d) Now a days microwaves are used to locate the flying
67. (b) The range of visible radiations is 4000Å to 8000 Å objects by Radar.
68. (d) 96. (b) 97. (c) 98. (a)
69. (a) The wavelength of infrared region is 8 × 10–5 cm to 99. (a) In vacuum velocity of all EM waves are same but their
3 × 10–3 cm. So maximum wavelength of infrared region wavelengths are different.
= 8 × 10–5 cm » 10–4 cm. 100. (a) Every body at all time, at all temperature emit radiation
70. (d) (except at T = 0), which fall in the infrared region.
71. (a) Radiowaves are reflected by ionosphere.
72. (a) Exercise - 2
73. (d)
74. (c) Light waves and X-rays are electromagnetic waves. 1. (c) As we know that,
75. (a) The basic difference between various types of E = hn
electromagnetic waves lies in their wavelengths or As given that h = 6.62 × 10–34 J-s
frequencies since all of them travel through vacuum E = 11 eV =11 × 1.6 × 10–19
with the same speed. Consequently, the waves differ n= ?
considerably in their mode of interaction with matter. 11 eV = hn
76. (d) 77. (c) 78. (d)
11 ´1.6 ´10 -19 11´1.6 ´10-19
79. (c) Here : Velocity of electromagnetic waves in free space So, n = J = J
and wavelength h 6.62 ´ 10-34
v = 3 × 108 m/s and l = 150 m = 2.65 × 1015 Hz
The frequency of radio waves is given by So, that frequency radiation belongs to ultraviolet
region.
v 3 ´108
= = = 2 ´106 Hz = 2 MHz. 2. (b) The type of wave doesn't change when a wave is
l 150 reflected from denser medium but only its phase
80. (b) changes by 180°. As E is along positive x-axis so
81. (d) Ozone layer will absorb ultraviolet rays; reflect the reflected ray will be along negative x-axis and its
infrared radiation and does not reflect back radiowaves. component will also be opposite to earlier in (–z)
82. (a) 83. (d) 84. (c) direction and phase will change.
1
85. (c) Speed of EM waves in vacuum = = constant For the reflected wave zˆ = - z,ˆ ˆi = - ˆi and additional
m 0 Î0 phase of p in the incident wave.
86. (b) 87. (c) 88. (b) As given that the incident electromagnetic wave is,
89. (c) (A) ® (4) ; (B) ® (3) ; (C) ® (2) ; (D) ® (1)
E = E ˆi cos ( kz - wt )
The frequency ranges of various waves are as under : 0

Radiowaves ; 5 × 105 to 109 Hz; g-rays ; 3 × 1018 to 5 × So, the reflected electromagnetic wave is
1022 Hz Microwaves; 1 × 109 to 3 × 1011 Hz; X-rays ; 1 E = E ( -ˆi ) cos ( k ( -z ) - wt + p )
r 0
× 1016 to 3 × 1021 Hz.
= - E 0 ˆi cos ( - ( kz + wt ) + p )
90. (a) From electromagnetic spectrum, frequencies of g-rays
is greater than frequency of X-rays. Frequency of X- = - E 0 ˆi cos ( p - ( kz + wt ) )
rays is greater than frequency of ultraviolet rays. = E0 iˆ cos ( kz + wt )
hc hc 6.6 ´ 10 -34 ´ 3 ´ 108 3. (b) As we know that
91. (a) E= Þl= = = 12.4 Å
l E 11´ 1000 ´ 1.6 ´10 -19 the momentum of incident light
92. (c) In vacuum X-rays, gamma rays and microwaves travel
U(total energy)
with same velocity, i.e., with the velocity of light c (= 3 =
× 108 m s–1) but have different wavelengths. c
93. (b) The wavelength of radiowaves being much larger than As given that the energy flux f = 20W/cm2
light, has a size comparable to those of buildings, hense Surface are A = 30 cm2
diffract from them. Time for total momentum delivered
94. (b) Infrared radiation help to maintain the earth warmth t = 30 min =30 × 60 sec
through the greenhouse effect. Incoming visible So, total energy falling in time t sec is
light which passes relatively easily through the U = fAt =20 × 30 × (30 × 60) J
ELECTROMAGNETIC WAVES 659

Momentum of the incident light 1 1


e E 2 = e E ( cB0 )
2
(Uav) =
U 2 0 0 2 0 0
= (Q c = 3 × 108) 1 æ 1 ö
c = e0 ´ c B çQ c =
2 2

2 ÷
Momentum of incident light è m 0 e0 ø

20 ´ 30 ´ ( 30 ´ 60) 1 1
= = 36 × 10–4 kg-ms–1 \ (Uav)Electric field = e0 ´ B02
3 ´ 108 2 m0e0
As no reflection from the surface and for comidete 1 B20
absorption. = =(Uav)Magnetic field
2 m0
Momentum of the reflected light = 0 So, the energy in electromagnetic wave is divided
Hence, momentum delivered to the surface equally between electric field vector and magnetic field
= Charge in momentum. = (pf – fi) vector.
= 36 × 10–4 – 0 = 36 × 10–4 kg-ms–1 Then, the ratio of contributions by the electric field and
4. (c) As we know that the electric field intensity on a surface magnetic field components to the intensity of an
due to incident radiation is, electromagnetic wave is
Iav µ E02 ( U av )electric field
Ratio = =1: 1
Pav ( U av )Magnetic field
µ E02 (A is constant) 7. (c) As we know that, the electric field is inversly
A
Here, Pav µ E02 æ 1ö
proportional to r, so ç E 0 µ ÷
è rø
So, E0 µ Pav
From a diode antenna, an electromagnetic waves are
( E0 )1 ( Pav )1 radiated outwards from dipole antenna with the
\ = amplitude of electric field vector (E0) which transports
( E0 )2 ( Pav )2 significant energy from the source falls off intensity
( E0 )1 100 é 2ù inversely as the distance (r) from the antenna, i.e.,
= =ê ú
( E0 )2 50 ë 1 û radiated energy æç E 0 µ 1 ö÷
è rø
( E0 ) 2 = ( E0 )1 2
8. (d) In electromagnetic wave, the electric field vector is
E = ( E1iˆ + E 2 ˆj) cos ( kz - wt )
5. (d) The direction of propagation of electromagnetic wave
is perpendicular to both electric field E and magnetic
and the associated magnetic field vector,
field B, i.e., in the direction of E × B by right thumb rule.
E E ˆi + E 2 ˆj
The diagram given below B= = 1 cos ( kz - wt )
x
c c
So, E and B are perpendicular to each other and the
E(x) B(–y) E(x) B(–y) propagation of electromagnetic wave is perpendicular
to E as well as B, so the electromagnetic wave plane
z polarised.
y B (y) E(–x) Dp 2IA
E(–x) B(y) 9. (b) Average force Fav = = (Q Power = F.V)
Dt c
So, electromagnetic wave is along the z-direction which
is give the cross product of E and B direction is 2 ´ 25 ´ 104 ´ 15 ´ 10 - 4
r r = = 2.50 × 10– 6 N
perpendicular to E and B from E to B . i.e., (E × B) in z- 3 ´ 108

direction. 10. (c) b -rays are the beam of fast moving electrons.
6. (c) Intensity in terms of electric field 11. (a) Ozone layer in the stratosphere helps in protecting life
of organism form ultraviolet radiation on earth. Ozone
1
Uav = e 0 E 0
2
layer is depleted due to of several factors like use of
2 chlorofluoro carbon (CFC) which is the cause of
Intensity in terms of magnetic field
environmental damages.
1 B20
Uav = 1
2 m0 12. (c) E0 = CB0 and C =
m0 e 0
We also know that the relationship between E and B is
E0 = cB0 1
Electric energy density = e 0 E02 = m E
So the average energy by electric field is 2
EBD_7418
660 PHYSICS

1 Bo 2 1
= mB 24. (c) Velocity of EM wave is given by v =
Magnetic energy density = mÎ
2 m0
Thus, mE = mB w C
Energy is equally divided between electric and Velocity in air = = C Velocity in medium = 2
k
magnetic field
13. (d) Infrared rays are used to treat muscular strain because Here, m1 = m2 = 1 as medium is non-magnetic
these are heat rays. 1
Radio waves are used for broadcasting because these Îr1 C Îr1 1
waves have very long wavelength ranging from few \ = =2 Þ =
1 æCö Îr2 4
centimeters to few hundred kilometers ç ÷
X-rays are used to detect fracture of bones because Îr2 è2ø
they have high penetrating power but they can't
penetrate through denser medium like dones.
Ultraviolet rays are absorbed by ozone of the Exercise - 3
atmosphere.
14. (d) Energy of x-ray is (100 ev to 100 kev) 1. (c) Given: Amplitude of electric field,
Hence energy of the order of 15 kev belongs to x-rays. E0 = 4 v/m
15. (c) Light wave is an electromagnetic wave in which E and Absolute permitivity,
e0 = 8.8 × 10–12 c2/N-m2
B are at right angles to each other as well as at right
Average energy density uE = ?
angles to the direction of wave propagation. Applying formula,
16. (d) To generate electromagnetic waves we need
accelerating charge particle. 1
Average energy density uE = e0 E 2
17. (c) Radio waves can be polarised because they are 4
transverse in nature. Sound waves in air are longitudinal 1
in nature. Þ uE = ´ 8.8 ´ 10 -12 ´ (4) 2 = 35.2 × 10–12 J/m3
4
E, Decreases 2. (d) Intensity of EM wave is given by
18. (c)
g-rays X-rays uv-rays Visible rays IR rays Radio P
P 1
VIBGYOR Microwaves waves I= = vav .c = e E2 ´ c Þ E0 =
Radio wave < yellow light < blue light < X-rays 4pR 2 2 0 0 2pR 2 e 0 c
(Increasing order of energy)
800 V
19. (a) Given, Erms = 6 V/m = = 54.77
E rms 2 ´ 3.14 ´ (4) ´ 8.85 ´ 10 -12 ´ 3 ´ 108
2
m
E
= c Þ Brms = rms ...(i) 3. (b)
Brms c r
4. ˆ
(d) Given : B = 1.2 ´ 10-8 kT
B r
Brms = 0 Þ B0 = 2 Brms
2 E=?
From formula,
E
B0 = 2 ´ rms From equation (i) E = Bc = (1.2 ´ 10-8 T)(3 ´ 108 ms -1 ) = 3.6 Vm -1
C
r
2 ´6 B is along Z-direction and the wave propagates along
= = 2.83 × 10–8 T r
3 ´ 108 X-direction. Therefore E should be along Y-direction.
20. (d) Electromagnetic wave consists of periodically r
Thus, E = 3.6ˆj Vm-1
oscillating electric and magnetic vectors in mutually
perpendicular planes but vibrating in phase. 5. (a) On comparing the given equation to
r
21. (d) The optical waves used in optical fibre communication E = a0iˆ cos (wt – kz), w = 6 × 108z,
are better carrier of signals than microwaves. The
speed of microwave and optical wave is the same in 2p w w 6 ´ 108
k= = , k= = = 2 m -1
vacuum. r c c 3 ´ 108
22. (b) As we know, 6. (b) Q The E.M. wave are transverse in nature i.e.,
r r r r r
E´B =V k ´E r
r = =H …(i)
( Ejˆ) ´ ( B) = Viˆ mw
r r r
(Q Electric field vector is along +y axis) r k ´H r
r where H = B and = -E … (ii)
So, B = Bk ˆ m we
i.e., direction of magnetic field vector is along +z r r r r
k is ^ H and k is also ^ to E
direction. r r r r r
23. (b) or In other words X || E and k || E ´ B
ELECTROMAGNETIC WAVES 661

7. (b) Displacement current, ID = conduction current, IC 13. (a) Intensity of light


dq d B0 2 c (4 ´ 10-6 )2 ´ (3 ´108 )
\ = [q0 cos2put] = – q0 2pu sin2put I= = = 1900 W/m2.
dt dt 2m0 2 ´ (4p´ 10-7 )
8. (a) E
9. (a) Erms = 720 14. (b) For electromagnetic waves we know that, = c
The average total energy density B
1 1 9 ´10-4
= Î0 E 20 = Î0 [ 2E rms ] =Î0 E rms
2 2 \ = 3 ´ 108 ms -1 B = 3 × 10–12 T.
2 2 B
15. (b) Wavelength of monochromatic green light
= 8.85 ´10-12 ´ (720)2 = 4.58 ´10-6 J / m3 = 5.5 × 10–5 cm
10. (b)
11. (b) Comparing with the equation of wave. Power 100 ´ ( 3 /100 ) 3
Ey = E0 cos (wt – kx) Intensity I = = = Wm -2
4p ( 5 )
Area 2 100 p
w = 2 pf = 2p × 106 \ f = 106 Hz
2p Now, half of this intensity (I) belongs to electric field
= k = p × 10–2 m–1, l = 200 m and half of that to magnetic field, therefore,
l
12. (d) Id = 1 mA = 10–3 A I 1 2I
= e0 E 02 C or E0 =
C = 2mF = 2 × 10–6 F 2 4 e0 C
d dV
ID = IC = (CV) = C æ 3 ö
dt dt 2´ç p÷
è 100 ø 6
dV I D 10-3 = = ´ 30 = 7.2
Therefore,
dt
=
C
=
2 ´10-6
= 500 Vs–1 æ
ç
1

è 4p ´ 9 ´10 ø
ö
(
´ 3 ´108 ) 25

Therefore, applying a varying potential difference of


500 V s–1 would produce a displacement current of \ E 0 = 2.68 V / m
desired value.
EBD_7418
662 PHYSICS

24. RAY OPTICS AND OPTICAL INSTRUMENTS

Exercise - 1 1 1 1
= -
f v2 30
1. (b) A plane mirror may form a real image when the rays
incident on it are converging. 1 -1 1 - 30 + 10 -20
= + =
2. (c) Because they form nearly point image of point source. v2 10 30 = 300 300
3. (d) v2 = – 15 cm
4. (d) The image formed by a convex mirror is always L = v2 – v1 = – 15 – (– 20) = 5 cm
virtual.
5. (a) æ ˆi 3 ˆö æ ˆi 3 ˆö
ç 2 + 2 j÷ .ç 2 - 2 j÷
6. (d) Focal length of the spherical mirror does not depend è ø è ø
12. (a) cos(180° - 2a ) =
on the medium in which it placed. 2 2 2
7. (a) To see his full image in a plane mirror a person requires æ 1ö æ 3ö æ 1ö æ 3ö
çè ÷ø + ç ÷ çè ÷ø + ç - ÷
a mirror of at least half of his height. 2 è 2 ø 2 è 2 ø
8. (a) Real, inverted and same in size because object is at 1
the centre of curvature of the mirror. \ cos(180° - 2a ) = -
2
9. (d) Convex mirror always forms, virtual, erect and smaller
image.
10. (c) A-2 : For u = –f,
1 1 1
+
v -f = f
f
\ v= \ 180° – 2a = 120° \ a = 30°
2
v f /2 Option (a) is correct.
and M = - = - = 0.5 .
13. (d)
u (- f )
B-4 : u = –2f, so v = –2f 14. (a) From mirror formula
v æ -2 f ö 1 1 1 dv v 2 æ du ö
M = - = –ç ÷ = -1 + = so, =- 2ç ÷
u è -2 f ø v u f dt u è dt ø
C-1 : In concave mirror, u = –2f, v = – ¥ 2
dv æ f ö du dv 1
v Þ = -ç Þ = m/s
\ M = - = – ¥. dt è u - f ÷ø dt dt 15
u
D-5 : In convex mirror u = – 2f 15. (c) Min. width of plane mirror to see full face
1 1 1 2f D - d 10 - 4
so + = Þv= . = = = 3cm.
v -2 f f 3 2 2
v 1 16. (c) See geometry of the figure.
Now M = – = .
u 3
11. (d) f = 10 cm

B A 80°

–10 cm –20 cm 80° 10°

60° 50° 50° 120° 70°


40° 40°

The focal length of the mirror 17. (b) For concave mirror
1 1 1 2 1 1 2 1 1
– = + = + or = +
f v u R v u - R v -u
For A end of the rod the image distance 1 1 2 R - 2U é RU ù
\ = - = or v = ê ú
When u1 = – 20 cm v U R UR ë R - 2U û
1 -1 1 -2 + 1 In spherical mirror, image velocity
= +
v1 10 20 = 20 é v2 ù é RU ù v0
2 2
é R ù
v1 = – 20 cm vi = - ê ú
2 0
v = - ê R - 2U ú 2
= -ê ú v0
êë u úû ë û U ë R - 2U û
For when u2 = – 30 cm
RAY OPTICS AND OPTICAL INSTRUMENTS 663

18. (a) We resolve the velocity vector vr of the person into 25. (c) Convex mirror is used as a shaving mirror.
r
two components, one parallel to the mirror, v|| and the other
r r r r
perpendicular to the mirror, v^ , i.e. v = v|| + v^ (figure). O

The velocity of the image will


r r r 10 cm 15 cm
obviously be v¢ = v|| - v^ .

////////////////////////////////////////////
Therefore, the velocity at
which the person v||
approaches his image is v|| v v' From question : v = 15 cm, u = – 10 cm
defined as his velocity Radius of curvature, R = 2f = ?
relative to the image. From the v v 1 1 1
formula vrel = 2v^ = 2v sin a. Using mirror formula, + =
v u f
19. (d) The ray diagram is shown in figure. Therefore, the 1 1 1
image will be real and between C and O. + = Þ f = – 30 cm
15 (-10) f
Therefore radius of curvature,
Normal at P will
pass through C R = 2f = – 60 cm
C C 26. (c) For M1 : V = – 60, m1 = –2
I 1 1 1
Image For M1 : u = +20, F = 10 = = Þ V = 20
V 20 10
20
P \ M2 = - = -1 \ M = m1 × m2 = +2
O O 20
27. (c) Let x be the length of the shadow of the boy, when he
20. (c) The magnitude of the velocity of the image will be is at a distance y from the lamp. From similar triangles,
same as that of the object. The difference will be we have
maximum when the velocity of the image is opposite to h 3h
that of object. The velocity must be along normal to = or x = y/2. Street lamp
x x+ y
the mirror. dx dy / dt
1 1 1 Thus =
21. (d) For P : + = dt 2
v u f
3h
1 1 1 3
or + = or vP = - m h
vP -3 -1 2 v
1 1 1 -5
For Q : + = or vQ = m x y
vQ -5 -1 4 sin i v1 m 2
So horizontal distance between image of P and Q is 28. (b) According to Snell’s law sin r = v = m
0.25 m 2 1
2
From fig. we see that
v2 æ f ö r > i Þ v2 > v1 i medium 1 (m1)
22. (d) The axial magnification dv = - (du) = ç ÷ l. from Snell’s Law denser
u2 èu- f ø
So v2 = nl2 > v1 = nl1 rare r
90 – r
BD
23. (a) In D ABD, = tan 45° or BD = 3H D Þ l2 > l1
3H medium 2 (m2)
y (Frequency of wave does not change on refraction)
29. (b)
Source 30. (c) As i > ic
C
H i1 i2 At i = ic angle of refraction
45°
iA 45° q B ‘r’ = 90°

H H H 2H sin ic
\ =µ=1
sin 90°
BC 1
And in D A'BC, = tan q = or BC = H 31. (d)
2H 2
Now, y = BD – BC = 3H – H = 2 H. 32. (d) Total thickness = t; Refrative index = m
c
ur V 2 ur Speed of light in Glass plate = m
24. (a) Vim = 2 Vom
u é Speed of light in vacuum ù
40 æ –10 ö æ
2
40 ö –10 2 êQ v = ú
0– = –ç =± ë R.I. of Medium û
ç 10 – ÷ø
13 è –10 + x ÷ø è 13 –10 + x 3 t mt
Time taken = c = c
x = 25, – 5cm
Object is infront of mirror m
Hence 25 cm. (where t = thickness of glass plate)
EBD_7418
664 PHYSICS

33. (c) Let x = Apparent position of the silvered surface in cm. 42. (c) : For total internal reflection
Since the image is formed due to relfection at the silvered sin i > sin C
face and by the property of mirror image distance of object where,
from the mirror = Distance of image from the mirror i = angle of incidence, C = critical angle
or x + 8 = 12 + 6 – x or x = 5 cm. 1 1 1
Therefore refractive index of glass But, sinC = \ sini > or m >
m m sin i
Re al depth 6
= Apparent depth = 5 = 1.2 . 1
m> (i = 45° (Given))
34. (d) sin 45°
35. (b) For critical angle qc, m> 2
1 43. (a) 44. (a) 45. (d)
sin qc =
m 46. (b) We know that frequency of electromagnetic radiation
For greater wavelength or lesser frequency m is less. remains the same when it changes the medium. Further
So, critical angle would be more. So, they will not suffer wavelength of light in vacuum l v
reflection and come out at angles less then 90°. m= =
wavelength of light in medium l m
5 4
36. (a) Given that w m g = and a m w = lv l
4 3
lm = =
5 4 5 m m
\ a mg = w mg ´a m w = ´ =
4 3 3 velocity of light in vacuum
37. (b) aµg = 1.5 Similarly, m =
velocity of light in medium
\ 1.5 = cosecC or C = 42°. Critical angle for glass
= 42°. Hence a ray of light incident at 50° in glass v
medium undergoes total internal reflection. d denotes lm =
m
the deviation of the ray.
\ d = 180° – (50° + 50°) or d = 80°. 47. (b)
2
sin i n1 a/
38. (a) m = Q i = 2r a/2
sin r n2
2r
sin 2 r 2 sin r. cos r
Þ m= = r
a/2
sin r sin r r
æmö æmö a /2
r = cos -1 ç ÷ , \ i = 2 cos -1 ç ÷
è 2ø è2ø
39. (a) The incident and emergent ray of a glass slab are
parallel therefore, the angle remains the same.
48. (b) From similar triangles APB and CBD, point P will be
Oil m1
just visible if CD = 4 R
40. (a) µ sin i = 1 × sin r
Water m2 µ sin (ÐCDQ) = 1 × sin (ÐCDB)
Real depth R 2R 2 17 85
As refractive index, m = m´ =1´ Þm= =
Apparent depth R 17 R 20 20 5
\ Apparent depth of the vessel when viewed from æ 1ö æ 1 ö
above is 49. (b) Shift = (l - m) ç1 - ÷ + m ç1 - ÷ = 0
è n1 ø è n2ø
x x xæ 1 1 ö
dapparent = + = ç + ÷ y0 æ sin q ö
2m1 2m 2 2 è m1 m 2 ø 50. (a) = d tan r ; ç ÷=n
2 è sin r ø
x æ m2 + m1 ö x (m1 + m 2 ) q
= ç ÷ = æ sin q ö
2 è m1m 2 ø 2m1m2 y 0 = 2d ç ÷ r
ç 2 2 ÷ r
è n - sin q ø r y0
1
sin C = =
1
\ C = sin -1 æ 1 ö r
41. (d) çç ÷÷ = 45 º
m 2 è 2ø q

sin C 1 sin 45º 1


Now = or =
sin r m sin r 2
sin r = 1 or r = 90º d
RAY OPTICS AND OPTICAL INSTRUMENTS 665

51. (a) See figure. The ray will come out from CD if it suffers 1 æ 1 1 ö
total internal reflection at surface AD, i.e., it strikes the surface 56. (a) = (m - 1)çç - ÷
÷
f R
è 1 R 2ø
AD at critical angle C ( the limiting case).
A P D 1 æ1 1ö
Applying Snell's law at P = (1.5 - 1)ç - ÷
C n n2 16 èR ¥ø
1
n2
n1 sin C = n2 or sin C = Q 90°–C 1 1
n1 a Þ = 0.5 ´ Þ R = 8 cm
16 R
Applying Snell's law at Q 100 100
B C 57. (d) P2 = P - P1 = - = - 3 .75 D
n2 sin a = n1 cos C 80 20
n1 ìï æ n ö üï 58. (b) R1 = 60 cm, R2 = µ , m = 1.6
Þ sin a = cos ísin -1 ç 2 ÷ ý 1 æ 1 1 ö
n2 ïî è n1 ø ïþ = (m - 1)çç - ÷
÷
f R
è 1 R 2ø
é n1 ìï æ n ö üï ù
or a = sin -1 ê cos ísin -1 ç 2 ÷ ý ú 1 æ 1 ö
êë n2 ïî è n1 ø ïþ úû = (1.6 - 1)ç ÷ Þ f = 100 cm.
f è 60 ø
52. (b) For the image of point P to be seen by the observer, it m 1 m -1 2 1 2 -1
should be formed at point Q. 59. (c) Using, - = or - =
v u R v ¥ R
In DQNS, NS = QS = 2h \ v = 2R
\ ÐNQS = 45° 60. (a) anl = 1.6, anw = 1.33
N
f = 20 cm
\ r = 45° We have,
Now in DQMA, r
1 æ 1 1 ö
M 2h = ( a nl -1) ç - ÷
ÐMQA = 45° f è R1 R 2 ø
i
\ MA = QA = h 1 æ 1 1 ö
= (1.6 - 1) ç
45°
Q S 2h -
è R1 R 2 ÷ø
A 20 ..... (1)
1 sin r sin 45°
m= = ...(i) h
2 sin i sin i 1 æ 1 1 ö
In D PMB, Also, = ( w n l - 1) ç - ÷
P B f' è R1 R 2 ø
PM2 = 4h2 + h2 = 5h2 2h
æ n öæ 1 1 ö
h 1 = ç a l - 1÷ç - ÷
\ sin i = = ...(ii) è a nw ø è R1 R 2 ø
5h 5
5 1 æ 1.6 öæ 1 1 ö
From (i) and (ii) 12 µ = =ç - 1÷ ç -
2 f ' è 1.33 ø è R1 R 2 ÷ø ..... (2)
Dividing equation (1) by (2)
53. (c) Real depth = 5 cm + 1cm = 6 cm
Water f' 0.6 0.6 ´ 20
d1 d2 m = 1.33 Þ = f'= = 60 cm.
Apparent depth = + + .... 5cm 20 (1.2 - 1) 0.2
m1 m2 Hence it's focal length is three times longer than in air.
5 1 m = 1.5 1cm | P1 | 2 f
= + 61. (a) = Þ 2 =2 ...(i)
1.33 1.5 Glass | P2 | 3 f1 3
; 3.8 + 0.7 ; 4.5 cm Focal length of their combination
\ Shift = 6 cm – 4.5 cm @ 1.5 cm
1 1 1 1 1 1´ 3
1 æ m 2 öæ 1 1 ö = - Þ = -
54. (b) =ç - 1÷ç - ÷ f f1 f 2 30 f1 2 f1
f çè m1 ÷øçè R1 R 2 ÷ø from (i)
m2 1 1 é 3ù 1 æ 1ö
As <1 Þ = ê1 - ú = ´ ç - ÷
m1 30 f1 ë 2 û f1 è 2 ø
\ f is negative. It acts as divergent lens. \ f1 = – 15 cm
2 2
m 2 m1 m -m \ f2 = ´ f1 = ´ 15 = 10 cm
55. (c) - = 2 1 3 3
v u R R 62. (b)
1 1.5 1 - 1.5 O
63. (a) Let the distance between the lenses be d.
or - =
v –R –R Then, equivalent power is
\ v = – R = – 6 cm P = P1 + P2 – d P1 P2
EBD_7418
666 PHYSICS

Given P1 = P2 = + 5 D æ3 ö
ç - 1÷
\ P = (10 – 25d) D amg -1
= f è
2 ø
73. (d) fw = f =4f
For P to be –ve, æ amg ö æ 3/ 2 ö
ç - 1÷ ç - 1 ÷
2 è a mw ø è 4/3 ø
10 – 25d < 0 Þ d > m
74. (b) Using lens maker’s formula,
5
or, d > 0.4 m or d > 40 cm 1 æ 1 1 ö
64. (a) Let refractive index of glass be µ. = (m – 1) ç – ÷
f è R1 R 2 ø
Let after first refraction, image distance be v then
m 1 m -1 mR 1 æ 1.5 öæ 1 1 ö
- = Þv= =ç – 1÷ç – ÷
v ¥ R m -1 f1 è 1 øè ¥ –20 ø
Now second refraction will take place. Þ f1 = 40cm
So distance of first image from O is 1 æ 1.7 öæ 1 1 ö
=ç – 1÷ç – ÷
mR R f2 è 1 øè –20 +20 ø
u1 = -R= and image is formed at R
m -1 m -1 100
Þ f2 = – cm
1 m (m - 1) 2 (1 - m ) 7
\ - =
R R R 1 æ 1.5 öæ 1 1 ö
and =ç – 1÷ç – ÷ Þ f3 = 40 cm
f3 è 1 øè ¥ –20 ø
3+ 5
Þ m 2 - 3m + 1 = 0 So, m = 1 1 1 1 1 1 1 1
2 = + + Þ = + +
65. (d) Since refractive index for different wave length of f eq f1 f 2 f3 f eq 40 –100 / 7 40
light is different. Hence the different colours of light \ feq = –50 cm
forms images at different position. This phenomenon Therefore, the focal length of the combination is
is called chromatic aberration. – 50 cm.
66. (d) The cause of chromatic aberration is that lens 75. (c) For the image to be real and of
focusses different colours at different points. same size as object, final image
67. (c) should be formed at the position
68. (a) In this case, one of the image will be real and the other of object itself. C O
virtual. Let us assume that image of S1 is real and that of S2 Let x be the distance of object from X
µx
is virtual. f = 9 cm plane surface. Apparent distance
1 1 1 from surface = mx
Applying - = I This should be centre of curve
v u f S1 S2
y
For S1 : 1 + 1 = 1 ...(i)
x \ mx = 30 Þ 1.5x = 30 Þ x = 20
y x 9 24 – x 76. (c) If fe be the focal length of the lens, then
1 1 1 1 2 1
For S2 : + = ... (ii) = +
y 24 - x 9 fe 15 ¥
Solving eqs. (i) and (ii), we get or f e = 7.5 cm
x = 6 cm Now using mirror formula, we have
69. (a) 1 1 1
70. (b) Cutting a lens in transverse direction doubles their + =
v -20 -7.5
focal length i.e., 2f. \ v = – 12 cm
Using the formula of equivalent focal length. 77. (c) Longitudinal chromatic aberration = w f
1 1 1 1 1 = 0.08 × 20 = 1.6 cm
= + + +
f f1 f 2 f3 f 4 78. (b) Here, P1 = 5 D
f P2 = P – P1 = 2 – 5 = –3 D
We get equivalent focal length as .
2 w1 f - P2 3
71. (c) Since lR > lV mR< mV =- 1 = =
w2 f2 P1 5
æ 1ö æ 1 ö 79. (d) If fe is the effective focal length of the system, then
çèQ m µ ÷ø Þ f V < fR çèQ µ (m - 1)÷ø
l f 1 2
+
1
fe = fe f m
72. (b) Due to difference in refractive indices images obtained
will be two. Two mediums will form images at two 2 1
= +
different points due to difference in focal lengths. 12 ¥
\ fe = 6 cm
RAY OPTICS AND OPTICAL INSTRUMENTS 667

Now using mirror formula, 87. (c) f = 20 cm


f = – 6 cm
l
1 1 1 O
+ =
v -20 -6
\ v = – 8.6 cm 30 cm 10 cm
æ mg ö 60 cm
ç - 1÷÷
ç
Pa è m a +5 1 1 1 1 1 1
80. (b) = ø= = -5 - = ; - = Þ v = 60 cm
P1 æ m g ö - 100 / 100 v u f v - 30 20
ç ÷
ç m - 1÷
Coincidence is possible when the image is formed at
è 1 ø the centre of curvature of the mirror. Only then the
æ m g ö mg rays refracting through the lens will fall normally on
- 5 çç - 1÷÷ = -1 the convex mirror and retrace their path to form the
è m1 ø m a image at O. So the distance between lens and mirror =
1 .5 -1 1 .5 5 60 – 10 = 50 cm.
-1 = (1 .5 - 1) = -0 .1 ; m1 = =
m1 5 0.9 3 88. (a) The angular dispersion q i.e., the angle between the
µ2 µ µ -µ m1 m2 m3 extreme rays of light,
81. (b) - 1 = 2 1 q = (dV – dR) where dV = (mV–1)A, dR = (mV–1)A & A is
V -u0 -2R R
2R angle of prism.
µ3 µ 2 µ 3 - µ 2 So if refractive index increases, then d increases & hence
- = m0 V
Vf V -R q increases.
µ3 µ1 µ1 - µ2 µ2 - µ3 m - mR dm
+ = + 89. (b) Dispersive power of a prism w = V = ,
Vf u 0 2R R my -1 m -1
mV + mR
µ1 - µ2 where m = m y =
\ < µ3 - µ2 Þ µ1 - µ2 < 2µ3 - 2µ2 2
2 90. (a)
Þ µ1 + µ2 < 2µ3 91. (a) In the position of minimum deviation, i1 = i2.
82. (b) 92. (d) We can combine two prisms in such a way
83. (c) For reading purposes : (i) deviation is zero but dispersion not
u = – 25 cm, v = – 50 cm, f = ? (ii) dispersion is zero but deviation is not.
1 1 1 1 1 1 But in any situation both deviation & dispersion
= - =- + = ;
f v u 50 25 50 can not be zero simultaneously.
100 93. (c) In the position of minimum deviation
P= = +2 D
f A + dm 60 + 30
i=e= = = 45°
For distant vision, f' = distance of far point = –3 m 2 2
94. (c) At the minimum deviation, A
1 1
P = = - D = -0.33 D f = Dm angle of incidence
f¢ 3
84. (b) If R1 = R, R2 = –2 R Ði = angle of emergence
Ði Ðe
Ðe and inside the prism
1 æ 1 1 ö
= (m - 1) çç - ÷
÷
refracted ray parallel to O
f R
è 1 R 2ø the base of the prism B C
1 æ1 1 ö 0 .5 ´ 3 95. (d)
= (1.5 - 1) çç + ÷÷ = Q
6 è R 2R ø 2R 96. (c)
R = 4.5 cm
85. (a) Considering refraction at the curved surface,
u = – 20, µ2 = 1
µ1 = 3/2 , R = + 20 P
R
m 2 m1 m 2 - m1
Applying - = There will be no refraction from P to Q and then from Q to R
v u R (all being identical). Hence the ray will now have the same
1 3 / 2 1 - 3/ 2 deviation.
Þ - = Þ v = -10
v -20 20 97. (c) Applying Snell's law at P, 30°
i.e., 10 cm below the curved surface or 10 cm above the sin r
actual position of flower. m=
sin 30° P r
86. (d) d
1.44 30°
sin r = = 0.72
2
EBD_7418
668 PHYSICS

\ d = r – 30 ° = sin –1 ( 0.72) – 30° 105. (b) A = 90° – q A


\ The rays make an angle of Þ r2 = A = 90° – q > qc
r2
2d = 2 [sin – 1(0.72) – 30 °] with each other.. 6/5 4
cos q > sin qc = =
98. (c) For the prism as the angle of incidence (i) increases, 2/3 5
the angle of deviation (d) first decreases goes to (qc is critical angle)
minimum value and then increases. B C
4
99. (a) r2 < qC ; A - r1 < qC A q < cos -1 = 37°
5
r1 > A - qC 60°
i A + d m 60 + 30
r1 r2 106. (b) i= = = 45 º
Þ sin r1 > sin (A - qC ) 2 2
sin i µ 3
Þ > sin (A - qC ) 107. (d) i1 = i 2 = A
m 4
B C
Þ sin i > m (sin A cos q C - sin qC cos A) As A + d = i1 + i 2
3 3 A 60 º
7æ 3 3 3 1ö 1 1 \ d = i1 + i 2 - A =A+ A-A = = = 30 º
ç 1- - . = 1- = 4 4 2 2
7 2 ÷ø
=
3è 2 7 2 2 108. (a) For a thin prism, D = (m – 1) A
1 Since lb < lr Þ mr < mb Þ D1 < D2
Þ sin i > or i > 30°
2 109. (a) In a prism : r + r' = A Þ r = A – r'
100. (d) Dispersion will not occur for a light of single wavelength \ r = 60° – (10 + t2) = 50 – t2
l = 4000 Å
101. (d) The deviation produced as light passes through a sin 60° 1 2
110. (d) = mÞm=
thin prism of angle A and refractive index m is d = sin 90° 3
A(m – 1). We want deviation produced by both prism 111. (b) Clearly, ic £ 60°
to be zero. So, maximum possible value of ic is 60°.
d = d' 1
Þ A(m - 1) = A '(m '- 1) Now, 1µg = sin i 60° 30°
c 60°
4 ´ (1.54 - 1) µg 1
Þ A' = = 4 ´ 0.75 = 3° =
(1.72 - 1) µ1 sin i c
102. (d) For T.I.R. 45° > C 3
\ sin 45° > sin C or µ1 = µg sin ic = 1.5sin 60° = 1.5 ´
45° 2
1 4/3 = 1.5 × 0.866 = 1.299 = 1.3
\ >
2 n 45° 112. (a) A ray of light incident on face AB will just suffer inter-
4 2 nal reflection at the other face BC. If it gets incident on face
\ n> BC at critical C angle for the material of the prism. If critical
3
angle for material is C, then B
1 1
103. (d) As we know m = sin C = …(i)
sin C m i r1r2
1 2 Now, for prism we have Q
m = =
sin 60º 3 r1 + r2 = B or r1 + C = B A
For total internal reflection to take place C
r1 = B – C
1 2 sin i
sinC = = At face AB , m =
m 3 sin r1
sin i sin i
2 m= =
C = sin–1 @ 42º sin( B – C ) sin B cos C – cos B sin C
3
q < 45º sin i
m=
Angle of incidence > critical angle, hence TIR takes sin B 1 – sin 2C – cos B sin C
place Using relation (i), we get
dv - dR (n v - n R ) A sin i
104. (b) Dispersive power w = d =
(n y - 1)A
m=
1 æ1ö
y sin B 1 – 2 – cos B. ç ÷
nv - nR m èmø
=
n y -1
So, it depends only on the material of the prism
RAY OPTICS AND OPTICAL INSTRUMENTS 669

éæ 1 ö æ 1 öù f o 150
sin i = m êç 1 – 2 ÷ sin B – cos B ç ÷ ú Also, MP = = = 30
ç
êëè m ø÷ è m ø úû fe 5
æ 1 ö 50 1
sin i + cos B = mç 1 – 2 ÷ sin B a= = rad
ç m ÷ø 1000 20
è
sin i + cos B sin i + cos B ö
2 1 3
= m 2 – 1 or m 2 = 1 + æç ÷
\ b = q = MP × a = 30 ´ = = 1.5 rad
sin B è sin B ø 20 2
1/ 2 124. (c) Resolving power of microscope,
é ì sin i + cos B ü 2 ù
m = ê1 + í ý ú 2 n sin q
sin B R.P. =
êë î þ úû l
113. (b) Deviation = zero l = Wavelength of light used to illuminate the object
So, d = d1 + d2 = 0 n = Refractive index of the medium between object
Þ (m1 – 1)A1 + (m2 – 1) A2 = 0 and objective
Þ A2 (1.75 – 1) = – (1.5 – 1) 15°
q = Angle
0.5 125. (d)
Þ A2 = - ´ 15°
0.75
f 0 æ f e ö - 200 æ 1 + 5 ö = - 48
or A2 = – 10°. 126. (a) (i) M = - ç1 + ÷ = ç ÷
Negative sign shows that the second prism is inverted fe è dø 5 è 25 ø
with respect to the first. (since least distance d = 25cm)
f 200
æ A + dm ö (ii) M = - 0 = - = -40
sin ç
è 2 ÷ø sin 45°
fe 5
114. (a) m = = 127. (a) Given : f0 = 1.2 cm; fe = 3.0 cm
sin ( A / 2) sin 26.5°
u0 = 1.25 cm; M¥ = ?
= 1.58
115. (b) The image formed by objective lens of compound 1 1 1 1 1 1
From = - Þ = -
microscope is real and enlarged, while final image formed f 0 v0 u0 1.2 v0 ( -1.25)
by compound microscope is inverted, virtual, enlarged 1 1 1
and at a distance D to infinite or from an eye, on same Þ = - Þ v0 = 30 cm
v0 1.2 1.25
side of eye piece.
Magnification at infinity,
116. (b) The aperture of objective lens of Astronomical
telescope is large to get better resolution. Since v D 30 25
M¥ = - 0 ´ = ´
D u0 f e 1.25 3
resolution of telescope power is R = , where D
1.22 l (Q D = 25cm least distance of distinct vision)
is the diameter of the objective lens of Telescope. = 200
117. (b) One should increase the power of lens i.e., decrease Hence the magnifying power of the compound
the focal length of a lens. microscope is 200
118. (d) f0
128. (a) = 9 , \ f0 = 9 fe
fe
119. (d) Resolving power = l plane transmission grating
dl Also f0 + fe = 20 (Q final image is at infinity)
Resolving power for telescope 9 fe + fe = 20, fe = 2 cm, \ f0 = 18 cm
1 d d 129. (d) Given, Focal length of objective, f0 = 30 cm
= = = 0
limit of resolution 1.22l dl focal length of eye lens, fe = 3.0 cm
by increasing the aperture of objective resolving power Magnifying power, M = ?
can be increased. Magnifying power of the Galilean telescope,
120. (a) f0 æ f e ö 30 æ 3 ö
121. (b) In normal adjustment,
MD = ç 1 - ÷ø =
fe è D ç1 - ÷ [Q D = 25 cm]
3 è 25 ø
f f 60 22
M = 0 = 20 , f e = 0 = = 3 cm = 10 ´ = 8.8 cm
fe 20 20 25
122. (d) x 1.22l 1.22 ´ 5 ´ 103 ´ 10 -10 ´ 10 3
123. (c) Magnifying power of telescope, 130. (d) Here = or x =
1000 D 10 ´ 10 -2
b (angle subtended by image at eye piece) –3
or x = 1.22 × 5 × 10 m = 6.1 m
MP =
a (angle subtended by object on objective) x is of the order of 5 mm.
EBD_7418
670 PHYSICS

Exercise - 2 1 1 1 1
sin i c = \ sin 45° > or >
n n 2 n
1. (a) As we know that the deviation
d = (µ – 1) A ..... (i) Þn> 2
By geometry, the angle of refraction by first surface is 15. (b)
5° and given µ = 1.5 16. (b) By Lens maker's formula for convex lens
So, d = (1.5 – 1) × 5°
1 æ m öæ 2 ö
= 2.5° =ç - 1÷ ç ÷ for, m L = 4 , f1 = 4 R
also, d = q – r, ..... (ii) f è mL øè R ø 1 3
By putting the value of d and r in equation (ii) 5
2.5° = q – 5° for m L 2 = , f 2 = -5 R Þ f2 = (–) ve
3
So, q = 5 + 2.5 = 7.5° 17. (c) For total internal reflection, incident angle (i) > critical
2. (d) As we know that when light ray goes from one medium angle (ic)
to other medium, the frequency of light remains
unchanged. So, sin i > sin ic
And, c = nl 1
So, c µ l the light of red colour is of highest wavelength sin 45° >
m i=45°
and therefore of highest speed. Thus, after travelling Þ m > 2 Þ 1.414
through the slab, the red colour emerge first, Since refractive index m of 45°
3. (c) According to the question, when object is at different green and voilet are greater
position, and if an object approaches towards a than 1.414 so they will total
convergent lens from the left of the lens with a uniform internal reflected. But red colour will be refracted.
speed of 5 m/s, the image move away from the lens to 18. (c) The minimum length of the mirror is half the length of
infinity with a non-uniform acceleration. the man. This can be proved from the fact that Ði = Ðr.
4. (b) When a passenger in an aeroplane then he may see f1 w 1
19. (a) =- 1 =- \ f 2 = -2 f1
primary and secondary rainbow such as concentric f2 w2 2
circles.
1 1 1
5. (c) 6. (c) As = +
7. (b) The reflection of radiowaves by ionosphere is similar F f1 f 2
to total internal reflection of light in air during a mirage 1 1 1 1
\ = - = \ f1 = 10 cm
because angle of incidence is greater than critical angle 20 f1 2 f1 2 f1
so that internal reflection of radio wave, take place. f2 = –20 cm
8. (b) The incident PQ ray of light passes through focus F
on the concave mirror, after reflection should become 20. (d) From similar triangles,
M
parallel to the principal axis, i.e., ray-2. 30°
3 0° R
9. (b) As we know, when the ray goes from rarer medium air QC R
=
to optically denser turpentine, then it bends towards sin 30° sin120° P Q
30°
C
the normal i.e., i > r whereas when it goes from optically
denser medium turpentine to rarer medium water, then sin 30° R
or QC = R ´ =
it bends away from normal i.e., i < r. sin120° 3 R
So, the path of ray 2 is correct. R 1 ö
Thus PQ = PC – QC = R - = R æç1 -
10. (d) As we know that, the image formed by convex mirror 3 è 3 ÷ø
does not depend on the relative position of object w.r.t.
21. (a) If F be the equivalent focal length, then
mirror.
1 1 1
So, when the car approaches in the rear side, initially it = +
appear at rest as images is formed at focus. Hence the F f1 f 2
speed of the image of the car would appear to increase 1 æ 1 1ö æ 1 1 ö µ – µ2
= (µ1 –1) ç + ÷ + (µ2 –1) ç – ÷= 1
as the distance between the cars decreases. F è ¥ R ø è –R ¥ø R
11. (a) 12. (b) R
13. (b) For minimum deviation, incident angle is equal to F=
µ1 – µ2
emerging angle. 22. (b) i = 45°; A = 60°;
\ QR is horizontal. Angle of minimum deviation, dm = 2i – A = 30°
14. (b) The incident angle is 45° incident angle > critical angle, Refractive index of material of prism.
i > ic æ A + dm ö
sin ç ÷
\ sin i > sin i c or sin 45 > sin i c è 2 ø sin 45° = 1 · 2 = 2
m= =
sin A / 2 sin 30° 2 1
RAY OPTICS AND OPTICAL INSTRUMENTS 671

23. (d) Given: Focal length of objective, f0 = 40cm from smell’s law
Focal length of eye – piece fe = 4 cm 1 sin i = n sin r1
image distance, v0 = 200 cm A
Using lens formula for objective lens sin A = n sin
2
1 1 1 1 1 1 A A A
- = Þ = + 2 sin cos = n sin
v0 u 0 f 0 v0 f 0 u 0 2 2 2
A
1 1 1 +5 - 1 2 cos = n
Þ = + = Þ v0 = 50 cm 2
v0 40 -200 200 when A = 90° = imin
Tube length l = |v0| + fe = 50 + 4 = 54 cm.
24. (a) The phenomenon of total internal reflection takes place then nmin = 2
during reflection at P. i = A = 0 nmax = 2
1 31. (c) By covering aperture, focal length does not change.
sin q = … (i) 1
w
g m But intensity is reduced by times, as aperture
4
When q is the angle of incidence at P d
l diameter is covered.
2
B A
q I 3I
q \ I' = I - =
R P 4 4
3I
\ New focal length = f and intensity = .
4
C
32. (d) When mirror is rotated by angle q reflected ray will be
a rotated by 2q.
w gh 1. 5
Now, gm =
w
= = 1.125
light
gm
4/3
y spot q
1 8 = 2q
Putting in (i), sin q = = x y
1.125 9
8 y source 2q
\ sin q should be greater than or equal to Þq= Mirror
9 2x (L) x
25. (a) For normally emerge e = 0
Therefore r2 = 0 and r1 = A 33. (a) For dispersion without deviation
Snell’s Law for Incident ray’s (m - 1)A1 + (m '- 1)A2 = 0
1sin i = m sin r1 = msin A
(m - 1)A1 = (m '- 1)A 2
For small angle
i = mA (1.42–1) × 10° = (1.7–1)A2
26. (c) 4.2 = 0.7A2 Þ A2 = 6°
27. (c) 34. (b) For critical angle qc,
28. (b) A telescope magnifies by making the object appearing
closer. 1
sin qc =
29. (c) m
30. (b) A Prism angle
A For greater wavelength or lesser frequency m is less.
angle of
Smin
minimum So, critical angle would be more. So, they will not suffer
deviation reflection and come out at angles less then 90°.
Incident C e angle of 35. (b) Plane mirror may form real image, if object is virtual.
r1 r2
angle emergence

(Real) I O (virtual)
B C
The angle of minimum deviation is given as 1 æ 3 öæ 1 1 ö 1
dmin = i + e–A 36. (c) -1 - ÷ =-
f1 = çè 2 ÷çøè ¥ 25 ø 50
,
for minimum deviation
dmin = A then 1 æ 4 öæ 1
-1
1 ö
+ ÷=
3
2A = i + e f 2 = çè 3 ÷çøè 25 20 ø 100
in case of dmin i = e 1 æ 3 öæ 1 1ö 1
A and -1 - ÷ =-
2A = 2i r1 = r2 = f3 = çè 2 ÷çøè -20 ¥ ø 40
2
i = A = 90°
EBD_7418
672 PHYSICS

1 1 1 1 1 1 1
Now = + + So, = +
f f1 f 2 f3 f v2 u2
1 3 1 1 1 1 1 1 1
=- + - = – Þ = –
50 100 40 –15 v2 20 v2 20 15
\ f = – 66.6 cm \ v2 = –60 cm
37. (a) Cmax = 60° Therefore image shifts away from mirror by = 60 – 24
= 36 cm
1
\ r md = 42. (c)
sin 60°
m 1 m -1
mg 43. (a) Using, - =
2 v u R
or ml
=
3 1.5 1 1.5 - 1
or - =
3 3 v -15 +30
\ µl = mg = ´ 1.5 \ v = – 30 cm
2 2
= 1.3 Exercise - 3
3 -10 3
x 1.22l 1.22 ´ 5 ´ 10 ´ 10 ´ 10 1. (c) Consider the activitiy A to B A u=0
38. (d) Here = or x = 2
1000 D 10 ´ 10 -2 2 2
Applying v - u = 2 as
a = 10 m/s
s = 7.2 m
or x = 1.22 × 5 × 10–3 m = 6.1 m 20 m
v 2 - 0 2 = 2 ´ 10 ´ 7.2 Þ v = 12 m/s B
x is of the order of 5 mm. The velocity of ball as perceived by
fish is v
f0 12.8 m
39. (d) For telescope, angular magnification = 4
fE v ' = w m ´ v = ´ 12 = 16 m/s
3
So, focal length of objective lens should be large.
fish
D
Angular resolution = So, D should be large. 2. (b) Convex lens forms the image at I1. I1 is at the second
1.22l focus of convex lens. Size of I1 = 2 cm.
So, objective lens of refracting telescope should have I1 acts as virtual object for concave lens. Concave
large focal length (f0) and large diameter D for larger lens forms the image of I1 and I2.
angular magnification.
40. (b) For retracing the path, light ray should be normally
I1
incident on silvered face. I2
A = r + O Þ r = 30°
Applying Snell’s law at point M, 30°
26 cm 4 cm
sin i 2 60°
= M
30°
30 cm
sin 30° 1 i

1 For concave lens,


Þ sin i = 2 ´ 1 1 1 1 1 1 4 1
2 m= 2 - =- or = - + = =
1 v 4 20 v 20 4 20 5
or, sin i = i.e., i = 45° or v = 5 cm = Distance of I2 from concave lens.
2
v size of image 5
\ Magnification = = =
f = 15 cm u size of object 4
O 40 cm size of image
41. (b) or = 1.25
2
or size of image due to concave lens = 2.5 cm
3. (c) The image I ' for first refraction (i.e., when the ray comes
1 1 1 out of liquid) is at a depth of
using mirror formula, f = v + u 33.25
1 = = 25cm
1 1 1 1 1 1 1.33
– = + Þ = +
15 v1 u v1 –15 40 é Re al depth ù
êQ Apparent depth = m ú
\ v1 = –24 cm ë û
When object is displaced by 20 cm towards mirror Now, reflection will occur at concave mirror. For this I'
Now, u2 = –20 behaves as an object.
\ u = – (15 + 25) = – 40 cm
RAY OPTICS AND OPTICAL INSTRUMENTS 673

é 25 ù For B :
and v = - ê15 +
ë 1.33 úû 1
=
1
-
1 1 1 4 5
= - 1 = - or v 2 = - m
v2 -1 -5 or v2 5 5 4
25
Where is the real depth of the image. 3 æ 5ö 3 5 1
1.33
Now, v1 - v 2 = - ç - ÷ = - + = - m = -0.25 m
Using mirror formula we get 2 è 4ø 2 4 4
1 1 1 l1 v v æ -3 ö æ -1ö
= + , f = – 18.31 cm Again, = - 1 or l1 = - 1 O = - çè ø÷ èç ø÷ = -1 m
f v u O u u 2 3
4. (a) Here fo = 2 cm and fe = 3 cm. l v æ 5ö æ 1 ö
Using lens formula for eye piece Again, 2 = - 2 or l 2 = - çè - ÷ø çè ÷ø 2 = -0.5 m
O u 4 -5
-1 1 1 8. (c)
Þ + = Þ ue = – 3 cm
ue µ 3 9. (b) Distance of first t image (I1) formed after refraction
from the plane surface of water is
But the distance between objective and eye piece is
15 cm (given) 10
= 7.5 cm
\ Distance of image formed by the objective 4/3
from water surface æç d app = actual ö÷ .
= v = 15 – 3 = 12 cm. d
Let u be the object distance from objective, then for è µ ø
objective lens Now distance of this image is 5 + 7.5 = 12.5 cm from
the place mirror. Therefore distance of second image
1 1 1 -1 1 1 (I2) will also be equal to 12.5 cm from the mirror.
- + = or + =
u0 v0 f 0 u 12 2
-1 1 1 5 12 O
Þ = - = , u = - = -2.4 cm 10. (c) 5 cm 40 cm
u 2 12 12 5 h
4
5. (d) Given, µ = h 5 1
3 = =
h = 15 cm 20 40 8
R= ? 20 5
or h = cm = cm = 2.5 cm
R 8 2
sin 90° 11. (b) Velocity of image in mirror
=m r
sin C v = -10$i + 10 $j
1 R 3 r r r
Þ sin C = = = C v rel = u - v = 20$i.
m R 2 + h2 4 h 12. (d) Hypotenuse comes out to be 5 cm.
3 cm r = 90°
Þ 16 R2 = 9 R2 + 9 h2 1 sin i
or, 7R2 = 9 h2 Since, =
m sin 90° i
3 3 1 5 4 cm i
or, R = h= ´ 15cm m= =
7 7 O sin i 3
6. (a) Len's formula is given by coin
1 1 1 c 3 ´ 108
< , Speed, v = = = 1.8 ´ 108 m/s
f v u m 5/3
For convex lens, 13. (b) When lenses are in contact
1 1 1 1 1 1 1 1
< ∗ Þ < +
f1 f 2 = … (i)
30 v 60 60 v 60
Similarly for concave lens When lenses are at separation,
1 1 1 1 1 1 1 10 1
< , Þ < + - = … (ii)
,120 v 40 v 60 f1 f 2 f1 f 2 30
Virtual object 10 cm behind plane mirror. On solving above equations, we get
Hence real image 10 cm infront of mirror or, 60 cm from f1 = 20cm , f 2 = –30cm.
convex lens. 14. (d) From the ray diagram.
7. (d) For A : M
In DANM and DADB A 2L X N
u = –3 m, v1 = ?, f = –1 m ÐADB = ÐANM = 90° d/2
D L B
1 1 1 1 1 1 2 ÐMAN = ÐBAN
= - = - = - 1 = - or v1 = - 3 d/2
u1 f u -1 -3 3 3 2 (laws of reflection) X
EBD_7418
674 PHYSICS

Also ÐBAN = ÐABD For a ray propagating at a distance h1, the distance
Þ ÐMAN = ÐABD x1 » R/2, with an error of about 0.5% since h12 << R 2 .
x d/2
\ DANM is similar to DADB \ = or x = d For a ray propagation at a distance h2, the distance
2L L
x2 = 3.125cm. Finally, we obtain Dx = x2 – x1
So, required distance = d + d + d = 3d.
D x » 0.6cm.
15. (d) The focal length of the lens
v æ f ö
1 1 1 20 + 1 240 17. (d) m = - = - ç ÷
= - = Þ f = cm u èu - f ø
f v u 240 21
æ 1ö æ 1 ö 1 æ f ö
Shift = t ç 1 - ÷ Þ 1ç1 - ÷ = 1´ Now m1 = - ç ÷ … (i)
è 3/ 2 ø 3 è -f ø
25
è mø
1 35 æ f ö
Now v' = 12 - = cm and m2 = - ç ÷ … (ii)
3 3 è 40 - f ø
Now the object distance u. m1 40 - f 40 - f
\ = or 4 = 25 - f or f = 20 cm.
1 3 21 1 é 48 - 49 ù m2 25 - f
= - Þ= ê
u 35 240 5 ë 7 ´ 16 úû
u = –7 ×16 × 5 = – 560 cm = – 5.6 m 18. (b) Maximum number of reflection = 2 3
x
16. (c) Let O be the centre of the spherical surface of the mirror,
where x = 0.2 tan 30° = 0.2 3 .
ABC is the ray incident at a distance BE from the mirror axis,
and OB = R (figure)./////////////// l
/ ////
/// x
A / B
// /
/ / //
/////

D
h 0.2m
/////// /////////// //////

30° 30°
O C
E
x
19. (a) The formula for spherical refracting surface is
R -m1 m2 m 2 - m1
////

+ = m1 m2
/

Glass
/ ////

u v R Air
///

///
/

Here u = –x, v = + x,
////
//////
/// /// P O C Q (Real Image)
u v
From the right triangle OBE, we find that sin a = h/R. The R = + R, µ1 = 1, µ2 = 1.5
triangle OBC is isosceles since -1 1.5 1.5 - 1
Ð ABO = Ð OBC according to the law of reflection, and + = Þ x = 5R
-x x R
Ð BOC = Ð ABO as the alternate-interior angles. Hence
20. (a) Maximum angular magnification is achieved when final
OD = DB = R/2. From the triangle ODC, we obtain
image is formed at near point (25 cm).
R R2
x= = 1 1 1
2cos a 2 R 2 - h 2 So, - = [v = –25 cm, f = 10 cm]
(C is the point of intersection of the ray reflected by the v u f
mirror and the optical axis). u = –7.1 cm
WAVE OPTICS 675

25. WAVE OPTICS


18. (a) 19. (c)
Exercise - 1
20. (d) It will be concentric circles.
1. (a) Wavefront is the locus of all points, where the Dl
particles of the medium vibrate with the same phase. 21. (a) b= . When d ® 0 , b ® ¥ , and so fringes will not
d
2. (c) Converging spherical be seen over the screen.
3. (b) Huyghen’s principle gives us a geometrical method of 22. (a)
tracing a wavefront.
343
4. (d) The Huygen’s construction of wavefront does not 23. (b) Path difference = 171.5l = l
explain the phenomena of origin of spectra. 2
= odd multiple of half wavelength .
5. (b) Phase reversal occurs i.e. phase change = p takes place It means dark fringe is observed.
on reflection, because glass is much denser than water.
343
6. (b) For coherent sources l is same and phase is also same According to question, 0.01029 = l
or phase diff. is constant. 2
0.01029 ´ 2
lD Þl = = 6 × 10–5 cm
7. (a) As b = \ b µ l. 343
d Þ l = 6000 Å.
As l for violet is least, therefore, fringe nearest to 24. (b) Path difference, P
central achromatic fringe will be violet. Dx = S1P – S2P = d cos q. q
8. (c) A slit would give divergent; a biprism would give \ d cos q = 4l S1 d S2
double; a glass slab would give a parallel wavefront. or cos q = (4l/d)
Edge is downward.
9. (a) Bright fringes are yellow and dark fringes are black. 25. (a) For strong reflection, the least optical path difference
I1 1 introduced by the film should by l/2. The optical path
10. (b) = Þ I = k and I = 4k difference between the waves reflected from the two
I2 4 1 2
surfaces of the film is 2 md.
Thus, for strong reflection, 2md = l/2.
2 I1I 2 2 k ´ 4k
\ Fringe visibility V = = = 0.8 l 589
( I1 + I 2 ) (4 + 4k ) d= = = 118 nm.
4m ´
4 1.25
2 26. (a) Locus of equal path difference are lines running parallel
Imax 25 æa +a ö 25
11. (d) = or ç 1 2 ÷ = to axis of the cylinder. Hence straight fringes will be
Imin 9 a -
è 1 2ø
a 9 observed.
where a denotes amplitude. 27. (d) : Resultant amplitude,
a1 + a 2 5 A= A 21 + A 22 + 2 A1 A2 cos f
= or 5a1 - 5a 2 = 3a1 + 3a 2
a1 - a 2 3 Here, A1 = A2 1 cm, f = 3p rad
or, 5a1 – 5a2 = 3a1 + 3a2 or 2a1 = 8a2
\A = 12 + 12 + 2 ´1 ´1 ´ cos 3p
2
a1 æa ö I1
or, = 4 or ç 1 ÷ = 16 = . = 2 + 2 ´ (-1) = 0
a2 è a2 ø I 2
2p
12. (a) 28. (b) Phase diff. = x
l
13. (b) P to Q: convergence increasing; Q to R : direction
2 p ´ 2.1´10 -6
changing. Path difference = = 7.692 p radian.
14. (c) There can be three minima from central point to ¥ 5460 ´ 10 -10
l 3l 5l 29. (b) For reflection at the air-soap solution interface, the
corresponding to , , path differences. phase difference is p
2 2 2
\ total number of minima = 2nmax = 6.
15. (b)
16. (b) Air
17. (c) I max = I + 4I + 2 I ´ 4 I = 9I, Soap solution
µ = 1.4
Glass
and Imin=I + 4I –2 I ´ 4 I = I. µ = 1.5
EBD_7418
676 PHYSICS

For reflection at the interface of soap solution to glass


l 589 ´ 10-9 1
also there will be a phase difference of p 37. (c) sin q = = = 10-3 = = 0.0001
\ The condition for max. intensity = 2µt = nl d 0.589 ´ 10-3 1000
for n, nl1 = (n – 1)l2 38. (c)
n 420 nm = (n – 1) 630 nm 39. (b) For constructive interference d sin q = nl
n(630 - 420) = 630, \ n(210) = 630 nm . n
given d = 2l Þ sin q =
630 2
\ n= =3 n = 0, 1, –1, 2, –2 hence five maxima are possible
210 40. (d)
This is the maximum order where they coincide
41. (b) Introducing a converging lens in the path of parallel
3 ´ 420 beam does not introduce any extra path differences
2 ´ 1.4 ´ t = 3 ´ 420 Þ t = = 450 nm.
2 ´ 1.40 in a parallel beam. Rather it gives a more intense
l pattern on the screen.
30. (c) Dx =
(2a ) 42. (d) The angular position
d d cos 2q b l lD
31. (b) In D OPM, OP = In DCOP, OC = q= = (Q b = )
cos q cos q D d d
O M The first dark fringe will be at half the fringe width from
Q R the mid point of central maximum. Thus the angular
q q position of first dark fringe will be-
C q 1 él ù
a= =
d 2 2 êë d úû
1 é 5000 ´ p ù 180
q a= ê ´ 10-10 ú
-
2 ë .1 ´ 10 3
û p
P a = 0.45º.
43. (c)
Path difference between the two rays reaching P is 44. (d) For dark fringe
l d cos 2q d l lD
= CO + OP + = + + x = (2n - 1)
2 cos q cos q 2 2d
d l l
= (cos 2q + 1) + = 2 d cos q + 2 xd 2 ´ 10 –3 ´ 0.9 ´10 -3
cos q 2 2 \l = =
(2n - 1) D (2 ´ 2 - 1) ´ 1
For constructive interference, path difference should
be nl l = 0.6 ´ 10 -6 m = 6 × 10–5 cm
l (2n - 1) l 45. (d) Intensity I µ 1 + cos f where f = phase difference
\ 2d cos q + = nl Þ cos q =
2 4 d
IP 1 + cos 0° 1+1 2
l \ = = =
For n = 1, cos q = IQ p 1+ 0 1
4d 1 + cos
2
32. (d) : If source are coherent IR = I1 + I2 + 2 I1 I 2 cosf 46. (c) Dxmax = d = 5000 Å. Given l = 3000 Å
I0 = I + I + 2I cos 0° = 4I As l < d < 2l, \ n = 3.
If source are incoherent, 47. (c) The double slit interference pattern has a central
maxima followed by minima. The dark and bright
4 I I0
IR = I1 + I2 + 2I = = fringes are of equal width and intensity.
2 2
47. (d) x µ D
lD 1
33. (b) Fringe width b =
d \ If d becomes thrice, then X becomes times.
3
æ la ö 49. (d) At the area of total darkness minima will occur for
34. (b) The wavelength of light in water ç l w = m ÷ is less both the wavelength.
è ø
(2n + 1) (2m + 1)
than that in air. When the set-up is immersed in \ l1 = l2
water, fringe width b(µ l) will decrease. 2 2
( 2n + 1) 560 7
35. (b) For minima, phase diff. = odd integral multiple of or = =
p = (2 n – 1) p. (2m + 1) 400 5
36. (d) Interference pattern will be invisible, because red and or 10 n = 14 m + 2
green are complimentary colours. byinspection form = 2, n = 3
for m = 7, n = 10
WAVE OPTICS 677

the distance between them will be the distance l¢ D¢ 3 l 3 D lD


between such points 60. (c) b= = = 27 .
d¢ d/3 d
Dl1 ì ( 2 n 2 + 1) – ( 2 n1 – 1) ü
i.e., Ds = í ý No. of fringes = 1 / 3 = d .
d î 2 þ b 81l D
put n2 = 10, n1 = 3
lD
on solving we get, Ds = 28 mm 61. (d) Fringe width b =
50. (c) b = b¢ d
D 'l lDD
Dl \ Db =
or = (2d ) \ D¢ = 2D d
d
D Dbd 10-3 ´ 0.03 ´ 10-3 10-3 ´ 3 ´ 10-5
51. (c) Distance of nth maxima, x = nl µl or, l = = =
d DD 5 ´ 10-2 5 ´ 10-2
As l b < l g \ x blue < x green –7
= 6 × 10 m = 6000 Å.
52. (c) If a is the amplitude of wave, then 62. (b) As optical path SB of lower slit is increased, therefore,
I fringe pattern shifts somewhat downwards.
I = 4 a2, then I 0 = a 2 = . 63. (c) Wavelength for which maximum obtained at the hole
4
53. (a) has the maximum intensity on passing. So,
D 5 ´ 6.5 ´ 10 -7 ´ 1 n lD
54. (c) x5 = n l = = 32.5 ´ 10 -4 m x=
d -3 d
10
xd 1 ´ 10 -3 ´ 0.5 ´ 10-3 1 ´ 10 -6 1000nm
1 Dl 5 ´ 6.5 ´ 10 -7 ´ 1 l= = = =
x 3 = ( 2 n - 1) = nD n ´ 50 ´ 10-2 n n
2 d 2 ´ 10 -3
n = 1, l = 1000 nm ® Not in the given range
= 16.25 ´ 10 -4 m n = 2, l = 500nm
x 5 - x3 = 16.25 ´ 10 -4 m = 1.625 mm. 64. (d)
55. (b) The position of n th dark fringe. So position of first 65. (d) b = lD / d Þ b µ l
dark fringe in x1 = lD / 2d . The position of the central maxima is independent of
d = 20 cm, D = 0.1mm, l = 5460 Å, x1 = 0.16 wavelength used.
n lD
56. (c) When the arrangement is dipped in water; yn = \ yn µ l
d
x 3 66. (a) n th minimum has a distance from the centre =
b¢ =b / m = = x = 0.75x
4/3 4 1 lD
b 0.133 x = (2n + 1)
57. (b) b¢ = = = 0.1 cm 2 d
m 1.33 For a point on the screen directly in front of one of
2lD the slits, x = d/2
58. (d) Width of central maxima in single slit pattern = \ for minimum intensity in front of one of the slits
a
lD d l D d2
Fringe width in double slit experiment b = = = (2 n + 1) \ l=
d 2 2 d (2n + 1) D
10 l D 2 l D 67. (d) Due to reflection virtual source will be formed at
So, required condition = distance D from mirror.
d a
The effective distance of the screen = 2D + 2D = 4D
d 1
Þ a== ´ 10–3 m = 0.2 mm 4Dl
5D 5 \ Fringe width =
59. (b) For path difference l, phase d
68. (a) Path difference between first and 11th bright fringe
æ 2p 2p ö = S1B (10 bright fringes) = 10 l
difference = 2p ç Q = x = .l = 2p ÷
è l l ø = 10 × (6000 × 10–7)m = 6 × 10–6 m
Þ I = I0 + I0 + 2I0 cos 2p Þ I = 4I0 (m - 1)tD lD D b
69. (c) n= but b = Þ =
l p d d d l
For x = , phase difference =
4 2 n = (m - 1) t b / l
p 20b = (m – 1) 2.5 × 10–3 (b/5000 ×10–8)
\ I' = I1 + I 2 + 2 I1 I 2 cos
2 20 ´ 5000 ´ 10 -8
I I m -1 = Þ m = 1.4
If I1 = I2 = I0 then I ' = 2I0 = 2. = 2.5 ´ 10 -3
4 2
EBD_7418
678 PHYSICS

L
( )
2
70. (d) E1= I0 = 2
, without mirror Imax = I1 + I2
d
s' d d s d
( ) = (2 I')
2 2
= I' + I' = 4I'
L L L é 1 ù 10I 0 I 3
an d E 2 = + = \ =
d (3d ) 2 2 ê1 + ú = 9 with I max 4
d2 ë 9û
mirror. nlD
71. (d) For maxima d sin q = nl 82. (a) Q y =
d
nl 8l 4 4 \ n1 l1 = n2l2
sin q = = Þ sin q = Þ tan q =
d 10l 5 3 Þ n1 × 12000 × 10–10 = n2 × 10000 × 10–10
y 4D or, n (12000 × 10–10) = (n + 1) (10000 × 10–10)
Also tan q = \ y=
D 3 Þn=5
72. (a)
nl1D
73. (a) Let q be the angular width in water. We know angular Hence, ycommon =
d
l
width =
d
Þ Angular width µ l =
(
5 12000 ´ 10-10 ´ 2 )
q l 2 ´ 10-3
= w ...... (i)
0.4° l a (Q d = 2 mm and D = 2m )
l l 4
Now, a m w = a Þ a = = 5 × 12 × 10–4 m
lw lw 3 = 6 × 10–3m = 6 mm
Hence from eq. (1), we have 83. (b) Width of central maximum in diffraction pattern due
q 3
= Þ q = 0.3° 2lD
0.4° 4 to single slit = where l is the wavelength, D
lD d
74. (a) Fringe width w = . is the distance between screen and slit and a is the
d
When the apparatus is immersed in a liquid, then l will slit width.
decrease µ times and hence w is reduced µ (refractive As the slit width a increases, width of central
index) times.10w ' = (5.5) w maximum becomes sharper or narrower. As same
æ Dö lD l 10 energy is distributed over a smaller area. therefore
or 10l ¢ ç ÷ = (5.5) or = = m or µ = 1.8
èdø d l¢ 5.5 central maximum becomes brigther.
84. (b) At the centre, all colours meet in phase, hence central
75. (a) (m - 1)t = nb
fringe is white.
(m1 - 1) ´ 1.8 ´ 10 -5 18b 85. (b) Light transmitted by nicol prism is plane polarised.
=
(m 2 - 1) ´ 3.6 ´ 105 9b 86. (c) Such substances rotate the plane of polarised light.
(m1 - 1) = 4(m2 - 1) 4m2 - m1 = 3 87. (c) Because both source & screen are effectively at infinite
76. (b) 77. (c) distance from the diffractive device
nDl1 ( n + 1) Dl 2 88. (d)
78. (c) yn = = 89. (b) Angle between plane of vibration and plane of
d d
æ l2 ö æ l1l 2 ö æ D ö polarisation is 90º.
or n = ç ÷ and yn = ç ÷ç ÷. 90. (d) Diffraction on a single slit is equivalent to interference
l
è 2 - l1ø è l 2 - l1 ø è d ø
of light from infinite number of coherent sources
b ( m - 1) b ( m - 1) b ( m - 1) contained in the slit.
79. (c) Shift =
l
t1 -
l
t2 =
l
( t1 - t2 ) 91. (c) In Fraunhoffer diffraction, for minimum intensity,
80. (d) l
81. (a) For path difference of l, the phase difference is 2p Dx = m
2
l l
For path difference of , the phase difference is For first minimum, m = 1 \ Dx =
6 2
2p ´ l / 6 l 92. (a)
= 93. (b) According to principle of diffraction, a sin q = nl
l 3
where, n = order of secondary minimum
p
\ Intensity I = I1 + I 2 + 2 I1 I 2 cos or, a sin 30° = 1 × (6500 × 10–-10)
3 or, a = 1.3 × 10–6 m, or, a = 1.3 micron.
\ I = I1 + I2 + I1 I2 94. (d) The angle of incidence for total polarization is given
when I1= I2 = I¢ (say) then I = 3I¢ by tan q = n Þ q = tan -1 n
WAVE OPTICS 679

95. (b) I = I 0 cos 2 q c 3 ´ 108


or, vg = = = 3 ´ 108 ms -1
I0 3 3
Intensity of polarized light = 105. (c) Suppose intensity of unpolarised light = 100.
2
\ Intensity of polarised light from first nicol prism
I0 I0
Þ Intensity of untransmitted light = I 0 - = Ι 1
2 2 = 0 = ´ 100 = 50
96. (c) Path difference between the opposite edges is l. 2 2
According to law of Malus,
For a phase difference of 2p we get a path diff. of l.
2
97. (c) When the wavelength of light used is comparable æ1ö
Ι = Ι0 cos 2 q = 50 (cos 60º )2 = 50 ´ ç ÷ = 12.5
with the separation between two points, the image è2ø
of the object will be a f diffraction pattern whose 106. (d)
size will be 107. (d) For diffraction pattern to be observed, the dimension
1.22l of slit should be comparable to the wave length of
q= rays. The wavelength of X-rays (1 – 100 Å) is less
D
where l = wavelength of light used than 0.6 mm.
D = diameter of the objective c 3 ´ 108
108. (c) l = f = = 0.214 m
Two objects whose images are closer than this 1420 ´ 106
distance, will not be resolved. 1.22l 1.22 ´ 0.219
sin q = = = 0.010
98. (c) At Brewster’s angle, only the reflected light is plane d 25
polarised, but transmitted light is partially polarised. or q = 0.6 °, and 2q = 1.2°
99. (c) Relation between intensities 2l
æI ö 45° B 109. (a) sin q2 =
IR = ç 0 ÷ cos 2 (45°) I0 (I0/2)
d
è2ø -1 æ 2l ö
(unpolarised) IR or q2 = sin ç ÷
I 1 I è d ø
= 0´ = 0 Given y = D(2 q2 ) = 8 × 10–2
2 2 4 A
100. (b) Intensity of light transmitted by polariser is half of
2l ö
intensity of unpolarised light. or 1.5 ´ 2 ´ sin -1 æç ÷ = 8 × 10
–2 Þ d
; 0.005 cm
è d ø
101. (a) For a circularly polarised light electric field remains
110. (d) Conditions for diffraction minima are
constant with time.
Path diff. Dx = nl and Phase diff. df = 2np
Path diff. = nl = 2l
Phase diff. = 2np = 4p (Q n = 2)
|E| 111. (b) Intensity of polarised light transmitted from 1st
polariser,
1
I1 = I0 cos2q but (cos2q)av =
t 2
102. (b) If the angular limit of resolution of human eye is R 1 32
So I1 = I0 = = 16Wm -2
then 2 2
1.22l 1.22 ´ 5 ´ 10 -7 112. (a) The angular width of central maxi. is
R= = rad
a 2 ´ 10 -3 l 2 ´ 6328 ´ 10 -10
2q = 2 = radian .
1.22 ´ 5 ´ 10 -7
180 a 2 ´ 10 - 4
= ´
´ 60 mi nute = 1 minute
2 ´ 10 -3 p 180
= 6328 ´ 10-6 ´ degree = 0.36º
103. (a) We know that for maxima p
l 113. (d) By principle of polarization, µ = tanqp
b sin q = (2n + 1) or µ = tan 54.74° or µ = 1.414
2
For an equilateral prism, ÐA = 60°
2n + 1 æ l ö
or sin q = ç ÷ æ A + dö æ 60° + d ö
2 èbø sin ç ÷ sin ç
è 2 ø è 2 ÷ø
So on decreasing the slit width, ‘b’, keeping l same, \ m= =
sin q and hence q increases. sin ( A / 2) sin ( 60° / 2)
104. (b) aµg = tan qP where qP = polarising angle. 1.141 ´ 1 æ 60° + d ö é
or, = sin ç Q1.414 = 2 ùû
or, aµg = tan 60° 2 è 2 ÷ø ë
c 1 æ 60° + d ö æ 60° + d ö
or, = 3 Þ = sin ç
vg è ÷ø or, sin 45° = sin çè 2 ÷ø
2 2
Þ d = 30°
EBD_7418
680 PHYSICS

114. (d) It is a one of Fraunhoffer diffraction from single slit. 5. (d) Consider the given figure there is a hole at point P2. By
so for bright fringe where a is the width of slit. Huygen’s principle, wave will propagates from the
l sources S1 and S2. Each point on the screen will acts as
a sin q = (2 n + 1) sources of secondary wavelets.
2
Wavefront starting from P2 reaches at S3 and S4 which
2 a sin q 2 ´1.2 ´10 -5 ´ 0.0906 will again act as two monochromatic or coherent sources.
l= = = 7248Å.
2 n +1 2 ´1 + 1 Hence, there will be always a regular two slit pattern on
115. (a) Where n is equivalent number of fringe by which the the second screen.
centre fringe is shifted due to mica sheet 6. (d) Given: D = 2m; d = 1 mm = 1 × 10– 3 m
l = 600 nm = 600 × 10– 6 m
(m - 1) t (1.5 - 1) 6 ´10 -6 Width of central bright fringe (= 2b)
l= = = 6000 Å
n 5
2lD 2 ´ 600 ´ 10 - 6 ´ 2
= = m = 2.4 mm
Exercise - 2 d 1 ´ 10 - 3
7. (c) For path difference l, phase difference = 2p rad.
1. (c) Let us consider the diagram shown below the light l p
beam incident from air to the glass slab at Brewster’s For path difference , phase difference = rad.
4 2
angle (ip). The angle between reflected ray BE and BC As K = 4I0 so intensity at given point where path
is 90°.
l
Then only reflected ray is plane polarised represented difference is
by arrows. 4
As the emergent and incident ray are unpolarised, so, 2 æ pö æ p ö K
K¢ = 4I0 cos ç ÷ ç cos = cos 45º÷ = 2I0 =
polaroid rotated in the way of CD then the intensity è 4ø è 4 ø 2
cannot be zero but varies in one complete rotation. 8. (a) As b µ l
N
Polarised 5
\ fringe width becomes times,
A E 4
iP 4
No, of fringes = ´ 10 = 8
5
B 90º 9. (b)
r 10. (d) According to malus law, intensity of emerging beam
is given by, I = I0cos2q
C
Now, IA' = I A cos230º and I B ' = I B cos 2 60º
3 1 IA 1
Polaroid D
As I A ' = I B ' Þ I A ´ = IB ´ Þ =
(P) 4 4 IB 3
2. (a) As given that the width of the slit 11. (a) Here the width of principal maxima is 2.5 mm, therefore
=104 Å = 10000 Å b 2.5
= 104×10–10 m = l0–6 m = 1 µm its half width is = = 1.25 ´ 10-3 m
2 2
Wavelength of visible sunlight varies from 4000 Å to
b / 2 1.25 ´ 10 -3
8000 Å. Diffraction angle q = =
Thus the width of slit is 10000 Å comparable to that of D 2
wavelength visible light i.e., 8000 Å. So diffraction occurs 1.25 ´ 10-3
with maxima at centre. Hence at the centre all colours \aq=l \ q = l/a =
2
appear i.e., mixing of colours form white patch at the
centre. 1.25 ´ 10 -3 1.25 ´ 10 -3 ´ 10-3
l= ´a =
3. (a) 2 2
4. (c) For sustained interference pattern to be formed on the l = 6.25 × 10–7 m = 6250 Å.
screen, the sources must be coherent and emits lights 12. (b)
of same frequency and wavelength. 13. (a) a sin q = nl
ax
In a Young’s double-slit experiment, when one of the = 3l
holes is covered by a red filter and another by a blue f
(since q is very small so sin q » tan q » q = x / f )
filter. In this case due to filteration only red and blue
lights are present which has different frequency. In this a x 0.3 ´10 -3 ´ 5 ´10 -3
or l = =
monochromatic light is used for the formation of fringes 3f 3 ´1
on the screen. So, in that case there shall be no = 5 ´ 10-7 m = 5000 Å.
interference fringes. 14. (a) When angle of incidence i is equal to angle of
polarisation i.e, then reflected light is completely plane-
WAVE OPTICS 681

polarised whose vibration is perpendicular to plane x is same for each.


of incidence. \ 5 ´ 6500 = 7 ´ l
15. (d)
5
Plane WF Þ l = ´ 6500 = 4642.8 Å
µ increases 7
16. (b) (Light bends Vel decreases 22. (a)
upwards) 23. (a)
Refracted B B¢
WF I 9
I/4 I
17. (d) Given: fringe with b = 0.03 cm, D = 1 m = 100 cm 64
Distance between images of the source = 0.8 cm. A A¢ 3
I
Image distance v = 80 cm 3 64
I 3
Object distance = u 4 I
16
Using mirror formula,
1 1 1 1 1 1
+ = Þ + = Þ u = 20 cm
v u f 60 u 16 I 9I I 9
From figure I1 = and I2 = Þ 2 =
v 80 4 64 I1 16
Magnification, m = = =4
u 20 2 2
æ I2 ö æ 9 ö
distances between images of slits
Magnification = ç I + 1÷ ç 16 + 1÷
distance between slits Imax 49
By using =ç 1 ÷ =ç ÷ =
0.8 0.8 Imin ç I2 ÷ ç 9 ÷ 1
= = = 4 Þ d = 0.2cm ç - 1÷ çè - 1÷
d d è I1 ø 16 ø
Dl 24. (c) According to question
Fringe width b = 8th bright fringe in medium = 5th dark fring in air
d
100l lD
or, b = = 0.03 ´ 10 -2 Y8th bright = 8
0.2 md
Therefore, wavelength of light used l = 6000 Å l D 9 lD
Y5th dark = (2 × 5 – 1) =
18. (b) The path difference when transparent sheet is 2d 2 d
introduced Dx = (m – 1)t 9 lD lD 16
Þ =8 or, refractive index m = = 1.78
If the central maxima occupies position of nth fringe, 2 d md 9
then (m – 1)t = n l = d sin q 25. (b)
( m - 1) t (1.17 - 1) ´1.5 ´10-7
Þ sin q = = = 0.085
d 3 ´ 10-7
Therefore, angular position of central maxima
q = sin -1 ( 0.085 ) = 4.88° » 4.9
For small angles, sin q » q » tan q
y According to malus law, I = I0cos2 q
Þ tan q =
D I
I1 = 0
y ( m - 1) t D ( m - 1) t 2
\ = Þ y= I0 1 I0
D d d I
I2 = 0 cos 45° = 2 ´ 2 = 4
2
19. (d) 2
20. (d) For the first minima,
I0 I
hl l 1 I3 = cos2 45° Þ I3 = 0
q= Þ sin30° = = 4 8
a a 2
First secondary maxima will be at 2m sin q
26. (b) Resolving power of a microscope =
3l 3 æ 1 ö æ 3ö l
sin q = = ç ÷ Þ q = sin–1 ç ÷
2a 2 è 2 ø è 4ø 1 R1 l 2
=
( 2n + 1 ) l D i.e., R µ
l
or,
R 2 l1
21. (c) x =
2a
R1 6000Å 3
( 4 + 1) D \ R = =
For red light, x = ´ 6500Å 2 4000Å 2
2a
27. (d) When red light is replaced by blue light the
( 6 + 1) D ´ lÅ
For other light, x = diffraction bands become narrow and crowded.
2a
EBD_7418
682 PHYSICS

28. (b) Ddu


29. (d) For common maxima, n1l1 = n2l2 v0 = (downwards)
2(x 0 - ut)2
n1 l 2 520 ´10-9 4
Þ = = = 34. (a) Path difference
n2 l1 650 ´ 10-9 5 nd
II maxima
5l
For l1 a sin q = y
2
q
n1l1 D 5l
y= , l1 = 650 nm a tan q =
d 2 D
4 ´ 650 ´ 10 -9 ´ 1.5 y 5l
y= or, y = 7.8 mn a× =
0.5 ´ 10-3 D 2
30. (b) Dxmax = 0 and Dxmax = 2l 2ay 2 ´ 0.7 ´ 10 -3 ´ 2 ´ 10 -3
l= =
Theortical maximas are = 2n + 1 = 2 × 2 + 1 = 5 5D 5 ´1
But on the screen there will be three maximas. 2.8
= ´ 10–6 meter
l 5
31. (b) Angular width =
d 28
= ´ 10–7 = 5600 Å
l 5
So, 0.20° =
2mm
Þ l = 0.20° × 2 15°
l I0 I0 /2
Again, 0.21° = 35. (a)
d
Now putting the value of l
0.20°´ 2mm
d=
0.21°
\ d = 1.9 mm I0
I= cos2 (15°)
32. (b) When reflected light rays and refracted rays are 2
perpendicular, reflected light is polarised with electric I0
field vector perpendicular to the plane of incidence. I= × 2 cos2 (15°)
4
Plane polarised I0
reflected light I= [1 + cos (30°)]
4
i
90° I0 é 3ù
I= × ê1 + ú
4 ë 2 û
m Partial polarised
I0
refracted light I= é2 + 3 ù
8 ë û
Also, tan i = m (i = Brewester angle) I = 0.46 I0d
d d 36. (c) Axis of transmission of A & B are parallel.
33. (a) tan q = and y0 = D tan q = D ×
2x 2x Polariser A Polariser B
while light
i = constant

d/2 q
q I I/2 I/2
d/2
D tan q
After introducing polariser C between A and B,
Polariser A Polariser C Polariser B

D
dy0 - Dd dx Dd
v0 = = × Þ v0 = ×u
dt 2x 2 dt 2x 2 I I/2 I/2Cos2q I/2Cos4q
WAVE OPTICS 683

I I 1 I0
cos4 q = Þ cos4 q = = cos 2 q sin 2 q
2 8 4 2
1 I0
Þ cos q = or, q = 45° = sin 2 2q
8
2 9. (b) wa = l/d Þ wa a l
2l (w a )water l water
37. (a) Angular width of central maxima = l
d = =
wa l m water l
d l´D 2´3
or, l = ; Fringe width, b = (wa)water = = 0.15º.
2 d' 4
d 50 ´ 10-2 10 -6 ´ 50 ´10 -2 10. (c)
10-2 = ´ =
2 d' 2´d'
Therefore, slit separation distance, d’ = 25mm
38. (a) 39. (c) d cos q 2d cos q

Exercise - 3 q q
d d
1. (c) y3 = y4
\ for minimam
3 ´ D ´ 590 4´ D´l l l 1
or = d cos q = Þ cos q = =
d d
\ l= 442.5 nm. 3 3d 9
x
Dl tan q = 4 5 = Þ x = 4 5D
2. (a) b = D
d
( DD ) l 11. (d) Optical path difference
or Db = Dx = (m2 – m1)t.
d
DP1 sin a – sin b a –b a+b
(5 ´ 10-2 )l 12. (d) = = tan cot
or 3 ´ 10-5 = DP2 sin a + sin b 2 2
10-3
\ l = 6000 Å . 13. (b) IA = I + 4I + 2 I ´ 4 I cos p / 2 = 5I
D –10 2
3. (b) x = (n )l = 3 ´ 5000 ´ 10 ´ and I B = I + 4I + 2 I ´ 4I cos p = I
d 0.2 ´ 10 – 3
–2 So I A - I B = 5 I – I = 4 I
= 1.5 ´ 10 m = 1.5 cm
14. (b) Two images of the source are shown in figure.
4. (a) 5. (c)
S1 acosq
6. (a) Suppose the mth bright fringe of 6500Å coincides with
the nth bright fringe of 5200Å acosq
ml1D nl 2 D q S
Xn = = d
d d q
m ´ 6500 ´ D n ´ 5200 ´ D acosq
Þ =
d d Screen
S2 acosq D
m 5200 4
Þ = = The separation between the two images,
n 6500 5
d = 2 (2acos q sin q)
\ distance y is = 2a sin 2q
ml1D 4 ´ 6500 ´ 1010 ´ 1.2 D = (2a cos q) cos q + 4 a
y= =
d 2 ´ 10-3 = 2a cos 2 q + 4a
y = 0.156 cm. For small q, sin q ; q and cos q = 1
7. (b) Constructive interference happens when \ d = 2a ´ 2q = 4a q ,
2t = ( m - 1/ 2)l . The minimum value for m = is m = 1, the and D = 6a.
Now fringe width,
maximum value is the integer portion of 6a ´ l
Dl 3l
b= = 4a q =
2d 1 2 ´ 0.034 ´ 10 -3
1 d 2q
+ = + = 100.5 D b
l 2 680 ´ 10-9 2 15. (a) Shift = (m - 1) t = (m - 1) t = 4b
d l
mmax = 100
4l 4 ´ 6000 ´ 10-10
t= = = 4.8 µm
éæ I ö
0 2 ù 2 m -1 1.5 - 1
8. (a) I = êç ÷ cos q ú cos (90° - q)
ëè 2 ø û
EBD_7418
684 PHYSICS

26. DUAL NATURE OF RADIATION AND MATTER

Exercise - 1 11. (a) The de-Broglie’s wavelength associated with the


h
1. (c) As cathode rays are deflected by electric and mangetic moving electron l =
fields, it shows that cathode rays carry charged P
particles (i.e. electrons) Now, according to problem
2. (b)
3. (b) Here the velocity of electron increases, so as per dl dp
=-
Einstein’s equation mass of the electron increases, l P
e
hence the specific charge decreases. 0.5 P
m =
4. (a) Specific charge (e/m) for cathode rays is constant as e 100 P '
and m for cathode rays are fixed. P¢ = 200 P
h 1- v2 / c2 12. (d) The force on cathode rays (i.e. charge e) moving with
5. (b) l =
h
= = 0 (Q v = c) r r
mv m0 v velocity v due to a perpendicular magnetic field B is
h r r r
6. (c) de-Broglie’s relation, l = -e(v ´ B) , which acts perpendicular to plane of v
p r
and B .
momentum p = 2mE 13. (d) de-Broglie wavelength,
h h
Þl=
2mE
=
2mK
(Q E = K ) l=
h
=
h
p 2.m.(K.E)
h h v p me
7. (c) l = = ; then mp vp = me ve or = 1
m p v p me v e ve m p \ lµ
K. E
1
Ep m p v 2p m æm ö
2
me
l
2 ´ç e ÷ = m <1
p
= = If K.E is doubled, l becomes
Ee 1 2 me ç mp ÷ p
2
me ve è ø
2 14. (b) As we know
\ Ep < Ee h h
1.227 h h l= =
nm , as l = = P 2mK
8. (c) l =
V p 2meV (Q P = 2mKE)
Substituting the numerical values of h, m and e we
l1 K2 16K 4
get the result. or = = =
9. (a) In discharge tube cathode rays (a beam of negative l2 K1 K 1
particles) and canal rays (positive rays) move opposite Therefore the percentage chanbe in de-Broglie
to each other. They will experience a magnetic force in 1- 4
the same direction, if a normal magnetic field is applied. wavelength = ´ 100 = - 75%
4
B 15. (b)
v + –
1
16. (d) qV = m v 2 or m v = 2q Vm ;
2
h h 1
F F So l= = i.e. l µ ;
mv 2q V m qm
1
10. (a) According to relation, E = mv 2
2 lp qama
so = = 2´ 4 = 2 2
2E la q pm p
=v
m 17. (d)
h eE
l= 18. (a) Here, u = 0 ; a = ;v=?;t=t
m
2mE eE
Because m1 < m3 < m2 \ v = u + at = 0 + t
m
So for same l, E1 > E 3 > E 2 .
DUAL NATURE OF RADIATION AND MATTER 685

h h h 36. (d) The work function of a metal depends upon the


de-Broglie wavelength, l = = =
mv m(eEt / m ) eEt properties of the metal and the nature of its surface.
Rate of change of de-Broglie wavelength 37. (a) For the longest wavelength to emit photo electron
dl h æ 1 ö -h
= ç - ÷÷ =
ç hc hc
dt eE è t ø e E t 2
2 =fÞl=
l f
h h h
19. (b) l= = = 6.63 ´ 10-34 ´ 3 ´ 108
p 2mE 2mqV Þl= = 310 nm
4 ´ 1.6 ´ 10-19
6.6 ´ 10-34 200
\ l=
(
2 ´ 1.6 ´10 -27
) ´ (1.6 ´10 ) ´1000
-19 38. (a) The energy of each photon =
4 ´ 1020
= 5 × 10–19 J

hc
6.6 ´ 10-34 ´ 10 22 Wavelength = l =
or l = = 0.9 ´ 10-12 m E
1.6 20
20. (b) (6.63 ´ 10-34 ) ´ (3 ´ 108 )
=
h 1 5 ´ 10-19
21. (c) l= So h µ
2mE m Þ l = 4.0 × 10–7 = 400 nm
since ma > mn > mp > me (l 0 )1 (W0 ) 2 4.5
so de-Broglie wave length in increasing order will be 39. (c) hc/ l 0 = W0 ; (l ) = ( W ) = 2.3 = 2 : 1.
0 2 0 1
ld, lm, lp, le 40. (a) For photoelectric effect
Vo
22. (d) 23. (d) 24. (b) hn f o
25. (d) For occurence of photoelectric effect, the incident light - = Vo
e e
should have frequency more than a certain minimum The slope is
which is called the threshold frequency (v0). q n
h
1 tan θ = = constant
We have, mv 2 = h v - hv0 e
2 \ The ratio will be 1.
For photoelectric effect emission n > n0
41. (d) hu – hu0 = EK, according to photoelectric equation,
where n is the frequency of the incident light.
when u = u0, EK = 0.
26. (b) E = hc/l = 6.6 × 10–34 × 3 × 108/5000 × 10–10
Graph (d) represents EK – u relationship.
= 3.973 × 10–19 J
27. (a) Emission of electron from a substance under the action 42. (a) Einstein equation E = huo + K.E
of light is photoelectric effect. Light must be at a where E = energy of incident photon.
sufficiently high frequency. It may be visible light, U.V, huo = work function of metal
X-rays. So U.V. cause electron emission.
K.E = max. kinetic energy of e–
28. (c) A certain minimum amount of energy is required to
pull the electron out from the surface of the metal. \ 4 eV = 2 eV + K.E or K.E = 2 eV
This minimum energy required by an electron to Stopping potential is the potential difference which
escape from the metal surface is called the work may stop this e–.
function of the metal. Let it be V, then eV = 2e Þ V = 2 volt.
29. (c) Beyond the threshold frequency the photoelectric hc
current increases with increase in intensity. 43. (b) Since work function for a metal surface is W =
30. (d) For the given frequency of incident light l0
where l0 is threshold wavelength or cut-off wavelength
Kmax = eV0
for a metal surface.
where Kmax is the maximum kinetic energy gained by
photoelectron and V0 is the minimum retarding here W = 4.125 eV = 4.125 × 1.6 × 10–19 Joule

6.6 ´ 10 -34 ´ 3 ´ 108


potential for which the photocurrent stops.
e = charge of electron. so l0 = = 3000Å
31. (b) Max. K.E. = hn – W0 ; so Max. K.E. µ n 4.125 ´ 1.6 ´ 10 -19
32. (a) 44. (c) By using E = W0 + Kmax
33. (c) Max. K.E. of phtoelectrons emitted is independent of
intensity of incident light. 12375
E= =2.475 eV and Kmax = eV0 = 1.36 eV
34. (b) The photoelectric current µ Intensity of light. 5000
So 2.475 = W0 + 1.36 Þ W0 = 1.1 eV.
E 10 ´ 1.6 ´ 10 -19
35. (d) Momentum of a photon µ = 45. (b) For electron and positron pair production, minimum
c 3 ´ 10 8 energy is 1.02 MeV.
= 5.33 ´10 - 27 kg ms -1
EBD_7418
686 PHYSICS

hc
-3 1.7 ´ 10-13 KB = - W0 ....(ii)
Energy of photon is given 1.7 ´ 10 J = lB
1.6 ´ 10-19 From eq. (i) and eq. (ii) we have,
= 1.06 MeV.
Since energy of photon is greater than 1.02 MeV. KB
so electron positron pair will be created. KA <
46. (c) In the given relation Ek stands for maximum K.E. of 2
emitted photoelectron. 58. (d) W0 = hn1 – eV1
47. (b) According to Einsten’s photoelectric effect, the K.E. = h n2 – e V2
of the radiated electrons
K.Emax = E – W eV2 = h(n2 – n1) + eV1
1 h ( n 2 - n1 )
mv 2 = (1 – 0.5) eV = 0.5 eV V2 = + V1
2 1 e
1 59. (c) hu = Wex + maximum kinetic energy
mv 2 = (2.5 – 0.5) eV = 2 eV hu0 = 8 eV = Wex + 2 eV Þ Wex = 6 eV
2 2
For incoming radiation, energy is
v1 0.5 1 1
= = = h × 1.25 u0 = 10 eV, Wex = 6 eV
v2 2 4 2 \ Kinetic energy (maximum) = 4 eV.
48. (c) The stopping potential is equal to maximum kinetic
energy. 60. (d) Intensity µ 1/ (distance)2 ; No. of photoelectrons
49. (a) K.E. = hn – hnth = eV0 (V0 = cut off voltage) emitted is proportional to intensity of incident light.
h 61. (a) The work function has no effect on photoelectric current
Þ V0 = (8.2 ´ 1014 - 3.3 ´ 1014 ) so long as hn > W0. The photoelectric current is
e
proportional to the intensity of incident light. Since there
6.6 ´ 10 -34 ´ 4.9 ´ 1014 is no change in the intensity of light, hence I1 = I2.
= » 2V.
1.6 ´ 10 -19 1
50. (d) From Eqn K.E = hn - f 62. (d) Iµ
slope of graph of K.E & n is h, d2
which is same for all metals. 63. (b) Photoelectrons are emitted in A alone. Energy of
51. (b) 52. (c) hu
53. (b) For emission of electrons incident energy of each electron needed if emitted from A = eV
e
photon must be greater than work function (threshold

54. (b)
energy).
\ EA =
( 6.6 ´10-34 ) ´ (1.8 ´ 1014 )
= 0.74 eV
55. (a) For a given photosensitive material and µ frequency 1.6 ´10-19
> threshold frequency photoelectric current µ
intensity.
hc EB =
( 6.6 ´10-34 ) ´ ( 2.2 ´1014 )
= 0.91 eV
56. (c) Work function, W0 = 1.6 ´10-19
l0
In electron volt, Incident energy 0.825 eV is greater than EA (0.74 eV)
hc 12375 but less than EB (0.91 eV).
W0 (in eV) = =
el 0 l 0 (Å) hc hc
64. (d) eV0 = - W0 and eV ¢ = - W0
12375 l0 2 l0
\l0 (Å) =
W0 (eV) Subtracting them we have
12375 hc é 1 ù hc hc
Þ For metal A, l0 = = 3867Å e ( V0 - V¢) = ê1- ú = or V¢ = V0 -
3.2 l0 ë 2 û 2 l0 2 e l0
12375 65. (a) From the graph it is clear that A and B have the same
For metal B, l 0 = = 6513Å
1.9 stopping potential and therefore the same frequency.
Q Incident wavelength (l) Also B and C have the same intensity.
< threshold wavelength l 0
hc æ 1 1 ö
for both the metals, hence photoelectrons will emit for 66. (a) Ek = - (in eV)
both. c çè l l 0 ÷ø
hc hc
57. (a) KE = - W0 Þ K A = l - W0 6.6 ´ 10-34 ´ 3 ´ 108 æ 1010 1010 ö
l A = ç 1800 - 2300 ÷ = 0.15eV
hc 1.6 ´ 10-19 è ø
KA = - W0 ....(i)
2l B 67. (b) From photoelectric equation,
(Q lA = 2 lB) hn' = hn + Kmax ...(i)
DUAL NATURE OF RADIATION AND MATTER 687

1 79. (d) The electron ejected with maximum speed vmax are
h.2n = hn + mV2max [\ n ¢ = 2 n] stopped by electric field E =4N/C after travelling a
2 distance d =1m
1 1
Þ hn = mV2max mv 2max = eEd = 4eV
2 2
2hn 1240
Þ Vmax = The energy of incident photon = = 6.2 eV
m 200
hc From equation of photo electric effect
68. (d) eVs =- W0 . If l decreases, V increases
l s
1
69. 2
(d) Intensity µ 1/ (distance) ; No. of photoelectrons mv 2max = hn - f0
2
emitted is proportional to intensity of incident light.
\ f0 = 6.2 - 4 = 2.2 eV
70. (b) E = W0 + eV0
1 hc 2( hc - lf )
For hydrogen atom, E = +13.6 eV 80. (c) mv 2 = -f Þ v =
\ + 13.6 = 4.2 + eV0 2 l lm
hc hc 12375
(13.6 - 4.2)eV 81. (d) Kmax = -W = - 5.01 = - 5.01
Þ V0 = = 9.4V l l l (in Å)
e
Potential at anode = – 9.4 V 12375
71. (d) The order of time is nano second. = –5.01 = 6.1875 – 5.01 = 1.17775 ; 1.2 V
2000
72. (a) In the Davisson and Germer experiment, the velocity
of electrons emitted from the electron gun can be nl 1´ 20
82. (b) 2 d sin q = nl or d = = = 20 nm
increased by increasing the potential difference 2 sin q 2 ´ sin 30º
between the anode and filament. 2d sin q 2 ´15 ´ sin 90º
83. (c) l max . = = = 30Å
1 n min . 1
73. (b) m n12 = 2 W0 - W0 = W0 and
2 I
I0 2.303 log10 0
1 I
m n 22 = 10 W0 - W0 = 9W0 84. (c) = 4, x = 7mm (given) Þ m =
2 I x
2.303 log10 4 2.303 ´ 0.6023
n1 W0 1 m= = = 0.198 mm–1
\ = = 7 7
n2 9 W0 3 85. (c) The energy of incident photons is given by
12400 12400 hn = eVs + f0 = 2 + 5 = 7eV
74. (a) V= Volt Þ V = = 6.2 KV
l min (Å) 2
(Vs is stopping potential and f0 is work function)
l m2 V1 6.22 ´ 10 3 Saturation current = 10–5 A
75. (a) = Þ l m2 = = 0.622 Å
l m1 V2 104 hP 10 -5 P
= e= e ( h is photon emission efficiency)
hn 7´e
hc 6.6 ´ 10 -34 ´ 3 ´ 108 \ P = 7W
76. (d) Energy = = J
l 0.5 ´ 10 -10 86. (b) DE. Dt ~ h
6.6 ´ 10 -34 ´ 3 ´ 108 hc
= eV = 24.82 keV E=
5 ´ 10 -11 ´ 1.6 ´ 10 -19 l
q ne
77. (a) I = = hc Dl
t t DE =
No. of electrons striking the target per second l l
I hc Dl
= 2 ´ 1016
= \ .Dt ~ h
e l l
78. (d) To study effect of intensity, the collector A is Now, c = vl
maintained at a positive potential with respect to
nDl + lDn = 0
emitter C so that electrons ejected from C are
attracted towards collector A. Keeping the frequency nDl = -lDn
of the incident radiation and the accelerating potential
fixed, the intensity of light is varied and the resulting Dl Dn
\ =–
phtoelectric current is measured each time. It is l n
found that the photocurrrent increases linearly with
ch Dn
intensity of incident light. \ Dt ~ h
l n
EBD_7418
688 PHYSICS

l 2 v1 3 v1
Dn h l l 600 ´ 10 –9 =
\ ~ . ~ = l1 v2 Þ 4 = v
n Dt hc Dtc 8 ´ 10 –9 ´ 3 ´ 108 2
v2 4 æ
= v2 - v1ö 100
Dn
~ 10 –6 v1 3 Þ çè v ÷ø ´ 100 = 3
1
n
87. (a) Give that, only 25% of 200W converter electrical energy 100
Hence, P.D. must be increased by %.
into light of yellow colour 3
æ hc ö 25 96 (a)
çè ÷ø ´ N = 200 ´
l 100 97. (b) According to Bragg’s law, Þ 2d sin q = nl,
Where N is the No. of photons emitted per second, n = 1 for first order
h = plank’s constant, c, speed of light.
Þ 2 × 2.82 sin 15.8 = l Þ l = 5.64 × 0.2723 = 1.53 Å
200 ´ 25 l
N= ´ l m2 V1 6.22 ´ 103
100 hc 98. (a) = Þ l m2 = = 0.622 Å
200 ´ 25 ´ 0.6 ´ 10 -6 l m1 V2 104
=
100 ´ 6.2 ´ 10-34 ´ 3 ´ 108 ch æ eV ö
= 1.5 × 1020 99. (d) lmin = \ h= çè ÷ø lmin
eV c
88. (a) Number of emitted electrons
1.6 ´ 10-19 ´ 30000 ´ 0.141 ´ 10-10
1012 ´ 2 ´ 10-4 ´ 25 h=
= = 5 × 104 electrons 3 ´ 108
105
Þ h = 6.624 × 10–34 J-sec
q = + ne = 8 × 10–15 C
89. (b) hv = f + Kmax hc
100. (a) Energy emitted/sec by S1 , P1 = n1
hv = 2 + 4 = 6 eV l1
æ5 ö hc
Now, h ç v÷ = 2 + K max Energy emitted/sec by S2 , P2 = n2
è3 ø l2
5 P2 n2 l1
\ = ×
Kmax = ´ 6 - 2 Þ Kmax = 8 eV.. P1 n1 l 2
3
90. (b)
91. (d) 1.02 ´ 1015 5000
= × = 1.0
92. (a) Give that, only 25% of 200W converter electrical energy 1015 5100
into light of yellow colour
æ hc ö 25 Exercise - 2
çè ÷ø ´ N = 200 ´
l 100
1. (d) Velocity of a body freely falling from a height H is
Where N is the No. of photons emitted per second,
h = plank’s constant, c, speed of light. v = 2gH
200 ´ 25 l h h h
N=
100
´
hc So, l = mv = m 2gH Þ = m 2g H

200 ´ 25 ´ 0.6 ´ 10 -6 (h, m and g are constant)


= -34 8 = 1.5 × 1020
100 ´ 6.2 ´ 10 ´ 3 ´ 10 h
93. (a) Here, is also constant
m 2g
94. (d) Number of emitted electrons NE µ Intensity
1
µ
1
So, h µ Þ or l µ H -1/2
(Distance)2 H
Therefore, as distance is doubled, NE decreases by (1/ hc
2. (b) Energy of a photon is E =
4) times. l
hc Where l is the minimum wavelength of the photon
95. (a) lmin =
ev0 required to eject the proton from nucleus.
hc hc Energy of photon must be equal to the binding energy
l1 = ; l2 =
ev1 ev2 of proton.
So, energy of a photon, E = 1 MeV Þ 106eV
(given)
DUAL NATURE OF RADIATION AND MATTER 689

æ hc ö
Now, çè E = ÷ø F eE 0ˆi
l
a= = (Q F = ma )
m m
Velocity of electron after time t, is v = (v0 + at)
hc æ 6.63 ´10-34 ´ 3 ´ 108 ö
So, l = =ç ÷ = 1.24 × 10–3 nm æ eE ˆi ö æ eE ö
E è 106 eV ø
v = v0 ˆi + ç 0 ÷ t = ç v 0 + 0 t ÷ ˆi
3. (d) When a beam of electrons of energy E0 is incident on è m ø è m ø
a metal surface kept in vacuum or evacuated chamber æ eE ö
so electrons can be emitted with maximum energy E0 v = v0 ç 1 + 0 t ÷ ˆi
è m ø
(due to elastic collision) and with any energy less than
Now for new de-Broglie wavelength associated with
E0, when part of incident energy of electron is used in
electron at time t is
liberating the electrons from the surface of metal. So
maximum energy of emitted electrons can be E0. h
l=
4. (c) mv
5. (b) The relation between l and K is given by
h l0
Þ l= =
h éæ eE0 t ö ˆù é eE 0 ù
l= v 0m êç 1 +
2mK ÷ i ú ê1 + tú
ëè mv 0 ø û ë mv 0 û
So, for the given value of kinetic energy K,
é h ù
[ ]
êQ l 0 = mv ú î = 1
h
is a constant. ë 0û
2K
1 l0
l=
Thus, l µ é æ eE 0 ö ù
m ê1 + ç mv ÷ t ú
ë è 0ø û
\ Þ l p : l n : l e : la
8. (c)
1 1 1 1 9. (b) According to Einstein’s photoelectric equation,
Þ = : : :
mp mn me ma hv = f0 + Kmax
We have hv = f0 + 0.5 ...(i)
if (mp = mn), then lp = ln and 1.2hv = f0 + 0.8 ...(ii)
if (ma > mp), then la < lp Therefore, from above two equations f0 = 1.0 eV.
if (me < mn), then le > ln 10. (a) Conservation of energy requires that the 15.0 eV
photon energy first provides the ionization energy to
Hence, la < lp = ln < le.
r r unbind the electron, and then allows any excess energy
6. (a) As given that, v = v0 i and B = B0 j to become the electron’s kinetic energy. The kinetic
Force on moving electron due to perpendicular energy in this case is 15.0 eV – 13.6 eV = 1.4 eV.
magnetic field B is, F = –e (v × B) 11. (b) Given : work function f of metal = 2.28 eV
Wavelength of light l = 500 nm = 500 × 10–9m
F = - e éë v 0 ˆi ´ B0 ˆi ùû = –e v0 B0 ( ˆi ´ ˆj )
hc 6.6 ´ 10-34 ´ 3 ´ 108
Þ = -ev 0 B0kˆ (Q kˆ = ˆi ´ ˆj) KEmax =
l
–f
5 ´ 10-7
– 2.82

So, the force is perpendicular to v and B, both as the KEmax = 2.48 – 2.28 = 0.2 eV
force is ^ to the velocity so the magnitude of v will h h
lmin = =
not change, so momentum is (= mv) will remain same p 2m ( KE )max
or constant in magnitude. Hence,
20
h ´ 10-34
de-Broglie wavelength l = remians constant. = 3 = 2.80 × 10–9 nm
mv -31 -19
7. (a) de-Brogile wavelength of electron, 2 ´ 9 ´10 ´ 0.2 ´1.6 ´10
\ l ³ 2.8 × 10–9 m
h 12. (d) Photoelectric equations
l0 = ...(i)
mv0
hc hc
Force on electron Ek1max = -f ...(i) and Ek2max = -f
l l /2
Þ F = -eE = ( - e ) ( - E 0 ˆi ) = eE 0 ˆi 2hc
Acceleration of electron EK2max = -f ...(ii)
λ
From question, Ek2max = 3Ek1max
EBD_7418
690 PHYSICS

( ) ( 2 ´10-2 ) ( 23 ´10-3 )
Multiplying equation (i) by 3 -19 2 2 2
1 1.6 ´ 10
æ hc ö
3Ek1max = 3 ç - f ÷
è l ø
...(iii) = 2
(9.1´10-31 )
From equation (ii) and (iii)
2.97 ´ 10 -15
3hc 2hc hc = 2.97 × 10–15 J = = 18.36 keV
- 3f = - f \ f (work function) = 1.6 ´ 10-19
l l 2l
13. (d) Photoelectric saturation current is independent of hc
Energy of the incident photon =
frequency. It only depends on intensity of light. l
14. (a) Both statement I and II are true; but even it radiation of 12.4
= = 24.8 keV
single wavelength is incident on photosensitive 0.50
surface, electrons of different KE will be emitted. Therefore, Binding energy = 24.8 – 18.6 = 6.2 keV
hc 17. (a) For electron De-Broglie wavelength,
15. (b) As we know, threshold wavelength ( l0 ) =
f h
le = ; For photon E = pc
Þ l0 =
( 6.63´10 ) ´ 3 ´10
-34 8
= 5.404 ´ 10 -7 m .
2mE
hc
2.3 ´ (1.6 ´10 )-19 Þ De-Broglie wavelength, l Ph =
E
1/2
Þ l0 = 5404 Å le h E æ E ö 1
\ = ´ =ç ÷
Hence, wavelength 4144 Å and 4972 Å will emit electron l Ph 2mE hc è 2m ø c
from the metal surface. 18. (d) According to Einstein's photoelectric effect,
For each wavelength energy incident on the surface hc hc
eV = - …(i)
per unit time l l0
= intensity of each × area of the surface wavelength hc hc
eV/4 = 2l - l …(ii)
0
3.6 ´10-3 Dividing equation (i) by (ii) by
´ (1cm ) = 1.2 ´ 10 -7 joule
2
=
3 1 1
-
Therefore, energy incident on the surface for each l l0
Þ 4= on solving we get, l0 = 3l
wavelength in 2s 1 1
-
E = (1.2 × 10–7) × 2 = 2.4 × 10–7 J 2l l 0
Number of photons n 1 due to wavelength 4144 Å 1 2 4 1 2
19. (c) hn02 – hn0 = mv \ hn 0 - hn 0 = mv '
2 3 2
n1 =
( 2.4 ´10 )( 4144 ´10 ) = 0.5 ´10
-7 -10
12 4 4
n - n0 n - n0
( 6.63 ´10 )( 3 ´10 )
-34 8
\
v '2 3
= \ v' = v 3
n - n0
v2 n - n0
Number of photons n 2 due to the wavelength 4972 Å
4

n2 =
( 2.4 ´10-7 )( 4972 ´10 -10 )
= 0.572 ´1012
\ v' > v
3
( 6.63 ´ 10 -34
)( )
3 ´ 108 20. (a) Pin = 25W, l = 6600 Å = 6600 × 10–10 m
nhv = P
Therefore total number of photoelectrons liberated in Þ Number of photons emitted/sec,
2s, P Pl 25 ´ 6600 ´ 10 -10
n= = =
hc hc 6.64 ´ 10-34 ´ 3 ´ 108
N = n1 + n 2
l 25
= 0.5 ´1012 + 0.575 ´1012 = 8.28 × 1019 = ´ 1019
3
= 1.075 ´ 1012 3% of emitted photons are producing current
3 3 25
16. (b) As we know, \ I= ´ ne = ´ × 1019 × 1.6 × 10–19 = 0.4A
100 100 3
v2 q 21. (a, d) Given; l0 = 3250 × 10–10 m
F = qvB = m Þ v = BR
R m l = 2536 × 10–10 m
The kinetic energy of the photoelectron
1 1 e 2 B2 R 2 hc 4.14 ´ 10-15 ´ 3 ´108
= mv2 = f= = = 3.82eV
2 2 m l0 3250 ´ 10-10
DUAL NATURE OF RADIATION AND MATTER 691

hc 4.14 ´10-15 ´ 3 ´108 Dividing eqn. (i) by (ii),


hv = l = = 4.89eV
1 v12
2536 ´ 10-10 =
v1 1
=
According to Einstein's photoelectric equation, 4 v22 \
v2 2
Kmax = hv – f
KEmax = (4.89–3.82)eV=1.077 eV 28. (c)
1 29. (c) hu = Wex + maximum kinetic energy
mv 2 = 1.077 × 1.6 × 10–19 hu0 = 8 eV = Wex + 2 eV Þ Wex = 6 eV
2
For incoming radiation, energy is
2 ´1.077 ´1.6 ´10-19
Þv= h × 1.25 u0 = 10 eV, Wex = 6 eV
9.1´10-31 \ Kinetic energy (maximum) = 4 eV.
or, v = 0.6 × 106 m/s or 6 × 105 m/s
22. (a) We know that, 30. (d) V = 3000 volt.
h h 1 2eV
de-Broglie wavelength l = = mv2 = eV Þ v =
P 2m(KE) 2 m
3 2 ´1.6 ´ 10-19 ´ 3000
K.E. of thermal neutron = kT v=
2 \
9.1 ´ 10 -31
h h
= = = 32.6 × 106 = 3.26 × 107 m/s.
æ3 ö 3mkT
2m ç kT ÷ Exercise - 3
è2 ø
23. (d) As l is increased, there will be a value of l above 1. (c) We know that
which photoelectrons will be cease to come out so 1 2
photocurrent will become zero. Hence (d) is correct h n - f = K max = mvmax
2
answer. According to question
24. (d) 5hn0 - hn0 v22
hc =
25. (c) In X-ray tube, l min = 2hn0 - hn0 v12
eV
æ hc ö v2 = 2v1 = 2 × 4× 106 = 8 × 106 m/s.
In l min = In ç ÷ - InV 2. (a) For one photocathode
è eø
Clearly, log lmin versus log V graph 1 2
hf1 - W = mv ....(i)
slope is negative hence option (c) correctly depicts. 2 1
For another photo cathode
26. (a) We have, E = W0 + K
1 2
hf 2 - W = mv2 ....(ii)
hc 2 1 2
or -9 = W0 + mv … (i) Subtracting (ii) from (i) we get
400 ´ 10 2
1 2 1 2
and
hc
250 ´ 10 -9
12
= W0 + m(2v ) … (ii)
2
( hf1 - W ) – ( hf 2 - W ) = mv - mv
2 1 2 2
On simplifying above equations, we get m 2 2
\ h( f1 - f 2 ) = (v 1 - v 2 )
W0 = 2hc ´ 106 J . 2
27. (a) Using Einstein’s photoelectric equation, 2 2 2h
\ v1 - v 2 = (f - f )
E = W0 + kmax m 1 2
When light of frequency, 2n0 is incident on a metal hC 1
plate, 3. (a) - W = mu12
l1 2
1 2
h(2n0) = hn0 + mv1
2 hC 1
and - W = mu22
1 2 l2 2
hn0 = mv1 ...(i)
2 Dividing the above two equations, we get
when light of frequency, 5n0 is incident on a metal
plate hC
-W 1240
1 2 l1 u2 -W
h(5n0) = hn0 + mv2 = 1 248 4
2 hC 2 \ =
- W u2 1240 1
1 2 l2 -W
4hn0 = mv2 ...(ii) 310
2
EBD_7418
692 PHYSICS

10. (b) Momentum imparted per unit time = np


1240 4 ´ 1240
\ -W = - 4W \ W = 3.7 eV nh nh
248 310 Þ F= \ Acceleration =
4. (a) From Bragg’s equation l ml
21A
d sin q = l 11. (d) = F and (Keq) paralle = 3 K
c
l
sin q = <1 2F 4IA
d DX = =
3K 3Kc
\ l<d 12. (b) lph = le
h é h ù h h E ph 2E e Ee v
<d = =
| py | êQ l = ú pph pe Þ = Þ Eph 2C
êë | p y | úû C v
\ h < | py | d hc
5. (c) From Einstein's photoelectric equation, we have 13. (a) E=
l
hc hc Number of photons emitted is
< ∗ eV ...(1)
l1 l0
Pt P lt
= n0 Þ n0 =
hc hc æ hc ö hc
< ∗ eV ...(2) çè ÷ø
l2 l0 l

hc hc
Since the radiation is sphericall symmetric, so total
< ∗ 3eV ' ...(3) number of photons entring the sensor is n 0 times the
l3 l0 ratio of aperture area to the area of a sphere of radius l.
2
From equation (1) & (2) p ( 2d ) Plt d 2
N = n0 =
3
,
2
<
1 4pl2 hc l 2
2l1 2l2 l0 1 hc
14. (d) mv 2 = -f
2 l
hc é 3 1 ùú
, hc ê , < eV ' 1
mv¢2 =
hc
-f =
4hc
-f
l1 ê 2l 2l ú
ë 1 2û 2 (5l / 4) 5l

hc éê 1 3 1 ùú Clearly, v¢ >
5
v
, ∗ < V'
e ë l3 2l1 2l2 úû
ê 3
15. (c) Photons have momentum (P = h/l) which they carry
6. (b) 7. (a) away; the spacecraft will acquire momentum in the
opposite direction according to law of conservation
1
8. (b) E - W0 = mv2 = eVs of momentum.
2 No. of photons per second from laser = n
Then, from energy considerations,
hc
or - W0 = eVs æ cö
l 0.5 ´ 10 -3 = nh ç ÷ Þ n = (0.5 × 10–3) l/ (ch)
è lø
hc Rate of change of momentum of spacecraft
Hence, - W0 = e ( 0.5) ....(i)
0.6 ´ 10 -6 h ( l æ h ö 0.5 ´ 10 -3
= np = n = 0.5 ´ 10 -3 ) ç ÷ =
l ch è l ø c
hc
and - W0 = e (1.5) ....(ii) nh
0.4 ´ 10-6 From Newton's second law, = ma
l
Solving, we get W0 = 1.5 eV
0.4 ´ 10-3 1
h 1000a = 8
= ´ 10-11
l= 3.00 ´ 10 6
9. (c)
2mqV v = at
mV = constant v 1000
t= = s = 6 ´ 1017 s
1837 V' = 1 × V a æ 1 ö 1 -11
çè ÷ ´ ´ 10
V 1000 ø 6
or V' = volt
1837
ATOMS 693

27. ATOMS

Exercise - 1 Vacuum Gold foil target


–8
about 10 m thick
Lead brick
1. (a) According to classical theory, the path on an electron
in Rutherford atomic model is spiral. Most pass
Beam of through
2. (b) 3. (b) a-particles
4. (b) The electric potentical energy of the atom increases in Source of
higher values of n. a-particles
5. (a) The significant result deduced from the Rutherford's ZnS Screen
Some are
scattering is that whole of the positive charge is deviated through
About 1 in 8000 a large angle q
concentrated at the centre of atom i.e. nucleus. is reflected back Detector
6. (c) At closest distance of approach, the kinetic energy of (microscope)
Schematic arrangement of the Geiger-Marsden experiment
the particle will convert completely into electrostatic
potential energy. Only about 0.14% of the incident a-particles scatter
1 2 by more than 1° and about 1 in 8000 deflected by
Kinetic energy K.E. = mv
2 more than 90°.32.
KQq
Potential energy P.E. = R1 n12 1 n 1
r 16. (d) = = \ 1 =
1 KQq 1 R2 n2 4 2 n2 2
mv 2 = Þ rµ 2 3 3
2 r v T1 æ n1 ö æ 1 ö 1
7. (d) When b = 0, scattering angle, q = 180º = çç ÷÷ = ç ÷ =
T2 è n 2 ø è 2 ø 8
8. (d) The orbital angular momentum of electron is 2 K ex P
17. (b) Ep =
independent of mass of orbiting particle & mass of 32
æ 2 b2 ö
nuclei. çx + 4 ÷ x
Ze(2e) è ø
9. (a) Distance of closest approach r0 = For maximum EP A B
æ 1 2ö +e b/2 0 b/2 +e
4pe 0 ç mv ÷
è2 ø dEP b
6 - 19 =0 Þ x=
Energy, E = 5 ´ 10 ´ 1.6 ´ 10 J dx 2 2
9 -19 -19
\ r0 = 9 ´ 10 ´ (92 ´ 1.6 ´ 10 ) (2 ´ 1.6 ´ 10 ) 18. (b) 19. (b) 20. (d) 21. (c)
5 ´ 106 ´ 1.6 ´ 10-19 2
22. (a) rn= ro.n , where ro is radius of ground-state & rn is
-14
Þ r = 5.2 ´ 10 m = 5.3 × 10 –12 cm. radius of nth state. (For first excited state n = 2).
10. (a) 23. (b) Jump to second orbit leads to Balmer series. When an
11. (a) T = 2 Ze2/4pe0 D electron Jumps from 4th orbit to 2nd orbit shall give rise
1 to second line of Balmer series.

D 24. (a) As r µ n 2 , therefore, radius of 2nd Bohr’s orbit = 4 r0
12. (a) The kinetic energy of the projectile is given by
ke2 n 2-h 2 1 v me4
1 Ze (2e) Z1 Z2 25. (b) v= ,r = ,n = = ,E =
mv 2 = = nh- mke2 T 2 pr 8e o n 2 h 2
2 4πε 0 r0 4πε 0 r0
Thus energy of the projectile is directly proportional 26. (a) Angular momentum = mrv = J
to Z1, Z2 J
\ v=
13. (d) a-particle cannot be attracted by the nucleus. mr
14. (c) According to postulates of Bohr's atom model the 1 1 æ J ö
2 J2
electron revolves around the nucleus in fixed orbit of K. E. of electron = mv2 = m ç ÷ =
2 2 è mr ø 2mr 2
definite radii. As long as the electron is in a certain
q æ 1 ö
orbit it does not radiate any energy. 27. (d) I= = qn = en çèQ = n & q = e÷ø
15. (d) Many of the a-particles pass through the foil. It means t t
that they do not suffer any collisions. 28. (b)
29. (a) Since 121.5 nm line of spectrum of hydrogen atom lies
in ultraviolet region, therefore it is Lyman series.
EBD_7418
694 PHYSICS

n(n - 1) 3(3 - 1) 47. (a) From conservation of momentum :


30. (b) No. of lines N E = = =3
2 2 h æ 1ö 3 hR
MV = = hR ç 1 - ÷ Þ V =
31. (b) l è 4 ø 4 M
32. (b) Transition from higher states to n = 2 lead to emission 48. (a) Velocity of electron in nth orbit of hydrogen atom is
of radiation with wavelengths 656.3 nm and 365.0 nm. given by :
These wavelengths fall in the visible region and
2pKZe2
constitute the Balmer series. Vn =
nh
33. (b) 34. (b) 35. (a) 36. (a) Substituting the values we get,
v c c
37. (c) vn = 1 \ v2 = = 2.2 ´ 106 Vn µ
1
n 137 ´ 2 274 Vn = m/s or
n n
38. (a) 39. (d) As principal quantum number increases, velocity
40. (d) l IR > l UV also wavelength of emitted radiation decreases.
49. (b)
1 50. (d)
lµ .
DE 51. (d) The shortest wavelength occurs when an electron
makes a transitions from n = ¥ to n = 2 state.
41. (d) Since out of the given four lines Hd line has smallest
wavelength. Hence the frequency of this line will be 1 æ 1 1ö R
\ = Rç – ÷=
maximum. l min è 22 ¥ ø 4
1 1
42. (d) As r µ \ r0¢ = r0 52. (b) When the atom gets appropriate energy from outside,
m 2
' then this electron rises to some higher energy level.
As E µ m \ E 0 = 2(–13.6) = –27.2 eV Now it can return either directly to the lower energy
43. (c) rn = r1 n2 level or come to the lowest energy level after passing
through other lower energy levels hence all possible
2 rn 21.2 ´ 10-11
\n = = =4 transitions take place in the source and many lines are
r1 5.3 ´ 10 -11 seen in the spectrum.
\n=2 53. (b) 54. (a)
m0 I e 55. (a) Energy of a H-like atom in it's nth state is given by
44. (d) Q B = and I =
2r T 2 13.6
m 0e En = - Z ´ 2 eV
B= [ r µ n 2 , T µ n5 ] ; 1 n
2rT Bµ For, first excited state of He+, n = 2, Z = 2
n5
45. (d) E = E4 – E3 4
\ EHe + = - 2 ´13.6 = -13.6 eV
2
13.6 æ 13.6 ö 56. (b) Obviously, difference of 11.1eV is not possible.
=- -ç- ÷ = -0.85 + 1.51 = 0.66 eV
4 2 è 32 ø –0.58eV
46. (b) The smallest frequency and largest wavelength in –0.85eV
ultraviolet region will be for transition of electron from –1.51eV 12.09eV n=2
orbit 2 to orbit 1. –3.4eV 10.2eV
–13.6eV n=1
1 æ 1 1ö
\ = Rç 2 – 2 ÷ 13.6 13.6
l è n1 n2 ø 57. (c) En = – Þ E2 = – = –3.4 eV..
2
n 22
1 é1 1ù é 1 ù 3R 58. (b) Energy of excitation,
Þ –9
= R ê 2 – 2 ú = R ê1 – ú =
122 ´ 10 m ë1 2 û ë 4û 4
æ 1 1ö
4 D E = 13.6 p2 çè h - h ÷ø eV
Þ R= m –1 1 2
3 ´ 122 ´ 10–9 æ1 1ö
Þ DE = 13.6 (3)2 çè - ÷ = 108.8 eV
The highest frequency and smallest wavelength for 12
32 ø
infrared region will be for transition of electron from ¥ 59. (c) Ist excited state corresponds to n = 2
to 3rd orbit. 2nd excited state corresponds to n = 3
1 æ 1 1ö 1 4 æ 1 1ö E1 n 22 32 9
\ = Rç 2 – 2 ÷ Þ = – = = =
l è n1 n2 ø l 3 ´ 122 ´ 10 –9 çè 32 ¥ ÷ø \
E 2 n12 2 2 4
3 ´ 122 ´ 9 ´ 10 –9 é1 1 ù
\ l= = 823.5nm 60. (a) Excitation energy DE = E2 –E1 = 13.6 Z2 ê 2 - 2 ú
4 ë1 2 û
3 2
Þ 40.8 = 13.6 ´ ´ Z Þ Z = 2.
4
ATOMS 695

Now required energy to remove the electron from and n1 = 2


+13.6Z 2 1 æ 1 1 ö 5R
ground state = = 13.6(Z) 2 = 54.4 eV. \ = R(1) 2 ç - ÷ = ...(i)
(1) 2 6561 è 4 9 ø 36
For the second spectral line in the Balmer series of
61. (a) Electron after absorbing 10.2 eV energy goes to its
singly ionised helium ion n2 = 4 and n1 = 2 ; Z = 2
first excited state (n = 2) from ground state (n = 1).
h 1 2 é1 1 ù 3R
\ Increase in momentum = \ = R (2) ê - ú = ...(ii)
2p l ë 4 16 û 4
6.6 ´ 10-34 Dividing equation (i) and equation (ii) we get
-34
= = 1.05 ´ 10 J-s. l 5R 4 5
6.28 = ´ =
62. (d) For ground state, the principal quantum n o 6561 36 3 R 27
(n) = 1. There is a 3rd excited state for principal \ l = 1215 Å
quantum number. 68. (c) For last line Balmer series, n1 = 2 and n2 = ¥
1 é 1 1 ù 7
4 3rd excited state = Rê - ú = 1.1 ´10 m –1
Pincipal quantum no. (n)

l 2 2
êë n 1 n 2 úû 4
6 –1
Energy states

= 2.75 × 10 m
3 2nd excited state
69. (c) For Balmer series, n1 = 2, n2 = 3 for 1st line and n2
= 4 for second line.
2 1st excited state
æ 1 1 ö
-
1 l1 çè 22 42 ÷ø
ground state = 3 /16 3 36 27
Possible number of spectral lines l2 æ 1 1 ö = = ´ =
ç 2 - ÷ 5 / 36 16 5 20
The possible number of the spectral lines is given è2 32 ø
n(n - 1) 4(4 - 1) 20 20
= = =6 l2 = l1 = ´ 6561 = 4860 Å
2 2 27 27
1 é 1 1 ù E1
63. (b) In Paschen series l = Rê 2 - 2 ú 70. (a) E n = 2
max ë (3) (4) û n
2 E -13.6
Þ l max =
144
=
144
= 1.89 ´ 10-6 m = 1.89 mm \n = 1 = = 16
7R 7 ´ 1.1 ´ 107 E n -0.85
9 9 1 é 1 1 ù 4
Similarly l min = = = 0.818 mm 71. (a) = R ê 2 - 2 ú Þ l max = » 1213Å
R 1.1 ´ 107 l max ë (1) (2) û 3R
1 é 1 1 ù 5R 1 é1 1 ù 3R 1 é 1 1ù 1
64. (a) =Rê - ú= , =Rê - ú= = R ê 2 - ú Þ l min = » 910Å.
2
l Balmer ë 2 3 û 36 lLyman
2 2 ë1 22 û 4 and l ¥û R
min ë (1)
5 72. (c) For longest wavelength of Balmer series,
\ l Lyman = l Balmer ´ = 1215.4Å
27 p = 2 and n = 3
1 é1 1 ù 1 æ 1 1 ö æ1 1ö
= R ê 2 - 2 ú . For first wavelength, n = 2, n = 3 = Rç - ÷ = R ç - ÷ 5R
65. (d) l l èp
2
n ø2 è 4 9 ø = 36
ë n1 n 2 û 1 2

Þ l1 = 6563 Å. For second wavelength, n1 = 2, n2 = 4 36 36


\ R = 5l = = 1.09 × 107 m–1
Þ l2 = 4861 Å 5 ´ 6.563 ´ 10-7
66. (c) For third line of Balmer series n 1 = 2, n2 = 5 73. (d) : By Bohr’s formula
1 é1 1 ù n2n2 1 é 1 1 ù
\ = RZ2 ê 2 - 2 ú gives Z2 = 2 1 22 = Z 2R ê - ú
l ë n1 n 2 û (n 2 - n1 )lR l êë n 12 n 22 úû
On putting values Z = 2 For first line of Lyman series n1 = 1, n2 = 2
1 3
13.6Z2 -13.6(2) 2 \ = Z2R
From E = - = = -54.4 eV l 4
n2 (1) 2 In the case of hydrogen atom, Z = 1
1 é1 1ù 1 3
67. (a) We know that = RZ 2 ê 2 - 2 ú =R
l ëê n1 n2 ûú l 4
The wave length of first spectral line in the For hydrogen like atom, Z = 11
Balmer series of hydrogen atom is 6561Å . Here n2 = 3
EBD_7418
696 PHYSICS

1 3 l 3R 4 1 78. (d) Radius of nth orbit rn µ n2, graph between r n and n is


= 121R Þ = ´ = 2
l' 4 l ' 4 121R ´ 3 121 rn æ n ö æ rn ö
l 1210 a parabola. Also, r = çè 1 ÷ø Þ log e ç r ÷ = 2log e (n)
= è 1ø
l' = = 10 Å 1
121 121 Comparing this equation with y = mx + c,
n ( n - 1)
Graph between log e æç n ö÷ and loge (n) will be a straight
74. (d) Number of possible emission lines = r
2 è r1 ø
75. (c)
n=3 line, passing from origin.
Case (I)
Similarly it can be proved that graph between
Energy states

æf ö
n=2
log e ç n ÷ and log n is a straight line. But with
Case (II) è f1 ø e

II negative slops.
n=1 c æ 1 1 ö æ1 1 ö
1 79. (c) n= = cR - = cR ç - ÷
l çè p2 n 2 ÷ø è 4 16 ø
The wave number ( n) of the radiation =
l 8 7
3 ´ 10 ´ 10 ´ 12 9
é1 1 ù = = ´ 1015 Hz
= R¥ ê - ú 64 16
2 2 80. (b)
ëê n1 n2 ûú
Now for case (I) n1 = 3, n2 = 2 mv 2
81. (c) qVB = ....(i)
1 é 1 -1ù
r
= R¥ ê , R = Rydberg constant nh
l1 ë 9 4 úû ¥ 2p
= mvr ....(ii)
1 é 4 - 9 ù -5R¥ - 36 qBnh
= m2v 2
= R¥ ê = l1 = Multiplying equation (i) and (ii),
l1 ë 36 úû 36 Þ 5 R¥ 2p
1 n qBh = 1 mv 2
1 é 1 1 ù -3R¥ Now multiplying both sides by ,
= R¥ ê - ú = 2m 4pm 2
l2 ë 4 1û 4
-4
é qBh ù
l2 = l1 -36 3R¥ i.e. KE = n ê ú
3 R¥ Þ l
= ´ ë 4pm û
2 5R¥ -4
1 æ 1 1ö
l1 27 82. (b) l = R ç 2 - 2 ÷
= è n1 n 2 ø
l2 5
76. (a) For emission, the wave number of the radiation is 1 æ 1 1ö æ 1 1 ö 5R
=R çè 2 - 2 ÷ø = R çè 4 - 9 ÷ø = 36
given as l0 2 3
1 æ 1 1ö 1 æ 1 1ö æ 1 1 ö 3R
= Rz 2 ç 2 - 2 ÷ =Rç - =Rç - ÷ =
l è n1 n2 ø l è 2 2 42 ÷ø è 4 16 ø 16
R = Rydberg constant, Z = atomic number l 5 16 20
= ´ =
æ1 1ö æ 1ö 1 24 l 0 36 3 27
= R ç 2 - 2 ÷ = R ç1 - ÷ Þ = R
è1 5 ø è 25 ø l 25 1 3 1
linear momentum
83. (a) R µ n2 ; V µ so VR is µ n ; T µ n Þ E µ 2
n n
h 24 84. (b)
P = = h × R× (de-Broglie hypothesis) 85. (c) Transition A (n = ¥ to 1) : Series lime of Lyman series
l 25
Transition B (n = 5 to n = 2) : Third spectral lien of
24hR 24hR Balmer series
Þ mv = Þ V =
25 25m Transition C (n = 5 to n = 3) : Second spectral line of
77. (d) For an atom following Bohr’s model, the radius is given Paschen series
r0 m 2 86. (c) Centripetal force = force of attraction of nucleus on
by rm = where r0 = Bohr’s radius and m = orbit electron
Z
number.
mv 2 1 e2 e
For Fm, m = 5 (Fifth orbit in which the outermost = v=
electron is present) a0 4 p e o a 20 4p e o m a 0
87. (a) For Lyman series
r0 52 1
\ rm = = nr0 (given) Þ n = 4 é1 1 ù
100 u = RC ê 2 - 2 ú
ë1 n û
ATOMS 697

where n = 2, 3, 4, ....... 31 1
For the series limit of Lyman series, n = ¥ On solving , R =
5 Dl . Z2
é1 1 ù æ 1 1ö
\ u1 = RC ê - ú = RC ...(i) 93. (a) E = hf = 13.6 Z2 ç 2 - 2 ÷
ë1 ¥ 2 û
2
è n 2 n1 ø
For the first line of Lyman series, n = 2
æ1 1ö
u2 = RC éê - ùú = RC 32 ç 2 - 2 ÷
1 1 3
\ ...(ii) f2 è1 2 ø 243
2 2 4
ë1 2 û = f2 = f.
For Balmer series f1 æ1 1ö 32
12 ç 2 - 2 ÷
é1 1 ù è1 3 ø
u = RC ê 2 - 2 ú
ë2 n û
94. (d) Energy of nth state in hydrogen is same as energy of
where n = 3, 4, 5 .....
3nth state in Li++.
For the series limit of Balmer series, n = ¥
\ 3 ® 1 transition in H would give same energy as
é1 1 ù RC the 3 × 3 ® 1 × 3 transitor in Li++.
\ u3 = RC ê - ú= 4 ...(iii)
2
ë2 ¥2 û 95. (a) 96. (c) 97. (a) 98. (a)
From equation (i), (ii) and (iii), we get hc hc
u1 = u2 + u3 or u1 - u2 = u3 99. (c) – = K.E.1 ...(i)
l1 l 0
88. (b) Radius of circular path followed by electron is given
hc hc
by, and – = K.E.2 ...(ii)
l2 l0
mu 2meV 1 2m
r= = = V
qB eB B e hc hc é l – l1 ù
Þ – = K.E.1 – K.E2 Þ hc ê 2 ú
2 2
B r e l1 l 2 ë l1l 2 û
Þ V= = 0.8V = K.E.1 – K.E2
2m
For transition between 3 to 2. (K.E.1 – K.E.2 ) l1l 2
\h =
æ 1 1 ö 13.6 ´ 5 c(l 2 – l1 )
E = 13.6 ç - ÷ = = 1.88eV
è 4 9ø 36 I.E.
100. (a) En = – for Bohr’s hydrogen atom.
Work function = 1.88 eV – 0.8 eV = 1.08 eV » 1.1eV n2
89. (b) For both the atoms the second excited state – 4R
Here, I.E. = 4R \ En = 2
corresponds to n
n = 3. Therefore, the angular momentum for each of –4 R æ 4 R ö
them is 3(h/2p). The energy, however, is proportional \ E2 – E1 = 2 – ç – 2 ÷ = 3R ..(i)
to Z2 where Z is the atomic number and hence 2 è 1 ø
numerical value of energy for hydrogen is less than hc
that for lithium. E2 – E1 = hn = ...(ii)
l
dU From (i) and (ii)
90. (a) - = F (conservative force field)
dr -K –34
Þ F= provides the centrifugal force for hc hc 6.6 ´ 10 ´ 3 ´ 108
r = 3R \ l = = = 300Å
circular motion of electron. l 3R 2.2 ´ 10 –18 ´ 3
mv 2 K nh
= Þ r=
r r 2p mK
1 2 1
K.E. of electron = mv = K Exercise - 2
2 2
P.E. of electron = K ln r 1. (c) According to Bohr's model of atom rading of an atom
E (n) = Total energy = K.E. + P.E. r
1 Ké n2h2 ù in ground state is r = 0 where r0 is Bohr's radius and
K + K ln r = 1 + log z
= ê ú z is a atomic number. Given r 0 = 53 pm
2 2ë 4p3 mk û
The atomic number of lithium is 3, therefore, the radius
E (2) - E (1)
Required ratio = =1 of Li++ ion in its ground state, on the basis of Bohr's
E (4) - E (2)
1
91. (a) model, will be about times to that of Bohr radius.
92. (c) Dl = l1 – l2 3
r 53
1 1 So, the radius of lithium ion is = 0 = » 18 pm .
=- z 3
é 1 1 ù R [1 - 0] Z 2
R ê 2 - 2 ú Z2
ë (2) (3) û
EBD_7418
698 PHYSICS

2. (c) If one decides to work in a frame of reference where n(n - 1) 4(4 - 1)


the electron is at rest, the given expression is not true = = =6
2 2
as it forms the non – inertial frame of reference.
1 é 1 1 ù 1
3. (a) The simple Bohr model cannot be directly applied to 8. (c) Wave number = RZ 2 ê - ú Þ l µ
calculate the energy levels of an atom with many l 2 2
êë n1 n úû Z2
electrons. So the nuclear the electrons not being By question n = 1 and n 1 = 2
subject to a central force. Then, l1 = l2 = 4l3 = 9l4
4. (a) Accroding Bohr's second postulate states that the
1 æ 1 1ö
electron evolves around the nucleus only in those 9. (c) As we know, = Re ç 2 - 2 ÷
orbits for which the angular momentum is some l è n1 n 2 ø
h For Lyman series, n1 = 1 and n2 = 2, 3, 4...
integral multiple of where h is the Planck's For Balmer series n1 = 2 and n2 = 3, 4, 5...
2p
–34
constant (= 6.6 × 10 J-s). So, the magnitude of 1 æ1 1 ö
= Re ç 2 - 2 ÷
angular momentum is kept equal to some integral l1 è1 2 ø
h
multiple of , where, h is Planck's constant and thus, 1 æ 1 1 ö l1 5
2p = Re ç - ÷ Þ l = 27
l2 è2 2
32 ø 2
the Bohr model does not gives correct value of
angular momentum. 10. (a) Absorption transition
5. (a) As we know that, C
The nuclear force is too much stronger than the
Coulomb force acting between charges or the B
gravitational forces between masses. The nuclear
binding force has to dominate over the Coulomb A
repulsive force between protons inside the nucleus.
The nuclear force between two nucleons falls rapidly
to zero as their distance is increase than a few Two possibilities in absorption transition.
femtometres. So in case of oxygen molecule, the
distance between atoms of oxygen is larger as
compared to the distance between nuclears in a
necleus. So, nuclear force between the nuclei of the
two oxygen atoms is not important because nuclear
forces are short – ranged.
6. (a) We know that,
Electron on the lowest state of the atom, called the
ground state have the lowest energy and the electron Three possibilities in emission transition.
revolving in the orbit of smallest radius, the Bohr Therefore, absorption transition < emission.
radius, r. The energy of this state (n = 1), E1 is – 13.6 eV. ze 2 k ze 2
Total energy of two H – atoms in the ground state 11. (c) U = –K ; T.E = –
r 2 r
collide in elastically = 2 × (–13.6 eV) = –27.2 eV.
The maximum amount by which their combined kinetic k ze 2
energy is reduced when any one H–atom goes into K.E = . Here r decreases
2 r
first excited state after the inelastic collision. So that 12. (b) For 2nd line of Balmer series in hydrogen spectrum
the total energy of the two H – atoms after the
inelastic collision 1 æ 1 1ö 3
= R (1) ç 2 - 2 ÷ = R
l è 2 4 ø 16
æ 13.6 ö (
=ç ÷ + 13.6 ) = 17.0eV é1 æ 1 1 ö 3R ù
è 22 ø
For Li2+ ê l = R ´ 9 çè 2 - 2 ÷ø = 16 ú
ë x 12 û
éëQ for excited state ( n = 2 ) ùû .
which is satisfied by n = 12 ® n = 6.
So, maximum loss of their combined kinetic energy. 13. (a) At closest distance of approach, the kinetic energy of
Due to inelastic collision the particle will convert completely into electrostatic
= 27.2 –17.0 = 10.2 eV potential energy.
æ 1
1 2
1 1 ö Kinetic energy K.E. = mv
7. (c) For the l = 975 Å; = Rç - ÷ 2
l ç n2 n 2 ÷
è 1 2ø KQq
where R is the Rydberg constant Potential energy P.E. =
r
Solving we get n 2 = n = 4 (Q n1 = 1 ground state) 1 2 KQq 1
Therefore number of spectral lines mv = Þ rµ
2 r m
ATOMS 699

14. (c) According to Bohr's theory, the wave number of the 21. (b) Bohr postulated that electrons in stationary orbits
last line of the Balmer series in hydrogen spectrum, around the nucleus do not radiate.
For hydrogen atom z = 1 This is the one of Bohr’s postulate. According to this
the moving electrons radiates only when they go from
1 æ 1 1 ö æ 1 1 ö
= RZ2 ç - one orbit to the next lower orbit.
2 ÷ = 10 × 1 ç 2 - 2 ÷
7 2
l n
è 2
2
n1 ø è2 ¥ ø 22. (a)
1
Þ wave number = 0.25 × 107 m–1 23. (b) In a Bohr orbit of the hydrogen atom
l Kinetic energy,
15. (d) For first line of Lyman series of hydrogen kze2
k=
hc æ1 1 ö 2rn
= Rhc ç - ÷
l1 è12
22 ø – kze2
Total energy, E =
For second line of Balmer series of hydrogen like ion 2rn
hc æ 1 1 ö So, Kinetic energy : total energy = 1 : –1
= Z2 Rhc ç - ÷ 24. (a) Energy of photon corresponding to first line of Balmer
l2 è2 2
42 ø
By question, l1 = l2 2 é1 1ù
series = (13.6) (2) ê - ú
æ1 1 ö 2 æ1 1 ö ë4 9û
Þ ç - ÷ = Z ç - ÷ or Z = 2 Energy need to eject electron from n = 2 level in H atom
è1 2 ø è 4 16 ø
16. (b) æ 1ö
= (13.6) çè ÷ø
4
1 é 1 1 ù
17. (c) = Rê 2 - 2 ú éæ 4 ö æ 1 ö ù æ 11 ö
l êë n1 n2 úû = (13.6) êçè1 - 9 ÷ø - çè 4 ÷ø ú eV = 13.6 ´ çè ÷ø = 4.155 eV
ë û 36
For limiting wavelength of Lyman series 25. (b) Rutherford confirmed that the repulsive force on
a-particle due to nucleus varies with distance
1 according to inverse square law and that the positive
n1 = 1, n2 = ¥ =R
lL charges are concentrated at the centre and not
For limiting wavelength of Balmer series distributed throughout the atom.
n1 = 2, n2 = ¥ 26. (a) Wavelength of emitted photon from nth state to the
1 æ1 1 ö
1 æ 1ö 4 ground state, = RZ 2 ç - ÷
= Rç ÷ Þ lB = Ln è12
n2 ø
lB è 4ø R -1
1 æ 1 ö
Ln = 1-
\ lB = 4lL = 4 × 912 Å. 2 ç ÷
RZ è n 2 ø
18. (b) For last line of Balmer series : n1 = 2 and n2 = ¥
Since n is very large, using binomial theorem
1 é 1 1 ù é1 1 ù
= RZ 2 ê - ú = R12 ê - ú Ln =
1 æ
1+
1 ö
lB 2 ç ÷
2 2 2
ë n1 n2 û ë2 ¥2 û RZ è n 2 ø
4
lB = ...(i) 1 1 æ 1 ö
R Ln = + ç ÷
2
For last line of Lyman series : n1 = 1 and n2 = ¥ RZ RZ 2 è n 2 ø
2pr æ n 2h2 ö 1
1 é 1 1 ù é1 1 ù
= RZ2 ê - ú = RZ2 ê - ú As we know, l n = n = 2p çç 2 2÷
÷ µn
lL 2 2
ë n1 n 2 û ë1 ¥ 2 û
2 è 4p mZe ø n
1 B
lL = ...(ii) Ln » A +
R ln2
4 27. (d) hnL = E ¥ - E1 ...(i)
lB R
= hn f = E ¥ - E 5 ...(ii)
Dividing equation (i) by (ii) l 1 =4
L 2
R z2 E5 æ 1 ö 1
Eµ Þ =ç ÷ =
19. (b) n 2 E1 è ø
5 25
ze 2 k ze 2 hn L E1
20. (c) U = –K ; T.E = – Eqn (i) / (ii) Þ n =
r 2 r h f E5
k ze 2 n L 25 n
K.E = . Here r decreases Þ = Þ nf = L
2 r nf 1 25
28. (d)
EBD_7418
700 PHYSICS

For the second spectral line in the Balmer series of


Exercise - 3
singly ionised helium ion n2 = 4 and n1 = 2 ; Z = 2
1 1 c–v 1 2 é1 1 ù 3R
1. (b) = \ = R (2) ê - ú = ...(ii)
l' l c+ v l ë 4 16 û 4
Here, l ' = 706 nm, l = 656 nm Dividing equation (i) and equation (ii) we get
2 2
c–v æ l ö æ 656 ö v 0.14 l 5R 4 5
\ =ç ÷ =ç ÷ = 0.86 Þ c = 1.86 = ´ = \ l = 1215 Å
c+ v è l'ø è 706 ø 6561 36 3 R 27
Þ v = 0.075 × 3 × 108 = 2.25 × 107m/s So, n2 = 4
2. (a) Number of emission spectral lines
n1 = 2 is verified.
n(n - 1) n (n - 1)
N= \3 = 1 1 , in first case. 7. (c) According to the question,
2 2 2
Or n1 - n1 - 6 = 0 or (n1 - 3)(n1 + 2) = 0 nh nh nh nh
Take positive root. \ n1 = 3 2pr = nl = = or mvr = or mv =
p mv 2 p 2 pr
n (n - 1)
Again, 6 = 2 2 , in second case. mv 2
2 F = qv B =
r
Or n 22 - n 2 - 12 = 0 or (n 2 - 4)(n 2 + 3) = 0. nh
mv nh
Take positive root, or n 2 = 4 or, q B = = or, r2 =
r 2pr.r 2pqB
2pKZe 2
Now velocity of electron u = nh
nh
or, r = i.e., r µ n1/2
u1 n 2 4 2pqB
\ = = .
u2 n1 3 8. (c) Magnetic moment of the hydrogen atom, when the
1 c 1 electron is in n th excited state, i.e., n’ = (n + 1)
3. (b) v = or v µ
137 n n As magnetic moment M n = In A = in (prn2 )
Since v is reduced to one - third, therefore
n=3 mz 2 e5 n 2h2 æ 1 ö
Now, r µ n2 i n = eVn = ; rn = çk = ÷
4e02 n 3 h 3 2
4p kzme è 4p Î0 ø
2

mv 2 3q 2 3q2
4. (a) = Þ mvr = ...(i) Solving we get magnetic moment of the hydrogen atom
r 4pe 0 r 2 4pe 0 v for nth excited state
nh æ e ö nh
and = mvr ...(ii) Mn ' = ç ÷
2p è 2m ø 2p
Using (i) and (ii) and putting n = 1
mv2 e2 æ 1 b ö
3q 2 3q 2 (c) As F = = +
4p Î0 çè r 2 r 3 ÷ø
h 9.
= Þ v= r
2p 4pe 0 v 2e 0 h
nh nh
5. (a) En = – 3.4 eV and mvr = Þv=
2p 2 pmr
The kinet2ic energy is equal to the magnitude of total
2
energy in this case. æ nh ö 1 e2 æ 1 b ö
\ mç ÷ ´ = ç + ÷
\ K.E. = + 3.4 eV è 2pmr ø r 4p Î0 è r 2 r3 ø
The de Broglie wavelength of electron 1 b mn 2 h 2 4p Î0 a0 n2 1 b
–34 or, 2 + 3 = or, = +
h 6.64 ´ 10 r r 4p2 m2 e2 r 3 r 3
r 2
r3
l= = eV
2mK 2 ´ 9.1 ´ 10 –31 ´ 3.4 ´ 1.6 ´ 10 –19 æ ö
Î h2
= 0.66 × 10–9 m çQa 0 = 0 Given ÷ \ r = a0n2 – b
ç mpe 2 ÷
è ø
1 é1 1ù
6. (a) We know that = RZ 2 ê 2 - 2 ú n(n – 1)
l ëê n1 n2 ûú 10. (c) =6
2
The wave length of first spectral line in the 4
Balmer series of hydrogen atom is 6561Å . Here n2 = 3 3
and n1 = 2 2
1 2æ1 1ö 5R 1
\ 6561 = R(1) ç 4 - 9 ÷ = 36 ...(i) n2 – n – 12 = 0
è ø (n – 4) (n + 3) = 0 or n =4
ATOMS 701

ke 2 dU 3ke2 1875 R æ 3ö
11. (b) U=- ,F = - =- For metal A : = R (Z1 - 1) 2 ç ÷ Þ Z1 = 26
2 4 è 4ø
2R dR 2R 4
2 æ 3ö
mv2 mv 2 3ke 2 For metal B : 675 R = R (Z2 - 1) çè ÷ø Þ Z2 = 31
But, F= Þ = 4
R R 2R 4 Therefore, 4 elements lie between A and B
nh
Also, mvR = 14. (b) KEmax = 10eV; f = 2.75 eV
2p
Total incident energy
6p 2 ke 2 m
Solve to get : R = E = f + KEmax = 12.75 eV
n 2h2
\ Energy is released when electron jumps from the
2
k mv excited state n to the ground state.
12. (b) When F = k = centripetal force, then =
r r r Q E4 – E1 = {– 0.85 – (–13.6) ev} = 12.75eV
2
Þ mv = constat Þ kinetic energy is constant Þ T is
\ value of n = 4
independent of n.
15. (a) Q T µ n3
1 é1 1 ù
13. (c) Using = R(Z - 1)2 ê 2 - 2 ú Tn1 = 8 Tn2 (given) Hence, n1 = 2n2
l ëê n 2 n1 úû
For a particle, n 1 = 2, n2 = 1
EBD_7418
702 PHYSICS

28. NUCLEI

Exercise - 1 understandable, as we have already seen that most


of the atom is empty. Ordinary matter consisting of
1. (b) Protons and neutrons are the constituents of atoms has a large amount of empty space.
nucleus. 16. (b) As momentum is conserved, therefore,
2. (c) Radius of nucleus R = R0 A1/3 where A is the mass
m1 A1 v 2 1
number of nucleus. = = =
m 2 A 2 v1 2
4 3 æ4 ö
\ Volume of nucleus = pR = ç p R 30 ÷ A 1/ 3 1/ 3
3 è3 ø R1 æ A1 ö æ1ö
\ =ç ÷ =ç ÷ = 1 : 21 / 3
\ Volume is proportional to A. R 2 çè A 2 ÷ø è2ø
3. (c)
4. (c) Nuclear radius = 10–15 m. 17. (b) R = R 0 (A)1/ 3
5. (c) As we know R = R0 A 1/3 æA ö
1/ 3 1/3
R æ 27 ö 3
\ R µ A1/3 \ 1 =ç 1÷ =ç ÷ =
Vl 8 1 R2 è A2 ø è 125 ø 5
As, V µ R3 or V µ A = = = 0.125
\
V2 64 8 5
R 2 = ´ 3.6 = 6 fermi
6. (a) Isotones means equal number of neutrons i.e., (A–Z) = 3
74 –34 = 71 – 31 = 40. 18. (a) Let the percentage of B10 atoms be x, then average
7. (b) R = R 0 (A)1/ 3 atomic weight
1/3
R1 æ A1 ö æ 256 ö
1/ 3
R 10x + 11(100 - x)
\ = =ç = 4 Þ R 2 = 1 = 2 fermi = = 10.81 Þ x = 19
R 2 çè A 2 ÷ø è 4 ÷ø 4 100
2 N 10 19
8. (d) and 1H3 are isotopes
1H \ B =
2 He3
and 1H3 are isobars N 11 81
197 B
79Au and 80Hg198 are isotones. 19. (c) We use the formula,
9. (c) Nucleus does not contain electron. R = R0 A1/3
1/ 3 1/ 3 This represents relation between atomic mass and
Rs æ A ö æ 32 ö
10. (c) =ç s ÷ =ç ÷ =2 radius of the nucleus.
R He è A He ø è 4ø For berillium, R1 = R0 (9)1/3
For germanium, R2 = R0 A1/3
A
11. (a) ZX + 0 n1 ¾¾
® 3 Li 7 + 2 He 4
R1 (9)1/ 3 1 (9)1/ 3
On comparison, = Þ =
R2 ( A)1/ 3 2 ( A)1/ 3
A = 7 + 4 – 1 = 10, z = 3 + 2 – 0 = 5
It is boron 5B10 1 9
Þ = Þ A = 8 ´ 9 = 72 .
12. (b) Both statements are separately correct. 8 A
13. (b) 20. (b) For nucleus of 8O16
Mass = (16) (1.67 × 10–27) kg
14. (a) Each atom of 6C14 contains 6 p, 6 e and 8 n
\ In 14 gram of 6C14 4 3
Volume = pR
p = 6 × 6 × 1023 = 36 × 1023 3
n = 8 × 6 × 1023 = 48 × 1023 4
e = p = 36 × 1023 = p(3 ´ 10 –15 )3 m3 = 36p × 10–45 m3
3
15. (d) As R µ A1/ 3 and volume of nucleus is proportional mass 16 × 1.67 × 10 –27 kg
to R3 is proportional to A. Thus, the density of Density = =
volume 36π ×10 –45 m3
nucleus is a constant, independent of A, for all
nuclei. Different nuclei are likes drop of liquid of = 2.35 × 1017 kgm–3
constant density. The density of nuclear matter is 21. (d) Average BE/nucleon increases first, and then
approximately 2.3 × 1017 kg m–3. decreases, as is clear from BE curve.
This density is very large compared to ordinary 22. (d) 23. (b) 24. (b)
matter, say water which is 1023 kg m–3. This is 25. (c) Control rods are made of cadmium.
NUCLEI 703

26. (a) E = mc2 = 10–6 × (3 × 108)2 = 10–6 × 9 × 1016 = 9 × 1010 J 46. (d) For 0 < A < 80, Ebn first increases then decreases.
27. (b) In case of Proton-Proton Electrostatic repulsive force 47. (b) B.E. = 0.042 × 931 ; 42 MeV
is also present which reduces the net force.
Number of nucleons in 37 Li is 7.
28. (c) 29. (d) 30. (b)
B.E 42
31. (d) Mass defect = \ B.E./ nucleon = = 6 MeV ; 5.6 MeV
c 2 7
Mass of nucleus = Mass of proton 48. (b) Fission rate
+ mass of neutron – mass defect total power 5 = 1.56 × 1011 s–1
= =
energy -13
32. (d) Atomic mass M(H) of hydrogen and nuclear mass 200 ´ 1.6 ´ 10
(Mn) are fission
M (H) = 1.007825 u and Mn = 1.008665 u 49. (c) Nuclear force is nearly same for all nucleus.
Mass defect, 50. (b) Q = 4 (x2 – x1)
Dm = [M (H) + Mn – M (D)] 51. (b) Liberated energy Q = 117 × 8.5 + 117 × 8.5 – 236 × 7.6 =
M (D) = mass of deuteron = 2.016490 u – 2.014102 u 200 MeV. Thus, in fission of one Uranium nuclei nearly
= 0.002388 u 200 MeV energy is liberated.
As 1 u corresponds to 931.494 MeV energy, therefore,
16
mass defect corresponds to energy, 52. (c) 4 42 He ¾
¾® 8O
33. (c)
B.E = DM × 931.5 MeV
34. (c) On an average 2.5 neutrons are released per fission of
the uranium atom. = (4 × 4.0026 – 15.9994) × 931.5 = 10.24 MeV
And the energy of the neutron released per fission of
the uranium atom is 2 MeV. æ Eö n ´ 200 ´ 106 ´ 1.6 ´ 10 -19
53. (a) P = n ç ÷ Þ 1000 =
è tø t
35. (a) 92U 235 + 0n1 ® 38 Sr 90 + 54 Xe143 + 30 n1 + energy
36. (c) Moderator slows down neutrons. n
Þ = 3.125 ´ 1013.
37. (d) Extremely high temps needed for fusion make K.E. large t
enough to overcome repulsion between nuclei.
54. (b) Mass defect Dm = 0.02866 a.m.u.
38. (c) 39. (c) 40. (c)
41. (c) Binding energy Energy = 0.02866 × 931 = 26.7 MeV
= [ZMP + (A – Z)MN – M]c2 As 1H2 + 1H2 ¾® 2He4
= [8MP + (17 – 8)MN – M]c2 Energy liberated per a.m.u = 13.35/2 MeV
= [8MP + 9MN – M]c2 = 6.675 MeV
= [8MP + 9MN – Mo]c2 55. (a) The chemical reaction of process is 212 H ® 42 He
2.22 Energy released = 4 ´ (7 ) - 4(1.1) = 23.6 eV
42. (a) B.E H = = 1.11
2
56. (b) Let E be the energy of proton, then
28.3
B.E He = = 7.08 E + 7 ´ 5.6 = 2 ´ [4 ´ 7.06]
4
492 Þ E = 56.48 - 39.2 = 17.28MeV
B.E Fe = = 8.78 = maximum
56
1
1786
57. (a) 0n ® 11H + -1e 0 + n + Q
¾¾
B.E U = = 7.6
235 The mass defect during the process
56 is most stable as it has maximum binding energy Dm = mn - mH - me
26 Fe
per nucleon. = 1.6725 × 10–27 – (1.6725 × 10–27+ 9 × 10–31kg)
43. (a) = – 9 × 10–31 kg
44. (b) (A) ® (2); (B) ® (1); (C) ® (4); (D) ® (3) The energy released during the process
0.3 E = Dmc2
45. (d) E = Dm.c2 Þ E = ´ (3 ´ 108 ) 2 = 2.7 ´ 1013 J
1000 E = 9 × 10–31× 9 × 1016 = 81 × 10–15 Joules

2.7 ´ 1013 81 ´ 10 -15


= = 7.5 ´ 106 kWh. E= = 0.511MeV
3.6 ´ 106 1.6 ´ 10 –19
EBD_7418
704 PHYSICS

58. (c) Energy released = (80 × 7 + 120 × 8 – 200 × 6.5) 185


83 Bi ¾® 184 1
82 Pb + 1 H
59. (d) All the characteristics given are true for
proton
radioactivity.
60. (c) b-rays are charged particles emitted by nucleus. 239
¾® 140 99
94 Pu 57 La + 37 X
61. (c) Fusion is not a mode of decay.
(c) is the correct option.
0.693 1 75. (c) No. of nuclide at time t is given by N = N o e - lt
62. (c) Half life Th = , Tm = Clearly,, Th < Tm .
l l Where N o = initial nuclide
thus this equation is equivalent to y = ae - kx
63. (d) 64. (a) 65. (a)
thus correct graph is
66. (d) Half life of a substance doesn’t depends upon amount,
temperature and pressure. It depends upon the nature N
of the substance.

67. (b) 0 is known as b– particle & n is known as


-1 e
antineutrino. Since in this reaction n is emitted with
t
0 (b– particle or electron), so it is known as b-
-1 e 76. (c) If A0 is the initial activity of radioactive sample, then
activity at any time
decay.
1
68. (c) An electron is accompanied by an antineutrino. A = A0e-lt or 1 × 106 = 4 × 106 e–l × 20 or e-20l =
4
69. (c) The count rate after 100 hour is given by
0.6931 ´ 1 0.6931 -l´100 =
A0e –100l = A0[e-20l ]5
70. (d) T = = year = 1620 years A ' = A0e
l 4.28 ´ 10 -4 5
6é1 ù
71. (b) A nucleus is denoted by ZXA = 4 ´ 10 ê ú =3.91 × 103 per second
ë4û
An isotope should have same Z. 77. (d) If N01 and N02 are initial numbers of U and
235 238
U
a–particle = 2He4; b–particle = –1b0 respectively, then after time t.
The emission of one a particle and the emission of N1 = N 01e-l1t
two b particles maintain the Z same.
and N2 = N02e-l 2t
Hence, for isotope formation 2b particles and one a
N1 N01 - (l1 -l 2 )t
particle are emitted. \ = e
N2 N 02
72. (c) Number of a-particles emitting
N01 N1 (l1 -l2 )t
dN or = e … (i)
per second = N02 N2
dt 0.693 0.693
= lN=n Where l1 = t1/ 2 = 7.04 ´ 108
n = 9.85 ´ 10 -10 year -1
\ l=
N 0.693
and l2 = 1.55 ´ 10-10 year
0.693 t1/ 2 =
\ T=
l -10
\ (l1 - l 2 )t = éë (9.85 – 1.55) ´ 10 ùû ´ (2 ´ 10 )
9

0.693N = 1.66
= Substituting this value in equation (i), and
n
N1
73. (b) The common time measure of how long any given = 0.0072 , we get
N2
type of radionuclide lasts is the half-life T1/2 of a
radionuclide, Which is the time at which both N and N01
N02 = 0.0072 × e1.66 = 0.0379
R have been reduced to one-half their initial values.
A sample may have nuclei which are not radioactive. ; 3.8 %
78. (a) Q - Value of b - decay is
74. (c) (A) ® (2); (B) ® (1); (C) ® (4); (D) ® (3)
Q = [m(19 O) - m(19 F )]c 2
15
8 O ¾® 15
7 N+
0
1b
=[19.003576 u – 18.998403 u ] ×(931 MeV/u)
+ = 4.816 MeV.
b particle
79. (c) The atomic number and mass number of resulting
238 234 4 atom will be,
92 U ¾® 90 Th + 2 He
a- particle Z = 88 – 3 × 2 + 1 = 83
and A = 238 – 3 × 4 = 226
NUCLEI 705

80. (d) The number of a-particles 92. (a) We assume that mass number of nucleus when it was
238 - 206 at rest = A
= =8
4 Q mass number of a -particle = 4
If n be the number of b-particles emitted then
\ mass number of remaining nucleus = A - 4
92 – 8 × 2 + n × 1 = 82
As there is no external force, so momentum of the
\ n = 6.
81. (d) system will remain conserved

dN 0.693N 4v
82. (c) Radioactivity = - = lN = Þ 0 = (A - 4)v¢ + 4v Þ v' = -
dt T1/ 2 (A - 4)
0.693 ´ 108 0.693 ´ 1.2 ´ 108 negative sign represents that direction is opposite
= = = 0.693 ´ 2 ´ 106. to the direction of motion of a -particle.
50 60
Radioactivity is proportional to 1/T1/2, and not to T1/2. 93. (b) A nucleus is denoted by ZXA
83. (c) 7 1
¾® 84 Be + 00 g An isotope should have same Z.
3 Li + 1p ¾
A 1 a–particle = 2He4; b–particle = –1b0
84. (a) Z X + 0 n ¾¾® 3 Li7 + 2 He 4
The emission of one a particle and the emission of
On comparison,
two b particles maintain the Z same.
A = 7 + 4 – 1 = 10, z = 3 + 2 – 0 = 5
Hence, for isotope formation 2b particles and one a
It is boron 5B10
particle are emitted.
85. (a) After every half-life, the mass of the substance reduces
to half its initial value. ln 2 ln 2
94. (a) T1/ 2 = \l =
5 years N 5 years N /2 l T1/ 2
N 0 ¾¾¾¾ ® 0 ¾¾¾¾ ® 0
2 2 In2 ln 2 l T
N 0 5years N 0 / 4 N 0 Þ lA = ,lB = Þ A = B.
= ¾¾¾¾ ® = TA TB l B TA
4 2 8 95. (b) Momentum
86. (c) Fraction remains after n half lives
E hn
N æ 1ö
n
æ 1ö
t /T Mu = =
=ç ÷ =ç ÷ c c
N0 è 2 ø è 2ø Recoil energy
T/2 2
N æ 1ö T æ 1ö
1/ 2
1 1 1 M2u 2 1 æ hn ö h 2 n2
\ =ç ÷ =ç ÷ = Mu 2 = = ç ÷ =
N0 è 2 ø è 2ø 2 2 2 M 2M è c ø 2Mc2
7 96. (a) Applying the principle of conservation of linear
87. (d) of Cu decays in 15 minutes. momentum
8 3
7 1 æ1ö 4u
\ Cu undecayed = N = 1 – = = ç ÷ (4) (u) = (v) (238) Þ v =
8 8 è2ø 238
\ No. of half lifes = 3 97. (b) Radioactive decay is a random process. Each decay is
t 15 a completely independent event. Therefore, which
n= or 3 =
T T particular nucleus will decay at a given instant of time
15 cannot be predicted. In other words when a particular
Þ T = half life period = = 5 minutes
3 nucleus will decay cannot be predicted. Each nucleus
88. (c) The range of energy of b-particles is from zero to some has same probability of disintegration.
maximum value.
89. (d) 90. (d) 1 æN ö 1 æ 5000 ö
98. (a) K = ln ç 0 ÷ Þ K = ln ç ÷
91. (d) Nuclear reactions conserve total charge, and also t è Nø 5 è 1250 ø
conserve the total approximate mass. The other
1 2
particle in the reaction will have mass = 236 – 140 – 94 = 2 ln (4) = ln 2 = 0.4ln 2
The other particles are two neutrons. Hence, (a) is 5 5
not correct. dN
For nuclei, number of protons tells the charge. So, 99. (c) = l N = activity R
dt
the other particle must have charge Z such that
R0 = lN0 at t = 0, R1 = lN1 at t = 5 minutes
92 = 54 + 38 + Z \ Z= 0
Therefore, the other particles have a total atomic mass 2.303 N
where l = log 0
2 and total charge 0. Hence, only (d) is correct. t N1
EBD_7418
706 PHYSICS

dN 1 108. (a) Activity of sample becomes 2500 from 5000 in 75 days


100. (a) We know that , = lN = N therefore its half life is 75 days, so
dt Tmean
R
R = 0 = 5000 Þ R 0 = 5000 ´ 4 = 20, 000
1 150
\ 1010 = ´N
109 2 75
109. (b) No. of a particles emitted = 8
\ N = 1019 No. of b– particles emitted = 4
i.e. 1019 radioactive atoms are present in the freshly
No. of b+ particles emitted = 2
prepared sample.
Z = 92 – 2 × 8 + 4 – 2 = 78
The mass of the sample
= 1019 × 10–25 kg = 10–6kg = 1 mg 110. (b) As a result of emission of 1 a-particle, the mass number
101. (d) Time taken to disintegrate completely by a substance decreases by 4 units and atomic number decreases by
2 units. And by the emission of 1 positron the atomic
N0
is infinity as log = lt number decreases by 1 unit but mass number remains
N constant.
N0 \ Mass number of final nucleus = A – 12
Þ log = lt
0 Atomic number of final nucleus = Z – 8
log ¥ = lt \ Number of neutrons = (A – 12) – (Z – 8)
hence when N ® 0, t ® ¥ . =A–Z –4
t 10 Number of protons = Z – 8
N æ 1 öT N æ 1 ö 20 A-Z -4
102. (a) =ç ÷ or =ç ÷
N0 è 2 ø 10000 è 2 ø \ Required ratio =
Z -8
10000 ´1 10000 111. (d)
or N = = = 7070.
2 1.414 112. (a) For substance A :
n 48/12
æ1ö N 1 æ 1ö N0 N0
103. (b) ç ÷ = = gives n = 4.32 2N0 ® N A = 2N 0 ç ÷ = =
è 2ø 23 8
è2ø N 0 20
For substance B :
t = nT = 4.3 × 3.8 = 16.5 days 48/16
104. (b) To work safely, intensity must reduce by 1/64 æ 1ö N0 N0
N0 ® N B = N 0 ç ÷ = =
t/T 6 t/T è 2ø 23 8
N 1 æ1ö æ1ö æ1ö NA : NB = 1 : 1
\ = =ç ÷ i.e. ç ÷ = ç ÷
N 0 64 è 2 ø è2ø è2ø 113. (b) Effective half life is calculated as
t 1 1 1
or = 6 or t = 6 T = 12 hrs = +
T T T1 T2
105. (d) 1 1 1
= + Þ T = 12 years
t / 5700 T 16 48
C14 1 æ 1 ö t
106. (c) = =ç ÷ Þ = 2 Þ t = 11400 years 3
C12 4 è 2 ø 5700 Time in which will decay is 2 half lives = 24 years
4
107. (c) Let N0 be the number of atoms of X at time t = 0. 180 a b
Then at t = 4 hrs (two half lives)
114. (a) 72 A ® 70 A1176 ¾¾
¾¾ ® 71 A 2176

N0 3N0 a g
Nx = and N y = ®69 A3172 ¾¾®69 A 4172
¾¾
4 4
115. (d) Radioactivity at T1 , R1 = l N1
\ Nx/Ny = 1/3
Radioactivity at T2, R2 = l N2
and at t = 6 hrs (three half lives)
\ Number of atoms decayed in time
N0 7N 0 (T1 – T2) = (N1 –N2)
Nx = and N y =
8 8 (R 1 - R 2 ) (R1 - R 2 ) T
Nx 1 = =
or = l 0.693
Ny 7
1 1 1 µ (R1 - R 2 ) T
The given ratio lies between and .
4 3 7 116. (a) Let no. of a-particles emitted be x and no. of b particles
Therefore, t lies between 4 hrs and 6 hrs. emitted be y.
NUCLEI 707

Diff. in mass no. 4x = 238 – 206 = 32 Þ x = 8 Þ ZX ® Z + 1A + –1e +


A Y 0
v + Q1
Diff. in charge no. 2x – 1y = 92 – 82 = 10
16 – y = 10, y = 6 Q1 = [mn (ZXA) – mn (z+1 YA) – me] c2
117. (b) Conservation of linear momentum requires: = [mn (ZXA) + Zme – mn (Z + 1 YA) – (Z + 1) me] c2
mradonvradon= mheliumvhelium with helium identified as the = [m (ZXA) – m (Z + 1 YA)] c2
alpha particle. The nuclear masses can be approximated = (Mx – My) c2
by their mass numbers (222 and 4). Thus, the recoil
5. (a) Tritium (1H3) has proton and 2 neutrons. If a neutron
speed of the radon is (4/222) × 1.5 × 107 m/s = 2.7 x 105
m/s. decays as n ® P + e + v , then the nucleus may have
2 protons and neutron, i.e., tritium atom will convert
118. (a)
into 2He3 (2 protons and 1 neutron).
dN A dN B Binding energy of 1 H3 is less than that of 2He3
119. (b) = -l1 N A , = 2l1 N A - l 2 N B ,
dt dt nucleus, So, tr ansformation is not possible
dN B energetically.
NB = maximum Þ =0
dt 6. (b) Stable heavy nuclei have more neutrons than protons
because electrostatic force between protons is
2l
Þ 2l1 N A = l 2 N B Þ NB = 1 NA repulsive, which causes unstability of nucleus.
max max l2
7. (b)
2l
Þ NB = 1 N 0 e -l1t = 2 8. (d) BE of 2He4 = 4 × 7.06 = 28.24 MeV
max l2
N 0 - l1t N BE of 73 Li = 7 × 5.60 = 39.20 MeV
120. (d) N x = e = 0.2N 0 N y = 0 e - l 2 t = 0.8N 0 7 1
2 2 3 Li +1 H ® 2 He 4 + 2He 4 + Q
( l1 -l 2 )
e = 4 Þ t = 8 ´ 10 years
9
39.20 28.24 ´ 2(= 56.48 MeV)
Therefore, Q = 56.48 – 39.20 = 17.28 MeV.
3
Exercise - 2 N 1 Nx 1 æ1ö
9. (c) As x = (Given) Þ = =ç ÷
1. (c) Radioactivity is a process due to which a radiactive Ny 7 Nx + Ny 8 è 2 ø
material spontaneously decays. Half-life time for a Therefore, age of the rock
radioactive remain average half of its or of radioactive
t = 3T1/2 = 3 × 1.4 × 109 yrs = 4.2 × 109 yrs.
atoms will decay. So, the containers will in general
have different number of atoms of the material,after dN
10. (c) = n - lN
one year means one half life i.e., average atom of dt
radioactive subtance remain after 1 year, in each
dN = (n - lN)dt
1
container is equal to of 10000 = 5000 atoms. N t N
2 dN 1 - ldN
2. (b) ò n - lN ò
= dt Þ -
l ò n - lN = t
N0 0 N0
3. (b) b-particle carries one unit of negative charge and a-
particle carries 2 units of positive charge and g-particle 1
carries no charge, so electronic energy levels of the
Þ- [log e (n - lN )]NN0 = t
l
atom charges in emission of a and b particle, but not
in g-decay. 1é æ n - lN ö ù
Þ- êlog e ç ú=t
4. (a) Let the nucleus is ZXA radiate b+ decay is represented l ëê è n - lN0 ÷ø ûú
as
é æ n - lN 0 öù
ZX ® Z–1 Y + +1e + v + Q2
A A 0
Þ lt = êlog e ç ÷ú
where Z and A have usual meaning : – ë è n - lN øû
\ Q2 = [mn (ZXA) – mn (Z –1 YA) – 2me] c2 n - lN 0
e lt =
= [mn (ZXA) + Zme – mn (Z – 1YA) – (Z–1) me – 2me]c2 n - lN
= [m (ZXA) – m (Z–1 YA) – 2me] c2 n - lN = (n - lN 0 ) e - lt
= (Mx – My – 2me ) c2
n æn ö
Let the parent nuclei ZXA is radioactive atom and b– - ç - N 0 ÷ e -lt = N
decay as under is represented as l èl ø
EBD_7418
708 PHYSICS

11. (b) b-particles, being emitted with very high speed Given W = 2Y
compared to a-particles, pass for very little time near BE of reactants = 120 × 7.5 = 900 MeV
the atoms of the medium. So the probability of the BE of products = 2 × (60 × 8.5) = 1020 MeV.
atoms being ionised is comparatively less. But due to 17. (b) For At½ = 20 min, t = 80 min, number of half lifes n = 4
this reason, their loss of energy is very slow and they
No
can penetrate the medium through a sufficient depth. \ Nuclei remaining = 4 . Therefore nuclei decayed
12. (d) In an explosion a body breaks up into two pieces of 2
unequal masses both part will have numerically equal No
= N0 - 4
momentum and lighter part will have more velocity. 2
U ® Th + He For Bt½ = 40 min., t = 80 min, number of half lifes n = 2
No
P2 P2 \ Nuclei remaining = 2 .
KETh = , KEHe = 2
2mTh 2m He
No
sinc mHe is less so KEHe will be more. Therefore nuclei decayed = N0 - 2
2
14 1 N No 1
C No - 4
13. (c) = 16 = N 1-
16 15 4 5
12
C 0 2
\Required ratio = No = 1 = ´ =
N n No - 2 1- 16 3 4
æ 1ö 2 4
Q N0 = ç ÷
è 2ø 18. (c) Let N0 be the number of atoms of X at time t = 0.
1 n 4 n Then at t = 4 hrs (two half lives)
æ 1ö æ 1ö æ 1ö
Þ = çè ÷ø Þ çè ÷ø = çè ÷ø
16 2 2 2 N0 3N0
Nx = and N y =
or, n = 4 4 4
t \ Nx/Ny = 1/3
or =4 and at t = 6 hrs (three half lives)
T
or t = 4 × T = 4 × 5730 = 22920 years N0 7N 0 Nx 1
14. (c) We know that energy is released when heavy nuclei Nx = and N y = or =
8 8 Ny 7
undergo fission or light nuclei undergo fusion.
Therefore Assertion is correct. 1 1 1
The given ratio lies between and .
The Reason is incorrect because for heavy nuclei the 4 3 7
binding energy per nucleon decreases with increasing Therefore, t lies between 4 hrs and 6 hrs.
Z and for light nuclei, B.E/nucleon increases with 19. (d) M(8O16) = M (7N15) + 1mP
increasing Z. binding energy of last proton
15. (c) Let the initial mass of uranium be M0 = M (N15) + mP – M (1O16)
3 = 15.00011 + 1.00783 – 15.99492
Final mass of uranium after time t, M = M 0 = 0.01302 amu = 12.13 MeV
4
According to the law of radioactive disintegration. 20. (a) Given, l A = 8l, l B = l
M æ1ö
t /T
M NA e -l A t
=ç ÷ Þ 0 = ( 2)
t /T
NB = Þ N o e -l B t = No
M0 è 2 ø M e e
æM ö t e -lt = e -8lt -1
\ log10 ç 0 ÷ = log10 ( 2 ) Comparing both side powers
è M ø T
1
æM ö æ4ö -lt = -8lt - 1 –1 = 7lt t = –
log10 ç 0 ÷ T log10 ç ÷ 7l
t =T è M ø= è3ø
21. (d) N = N0e–lt
log10 ( 2 ) log10 ( 2 )
Here, t = 5 minutes
T log10 (1.333) æ 0.1249 ö
= = 4.5 ´109 ç ÷ N0
log10 ( 2 ) è 0.3010 ø = N 0 × e -5l
e
Þ t = 1.867 × 109 yr. 1
16. (c) Energy is released in a process when total binding Þ 5l = 1 , or l= ,
5
energy (BE) of products is more than the reactants.
By calculations we can see that this happens in l n2
option (c). Now, T1/2 = = 5 l n2
l
NUCLEI 709

22. (d) We know that energy is released when heavy nuclei d/N
undergo fission or light nuclei undergo fusion. 27. (d) = lN
dt
Therefore Assertion is correct.
The Reason is incorrect because for heavy nuclei the NA m
Number of radius nuclei in mg =
binding energy per nucleon decreases with increasing 226
Z and for light nuclei, B.E/nucleon increases with 0.693 0.693
increasing Z. Decay constant, l = t =
1/ 2 1620 ´ 3.16 ´ 107
23. (d) Let initially there are total N0 number of nuclei
NB dN 6.02 ´ 1023 m 0.693
At time t N = 0.3(given) Þ NB = 0.3NA = 10 = ´
A dt 226 1620 ´ 3.16 ´ 107
N0
N0 = NA + NB = NA + 0.3NA \ NA =
1.3 10 ´ 226 ´ 162 ´ 3.16 ´ 107
N \ m=
Nt = N0 e– lt or, 0
= N0 e– lt 6.02 ´ 1023 ´ 0.693
1.3
= 2.77 × 10–10 g = 2.77 × 10–13 kg
1
= e–lt Þ ln(1.3) = lt
1.3 N
dN
t
dN N
or, t =
ln(1.3)
Þ t=
ln(1.3) ln(1.3) T
= ln(2)
28. (d)
dt
= 50 -
0.5
ò 50 - 2 N = ò dt
l ln(2) 0 0

T N = (100 (1 – e–t/2)) = 25
æ1ö
n t = 2 ln (4/3)
24. (b) N = N0 ç ÷
è2ø
n
Exercise - 3
N0 æ1ö
or = N0 ç ÷ 1. (c) l A = 5l and l B = l
16 è2ø
or n =4 2
N A æ 1ö
At t = 0, ( N0 ) A = ( N 0 ) B Given, =ç ÷
t 2 1 NB è eø
Half life t 1/2 = = = h According to radioactive decay,
n 4 2
N NA
25. (d) Mass of uranium changed into energy = e -lt \ = e -l At ..... (1)
N0 ( N0 ) A
0.1
= ´1 NB
100 = e -l Bt ..... (2)
( N0 )B
= 10–3 kg.
From (1) and (2),
The energy released = mC2 2 4l t
NA
= e - (5 l-l )t Þ æç ö÷ = e -4lt = æç ö÷
= 10–3 × (3 × 108)2 1 1
NB è eø è eø
= 9 × 1013 J.
26. (a) Number of nuclei remaining, N = 600 – 450 = 150 after 1
Þ 4lt = 2 \ t = .
time ‘t’ 2l
N 150 1 2. (d) The nuclear reaction for the process is
= = 235
N0 600 4 92U ® 2 46 Pd 116 + 2g
N0 The binding energy of 92 U 235 nucleus
N = N0e–lt Þ ln = lt
N = 7.2 × 235 = 1692 MeV
1 N0 Binding energy of two 46 Pd 116 nuclei
Þ t= ln
l N = 2 × 8.2 × 116 = 1902.4 MeV
2.303 ´ T 1 Energy of two g rays = 2 × 5.2 = 10.4 MeV
2 N0 \ Energy released = 1902.4 – (1692 + 10.4) = 200 MeV
Þ t= log10
0.693 N 3. (c) The net reaction is

=
2.303 ´10
0.693
log10 4 3 ( 12 H) ®42 He + n + p
Q = [3 × m(2H) – m(4He) – m(p)] × 931 MeV = 3.87 × 10–12 J
EBD_7418
710 PHYSICS

This is the energy produced by the consumption of 3 9. (d) If m kg is the required mass of the uranium, then number
deuteron atoms. So, the total energy released by 1040 of nuclei
deuterons is ( m ´ 1000) ´ 6.02 ´ 1023
-12 =
3.8 ´ 10 235
´ 1040 = 1.29 ´ 10 28 J
3 Each U 235 nucleus releases energy 200 MeV,,
Let total supply of deuterons in star be exhausted in t \ total energy released in 10 years
seconds.
Then, m ´ 6.02 ´ 1026
Ein = ´ 200
1016 × t = 1.29 × 1028 Þ t = 1.295 ´ 1012 s 235
1 Energy required in 10 years, Eour= Pt
4. (d) After first half hrs N = N0
2 6
=(1000 ×10 ) × (10 ×365× 24 ×3600)
4 5
1 1 æ 1ö é 1 ù æ 1ö Eout
for t = to t = 1 N = ç N0 ÷ ê ú = N0 ç ÷ Efficiency h =
2 2 è ø
2 ë2û è 2ø Ein
1 Substituting the values, we get
for t = 1 to t = 2 hrs. [for both A and B
2 m = 3.8 ´104 kg .
1 1 1
= + = 2 + 4 = 6 ; t = 1/6 hrs] 10. (d) If d is the distance of closedt approach given, then
t1/ 2 1/ 2 1/ 4 1/2
the angular mometum = mvd = 10–33 J s
é æ 1 ö5 ù æ 1ö 3 æ 1ö
8 1
N = ê N0 ç ÷ ú ç ÷ = N 0 ç ÷ E = mv2 = 1 MeV = 1.6 ´ 10 -13 J
è 2 ø ú è 2ø è 2ø 2
ëê û
Momentum,
5. (d) In time t, the total number of b-particles emitted = 5 ×
1010 t. As only 40% escape from the surface, so N = P = 2m n E = 2 ´ 1.6 ´ 10 -27 ´ 1.6 ´ 10 -13
0.40 × (5 × 1010)t = 2 × 1010t. The charge develops due
= 1.6 2 ´ 10-20 kg m s -1
to escape of each b- particle is 1.6 × 10–19 C, so total
charge escapes in time t, q = (2 ´ 1010 )t ´ (1.6 ´ 10 -19 )C . Distance of closest approach,
Thus we can write, 10 -33
d=
1 q 1.6 2 ´ 10-20
V = 4p Î0 R 1 100
= ´ 10-13 = fm = 0.44 fm
1 (2 ´ 1010 )t ´ (1.6 ´ 10-19 ) 1.6 2 1.6 2
or 2 =
4p Î0 10-2 11. (c) Suppose an initial radionuclide I decays to a final
\ t = 6.94 × 10–4s product F w ith a half – life T1/2.
6. (d) At any time, NI = N0e–lt
7. (d) The rate dm / dt can be calculate as ;
Number of product nuclei = NF = N0 – NI
dE dE dm DE dm
Power, P = = ´ = ´
dt dm dt Dm dt N F N0 - N I æ N0 ö
= =ç -I
dm Dm NI NI è N1 ÷ø
\ = P … (i)
dt DE N0 æ NF ö
We known that 26.2 MeV = 4.20 × 10–12 J of thermal energy = 1+ = 1 + 0.5 = 1.5
N1 çè N I ÷ø
is produced when four protons are consumed. This is
æ 3ö
DE = 4.20× 10–12 J for Dm = 4 × (1.67 × 10–27 kg). 1n ç ÷
T1/ 2 ln (1.5) 9 è 2ø
Substituting these values in equation (i), we have \ = 4.5 ´ 10 year
1n2 1n 2
dm Dm 4(1.67 ´ 10 -27 ) 12. (b) Let number of atoms in X = Nx
= P = ´ (3.90 ´ 10 26 )
dt DE 4.20 ´ 10-12 Number of atoms in Y = Ny
= 6.2 × 1011 kg /s By question
8. (a) The mass defect in the process
Nx 1
=
Dm = m( 238 Pu ) - m (234U ) - m (4 He) N y 15
= 238.04955 u – 234.04095 u – 4.002603 u 1 1
= 0.005997 u \ Part of Nx = (N x + N y ) = 4 (Nx + N y )
16 2
The kinetic energy of a- particle So, total 4 half lives are passed, so, age of rock is 4 ×
K = Dmc2 = 0.005997 × 931 MeV/u = 5.58 MeV.. 50 = 200 years
NUCLEI 711

1 15. (b) Initially P ® 4N0


13. (a) The value of X is Q ® N0
8
x0 x Half life TP = 1 min.
= = 0 Þ t = 3T = 3 × 20 = 60 years
8 23 TQ = 2 min.
Hence the estimated age of the rock is 60 years
Let after time t number of nuclei of P and Q are equal,
1
14. (b) N1 = N0 e–lt N1 = N0 that is
3
N0 1 4N0 N0 4N 1
= N 0 e -lt2 Þ = e-lt2 ...(i) t /1
= t/2
Þ t /1
= t /2
3 3 2 2 2 2
2 Þ 2t/1 = 4.2t/2 = 2(2 + t/2)
N2 = N0
3 t t t
2 2 Þ = 2 + Þ = 2 Þ t = 4 min
N 0 = N0e -lt1 Þ = e -lt1 ...(ii) 1 2 2
3 3
(4N 0 ) N0
Dividing equation (i) by equation (ii) NP = 4 /1
=
2 4
1 ln 2
= e -l (t2 -t1 ) Þ t 2 - t1 = = T1/2 = 50 days at t = 4 min.
2 l
N0 N 0
N0 = =
4 4
or population of R
æ N0 ö æ N0 ö 9N 0
ç 4N0 - ÷ + ç N0 - ÷ =
è 4 ø è 4 ø 2
EBD_7418
712 PHYSICS

29. SEMICONDUCTOR ELECTRONICS : MATERIALS,


DEVICES AND SIMPLE CIRCUITS
Exercise - 1 19. (b) Conductivity s = n i em e = 1017 ´ (1.6 ´10 -19 ) ´ 3800
1. (a) At absolute zero kelvin temperature, covalent bonds = 60.8 mho/cm
are very strong and there are no free electrons and hence 20. (b) Given : µe = 2.3 m2 V–1 s–1
semiconductor behaves as perfect insulator. µh = 0.01 m2 V–1 s–1, ne = 5 × 1012 / cm3
2. (d) = 5 × 1018/m3 nh = 8 × 1013/cm3 = 8 × 1019/m3.
3. (b) Resistivity of a semiconductor at room temp. is in Conductivity s = e[neµe + nhµh]
between 10–3 to 106 Wcm
= 1.6 × 10–19 [5 × 1018 × 2.3 + 8 × 1019 × 0.01]
4. (c) Electric conduction, in a semi conductors occurs due to
both electrons & holes. = 1.6 × 10–1 [11.5 + 0.8]
5. (c) By doping, the band gap reduce from 1eV to 0.3 to 0.7 = 1.6 × 10–1 × 12.3 = 1.968 W–1 m–1.
eV & electron can achieve this energy (0.3eV to 0.7eV) 21. (a) In the circuit the diode is in reverse bias and so no
at room temperature & reach in C.B (conduction band). current in the circuit.
6. (d) In extrinsic semi conductor the number of holes are not V 0.1 5
equal to number of electrons i.e., 22. (c) E = = - 6 = 10 V / m
d 10
nh ¹ ne 23. (b) Reverse resistance
DV 1
In p - type nh > ne = = = 2 ´ 106 W
D I 0.5 ´10-6
In n - type ne > nh 24. (d) Current in junction diode, I = I 0 (e eV / kT - 1)
7. (b) 8. (a) 9. (d) In forward biasing, V is positive ; In reverse bias V
10. (a) The average value of output direct current in a half is negative. Then Ir = I0
wave rectifier is = (average value of current over a cycle)/ I F I 0 (e eV / kT - 1)
2 = (2 I0/p)/2 = I0/p = = (e eV / kT - 1)
Ir I0
11. (c) Copper is a conductor so its resistance decreases on 25. (a) The current will flow through RL when the diode is
decreasing temperature as thermal agitation decreases forward biased.
whereas germanium is semiconductor therefore on 26. (c)
decreasing temperature resistance increases.
DV
12. (a) 27. (c) Forward bias resistance =
DI
13. (c) When small amount of antimony (pentavalent) is added (0.7 - 0.6)V 0.1
= = = 10 W.
to germanium crystal then crystal becomes n-type semi (15 - 5 ) mA 10 ´10-3
conductor. Therefore, there will be more free electrons 28. (c) A crystal structure is composed of a unit cell, a set of
than holes in the semiconductor. atoms arranged in a particular way; which is periodically
repeated in three dimensions on a lattice. The spacing
Eg
- between unit cells in various directions is called its
14. (d) For semiconductor, n = AT3 / 2e 2KT ; lattice parameters or constants. Increasing these lattice
constants will increase or widen the band-gap (Eg),
so n µ T 3 / 2 which means more energy would be required by
15. (a) electrons to reach the conduction band from the
16. (d) The average value of output direct current in a full wave valence band. Automatically Ec and Ev decreases.
rectifier = average value of current over a cycle = 2 I 0/p 29. (d) ni2 = nenh
17. (d) The input signal should be connected between two (1.5 × 1016)2 = ne (4.5 × 1022)
points of bridge rectifier such that in positive half wave Þ ne = 0.5 × 1010
of input signal, one p-n junction should be forward or ne = 5 × 109
biased and other should be reverse biased and in
Given nh = 4.5 × 1022
negative half wave of input signal, the reverse should
take place. It will be so when input is connected between Þ nh >> ne
B and D. \ Semiconductor is p-type and
18. (d) ne = 5 × 109 m–3.
SEMICONDUCTOR ELECTRONICS : MATERIALS, DEVICES AND SIMPLE CIRCUITS 713

46. (d) Voltage across zener diode is constant.


V 10
30. (c) V= o = V
p p 250W i i1kW
31. (b) In half wave rectifier, we get the output only in one 5V i–i1kW
half cycle of input a.c. therefore, the frequency of the
20v 1kW
ripple of the output is same as that of input a.c. i.e. 50 Hz
32. (d) In forward biasing, the diode conducts. For ideal 15V
15V
junction diode, the forward resistance is zero;
therefore, entire applied voltage occurs across external Current in 1kW resistor,
resistance R i.e., there occur no potential drop, but 15volt
potential across R is V in forward biased. (i)1kW = = 15 mA
33. (d) Positive terminal is at lower potential (0V) and negative 1kW
terminal is at higher potential 5V. Current in 250W resistor,
34. (c) When no current flows at the junction plane, then (20 - 15)V 5V
(i)250W = =
contact potential of junction plane is equal to the 250W 250W
forward voltage applied = 0.045 V 20
= A = 20 mA
35. (a) In semiconductors, by increasing temperature, 1000
covalent bond breaks and conduction hole and
electrons increase.
\ (i) zener diode = (20 - 15) = 5mA.
36. (d) For a p-type semiconductor, the acceptor energy level, hc 6.62 ´ 10 -34 ´ 3 ´108
as shown in the diagram, is slightly above the top Ev 47. (b) l= = = 2.07 ´10 -5 m
-3 -19
E (60 ´ 10 ´1.6 ´10 )
of the volume band. With very small supply of energy
an electron from the valence band can jump to the 48. (d) In pure semiconductor electron-hole pair = 7 × 1015/
level EA and ionise acceptor negatively m3
DI p ninitial = nh + ne = 14 × 1015 after doping donor Impurity
37. (b) gm =
DVg 5 ´ 1028 ND
ND = 7 = 5 × 1021 and ne = = 2.5 × 1021
-4
or DIp = gm ´ DVg = 3 ´ 10 ´ [-3 - (-1)] 10 2
= –0.6 × 10–3 A = shortage of 0.6 × 10–3 A So, nfinal = nh + ne
38. (d) In forward biasing, the diode conducts. For ideal Þ nfinal » ne » 2.5 × 1021 (Q ne >> nh)
junction diode, the forward resistance is zero; n final - n initial
Factor =
therefore, entire applied voltage occurs across external n initial
resistance R i.e., there occur no potential drop, but 2.5 ´ 1021 - 14 ´1015 2.5 ´1021
potential across R is V in forward biased. = » = 1.8 × 105
14 ´1015 14 ´1015
39. (c)
V 20
40. (c) i = R = = 10 mA. mR mR
2 ´ 103 49. (b) An = =
rp + R (m / g m ) + R
41. (d) Here, ni = 1016 m–3, nh = 5 × 1022 m–3
42 ´ (50 ´103 )
As nenh = ni2 = = 29.57
42 /(2 ´10 -3 ) + 50 ´103
n i2 (1016 m-3 )2 1
\ ne = = = 2 ´109 m -3
n h 5 ´1022 m -3 50. (a) Conductivity, σ = = e(n eμ e + n h μ h )
ρ
42. (d) It is the circuit of full wave rectifier. ie, 2.13 = 1.6 × 10–19(0.38 + 0.18) ni
43. (a)
(Since in intrinsic semi-conductor, ne = nh= ni)
44. (c) In circuit A, both the diodes are in forward bias and in
parallel, so \ density of charge carriers, n i
R = 2W 2.13
= -19
= 2.37 ´ 1019 m -3
V 8 1.6 ´ 10 ´ 0.56
Current, i = = = 4 A.
R 2
51. (c) V' = V + IR = 0.5 + 0.1 × 20 = 2.5 V
In circuit B, only one diode is in forward bias,
8 0.5V 20W
so current, i= = 2A.
4 0.1A
45. (b) I = nA evd or I µ nvd

Ie n v n e Ie v h 7 4 7
\ = e e = ´ = ´ = V
I h n h v h or n h Ih v e 4 5 5
EBD_7418
714 PHYSICS

52. (a) Here, E = 9V; Vz = 6; RL = 1000W and Rs = 100W, hoe= output admittance.
Potential drop across series resistor 63. (a) IC = 5.488 mA, Ie = 5.6 mA
V = E – VZ = 9 – 6 = 3V
5.488 a
Current through series resistance RS is a= ,b= = 49
V 3 5. 6 1- a
I= = = 0.03 A 64. (c) IC = IE –IB = 90 – 1 = 89 m A
R 100
Current through load resistance RL is Ro 5000
VZ 6 65. (c) Voltage amplification A v = b = 60 ´ = 600
Ri 500
IL = = = 0.006 A
R L 1000
A V 2800
Current through Zener diode is 66. (c) Current gain, a = = = 0.93
A R 3000
IZ = I – IL = 0.03 – 0.006 = 0.024 amp.
RL 24
Power dissipated in Zener diode is 67. (b) Voltage gain, A v = b R = 0.6 ´ 3 = 4.8
i
PZ = VZ IZ = 6 × 0.024 = 0.144 Watt 68. (d) Current gain in common emitter mode
0.406 a 0.995 0.995
53. (a) h= = = = = 199.
R 1 - a 1 - 0.995 0.005
1– f
RL 69. (b) In the circuit emitter is common.
% hmax = 40.6% 70. (c) No. of electrons reaching the collector,
96
nC = ´ 1010 = 0.96 ´ 1010
æ Rf ö 100
çè if R = 1÷ø
L n ´e
Emitter current, IE = E
Vm 25 t
54. (a) Im = = = 24.75 mA nC ´ e
R f + R L (10 + 1000) Collector current, IC =
t
\ Current transfer ratio,
Im 24.75
Idc = = = 7.87 mA IC n C 0.96 ´1010
p 3.14 a= = = 0.96
=
Im 24.75 IE n E 1010
Irms = = = 12.37 mA Vo R 5 ´103 ´ 62
2 2 = o ´b = = 10 ´ 62 = 620
Pdc = Idc × RL = (7.87 × 10–3)2 × 103 = 61.9 mW
2 71. (b)
Vin R in 500
Pac = Irms 2(Rf + RL) = (12.37 × 10–3)2 × (10 + 1000) Vo = 620 × Vin= 620 × 0.01 = 6.2 V
= 154.54 mW \ Vo = 6.2 volt.
Rectifier efficiency 72. (c) Power gain = voltage gain × current gain
Pdc 61.9 V0 I0
h= × 100 = × 100 = 40.05% = VG × IG = ×
Pac 154.54 Vi Ii
55. (b) In half wave rectifier only half of the wave is rectified V02 Ri 100
= × = 50 ´ 50 ´ = 1250
56. (a) transistor is usually made of metal oxides with high Vi 2 R
0 200
temperature coefficient of resistivity. 73. (b) I ® ON II ® OFF
57. (d) The biasing of the transistor is done differently for In IInd state it is used as a amplifier it is active region.
different uses. The transistor works as an amplifier with DI
its emitter-base junction forward biased and the base- 74. (a) b = C or DIC = b DIB = 40 ´ 100 m A
DI B
collector junction reverse biased. 75. (a) The current gain
58. (d) DIC 10mA 10 ´ 103
59. (d) Holes move from base to emmitter. b= = = = 50
DIB 200mA 200
Ic 76. (c) Voltage gain,
60. (d) b= > 1 or Ic > Ib
Ib V0 R é R2 ù é 10 ù -3
b 2 êb ú ´ Vi = ê100 ´ ú ´ 1 ´ 10 = 1 V.
Vi = R1 or 0 = ë R1 û
V
61. (d) Zero; In common base amplifier circuit, input and output ë 1û
voltage are in the same phase. D IC
62. (d) In common emitter configuration current gain 77. (b) Current gain = when VCE is constant.
DI B
-h f e -50 2.5 ´ 10 -3
Ai = = = = 0.1´ 103 = 100
1 + h oe R L 1 + 25 ´ 10-6 ´ 1 ´ 103 = – 48.78 25 ´ 10 -6
[DIB = 125 µA – 100 µA = 25 µA
Where hfe= forward current ratio
SEMICONDUCTOR ELECTRONICS : MATERIALS, DEVICES AND SIMPLE CIRCUITS 715

DIC = 7.5 mA – 5 mA = 2.5 mA]


Vout I out Rout
78. (a) Voltage gain (AV) = V = I × R
in in in

2 ´ 10 –34 ´ 103
AV = ´ = 2 × 100 = 2000
40 ´ 10-6 100
R
79. (d) Voltage gain Av = b out
R in
R out
Þ G = 25 R ...(i)
in
iC
b b 25 \ iB =
Transconductance gm = Þ Rin = = b dc
R in gm 0.03
Putting this value of Rin in eqn. (i) 4mA
=
100
R
G = 25 out × 0.03 ...(ii) = 0.04 mA
25
R out From figure
\ G' = 20 × 0.02 ...(iii) iB RB + VBE = VCC
20
From eqs. (ii) and (iii) VCC - VBE (8 - 0.6)
\ RB = =
2 iB 0.04 ´ 10 -3
Voltage gain of new transistor G' = G
3 = 185 kW
80. (a) Given: amplification factor a = 0.96 85. (a)
load resistance, RL = 800 W 86. (b) Combination of NAND & NOR gates can produce OR,
input resistance, Ri = 192W AND & NOT gates
a 0.96 87. (d) Relation between A, B and C shows that C = AB
So, b = = Þ b = 24
1 - a 0.04 So NAND Gate
Voltage gain for common emitter configuration 88. (b)
RL 800 89. (d) The given truth table is for NAND gate.
Av = b. = 24 × = 100
Ri 192
A 0 1 1 0
Power gain for common emitter configuration B 0 0 1 1
90. (a)
Pv = bAv = 24 × 100 = 2400 C 1 1 1 1
Voltage gain for common base configuration OR gate
RL 800
Av = a, = 0.96 × =4 91. (d) (A) is a NAND gate so output is 1´ 1 = 1 = 0
RP 192
Power gain for common base configuration
Pv = Ava = 4 × 0.96 = 3.84 (B) is a NOR gate so output is 0 + 1 = 1 = 0
81. (a) Voltage gain
(C) is a NAND gate so output is 0 ´1 = 0 = 1
R 5 ´103
A v = b C = 60 ´ = 150 (D) is a XOR gate so output is 0 Å 0 = 0
RB 2 ´103
R
Power gain = b 2 C = 60 ´ 60 ´ 2.5 = 9000 0
RB
1
12 ´ 10–3 –6 1
Base current = IB = = 6 ´ 10 A
2 ´ 103
IC = bIB = 60 × 6 × 10–6 = 3.6 × 10–4A Following is NAND Gate Y = AB
Output = ICRL = 3.6 × 10–4 × 5 × 103W = 1.8V
82. (a) 92. (b) Output of upper AND gate = AB
83. (c) Output of lower AND gate = AB
84. (d) By the definition \ Output of OR gate, Y = A B + BA
iC
b dc = This is boolean expression for XOR gate.
iB
EBD_7418
716 PHYSICS

93. (d) A 98. (b)


99. (c) (W + X) . (W + Y) = W + (X . Y)
C 100. (a) The truth table of the output is as follows :

B A B A B A+B A+ B
The truth table for the above logic gate is : 1 1 0 0 0 1
0 0 1 1 1 0
A B C 0 1 1 0 1 0
1 1 1 1 0 0 1 1 0
1 0 1 1 1 0 0 0 1
0 1 1 0 1 1 0 1 0
0 0 0 So option (a) is the correct option.
This truth table follows the boolean algebra
C = A + B which is for OR gate Exercise - 2
1. (d) In semiconsuctor the density of change carriers
A Y1 (electron hole) are very small, so its resistance is high
94. (b)
B when the conductivity of a semiconductor increases
Y with increase in temperature, because the number
density of current carries increases then the speed of
free electron increase and relaxation time decreases
Y2 but effect of decrease in relaxation is much less than
increase in number density.
2. (b) When p–n junction is forward biased then the depleton
Y1 = A + B, Y2 = A . B
layer is compresses or decrease so it opposes the
potential junction resulting decrease in potential
Y = (A + B)gAB = AgA + AgB + BgA + BgB barrier junction when p–n junction is reverese biased,
it supports the potential barrier junction, resulting
= 0 + AgB + BgA + 0 = AgB + BgA increase in potential across the junction.
This expression is for XOR 3. (b) As the given circuit, p–side of p–n function D1 is
connected to lower voltage and n –side of D1 of higher
95. (c)
voltage.
A So, D1 is reverse biased.
A Y' In circuit A is at –10V and B is at 0 (zero) V. So B is
Y
B positive then A or
B The p–side of p–n junction D2 is at higher potential
and n–side of D2 is at lower potential.
Y ' = A + B. Y = A + B = A + B. So, D2 is forward biased.
Truth table of the given circuit is given by Hence, No current flows through the junction B to A
and vice–versa.
A B Y' Y 4. (d) As the given figure p–n junction conducts during
0 0 1 0 positive half cycyle only, then diode connected here
will work is positive half cycle. Potential difference
0 1 0 1
1 across C will be peak voltage when diode is in forword
1 0 0 1 bias then the peak voltage of the given AC voltage
1 1 0 1
= V0 = Vrms 2 = 220 2V
96. (a) The final boolean expression is,
5. (b) Atom of semiconductor are bounded by covalent
X = ( A . B ) = A + B = A + B Þ OR gate
bonds between the atoms of same or different type.
The concept of hole describes the lack of an electron
97. (d) Truth table of the gate is as follows : at a position where one could exist in an atom or
atomic lattice. If an electron is excited into a higher
A B C A + B Y = ( A + B ).C Y state, it leaves a hole in its old state. So, hole can be
0 0 0 0 0 1 defined as a vacancy created when an electron leaves
1 1 0 1 0 1 a covalent bond.
SEMICONDUCTOR ELECTRONICS : MATERIALS, DEVICES AND SIMPLE CIRCUITS 717

6. (c) When the diode will be in forward biased during 15. (c)
positive half cycle of input AC voltage, the resistance 16. (a) Due to strong electronegativity of carbon.
of p–n junction is low. The current in the circuit is 17. (a) In semiconductors the energy gap between conduction
maximum. So, a maximum potential difference will band and valence band is small (»1 eV). Due to
appear across resistance connected in series of circuit. temperature rise, electron in the valence band gain
So, potential across PN junction will be zero. When thermal energy and may jumpy across the small energy
the diode will be in reverse biase during negative half gap, (to the conduction band). Thus conductivity
cycle of AC voltage, the resistance of p–n juntion increases and hence resistance decreases.
becomes high which will be more than resistance in 18. (c) Given : Voltage across the collector V0 = 2 V; collector
series. So, there will be voltage across p–n junction resistance, Rc = 2 × 103 W; Base resistance RB = 1 ×
with negative cycle in output. 103 W; Input signal voltage, Vi = ?
7. (b) Let the potential difference between A and B is V, Given IC
here r1 = 5 kW and r2 = 5 kW are resistance in series
connection. RC Output = 2V
So, Input RB
VAB – 0.3 = [(r1 + r2) 103] × (0.2 × 10 – 3)] [Q V = ir]
V0 = 2V
(VAB – 0.3) = 10 × 10 × 0.2 × 10 = 2
3 –3
V0 = ICRC = 2
So, VAB = 2 + 0.3 = 2.3 V
2
8. (c) As the given figure the output of C,D and E are : Þ IC = 3
= 10-3 A
2 ´10
C = A.B and D = A.B IC
Current gain a = = 100
E = C + D = (A.B) + ( A.B ) IB
So, the truth table of given arrangement of gates can IC 10-3
be written as : Þ IB = = = 10-5 A
100 100
Vi = RBIB Þ Vi = 1 × 103 × 10–5
A B A C = A.B D = A.B E = ( C + D)
Þ Vi = 10–2 V Þ Vi = 10 mV
0 0 1 0 0 0
0 1 1 0 1 1 19. (b) Since diode is in forward bias, so the value of current
1 0 0 0 0 0 DV 4 - ( -6 ) 10
flowing through AB i = = = 3 = 10–
1 1 0 1 0 1 R 1´103 10
2A
9. (a) The given graph represents V-I characteristics of solar 20. (a) Given: amplification factor a = 0.96
cell.
10. (d) The barrier potential of a p-n junction depends on load resistance, RL = 800 W
amount of doping, type of semiconductor material and input resistance, Ri = 192W
temperature.
11. (a) a 0.96
So, b= = Þ b = 24
1 - a 0.04
12. (b) A
Y Voltage gain for common emitter configuration
B X
RL 800
X = AB Av = b. = 24 × = 100
Ri 192
\ Y = X = AB Power gain for common emitter configuration
Y = AB by Demorgan theorem Pv = bAv = 24 × 100 = 2400
\ This diagram performs the function of AND gate. Voltage gain for common base configuration
13. (a) P n RL 800
Av = a, = 0.96 × =4
For forward bias, p-side must be at higher potential RP 192
than n-side. DV = (+ )Ve Power gain for common base configuration
V (3.5 - 0.5) 3 Pv = Ava = 4 × 0.96 = 3.84
14. (d) Current I = = A = = 30mA
R 100 100 21. (d)
[ Q Barrier potential VB = 0.5V]
EBD_7418
718 PHYSICS

22. (a) Input impedance of common emitter configuration. 31. (c)


DVBE 32. (d) During the operation, either of D1 and D2 be in forward
= bias. Also R1 and R2 are different, so output across R
DiB V
CE = constant will have different peaks.
where DVBE = voltage across base and emitter (base A
emitter region is forward biased) 33. (c) A A× B
B
DiB = base current which is order of few microampere. B
A Y
23. (c) Graph (a) is for a simple diode.
Graph (b) is showing the V Break down used for zener A×B
diode. B
Graph (c) is for solar cell which shows cut-off voltage Y = (A × B + A × B)
and open circuit current. 34. (c) From question, VBE = = 0, Vi = 20 V
Graph (d) shows the variation of resistance h and
VCE = 0
hence current with intensity of light.
Vb = 0 (earthed)
r P
24. (a) IB = ?, IC = ?, b = ? 20 V
(20 – 0)
IC = –3
= 5 × 10 = 5 mA IC RC = 4 k W
4 ´ 103
Vi = VBE + IBRB
RB
T T or, Vi = 0 + IBRB Vi Vb
Metal (for limited Semiconductor Ib 500 kW
–Eg Þ 20 = IB × 500 × 10 3
range of temperature) K T B

20
25. (a) In case of an 'OR' gate the input is zero when all inputs Þ IB = = 40 mA
are zero. If any one input is ' 1', then the output is '1'. 500 ´ 103
IC 25 ´ 10 –3
Y1 Y2 Y b= = = 125
A Ib 40 ´ 10–6
26. (b) B
35. (c) On heating, number of electron-hole pairs increases,
y1 = A + B so overall resistance of diode will change.
Hence forward biasing and reversed biasing both are
y2 = y1 + y1 = y1 = A + B
changed.
y = y 2 = A + B i.e. NOR gate 36. (a) In ‘n’ region; number of e– is due to As:
27. (b) Given, current gain b = 100, Rc = 3kW, Rb = 2kW ne = ND = 1 ´ 10–6 ´ 5 ´ 1028 atoms/m3
ne = 5 ´ 1022/m3
Rc æ3ö
Voltage gain (Av) = b = 100 ç ÷ = 150 The minority carriers (hole) is
Rb è2ø
Power gain = Av b = 150 (100) = 15000 n2 (1.5 ´ 1016 )2 2.25 ´ 1032
nh = i = =
V2
ne 5 ´ 1022 5 ´ 1022
V1
28. (d) nh = 0.45 ´ 10/m3
In forward bias, V1 > V2 i.e., in figure (d) p-type semi- Similarly, when Boron is implanted a ‘p’ type is created
conductor is at higher potential w.r.t. n-type semicon- with holes
ductor. nh = NA = 200 ´ 10–6 ´ 5 ´ 1028 = 1 ´ 1025/m3
29. (b) Here D1 is in forward bias and D2 is in reverse bias so, This is far greater than e– that existed in ‘n’ type wafer
D1 will conduct and D2 will not conduct. Thus, no on which Boron was diffused.
current will flow through DC. Therefore, minority carriers in created ‘p’ region
V 5 1 n 2 2.25 ´ 1032
I= = = A ne = i = = 2.25 ´ 107/m3
R 10 2 nh 1 ´ 1025
30. (a) 37. (c) Peak value of rectified output voltage
A = peak value of input voltage – barrier voltage
Y2 = A.AB = 2 – 0.7 = 1.3 V.
Y = A.AB B.AB 38. (c) Clearly from fig. given in question, Silicon diode is
Y1 = AB in forward bias.
B \ Potential barrier across diode
Y3 = B.AB DV = 0.7 volts
By expanding this Boolen expression V - DV 3 - 0.7 2.3
Current, I = = = = 11.5mA
R 200 200
Y = A.B + B.A
SEMICONDUCTOR ELECTRONICS : MATERIALS, DEVICES AND SIMPLE CIRCUITS 719

RL 24 Þ 12.5 = IZ + 5
39. (b) Voltage gain, A v = b R = 0.6 ´ 3 = 4.8 Þ IZ = 12.5 – 5 = 7.5 mA
i
10. (c) Voltage gain in common-emitter configuration is given
by
Exercise - 3 R 3
L 6 ´ 10
1. (b) The electric field retarded the electron. The retarda- Av = b R = 50 ´ = 300
in 1 ´ 103
tion 11. (a) When positive terminal connected to A then diode
Ee
a = . 2
m D1 is forward biased, current, I = = 0.4 A
5
When positive terminal connected to B then diode
2
D2 is forward biased, current, I = = 0.2 A
10
12. (a) Here, diodes D1 and D2 are forward biased and D3 is
The speed of the electron on entering p-side reverse biased.
2 2 eE Therefore current through R3
v2 = u - 2as = u - 2 .d
m
V 6 1
2 ´ 1.6 ´ 10 -19 ´ 3 ´ 105 i= = = A = 50 mA
5 2
= (5 ´ 10 ) - ´ (1 ´ 10-6 ) R ' 120 20
9.1 ´ 10-31 æ 0.7 ö
= 3.8 × 105 m/s 13. (b) VB = I (R D + R) Þ 5 = 1 ´ 10-3 ç + R÷
è 10-3 ø
2. (a) Given eev / kT = 100 Þ R = 4.3 kW
ev
or = ln 100
kT 14. (d) As each phosphorus atom contributes one electron
kT ln100 1.38 ´ 10-23 ´ 300 ´ ln100 and we want total number density of conduction elec-
V= = = 0.12V trons to be 106 n0, so the number density of phospho-
e 1.6 ´ 1019
3. (b) The output r.m.s. voltage in a half wave rectifier circuit rus atoms np must be given by
is Vrms = V0 / 2 = 200 / 2 = 100 V 106n0 = n0 + np
4. (a) The conductivity of semiconductor Then n p = 106 n0 - n0 ; 106 n0
s = e (heµe + hhµh) = 106 × 1016 = 1022 m–3
= 1.6 × 10–19(5 × 1018 ×2 + 5 × 1019 × 0.01) This gives us an ideal that we must add 1022 atoms of
phosphorus per cubic metre.
= 1.6 × 1.05 = 1.68
5. (d) The number of silicon atoms per unit volume is given
6. (a) by
rN A (2330) ´ (6.02 ´ 10 23 )
7. (b) X = 1 = A × B × (BC) nSi = =
M si 0.0281
Therefore, A = 1, B = 1, C = 0
= 5 × 1028 m–3.
3
DIC 10mA 10 ´ 10 The fraction we want
8. (a) The current gain b = = = = 50
DIB 200mA 200 np 10 22 1
= =
9. (d) Here, R = 4 kW = 4 × 103 W nSi 5 ´ 1028 5 ´ 106
Vi = 60 V 15. (a) By the definition
Zener voltage Vz = 10 V iC
RL = 2 kW = 2 × 103 W bdc = iB
VZ 10
Load current, IL = = = 5 mA iC
RL 2 ´ 103 \ iB = b dc
Vi - VZ 4mA
Current through R, I = =
R 100
60 - 10 50 = 0.04 mA
= = = 125mA RLiC + VCE = VCC
4 ´ 10 3
4 ´ 10 3 Also
Fom circuit diagram, VCC - VCE 8-4
\ RL = = = 1kW
I = IZ + IL iC 4 ´10-3
EBD_7418
720 PHYSICS

30. COMMUNICATION SYSTEMS

Exercise - 1 resistance in the flow of induced current due to which


the waves are attenuated.
1. (b) Exchange of information is called communication 16. (d) On superimposition of two waves the audible
2. (d) frequency wave is the modulating wave and radio-
3. (b) The message from the information source may not be wave is the carrier wave, thus the resultant wave
obtained is known as modulated wave as it is obtained
in electrical form so to convert these information i.e. in
by the process of modulation.
form of sound to electrical form a transducer like a
17. (b)
microphone is used.
18. (a) wc + wm
4. (b) Channel indicate frequency range at which different
= 2.5 × 106 + 0.0005 × 106Hz
R.F. signals all transmitted.
wc – wm
5. (d) Buffer amplifier isolate master oscillator from the
= 2.5 × 106 – 0.0005 × 106 Hz
influence of modulation done at a later stage.
= 2.4995 × 106 Hz
6. (d) The RF-signal coming from the transmitter needs to
19. (c) Carrier + signal ® modulation.
be demodulated by a detector inorder to remove the
20. (b) 21. (d)
carrier frequency and receive back the low-frequency
22. (d) When ma > 1 then carrier is said to be over modulated.
original signal.
23. (d) The maximum line-of-sight distance dM is given by
7. (a)
8. (a) Since the attenuation of ground waves increases with d M = 2Rh T + 2Rh R
increase in frequencies so only low frequency- radio 24. (c)
waves uses this mode of propagation for short 25. (d) For good demodulation,
distances.
9. (d) TV-signal ( 30 MHz -300 MHz) are not reflected by 1 1
<< RC or, RC >>
ionosphere. Therefore, sky-wave propagation is not f f
possible and similarly ground & space wave is also
26. (c)
not feasible hence they are transmitted to artificial
27. (d) Since maximum part of the power of modulated wave
satellite from where they are transmitted back to the
is contained with the carrier wave which does not
earth.
transmit any desired information, hence to avoid
10. (a)
wastage of power to suppress carrier balanced
11. (a) space wave propagation takes place in such a way
modulator is used.
that the radio waves transmitted at an angle from
earth's surface gets reflected by the troposphere and ( Df )actual
28. (c) m= ´100 Þ m ¥ ( Df )actual
then reaches the receiving antenna since the height ( Df )max
of troposphere is quite small, long-range propagation
by this mode is not possible. i.e. if frequency deviation is doubled then modulation
is doubled.
12. (b) Critical frequency fc = 9 N m 29. (c) TV signal comprises of video and audio signals.
where N m represents election density of layers Video signal is AM and sound signal is FM.
30. (d) When transmission path is long more repeater stations
Q fc µ N m are needed at intermediate points as repeater receives
signal, remove the noise, amplify it and retransmit it
Þ fc is different for different layers.
along the channel.
13. (a) Encoding modulation of signal i.e. to be transmitted is
31. (c) The limitter removes from the carrier all amplitude
done with carrier frequency at transmitting end to avoid
variations which may caused by changes in the
interference with other signals that are also transmission path, by man-made static or natural static.
transmitted. This suppression of amplitude variation is necessary
14. (d) because FM-receives, a vary large improvement in S/
15. (d) During ground wave propagation of radio waves, a N results from this.
charge induced on the earth's surface which takes the 32. (c)
form of current as the wave propagate. The earth offers
COMMUNICATION SYSTEMS 721

1 40. (b) The modulated voltage comprises of


33. (d) Phase velocity of e.m.wave in free space c =
m 0 Î0 (i) Carrier wave of frequency wc

Phase velocity of e.m.wave in ionised medium (ii) Lower side frequency ( wc - wm ) wave
1 (iii) Upper side frequency ( wc + wm ) wave
vp =
æ Ne2 ö Thus in an AM wave information is contained in lower
m 0 e0 ç 1 - ÷÷
ç e mw2
è 0 ø ( wc - wm ) and upper ( wc + wm ) side frequencies.
\ vp >c but vg < c in ionised medium. 41. (c)
34. (a) Radio wave enters from an un-ionised medium to an 42. (a) In statellite communication, the frequency used is
ionised medium, the wave incident on the boundary more than 40 MHz. The uplink and downlink
of the medium deviates from its straight path i.e. will frequencies are different to avoid distortion of
bend away from normal because ionised medium signal and the orbit of geostationary satellite lies in
behaves as a rarer medium w.r.t. to unionised medium.
the equatorial plane at an inclination of 0°.
35. (d) Following are the problems which are faced while 43. (c)
transmitting audio signals directly, 44. (a) (A) ® (2); (B) ® (1); (C) ® (4); (D) ® (3)
(i) These signals are relatively of short range. 45. (b) The frequency of AM channel is 1020 kHz whereas for
(ii) If every body started transmitting these low the FM it is 89.5 MHz (given). For higher frequencies
frequency signals directly, mutual interference will (MHz), space wave communication is needed. Very
render all of them ineffective. tall towers are used as antennas.
(iii) Size of antenna required for their efficient radiation 46. (b) In frequecy modulation the frequency of the
would be larger, i.e., about 75 km. modulated wave is the linear function of the amplitude
36. (c) Maximum range of space wave propagation of the modulating wave.
47. (a)
4
d= ´ 1.23 é H t + H r ù 48. (c) A transmitter processes the incoming message
3 ë û
signal, so as make it suitable for transmission
Þ d ¥ Ht through a channel and subseqeuent reception.

d ¥ Hr Pt 12 12
49. (c) Pc = = = = 9.6 kW
1+
m a2
1+
( 0.5) 2 1.25
\ d increases if H t and H r i.e. height of transmitting
2 2
and receiving antenna increases.
æ m2 ö
actual frequency deviation 50. (b) Pc = Pt ç1 + ÷
37. (b) m= ´ 100% ç 2 ÷ø
max.allowed frequency deviation è

(Df )actual æ m2 ö
= ´100% Þ 11.8 = 10 ç1 + ÷
( Df )max ç 2 ÷
è ø
if (Df) actual = (Df ) max Þ m = 0.6 Þ % modulation = 60%.
m = 100 %
51. (c) FM (I.F)=11.5MHz
38. (d) Pre-emphasis of higher frequency component is
required in FM-system because high frequency terms F(image) = fs+2 fIF = 98.6 + 2 ´11.5 = 121.6 MHz
of modulating signal have small amplitude and
therefore small power relative to those of low
é m 2ù é ( 0.8 ) 2 ù
frequency term. 52. (a) P = Pc ê1 + a ú = 1 ê1 + ú = 1.32kW.
In the reproduced program at the o/p, these high ë 2 û ë 2 û
frequency terms have poor S/N ratio and at time noise
may completely mask the signal at these high 53. (b) For given tran smission ban d 88-108 MHz
frequencies, so it is necessary to provide pre-emphasis ( Df )max = 75 kHz , ( Df )actual = 18.75 kHz
of high frequencies.

39. (c)
E max – E min
= m a -Depth of modulation. ( Df )actual 18.75
E max + E min \ % modulation m = ´100 = =25%
( Df )max 75
EBD_7418
722 PHYSICS

E c2 E max - E min 90 - 30 60 6
54. (b) Pc = = 10 kW 61. (b) ma = = = = = 0.5
2 E max + E min 90 + 30 120 12

ma = m12 + m 22 = 0.30 2 + 0.40 2 = 0.50 62. (b) 100% modulation Þ ma = 1

æ m 2 ö æ ( 0.5 )2 ö useful power ma 2 1 1


\ Pt = Pc ç1 + a ÷ = 10 ç 1 + ÷ = 11.25 kW = = =
ç 2 ÷ ç 2 ÷ total power radiated 2 + m 2 2 + 1 3
è ø a
è ø
1
55. (c) Carrier Swing = 2 ´ Df Þ Useful power = ( total power radiated )
3
= 2 ´ 105.03 - 105 = 2 ´ 0.03 MHz
63. (c) The range covered is not proportional to the height
= 0.06 MHz = 60 kHz of the TV transmission tower. The range depends
56. (a) For transmitting a signal, we need an antenna or an directly on square root of the height of the antenna
aerial. This antenna should have a size comparable i.e.
to the wavelength of the signal so that the antenna Sµ h .
properly senses the time variation of the signal. For Let the height of the TV transmission tower be h
an electromagnetic wave frequency 20 kHz, the and h' which covers the range S and S' respectively.
wavelength is 15 km. Obviously such a long
antenna is not possible to cosntruct and operate. \ S= 2hR and S' = 2h ' R
Hence direct transmission of such baseband signals For S' = 2S i.e. 2h ' R = 2 2h ' R
is not practical. Therefore there is a need of 64. (c) 65. (d)
translating the information constained in our original 66. (d) If the output of information source is non-electrical
low frequency baseband signal into high or radio signal like a voice signal, a transducer converts it to
frequencies before trasnsmission.
electrical form before giving it as an input to the
Vmax - Vmin 10 - 4 6 transmitter.
57. (b) ma = = = = 0.43 67. (b) There is a need of translating the information
Vmax + Vmin 10 + 4 14
contained in our original low frequency baseband
æ m 2ö signal into high or radio frequencies before
58. (d) PT = PC ç1 + a ÷ transmission.
ç 2 ÷ø
è 68. (c)
æ m 2ö 3 ma 2 69. (c) modulutaion index (m)
\ 600 = 400 ç 1 + a ÷ Þ = 1 +
ç 2 ÷ø 2 2 A m M1 - M 2 20 - 4 16 2
è = = = = =
m 2
1 A c M1 + M 2 20 + 4 24 3
or a = Þ m a = 1
2 2 E max - E min 90 - 30 60 6
70. (b) ma = = = = = 0.5
é m 2ù V 2rms V 2c é ma 2 ù E max + E min 90 + 30 120 12
59. (b) Pt = Pc ê1 + a ú Þ = ê1 + ú 71. (c) Modulation index m is kept £ 1 to avoid distortion.
ë 2 û 2 2 ë 2 û
72. (a) The sky wave are the radio waves of frequency
é m 2ù between 2 MHz to 30 MHz.
V 2 rms = V 2 c ê1 + a ú 73. (d) The maximum line-of-sight distance between the
ë 2 û
transmitting and receiving antennas is
ma 2 (0.3)2 dM = 2 RhT + 2 RhR
Þ Vrms = Vc 1 + Þ Vrms = 100 1 +
2 2 where hT and hR are the heights of transmitting and
= 104.5 volts. receiving antennas respectively.
\ dM = 2 Rh + 2 Rh = 2 2 Rh = 8Rh
m
60. (c) Average side-band power Pav = a Pc 2 (Q hT = hR = h)
4
74. (a) The critical frequency of a sky wave for reflection from
Here ma = 0.5 a layer of atmosphere is given by fc = 9(Nmax)1/2
Pc = 10 Þ 10 × 106 = 9(Nmax)1/2
2
0.5 ´ 10 ´ 10 æ 10 ´ 106 ö
÷ ; 1.2 ´10 m
12 –3
\ Pav = = 6.25 Þ N max = ç
4 è 9 ø
COMMUNICATION SYSTEMS 723

75. (d) f c = 9 N m = 9 ´ 9 ´ 1010 85. (d) Total signal B.W= 12 ´ 5 = 60 kHz


11 guard band are required between 12 signal
= 2.7 ´ 106 Hz = 2.7 MHz
\ guard bandwidth = 11 ´ 1kHz = 11kHz
76. (b) The frequency of AM channel is 1020 kHz whereas for
\ total bandwidth = 60 + 11 = 71kHz
the FM it is 89.5 MHz (given). For higher frequencies
86. (a) SSB transmission to signal are possible at load
(MHz), space wave communication is needed. Very
wc + wm or wc – wm
tall towers are used as antennas.
77. (b) Area covered by T.V. signals Þ (1000 + 10) kHz or (100 – 10) kHz.
A = 2phR = 2 × 3.14 × 100 × 6.4 × 106 = 128p × 108 Þ 1010 kHz or 990 kHz
Þ A = 1.28p × 103 km2 B
78. (a) 87. (b) Modulation index =
A
79. (c) E max = ( 1+ ma) Ec = ( 1+ 0.6) × 10 = 16 V
Emin = ( 1– ma) Ec = ( 1– 0.6) × 10 = 4V. B = 25, A = 60
25
E – E min 10 – 4 6 Þ M.I. = = 0.416 Þ m% = 41.6%
80. (b) m a = max = = = 0.428 = 42.8% 60
E max + E min 10 + 4 14
88. (b) Ec = 100 V, ma = 0.4, R = 100 W,
1
81. (c) R max µ d µ P4 E 2 (100) 2
Pc = c = = 50 watt
R 2 (new range) D2 (new diameter) 2R 2 ´ 100
Þ =
R1 (original range) D1 (original diameter) æ ö
m 2 é (0.4) 2 ù
1 P = ç1 + a ÷ Pc = ê1 + ú ´ 50 = 54 watt
ç 2 ÷ êë 2 úû
P24 (new transmitted power) è ø
=
1
Frequency deviation 50
P14 (original transmitted power) 89. (a) Carrier Swing = = = 7.143
Modulating frequency 7
1

= ´
(
3D1 81 P1 4 )
= 3´ 3 = 9
10
1 90. (c) modulation index = = 0.5 = 50%
D1 20
4 91. (c)
82. (b) Frequency of radio waves for sky wave propagation 92. (c) λ = 3m, H =100m, h = 60m
is 2 MHZ to 30 MHZ. Path difference between direct wave and wave obtained
83. (b) Given : Resistance R = 100 kilo ohm ofter reflection from earth surface is
= 100 × 103 W
2π æ 2 Hh ö
Capacitance C = 250 picofarad ç ÷ + π = 2π
= 250 × 10–12F λ è d ø
t = RC = 100 × 103 × 250 × 10–12 sec æ 4 Hh ö 4 Hh
or ç ÷ =1 or d =
= 2.5 × 107 × 10–12 sec è λd ø λd
= 2.5 × 10–5 sec 4 ´100 ´ 60
Þd= = 8×103 or 8km
The higher frequency whcih can be detected with 3
tolerable distortion is 93. (b) Here : Velocity of electromagnetic waves in free space
1 1 and wavelength
f = = Hz
2pma RC 2p ´ 0.6 ´ 2.5 ´ 10 -5 v = 3 × 108 m/s and l = 150 m
The frequency of radio waves is given by
100 ´ 104 4
= Hz = ´ 10 4 Hz v 3 ´108
25 ´ 1.2p 1.2 p = = = 2 ´106 Hz = 2 MHz.
= 10.61 KHz l 150
94. (a) Area covered by T.V. signals
This condition is obtained by applying the condition
A = 2phR = 2 × 3.14 × 100 × 6.4 × 106 = 128p × 108
that rate of decay of capacitor voltage must be equal
or less than the rate of decay modulated singnal Þ A = 1.28p × 103 km2
voltage for proper detection of mdoulated signal. 95. (d) Since radio frequency waves can travel long distances
84. (a) Comparing (x AM)t = 100 [1 + 0.5 t] coswct for 0<t<1 because these waves are of wave length of the order
of 100 m and their energy content is quite large
with standard AM signal x AM = Ec [1+ma cos wmt] therefore e.m.wave of audible frequency are
cos wct superimposed with radio frequency waves.
We have modulating signal t and ma = 0.5.
EBD_7418
724 PHYSICS

96. (b) Frequency deviation, Df = 25 kHz 40 ´103 = 2 ´ 6.4 ´ 10 6 ´ h + 24 ´103


Frequency of modulating signal, fm = 5 kHz [(40 – 24)103 ]2
h= = 20 m.
Df 25 2 ´ 6.4 ´106
Modulation index, mf = = =5
fm 5
97. (a)
f 60 Exercise - 2
98. (c) MUF = c = = 175 MHz
cos q cos 70° 1. (b) As we know that the range of frequencies for
A 0.5V communication
99. (a) Modulation index, M = m = = 0.05
AC 10V Ground wave propagation – 530 kHz to 1710 kHz
The side band frequencies are Sky wave propagation – 1710 kHz to 40 MHz
VSB = VC ± Vm = 1 ± 0.010 MHz Space wave propagation – 54 MHz to 42 GHz
100. (b) So given frequency A, B and C are to be transmitted
101. (d) Carrier frequency > audio frequency by ground wave, sky wave and space wave.
102. (b) Let h be the height of a transmission tower. 2. (d) Given, length of the building (l) is = 500m, Height or
Population density, r = 1000 km–2 = 1000 × 10–6 m–2 length of antenna from the ground height of building
= 10–3 m2 + hight of antenna
Now, d = 2Rh (where R is radius of the earth) = (500 + 100) = 600 m
Population covered = rpd2 = rp.2hR
22 we know that, wavelength of the wave which can be
\ 20 × 105 = 10–3 × ´ 2 ´ h ´ 6400 ´ 103 transmitted by
7
\ h = 50m l = 4l = 4 × 600 = 2400 m
R 68 3. (b) As given, that power of signal transmitted is given Pi
103. (d) h= = = 0.872
R + R L 68 + 10 = 1kW = 1000 W
Rate of attenuation of signal = –2dB/km
Þ Power gain = h G
Length of total path = 5 km
Power gain 16
Þ directive gain G = = = 18.35 So, gain in dB = 5 × (–2)dB = –10 dB
h 0.872
1 1 æ P0 ö
104. (b) v= = = 1592 kHz Also, gain in dB = 10 log ç P ÷ ...(i)
2p LC 2 ´ 3.14 10 ´ 10 –6 ´ 1 ´ 10 –9 è iø
105. (d) Frequency of EM wave u = 830 KHz where P0 is the power of the received signal
= 830 × 103 Hz. Putting the dB values in Eq. (i),
Magnetic field, B = 4.82 × 10–11 T æP ö æPö
-10 = 10 log ç o ÷ Þ 10 log ç i ÷ = 10
c è Pi ø è P0 ø
As we know, frequency, u =
l
P æPö
c 3 ´ 108 log i = 1 Þ loge ç i ÷ = loge 10 (Q log e 10 = 1)
or l = = l ; 360 m P0 è P0 ø
v 830 ´ 103 taking antilog
And, E = BC = 4.82 × 10–11 × 3 × 108 = 0.014 N/C Pi
nhc So, = 10
106. (b) Power = P0
l 1000W
(where, n = no. of photons per second) P0
= 10 (Q Pi = 1000W )
10 ´ 103 ´ 500
Þ n= ; 1031 Þ P0 = 100 W
6.6 ´ 10 -34 ´ 3 ´ 108 4. (a) Given, frequency of carrier signal is
107. (d) fc = 9 N m = 9 ´ 9 ´1010
wc = 1MHz
= 2.7 × 106 Hz = 2.7 MHz frequency of speech signal Wm = 3kHz
108. (a) = 3 × 10–3 MHz
109. (b) Core of acceptance angle q = sin –1 n12 – n22 wm = 0.003 MHz
110. (b) d = 2 Rhr + 2 RhR So frequency of side bands are :
= (wc ± wm) = (1 ± 0.003)
\ 40 ´1000 = 2 ´ 6.4 ´ 106 ´ h + 2 ´ 6.4 ´ 106 ´ 45 = 1.003 MHz and 0.997 MHz
COMMUNICATION SYSTEMS 725

5. (b) As we know that in amplitude modulation, the


Audio signal
frequency of modulated wave is equal to the
frequency of carrier wave.
Carrier wave
Here, the frequency of carrier wave is wc.
So the amplitude modulated wave also has frequency wc.
Amplitude modulated wave
6. (c) The device which follows square law can be used for
modulation purpose. Characteristics shown by (i) and
(iii) corresponds to linear devices as graphs are 12. (c) Modulated carrier wave contains frequency wc and
straight line. Characteristics shown by Graph (ii) wc ± w m
corresponds to the square law device. Some part of
graph (iv) also shows square law. 13. (c) If n = no. of channels
10% of 10 GHz = n × 5 KHz
Hence, (ii) and (iv) can be used for modulation.
10
7. (b) We know that or, ´ 10 ´ 109 = n ´ 5 ´ 103
The frequency of male voice smaller than that of a 100
female voice. Þ n = 2 × 105
So, the frequency of modulated signal is more than Exercise - 3
input signal which is possible with the poor bandwidth
selection of amplifiers. 1. (a) Here, fc = 1.5 MHz = 1500 kHz, fm = 10 kHz
This happens because bandwidth in amplitude \ Low side band frequency
modulation is equal to twice the frequency of = fc – fm = 1500 kHz – 10 kHz = 1490 kHz
modulated signal.
Upper side band frequency
8. (b) Communication system is the set–up used in the
transmission and reception of information travel from = fc + fm = 1500 kHz + 10 kHz = 1510 kHz
one place to another. c
9. (c) Let us consider a sinusoidal signal to be modulating 2. (d) Optical source frequency v =
l
is
m(t) = Am sin wmt 3 ´108 ms –1
where, Am = Amplitude of modulating signal Þ v= –9
= 2.5 ´1014 Hz
1200 ´10 m
wm = (Angular frequency) = wm =
Bandwidth of channel (2% of the source frequency)
2p Vm = f Vm ...(i)
Am = Ac + m (t) = 5 × 1012 Hz
Now Let us consider a sinusoidal carrier wave is Total bandwidth
Number of channels =
C (t) = Ac sin wct ...(ii) Bandwidth needed per channel
So, Cm (t) = Am sin wc(t)
5 ´ 1012 Hz
Cm (t) = (Ac + Am sin wmt) sinwct = = 106 = 1 million.
5 ´ 1016 Hz
é A ù æ Am ö 3. (b) Voltage observed on CRO of modulated signal (peak
= A c ê1 + m sin wm t ú sin wm t Q çè A = µ÷ø
ë A c û c
to peak)
= (carrier voltage + modulating signal voltage) ´ 2
Þ Cm (t) = (Ac + Ac × µ sinwmt) sinwmt ...(iii)
Now, we know that Ac × µ = K[wave constant] = (10 + 6 ) ´ 2 = 16 ´ 2 = 32 kV
and sin wmt = Vm (t) [wave velocity]
4. (b) For high-side tunning
Thus, Eq. (iii) becomes
Cm (t) = (Ac + K × Vm (t)) sin wct fLO = fm + fIF , PIF = 600 kHz = 0.6 MHz
Let us consider a change in phase angle by f then Þ f LOL = 5 + 0.6 = 5.6 MHz
sin wct ® sin (wct + f)
So, Cm (t) = (Ac + KVm (t) (sin wc + f) f LOU = 10 + 0.6 = 10.6 MHz
10. (a) Amplitude modulated wave consists of three 5. (b) Given : hR = 32 m
frequencies are wc + wm, w,wc – wm hT = 50 m
i.e. 2005 KHz, 2000KHz, 1995 KHz Maximum distance, dM = ?
11. (c) In amplitude modulation, the amplitude of the high
frequency carrier wave made to vary in proportional Applying, dM = 2Rh T + 2Rh R
to the amplitude of audio signal.
= 2 ´ 6.4 ´106 ´ 50 + 2 ´ 6.4 ´106 ´ 32 = 45.5 km
EBD_7418
726 PHYSICS

6. (c) l = 3m, H = 100 m, h = 60 m 10. (a) Let d is the maximum distance, upto it condetict the
Path difference between direct wave and wave obtained objects C
after reflection from earth surface is From DAOC
2 p æ 2Hh ö h
+ p = 2p OC 2 = AC 2 + AO 2
d
l çè d ÷ø B
(h + R )2 = d 2 + R 2
4Hh 4Hh A q R
or = 1 or d= Þ d 2 = (h + R )2 - R 2
ld l R
4 ´ 100 ´ 60 2 2 O
Þd= = 8 ´ 103 m or 8km d = ( h + R) - R
3
7. (b) h = 100 m, R = 6.4 × 106 m d = h2 + 2hR

d = 5002 + 2 ´ 6.4 ´ 10 6 = 80 km
d d 11. (b) The frequencies present in amplitude modulated
h
wave are :
Carrier frequency = wc
Upper side band frequency = wc + wm
R
Lower side band frequency = wc – wm.
12. (c) For x(t), BW = 2(Dw + w)
Average population density = 1000 km–2
Dw is deviation and w is the band width of modulating
= 10–3 m–2 signal.
Maximum range of TV transmission \ BW = 2(90 + 5) = 190
1 2
For x (t), BW = 2 × 190 = 380
d = 2hR = (2 ´100 ´ 6.4 ´106 ) 2
13. (b) Comparing the given equation with standard
\ Maximum area covered by T.V. transmission
modulated signal wave equation, m = Ac sin wc t +
= pd2 = p × 2hR = 2 × 3.14 × 100 × 6.4 × 106
mA c mA c
= 4.02 × 109 m2 cos (wc – ws) t – cos (wc + ws) t
2 2
\ the population covered by T.V. transmission Ac 2
m = 10 Þ m = (modulation index)
= (4.02 × 109) × 10–3 = 4.02 × 106 2 3
8. (a) Length of antenna = comparable to l Ac = 30
wc – ws = 200p
Power radiated by linear antenna inversely depends
wc + ws = 400p
on the square of wavelength and directly on the length
Þ fc = 150, fs = 50 Hz.
of the antenna. Hence,
Vm 5
2 14. (d) Modulation index (m) = = = 0.2
Power P = m æç ö÷
1 V0 25
è lø Given, frequency of carrier wave (fc) = 1.2 × 106 Hz
here µ = K = 1200 kHz.
9. (c) Modulation index Frequency of signal (f0) = 20 kHz.
Em A Side frequency bands = fc ± f0
ma = = =1
Ec A f1 = 1200 – 20 = 1180 kHz
f2 = 1200 + 20 = 1220 kHz
Equation of modulated signal [Cm(t)]
15. (c) Ratio of AM signal Bandwidths
= E(C) + maE(C) sin wmt
15200 - 200 15000
= A (1+ sin wCt) sin wmt = = = 6.
2700 - 200 2500
(As E(C) = A sinwCt)
M ck Test - 1
Time : 1 hrs Max. Marks -180

r A 8. If the magnetic dipole moment of an atom of diamagnetic


1. The electric field is given by E = 3 iˆ + Byjˆ + Cz 2kˆ . The SI
x material, paramagnetic material and ferromagnetic material
units of A, B and C are respectively: [here x, y, z are in m] are denoted by md, mp and mf respectively, then
(a) md = 0 and mp ¹ 0 (b) md ¹ 0 and mp = 0
Nm3
(a) , V/m2, N/m2-C (b) V-m2, V/m, N/m2-C (c) mp = 0 and mf ¹ 0 (d) md ¹ 0 and mf ¹ 0
C
(c) V/m2, V/m, N-C/m2 (d) V/m, N-m3/C, N-C/m 9. A car runs at a constant speed on a circular track of radius
100 m, taking 62.8 seconds in every circular loop. The
2. The acceleration versus time graph a (m/s)²
average velocity and average speed for each circular loop
for a particle moving along a
4 respectively, is
straight line is shown in the figure.
10 20 30 t (sec) (a) 0, 10 m/s (b) 10 m/s, 10 m/s
If the particle starts from rest at t =
0, then its speed at t = 30 sec will be –4 (c) 10 m/s, 0 (d) 0, 0
10. A mass is tied to a string and rotated in a vertical circle, the
(a) 20 m/sec (b) 0 m/sec minimum velocity of the body at the top is
(c) – 40 m/sec. (d) 40 m/sec. (a) gr (b) g/r
3. A body of mass 5 kg falls from a height of 20 metres on the 3/ 2
ground and it rebounds to a height of 0.2 m. If the loss in ægö
(c) ç ÷ (d) gr
potential energy is used up by the body, then what will be the èrø
temperature rise? 11. Power dissipated across the 8W resistor in the circuit shown
(specific heat of material = 0.09 cal gm–1 ºC–1) here is 2 watt. The power dissipated in watt across the 3W
resistor is 1W 3W
(a) 0ºC (b) 4ºC i1
(a) 1.0
8 11 (b) 0.5
i
(c) °C (d) °C
33 32
(c) 3.0
4. A monoatomic gas at 77°C is compressed adiabatically to 8W i2
(d) 2.0
125 th
of its original volume. The rise in temperature will be 12. The energy of hydrogen atom in the 5 orbit is E, then the
343 energy in the 7th orbit is
(a) 186 K (b) 235 K (c) 336 K (d) 586 K 27 49
5. A man projects a coin upwards from the gate of a uniformly (a) E (b) E
8 25
moving train. The path of coin for the man will be
25 5E
(a) parabolic (c) E (d)
49 7
(b) inclined straight line
13. Two identical particles move towards each other with
(c) vertical straight line velocity 2v and 5v respectively. The velocity of centre of
(d) horizontal straight line mass is
6. What is the disintegration constant of radon, if the number (a) 5v/2 (b) 9v/5 (c) 3v/2 (d) v/8
of its atoms diminishes by 18% in 24 h? 14. The mass number of He is 4 and that for sulphur is 32. The
(a) 2.1 × 10–3 s–1 (b) 2.1 × 10–4 s–1 radius of sulphur nuclei is larger than that of helium by
(c) 2.1 × 10–5 s–1 (d) 2.1 × 10–6 s–1 (a) 8 (b) 4
7. The refractive index of water measured by the relation (c) 2 (d) 8
real depth 15. According to Newton’s law of cooling, the rate of cooling
m= is found to have values of 1.34, 1.38,
apparent depth of a body is proportional to ( Dq)n , where Dq is the
1.32 and 1.36; the mean value of refractive index with difference of the temperature of the body and the
percentage error is surroundings, then n is equal to
(a) 1.35 ± 1.48 % (b) 1.35 ± 0 % (a) two (b) three (c) four (d) one
(c) 1.36 ± 6 % (d) 1.36 ± 0 %
EBD_7418
MT- 2 PHYSICS

16. A body having initial velocity of 10 m/s moving on a rough 25. When a tuning fork produces sound waves in air, which
surface comes to rest after moving 50 m. What is coefficient one of the following is same in the material of tuning fork as
of friction between the body and surface? (g = 10 m/s2) well as in air?
(a) 0.5 (b) 0.2 (a) Wavelength (b) Frequency
(c) 0.3 (d) 0.1 (c) Velocity (d) Amplitude
17. The separation between successive fringes in a double slit 26. The fermi energy for a substance is
arrangement is x. If the whole arrangement is dipped under (a) independent of T
water what will be the new fringe separation? [The (b) directly proportional to T
wavelenght of light being used is 5000 Å] (c) directly proportional to T
(a) 1.5 x (b) x (d) directly proportional to T2
(c) 0.75 x (d) 2 x 27. ABC is a triangular plate of uniform
18. A thin, metallic spherical shell contains a charge Q on it. A thickness. The sides are in the ratio
A
point charge q is placed at the cente of the shell and another shown in the figure. IAB, IBC and
charge q1 is placed outside it as shown in figure. All the ICA are the moments of inertia of the
three charges are positive. The force on the charge at the plate about AB, BC and CA as axes 5
3
centre is respectively. Which one of the
following relations is correct? 90°
Q B C
3
(a) I AB > I BC (b) I BC > I AB
q q
(c) I AB + I BC = I CA (d) ICA is maximum
(a) towards left (b) towards right 28. Consider the following statement:
(c) upward (d) zero When jumping from some height, you should bend your
19. A charge q is moving with a velocity v parallel to a magnetic knees as you come to rest, instead of keeping your legs
field B. Force on the charge due to magnetic field is stiff. Which of the following relations can be useful in
(a) q v B (b) q B/v explaining the statement? Where symbols have their usual
(c) zero (d) B v/q meanings.
r r
20. A bucket full of hot water is kept in a room and it cools from (a) D p1 = -D p2 (b) DE = D(PE + KE) = 0
75ºC to 70ºC in T1 minutes, from 70ºC to 65ºC in T2 minutes r r
and from 65ºC to 60ºC in T3 minutes. Then (c) FDt = mD v (d) Dx µ DF
(a) T1 = T2 = T3 (b) T1 < T2 < T3 29. The amplitude of magnetic field of an electromagnetic wave
(c) T1 > T2 > T3 (d) T1 < T2 > T3 is 2 × 10–7T. It's electric field amplitude if the wave is travelling
in free space is
21. In uniform circular motion, the velocity vector and
(a) 6 Vm–1 (b) 60 Vm–1
acceleration vector are
(c) 10/6 Vm–1 (d) None of these
(a) perpendicular to each other 30. The magnetic flux density B at a distance r from a long
(b) in same direction straight wire carrying a steady current varies with distance r as
(c) in opposite direction
(d) not related to each other
B B
22. The current in a coil of L = 40 mH is to be increased uniformly
from 1A to 11A in 4 milli sec. The induced e.m.f. will be (a) (b)
(a) 100 V (b) 0.4 V
(c) 440 V (d) 40 V r r
23. Two capacitors when connected in series have a capacitance
of 3 mF, and when connected in parallel have a capacitance B B
of 16 mF. Their individual capacities are
(a) 1 mF, 2 mF (b) 6 mF, 2 mF (c) (d)
(c) 12 mF, 4 mF (d) 3 mF, 16 mF
r r
24. The weight of a body will be the least at
31. A metal piece is heated upto Tº abs. The temperature of
(a) poles the surrounding is tº abs.The heat in the surrounding due
(b) equator to radiation is proportional to
(c) at height equal to R (a) (T – T)4 (b) T4 – t4
(d) centre of the earth (c) (T – t) 1/4 (d) T2 – t2
MOCK TEST 1 MT- 3

4 39. A ray of light is incident on the surface of separation of a


3
32. Given, aµg = , a m w = , if a convex lens of focal length medium with the velocity of light at an angle 45° and is
2 3
refracted in the medium at an angle 30°. Velocity of light in
10 cm is placed in water, then its focal length in water is
(a) equal to 40 cm (b) equal to 20 cm the medium will be (velocity of light in air = 3 ´ 10 8 m/s)
(c) equal to 10 cm (d) None of these
33. If an alternating current is flowing in a spring, then the (a) 3.8 ´ 108 m/s (b) 3.38 ´ 108 m/s
spring will be changing
(a) in a straight line (b) periodically (c) 2.12 ´ 108 m/s (d) 1.56 ´ 108 m/s
(c) elliptically (d) first (c) then (a) 40. The rain drops are in spherical shape due to
34. The unit vector along 2i – 3j + k is
(a) residual pressure (b) thrust on drop
2i - 3j + k 2i - 3 j + k
(a) (b) (c) surface tension (d) viscosity
14 5
41. A dip circle is so set that its needle moves freely in the
2i - 3 j + k magnetic meridian. In this position, the angle of dip is 40º.
(c) (d) None of these Now the dip circle is rotated so that the plane in which the
15
35. An iron rod of length 2m and cross-sectional area of 50 needle moves makes an angle of 30º with the magnetic
mm2 is stretched by 0.5 mm, when a mass of 250 kg is hung meridian. In this position, the needle will dip by an angle
from its lower end. Young’s modulus of iron rod is (a) 40º (b) 30º
(a) 19.6 ´ 10 20 N / m 2
18
(b) 19.6 ´10 N / m
2 (c) more than 40º (d) less than 40º
42. If the critical angle for total internal reflection from a medium
(c) 19.6 ´ 1010 N / m 2 (d) 19.6 ´1015 N / m 2 to vacuum is 30°. Then velocity of light in the medium is
36. A particle of mass 1 kg is moving in S.H.M. with an
amplitude 0.02 and a frequency of 60 Hz. The maximum (a) 1.5 ´108 m / s (b) 2 ´108 m / s
force acting on the particle is
(a) 144 p2 (b) 188 p2 (c) 3 ´108 m / s (d) 0.75 ´108 m / s
(c) 288 p 2 (d) None of these
37. According to Maxwell’s hypothesis, a changing electirc 43. An oscillator is nothing but an amplifier with
field gives rise to (a) positive feedback (b) large gain
(a) an e.m.f (b) magnetic field
(c) no feedback (d) negative feedback
(c) electric current (d) pressure gradient.
38. The load versus elongation graph for four wires has been 44. For an AM-system the total power of modulated signal is
shown in the figure. The thinest wire is 600 W and that of carrier is 400 W, the modulation index is
(a) 0.25 (b) 0.36
Load a (c) 0.54 (d) 1
b
45. The drift current in a p-n junction is
c
(a) from the n-side to the p-side
(b) from the p-side to the n-side
d
(c) from the n-side to the p-side if the junction is forward-
baised and in the opposite direction if it is reverse
O biased
Elongation
(d) from the p-side to the n-side if the junction is forward-
(a) a (b) b baised and in the opposite direction if it is reverse-
(c) c (d) d baised
EBD_7418
MT- 4 PHYSICS

M ck Test - 2
Time : 1 hrs Max. Marks -180

1. If power dissipated in the 9-W resistor in the circuit shown 9. The distance travelled by a body moving along a line in
is 36 watt, the potential difference across the 2-W resistor is time t is proportional to t3.
9W The acceleration-time (a, t) graph for the motion of the
(a) 4 volt body will be
6W
(b) 8 volt
a
a
(c) 10 volt
(a) (b)
(d) 2 volt V 2W
2. Error in the measurement of volume of a sphere is 9%. Then t t
error in the measurement of radius is
(a) 9% (b) 10% a a
(c) 3% (d) 8% (c) (d)
3. The spring constant from the adjoining combination of
t t
springs is
10. Minimum wavelength of radiation in balmar series of
(a) K hydrogen spectrum will be
2K
(b) 2 K (a) 3646 A° (b) 6536 A°
m
(c) 2241 A° (d) 8128 A°
(c) 4 K K K 11. In the given network of capacitors as shown in Fig. given
(d) 5 K/2 that C1 = C2 = C3 = 400 pF and C4 = C5 = C6 = 200 pF. The
effective capacitance of the circuit between X and Y is
4. A body of mass 2 kg is rotating on a circular path of radius
0.8 m with an angular velocity 44 rad/sec. If radius of path C1 C2 C3
X Y
becomes 1 m then value of angular velocity will be
C4
(a) 35.28 rad/sec (b) 14.08 rad/sec
(c) 28.16 rad/sec (d) 24.08 rad/sec C5
5. The motion of particle is described by the equation x = a + C6
bt2, where a = 15 cm and b = 3 cm/sec2. Its instant velocity
at time 3 sec will be (a) 810 pF (b) 205 pF (c) 600 pF (d) 410 pF
(a) 36 cm/sec (b) 9 cm/sec 12. According to Einstein’s photoelectric equation, the plot of
the kinetic energy of the emitted photoelectrons from a
(c) 4.5 cm/sec (d) 18 cm/sec metal vs the frequency of the incident radiation gives a
6. At S.T.P. 201.6 L of NH3 contains how many grams of it? straight line whose slope
(a) 250 gram (b) 153 gram (a) depends on the intensity of the radiation
(b) depends of the mature of the metal used
(c) 90 gram (d) 280 gram
(c) depends both on the intensity of the radiation and the
7. A coil of insulated wire is connected to a battery. If it is metal used
taken to galvanometer, its pointer is deflected, because (d) is the same for all metals and independent of the
(a) induced current is produced intensity of the radiation
(b) the coil acts like a magnet 13. The moment of inertia of a solid cylinder about geometrical
axis
(c) the number of turns in the coil of the galvanometer are
(a) MR2 (b) MR2/2
changed
(c) MR2/4 (d) 2 MR2/5
(d) None of these 14. In a nuclear fission, 0.1% mass is converted into energy.
8. The dimensions of mo ´ eo in M, L, T is given as The energy released by the fission of 1 kg mass will be
(a) [MoL–2 T2] (b) [M L T–2] (a) 9 × 1019 J (b) 9 × 1017 J
(c) 9 × 10 J16 (d) 9 × 1013 J
(c) [MT–1] (d) [ML2 T–1]
MOCK TEST 2 MT- 5

15. If the coefficient of cubical expansion is x times of the 24. The kinetic energy of a body becomes four times its inital
coefficient of superficial expansion, then value of x is value. The new momentum will be
(a) 3 (b) 2.5 (a) same as initial value
(c) 1.5 (d) 2 (b) twice the initial value
16. When a bus suddenly takes a turn, the passengers are (c) thrice the initial value
thrown outwards because of (d) four times the initial value
(a) speed of motion 25. An observer moves towards a stationary source of sound
(b) inertia of motion with a speed 1/5th ofthe speed of sound. The wavelength
(c) acceleration of motion and frequency of the source emitted any l and f
(d) None of these respectively. The apparent frequency and wavelength
17. An equilateral prism is placed on a horizontal surface. A recorded by the observer are respectively
ray PQ is incident onto it. For minimum deviation (a) 0.8f, 0.8l (b) 1.2f, 1.2l
(c) 1.2f, l (d) f, 1.2l
R 26. The photoelectric effect is based upon the law of
Q S conservation of
P
(a) momentum
(b) energy
(a) PQ is horizontal (c) angular momentum
(b) QR is horizontal (d) mass
(c) RS is horizontal 27. A choke is preferred to a resistance for limiting current in
(d) any one will be horizontal AC circuit because
18. A tin nucleus (atomic number Z = 50) has a radius of 6.6 × (a) choke is cheap
10–15 m. The potential on its surface will be (the charge on (b) there is no wastage of power
the proton = 1.6 × 10–19 C) (c) choke is compact in size
(a) 1.1 × 107 V (b) 2.1 × 107 V (d) choke is a good absorber of heat
(c) 3.1 × 10 V7 (d) 0.15 × 107 V 28. If a body of mass 3 kg is dropped from the top of a tower of
19. A relation between Faraday’s constant F, chemical height 25m. Then its kinetic energy after 3 sec is
equivalent E and electrochemical equivalent Z is (a) 557 J (b) 748 J
E (c) 1050 J (d) 1296 J
(a) F = EZ 2 (b) F =
Z2 29. A gymnast takes turns with her arms & legs stretched.
2 When she pulls her arms & legs in
E E
(c) F = (d) F = (a) the angular velocity decreases
Z Z
20. Two rods of the same length and areas of cross-section A 1 (b) the moment of inertia decreases
and A2 have their ends at the same temperature K1 and K2 (c) the angular velocity stays constant
are the thremal conductivities of the two rods. The rate of (d) the angular momentum increases
flow of heat is same in both rods if 30. A current of I ampere flows in a wire forming a circular arc
A 1 K1 A1 K2 of radius r metres subtending an angle q at the centre as
(a) = (b) =
A2 K 2 A 2 K1 shown. The magnetic field at the centre O in tesla is
m0 I q
(c) A1A2 = K1K2 (d) A1 K12 = A2 K22 (a)
4pr I
21. A stone is just released from the window of a train moving
m0 I q q
along a horizontal straight track. The stone will hit the (b)
ground following a 2 pr
m0 I q O
(a) straight line path (c)
(b) circular path 2r
(c) parabolic path m0 I q
(d)
(d) hyperbolic path 4r
22. Lenz’s law is a consequence of the law of conservation of 31. The number of degrees of freedom for each atom of a
monoatomic gas is
(a) charge (b) mass
(a) 3 (b) 5
(c) energy (d) momentum (c) 6 (d) 1
23. An equipotential surface is that surface 32. Ability of the eye to see objects at all distances is called
(a) on which each and every point has the same potential (a) binocular vision
(b) which has negative potential (b) myopia
(c) which has positive potential (c) hypermetropia
(d) which has zero potential (d) accommodation
EBD_7418
MT- 6 PHYSICS

33. The velocity of electromagnetic radiation in a medium of 40. Surface tension of a solution is 30×10–2 N/m. The radius of
permittivity e0 and permeability m0 is given by the soap bubble is 5 cm. The surface energy of soap bubble
Î0 is
(a) (b) m0 Î0 2
m0 (a) 1.8 ´ 101 J (b) 1.8 ´ 10 J
1 m0 (c) 1.0 ´ 10 -1 J (d) 1.8 ´ 10-2 J
(c) (d) 41. A cylindrical resonance tube, open at both ends, has a
m0 Î0 Î0
34. The numerical ratio of displacement to distance is fundamental frequency f in air. If half of the length is dipped
(a) always less than one vertically in water, the fundamental frequency of the air
(b) always equal to one column will be
(c) always more than one 3f
(a) (b) 2f
(d) equal to or less than one 2
35. The change in the value of ‘g’ at a height ‘h’ above the f
(c) f (d)
surface of the earth is the same as at a depth ‘d’ below the 2
surface of the earth. When both ‘d’ and ‘h’ are much smaller 42. Which of the following waves have the maximum
than the radius of earth, then which one of the following is wavelength ?
correct? (a) Infrared rays (b) UV rays
h (c) Radio waves (d) X – rays
3h
(a) d = (b) d = 43. In an p-n-p transistor working as a common base amplifier
2 2
(c) d = h (d) d =2 h current gain is 0.96 and emitter current is 7.2 mA. The base
36. Pressure exerted by a perfect gas is equal to current is
(a) mean kinetic energy per unit volume (a) 0.2 mA (b) 0.29 mA
(b) half of mean kinetic energy per unti volume (c) 0.35 mA (d) 0.4 mA
(c) two third of mean kinetic energy per unit volume 44. The forward biasd diode is
(d) one third of mean kinetic energy per unit volume –2V +2V
(a)
37. Transformer is based upon the principle of
(a) self induction (b) mutual induction
(b) –4V –3V
(c) eddy current (d) None of these
38. A spherical ball of iron of radius 2 mm is falling through a
column of glycerine. If densities of glycerine and iron are (c) 3V 5V
respectively 1.3 × 103 kg/m3 and 8 × 103 kg/m3, h for glycerine
= 0.83 Nm–2 sec, then the terminal velocity is (d) 0V –2V
(a) 0.7 m/s (b) 0.07 m/s
(c) 0.007 m/s (d) 0.0007 m/s 45. In V-I characteristic of a p-n junction reverse biasing results
39. The time taken by light to pass through 4 mm thick glass in
slab of refractive index 1.5 will be (velocity of light in air = (a) leakage current
3 × 108 m/s) (b) the current barrier across junction increases
(a) 8 × 10–11 s (b) 2 × 10–11 s (c) no flow of current
(c) 8 × 10 s –8 (d) 8 × 10–8 s (d) large current
M ck Test - 3
Time : 1 hrs Max. Marks -180

1. When the speed of a car is v, the minimum distance over 8. If Q, E and W denote respectively the heat added, change
which it can be stopped is s. If the speed becomes nv, what in internal energy and the work done in a closed cyclic
will be the minimum distance over which it can be stopped process, then
during same retardation? (a) W = 0 (b) Q = W = 0
(a) s/n (b) ns (c) s/n 2 (d) n 2 s (c) E = 0 (d) Q = 0
2. An eye specialist prescribes spectacles having combination
9. The moment of inertia of annular disc of inner and outer
of convex lens of focal length 40 cm in contact with a
radii R1 & R2 about diameter in its plane is
concave lens of focal length 25 cm. The power of this lens
combination in diopter is M (R 12 + R 22 )
(a) + 1.5 (b) – 1.5 (a)
4
(c) + 6.67 (d) – 6.67
3M (R1 + R 2 )2
3. A ball is released from the top of a tower of height h meters. (b)
It takes T seconds to reach the ground. What is the position 2
T 5
of the ball at second? (c) M (R1 + R 2 ) 2
3 4
8h 7M (R1 + R 2 ) 2
(a) meters from the ground (d)
9 4
7h 10. In Fig. find the total magnification after two successive
(b) meters from the ground
9 reflections first on M1 and on M2. f = 10 cm f = 20 cm
h (a) + 1
(c) meters from the ground
9
17 h (b) –2
(d) meters from the ground
18 (c) +2 M2 M1
4. A boat is sent across a river with a velocity of 8 km h –1. If
(d) –1 10 cm 30 cm
the resultant velocity of boat is10 km h–1 , then the velocity
of the river is 11. How many electrons make up a charge of 20 µC?
(a) 12.8 km h–1 (b) 6 km h–1 (a) 1.25 × 1014 (b) 2.23 × 1014
(c) 8 km h–1 (d) 10 km h–1 (c) 3.25 × 1014 (d) 5.25 × 1014
5. Energy required to move a body of mass m from an orbit of 12. Two oscillators are started simultaneously in same phase.
radius 2R to 3R is
After 50 oscillations of one, they get out of phase by p, that
(a) GMm/12R2 (b) GMm/3R2 is half oscillation. The percentage difference of frequencies
(c) GMm/8R (d) GMm/6R of the two oscillators is nearest to
6. A force of – Fk̂ acts on O, the origin of the coordinate (a) 2% (b) 1% (c) 0.5% (d) 0.25%
system. The torque about the point (1, –1) is Z 13. When sound waves travel from air to water, which one of
the following remains constant?
(a) F(î - ˆj) (b) - F(î + ĵ) (a) Time period (b) Frequency
O Y
(c) Velocity (d) Wavelength
(c) F(î + ĵ) (d) - F(î - ˆj) X 14. An ideal gas A and a real gas B have their volumes increased
7. If I is the moment of inertia and w is the angular velocity,, from V to 2V under isothermal conditions. The increase in
what is the dimensional formula of rotational kinetic energy internal energy
1 2?
Iw (a) will be same in both A and B
2
(b) will be zero in both the gases
(a) [ ML2T -1 ] (b) [ M 2 L-1T -2 ]
(c) of B will be more than that of A
(c) [ ML2 T - 2 ] (d) [ M 2 L-1T -2 ]
(d) of A will be more than that of B
EBD_7418
MT- 8 PHYSICS

15. In the circuit shown, the effective capacitance between 22. The escape velocity for a body projected vertically upwards
points x and y is from the surface of earth is 11 km/s. If the body is projected
6 µF 3 µF
a b
at an angle of 45°with the vertical, the escape velocity will be
8 µF 2 µF 3 µF (a) 11 2 km/s (b) 22 km/s
x 11
(c) 11 km/s (d) km / s
y 1 µF 1 µF 2
23. An open U-tube contains mercury. When 11.2 cm of water
is poured into one of the arms of the tube, how high does
8 µF the mercury rise in the other arm from its initial level?
(a) 3.33 µF (b) 1 µF (a) 0.56 cm (b) 1.35 cm
(c) 0.44 µF (d) None of these (c) 0.41 cm (d) 2.32 cm
16. A particle of mass m1 is moving with a velocity v1 and another 24. A steel wire of length 20 cm and uniform cross-section 1
particle of mass m2 is moving with a velocity v2. Both of mm2 is tied rigidly at both the ends. The temperature of the
them have the same momentum but different kinetic energies wire is altered from 40ºC to 20ºC. Coefficient of linear
are E1 and E2 respectively. If m1 > m2 then expansion for steel a = 1.1 × 10–5/ºC and Y for steel is 2.0 ×
(a) E1 = E2 (b) E1 < E2 1011 N/m2. The change in tension of the wire is
E1 m1 (a) 2.2 × 106 newton (b) 16 newton
(c) E = m (d) E1 > E2 (c) 8 newton (d) 44 newton
2 2 25. The fastest mode of transfer of heat is
17. A force F acting on an object varies with distance x as (a) conduction (b) convection
shown here. The force is in N and x in m. The work done by (c) radiation (d) None of these
the force in moving the object from x = 0 to x = 6 m is 26. The B – H curve (i) and (ii) shown in fig associated with
B
F(N) (i)
(ii)

3 H
2
1
x(m) (a) (i) diamagnetic and (ii) paramagnetic substance
0
1 2 3 4 5 6 7 (b) (i) paramagnetic and (ii) ferromagnetic substance
(c) (i) soft iron and (ii) steel
(a) 18.0 J (b) 13.5 J (c) 9.0 J (d) 4.5 J
(d) (i) steel and (ii) soft iron
18. If a spherical ball rolls on a table without slipping the fraction 27. The pressure P of an ideal gas and its mean K.E. per unit
of its total energy associated with rotational energy is volume are related as
3 2 2 3 E
(a) (b) (c) (d) (a) P = (b) P = E
5 7 5 7 2
3E 2E
19. Shown below is a distribution of charges. The flux of electric (c) P = (d) P =
field due to these charges through the surfaces S is 2 3
28. A body cools from 50.0°C to 48°C in 5s. How long will it
(a) 3q/e0 take to cool from 40.0°C to 39°C? Assume the temperature
+q –q of surroundings to be 30.0°C and Newton's law of cooling
(b) 2q/e0 to be valid.
(a) 2.5 s (b) 10 s (c) 20 s (d) 5 s
(c) q/e0
29. The spatial distribution of the electric field due to two
+q
(d) zero charges (A, B) is shown in figure. Which one of the
20. 10 cm is a wavelength corresponding to the spectrum of following statements is correct ?
(a) infrared rays (b) ultraviolet rays
(c) microwaves (d) g-rays A
B
21. The transformer voltage induced in the secondary coil of a
transformer is mainly due to
(a) a varying electric field (a) A is +ve and B –ve; |A| > |B|
(b) a varying magnetic field (b) A is –ve and B +ve; |A| = |B| :
(c) the vibrations of the primary coil (c) Both are +ve but A > B
(d) the iron core of the transformer (d) Both are –ve but A > B
MOCK TEST 3 MT- 9

30. The equation of plane wave is given by 37. When a ray of light enters a glass slab from air,
(a) its wavelength decreases
æ xö (b) its wavelength increases
y = 2sin p ç 200t - ÷
è 15 ø (c) its frequency decreases
where displacement y is given in cm and time t in second, (d) neither its wavelength nor its frequency changes.
then the velocity of the wave is 38. Main function of the RF amplifiers in a superheterodyne
(a) 3000 cm/sec (b) 200 cm/sec receiver is to
(c) 150 cm/sec (d) 2 cm /sec (a) amplify signal
31. A primary cell has an e.m.f. of 1.5 volt. When short-circuited (b) reject unwanted signal
it gives a current of 3 ampere. The internal resistance of the (c) discriminate against image frequency signal and IF-signal
cell is (d) All the above
(a) 4.5 ohm (b) 2 ohm 39. An electric fan and a heater are marked as 100 W, 220 V and
(c) 0.5 ohm (d) (1/4.5) ohm 1000 W, 220 V respectively. The resistnace of heater is
32. The magnetic lines of force inside a bar magnet (a) equal to that of fan
(a) arefrom N-pole to S-pole of magnet (b) lesser than that of fan
(b) do not exist (c) greater than that of fan
(c) depend upon the area of cross-section of bar magnet (d) zero
(d) are from S-pole to N-pole of magnet 40. A straight wire of length 0.5 metre and carrying a current of
33. Five resistances have been connected as shown in the 1.2 ampere is placed in uniform magnetic field of induction
figure. The effective resistance between A and B is 2 tesla. The magnetic field is perpendicular to the length of
the wire. The force on the wire is
3W 4W (a) 2.4 N (b) 1.2 N (c) 3.0 N (d) 2.0 N
41. Monochromatic light of frequency 6.0 × 10 14 Hz is
A 7W B produced by a laser. The power emitted is 2 × 10–3 W. The
6W 8W number of photons emitted, on the average, by the source
per second is
(a) 14/3W (b) 20/3W (c) 14W (d) 21W (a) 5 × l 016 (b) 5 × 1017
34. Yellow light is used in a single slit diffraction experiment (c) 5 × 1014 (d) 5 × 1015
with slit width of 0.6 mm. If yellow light is replaced by X– 42. Of the various series of the hydrogen spectrum, the one
rays, then the observed pattern will reveal, which lies wholly in the ultraviolet region is
(a) that the central maximum is narrower (a) Lyman series (b) Balmer series
(b) more number of fringes (c) Paschen series (d) Bracket series
(c) less number of fringes 43. An artificial satellite moving in a circular orbit around the
(d) no diffraction pattern earth has a total (kinetic + potential) energy E0. Its potential
35. Which one is possible? energy is
19 16 1 (a) –E0 (b) 1.5 E0 (c) 2E0 (d) E0
(a) 7N + 0 n1 ® 7 N + 1H 44. Of the diodes shown in the following diagrams, which one
(b) 16 S
32
+ 1 H 1 ® 17Cl 35 + 2 He4 is reverse biased ?
16 1 14 1 0 +10 V
(c) 8 O + 0 n ® 7 N + 3 1H + 2 - 1b
(a) R
(d) 1 H 1 + 1H 1 ® 2 He 4
+5 V
36. A conducting square loop of side L and resistance R moves
in its plane with a uniform velocity v perpendicular to one –12 V R
of its side. A magnetic induction B constant in (b)
–5 V
x x x x x x
x x x x
x x x x
R
x x x x x vx B (c)
x x x x x –10 V
x x x x x x
+5 V
time and space, pointing perpendicular and into the plane
of the loop exists everywhere. The current induced in the R
loop is (d)
Bl v
(a) clockwise 45. Which of the following gates will have an output of 1?
R
1 0
Bl v 1 1
(b) anti-clockwise
R A B
0 0
2 Bl v
(c) anti - clockwise 1 0
R C D
(d) zero (a) D (b) A (c) B (d) C
EBD_7418
MT- 10 PHYSICS

M ck Test - 4
Time : 1 hrs Max. Marks -120

1. The potential energy of a particle is given by the expression 8. The gravitational field in a region is given by
x ®
U ( x) = – ax + b sin . A dimensionless combination of the
g g = 5N / kgiˆ + 12N / kgjˆ . The change in the gravitational
constants a, b and g is : potential energy of a particle of mass 1 kg when it is taken
a a2 g ag from the origin to a point (7 m, – 3 m) is:
(a) (b) (c) (d)
bg ba ab b (a) 71 J (b) 13 58J (c) – 71 J (d) 1 J
2. Among the four graph shown in the figure there is only 9. A metallic bar is heated from 0ºC to 100ºC. The coeficient of
one graph for which average velocity over the time interval linear expansion is 10–5 K–1. What will be the percentage
(O, T) can vanish for a suitably chosen T. Which one is it? increase in length?
x x
(a) 0.01% (b) 0.1%
(c) 1% (d) 10%
(a) t (b) 10. A spring is compressed between two blocks of masses m1
t and m2 placed on a horizontal frictionless surface as shown
in the figure. When the blocks are released, they have initial
x x velocity of v1 and v2 as shown. The blocks travel distances
x1 and x2 respectively before coming to rest. The ratio
(c) (d) æ x1 ö
is
t t çè x ÷ø m2
2 m1

3. A particle undergoes simple harmonic motion having time


v1 v2
period T. The time taken in 3/8th oscillation is
3 5 5 7
(a) T (b) T (c) T (d) T
8 8 12 12
4. A uniform solid cylindrical roller of mass ‘m’ is being pulled m2 m1 m2 m1
(a) m1 (b) m2 (c) m1 (d) m2
on a horizontal surface with force F parallel to the surface
and applied at its centre. If the acceleration of the cylinder
11. A thin convex lens of focal length ‘f’ is put on a plane
is ‘a’ and it is rolling without slipping then the value of ‘F’ is: mirror as shown in the figure. When an object is kept at
5 3 a distance ‘a’ from the lens - mirror combination, its image
(a) ma (b) ma (c) ma (d) 2 ma
3 2 a
is formed at a distance in front of the combination.
5. The work function of aluminium is 4.2 eV. If two photons 3
each of energy 3.5 eV strike an electron of aluminium, then The value of ‘a’ is :
emission of electron will
3
(a) depend upon the density of the surface (a) 3f (b) f
(b) possible 2
(c) not possible (c) f (d) 2f
(d) None of these 12. Electric lines of force about a positive point charge are
6. Resonance frequency of LCR series a.c. circuit is f0. Now (a) circular, anti-clockwise
the capacitance is made 4 times, then the new resonance (b) circular, clockwise
frequency will become (c) radial, inwards
(a) f0/4 (b) 2f0 (c) f0 (d) f0/2. (d) radial, outwards
7. Two wires A and B of the same material, having radii in the 13. A gun fires two bullets at 60º and 30º with horizontal. The
ratio 1 : 2 and carry currents in the ratio 4 : 1. The ratio of bullets strike at some horizontal distance. The ratio of
drift speed of electrons in A and B is maximum height for the two bullets is in the ratio of
(a) 16 : 1 (b) 1 : 16 (a) 2 : 1 (b) 3 : 1
(c) 1 : 4 (d) 4 : 1 (c) 4 : 1 (d) 1 : 1
MOCK TEST 4 MT- 11

14. A generator has an e.m.f. of 440 Volt and internal resistance 23. A galvanometer coil has a resistance of 15W and gives full
of 400 Ohm. Its terminals are connected to a load of 4000 scale deflection for a current of 4 mA. To convert it to an
Ohm the voltage across the load is ammeter of range 0 to 6 A
(a) 220 volt (b) 440 volt (c) 200 volt (d) 400 volt (a) 10 mW resistance is to be conncected in parallel to the
15. The radius vector, drawn from the sun to a planet, sweeps galvanometer
(b) 10 mW esistance is to be connected in series with the
out equal areas in equal intervals of time. This is the
galvanometer
statement of (c) 0.1 W resistance is to be connected in parallel to the
(a) Kepler’s first law (b) Kepler’s second aw galvanometer
(c) Newton’s first law (d) Kepler’s third law (d) 0.1 W resistance is to be connected in series with the
16. The path difference between the two waves : galvanometer
æ 2 px ö 24. A uniform rod of mass m, length l , area of cross-section A
y1 = a1 sin ç wt - ÷
è l ø has Young’s modulus Y. If it is hanged vertically, elongation
æ 2px ö under its own weight will be
and y 2 = a 2 sin ç wt - + f ÷ will be mgl 2mgl mgl mgY
è l ø (a) (b) (c) (d)
2AY AY AY Al
2p 2 p æ pö 25. The magnifying power of a telescope is 9. When it is
(a) f (b) çf - ÷
l l è 2ø adjusted for parallel rays, the distance between the objective
l 2p æ pö and the eye piece is found to be 20 cm. The focal length of
(c) f (d) çf + ÷ lenses are
2p l è 2ø
(a) 18 cm, 2 cm (b) 11 cm, 9 cm
17. The diagram shows the energy levels for an electron in a (c) 10 cm, 10 cm (d) 15 cm, 5 cm
certain atom. Which transition shown represents the 26. If two soap bubbles of different radii are connected by a
emission of a photon with the most energy? tube. Then
n=4 (a) air flows from the smaller bubble to the bigger
n=3 (b) air flows from bigger bubble to the smaller bubble till
the sizes are interchanged
n=2 (c) air flows from the bigger bubble to the smaller bubble
till the sizes become equal
(d) there is no flow of air.
n =1 27. An object undergoing SHM takes 0.5 s to travel from one
I II III IV point of zero velocity to the next such point. The distance
(a) IV (b) III (c) II (d) I between those points is 50 cm. The period, frequency and
18. The direction of propagation of electromagnetic waves is amplitude of the motion is
given by the direction of (a) 1s, 1Hz, 25 cm (b) 2s, 1Hz, 50 cm
ur ur (c) 1s, 2Hz, 25 cm (d) 2s, 2Hz, 50 cm
(a) Vector E (b) Vector B
ur ur 28. A plane wave of wavelength 6250 Å is incident normally on
(c) Vector ( E ´ B ) (d) None of these a slit of width 2 × 10–2 cm. The width of the principal maximum
19. A body of mass 10 kg and velocity 10 m/s collides with a on a screen distant 50 cm will be
stationary body of mass 5 kg. After collision both bodies stick (a) 312.5 × 10–3 cm (b) 312.5 × 10–3 m
to each other, velocity of the bodies after collision will be (c) 312.5 × 10–3 m (d) 312 m
3 18 9 20 29. In a p-type semi-conductor germanium is doped with
(a) m/s (b) m/s (b) m/s (d) m/s (a) aluminium (b) boron
10 3 20 3
20. Two particles of mass m1 and m2 (m1 > m2) attract each (c) gallium (d) all of these
other with a force inversely proportional to the square of 30. The table given below represents the truth table for which
the distance between them. If the particles are initially held of the following combinations of logic gates?
at rest and then released, the centre of mass will A B Y
(a) move towards m1 (b) move towards m2 0 0 1
(c) remain at rest (d) Nothing can be said 0 1 0
21. The r.m.s. velocity of oxygen molecule at 16ºC is 474 m/sec. 1 0 1
The r.m.s. velocity in m/s of hydrogen molecule at 127ºC is 1 1 0
(a) 1603 (b) 1896 (c) 2230.59 (d) 2730
(a) A
22. To demonstrate the phenomenon of interference, we require B Y
two sources which emit radiation
A
(a) of the same frequency (b) Y
B
(b) of different wavelengths
(c) of the same frequency and having a definite phase A
(c) B Y
relationship
(d) of nearly the same frequency (d) None of these
EBD_7418
MT- 12 PHYSICS

M ck Test - 5
Time : 1 hrs Max. Marks -120

1. Which of the following do not have the same dimensional 8. A bomb is dropped on an enemy post by an aeroplane flying
formula as the velocity? horizontally with a velocity of 60 km h–1 and at a height of
Given that m0 = permeability of free space, e0 = permittivity 490 m. At the time of dropping the bomb, how far the
of free space, n = frequency, l = wavelength, P = pressure, r aeroplane should be from the enemy post so that the bomb
= density, w = angular frequency, k = wave number, may directly hit the target ?
400 500 1700
(a) 1 m 0 eo (b) n l (c) P/r (d) wk (a) m (b) m (c) m (d) 498 m.
3 3 3
2. A mass m is moving with velocity v collides inelastically 9. A sphere of radius R has uniform volume charge density.
with a bob of simple pendulum of mass m and gets embedded The electric potential at a point (r < R) is
into it. The total height to which the masses will rise after (a) due to the charge inside a sphere of radius r only
collision is (b) due to the entire charge of the sphere
(c) due to the charge in the spherical shell of inner and
v2 v2 v2 2v 2
(a) (b) (c) (d) outer radii r and R, only
8g 4g 2g g (d) independent of r
3. In the system shown in figure, the pulley is smooth and 10. A point object is located at a distance 15cm. from the pole of
massless, the string has a total mass 5g, and the two a concave mirror of focal length 10cm on its principal axis is
suspended blocks have masses 25 g and 15 g. The system moving with a velocity (8iˆ + 11 ˆj ) cm/s and velocity of
is released from state l = 0 and is studied upto stage l¢ = 0
mirror is (4iˆ + 2 ˆj ) cm/s as shown. If vr is the velocity of
During the process, the acceleration of block A will be r
g image. Then find the value of | v | in (cm/s).
(a) constant at
9 y
g l l'
(b) constant at
4 A
25 g ^ ^
////
(c) increasing by factor of 3 B (8i + 11j) cm/s x
/////// /////////////
15 g
(d) increasing by factor of 2
^
(4i + 2^j) cm/s
////

4. A particle starting from rest falls from a certain height. O


Assuming that the acceleration due to gravity remain the 15cm
same throughout the motion, its displacements in three
successive half second intervals are S1, S2 and S3 then
(a) S1 : S2 : S3 = 1 : 5 : 9 (b) S1 : S2 : S3 = 1 : 3 : 5 (a) 20 (b) 30 (c) 10 (d) 40
(c) S1 : S2 : S3 = 9 : 2 : 3 (d) S1 : S2 : S3 = 1 : 1 : 1 11. The meniscus of a liquid contained in one of the limbs of a
5. A cube is subjected to a uniform volume compression. If narrow U-tube is held in an electromagnet with the meniscus
the side of the cube decreases by 2% the bulk strain is in line with the field. The liquid is seen to rise. This indicates
(a) 0.02 (b) 0.03 (c) 0.04 (d) 0.06 that the liquid is
(a) ferromagnetic (b) paramagnetic
6. A transformer is employed to
(c) diamagnetic (d) non-magnetic
(a) convert A.C. into D.C. 12. Under a constant torque the angular momentum of a body
(b) convert D.C. into A.C. changes from A to 4A in 4 second. The torque on the body
(c) obtain a suitable A.C. voltage will be
1 4 3
(d) obtain a suitable D.C. voltage (a) 1A (b) A (c) A (d) A
4 3 4
7. A square coil of side 25cm having 1000 turns is rotated with
a uniform speed in a magnetic field about an axis 13. The kinetic theory of gases
perpendicular to the direction of the field. At an instant t, (a) explains the behaviour of an ideal gas
the emf induced in the coil is e = 200 sin 100pt. The magnetic (b) describes the motion of a single atom or molecule
induction is (c) relates the temperature of the gas with K.E. of atoms of
(a) 0.50 T (b) 0.02 T (c) 0.01 T (d) 0.1 T the gas
(d) all of the above
MOCK TEST 5 MT- 13

14. An electric bulb marked 40 W and 200V, is used in a circuit of (a) E a = 4E q (b) E q = 2E a
supply voltage 100V. Now its power is
(c) E a = 2E q (d) E q = 3E a
(a) 10 W (b) 20 W (c) 40 W (d) 100 W
15. The height at which the acceleration due to gravity becomes 23. Two coherent monochromatic light beams of intensities I
and 4 I are superimposed. The maximum and minimum
g possible intensities in the resulting beam are
(where g = the acceleration due to gravity on the surface
9 (a) 5I and I (b) 5I and 3I
of the earth) in terms of R, the radius of the earth, is (c) 9I and I (d) 9I and 3I
R 24. When an electron jumps from the fourth orbit to the second
(a) (b) R / 2 (c) 2 R (d) 2 R orbit, one gets the
2
16. A thermodynamic system is taken from state A to B along (a) second line of Lyman series
ACB and is brought back to A along BDA as shown in the PV (b) second line of Paschen series
diagram. The net work done during the complete cycle is (c) second line of Balmer series
given by the area (d) first line of Pfund series
25. A uniform magnetic field acts at right angles to the direction
P
of motion of electron. As a result, the electron moves in a
B
P2 circular path of radius 2 cm. If the speed of electron is
C
doubled, then the radius of the circular path will be
P1 D (a) 2.0 cm (b) 0.5 cm (c) 4.0 cm (d) 1.0 cm
A 26. A photoelectric cell is illuminated by a point source of light
A¢ B¢ V 1 m away. When the source is shifted to 2 m, then
(a) P1ACBP2P1 (b) ACBB'A'A (a) number of electrons emitted is a quarter of the initial
(c) ACBDA (d) ADBB'A'A number
(b) each emitted electron carries one quarter of the initial
17. A man is watching two trains, one leaving and the other energy
coming with equal speed of 4 m/s. If they sound their (c) number of electrons emitted is half the initial number
whistles each of frequency 240 Hz, the number of beats heard (d) each emitted electron carries half the initial energy
by man (velocity of sound in air = 320 m/s) will be equal to 27. A hollow cylinder has a charge q coulomb within i. If f is
(a) 12 (b) 0 (c) 3 (d) 6 the electric flux in units of voltmeter associated with the
18. When a wave travels in a medium the particles displacement curved surface B, the flux linked with the plane surface A in
is given by the equation y = 0.03 sin p(2t – 0.01x), where x units of voltmeter will be
and y are in seconds. The wavelength of the wave is B

(a) 200 m (b) 100 m (c) 20 m (d) 10 m C A


19. The value of current I in the circuit shown in figure is
R2= 5W
q f q 1æ q ö
(a) (b) (c) - f (d) ç - f÷
2e0 3 e0 2 è e0 ø
3V R1= 5W R3= 5W
28. Two flat circular coils have a common center, but their planes
are at right angles to each other. The inner coil has 150 turns
R4= 5W and radius of p cm. The outer coil has 400 turns and a radius
(a) 1.8 A (b) 0.8 A (c) 0.2 A (d) 1.6 A of 2p cm. The magnitude of the resultant magnetic induction
20. The oscillating electric and magnetic field vectors of at the common centers of the coils when a current of 200 mA
electromagnetic wave are oriented along is sent through each of them is
(a) the same direction and in phase (a) 10–3 Wb/m2 (b) 2 × 10–3 Wb/m2
(b) the same direction but have a phase difference of 90º –3
(c) 5 × 10 Wb/m 2 (d) 7 × 10–3 Wb/m2
(c) mutually perpendicular directions and are in phase 29. Optical fibres transmit light along its axis, by the process of
(d) mutually perpendicular directions but has a phase (a) total internal reflection
difference of 90º (b) refraction
21. If the cold junction of a thermo-couple is kept at 0°C and the (c) interference
hot junction is kept at T°C then the relation between neutral (d) diffraction
temperature (Tn) and temperature of inversion (Ti) is 30. When a p-n junction diode is reverse biased the flow of
(a) Tn = 2Ti (b) Tn = Ti – T current across the junction is mainly due to
(c) Tn = Ti + T (d) Tn = Ti/2 (a) diffusion of charges
22. Let E a be the electric field due to a dipole in its axial plane (b) drift of charges
distant l and Eq be the field in the equatorial plane distant (c) depends on the nature of material
l', then the relation between Ea and Eq will be (d) both drift and diffusion of charges
EBD_7418
MT- 14 PHYSICS

HINTS & SOLUTIONS


MOCK TEST (1-5)
MOCK TEST - 1
ANSWER KEYS
1 (a) 6 (d) 11 (c) 16 (d) 21 (a) 26 (a) 31 (b) 36 (c) 41 (d)
2 (c) 7 (a) 12 (c) 17 (c) 22 (a) 27 (b) 32 (a) 37 (b) 42 (a)
3 (d) 8 (a) 13 (c) 18 (d) 23 (c) 28 (c) 33 (b) 38 (d) 43 (a)
4 (c) 9 (a) 14 (c) 19 (c) 24 (d) 29 (b) 34 (a) 39 (c) 44 (d)
5 (b) 10 (a) 15 (d) 20 (b) 25 (b) 30 (a) 35 (c) 40 (c) 45 (a)

EXPLANATIONS
6. (d) For nuclear disintegration
[E] [E]
1. (a) [A] = [E] [x3]; [B] = ; [C] = 2 2.303 No
[y] [z ] l= log
t No - N
2. (c) For a–t curve, area under the graph gives change in velocity
at t = 10 sec, v = 40 m/s. 2.303 100
= log = 2.1 × 10–6sec–1
For 10 – 30 sec, Dv = – 80, v30sec – 40 = –80. 24 ´ 60 ´ 60 82
Speed at 30 sec = – 40 m/s 7. (a) The mean value of refractive index,
3. (d) W = W1 - W2 = mgh - mg h ¢ = mg (h - h ¢) 1.34 + 1.38 + 1.32 + 1.36
m= = 1.35
= 5 ´ 10 ( 20 - 0 . 2 ) = 5 ´ 10 ´ 19 .8 4
and
= 5 ´ 198 = 990 joule
| (1.35 - 1.34) | + | (1.35 - 1.38) | + | (1.35 - 1.32) | + | (1.35 - 1.36) |
Dm =
This energy is converted into heat when the ball strikes 4
the earth. Heat produced is = 0.02
990 Dm 0.02
Q= calorie Thus ´ 100 = ´ 100 = 1.48
4.2 m 1.35
Q 99 ´ 100 11 8. (a) The magnetic dipole moment of diamagnetic material
DT = = = ºC is zero as each of its pair of electrons have opposite
mc 42 ´ 5000 ´ 0 .09 32
spins, i.e., md = 0.
4. (c) T1= 77ºC = 350 K
Paramagnetic substances have dipole moment > 0, i.e.
125V mp ¹ 0, because of excess of electrons in its molecules
V1 = V and V2 =
343 spinning in the same direction.
5 Ferro-magnetic substances are very strong magnets
Ratio of specific heats for monoatomic gas, g = and they also have permanent magnetic moment, i.e.
3
mf ¹ 0.
In an adiabatic process,
9. (a) Distance covered in one circular loop = 2pr
T1V1g–1 = T2V2g–1
= 2 × 3.14 × 100 = 628 m
g -1
æ V1 ö 628
or T2 = ç V ÷ × T,, Speed = = 10 m / sec
è 2ø 62.8
(5/3) -1
Displacement in one circular loop = 0
æ 1 ö 0
T2 = 350 × ç
è 125 / 343 ÷ø Velocity = =0
time
= 686 K
10. (a) Let velocity at A = v A and velocity at B = v B
Hence, rise in temperature
= T2 – T1 = 686 – 350 = 336 K
5. (b)
MOCK TEST 1 MT- 15

A r r
vA 19. (c) Fn =q(v ´ B)
r T
= qvB sinq = 0 (because q = 0º )
20. (b) The time of cooling increases as the difference
vB between the temperature of body & surrounding is
B
reduced. So T1<T2<T3 (according to Newton’s law of
Applying conservation of energy at A & B
cooling).
1 1
mv 2A + 2gmr = mv 2B 21. (a) In uniform circular motion speed is constant. So, no
2 2
tangential acceleration.
v 2B = v 2A + 4gr..........(i ) It has only radial acceleration
Now as it is moving in circular path it has centripetal v2
force. aR = [directed towards center]
R
mv 2A
At point A Þ T + mg =
r
r
for minimum velocity T ³ 0 vR2

mv 2A
or ³ mg Þ v 2A ³ gr Þ v A ³ gr
r
11. (c) Power = V . I = I2R and its velocity is always in tangential direction. So these
Power 2 1 1 two are perpendicular to each other.
i2 = = = = A -3
R 8 4 2 LdI 40 ´10 (11 - 1)
22. (a) e= = = 100V
Potential over 8W = Ri 2 = 8 ´
1
= 4V
dt 4 ´10-3
2
C1 C2
This is the potential over parallel branch. So, 23. (c) Cs = =3
C1 + C2
4
i1 = =1A
4 Cp = C1 + C2 = 16 \ C1 C2 =48
Power of 3W = i12R = 1 × 1 × 3 = 3W
2 p 2 em 4 z 2 C1 – C2 = (C1 +C 2 ) 2 – 4 C1 C 2
12. (c) For nth orbit, energy, E n =
n 2h2
E5 7 2 = 162 - 4 ´ 48 = 64 = 8
=
E 7 55 \ C1 = 12 mF and C2 = 4 mF
25 24. (d) At the centre of the earth gravity is zero.
Þ E = E7
49 25. (b) Frequency does not depend upon the medium, so, it
13. (c) Conserving Linear Momentum will remain same in the material of the tuning fork and
2Mvc = 5Mv – 2Mv Þ vc = 3v/2. in air.
26. (a) The highest energy level occupied by an electron in
1/ 3 1/ 3
Rs æ A s ö æ 32 ö the energy band at zero kelvin is called Fermi level and
14. (c) =ç ÷ =ç ÷ =2 the energy associated is known as Fermi energy. So, it
R a çè A a ÷
ø è 4 ø is independent of T.
dH 27. (b) The intersection of medians is the centre of mass of
15. (d) µ (q 2 - q1 ) = (Dq) n Þ n = 1 the triangle. Since distances of centre of mass from
dt
the sides are related as : xBC < xAB < xAC therefore IBC
16. (d) Use a = µg and v2 = u2 + 2as > IAB > IAC or IBC > IAB.
17. (c) When the arrangement is dipped in water; r
28. (c) If F force acts for short interval Dt, then
x 3 r
b¢ =b / m = = x = 0.75x r
4/3 4 F DT = mD v .
18. (d) The charge q, which is kept at the centre of metallic 29. (b) For electromagnetic wave,
spherical shell transfered to the outer surface of shell FE = FM Þ eE = BeC Þ E = B.C
& inside the shell the electric field is zero & hence
force is also zero. = 2 ´ 10-7 ´ 3 ´ 108 = 60 V / m
EBD_7418
MT- 16 PHYSICS

30. (a) According to Biot Savart’s law 39. (c)


m 0 Idl sin q
dB = . 45°
4p r2
1 30°
dB µ 2
r
So, graph (a) is correct.
31. (b) According to Stefan’ law heat emitted per second per
sin 45º
unit area a(T14 - T24 ) where T1 is the temperature of Refractive index, m =
sin 30º
body & T2 is the temperature of surrounding (T1 & T2 1
are on Kelvin scale). m= ´2 = 2
2
32. (a) 1
fw
= ( µ -1)æçç r1 - r1 ö÷÷
w
g Now, m =
Velocity of light in air
Velocity of light in medium
= 2
è 1 2 ø
Velocity of light in medium
[ f w is focal length of lens in water]
8 1
= 3 ´ 10 ´ = 2.12 ´ 108 m / sec
1 a æ1 1ö 2
= ( µg - 1) ç - ÷ [f is focal length of lens in air]
f è r1 r2 ø 40. (c) Rain drops are in spherical shape due to surface
tension.
Dividing,
41. (d) d1 = 40º , d2 = 30º , d = ?
f (w µg - 1) a
µg
3/ 2 3 3 9
= ;w µ g = = = ´ = cot d = cot 2 d1 + cot 2 d 2 = cot 2 40º + cot 2 30º
fw (a µg - 1) a
µw 4/3 2 4 8
cot d = 1.19 2 + 3 = 2.1
f 9 / 8 - 1 1/ 8 1 \ d = 25º i.e. d < 40 º.
= = =
fw 3 1/ 2 4
-1 1 1 1
2 42. (a) m= = = =2
sin C sin 30° 1/ 2
f w = 4 ´ f = 4 ´ 10 = 40 cm. Velocity of light in the medium
33. (b) If current passes through a spring, it shrinks as in two
3 ´108 8
adjacent wire, current is flowing in the same direction. = = 1.5 ´10 m / sec
Even if direction of current is reversed still the spring 2
will shrink. As AC current is a periodically changing 43. (a)
current, the process of shrinking will also be periodic æ m 2 ö
in nature. 44. (d) PT = PC ç1 + a ÷
ç 2 ÷
250 ´ 10 è ø
F / A 50 ´ 10-6 = 250 ´ 9.8 ´ 2 æ m 2 ö 3 ma 2
35. (c) Y= =
Dl / l 0.5 ´ 10 -3 50 ´ 10 -6
0.5 ´ 10 -3 \ 600 = 400 ç 1 + a ÷ Þ = 1+
ç 2 ÷ 2 2
2 è ø

Þ 19.6 ´1010 N / m 2 ma 2 1
or = Þ ma = 1
2 2
36. (c) Max. force = mass × max. acceleration
= m 4 p2 n2 a = 1 × 4 ×p2 × (60)2 × 0.02 = 288 p2
37. (b) Changing electric field gives rise to displacement
current which creates magnetic field around it.
MOCK TEST - 2
ANSWER KEYS
1 (c) 6 (b) 11 (d) 16 (b) 21 (c) 26 (b) 31 (a) 36 (c) 41 (c)
2 (c) 7 (a) 12 (d) 17 (b) 22 (c) 27 (b) 32 (d) 37 (b) 42 (c)
3 (c) 8 (a) 13 (b) 18 (a) 23 (a) 28 (b) 33 (c) 38 (b) 43 (b)
4 (c) 9 (b) 14 (d) 19 (d) 24 (b) 29 (b) 34 (d) 39 (b) 44 (d)
5 (d) 10 (a) 15 (c) 20 (b) 25 (c) 30 (a) 35 (d) 40 (d) 45 (a)

EXPLANATIONS

V2 V2 Þ 44 ´ (0.8) 2 = w2 ´ (1) 2
1. (c) P = Þ 36 = Þ V = 18V
R 9
Þ w2 = 44 ´ 0.8 ´ 0.8 = 28.16 radian/sec
Current passing through the 9W resistor is
V 18 5. (d) x = a + bt2 = 15 + 3t2
i1 = = = 2A
R 9
dx
The 9W and 6W resistors are in parallel, therefore v= = 3 × 2t = 6t Þ v t = 3s = 6 × 3 = 18cm/s
dt
6 6. (b) A/C to Avogadro’s Law
i1 = ´i
9+ 6
where i is the current delivered by the battery. 1 mol at STP º 22.4 L

2 ´ 15 Þ 1 mol NH3 º 14 + 3 = 17g


\i= = 5A
6
201.6
Thus, potential difference across 2W resistor is Total wt = ´ 17 = 153 g
22.4
V = iR 7. (a)
=5×2
1 1 1
= 10V 8. (a) C = Þ moeo = = = [Mo L–2 T2]
m o eo C 2
[M L T -1 ]2
o

4 3
2. (c) V= pr ; 9. (b) Distance along a line i.e., displacement (s)
3
= t3 (Q s µ t 3 given)
DV æ Dr ö By double differentiation of displacement, we get
´ 100 = 3 ç ÷ ´ 100 acceleration.
V è r ø
ds dt 3
Þ 9% = 3 × x% Þ x% = 3% V= = = 3t 2 and
dt dt
3. (c) Here all the three springs are connected in parallel to
mass m. Hence equivalent spring constant dv d 3t 2
a= = = 6t Þ a µ t
k = K + K + 2 K = 4 K. dt dt
4. (c) We shall apply conservation of angular momentum; Hence graph (b) is correct.
that is 10. (a) For Balmar series
1 é1 1 ù
I1w1 = I2 w 2 =Rê - ú
l min ë 22 ¥2 û
2 2 4
Þ mv1r1 = mv2 r2 Þ w1r1 = w 2 r2 Þ lmin = = 3646 A°
R
EBD_7418
MT- 18 PHYSICS

11. (d) Start with C2 and C4 in parallel, then C2 in series, then 1 q -18
× = 9 ´ 10 ´ 8 ´ 10 = 1.1 × 107 V
9
V=
C5 in parallel, then C1 in series and finally C6 in 4pe0 r 6.6 ´ 10 -15
parallel.
19. (d) One faraday charge is required to losing about
1 electrolysis of one gram equivalent of a body. Now
12. (d) hn = f + mv2 Þ hv = f + K.E.
2 using this information in the relation,

1 m = ZQ
n = f+ K.E.
h
E
This is equation of a straight line, if we plot n, K-E E = ZF Þ F =
Z
1
curve. The slope of this curve is which is constant. 21. (c) The horizontal velocity of the stone will be the same
h
So, it is independent of metals used. as that of the train. In this way, the horizontal motion
will be uniform. The vertical motion will be controlled
Option (d) is correct. by the force of gravity. Hence it is accelerated motion.
The resultant motion is a parabolic trajectory.
13. (b) Moment of inertia of a body is given by the expression

n P2
24. (b) K= Þ P = 2 Km
å mi r i 2 2
I= MR 2m
= for solid cylinder about
i =1 2
as K is made four times P will become two times.
geometrical axis.
This is the sum of product of mass of each particle æ u + u/5 ö 6
25. (c) fapparent = ç ÷ f = f =1.2f
and square of its distance from the axis. è u ø 5
14. (d) 0.1% of 1 kg = 0.001 kg
wavelength remains constant (unchanged) in this case.
From the formula, E = mc 2 26. (b) Photoelectric effect is based on law of conservation
of energy
Energy released = 0.001 ´ (3 ´ 108 )2 joules
27. (b) Choke coil is a device having high inductance and
16 -3 13 negligible resistance. It is used to control current in ac
= 9 ´ 10 ´ 10 = 9 ´ 10 J
circuits and the power loss in a circuit containing choke
15. (c) 3k = x. 2k coil is least.
[Here a is coefficient of linear expansion so 28. (b) After 3 sec its velocity
coefficient of cubical expansion = 3a & coefficient
of superfacial expansion = 2a] v 2 = u 2 + 2gh = 0 + 2 ´ 9.8 ´ 25

3
x= = 1.5 1 1
2 K.E = mv 2 = ´ 3 ´ 2 ´ 9.8 ´ 25 = 75 × 9.8 = 748 J.
2 2
16. (b) When a bus suddenly take a turn, the passengers are
thrown outwards because of inertia of motion. 29. (b) Since no external torque action gymnast, so angular
momentum (L = Iw) is conserved. After pulling her
17. (b) For minimum deviation, incident angle is equal to arms & legs, the angular velocity increases but moment
emerging angle. of inertia of gymnast, decreases in, such a way that
\ QR is horizontal. angular momentum remains constant.
18. (a) Charge on the tin nucleus = n.Q
µ0 I q m0 I q
30. (a) B= ´ =
= (1.6 × 10–19 × 50) C = 8 × 10–18 C 2r 2 p 4 pr
This charge is supposed to be concentrated at its 32. (d) Ability of the eye to see object at all distance is called
centre. Hence, potential on the surface,
MOCK TEST 2 MT- 19

accommodation. This is the power of the eye to change 40. (d) Surface tension of solution
the focal length of eye lens so that it can focus rays
coming from different distances. (T) = 30 ´ 10-12 N / m
33. (c) The velocity of electromagnetic radiation in a medium Radius of the soap bubble (r) = 5 cm = 0.05 m
of permittivity Î 0 and permeability m0 is equal to
Since, the bubble has two spherical surface,
1
.
m0 Î0 hence, DA = 2 ´ surface area = 2 × 4pr 2

34. (d) As displacement £ distance, therefore, the ratio is Therefore, surface energy = T × DA
equal to or less than one.
= 30 × 10–2 × 2 × [4 p (0.05)2] = 1.8 × 10–2 J

é 2h ù
35. (d) Variation of g with altitude is, g h = g ê1 - ú ; 41. (c)
ë Rû
/2

é dù
variation of g with depth is, g d = g ê1 - ú
ë Rû

Equating g h and g d , we get d = 2h From the figure it is clear that in both the cases
wavelength is same. So, frequency of fundamental
2/3(½ m N v 2rm s ) tone will also be same. Hence the answer will be (c).
36. (c) P= where m is mass of molecule
V 42. (c) Radio waves are low frequency electro-magnetic waves.
So, their wavelength will be comparatively large.
of gas, N is number of molecules in gas & v rms is root
43. (b) For common base
mean squar velocity of the molecule.

37. (b) Transformer is based upon the principle of mutual Ic


Current gain = = 0.96
induction. Current is induced in the secondary coil due Ie
to change in current in primary coil of the transformer.
I c = I e ´ 0.96 = 0.96 ´ 7.2 = 6.912

2 r 2 (r - r0 )g Ib = Ie - Ic = 7.2 - 6.912
38. (b) Terminal velocity, v 0 =
9h = 0.288 » 0.29 mA

44. (d) 0V –2V It can be sketched as


2 ´ (2 ´10 -3 ) 2 ´ (8 - 1.3) ´103 ´ 9.8
= = 0.07 ms–1
9 ´ 0.83
Immobile ions

– +
3 ´108 P – + N
39. (b) Velocity of light in glass = = 2 ´108 m / sec 0V – +
1.5 depletion
layer –2V

distance 4 ´10-3 45. (a) Leakage current is the name given to the reverse
Time taken = = = 2 ´10-11 sec
velocity 2 ´ 108 current
EBD_7418
MT- 20 PHYSICS

MOCK TEST - 3
ANSWER KEYS
1 (d) 6 (c) 11 (a) 16 (b) 21 (b) 26 (c) 31 (c) 36 (d) 41 (d)
2 (b) 7 (c) 12 (b) 17 (b) 22 (c) 27 (d) 32 (d) 37 (d) 42 (a)
3 (a) 8 (c) 13 (b) 18 (b) 23 (c) 28 (b) 33 (a) 38 (d) 43 (c)
4 (b) 9 (a) 14 (b) 19 (d) 24 (d) 29 (a) 34 (d) 39 (b) 44 (d)
5 (d) 10 (c) 15 (b) 20 (c) 25 (c) 30 (a) 35 (c) 40 (b) 45 (d)

EXPLANATIONS
1. (d) v2 = u2 + 2 a s or v2 – u2 = 2 a s
Maximum retardation, a = v2/2 s GMm -GMm GMm GMm GMm
DU = - - = - =
When the initial velocity is n v, then the distance over 3R 2R 2R 3R 6R
ur r ur
which it can be stopped is given by 6. (c) Torque t = r ´ F = (iˆ - ˆj) ´ (-Fk)
ˆ
u 2 (n v) 2
sn = 0 = = n2 s = F[- ˆi ´ kˆ + ˆj ´ k]
ˆ = F(ˆj + ˆi) = F(iˆ + ˆj)
2a 2 (v 2 / 2s)
éSince kˆ ´ ˆi = ˆj and ˆj ´ kˆ = iˆ ù
2. (b) f1 = + 40 cm (for convex lens) ë û
= 0.4 m 7. (c) Dimensionally K.E = Work
f2 = – 25 cm (for concave lens) 8. (c) In a cyclic process, the initial state coincides with the
final state. Hence, the change in internal energy is
= – 0.25 m
zero, as it depends only on the initial and final states.
\ Focal length (f) of the combination But Q & W are non-zero during a cycle process.
1 1 1 1 1 0.25 - 0.4 9. (a) MOI of annular disc about diameter
= + = - =
f f1 f 2 0.40 0.25 0.40 ´ 0.25
R2
1 M ´ 2p x 3 dx M 2
=-
0.15
= -1.5 dioptre.
=
2 ò p (R 22 - R12 )
=
4
(R 2 + R12 )
0.1 R1

1 10. (c) For M1 : V = – 60, m1 = –2


Þ P= = -1.5 dioptre For M1 : u = +20, F = 10
f
1 1 1
1 2 = = Þ V = 20
3. (a) We have s = ut + gt , V 20 10
2
20
1 2 \ M2 = -= -1
or h = gT (Q u = 0) 20
2
\ M = m1 × m2 = +2
now for T/3 second, vertical distance moved is given
11. (a) Charge on an electron
by
2 = 1.6 ´ 10 -19 coulomb
1 æTö 1 gT 2 h
h' = g ç ÷ Þ h' = ´ =
2 è 3ø 2 9 9 20 ´ 10 -6
No. of electrons required =
1.6 ´ 10 -19
h
\ position of ball from ground = h - = 8h
9 9 20
= ´1013 = 1.25 ´ 1014
1. 6
4. (b) vr = vR 2 – vB2 = 102 - 82 = 6 km h –1
12. (b) Phase change p in 50 oscillations.
5. (d) Energy required to move a body of mass m from an Phase change 2p in 100 oscillations.
orbit of radius 2R to 3R is So frequency different ~ 1 in 100.
MOCK TEST 3 MT- 21

13. (b) When sound travels from one medium to another its
1
speed changes due to change in wavelength. Its mv 2
frequency remains constant. kinetic energy = 5
7
14. (b) Under isothermal conditions, there is no change in mv 2
10
internal energy.
15. (b) 1 10 2
= ´ =
5 7 7
p2
16. (b) E= 19. (d) Net charge inside the surface is zero. So, flux through
2m
the surface is zero.
2 2
20. (c) Microwave region wavelength = 10 -3 m to 1m
or, E1 = p1 , E2 = p2
2m1 2m2 22. (c) ve = 2 gR
The escape velocity is independent of the angle at
p12 p2
or, m1 = , m2 = 2 which the body is projected.
2 E1 2 E2
23. (c) mercury
water
m1 x
m1 > m2 Þ >1 2x
m2 x
B A
p12 E2 E2
\ >1 Þ >1 [Q p1 = p2 ]
E1P22 E1 At point A and B pressure will be same. So pressure at
A = P + h1r1g
or, E2 > E1 pressure at B = P + h 2r2g
17. (b) P + h1r1g = P + h2r2g
F(N) or, 11.2 × 1 × g = 2x × 13.6 × g
11.2
or, x = = 0.41 cm
A B 2 ´ 13.6
3
2 24. (d) F = Y A a t = (2.0 × 1011) (10–6) (1.1 × 10–5) (20) = 44
newton
1 C 25. (c) Radiation is the fastest mode of transfer of heat.
x(m)
0 26. (c) The loop (i) is for soft iron and the loop (ii) is for steel
1 2 3 4 5 6 7
in fig.
Work done = area under F-x graph 1 2 æ1 ö 2
= area of trapezium OABC 27. (d) P = r v2 = ´ ç r v 2 ÷ = E
3 3 è 2 ø 3
1
=
(3 + 6) (3) = 13.5 J 28. (b) Rate of cooling µ temperature difference
2
between system and surrounding.
18. (b) Total kinetic energy of a rolling ball
As the temperature difference is halved, so the rate of
K.E. = linear K.E. + rotational K.E.
cooling will also be halved
K.E. = 1 mv 2 + 1 Iw2 29. (a) A is positive as electric lines are coming out of it. B is
2 2 negative as electric lines are entering into it.

1 1 2 v2 30. (a) ì xü
= mv 2 + ´ mr 2 . y = 2 sin pí200 t - ý
2 2 5 r2 î 15 þ

æ 5+2 ö 2 7 ì xp ü
= ç ÷ mv = 10 mv
2 = 2 sin í200pt - ý
è 10 ø î 15 þ
Fraction of total energy associated with rotational Comparing it with the equation of a wave,
y = a sin(wt - kx)
EBD_7418
MT- 22 PHYSICS

39. (b) As R µ V2/P or R µ 1/P, so resistance of heater is less


p
w = 200 p ;k = than that of fan.
15
40. (b) F = Bi l = 2 ×1.2 × 0.5 = 1.2 N
w 200p ´ 15 41. (d) Since p = nhn
v= = = 3000 cm / sec
k p
p 2 ´ 10 -3
31. (c) r = E / I = 1.5 / 3 = 0.5 ohm. Þn= =
hn 6.6 ´ 10 -34 ´ 6 ´ 1014
33. (a) The wheatstone bridge is balanced, when P/Q = R/S,
3 4 = 5 ´ 1015
In this case = , so bridge is balanced & 7W
4 8 42. (a) The range of wavelengths of the lines of Lyman series
resistance is not effective) varies from 912 A° to 1216 A°.
34. (d) For diffraction pattern to be observed, the dimension 43. (c) P.E. = 2T.E.
of slit should be comparable to the wavelength of light. Þ P.E. = 2E0
35. (c) By law of lepton numbers, mass number and atomic
In case of circular motion of satellite around the earth
number is to be conserved on both sides.
/ planets revolving around sun/electrons revolving in
36. (d) Since the magnetic field is uniform the flux f through circular orbit P.E. = 2 T.E. and |K.E.| = |T.E.|].
the square loop at any time t is constant, because
44. (d) Positive terminal is at lower potential (0V) and negative
f = B × A = B × L2 = constant
terminal is at higher potential 5V.
df
\ e=- = zero 45. (d) (A) is a NAND gate so output is 1´ 1 = 1 = 0
dt
37. (b) When the ray enters a glass slab from air, its frequency (B) is a NOR gate so output is 0 + 1 = 1 = 0
remains unchanged.
(C) is a NAND gate so output is 0 ´1 = 0 = 1
Since glass slab in an optically denser medium, the
velocity of light decreases and therefore we can (D) is a XOR gate so output is 0 Å 0 = 0
conclude that the wavelength decreases.
(Q n = nl)
0
1
1
MOCK TEST - 4
ANSWER KEYS
1 (d) 4 (c) 7 (a) 10 (a) 13 (b) 16 (c) 19 (d) 22 (c) 25 (a) 28 (a)
2 (b) 5 (c) 8 (d) 11 (d) 14 (d) 17 (b) 20 (c) 23 (a) 26 (a) 29 (d)
3 (c) 6 (d) 9 (b) 12 (c) 15 (b) 18 (c) 21 (c) 24 (c) 27 (a) 30 (a)

EXPLANATIONS
1. (d) 7. (a) Current flowing through the conductor,
I = n e v A. Hence
2. (b) 2
4 nevd1 p(1) vd 4 ´ 4 16
T = or 1 = = .
3. (c) Time to complete 1/4th oscillation is s. Time to 1 nev d p(2) 2 vd2 1 1
4 2
1
complete th vibration from extreme position is
8
8. (
(d) Gravitational field, I = 5iˆ + 12ˆj N/kg )
obtained from dv
I =-
dr
a 2p T
y= = a cos w t = a cos t or t = s
2 T 6 éx y ù
So time to complete 3/8th oscillation v = - ê ò I x dx + ò I y dy ú = – éë I x .x + I y .y ùû
êë 0 0
úû
T T 5T
= + =
4 6 12 a = – éë5 ( 7 - 0 ) + 12 ( -3 - 0 ) ùû
4. (c) From figure,
ma = F – f ....(i) Mass = m F = - éë35 + ( -36 ) ùû = 1 J / kg
And, torque t = Ia O
i.e., change in gravitational potential 1 J/kg.
mR 2 Hence change in gravitational potential energy 1 J
a = fR f
2 9. (b) Dl = a D T = 10 -5 ´ 100 = 103
mR 2 a é aù l
= fR êQ a = ú
2 R ë Rû Dl
´ 100% = 10 -3 ´100
ma l
= f ...(ii)
2
Put this value in equation (i), = 10 -1 = 0.1%
10. (a)
ma 3ma
ma = F – or F = 11. (d)
2 2
5. (c) Work function of aluminium is 4.2 eV. The energy of 12. (c) For a single negative point charge, electric lines of
two photons can not be added at the moment force are radial and inwards.
photons collide with electron all its energy will be
dissipated or wasted as this energy is not sufficient 13. (b) The bullets are fired at the same initial speed
to knock it out. H u 2 sin 2 60º 2g sin 2 60º
Hence emmission of electron is not possible. = ´ =
H¢ 2g u 2 sin 2 30º sin 2 30º
6. (d) In LCR series circuit, resonance frequency f0 is given
by
( 3 / 2)2 3
1 1 1 = =
Lw = Þ w2 = \ w= = 2pf0 (1/ 2) 3 1
Cw LC LC
14. (d) Total resistance of the circuit
1 1
\ f0 = or f0 a = 4000 + 400 = 4400 W
2p LC C
When the capacitance of the circuit is made 4 times, V 440
its resonant frequency become f 0' Current flowing i = = = 0.1 amp.
R 4400
f0' C f0 Voltage across load = R i
\ = or f 0' =
f0 4C 2 = 4000 × 0.1 = 400 volt.
EBD_7418
MT- 24 PHYSICS

16. (c) Phase difference = f f0


25. (a) =9, \ f0 = 9f e
l l fe
Path diff = ´ phase diff . = f
2p 2p Also f0 + fe = 20 (Q final image is at infinity)
é 1 1 ù 9 fe + fe = 20, fe = 2 cm, \ f0 = 18 cm
17. (b) E = Rhc ê - ú
êë n1 2
n 2 2 úû 26. (a) Let pressure outside be P0
E will be maximum for the transition for which 2T
\ P1 ( in smaller bubble ) = P0 +
r
é 1 1 ù
ê 2 - 2 ú is maximum. Here n 2 is the higher 2T
ëê n1 n 2 ûú P2 ( in bigger bubble ) = P0 + ( R > r)
R
energy level.
\ P1 > P2
é 1 1 ù
Clearly, ê - ú is maximum for the third hence air moves from smaller bubble to bigger bubble.
2
ëê n1 n 2 2 ûú 27. (a) Given : T/2 = 0.5 s
transition, i.e. 2 ® 1 . I transition represents the \ T = 1s
absorption of energy. 1 1
ur ur Frequency, f = = = 1Hz
18. (c) Vector ( E ´ B ) T 1
19. (d) Velocity after the collision If A is the amplitude, then
2A = 50 cm Þ A = 25 cm.
10 ´10 + 5 ´ 0 100 20
= = = m / sec . 28. (a) Width of central maximum
15 15 3
20. (c) The centre of mass remains at rest because force of 2 l D 2 ´ 6250 ´ 10 -10 ´ 0.5
= =
attraction is mutual. No external force is acting. a 2 ´ 10 -4
3R ´ 289 æ 3RT ö = 3125 ´ 10 -6 m = 312.5 ´ 10 -3 cm.
21. (c) v oxg . = çè vrms = M ÷ø
32

3R ´ 400
vH = so vH = 2230.59 m/sec
2 sin q = 2la
22. (c) For the observation of interference phenomenon, the
two source must be coherent & must have same sin q = a l
frequency. a q o sin q = 0
23. (a) G = 15W, ig = 4 mA, i = 6 A –l
sin q = a
Required shunt,
Intensity
æ 4 ´ 10-3 ö sin q = –2l
a
æ ig ö
S=ç ÷ G = ç ÷ ´ 15 D
è i - ig ø è 6 - 4 ´ 10-3 ø

4 ´ 10-3 Screen position of various minima for Fraunhoffer


= ´ 15 = 0.01W
5.996 diffraction pattern of a single slit of width a.
= 10mW (in parallel) 29. (d) In a p-type semiconductor germanium is doped with
trivalent element. All the elements, like aluminium,
Fl Fl mgl
24. (c) Y= Þ Dl = = boron & gallium are trivalent. So, the correct
A l
D YA YA alternative is (d).
MOCK TEST - 5

ANSWER KEYS
1 (d) 4 (b) 7 (c) 10 (a) 13 (d) 16 (c) 19 (b) 22 (c) 25 (c) 28 (a)
2 (a) 5 (d) 8 (b) 11 (b) 14 (a) 17 (d) 20 (c) 23 (c) 26 (a) 29 (a)
3 (c) 6 (c) 9 (a) 12 (d) 15 (d) 18 (a) 21 (d) 24 (c) 27 (d) 30 (b)

EXPLANATIONS
1 1
1. (d) wk = ´ = [ L-1 T -1 ] æ 5ö
\ x = vt = ç 60 ´ ÷ ´ 10 =
500
m
T L 18 ø 3
è
The dimensions of the quantities in a, b, c are of velocity
9. (a) Due to the charge inside a sphere of radius r only.
[LT–1]
r
2. (a) For inelastic collision, linear momentum is conserved 10. (a) vobj , mirror = 4iˆ + 9 ˆj

v1 dx dy
Þ mv1 = 2mv 2 Þ v 2 = dt
= 4,
dt
= 9; u=–x
2
Loss in K.E. = Gain in P.E. 1 1 1 -10 x
- = + ; V=
10 V - x x - 10
1 1
= mv12 - (2m) v 22 = 2 mgh 2
2 2 dV æ 10 ö æ dx ö
v1x = =ç ÷ ç ÷ ; v = – 16
dt è x - 10 ø è dt ø 1x
2
mv12 mv1 mv2 v2
Þ 4 mgh = mv12 – = = Þ h= V -10 y0
2 2 2 8g m=- = = 1
- x x - 10 y
3. (c) Considering the two masses and the rope a system,
then æ 10 ö dy
v1 y = - ç ÷ ; v1y = –18
Initial net force = [ 25 - (15 + 5)] g = 5g è x - 10 ø dt
r r
vimage, mirror = -16iˆ - 18 ˆj; vimage = -12iˆ - 16 ˆj
Final net force = ëé( 25 + 5 ) - 15 ûù g = 15 g
| v | = 20 cm/s
Þ (acceleration)final = 3 (acceleration)initial 11. (b) Paramagnetic liquid tends to flow from region of weaker
magnetic fields to stronger magnetic fields.
1 æ 1 ö2 æ 1ö
4. (b) S1 = g ç ÷ , S1 + S2 = çè ÷ø g(1)
2
dL 4A - A 3
2 è2ø 2 12. (d) Torque = = = A

You might also like